You are on page 1of 789

Solutions Manual

Copyright by The McGraw-Hill Companies, Inc. All rights reserved. Printed in the United
States of America. Permission is granted to reproduce the material contained herein on the
condition that such material be reproduced only for classroom use; be provided to students,
teachers, and families without charge; and be used solely in conjunction with Glencoe
Algebra 1. Any other reproduction, for use or sale, is prohibited without prior written
permission of the publisher.
Send all inquiries to:
Glencoe/McGraw-Hill
8787 Orion Place
Columbus, OH 43240-4027
ISBN: 0-07-827752-3
1 2 3 4 5 6 7 8 9 10

009

07 06 05 04 03 02

Algebra 1
Solutions Manual

Contents
Chapter 1

The Language of Algebra . . . . . . . . . . . . . . . . . . . . . . . 1

Chapter 2

Real Numbers . . . . . . . . . . . . . . . . . . . . . . . . . . . . 33

Chapter 3

Solving Linear Equations . . . . . . . . . . . . . . . . . . . . . . 62

Chapter 4

Graphing Relations and Functions . . . . . . . . . . . . . . . . . 122

Chapter 5

Analyzing Linear Equations . . . . . . . . . . . . . . . . . . . . 191

Chapter 6

Solving Linear Inequalities . . . . . . . . . . . . . . . . . . . . . 244

Chapter 7

Solving Systems of Linear Equations and Inequalities . . . . . . 305

Chapter 8

Polynomials . . . . . . . . . . . . . . . . . . . . . . . . . . . . . 364

Chapter 9

Factoring . . . . . . . . . . . . . . . . . . . . . . . . . . . . . . 400

Chapter 10

Quadratic and Exponential Functions . . . . . . . . . . . . . . . 438

Chapter 11

Radical Expressions and Triangles . . . . . . . . . . . . . . . . . 491

Chapter 12

Rational Expressions and Equations . . . . . . . . . . . . . . . . 533

Chapter 13

Statistics. . . . . . . . . . . . . . . . . . . . . . . . . . . . . . . 578

Chapter 14

Probability . . . . . . . . . . . . . . . . . . . . . . . . . . . . . 601

Prerequisite Skills . . . . . . . . . . . . . . . . . . . . . . . . . . . . . . . . . . 633


Extra Practice . . . . . . . . . . . . . . . . . . . . . . . . . . . . . . . . . . . . 643
Mixed Problem Solving . . . . . . . . . . . . . . . . . . . . . . . . . . . . . . . 766

iii

PQ249-6481F-01[001-009] 26/9/02 4:06 PM Page 1 Sahuja Ahuja_QXP_06:Desktop Folder:Chandra:Algebra_FNL_Delivery:

Chapter 1
Page 5

The Language of Algebra


1-1

Getting Started

1. 8  8  64
4. 23  6  138
6. 68  4 

68
4

2. 4  16  64 3. 18  9  162
57
5. 57  3  3  19
 17 7.

72
3

 24

8.

90
6

Page 8

 15

1 32
11
 41 8 2
 4 18

7. 44  4  4  4  4 4
 256
8. the product of 4 and m to the fourth power
9. one half of n cubed
10. Lorenzo will receive the difference of $20 and p
the cost of the peanuts. The word difference
implies subtract, so the expression can be written
as 20  p.

11
2
1
52

The perimeter is 5 2 feet.


12. P  2l  2w

 2 42 8  2 25 4





10
2
84 8  50 4
2
2
85 8  50 4
1
2
85 4  50 4
3
135 4

Pages 89
3

14.

1.2
6
7.2

1.8
15. 1.83.2
 4
1 8
14 4
1 4 4
0
17.

3
4

3.9
 0.5
1.95

7.6
16. 1.4
 10.6
4
9 8
84
8 4
0

12
1

3
4

 12  4 

19.

12
1

 34
1

5
16

18. 1 3  4  3  4
3

1

4

9

 14

9
12

5
16

12
 9

20.

5
6

2
3

5
6
2

18. One-half implies multiply by 2, and cube implies


raise to the third power. So the expression can be
1
written as 2 n3.

3
2

 16 
4

15

 36


12
9

Practice and Apply

11. The word sum implies add, so the expression can


be written as 35  z.
12. Sample answer: Let x be the number. The word
sum implies add, so the expression can be written
as x  7.
13. The word product implies multiply, so the
expression can be written as 16p.
14. Sample answer: Let y be the number. The word
product implies multiply, so the expression can be
written as 5y.
15. Sample answer: Let x be the number. Increased
by implies add, and twice implies multiply by 2.
So the expression can be written as 49  2x.
16. And implies add, and times implies multiply. So
the expression can be written as 18  3d.
17. Sample answer: Let x be the number. Two-thirds
2
implies multiply by 3, and square implies raise
to the second power. So the expression can be
2
written as 3 x 2.

The perimeter is 135 4 feet.


13.

Check for Understanding

1. Algebraic expressions include variables and


numbers, while verbal expressions contain words.
2. Sample answer: The perimeter is the sum of the
four sides of the rectangle. Thus, the perimeter is
the sum of two ls and two ws, or 2l  2w.
3. Sample answer: Let a be the variable. Then, a to
the fifth power is a5.
4. The word sum implies add, so the expression can
be written as j  13.
5. Sample answer: Let x be the number. Less than
implies subtract in reverse order, and times
implies multiply. So the expression can be written
as 3x  24.
6. 92  9  9
 81

9. P  2(l  w)
 2(5.6  2.7)
 2(8.3)
 16.6
The perimeter is 16.6 meters.
10. P  2(l  w)
 2(6.5  3.05)
 2(9.55)
 19.1
The perimeter is 19.1 centimeters.
11. P  4s

Variables and Expressions

4

19. The word for implies multiply. So the amount of


money Kendra will have is represented by the
expression s  12d.

 14

5
12

Chapter 1

PQ249-6481F-01[001-009] 26/9/02 4:06 PM Page 2 Sahuja Ahuja_QXP_06:Desktop Folder:Chandra:Algebra_FNL_Delivery:

47. You can use the expression 4s to find the


perimeter of a baseball diamond. Answers should
include the following.
four times the length of the sides and the sum
of the four sides
ssss
48. D; More than implies add, and times implies
multiply. So the expression can be written as
6  2x or 2x  6.
49. B; 4  4  4  43 and c  c  c  c  c4

20. The word square implies raise to the second


power. So the area of the circle is represented by
the expression r2.2
21. 62  6  6 2
 36
22. 82  8  8
 64
23. 34  3  3  3  3
 81
24. 63  6  6  6
 216
25. 35  3  3  3  3  3
 243
26. 153  15  15  15
 33753
27. 106  10  10  10  10  10  10
 1,000,000

Page 9
50.

28. 1003  100  100  100


 1,000,000
29. The word for implies multiply. So the cost of
buying b dozen bagels for $8.50 a dozen can be
represented by the expression 8.5b. Also the cost
of buying d dozen donuts for $3.99 a dozen can be
represented by the expression 3.99d. The word
and implies add. So the cost of buying b dozen
bagels and d dozen donuts can be represented by
the expression 8.5b  3.99d.
30. The word for implies multiply. So the amount of
miles driven for two weeks can be represented by
the expression 14m. The mileage on Saris
odometer after her trip is the sum of 23,500 and
14m or the expression 23,500  14m.
31. 7 times p
32. 15 times r
33. three cubed
34. five to the fourth power
35. three times x squared plus four
36. 2 times n cubed plus 12
37. a to the fourth power times b squared
38. n cubed times p to the fifth power
39. Sample answer: one-fifth 12 times z squared
40. Sample answer: one-fourth 8 times g cubed
41. 3 times x squared minus 2 times x
42. 4 times f to the fifth power minus 9 times k cubed

54.

Maintain Your Skills


1

14.3
 1.8
16.1

1
3

51.

09 91

10.00 52.
3.24
6.76

 5  15  15

55.

3.2
11
15.3
1.04 53. 4.8
 6
14 4
 4.3
96
312
9 6
416
0
4.472
3
4

11

3
8

 15
56.

 6  12  12
 12

989
1

6
57.

7
10

 5  10  3
1

 10  3
2

 6 or 16

Page 10

Reading Mathematics

c; 9  2  n
2. b; 4  (n  6)
3. f; n  52
h; 3(8  n)
5. g; 9  (2  n)
6. d; 3(8)  n
a; (n  5)2
8. e; 4  n  6
Sample answer: one more than five times x
Sample answer: five times the quantity x plus one
Sample answer: three plus the product of seven
and x
12. Sample answer: the sum of three and x multiplied
by seven
13. Sample answer: the sum of six and b divided by y
14. Sample answer: six plus the quotient of b and y

1.
4.
7.
9.
10.
11.

43. Sum implies add, and product implies multiply.


1
So the expression can be written as x  11x.
44. Sum implies add, product implies multiply, and
twice implies multiply by 2. So the expression can
be written as 2lw  2lh  2wh.
45. The word by implies multiply, so the expression
can be written as 3.5m.

1-2
Page 13

Check for Understanding

1. Sample answer: First add the numbers in


parentheses, (2  5). Next square 6. Then
multiply 7 by 3. Subtract inside the brackets.
Multiply that by 8. Divide, then add 3.
2. Sample answer: (2  4)  3
3. Chase; Laurie raised the incorrect quantity to the
second power.

46. The area of the square can be represented by the


expression a2 or a  a . The perimeter can be
represented as a  a  a  a or 4a. If a  4,
then a  a  4  4  16 and 4  a  4  4  16.
Thus, the value of a is 4.

Chapter 1

Order of Operations

PQ249-6481F-01[001-009] 26/9/02 4:06 PM Page 3 Sahuja Ahuja_QXP_06:Desktop Folder:Chandra:Algebra_FNL_Delivery:

4. (4  6)7  (10)7
5. 50  (15  9)  50  24
 70
 26
6. 29  3(9  4)  29  3(5)
 29  15
 14
7. [7(2)  4]  [9  8(4)]  [14  4]  [9  8(4)]
 10  [9  8(4)]
 10  [9  32]
 10  41
 51
8.

(4  3) 2  5
9  3






17. 15  3  2  15  6
18. 22  3  7  22  21
 21
 43
19. 4(11  7)  9  8  4(18)  9  8
 72  9  8
 72  72
0
20. 12(9  5)  6  3  12(14)  6  3
 168  6  3
 168  18
 150
21. 12  3  5  42  12  3  5  16
 4  5  16
 20  16
4
22. 15  3  5  42  15  3  5  16
 5  5  16
 25  16
9
23. 288  [3(9  3)]  288  [3(12)]
 288  [36]
8
24. 390  [5(7  6)]  390  [5(13)]
 390  [65]
6

(12) 2  5
9  3
144  5
9  3
720
9  3
720
12

 60
9.

3  23
52 (4)




3  8
52 (4)
11
52 (4)
11

 25 (4)
11

 100
10. hk  gj  6  12  4  8
 72  4  8
 72  32
 40
11. 2k  gh2  j  2  12  4  62  8
 2  12  4  36  8
 24  4  36  8
 24  144  8
 168  8
 160
12.

2g(h  g)
gh  j

25.






2  64  4  8
2  8
128  32
2  8
96
2  8
96
16

6
26.

4  62  42  6
4  6




2  4(6  4)
 468
2  4(2)
468
8(2)
468
16
468
16
 24  8
16
 16




4  36  16  6
4  6
144  96
4  6
48
4  6
48
24

2
27.

1
13. 3 packages
2 additional
of software plus packages
1442443 123 1442443

20.00
2  9.95

or 20.00
2  9.95
14. Evaluate 20.00  2  9.95.
20.00  2  9.95  20.00  19.90
 39.90
The cost of 5 software packages is $39.90.

Pages 1415

2  82  22  8
2  8

[ (8  5) (6  2) 2 ]  (4  17  2)
[ (24  2)  3 ]

28. 6 

Practice and Apply

[ (13) (4) 2 ]  (4  17  2)
[ (24  2)  3 ]

[ (13) (16) ]  (4  17  2)
[ (24  2)  3 ]

208  (68  2)
[ (24  2)  3 ]

208  34
[ (24  2)  3 ]

174
[ (24  2)  3 ]

174
[ 12  3 ]

174
4

87
2

or 43 2

3 2 3 7  (2  3  5) 4  6  3 2 3 7  (6  5) 4
2  7
 6  3 3  14
9
 6  33  14
 6  [ 3  1]
62
4

15. (12  6)  2  6  2
 12
16. (16  3)  4  13  4
 52

Chapter 1

PQ249-6481F-01[001-009] 26/9/02 4:06 PM Page 4 Sahuja Ahuja_QXP_06:Desktop Folder:Chandra:Algebra_FNL_Delivery:

29. Evaluate n(2n  3) for n  4.


n(2n  3)  4(2  4  3)
 4(8  3)
 4(11)
 44
The area of the rectangle is 44 cm2.2.
Derrick
30. Samantha
sells 60
floor seats

and

70 balcony
seats

and

60(7.5)

70(5)

or 60(7.5)

70(5)

sells 50
floor seats

38.

1xy 2

3y  z
(x  y) 2


90 balcony
seats.

90(5)

90(5)





31. Evaluate 60(7.5)  70(5)  50(7.5)  90(5).


60(7.5)  70(5)  50(7.5)  90(5)  450  350
 375  450
 800  375  450
 1175  450
 1625
Samantha and Derrick have collected $1625.
32. x  y2  z2  12  82  32
 12  64  32
 12  64  9
 76  9
 85
33. x3  y  z3  123  8  33
 1728  8  33
 1728  8  27
 1736  27
 1763
34. 3xy  z  3  12  8  3
 36  8  3
 288  3
 285
35. 4x  yz  4  12  8  3
 48  8  3
 48  24
 24
36.

2xy  z3
z









39.

37.







x  z
y  x

2y  x
y2  2





9
3  8  3
 (12  8) 2
4
9
24  3
 (12  8) 2
4
9
21
 (12  8) 2
4
9
21
 42
4
9
21
 16
4
36
21
 16
16
15
16

12  32
2  8  12
 82  2
8  12
12  9
16  12
 64  2
8  12
3
4
 64  2
8  12
3
4

2
32
3

 3  2  32
9

 2  32
9  16

 2  16  32




144
4
 32
32
148
32
37
5
or 4 8
8

40. 100 cells after


20 minutes

250 cells after


20 minutes
plus
144424443 123 144424443
100.2
250.2

or 100  2  250  2
Evaluate 100  2  250  2.
100  2  250  2  200  250  2
 200  500
 700
The total number in both dishes is 700 bacteria
cells.
41. the sum of salary, commission, and 4 bonuses
4 bonuses
salary plus commission plus 1442443
42. 14243
123 1442443 123

2  12  8  33
3
2  12  8  27
3
24  8  27
3
192  27
3
165
3

4b
s

c

or his earnings e is s  c  4b.
43. Evaluate s  c  4b for s  42,000,
c  12  825 or 9900, and b  750.
s  c  4b  42,000  9900  4  750
 42,000  9900  3000
 51,900  3000
 54,900
Mr. Martinez earns $54,900 in a year.

12  82  3  3
3
12  64  3  3
3
768  3  3
3
768  9
3
759
3

 253

Chapter 1

 55
xy2  3z
3

and

1442443 123 1442443 123 1442443 123 1442443

50(7.5)
50(7.5)

3  8  3
1128 2  (12
 8)
3
3  8  3
 1 2 2  (12  8)

PQ249-6481F-01[001-009] 26/9/02 4:06 PM Page 5 Sahuja Ahuja_QXP_06:Desktop Folder:Chandra:Algebra_FNL_Delivery:

44. Sample answer: Using 1, 2, 3:


1  2  3  6; 1  2  3  7; 1  2  3  5;
3  2  1  1; (2  1)  3  3
45. Use the order of operations to determine how
many extra hours were used and then how much
the extra hours cost. Then find the total cost.
Answers should include the following.
6 [4.95  0.99(n)]  25.00
You can use an expression to calculate a specific
value without calculating all possible values.
46. A; Evaluate a  b  c for a  10, b  12, and c  17.
a  b  c  10  12  17
 22  17
 39
47. B; (5  1)3  (11  2)2  (7  4)3  43  92  33 3
 64  81  27
 145  27
 172
48. Enter:

49. Enter:

4 9 91

0.5000
0.0075
0.4925

2.884
66. 5.214.9968
 
10 4
4 59
4 16
436
416
208
208
0
1

68. 4 8  1 2 




( .25 .75
2 ) ( 7
.75
3 ) ENTER 0.0476190476
( 2 27.89
2

50. Enter:

64.

( 12.75
( 12.75

2 )

27.89

ENTER 2.074377092
12.75
2 )

)
2

27.89

12.75


70.

5
6

3
2
3  4
2  4
12
8

5.600
 1.612
7.212

67.

6.42 5
 2.3
1 9 27 5
12 8 50
14.7 77 5

69.

3
5

1 11

19
7
21
95

35
35
116
35
11
3 35

 27  5 




5  6 5


71. 8 

2
9

2
3
8

19
8

 12

ENTER 1.170212766

Page 15

33

8
33

8
33

8
21
8
5
28

65.

Maintain Your SkillsMaintain Your Skills

8
1

36
1

2

 36

51. Product implies multiply, so the expression can be


written as a3  b4.
52. Less than implies subtract in reverse order, and
times implies multiply. So the expression can be
written as 3y2  6.
53. Sum implies add, increased by implies add, and
quotient implies divide. So the expression can be
b
written as a  b  a.
54. Times implies multiply, sum implies add,
increased by implies add, twice implies multiply
by 2, and difference implies subtract. So the
expression can be written as 4(r  s)  2(r  s).
55. Triple implies multiply by 3, and difference
implies subtract. So the expression can be written
as 3(55  w3).3).
56. 24  2  2  2  2
 16
57. 121  121
58. 82  8  8
59. 44  4  4  4  4
 64
 256
60. Five times n plus n divided by 2
61. 12 less than q squared
62. the sum of x and three divided by the square of
the quantity x minus two
63. x cubed divided by nine

1-3
Page 18

Open Sentences
Check for Understanding

1. Sample answer: An open sentence contains an


equals sign or inequality sign.
2. Sample answer: x  7
3. Sample answer: An open sentence has at least
one variable because it is neither true nor false
until specific values are used for the variable.
4. Replace x in 3x  7  29 with each value in the
replacement set.
x

3x  7  29

True or False?

10 31102  7  29 S 23 29
?

11 31112  7  29 S 26 29
?

12 31122  7  29 S 29  29
?

13 31132  7  29 S 32 29
?

14 31142  7  29 S 35 29
?

15 31152  7  29 S 38 29

false
false
true
false
false
false

The solution of 3x  7  29 is 12.

Chapter 1

PQ249-6481F-01[001-009] 26/9/02 4:06 PM Page 6 Sahuja Ahuja_QXP_06:Desktop Folder:Chandra:Algebra_FNL_Delivery:

11. Replace x in 3(12  x)  2 28 with each value


in the replacement set.

5. Replace x in 12(x  8)  84 with each value in


the replacement set.
12(x  8)  84

True or False?

10 12110  82  84 S 24 84

false

11 12111  82  84 S 36 84

1.5 3(12  1.5)  2 28 S 29.5 28

false

12 12112  82  84 S 48 84

false

13 12113  82  84 S 60 84
?

false

15 12115  82  84 S 84  84
3

x  5  1 20
2 ?

or C 

13. Solve C 
C

3
1 20

false

C

 5  1 20 S 1 20  1 20

 5  1 20 S 20 1 20

2 ?

2 ?

2 ?

13

false

1 4  5  1 20 S 1 20 1 20
2

Pages 1920

True or
False?

7.2(x  2)  25.92

1.2

7.211.2  22  25.92 S 23.04 25.92

1.4

7.211.4  22  25.92 S 24.48 25.92

1.6

7.211.6  22  25.92 S 25.92  25.92

1.8

7.211.8  22  25.92 S 27.36 25.92

?
?
?
?

12
17

false

18
false
21

true

25

false

w2

2
3
4
5
6

true

true

true

true

true

true

24  2122
13 S 20
13
24  2132
13 S 18
13
24  2142
13 S 16
13
24  2152
13 S 14
13
?

24  2162
13 S 12 13

12
17
18
21

True or False?

24  2112
13 S 22
13

25

14

3500  4
.
14
3500  4
14
14,000
14

Practice and Apply

b  12  9

True or False?

12  12  9 S 0 9

false

17  12  9 S 5 9

false

18  12  9 S 6 9

false

21  12  9 S 9  9

true

25  12  9 S 13 9

false

34  b  22

True or False?

34  12  22 S 22  22
?

34  17  22 S 17 22
?

34  18  22 S 16 22
?

34  21  22 S 13 22
?

34  25  22 S 9 22

The solution of 34  b  22 is 12.

false

The solution set for 24  2x


13 is {0, 1, 2, 3, 4, 5}.
Chapter 1

15. Replace b in 34  b  22 with each value in the


replacement set.

27  x
w3
The solution is 27.
The solution is 3.
10. Replace x in 24  2x
13 with each value in the
replacement set.

The solution of b  12  9 is 21.

The solution of 7.2(x  2)  25.92 is 1.6.


14  8
8. 4(6)  3  x
9. w  2

24  2102
13 S 24
13

3500

14. Replace b in b  12  9 with each value in the


replacement set.

7. Replace x in 7.2(x  2)  25.92 with each value in


the replacement set.

24  2x  13

the number
of days.

false

The solution of x  5  1 20 is 4.

24  3  x

divided
by

C  1000
The number of Calories a person would have to
burn each day is 1000 Calories.

true

1  5  1 20 S 1 5 1 20

the number
of Calories
the number
perpound times of pounds

3500  4
14

9
S 10

3
4

3
1 20

3
4

false

is

1
2

The number
of Calories

13

1
2

14

true

3(12  3)  2 28 S 25 28

1
4

true

1442443 123 1442443 123 1442443 14243 1442443

True or False?

1
4

2 ?

5

true

12.

6. Replace x in x  5  1 20 with each value in the


replacement set.
x

3(12  2)  2 28 S 28 28

The solution set for 3(12  x)  2 28 is {2, 2.5, 3}.

true

The solution of 12(x  8)  84 is 15.


2

false

2.5 3(12  2.5)  2 28 S 26.5 28

false

14 12114  82  84 S 72 84

True or
False?

3(12  x)  2  28

true
false
false
false
false

PQ249-6481F-01[001-009] 26/9/02 4:06 PM Page 7 Sahuja Ahuja_QXP_06:Desktop Folder:Chandra:Algebra_FNL_Delivery:

16. Replace a in 3a  7  31 with each value in the


replacement set.
3a  7  31

True or False?

3(0)  7  31 S 7 31

3(3)  7  31 S 16 31

3(5)  7  31 S 22 31

3(8)  7  31 S 31  31

?
?
?

10 3(10)  7  31 S 37 31

4a  5  17

false

4(0)  5  17 S 5 17

4(3)  5  17 S 17  17

?
?

4(5)  5  17 S 25 17

4(8)  5  17 S 37 17

?
?

10 4(10)  5  17 S 45 17

false

40

40
5

 4  0S4 0

40
8

10

40
10

1
S 93

 4  0S0  0

17
8

false

7 ? 17
8

21
8

17
8

false

The solution of x  4 

17
8

is 8.

false
true

1
2

false

12
3

 2  4S2 4

17

17
3

 2  4 S 33 4

18

18
3

 2  4S4  4

21

21
3

 2  4S5 4

25

25
3

false

25

1
2

True or false?

7 ? 25
 12
12

S 12 12

? 25
12

S 12 12

? 25
12

S 12  12

? 25
12

S 12 12

1  12 

false

12

1 2  12 

2  12 

13

25

false

19

25

false

25

25

true

31

25

false

25

The solution of x  12  12 is 1 2.
8

22. Replace x in 5 (x  1)  15 with each value in the


replacement set.

1
6
1
3
1
2
2
3

2
1x
5

 12  15

1  12  158 S 157 158


? 8
8
8
2 1
 1 2  15 S 15  15
5 13
? 8
3
8
2 1
 1 2  15 S 5 15
5 12
? 8
2 2
2
8
 1 2  15 S 3 15
5 13

True or False?

2 1
5 6

false
true
false
false
1

The solution of 5 (x  1)  15 is 3.
23. Replace x in 2.7(x  5)  17.28 with each value in
the replacement set.

True or False?

12

25

x  12  12

 4  0 is 10.

21. Replace x in x  12  12 with each value in the


replacement set.

true

24

false

2.7(x  5)  17.28
?

?
?

24
b
3

19
8

4

The solution of

7
8

19. Replace b in 3  2  4 with each value in the


replacement set.

7 ? 17
8

7
8

false

true

false

false

 4  0S1 0

b
3

17
8

4

false

40
a

The solution of

17
8

5
8

false

40
3

5
8

 4  0 S undefined 0

false

7 ? 17
 8
4

true

True or False?

40
0

17
8

18. Replace a in  4  0 with each value in the


replacement set.

3
8

40

True or False?

15
8

4

3
8

40
a

40
a

17
8

1
8

false

The solution of 4a  5  17 is 3.

with each value in the

7 ? 17
8

1
8

True or False?

17
8

x4

The solution of 3a  7  31 is 8.
17. Replace a in 4a  5  17 with each value in the
replacement set.
a

20. Replace x in x  4 
replacement set.

1
S 63

1.2 2.7(1.2  5)  17.28 S 16.74 17.28

false

1.3 2.7(1.3  5)  17.28 S 17.01 17.28

true

1.4 2.7(1.4  5)  17.28 S 17.28  17.28

false

1.5 2.7(1.5  5)  17.28 S 17.55 17.28

True or
False?
false
false
true
false

The solution of 2.7(x  5)  17.28 is 1.4.

false

 2  4 is 18.

Chapter 1

PQ249-6481F-01[001-009] 26/9/02 4:06 PM Page 8 Sahuja Ahuja_QXP_06:Desktop Folder:Chandra:Algebra_FNL_Delivery:

24. Replace x in 16(x  2)  70.4 with each value in


the replacement set.

p  4 [7(8)  4(25)  12]

True or
False?

16(x  2)  70.4

2.2

16 (2.2  2)  70.4 S 67.2 70.4

2.4

16 (2.4  2)  70.4 S 70.4  70.4

2.6

16 (2.6  2)  70.4 S 73.6 70.4

?
?
?
?

16 (2.8  2)  70.4 S 76.8 70.4

2.8

35. p  4 [7(23 )  4(52 )  6(2) ]


1

p  4 [56  100  12]


1

false

p  4 [156  12]

true

p  4 [144]

false

p  36
The solution is 36.

false

36. n  8 [6(32 )  2(43 )  2(7) ]

The solution of 16(x  2)  70.4 is 2.4.


25. Replace x in 21(x  5)  216.3 with each value in
the replacement set.

n  8 [54  128  14]


1

True or

n  8 [182  14]

False?

n  8 [164]

false

n  21
The solution is 21.

21(x  5)  216.3

n  8 [6(9)  2(64)  14]

3.1 21 (3.1  5)  216.3 S 170.1 216.3


?

4.2 21 (4.2  5)  216.3 S 193.2 216.3

false

37. Replace a in a  2  6 with each value in the


replacement set.

true

5.3 21 (5.3  5)  216.3 S 216.3  216.3


?

6.4 21 (6.4  5)  216.3 S 239.4 216.3

false

The solution of 21(x  5)  216.3 is 5.3.

26.

The
adult
the
student
is no
number
price
number
price
more
$30.
of adults times 1442443
per ticket plus 1442443
of students times 1442443
per ticket 123
than 123
1442443
123

123

123

$30

or 2a  3s 30
27. Evaluate 2a  3s for a  4.50 and s  4.50.
2a  3s  2(4.50)  3(4.50)
 9  13.50
 22.50
The cost for the family to see a matinee is $22.50.
28. Evaluate 2a  3s for a  7.50 and s  4.50.
2a  3s  2(7.50)  3(4.50)
 15  13.50
 28.50
The cost for the family to see an evening show is
$28.50.
29. 14.8  3.75  t
30. a  32.4  18.95
11.05  t
a  13.45
The solution is 11.05.
The solution is 13.45.
12  5
 3
60
12

31. y  15
y

g

7(3)  3

33. d  4(3  1)  6
21  3
4(2)

d

24
8

11

a
13
14
15
16
17

15  6
 7
90
9

6

6

d36
d9
The solution is 9.

a

4(14  1)
3(6)  5
4(13)

18  5

a

52
13

true

true

6  2 6 6S4 6 6
7  2 6 6S5 6 6
?

8  2 6 6S6  6
?

9  2 6 6S7  6
?

10  2 6 6 S 8  6
?

11  2 6 6 S 9  6

a  7  22

false
false
false

True or False?

true

true

13  7 6 22 S 20 6 22
14  7 6 22 S 21 6 22
?

15  7 6 22 S 22  22
?

16  7 6 22 S 23  22
?

17  7 6 22 S 24  22

false
false
false

39. Replace a in 5
2 with each value in the
replacement set.
a
5

 2

True or False?

7

5 ?

2S1 2

false

10

10 ?

2S2
2

true

15

15 ?

2S3
2

true

20

20 ?

2S4
2

true

25

25 ?

2S5
2

true

7

a47
a  11
The solution is 11.

The solution set for


Chapter 1

false

The solution set for a  7  22 is {13, 14}.

g  10
The solution is 10.
34. a 

True or False?

The solution set for a  2  6 is {6, 7}.


38. Replace a in a  7  22 with each value in the
replacement set.

32. g  16

y5
The solution is 5.
d

10

a26

a
5

2 is {10, 15, 20, 25}.

PQ249-6481F-01[001-009] 26/9/02 4:06 PM Page 9 Sahuja Ahuja_QXP_06:Desktop Folder:Chandra:Algebra_FNL_Delivery:

44. Replace b in 2b  5 with each value in the


replacement set.

2a

40. Replace a in 4 8 with each value in the


replacement set.
2a
4

a
12
14
16
18
20
22

8

21122 ?
4

8S6 8

8S7 8
8S8 8

21182 ?
4

8S9 8

8 S 10 8

21222 ?
4

8 S 11 8

The solution set for

2a
4

45.

3.4

4(3.4)  3
10.6 S 10.6
10.6

3.6
3.8

false

true

true

true

413.62  3
10.6 S 11.4
10.6
413.82  3
10.6 S 12.2
10.6
4(4)  3
10.6 S 13
10.6

4.2

614.22  5
23.8 S 20.2 23.8

4.5

614.52  5
23.8 S 22 23.8

$39

4.8

614.82  5
23.8 S 23.8
23.8

5.1

615.12  5
23.8 S 25.6
23.8

5.4

?
?

615.42  5
23.8 S 27.4
23.8

3a  4
3102 4 S 0 4

0
1
3
2
3

1
1 13

1 12

18,995

$10.95

true

2 39122  10.95 102.50 S88.95 102.50

true
true
1

39n  10.95  102.50


?

$102.50

Reasonable?
too low

3 39132  10.95 102.50 S127.95 102.50 too high

true

1 2

Examine The solution set is {0, 1, 2}. You can buy


2 sweaters and stay within your budget.
49. The solution set includes all numbers less than or
1
equal to 3.
50. You can use equations to determine how much
money you have to spend and how you can spend
your money. Answers should include the following.
Sample answers: calculating gasoline required
for a trip: cooking time

true

6220

true

3 13 4 S 4 4

123 14243 123 14243 123 14243

true

3112 4 S 3 4

15,579

glasses
of soda.

39  10.95 102.50
49.95 102.50
The estimate is too low. Increase the value of n.

false

true

113 2 4 S 1 4
2 ?
313 2 4 S 2 4

plus

True or False?

39n  10.95 102.50

The solution set for 6a  5


23.8 is {4.8, 5.1, 5.4}.
43. Replace a in 3a 4 with each value in the
replacement set.
a

plus

glasses
of juice

39(1)  10.95 102.50

false

is

glasses
of milk

or 39n  10.95 102.50


48. Explore You can spend no more than $102.50.
So the situation can be represented by
the inequality 39n  10.95 102.50.
Plan
Since no replacement set is given,
estimate to find reasonable values for
the replacement set.
Solve
Start by letting n  1 and then adjust
values up or down as needed.

True or
False?

6a  5  23.8

Total number
of glasses

1442443 123 1442443 123 1442443 14243

The solution set for 4a  3


10.6 is
{3.4, 3.6, 3.8, 4}.
42. Replace a in 6a  5
23.8 with each value in the
replacement set.
a

$102.50.
each sweater times of sweaters plus for shipping more than 14243

true

false

or g  15,579  6220  18,995


46. Evaluate 15,579  6220  18,995.
15,579  6220  18,995  21,799  18,995
 40,794
The average American drinks 40,794 glasses.
47. The cost of
the number
the charge
is not

True or
False?

4(3.2)  3
10.6 S 9.8 10.6

false

41. Replace a in 4a  3
10.6 with each value in the
replacement set.

3.2

1 2 6 5S5  5

true

1
22

2132 6 5 S 6  5

1442443

8 is {12, 14, 16}.

4a  3  10.6

true

The solution set for 2b  5 is 1, 12, 2 .

false

false

1 2 6 5S3 6 5

true

1
12

2122 6 5 S 4 6 5

2 12

false

21202 ?

true

True or False?

2112 6 5 S 2 6 5

1 12

true

21162 ?
4

true

21142 ?

2b  5

True or False?

The solution set for 3a 4 is 0, 3, 3, 1, 1 3 .

Chapter 1

51. B; Replace n in

15  n2 2  5

57. 53  3(42)  53  3(16)


 125  3(16)
 125  48
 173

6 28 with each value in

19  32 2  n

the replacement set.


15  n2 2  5

 28

11

19  3 2  n

15  52 2  5

15  72 2  5

15  92 2  5

True or False?

19  33 2  7

6 28 S 16 37 6 28

15  112 2  5

13

15  132 2  5

19  3 2  11
2

19  3 2  13
2

23

6 28 S 28 37  28
?

6 28

7
S 62 34

The solution set for

 26
1
59. [5(2  1)]4  3  [5(3)]4  3
 [15]4  3
 50,625  3
 50,628

true
false
false

60.

1
6

2
5

6
3

 28

15  n2 2  5

19  32  n
2

false

6 28 is {5, 7}.

5
6

15
16

9
3

3
7

15
16

63.

6
14

12
18

25
32

12
18

 14 
7
2

6
21
7

7
8
13

2
11

66.

3
11

7
16

8  2
 11
16
143

 13


65.

4
9

4  4
 9
16
63

7

3  7
 16

Page 20

4
7

 11


67.

21
176

2
9

24
25

9
3

24
25

16
75

Practice Quiz 1

1.
2.
3.
4.
5.

twenty less than x


five times n plus two
a cubed
n to the fourth power minus one
6(9)  2(8  5)  6(9)  2(13)
 54  2(13)
 54  26
 28
6. 4[2  (18  9)3]  4[2  (2)3]
 4[2  8]
 4[10]
 40
7. 9(3)  42  62  2  9(3)  16  62  2
 9(3)  16  36  2
 27  16  36  2
 27  16  18
 11  18
 29

t 14s  r2  2 14  5  22
1

 2 120  22
1

 2 1222
1

11
55. Sum implies add, and times implies multiply. So
the expression can be written as (r  s)t2.
1

Evaluate (r  s)t2 for r  2, s  5, and t  2.

4

8.

56. Decreased by implies subtract, so the expression


can be written as r5  t.
1
Evaluate r5  t for r  2 and t  2.

15  22 2

314  2  72

 314



r5  t  25  2
 32  2

10

132 2

 2  72
9
314  2  72
9
318  72
9
3112
9
3

3

 31 2
Chapter 1

3
7

Evaluate t(4s  r) for r  2, s  5, and t  2.

112 22
1
 172 1 4 2

 21

6

Maintain Your Skills

 172

4
9

62.

53. Increased by implies add, and times implies


multiply. So the expression can be written as
r 2  3s.
Evaluate r2  3s for r  2 and s  5.
r2  3s  22  3  5
43 5
 4  15
 19
54. Times implies multiply, and sum implies add. So
the expression can be written as t(4s  r).

112 22

61.

64.

1r  s2t2  12  52

2
5

 15

52. C; 27  3  (12  4)  27  3  8
98
 17

Page 20

6 28 S 43 7  28
?

26

 2  13
26

6 28 S 8 38 6 28

11

38  12
2  13

true

19  32 2  5

19  32 2  9

58.

9.

5  42  10  2
6  5






11. From Exercises 9 and 10 we determine that four


score and seven years is 87 years.

5  16  10  2
6  5
80  10  2
6  5
90  2
6  5
88
11

Pages 2325

8
10. Replace n in 2n2  3  75 with each value in the
replacement set.
2n2  3  75

n
4
5
6
7
8
9

True or False?

2  42  3  75 S 35  75
2

2  5  3  75 S 53  75
2  6  3  75 S 75  75
2  7  3  75 S 101 75
?

2  82  3  75 S 131 75
2

92

 3  75 S 165 75

true
true
true
false
false

18. Multiplicative Inverse Property;


1
1
n  2, since 1  2  2.

false

The solution set for 2n2  3  75 is {4, 5, 6}.

1-4

19. Multiplicative Inverse Property;


1
n  1, since 4  4  1.
20. Substitution Property;
n  5, since (9  7)(5)  2(n).
21. Substitution Property;
n  3, since 3  (2  8)  n  10.
22. Multiplicative Identity Property;

Identity and Equality Properties

Page 23

Check for Understanding

1. No; for 1 to be an additive identity, the sum of


any number a and 1 must be a. However,
3  1  4  3.
2. Sample answer: 5  3  2, and 3  2  4  1, so
5  4  1; 5  7  8  4, and 8  4  12, so
5  7  12.
3. Sample answer: You cannot divide by zero.
4. Multiplicative Property of Zero;
n  0, since 13  0  0.
5. Additive Identity Property;
n  17, since 17  0  17.

n  3, since 3 52 

n  2, since 6
24.

2  3 125  251 2

3
4

112  22  6  1  6.

[ 4  (7  4) ]
3

 4 [4  3] Substitution; 7  4  3

34

3 4
3

4

1
25.

7. 6(12  48  4)  6(12  12) Substitution; 48  4  12


 6(0)
Substitution; 12  12  0
0
Multiplicative Property
of Zero; 6  0  0
8.

1
25

3 1
 3.
23. Multiplicative Identity Property;

6. Multiplicative Inverse Property;


1
n  6, since 6  6  1.

1
15  15

Practice and Apply

12. Multiplicative Identity Property;


n  1, since 12  1  12.
13. Multiplicative Identity Property;
n  5, since 5  1  5.
14. Reflexive Property;
n  5, since 8  5  8  5.
15. Reflexive Property;
n  0.25, since 0.25  1.5  0.25  1.5
16. Additive Identity Property;
n  0, since 8  0  8.
17. Additive Identity Property;
1
1
1
n  3, since 3  0  3.

2
3

Substitution; 4  3  3
3
4

Multiplicative Inverse;

4
3

1

[ 3  (2  1) ]
2

 3 [3  2] Multiplicative Identity; 2  1  2

34

2 3
2

3

1

 8  0  12

Substitution; 3  2  2
Multiplicative Inverse;

26. 2 (3  2  5) 

 (1 + 8  0)  12 Multiplicative Inverse;
1
15  15  1
 (1  0)  12
Multiplicative Property of Zero;
8 00
 1  12
Additive Identity; 1  0  1
 12
Multiplicative Identity;
1  12  12

2
3

3
2

1

1
33
1

 2(6  5)  3  3 Substitution; 3  2  6
 2 112  3 
1

 2  33

1
3

Substitution; 6  5  1
Multiplicative Identity;
2 12

9. four score and seven


1
442443 123
1
424
3


4(20)
7
or 4(20)  7
10. 4(20)  7  80  7 Substitution; 4  20  80
 87
Substitution; 80  7  87

11

21

Multiplicative Inverse;
1
331

3

Substitution; 2  1  3

Chapter 1

35.

27. 6  6  5 (12  4  3)

Eight
players times
14243 123

 6  6  5(3  3) Substitution; 12  4  3
6

1
6

 5102

Substitution; 3  3  0

 1  5(0)

Three
players times
3

36.

37.

38.

39.
40.
41.

1169.10

or 1169.10  y  1169.10
y  0, since 1169.10  0  1169.10.
33.

increase
Pay for
from
grade
yr to 10 yr
E-4 at 6 yr times 6
1442443
1442443
123

1653

is

1653

123

pay for
grade
E-4 at 10 yr.
144424443

42.

Base
salary

is the
bonus for
plus 12 touchdowns same as

base
salary

$50,000

$350,000

bonus for
plus 76 points.


$50,000

or 350,000  50,000  350,000  50,000

Chapter 1

($400,000

$50,000

base
salary

plus

($400,000

1601
yards

14243

$1,000,000

$50,000)

$50,000)

12

4.5 yards
plus per carry

123 14243

$50,000)

or 4(400,000  1,000,000  50,000)


So the expression can be written as 8(400,000
 100,000  50,000)  3(400,000  50,000
 50,000)  4(400,000  1,000,000  50,000).
8(400,000  100,000  50,000)  3(400,000 
50,000  50,000)  4(400,000  50,000  50,000)
 8(550,000)  3(400,000  50,000  50,000) 
4(400,000  50,000  50,000) Substitution;
400,000  100,000  50,000  550,000
 8(550,000)  3(500,000)  4(500,000)
Substitution; 400,000  1,000,000  50,000
 1,450,000
 4,400,000  1,500,000  2,000,000
Substitution; 4  1,450,000  5,800,000
 7,900,000 Substitution; 4,400,000  1,500,000
 5,800,000  11,700,000
Sometimes; Sample answer: true: x  2, y  1,
z  4, w  3, 2
1 and 4
3 and 2  4  8
3
 1  3; false: x  1, y  1, z  2, w  3,
1
1 and 2
3 however, 1(2)  2 3 
(1)(3).
You can use the identity and equality properties
to see if data is the same. Answers should include
the following:
Reflexive: r  r, or Symmetric: a  b so b  a.
Oklahoma, week 1  a, week 2  b, week
3  c. a  b and b  c so a  c.
A; In words: If one quantity equals a second
quantity, then the second quantity equals the first.
B; Substitution Property, since 10  8  2.
False, since 4  5  1 and 1 is not a whole
number.
True, for any whole numbers a and b the product
ab is also a whole number.
1

14243 123 144424443 14243 14243 123 14243

$350,000

76 points
12
plus touchdowns plus scored

43. False, since 1  2  2 and


number.

or 1653y  1653
y  1, since 1653  1  1653.
34.

Four
players times

grade
E-2 at 12 yr.
144424443

base
salary

14243 123 14243 123

 80(2.5  1)  40(10  6)
25(5  3)
or 25(5  3) +
80(2.5 1)
+ 40(10  6)
31. 25(5  3)  80(2.5  1)  40(10  6)
 25(2)  80(2.5  1)  40(10  6) Substitution;
532
 25(2)  80(1.5)  40(10  6)
Substitution;
2.5  1  1.5
 25(2)  80(1.5)  40(4)
Substitution;
10  6  4
 50  120  160
Substitution;
40  4  160
 330
Substitution;
170  160
 330
increase
pay for
32. Pay for
123

$100,000

or 3(400,000  50,000  50,000)

Additive Identity; 1  0  1
1
2
28. 3  5(4  2 )  1
 3  5(4  4)  1 Substitution; 22  4
 3  5(0)  1
Substitution; 4  4  0
301
Multiplicative Property of
Zero; 5  0  0
31
Additive Identity; 3  0  3
2
Substitution; 3  1  2
29. 7  8(9  32)
 7  8(9  9) Substitution; 32  9
 7  8(0)
Substitution; 9  9  0
70
Multiplicative Property of Zero;
8 00
7
Additive Identity; 7  0  7
30. Profit for
profit for
profit for
and 80 buttons and 40 caps
25 peanuts 123
1442443
1442443 123 14243

is

14243 123 14243 123 1442443 123 14243

Multiplicative Property of
Zero; 5  0  0

from
grade
plus
yr to 12 yr
E-2 at 3 yr 123 3
1442443
1442443

y
1169.10

($400,000

or 8(400,000  100,000  50,000)

Multiplicative Inverse;
1
661

10

35% of
offensive
base
weight
salary plus below 240 lb plus plays
14243 123 1442443 123 14243

1
2

is not a whole

Page 25

Maintain Your Skills

10  x  6

10  3 7 6 S 7 7 6

10  5 7 6 S 5  6

10  6 7 6 S 4  6

10  8 7 6 S 2  6

True or False?

1
2

true

1
3

false

1
4

false
false

1
5

The solution set for 10  x


6 is {3}.
45. Replace x in 4x  2 58 with each value in the
replacement set.
4x  2  58

x
11

4  11  2 6 58 S 46 6 58

12

4  12  2 6 58 S 50 6 58

13
14

?
?

4  13  2 6 58 S 54 6 58
?

4  14  2 6 58 S 58  58
?

4  15  2 6 58 S 62  58

15

1
6

false

false

5.9

5.9 ?

2

3 S 2 20 3

false

3S3 3

true

1
S 3 20

6.1

6.1 ?

2

true

6.2

6.2 ?

2

3 S 3 10 3

true

6.3

6.3 ?

2

3
S 3 20

The solution set for

x
2

3
1
32

3
3.25
3.5
3.75
4

8  3  32 S 24  32
?

8  3.25  32 S 26  32
?

8  3.5  32 S 28  32
?

8  3.75  32 S 30  32
?

8  4  32 S 32  32

3
10

S 30 6

 24 6

3
10

S5 6

1
5

3
10

S 10  10

false

3
10

S 30  10

11

false

2
2

1
6

6
?

7
10

3
10

true

3
10

true

3
10

 2x 6

true

3
10

2x  1  2

is

512, 13, 14 6.

True or False?
1

true

false

2  3  1  2S5 2
1
2  32

false

 1  2S6 2

50. (3  6)  32  (9)  32
99
1
51. 6(12  7.5)  7  6(4.5)  7
 27  7
 20
52. 20  4  8  10  5  8  10
 40  10
4
53.

true

16  22 2
16

 3192 


3 is {6, 6.1, 6.2, 6.3}.

8x  32

1
3

5 16

false

The solution set for 2x  1  2 is 1 4 .

47. Replace x in 8x  32 with each value in the


replacement set.
x

S 10 6

2  2  1  2S3 2

3
10

false

6 ?

2

2  14  1  2 S 12  2

5.8

7
10

2

7
10
7
10

 22 6

14

True or False?

19

7
10

True or False?

true

5.8 ?

2

9
S 2 10

7
10

3

 2x  10

true

46. Replace x in 2 3 with each value in the


replacement set.
x

7
10

with each value in the

49. Replace x in 2x  1  2 with each value in the


replacement set.

true

The solution set for 4x  2 58 is {11, 12, 13}.

x
2

3
10

The solution set for

True or False?

48. Replace x in 10  2x 6
replacement set.

44. Replace x in 10  x
6 with each value in the
replacement set.

54.

True or False?
true
true
true

55.

true
true

The solution set for 8x  32 is {3, 3.25, 3.5, 3.75, 4}.

56.

57.

13

182 2

 3192

16
64

16

3192

 4  3(9)
 4  27
 31
[62  (2  4)2] 3  [62  (6)2]3
 [36  (6)2]3
 [36  12]3
 [24]3
 72
2
2
9(3)  4  6  2  9(3)  16  36  2
 27  16  36  2
 27  16  18
 11  18
 29
Sample answer: Let x be the number. Sum
implies add, and twice implies multiply by 2. So
the expression can be written as 2x2  7.
10(6)  10(2)  60  10(2)
 60  20
 80

Chapter 1

58. (15  6)  8  9  8
 72
59. 12(4)  5(4)  48  5(4)
 48  20
 28
60. 3(4  2)  3(6)
61. 5(6  4)  5(2)
 18
 10
62. 8(14  2)  8(16)
 128

4. 3(x  3)  3x  3
The rectangle has 3 x-tiles and 9 1-tiles. The area
of the rectangle is
x111x111x111
or 3x  9.
Therefore, 3(x  3)  3x  3 is false.
x3

The Distributive Property

1-5
Page 28

Algebra Activity

x
1
1

6. Rachel; Sample answer: 3(x  4)  3(x)  3(4) 


3x  12
The rectangle has 3 x-tiles. The area of the
rectangle is 3x  12. Therefore, Rachel is correct.

x4

1
1
1
1
1

 5x  10

Pages 2930

2x  1

Chapter 1

x
x

x
x

1
1

x
x
x

1
1
1

1 1
1 1
1 1

1
1
1

 3x  12

Check for Understanding

1. Sample answer: The numbers inside the


parentheses are each multiplied by the number
outside the parentheses then the products are
added.
2. Sample answer: 4ab  3b  a  2ab  7ab
3. Courtney; Ben forgot that w4 is really 1  w4.
4. 6(12  2)  6  12  6  2
 72  12
 60
5. 2(4  t)  2  4  2  t
 8  2t
6. (g  9)5  g  5  9  5
 5g  45
7. 16(102)  16(100  2)
 16(100)  16(2)
 1600  32
 1632

3. 2(2x  1)
The rectangle has 4 x-tiles and 2 1-tiles. The area
of the rectangle is
x  x  1  x  x  1 or 4x  2.
Therefore, 2(2x  1)  4x  2.

 5x

x x x x x

x2
1
1
1
1
1

 3x  9

 2x  2

2. 5 (x  2)
The rectangle has 5 x-tiles and 10 1-tiles. The
area of the rectangle is
x11x11x11x11
x  1  1 or 5x  10.
Therefore, 5(x  2)  5x  10.

x
x
x
x
x

1 1
1 1
1 1

32

x1

x
x

1
1
1

5. x(3  2)  3x  2x
The rectangle has 5 x-tiles. The area of the
rectangle is x  x  x  x  x or 3x  2x or 5x.
Therefore, x(3  2)  3x  2x is true.

1. 2(x  1)
The rectangle has 2 x-tiles and 2 1-tiles. The area
of the rectangle is
x  1  x  1 or 2x  2.
Therefore, 2(x  1)  2x  2.

x
x
x

 4x  2

14

1 12

8. 3 17 1172  3  17 17

9.
10.

11.
12.
13.

28. 4(8p  4q  7r)  4(8p)  4(4q)  4(7r)


 32p  16q  28r
29.
number of
number of
people at
people at
number
Olympic
aquatics
of days times Stadium plus center
14243 123 14243 123 14243

 3 1172  17 1172
1

 51  1
 52
13m  m  (13  1)m
 14m
3(x  2x)  3(x)  3(2x)
 3x  6x
 (3  6)x
 9x
There are no like terms. 14a2  13b2  27 is
simplified.
4(3g  2)  4(3g)  4(2)
 12g  8
average
number of
price of
plus
tip
haircuts times
each
haircut
14243 123 1442443 123 14243

($19.95

$2)
12

30.

31.

32.

or 12(19.95  2)
14. 12(19.95  2)  12(19.95)  12(2)
 239.40  24
 263.4
Ms. Curry earned $263.40.

Pages 3031

33.

Practice and Apply

34.

15. 8(5  7)  8  5  8  7
 40  56
 96
16. 7(13  12)  7  13  7  12
 91  84
 175
17. 12(9  5)  12  9  12  5
 108  60
 48
18. 13(10  7)  13  10  13  7
 130  91
 39
19. 3(2x  6)  3  2x  3  6
 6x  18
20. 8(3m  4)  8  3m  8  4
 24m  32
21. (4  x)2  4  2  x  2
 8  2x
22. (5  n)3  5  3  n  3
 15  3n

23. 28 y  7  281y2  28

24. 27 2b 

1
3

 28y  4

4

(110,000 
17,500)
or 4(110,000  17,500)
4(110,000  17,500)  4(110,000)  4(17,500)
 440,000  70,000
 510,000
The attendance for the 4-day period was 510,000
people.
5  97  5(100  3)
 5(100)  5(3)
 500  15
 485
8  990  8(1000  10)
 8(1000)  8(10)
 8000  80
 7920
17  6  (20  3)6
 20(6)  3(6)
 120  18
 102
24  7  (20  4)7
 20(7)  4(7)
 140  28
 168

1 12

35. 18 2 9  18 2  9

 18(2)  18
 36  2
 38

37.

12

1 12

36. 48 3 6  48 3  6

1
9

 48132  48
 144  8
 152

116 2

hours
hours
hours on
number
using
meeting
telephone
of weeks times e-mail plus in person 123
plus 1
14243 123 14243 123 1
4424
43
4424
43
12

(5

12

18)

or 12(5  12  18)

38. 12(5  12  18)  12(5)  12(12)  12(18)


 60  144  216
 420
She should plan 420 h for contacting people.
monthly
monthly
monthly
39.
charge
number
charge
charge
of months times for medical plus for dental plus for vision

1442443 123 1442443 123 1442443 123 1442443

12

1
7

(78

20

12)

or 6(78  20  12)

40. 6(78  20  12)  6(78)  6(20)  6(12)


 468  120  72
 660
The cost to the employee is $660.

2  2712b2  27 1 2
1
3

 54b  9
25. a(b  6)  a(b)  a(6)
 ab  6a
26. x(z  3)  x(z)  x(3)
 xz  3x
27. 2(a  3b  2c)  2(a)  2(3b)  2(2c)
 2a  6b  4c

15

Chapter 1

41.

number
monthly charge
monthly charge
of months times for medical
for dental
plus

l  w  13 2

1442443 123 144424443 123 144424443

42.
44.
45.
46.
47.
48.
49.
50.

51.

(78  50

12

(20  15)

3
w
2

or 12[(78  50)  (20  15)]


12[(78  50)  (20  15)]  12 [128  35]
 12(128)  12(35)
 1536  420
 1956
The cost to the employee is $1956.
2x  9x  (2  9)x
43. 4b  5b  (4  5)b
 11x
 9b
There are no like terms. 5n2  7n is simplified.
3a2  14a2  (3  14)a2
 17a2
12(3c  4)  12(3c)  12(4)
 36c  48
15(3x  5)  15(3x)  15(5)
 45x  75
2
6x  14x  9x  6x2  (14  9)x
 6x2  5x
3
3
4
4y  3y  y  (4  3)y3  y4
 7y3  y4
6(5a  3b  2b)  6(5a)  6(3b)  6(2b)
 30a  18b  12b
 30a  (18  12)b
 30a  6b
5(6m  4n  3n)  5(6m)  5(4n)  5(3n)
 30m  20n  15n
 30m  (20  15)n
 30m  5n

3 2
w
2

 13 2

3 2
w
2

2 3 2
w
3 2

27
2

2  23 1272 2

w2  9
w3
Substitute w  3 into the area formula.
1

l  w  13 2
1

l  3  13 2
1

l  42
1

Therefore, the rectangle has a length of 4 2 units


and a width of 3 units.
55. You can use the Distributive Property to calculate
quickly by expressing any number as a sum or
difference of more convenient numbers. Answer
should include the following.
Both methods result in the correct answer. In
one method you multiply then add, and in the
other you add then multiply.
56. D; 3(x  y)  2(x  y)  4x  3x  3y  2x  2y  4x
 3x  2x  4x  3y  2y
 (3  2  4)x  (3  2)y
 1x  5y
 x  5y
57. C; c  7(2  2.8  3  4.2)
 7(5.6  12.6)
 7(5.6)  7(12.6)
 39.2  88.2
 127.4

7
x
7
1
52. x2  8 x  8  x2  8 x  8x

178  18 2x

 x2 

 w  13 2

 x2  8 x
3

 x2  4 x
53. a 

a
5

Page 31

 5a  5a  5a  5a


155  15  25 2 a

58.
59.
60.
61.
62.
63.
64.

 5a

54. Area  l  w  13 2

w  5  perimeter  5  121l  w2 2
1

 5  12l  2w2
1
2

Solve w 
w
w

2
 5w
3
w
5
5 3
w
2 5
3
w
2

12





2
w
5
2
w
5

is

number of
number of
feet per second times seconds
1129

or d  1129(2)
65. 1129(2)  (1000  100  30  1)2
 1000(2)  100(2)  30(2)  1(2)
 2000  200  60  2
 2258
Sound travels 2258 feet in 2 seconds.
66. 3ab  c2  3  4  6  32
3469
 12  6  9
 72  9
 63

for l.

12

l

Substitute 2 w for l into the Area formula.

Chapter 1

total
distance

14243 123 144424443 123 1442443

 5l  5w
2
l
5
2
l
5
2
l
5
2
l
5
5 2
l
2 5

Maintain Your Skills

Symmetric Property
Substitution Property
Multiplicative Identity Property
Multiplicative Inverse Property
Multiplicative Inverse Property
Reflexive Property

16

67. 8(a  c)2  3 






68.

6ab
c 1a  22

7.

6  4  6

 314  22





69. 1a  c2

6. 5  3  6  4  5  4  3  6
 (5  4)  (3  6)
 20  18
 360

8(4  3)2  3
8(1)2  3
813
83
11

1a 2 b 2  14  32 14 2 6 2
4  6
 172 1 2 2
10
 172 1 2 2

 12 1142
 168
Area is 168 cm2.2.
72. A  s2
 8.52
 72.25
Area is 72.25 m2.a i

Commutative and Associative


Properties
Check for Understanding

1. Sample answer: The Associative Property says the


way you can group numbers together when
adding or multiplying does not change the result.
2. Sample answer: Division is not Commutative. For
example, 10  2  2  10.
3. Sample answer: 1  5  8  8  1  5;
(1  5)8  1(5  8)
4. 14  18  26  14  26  18
 (14  26)  18
 40  18
 58

64
 10

114 q  234 q2  2q
3
1
 1 4  24 2 q  2q

1442443
1
(p
2

1
1p
2

 2q)

 2q2  4q  2 1 p2  2 12q2 
3

 2 p  q  4q

1
2
3
1
 2 p  11  4 2q


1
1
32  22  4
1
1
32  22  4

5
6

5
6

 3q  2q
 13  22q
 5q
11. 3(4x  2)  2x  3(4x)  3(2)  2x
 12x  6  2x
 12x  2x  6
 (12x  2x)  6
 (12  2)x  6
 14x  6
12. 7(ac  2b)  2ac  7(ac)  7(2b)  2ac
 7ac  14b  2ac
 7ac  2ac  14b
 (7ac  2ac)  14b
 (7  2)ac  14b
 9ac  14b
13. 3(x  2y)  4(3x  y)  3(x)  3(2y)  4(3x)  4(y)
 3x  6y  12x  4y
 3x  12x  6y  4y
 (3  12)x  (6  4)y
 15x  10y
14. half the sum
by three-fourths q
of p and 2q increased
1442443 144424443

 2 1242 1142

4

5.

10. 4 q  2q  24 q  4 q  24 q  2q

71. A  2 bh

1
22

5
6

 130
8. 4x  5y  6x  4x  6x  5y
 (4x  6x)  5y
 (4  6)x  5y
 10x  5y
9. 5a  3b  2a  7b  5a  2a  3b  7b
 (5a  2a)  (3b  7b)
 (5  2)a  (3  7)b
 7a  10b

70. A  l  w
95
 45
Area is 45 in2.2.

1
32

 156 

8

Page 34

 16  94  16  94 

 16  94 

24  6
314  22
144
314  22
144
3162
144
18

 172 152
 35

1-6

5
6

1
p
2

 q

 2 p  14 q

17

3
q
4

3
q
4
3
q
4

Distributive
Property
Multiply.
Associative ()
Distributive
Property
Substitution

Chapter 1

15. number of
area of
triangles times
each
triangle
1442443 123 1444244
43
1

4
bh
2

11 2

26. 3 7  14  14  3 7  14  14
1

 48  14

112  5.2  7.862


1
 1 4  2 2  15.2  7.862

 60

4 2bh  4

27.

5
28

2
 24  6 3

 2 8  24  6 3
2

 63  6 3

 2  40.872
 81.744
The area of the large triangle is 81.744 cm2.

28.

 420
cost
cost
cost
Friday Saturday Sunday

cost
Monday

14243 14243 14243 14243

Pages 3436

$72  $63  $63  $72


72  63  63  72  72  63  72  63
 (72  63)  (72  63)
 135  135
 270
The total cost is $270.

Practice and Apply

16. 17  6  13  24  17  13  6  24
 (17  13)  (6  24)
 30  30
 60
17. 8  14  22  9  8  22  14  9
 (8  22)  (14  9)
 30  23
 53
18. 4.25  3.50  8.25  4.25  8.25  3.50
 (4.25  8.25)  3.50
 12.50  3.50
 16
19. 6.2  4.2  4.3  5.8  6.2  4.3  4.2  5.8
 (6.2  4.3)  (4.2  5.8)
 10.5  10
 20.5
1

29.
cost
tax
cost
tax
cost
tax
cost
tax
for
for
for
for
for
for
for
for
Friday
123 Saturday
1424
3
1424
3 Monday
14243 Saturday
123 Friday
14243 Sunday
1424
3 Sunday
1424
3 Monday

$72 $5.40 $63  $5.10  $63 $5.10 $72  $5.40

72  5.40  63  5.10  63  5.10  72  5.40


 72  63  72  63  5.40  5.10  5.40  5.10
 (72  63)  (72  63)  (5.40  5.10)  (5.40  5.10)
 135  135  10.50  10.50
 (135  135)  (10.50  10.50)
 270  21
 291
The total cost including tax is $291.
30. Sample answer:
Sales from Sales from Sales from
Sales from 3 new
5 used
2 older
2
DVDs
videos
releases
videos
14243

20. 6 2  3  2  2  6 2  2  3  2

 62  2  13  22
1

75
 12
3

14243

21. 2 8  4  3 8  28  3 8  4

 28  3 8  4




6
58
6
98
3
94

4

31.

22. 5  11  4  2  5  2  11  4
 (5  2)  (11  4)
 10  44
 440
23. 3  10  6  3  3  3  10  6
 (3  3)  (10  6)
 9  60
 540
24. 0.5  2.4  4  0.5  4  2.4
 (0.5  4)  2.4
 2  2.4
 4.8
25. 8  1.6  2.5  8  2.5  1.6
 (8  2.5)  1.6
 20  1.6
 32

Chapter 1

32.

33.

34.

18

14243

14243

2(3.99)  3(4.49)  2(2.99)  5(9.99)


2(3.99)  3(4.49)  2(2.99)  5(9.99)
 2(3.99)  2(2.99)  3(4.49)  5(9.99)
 2(3.99  2.99)  3(4.49)  5(9.99)
Two expressions to represent total sales can be
2(3.99)  3(4.49)  2(2.99)  5(9.99) and
2(3.99  2.99)  3(4.49)  5(9.99).
2(3.99  2.99)  3(4.49)  5(9.99)
 2(6.98)  3(4.49)  5(9.99)
 13.96  13.47  49.95
 27.43  49.95
 77.38
The total sales of the clerk are $77.38.
4a  2b  a  4a  a  2b
 (4a  a)  2b
 (4  1)a  2b
 5a  2b
2y  2x  8y  2y  8y  2x
 (2y  8y)  2x
 (2  8)y  2x
 10y  2x
 2x  10y
x2  3x  2x  5x2  x2  5x2  3x  2x
 (x2  5x2)  (3x  2x)
 (1  5)x2  (3  2)x
 6x2  5x

44. twice the sum


of s and t decreased by
1442443

35. 4a3  6a  3a3  8a  4a3  3a3  6a  8a


 (4a3  3a3)  (6a  8a)
 (4  3)a3  (6  8)a
 7a3  14a
36. 6x  2(2x  7)  6x  2(2x)  2(7)
 6x  4x  14
 (6x  4x)  14
 (6  4)x  14
 10x  14
37. 5n  4(3n  9)  5n  4(3n)  4(9)
 5n  12n  36
 (5n  12n)  36
 (5  12)n  36
 17n  36
38. 3(x  2y)  4(3x  y)  3(x)  3(2y)  4(3x)  4(y)
 3x  6y  12x  4y
 3x  12x  6y  4y
 (3x  12x)  (6y  4y)
 (3  12)x  (6  4)y
 15x  10y
39. 3.2(x  y)  2.3(x  y)  4x
 3.2(x)  3.2(y)  2.3(x)  2.3(y)  4x
 3.2x  3.2y  2.3x  2.3y  4x
 3.2x  2.3x  4x  3.2y  2.3y
 (3.2x  2.3x  4x)  (3.2y  2.3y)
 (3.2  2.3  4)x  (3.2  2.3)y
 9.5x  5.5y
40. 3(4m  n)  2m  3(4m)  3(n)  2m
 12m  3n  2m
 12m  2m  3n
 (12m  2m)  3n
 (12  2)m  3n
 14m  3n
41. 6(0.4f  0.2g)  0.5f  6(0.4f )  6(0.2g)  0.5f
 2.4f  1.2g  0.5f
 2.4f  0.5f  1.2g
 (2.4f  0.5f )  1.2g
 (2.4  0.5)f  1.2g
 2.9f  1.2g
42.

3
4

2(s  t)

s
2(s  t)  s  2(s)  2(t)  s Distributive
Property
 2t  2s  s
Comm. ()
 2t  s(2  1)
Dist.
 2t  s(1)
Sub.()
 2t  s
Mult. Id.
 s  2t
Comm. ()
45.
five times the
by 3xy
product of x and y increased
1442443 123
14444244443


3xy
5(x  y)
5(xy)  3xy  5(xy)  3(xy) Associative ()
 xy(5  3)
Distributive Property
 xy(8)
Substitution
 8xy
Commutative ()
46. the product of
6 and the
the sum of
square
of
z
increased
by
seven,
z2, and 6
1442443
1442443 144424443

6  z2

(7  z2  6)
2
2
6z  (7  z  6)
 6z2  (z2  7  6) Commutative ()
 (6z2  z2)  (7  6) Associative ()
 z2(6  1)  (7  6) Distributive Property
 z2(7)  13
Substitution
 7z2  13
Commutative ()
47.
three times the
six times
sum of x and half
the sum of
x144424443
and y squared decreased by 144424443
of y squared
1442443
6(x  y2)

6 1x 

 3x  42y2

123s  s2  43t
3
2
4
 4  1 3  1 2 s  3t
3

 4  3s  3t

1 2

3 1

1 2

43. 2p  5 2 p  2q  3  2p  5 2 p  5 2q  3
3

 2p  10 p  5q  3

3
1
2 6 2
3
6
2
 1 2  10 2 p  5 q  3

 2p  10 p  5 q  3

23

 10 p  5 q  3


2
3

23

 3 1x2  3
3

112 y22

Distributive
Property
Commutative ()
Distributive
Property
Commutative ()

48. Sometimes; Sample answer: 4  3  3  4,


but 4  4  4  4.
49. You can use the Commutative and Associative
Properties to rearrange and group numbers for
easier calculations. Answers should include the
following.
d  (0.4  1.1)  (1.5  1.5)  (1.9  1.8  0.8)
50. C; 6(ac  2b)  2ac  6(ac)  6(2b)  2ac
 6ac  12b  2ac
 6ac  2ac  12b
 (6ac  2ac)  12b
 (6  2)ac  12b
 8ac  12b

4

3 1

6 1 y2 2

 x(6  3)  y2 6  2

 4  3s  s  3t

3 x

1 2
y
2

3 x  2y2

 6x  3x  6y2  2 y2

 4  3s  3t  s

y2 2

 6 1x2 

 3 1s  2t2  s  4  3 1s2  3 12t2  s


2

1442443 123

 10 p  5 q

19

Chapter 1

66. 4a  3b  4  2  3  5
835
 8  15
 23

51. B; the Commutative Property implies that


6  5  5  6.

Page 36

Maintain Your Skills

52. 5(2  x)  7x  5(2)  5(x)  7x


 10  5x  7x
 10  (5  7)x
 10  12x
 12x  10
53. 3(5  2p)  3(5)  3(2p)
 15  6p
54. 3(a  2b)  3a  3(a)  3(2b)  3a
 3a  6b  3a
 3a  3a  6b
 (3a  3a)  6b
 (3  3)a  6b
 0a  6b
 6b
55. 7m  6(n  m)  7m  6(n)  6(m)
 7m  6n  6m
 7m  6m  6n
 (7m  6m)  6n
 (7  6)m  6n
 13m  6n
56. (d  5)f  2f  (d)(f )  5( f )  2f
 df  5f  2f
 df  (5  2)f
 df  7f
57. t2  2t2  4t  (1  2)t2  4t
 3t2  4t
58. 3(10  5  2)  21  7
 3(10  10)  21  7 Substitution; 5  2  10
 3(0)  21  7
Substitution; 10  10  0
 0  21  7
Multiplicative Property
of Zero; 3  0  0
03
Substitution; 21  7  3
3
Additive Identity;
033
59. 12(5)  6(4)  60  6(4)
 60  24
 36
60. 7(0.2  0.5)  0.6  7(0.7)  0.6
 4.9  0.6
 4.3
61. 8[62  3(2  5)]  8  3  8[62  3(7)]  8  3
 8[36  3(7)]  8  3
 8[36  21]  8  3
 8[15]  8  3
 120  8  3
 15  3
 18
62. 2x  7  2  4  7
63. 6x  12  6  8  12
87
 48  12
 15
 60
64. 5n  14  5  6  14 65. 3n  8  3  7  8
 30  14
 21  8
 16
 13

Chapter 1

Page 36

Practice Quiz 2

1. j; Additive Identity Property, since a  0  a


2. c; Substitution Property of Equality, since
18  7  11
3. i; Commutative Property, since a  b  b  a
4. f; Reflexive Property of Equality, since a  a
5. g; Associative Property, since (a  b)  c 
a  (b  c)
6. d; Multiplicative Identity Property, since
1aa
7. b; Multiplicative Property of 0, since a  0  0
8. a; Distributive Property, since a(b  c)  ab  ac
9. h; Symmetric Property of Equality, since if a  b,
then b  a
10. e; Multiplicative Inverse Property, since
a b
 1
b a

1-7
Page 39

Logical Reasoning
Check for Understanding

1. Sample answer: If it rains, then you get wet.


Hypothesis: it rains
Conclusion: you get wet
2. Sample answer: Counterexamples are used to
disprove a statement.
3. Sample answer: You can use deductive reasoning
to determine whether a hypothesis and its
conclusion are both true or whether one or both
are false.
4. Hypothesis: it is January
Conclusion: it might snow
5. Hypothesis: you play tennis
Conclusion: you run fast
6. Hypothesis: 34  3x  16
Conclusion: x  6
7. Hypothesis: Lance does not have homework
Conclusion: he watches television
If Lance does not have homework, then he
watches television.
8. Hypothesis: a number is divisible by 10
Conclusion: it is divisible by 5
If a number is divisible by 10, then it is divisible
by 5.
9. Hypothesis: a quadrilateral has four right angles
Conclusion: it is a rectangle
If a quadrilateral has four right angles, then it is
a rectangle.
10. The last digit of 10,452 is 2, so the hypothesis is
true. Conclusion: the number is divisible by 2.
Check:
10,452  2  5226
2 divides 10,452.

20

33. The hypothesis is false. If the VCR cost less than


$150, we know Ian will buy one. However, the
hypothesis does not say Ian wont buy a VCR if it
costs $150 or more. Therefore, there is no valid
conclusion.
34. The conditional statement does not mention DVD
players. There is no way to determine if the
hypothesis is true. Therefore, there is no valid
conclusion.
35. The conclusion is true. If the cost of the VCRs is
less than $150, the hypothesis is true, also.
However, if the cost of the VCRs is $150 or more,
the hypothesis is false. There is no way to
determine the cost of the VCRs. Therefore, there
is no valid conclusion.
36. People move to other states.
37. There is a professional team in Canada.
38. Girls can wear blue clothes.
39. Left-handed people can have right-handed
parents.
40. x  2, y  3; Is 2  3 even? 2  3  6 and 6 is
even, but 3 is not even.
?
41. 8 is greater than 7. 42. n  15; 4  15  8 52
2(8)  16
52 52
but 15  15
16  16

11. The conclusion is true. If the last digit of the


number is 2, the hypothesis is true also. However,
if the last digit is an even number other than 2,
the hypothesis is false. There is no way to
determine if the last digit is 2. Therefore, there is
no valid conclusion.
12. The conclusion is true. 946 is divisible by 2.
However, since the last digit is 6, the hypothesis
is false. Therefore, there is no valid conclusion.
13. Anna could have a schedule without a science class.
14. a book that has more than 384 pages
?

12 7 1
11
?
16. x  15; 3  15  7 52
52 52
but 15  15
15. x  1;

17. A;

?
x  1, 12 7 1
11

Pages 4042

Practice and Apply

18. Hypothesis: both parents have red hair


Conclusion: their children have red hair
19. Hypothesis: you are in Hawaii
Conclusion: you are in the tropics
20. Hypothesis: 2n  7
25
Conclusion: n
16
21. Hypothesis: 4(b  9)  68
Conclusion: b  8
22. Hypothesis: a  b
Conclusion: b  a
23. Hypothesis: a  b, and b  c
Conclusion: a  c
24. Hypothesis: it is Monday
Conclusion: the trash is picked up
If it is Monday, then the trash is picked up.
25. Hypothesis: it is after school
Conclusion: Greg will call
If it is after school, then Greg will call.
26. Hypothesis: a triangle has all sides congruent
Conclusion: it is an equilateral triangle
If a triangle has all sides congruent, then it is an
equilateral triangle.
27. Hypothesis: a number is divisible by 9
Conclusion: the sum of its digits is a multiple of 9
If a number is divisible by 9, then the sum of its
digits is a multiple of 9.
28. Hypothesis: x  8
Conclusion: x 2  3x  40
If x  8, then x 2  3x  40.
29. Hypothesis: s
9
Conclusion: 4s  6
42
If s
9, then 4s  6
42.
30. $139 is less than $150, so the hypothesis is true.
Conclusion: Ian will buy a VCR.
31. $99 is less than $150, so the hypothesis is true.
Conclusion: Ian will buy a VCR.
32. The conclusion is false. Ian did not buy a VCR, and
he would have bought one if the cost were less than
$150. Therefore, the VCR cost $150 or more.

43. x  3, y  2;

6
3

1
2

1

11
but

6
3

 1 and

1
2

1

44. Sample answer:


P

45. Sample answer:


R

46. See students work. There will probably be both


examples and counterexamples.
47. Numbers that end in 0, 2, 4, 6, or 8 are in the
divisible by 2 circle. Numbers whose digits have
a sum divisible by 3 are in the divisible by 3
circle. Numbers that end in 0 or 5 are in the
divisible by 5 circle.
48. Sample answer: If a number is divisible by 2 and
3, then it must be a multiple of 6.
49. There are no counterexamples to the conclusions
obtained in Exercises 47 and 48.
50. No; Sample answer: Let a  1 and b  2; then
1 * 2  1  2(2) or 5 and 2 * 1  2  2(1) or 4.
51. You can use if-then statements to help determine
when food is finished cooking. Answers should
include the following.
Hypothesis: you have small, underpopped
kernels
Conclusion: you have not used enough oil in
your pan
If the gelatin is firm and rubbery, then it is
ready to eat. If the water is boiling, lower the
temperature.

21

Chapter 1

52. 8, since 14  8  12  112  12


 100
53. C; # 4 


66. Multiplicative Property of Zero


n  0, since 36  0  0.
67. 5(7)  6  x
35  6  x
41  x
The solution is 41.
68. 7(42)  62  m
7(16)  36  m
112  36  m
76  m
The solution is 76.

43
2
64
2

 32

Page 42

Maintain Your Skills

54. 2x  5y  9x  2x  9x  5y
 (2x  9x)  5y
 (2  9)x  5y
 11x  5y
55. a  9b  6b  a  (9b  6b)
 a  (9  6)b
 a  15b
3

69. p 



56. 4g  5f  8g  4g  8g  5f

134 g  58 g2  25f
3
5
2
 1 4  8 2 g  5f


70.
71.

11
2
g  5f
8
3
2
1 8g  5f

72.

57. 4(5mn  6)  3mn  4(5mn)  4(6)  3mn


 20mn  24  3mn
 20mn  3mn  24
 (20mn  3mn)  24
 (20  3)mn  24
 23mn  24
58. 2(3a  b)  3b  4  2(3a)  2(b)  3b  4
 6a  2b  3b  4
 6a  (2b  3b)  4
 6a  (2  3)b  4
 6a  5b  4
59. 6x2  5x  3(2x2)  7x  6x2  5x  6x2  7x
 6x2  6x2  5x  7x
 (6  6)x2  (5  7)x
 12x2  12x
60. gallons
gallons
used
used
gallons
number
flushing showering
used
of days
toilet and bathing in sink times
123 14243

73.
74.
75.
76.
77.

78.

14243 1442443 14243

61.
62.
63.
64.
65.

Chapter 1

28  2
22  8
28  4
14
7

2
The solution is 2.
The word product implies multiply, so the
expression can be written as 8x4.
Times implies multiply, and decreased by implies
subtract. So the expression can be written as
3n  10.
More than implies add, and quotient implies
divide. So the expression can be written as
12  (a  5).
40%  90  0.4(90)
 36
23%  2500  0.23(2500)
 575
18%  950  0.18(950)
 171
38%  345  0.38(345)
 131.1
42.7%  528  0.427(528)
 225.456
 225.5
67.4%  388  0.674(388)
 261.512
 261.5

1-8

(100 
80
 8)

d
(100  80  8)d  (100)(d)  (80)(d)  (8)(d)
 100d  80d  8d
Two expressions to represent the amount of
water used in d days can be (100  80  8)d and
100d  80d  8d.
Multiplicative Identity Property
n  64, since 1(64)  64.
Reflexive Property
n  7, since 12  7  12  7.
Substitution Property
n  5, since (9  7)5  (2)5.
Multiplicative Inverse Property
1
n  4, since 4  4  1.
Additive Identity Property
n  0, since 0  18  18.

22  113  52

Page 46

Graphs and Functions


Check for Understanding

1. The numbers represent different values. The first


number represents the number on the horizontal
axis and the second represents the number on the
vertical axis.
2. Sample answer: A dependent variable is
determined by the independent variable for a
given function.
3. See students work.
4. Sample answer: Alexis speed decreases as he
rides uphill, then increases as he rides downhill.

22

Height (cm)

5. Just before jumping from a plane, the skydivers


height is constant. After she jumps, her height
decreases until she lands. When she lands the
skydivers height above the ground is zero. Graph
B shows this situation.
6. Time is the independent variable, as it is
unaffected by the height of the object above the
ground. Height is the dependent quantity, as it is
affected by time.
7. The ordered pairs can be determined from the
table. Time is the independent variable, and
the height above the ground is the dependent
variable. So, the ordered pairs are (0, 500),
(0.2, 480), (0.4, 422), (0.6, 324), (0.8, 186), and
(1, 10).
500
8.

40
35
30
Cost

450
400
350
300
250
200
150
100
50
0

14. The ordered pairs can be determined from the


table. The time parked is the independent
variable, and the cost is the dependent variable.
So, the ordered pairs are (0, 0), (1, 1), (2, 2),
(3, 2), (4, 4), (5, 4), (6, 4) (7, 5), (8, 5), (9, 5),
(10, 5), (11, 5), (12, 30), (13, 30), (14, 30), (15, 30),
(16, 30), (17, 30), (18, 30), (19, 30), (20, 30),
(21, 30), (22, 30), (23, 30), (24, 30), (25, 45),
(26, 45), (27, 45), (28, 45), (29, 45), (30, 45),
(31, 45), (32, 45), (33, 45), (34, 45), (35, 45), and
(36, 45).
15.
45

25
20
15
10
5
0

0.2

0.4 0.6
Time (s)

0.8

8 12 16 20 24 28 32 36
Time

16. From 7:00 A.M. Monday to 7:00 A.M. Tuesday is


24 hours. From 7:00 A.M. Tuesday to 7:00 P.M.
Tuesday is 12 hours. From 7:00 P.M. Tuesday to
9:00 P.M. Tuesday is 2 hours. Therefore, from
7:00 A.M. Monday to 9:00 P.M. Tuesday is 24  12
 2  38 hours. The cost for the first 24 hours is
$30. The cost for the next 14 hours is $15. Thus,
the cost of parking from 7:00 A.M. Monday to
9:00 P.M. Tuesday is 30  15 or $45.
17. The number of sides of the polygon is the
independent variable as it is unaffected by the
sum of the interior angles, and the sum of the
interior angles is the dependent variable as it is
affected by the number of sides of the polygon.
18.

1.0

Height

9. Paul releases the ball above the ground. The


height of the ball then decreases as it approaches
the height at which the catcher catches the ball.

Time

900

Practice and Apply

Sums

Pages 4648

10. Michelle gets a fever and takes some medicine.


After a while her temperature comes down, then
slowly begins to go up again.
11. Rashaads account is increasing as he makes
deposits and earns interest. Then he pays some
bills. He then makes some deposits and earns
interest, and so on.
12. As it moves along, a radio-controlled car has a
constant speed. When it hits the wall, its speed
is zero. Graph C shows this situation.
13. A persons income starts at a certain level, then,
in general, their income increases. This is
represented by starting at a certain height, then
gradually increasing. Graph B shows this
situation.

720
540
360
180
0

4 5
Sides

19. If we look for a pattern in the numbers 180, 360,


540, 720, and 900, we see that the numbers are
all multiples of 180. In fact:
180  1  180
360  2  180
540  3  180
720  4  180
900  5  180
Thus, the next three numbers would be 6  180 
1080, 7  180  1260, and 8  180  1440.
Therefore, we would predict that the sum of the
interior angles for an octagon is 1080, for a
nonagon is 1260, and for a decagon is 1440.

23

Chapter 1

20. A cars value decreases as it gets older, but if it is


taken care of, the value of the car again
increases.

24. B; The value 4 on the horizontal axis corresponds


to the highest point on the curve.
25. A;
cost for
charge for
minus
profit is
each CD
each CD
123 123 14243

123 14243


(2.50
0.35)

number
cost of
times
minus
equipment
of CDs
123 14243 14243 1442443

Value

Years



n
or p  2.15n  875

21. When the block of ice is removed from the


freezer the temperature starts to increase. The
temperature increases until it approaches the
room temperature, at which time the
temperature gradually becomes constant.

Page 48

Temperature (F )

22a. At each value of the independent variable the


dependent variable is 23 units greater. The
following table shows this relationship.
Lisas Age 5 10 15 20 25 30 35 40
Mallorys 28 33 38 43 48 53 58 63
Age
The ordered pairs for this data are (5, 28),
(10, 33), (15, 38), (20, 43), (25, 48), (30, 53),
(35, 58), and (40, 63). Graph the ordered pairs.
Then draw a line through the points.
90

Mallorys Age

80
70

U.S. Commercial Radio Stations


by Format, 2000

60
50
40
30

Number of Stations

20
10
5 10 15 20 25 30 35 40 45
Lisas Age

2400
2200
2000
1800
1600
1400
1200
1000
800
600
0

24

ck

Format

Ro

Co

s
ie
ld
O
k
al
s/T
w
y
Ne
ar
t or
ul mp
Ad te
n
Co ry
t
un

22b. We know Mallory is 23 years older than Lisa.


So, for Mallory to be twice as old as Lisa, Lisa
must be 23 years old. The point on the graph
that corresponds to this situation is (23, 46).
23. Real-world data can be recorded and visualized
in a graph and by expressing an event as a
function of another event. Answers should
include the following.
A graph gives you a visual representation of
the situation, which is easier to analyze and
evaluate.
During the first 24 hours, blood flow to the
brain decreases to 50% at the moment of the
injury and gradually increases to about 60%.
Significant improvement occurs during the
first two days.

Chapter 1

Maintain Your Skills

26. Hypothesis: you use a computer


Conclusion: you can send e-mail
27. Hypothesis: a shopper has 9 or fewer items
Conclusion: the shopper can use the express lane
28. ab(a  b)  (ab)a  (ab)b
Distributive Prop.
 a(ab)  (ab)b
Commutative ()
 (a  a)b  a(b  b) Associative ()
 a2b  ab2
Substitution
29. Substitution Property
n  3, since (12  9)(4)  (3)(4).
30. Multiplicative Property of Zero
n  0, since 7(0)  0.
31. Multiplicative Identity Property
n  1, since (1)(87)  87.
32. Step 1 Draw a horizontal axis and a vertical
axis. Label the axes. Add a title.
Step 2 Draw a bar to represent each category. The
vertical scale is the number of radio
stations using each format. The horizontal
scale identifies the formats used.

Time

875

Page 49

Algebra Activity
(Follow-Up of Lesson 1-8)

1-9

1. Sample answer: In 1900 there were 15,503,000


students, and in 1920 there were 21,578,000
students. Therefore, we could estimate that there

Pages 5354

were
 18,540,500, or about
18,540,000 students in 1910. In 1970
there were 45,550,000 students, and in 1980
there were 41,651,000 students. Therefore,
we could estimate that there were
45,550,000  41,651,000
2

4.

5.

 43,600,500 or
about 43,600,000 students in 1975.
Sample answer: 55,000,000. By looking at general
trends of the graph, we see that it is increasing,
and the amount of students will be around
55,000,000 in 2020.
Sample answer: For Exercise 1 we averaged the
enrollments for 1900 and 1920 and then for 1970
and 1980. For Exercise 2 we calculated the
increase in students per year from 1900 to 1980,
then added 40 times that amount to the 1980
enrollment.
If the U.S. population does not increase as
quickly as in the past, then the number of
students may be too high.
The year is the independent variable since it is
unaffected by the number of students per
computer, and the number of students per
computer is the dependent variable since it is
affected by the year. The ordered pairs can be
determined from the table with x representing
the number of years since 1984. The orderd pairs
are (0, 125), (1, 75), (2, 50), (3, 37), (4, 32), (5, 25),
(6, 22), (7, 20), (8, 18), (9, 16), (10, 14), (11, 10.5),
(12, 10), (13, 7.8), (14, 6.1), (15, 5.7). Graph the
ordered pairs.

Students per Computer

3.

14243

123

8940


8940 of the students were from Germany.
7. The percentage representing the number of
students from Canada is 0.15%. The percentage
representing the number of students from the
United Kingdom is 0.05%. So, find 0.15  0.05 or
0.1% of 14.9 million.
0.1% of
14,900,00 equals 14,900
123
1442443
14243
0.0006

14,900,000

14243

123

8.
9.

130
120
110
100
90
80
70
60
50
40
30
20
10
0

Check for Understanding

1. A circle graph compares parts to the whole. A bar


graph compares different catagories of data. A
line graph shows changes in data over time.
2. See students work.
3. Sample answer: The percentages of the data do
not total 100.
4. The bar for the number of schools participating in
basketball shows 321 and the bar for the number
of schools participating in golf shows 283. So,
there were 321 283 or 38 more schools
participating in basketball than in golf.
5. The bar showing the least amount is for tennis.
Therefore, of the sports listed, tennis is offered at
the fewest schools.
6. The percentage representing the number of
students from Germany is 0.06%. So, find 0.06%
of 14.9 million.
0.06%
14,900,00 equals 8940.
of 1442443
14243
123

15,503,000  21,578,000
2

2.

Statistics: Analyzing Data by


Using Tables and Graphs

10.
11.

0.001  14,900,000
14,900

14,900 more students were from Canada than
from the United Kingdom.
No; the data do not represent a whole set.
A bar graph would be more appropriate, since a
bar graph is used to compare similar data in the
same category.
Sample answer: The vertical axis scale shows
only partial intervals.
The vertical axis needs to begin at 0.

Pages 5455

Practice and Apply

film
stock plus processing plus prep for telecine plus
12. 1
442443 123 1442443 123 144424443 123
1 2 3 4 5 6 7 8 9 10 11 12 13 14 15
Years Since 1984

3110.40

621.00

60.00

telecine plus tape stock equals total cost.

14243 123 1442443 14


424
43 1442443

A prediction is 1 student per computer because it


does not seem likely that schools would have
more computers than students.

1000.00

73.20

4864.60

The total cost is $4864.60.


13. original

backup
total cost of
tape stock plus tape stock equals digital video .
1442443 123 1442443 14
424
43 144424443
10.00
10.00
20.00



The total cost of using digital video is $20.00, and


from Exercise 12 the total cost of using
35-millimeter film is $4864.60.
4864.60  20.00  243.23
The cost of 35-millimeter film is about 250 times
as great.

25

Chapter 1

Page 55

14. The section of the graph representing books


purchased in the spring is 19%, so find 19% of
25 million.
19%
25,000,000 equals 4,750,000
1
23 of
{ 14243 123 14243
0.19  25,000,000 
4,750,000

Heart rate

About 5 million books were purchased in the spring.


15. The section of the graph representing books
purchased in the summer is 15%, so find 15% of
15,000.
15%
15,000 equals 2250.
1
23 of
{ 14243 123 1
23
0.15  15,000

2250

Distance (mi)

23. 4x  5  42
4x  47

She should expect to sell about 2250 books.


16. The vertical axis is extended and does not begin
at 0. It gives the impression that the number of
cable television systems is decreasing rapidly.
17. Yes, the graph is misleading because the sum of
the percentages is not 100. To fix the graph, each
section must be drawn accurately and another
section that represents other toppings should be
added.
18a. To show little increase, bunch the values on the
vertical scale closer together.

x  11 4
Thus, x  12 is a sample counterexample.
24. Sample answer:
3

x  2;

but

Percentage of
U.S. Households

3 ?
7 1
2
3
7 1
2
1
3
 3
2
2

25. a rectangle with length 6 inches and width


2 inches
6  6  2  2  16
16  16
However, each side is not 4 inches long.
26. 7a  5b  3b  3a  7a  3a  5b  3b
 (7a  3a)  (5b  3b)
 (7  3)a  (5  3)b
 10a  8b
27. 4x2  9x  2x2  x  4x2  2x2  9x  x
 (4x2  2x2)  (9x  x)
 (4  2)x2  (9  1)x
 6x2  10x

Color Television Ownership,


19802000
100
90
80
70
60
80 85 90 95 00
Year

18b. To show rapid increase spread the values on the


vertical scale further apart.
Color Television Ownership,
19802000
Percentage of
U.S. Households

Maintain Your Skills

22. Pedros heart rate increases as he exercises, and


continues to increase until he is done sprinting.
His heart rate decreases during his last walk
until it returns to a normal rate.

100

112 n  13 n2  123 m  12 m2
1
1
2
1
 12  3 2n  13  2 2m

95
90
85

 6n  6m

80

 6n  1 6m

80 85 90 95 00
Year

18c. See students graphs and explanations.


19. Tables and graphs provide an organized and
quick way to examine data. Answers should
include the following.
Examine the existing pattern and use it to
continue a graph to the future.
Make sure the scale begins at zero and is
consistent. Circle graphs should have all
percents total 100%. The right kind of graph
should be used for the given data.
20. C; From the second to the third day the
temperature increase was about 8F.
21. C; A line graph shows changes in data over time.

Page 56

Spreadsheet Investigation
(Follow-Up of Lesson 1-9)

1. Enter the data in a spreadsheet. Use Column A


for the years and Column B for the sales. Select
the data to be included in the graph. Then use
the graph tool to create a line graph.
Snowmobile Sales
Sales (millions)

1200
1000
800
600
400
200
0
1990

Chapter 1

28. 2n  3m  2m  3n  2n  3n  3m  2m

26

1992

1994
Year

1996

1998

21. 3  2  4  3  8
 11

2. Enter the data in a spreadsheet as in Exercise 1.


Select the data to be included in your graph.
Then use the graph tool to create a bar graph.

22.

Snowmobile Sales

110  62
8

2

Sales (millions)

1200
1000

23. 18  42  7  18  16  7
27
9
24. 8(2  5)  6  8(7)  6
 56  6
 50
25. 4(11  7)  9  8  4(18)  9  8
 72  9  8
 72  72
0
26. 288 [3(9  3)]  288 [3(12)]
 288 36
8
27. 16 2  5  3 6  8  5  3 6
 40  3 6
 120 6
 20
28. 6(43  22)  6(64  22)
 6(64  4)
 6(68)
 408

800
600
400
200
0
1990

1992

1994
Year

1996

1998

3. Yes; you can change the scales to begin at values


other than zero, or change the intervals on the
scale to be misleading.

Chapter 1 Study Guide and Review


Page 57
1.
2.
3.
4.
5.
6.
7.
8.
9.
10.

8

Vocabulary and Concept Check

a; Additive Identity Property


e; Multiplicative Identity Property
g; Multiplicative Property of Zero
f; Multiplicative Inverse Property
h; Reflexive Property
j; Symmetric Property
i; Substitution Property
k; Transitive Property
b; Distributive Property
d; Associative Property

29. 13  12 3 

14  62
15  22

 33 




Pages 5762

Lesson-by-Lesson Review

11. a number x to the fifth power

30. t2  3y 




14444444244444443
x5

The algebraic expression is x5.


12. five times a number x squared
123 1444
4424
44443
123

ty
x

14  62

15  22
10
 15  22
10
33  10
10
27  10

33

 27  1
 26
42  3  2
31. xty3  3  4  23
16  3  2
3 4 8
16  6
 12  8
22
 96

4  2
3
8
3
2
23


x2
5
The algebraic expression is 5x2.
twenty-one
13. a number x sum of 1442443

32.

21
x

The algebraic expression is x  21.
14. twice a number x difference of eight

33. x  t2  y2  3  42  22
 3  16  22
 3  16  4
 19  4
 23
34. 3ty  x2  3  4  2  32
3 4 29
 12  2  9
 24  9
 15
35. 8(x  y)2  2t  8(3  2)2  2  4
 8(1)2  2  4
8 12 4
82 4
88
 16




1442443 14243

14444244443 144
424
443 14243

2x
8

The algebraic expression is 2x  8.
15. 33  3  3  3
16. 25  2  2  2  2  2
 27
 32
17. 54  5  5  5  5
 625
18. the product of two and a number p squared
19. the product of three and a number m to the fifth
power
20. the sum of one half and two

27

Chapter 1

36. x  22  13
9
The solution is 9.
38. m 


14  28
4  3
42
7

6
The solution is 6.
42. b 



4 2  4  5 15 S 13  15

714  32

5 2  5  5 15 S 15 15

18
9

6
84
6

6 2  6  5 15 S 17 15

4
5
6
7
8

x27
?

4  2 7 7S6 7
?

5  2 7 7S7 7
?

6  2 7 7S8 7 7
?

7  2 7 7S9 7 7
?

8  2 7 7 S 10 7 7

7 2  7  5 15 S 19 15

96 6
8 2
16
4

10  x  7
?

10  4 6 7 S 6 6 7
?

10  5 6 7 S 5 6 7

10  6 6 7 S 4 6 7

10  7 6 7 S 3 6 7

10  8 6 7 S 2 6 7

?
?
?

true

8 2  8  5 15 S 21 15

true

The solution set for 2x  5 15 is {5, 6, 7, 8}.


48. 2[3 (19  42)]
 2[3 (19  16)] Substitution; 42  16
 2[3 3]
Substitution; 19  16  3
2 1
Substitution; 3 3  1
2
Multiplicative Identity;
2 12

6172  2132
2

4  6122
42  6
16  12
36
4

49.

1
2

 2  2[2  3  1]
1

 2  2  2[6  1] Substitution; 2  3  6
1

 22  25
125

Substitution; 6  1  5
Multiplicative Inverse;
21

1
2

Substitution; 2  5  10

 1  10

true

 11Substitution; 1 + 10  11
50. 42  22  (4  2)
 42  22  2 Substitution; 4  2  2
 16  22  2 Substitution; 42  16
 16  4  2 Substitution; 22  4
 12  2
Substitution; 16  4  12
 10
Substitution; 12  2  10
51. 1.2  0.05  23
 1.2  0.05  8 Substitution; 23  8
 1.15  8
Substitution; 1.2  0.05 
1.15
 9.15
Substitution; 1.15  8  9.15
52. (7  2)(5)  52
 5(5)  52 Substitution; 7  2  5
 5(5)  25 Substitution; 52  25
 25  25 Substitution; 5  5  25
0
Substitution ()

true

53. 3(4 4) 2  4 (8)

True or False?
false
false
true
true
true

The solution set for x  2


7 is {6, 7, 8}.
46. Replace x in 10  x 7 with each value in the
replacement set.
x

true

 14
9
The solution is 14.
The solution is 9.
44. y  5[2(4)  13]
 5[8  1]
 5[7]
 35
The solution is 35.
45. Replace x in x  2
7 with each value in the
replacement set.
x

true

4
The solution is 4.
43. x 

18 3
71122

false

2
The solution is 2.
41. n 

True or False?

39. x  12  3


2x  5  15

21  3

64  4
17
68
17

4
The solution is 4.
40. a 

47. Replace x in 2x  5 15 with each value in the


replacement set.

37. y  4  32
49
 13
The solution is 13.

True or False?

true

 3(1) 2  4 (8) Substitution; 4 4  1

true

31
3

true

The solution set for 10  x 7 is {4, 5, 6, 7, 8}.

1
182
4

1
(8)
4

32

Substitution; 12  1
Multiplicative Identity;
313
Substitution;

1
4

82

1
Substitution; 3  2  1
54. 2(4  7)  2(4)  2(7) 55. 8(15  6)  8(15)  8(6)
 8  14
 120  48
 22
 72

Chapter 1

28

56. 4(x  1)  4(x)  4(1)


 4x  4
57. 3

1
3

p 3

72. five times the


decreased by 2x
sum of x and y 1
44424443 123
144424443

1 2  31 p2


2x
5(x  y)
5(x  y)  2x  5(x)  5(y)  2x Distributive
Property
 5x  2x  5y
Commutative
()
 3x  5y
Substitution
73. twice the product
the product
increased by of p and q
of p and q
144424443

1
3

 1  3p
58. 6(a  b)  6(a)  6(b)
 6a  6b
59. 8(3x  7y)  8(3x)  8(7y)
 24x  56y
60. 4a  9a  (4  9)a
 13a
61. There are no like terms.
4np  7mp is simplified.
62. 3w  w  4v  3v  (3w  w)  (4v  3v)
 (3  1)w  (4  3)v
 2w  1v
 2w  v
63. 3m  5m  12n  4n  (3m  5m)  (12n  4n)
 (3  5)m  (12  4)n
 8m  8n
64. 2p(1  16r)  2p(1)  2p(16r)
 2p  32pr
65. 9y  3y  5x  (9y  3y)  5x
 (9  3)y  5x
 12y  5x
66. 3x  4y  2x  3x  2x  4y
 (3x  2x)  4y
 (3  2)x  4y
 5x  4y
2
67. 7w  w  2w2  7w2  2w2  w
 (7w2  2w2)  w
 (7  2)w2  w
 9w2  w
1

14444244443

1442443


5( p  q)
pq
2pq  pq  (2  1)pq Distributive Property
 3pq
Substitution
74.
the sum of eight
six times a plus times b and twice a
1442443 123 1444442444443
6a

(8b  2a)
6a  8b  2a
 6a  2a  8b Commutative ()
 (6a  2a)  8b Associative ()
 (6  2)a  8b Distributive Property
 8a  8b
Substitution
75. three times
the sum of x squared
the
square
of
x
and seven times x
plus
144424443 123 1444442444443
3  x2
(x2  7  x)

3x2  x2  7x
 (3x2  x2)  7x Associative ()
 (3  1)x2  7x Distributive Property
 4x2  7x
Substitution
76. Hypothesis: it is 7:30 A.M.
Conclusion: school begins
If it is 7:30 A.M., then school begins.
77. Hypothesis: a figure is a triangle
Conclusion: it has three sides
If a figure is a triangle, then it has three sides.

1 1 1 2
1
1
 13 2  2 2m  n

68. 3 2m  2m  n  3 2m  2m  n

78. x  13, y  12;

13 7 12
13 7 12
but 21132 31122

 4m  n
69. 6a  5b  2c  8b  6a  5b  8b  2c
 6a  (5b  8b)  2c
 6a  (5  8)b  2c
 6a  13b  2c
70. 3(2  3x)  21x  3(2)  3(3x)  21x
 6  9x  21x
 6  (9x  21x)
 6  (9  21)x
 6  30x
 30x  6
71. 6(2n  4)  5n  6(2n)  6(4)  5n
 12n  24  5n
 12n  5n  24
 (12n  5n)  24
 (12  5)n  24
 17n  24

79. a  15, b  1, c  12;

15 7 1 and 15 7 12
15 7 1 and 15 7 12
but 1 12
80. When taking off, the airplane has zero altitude.
Then the plane gains altitude. The planes
altitude remains constant as it flies around. Then
the altitude decreases until the plane lands.
Graph C shows this situation.

29

Chapter 1

7 difference of a number x squared


5. {
44443
144424443 14444424
7
x2

The algebraic expression is 7  x2.
6. 5(9  3)  3  4  5(12)  3  4
 60  3  4
 60  12
 48
7. 12  6 3  2 8  72 3  2 8
 24  2 8
 48 8
6
8. a2b  c  22  5  3
4 53
 20  3
 23
9. (cd)3  (3  1)3
10. (a  d )c  (2  1)3
 33
 (3)3
 27
9
11. y  (4.5  0.8)  3.2 12. 42  3(4  2)  x
y  5.3  3.2
42  3(2)  x
y  2.1
16  3(2)  x
The solution is 2.1.
16  6  x
10  x
The solution is 10.

Helium (cm3 )

81. The amount of helium in a balloon increases from


zero at a steady rate until it bursts. When the
balloon bursts, the amount of helium in the
balloon is again zero.

Time

82. Construct a table with 2 rows. The first row is


labeled Earth Years and the second row is labeled
Mars Years. Since one year on Earth is about 0.54
years on Mars, pick some value for Earth Years and
multiply by 0.54 to get values for Mars Years.
Earth Years

10

15

20

25

Mars Years

2.7

5.4

8.1

10.8

13.5

83.

50

Mars Years

40
30
20

13.

10

10

20 30 40
Earth Years

50

84. The section of the graph representing students


who chose the amusement park is 45% of the
circle, so find 45% of 120.
45%
120
equal 54.
{ 1
23 123
{
1
23 of

n
n
n
7
3

or 2 3.

14. 32  2  (2  2)
 9  2  (2  2) Substitution; 32  9
920
Substitution; 2  2  0
70
Substitution; 9  2  7
7
Additive Identity; 7  0  7
15. (2  2  3)  22  32
 (4  3)  22  32 Substitution; 2  2  4
 1  22  32
Substitution; 4  3  1
 1  4  32
Substitution; 22  4
149
Substitution; 32  9
59
Substitution; 1  4  5
 14
Substitution; 5  9  14
16. 2m  3m  (2m  3m)
 (2  3)m
 5m
17. 4x  2y  2x  y  4x  2x  2y  y
 (4x  2x)  (2y  y)
 (4  2)x  (2  1)y
 2x  3y
18. 3(2a  b)  5a  4b  3(2a)  3(b)  5a  4b
 6a  3b  5a  4b
 6a  5a  3b  4b
 (6a  5a)  (3b  4b)
 (6  5)a  (3  4)b
 1a  7b
 a  7b

Chapter 1 Practice Test


Page 63
d; Reflexive Property of Equality
a; Substitution Property of Equality
c; Transitive Property of Equality
a number x sum of 13
1442443 14243 123

13
x

The algebraic expression is x  13.

Chapter 1

n

The solution is

0.45  120
 54
54 students chose the amusement park.
85. The section of the graph representing students
who chose the amusement park is 45% of the
circle. The section of the graph representing
students who chose the water park is 25%. So,
find 45 25 or 20% of 180.
20%
180 equal 36.
1
23 of
{ 1
23 123 {
0.2%  180
 36
There were 36 more students who chose the
amusement park than the water park.

1.
2.
3.
4.

23  13
2  1
8  1
3
7
3
1
23

30

2. C; 2    r  2  3.14  4
 6.28  4
 25.12
3. B; There are about 50 weeks in a year.
80  50  4000

19. Running for 15 minutes does not mean you can


run for a few hours.
20. 2x  3 9
2x 12
x 6
Thus, x  6 is a counterexample.
21. When a basketball is shot, its height starts above
the ground. Then it increases until the basketball
reaches its maximum height. The height of the
ball decreases until it gets to the basket, at which
time it falls back to the ground.

4
1250
48
2
5. D; charge
number
cost of replaced
plus
per hour times
of
hours
parts
14243 123 14243 123 144424443
4. A;

Height



($36
h)
$85
or (36  h)  85
6. D; 3(2x  3)  2(x  1)  3(2x)  3(3)  2(x)  2(1)
 6x  9  2x  2
 6x  2x  9  2
 (6x  2x)  (9  2)
 (6  2)x  11
 8x  11
7. B; 3 is a positive integer, but 9 is not divisible by 2.
8. B; The section of the graph representing foreignborn people from Asia is 25.2% of the circle.
The section of the graph representing foreignborn people from Central America is 34.5% of
the circle. 25.2  34.5

Time

Height

22. The nickels height is at a maximum when it is


dropped. Its height decreases until it hits the
stack of pennies. The height of the nickel
increases as it bounces off the pennies.

9. Five eighths of the students are girls, so 1  8 or


3
3
of the students are boys. Find 8 of 32.
8
3
of 32
8 { {
{
3
 32
8

Time

23. The section of the graph representing students


who chose chocolate is 62% of the circle. The
section of the graph representing students who
chose vanilla is 32% of the circle. So, find 62 32
or 30% of 200.
30%
200 equal 60.
1
23 of
{ 12
3 123 {
0.3  200
 60
There were 60 more students who chose chocolate
than vanilla.
24. The section of the graph representing students
who chose chocolate is 62% of the circle. The
section of the graph representing students who
chose vanilla is 32% of the circle. So, 62  32 or
94% of the students chose either chocolate or
vanilla.
25. D; 2 is a prime number, but is not odd.

There are 12 boys in the class.


10.

cost of
the books

plus

tax

[ (8.99  13.99)

0.06(8.99  13.99) ]

$25 minus

123 14243 144424443 123 14444244443

25

25
 [(8.99  13.99)  0.06 (8.99  13.99)]
25  [(8.99  13.99)  0.06 (8.99  13.99)]
 25  [22.98  0.06(22.98)]
 25  [22.98  1.38]
 25  24.36
 0.64
She should receive $0.64 in change.
11. The bars with the least difference in height are
representing 1999. Therefore, 1999 was the year
with the least difference in home runs.
12.
15% of 80 equals 1
12. B;123
23
123
123

1
424
3

12

 80
12.5
of 50 equals
123 123 123 1
424
3 123
12.5
0.25  50

12 12.5
0.15
25%

Chapter 1 Standardized Test Practice


Pages 6465

equal 12.
123 {
 12

1. B; 1  4  800  200  4  800  200


 200  200
 40,000

31

Chapter 1

13. A;

10
2
3

15. C;

 10 3

1
1a
4

 10  2


10 3

1 2

10  3
12

53
11
15
1

 15
15 7 15
14. B; Replace x in 2x  1 2x  1 with each value
in {1, 2, 3, 4, 5}.
2x  1  2x  1

2  1  1 6 2  1  1S1 6 3

?
?

2  2  1 6 2  2  1S3 6 5
?

2  3  1 6 2  3  1S5 6 7
?

2  4  1 6 2  4  1S7 6 9

2  5  1 6 2  5  1 S 9 6 11

ac  bc
4

True or
False?
true
true
true

Hours

true

17c. This graph shows that only the old pump was
pumping at first, and then after some hours the
new pump started pumping also.

true

Notice that the expression 2x  1 is always 2


units greater than the expression 2x  1.

Chapter 1

Gallons

 4 1ac  bc2

16. C; (26  39)  (39  13)  (26  39)  (13  39)


 (26  13)  39
 39  39
 392
17a. Sample answer: The new pump pumps many
more gallons per hour than the old pump. The
new pump pumps about twice as many gallons
per hour as the old pump.
17b. The gallons pumped per hour by both pumps at
the same time is the sum of the number of gallons
pumped per hour by each pump individually.

 b2c  4 3a 1c2  b 1c2 4

32

Chapter 2 Real Numbers


Page 67
1.

5.

Getting Started

2.2
0.16
2.36

1
4

2. 0 12 1
1 3 .4
4.5
8.9

3.

6.

 3  12  12

19. mean 
4.

33

6.4
8.8
512
51 2
56.3 2
1
2

11

1
1

 10  4  10

8.

8
There are six numbers in the set. When ordered
from least to greatest, the third number is 7 and
7  7
the fourth is 7. So, the median is 2 or 7.
7 appears two times, and the other numbers of
the set appear only once. So, the mode is 7.

6

5
4

366

 12
7.

18
4.25
76
.
50


4 25
34 00
34 00
0

4
9

 3  91

3
8

4
3

or

14. b  3 


36
4

 0.3(8)
 2.4
17. mean 



15. xy  7  0.3
 2.1

2  4  7  9  12  15
6
49
6
1
86




145 22  45  45
16

 25

2-1

Rational Numbers on the


Number Line

Page 70

Check for Understanding

6.

5432 1 0 1 2 3 4 5 6 7 8

7.
4 3 2 1

8.

There are six numbers in the set. When ordered


from least to greatest, the third number is 7 and
7  9
the fourth is 9. So, the median is 2 or 8. Every
number in the set appears only once. So, there is
no mode.
18. mean 

23.

123 22  23  23

1. Let a be any integer. Since a can always be


a
written in the form 1 , a is also a rational number.
The statement is always true.
2. Sample answer: Absolute value is how far from
zero a number is.
3. Sample answer: describing distances in a given
direction such as north versus south, or left
versus right
4. The dots indicate each point on the graph. The
coordinates are {2, 1, 2, 5}.
5. The bold arrow on the left means that the graph
continues indefinitely in that direction. The
11
9
7
5
3
coordinates are . . .,  2 , 2, 2, 2, 2 .

16. y(a  b)  0.3 2  4

22.

 12

3

1
34

21. 0.92  0.9  0.9


 0.81

9

9
1
4

20. 112  11  11
 121
1
13

9. 3a  2  3  2  2
10. 2x  5  2  7  5
62
 14  5
4
 19
11. 8( y  2.4)  8(0.3  2.4)
 8(2.7)
 21.6
1
1
1
12. 4(b  2)  4 4  2
13. a   2 
2
2

1 2
1
8
 414  4 2
9
 414 2

7  19  2  7  4  9
6
48
6

1

1
2

0 1 2
4 5

5
3

9.
9 8 76 5 4 3 2 1 0 1

10. 2 is two units from zero in the negative


direction.
02 0  2
11. 18 is eighteen units from zero in the positive
direction.
018 0  18
12. 2.5 is two and one half units from zero in the
positive direction.
02.5 0  2.5

23  23  23  12  12  14
6
107
6
5
176

There are six numbers in the set. When ordered


from least to greatest, the third number is 14 and
14  23
1
the fourth is 23. So, the median is
or 182.
2
The number 23 appears three times, and the
other numbers of the set appear only once. So, the
mode is 23.

13. 6 is five-sixths unit from zero in the negative


direction.

0 56 0  56

33

Chapter 2

14. 57  0x  34 0  57  018  34 0
 57  052 0
 57  52
5
15. 19  021  y 0  19  021  4 0
 19  017 0
 19  17
 36
16. 0z 0  0.26  00.76 0  0.26
 0.76  0.26
 0.50
17.
Rational Numbers
Integers

1
2

Whole
Numbers
0

40 4 2
53
3
13

Pages 7172

34. 38 is thirty-eight units from zero in the negative


direction.
038 0  38
35. 10 is ten units from zero in the positive direction.
010 0  10
36. 97 is ninety-seven units from zero in the positive
direction.
097 0  97
37. 61 is sixty-one units from zero in the negative
direction.
061 0  61
38. 3.9 is three and nine tenths units from zero in the
positive direction.
03.9 0  3.9
39. 6.8 is six and eight tenths units from zero in the
negative direction.
06.8 0  6.8

1.25
2 1 5
 
3 5 2
0.33 2.98
49.98

23

40. 56 is twenty-three fifty-sixths unit from zero in


the negative direction.

Practice and Apply

41.

18. The dots indicate each point on the graph. The


coordinates are {4, 2, 0, 2, 4}.
19. The dots indicate each point on the graph. The
coordinates are {7, 6, 5, 3, 2}.
20. The bold arrow on the right means that the graph
continues indefinitely in that direction. The
coordinates are {2, 3, 4, 5, 6, . . .}.
21. The bold arrow on the left means that the graph
continues indefinitely in that direction. The
coordinates are {. . ., 0, 0.2, 0.4, 0.6, 0.8}.
22. The dots indicate each point on the graph. The
5
2
coordinates are 2, 3, 1, 3, 1 .

5
5

42.

43.
44.
45.

23. The dots indicate each point on the graph. The


1 4 7 8
coordinates are 5, 5, 5, 5, 2 .
24.
5 4 3 2 1

46.

25.
1 0 1 2 3 4 5 6 7 8 9 10

26.
7

6

5

4

3

2

47.

27.
2 1

28.
9 8 7 6 5 4 3 2

48.

29.
7 6 5 4 3 2 1

30.
1  2  1
3

1
3

2
3

1 11 12 2
3

49.

31.
4 3 2 1

32.
9 8 7 6 5 4 3 2 1 0 1

50.

33.
6 4 2

Chapter 2

35
80

is thirty-five eightieths unit from zero in the


positive direction.

0 3580 0  3580

108 6 4 2 0 2 4 6 8 10

0 2356 0  2356

10

34

Therefore, the percents of change from least to


greatest are 10.6, 2.9, 1.4, 0.2, 1.7, 4.3, 4.7,
5.3, 8.5, and 10.0.
Philadelphia, PA; Sample answer: It had the
greatest absolute value.
Wayne, MI; Sample answer: It had the least
absolute value.
48  0x  5 0  48  012  5 0
 48  07 0
 48  7
 55
25  017  x 0  25  017  12 0
 25  029 0
 25  29
 54
017  a 0  23  017  6 0  23
 011 0  23
 11  23
 34
043  4a 0  51  043  4  6 0  51
 043  24 0  51
 019 0  51
 19  51
 70
0z 0  13  4  05 0  13  4
 5  13  4
 18  4
 14
28  13  0z 0  28  13  05 0
 28  13  5
 15  5
 20

51. 6.5  08.4  y 0  6.5  08.4  3.2 0


 6.5  05.2 0
 6.5  5.2
 1.3
52. 7.4  0y  2.6 0  7.4  03.2  2.6 0
 7.4  00.6 0
 7.4  0.6
8
53.

1
6

60. Sample answer: You can plot the data on a


number line to visualize their relationship.
Answers should include the following.
Determine the least and greatest values of the
data, and use those as the endpoints of the line.
Find the absolute value of each number.
61. D; 08 0  2  8  2
6
62. C; Zero is a whole number, but zero is not a
natural number.

0 23  127 0
1
1
 6  0 12 0
1

 b  12  6 
1

 6  12

Page 72

4

2 0 5 0 123  12 2  0 56 0
2
1
5
 13  2 2  6

54. b  2  6 

66
1

3

0 54  1 0  25
1
2
 040  5

55. 0 c  1 0  5 

45
13

 20

2 0 5 0 1 12
5
1
 4  12  2 2

56. 0c 0  2  2  4  2  2
1

15
4

or 3 4

42


57.  0x 0  1  0x 0
Therefore, 0x 0   0x 0 can be thought of as what
number when multiplied by 1 is equal to itself?
The number 0 is the only number that satisfies
that characteristic. i.e. 1  0  0. So, 0x 0  0.
What number is zero units from zero? x  0, or
the solution is 0.
58.
10

10

Maintain Your Skills

63. The graph shows that Mr. Michaelss sales were


less than 10 cars for each month except
December. In December Mr. Michaels sold
12 cars. Therefore, his greatest sales happened
in December.
64. The graph shows that Mr. Michaelss change in
sales between any two consecutive months was
always less than 7 cars except between November
and December when it was 8 cars. Therefore, the
greatest change in sales occurred between
November and December.
65. The graph shows that Mr. Michaels sold 3 cars in
February, July, and October.
66. The volume starts high, decreases when she turns
it down, remains constant while she is on the
phone, then increases to its starting volume when
she gets off the phone.

20

Volume

Time

67. 8x  2y  x  8x  x  2y
 (8x  x)  2y
 (8  1)x  2y
 9x  2y
68. 7(5a  3b)  4a  7(5a)  7(3b)  4a
 35a  21b  4a
 35a  4a  21b
 (35a  4a)  21b
 (35  4)a  21b
 31a  21b
69. 4[1  4(5x  2y)]  4[1  4(5x)  4(2y)]
 4[1  20x  8y]
 4[1]  4[20x]  4[8y]
 4  80x  32y

30

59. 11 is eleven units from zero in the negative


direction. For Bismarck, 011 0  11.
5 is five units from zero in the negative
direction. For Caribou, 05 0  5.
4 is four units from zero in the negative
direction. For Chicago, 04 0  4.
9 is nine units from zero in the negative
direction. For Fairbanks, 09 0  9.
13 is thirteen units from zero in the negative
direction. For International Falls, 013 0  13.
7 is seven units from zero in the positive
direction. For Kansas City, 07 0  7.
34 is thirty-four units from zero in the positive
direction. For Sacramento, 034 0  34.
33 is thirty-three units from zero in the positive
direction. For Shreveport, 033 0  33.

70.

3
8

88

71.

7
12

2
72.

7
10

 12  12
3

 5  10  10
9

 10

73.

3
8

16

 3  24  24
25

 24
1

 1 24

35

Chapter 2

74.

5
6

266

75.

3
4

6
9
15

18

15

 2  30  30

77.

14

 18  18  18
7

 30


7
9

 18

1
10

Page 76

2 1

Check for Understanding


3

13

16. 134  (80)  134  (80)


 134  80
 214
The difference is 214.

Pages 7678

1 72
8
7
  1 0 14 0  0 14 0 2
8
7
  1 14  14 2
8

 14
8

25

32

9. 12  15  60  60

1 0 3260 0  0 2560 0 2
32
25
  1 60  60 2


 60

10. 18  23  18  (23)
 ( 023 0  018 0 )
 (23  18)
 5
11. 12.7  (18.4)  12.7  (18.4)
 12.7  18.4
 31.1
12. (3.86)  1.75  (3.86)  (1.75)
 ( 03.86 0  01.75 0 )
 (3.86  1.75)
 5.61
13. 32.25  (42.5)  32.25  (42.5)
 32.25  42.5
 ( 042.5 0  032.25 0 )
 (42.5  32.25)
 10.25
Chapter 2

Practice and Apply

17. 8  13  ( 013 0  08 0 )


 (13  8)
5
18. 11  19  ( 019 0  011 0 )
 (19  11)
8
19. 41  (63)  ( 063 0  041 0 )
 (63  41)
 22
20. 80  (102)  ( 0102 0  080 0 )
 (102  80)
 22
21. 77  (46)  ( 077 0  046 0 )
 (77  46)
 123
22. 92  (64)  ( 092 0  064 0 )
 (92  64)
 156
23. 1.6  (3.8)  ( 01.6 0  03.8 0 )
 (1.6  3.8)
 5.4
24. 32.4  (4.5)  ( 032.4 0  04.5 0 )
 (32.4  4.5)
 36.9
25. 38.9  24.2  ( 038.9 0  024.2 0 )
 (38.9  24.2)
 14.7
26. 7.007  4.8  ( 07.007 0  04.8 0 )
 (7.007  4.8)
 2.207
27. 43.2  (57.9)  ( 057.9 0  043.2 0 )
 (57.9  43.2)
 14.7

 2  14  14

2 1 42 2 1 55 2
42
55
 1 60 2  1 60 2
42
55
 1 60 2  60
55
42
  1 0 60 0  0 60 0 2
55
42
  1 60  60 2
 60

4. 15  (12)  ( 015 0  012 0 )


 (15  12)
 27
5. 24  (45)  ( 024 0  045 0 )
 (24  45)
 69
6. 38.7  (52.6)  ( 052.6 0  038.7 0 )
 (52.6  38.7)
 13.9
7. 4.62  (12.81)  ( 04.62 0  012.81 0 )
 (4.62  12.81)
 17.43

1 12

11

15. 10  12  60  60

1. Sample answer: 5  5
2. Sample answer: To subtract a real number, add
its opposite.
6
6
3. Gabriella; subtracting 9 is the same as adding 9.

4
7

47

 90

Adding and Subtracting


Rational Numbers

2-2

8.

1 27 2
20
27
  1 0 90 0  0 90 0 2
20
27
  1 90  90 2
 90  90

 3 or 1 3
76.

27

20

 12

20

14. 9  10  90  90

 3  12  12

36

28. 38.7  (61.1)  ( 061.1 0  038.7 0 )


 (61.1  38.7)
 22.4
29.

6
7

18

37. To find the total score, find the sum of her scores.
100  200  500  (300)  400  (500)
 300  500  (300)  400  (500)
 800  (300)  400  (500)
 ( 0800 0  0300 0 )  400  (500)
 (800  300)  400  (500)
 500  400  (500)
 900  (500)
 ( 0900 0  0500 0)
 (900  500)
 400
Her total score was 400 points.
38. First find the net yards gained by finding the sum
of 6, 8, and 3.
6  (8)  3  ( 08 0  06 0 )  3
 (8  6)  3
 2  3
 ( 03 0  02 0)
 (3  2)
1
Over the three plays, they gained 1 yard from
their 20-yard line. Therefore, they were on the
21-yard line.
39. 19  8  19  (8)
 ( 019 0  08 0 )
 (19  8)
 27
40. 16  (23)  16  (23)
 16  23
 39
41. 9  (24)  9  (24)
 9  24
 33
42. 12  34  12  (34)
 ( 034 0  012 0 )
 (34  12)
 22
43. 22  41  22  (41)
 ( 041 0  022 0)
 (41  22)
 19
44. 9  (33)  9  (33)
 9  33
 ( 033 0  09 0 )
 (33  9)
 24
45. 58  (42)  58  (42)
 58  42
 ( 058 0  042 0)
 (58  42)
 16
46. 79.3  (14.1)  79.3  (14.1)
 79.3  14.1
 93.4
47. 1.34  (0.458)  1.34  (0.458)
 1.34  0.458
 1.798
48. 9.16  10.17  9.16  (10.17)
 ( 09.16 0  010.17 0 )
 (9.16  10.17)
 19.33

14

 3  21  21
32

11

 21 or 1 21
30.

3
18

51

108

 17  306  306
159

 306
53

 102
4

20

33

31. 11  5  55  55

1 0 3355 0  0 2055 0 2
33
20
  1 55  55 2


13

 55
2

17

17

32. 5  20  20  20

1 0 1720 0  0 208 0 2
17
8
  1 20  20 2


 20
9

1 135 2
64
135
  1 0 240 0  0 240 0 2
64
135
  1 240  240 2
64

33. 15  16  240  240

16

13

199

 240

1 26 2
16
26
  1 0 40 0  0 40 0 2
16
26
  1 40  40 2
16

34. 40  20  40  40

42

 40
1

21

 20 or 1 20

2
33
12
 1 0 8 0  0 8 0 2
33
12
 1 8  8 2

35. 4 8  1 2 

33
8

12

 8

21
8
5
 28
17
67
3 25  50

17

36. 1 50 

184

  50

1 0 0  0 6750 0 2
184
67
  1 50  50 2


184
 50

117

  50

17

 2 50

37

Chapter 2

58. 2  (6)  (4)  (4)


 ( 02 0  06 0 )  (4)  (4)
 (2  6)  (4)  (4)
 8  (4)  (4)
 ( 08 0  04 0 )  (4)
 (8  4)  (4)
 12  (4)
 ( 012 0  04 0 )
 (12  4)
 16
59. Under; yes; it is better than par 72.
60. Subtract to find the change in value.
week 8
week 1
value 123
minus 123
value
123

49. 67.1  (38.2)  67.1  (38.2)


 67.1  38.2
 105.3
50. 72.5  (81.3)  72.5  (81.3)
 72.5  81.3
 153.8
1

51. 6  3  6  6

1 42
1
4
  1 0 6 0  0 6 0 2
1
4
 16  6 2
1

 6  6

 6
52.

1
2

 5  10  10
5
10

8
10

11,257.24 
9791.09  1466.15.
The value changed by 1466.15.

1 0 0  0 105 0 2
8
5
  1 10  10 2
8
10



61. From week 1 to week 2 the change was:


10,126.94  9791.09  335.85.
From week 2 to week 3 the change was:
10,579.85  10,126.94  452.91.
From week 3 to week 4 the change was:
10,810.05  10,579.85  230.20.
From week 4 to week 5 the change was:
10,951.24  10,810.05  141.19.
From week 5 to week 6 the change was:
10,821.31  10,951.24
 10,821.31  (10,951.24)
 ( 010,951.24 0  010,821.31 0 )
 (10,951.24  10,821.31)
 129.93.
From week 6 to week 7 the change was:
11,301.74  10,821.31  480.43.
From week 7 to week 8 the change was:
11,257.24  11,301.74.
 11,257.24  (11,301.74)
 ( 011,301.74 0  011,257.24 0 )
 (11,301.74  11,257.24)
 44.50.
Therefore, week 7 had the greatest change from
the previous week.
62. Week 8 had the least change from the previous week.
63. Sometimes; the equation is false for positive values
of x, but true for all other values of x.
64. Sample answer: If a team gains yards, move right on
the number line. If a team loses yards, move left on the
number line. Answers should include the following.
Move right or left, depending on whether the
Giants gained or lost yards on each play.
Where you end will tell you how many yards
the Giants lost or gained.
Instead of using a number line, you can use the
rules for adding and subtracting rational
numbers.
65. C; 57  87  57  (87)
 ( 057 0  087 0 )
 (57  87)
 144

 10
7

1 32
14
3
 16  1 16 2
14

53. 8  16  16  16

14

 16  16

1 0 14 0 0 163 0 2
14
3
  1 16  16 2
  16 

1 32

11

 16

1 92
1
9
 12  1 12 2
1

54. 12  4  12  12

 12  12

1 0 129 0  0 121 0 2
9
1
  1 12  12 2


 12
2

3
1

55. 2 4  6 3  4 
27

19
3

76

 12  12

1 0 0  0 2712 0 2
76
27
  1 12  12 2


76
12

49

 12 or 4 12

31

53

81

56. 5 10  1 50  10  50



265
81
 50
50
92
17
or 325
25

57. A score of 70, or 2 under par, is written as 2.


A score of 66, or 6 under par, is written as 6.
A score of 68, or 4 under par, is written as 4.
During the four rounds, he shot 2, 6, 4, and 4.

Chapter 2

38

73. Replace a in 3a  5 7 with each value in the


replacement set A.

66. B; 5  (8)  5  (8)


58
85

3a  5  7

32  5 7 7S1
7

Page 78

Maintain Your Skills

33  5 7 7S4
7
34  5 7 7S7
7
3  5  5 7 7 S 10 7 7
3  6  5 7 7 S 13 7 7

c  12  214

True or False?

1 ?

1 ?

1 ?

true

1 ?

true

1 ?

true

1
4

1
4

 2 6 24 S 4 6 24

1
2

1
2

 2 6 24 S 1 6 24

3
4

3
4

 2 6 24 S 14 6 24

true

true

1  2 6 24 S 12 6 24

114

14  2 6 24 S 14 6 24

The solution set for c  2 6 24 is

514, 12, 34, 1, 114 6.

75. Less than implies subtract in reverse order, and


square implies raised to the second power. So the
expression can be written as q2  8.
76. Less than implies subtract in reverse order, and
times implies multiply. So the expression can be
written as 2k  37.
1
2

3  23
1

Other
8%

b  1.3  1.8

0.3

0.3  1.3  1.8 S 1.6 1.8

?
?

0.4  1.3  1.8 S 1.7 1.8


?

0.5  1.3  1.8 S 1.8  1.8


?

0.6  1.3  1.8 S 1.9  1.8


0.7  1.3  1.8 S 2.0  1.8




True or False?

1
4

5  45

79.

3
4

12
5
2
25

1
3

6  46

 10

80. 4  5  1  5

72. Replace b in b  1.3  1.8 with each value in the


replacement set B.

78.

3

0.7

true

The solution set for 3a  5 7 is {5, 6}.


1
1
74. Replace c in c  2 6 24 with each value in the
replacement set C.

77.

true

0.6

false

Leave It
25%

0.5

false

Drink It
67%

0.4

false

67. 12.2  08  x 0  12.2  08  4.8 0


 12.2  03.2 0
 12.2  3.2
 15.4
68. 0y 0  9.4  3  07.4 0  9.4  3
 7.4  9.4  3
 16.8  3
 13.8
69. 24.2  018.3  z 0  24.2  018.3  10 0
 24.2  08.3 0
 24.2  8.3
 15.9
70. Sample answer: A category labeled other
representing 8% would have to be added so that
the data would sum to 100%.
71. Draw a circle with three sections. Make one section
67% of the circle and label it with the phrase drink
it. Make another section 25% of the circle and label
it with the phrase leave it. Make the third section
8% of the circle and label it with the word other.
Cereal Milk

True or False?

8

81. 8  58  18  58

82.

1
5

7
9

 12  9  12
1



3
28
3
1
93

false

2-3

false

Multiplying Rational Numbers

true

Page 81

true

Check for Understanding

1. ab will be negative if either one factor is negative,


and the other is positive. Let a  2 and b  3:
2(3)  6. Let a  2 and b  3: 2(3)  6.
2. Sample answer: calculating a $2.00 monthly
banking fee for the entire year: 12  ($2)  $24
3. Since multiplication is repeated addition,
multiplying a negative number by another
negative number is the same as adding
repeatedly in the opposite, or positive, direction.

true

The solution set for b  1.3  1.8 is {0.5, 0.6, 0.7}.

39

Chapter 2

4. (6)(3)  18
6. (4.5)(2.3)  10.35
8.

1 21 2 
5
3

10
21

2
7

5. 5(8)  40
7. (8.7)(10.4)  90.48
9.

1 21 2 
4 7
9 15

32. 5(5)(2)  (3)(2)


6

28
135

33.

10. 5s(6t)  5(6)st


 30st
11. 6x(7y)  (15xy)  6(7)xy  15xy
 42xy  15xy
 (42  15)xy
 57xy

1 22
12

 3
 4
1

13. np  2 3 4

15

  8 or 18

112 22123  22
1
4
 12 2213 2
1 4
 1 4 21 3 2

14. n2(m  2) 

 12
1

3
15. To find how much honey 675 honeybees make,
multiply the number of bees by the amount an
average bee makes.
1

675  12 



675
12
225
4
1
56 4
1

675 bees make 56 4 teaspoons of honey.

Pages 8183 Practice and Apply


16. 5(18)  90
18. 12(15)  180
20. 47(29)  1363

17. 8(22)  176


19. 24(8)  192
21. 81(48)  3888

22.

23.

145 2138 2  1240


3

 27

1 3 2156 2  1530

24. 5

4
15

21

2 1

16

21

15

26. 35 72   5  2

27.

21

1 2 2167 2  1235

25. 5

 2
1

20
1125 2149 2  108
5

 10

240
10

 24

1
22

2  195 2 152 2
45

 10
9

2
1

 42
28. 7.2(0.2)  1.44
30. (5.8) (2.3)  13.34
Chapter 2

 (2)(4)

8
34. 6(2x)  14x  6(2)x 14x
 12x  14x
 (12  14)x
 26x
35. 5(4n)  25n  5(4)n25n
 20n  25n
 (20  25)n
 45n
36. 5(2x  x)  5(2  1)x
 5(1)x
 5x
37. 7(3d  d )  7(3  1)d
 7(4)d
 28d
38. 2a(3c)  (6y)(6r)  2(3)ac  (6)(6)yr
 6ac  (36)yr
 6ac  36yr
 6ac  36ry
39. 7m(3n)  3s(4t)  7(3)mn  3(4)st
 21mn  (12)st
 21mn  12st
40. To find the change in price of 35 shares, multiply the
number of shares by the change in price of one share.
35  (61.66  63.66)  35  (2)
 70
The change in price of 35 shares was $70.
41. To find how much money you gained or lost,
multiply the number of shares by how much you
gained or lost on one share.
50  (61.69  64.38)  50  (2.69)
 134.5
You lost $134.50 or $134.50.
42. 5c2  5(4.5)2
43. 2b2  2(3.9)2
 5(20.25)
 2(15.21)
 101.25
 30.42
44. 4ab  4(2.7)(3.9)
 10.8 (3.9)
 42.12
45. 5cd  5(4.5)(0.2)
 22.5(0.2)
 4.5
46. ad  8  (2.7)(0.2)  8
 0.54  8
 7.46
47. ab  3  (2.7)(3.9)  3
 10.53  3
 13.53
2
48. d (b  2a)  (0.2)2 [3.9  2(2.7)]
 (0.2)2 [3.9  (5.4)]
 (0.2)2 [3.9  5.4]
 (0.2)2 [9.3]
 (0.04)[9.3]
 0.372

12. 6m  6 3

1 32
1
15
 2 1 4 2

2
(11)(4)
11

29. 6.5(0.13)  0.845


31. (0.075)(6.4)  0.48

40

49. b2 (d  3c)  (3.9)2 (0.2  3  4.5)


 (3.9)2 (0.2  13.5)
 (3.9)2 (13.7)
 (15.21)(13.7)
 208.377

60.

4
5

1 32

 20

50. To find the length of the union, multiply 5 by the fly.


2
5

6 


61. 42  (14)  42  (14)


 42  14
 56
62. 14.2  6.7  14.2  (6.7)
 ( 014.2 0  06.7 0 )
 (14.2  6.7)
 20.9
63.

12
5
2
25
2

The union is 2 5 feet long.


51. To find the price after 7 months, subtract the
number of months times the drop in price per
month from the starting price.
1450  7(34.95)  1450  244.65
 1205.35
The price of a computer was $1205.35.
52. To find the degrees difference, multiply the
number of 530-foot rises in altitude by the
temperature drop in one 530-foot rise.
64,997
530

53.

54.

55.

56.

57.
58.

1 15 2
16
15
  1 0 20 0  0 20 0 2
16
15
  1 20  20 2
16

 4  20  20

4 32 1 0 1 2 3 4 5 6

64.
2 1

65.
1

1  3 0

2
1
3

66. Graph c; Before Brandon fills the balloon with


air, the balloon has no air in it. The amount of air
in the balloon increases as he fills it. The amount
of air decreases after Brandon lets it go until the
balloon has a minimal amount of air left in it.
Graph c shows this situation.
67. Sample answer:

129,994
530

 (2)  

 245
The amount of degrees difference was about 245F.
To find how many plastic bottles are used in one
day, multiply the number of hours in a day by the
number of plastic bottles used every hour.
24  2,500,000  60,000,000
About 60 million plastic bottles are used in one day.
To find how many plastic bottles are used in one
week, multiply the number of hours in a week by
the number of plastic bottles used every hour.
168  2,500,000  420,000,000
About 420 million plastic bottles are used in one week.
Positive; the product of two negative numbers is
positive and the even number of negative factors
can be divided into groups of two.
Sample answer: Multiplying lets consumers
calculate quickly the total of several similar
items. Answers should include the following.
Coupons are negative values because adding a
negative number is the same as subtracting a
positive number.
Multiply 13.99 and 1.50 by three, then, add
the products.
B; 2x(4y)  2(4)xy
 8xy
B; 2ab  2(4)(6)
 (8)(6)
 48

x  5;

25  4  6
66
but 5
5
68. Sample answer:
?
a  4; 04 0 7 3
4 7 3
but 4
3
69.

5
8

 2  82

70.

2
3

 16
3

6
4

71. 5  4  5  3
20
3
2
 63
4
1 8
 23
1
4
3
1
 13
2
1
4
5
56
1
3
2
3

75.

1
2

4
5

8

5

72. 1  5  1  2
5

2

73.

2
 4  32

 22
74.

7
9

2
6

 6  95
3
14

 15

76.

7
8

 3  82
21

 16
5

 116

Page 83

Maintain Your Skills

59. 6.5  (5.6)  ( 06.5 0  05.6 0 )


 (6.5  5.6)
 12.1

41

Chapter 2

Page 83

Practice Quiz 1

10.

1. The dots indicate each point on the graph. The


coordinates are {4, 1, 1, 6}.
2. 32  0x  8 0  32  015  8 0
 32  023 0
 32  23
9
3. 15  7  ( 015 0  07 0 )
 (15  7)
 8
4. 27  (12)  27  (12)
 27  12
 39
5. 6.05  (2.1)  ( 06.05 0  02.1 0 )
 (6.05  2.1)
 8.15

1 22

28

 4

12.

 650a

13.

2ab
ac





1 82
15
8
 20  1 20 2

 (6b  18)  (2)

1 12
1
1
 6b 1 2 2  18 1 2 2

2(3)(4.5)
(3)(7.5)
(6) (4.5)
(22.5)
27
22.5

14.

15. b 

a
c

 (4.5)  7.5
7.5
3

22.5

 (13.5)
 (1.67)
 1.67
16. To find the number of visitors the site had in 1999,
divide the number of visitors in 2000 by eight.
419,000  8  52,375
There were 52,375 visitors in 1999.

7. 9(12)  108
8. (3.8)(4.1)  15.58
9. (8x)(2y)  (3y)(z)  (8)(2)xy  (3)(1)yz
 16xy  (3)yz
 16xy  3yz
10. mn  5  (2.5)(3.2)  5
 8  5
 ( 08 0  05 0 )
 (8  5)
 3

Pages 8687
17.
19.
21.
23.
24.
25.
26.

Dividing Rational Numbers

Practice and Apply

18. 78  (4)  19.5


64  (8)  8
20. 108  (0.9)  120
78  (1.3)  60
42.3  (6)  7.05 22. 68.4  (12)  5.7
23.94  10.5  2.28
60.97  13.4  4.55
32.25  (2.5)  12.9
98.44  (4.6)  21.4
1

1. Sample answer: Dividing and multiplying


numbers with the same signs both result in a
positive answer while dividing or multiplying
numbers with different signs results in a negative
answer. However, when you divide rational
numbers in fractional form, you must multiply by
a reciprocal.
1
1
1
2. Sample answer: 2, since 1  2 and 2 7 2.




Chapter 2

35

  3 or 11 3
2

30. 5  7  5  2
35

  2 or 17 2
31.

16
36

24

16

60

24

960

33.

5
4
10
12
5
6

14
32


12

10
9

42

63

1512

14

25

 25  32  12


24

 1736

or 1 9

31

32. 56  63  56  31

 60  36  24
 864

2
3

29. 7  5  7  3

3. To divide by a rational number, multiply by its


reciprocal.
4. 96  (6)  16
5. 36  4  9
6. 64  5  12.8
7. 64.4  2.5  25.76


 48
 16

9.

 12

Check for Understanding

4
5

28. 4  12  4  12

27. 3  4  3  4

 12 

(7.5)(4.5)
4(3)
33.75
12

 2.81

 20

8.




 (4.5) 

2
3

cb
4a

 1.2

15

2 1
3  12
2
36
1
18

1101 2

 3b  (9)
 3b  9

1 0 15 0 0 208 0 2
15
8
  1 20  20 2

2
3

 650a  10

 (6b  18) 2

  20 

Page 86

650a
10

 65a
6b  18
2

 20  20

2-4

11.

 7

6. 4  5  20  20
15

25  3
4

350
384
175
192

27

 31

34.

80
25

1 22

1 32

80

45. To find the number of pillows, divide the total


amount of fabric by the amount of fabric needed
for each pillow.

 3  25  2
240

  50
24

1 52

1 32

1 32

222
5

36

1 82

1248
3




81c
9

 81c  9
 81c
 9c

39.

8r  24
8

12

38.

105g
5

1
9

 105g  5
 105g
 21g

115 2

 (8r  24)  (8)

1 12
1
1
 8r 1 8 2  24 1 8 2

 (8r  24) 8

47.

7h  35
7

 (7h  35)  (7)

1 12
1
1
 7h 1 7 2  35 1 7 2

 1h  (5)
 h  5
40a  50b
2

(8) (6.5)
(3.2)
52
3.2

51.

n  p
m




 (40a  50b)  2

112 2
1
1
 40a 1 2 2  50b 1 2 2

53.

m  2n
n  q

54.

m  p
q




8  3.2
5.4
4.8
5.4

 0.89

8  2(6.5)
 (5.4)
8  13
6.5  5.4

 6.5

p  3q
q  m

3.2  3(5.4)

 (5.4)  (8)
3.2  (16.2)

 (5.4)  (8)


 [8f  (16g)]  8

118 2
1
1
 8f 1 8 2  (16g) 1 8 2

 [8f  (16g)]

3.2  16.2
5.4  8
19.4
13.4

 1.45
55. To find the average loss per month, divide the
total loss by the number of months.

 1f  (2g)
 f  2g
44.

52.

 1.76

 14c  6d

5x  (10y)
5

(6.5) (3.2)
8
20.8
8

21

113 2
1
1
 42c 1 3 2  18d 1 3 2

8f  (16g)
8

 11.9

 (42c  18d)  3
 (42c  18d)

43.

6.5  3.2
8
9.7
8

 20a  25b
42c  18d
3

np
m

 1.21

 (40a  50b)

42.

48.

 16.25
 2.6
49. mq  np  (8) (5.4)  (6.5) (3.2)
 43.2  20.8
 2.08
50. pq  mn  (3.2) (5.4)  (8) (6.5)
 17.28  (52)
 0.33

 (7h  35) 7

41.

mn
p

 1r  (3)
 r  3
40.

18
7
4
27

She can make 2 pillows.


46. To find the average change in revenue, divide the
change in revenue by the number of years.
(2,764,000,000  2,800,000,000)  8
 36,000,000  8  4,500,000
The average change in revenue for each of the
8 years was $4,500,000.

 416
37.

 14

or 44 5

36. 156  8  156  3




 27

35. 74  3  74  5




42  14  2  4

  5 or 4 5

23,985  12  1998.75
The average loss per month was $1998.75.
56. To find the fraction that is pure gold, divide the
karat value by the karat value of pure gold.

 [5x  (10y)]  5

115 2
1
1
 5x 1 5 2  (10y) 1 5 2

 [5x  (10y)] 

10

10  24  24

 1x  (2y)
 x  2y

 12
Therefore, 10-karat gold is
5
7
1  12 or 12 not gold.

43

5
12

pure gold, and

Chapter 2

70. Substitution Property; 1.2  3.8  5


71. 8b  12(b  2)  8b  12(b)  12(2)
 8b  12b  24
 (8  12)b  24
 20b  24
72. 6(5a  3b  2b)  6(5a)  6(3b)  6(2b)
 30a  18b  12b
 30a  (18  12)b
 30a  6b
73. 3(x  2y)  2y  3(x)  3(2y)  2y
 3x  6y  2y
 3x  (6  2)y
 3x  4y

57. Build up the fraction so that the denominator is


24. The numerator will be the karat value of the
gold.
2
3

2.8

 3.8
16

 24
Therefore, jewelry that is

2
3

gold is 16-karat gold.

58. Since 4 is even, 4 is divisible by 2. Therefore, if a


number is divisible by 4, then it is divisible by 2.
Note that 3 . 4 . 6  72 and 8 . 9  72. Therefore,
if a number is divisible by 72, then it is divisible
by 3, 4, 6, 8, and 9. Since 8 and 9 have no
common factor, no number smaller than their
product will be divisible by both 8 and 9. Now,
note that 72 . 5 . 7  2520, so 2520 is divisible by
1, 2, 3, 4, 5, 6, 7, 8, and 9. Since 72, 5, and 7 have
no common factor, no number smaller than their
product will be divisible by 72, 5, and 7. Thus,
2520 is the least positive integer that is divisible
by all whole numbers from 1 to 9.
59. Sample answer: You use division to find the mean
of a set of data. Answers should include the
following.
You could track the mean number of turtles
stranded each year and note if the value
increases or decreases.
Weather or pollution could affect the turtles.
60. D; 6.25  10  12  0.625  12
 7.5
61. E; 6x  1  6 

17
3

74. mean 


 36
There are five numbers in the set. When ordered
from least to greatest, the third number is 38.
So, the median is 38. The number 40 appears
twice, and the other numbers of the set appear
only once. So, the mode is 40.
75. mean 


1

76. mean 


64.

1
(5)
4

5
4

1
14

63. 2.5(1.2)  3
65. 1.6(0.3)  0.48

66. 8  (6)  8  (6)


86
 14
67. 15  21  15  (21)
 ( 021 0  015 0 )
 (21  15)
 6
68. 7.5  4.8  7.5  (4.8)
 ( 07.5 0  04.8 0 )
 (7.5  4.8)
 12.3
5

1 12

77. mean 


1 42
15
4
 24  1 24 2
15

 24  24

1 0 15 0 0 244 0 2
15
4
  1 24  24 2
  24 
11

 24

Chapter 2

79  84  81  84  75  73  80  78
8
634
8

 79.25
There are eight numbers in the set. When ordered
from least to greatest, the fourth number is 79 and
79  80
the fifth is 80. So, the median is
or 79.5. The
2
number 84 appears twice, and the other numbers
of the set appear only once. So, the mode is 84.

69. 8  6  24  24


15

1.2  1.7  1.9  1.8  1.2  1.0  1.5


7
10.3
7

 1.47
There are seven numbers in the set. When
ordered from least to greatest, the fourth number
is 1.5. So, the median is 1.5. The number 1.2
appears twice, and the other numbers of the set
appear only once. So, the mode is 1.2.

Maintain Your Skills

62. 4(11)  44

3  9  0  2  11  8  14  3
8
50
8

 6.25
There are eight numbers in the set. When ordered
from least to greatest, the fourth number is 3 and
3  8
1
the fifth is 8. So, the median is 2 or 52  5.5.
The number 3 appears twice, and the other numbers
of the set appear only once. So, the mode is 3.

 34  1
 35

Page 87

40  34  40  28  38
5
180
5

44

2-5

Statistics: Displaying and


Analyzing Data

Page 91

Check for Understanding

11. The greatest common place value is tens, so the


digits in the tens place are the stems.
Stem Leaf
5 4556
6 0149
7 03578
8 0035888
9 0
10 0 2 5
11 0
5 0 4  54
12. Occurring three times, the most frequent value is 88.
13. The mode is not the best measure as it is higher
than most of the values.

1. They describe the data as a whole.


2. Sample answer:
Line Plot:


1








Line Graph:

Pages 9294

Practice and Apply

14. The lowest value is 35, and the highest value is


54, so use a scale that includes those values.
Place an  above each value for each occurrence.
 
1

15. The lowest value is 2.0, and the highest value is


2.5, so use a scale that includes those values.
Place an  above each value for each occurrence.

 
 

2

1




 
 
 
 
 
 
 
 

10 12 14 16 18 20 22 24

5. The lowest value is 0, and the highest value is 14,


so use a scale that includes those values. Place an
 above each value for each occurrence.


 

  
  

1
0










16. The lowest value is 1, and the highest value is 8,


so use a scale that includes those values. Place an
 above each value for each occurrence.






 








34 36 38 40 42 44 46 48 50 52 54

3. A set of data with one extremely high value would


be better represented by the median, not by the
mean. Sample answer: 13, 14, 14, 28
4. The lowest value is 10, and the highest value is
22, so use a scale that includes those values.
Place an  above each value for each occurrence.



  

10 12 14










 
  



7

17.
18.
19.
20.

23 of the 44 teams were not number 1 seeds.


29 of the 44 teams were seeded higher than third.
Sample answer: Median; most of the data are near 2.
The greatest common place value is ones, so the
digits in the ones place are the stems.
Stem Leaf
5 89
6 03569
7 01123
5 08  5.8
21. The greatest common place value is tens, so the
digits in the tens place are the stems.
Stem Leaf
1 88
2 2366689
3 011234
4 7
1 08  18

6. Occurring four times, the most frequent value is 0.


7. The mean, 6.75, and the median, 6.5, both represent
the data accurately as they are fairly central.
8. The greatest common place value is tens, so the
digits in the tens place are the stems.
Stem Leaf
6 46888
7 1236
8 01688
9 3
6 0 4  64
9. The greatest value is 12.9, and the least value is
9.3. Therefore, the difference is 12.9  9.3 or 3.6.
10. Median; most of the data clusters higher, near the
median, 11.8.

45

Chapter 2

38. Sample answer: 4, 4, 4, 5, 5, 5, 7, 8, 8, 8, 8, 18

22. The greatest common place value is hundreds, but


the hundreds digit in every number is a one. Thus
we use the digits in the tens place as the stems.
Stem Leaf
10 0 0 4 5 5 6 6 7 8 9 9
11 0 0 0 0 1 1 2 2 2 2 3 3 4 4 4 4 4 5
67778888889
12 0 0 0 0 1 1 2 5 8
13 4
10 00  100
23. Occuring six times, the most frequent temperature
is 118.
24. No; the mode is higher than most of the data.
25. See students work.
26. Occuring three times, the most frequent
magnitude was 7.5.
27. Mean or median; both are centrally located and
the mode is too high.
28. The lowest value is 8, and the highest value is 40,
so use a scale that includes those values. Place an
 above each value for each occurrence.

   

mean 


39.

40.

41.

42.

12 16 20 24 28 32 36 40

29. 7 of the 10 countries won fewer than 25 gold medals.


30. When ordered from least to greatest, the fifth
value is 13 and the sixth is 14. So, the median is
13  14
or 13.5.
2
31. Sample answer: Yes; most of the data are near the
median.
32. The greatest common place value is tens, so the
digits in the tens place are the stems.
Stem Leaf
1 688999
2 00000112334568889
9999
3 00001334467
4 37
1 06  16
33. 22 of the 40 vehicles get more than 25 miles per
gallon.
34. Sample answer: Mean; the median is too high,
and the modes are either too high or too low.
35. The greatest common place value is tens, so the
digits in the tens place are the stems.
Stem Leaf
3 047
4
5 29
6 27
7 7
8 4 5 3 00  30
36. The interval that corresponds to the stem value 3
has the most values. The interval is from 30 to 39.
37. Every number in the set appears only once. So,
there is no mode.

Chapter 2

43.

4  4  4  5  5  5  7  8  8  8  8  18
12
84
12

7
There are twelve numbers in the set. When
ordered from least to greatest, the sixth number
5  7
is 5 and the seventh is 7. So, the median is 2
or 6. The number 8 appears four times, and the
other numbers of the set appear three times or
less. So, the mode is 8.
High school: $33,184  $23,061  $10,123; some
college: $39,221  $27,757  $11,464; Bachelors
degree: $60,201  $41,747  $18,454; Doctoral
degree: $81,687  $60,079  $21,608
Sample answer: The highest median salaries are
earned by people whose highest level of education
is a bachelors degree.
Sample answer: because the range in salaries is
often very great with extreme values on both the
high end and low end
Sample answer: They can be used in marketing or
sales to sell the most products to a specific group.
Answers should include the following.
a line plot showing the number of males with
the names from the beginning of the lesson
By finding out the most popular names, you can
use the popular names on more of your items.
C;

mean 


7.5  7.5  7.5  7.5  7.9  7.9  7.9  8.3  9.1  11


10
82.1
10

 8.21
44. C; 7 of the 10 wingspans are less than 8 inches.

Page 94

Maintain Your Skills

45. 56  (14)  4
46. 72  (12)  6
47. 40.5  3  13.5
48. 102  6.8  15
49. 2(6x)  5x  2(6)x  5x
 12x  5x
 (12  5)x
 17x
50. 3x(7y)  4x(5y)  3(7)xy  4(5)xy
 21xy  20xy
 (21  20)xy
 41xy
51. 5(3t  2t)  2(4t)  5(3t)  5(2t)  2(4t)
 5(3)t  5(2)t  2(4)t
 15t  10t  8t
 (15  10  8)t
 3t
52.
x dollars
d dollars plus per week for 12 weeks
14243 123 14243 123 123
d

x

12
or d  12x
53. y  3x  16  3.5
54. xz  3  5.9  3
 16  15
 45  3
1
 15

46

55. 2x  x  (y  4)  2  5  5  (16  4)
2554
 10  5  4
54
9
56.

58.

60.

62.

64.

x  z
2y

5  9
2  16
25  9
 2  16
16
 2  16
16
 32
1
2
54
54  6
 60  6
60
9
 10
42
42  6
 48  6
48
7
8
28
28  4
 52  4
52
7
 13
84
84  6
 90  6
90
14
 15

Page 95

57.

12
18

4. There are four 5s and 52 total cards.


4

P(5)  52
1

 13
1

The probability of selecting a 5 is 13 or about 8%.


5. There are two red 10s and 52 total cards.

12  6

 18  6

P(red 10)  52

3

 26
1

The probability of selecting a red 10 is 26 or


about 4%.
6. There are 16 odd numbered cards and 52 total cards.
59.

21
30

21  3

16

 30  3

P(odd number)  52

 10
61.

32
64

 13

32  32

 64  32

The probability of selecting an odd numbered card


4
is 13 or about 31%.
7. There is 1 way to pick the queen of hearts and
1 way to pick the jack of diamonds. So there are
1  1 or 2 ways to pick the queen of hearts or
jack of diamonds

2
63.

16
36

16  4

 36  4
4

9

P(queen of hearts or jack of diamonds)  52


1

 26
The probability of selecting the queen of hearts or
1
jack of diamonds is 26 or about 4%.
8. There are three numbers on the spinner that are
multiples of 3, and there are 10 3 or 7 numbers
that are not multiples of 3.

Reading Mathematics

1. Sample answer: The data show the acreage and


number of visitors in thousands for selected state
parks and recreation areas in 1999.
2. Sample answer: The footnote indicates that the
number of visitors includes those staying overnight.
3. The data is for 1999.
4. The unit indicator is thousands.
5. The table has the value 1016 in the row labeled
New York and the column labeled Acreage.
Therefore, New York has 1,016,000 acres.
6. The largest number in the column labeled Visitors
is 76,736, which is in the row labeled California.
Therefore, California had the greatest number of
visitors with 76,736,000 visitors.

odds of a multiple of 3  7
The odds of getting a multiple of 3 on the spinner
are 3:7.
9. There are three even numbers less than 8 on the
spinner, and there are 10  3 or 7 numbers that
are not even or not less than 8.
3

odds of an even number less than 8  7


The odds of getting an even number less than 8
on the spinner are 3:7.
10. There are seven numbers on the spinner that are
odd or blue, and there are 10  7 or 3 numbers
that are even and not blue.
7

2-6
Page 98

odds of an odd number or blue  3

Probability: Simple
Probability and Odds

The odds of getting an odd number or blue on the


spinner are 7:3.
11. There are six numbers on the spinner that are
red or yellow, and there are 10  6 or 4 numbers
that are blue.

Check for Understanding

1. Sample answers: impossible event: rolling a


number greater than 6; certain event: rolling a
number from 1 to 6; equally likely event: rolling
an even number.
3
2. The probability is 5 which means there are
3 favorable outcomes for every 5  3 or
2 unfavorable outcomes. Thus the odds are 3:2.
3. Doug; Mark determined the odds in favor of
picking a red card.

odds of red or yellow  4


The odds of getting a red or yellow number on the
spinner are 6:4.

47

Chapter 2

18. There are 70  100  80 or 250 coins with value


less than $1.00 and 300 total coins.

12. The factors of 48 are 1, 2, 3, 4, 6, 8, 12, 16, 24,


and 48, of which 4, 8, 12, 16, 24, and 48 are
multiples of 4. There are 4 factors that are not a
multiple of 4 and 10 total factors.

250

P(value less than $1.00)  300


5

6

P(not a multiple of 4)  10

 0.83
The probability of selecting a coin with value less
than $1.00 is 65 or about 83%.
19. There are 80  50 or 130 coins with value greater
than $0.10 and 300 total coins.

5
The probability of selecting a factor that is not a
2
multiple of 4 is 5 or 40%.
13. The factors of 48 are 1, 2, 3, 4, 6, 8, 12, 16, 24,
and 48, of which 12, 24, and 48 have both 4 and 6
as factors. There are 3 factors of 48 that have
4 and 6 as two of their factors and 10 total factors
of 48.

130

P(value greater than $0.10)  300


13

 30
 0.43
The probability selecting a coin with value greater
13
than $0.10 is 30
or about 43%.
20. There are 80  50 or 130 coins with value at least
$0.25 and 300 total coins.

P(has 4 and 6 as factors)  10 or 0.3


The probability of selecting a factor that has 4
3
and 6 as two of its factors is 10 or 30%.

130

Pages 99101

P(value at least $0.25)  300

Practice and Apply

13

 30

14. There are 80 quarters and 300 total coins.

 0.43
The probability of selecting a coin with value at
least $0.25 is 13
or about 43%.
30
21. There are 70  100  80  50 or 300 coins with
value at most $1.00 and 300 total coins.

80

P(quarter)  300


4
15

 0.27
4
The probability of selecting a quarter is 15 or
about 27%.
15. There are 100 dimes and 300 total coins.

300

P(value at most $1.00)  300


1
The probability of selecting a coin with value at
most $1.00 is 1 or 100%.
22. There are 15 ways for the sum of two dice to be
less than 7 and 36 total outcomes.

100

P(dime)  300
1

3
 0.33
1

The probability of selecting a dime is 3 or about 33%.


16. There are 70 ways to pick a nickel and 50 ways to
pick a dollar. So there are 70  50 or 120 ways to
pick a nickel or a dollar and 300 total coins.

15

P(sum less than 7)  36


5

 12
 0.42
5
The probability of the sum being less than 7 is 12
or about 42%.
23. There are 21 ways for the sum of two dice to be
less than 8 and 36 total outcomes.

120

P(nickel or dollar)  300


2

5
 0.40
The probability of selecting a nickel or a dollar is
2
or 40%.
5
17. There are 80 ways to pick a quarter and 70 ways
to pick a nickel. So there are 80  70 or 150 ways
to pick a quarter or a nickel and 300 total coins.

21

P(sum less than 8)  36


7

 12
 0.58
The probability of the sum being less than 8 is
or about 58%.
24. There are 0 ways for the sum of two dice to be
greater than 12 and 36 total outcomes.

150

P(quarter or nickel)  300


1

 2 or 0.5
The probability of selecting a quarter or a nickel
1
is 2 or 50%.

P(sum is greater than 12) 

7
12

0
36

0
The probability of the sum being greater than 12
is 0 or 0%.

Chapter 2

48

33. There are 3 polygons that have more than one


right angle and 6 total polygons.

25. There are 36 ways for the sum of two dice to be


greater than 1 and 36 total outcomes.
36

P(sum is greater than 1)  36

P(more than one right angle)  6

1
The probability of the sum being greater than 1 is
1 or 100%.
26. There are 20 ways for the sum of two dice to be
between 5 and 10 and 36 total outcomes.

 2 or 0.5
The probability of selecting a polygon that has
more than one right angle is 1 or 50%.
2
34. There is one day in April that is the 29th and 30
total days in April. The probability that the
1
persons birthday is the 29th is 30 or about 3%.
35. There are 15 days in July after the 16th and 31
total days in July. The probability that the
15
persons birthday is after the 16th is 31 or
about 48%.
36. There are three as in the name, and there are
24  3 or 21 letters that are not an a.

20

P(sum is between 5 and 10)  36


5

9
 0.56
The probability of the sum being between 5 and
5
10 is 9 or about 56%.
27. There are 25 ways for the sum of two dice to be
between 2 and 9 and 36 total outcomes.
P(sum is between 2 and 9) 

odds of an a  21

25
36

7

 0.69
The probability of the sum being between 2 and 9
25
is 36 or about 69%.
28. There are 3 triangles and 6 total polygons.

The odds of selecting an a from the name are 1:7.


37. There are four ts in the name and there are
24  4 or 20 letters that are not a t.
4

odds of a t  20 or

P(triangle)  6

1
5

The odds of selecting a t from the name are 1:5.


38. There are eleven vowels in the name and there
are 24  11 or 13 consonants.

 2 or 0.5
1

The probability of selecting a triangle is 2 or 50%.


29. There is 1 pentagon and 6 total polygons.

11

odds of a vowel  13

P(pentagon)  6

The odds of selecting a vowel from the name are


11:13.
39. There are thirteen consonants in the name and
there are 24  13 or 11 vowels.

 0.17
1
The probability of selecting a pentagon is 6 or
about 17%.
30. There are 3 polygons that are not triangles and 6
total polygons.

13

odds of a consonant  11
The odds of selecting a consonant from the name
are 13:11.
40. There are four uppercase letters in the name and
there are 24  4 or 20 lowercase letters.

P(not a triangle)  6
1

 2 or 0.5
The probability of selecting a polygon that is not a
1
triangle is 2 or 50%.
31. There are 4 polygons that are not quadrilaterals
and 6 total polygons.

odds of an uppercase letter  20 or

1
5

The odds of selecting an uppercase letter from the


name are 1:5.
41. There are nine lowercase vowels in the name and
there are 24  9 or 15 letters that are not
lowercase vowels.

P(not a quadrilateral)  6
2

3

odds of a lowercase vowel  15 or

 0.67
The probability of selecting a polygon that is not a
2
quadrilateral is 3 or about 67%.
32. There are 3 polygons that have more than three
sides and 6 total polygons.

3
5

The odds of selecting a lowercase vowel from the


name are 3:5.
42. There are five stamps from Canada and there are
32  5 or 27 stamps that are not from Canada.

P(more than three sides)  6

odds of being from Canada  27

 2 or 0.5

The odds the stamp is from Canada are 5:27.


43. There are twelve stamps from Mexico and there
are 32  12 or 20 stamps that are not from Mexico.

The probability of selecting a polygon that has


1
more than three sides is 2 or 50%.

12

odds of being from Mexico  20 or

3
5

The odds the stamp is from Mexico are 3:5.

49

Chapter 2

52. 100  40  25(100  2  25)


 100  40  25(100  50)
 100  40  25(50)
 4000  1250
 5250
2  252  100  35  2  625  100  35
 1250  3500
 4750
The area of the region not shaded is 5250 cm2,
and the area of the shaded region is 4750 cm2.

44. There are 32  3 or 29 stamps not from France


and there are 3 stamps from France.
odds of not being from France 

29
3

The odds the stamp is not from France are 29:3.


45. There are 3  8  1  3 or 15 stamps not from a
North American country and there are 32 15 or
17 stamps from a North American Country.
odds of not being from a North American
15
country  17

5250

odds against shaded region  4750

The odds the stamp is not from a North American


country are 15:17.
46. There are 3  1 or 4 stamps from Germany or
Russia and there are 32  4 or 28 stamps not
from Germany or Russia.
4
1
odds of being from Germany or Russia  28 or 7

21

 19
The odds against placing a piece on a shaded
region are 21:19.
53.
100  35  3500
100(40  25)  100  65
 6500
The area of the green rectangle is 3500 cm2, and
the area outside the green rectangle is 6500 cm2.

The odds the stamp is from Germany or Russia


are 1:7.
47. There are 5  8 or 13 stamps from Canada or
Great Britain and there are 32  13 or 19 stamps
not from Canada or Great Britain.
13
odds of being from Canada or Great Britain  19

3500

odds of green rectangle  6500


7

 13

The odds the stamp is from Canada or Great


Britain are 13:19.
3
48. The probability that it will occur is 7, so the

The odds of a piece being placed within the green


rectangle are 7:13.
54. There are 24 players who hit more than 35 home
runs and 46 total players.

probability that it will not occur is 74.


3

24

P(more than 35)  46

odds it occurs  7 : 7 or 3:4

12

 23

The odds that the event will occur are 3:4.

 0.52
The probability of selecting a player who hit more
12
than 35 home runs is 23 or about 52%.
55. There are forty-two players who hit less than
45 home runs and there are 46  42 or 4 players
who hit 45 or more.

49. The probability that it will occur is 3, so the


1

probability that it will not occur is 3.


1

odds against it occuring  3 : 3 or 1:2.


The odds that the event will not occur are 1:2.
50. There are four cards from the coworkers in the
bowl and there are 80  4 or 76 cards not from
the coworkers.

odds of less than 45 




odds a coworker wins  76

The odds of selecting a player who hit less than


45 home runs are 21:2.
56. The number of home runs is 38 and the total
number of bats is 439.

 19
51.

The odds one of the coworkers will win are 1:19.


2  252  100  35  2  625  100  35
 1250  3500
 4750
100(35  40  25)  100(100)
 10,000
The area of the shaded region is 4750 cm2, and
the total area is 10,000 cm2.

38

P(home run)  439


 0.09
The probability the player hits a home run the
38
next time the player bats is 439 or about 9%.
57. There is one winning game card and 1,000,000
nonwinning game cards, so there is one winning
card and a total of 1,000,001 cards.

4750

P(shaded region)  10,000

19

P(grand prize)  1,000,001

 40 or 0.475

 0.000001
The probability of selecting the winning game
1
card is 1,000,001 or about 0.0001%.
58. No; even with 100 game cards the odds of winning
are only 100:999,901. It would require several
hundred thousand cards to significantly increase
the odds of winning.

The probability the piece is placed on a shaded


19
region is 40 or 47.5%.

Chapter 2

42
4
21
2

50

68. 12.2  7.8  ( 012.2 0  07.8 0 )


 (12.2  7.8)
 4.4

59. There are 6 ways for a head to appear on at least


one of them and 7 total possible outcomes.
6

P(at least one head)  7

13

70.

12




2
5
4
5

74.






66.

ab
c







1 0 10 0 0 127 0 2
10
7
  1 12  12 2
  12 

0 23 0  23

1
6

is one-sixth unit from zero in the positive


direction.

0 16 0  16

75. 62  6  6
 36
77. (8)2  (8)(8)
 64

76. 172  17  17
 289
78. (11.5)2  (11.5)(11.5)
 132.25

79. 1.62  1.6  1.6


 2.56

80.

81. 49 2  49 49

2  16 16
82. 16
15
15
15

1 2

65. 2a  b  2 3  5

2
1

or 80%

1 12

10

73. 3 is two-thirds unit from zero in the negative


direction.

63. The greatest common place value is tens, so the


digits in the tens place are the stems.
Stem Leaf
5 8.3
6 4.3 5.1 5.5 6.7 7.0 8.7 9.3
7 0.0 2.8 3.2 5.8 7.4 7.4
5 0 8.3  58.3
1

71. 4.25 is four and twenty-five hundredths units


from zero in the positive direction.
04.25 0  4.25
72. 8.4 is eight and four tenths units from zero in
the negative direction.
08.4 0  8.4

Maintain Your Skills

12

64. b  c  5  2

1 52

 6  12  12

 4

P(satisfy inequality)  15

Page 101

7
12

 8

 13:12
62. D; There are 12 numbers that satisfy the
inequality and 15 total numbers.


1 32
2
3
  1 0 8 0  0 8 0 2
2
3
 18  8 2
2

 12

odds against occurring  25 : 25

4
5

1 32

69. 4  8  8  8

 0.86
6
The probability of at least one head is 7 or
about 86%.
60. Sample answer: Probabilities are often used for
strategy like placing a certain pitcher against a
batter who has a low probability of hitting a pitch
from that pitcher. Answers should include the
following.
baseball: using the probability that a team can
get base runners out; basketball: the
probability that a player can make a basket
from a certain place on the court; auto racing:
the probability that a set of tires will hold out
for the remainder of a race
Odds in favor of an event and odds against an
event are frequently used.
13
61. B; The probability the event will not occur is 25.

2
2
3  5
2 5
3  2
10
6
5
2
3 or 13

Page 101

1 21 2

1125 22  125  125


1

16
81

Practice Quiz 2

1. 136  (8)  17

3. (46.8)  4  11.7

256
225

1 32

21

or

31
1225

1 82

2. 15  8  15  3

3  5
1
2
1 2
1
3  5  2
2
1
15  2
2 2
15  1
4
15

25

 144

4.

3a  9
3

120

  3
  40

 (3a  9)  3

113 2
1
1
 3a 1 3 2  9 1 3 2

 (3a  9)
 1a  3
a3

67. 4.3  (8.2)  ( 08.2 0  04.3 0 )


 (8.2  4.3)
 3.9

51

Chapter 2

5.

4x  32
4

2.

 (4x  32)  4

114 2
1
1
 4x 1 4 2  32 1 4 2

4 boys
BBBB

 (4x  32)

 1x  8
x8
6.

15n  20
5

 (15n  20)  (5)

1 12
1
1
 15n 1 5 2  20 1 5 2

 3n  (4)
 3n  4

7. The lowest value is 0.4, and the highest value is


5.0, so use a scale that includes those values.
Sample answer: scale from 0 to 5 Place an 
above each value for each occurrence.

 

    


  




1 boy,
3 girls
BGGG
GBGG
GGBG
GGGB

4 girls
GGGG

3. There are 2 possibilities for a one-child family,


4 possibilities for a two-child family, 8 possibilities
for a three-child family, and 16 possibilities for a
four-child family. Following the pattern, there
should be 32 possibilities for a five-child family
and 64 possibilities for a six-child family. The
pattern represents powers of 2.
4. There are 3 ways to have 2 boys and 1 girl and
8 total outcomes. Therefore, the probability of
having 2 boys and 1 girl in a three-child family is
3
or 37.5%.
8
5. There are 6 ways to have 2 boys and 2 girls and
16 total outcomes. Therefore, the probability of
having 2 boys and 2 girls in a four-child family is
6
3
 8 or 37.5%.
16
6. Sample answer: Each number in each row shows
the number of ways to have boys and girls for a
given number of children.
7. The fifth row labeled Row 4 represents a fourchild family. The second number from the right is
4 and represents how many ways to have 3 girls
and 1 boy. There are 1  4  6  4  1 or 16 total
outcomes. Therefore, the probability of having
1 boy in a four-child family is 4  1 or 25%.

 (15n  20) 5

3 boys, 2 boys,
1 girl 2 girls
BBBG
BBGG
BBGB
BGBG
BGBB
BGGB
GBBB
GBBG
GBGB
GGBB

8. There are 3 ways for the sum of two dice to be 10


and 36 total outcomes.
3

P(sum of 10)  36
1

 12
 0.08
1
The probability of the sum being 10 is 12 or
about 8%.
9. There are 26 ways for the sum of two dice to be
greater than or equal to 6 and 36 total outcomes.
26

P(sum  6)  36

16

13

 18

2-7

 0.72
The probability of the sum being greater than or
equal to 6 is 13
or about 72%.
18
10. There are 30 ways for the sum of two dice to be
less than 10 and 36 total outcomes.

Page 107

30

6

Page 102

5
6

Rational numbers: 0.5, 0.125, 0.3;


Irrational numbers: 22, 23,
3. There is no real number that can be multiplied by
itself to result in a negative product.

Algebra Activity
(Follow-Up of Lesson 26)

4. 225 represents the negative square root of 25.

1. There are 2 more possibilities for 2 boys, 1 girl


and 2 more possibilities for 1 boy, 2 girls.
3 boys
BBB

Chapter 2

2 boys,
1 girl
BBG
BGB
GBB

1 boy,
2 girls
BGG
GBG
GGB

Check for Understanding

1. Sometimes; the square root of a number can be


negative, such as 4 and 4, which are both
square roots of 16.
2. Rational numbers are numbers that, when
written as decimals, terminate or repeat.
Irrational numbers do not terminate nor do they
repeat. Sample answer:

P(sum 10)  36
 0.83
The probability of the sum being less than 10 is
or about 83%.

Square Roots and Real Numbers

25  52 S 225  5
5. 21.44 represents the positive square root of 1.44.

3 girls
GGG

1.44  1.22 S 21.44  1.2

52

18. Write each number as a decimal.

16

6. ; 3 49 represents the positive and negative square


roots of
16
49

147 22 and 1649  147 22

;3

230  5.477225575 . . . or about 5.48

16
.
49

16
49

;

59  5.444444444 . . . or about 5.44


13  13.0

4
7

1
230

7. 232 represents the positive square root of 32.


32 

5.662

0.18 6 5.44 6 5.48 6 13.0


The numbers arranged in order from least to
1
4
greatest are 30, 59, 230, 13.

S 232  5.66

8. Because 264  8, this number is an integer


and a rational number.
9. Because 8 and 3 are integers and 8  3  2.66666 . . .
is a repeating decimal, this number is a rational
number.
10. Because 228  5.29150262 . . . , which is not a
repeating or terminating decimal, this number is
irrational.
56
11. Because 7  8, this number is a natural number,
a whole number, an integer, and a rational number.
12.
6

5

4

3

19. C; Replace a in 2a 6 


?

22 6 

21. 281 represents the positive square root of 81.


81  92 S 281  9
22. 25.29 represents the positive square root of 5.29.
5.29  2.32 S 25.29  2.3

23. 26.25 represents the positive square root of 6.25.


6.25  2.52 S 26.25  2.5
24. 278 represents the negative square root of 78.
78  8.832 S 278  8.83
25. 294 represents the negative square root of 94.

 0.333333 . . .

94  9.702 S 294  9.70

1
.
3

36

26. ; 3 81 represents the positive and negative square

15. You can use a calculator to find an approximation


2
for 9.
2
9

roots of
36
81

 0.222222 . . .

0.2  0.222222 . . .
Therefore,

2
9

100

27. ; 3 196 represents the positive and negative


square roots of
100
196

6 26.

11014 22 and

100

10

; 3 196  ; 14

17. Write each number as a decimal.


1
8
1
8

;3

26  2.449489743 . . .
Therefore,

169 22 and 8136  169 22

36

 0.166666 . . .
1
6

36
.
81

; 3 81  ; 9

 0.2.

16. You can use a calculator to find approximations


1
for 6 and 26.
1
6

Practice and Apply

49  72 S 249  7

The heavy arrow indicates that all points to the


right of 7 are included in the graph. The dot at
7 indicates that 7 is included in the graph.
14. You can use a calculator to find an approximation
1
for 3.
Therefore, 0.3 6

20. 249 represents the positive square root of 49.

13.

1
3

with 2.

22

Pages 107109

2

1
2a

1.41 6 0.71

The heavy arrow indicates that all numbers to the


left of 3.5 are included in the graph. The circle at
3.5 indicates 3.5 is not included in the graph.
8 7 6 5 4 3 2 1

 0.1825741858 . . . or about 0.18

100
.
196
100

196

11014 22

;7

 0.125
 0.3535533906 . . . or about 0.35

0.15  0.15151515 . . . or about 0.15


15  15.0
15.0 6 0.125 6 0.15 6 0.35
The numbers arranged in order from least to
1
1
greatest are 15, 8, 0.15, 3 8.

53

25

25

28.

3 14 represents the positive square root of 14.


9
9
 0.802 S 3 14  0.80
14

29.

3 42 represents the positive square root of 42.


25
25
 0.772 S 3 42  0.77
42

Chapter 2

49. Because  3.141592654 . . . , which is not a


repeating or terminating decimal, this number is
irrational.
d
50. Replace d in 3 16 with 28.

30. ; 2820 represents the positive and negative


square roots of 820.
820  28.642 and 820  (28.64)2
; 2820  ; 28.64
31. ; 2513 represents the positive and negative
square roots of 513.
513  22.652 and 513  (22.65)2
; 2513  ; 22.65
32. Because 222  4.69041576 . . . , which is not
a repeating or terminating decimal, this number
is irrational.
36
33. Because 6  6, this number is a natural number,
a whole number, an integer, and a rational number.
34. Because 1 and 3 are integers and 1  3 
0.333333 . . . is a repeating decimal, this number
is a rational number.
35. Because 5 and 12 are integers and (5  12) 
0.416666 . . . is a repeating decimal, this
number is a rational number.

28

3 16  1.32
The ball would take 1.32 s to reach the ground.
3
51. Replace d in 224d with 434.

3 24 1 43 4 2  21050  32.40
3

Jerome should not get a ticket. He was traveling


at about 32.4 mph.
52.
14 13121110 9 8 7 6

The heavy arrow indicates that all numbers to


the right of 12 are included in the graph. The
circle at 12 indicates that 12 is not included in
the graph.
53.
1

82
20

36. Because 3  2.024845673 . . . , which is not a


repeating or terminating decimal, this number is
irrational.
37. Because 246  6.782329983 . . . , which is
not a repeating or terminating decimal, this
number is irrational.
38. Because 210.24  3.2, which is a terminating
decimal, this number is a rational number.
39. Because 54 and 19 are integers and 54  19 
2.8421052631579 is a terminating decimal, this
number is a rational number.
40. Because 3 and 4 are integers and (3  4) 
0.75 is a terminating decimal, this number is a
rational number.
41. Because 220.25  4.5, which is a terminating
decimal, this number is a rational number.
18
42. Because 3  6, this number is a natural number,
a whole number, an integer, and a rational
number.
43. Because 22.4025  1.55, which is a terminating
decimal, this number is a rational number.
44. Because 68 and 35 are integers and 68  35 
1.9428571428571 is a terminating decimal, this
number is a rational number.
45. Because 6 and 11 are integers and
6  11  0.54545454 . . . is a repeating decimal,
this number is a rational number.
46. Because 25.5696  2.36, which is a terminating
decimal, this number is a rational number.

54.
10

9

The heavy arrow indicates that all points to the


right of 10.2 are included in the graph. The dot
at 10.2 indicates that 10.2 is included in the
graph.
55.
2

1

The heavy arrow indicates that all numbers to


the left of 0.25 are included in the graph. The
circle at 0.25 indicates 0.25 is not included in
the graph.
56.
5 4 3 2 1

The heavy arrows indicate that all numbers to


the right and left of 2 are included in the graph.
The circle at 2 indicates that 2 is not included
in the graph.
57.
8 6 4 2

The heavy line and arrows indicate that all numbers


to the right and left of 6 and 6 are included in the
graph. The circles at 6 and 6 indicate that 6 and
6 are not included in the graph.
58. You can use a calculator to find an approximation
for 15.
25  2.23606797 . . .
5.72  5.727272 . . .
Therefore, 5.72 7 25.
59. You can use a calculator to find an approximation
for 18.

78

47. Because 3 42  1.36277028 . . . , which is not a


repeating or terminating decimal, this number is
irrational.
48. Because 29.16  3.02654919 . . . , which is
not a repeating or terminating decimal, this
number is irrational.

Chapter 2

The heavy arrow indicates that all points to the


left of 8 are included in the graph. The dot at
8 indicates that 8 is included in the graph.

18  2.82842712 . . .
2.63  2.636363 . . .
Therefore, 2.63 6 28.

54

67. Write each number as a decimal.


4.83  4.83838383 . . . or about 4.84
0.4
 2.82842712 . . . or about 2.83

60. You can use a calculator to find approximations for


1
1
and
.
7
1
7
1
27

27

 0.142857142 . . .

28
3

8  0.375
4.84 6 0.375 6 0.4 6 2.83
The numbers arranged in order from least to
3
greatest are 4.83, 8, 0.4, 18.

 0.37796447 . . .

Therefore,

1
7

1
27

61. You can use a calculator to find approximations for


2
2
and
.
3
2
3
2
23

23

68. Write each number as a decimal.

 0.666666 . . .

165  8.06225774 . . . or about 8.06


2
65  6.4

 1.15470053 . . .

Therefore,

2
3

2
23

127  5.19615242 . . . or about 5.20


8.06 6 6.4 6 5.20
The numbers arranged in order from least to
2
greatest are 165, 65, 127.
69. Write each number as a decimal.
2122  11.0453610 . . . or about 11.05
4
79  7.444444 . . . or about 7.44

62. You can use a calculator to find approximations


1
for
and 131
.
31
1

231

231
231
31

 0.17960530 . . .
 0.17960530 . . .

Therefore,

1
131

131
.
31

2200  14.1421356 . . . or about 14.14


7.44 6 11.05 6 14.14
The numbers arranged in order from least to
4
greatest are 79, 1122, 1200.

63. You can use a calculator to find an approximation


12
for 2 .
22
2
1
2

 0.707106781 . . .

70. Replace h in 1.41h with 1500.


1.421500  1.4  38.73
 54.222
The tourists can see about 54.2 miles.
71. Replace h in 1.41h with 135.
1.4 2135  1.4  11.62
 16.268
Replace h in 1.41h with 85.

 0.5

Therefore,

22
2

7 2.

64. Write each number as a decimal.


20.42  0.648074069 . . . or about 0.648
0.63  0.63636363 . . . or about 0.636
14
 0.66666666 . . . or about 0.667
3
0.636 6 0.648 6 0.667
The numbers arranged in order from least to
14
greatest are 0.63, 10.42, 3 .

1.4185  1.4  9.22


 12.908
Marissa can see about 16.268  12.908  3.36 or
about 3.4 miles farther than Dillan.
72. Replace h in 1.41h with 120.

65. Write each number as a decimal.


10.06  0.244948974 . . . or about 0.245
0.24  0.24242424 . . . or about 0.242
19
12

1.41120  1.4  10.95


 15.33
The lighthouse keeper cannot see the boat. The
lighthouse keeper can only see about 15.3 miles.
73. They are true if q and r are positive and q 7 r.

 0.25

0.242 6 0.245 6 0.25


The numbers arranged in order from least to
19
greatest are 0.24, 10.06, 12 .
66. Write each number as a decimal.
1.46  1.46464646 . . . or about 1.46
0.2
 1.41421356 . . . or about 1.41
12
1

6  0.1666666 . . . or about 0.17


1.46 6 0.17 6 0.2 6 1.41
The numbers arranged in order from least to
1
greatest are 1.46, 6, 0.2, 12.

55

Chapter 2

74. The area of a square is given by A  s2 where s is


the side length. If A  s2 then 1A  s. To find
the side lengths, find the square root of the area.
1  12 S 21  1
4  22 S 24  2
9  32 S 29  3
16  42 S 216  4
25  52 S 225  5

81. There are 20 even numbers in a deck of cards,


and there are 52  20 or 32 cards that are not
even numbers.
20

odds of an even number  32


5

8
The odds of selecting an even number from a deck
of cards are 5:8.
82. There are 12 face cards in a deck of cards, and
52  12 or 40 cards that are not face cards.

The perimeter of a square is 4 times the side


length. To find the perimeter of each square,
multiply each of the side lengths by 4.
414
428
4  3  12
4  4  16
4  5  20

40

odds against a face card  12




The odds against selecting a face card from a deck


of cards are 10:3.
83. There are four aces in a deck of cards, and 52  4
or 48 cards that are not aces.

Squares
Area (Units2 )
1
4
9
16
25

Side Length
1
2
3
4
5

odds against an ace 


Perimeter
4
8
12
16
20

84.
85.

75. The length of the side is the square root of the area.
76. The perimeter of a square is 4 times the side
length. The side length of a square is the square
root of the area. Therefore, the perimeter of a
square is 4 times the square root of the area. For
a square whose area is a units2, the perimeter is
41a units.
77. Sample answer: By using the formula Surface

86.

87.

height  weight

Area  3
, you need to use square
3600
roots to calculate the quantity. Answers should
include the following.
You must multiply height by weight first.
Divide that product by 3600. Then determine
the square root of that result.
Sample answers: exposure to radiation or
chemicals; heat loss; scuba suits
Sample answers: determining height, distance

88.

78. B; 27
79. B; Write each number as a decimal.
3

6  0.5
6

3  2
0.5 7 2

Page 109

48
4
12
1

The odds against selecting an ace from a deck of


cards are 12:1.
Sample answer: Mean; the median and mode are
too low.
4(7)  3 (11)  28  33
 28  (33)
 ( 028 0  033 0 )
 (28  33)
 61
3(4)  2(7)  12  (14)
 ( 012 0  014 0 )
 (12  14)
 26
1.2(4x  5y)  0.2(1.5x  8y)
 1.2(4x)  1.2(5y)  0.2(1.5x)  0.2(8y)
 1.2(4)x  1.2(5)y  0.2(1.5)x  0.2(8)y
 4.8x  6y  (0.3)x  1.6y
 4.8x  6y  0.3x  1.6y
 4.8x  0.3x  6y  1.6y
 (4.8  0.3)x  (6  1.6)y
 5.1x  7.6y
4x(y  2z)  x(6z  3y)
 4x(y)  4x(2z)  x(6z)  x(3y)
 4(1)xy  4(2)xz  1(6)xz  1(3)xy
 4xy  (8)xz  6xz  (3)xy
 4xy  8xz  6xz  3xy
 4xy  3xy  8xz  6xz
 (4  3)xy  (8  6)xz
 7xy  14xz

Chapter 2 Study Guide and Review

Maintain Your Skills

80. There are two red 4s in a deck of cards, and there


are 52  2 or 50 cards that are not a red 4.

Page 110

Vocabulary and Concept Check

1. true; 26 is twenty-six units from zero in the


negative direction.
2. true; definition of a rational number
3. true; 2144 represents the principal square root
of 144.
144  122 S 2144  12

odds of a red 4  50
1

 25
The odds of selecting a red 4 from a deck of cards
are 1:25.

Chapter 2

10
3

56

22. 2  10  2  (10)
 ( 02 0  010 0 )
 (2  10)
 12
23. 9  (7)  9  (7)
97
 16
24. 1.25  0.18  1.07
25. 7.7  (5.2)  7.7  (5.2)
  7.7  5.2
 ( 07.7 0  05.2 0 )
 (7.7  5.2)
  2.5

4. false; Because 2576  24, this number is an


integer and a rational number.
5. true; 152  225
6. false; sample answer: 3
7. false; sample answer: 0.6 or 0.666 . . .
8. true; key concept of multiplication

Pages 110114

Lesson-by-Lesson Review

9.
4 32 1 0 1 2 3 4 5 6

10.
3 2 1

6 5 4 3 2 1

26.

11.
12. 32  0y  3 0  32  08  3 0
 32  05 0
 32  5
 27
13. 3 0x 0  7  3 04 0  7
 34  7
 12  7
5
14. 4  0z 0  4  09 0
49
 13
15. 46  y 0x 0  46  8 04 0
 46  8 # 4
 46  32
 14
16. 4  (4)  ( 04 0  04 0 )
 (4  4)
0
17. 2  (7)  ( 07 0  02 0 )
 (7  2)
 5
18. 0.8  (1.2)  ( 00.8 0  01.2 0 )
 (0.8  1.2)
 2
19. 3.9  2.5  ( 03.9 0  02.5 0 )
 (3.9  2.5)
 1.4
1

1 12

5
6

1 12

1 12

1 12

 2  2  2
9

22


10
2

5

27.

1
8

1 2  243  11624 2
3
16
 24  1 24 2
2
3

16

 24  24

1 0 1624 0  0 243 0 2
16
3
  1 24  24 2



13

 24

28. (11)(9)  99


30. 8.2(4.5)  36.9
32.

3
4

29. 12(3)  36


31. 2.4(3.6)  8.64

1 1 21

 12  48

21

33. 3 10  30

 10

 16

34. 8(3x)  12x  8(3)x  12x


 24x  12x
 (24  12)x
 12x
35. 5(2n)  9n  5(2)n  9n
 10n  9n
 (10  9)n
 1n
n
36. 4(6a)  (3)(7a)  4(6)a  (3)(7)a
 24a  21a
 (24  21)a
 45a

1 12
2
1
  1 0 8 0  0 8 0 2
2
1
 18  8 2
2

20. 4  8  8  8

21.

9
2

37.

54
6

38.

74
8

 54  6
 9

 8

1 22
5
2
  1 0 6 0  0 6 0 2
5
2
  16  6 2
5

 3  6  6

 (74  8)

 9.25
39. 21.8  (2)  10.9
40. 7.8  (6)  1.3
41. 15 

6

134 2  15  43


2

60
3

 20

57

Chapter 2

42.

21
24

21

52. There is 1 S and 12 total letters.

 3  24  1

P(S)  12

63

 24

43.

14  28x
7

21
8

 0.08
1
The probability of selecting an S is 12 or about 8%.

or 28

 (14  28x)  (7)

1 12
1
1
 14 1 7 2  28x 1 7 2

53. There are 3 Es and 12 total letters.

 (14  28x) 7

P(E)  12
1

 4 or 0.25

 2  (4x)
 2  4x
44.

5  25x
5

The probability of selecting an E is

115 2
1
1
 5 1 5 2  25x 1 5 2

10

P(not N)  12
5

6
 0.83
The probability of selecting a letter that is not
N is 5 or about 83%.
6
55. There are 2 Rs and 1 P. So there are 2
1 or 3 letters that are R or P and 12 total letters.

 1  5x
4x  24y
4

 (4x  24y)  4

114 2
1
1
 4x 1 4 2  24y 1 4 2

 (4x  24y)

P(R or P)  1

 1x  6y
 x  6y
46. xz  2y  (4)(3)  2 (2.4)
 12  4.8
 16.8
47. 2

 4 or 0.25
The probability of selecting an R or P is

12yz 2  212 324 2


4.8
 2 1 3 2

2x  z
4

 3y 


50

10

 39
The odds of selecting a dime are 10:39.
57. There are 90 pennies, and there are 75  50  30
or 155 coins that are not a penny.
90

odds of a penny  155


18

 31


 2.75  7.2
 4.45
49. The lowest value is 12, and the highest value is
30, so use a scale that includes those values.
Place an  above each value for each occurance.

10

15

20

The odds of selecting a penny are 18:31.


58. There are 90  50  30 or 170 coins that are not
a nickel, and there are 75 nickels.
odds of not a nickel 


25

170
75
34
15

The odds of selecting a coin that is not a nickel


are 34:15.
59. There are 75  50 or 125 coins that are a nickel
or a dime, and there are 90  30 or 120 coins that
are not a nickel or a dime.


  

30

The greatest common place value is tens, so the


digits in the tens place are the stems.
Stem Leaf
1 222344555566
77778899999
2 01112668
3 0
1 0 2 = 12
50. Sample answer: Mean; the median and mode are
too low.
51. Sample answer: Median; it is closest in value to
most of the data.

Chapter 2

or 25%.

odds of a dime  195

2(4)  3
 3(2.4)
4
8  3

7.2
4
11
 7.2
4


  
    
  


1
4

56. There are 50 dimes, and there are 90  75  30


or 195 coins that are not a dime.

 2(1.6)
 3.2

48.

or 25%.

54. There are 10 letters that are not N and 12 total


letters.

 (5  25x)  5
 (5  25x)

45.

1
4

125

odds of a nickel or a dime  120


25

 24
The odds of selecting a coin that is a nickel or a
dime are 25:24.
60. 2196 represents the positive square root of 196.
196  142 S 2196  14
61. ;21.21 represents the positive and negative
square roots of 1.21.
1.21  1.12 and 1.21  (1.1)2
;21.21  ;1.1

58

4.  0x 0  38   02 0  38
 2  38
 40
5. 34  0x  21 0  34  07  21 0
 34  014 0
 34  14
 20
6. 12  0x  8 0  12  01.5  8 0
 12  06.5 0
 12  6.5
 5.5
7. 19  12  ( 019 0  012 0 )
 (19  12)
 7
8. 21  (34)  21  (34)
 21  34
 ( 034 0  021 0 )
 (34  21)
 13
9. 16.4  (23.7)  ( 023.7 0  016.4 0 )
 (23.7  16.4)
 7.3
10. 6.32  (7.41)  6.32  (7.41)
 6.32  7.41
 13.73

62. 2160 represents the negative square


root of 160.
160  12.652 S 2160  12.65
4

63. ; 3 225 represents the positive and negative


4
square roots of 225.
4
225


4

1152 22 and 2254  1152 22


2

; 3 225  ; 15

64. Because 16 and 25 are integers and 16  25  0.64


is a terminating decimal, this number is a rational
number.
264

65. Because 2  4, this number is a natural


number, a whole number, an integer, and a
rational number.
66. Because 248.5  6.96419413 . . ., which is
not a repeating or terminating decimal, this
number is irrational.
67. You can use a calculator to find an approximation
1
for
.
249
1
 0.125
8

1
249

 0.142857142 . . .

Therefore,

1
8

1
249

33

70. Replace d in
93

3 216

 36

3 3.

13. 5(19)  95


15. 96  (0.8)  120

d3

729

3 216

38

20

1 0 21 0 0 2036 0 2
20
21
  1 36  36 2

3 216 with 9.

1 20 2
20
21
 36  1 36 2
21

  36 

1 52

21

3 3  0.577350269 p
34

 36  36

 0.866025403 p

Therefore,

1 0 7 0 0 166 0 2
7
6
  1 16  16 2

12. 12  9  36  36

7 9.

69. You can use a calculator to find approximations


3
1
for 3 4 and 3 3.
3
4

 16

 0.4444444 . . .

Therefore,

  16 

3 3  0.816496580 . . .
4
9

11. 16  8  16  16

68. You can use a calculator to find approximations


2
4
for 3 3 and 9.

1 12

17. 8  (5)  8  5
1

 40

27

14. 56  (7)  8


16. (7.8)(5.6)  43.68
15

15

18. 32  4  32  3


60

 96
5

 1.84
The worst part of the hurricane will last about
1.8 hours.

 8
19. 5(3x)  12x  5(3)x  12x
 15x  12x
 (15  12)x
 27x
20. 7(6h  h)  7(6h)  7(h)
 7(6)h  7h
 42h  7h
 (42  7)h
 35h
21. 4m(7n)  (3d)(4c)  4(7)mn  3(4)dc
 28mn  (12)dc
 28mn  12cd

Chapter 2 Practice Test


Page 115
1. absolute value
2. rational
3. sample space

59

Chapter 2

22.

36k
4

31. The lowest value is 58, and the highest value is


74, so use a scale that includes these values.
Place an  above each value for each occurance.

 36k  4
1

 36k  4
 36
 9k
23.

9a  27
3

114 2k


 


    
 

 (9a  27)  (3)

1 12
1
1
 9a 1 3 2  27 1 3 2

58 60 62 64 66 68 70 72 74

 (9a  27) 3

32. Sample answer: The median and mode can be


used to best represent the data. The mean is too
high.
33. B; There are 7 hard-rock songs and 8  7  5 or
20 total songs.

 3a  (9)
 3a  9
24.

70x  30y
5

 (70x  30y)  (5)

1 2
1
 70x 1 2  30y 1 5 2
 (70x  30y)

P(hard-rock)  20 or 0.35

1
5

The probability a hard-rock song will be playing


7
is 20 or 35%.

1
5

 14x  (6y)
 14x  (6y)
 14x  6y

Chapter 2 Standardized Test Practice

25. 264 represents the negative square root of 64.

Pages 116117

64  82 S 264  8

1. B; 518.4  (9  8)  518.4  72
 7.2
2. C; 20 in 10 minutes, 40 in 20 minutes, 60 in
30 minutes, 80 in 40 minutes. Extending the
pattern would give 100 in 50 minutes and 120 in
60 minutes.
3. C; The points on the number line show 1, 0, 1,
2, and 3.
4. D; 0 4 0  4
04 0  4
07 0  7
09 0  9
4 6 7 6 9
5. A; 3.8  4.7  ( 04.7 0  03.8 0 )
 (4.7  3.8)
 0.9
6. D; 3(2m)  7m  3(2)m  7m
 6m  7m
 (6  7)m
 13m
7. D; The least value is 3|1  31.
8. A; 1 box has a prize and 4  1 or 3 boxes do not
have a prize.

26. 23.61 represents the positive square root of 3.61.


3.61  1.92 S 23.61  1.9
16

27. ; 3 81 represents the positive and negative square


16
roots of 81.
16
81

149 22 and 8116  149 22

16

; 3 81  ; 9

28. You can use a calculator to find an approximation


1
for
.
23

1
23
1
3

 0.577350269 p
 0.33333333 p

Therefore,

23

7 3.

29. You can use a calculator to find an approximation


for

3 2.

3 2  0.7071067812
8
11

 0.727272727 p

Therefore,

32

8
.
11

odds of choosing a prize  3

30. You can use a calculator to find approximations


23
for 20.56 and 2 .

The odds of choosing a prize are 1:3.


9. B; 210  3.16
10. The length of the label will be the circumference
of the jar.
2 r  2(3.14)(4)
 6.28(4)
 25.12
The length of the label is 25.12 cm.

20.56  0.748331477 p
23
2

 0.866025403 p

Therefore, 20.56 6

Chapter 2

23
.
2

60

11.

5  1
4  12  3  2

4
 12  3  2
4
4  4  2
4
4  8
4
12
1
3

18. D; 49  72 and 49  (7)2 a could be 7 or 7.


19a.
cost per
number
monthly
Total cost 1is
minute 123
times 14243
of minutes 123
plus 14243
fee.
14243
23
14243

4





C

m

y

x
or C  my  x
19b. For plan A, evaluate my  x with m  0.3,
y  150, and x  5.95.
my  x  0.3  150  5.95
 45  5.95
 50.95
For plan B, evaluate my  x with m  0.1,
y  150, and x  12.95.
my  x  0.1  150  12.95
 15  12.95
 27.95
For plan C, evaluate my  x with m  0.08,
y  150, and x  19.99.
my  x  0.08  150  19.99
 12  19.99
 31.99
Plan B is the least expensive.
20a. Without a key, you cannot determine what the
values are.
20b. If the key is 3 02  3.2, then the data are ten
times as great as they would be if the key is
3 02  0.32.

12. Replace m in 4m  3  9 with each value in the


replacement set.
4m  3  9
4  0  3  9 S 3  9
42  3 9S5  9
43  3 9S9  9
4  5  3  9 S 17  9

m
0
2
3
5

True or False?
false
false
true
false

The solution of 4m  3  9 is 3.
three times the
2p
plus
difference
of m and n
123 123 1444424443
2p 
3  (m  n)
or 2p  3(m  n)
14. Hypothesis; 3x  3 7 24
15. C; x is x units from zero, and x is x units away
from zero.
0x 0  0x 0
16. B; y 6 x
1
1
 y 6 x  xy
xy
13.

1
x

1
y

17. A; n 7 1
1
nn 7 1n
1 7 n

61

Chapter 2

PQ249-6481F-03[062-092] 26/9/02 4:29 PM Page 62 Sahuja Ahuja_QXP_06:Desktop Folder:Chandra:Algebra_FNL_Delivery:

Chapter 3
Page 119

Solving Linear Equations


16. The base is 300, and the part is 21. Let p
represent the percent.

Getting Started

1. Greater than implies add, and half of implies


1
multiply by 2. So, the expression can be written as
1
t  5.
2

a
b
21
300

2. Product implies multiply, and divided by implies


divide. So, the expression can be written as
7s  8y.
3. Sum implies add, times implies multiply, and
square implies raise to the second power. So, the
expression can be written as 3a  b2.
4. Decreased by implies subtract, so the expression
can be written as w5  37.
5. Times implies multiply, and subtracted from
implies subtract. So, the expression can be
written as 95  9y.
6. Sum implies add, and divided by implies divide.
So, the expression can be written as (r  6)  12.
12
4

7. 3  6 

 18 

2  6  4
2




2100
300

19  5
7

a
b
15
5

14
7

1500
5

12
4

a
b
12
60

1200
60

a
b
16
10

1600
10

 100
p

 100

60p
60

 100
p

 100

10p
10

160  p
Sixteen is 160% of 10.
20. The base is 50, and the part is 37.5. Let p
represent the percent.
a
b
37.5
50

 100

 100
p

 100

37.5(100)  50p
3750  50p

 100

5(100)  20p
500  20p

3750
50

50p
50

75  p
Thirty-seven and one half is 75% of 50.

20p
20

25  p
Five is 25% of 20.

Chapter 3

5p
5

16(100)  10p
1600  10p

3

0
15. The base is 20, and the part is 5. Let p represent
the percent.

20  p
Twelve is 20% of 60.
19. The base is 10, and the part is 16. Let p represent
the percent.

500
20

 100

12(100)  60p
1200  60p

14. 4 (24)  2 (12)  6  6

a
b
5
20

 100

300  p
Fifteen is 300% of 5.
18. The base is 60, and the part is 12. Let p represent
the percent.

23
5
1

300p
300

15(100)  5p
1500  5p

12  4
2
8
2

3

7p
Twenty-one is 7% of 300.
17. The base is 5, and the part is 15. Let p represent
the percent.

4
11. (25  4)  (22  1)  21  (4  1)
 21  3
7
12. 36  4  2  3  9  2  3
73
 10
13.

 100

21(100)  300p
2100  300p

 18  3
 15
8. 5(13  7)  22  5(6)  22
 30  22
8
9. 5(7  2)  32  5(5)  32
 5(5)  9
 25  9
 16
10.

 100

62

PQ249-6481F-03[062-092] 26/9/02 4:29 PM Page 63 Sahuja Ahuja_QXP_06:Desktop Folder:Chandra:Algebra_FNL_Delivery:

5.

the sum of is the


Five times m and n same as seven times n.
123 123 14243 1
424
3 123 123 {
5

(m  n)

7

n
The equation is 5(m  n)  7n.
6. Words: Area equals one half times the base
times the height.
Variables: Let A  area, b  base, and h  height.

Writing Equations

3-1
Page 122

Algebra Activity

1. The area of the front is the product of the length


and the height or lh.
2. The area of the back is the product of the length
and the height or lh.
3. The area of one side is the product of the width
and the height or wh.
4. The area of the other side is the product of the
width and the height or wh.
5. The area of the top is the product of the length
and the width or lw.
6. The area of the bottom is the product of the
length and the width or lw.
surface area is
7. The
the sum of the areas of the faces.
1
4444244
443 { 1444444442444444443


one
the
the
Area
half
base
height.
1
23 equals
123 1
23 times
123 1
23 times
123 123

Formula:

Circumference equals the product of


two, pi, and the radius.
Variables: Let C  circumference and r  radius.
Circumference
1442443

Fourteen
9.

1
b
3
{

plus

equals

3
4
{

six times d.

2a
{

One-third of b minus three-fourths equals two times a.

10. Sample answer: The original cost of a suit is c.


After a $25 discount, the suit costs $150. What is
the original cost of the suit?
Darius
number of pounds
11.
current
weight
14
442444
3
155

plus
1
23


he wants
to gain
144
442444
43
g

equals
1
4243


160.
123
160

The equation is 155  g  160.


12. 155  g  160
Find g mentally by asking, What number added
to 155 equals 160?
g5
Darius needs to gain 5 pounds to reach his goal.

Pages 124126
13.

Two
hundred
1
4
44
244
43
200

Practice and Apply


minus
123


three
123
3

times
123


is equal to
14
424
43


nine.
123
9

The equation is 200  3x  9.


14. Rewrite the sentence so it is easier to translate.
Twice
plus
three
times
s 1
is44244
identical to
1
424
3r 12
3 1
23 1
23 {
3 thirteen.
1
424
3
2r

3

s

13

The equation is 2r  3s  13.


15. Rewrite the sentence so it is easier to translate.
One-third
q 1
plus
is44244
as much as
q.
1
4424
43
23 25
{
1
3 twice
1
424
3

30

w
or 30w
2d. No; 1900  52(30)  3460, which is less than
3500.
3. Sample answer: After sixteen people joined the
drama club, there were 30 members. How many
members did the club have before the new
members?
4. 1
Two
t decreased
23 times
123 {
14
4244by
3 eight
123 equals
123 seventy.
14243


the product of two,


pi, and the radius.
144424443

equals
123

Formula:
C

2r
The formula for the circumference of a circle is
C  2r.
8.
14

d

6d
{
{
{
{
{

lh  lh  wh  wh  lw  lw

30

10
30 . 10  300
money
number
per
week
times
of
weeks
14243 123 14
243
30

20
30 . 20  600
She will add $300 in 10 weeks and $600 in
20 weeks.
2c. money
number
per
week
times
of
weeks
14243 123 14
243

7. Words:

Check for Understanding

1. Explore the problem, plan the solution, solve the


problem, and examine the solution.
2a. Misae has $1900 in her account at the
beginning.
2b. money
number
per
week
times
of
weeks
14243 123 14
243

The formula for the area of a triangle is A  2 bh.

or S  2lh  2wh  2lw


8. If s represents the length of the side of a cube,
then the surface area is given by S  2lh  2wh 
2lw with l  s, h  s, and w  s.
S  2lh  2wh  2lw
 2s  s  2s  s  2s  s
 2s2  2s2  2s2
 6s2
The surface area is given by S  6s2.

Page 123

1
2

1
q
3

The equation is

1
q
3

25

2q

 25  2q.

16. 14
The44244
square of
m 123
minus the
cube 43
of n is
sixteen.
{ 1
4243
43
14
424
2
3

n

16
m
The equation is m2  n3  16.

70

The equation is 2t  8  70.

63

Chapter 3

PQ249-6481F-03[062-092] 26/9/02 4:29 PM Page 64 Sahuja Ahuja_QXP_06:Desktop Folder:Chandra:Algebra_FNL_Delivery:

Volume equals the product of , the


square of the radius of the base, and
the height.
Variables: Let V  volume, r  radius of base,
and h  height.

17.

the sum of is equal


Two
times
v and w 1
to43 1
two
z.
123 123 14243
42
23 times
123 {
2

(v  w)

2
 z
The equation is 2(v  w)  2z.
18.
the sum of is the
Half
of
nine and p same as p minus three.
123 { 14243
1
424
3 { 123 123
1

(9

p)

p

3
2

26. Words:

the square of the


the
Volume
times radius of the base times height.
1
424
3 equals
123 
{ 123 14
4424443 123 123
2

Formula: V
  
r

h
The formula for the volume of a cylinder is
V  r2h.
27. Words:
The square of the measure of the
hypotenuse is the square of the
measure of one leg plus the square of
the measure of the other leg.
Variables: Let c  measure of hypotenuse,
a  measure of one leg, and
b  measure of other leg.
Formula:

The equation is 2(9  p)  p  3.


19. Rewrite the sentence so it is easier to translate.
The
divided
the
is the
twice the
number
g 1
by 3 number
h same
as
sum of g and h plus seven.
14243
424
14243
1
424
3 1442443
123 123

2(g  h)

The equation is g  h  2(g  h)  7.


20. Rewrite the sentence so it is easier to translate.
the square of the
the square
the square
Five-ninths times sum of a, b, and c equals
of a
plus
of c.
14243 123 144424443 123 14243 123 14243
5
9

(a  b  c)2

a2

c2

The square of the


the square of the
the square of the
measure
of the hypotenuse
measure
of one43
leg 1
plus
of the other leg.
144
44424444
43 is
{1
444244
23 measure
14444244443

The equation is 9(a  b  c)2  a2  c2.

c2

21. Rewrite the sentence so it is easier to translate.


The Pacific
Ocean
46%
Earth.
{ 1
23 of
{ 123
144
42444
3 is

This formula for a right triangle is c  a2  b2.


28. Words:
Temperature in degrees Fahrenheit
is nine-fifths of the degrees Celsius
plus thirty-two.
Variables: Let F  degrees Fahrenheit and
C  degrees Celsius.
Temperature in
the degrees
degrees Fahrenheit is nine-fifths of Celsius plus thirty-two.
144424443 { 14243 { 14243 123 14243

Formula:

d
30.

32.

minus
2f
{
k2

fourteen
6
{

equals

{

five.
19
{

equals

nineteen.

2a
{
3
p
4
{

six

17

Two times a
33.

plus

k squared plus

32.


{

34.

2
w
5{

53

seventeen equals fifty-three minus j.

7a
{

equals


Three-fourths of p plus

(B  h)

14
{

Two times f
31.

The formula for the volume of a pyramid is


1
V  3Bh.

seven times a
1
2
{

one-half
1
w
2{

minus b.


equals p.


Two-fifths times w equals one-half times w plus three.


35. 7(m
 n)

10n

17
1
424
3
{
1
23
{
{
Seven times
equals
ten
plus seventeen.
the sum of m and n
times n
36. 4(t  s)

5s

12
{
{
{
{
123
Four times the
equals five times s plus twelve.
quantity t minus s

25. Words:

Perimeter is twice the sum of the


lengths of the two adjacent sides.
Variables: Let P  perimeter, a  one side, and
b  an adjacent side.
the sum of the lengths
Perimeter is two times of the two adjacent sides.
44424444
43
14243 { 123 123 144

Formula:
P  2 
(a  b)
The formula for the perimeter of a parallelogram
is P  2(a  b).

Chapter 3

29. {
d

the product of the area


Volume
is one-third
the44424444
base and its height.
14243 times
123 of
144
43
123 {
1
3

9
5

The formula for temperature in degrees Fahrenheit


9
is F  5C  32.

Formula:
A 
b

h
The formula for the area of a parallelogram is
A  bh.
24. Words:
Volume is one-third times the
product of the area of the base and
its height.
Variables: Let V  volume, B  area of base,
and h  height.

V 

b2

P
 0.46  E
The equation is P  0.46E.
22. Rewrite the sentence so it is easier to translate.
The cost
number
per
yard
times
of
yards
plus
3.50
equals 73.50.
14243 123 1
424
31
23 1
23 123
123
1.75

f
 3.50

73.50
The equation is 1.75f  3.50  73.50.
23. Words:
Area is the base times the height.
Variables: Let A  area, b  base, and h 
height.
Area
the
base
times
the height.
1
23 is
{ 1
424
3 1
23 14243

Formula:

a2

64

PQ249-6481F-03[062-092] 26/9/02 4:29 PM Page 65 Sahuja Ahuja_QXP_06:Desktop Folder:Chandra:Algebra_FNL_Delivery:

Plan:

37. A  2h (a  b)

Write the formula for the volume of


a sphere.

The area A of a trapezoid equals one-half times


the product of the height h and the sum of the
bases, a and b.
38.
rt

d
{

The
fourVolume
is thirds times
1
424
3 {
123 123

Rate times time equals distance.


39. Sample answer: Lindsey is 7 inches taller than
Yolanda. If 2 times Yolandas height plus
Lindseys height equals 193 inches, find Yolandas
height.
40. Sample answer: The price of a new backpack is p
dollars and the tax on the backpack is 0.055p. If
the total cost of the backpack and the tax is
$31.65, what is the price of the backpack?
41. Words:
Volume equals one-third times the
product of , the square of the
radius of the base, and the height.
Variables: Let V  volume, r  radius of base,
and h  height.

Solve:






1
3

r2

h
1

Solve:

1
3

1912

V  3r2h
1

 3 (10)2(30)
1

 3 (100)(30)

11003 2(30)

 1000
 3141.6
Examine: The volume of a cone with r  10 and
h  30 is about 3142 cm3.
43. Words:
Volume is four-thirds times  times
the radius of the sphere cubed.
Variables: Let V  volume and r  radius of
sphere.

Formula:

fourVolume is thirds times


1
424
3 { 123 123
4
V  3


r3

1928

a

(4a  15)
 60
The equation is a  (4a  15)  60.
50. Find the solution for a  (4a  15)  60. Start by
letting a  10 and then adjust values up or down
as needed.
a

a  (4a  15)  60
?

10 10  (4  10  15)  60 S 65  60


{

times
123

the radius
of the
sphere
cubed.
14243

r3

The formula for the volume of a sphere is V  3r3.


44. Explore:

4
r3
3
4
 (4)3
3
4
 (64)
3
4
(64)
3
256

3

4

a

15
The expression is 4a  15.
49. Rewrite the sentence so it is easier to translate.
The advertising
15 more than 4 times
portion
plus
advertising portion {
is {
60.
1442443
1
23 the
14444244443

 3 (100)(30)


times
123

The equation is 1912  y  1928.


47. 1912  y  1928
Ask yourself, What number added to 1912 equals
1928?
y  16
There were 16 years between the first Tarzan
story and the naming of the town.
48. Rewrite the phrase so it is easier to translate.
the advertising
Four
times
portion
plus
1
23
1
23 1442443
1
23 15
{

(r2h)

The year Tarzan


the number of years before
the year the
was published plus
the town was renamed
equals town was renamed.
1
4442444
3 123 1444442444443 123 144424443

42. Explore: You know the formula for the volume


of a cone. You need to evaluate the
formula with r  10 and h  30.
Plan:
Write the formula for the volume of a
cone.
the product of
, the square
The
volume
equals 1
one-third
times 14
of 424
r, and43
h.
14
4244
3 123
4243 123





1912

y
The expression is 1912  y.
46. Rewrite the sentence so it is easier to translate.

The formula for the volume of a cone is V  3 r2h.

 268.1
The volume of a sphere with r  4
is about 268 in3.
45. Rewrite the phrase so it is easier to translate.
the number of years
The year Tarzan
before the town was
was
first
published
plus
named Tarzana
144424443 123 144424443

the square of the


the
Volume equals one-third times  times radius of the base times height.
1
424
3 123 14243 123 { 123 14444244443 123 123


4
3

Examine:

Formula:

V

the radius
of the
sphere
cubed.
14243

5  (4  5  15)  60 S 40  60

7  (4  7  15)  60 S 50  60

?
?

8  (4  8  15)  60 S 55  60
?

9  (4  9  15)  60 S 60  60

Reasonable?
too high
too low
too low
almost
true

The solution of a  (4a  15)  60 is 9. Therefore,


9 minutes were spent on advertising.
51. See students work.

You know the formula for the volume


of a sphere. You need to evaluate the
formula with r  4.

65

Chapter 3

PQ249-6481F-03[062-092] 26/9/02 4:29 PM Page 66 Sahuja Ahuja_QXP_06:Desktop Folder:Chandra:Algebra_FNL_Delivery:

63. 12d  3  4d  12d  4d  3


 (12d  4d)  3
 (12  4)d  3
 8d  3
2
64. 7t  t  8t  7t2  (t  8t)
 7t2  (1  8)t
 7t2  9t
65. 3(a  2b)  5a  3(a)  3(2b)  5a
 3a  6b  5a
 3a  5a  6b
 (3a  5a)  6b
 (3  5)a  6b
 8a 6b
66. 5(8  3)  7  2  5(5)  7  2
 25  14
 39
67. 6(43  22)  6(64  4)
 6(68)
 408
68. 7(0.2  0.5)  0.6  7(0.7)  0.6
 4.9  0.6
 4.3
69. 1
70. 1
0.57
5.67
2.80
3.70
9.37
3.37
71. 4
72. 8 9
5.128
9.010
3. 4 0
7.3 5
1. 8 8
1.6 5

52. Words:

Surface area is the sum of the area


of the faces.
Variables: Let S  surface area, a  length of
the sides of an equilateral triangle,
and h  height of a prism. The
height of the equilateral triangle is
a 23
given by 2 .

Surface
area of
area of
area
is
three
times
a
side
plus
two
times
a
base.
1
4243 { 123 123 123 123 123 123 123
Formula:
1 a 23
S
 3

ah

2
 2a 2
The formula for the surface area of this
a2 23

triangular prism is S  3ah  2 .


53. Equations can be used to describe the
relationships of the heights of various parts of a
structure. Answers should include the following.
The equation representing the Sears Tower is
1454  a  1707.
54. B;
a
the
One-fourth of
number
plus
five
equals
number
minus 123
seven.
14243
{ 1
424
3 1
23 1
231
424
31
424
3 123
1
4

The equation is

1
n
4


{

55. D;{
7
Seven

times

Page 126

 5  n  7.
(x  y)
123
the sum of x and y

35
{

equals

35.

Maintain Your Skills

56. 28100 represents the positive square root of


8100.
8100  902 S 28100  90
25

57.  3 36 represents the negative square root of


25
36

156 22 S  3 2536  56

73.

2
3

10

 5  15  15

74.

1
6

13

75.

7
9

 15
25
.
36

366
6

399

76.

3
4

 6  12  12
7

 12

9

58. 290 represents the positive square root of 90.


90  9.492 S 290  9.49

Page 127

Algebra Activity
(Preview of Lesson 32)

59. 255 represents the negative square root of 55.


1. Step 1

55  7.422 S 155  7.42


60. There is 1 six and 6 total outcomes.

Model the equation.

P(6)  6
 0.17
The probability of rolling a 6 is

1
1
6

1

or about 17%.

1

61. There are 3 even numbers and 6 total outcomes.


P(an even number) 


3
6
1
2

1
1

or 0.5

1
1
1
1
1

x57
x  5  (5)  7  (5)
Place 1 x tile and 5 positive 1 tiles on one side of
the mat. Place 7 positive 1 tiles on the other side
of the mat. Then add 5 negative 1 tiles to each
side.

The probability of rolling an even number is 2 or


50%.
62. There are 4 numbers greater than two and 6 total
outcomes.
4

P(a number greater than 2)  6


2

3
The probability of rolling a number greater than
2
2 is 3.

Chapter 3

66

PQ249-6481F-03[062-092] 26/9/02 4:29 PM Page 67 Sahuja Ahuja_QXP_06:Desktop Folder:Chandra:Algebra_FNL_Delivery:

Step 2

Place 1 x tile and 4 positive 1 tiles on one side of


the mat. Place 27 positive 1 tiles on the other
side of the mat. Then add 4 negative 1 tiles to
each side.
Step 2 Isolate the x term.

Isolate the x term.

x
1

1

1

1

1

1

1

1

1

1

x2

1

1


1

Group the tiles to form zero pairs. Then remove


all the zero pairs. The resulting equation is x  2.
2. Step 1 Model the equation.

1

1

x
1 1 1

x  (2)  28
x  (2)  2  28  2
Place 1 x tile and 2 negative 1 tiles on one side of
the mat. Place 28 positive 1 tiles on the other side
of the mat. Then add 2 positive 1 tiles to each side.
Step 2 Isolate the x term.
1

x  23
Group the tiles to form zero pairs. Then remove
all the zero pairs. The resulting equation is x  23.
4. Step 1 Model the equation.

x  (3)  4
x  (3)  3  4  3
Place 1 x tile and 3 negative 1 tiles on one side of
the mat. Place 4 positive 1 tiles on the other side
of the mat. Then add 3 positive 1 tiles to each side.
Step 2 Isolate the x term.
\
x

x  30
Group the tiles to form zero pairs. Then remove
all the zero pairs. The resulting equation is x  30.
3. Step 1 Model the equation.

x7
x
1

1
1
1

1
1

Group the tiles to form zero pairs. Then remove


all the zero pairs. The resulting equation is x  7.

1

1

1

1
1

x  4  27
x  4  (4)  27 (4)

67

Chapter 3

PQ249-6481F-03[062-092] 26/9/02 4:29 PM Page 68 Sahuja Ahuja_QXP_06:Desktop Folder:Chandra:Algebra_FNL_Delivery:

5. Step 1

Group the tiles to form zero pairs. Then remove all


the zero pairs. The resulting equation is x  5.
7. You can model the equation a  b by putting an a
tile on one side of the mat, and putting a b tile on
the other side of the mat. You can add a c tile to
each side of the mat. The resulting equation
would be a  c  b  c.
8. You can model the equation a  b by putting an a
tile on one side of the mat, and putting a b tile on
the other side of the mat. You can add a negative
c tile to each side of the mat. The resulting
equation would be a  (c)  b  (c)
or a  c  b  c.

Model the equation.

x
1

1

1

1

1

1

1

1

1

1

1

x  3  4
x  3  (3)  4  (3)
Place 1 x tile and 3 positive 1 tiles on one side of
the mat. Place 4 negative 1 tiles on the other side
of the mat. Then add 3 negative 1 tiles to each side.
Step 2 Isolate the x term.

3-2
Page 131

1

1

1

1

1

1

1

1

1

1

Group the tiles to form zero pairs. Then remove all


the zero pairs. The resulting equation is x  7.
6. Step 1 Model the equation.

1

1

1

1
1

1

1
1
1

1

1

1

1

x72
x  7  (7)  2  (7)
Place 1 x tile and 7 positive 1 tiles on one side of
the mat. Place 2 positive 1 tiles on the other side of
the mat. Then add 7 negative 1 tiles to each side.
Step 2 Isolate the x term.

x
1
1
1
1
1
1
1

1

1

1

1

1
1

1
1
1

1
1

1

x  5
Chapter 3

Check for Understanding

1. Sample answers: n  13, n  16  29, n  12  25


2. The Addition Property of Equality and the
Subtraction Property of Equality can both be
used to solve equations. The Addition Property
of Equality says you can add the same number
to each side of an equation. The Subtraction
Property of Equality says you can subtract the
same number from each side of an equation.
Since subtracting a number is the same as
adding its inverse, either property can be used
to solve any addition equation or subtraction
equation.
3. (1) Add 94 to each side.
g  94  (94)  75  (94)
g  19
(2) Subtract 94 from each side.
g  94  94  75  94
g  19
4.
t  4  7
t  4  4  7  4
t  3
t  4  7
Check:
?
3  4  7
7  7
The solution is 3.
5.
p  19  6
p  19  19  6  19
p  13
p  19  6
Check:
?
13  19  6
66
The solution is 13.
6.
15  r  71
15  r  15  71  15
r  56
15  r  71
Check:
?
15  56  71
71  71
The solution is 56.

x  7

1

Solving Equations by Using


Addition and Subtraction

68

PQ249-6481F-03[062-092] 26/9/02 4:29 PM Page 69 Sahuja Ahuja_QXP_06:Desktop Folder:Chandra:Algebra_FNL_Delivery:

104  y  67
104  67  y  67  67
171  y
Check:
104  y  67
?
104  171  67
104  104
The solution is 171.
8.
h  0.78  2.65
h  0.78  0.78  2.65  0.78
h  3.43
h  0.78  2.65
Check:
?
3.43  0.78  2.65
2.65  2.65
The solution is 3.43.

14. The average


time
in Europe
1442443

the average
minus
time
in Japan
123 14424
43
35.5

16.8
35.5  16.8  18.7
The difference is 18.7 hours.

7.

2
3

9.
2
3

Pages 132134

 w  12
2

 w  3  12  3
5

w6
2
w
3
2
5 ?
6
3
1
12 
5
solution is 6.

Check:

The

12
1

12
1

12

10. 1
A4
number
minus
twenty-one
42443
1
424
3
1
442443
n

21
n  21  8
n  21  21  8  21
n  13
n  21  8
Check:
?
13  21  8
8  8
The solution is 13.
11. 1
A4
number
increased
by
37
42443 1
4442444
3
1
23

is
123


8.
1
23
8

is
91.
{
123
n

37

91
n  (37)  91
n  (37)  37  91  37
n  54
Check:
n  (37)  91
?
54  (37)  91
91  91
The solution is 54.
12. Words: The average time in Japan plus 8.1
equals the average time in the United
States.
Variable: Let /  the average time in Japan.
The average
the average
time
in
Japan
plus
8.1
equals
time
in the U.S.
1442443 123123 123 1442443
Equation:
/
 8.1

13.
/  8.1  24.9
/  8.1  8.1  24.9  8.1
/  16.8
The average time is 16.8 hours.

Practice and Apply

15.
 9  14
 9  9  14  9
 23
 9  14
Check:
?
23  9  14
14  14
The solution is 23.
16.
s  19  34
s  19  19  34  19
s  15
s  19  34
Check:
?
15  19  34
34  34
The solution is 15.
17.
g  5  33
g  5  5  33  5
g  28
Check:
g  5  33
?
28  5  33
33  33
The solution is 28.
18.
18  z  44
18  z  18  44  18
z  26
18  z  44
Check:
?
18  26  44
44  44
The solution is 26.
19.
a  55  17
a  55  55  17  55
a  38
a  55  17
Check:
?
38  55  17
17  17
The solution is 38.
20.
t  72  44
t  72  72  44  72
t  28
Check:
t  72  44
?
28  72  44
44  44
The solution is 28.
21.
18 61  d
18  61 61  d 61
43  d
18  61  d
Check:
?
18  61  43
18  18
The solution is 43.

24.9

69

Chapter 3

PQ249-6481F-03[062-092] 26/9/02 4:29 PM Page 70 Sahuja Ahuja_QXP_06:Desktop Folder:Chandra:Algebra_FNL_Delivery:

22.

23.

24.

25.

26.

27.

28.

6  m  (3.42)
6  3.42  m  (3.42)  3.42
2.58  m
6  m  (3.42)
Check:
?
6  2.58  (3.42)
6  6
The solution is 2.58.
30.
6.2  4.83  y
6.2  4.83  4.83  y  4.83
11.03  y
6.2  4.83  y
Check:
?
6.2  4.83  11.03
6.2  6.2
The solution is 11.03.
31.
t  8.5  7.15
t  8.5  8.5  7.15  8.5
t  15.65
Check:
t  8.5  7.15
?
15.65  8.5  7.15
7.15  7.15
The solution is 15.65.
32.
q  2.78  4.2
q  2.78  2.78  4.2  2.78
q  6.98
q  2.78  4.2
Check:
?
6.98  2.78  4.2
4.2  4.2
The solution is 6.98.

25  150  q
25  150  150  q  150
125  q
25  150  q
Check:
?
25  150  125
25  25
The solution is 125.
r  (19)  77
r  19  77
r  19  19  77  19
r  96
r  (19)  77
Check:
?
96  (19)  77
77  77
The solution is 96.
b  (65)  15
b  65  15
b  65  65  15  65
b 50
Check:
b  (65)  15
?
50  (65)  15
15  15
The solution is 50.
18  (f )  91
18  f  91
18  f  18  91  18
f  73
18  (f )  91
Check:
?
18  (73)  91
91  91
The solution is 73.
125  (u)  88
125  u  88
125  u  125  88  125
u  37
125  (u)  88
Check:
?
125  [(37) ]  88
88  88
The solution is 37.
2.56  c  0.89
2.56  c  2.56  0.89  2.56
c  3.45
Check:
2.56  c  0.89
?
2.56  3.45  0.89
0.89  0.89
The solution is 3.45.
k  0.6  3.84
k  0.6  0.6  3.84  0.6
k  4.44
k  0.6  3.84
Check:
?
4.44  0.6  3.84
3.84  3.84
The solution is 4.44.

Chapter 3

29.

33.

x46
3

x4464
7

x  112
Check:

x46

3 ? 5
6
5
5


6
6
7
The solution is 112.
3
7
a  5  10
3
3
7
3
a  5  5  10  5
1
a  10
3
7
Check:
a  5  10
1
3 ?
7
10  5  10
7
7
10  10
1
The solution is 10.
1
5
2  p  8
1
1
5
1
2  p  2  8  2
1
p  18
1
5
Check:
2  p  8
1
1 ? 5
2  18  8
5
5
8
8
1
The solution is 18.
7

112  4 

34.

35.

70

PQ249-6481F-03[062-092] 26/9/02 4:29 PM Page 71 Sahuja Ahuja_QXP_06:Desktop Folder:Chandra:Algebra_FNL_Delivery:

2
3

36.
2
3

43.

A
number
eighteen
equals 31.
1424
3 minus
1231
424
3 123
{
Equation:
n

18
 31
n  18  31
n  18  18  31  18
n  49
n  18  31
Check:
?
49  18  31
31  31
The solution is 49.
44.
What
number
equals
18?
14442
4443 decreased
14
44244
4by
3 77
{ 1
424
31
23

 r  9
2

 r  3  9  3
1

r  19
2
3

Check:
2
3

 r  9
1

 19  9
4

9  9
1

The solution is 19.


37.

2
3
2
4
5
3
2
15

v5
4

38.
4

v55
v

2
4
Check:
v5
3
2 ?
2
4
 15  5
3
2
2
3
3
2
The solution is 15.

2
5
2
3
4
5
7
20

w4
3


77
 18
n 77  18
n  77  77  18  77
n  59
n  77  18
Check:
?
59  77  18
18  18
The solution is 59.
45.
A
number
increased
by
16
21.
1424
3 14
4244
31
23 is
{ 1
23
Equation:
n

(16)  21
n  (16)  21
n  (16)  16  21  16
n  5
Check:
n  (16)  21
?
5  (16)  21
21  21
The solution is 5.
46.
A
number
plus
43
is 1
102.
23
14
243 1
23 1
23 {
Equation: n

w44
w

2
3
w4
5
2 ?
7
3
 20  4
5
2
2
5
5
7
solution is 20.

Check:

The

39. First, solve x  7  14.


x  7  14
x  7  7  14  7
x  21
Now, replace x in x  2 with 21.
x  2  21  2
 19
The value is 19.
40. First, solve t  8  12.
t  8  12
t  8  8  12  8
t  20
Now, replace t in t  1 with 20.
t  1  20  1
 19
The value is 19.
41.
The
length
of the rectangle
1444
4424444
43 is 78.
{

n
 (43)  102
n  (43)  102
n  (43)  43  102  43
n  145
Check:
n  (43)  102
?
145  (43)  102
102  102
The solution is 145.
Equation:

x  55
 78
x  55  78
x  55  55  78  55
x  23
x  55  78
Check:
?
23  55  78
78  78
The solution is 23.
42.
The
width of the rectangle
144444244444
3 is 24.
Equation:

y  17
y  17  24
y  17 17  24  17
y  41
y  17  24
Check:
?
41  17  24
24  24
The solution is 41.

Equation:

47.

What
number
minus
1
424
3 123

Equation:

one- is equal
half
to 3
1
23 1424
1
2

negative
three-fourth?
1
442443
3

4

n  2  4
1

n  2  2  4  2
1

n  4
Check:

{ {

1 ?

n  2  4
1

4  2  4

 24

4  4
1

The solution is 4.

71

Chapter 3

PQ249-6481F-03[062-092] 26/9/02 4:29 PM Page 72 Sahuja Ahuja_QXP_06:Desktop Folder:Chandra:Algebra_FNL_Delivery:

58. Words:

The number of volumes at Harvard


University plus the difference equals
the number of volumes at the Library
of Congress.
Variable: Let x  the difference in volumes
between the two libraries.

48.

is equal
19 plus 42 plus a number
to
87.
{ 1
23 { 123 14
4244
3 14
4244
3 {
Equation:19  42 
n

87
19  42  n  87
61  n  87
61  n  61  87  61
n  26
Check:
19  42  n  87
?
19  42  26  87
87  87
The solution is 26.
49. Sometimes; if x  0, x  x  x is true.
50. Always; any number plus 0 is always the number.
51. Words:
The miles per gallon of a luxury car
plus 10 equals the miles per gallon of
a midsize car.
Variable: Let   the miles per gallon of a
luxury car.
The miles per gallon
of a luxury car
plus
equals
144424443
1
23 10
{
123

The number of volumes


the
the number of volumes at
at Harvard University plus difference equals the Library of Congress.
14444244443 123 14243 123 1444442444443

Equation: 13.6 
x

24.0
13.6  x  24.0
13.6  x  13.6  24.0  13.6
x  10.4
The Library of Congress has 10.4 million volumes
more.
59. Words:
The number of volumes at New York
Public plus the difference equals the
number of volumes at Harvard
University.
Variable: Let x  the difference in volumes
between the two libraries.

the miles per gallon


of a midsize car.
144424443

Equation: 
 10 
34
The equation is   10  34.
/  10  34
52.
/  10  10  34  10
/  24
A luxury car travels 24 miles on a gallon of gasoline.
the miles per gallon
53. The miles per gallon
of
a subcompact car
144424443
x

is
{


of a luxury car
144424443
24

plus
123


13.
{
13

37

34
37  34  3
A subcompact car travels 3 miles more.
55. Sample answer: 29 miles; 29 is the average of 24
(for the 8-cylinder engine) and 34 (for the 4-cylinder
engine).
56. Words:
The height of the Great Pyramid plus
the amount of decrease equals the
original height.
Variable: Let d  the amount of decrease.
the amount
plus
of4decrease
1
23 1
42443

Equation: 450

d
The equation is 450  d  481
57.
450  d  481
450  d  450  481  450
d  31
The decrease in height is 31 ft.

Chapter 3

equals
123

The number
of volumes at
the
New York Public 1
plus
difference
144424443
23 14243

the number
of volumes at the
equals Library
of Congress.
123
144424443

Equation: 11.4 
x

24.0
11.4  x  24.0
11.4  x  11.4  24.0  11.4
x  12.6
The Library of Congress has 12.6 million volumes
more.
61. Words:
The total number of volumes is equal
to the sum of the volumes at the
Library of Congress, Harvard
University, and New York Public.
Variable: Let x  the total number of volumes.

the original
equals
123

the number
of volumes at
Harvard
University.
144424443

Equation: 11.4 
x

13.6
11.4  x  13.6
11.4  x  11.4  13.6  11.4
x  2.2
Harvard University has 2.2 million volumes more.
60. Words:
The number of volumes at New York
Public plus the difference equals the
number of volumes at the Library of
Congress.
Variable: Let x  the difference in volumes
between the two libraries.

x  24  13
x  37
A subcompact car travels 37 miles on a gallon of
gasoline.
54. The miles per gallon
the miles per gallon
of
a
subcompact
car
minus
of a midsize car
144424443 123 144424443

The height of the


Great Pyramid
144424443

The number
of volumes at
the
New York Public 1
plus
difference
144424443
23 14243

height.
14
42443

The total
volumes at
volumes at
volumes at
number
is
Lib.
of
C.
plus
Harvard
plus
NY Public.
14243 { 14243 123 14243 123 14243

481

Equation: x  24.0

13.6

11.4
x  24.0  13.6  11.4
x  49
The total number of volumes at the three largest
U.S. libraries is 49 million.

72

PQ249-6481F-03[062-092] 26/9/02 4:29 PM Page 73 Sahuja Ahuja_QXP_06:Desktop Folder:Chandra:Algebra_FNL_Delivery:

68. A;
x 167  52
x 167  167  52  167
x  115

62. Words:

The number of one-year olds plus the


difference equals the number of
newborns.
Variable: Let x  the difference between the
number of one-year olds and
newborns.
The number of
the
one-year olds 1
plus
1442443
23 difference
14243 equals
123

Page 134

Maintain Your Skills

69. Words:
Area is  times the radius squared.
Variables: Let A  area and r  radius.

the number of
newborns.
1442443

Equation: (679  634)


x
 (1379  1286)
679  634  x  1379  1286
1313  x  2665
1313  x  1313  2665  1313
x  1352
There are 1352 more newborns.
63. Words:
The number of males plus the
difference equals the number of
females.
Variable: Let x  the difference between the
number of males and females.

Area
1
23

is

times
123

the radius squared.


144424443

Formula:
A
 

r2
The formula for the area of a circle is A  r2.
70. Replace r in r2 with 16.
r2  (16)2
 (256)
804.2
The area is about 804 in2.
71. You can use a calculator to find an approximation
for 22.
1
2

The number
the
the number
of4males
plus
difference
equals 1
of4females.
14
2443 1
23 1
442443 123
42443

 0.5

22  1.41421356 p

Equation: (1379  679 


x
 (1286 
1707)
634  3714)
1379  679  1707  x  1286  634  3714
3765  x  5634
3765  x  3765  5634  3765
x  1869
There are 1869 more females.
64. Words:
The total population is equal to the
sum of the number of newborns,
one-year olds, and adults.
Variable: Let t  the total population.

Therefore,

1
2

6 22.

72. You can use a calculator to find an approximation


2
for 3.
3
4
2
3

 0.75
 0.66666666 p

Therefore,

3
4

7 3.
3

73. You can use a calculator to find the value of 8.


3
8

The total
the number of
the number of
the number
population is newborns
plus one-year olds plus of adults.
14243
{ 14424431
23 1442443 123 14243

 0.375
3

Therefore, 0.375  8.

Equation:
t  (1379  1286)  (679  634)  (1707  3714)
t  1379  1286  679  634  1707  3714
t  9399
The total deer population is 9399.
65. If a  b  x, then a  b  x. If a  x  b  x, then
by the transitive property a  a  x. If a  a  x,
then x  0. If x  0, then a  x  b  x implies
a  b. Therefore, if a  b  x, then a  b and
x  0 would make a  x  b  x true.
66. Equations can be used to describe the
relationships of growth and decline in job
opportunities. Answer should include the
following.
To solve the equation, add 66 to each side. The
solution is m  71.
An example such as The percent increase in
growth for paralegals is 16 more than the
percent increase in growth for detectives. If
the growth rate for paralegals is 86%, what is
the growth rate for detectives? d  16  86;
70%
67. C;
b 15  32
b  15  2  32  2
b 13  34

74. The greatest common place value is tens, so the


digits in the tens place are the stems.
Stem
Leaf
3
12456
4
0123
5
246
3 01  31
75. The greatest common place value is ones, so the
digits in the ones place are the stems.
Stem
Leaf
0
58
1
1247
2
3689
3
4
15
0 05  0.5
76. Hypothesis: y  2
Conclusion: 4y  6  2
77. Hypothesis: it is Friday
Conclusion: there will be a science quiz
78. 4(16  42)  4(16  16) Substitution; 42  16
 4(1)
Substitution; 16  16  1
4
Multiplicative
Identity; 4  1
4

73

Chapter 3

PQ249-6481F-03[062-092] 26/9/02 4:29 PM Page 74 Sahuja Ahuja_QXP_06:Desktop Folder:Chandra:Algebra_FNL_Delivery:

4. 2g  84

79. (25  52)  (42  24)  (32  25)  (16  16)


Substitution
 7  0 Substitution
7
Additive Identity
80. Replace x in 3x  2
2 with each value in the
replacement set.
3x  2  2

30  2 7 2S2 2

true

true

31  2 7 2S5 7 2

32  2 7 2S8 7 2

2y2  1  0

True or False?

true

2(1)2  1 7 0 S 1 7 0

2(3)2  1 7 0 S 17 7 0

2(5)2  1 7 0 S 49 7 0

true

true

14

84.

t  35

2
3

85.

8 38

87.

5
9

6.
36

88.

2
5




1
2
5
4

8
9

4
15




 14




7.

9

1 2  36149 2
a
4
9
36
16 ? 4
9
36
4
4
9
9

Solving Equations by Using


Multiplication and Division

Page 138

Check for Understanding

12

9
4

12

1 2

8. 3.15  1.5y
3.15
1.5

8 15

9 4
120
36
10
3
1
33

3-3

4
k
5
5 4
k
4 5

5 8
9
40
10
k  36 or 9
1
k  19
4
8
Check:
k9
5
4 1 ? 8
1 9
5 9
8
8
9
9
1
The solution is 19.

 1.5y
1.5

2.1  y
Check:

3.15  1.5y
?
3.15  1.5(2.1)
3.15  3.15
The solution is 2.1.
9.

1314 2p  212
1134 2p  52
4 5
4 13
p  13 1 2 2
13 1 4 2
20

10

p  26 or 13

1. Sample answer: 4x  12


2. Dividing each side of an equation by a number is
the same as multiplying each side of the equation
by the numbers reciprocal.
3. Juanita; to find an equivalent equation with
1n on one side of the equation, you must divide
1
each side by 8 or multiply each side by 8.

Chapter 3

5

The solution is 16.

89.

a
36
a
36

Check:

6

5
2

5

a  16

 10  9  10

 12
1
2

t

7
35 ?

7

5  5
The solution is 35.

7 0.3
 0.1 5
35 1 5
70 3
10.5 4 5
0.22
1.50.3
30
 
30
30
30
0

86.

17t 2  7(5)

83.

6 .5
 2 .8
52 0
13 0
18.2 0
7.12
2.5
17.8
00


17 5
30
25
50
50
0

 5

Check:

The solution set for 2y2  1


0 is {1, 3, 5}.
82.

t
7

5.

The solution set for 3x  2


2 is {1, 2}.
81. Replace y in 2y2  1
0 with each value in the
replacement set.
y

84
2

2(42)  84
84  84
The solution is 42.

false

g  42
Check:
2g  84

True or False?

2g
2

Check:

1314 2p  212
?
1314 211013 2  212
1

22  22

The solution is

74

10
.
13

PQ249-6481F-03[062-092] 26/9/02 4:29 PM Page 75 Sahuja Ahuja_QXP_06:Desktop Folder:Chandra:Algebra_FNL_Delivery:

16. 1634  86s

120.
Five times a number is 1
10. 123
123 14243 { 23
5
n
 120

5n  120
5n
5

1634
86

17.

12

 24

The discharge
the
the
of a river
is width times depth times
144424443 { 123 123 123 123

Equation: 3198
 533 
d
3198  533 d (0.6)
3198  533(0.6)d
3198  319.8 d
3198
319.8

11

1 v2

v  225
v

5  45

Check:

225 ?

5

319.8d
319.8

45

45  45
The solution is 225.
19.

Practice and Apply

2
n
3
3 2
n
2 3

12

 14
3

 2 (14)

n  21
2
n
3

Check:
2
3

55

 14

1212  14
?

14  14
The solution is 21.

r  11
5r  55
Check:
?

20.

5(11)  55
55  55
The solution is 11.
14. 8d  48

2
g
5
5 2
g
2 5

12

 14
5

g  35

2
g
5

2
5

d6
Check:

 2 14

Check:

48
8

 14

1352  14
?

14  14
The solution is 35.

8d  48
?
8(6)  48
48  48
The solution is 6.
15. 910  26a


11

5 5  5(45)

0.6

 5

910
26

b

7
77 ?

7

5  45

18.

the
speed.
123

13. 5r  55

1 2  7(11)

11  11
The solution is 77.

10  d
The Mississippi River is 10 m deep at this
location.

8d
8

 11

Check:

n  60
The solution is 60.
12. Words:
The discharge of a river is the width
times the depth times the speed.
Variable: Let d  the depth of the river.

5r
5

b
7
b
7

b  77

 2 (24)

Pages 138140

86s
86

19  s
Check:
1634  86s
?
1634  86(19)
1634  1634
The solution is 19.

120
5

n  24
The solution is 24.
equals 24.
11. Two fifths of a number 1
1442443 { 14243 4243 123
2

n
24

2
n
5
5 2
n
2 5

21.
24

g
24
g
24

 12

1 2  241125 2
g  10

26a
26

Check:

35  a
Check:
910  26a
?
910  26(35)
910  910
The solution is 35.

5
g
 12
24
10 ? 5
 12
24
5
5
 12
12

The solution is 10.

75

Chapter 3

PQ249-6481F-03[062-092] 26/9/02 4:29 PM Page 76 Sahuja Ahuja_QXP_06:Desktop Folder:Chandra:Algebra_FNL_Delivery:

22.
45

z
45
z
45

5

28.

1 2  45125 2
z  18

z
2
5
45
18 ? 2
5
45
2
2
5
5

Check:

33

x  22 or

Check:

11.78
1.9

29. 5h  33


2

5h
5

6.272
0.49

11
.
15

30. 3p  45
1

3p
3

45

3
21 1

5 3
21
7
or 5
15
2
15

p
p
p

97.41
5.73

Check:

3p  45

1 22

3 15  45
1

45  45
2

The solution is 15.

1235 2t  22
1135 2t  22
5
13
5
13 1  5 2 t  13(22)

31. First, solve 4m  10.


4m  10
4m
4

m
m

110
 13
6
 813
3
25

10
4
5
2
1
22
1

Now, replace m in 12m with 22.

1 12
5
 12 1 2 2

2t  22
?
1235 218136 2  22

12m  12 22

 30
The value is 30.

22  22
6

The solution is 813.

Chapter 3

The solution is

q  17
5.73q  97.41
Check:
?
5.73(17)  97.41
97.41  97.41
The solution is 17.

Check:

11115 2  323
2

2.8(3.5)  9.8
9.8  9.8
The solution is 3.5.
26. 5.73q  97.41

5h  33

33  33

9.8

1 2

5

27.

11
15

Check:

 2.8

h

m  3.5
2.8m  9.8
Check:

5.73q
5.73

33

5
11
1
h   3 5

k  12.8
Check:
0.49k  6.272
?
0.49(12.8)  6.272
6.272  6.272
The solution is 12.8.
25. 2.8m  9.8
2.8m
2.8

The solution is 12.

1.9(6.2)  11.78
11.78  11.78
The solution is 6.2.
24. 0.49k  6.272


1323 2x  512
?
1323 21112 2  512

52  52

f  6.2
Check:
1.9f  11.78

0.49k
0.49

3
2

x  12

The solution is 18.


23. 1.9f  11.78
1.9f
1.9

1323 2x  512
1113 2x  112
3
11
3 11
x  11 1  2 2
11 1 3 2

76

PQ249-6481F-03[062-092] 26/9/02 4:29 PM Page 77 Sahuja Ahuja_QXP_06:Desktop Folder:Chandra:Algebra_FNL_Delivery:

32. First, solve 15b  55.


15b  55
15b
15

b
b

38.

12

1 2
11
 31 3 2
2
33

39.

1
p
7

41. Replace  in /  7p with 65 and solve.


1

/  7p
1

65  7p
7(65)  7

50


12

126t
126

8n  12

n  32
The solution is 32.
Two and one half times a number equals one and one fifth.

14444244443 123 1
44244
3 14243 14444244443
1

1212 2n  115
12

5

15

132t
132

12

The solution is

60.5
126

60.5
132

t  0.46
It takes about 0.46 second.
time for a
time for a
45.
two-seam
fastball
minus
four-seam
fastball
14444244443 1
424
3 14444244443
0.48

0.46
0.48  0.46  0.02
The difference is about 0.02 second.

2 6
5
12
25

5

n

d


t  0.48
It takes about 0.48 second.
44. Replace r with 132 and d with 60.5 in rt  d and
solve.
rt  d
132t  60.5

3 8 n  3(12)

22

15.9  d
The diameter was about 16 feet.
43. Replace r with 126 and d with 60.5 in rt  d and
solve.
rt  d
126t  60.5

Negative three eighths times a number equals 12.


1444442444443 123 1
44244
3 14243 123
8

117 2p

455  p
There are 455 people in the group.
42. Replace C in C  d with 50 and solve.
C  d
50  d

n  60
The solution is 60.

5
n
2
2 5
n
5 2

 7(350)

 50
There are 50 left-handed people.

12

37.

The number of
people in
left-handed people is one seventh of the world.
{ 1442443
{ 14
42443
1


p
/

117
9

1 32

 4(4.82)

40. Replace p in 7p with 350 and solve.

5
1
n
 12
5
5 1
5
n  1(12)
1 5

 4.82

n  13
The solution is 13.
35. One fifth of a number is
12.
14243 { 1
44244
3 { 123
1
n
 12


36.

 4.82

84
7

4.82

The equation is /  7p.

n  12
The solution is 12.
Negative nine times a number is 117.
34. 1
44424443 123 1
44244
3 { 123
n
9
 117

9n  117
9n
9

14444244443

 11
The value is 11.
Seven
equals 84.
33. 1
times a number 1
424
3 123 1
44244
3 4243 123
7
n
84


7n  84


n  3.615
The solution is 3.615.

Now, replace b in 3b with 33.

7n
7

13

1
13 n
4
n
3
3 4
n
4 3

1 2

55
15
11
3
2
33

3b  3

One and one third times a number is 4.82.


14444244443 123 1
44244
3 { 14243

12
.
25

77

Chapter 3

PQ249-6481F-03[062-092] 26/9/02 4:29 PM Page 78 Sahuja Ahuja_QXP_06:Desktop Folder:Chandra:Algebra_FNL_Delivery:

46.
eight
123

times
123

The side length of each square is 4 inches.


Area
is
side length squared.
123
{
144424443
A

42
2
A4
A  16
The area of the square is 16 in2.
53. A; 4t  20

the number of
grams
of hydrogen
14444244443

8

x
The expression for oxygen is 8x.
47.

number of grams
number of grams
Number of grams
of oxygen
of water.
plus
equals
of hydrogen
14444244443 123
1
424
3


14444244443

8x

14444244443

477

The equation is x  8x  477.


48. Solve x  8x  477
x  8x  477
(1  8)x  477
9x  477
9x
9

4t
2

2t  10

Page 140

477
9

y

11
6
5
16
5

Now, replace y in 18y  21 with 16.

1 52
11
 18 1 6 2  21

18y  21  18 16  21
 33  21
 12
The value is 12.
51. You can use the distance formula and the speed of
light to find the time it takes light from the stars
to reach Earth. Answers should include the
following.
Solve the equation by dividing each side of the
equation by 5,870,000,000,000.
The answer is 53 years.
The equation 5,870,000,000,000t 
821,800,000,000,000 describes the situation
for the star in the Big Dipper farthest from
Earth.
52. C;
2 times
2 times
Perimeter
is
length
plus
width.
1442443
{
14243
123
14243
48

48  2(5x)  2x
48  10x  2x
48  (10  2)x
48  12x
48
12

2(5x)

Ten
1
23 times
123
10


a4
number
a
1
42443
a

to43 {
5
142

5

times
1
23


The equation is 10a  5(b  c).


58. (5)(12)  60
59. (2.93)(0.003)  0.00879
60. (4)(0)(2)(3)  (0)(2)(3)
 (0)(3)
 (0)
61.
4 3 2 1

62.
654321 0 1 2 3 4 5 6 7 8 9 10

2x

63.
7 6 5 4 3 2 1

64.
7 6 5 4 3 2 1

65. Commutative Property of Addition


66. Associative Property of Multiplication
67. 2  8  9  16  9
 25
68. 24  3  8  8  8
0

12x
12

4x

Chapter 3

Maintain Your Skills

m  14  81
m  14  14  81  14
m  67
Check:
m  14  81
?
67  14  81
81  81
The solution is 67.
55.
d  27  14
d  27  27  14  27
d  13
Check:
d  27  14
?
13  27  14
14  14
The solution is 13.
56. 17  (w)  55
17  w  55
17  w  17  55  17
w  72
Check:
17  1w2  55
?
17  31722 4  55
55  55
The solution is 72.
57.
is equal

54.

x  53
There are 53 grams of hydrogen.
49. Replace x in 8x with 53.
8x  8(53)
 424
There are 424 grams of oxygen.
50. First, solve 6y  7  4.
6y  7  4
6y  7  7  4  7
6y  11
6y
6

20

 2

78

the sum of
b2
and
c.
14
443
(b  c)

PQ249-6481F-03[062-092] 26/9/02 4:29 PM Page 79 Sahuja Ahuja_QXP_06:Desktop Folder:Chandra:Algebra_FNL_Delivery:

69.

3
(17
8

 7)  8 (24)


7.

72
8

9
70.

15  9
26  12





Page 140
1.

Surface
area
14
243
S

2.

3.

4.

5.

6.

6
26  12
6
38
3
19

12

 18
3

 2 (18)

p  27
2
p
3

Check:

 18

?
2
(27) 
3

18
18  18
The solution is 27.
8. 17y  391

Practice Quiz 1
equals
123


2
p
3
3 2
p
2 3

four
1
23
4

times
123


times
123


17y
17

the square of
the
radius.
1
4424
43

391

 17

y  23
17y  391
Check:

r2

The formula for the surface area of a sphere is


S  4r2.
Replace r in 4r 2 with 7.
4r2  4 (7)2
 4 (49)
 4(49)
 196
 615.8
The surface area is about 615.8 cm2.
d  18  27
d  18  18  27  18
d  45
Check:
d  18  27
?
45  18  27
27  27
The solution is 45.
m  77  61
m  77  77  61  77
m  16
Check:
m  77  61
?
16  77  61
61  61
The solution is 16.
12  a  36
12  a  12  36  12
a  24
Check:
12  a  36
?
12  1242  36
36  36
The solution is 24.
t  (16)  9
t  16  9
t  16  16  9  16
t  7
Check:
t  (16)  9
?
7  (16)  9
99
The solution is 7.

17(23)  391
391  391
The solution is 23.
9. 5x  45
5x
5

45
5

x  9
Check:

5x  45
?

5 (9)  45
45  45
The solution is 9.
2

5 d  10

10.
5

1 22

2 5 d  2 (10)
d  25
2

5 d  10

Check:

5 (25)  10
10  10
The solution is 25.

Page 141
1. Step 1

Algebra Activity
Model the equation.

x
1 1 1


1 1 1

1 1 1
1 1 1

2x  3  9

Place 2 x tiles and 3 negative 1 tiles on one side of


the mat. Place 9 negative 1 tiles on the other side
of the mat.

79

Chapter 3

PQ249-6481F-03[062-092] 26/9/02 4:29 PM Page 80 Sahuja Ahuja_QXP_06:Desktop Folder:Chandra:Algebra_FNL_Delivery:

Step 2

Place 3 x tiles and 5 positive 1 tiles on one side of


the mat. Place 14 positive 1 tiles on the other side
of the mat.
Step 2 Isolate the x term.

Isolate the x term.

x
1 1 1

1 1 1
1

1 1 1

1 1 1

2x  3  3  9  3

Since there are 3 negative 1 tiles with the x tiles,


add 3 positive 1 tiles to each side to form zero pairs.
Step 3 Remove zero pairs.

1 1 1

1 1 1

1

1 1 1

1 1 1

1 1

1 1

3x  5  5  14  5

Since there are 5 positive 1 tiles with the x tiles,


add 5 negative 1 tiles to each side to form zero
pairs.
Step 3 Remove zero pairs.

1

1

1

1

1

2x  6

1 1 1

1 1 1


1

1 1

1 1

1 1

1 1

1 1

1 1

1 1

1 1

1 1

9
3x
3
3

Separate the tiles into 3 equal groups to match


the 3 x tiles. Each x tile is paired with 3 positive
1 tiles. Thus, x  3.

3x  5  14

Chapter 3

Group the tiles to form zero pairs and remove the


zero pairs.
Step 4 Group the tiles.

x
1

3x  9

Separate the tiles into 2 equal groups to match


the 2 x tiles. Each x tile is paired with 3 negative
1 tiles. Thus, x  3.
2. Step 1 Model the equation.

1 1

6
2x
 2
2

Group the tiles to form zero pairs and remove the


zero pairs.
Step 4 Group the tiles.

80

PQ249-6481F-03[062-092] 26/9/02 4:29 PM Page 81 Sahuja Ahuja_QXP_06:Desktop Folder:Chandra:Algebra_FNL_Delivery:

3. Step 1

Separate the tiles into 3 equal groups to match


the 3 x tiles. Each x tile is paired with 4 positive
1 tiles. Thus, x  4.
4. Step 1 Model the equation.

Model the equation.

x
x
x


1 1

x
x
1 1 1

1 1 1

1 1
3x  2  10

Place 3 x tiles and 2 negative 1 tiles on one side of


the mat. Place 10 positive 1 tiles on the other side
of the mat.
Step 2 Isolate the x term.

1

1

1 1

Since there are 2 negative 1 tiles with the x tiles,


add 2 positive 1 tiles to each side to form zero pairs.
Step 3 Remove zero pairs.

1 1

1 1

1 1

1 1

8  4  2x  4  4

Since there are 4 positive 1 tiles with the x tiles,


add 4 negative 1 tiles to each side to form zero
pairs.
Step 3 Remove zero pairs.

1

1 1 1

3x  2  2  10  2

1 1 1
1

1

Place 2 x tiles and 4 positive 1 tiles on one side of


the mat. Place 8 negative 1 tiles on the other side
of the mat.
Step 2 Isolate the x term.

8  2x  4

x
x

1 1 1

1
1

1 1 1

1

1

1 1

1

1 1

1

1 1

3x  12

Group the tiles to form zero pairs and remove the


zero pairs.
Step 4 Group the tiles.

12  2x

Group the tiles to form zero pairs and remove the


zero pairs.

3x
 12
3
3

81

Chapter 3

PQ249-6481F-03[062-092] 26/9/02 4:29 PM Page 82 Sahuja Ahuja_QXP_06:Desktop Folder:Chandra:Algebra_FNL_Delivery:

Step 4

Step 3

Group the tiles.

1 1 1
1 1 1

1 1 1
1 1 1

Remove zero pairs.

x
x
x
x

1

1
1

1
1

1

1

1

1

1

x
x
x
x

Group the tiles to form zero pairs and remove the


zero pairs.
Step 4 Group the tiles.

Separate the tiles into 2 equal groups to match


the 2 x tiles. Each x tile is paired with 6 negative
1 tiles. Thus, x  6.
5. Step 1 Model the equation.

4x  8

12
 22x
2


1

8
4x
4
4

3  4x  11

Separate the tiles into 4 equal groups to match


the 4 x tiles. Each x tile is paired with 2 positive
1 tiles. Thus, x  2.
6. Step 1 Model the equation.

Place 4 x tiles and 3 positive 1 tiles on one side of


the mat. Place 11 positive 1 tiles on the other side
of the mat.
Step 2 Isolate the x term.

x
x
x
x
x
1


1

1 1 1

1 1 1
2x  7  1

3  4x  3  11  3

Place 2 x tiles and 7 positive 1 tiles on one side of


the mat. Place 1 positive 1 tile on the other side of
the mat.
Step 2 Isolate the x term.

Since there are 3 positive 1 tiles with the x tiles,


add 3 negative 1 tiles to each side to form zero
pairs.

x
x
1

1 1 1 1

1 1 1 1

1 1 1

1 1 1

2x  7  7  1  7

Chapter 3

82

PQ249-6481F-03[062-092] 26/9/02 4:30 PM Page 83 Sahuja Ahuja_QXP_06:Desktop Folder:Chandra:Algebra_FNL_Delivery:

Step 2

Since there are 7 positive 1 tiles with the x tiles,


add 7 negative 1 tiles to each side to form zero
pairs.
Step 3 Remove zero pairs.

1 1 1

1 1 1

1 1

1 1

1 1

1 1

1 1

1 1

x
x
x
x
1 1 1 1
1 1 1

Group the tiles to form zero pairs and remove the


zero pairs.
Step 4 Group the tiles.

1 1 1
1

1 1

1 1

1 1

1 1

1 1

1 1

1 1

1 1 1

Since there are 7 negative 1 tiles with the x tiles,


add 7 positive 1 tiles to each side to form zero pairs.
Step 3 Remove zero pairs.

2x  6

9  7  4x  7  7

1 1

1 1

Isolate the x term.

16  4x

Group the tiles to form zero pairs and remove the


zero pairs.
Step 4 Group the tiles.

2x
 6
2
2

Separate the tiles into 2 equal groups to match


the 2 x tiles. Each x tile is paired with 3 negative
1 tiles. Thus, x  3.
7. Step 1 Model the equation.

x
x
1

x
x

1 1 1

16
 44x
4

1 1 1

Separate the tiles into 4 equal groups to match


the 4 x tiles. Each x tile is paired with 4 positive
1 tiles. Thus, x  4.
8. Step 1 Model the equation.

1
9  4x  7

Place 4 x tiles and 7 negative 1 tiles on one side of


the mat. Place 9 positive 1 tiles on the other side
of the mat.

x
x
1 1 1

x
1

1 1 1
1 1

1
7  3x  8

83

Chapter 3

PQ249-6481F-03[062-092] 26/9/02 4:30 PM Page 84 Sahuja Ahuja_QXP_06:Desktop Folder:Chandra:Algebra_FNL_Delivery:

Place 3 x tiles and 7 positive 1 tiles on one side of


the mat. Place 8 negative 1 tiles on the other side
of the mat.
Step 2 Isolate the x term.

x
x
x

Page 145

1 1 1

1 1

1 1 1 1

1 1 1 1

1 1 1

1 1 1

7  3x  7  8  7

Since there are 7 positive 1 tiles with the x tiles,


add 7 negative 1 tiles to each side to form zero
pairs.
Step 3 Remove zero pairs.

x
x
x

1 1 1
1 1 1

1 1

1 1

1 1

1 1

1 1

1 1

Statement
The result is 55.
The product is added to 13.
A number is multiplied
by seven.

1 1

1 1 1 1
1 1 1

1 1

3x  15

Statement
There are 2,187,000
bacteria on the seventh
day.
A bacteria population
triples in number
each day.

1 1 1
1 1

1 1 1

7.

1 1
1 1 1

42  7  6

Undo the Statement


2,187,000

2,187,000  36  3000

There were 3000 bacteria.


4g  2  6
4g  2  2  6  2
4g  4
4g
4

1 1

4
4

g  1
4g  2  6
?
4(1)  2  6
?
4  2  6
6  6
The solution is 1.
8.
18  5p  3
18  3  5p  3  3
15  5p
Check:

3x
 15
3
3

Separate the tiles into 3 equal groups to match


the 3 x tiles. Each x tile is paired with 5 negative
1 tiles. Thus, x  5.
9. First add 12 to each side, and then divide each
side by 7.

15
5

5p
5

3p

Chapter 3

Undo the
Statement
55
55  13  42

The number is 6.
6. Start at the end of the problem and undo each step.

Group the tiles to form zero pairs and remove the


zero pairs.
Step 4 Group the tiles.

Check for Understanding

1. Sample answers: 2x  3  1, 3x  1  7


2. (1) Add 4 to each side. (2) Multiply each side by 5.
(3) Subtract 3 from each side.
3. Odd integers are two units apart. If we want to
signify the odd integer before odd integer n, we
need to subtract 2 units. Therefore, the
expression can be written as n 2.
4a. Subtract 3 from each side.
4b. Simplify.
4c. Multiply each side by 5.
4d. Simplify.
4e. Subtract 4 from each side.
4f. Simplify.
4g. Divide each side by 2.
4h. Simplify.
5.Start at the end of the problem and undo each step.

1 1 1

Solving Multi-Step Equations

3-4

84

PQ249-6481F-03[062-092] 26/9/02 4:30 PM Page 85 Sahuja Ahuja_QXP_06:Desktop Folder:Chandra:Algebra_FNL_Delivery:

18  5p  3
?
18  5(3)  3
?
18  15  3
?
18  18
The solution is 3.

13. Twelve
a number equals
34.
123 decreased
144244by
3 twice
1442443
123 1
23
2n

34
12

12  2n  34
12  2n  12  34  12
2n  46

Check:

3
a
2

9.
3
a
2

2n
2

 8  11

12

 19
2

 3(19)

a

38
3
2

a  123
3
a
2
3
2
123
2

Check:

n  (n  1)  (n  2)
n  (n  1)  (n  2)  42
3n  3  42
3n  3  3  42  3
3n  39

 8  11

1 2  8  11
?
?

19  8  11
11  11

3n
3

The solution is 123.


10.
2

b  4
2
b  4
2

2  2(17)
b  4

2
30  4 ?

2
34 ?

2

17

26  2  2n
26  2  2  2n  2
24  2n

17
17

24
2

2n
2

Pages 145148

5.6

Practice and Apply

16. Start at the end of the problem and undo each step.

n  28
0.2n  3  8.6
?
0.2(28)  3  8.6
?
5.6  3  8.6
8.6  8.6
The solution is 28.
12.
3.1y  1.5  5.32
3.1y  1.5  1.5  5.32  1.5
3.1y  6.82

Statement
The result is 25.
The quotient is added
to 17.
A number is divided
by 4.

Check:

12  n
The Hawaiian alphabet has 12 letters.

 0.2

3.1y
3.1

39
3

The
English alphabet 123
equals 2 more than twice the Hawaiian alphabet.
14444244443
{ 14243 1444442444443
26
 2
+
2n

17  17
The solution is 30.
0.2n  3  8.6
11.
0.2n  3  3  8.6  3
0.2n  5.6
0.2n
0.2

 42

n  13
n  1  13  1 or 14
n  2  13  2 or 15
The consecutive integers are 13, 14, and 15.
15. Let n  the number of letters in the Hawaiian
alphabet.

 17

b  4  34
b  4  4  34  4
b  30
Check:

46
2

n  23
The solution is 23.
14. Let n  the least integer.
Then n  1  the next greater integer, and
n  2  the greatest of the three integers.
The
sum
of three consecutive integers is
42.
{ {
1444444442444444443

 8  8  11  8
3
a
2
2 3
a
3 2

Undo the Statement


25
25  17  8
8  4  32

The number is 32.


17. Start at the end of the problem and undo each
step.

6.82
3.1

Statement
The result is 75.
The difference is
multiplied by 5.
Nine is subtracted
from a number.

y  2.2
3.1y  1.5  5.32
Check:
?
3.1(2.2)  1.5  5.32
?
6.82  1.5  5.32
5.32  5.32
The solution is 2.2.

Undo the Statement


75
75  5  15
15  9  24

The number is 24.

85

Chapter 3

PQ249-6481F-03[062-092] 26/9/02 4:30 PM Page 86 Sahuja Ahuja_QXP_06:Desktop Folder:Chandra:Algebra_FNL_Delivery:

18. Start at the end of the problem and undo each


step.

21. Start at the end of the problem and undo each step.
Statement
He had $13.45 left.
He bought lunch for
$6.55.
He spent half for a
haircut.
He spent one third for
gasoline.

Statement
Undo the Statement
The fourth question is
worth $6000.
$6000
The fourth question is
worth twice as much
$6000  2  $3000
as the third question.
The third question is
worth twice as much
$3000  2  $1500
as the second question.
The second question is
worth twice as much
$1500  2  $750
as the first question.

1
2

The sculpture lost its


weight the 8th hour.

2

25.
24

5
4

22

The sculpture lost 2 its


weight the 5th hour.

5
2

10  2  20

The sculpture lost 2 its


weight the 3rd hour.
1
2

The sculpture lost its


weight the 2nd hour.
1

The sculpture lost 2 its


weight the 1st hour.

10
5

3c
3

18
3

c  6

4a
4

7g
7

7  g
c
3

26.
c
3

57
c
3
c
3

y
5

27.
y
5

5575
3

96

9969

2

1 2  3(2)

c  6

3

28.

20  2  40

3

a
7

40  2  80
7

a
7

 2

9 

p
4

30.
t
8

Statement
Undo the Statement
She went into the
building.
0
She climbed the
044
remaining 4 rungs.
4  7  11
She went up 7 rungs.
She backed down
11  5  6
5 rungs.
639
She moved up 3 rungs.
She stood on the
2(9)  1  19
middle rung.

 6  12

p  56

1 2  8(6)
 10

17  s  40
17  s  17  40  17
s  57
s
1

57
1

s  57

The ladder has 19 rungs.

86

m
5

31.

t
8

2  4(10)

959

1 p2

t  48

32.

5

4 4  4(14)

m
5

 6  31

 6  6  31  6

 6

17  s
4
17  s
4

p
4

 6  6  12  6
t
8

1 2  5(3)

4  14

 5

1 2  7(5)

t
8

 3

y  15

 3  2  3
a
7
a
7

y
5
y
5

9 

29.

a  35

The sculpture weighed 80 lb.


20. Start at the end of the problem and undo each
step. Count the rungs from the top.

Chapter 3

49
7

25

5  2  10

The sculpture lost its


weight the 4th hour.

$40  2  $60

5a
63  7g  14
63  14  7g  14  14
49  7g

1
2

$20  2  $40

n  2
15  4a  5
15  5  4a  5  5
20  4a
20
4

5
8

5
8

The sculpture lost 2 its


weight the 6th hour.

24.

5
16

The sculpture lost 2 its


weight the 7th hour.

5n
5

Undo the Statement

5
16

$13.45  $6.55  $20

He withdrew $60.
Exercises 2239 For checks, see students work.
22.
23.
5n  6  4
7  3c  11
5n  6  6  4  6
7  3c  7  11  7
5n  10
3c  18

The first question is worth $750.


19. Start at the end of the problem and undo each
step.
Statement
After 8 hours, it
5
weighs 16 of a pound.

Undo the Statement


$13.45

5

m
5
m
5

 25

1 2  5(25)
m  125

PQ249-6481F-03[062-092] 26/9/02 4:30 PM Page 87 Sahuja Ahuja_QXP_06:Desktop Folder:Chandra:Algebra_FNL_Delivery:

33.
6

3j  (4)
6
3j  (4)
6

Now, replace a in 5a  2 with 5.


5a  2  5(5)  2
 25  2
 27
The value is 27.
41. First, solve 2x  1  5.
2x  1  5
2x  1  1  5  1
2x  4

 12

4  6 (12)

3j  (4)  72


3j  (4)  (4)  72  (4)
3j  76
3j
3

j
j
34.

76
3
76
3
1
253

2x
2

3d  1.2  0.9


3d  1.2  1.2  0.9  1.2
3d  2.1
3d
3

x2
Now, replace x in 3x  4 with 2.
3x  4  3(2)  4
64
2
The value is 2.
42. Two-thirds
a number minus six is negative ten.
14243 of
{ 14243 123 { { 1442443
2

n

6 
10
3

2.1
3

d  0.7
35.
2.5r  32.7  74.1
2.5r  32.7  32.7  74.1  32.7
2.5r  106.8
2.5r
2.5

106.8
2.5

2
n
3

r  42.72
36.
0.6  (4a)  1.4
0.6  (4a)  0.6  1.4  0.6
4a  0.8
4a
4

2
n
3

p
7
p
7

 1.8

1 2  7(1.8)

p  12.6
38. 3.5x  5  1.5x  8
(3.5  1.5)x  5  8
2x  5  8
2x  5  5  8  5
2x  3
2x
2

16
4

2
1

9z  4
8
5
9z  4
88
5
9z  4
5
9z  4
5 5

 5.4  8
 13.4

3n
3

9z  4  67
9z  4  4  67  4
9z  63


4n
4

n  (n  2)  (n  4)
n  (n  2)  (n  4)  51
3n  6  51
3n  6  6  51  6
3n  45

 5.4

2  5(13.4)

9z
9

4n
The solution is 4.
44. Let n  the least odd integer.
Then n  2  the next greater odd integer, and
n  4  the greatest of the three odd integers.
The
sum of three consecutive odd integers {
is 51.
{
144444444424444444443

x  12 or 1.5
39.

 4

n  6
The solution is 6.
43. Twenty-nine
is thirteen added to four times a number.
1442443 {
14243 14243 123 123 14243
29
 13

4

n
29  13  4n
29  13  13  4n  13
16  4n

 0.5  0.5  1.3  0.5


7

 6  6  10  6

1 2  32(4)

0.8
4

 0.5  1.3
p
7
p
7

 6  10

2
n
3
3 2
n
2 3

a  0.2
37.

2

 51

45
3

n  15
n  2  15  2 or 17
n  4  15  4 or 19
The consecutive odd integers are 15, 17, and 19.

63
9

z7
40. First, solve 3a  9  6.
3a  9  6
3a  9  9  6  9
3a  15
3a
3

15
3

a5

87

Chapter 3

PQ249-6481F-03[062-092] 26/9/02 4:30 PM Page 88 Sahuja Ahuja_QXP_06:Desktop Folder:Chandra:Algebra_FNL_Delivery:

49. Let x  the length of the shortest side.


Then x  2  the length of the next longer side,
and x  4  the length of the longest side.
The
sum of the three sides is 54.
1444442444443

45. Let n  the least even integer.


Then n  2  the next greater even integer, and
n  4  the greatest of the three even integers.
The sum of three consecutive even integers is 30.
1444444442444444443 { 123
 30
n  (n  2)  (n  4)
n  (n  2)  (n  4)  30
3n  6  30
3n  6  6  30  6
3n  36
3n
3

{ {

x  (x  2)  (x  4)
 54
x  (x  2)  (x  4)  54
3x  6  54
3x  6  6  54  6
3x  48

36
3

3x
3

9

50.

92
7
2000(7) 

{ {

n  (n  1)  (n  2)  (n  3)  94
n  (n  1)  (n  2)  (n  3)  94
4n  6  94
4n  6  6  94  6
4n  88


88
4

n  1  22  1 or 23
n  3  22  3 or 25
The consecutive integers are 22, 23, 24, and 25.
47. Let n  the least odd integer.
Then n  2  the next greater odd integer,
n  4  the next greater odd integer, and
n  6  the greatest of the four odd integers.
The
sum of four consecutive odd integers is 8.
1444444442444444443

20
2

{ {

4
4

0.10

number
times
of4miles
123 1
243

x

is

 2000

1500
0.02

The area
the area of
of the square minus the rectangle is
1442443 123 1442443 {
4/
10  10



cost for
one day.
14243
60

4
of
5
{ {
4

5

10  10  4/  5 (10  10)
4

100  4/  5(100)
100  4/  80
100  4/  100  80  100
4/  20

45.05
0.1

4/
4

x  450.5
Ms. Jones can drive 450.5 mi.

Chapter 3

2/
2

x  75,000
Mr. Goetz must have $75,000 in sales.
54. Let   the length of the rectangle.

14.95  0.1x  60
14.95  0.1x  14.95  60  14.95
0.1x  45.05
0.1x
0.1

0.02x
0.02

n  2  1  2 or 1
n  6  1  6 or 5
The consecutive odd integers are 1, 1, 3, and 5.
48. Let x  the number of miles driven in one day.
cost per
mile
14
243

500
 0.02 
x
500  0.02x  2000
500  0.02x  500  2000  500
0.02x  1500

n  1
n  4  1  4 or 3

Price of car
per day
plus
14243
1
23
14.95


10  /
The persons foot is about 10 in. long.
52. See students work.
53. Let x  the amount of sales.
The monthly
the amount
salary
plus
of of sales
is $2000.
1442443
1
23 2%
{ { 1442443 { 123

 8
n  (n  2)  (n  4)  (n  6)
n  (n  2)  (n  4)  (n  6)  8
4n  12  8
4n  12  12  8  12
4n  4


a  7000
2
2000
a  7000
2
2000
a  7000
2000
a  7000
2000 2000

14,000  a  7000
14,000  7000  a  7000  7000
21,000  a
They can climb to about 21,000 ft.
51.
8  2/  12
8  12  2/  12  12
20  2/

n  22
n  2  22  2 or 24

4n
4

48
3

x  16
x  2  16  2 or 18
x  4  16  4 or 20
The lengths of the sides are 16 cm, 18 cm, and
20 cm.

n  12
n  2  12  2 or 10 n  4  12  4 or 8
The consecutive even integers are 12, 10,
and 8.
46. Let n  the least integer.
Then n  1  the next greater integer,
n  2  the next greater integer, and
n  3  the greatest of the four integers.
The
sum of four consecutive integers is 94.
1444444442444444443

4n
4

20
4

/5
The length of the rectangle is 5 in.

88

the area of
the square.
1442443
(10  10)

PQ249-6481F-03[062-092] 26/9/02 4:30 PM Page 89 Sahuja Ahuja_QXP_06:Desktop Folder:Chandra:Algebra_FNL_Delivery:

55. Never; Let n, n  2 be the even numbers, and


m, m  2 be the odd numbers.
The sum of two
the sum of two
consecutive
consecutive
even
numbers
equals
odd
numbers.
1442443 123 1442443

Step 2

n  (n  2)

m  (m  2)
n  (n  2)  m  (m  2)
2n  2  2m  2
2n  2  2  2m  2  2
2n  2m
2n
2

58. D;
3
a
5

Step 2

22

Step 2
Step 3

 3

1 2  53 (3)

a  5
0  11y  33
Press

Step 3 Enter (

and choose 0, for solve.

7t
7

0  18 
Step 2

Press

Press

and choose 0, for solve.


ALPHA R

0.8 )

Press

and choose 0, for solve.

h
7
h
7
h
7

Maintain Your Skills


91

 7

66.
15

4.
Step 4 Press ALPHA [SOLVE] to reveal the
solution. W  18.
6  6  12 

 0.7

7(13)  91
91  91
The solution is 13.

and choose 0, for solve.


ALPHA W  2 )

6  12 

0

t  13
7t  91
Check:

61. Step 1

 0.7

r  0.8
6

65. 7t  91

Enter 11 ALPHA Y  33.


Step 4 Press ALPHA [SOLVE] to reveal the
solution. Y  3.
w  2
40
5
60. Step 1
Press

0.7  0.7 

r  0.8
6
r  0.8
6

Page 148

Step 3

Step 2

6.

Enter 7.2 ALPHA T


33.84.
ALPHA
Step 4 Press
[SOLVE] to reveal the
solution. T  4.7.

5 3
a
3 5

Step 2

( ) 2

( ) 5

0.7.
Step 4 Press ALPHA [SOLVE] to reveal the
solution. R  5.
4.91  7.2t  38.75
64. Step 1
4.91  7.2t  38.75  38.75  38.75
7.2t  33.84  0

 19  19  16  19

59. Step 1

 19  16
3
a
5

and choose 0, for solve.

Step 3 Enter (

15  3n  22
3
a
5

0

ALPHA P

0.7 

63. Step 1

66

Step 4 Press ALPHA [SOLVE] to reveal the


solution. P  17.

2m
2

The alligator is about 93 or 10 years old.


57. B; 1
Fifteen minus three times a number equals negative twenty-two.
4243 123 123 123 14243 123 14444244443
15

6

Press

Step 3 Enter (
)
)

nm
Thus, n must equal m, so n is not even or m is not
odd.
56. By using the length at birth, the amount of
growth each year, and the current length, you can
write and solve an equation to find the age of the
animal. Answers should include the following.
To solve the equation, subtract 8 from each
side and then divide each side by 12.

or

p  (5)
2
p  (5)
6
2
p  (5)
6
2

62. Step 1

r
15
r
15

 8

1 2  15(8)
r  120

Check:

r

15
120 ?

15

8
8

8  8
The solution is 120.

6

and choose 0, for solve.

ALPHA H ( )
Step 3 Enter ( ) 18
7.
Step 4 Press ALPHA [SOLVE] to reveal the
solution. H  126.

89

Chapter 3

PQ249-6481F-03[062-092] 26/9/02 4:30 PM Page 90 Sahuja Ahuja_QXP_06:Desktop Folder:Chandra:Algebra_FNL_Delivery:

3b  1 2

67.

2
3b
3
2
2 3 b

75.

3
 2
3
3
 2 2
9
b4
1
b  24
2
1
Check:
3b  1 2
2 1 ?
1
3 2 4  1 2
1
1
1 2  1 2
1
The solution is 2 4.

1 2

76.

Equation:
m
 18 
47
The equation is m  18  47.
69. Solve m  18  47.
m  18  47
m  18  18  47  18
m  29
There were 29 models in 1990.
70. There are two numbers divisible by 3, and there
are 8  2 or 6 numbers not divisible by 3.

1 12
1 12

The odds of spinning a number divisible by 3


are 1:3.
71. There are four numbers equal to or greater than
5, and there are 8  4 or 4 numbers less than 5.


 16(1)  16
 16  4
 20

 18(2)  18

114 2
119 2

82. the
quantity 3 plus b 14243
divided by {y
14444244443
(3  b)

y
Thus, the algebraic expression is (3  b)  y.
83. {
3 times
a 1
plus
square
b
123 {
23 the
144
4244of
43

odds of less than 7  2

84.

1
86.

The odds of spinning a number less than 7


are 3:1.

88.

73. 7  3  7  3
6

 21

89.

2
7

38
1

 38
2

 24
1

 12

Chapter 3

Thus, the algebraic expression is 5m  2 .

4
4
1
1

The odds of spinning a number equal to or greater


than 5 are 1:1.
72. There are six numbers less than 7, and there are
8  6 or 2 numbers not less than 7.

 36  2
 38
81. the
product
of 5 and m, 1
plus
n
14444244443
23 half
142of
43
1

5m
n
2

3

odds of equal to or greater than 5 

80. 18 2 9  18 2  9

1 12
1
1
 15t 1 5 2  25 1 5 2

79. 16 1 4  16 1  4

odds of divisible by 3  6

74.

 (15t  25)  (5)

 3t  (5)
 3t  5
77. 17  9  17(10  1)
 17(10)  17(1)
 170  17
 153
78. 13(101)  13(100  1)
 13(100)  13(1)
 1300  13
 1313

The number of
the number of
models
in 1990 1
plus
equals models in 2000.
1442443
23 18
{ 123 1442443

15t  25
5

 (15t  25) 5

The number of models in 1990 plus 18


equals the number of models in 2000.
Variable: Let m  the number of models in 1990.

2
3

114 2
1
1
 3a 1 4 2  16 1 4 2
 4a  4

68. Words:

 (3a  16)  4
 (3a  16)

1 2

1 2

3a  16
4

90

3
a 
b2

Thus, the algebraic expression is 3a  b2.
5d  2d  (5  2)d 85. 11m  5m  (11  5)m
 3d
 6m
8t  6t  (8  6)t
87. 7g  15g  (7  15)g
 14t
 8g
9f  6f  (9  6)f
 3f
3m  (7m)  [3  (7)]m
 10m

PQ249-6481F-03[062-092] 26/9/02 4:30 PM Page 91 Sahuja Ahuja_QXP_06:Desktop Folder:Chandra:Algebra_FNL_Delivery:

Solving Equations with the


Variable on Each Side

3-5

Pages 151152

3
8

2g
2

13
13

The solution is
7.

3a
3

13z
13

13
5

10.

1
1 2.

3

5d
5

c  1
8
c  1
8

4

2  814c 2

c  1  2c
c  1  c  2c  c
1c
Check:

 2t  4
1

 2t  4  2t
3

 4
3

4

2.6  d
Check:
6  3  5(d  2)
?
6  3  5(2.6  2)
?
6  3  5(0.6)
?
633
66
The solution is 2.6.

60

8

11 12 2  34

a3
Check:
3(a  5)  6
?
3(3  5)  6
?
3(2)  6
6  6
The solution is 3.
8.
7  3r  r  4(2  r)
7  3r  r  8  4r
7  3r  8  3r
7  3r  3r  8  3r  3r
7  8
Since 7  8 is a false statement, this equation
has no solution.
9.
6  3  5(d  2)
6  3  5d  10
6  5d  7
6  7  5d  7  7
13  5d

 15

1 12

3(a  5)  6
3a  15  6
3a  15  15  6  15
3a  9

12
2

c4
Check:
20c  5  5c  65
?
20(4)  5  5(4)  65
?
80  5  20  65
85  85
The solution is 4.

 4 12

00

1z
The solution is 1.
2. If both sides of the equation are always equal, the
equation is an identity.
3. Sample answer: 2x  5  2x  5
4a. Subtract 6n from each side.
4b. Simplify.
4c. Add 13 to each side.
4d. Simplify.
4e. Divide each side by 2.
4f. Simplify.
5.
20c  5  5c  65
20c  5  5c  5c  65  5c
15c  5  65
15c  5  5  65  5
15c  60

3
1
 4t
8
3
1
1
 4t  2t
8
3
3
 4t
8
3
3
3
 4t  8
8
3
4t
4
3
3 4 t

? 1
2
? 3
4

 4t  2t  4

3
8

g6
The solution is 6.
1b. correct
1c. Incorrect; to eliminate 6z on the left side of the
equals sign, 6z must be added to each side of the
equation.
6z  13  7z
6z  13  6z  7z  6z
13  13z

6.

Check for Understanding

1a. Incorrect; the 2 must be distributed over both g


and 5.
2(g  5)  22
2g  10  22
2g  10  10  22  10
2g  12

15c
15

3
8

Check:

c  1
c
4
8
1  1 ? 1
4
8
2 ? 1
4
8
1
1
4
4

The solution is 1.
11. 5h  7  5(h  2)  3
5h  7  5h  10  3
5h  7  5h  7
Since the expressions on each side of the equation
are the same, this equation is an identity. The
statement 5h  7  5(h  2)  3 is true for all
values of h.

 4  8
9

 8

2  43 198 2
3

t2

t  12

91

Chapter 3

PQ249-6481F-03[062-092] 26/9/02 4:30 PM Page 92 Sahuja Ahuja_QXP_06:Desktop Folder:Chandra:Algebra_FNL_Delivery:

12.

5.4w  8.2  9.8w  2.8


5.4w  8.2  9.8w  9.8w  2.8  9.8w
8.2  4.4w  2.8
8.2  4.4w  8.2  2.8  8.2
4.4w  11
4.4w
4.4

Exercises 1639 For checks, see students work.


16.
3  4q  10q  10
3  4q  10q  10q  10  10q
3  14q  10
3  14q  3  10  3
14q  7

11

 4.4

14q
14

w  2.5
5.4w  8.2  9.8w  2.8
Check:
?
5.4(2.5)  8.2  9.8(2.5)  2.8
?
13.5  8.2  24.5  2.8
21.7  21.7
The solution is 2.5.
13. D; Solve by substitution.
A
75  9t  5(4  2t)
?
75  9(5)  5[4  2(5)]
?
75  (45)  5[4  (10)]
?
75  45  5[14]
120  70
B
75  9t  5(4  2t)
?
75  9(4)  5[4  2(4)]
?
75  (36)  5[4  (8)]
?
75  36  5[12]
111  60
C
75  9t  5(4  2t)
?
75  9(4)  5[4  2(4)]
?
75  36  5[4  8]
?
39  5[4]
39  20
D
75  9t  5(4  2t)
?
75  9(5)  5[4  2(5)]
?
75  45  5[4  10]
?
30  5[6]
30  30

 14
1

q  2 or 0.5
17.

3k  5  7k  21
3k  5  7k  7k  21  7k
4k  5  21
4k  5  5  21  5
4k  16
4k
4

18.

16
4

k4
5t  9  3t  7
5t  9  3t  3t  7  3t
8t  9  7
8t  9  9  7  9
8t  16
8t
8

16
8

t2
19.
8s  9  7s  6
8s  9  7s  7s  6  7s
s96
s9969
s  3
3
n
4

20.
3
n
4

 16  8n  2  8n  8n
7
n
8

7
n
8

 16  2  8n
 16  2

 16  16  2  16
7
n
8
8 7
n
7 8

 14

1 2  87 1142

The answer is D.

n  16

Pages 152154

Practice and Apply

21.

14a. Multiply each side by 10.


14b. Simplify.
14c. Distributive Property
14d. Add 4 to each side.
14e. Simplify.
14f. Divide each side by 6.
14g. Simplify.
15a. Subtract v from each side.
15b. Simplify.
15c. Subtract 9 from each side.
15d. Simplify.
15e. Divide each side by 6.
15f. Simplify.

1
4

2
 3y
1
2
1
 3y  3y
4
1
1
 3y
4
1
1
1
 3y  4
4
1
3y
1
3 3y

 4  3y  3y
3

4
3

44
1

2

2  3112 2
3

y  2

y  1 2
22.

8  4(3c  5)
8  12c  20
8  20  12c  20  20
12  12c
12
12

12c
12

1  c

Chapter 3

 4  3y

92

23.

7(m  3)  7
7m  21  7
7m  21  21  7  21
7m  28
7m
7

30.

1
y
2

28
7

 y  4  2y

1
y
2
1
y
2

4
b
5
5 4
b
4 5

8

1 2  54(8)
b  10

32.

8848
2x  4

2  2(4)

x8
26.
4(2a  1)  10(a  5)
8a  4  10a  50
8a  4  10a  10a  50  10a
18a  4  50
18a  4  4  50  4
18a  54

1
(7  3g)
4
7
3
 4g
4
7
3
g
 4g  8
4
7
7
 8g
4
7
7
7
 8g  4
4
7
g
8
8 7
g
7 8

 8
g

 8
g

 8  8
0
7

04
7

 4

1 2  87 174 2
g  2

33.

54

 18

a3
27.
4(f  2)  4f
4f  8  4f
4f  8  4f  4f  4f
8  0
Since 8  0 is a false statement, this equation
has no solution.
28. 3(1  d)  5  3d  2
3  3d  5  3d  2
3d  2  3d  2
Since the expressions on each side of the equation
are the same, this equation is an identity. The
statement 3(1  d)  5  3d  2 is true for all
values of d.
29. 2(w  3)  5  3(w  1)
2w  6  5  3w  3
2w  1  3w  3
2w  1  3w  3w  3  3w
w  1  3
w  1  1  3  1
w  2
w
1

2x  8  4

18a
18

3  5b  3  11  3

5  2x  3  4

2

3  5b  11

18
6

r  3

1
2x

3  5b  5b  11  5b  5b

 4  2y  2y

3  5b  11  5b

31.

25. 5  2 (x  6)  4

1
2x

 4  2y

04
Since 0  4 is a false statement, this equation has
no solution.

m4
24. 6(r  2)  4  10
6r  12  4  10
6r  8  10
6r  8  8  10  8
6r  18
6r
6

3
y
2

1
(a  4)
6
1
2
a3
6
1
2
2
a  3  3a
6
1
2
2a  3
1
2
2
2a  3  3
1
2a
1
2 2a

34.

 3(2a  4)
2

 3a  3
4

3
4

33
2

2  2(2)

a  4
28  2.2x  11.6x  262.6
28  2.2x  11.6x  11.6x  262.6  11.6x
28  13.8x  262.6
28  13.8x  28  262.6  28
13.8x  234.6
13.8x
13.8

35.

2

 1

234.6

 13.8

x  17
1.03p  4  2.15p  8.72
1.03p  4  2.15p  2.15p  8.72  2.15p
3.18p  4  8.72
3.18p  4  4  8.72  4
3.18p  12.72
3.18p
3.18

w2

 3a  3  3a

12.72
3.18

p4

93

Chapter 3

36.

18  3.8t  7.36  1.9t


18  3.8t  1.9t  7.36  1.9t  1.9t
18  1.9t  7.36
18  1.9t  18  7.36  18
1.9t  10.64
1.9t
1.9

2(n  2)  3n  13
2n  4  3n  13
2n  4  3n  3n  13  3n
n  4  13
n  4  4  13  4
n  17

10.64
1.9

n
1

t  5.6
37.
13.7v  6.5  2.3v  8.3
13.7v  6.5  2.3v  2.3v  8.3  2.3v
16v  6.5  8.3
16v  6.5  6.5  8.3  6.5
16v  14.8
16v
16

14.8
16

Three
greatest
equals twice
the least 1
plus
38.
123 times
123 the
14
4244
3 123
1442443
23 1
23
3

(n  4)

2 n

38

38. 2[s  3(s  1) ]  18


2[ s  3s  3]  18
2[ 4s  3]  18
8s  6  18
8s  6  6  18  6
8s  24


3(n  4)  2n  38
3n  12  2n  38
3n  12  2n  2n  38  2n
n  12  38
n  12  12  38  12
n  26
n  2  26  2 or 28
n  4  26  4 or 30
The consecutive even integers are 26, 28, and 30.
44.
0.8(220  a)  152
176  0.8a  152
176  0.8a  176  152  176
0.8a  24

24
8

s3
39. 3(2n  5)  0.5(12n  30)
6n  15  6n  15
Since the expressions on each side of the equation
are the same, this equation is an identity. The
statement 3(2n  5)  0.5(12n  30) is true
for all values of n.
40. One half of increased
two thirds of
a number
by
14243 14243
1
2

n


1
n
2

1
n
2

16
123

16

is

the number minus


144244
3 123
2
3

n

0.8a
0.8

22  1  4(x  2)

45.

2
2

22  2  4x

6n  16  4
1

2  4x

4(2)  4 4x

6n  20

n  120
41. The sum of one half
of
a44244
number and
6
14
443
1
n
2
1
n
2

equals
123


n6
6
1

 6  2n 
6
6(6) 

1
n
3
1
1
n  2n
3
1
6n
1
6 6n

one third of
the
number.
14
424
43
1
3

n

0.425x
0.425

2.4

 0.425

x  5.647
It will take about 5.6 years.

36  n
42. Let n  the lesser odd integer.
Then n  2  the greater odd integer.
Twice the greater
three times the
odd
integer
is
lesser number minus 13.
144424443 123 123
123 123
2(n  2)

3 n

13

Chapter 3

11 2

8x
The name is 8-penny.
46.
4.9  0.275x  2.5  0.7x
4.9  0.275x  0.7x  2.5  0.7x  0.7x
4.9  0.425x  2.5
4.9  0.425x  4.9  2.5  4.9
0.425x  2.4

6 6n  6(20)

1
2

22  2  2  4x  2

6n  16  16  4  16
1

22  1  4x  2

 16  3n  3n  4  3n

24

 0.8

a  30
The age is 30 years.

4.

 16  3n  4
2

17
1

n  17
n  2  17  2 or 19
The consecutive odd integers are 17 and 19.
43. Let n  the least even integer.
Then n  2  the next greater even integer, and
n  4  the greatest of the three even integers.

v  0.925

8s
8

94

47. 2(3x  1)  2x  4(3x)


6x  2  2x  12x
2  8x  12x
2  8x  8x  12x  8x
2  4x
2
4
1
2

54.
7

4x
4

0.5  x
3x  1  3(0.5)  1 or 2.5
3x  3(0.5) or 1.5
The dimensions of the rectangle are 2.5 by 0.5,
and the dimensions of the square are 1.5 by 1.5.
48. 0.60(10)(1000)  50(8)(tf  50)
6000  400(tf  50)
6000  400tf  20,000
6000  20,000  400tf  20,000  20,000
26,000  400tf
26,000
400

400t

The number of
the number of
the number
Calories burned is Calories per minute times of minutes.
144
4244
43 123 144424443 123 14
424
43

Equation: C 
4.5
The equation is C  4.5m.
57. 150  4.5m

400

150
4.5
100
3
1
333

m
m



 
  

 
  
10

12

14

16

18

20

59. The lowest value is 19, and the highest value is


28, so use a scale that includes those values.
Place an  above each value for each occurrence.


 
 

 
 


 2

18

20

22

24

26

28

60. 10  (17)  ( 010 0  017 0 )


 (10  17)
 27
61. 12  (8)  12  (8)
 12  8
 ( 012 0  08 0 )
 (12  8)
 4
62. 6  14  6  (14)
 ( 014 0  06 0 )
 (14  6)
 8
63. Sample answer: 1 and 3
1  3  4 is even, but 1 and 3 are odd.
64. Sample answer: You could bake sugar cookies,
which do not require chocolate chips.

Maintain Your Skills


 6  14

 6  6  14  6
2
v
9
9 2
v
2 9

4.5m
4.5

58. The lowest value is 11, and the highest value is


17, so use a scale that includes those values.
Place an  above each value for each occurrence.

Exercises 5355 For checks, see students work.

2
v
9

x  4
5n  4  7(n  1)  2n
5n  4  7n  7  2n
5n  4  5n  7
5n  4  5n  5n  7  5n
47
Since 4  7 is a false statement, this equation has
no solution.

2
v
9

It will take 333 minutes.

52. C;

53.

18

 9

w  2
56. Words:
The number of Calories burned is
the number of Calories per minute
times the number of minutes.
Variables: Let C  the number of Calories
burned, and m  the number of
minutes.

The final temperature is 65F.


49. Sample answer: 3(x  1)  x  1
50. Set two expressions equal to each other and solve
the equation. Answers should include the following.
The steps used to solve the equation are (1)
subtract 7.6x from each side, (2) subtract 6
from each side, and (3) divide each side by 0.4.
The number of male and female Internet users
will be the same in 2010.
If two expressions that represent the growth
in use of two items are set equal to each other,
the solution to the equation can predict when
the number of items in use will be equal.
51. D;
8x  3  5(2x  1)
8x  3  10x  5
8x  3  10x  10x  5  10x
2x  3  5
2x  3  3  5  3
2x  8

Page 154

2  7(2)

9w
9

65  tf

2x
2

 2

x  3  14
x  3  3  14  3
x  11
5  9w  23
5  9w  5  23  5
9w  18

55.

x

x  3
7
x  3
7

 20

1 2  92(20)
v  90

95

Chapter 3

65.

3a2
b  c

3152 2

7.

87
75

87

3x
3

75
15

70.

72.

12  3
 15  3
4
5
36  12
36
 60  12
60
3
5
108
108  9
 99
9
12
 1
12
15

69.

71.

73.

9.

16
40

16  8

 40  8

75.

5.082
1.1

5
120

1750
120
175
12
7
1412

19  19

11.

 57  19

12.

6.

13.

 350

120g
120

g
g

3 ? 21
 14
2
?

8 ? 12
 18
9
?

14.

16
17

15.

8
.
9

2.3
3.4

3.0

 3.6.

4.2 ? 1.68
 2.24
5.6
?

21.1 ? 1.1
 1.2
14.4
?

21.1(1.2)  14.4(1.1)
25.32  15.84
No, the cross products are not equal, so
The ratios do not form a proportion.
16.

12

 18.

4.2(2.24)  5.6(1.68)
9.408  9.408
1.68
4.2
Yes, the cross products are equal, so 5.6  2.24.
Since the ratios are equal, they form a proportion.

2.1 ? 0.5
 0.7
3.5
?

2.1
3.5

8
9

2.3 ? 3.0
 3.6
3.4
?

2.3(3.6)  3.4(3.0)
8.28  10.2
No, the cross products are not equal, so
The ratios do not form a proportion.

16 ? 8
9
17
?

Chapter 3

1.1n
1.1

No, the cross products are not equal, so


The ratios do not form a proportion.

4 ? 12
 33
11
?

2.1(0.7)  3.5(0.5)
1.47  1.75
No, the cross products are not equal, so
The ratios do not form a proportion.

8(18)  9(12)
144  108

Check for Understanding

16(9)  17(8)
144  136
No, the cross products are not equal, so
The ratios do not form a proportion.

3(14)  2(21)
42  42
3
21
Yes, the cross products are equal, so 2  14. Since
the ratios are equal, they form a proportion.

3

4(33)  11(12)
132  132
12
4
Yes, the cross products are equal, so 11  33. Since
the ratios are equal, they form a proportion.
5.

a  15
n
8.47

Pages 158159 Practice and Apply

1. See students work.


2. A ratio is a comparison of two numbers, and a
proportion is an equation of two equal ratios.
3. Find the cross products and divide by the value
with the variable.
4.

225
15

The Lehmans need about 14.6 gallons.

Ratios and Proportions

Page 158

5(350)  120(g)
1750  120g

28  7
 49  7
4
7
8
8  8
 120  8
120
1
 15
28
28  14
 42  14
42
2
3

5

3-6

15a
15

4.62  n
10. Let g represent the amount of gasoline needed for
a 350-mile trip.

28
49

19
57

0.6
1.1

 15

a(15)  45(5)
15a  225

24
3

a
45

0.6(8.47)  1.1(n)
5.082  1.1n

 12
74.

8.

x8

5
66. x(a  2b)  y  2(5  2  8)  1
 2(5  16)  1
 2(21)  1
 42  1
 41
67. 5(x  2y)  4a  5(2  2  1)  4(5)
 5(2  2)  4(5)
 5(4)  4(5)
 20  20
0
68.

6
x

3(x)  4(6)
3x  24

3(25)

87


3
4

0.5
.
0.7

5 ? 4
 1.6
2
?

5(1.6)  2(4)
88

96

21.1
14.4

1.1

 1.2.

Yes, the cross products are equal, so 2  1.6. Since


the ratios are equal, they form a proportion.
17. For each row in the table write the ratio of the
number in the first column to the number in the
871
498
fourth column. USA: 2116; USSR/Russia: 1278;

5
3

30.

5x
5

374
180
; Great Britain: 638;
1182
136
179
188
; Italy: 479; Sweden: 469
598

19.

2
10

20.

4(10)  x(2)
40  2x
40
2

15
3

20  x
6
5

21.

 15

22.

23.

a
25.

1
0.19

56
6
28
3

152d
152

3y
3
n

 21

24.

16
7

248
4

28n
28

248h
248

b

or

1
93

b

63
16
63
16

2
0.21

2.5
10

30
10

1
1

64

28.

h

3.67s
3.67

3
20

x

x

 122

3(122)  20(n)
366  20n
366
20

7
 3

116
6
58
or
3

20n
20

18.3  n
You would expect about 18 animals.

6(x  3)  14(7)
6x  18  98
6x  18  18  98  18
6x  116
6x
6

or 46

35. Let n represent the number of pets from


a breeder.

1.066z
1.066
6
14

25
6

63.37  z
29.

The car is 46 feet high.

z
9.65

7(9.65)  1.066(z)
67.55  1.066z
67.55
1.066

2
3

1(h)  64

1.23  s
7
1.066

1 1 2123 2
25 2
h  4 13 2

s
1.88

2.405(1.88)  3.67(s)
4.5214  3.67s


10d
10

3d
The wall in the blueprint is 3 in. long.
34. Let h represent the actual height.

n  0.84

4.5214
3.67

 12

2.5(12)  10(d)
30  10d

1.68
2

33. Let d represent the scale length.


15

or 316

27.

372

 248
1

12
n

2.405
3.67

93
h

It will take 12 hours.

n

2(n)  0.21(8)
2n  1.68
2n
2

2128
152

h  2 or 12

1(n)  0.19(12)
n  2.28
26.

248(h)  4(93)
248h  372

16(b)  7(9)
16b  63
16b
16

532
d

d  14
It will take 14 days.
32. Let h represent the number of hours needed to
drive 93 miles.

15  n

7
a

6(a)  8(7)
6a  56
6a
6

20
28

420
28

18  x
6
8

152(d)  4(532)
152d  2128

20(21)  28(n)
420  28n

5x
5

152
4

3
15

5y

6(15)  5(x)
90  5x
90
5

31. Let d represent the number of days needed to


earn $532.

1(15)  y(3)
15  3y

2x
2

1
y

5

x  5 or 15

18. No; if two of these ratios formed a proportion, the


two countries would have the same part of their
medals as gold medals.
4
x

6
 2

5(x  2)  3(6)
5x  10  18
5x  10  10  18  10
5x  8

Germany:
France:

x

193

97

Chapter 3

36. Evaluate
2a  3b
4b  3c

The

2a  3b
with
4b  3c
2(3)  3(1)

 4(1)

43.

a  3, b  1, and c  5.

 3(5)

5  9w  23
5  9w  5  23  5
9w  18
9w
9

6  3
 4  15
9
 19
9
value is 19.

w  2
m
5

44.
m
5

37. Sample answer: Ratios are used to determine how


much of each ingredient to use for a given number
of servings. Answers should include the following.
To determine how much honey is needed if you
use 3 eggs, write and solve the proportion
2
3
 , where h is the amount of honey.
3
h

45.
5

48.

3

15  y
5

15
z

5

15(5)  z(3)
75  3z
75
3

3z
3

25  z

Page 159

2  5(3)

Maintain Your Skills

Temperature

Exercises 4045 For checks, see students work.


40.
8y  10  3y  2
8y  10  3y  3y  2  3y
11y  10  2
11y  10  10  2  10
11y  12
11y
11

12

 11
12

1 21 2  499
3 3
7 7

49. (0.075)(5.5)  0.4125


50. 33 is thirty-three units from zero in the
negative direction.
033 0  33
51. 77 is seventy-seven units from zero in the positive
direction.
077 0  77
52. 2.5 is two and five tenths units from zero in the
positive direction.
02.5 0  2.5
53. 0.85 is eighty-five hundredths unit from zero in
the negative direction.
|0.85|  0.85
54. The temperature is high when you enter the
house, but decreases to a lower constant
temperature due to the air conditioner.

2y
2

y
z

y  11 or 111

Time

17  2n  21  2n
17  2n  2n  21  2n  2n
17  21
Since 17  21 is a false statement, this equation
has no solution.
42.
7(d  3)  4
7d  21  4
7d  21  21  4  21
7d  25
41.

7d
7

d
Chapter 3

 3

 1

3

z  7
5
z  7
5

47. 9

10(3)  y(2)
30  2y
30
2

1 2  5(25)

1 8 2198 2  7272

9(27)  12(18)
243  216
10
y

 25

z  7  15
z  7  7  15  7
z  8
46. (7)(6)  42

9 ? 18
 27
12
?

x
y

m
5
m
5

m  125

39. C;

 6  31

 6  6  31  6
5

To alter the recipe to get 5 servings, multiply


1
each amount by 14.
38. D;

18

 9

25
7
25
or
7

55. The base is 60, and the part is 18. Let p represent
the percent.
a
b
18
60

 100
p

 100

18(100)  60(p)
1800  60p
1800
60

60p
60

30  p
Eighteen is 30% of 60.

37

98

Find the percent using the original number, 72, as


the base.

56. The base is 14, and the part is 4.34. Let p


represent the percent.
a
b
4.34
14

36
72

 100
p

36(100)  72(r)
3600  72r

 100

4.34(100)  14(p)
434  14p
434
14

3600
72

14p
14

 100
p

 100

5
45

6(100)  15(p)
600  15p
600
15

500
45

 100
p

 100

2p
2

2
14

400  p
Eight is 400% of 2.

Page 162

45r
45

 100

2(100)  14(r)
200  14r
200
14

3-7

 100

11  r
The percent of increase is about 11%.
6. Find the amount of change. Since the new
amount is greater than the original, the percent
of change is a percent of increase.
16  14  2
Find the percent using the original number, 14, as
the base.

8(100)  2(p)
800  2p
800
2

72r
72

5(100)  45(r)
500  45r

15p
15

40  p
Six is 40% of 15.
58. The base is 2, and the part is 8. Let p represent
the percent.
a
b
8
2

50  r
The percent of decrease is 50%.
5. Find the amount of change. Since the new
amount is greater than the original, the percent
of change is a percent of increase.
50  45  5
Find the percent using the original number, 45, as
the base.

31  p
Four and thirty-four hundredths is 31% of 14.
57. The base is 15, and the part is 6. Let p represent
the percent.
a
b
6
15

 100

14r
14

14  r
The percent of increase is about 14%.
7. The percent of change is a percent of decrease
because the new amount is less than the original.
Find the change.
150  120  30
Find the percent using the original number, 150,
as the base.

Percent of Change
Check for Understanding

1. Percent of increase and percent of decrease are


both percents of change. If the new number is
greater than the original number, the percent of
change is a percent of increase. If the new
number is less than the original number, the
percent of change is a percent of decrease.
2. Sample answer: If the original number is 10 and
the new number is 30, the percent proportion is
30  10
r
 100 and the percent of change is 200%,
10
which is greater than 100%.
3. Laura; Cory used the new number as the base
instead of the original number.
4. The percent of change is a percent of decrease
because the new amount is less than the original.
Find the change.
72  36  36

30
150

 100

30(100)  150(r)
3000  150r
3000
150

150r
150

20  r
The percent of decrease is 20%.
8. The tax is 6.5% of the price of the software.
6.5% of $39.50  0.065  39.50
 2.5675
Round $2.5675 to $2.57 since tax is always
rounded up to the nearest cent. Add this amount
to the original price.
$39.50  $2.57  $42.07
The total price of the software is $42.07.

99

Chapter 3

Find the percent using the original number, 25, as


the base.

9. The tax is 5.75% of the price of the compact disc.


5.75% of $15.99  0.0575  15.99
 0.919425
Round $0.919425 to $0.92 since tax is always
rounded up to the nearest cent. Add this amount
to the original price.
$15.99  $0.92  $16.91
The total price of the compact disc is $16.91.
10. The discount is 25% of the original price.
25% of $45  0.25  45
 11.25
Subtract $11.25 from the original price.
$45.00  $11.25  $33.75
The discounted price of the jeans is $33.75.
11. The discount is 33% of the original price.
33% of $19.95  0.33  19.95
 6.5835
Subtract $6.58 from the original price.
$19.95  $6.58  $13.37
The discounted price of the book is $13.37.
12. Find the amount of change.
40,478  22,895  17,583
Write the equation using the original number,
22,895, as the base.
17,583
22,895

7
25

7(100)  25(r)
700  25r
700
25

36
66

17,583
22,895

3600
66

 100 .

 100

94
58

9400
58

77  r
The percent of increase is about 77%.

Practice and Apply

2650
13.7

58r
58

 100

13.7r
13.7

193  r
The percent of increase is about 193%.
19. The percent of change is a percent of decrease
because the new amount is less than the original.
Find the change.
15.6  11.4  4.2

50r
50

40  r
The percent of increase is 40%.
15. The percent of change is a percent of decrease
because the new amount is less than the original.
Find the change.
25  18  7

Chapter 3

26.5(100)  13.7(r)
2650  13.7r

 100

 100

26.5
13.7

20(100)  50(r)
2000  50r
2000
50

66r
66

162  r
The percent of increase is about 162%.
18. Find the amount of change. Since the new
amount is greater than the original, the percent
of change is a percent of increase.
40.2  13.7  26.5
Find the percent using the original number, 13.7,
as the base.

14. Find the amount of change. Since the new


amount is greater than the original, the percent
of change is a percent of increase.
70  50  20
Find the percent using the original number, 50, as
the base.
20
50

94(100)  58(r)
9400  58r

22,895r
22,895

Pages 162164

 100

55  r
The percent of decrease is about 55%.
17. Find the amount of change. Since the new
amount is greater than the original, the percent
of change is a percent of increase.
152  58  94
Find the percent using the original number, 58, as
the base.

17,583(100)  22,895(r)
1,758,300  22,895r
1,758,300
22,895

25r
25

36(100)  66(r)
3600  66r

17,583
22,895

28  r
The percent of decrease is 28%.
16. The percent of change is a percent of decrease
because the new amount is less than the original.
Find the change.
66  30  36
Find the percent using the original number, 66, as
the base.

 100

13. Find the percent by solving

 100

100

Find the percent using the original number, 9.8,


as the base.

Find the percent using the original number, 15.6,


as the base.
4.2
15.6

2.3
9.8

 100

2.3(100)  9.8(r)
230  9.8r

4.2(100)  15.6(r)
420  15.6r
420
15.6

230
9.8

15.6r
15.6

7.5
40

 100

750
40

132r
132

3.5
25

 100

350
25

85r
85

25r
25

 100

64,000,000(100)  253,000,000(r)
6,400,000,000  253,000,000r

 100

 100

64,000,000
253,000,000

2.5(100)  32.5(r)
250  32.5r
250
32.5

40r
40

14  r
The percent of decrease is 14%.
26. Find the amount of change.
317,000,000  253,000,000  64,000,000
Find the percent using the original number,
253,000,000, as the base.

6r
The percent of increase is about 6%.
22. The percent of change is a percent of decrease
because the new amount is less than the original.
Find the change.
32.5  30  2.5
Find the percent using the original number, 32.5,
as the base.
2.5
32.5

3.5(100)  25(r)
350  25r

5(100)  85(r)
500  85r
500
85

 100

19  r
The percent of decrease is about 19%.
25. The percent of change is a percent of decrease
because the new amount is less than the original.
Find the change.
25  21.5  3.5
Find the percent using the original number, 25, as
the base.

14  r
The percent of increase is about 14%.
21. Find the amount of change. Since the new
amount is greater than the original, the percent
of change is a percent of increase.
90  85  5
Find the percent using the original number, 85, as
the base.
5
85

9.8r
9.8

7.5(100)  40(r)
750  40r

18(100)  132(r)
1800  132r
1800
132

23  r
The percent of increase is about 23%.
24. The percent of change is a percent of decrease
because the new amount is less than the original.
Find the change.
40  32.5  7.5
Find the percent using the original number, 40, as
the base.

27  r
The percent of decrease is about 27%.
20. Find the amount of change. Since the new
amount is greater than the original, the percent
of change is a percent of increase.
150  132  18
Find the percent using the original number, 132,
as the base.
18
132

 100

6,400,000,000
253,000,000

253,000,000r
253,000,000

25  r
The percent of increase is about 25%.
27. Find the change.
2,000,000  1,400,000  600,000
Find the percent using the original number,
2,000,000, as the base.

32.5r
32.5

8r
The percent of decrease is about 8%.
23. Find the amount of change. Since the new
amount is greater than the original, the percent
of change is a percent of increase.
12.1  9.8  2.3

600,000
2,000,000

 100

600,000(100)  2,000,000(r)
60,000,000  2,000,000r
60,000,000
2,000,000

2,000,000r
2,000,000

30  r
The percent of decrease is 30%.

101

Chapter 3

28. Let n  the original number. Since 16% is a


percent of increase, the new number is more than
the original number. Therefore, 522  n
represents the amount of change.
522  n
n

16

 100
34.

(522  n)(100)  n(16)


52,200  100n  16n
52,200  100n  100n  16n  100n
52,200  116n
52,200
116

116n
116

450  n
The original number is 450.
29. Let f  the amount of fat is one ounce of regular
chips. Since 25% is a percent of decrease, the
amount of fat in regular chips is more than the
amount of fat in reduced fat chips. Therefore,
f  6 represents the amount of change.
f  6
f

35.

25

 100

(f  6)(100)  f(25)
100f  600  25f
100f  600  100f  25f  100f
600  75f
600
75

36.

75f
75

8f
The least amount of fat in one ounce of regular
chips is 8 grams.
30. Let n  the number of internet hosts in 1996.
Since 1054% is a percent of increase, the number
of internet hosts in 1996 is less than the number of
internet hosts in 2001. Therefore, 109,600,000  n
represents the amount of change.
109,600,000  n
n

37.

1054
100

(109,600,000  n)(100)  n(1054)


10,960,000,000  100n  1054n
10,960,000,000  100n  100n  1054n  100n
10,960,000,000  1154n
10,960,000,000
1154

38.

1154n
1154

9,497,400  n
The number of internet hosts in 1996 was about
9.5 million.
31. The tax is 5.5% of the price of the umbrella.
5.5% of $14  0.055  14
 0.77
Add this amount to the original price.
$14.00  $0.77  $14.77
The total price of the umbrella is $14.77.
32. The tax is 7% of the price of the backpack.
7% of $35  0.07  35
 2.45
Add this amount to the original price.
$35.00  $2.45  $37.45
The total price of the backpack is $37.45.
33. The tax is 5.75% of the price of the candle.
5.75% of $7.50  0.0575  7.50
 0.43125

Chapter 3

39.

40.

41.

102

Round $0.43125 to $0.43 since tax is always


rounded to the nearest cent. Add this amount to
the original price.
$7.50  $0.43  $7.93
The total price of the candle is $7.93.
The tax is 6.25% of the price of the hat.
6.25% of $18.50  0.0625  18.50
 1.15625
Round $1.15625 to $1.16 since tax is always
rounded to the nearest cent. Add this amount to
the original price.
$18.50  $1.16  $19.66
The total price of the hat is $19.66.
The tax is 6.75% of the price of the clock radio.
6.75% of $39.99  0.0675  39.99
 2.699325
Round $2.699325 to $2.70 since tax is always
rounded to the nearest cent. Add this amount to
the original price.
$39.99  $2.70  $42.69
The total price of the clock radio is $42.69.
The tax is 5.75% of the price of the sandals.
5.75% of $29.99  0.0575  29.99
 1.724425
Round $1.724425 to $1.72 since tax is always
rounded to the nearest cent. Add this amount to
the original price.
$29.99  $1.72  $31.71
The total price of the sandals is $31.71.
The discount is 40% of the original price.
40% of $45  0.40  45
 18
Subtract $18 from the original price.
$45.00  $18.00  $27.00
The discounted price of the shirt is $27.00.
The discount is 20% of the original price.
20% of $6  0.20  6
 1.20
Subtract $1.20 from the original price.
$6.00  $1.20  $4.80
The discounted price of the socks is $4.80.
The discount is 35% of the original price.
35% of $37.55  0.35  37.55
 13.1425
Subtract $13.14 from the original price.
$37.55  $13.14  $24.41
The discounted price of the watch is $24.41.
The discount is 33% of the original price.
33% of $24.25  0.33  24.25
 8.0025
Subtract $8.00 from the original price.
$24.25  $8.00  $16.25
The discounted price of the gloves is $16.25.
The discount is 45% of the original price.
45% of $175.95  0.45  175.95
 79.1775

42.

43.

44.

45.

46.

x  1.24
1.24

Subtract $79.18 from the original price.


$175.95  $79.18  $96.77
The discounted price of the suit is $96.77.
The discount is 30% of the original price.
30% of $79.99  0.30  79.99
 23.997
Subtract $24.00 from the original price.
$79.99  $24.00  $55.99
The discounted price of the coat is $55.99.
The discount is 20% of the original price.
20% of $120  0.20  120
 24
Subtract $24 from the original price.
$120  $24  $96
The discounted price of the lamp is $96.
The tax is 6% of the discounted price of the lamp.
6% of $96  0.06  96
 5.76
Add this amount to the discounted price.
$96.00  $5.76  $101.76
The total price of the lamp is $101.76.
The discount is 30% of the original price.
30% of $70  0.30  70
 21
Subtract $21 from the original price.
$70  $21  $49
The discounted price of the dress is $49.
The tax is 7% of the discounted price of the dress.
7% of $49  0.07  49
 3.43
Add this amount to the discounted price.
$49.00  $3.43  $52.43
The total price of the dress is $52.43.
The discount is 25% of the original price.
25% of $58  0.25  58
 14.50
Subtract $14.50 from the original price.
$58.00  $14.50  $43.50
The discounted price of the camera is $43.50.
The tax is 6.5% of the discounted price of the
camera.
6.5% of $43.50  0.065  43.50
 2.8275
Round $2.8275 to $2.83 since tax is always
rounded up to the nearest cent.
Add this amount to the discounted price.
$43.50  $2.83  $46.33
The total price of the camera is $46.33.
Let x  the population in China in 2050. Since
22.6% is a percent of increase, the population in
2050 will be greater than the population in 1997.
Therefore, x  1.24 represents the amount of
change.

22.6
100

(x  1.24)(100)  1.24(22.6)
100x  124  28.024
100x  124  124  28.024  124
100x  152.024
100x
100

152.024
100

x  1.52024
The projected population of China is about 1.52
billion people in 2050.
Let x  the population in India in 2050.
Since 57.8% is a percent of increase, the
population in 2050 will be greater than the
population in 1997. Therefore, x  0.97 represents
the amount of change.
x  0.97
0.97

57.8
100

(x  0.97)(100)  0.97(57.8)
100x  97  56.066
100x  97  97  56.066  97
100x  153.066
100x
100

153.066
100

x  1.53066
The projected population of India is about 1.53
billion people in 2050.
Let x  the population in the U.S. in 2050.
Since 44.4% is a percent of increase, the
population in 2050 will be greater than the
population in 1997. Therefore, x  0.27 represents
the amount of change.
x  0.27
0.27

44.4
100

(x  0.27)(100)  0.27(44.4)
100x  27  11.988
100x  27  27  11.988  27
100x  38.988
100x
100

38.988
100

x  0.38988
The projected population of the U.S. is about 0.39
billion people in 2050.
47. Since the projected population for China is 1.52
billion people, for India is 1.53 billion people, and
for the U.S. is 0.39 billion people, India will be
the most populous in 2050.
48. See students work.
x

49. Always; x% of y 1 100 


y

y% of x 1 100 

103

P
y
P
x

xy

or P  100;
xy

or P  100

Chapter 3

50. Find the amount of change and express this


change as a percent of the original number.
Answers should include the following.
To find the percent of increase, first find the
amount of increase. Then find what percent
the amount of increase is of the original
number.
The percent of increase from 1996 to 1999 is
about 67%.
An increase of 100 is a very large increase if
the original number is 50, but a very small
increase if the original number is 100,000. The
percent of change will indicate whether the
change is large or small relative to the
original.
51. B; Find the amount of change.
910  840  70
Write the proportion using the original number,
840, as the base.
70
840

52. C; The discount is 30% of the original price.


30% of $249.00  0.30  249.00
 74.70
Subtract $74.70 from the original price.
$249.00  $74.70  $174.30

3t
3

67.
53.

Maintain Your Skills




3
15

a(15)  45(3)
15a  135
15a
15

2
3

d

55.

2(d)  3(8)
2d  24
2d
2

5.22
13.92

250.56
13.92

 48

13.92t
13.92

6
6

5c
5

n  1
58.
18  4a  2
18  2  4a  2  2
20  4a
20
4

69.

30
5

c  6

2
5

70.

4a
4

 4  54

60. 5  3  5  2
 10

 5 or 1 5

 10

12
6

104

2a
2

9.5  a

28
7

22
2

p  11
5  4  2(a  5)
5  4  2a  10
5  14  2a
5  14  14  2a  14
19  2a
19
2

 20

Chapter 3

12
4

d4
6(p  3)  4(p  1)
6p  18  4p  4
6p  18  4p  4p  4  4p
2p  18  4
2p  18  18  4  18
2p  22
2p
2

5a
59.

7d
7

d  12
18  t
Exercises 5658 For checks, see students work.
56.
57.
6n  3  3
7  5c  23
6n  3  3  3  3
7  5c  7  23  7
6n  6
5c  30
6n
6

45
3

y  3
68. 7(d  3)  2  5
7d  21  2  5
7d  23  5
7d  23  23  5  23
7d  28

5.22(48)  13.92(t)
250.56  13.92t

24
2

t  15
7y  7  3y  5
7y  7  3y  3y  5  3y
4y  7  5
4y  7  7  5  7
4y  12
4y
4

135
15

a9
54.

 27

64. False; 8y  y2  y  10
?
8  4  42  4  10
?
8  4  16  14
?
32  16  14
16  14
65. True; 16p  p  15p
?
16(2.5)  2.5  15(2.5)
?
40  2.5  37.5
37.5  37.5
Exercises 6671 For checks, see students work.
66.
43  3t  2  6t
43  3t  6t  2  6t  6t
43  3t  2
43  3t  43  2  43
3t  45

a
45

1 42

62. True; a2  5  17  a 63. False; 2v2  v  65


?
?
32  5  17  3
2(5)2  5  65
?
?
9  5  14
2(25)  5  65
?
50  5  65
14  14
55  65

 100

Page 164

1 32

61. 9  4  9  3

71.

8x  4  10x  50
8x  4  10x  10x  50  10x
18x  4  50
18x  4  4  50  4
18x  54
18x
18

5. 7  2(w  1)  2w  9
7  2w  2  2w  9
2w  9  2w  9
Since the expressions on each side of the equation
are the same, this equation is an identity. The
statement 7  2(w  1)  2w  9 is true for all
values of w.
6.
8(4  9r)  7(2  11r)
32  72r  14  77r
32  72r  77r  14  77r  77r
5r  32  14
5r  32  32  14  32
5r  18

54

 18

x3

Page 164
1.

Practice Quiz 2

3x  7  18
3x  7  7  18  7
3x  25
3x
3

5r
5

25

 3
25

r

x   3 or 8 3

Check:

?
?

25  7  18
18  18
1

The solution is 8 3.
5
4(5) 

m  5
4
m  5
4 4

7.

2a
2

m  5
4
? 25  5
 4
? 20
 4

4y
4

h  2 or 1.5

24
10

4h  5  11
?
4(1.5)  5  11
?
6  5  11
11  11
The solution is 1.5.
4.
5d  6  3d  9
5d  6  3d  3d  9  3d
2d  6  9
2d  6  6  9  6
2d  15
Check:

d

15
2
15
2

24
x

3(x)  5(24)
3x  120
3x
3

120
3

x  40
y  5
8

20
4

y5
10. Find the amount of change.
34  10  24
Find the percent using the original number, 10, as
the base.

3
5

y(8)  4(y  5)
8y  4y  20
8y  4y  4y  20  4y
4y  20

4

2d
2

8.

10
2
y
4

9.

55
The solution is 25.
3.
4h  5  11
4h  5  5  11  5
4h  6
4h
4

a

a5

5
5

2
10

2(a)  10(1)
2a  10

20  m  5
20  5  m  5  5
25  m
Check:

or 3.6

8(4  9r)  7(2  11r)


?
8[4  9(3.6)]  7[2  11(3.6)]
?
8[4  32.4]  7[2  39.6]
?
8[36.4]  7[41.6]
291.2  291.2
The solution is 3.6.

3 8 3  7  18

2.

18
5
18
5

Check:

3x  7  18
1

 100

24(100)  10(r)
2400  10r
2400
10

10r
10

240  r
The percent of increase is 240%.

Page 165

Reading Mathematics

1. original number: 166 lb


amount of change: 166  158 or 8 lb
166  158
166
8
166

or 7.5

5d  6  3d  9
?
5(7.5)  6  3(7.5)  9
?
37.5  6  22.5  9
31.5  31.5
The solution is 7.5.
Check:

 100
r

 100

8(100)  166(r)
800  166r
800
166

166r
166

5r
There was about a 5% decrease in Monsas weight.

105

Chapter 3

2. original number: 75 points


amount of change: 94  75 or 19 points
94  75
75
19
75

6.

 100
r

 100

4z  b  2z  c
4z  b  2z  2z  c  2z
2z  b  c
2z  b  b  c  b
2z  c  b
2z
2

19(100)  75(r)
1900  75r
1900
75

z

75r
75

The value of z

25  r
There was about a 25% increase in her score.
3. original number: 12 orders
amount of change: 18  12 or 6 orders
18  12
12
6
12

7.
3

 100

p
b  c
p
b  c

12r
12

50  r
There was a 50% increase in production

Check for Understanding

1. (1) Subtract az from each side. (2) Add y to each


side. (3) Use the Distributive Property to write
ax  az as a(x  z). (4) Divide each side by x  z.
2. t can be any number except 2.

10. A  2bh
1

A  63
The area is 63 ft2.
12. h 

bh
h

h

b

The value of b is

b
9
b
9

13.

x

2 1

 b 2bh
h

5a  y  54
5a  y  y  54  y
5a  54  y

Practice and Apply

14.
5g  h  g
8t  r  12t
5g  h  5g  g  5g
8t  r  8t  12t  8t
h  4g
r  4t
h
4
h
4

Chapter 3

2
(A)
b
2A
b

2A
b
2(28)
8

Pages 168170

9x
9

The value of x is 9.

The value of

A  2bh

h7
The height is 7 ft.

2A
.
h

3x  b  6x
3x  b  3x  6x  3x
b  9x

11.

A  2(18) (7)

11 2

5a
5

5  t

m2
5  t

2A  bh

5.

mw  t  2w  5
mw  t  2w  2w  5  2w
mw  t  2w  5
mw  t  2w  t  5  t
mw  2w  5  t
w(m  2)  5  t

The value of w is m  2. Since division by 0 is


undefined, m  2  0 or m  2.

2A  2 2bh

4.

a

5  t

A  2bh

a(b  c)
b  c

wm2

2A
h
2A
h

w(m  2)
m  2

3. Sample answer: For a triangle, area is 2 times the


product of the length of the base times the height
1
or A  2bh. Solve for b.
1

2  3(c)

Solving Equations and Formulas

Page 168

c

The value of a is b  c. Since division by 0 is


undefined, b  c  0 or b  c.
9.

3-8

y  a
3
y  a
3

c  b
2
c  b
2
c  b
is 2 .

y  a  3c
y  a  a  3c  a
y  3c  a
The value of y is 3c  a.
8.
p  a(b  c)

 100

6(100)  12(r)
600  12r
600
12

4g
4

g

The value of g is

54  y
5
54  y
54  y
 5 or 5
54  y
a is 5 .

106

h
4 .

r
4
r
4

4t
4

t
r

The value of t is 4.

15.

y  mx  b
y  b  mx  b  b
y  b  mx
y  b
x
y  b
x

m
y  b
.
x

3ax
3a

22.

a

y
The value of y

19.

23.

am  z
7
am  z
7
am  z
is 7 .

4b
4

b
The value for

by  2
3
by  2
3

5x  y
.
2

c

2  3(c)

3c  2
b
3c  2
y b
3c  2
value of y is b .
by
b

The
undefined, b  0.
24.
7

6c  t
7
6c  t
7

Since division by 0 is

b

2  7(b)

6c  t  7b
6c  t  t  7b  t
6c  7b  t

km  5x  6y
km  5x  5x  6y  5x
km  6y  5x

6c
6

The value
undefined, k  0.
20.
4b  5  t
4b  5  5  t  5
4b  t  5

a

by  2  3c
by  2  2  3c  2
by  3c  2

6y  5x
k
6y  5x
m k
6y  5x
of m is
.
k

2a
2

The value of a is

c  2b
5
c  2b
2b  c
a  5 or 5
2b  c
of a is 5 .

km
k

2

2  a(2)

5x  y
2
5x  y
2

9a  2b  c  4a
9a  2b  4a  c  4a  4a
5a  2b  c
5a  2b  2b  c  2b
5a  c  2b
5a
5

5x  y
a
5x  y
a

by 0 is

5x  y  2a

v  r

7y
7

20  n
3a
20  n
n  20
 3a or 3a
n  20
x is 3a . Since division

The value of
undefined, a  0.

at
t

The value

 4

2  5(4)

Since division by 0 is

The value of a is t . Since division by 0 is


undefined, t  0.
17.
3y  z  am  4y
3y  z  4y  am  4y  4y
7y  z  am
7y  z  z  am  z
7y  am  z

18.

3ax  n
5
3ax  n
5

3ax  n  20
3ax  n  n  20  n
3ax  20  n

mx
x

The value of m is
undefined, x  0.
16.
v  r  at
v  r  r  at  r
v  r  at
v  r
t
v  r
t

21.

c
The value of
Since division by 0 is

7b  t
6
7b  t
6
7b  t
c is 6 .

c  4y  b

25.

c  b  4y  b  b
3

c  b  4y
4
(c
3
4
(c
3

t  5
4
t  5
5  t
or 4
4
5  t
b is 4 .

1 2

4 3

 b)  3 4y
 b)  y

The value of y is 3(c  b).


3
m
5

26.
3
m
5

ab

aaba
3
m
5
5 3
m
3 5

ba

1 2  53(b  a)
5

m  3(b  a)
5

The value of m is 3(b  a).

107

Chapter 3

S  2 (A  t)

27.

2
(S)
n
2S
n
2S

t
n
2S
t
n
2S  nt
n

2 n

 n 2 (A  t)
At

33. Five-eighths of

a number
x
14
42443
5
8

Att
A
A

The value of A is
undefined, n  0.
28. p(t  1)  2
p(t  1)
p

2S  nt
.
n

2
Since division by 0 is

c  b
t  r
c  b
c  b
a  t  r or r  t
c  b
a is r  t . Since division

35.

2
2
1
(A)  a  b 2h(a
a  b
2A
h
a  b
2A
hab
2(60)
h  8  12

S

by 0 is
37. e
e

 b)

w  10e
m
w  10e
m m

ms  w
10e
 10
10
ms  w
e
10
w  ms
e
10
w  ms
 10
410  5(76)

10

e3
Miguel made 3 errors.
38.

P
H2
(P)
1.2

39.

1.2W
H2

H 2 1.2W
H2

 1.2

H 2P
W
1.2
H2P
W  1.2

W

32  30
1.2

W  225
The person weighs 225 lb.
40. Solve the formula for F.
R
(S  P)R 

six times
twice a
another
minus
p 1424
equals3 number
plus
one.
1
424
3 number
14243
1442443q 1
23 1
23
2p
6q



1

S  F  P
S  P
S  F  P
(S  P) S  P

(S  P )R  S  F  P
(S  P)R  S  S  F  P  S
(S  P)R  S  F  P
(S  P)R  S  P  F  P  P
(S  P)R  S  P  F
F  (S  P)R  S  P
F  (900  900)6  900  900
F  9000
9000 grams of fuel should be loaded.

5  2p  6q  1
5  2p  5  6q  1  5
2p  6q  4
6q  4
2

p  3q  2 or 2  3q
The value of p is 2  3q.
Chapter 3

33

ms  w  10e
ms  w  w  10e  w
ms  w  10e

r6
r65
 r  11
of t is r  11.

y

m(s) 

A
number
t 123
minus 1
five
equals another
number
plus
six.
14
4244
3
23 123
1444244
43r 1
23 1
23
t

5

r

6

2p
2

2 1 2

36.

The value of g
division by 0 is
undefined, 2  h  0 or h  2.

Five
1
23
5

y

h6
The height is 6 meters.

5  m
2  h
5  m
g 2h
5  m
is 2  h . Since

32.

1
y
2
1
y
2
1
y
2
1
2 2y

A  2h(a  b)

The value of
undefined, r  t  0 or r  t.
30.
2g  m  5  gh
2g  m  gh  5  gh  gh
2g  m  gh  5
2g  m  gh  m  5  m
2g  gh  5  m
g(2  h)  5  m

t5
t55
t
The value

5
x3
8
5
x3
8
5
x3
8
5
x6
4

34.

Since division by 0 is
undefined, p  0.
29.
at  b  ar  c
at  b  ar  ar  c  ar
at  b  ar  c
at  b  ar  b  c  b
at  ar  c  b
a(t  r)  c  b

31.

5
x
8

plus
1
23

2
p
2
t1 p
2
t11 p 1
2
t p 1
2  p
t p
2  p
The value of t is p .

g(2  h)
2  h

x

1
2

The value of y is 4x  6.

a(t  r)
t  r

one-half of
another
number
14
44244
43y

is
{

108

three.
1
23
3

Subtract $5.25 from the original price.


$15.00  $5.25  $9.75
The discounted price of the scarf is $9.75.
48. The discount is 15% of the original price.
15% of $299  0.15  299
 44.85
Subtract $44.85 from the original price.
$299.00  $44.85  $254.15
The discounted price of the television is $254.15.

41. Solve for h.


V  r2h
V
r2h
 r2
r2
V
h
r2
V
h  r2

h

1202 22

5453


h  17.4
The height of the container should be about
17.4 cm.
42. The area of the arrow is the sum of the area of
the triangle with base  3s and height  s and
the square with side length  s.

2
9

49.

a

2(a)  9(5)
2a  45
2a
2

45
2
45
2

a

a  222 or 22.5

A  2(3s)(s)  s2

A  2s2  s2

x
4x  4

120
32
15
4
3
34

15 2 y 2

32t
32

t
 t or 3.75  t
3

4

(x  1)(4)  8(3)
4x  4  24
4x  4  4  24  4
4x  20
4x
4

20
4

x5
52. Write each number as a decimal.

5  y
.
2

1
4

 0.25

1
4

 0.5

0.5  0.555555 p or about 0.56


0.2  0.2
0.2 6 0.25 6 0.5 6 0.56
The numbers arranged in order from least to

greatest are 0.2, 4, 3 4, 0.5.

 2(5  y)
 10  2y
45. C; A  2bh

A  2(16)(7)

53. Write each number as a decimal.


25  2.23606797 p or about 2.2.
3  3.0

A  56
The area of the triangle is 56 m2.

2
3

Page 170

x  1
8

51.

5  y
2
5  y
2

Now, replace x in 4x with

8

15(8)  32(t)
120  32t

43. Equations from physics can be used to determine


the height needed to produce the desired results.
Answers should include the following.
Use the following steps to solve for h. (1) Use
the Distributive Property to write the
1
equation in the form 195g  hg  2mv2.
(2) Subtract 195 from each side. (3) divide
each side by g.
The second hill should be 157 ft.
44. B; First, solve 2x  y  5 for x.
2x  y  5
2x  y  y  5  y
2x  5  y
2x
2

15
32

50.

A  2 s2

 0.666666 p or about 0.7.

1.1  1.1
0.7 6 1.1 6 2.2 6 3.0
The numbers arranged in order from least to
2
greatest are 3, 1.1, 25, 3.

Maintain Your Skills

46. The discount is 20% of the original price.


20% of $85  0.20  85
 17
Subtract $17 from the original price.
$85  $17  $68
The discounted price of the camera is $68.00.
47. The discount is 35% of the original price.
35% of $15  0.35  15
 5.25

54. 2.18  (5.62)  ( 05.62 0  02.18 0 )


 (5.62  2.18)
 3.44

109

Chapter 3

1 32

5.

55. 2  4  4  4
 `

1

4
2

10

3
2
` ` `
4
4

10

The equation is 0.10(6  p)  1.00p  0.40(6).


6. 0.10(6  p)  1.00p  0.40(6)
0.6  0.1p  p  2.4
0.6  0.9p  2.4
0.6  0.9p  0.6  2.4  0.6
0.9p  1.8

134  24 2
6

56. 3  5  15  15

 15  15

0.9p
0.9

11015  156 2

16

 15 or 115
57.
58.
59.
60.
61.

Multiplicative Identity Property


Symmetric Property of Equality
Reflexive Property of Equality
Substitution Property of Equality
6(2  t)  6(2)  6(t)
 12  6t
62. (5  2m)3  (5)3  (2m)3
 15  6m
63. 7(3a  b)  7(3a)  (7)(b)
 21a  (7b)
 21a  7b
2

Units(lb) Price per Unit (lb) Total Price

 4h  6
65.

Walnuts

10

$4.00

Cashews

$7.00

1 32

 5t)  5(15)  5 (5t)

 9  (3t)
 9  3t
66. 0.25(6p  12)  0.25(6p)  0.25(12)
 1.5p  3

7.00 c

Mixture

10  c

$5.50

5.50(10  c)

1.5c
1.5

15

 1.5

c  10
10 pounds of cashews should be mixed with
10 pounds of walnuts.
4(1)  3(1)  4(1)  3(1)  4

3-9

9.

Weighted Averages

Page 174

11112

1. Sample answer: grade point average


2. The formula d  rt is used to solve uniform
motion problems. In the formula, d represents
distance, r represents rate, and t represents time.
3. Let d  the number of dimes.

Quarters

Number
of Coins

Value of
Each Coin

$0.10

0.10d

d8

$0.25

0.25(d  8)

Total
Value

First Cyclist
Second Cyclist
Distance traveled
by first cyclist
1444244
43
20t

Quarts

Amount of Orange Juice

10% Juice

6p

0.10(6  p)

100% Juice

1.00p

40% Juice

0.40(6)

43432
1

42

16
412

 3.56
Her GPA was about 3.56.
10. Let t  the number of hours until the two cyclists
are 15 miles apart.

4.

Chapter 3

112 2

Check for Understanding

Dimes

4.00(10)

Price of
price of
price of
walnuts
plus
cashews
equals
mixture.
1
424
3 123 1
424
3 123
1
424
3
4.00(10)

7.00c

5.50(10  c)
4.00(10)  7.00c  5.50(10  c)
40  7c  55  5.5c
40  7c  5.5c  55  5.5c  5.5c
40  1.5c  55
40  1.5c  40  55  40
1.5c  15

64. 3(6h  9)  3(6h)  3(9)


3
5(15

1.8

 0.9

p2
2 quarts of pure orange juice
7. Replace p in 6  p with 2.
6p62
4
4 quarts of 10% juice
8. Let C  the number of pounds cashews in the
mixture.

10
6
  ` `  ` `
15
15


Amount of orange
amount of orange
amount of orange
juice in 10% juice plus juice in 100% juice equals juice in 40% juice.
144424443 123 144424443 123 144424443
0.10(6  p)

1.00p

0.40(6)

minus
123


r
20
14

t
t
t

distance traveled
by
second cyclist
1
444244
43
14t

d  rt
20t
14t
equals
123


15.
1
23
15

20t  14t  15
6t  15
6t
6

15
6

t  2.5
The cyclists will be 15 miles apart in 2.5 hours.

110

Pages 174177

21. 40t  30t  245


70t  245

Practice and Apply

11.
Number of Price per
Dozens
Dozen
Peanut Butter
Cookies
Chocolate Chip
Cookies
12.

$6.50

6.50p

p  85

$9.00

9.00(p  85)

1444442444443 123

t

sales of the
chocolate chip cookies

equals

9.00(p  85)

1444442444443 1
424
3

15.5p
15.5

22. Let s  the number of rolls of solid gift wrap sold.


Rolls Price per Roll Total Price
Solid Wrap
s
$4.00
4.00s
Print Wrap 480  s
$6.00
6.00(480  s)

total
sales.
123
4055.50

Price
of solid wrap 1
plus
of print wrap 123
equals $2340.
144424443
23 price
144424443
123
4.00s

6.00(480  s)

2340

4.00s  6.00(480  s)  2340


4s  2880  6s  2340
2880  2s  2340
2880  2s  2880  2340  2880
2s  540

4820.5
15.5

2s
2

p  311
311 doz peanut butter cookies were sold.
14. Replace p in p  85 with 311.
p  85  311  85
 226
226 doz chocolate chip cookies were sold.
15.

16.

Value
of 43
gold
14424
270g

Number of
Ounces
g
15  g
15
plus
1
23


Price per
Ounce
$270
$5
$164

value of silver
1442443
5(15  g)

equals
123


265g
265

Value
270g
5(15  g)
164(15)

Units (lb)

Price per
Unit (lb)

Total Price

$6.40 coffee

$6.40

6.40(9)

$7.28 coffee

$7.28

7.28p

$6.95 coffee

9p

$6.95

6.95(9  p)

0.33p
0.33

g9
9 oz of gold was used.
18. Replace g in 15  g with 9.
15  g  15  9
6
6 oz of silver was used.
19.
r
40
30

540
2

Price of
price of
price of
$6.40
coffee
plus
$7.28
coffee
equals
$6.95
coffee.
1442443 123 1442443 123 14243
6.40(9)

7.28p
 6.95(9  p)
6.40(9)  7.28p  6.95(9  p)
57.6  7.28p  62.55  6.95p
57.6  7.28p  6.95p  62.55  6.95p  6.95p
57.6  0.33p  62.55
57.6  0.33p  57.6  62.55  57.6
0.33p  4.95

value
of alloy.
14424
43
164(15)

2385
265

Eastbound Train
Westbound Train

s  270
480  s  480  270 or 210
270 rolls of solid wrap and 210 rolls of print wrap
were sold.
23. Let p  the number of pounds of the $7.28 coffee
in the mixture.

The equation is 270g  5(15  g)  164(15).


17. 270g  5(15  g)  164(15)
270g  75  5g  2460
265g  75  2460
265g  75  75  2460  75
265g  2385

20.

245
70
1
32

The trains will be 245 miles apart in 32 hours.

The equation is 6.50p  9.00( p  85)  4055.50.


13. 6.50p  9.00( p  85)  4055.50
6.5p  9p  765  4055.5
15.5p  765  4055.5
15.5p  765  765  4055.5  765
15.5p  4820.5

Gold
Silver
Alloy

Sales of the
peanut butter cookies plus
6.50p

70t
70

Total
Price

4.95

 0.33

p  15
15 pounds of the $7.28 coffee should be mixed
with 9 pounds of the $6.40 coffee.

t
t
t

d  rt
40t
30t

Distance traveled
distance traveled
by
eastbound train 123
plus by
westbound train equals
144424443
144424443
123
40t

30t


245
1
23.
245

The equation is 40t  30t  245.

111

Chapter 3

27. Let x  the amount of 25% solution to be added.

24. Let w  the number of gallons of whipping cream.

Whipping
Cream
2% Milk
4% Milk

Units
(gal)

Percent
Butterfat

Total
Butterfat

w
35w
35

9%
2%
4%

0.09w
0.02(35  w)
0.04(35)

25% Solution
60% Solution
30% Solution

0.25x  0.60(140  x)  0.30(140)


0.25x  84  0.60x  42
84  0.35x  42
84  0.35x  84  42  84
0.35x  42
0.35x
0.35

w  10
35  w  35  10 or 25
10 gallons of whipping cream should be mixed
with 25 gallons of 2% milk.
25. Let x  the amount of 25% copper to be added.

40% Glycol
60% Glycol
48% Glycol

Amount of Alloy Amount of Copper


x

0.25x

50% Copper

1000  x

0.50(1000  x)

45% Copper

1000

0.45(1000)

Amount of
copper in
25% copper

plus
14243 1
23
0.25x

amount of
copper in
50% copper

amount of
copper in
equals 45% copper.


0.45(1000)

0.20x
0.20

r


 500

29.

85(3)  92(3)  82(1)  75(1)  95(1)


3  3  1  1  1
255  276  82  75  95

9
783
 9

 87
The average grade is 87.
30. Let t  the number of hours until the helicopter
reaches the trawler.

d
r t
1000
 3
1
 3333
1
or 3333 miles

or 500 miles per hour


per hour
You must find the weighted average.
1
500(2)  3333(3)
M
23
2000
 5

Trawler
Helicopter

r
30
300

t
t
t

d  rt
30t
300t

Distance traveled
distance traveled
by trawler
plus
by helicopter
equals
660
144424443
123
144424443
123 1
42km.
43
30t

300t

660

 400
The airplanes average speed was 400 mph.
Chapter 3

12

 0.20

x  60
100  x  100  60 or 40
60 gallons of 40% antifreeze should be mixed with
40 gallons of 60% antifreeze.

50

 0.25

x  200
1000  x  1000  200 or 800
200 g of 25% copper alloy should be mixed with
800 g of 50% copper alloy.
26. To find the average speed for each leg of the trip,
d
rewrite d  rt as r  t .
East
South
d
t
1000
2

Amount of
Glycol
0.40x
0.60(100  x)
0.48(100)

0.40x  0.60(100  x)  0.48(100)


0.40x  60  0.60x  48
60  0.20x  48
60  0.20x  60  48  60
0.20x  12

0.25x  0.50(1000  x)  0.45(1000)


0.25x  500  0.50x  450
500  0.25x  450
500  0.25x  500  450  500
0.25x  50
0.25x
0.25

Amount of
Solution
x
100  x
100

Amount of
amount of
amount of
glycol in
glycol in
glycol in
40% solution 1
plus
60% solution 123
equals 1442443
48% solution.
1442443
23 1442443
0.40x

0.60(100  x)

0.48(100)

14243 123 14243

0.50(1000  x)

42

 0.35

x  120
140  x  140  120 or 20
120 mL of 25% solution should be mixed with
20 mL of 60% solution.
28. Let x  the amount of 40% glycol to be added.

0.7
0.07

25% Copper

Amount of
Copper Sulfate
0.25x
0.60(140  x)
0.30(140)

Amount of
amount of
amount of
copper sulfate
copper sulfate
copper sulfate
in
25% solution 123
plus in
60% solution 123
equals in
30% solution.
1442443
1442443
1442443
0.25x

0.60(140  x)

0.30(140)

Amount of
butterfat in
amount of
amount of
whipping
butterfat
butterfat
cream
plus
in
2%
milk
equals
in
4%
milk.
14243 123 144244
3 123 1
44
244
3
0.09w
 0.02(35  w)

0.04(35)
0.09w  0.02(35  w)  0.04(35)
0.09w  0.7  0.02w  1.4
0.07w  0.7  1.4
0.07w  0.7  0.7  1.4  0.7
0.07w  0.7
0.07w
0.07

Amount of
Solution
x
140  x
140

112

30t  300t  660


330t  660
330t
330

0.25(16  x)  1.00x  0.40(16)


4  0.25x  x  6.4
4  0.75x  6.4
4  0.75x  4  6.4  4
0.75x  24

660

 330

t2
The helicopter will reach the trawler in 2 hours.
31. Let t  the number of seconds until the cheetah
catches its prey.
r
90
70

Cheetah
Prey
equals

distance traveled
by prey

90t

70t

1442443

123

1444442444443

plus

300 ft.

300

14243

Express Train
Local Train

123

90t  70t  300


90t  70t  70t  300  70t
20t  300
20t
20

t  15
The cheetah will catch the prey in 15 seconds.
32. Let t  the number of seconds until the sprinter
catches his opponent.
r
8.2
8

Distance traveled
by sprinter
equals
1442443
1
4243

8.2t

8.2t  8(t  1)
8.2t  8t  8
8.2t  8t  8t  8  8t
0.2t  8
0.2t
0.2

t
t
t1

32t
32

8(t  1)

 0.2

t  40
Distance traveled by sprinter is 8.2t  8.2(40) or
328 meters. Therefore, the sprinter would not
catch his opponent in a 200-meter race.
33. Let x  the amount of 100% antifreeze to be
added.

25% of Antifreeze
100% Antifreeze
40% Antifreeze

t
d  rt
t
80t
t  2 48(t  2)

96

 32

t3
The distance from Ironton to Wildwood is
80t  80(3) or 240 km.
35. R  [50  2000(C  A)  8000(T  A)
10,000(I  A)  100(Y  A)]  24
 [50  2000(297  474)  8000(33  474)
10,000(16  474)  100(3937  474)]  24
 98.0
Daunte Culpeppers rating was about 98.0.
36. Sample answer: How many grams of salt must be
added to 40 grams of a 28% salt solution to obtain
a 40% salt solution?
37. A weighted average is used to determine a
skaters average. Answers should include the
following.
The score of the short program is added to
twice the score of the long program. The sum
is divided by 3.

d  rt
8.2t
8(t  1)

distance traveled
by opponent.
1444244
43

Amount of
Solution
16  x
x
16

r
80
48

Distance traveled
distance traveled
by
express
train
equals
by local
train.
144424443 123 144
424
443
80t

48(t  2)
80t  48(t  2)
80t  48t  96
80t  48t  48t  96  48t
32t  96

300
20

Sprinter
Opponent

2.4

 0.75

x  3.2
3.2 quarts of pure antifreeze must replace 25%
antifreeze solution.
34. Let t  the time the express train travels.

d  rt
90t
70t

t
t
t

Distance traveled
by cheetah

0.75x
0.75

4.9(1)  5.2(2)
1  2

Amount of
Antifreeze
0.25(16  x)
1.00x
0.40(16)

 5.1

38. D;
Amount
Amount at 4.5%
d
Amount at 6% 6000  d

Interest Amount of
Rate
Interest
4.5%
0.045d
6%
0.06(6000  d)

39. C;

Amount of
amount of
amount of
antifreeze
antifreeze
antifreeze
in 25%
in 100%
in 40%
solution
plus
solution
equals
solution.
14243 123 14243 123 14243
0.25(16  x) 
1.00x

0.40(16)

r

d
t
616

 16  2


616
14

 44 mph

113

Chapter 3

Page 177
40.

46. (2b)(3a)  2(3)ab


 6ab
47. 3x(3y)  (6x)(2y)  3(3)xy  (6)(2)xy
 9xy  12xy
 (9  12)xy
 3xy
48. 5s(6t)  2s(8t)  5(6)st  2(8)st
 30st  (16)st
 30st  16st
 (30  16)st
 46st
49. The bold arrow on the left means that the graph
continues indefinitely in that direction. The
coordinates are {. . ., 2, 1, 0, 1, 2, 3}.
50. The dots indicate each point on the graph.
The coordinates are {0, 2, 5, 6, 8}.

Maintain Your Skills

3t  4  6t  s
3t  4  6t  6t  s  6t
3t  4  s
3t  4  4  s  4
3t  s  4
3t
3

t
a6

41.

4(a  6) 

s  4
3
s  4
s  4
or 3
3
b  1
4
b  1
4 4

4a  24  b  1
4a  24  1  b  1  1
4a  25  b
42. The percent of change is a percent of decrease
because the new amount is less than the original.
Find the change.
25  14  11
Find the percent using the original number, 25, as
the base.
11
25

Page 178
1.

 100

16.95(56)  12.59(97)  10.75(124)  10.15(71)  11.25(69)  9.95(45)


56  97  124  71  69  45
949.20  1221.23  1333  720.65  776.25  447.75

56  97  124  71  69  45
5448.08
 462

11(100)  25(r)
1100  25r
1100
25

25r
25

 11.7923810
The average price was about $11.79.

44  r
The percent of decrease is 44%.
43. Find the amount of change. Since the new
amount is greater than the original, the percent
of change is a percent of increase.
42  35  7
Find the percent using the original number, 35, as
the base.
7
35

2.

18.65(56)  13.85(97)  11.83(124)  11.17(71)  12.38(69)  10.95(45)


56  97  124  71  69  45

35r
35

20  r
The percent of increase is 20%.
44. Find the amount of change. Since the new
amount is greater than the original, the percent
of change is a percent of increase.
300  244  56
Find the percent using the original number, 244,
as the base.
56
244

5994.81
462

4.
16.95(50)  12.59(50)  10.75(50)  10.15(50)  11.25(50)  9.95(50)
50  50  50  50  50  50
847.50  629.50  537.50  507.50  562.50  497.50

50  50  50  50  50  50
3582
 300

 11.94

The weighted average is $11.94.

244r
244

16.95  12.59  10.75  10.15  11.25  9.95


6

23  r
The percent of increase is about 23%.

71.64
6

 11.94
The average of the per-pound coffee prices is $11.94.
The average of the prices per pound is the same
as the weighted average if the same number of
pounds of each type are sold. This is because each
price is multiplied by the same weight, and then
that weight is divided out.

45. The probability that the event will occur is 3, so


1
the probability that it will not occur is 3.
2 1
odds of event occurring  3:3 or 2:1
The odds that the event will occur are 2:1.

Chapter 3

1044.40  1343.45  1466.92  793.07  854.22  492.75


56  97  124  71  69  45

 12.9757792

 100

The weighted average increased by 10% to about


$12.98.

56(100)  244(r)
5600  244r
5600
244

1005.20  1318.23  1457  791.65  845.25  492.75


56  97  124  71  69  45
5910.08
462

 12.7923809
The weighted average increases by $1.00 to about
$12.79.
3. Multiply each price by 1.1.

 100

17.95(56)  13.59(97)  11.75(124)  11.15(71)  12.25(69)  10.95(45)


56  97  124  71  69  45

7(100)  35(r)
700  35r
700
35

Spreadsheet Investigation
(Follow-Up of Lesson 39)

114

Chapter 3 Study Guide and Review


Page 179
1.
2.
3.
4.
5.
6.
7.
8.
9.
10.

23.

Addition
ratio
different
explore
identity
3x  2
increase
Dimensional analysis
weighted average
8 and 9

25.

27.

29.

4


z
3z
The equation is 4  3z  z  2.
13. The sum of the square
of a and the cube of b is 16.
1444442444443 { {
a2  b3
 16

12r 2

12
4

4
4

4v
4

1v
c
4

30.
c
4

 8  8  42  8

 51

1 2  3(51)

 8  42

4

c
4
c
4

 34

1 2  4(34)
c  136

7

2  7(7)

4d  5  49
4d  5  5  49  5
4d  44

32.


44
4

7n  (1)
8
7n  (1)
8

8

2  8(8)

7n  (1)  64
7n  (1)  1  64  1
7n  65

Sixteen minus the product of 9 and a number r is equal to r.

7n
7

n
33.

65
7
65
7

or 97

n  2  4  2n
n  2  2n  4  2n  2n
3n  2  4
3n  2  2  4  2
3n  6
3n
3

3

n2
34. 3t  2(t  3)  t
3t  2t  6  t
t6t
t6ttt
6  0
Since 6  0 is a false statement, this equation
has no solution.

r  2  4
3

or 833

10  r
6  4v  2
6  2  4v  2
4  4v

y  153

31.

250
3

d  11

 6  6  45  6

4d  5
7
4d  5
7

y
r

28.

 6  45
y
3
y
3

2(5)  2

a  10
4p  7  5
4p  7  7  5  7
4p  12

y
3

52

26.

1 2  25(25)

Exercises 1538 For checks, see students work.


15.
r  21  37
r  21  21  37  21
r  16
16.
14  c  5
14  c  14  5  14
c  19
17.
27  6  p
27  6  6  p  6
21  p
18.
b  (14)  6
b  (14)  14  6  14
b  20
19.
d  (1.2)  7.3
d  1.2  7.3
d  1.2  1.2  7.3  1.2
d  8.5

1 12

 25

4d
4

The equation is a2  b3  16.


14. 16

9r

1 12

5y  50

3 5y  3(50)

p3

11. Three times


a number n decreased by 21 is 57.
1442443 1442443 { { {

21  57
3n
The equation is 3n  21  57.
12.
three is equal
Four
minus
times
to
z decreased by 2.
123 14243 14243z 14243
{ 144424443 {

20.

5
a
2
2 5
a
5 2

Lesson-by-Lesson Review

24.

1 2  43(30)

4p
4

y
3

 30

n  40

Vocabulary and Concept Check

Pages 179184

3
n
4
4 3
n
3 4

r  2  2  4  2
21. 6x  42
6x
6

42
6

x  7

r  4
22. 7w  49
7w
7

49
7

w7

115

Chapter 3

3  6y  2  6y

35.

3y2
3y323
y  1

6s
6

1

 1

x  2
6
x  2
6

2

2  612x 2

x  2  3x
x  2  x  3x  x
2  2x
2
2

8
40

2x
2

38.

800
40

b  3
8.3h  2.2  6.1h  8.8
8.3h  2.2  6.1h  6.1h  8.8  6.1h
2.2h  2.2  8.8
2.2h  2.2  2.2  8.8  2.2
2.2h  6.6


6
15

 45

6(45)  15(n)
270  15n
270
15

3800
50

55x
55

18  n
41.

12
d

42.

20d
20

9d
2
3

43.

14
20

385
55

21
m

14(m)  20(21)
14m  420
14m
14

2.1
35

m  30

210
35

b  5
9

50r
50

 100

35r
35

6r
The percent of increase is 6%.
48. The tax is 6.25% of the price of the book.
6.25% of $14.95  0.0625  14.95
 0.934375
Round $0.934375 to $0.93 since tax is always
rounded to the nearest cent. Add this amount to
the original price.
$14.95 + $0.93 = $15.88
The total price of the book is $15.88.

3b
3

1b

Chapter 3

 100

2.1(100)  35(r)
210  35r

420
14

2(9)  3(b  5)
18  3b  15
18  15  3b  15  15
3  3b
3
3

40r
40

76  r
The percent of increase is 76%.
47. Find the amount of change. Since the new
amount is greater than the original, the percent
of change is a percent of increase.
37.1  35  2.1
Find the percent using the original number, 35, as
the base.

35

 55

x7

20

 15

12(15)  d(20)
180  20d
180
20

x
11

x (55)  11(35)
55x  385

15n
15

38(100)  50(r)
3800  50r

6.6
2.2

40.

 100

38
50

h  3
39.

96
6

20  r
The percent of decrease is 20%.
46. Find the amount of change. Since the new
amount is greater than the original, the percent
of change is a percent of increase.
88  50  38
Find the percent using the original number, 50, as
the base.

 1

2.2h
2.2

8(100)  40(r)
800  40r

1  x
37.
2(b  3)  3(b  1)
2b  6  3b  3
2b  6  3b  3b  3  3b
b  6  3
b  6  6  3  6
b  3
b
1

9
 4

s  16
45. The percent of change is a percent of decrease
because the new amount is less than the original.
Find the change.
40  32  8
Find the percent using the original number, 40, as
the base.

y1
36.

s

6(s  4)  8(9)
6s  24  72
6s  24  24  72  24
6s  96

3  6y  6y  2  6y  6y

y
1

6
8

44.

116

55. Let r  the speed of the slower airplane.

49. The discount is 20% of the original price.


20% of $12.99  0.20  12.99
 2.598
Subtract $2.60 from the original price.
$12.99  $2.60  $10.39
The discounted price of the T-shirt is $10.39.
50. 5x  y
5x
5

Slower Airplane
Faster Airplane

r132  1r  8023  2940


3r  3r  240  2940
6r  240  2940
6r  240  240  2940  240
6r  2700

5
y

x5

6r
6

The value of x is 5.
51.
ay  b  c
ay  b  b  c  b
ay  c  b


1.
2.
3.
4.

number
every
greater than
The sum of twice x
and three times y 14243
is equal to 1
thirteen.
144424443
4243
2x  3y

13
The equation is 2x  3y  13.
5. Two
thirds of a number is negative eight-fifths.
14243
{ 14243 { 144
4424
4443
2
8

n


3
5

(c  y)x
c  y

x
x
a

2y  a
3

a  3b
4

The value

15  k  8
15  k  15  8  15
k  23
15  k  8
Check:
?
15  23  8
88
The solution is 23.
7. 1.2x  7.2
6.

7a  9b
8
7a  9b
y
8
7a  9b
of y is
.
8

54. Let x  the number of pounds of the $7.28 coffee


to be added.
Units
(lb)
$8.40 Coffee
9
$7.28 Coffee
x
$7.95 Coffee 9  x

Price per
Unit (lb)
$8.40
$7.28
$7.95

1.2x
1.2

7.2

 1.2

x  6
1.2x  7.2
Check:
?
1.2(6)  7.2
7.2  7.2
The solution is 6.
8.
k  16  21
k  16  16  21  16
k  5
k  16  21
Check:
?
5  16  21
21  21
The solution is 5.

Total
Price
8.40(9)
7.28x
7.95(9  x)

8.40(9)  7.28x  7.95(9  x)


75.60  7.28x  71.55  7.95x
75.60  7.28x  7.95x  71.55  7.95x  7.95x
75.60  0.67x  71.55
75.60  0.67x  75.60  71.55  75.60
0.67x  4.05
0.67x
0.67

The equation is 3 n  5.

4(2y  a)  3(a  3b)


8y  4a  3a  9b
8y  4a  4a  3a  9b  4a
8y  7a  9b
8y
8

2700
6

Page 185

The value of x is y  c. Since division by 0 is


undefined, y  c  0 or y  c.
53.

d  rt
r(3)
1r  8023

Chapter 3 Practice Test

The
Since division by 0 is
undefined, a  0.
52.
yx  a  cx
yx  a  yx  cx  yx
a  cx  yx
a  (c  y)x
a
c  y
a
c  y
a
y  c

t
3
3

r  450
r  80  450  80 or 530
The speed of the slower plane was 450 mph, and
the speed of the faster plane was 530 mph.

c  b
a
c  b
y a
c  b
value of y is a .
ay
a

r
r
r  80

4.05

 0.67

x  6.04477612
About 6 lb of $7.28 coffee should be mixed with
9 lb of $8.40 coffee.

117

Chapter 3

9.
4

t  7
4
t  7
4

14. 5a  125

 11

2  4(11)

5a
5

t  7

4
51  7 ?

4
44 ?

4

5a  125
?
5(25)  125
125  125
The solution is 25.

11
11
11

r
5

2r
5
2r
5

3

3

2r
5

 16
 16 

5  3  3  16  3

y  36

5 5  5(19)

1 2  43(27)

5  19

3
y
4

1 r2

r  95

 27

?
3
(36) 
4

12  7 
12  7  7 
19 

3(19)  3
57  y
Check:

y
3
y
3

0.1r
0.1

1 2
?

0.1r  19
?

0.1(190)  19

y
3
57
3

19  19
The solution is 190.

12  7  19

17.

12  12
The solution is 57.
12.
t  (3.4)  5.3
t  (3.4)  (3.4)  5.3  (3.4)
t  8.7
t  (3.4)  5.3
Check:
?
8.7  (3.4)  5.3
5.3  5.3
The solution is 8.7.
13.
3(x  5)  8x  18
3x  15  8x  18
3x  15  8x  8x  18  8x
11x  15  18
11x  15  15  18  15
11x  33


19

 0.1

r  190
Check:

12  7 

3

3z  9
2

1 42

2 3z  2 9
2

z3

The solution

12

2 2 ?

3
4
9 
2
is 3.

3

18.

3z  9

Check:

9
4

9

w  11  4.6
w  11  11  4.6  11
w  6.4
w
1

6.4
1

w  6.4
w  11  4.6
?
6.4  11  4.6
4.6  4.6
The solution is 6.4.
19.
2p  1  5p  11
2p  1  5p  5p  11  5p
3p  1  11
3p  1  1  11  1
3p  12
Check:

33
11

x  3
3(x  5)  8x  18
Check:
?
3(3  5)  8(3)  18
?
3(2)  24  18
6  6
The solution is 3.

3p
3

12
3

p4
Chapter 3

16

19  3  38  16
22  22
The solution is 95.
16. 0.1r  19

7

12  7 

2r
 16
5
? 2(95)
 5 

3
?

y
3

11x
11

r
5
95
5

Check:

27

27  27
The solution is 36.

y
3

2r
5

5  3  16

 27

3
y
4
4 3
y
3 4

Check:

11.

r
5

15.

11  11
The solution is 51.
10.

125
5

a  25
Check:

t  7  44
t  7  7  44  7
t  51
Check:

118

2p  1  5p  11
?
2(4)  1  5(4)  11
?
8  1  20  11
99
The solution is 4.
20.
25  7w  46
25  7w  25  46  25
7w  21
Check:

7w
7

26.

h  0.25vt2
t
h  0.25vt2
t

21

 7

 11

22.

36(11)  t(9)
396  9t
396
9

9t
9

52n
52

x

2.00x  2.50(30  x)  178.50


2x  75  2.5x  178.50
4.5x  75  178.50
4.5x  75  75  178.50  75
4.5x  103.50

10
 1

5(x  1)  12(10)
5x  5  120
5x  5  5  120  5
5x  125
5x
5

4.5x
4.5

125
5

r
100

36(100)  45(r)
3600  45r
3600
45

t
t

The River Rover

10

t2

10 t  2

8t  10t  5
8t  10t  10t  5  10t
2t  5
2t
2

5

 2
5

t  2 or 2.5
The River Rover overtakes The Yankee Clipper
2.5 hours after 9:00 A.M. or at 11:30 A.M.
30. B;

 100

d  rt
8t

r
8

8(100)  12(r)
800  12r
800
12

103.50
4.5

The Yankee Clipper

8t  10 t  2

45r
45

80  r
The percent of decrease is 80%.
25. Find the amount of change. Since the new
amount is greater than the original, the percent
of change is a percent of increase.
20  12  8
Find the percent using the original number, 12, as
the base.
8
12

x  23
30  x  30  23 or 53
23 cups of espresso were sold, and 53 cups of
cappuccino were sold.
29. Let t  the number of hours the Yankee Clipper
traveled.

x  25
24. The percent of change is a percent of decrease
because the new amount is less than the original.
Find the change.
45  9  36
Find the percent using the original number, 45, as
the base.
36
45

Cups Price per Cup Total Price


Espresso
x
$2.00
2.00x
Cappuccino 30  x
$2.50
2.50(30  x)

13

 52

n  4 or 0.25

5
12

Since division by 0 is

The
Since division by 0 is
undefined, a  0.
28. Let x  the number of cups of espresso sold.

3.25
52

n(52)  4(3.25)
52n  13

44  t
23.

a

b  a
a
b  a
y a
b  a
value of y is a .
ay
a

n
4

at
t

h  0.25vt2
.
t

w  3
25  7w  46
?
25  7(3)  46
?
25  (21)  46
46  46
The solution is 3.
36
t

The value of a is
undefined, t  0.
27.
a(y  1)  b
ay  a  b
ay  a  a  b  a
ay  b  a

Check:

21.

h  at  0.25vt2
h  0.25vt2  at  0.25vt2  0.25vt2
h  0.25vt2  at

4
5

20

4  54
12
20
12
20

2x

 20

1 2  201202x 2
12  2x

12r
12

12
2

67  r
The percent of increase is about 67%.

2x
2

6x

119

Chapter 3

Chapter 3 Standardized Test Practice

10. F  5C  32
9

 5(5)  32
 9  32
 23
The temperature is 23F.
11. There are five pairs of black socks, and there are
3 pairs of socks that are not black.
5
odds of black  3

Pages 186187
1. C; P  2/  2w
 2(15)  2(6)
 30  12
 42
2. C; 65 percent is less than 100 percent, and 100
percent equals 1. Therefore, 65 percent of 20 is
less than 1 times 20 or 20.
3. B; From the bar graph we see that the plant is
about 4 cm tall at the end of the fifth week and
growing about 1 cm a week. Therefore, at the end
of the sixth week the plants height should be
about 4  1 or 5 cm.
4. D; Write each probability as a decimal.
25%  0.25
1 in 4  0.25
1
5

The odds of choosing a black pair are 5:3.


12. Let n  age of the youngest sister.
Then n  1  age of the next oldest sister, and
n  2  age of the oldest sister.
The
sum of the ages of the three sisters {
is 39.
1444444442444444443
{
n  (n  1)  (n  2)
n  (n  1)  (n  2)  39
3n  3  39
3n  3  3  39  3
3n  36

 0.20

3n
3

0.3  0.30
0.20 6 0.25 6 0.30
Therefore, 0.3 is the greatest probability.
5. A; Profit  Revenue minus costs.
p  24.95n  (0.8n  575)
p  24.95n  0.8n  575
p  (24.95  0.8)n  575
6. D; 8(x  2)  12
8x  16  12
1
(8x
4

Then Tyson drove

3
x
2

x
2

36  x
1

8. D; The sum of x and y is


144424443 {
1
xy

x
x

1
y

0
0

1
y

x(xy  5)
4







Chapter 3

 500
 435

 65 

290
2

 65

 145  65
 210
Tyson drove 210 miles.
14. 7(x  2)  4(2x  3)  47
7x  14  8x  12  47
15x  2  47
15x  2  2  47  2
15x  45

0.

x  y
9.

 500

x  290

6x
6

1
y
1
y

 65 miles.

1 2  23(435)

8(27)  6(x)
216  6x
216
6

36
3

 65  65  500  65
3
x
2
2 3
x
3 2

x
27

x
2

2  500

x
x  2  65
x
x  2  65
3
x  65
2

 16)  4(12)

39

n  12
n  1  12  1 or 13
n  2  12  2  14
The middle sister is 13 years old.
13. Let x  the number of miles Pete drove.

2x  4  3
7. C; Let x  the amount needed for 27 servings.
8
6

15x
15

2[2(3)  5]
4
2[6  5]
4
2[1]
4
2
4
1
2 or 0.5

45

 15

x3
The solution is 3.
15. The discount is 45% of the original price.
45% of $22.95  0.45  22.95
 10.3275
$22.95  $10.33  $12.62
The discounted price of the book is $12.62.
16. C; a and a are the same number of units from
zero, but in opposite directions. Therefore,
0a 0  0a 0 .

120

17. B; 3x  7  10
3x  7  7  10  7
3x  3
3x
3

Kirbys pickup travels at 36 mph.


15 mi
25 min

15(60)  25(x)
900  25x

3

x1
4y  2  6
4y  2  2  6  2
4y  8
4y
4

900
25

4

 100

75r
75

0.20(200  x)

r

0.80x

0.50(200)

The equation is 0.20(200  x)  0.80x  0.50(200).


20c.
0.20(200  x)  0.80x  0.50(200)
40  0.20x  0.80x  100
40  0.60x  100
40  0.60x  40  100  40
0.60x  60

 100

0.60x
0.60

50(100)  150(r)
5000  150r


Liters of
Acid
0.20(200  x)
0.80x
0.50(200)

20b.

5000
150
1
33 3

Liters of
Solution
200  x
x
200

The amount of acid


the amount of acid
the amount of acid
in
the 20% solution 1
plus
in the 80% solution 123
equals 144424443
in the 50% solution
144424443
23 144424443

The rate of increase from 75 to 100 is 33 3 %.


Find the amount of change.
200  150  50
Find the percent using the original number, 150,
as the base.
50
150

25x
25

20% Solution
80% Solution
50% Solution

25(100)  75(r)
2500  75r
2500
75
1
33 3

36  x
Nolas SUV travels at 36 mph. Both are traveling
at 36 mph, and therefore, neither is exceeding the
speed limit.
20a.

y2
18. C; Find the amount of change.
100  75  25
Find the percent using the original number, 75, as
the base.
25
75

x mi

 60 min

60

 0.60

x  100
200  x  200  100 or 100
100 L of the 20% solution and 100 L of the 80%
solution are needed.

150r
150

r
1

The rate of increase from 150 to 200 is 33 3 %.


19. Calculate the miles per hour rate for each
vehicle.
6 mi
10 min

x mi

 60 min

6(60)  10(x)
360  10x
360
10

10x
10

36  x

121

Chapter 3

PQ249-6481F-04[122-168] 31/7/02 10:59 PM Page 122 Sahuja Ahuja_QXP_06:Desktop Folder:

Chapter 4

Graphing Relations and Functions

Page 191

16. 2c  b  2(3)  (4)


 6  4
 10
17. c  3a  (3)  3(1)
 3  (3)
 3  3
0
18. 3a  6b  2c  3(1)  6(4)  2(3)
 3  24  (6)
 27  6
 21

Getting Started

1.
1 0

5 4 3 2 1

8 7 6 5 4 3 2 1

2.
3.
4.
2 1 2 11 1  1 0
2

1
2

1 11 2 21
2

5. 3(7  t)  3  7  3  t
 21  3t
6. 4(w  2)  (4)(w)  14)(2)
 4w  (8)
 4w  8
7. 5(3b  2)  (5)(3b)  (5)(2)
 15b  (10)
 15b  10
8.

1
(2z
2

1
2

19. 8a  2b  3c  8(1)  2 (4)  3(3)


 8  2  (9)
 6  9
3
2

20. 6a  8b  3c  6(1)  8(4)  3 (3)


 6  32  (2)
 26  2
 24

1
2

 4)   2z   4

z2
2x  y  1
2x  y  2x  1  2x
y  1  2x
10.
x8y
xy8yy
xy8
xyx8x
y8x
6x  3y  12
11.
6x  3y  6x  12  6x
3y  12  6x
9.

Page 194

Check for Understanding

1. Draw two perpendicular number lines. Label the


horizontal line x since this is the x-axis, and label
the vertical line y since this is the y-axis. Label
the point where the two axes meet 0 since this
is the origin. The axes divide the coordinate plane
into four regions: the upper right region is
quadrant I, the upper left region is quadrant II,
the lower left region is quadrant III, and the
lower right region is quadrant IV.

13 (3y)  13 (12  6x)


y  4  2x or 2x  4
2x  3y  9
12.
2x  3y  2x  9  2x
3y  9  2x
1
(3y)
3

The Coordinate Plane

4-1

II

y  3  23x or 23x  3
9

13.
9

1
y
2

1
y
2

 4x

III

IV

 9  4x  9
1

2 y  4x  9
1

2. To graph (1, 4), move 1 unit left from the origin


and 4 units up. This point is in quadrant II. To
graph (4, 1), move 4 units right from the origin
and 1 unit down. This point is in quadrant IV.
3. Sample answer: (3, 3) is in quadrant I since both
coordinates are positive; (3, 3) is in quadrant II
since the x-coordinate is negative and the
y-coordinate is positive; (3, 3) is in quadrant
III since both coordinates are negative; (3, 3) is
in quadrant IV since the x-coordinate is positive
and the y-coordinate is negative.

y  18  8x

y  5
3
y  5
3

x2

2  3(x  2)

y  5  3x  6
y  5  5  3x  6  5
y  3x  1
15. a  b  c  (1)  (4)  (3)
 1  4  3
33
6

Chapter 4

2 2 y  2(4x  9)
14.

 13 (9  2x)

122

PQ249-6481F-04[122-168] 31/7/02 10:59 PM Page 123 Sahuja Ahuja_QXP_06:Desktop Folder:

11. M(2, 2)


Start at the origin.
Move left 2 units since the x-coordinate is 2.
Move down 2 units since the y-coordinate is 2.
Draw a dot and label it M.
811.
y
J
K

4. Follow along a vertical line through point E to


find the x-coordinate on the x-axis. The
x-coordinate is 2.
Follow along a horizontal line through point E
to find the y-coordinate on the y-axis. The
y-coordinate is 5.
So, the ordered pair for point E is (2, 5).
Since both coordinates are negative, point E is
located in quadrant III.
5. Follow along a vertical line through point F to
find the x-coordinate on the x-axis. The
x-coordinate is 1.
Follow along a horizontal line through point F
to find the y-coordinate on the y-axis. The
y-coordinate is 1.
So, the ordered pair for point F is (1, 1).
Since the x-coordinate is negative and the
y-coordinate is positive, point F is located in
quadrant II.
6. Follow along a vertical line through point G to
find the x-coordinate on the x-axis. The
x-coordinate is 4.
Follow along a horizontal line through point G
to find the y-coordinate on the y-axis. The
y-coordinate is 4.
So, the ordered pair for point G is (4, 4).
Since both coordinates are positive, point G is
located in quadrant I.
7. Follow along a vertical line through point H to
find the x-coordinate on the x-axis. The
x-coordinate is 4.
Follow along a horizontal line through point H
to find the y-coordinate on the y-axis. The
y-coordinate is 2.
So, the ordered pair for point H is (4, 2).
Since both coordinates are negative, point H is
located in quadrant III.
8. J(2, 5)
Start at the origin.
Move right 2 units since the x-coordinate is 2.
Move up 5 units since the y-coordinate is 5.
Draw a dot and label it J.
(See coordinate plane after Exercise 11.)
9. K(1, 4)
Start at the origin.
Move left 1 unit since the x-coordinate is 1.
Move up 4 units since the y-coordinate is 4.
Draw a dot and label it K.
(See coordinate plane after Exercise 11.)
10. L(0, 3)
Start at the origin.
Since the x-coordinate is 0, the point will be
located on the y-axis.
Move down 3 units since the y-coordinate is 3.
Draw a dot and label it L.
(See coordinate plane after Exercise 11.)

M
L

12. Sketch the given figure on the coordinate plane


by plotting point A at (40, 10). From point A,
move 40 units right and label point B. From point
B, move up 10 units and label point C. From point
C, move left 20 units and label point D. From
point A, move up 30 units and label point E.
y

C
D
A

40
30
20
10

20 151010 O10 20 30 40 x
10
20
30
40

Point B has coordinates (0, 10), C(0, 20),


D(20, 20), and E(40, 40).

Pages 195196

Practice and Apply

xCoordiPoint
nate
13.
N
4
14.
P
5
15.
Q
1
16.
R
5
17.
S
3
18.
T
2
19.
U
2
20.
V
4
21.
W
0
22.
Z
3

yCoordinate
5
3
3
2
3
0
1
2
4
3

Ordered
Pair
(4, 5)
(5, 3)
(1, 3)
(5, 2)
(3, 3)
(2, 0)
(2, 1)
(4, 2)
(0, 4)
(3, 3)

Quadrant
II
IV
III
I
II
none
IV
III
none
I

23. A point 12 units down from the origin has


y-coordinate 12. A point 7 units to the right of
the origin has x-coordinate 7. So, the ordered pair
is (7, 12).

123

Chapter 4

PQ249-6481F-04[122-168] 31/7/02 10:59 PM Page 124 Sahuja Ahuja_QXP_06:Desktop Folder:

24. A point 9 units to the left of the origin has


x-coordinate 9. A point that lies on the x-axis
has y-coordinate 0. So, the ordered pair is (9, 0).
25. A(3, 5)
Start at the origin.
Move right 3 units and up 5 units.
Draw a dot and label it A.
(See coordinate plane after Exercise 36.)
26. B(2, 2)
Start at the origin.
Move left 2 units and up 2 units.
Draw a dot and label it B.
(See coordinate plane after Exercise 36.)
27. C(4, 2)
Start at the origin.
Move right 4 units and down 2 units.
Draw a dot and label it C.
(See coordinate plane after Exercise 36.)
28. D(0, 1)
Start at the origin.
Since the x-coordinate is 0, the point is on the
y-axis.
Move down 1 unit.
Draw a dot and label it D.
(See coordinate plane after Exercise 36.)
29. E(2, 5)
Start at the origin.
Move left 2 units and up 5 units.
Draw a dot and label it E.
(See coordinate plane after Exercise 36.)
30. F(3, 4)
Start at the origin.
Move left 3 units and down 4 units.
Draw a dot and label it F.
(See coordinate plane after Exercise 36.)
31. G(4, 4)
Start at the origin.
Move right 4 units and up 4 units.
Draw a dot and label it G.
(See coordinate plane after Exercise 36.)
32. H(4, 4)
Start at the origin.
Move left 4 units and up 4 units.
Draw a dot and label it H.
(See coordinate plane after Exercise 36.)
33. I(3, 1)
Start at the origin.
Move right 3 units and up 1 unit.
Draw a dot and label it I.
(See coordinate plane after Exercise 36.)
34. J(1, 3)
Start at the origin.
Move left 1 unit and down 3 units.
Draw a dot and label it J.
(See coordinate plane after Exercise 36.)
Chapter 4

35. K(4, 0)
Start at the origin.
Move left 4 units.
Since the y-coordinate is 0, the point is on the
x-axis.
Draw a dot and label it K.
(See coordinate plane after Exercise 36.)
36. L(2, 4)
Start at the origin.
Move right 2 units and down 4 units.
Draw a dot and label it L.
2536.
y
E
A
H
G

B
I
K
x

D
C
J
F

37. Latitude lines run east and west. Sample answer:


Louisville and Richmond.
38. Longitude lines run north and south. Sample
answer: Austin and Oklahoma City.
39.
xyOrdered
Artifact Coordinate Coordinate
Coins
3
5
Plate
7
2
Goblet
8
4
Vase
5
9

Pair
(3, 5)
(7, 2)
(8, 4)
(5, 9)

40. The Shapiro Undergraduate Library is in column


C and in row 5, so is located in sector C5.
41. The Natural Science, Chemistry, and Natural
Resources and Environment Buildings are in
column C and in row 4, so are located in sector C4.
42. E. Huron St. runs horizontally from sector A2 to
sector D2.
43. The four sectors that have bus stops are B5, C2,
D4, and E1.
44a. xy 7 0 indicates that the product of x and y is
positive. This is true if both x and y are positive,
placing (x, y) in quadrant I; it is also true if both
x and y are negative, placing (x, y) in quadrant III.
44b. xy 7 0 indicates that the product of x and y is
negative. This is true if x is positive and y is
negative, placing (x, y) in quadrant IV; it is also
true if x is negative and y is positive, placing
(x, y) in quadrant II.
44c. xy 7 0 indicates that the product of x and y is 0.
This is true if both x and y are 0, placing (x, y) at
the origin; it is also true if x is 0, placing (x, y)
on the y-axis; it is also true if y is 0, placing (x, y)
on the x-axis.

124

PQ249-6481F-04[122-168] 31/7/02 10:59 PM Page 125 Sahuja Ahuja_QXP_06:Desktop Folder:

The distance traveled by each plane is the same.


Write an equation.
7r  2240

45. Archaeologists use coordinate systems as


mapping guides and as systems to record
locations of artifacts. Answers should include the
following.
The grid gives archaeologists points of
reference so they can identify and explain to
others the location of artifacts in a site they
are excavating. You can divide the space so
more people can work at the same time in
different areas.
Knowing the exact location of artifacts helps
archaeologists reconstruct historical events.
46. C
Vertex B is located 3 units right and 2 units up
from the origin. Since the rectangle is centered at
the origin, vertex A must be located 3 units left
and 2 units up from the origin at (3, 2).

7r
7

47. B
Vertex A is 2 units above the x-axis and vertex D
is 2 units below the x-axis. So, the length of AD is
2  2  4 units.
48. Let (a, b)  (7, 1) and (c, d )  (3, 1).

1a 2 c, b 2 d 2  1 2
 1 42, 22 2

7  132 1  1
, 2

4c
4

21
 1 14
, 10
2
2 2

5  9 2  (8)
,
2
2

3h
3

55.
4

Maintain Your Skills

t

58. You can use a calculator to find an approximation


for 1180.
1180  13.41640786 p
Rounded to the nearest hundredth, 1180 is about
13.42.
59. 1256 represents the negative square root of 256.

8t
So, the first airplane arrived in Baltimore after 8 h.
Since the second airplane left Tucson 45 min after
the first airplane, and is scheduled to land in
Baltimore 15 min before the first airplane, the
second airplane travels 60 min, or 1 hr, less than
the first airplane. So, the second airplane will
travel for 7 h.
Make a table of the information.
t
8
7

11t
11

163  7.937253933 p
Rounded to the nearest hundredth, 163 is about
7.94.

280t
280

r
280
r

56. 181 represents the negative square root of 81.


81  92 S 181  9
57. You can use a calculator to find an approximation
for 163.

51. Since the first airplane traveled 2240 mi at


280 mph, substitute d  2240 and r  280 into
the formula d  rt.
d  rt
2240  280t

First Airplane
Second Airplane

 2t

4  4(2t)

3a
11
3a
11

 (0, 0)

3(a  t)
4
3(a  t)
4

b  6w
3
6w  b
3

3(a  t)  8t
3a  3t  8t
3a  3t  3t  8t  3t
3a  11t

1a 2 c, b 2 d 2  14 2 4, 4 2(4) 2
0 0
 1 2, 2 2

2240
280

h

 (7, 5)
50. Let (a, b)  (4, 4) and (c, d)  (4, 4).

Page 196

4d

 4
cd
54.
6w  3h  b
6w  3h  6w  b  6w
3h  b  6w

 (2, 1)
49. Let (a, b)  (5, 2) and (c, d )  (9, 8).
a  c b  d
, 2
2

2240

 7
r  320
The second plane must travel at 320 mph to
arrive on schedule.
52.
3x  b  2x  5
3x  b  b  2x  5  b
3x  2x  5  b
3x  2x  2x  5  b  2x
x5b
53.
10c  2(2d  3c)
10c  2  2d  2  3c
10c  4d  6c
10c  6c  4d  6c  6c
4c  4d

256  162 S 1256  16


60. 52  018  7 0
61. 081  47 0  17
 52  011 0
 034 0  17
 52  11
 34  17
 63
 51
62. 42  060  74 0
63. 36  015  21 0
 42  014 0
 36  06 0
 42  14
 36  6
 28
 30

d  rt
2240
7r

125

Chapter 4

PQ249-6481F-04[122-168] 31/7/02 10:59 PM Page 126 Sahuja Ahuja_QXP_06:Desktop Folder:

64. 010  16  27 0
65. 038  65  21 0
 06  27 0
 027  21 0
 021 0
 048 0
 21
 48
66. 4(x  y)  4x  4y
67. 1(x  3)  (1)(x)  (1)(3)
 x  3
68. 3(1  6y)  3  1  3  6y
 3  18y
69. 3(2x  5)  (3)(2x)  (3)(5)
 6x  (15)
 6x  15
1

5. To reflect the triangle over the x-axis, multiply


the y-coordinate of each vertex by 1.
(x, y) S (x, y)
P(1, 2) S P(1, 2)
Q(4, 4) S Q(4, 4)
R(2, 3) S R(2, 3)
y

Q
R
P

 3x  2y
1

70. 3(2x  6y)  3(2x)  3(6y)


1

R
1

71. 4(5x  2y)  4(5x)  4(2y)




4-2

5
x
4

1
y
2

6. To translate the quadrilateral 3 units up, add 3 to


the y-coordinate of each vertex.
(x, y) S (x, y  3)
A(4, 2) S A(4, 2  3) S A(4, 5)
B(4, 2) S B(4, 2  3) S B(4, 1)
C(1, 3) S C(1, 3  3) S C(1, 0)
D(3, 2) S D(3, 2  3) S D(3, 5)

Transformations on the
Coordinate Plane

Pages 200201

Check for Understanding

1. Transformation
Reflection
Rotation
Translation
Dilation

Size
same
same
same
changes

Shape
same
same
same
same

Orientation
changes
changes
same
same

A
A

B
x

C O
2. Sample answer: The preimage is a square
centered at the origin with a side 4 units long.
The image after a dilation that is an enlargement
3
having a scale factor of 2 is the square centered
at the origin with a side 6 units long. The image
after a dilation that is a reduction having a scale
1
factor of 2 is the square centered at the origin
with a side 2 units long.

B
C

7. To dilate the parallelogram, multiply the


coordinates of each vertex by 2.
(x, y) S (2x, 2y)
E(1, 4) S E(2  (1), 2  4) S E(2, 8)
F(5, 1) S F(2  5, 2  (1)) S F(10, 2)
G(2, 4) S G(2  2, 2  (4)) S G(4, 8)
H(4, 1) S H(2  (4), 2  1) S H(8, 2)

y
x

E
E
H
H

2
2

3. The figure has been shifted horizontally to the


right. This is a translation.
4. The figure has been turned around a point. This
is a rotation.

Chapter 4

G
G

126

PQ249-6481F-04[122-168] 31/7/02 10:59 PM Page 127 Sahuja Ahuja_QXP_06:Desktop Folder:

17b.

8. To rotate the triangle 90 counterclockwise about


the origin, switch the coordinates of each vertex
and then multiply the new first coordinate by 1.
(x, y) S (y, x)
J(0, 0) S J(0, 0)
K(2, 5) S K(5, 2)
L(4, 5) S L(5, 4)

T
R

J
J

18a. To reflect the trapezoid over the x-axis, multiply


the y-coordinate of each vertex by 1.
(x, y) S (x, y)
A(2, 3) S A(2, 3)
B(5, 3) S B(5, 3)
C(6, 1) S C(6, 1)
D(2, 1) S D(2, 1)
18b.
y

K
L
K
9. Draw a dot and label it A. Move 10 units left
(10 mi west) and 7 units down (7 mi south). Draw
a dot and label it B.

10 mi

D
B

7 mi

19a. To translate the quadrilateral 8 units right, add


8 to the x-coordinate of each vertex.
(x, y) S (x  8, y)
R(6, 3) S R(6  8, 3) S R(2, 3)
S(4, 2) S S(4  8, 2) S S(4, 2)
T(1, 5) S T(1  8, 5) S T(7, 5)
U(3, 7) S U(3  8, 7) S U(5, 7)
19b.
U
U

B
10. To translate the point A(x, y) 10 units left, add
10 to the x-coordinate of A. To translate A(x, y)
7 units down, add 7 to the y-coordinate of A.
(x, y) S (x  10, y  7)
The ships current location is represented by
(x  10, y  7).

Pages 201203

Practice and Apply

R
S

11. The figure has been shifted horizontally to the


right. This is a translation.
12. The figure has been turned around a point. This
is a rotation.
13. The figure has been flipped over a line. This is a
reflection.
14. The figure has been increased in size. This is a
dilation.
15. The figure has been flipped over a line. This is a
reflection.
16. The figure has been shifted vertically down. This
is a translation.
17a. To reflect the triangle over the y-axis, multiply
the x-coordinate of each vertex by 1.
(x, y) S (x, y)
R(2, 0) S R(2, 0)
S(2, 3) S S(2, 3)
T(2, 3) S T(2, 3)

S
O

20a. To translate the parallelogram 3 units right, add


3 to the x-coordinate of each vertex. To translate
the parallelogram 2 units down, add 2 to the
y-coordinate of each vertex.
(x, y) S (x  3, y  2)
M(6, 0) S M(6  3, 0  2) S M(3, 2)
N(4, 3) S N(4  3, 3  2) S N(1, 1)
O(1, 3) S O(1  3, 3  2) S O(2, 1)
P(3, 0) S P(3  3, 0  2) S P(0, 2)

127

Chapter 4

PQ249-6481F-04[122-168] 31/7/02 10:59 PM Page 128 Sahuja Ahuja_QXP_06:Desktop Folder:

20b.

23b.

H
F

x
x

F
H
G

21a. To dilate the trapezoid by a scale factor


1
1
of 2, multiply the coordinates of each vertex by 2.

11

(x, y) S 2x, 2y

24a. To rotate the quadrilateral 90 counterclockwise


about the origin, switch the coordinates of each
vertex and then multiply the new first
coordinate by 1.
(x, y) S (y, x)
T(4, 2) S T(2, 4)
U(2, 4) S U(4, 2)
V(0, 2) S V(2, 0)
W(2, 4) S W(4, 2)
24b.
y
U

1
1
2
1
1
K(2, 4) S K 1 2  (2), 2  4 2 S K(1, 2)
1
1
L(4, 4) S L 1 2  4, 2  4 2 S L(2, 2)
1
1
M(4, 4) S M 1 2  (4), 2  (4) 2 S M(2, 2)
1
2

J(4, 2) S J  (4), 2  2 S J(2, 1)

21b.

K
J

W T

M
25a. To reflect the parallelogram over the y-axis,
multiply the x-coordinate of each point by 1.
To then rotate the result 180 about the origin,
multiply both coordinates of the reflected point
by 1.
(x, y) S (x, y) S (x, y)
(x, y) S (x, y)
W(1, 2) S W(1, 2)
X(3, 2) S X(3, 2)
Y(0, 4) S Y(0, 4)
Z(4, 4) S Z(4, 4)
25b.
y

22a. To dilate the square by a scale factor of 3,


multiply the coordinates of each vertex by 3.
(x, y) S (3x, 3y)
A(2, 1) S A(3  (2), 3  1) S A(6, 3)
B(2, 2) S B(3  2, 3  2) S B(6, 6)
C(3, 2) S C(3  3, 3  (2)) S C(9, 6)
D(1, 3) S D(3  (1), 3  (3)) S D(3, 9)
22b.
8

A
8

4

4

8

8x

X
x

23a. To rotate the triangle 180 about the origin,


multiply both coordinates of each vertex by 1.
(x, y) S (x, y)
F(3, 2) S F(3, 2)
G(2, 5) S G(2, 5)
H(6, 3) S H(6, 3)

Chapter 4

128

PQ249-6481F-04[122-168] 31/7/02 10:59 PM Page 129 Sahuja Ahuja_QXP_06:Desktop Folder:

31. The x-coordinate of each vertex of the dilated


1
triangle is 2 of the x-coordinate of the
corresponding preimage vertex. The y-coordinate
1
of each vertex of the dilated triangle is 2 of the
y-coordinate of the corresponding preimage
vertex. So, triangle QRS was dilated by a scale
1
factor of 2.
32. The x-coordinate of each vertex of the image is 1
times the x-coordinate of the corresponding
preimage vertex. The y-coordinate of each vertex
of the image is equal to the y-coordinate of the
corresponding preimage vertex. So, parallelogram
WXYZ was reflected over the y-axis.
33. To x-coordinate of each vertex of the image is 1
times the y-coordinate of the corresponding
preimage vertex. The y-coordinate of each vertex
of the image is equal to the x-coordinate of the
corresponding preimage vertex. That is, for each
ordered pair, the coordinates were switched and
the new first coordinate was multiplied by 1. So,
triangle XYZ was rotated 90 counterclockwise
about the origin.

26a. To reflect the pentagon over the x-axis, multiply


the y-coordinate of each vertex by 1. To then
translated the result as described, add 2 to the
x-coordinate and 1 to the y-coordinate of each
reflected vertex.
(x, y) S (x, y) S (x  2, y  1)
(x, y) S (x  2, y  1)
P(0, 5) S P(0  2, 5  1) S P(2, 4)
Q(3, 4) S Q(3  2, 4  1) S Q(1, 3)
R(2, 1) S R(2  2, 1  1) S R(0, 0)
S(2, 1) S S(2  2, 1  1) S S(4, 0)
T(3, 4) S T(3  2, 4  1) S T(5, 3)
26b.
y P
T
Q

Q
P

34. Multiply each dimension by 22.


1

1800  22  4500

27. A(5, 1), B(3, 3), C(5, 5), D(5, 4),


E(8, 4), F(8, 2), and G(5, 2) are the
vertices of the arrow.
28. To translate the arrow 2 units right, add 2 to the
x-coordinate of each vertex. To reflect the
translated arrow across the x-axis, multiply the
y-coordinate of each vertex by 1.
(x, y) S (x  2, y)
A(5, 1) S A(3, 1)
B(3, 3) S B(1, 3)
C(5, 5) S C(3, 5)
D(5, 4) S D(3, 4)
E(8, 4) S E(6, 4)
F(8, 2) S F(6, 2)
G(5, 2) S G(3, 2)
29.
y
C

1600  22  4000
The new digital photograph will be 4500 pixels
wide by 4000 pixels high.
3536.
4800
(0, 4000)

3200
2400
(0, 1600)

800
(0, 0)
0

B
A

G
B

(1800, 0)

(4500, 0)

800 1600 2400 3200 4000 4800

35. The other three vertices of the 1800  1600


digital photograph are (0, 1600), (1800, 1600), and
(1800, 0).
36. The vertices of the enlarged 4500  4000
photograph are (0, 0), (0, 4000), (4500, 4000), and
(4500, 0).
37.

G
A

(1800, 1600)

1600

(4500, 4000)

4000

D
C

30. The x-coordinate of each vertex of the translated


trapezoid is 3 more than the x-coordinate of the
corresponding preimage vertex. The y-coordinate
of each vertex of the translated trapezoid is 2 less
than the y-coordinate of the corresponding
preimage vertex. So, trapezoid JKLM was
translated 3 units right and 2 units down.
Sample answer: The pattern resembles a
snowflake.

129

Chapter 4

PQ249-6481F-04[122-168] 31/7/02 10:59 PM Page 130 Sahuja Ahuja_QXP_06:Desktop Folder:

45.

38. Yes, the same pattern could be drawn using


translations since all octagons in the figure have
the same size, shape, and orientation.
39. To rotate a point 90 clockwise about the origin,
we switch the coordinates of the point and
multiply the new second coordinate by 1.
(x, y) S ( y, x)
40. To reflect the point A(x, y) over the x-axis, we
multiply the y-coordinate by 1. Then, to reflect
this image over the y-axis, we multiply the
x-coordinate by 1. So, the final image is
A(x, y).
To rotate a point 180 about the origin, we
multiply both coordinates by 1. So,
A(x, y) S A(x, y).
Since the final image in each case is the same,
the statement is always true.
41. Artists use computer graphics to simulate
movement, change the size of objects, and create
designs.
Objects can appear to move by using a series
of translations. Moving forward can be
simulated by enlarging objects using dilations
so they appear to be getting closer.
Computer graphics are used in special effects
in movies, animated cartoons, and web design.
42. C; For the translations described, add 1 to each
x-coordinate and add 3 to each y-coordinate.
S(4, 2) S S(4  1, 2  3) S S(5, 1)
43. C
2

y3

46.

R
x
yx

The coordinates of the vertices of the image are


R(3, 3), S(0, 4), and T(4, 1).

Page 203

84  z
y

A
B
D x
C

x0

The coordinates of the vertices of the image are


A(3, 4), B(2, 2), C(3, 2), and D(4, 0).

Chapter 4

Maintain Your Skills

47. A(2, 1)


Start at the origin.
Move right 2 units and down 1 unit.
Draw a dot and label it A.
(See coordinate plane after Exercise 52.)
48. B(4, 0)
Start at the origin.
Move left 4 units.
Since the y-coordinate is 0, the point is on the
x-axis.
Draw a dot and label it B.
(See coordinate plane after Exercise 52.)
49. C(1, 5)
Start at the origin.
Move right 1 unit and up 5 units.
Draw a dot and label it C.
(See coordinate plane after Exercise 52.)
50. D(1, 1)
Start at the origin.
Move left 1 unit and down 1 unit.
Draw a dot and label it D.
(See coordinate plane after Exercise 52.)
51. E(2, 3)
Start at the origin.
Move left 2 units and up 3 units.
Draw a dot and label it E.
(See coordinate plane after Exercise 52.)

4  21  4  4z

If y  4z and y  21, then 21  4z.

The coordinates of the vertices of the image are


J(3, 5), K(2, 8), L(1, 8), and M(3, 5).

21  y

44.

 14  2  3 y
1

If x  3y and x  14, then 14  3y.


3
2

130

PQ249-6481F-04[122-168] 31/7/02 11:00 PM Page 131 Sahuja Ahuja_QXP_06:Desktop Folder:

52. F(4, 3)


Start at the origin.
Move right 4 units and down 3 units.
Draw a dot and label it F.
4752.
y
C

Page 204

Graphing Calculator Investigation


(Preview of Lesson 4-3)

1. {(10, 10), (0, 6), (4, 7), (5, 2)}


Step 1 Enter the data.
Enter the x-coordinates in L1 and the
y-coordinates in L2.

STAT ENTER 10 ENTER


0 ENTER 4 ENTER 5 ENTER
10
ENTER 6 ENTER 7 ENTER 2 ENTER

KEYSTROKES:

B
D

Step 2 Format the graph.


Turn on the statistical plot.

A
F

KEYSTROKES:

30% Solution
15% Solution
25% Solution

Amount of
Nitric Acid
0.30(20)
0.15 x
0.25(20  x)

ENTER

ENTER ENTER

2nd

L2

ENTER

Step 3 Choose the viewing window.


Be sure you can see all of the points.
[10, 15] scl : 1 by [10, 15] scl: 1
WINDOW 10 ENTER 15
ENTER 1 ENTER 10 ENTER 15 ENTER 1

KEYSTROKES:

0.30(20)  0.15x  0.25(20  x)


6  0.15x  5  0.25x
6  0.15x  0.15x  5  0.25x  0.15x
6  5  0.10x
6  5  5  0.10x  5
1  0.10x
1
0.10

STAT
PLOT

Select the scatter plot, L1 as the Xlist and L2


as the Ylist.
ENTER
2nd
L1
KEYSTROKES:

53. Let x  the amount of 15% solution to be added.


Amount of
Solution (mL)
20
x
20  x

2nd

Step 4 Graph the relation.


Display the graph.
KEYSTROKES:

GRAPH

0.10x
0.10

10  x
Jamaal should add 10 mL of the 15% solution to
the 20 mL of the 30% solution.
54. There are 26 favorable outcomes of the 36 total
possible outcomes.
26

[10, 15] scl: 1 by [10, 15] scl: 1

13

P (sum is less than 9)  36  18 or about 72%

2. {(4, 1), (3, 5), (4, 5), (5, 1)}


Step 1 Enter the data.
Enter the x-coordinates in L1 and the
y-coordinates in L2.

55. There are three favorable outcomes of the 36 total


possible outcomes.
3

P (sum is greater than 10)  36  12 or about 8%

STAT ENTER 4 ENTER


ENTER
ENTER
3
4
5 ENTER
1
ENTER 5 ENTER 5 ENTER 1 ENTER

KEYSTROKES:

56. There are 15 favorable outcomes of the 36 total


possible outcomes.
15

15

P (sum is less than 7)  36  12 or about 42%

Step 2 (See Step 2 in Exercise 1.)


Step 3 Choose the viewing window.
Be sure you can see all of the points.
[10, 10] scl: 1 by [10, 10] scl: 1

57. There are 30 favorable outcomes of the 36 total


possible outcomes.
30

P (sum is greater than 4)  36  6 or about 83%


58. The number of toppings is the independent
variable, and the cost is the dependent variable.
So the data in the table can be represented as the
set of ordered pairs {(1, 9.95), (2, 11.45), (3, 12.95),
(4, 14.45), (5, 15.95), (6, 17.45)}.
59. Time is the independent variable, and the
temperature is the dependent variable. So the
data in the table can be represented as the set of
ordered pairs {(0, 100), (5, 90), (10, 81), (15, 73),
(20, 66), (25, 60), (30, 55)}.

ENTER 10
ENTER 1 ENTER 10 ENTER 10 ENTER 1

KEYSTROKES:

Step 4

WINDOW 10

(See Step 4 in Exercise 1.)

[10, 10] scl: 1 by [10, 10] scl: 1

131

Chapter 4

PQ249-6481F-04[122-168] 31/7/02 11:00 PM Page 132 Sahuja Ahuja_QXP_06:Desktop Folder:

3. {(12, 15), (10, 16), (11, 7), (14, 19)}


Step 1 Enter the data.
Enter the x-coordinates in L1 and the
y-coordinates in L2.

Page 207

Algebra Activity
y

ENTER 12 ENTER 10
ENTER 11 ENTER 14 ENTER
15
ENTER 16 ENTER 7 ENTER 19 ENTER

KEYSTROKES:

STAT

Step 2 (See Step 2 in Exercise 1.)


Step 3 Choose the viewing window.
Be sure you can see all of the points.
[15, 15] scl: 2 by [20, 20] scl: 2

FOLD LINE

ENTER 15
ENTER 2 ENTER 20 ENTER 20 ENTER 2

KEYSTROKES: WINDOW 15

Step 4

1. The inverse of each point matches the point.


2. The inverse of each point is a reflection of the
point across the fold.
3. Sample ordered pairs: (1, 1), (0, 0), (2, 2)
For each ordered pair (x, y), x  y.
4. Reflect the points across the line in which the
x-coordinate equals the y-coordinate.

(See Step 4 in Exercise 1.)

4-3

[15, 15] scl: 2 by [20, 20] scl: 2

4. {(45, 10), (23, 18), (22, 26), (35, 26)}


Step 1 Enter the data.
Enter the x-coordinates in L1 and the
y-coordinates in L2.

Page 208

Check for Understanding

1. A relation can be represented as a set of ordered


pairs, a table, a graph, or a mapping.
2. Sample answer:
{(1, 2), (3, 4), (5, 6), (7, 8), (9, 8)} has five elements
in its domain {1, 3, 5, 7, 9} and four elements in
its range {2, 4, 6, 8}.
3. The domain of a relation is the range of the
inverse, and the range of a relation is the domain
of the inverse.
4.
Table
Graph
List the set of
Graph each
x-coordinates in the
ordered pair on
first column and the
a coordinate
corresponding y-coordinates
plane.
in the second column.

STAT ENTER 45 ENTER


23 ENTER 22 ENTER 35 ENTER
10 ENTER 18 ENTER 26 ENTER 26 ENTER

KEYSTROKES:

Step 2 (See Step 2 in Exercise 1.)


Step 3 Choose the viewing window.
Be sure you can see all of the points.
[2, 50] scl: 2 by [2, 30] scl: 2
WINDOW 2 ENTER 50
ENTER 2 ENTER 2 ENTER 30 ENTER 2

KEYSTROKES:

Step 4

Relations

(See Step 4 in Exercise 1.)

x
5
8
7

y
2
3
1

4
2

O
8642

[2, 50] scl: 2 by [2, 30] scl: 2

2

5. The scale of the x-axis should include the least


and greatest values in the domain, and the scale
of the y-axis should include the least and greatest
values in the range.

Chapter 4

4

132

2 4 6 8x

PQ249-6481F-04[122-168] 31/7/02 11:00 PM Page 133 Sahuja Ahuja_QXP_06:Desktop Folder:

Mapping
List the x values in set X and the y values in set
Y. Draw an arrow from each x value in X to the
corresponding y value in Y.

Mapping
List the x values in set X and the y values in set
Y. Draw an arrow from each x value in X to the
corresponding y value in Y.

5
8
7

2
3
1

7
3
2

1
0
5

The domain for this relation is {7, 5, 8}. The


range is {2, 1, 3}.
5.
Table
Graph
List the set of
Graph each
x-coordinates in the
ordered pair
first column and the
on a coordinate
corresponding y-coordinates
plane.
in the second column.
x
6
3
1
5

y
4
3
9
3

10
8
6
4
2
2

2
4

The domain for this relation is {2, 3, 7}. The


range is {0, 1, 5}.
7.
Table
Graph
List the set of
Graph each
x-coordinates in the
ordered pair
first column and the
on a coordinate
corresponding y-coordinates
plane.
in the second column.
x
4
1
4
6

6
3
1
5

4
3
9

y
1
0
5

8.
9.
10.

11.
12.
13.

4

2

2
4
6
8

O
2

Mapping
List the x values in set X and the y values in set
Y. Draw an arrow from each x value in X to the
corresponding y value in Y.

The domain for this relation is {1, 3, 5, 6}. The


range is {3, 4, 9}.
6.
Table
Graph
List the set of
Graph each
x-coordinates in the
ordered pair on
first column and the
a coordinate
corresponding y-coordinates
plane.
in the second column.
x
7
3
2

8
6
4
2

6x

Mapping
List the x values in set X and the y values in set
Y. Draw an arrow from each x value in X to the
corresponding y value in Y.

y
8
9
7
9

14.
15.

133

4
1
6

8
9
7

The domain for this relation is {4, 1, 6}. The


range is {7, 8, 9}.
relation: {(3, 2), (6, 7), (4, 3), (6, 5)}
inverse: {(2, 3), (7, 6), (3, 4), (5, 6)}
relation: {(4, 9), (2, 5), (2, 2), (11, 12)}
inverse: {(9, 4), (5, 2), (2, 2), (12, 11)}
relation: {(3, 0), (5, 2), (7, 4)}
inverse: {(0, 3), (2, 5), (4, 7)}
relation: {(2, 8), (3, 7), (4, 6), (5, 7)}
inverse: {(8, 2), (7, 3), (6, 4), (7, 5)}
relation: {(1, 2), (2, 4), (3, 3), (4, 1)}
inverse: {(2, 1), (4, 2), (3, 3), (1, 4)}
relation: {(4, 4), (3, 0), (0, 3), (2, 1), (2, 1)}
inverse: {(4, 4), (0, 3), (3, 0), (1, 2), (1, 2)}
Sample answer: (1989, 25), (1991, 20), (1996, 10)
The domain of the relation is {1988, 1989, 1990,
1991, 1992, 1993, 1994, 1995, 1996, 1997, 1998,
1999}.

Chapter 4

PQ249-6481F-04[122-168] 31/7/02 11:00 PM Page 134 Sahuja Ahuja_QXP_06:Desktop Folder:

16. The least value in the range is about 5.7 students,


and the greatest value in the range is 25
students.
17. There are fewer students per computer in more
recent years. So the number of computers in
schools has increased.

Pages 209210
18.

19.

x
4
1
1
2

8
6
4
2
4

4
1
2

3
7
9

2

O
2

2
4
6
8

4x

22.

The domain of this


relation is {1, 2, 4}.
The range is {7, 3, 9}.

y
2
0
4
7

5
5
6
2

2
0
4
7

The domain of this


relation is {5, 2, 5, 6}.
The range is {0, 2, 4, 7}.

4321
2
4
6
8

0
6
5
4

0
1
6
2

3
2
1

8
7
9

x
4
3
1
6

O
1 2 3 4x

The domain of this


relation is {1, 2, 3}.
The range is {9, 7, 8}.

y
2
4
2
4

4
3
1
6

2

The domain of this


relation is {1, 3, 4, 6}.
The range is {2, 4}.

x
0
5
0
1

y
2
1
6
9

12
10
8
6
4
2

O
1 2x

4

6543 21
2

y
0
1
6
2

Chapter 4

8
6
4
2

23.

x
0
6
5
4

y
8
7
9
9

20.

x
3
3
2
1

Practice and Apply

y
3
7
3
9

x
5
5
6
2

21.

The domain of this


relation is {0, 4, 5, 6}.
The range is {1, 0, 2, 6}.

134

0
5
1

2
1
6
9

The domain of this


relation is {5, 1, 0}.
The range is {1, 2, 6, 9}.

PQ249-6481F-04[122-168] 31/7/02 11:00 PM Page 135 Sahuja Ahuja_QXP_06:Desktop Folder:

25.

x
3
4
2
5
4

3
4
2
5
4

4
3
2
4
5

x
7
3
4
2
3

35. relation: {(2, 0), (2, 4), (3, 7), (5, 0), (5, 8), (7, 7)}
inverse: {(0, 2), (4, 2), (7, 3), (0, 5), (8, 5), (7, 7)}
36. relation: {(3, 3), (2, 2), (1, 1), (1, 1), (2, 2),
(3, 3)}
inverse: {(3, 3), (2, 2), (1, 1), (1, 1), (2, 2),
(3, 3)}
37. relation: {(3, 1), (3, 3), (3, 5), (0, 3),
(2, 3), (4, 3)}
inverse: {(1, 3), (3, 3), (5, 3), (3, 0),
(3, 2), (3, 4)}
38.
y

y
4
3
2
4
5

The domain of this


relation is {4, 2, 3, 4, 5}.
The range is {4, 2, 3, 4, 5}.

222
218
Boiling Points (F)

24.

y
6
4
5
6
2

214
210
206
202
198
194
190
186
0

x
1000 3000 5000 7000 9000
2000 4000 6000 8000 10,000
Altitude (ft)

7
3
4
2
3

6
4
5
2

39. The inverse of the relation, as a set of ordered


pairs, is {(212.0, 0), (210.2, 1000), (208.4, 2000),
(206.5, 3000), (201.9, 5000), (193.7, 10,000)}.
40. Use the inverse relation to find the corresponding
altitude for a given boiling point.
41. The domain of the relation is {1991, 1992, 1993,
1994, 1995, 1996, 1997, 1998, 1999, 2000}. The
range of the relation is approximately
{6.3, 7.5, 9.2, 9.5, 9.8, 10, 10.4}.
42. The lowest production, about 6.3 billion bushels,
occurred in 1993. The highest production, about
10.4 billion bushels, occurred in 2000.
43. Production seems to alternately increase and
decrease each year. However, since 1995, the
trend has shown an overall production increase.
44. Body Weight (lb)
Muscle Weight (lb)

The domain of this


relation is {3, 2, 3, 4, 7}.
The range is {2, 4, 5, 6}.

26. relation: {(1, 2), (3, 4), (5, 6), (7, 8)}
inverse: {(2, 1), (4, 3), (6, 5), (8, 7)}
27. relation: {(0, 3), (5, 2), (4, 7), (3, 2)}
inverse: {(3, 0), (2, 5), (7, 4), (2, 3)}
28. relation: {(6, 2), (4, 5), (3, 3), (1, 7)}
inverse: {(2, 6), (5, 4), (3, 3), (7, 1)}
29. relation: {(8, 4), (1, 1), (0, 6), (5, 4)}
inverse: {(4, 8), (1, 1), (6, 0), (4, 5)}
30. relation: {(4, 2), (2, 1), (2, 4), (2, 3)}
inverse: {(2, 4), (1, 2), (4, 2), (3, 2)}
31. relation: {(3, 3), (1, 3), (4, 2), (1, 5)}
inverse: {(3, 3), (3, 1), (2, 4), (5, 1)}
32. relation: {(0, 0), (4, 7), (8, 10.5), (12, 13),
(16, 14.5)}
inverse: {(0, 0), (7, 4), (10.5, 8), (13, 12), (14.5, 16)}
33. relation: {(1, 16.50), (1.75, 28.30), (2.5, 49.10),
(3.25, 87.60), (4, 103.40)}
inverse: {(16.50, 1), (28.30, 1.75), (49.10, 2.5),
(87.60, 3.25), (103.40, 4)}
34. relation: {(3, 2), (3, 8), (6, 5), (7, 4), (11, 4)}
inverse: {(2, 3), (8, 3), (5, 6), (4, 7), (4, 11)}

100
105
110
115
120
125
130

40
42
44
46
48
50
52

45. The domain of the relation is {100, 105, 110, 115,


120, 125, 130}. The range is {40, 42, 44, 46, 48,
50, 52}.

135

Chapter 4

PQ249-6481F-04[122-168] 31/7/02 11:00 PM Page 136 Sahuja Ahuja_QXP_06:Desktop Folder:

46.

52. The inverse of the relation is


{(4, 4), (2, 2), (0, 0), (2, 2), (4, 4)}.

y
52

Muscle Weight (lb)

50
48
O

46

44
42

53a. Sample answer:

40
100 105 110 115 120 125 130
Body Weight (lb)

47. The domain of the inverse of the relation is


{40, 42, 44, 46, 48, 50, 52}. The range of the
inverse is {100, 105, 110, 115, 120, 125, 130}.
48.
8

4
4

4

53b. Sample answer:


[10, 10] scl: 1 by [10, 12] scl: 1
53c. The inverse of the relation is
{(10, 0), (8, 2), (6, 6), (4, 9)}.
Sample answer:

A
8

[10, 10] scl: 1 by [10, 12] scl: 1

8x

8

49. Sample answer:


F  {(1, 1), (2, 2), (3, 3)}
G  {(1, 2), (2, 3), (3, 1)}
The elements in the domain and range of F
should be paired differently in G.
50. Expressing real-world data as relations shows
how the members of a domain relate to the
members of the range. For example, a table helps
to organize the data or a graph may show a
pattern in the data. Answers should include the
following.
Graph

Strikeouts

130

[10, 12] scl: 1 by [10, 10] scl: 1

53d.

Relation
Point Quadrant
(0, 10)
none
(2, 8)
IV
(6, 6)
I
(9, 4)
IV

Inverse
Point Quadrant
(10, 0)
none
(8, 2)
II
(6, 6)
I
(4, 9)
II

54a. Sample answer:

110
90
70
50
0

20

40 60 80
Home Runs

[10, 10] scl: 1 by [6, 40] scl: 2

54b. Sample answer:


[10, 10] scl: 1 by [6, 40] scl: 2

There seems to be a positive relationship


between the number of home runs and
strikeouts. In years when Griffey hit more
home runs, he also struck out more.
51. B; The set of ordered pairs graphed is {(4, 4),
(2, 2), (0, 0), (2, 2), (4, 4)}. The domain of this
relation is {4, 2, 0, 2, 4}. The range is {0, 2, 4}.

Chapter 4

136

PQ249-6481F-04[122-168] 31/7/02 11:00 PM Page 137 Sahuja Ahuja_QXP_06:Desktop Folder:

54c. The inverse of the relation is


{(18, 1), (23, 2), (28, 3), (33, 4)}.
Sample answer:

56c. The inverse of the relation is


{(77, 92), (200, 93), (50, 19)}.
Sample answer:

[6, 40] scl: 2 by [10, 10] scl: 1

54d.

Relation
Point Quadrant
(1, 18)
II
(2, 23)
II
(3, 28)
II
(4, 33)
II

[100, 250] scl: 10 by [100, 50] scl: 10

56d.

Inverse
Point Quadrant
(18, 1)
IV
(23, 2)
IV
(28, 3)
IV
(33, 4)
IV

Relation
QuadPoint
rant
(92, 77)
III
(93, 200)
II
(19, 50)
IV

Inverse
QuadPoint
rant
(77, 92)
III
(200, 93)
IV
(50, 19)
II

55a. Sample answer:

Page 211

[10, 80] scl: 5 by [10, 60] scl: 5

55b. Sample answer:


[10, 80] scl: 5 by [10, 60] scl: 5
55c. The inverse of the relation is
{(12, 35), (25, 48), (52, 60)}.
Sample answer:

xyCoordi- Coordi- Ordered


Point nate
nate
Pair
Quadrant
60. A
4
4
(4, 4)
IV
61. K
3
2
(3, 2)
I
62. L
1
3
(1, 3)
III
63. W
1
1
(1, 1)
IV
64. B
4
0
(4, 0)
none
65. P
4
2
(4, 2)
III
66. R
5
3
(5, 3)
II
67. C
2
5
(2, 5)
II

[10, 60] scl: 5 by [10, 80] scl: 5

55d.

Relation
Point Quadrant
(35, 12)
I
(48, 25)
I
(60, 52)
I

Maintain Your Skills

57. The figure has been turned around a point. This


is a rotation.
58. The figure has been flipped over a line. This is a
reflection.
59. The figure has been shifted vertically down. This
is a translation.

68. Find the amount of change.


10.15  9.75  0.40
Find the percent using the original number, 9.75,
as the base.

Inverse
Point Quadrant
(12, 35)
I
(25, 48)
I
(52, 60)
I

0.40
9.75

 100

0.40(100)  9.75 r
40  9.75 r

56a. Sample answer:

40
9.75

9.75 r
9.75

4.1  r
The percent of increase in Dominiques salary is
about 4.1%.
69. 72  9  8
70. 105  15  7
3

71. 3  3  3  1  1  9

[100, 50] scl: 10 by [100, 250] scl: 10

72. 16  4  16  1 

56b. Sample answer:


[100, 50] scl: 10 by [100, 250) scl: 10

137

64
1

 64

Chapter 4

PQ249-6481F-04[122-168] 31/7/02 11:00 PM Page 138 Sahuja Ahuja_QXP_06:Desktop Folder:

73.

54n  78
6



98x  35y
7

12
12

1
(54n  78) 6
1
1
54n 6  78 6

12

 9n  13
74.

79.

 (54n  78)  6

 (98x  35y)  7

12
1
1
 98x 1 7 2  35y 1 7 2
1
7

 (98x  35y)

75. a

a  15  20
?

3  15  20 S 18 20

4  15  20 S 19 20

5  15  20 S 20  20

?
?

6  15  20 S 21 20

7  15  20 S 22 20

8  15  20 S 23 20

True or False?
false

r62

3  6  2 S 3 2

?
?

4  6  2 S 2 2

5  6  2 S 1 2

80.

true

9  5(6)  6 S 9 24

9  5(7)  6 S 9 29

9  5(8)  6 S 9 34

3
4
5
6
7
8

3  8w  35
?

3  8(3)  35 S 27 35
?

3  8(4)  35 S 35  35
?

3  8(5)  35 S 43 35
?

3  8(6)  35 S 51 35
?

3  8(7)  35 S 59 35
?

3  8(8)  35 S 67 35

false

8
3

 15  17 S 173 17

m
5

m
3
5
4
5

52

3 ?

false

false

false

false

3 ?

5

2 S 15 2

3 ?

5

2 S 15 2

 5  2 S 25 2

false

3 ?
3 ?
3 ?

True or False?

 5  2 S 15 2

8
5

true
false

Since m  7 makes the equation true, the


solution set is {7}.

Page 211

Practice Quiz 1

1. Q(2, 3)
Start at the origin.
Move right 2 units and up 3 units.
Draw a dot and label it Q.
(See coordinate plane after Exercise 4.)
2. R(4, 4)
Start at the origin.
Move left 4 units and down 4 units.
Draw a dot and label it R.
(See coordinate plane after Exercise 4.)
3. S(5, 1)
Start at the origin.
Move right 5 units and down 1 unit.
Draw a dot and label it S.
(See coordinate plane after Exercise 4.)

false
false
false
false
false

True or False?
false
true
false
false
false
false

Since w  4 makes the equation true, the solution


set is {4}.
Chapter 4

 15  17 S 173 17

Since n  3 makes the equation true, the solution


set is {3}.
78. w

false

7
3

false

true

5  2S 22

9  5(3)  6 S 9  9

true

7
5

True or False?

9  5(5)  6 S 9 19

 15  17 S 17  17

true

false

false

22

9  5(4)  6 S 9 14

 5  2 S 15 2

false

6
3

6
5

1 2

 15  17 S 163 17

false

True or False?

9  5n  6

5
3

Since r  8 makes the equation true, the solution


set is {8}.
77.

false

5
5

86 2S

 15  17 S 163 17

76 2S

4
3

false

false

false

0 2

66 2S

 15  17 S 16 17

false

3
3

True or False?

Since g  6 makes the equation true, the solution


set is {6}.

false

Since a  5 makes the equation true, the solution


set is {5}.
76.

 15  17

 14x  5y

g
3

138

PQ249-6481F-04[122-168] 31/7/02 11:00 PM Page 139 Sahuja Ahuja_QXP_06:Desktop Folder:

7. The domain of the relation is {1, 2, 4}. The range


is {3, 5, 6}. The inverse of the relation is
{(3, 1), (6, 4), (3, 2), (5, 1)}.
8. The domain of the relation is {2, 0, 4, 8}. The
range is {5, 2, 3, 6}. The inverse of the relation
is {(6, 2), (3, 0), (2, 4), (5, 8)}.
9. The domain of the relation is {8, 11, 15}. The
range is {3, 5, 22, 31}. The inverse of the relation
is {(5, 11), (3, 15), (22, 8), (31, 11)}.
10. The domain of the relation is {5, 1, 2, 6}. The
range is {0, 3, 4, 7, 8}. The inverse of the relation
is {(8, 5), (0, 1), (4, 1), (7, 2), (3, 6)}.

4. T(1, 3)
Start at the origin.
Move left 1 unit and up 3 units.
Draw a dot and label it T.
y

S
R

Equations as Relations

4-4
5. To reflect the triangle over the x-axis, multiply
the y-coordinate of each vertex by 1.
(x, y) S (x, y)
A(4, 8) S A(4, 8)
B(7, 5) S B(7, 5)
C(2, 1) S C(2, 1)
The coordinates of the vertices of the image are
A
(4, 8), B
(7, 5), and C
(2, 1).
8

Pages 214215

A
B

C
O

4

7x

8

1

10

12

6. To translate the quadrilateral 5 units to the left,


add 5 to the x-coordinate of each vertex. To
translate the quadrilateral 4 units down, add 4
to the y-coordinate of each vertex.
(x, y) (x  5, y  4)
W(1, 0) W
(1  5, 0  4) W
(4, 4)
X(2, 3) X
(2  5, 3  4) X
(3, 1)
Y(4, 1) Y
(4  5, 1  4) Y
(1, 3)
Z(3, 3) Z
(3  5, 3  4) Z
(2, 7)
The coordinates of the vertices of the image are
W
(4, 4), X
(3, 1), Y
(1, 3), and
Z
(2, 7).
y

6.

Y
Z

true
true
false
false

x
7

y
3

2

1

2x  5y  1
2(7)  5(3)  1
11
2(7)  5(3)  1
1  1
2(2)  5(1)  1
1  1
2(2)  5(1)  1
11

True or False?
true
false
false
true

The solution set is {(7, 3), (2, 1)}.

W
O

1  3(1)  4
11
10  3(2)  4
10  10
12  3(3)  4
12  13
1  3(7)  4
1  25

The solution set is {(1, 1), (2, 10)}.


5.

Y
X

Check for Understanding

1. To find the domain of an equation if you are given


the range, substitute the values for y and solve
for x.
2. Sample answer: y  x  5 is an equation in two
variables. Two solutions for this equation are
(9, 4) and (10, 5).
3. Bryan is correct. x represents the domain and y
represents the range. So, replace x with 5 and y
with 1.
4. x
y
y  3x  4
True or False?

x
3
1
0
2

2x  1
2(3)  1
2(1)  1
2(0)  1
2(2)  1

y
7
3
1
3

(x, y)
(3, 7)
(1, 3)
(0, 1)
(2, 3)

The solution set is


{(3, 7), (1, 3), (0, 1), (2, 3)}.

139

Chapter 4

PQ249-6481F-04[122-168] 31/7/02 11:00 PM Page 140 Sahuja Ahuja_QXP_06:Desktop Folder:

7.

4x
4  (3)
4  (1)
40
42

x
3
1
0
2

y
7
5
4
2

11. First solve the equation for y in terms of x.


2y  x  2

(x, y)
(3, 7)
(1, 5)
(0, 4)
(2, 2)

2y
2

y

The solution set is {(3, 7), (1, 5), (0, 4), (2, 2)}.
8. First solve the equation for y in terms of x.
2y  2x  12
2y  2x  2x  12  2x
2y  12  2x
2y
2

x2
2

(x, y)

4

4  2
2

1

(4, 1)

2

2  2
2

(2, 0)

0  2
2

(0, 1)

2  2
2

(2, 2)

4  2
2

(4, 3)

12  2x
2

6x
6  (3)
6  (1)
60
62

y
9
7
6
4

(x, y)
(3, 9)
(1, 7)
(0, 6)
(2, 4)

x  2
2
x  2
2

y6x
x
3
1
0
2

Graph the solution set


{(4, 1), (2, 0), (0, 1), (2, 2), (4, 3)}.
y

The solution set is {(3, 9), (1, 7), (0, 6), (2, 4)}.
9. First solve the equation for y in terms of x.
3x  2y  13
3x  2y  3x  13  3x
2y  13  3x


y

12.
y

(x, y)

3

13  3x
2
13  3(3)
2

11

(3, 11)

1

13  3(1)
2

(1, 8)

13  3(0)
2

6.5

(0, 6.5)

13  3(2)
2

3.5

(2, 3.5)

10.

13  3x
2
13  3x
2

3x
3(3)
3(2)
3(1)
3(0)
3(1)
3(2)
3(3)

x
3
2
1
0
1
2
3

y
9
6
3
0
3
6
9

(x, y)
(3, 9)
(2, 6)
(1, 3)
(0, 0)
(1, 3)
(2, 6)
(3, 9)

4

Chapter 4

2

2
4
6
8

10

25(10)
6

413

14

25(14)
6

583

18

25(18)
6

75

24

25(24)
6

100

2
1

80
70
60
50
40
0

25k
6

90

Graph the solution set {(3, 9), (2, 6),


(1, 3), (0, 0), (1, 3), (2, 6), (3, 9)}.
8
6
4
2

100

Gold %

2y
2

4x

140

x
10 12 14 16 18 20 22 24 26
Number of Karats

(k, g)

110, 4123 2
114, 5813 2
(18, 75)

(24, 100)

PQ249-6481F-04[122-168] 31/7/02 11:00 PM Page 141 Sahuja Ahuja_QXP_06:Desktop Folder:

25k

13. Solve the equation g  6 for k since the value of


k will depend on the given value of g.
g
6g 

18. x
0

25k
6
25k
6 6

1 2

6g  25k
6g
25
6g
25

y
0.5

0.75

25k
25

k

For a ring that is 50% gold, g  50. So


6(50)
k  25  12. A 12-karat ring is 50% gold.

14. x
2

y
1

True or False?

3

x
4

y
4

y  8  3x
4  8  3(4)
4  4
0  8  3(8)
0  16
2  8  3(2)
22
3  8  3(3)
3  1

true

x
1

y
2

1

x  3y  7
1  3(2)  7
7  7
2  3(2)  7
4  7
2  3(4)  7
10  7
2  3(3)  7
7  7

x
3

y
0

2

1

1

2x  2y  6
2(3)  2(0)  6
66
2(2)  2(1)  6
66
2(2)  2(1)  6
6  6
2(4)  2(1)  6
66

false
false
false

3.5

2y  4x  8
2(2)  4(0)  8
48
2(0.5)  4(3)  8
11  8
2(3.5)  4(0.25)  8
88
2(2)  4(1)  8
88

True or
False?
false
false
true
true

The solution set is {(0.25, 3.5), (1, 2)}.


true

20.

True or False?
true
false

x
2
1
1
3
4

4  5x
4  5(2)
4  5(1)
4  5(1)
4  5(3)
4  5(4)

y
14
9
1
11
16

(x, y)
(2, 14)
(1, 9)
(1, 1)
(3, 11)
(4, 16)

The solution set is


{(2, 14), (1, 9), (1, 1), (3, 11), (4, 16)}.

true

21.

false

True or False?
true

x
2
1
1
3
4

2x  3
2(2)  3
2(1)  3
2(1)  3
2(3)  3
2(4)  3

y
1
1
5
9
11

(x, y)
(2, 1)
(1, 1)
(1, 5)
(3, 9)
(4, 11)

The solution set is


{(2, 1), (1, 1), (1, 5), (3, 9), (4, 11)}.
22. First solve the equation for y in terms of x.
xy4
x4y44
x4y

false
false
true

x
2
1
1
3
4

The solution set is {(1, 2), (2, 3)}.


17.

0.5

0.25

false

The solution set is {(4, 4), (2, 2)}.


16.

y
2

false

The solution set is {(1, 5), (0, 1)}.


15.

true

The solution set is {(0, 0.5)}.

Practice and Apply


y  4x  1
1  4(2)  1
1  9
5  4(1)  1
55
2  4(9)  1
2  37
1  4(0)  1
11

True or False?

19.
x

Pages 215217

3x  8y  4
3(0)  8(0.5)  4
4  4
3(4)  8(1)  4
4  4
3(2)  8(0.75)  4
0  4
3(2)  8(4)  4
26  4

True or False?
true
true

x4
2  4
1  4
14
34
44

y
6
5
3
1
0

(x, y)
(2, 6)
(1, 5)
(1, 3)
(3, 1)
(4, 0)

The solution set is


{(2, 6), (1, 5), (1, 3), (3, 1), (4, 0)}.

false
true

The solution set is {(3, 0), (2, 1), (4, 1)}.

141

Chapter 4

PQ249-6481F-04[122-168] 31/7/02 11:00 PM Page 142 Sahuja Ahuja_QXP_06:Desktop Folder:

26. First solve the equation for y in terms of x.


8x  4y  12
8x  4y  8x  12  8x
4y  12  8x

23. First solve the equation for y in terms of x.


x7y
xy7yy
xy7
xyx7x
y7x
7x
7  (2)
7  (1)
71
73
74

x
2
1
1
3
4

y
9
8
6
4
3


4
y  3  2x

(x, y)
(2, 9)
(1, 8)
(1, 6)
(3, 4)
(4, 3)

3  2x
3  2(2)
3  2(1)
3  2(1)
3  2(3)
3  2(4)

x
2
1
1
3
4

The solution set is


{(2, 9), (1, 8), (1, 6), (3, 4), (4, 3)}.
24. First solve the equation for y in terms of x.
6x  3y  18
6x  3y  6x  18  6x
3y  18  6x
3y
3

18  6x
3

6  2x
6  2(2)
6  2(1)
6  2(1)
6  2(3)
6  2(4)

2x
2

y
10
8
4
0
2

3  6x
3  6(2)
3  6(1)
3  6(1)
3  6(3)
3  6(4)

y
9
3
9
21
27

2y
2

(x, y)

2
1
1
3
4

2
1
1
3
4

(2, 2)
(1, 1)
(1, 1)
(3, 3)
(4, 4)

The solution set is


{(2, 2), (1, 1), (1, 1), (3, 3), (4, 4)}.
28. First solve the equation for y in terms of x.
5x  10y  20
5x  10y  5x  20  5x
10y  20  5x
10y
10

(x, y)
(2, 9)
(1, 3)
(1, 9)
(3, 21)
(4, 27)

20  5x
10

y  2  0.5x
x
2
1
1
3
4

The solution set is


{(2, 9), (1, 3), (1, 9), (3, 21), (4, 27)}.

Chapter 4

(x, y)
(2, 7)
(1, 5)
(1, 1)
(3, 3)
(4, 5)

xy

(x, y)
(2, 10)
(1, 8)
(1, 4)
(3, 0)
(4, 2)

The solution set is


{(2, 10), (1, 8), (1, 4), (3, 0), (4, 2)}.
25. First solve the equation for y in terms of x.
6x  y  3
6x  y  6x  3  6x
y  3  6x
1(y)  1(3  6x)
y  3  6x
x
2
1
1
3
4

y
7
5
1
3
5

The solution set is


{(2, 7), (1, 5), (1, 1), (3, 3), (4, 5)}.
27. First solve the equation for y in terms of x.
2x  2y  0
2x  2y  2y  0  2y
2x  2y

y  6  2x

x
2
1
1
3
4

12  8x

4y
4

2  0.5x
2  0.5(2)
2  0.5(1)
2  0.5(1)
2  0.5(3)
2  0.5(4)

y
3
2.5
1.5
0.5
0

(x, y)
(2, 3)
(1, 2.5)
(1, 1.5)
(3, 0.5)
(4, 0)

The solution set is


{(2, 3), (1, 2.5), (1, 1.5), (3, 0.5), (4, 0)}.

142

PQ249-6481F-04[122-168] 31/7/02 11:00 PM Page 143 Sahuja Ahuja_QXP_06:Desktop Folder:

29. First solve the equation for y in terms of x.


3x  2y  14
3x  2y  3x  14  3x
2y  14  3x
2y
2

y

14  3x
2
14  3x
2

14  3x
2

(x, y)

2

14  3(2)
2

10

(2, 10)

1

14  3(1)
2

8.5

(1, 8.5)

14  3(1)
2

5.5

(1, 5.5)

14  3(3)
2

2.5

(3, 2.5)

14  3(4)
2

32.

2x  3
2(3)  3
2(2)  3
2(1)  3
2(1)  3
2(2)  3
2(3)  3

x
3
2
1
1
2
3

y
3
1
1
5
7
9

(x, y)
(3, 3)
(2, 1)
(1, 1)
(1, 5)
(2, 7)
(3, 9)

Graph the solution set


{(3, 3), (2, 1), (1, 1), (1, 5), (2, 7), (3, 9)}.
y

(4, 1)

The solution set is


{(2, 10), (1, 8.5), (1, 5.5), (3, 2.5), (4, 1)}.
30. First solve the equation for y in terms of x.

x2y8
1

x  2y  x  8  x

1
y
2
1
y
2

33.

8x

1 2  2(8  x)
y  16  2x

16  2x

(x, y)

2
1
1
3
4

16  2(2)
16  2(1)
16  2(1)
16  2(3)
16  2(4)

20
18
14
10
8

(2, 20)
(1, 18)
(1, 14)
(3, 10)
(4, 8)

2x 
2x 

1
y
3

8
6
4
2
4

4

(3) 3 y  (3)(4  2x)


x
2
1
1
3
4

y  12  6x
12  6x
12  6(2)
12  6(1)
12  6(1)
12  6(3)
12  6(4)

2

4
6
8
10
12
14
16

3 y  4  2x
1

(x, y)
(5, 16)
(2, 7)
(1, 2)
(3, 8)
(4, 11)

 2x  4  2x
1

y
16
7
2
8
11

Graph the solution set


{(5, 16), (2, 7), (1, 2), (3, 8), (4, 11)}.

The solution set is


{(2, 20), (1, 18), (1, 14), (3, 10), (4, 8)}.
31. First solve the equation for y in terms of x.
1
y
3

3x  1
3(5)  1
3(2)  1
3(1)  1
3(3)  1
3(4)  1

x
5
2
1
3
4

y
24
18
6
6
12

(x, y)
(2, 24)
(1, 18)
(1, 6)
(3, 6)
(4, 12)

The solution set is


{(2, 24), (1, 18), (1, 6), (3, 6), (4, 12)}.

143

Chapter 4

PQ249-6481F-04[122-168] 31/7/02 11:00 PM Page 144 Sahuja Ahuja_QXP_06:Desktop Folder:

34. First solve the equation for y in terms of x.


3x  2y  5
3x  2y  3x  5  3x
2y  5  3x
 2y
2

y

Graph the solution set {(4, 7), (1, 3.25), (0, 2),
(2, 0.5), (4, 3), (6, 5.5)}.
y

5  3x
2
3x  5
2

3x  5
2

(x, y)

3

3(3)  5
2

7

(3, 7)

1

3(1)  5
2

4

(1, 4)

3(2)  5
2

0.5

(2, 0.5)

3(4)  5
2

3.5

(4, 3.5)

3(5)  5
2

36. First solve the equation for y in terms of x.

(5, 5)

1
x
2

Graph the solution set


{(3, 7), (1, 4), (2, 0.5), (4, 3.5), (5, 5)}.

1
x
2

y2
1

 y  2x  2  2x
1

y  2  2x

35. First solve the equation for y in terms of x.


5x  4y  8
5x  4y  5x  8  5x
4y  8  5x
4y
4

y

(x, y)

(4, 4)

4

2  2(4)

1

2  2(1)

2.5

(1, 2.5)

2  2 112
1

1.5

(1, 1.5)

2  2(4)

2

1
(7)
2

1.5

2  2(8)

2

(4, 0)
(7, 1.5)
(8, 2)

8  5x
4
8  5x
4

(x, y)

4

8  5(4)
4

(4, 7)

1

8  5(1)
4

3.25

8  5(0)
4

8  5(2)
4

0.5

8  5(4)
4

3

8  5(6)
4

5.5

Chapter 4

Graph the solution set {(4, 4), (1, 2.5), (1, 1.5),
(4, 0), (7, 1.5), (8, 2)}.

8  5x
4

2  2x

(1, 3.25)
(0, 2)
(2, 0.5)
(4, 3)
(6, 5.5)

144

PQ249-6481F-04[122-168] 31/7/02 11:00 PM Page 145 Sahuja Ahuja_QXP_06:Desktop Folder:

37.

1
x
4

3

4

1
(4)
4

3

4

2

1
(2)
4

3

3.5

1
(0)
4

3

3

1
(2)
4

1
(4)
4

3

2

1
(6)
4

3

1.5

3

2.5

41.

(x, y)
(4, 4)
(2, 3.5)
(0, 3)
(2, 2.5)
(4, 2)
(6, 1.5)

42.

8  3x
8  3(1)
8  3(2)
8  3(5)
8  3(8)

2y  6
2(4)  6
2(3)  6
2(1)  6
2(6)  6
2(7)  6

34

34  32
1.8

1.1

Chicago

23

23  32
1.8

San Francisco

55

55  32
1.8

12.7

72

72  32
1.8

22.2

40

40  32
1.8

4.4

5

P  2/  2w
24  2/  2w
24  2w  2/  2w  2w
24  2w  2/


2/
2

12  w
12  1
12  2
12  3
12  4
12  5

w
1
2
3
4
5


11
10
9
8
7

(w, )
(1, 11)
(2, 10)
(3, 9)
(4, 8)
(5, 7)

For the values of w chosen, the solution set is


{(1, 11), (2, 10), (3, 9), (4, 8), (5, 7)}.

y
11
2
7
16

45.

x
14
12
4
6
8

Length of
Tibia (cm)
30.5
34.8
36.3
37.9

Male
Height
(cm)
81.7  2.4(30.5)
81.7  2.4(34.8)
81.7  2.4(36.3)
81.7  2.4(37.9)

(T, H )
(30.5, 154.9)
(34.8, 165.2)
(36.3, 168.8)
(37.9, 172.7)

Length of
Tibia (cm)
30.5
34.8
36.3
37.9

Female
Height
(cm)
72.6  2.5(30.5)
72.6  2.5(34.8)
72.6  2.5(36.3)
72.6  2.5(37.9)

(T, H )
(30.5, 148.9)
(34.8, 159.6)
(36.3, 163.4)
(37.9, 167.4)

176
172
168
164
160
156
152
148
144

The domain is {14, 12, 4, 6, 8}.


40.
F  1.8C  32
F  32  1.8C  32  32
F  32  1.8C


New York

12  w  /
43. Since the equation is solved for , the dependent
variable is  and the independent variable is w.
44. Sample answer:

The range is {16, 7, 2, 11}.


39. Since the range is given, first solve the equation
for x in terms of y.
2y  x  6
2y  x  2y  6  2y
x  6  2y
1121x2  11216  2y2
x  2y  6

F  32
1.8
F  32
1.8

24  2w
2

38. First solve the equation for y in terms of x.


3x  y  8
3x  y  3x  8  3x
y  8  3x

y
4
3
1
6
7

F  32
1.8

Washington, D.C.

x
1
2
5
8

Miami

Graph the solution set {(4, 4), (2, 3.5),


(0, 3), (2, 2.5), (4, 2), (6, 1.5)}.

City

1.8C
1.8

C

Male

Female

T
30 31 32 33 34 35 36 37 38

46. See students work.

145

Chapter 4

PQ249-6481F-04[122-168] 31/7/02 11:00 PM Page 146 Sahuja Ahuja_QXP_06:Desktop Folder:

47c. Substitute each given range value for y and


solve for x.
y  04x  16 0
0  04x  16 0
The only number whose absolute value gives 0 is
0, so the expression 4x 16 must represent 0.
4x  16  0
4x  16  16  0  16
4x  16

47a. Substitute each given range value for y and


solve for x.
y  x2
0  x2
The only number that gives 0 when squared is
0, so x  0 when y  0.
16  x2
Both 42 and (4)2 give 16, so x  4 or x  4
when y  16.

4x
4

36  x2
Both 62 and (6)2 give 36, so x  6 or x  6
when y  36.
Thus, if the range of y  x2 is {0, 16, 36}, then
the domain is {6, 4, 0, 4, 6}.
47b. Substitute each given range value for y and
solve for x.
y  04x 0  16
0  04x 0  16
0  16  04x 0  16  16
16  04x 0
Since both 016 0 and 016 0 give 16, the expression
4x may represent either 16 or 16.
4x  16
4x
4

16
4

4x  16  16
4x  16  16  16  16
4x  32
4x
4

4x
4

16
4

4x
4

4x
4

0
4

x0

52
4

4x
4

20
4

x  13
x  5
So, x  13 or x  5 when y  36.
Thus, if the range of y  04x  16 0 is {0, 16, 36},
then the domain is {5, 0, 4, 8, 13}.
48. Sample answer:
x
1
2
3
4
5

x4
14
24
34
44
54

y
5
6
7
8
9

(x, y)
(1, 5)
(2, 6)
(3, 7)
(4, 8)
(5, 9)

For the values of x chosen, the relation is


{(1, 5), (2, 6), (3, 7), (4, 8), (5, 9)} and the inverse
relation is {(5, 1), (6, 2), (7, 3), (8, 4), (9, 5)}.
Since each y-coordinate is 4 less than each
x-coordinate in each ordered pair of the inverse
relation, the equation of the inverse relation is
y  x  4.

52
4

x  13
x  13
So, x  13 or x  13 when y  36.
Thus, if the range of y  04x 0  16 is {0, 16, 36},
then the domain is {13, 8, 4, 4, 8, 13}.

Chapter 4

32
4

x8

x8
x  8
So, x  8 or x  8 when y  16.
36  04x 0  16
36  16  04x 0  16  16
52  04x 0
Since both 052 0 and 052 0 give 52, the expression
4x may represent either 52 or 52.
4x  52
4x  52
52
4

4x  16  16
4x  16  16  16  16
4x  0

So, x  8 or x  0 when y  16.


36  04x  16 0
Since both 036 0 and 036 0 give 36, the expression
4x  16 may represent either 36 or 36.
4x  16  36
4x  16  36
4x  16  16  36  16 4x  16  16  36  16
4x  52
4x  20

x4
x  4
So, x  4 or x  4 when y  0.
16  04x 0  16
16  16  04x 0  16  16
32  04x 0
Since both 032 0 and 032 0 give 32, the expression
4x may represent either 32 or 32.
4x  32
4x  32
4x
32
4x
32

 4
4
4
4

4x
4

16
4

x4
So, x  4 when y  0.
16  04x  16 0
Since both 016 0 and 016 0 give 16, the expression
4x 16 may represent either 16 or 16.

4x  16
4x
4

146

PQ249-6481F-04[122-168] 31/7/02 11:01 PM Page 147 Sahuja Ahuja_QXP_06:Desktop Folder:

49. When traveling to other countries, currency and


measurement systems are often different. You
need to convert these systems to the system with
which you are familiar. Answers should include
the following.
At the current exchange rate, 15 pounds is
roughly 10 dollars and 10 pounds is roughly
7 dollars. Keeping track of every 15 pounds
you spend would be relatively easy.
If the exchange rate is 0.90 compared to the
dollar, then items will cost less in dollars.
For example, an item that is 10 in local
currency is equivalent to $9.00. If the
exchange rate is 1.04, then items will cost
more in dollars. For example, an item that
costs 10 in local currency is equivalent to
$10.40.
50. D;
3x  y  18
3x  3  18
3x  3  3  18  3
3x  21
3x
3

54.

X
.4
.6
1.8
2.2
3.1

55.

TABLE
ENTER 2nd
TABLE 2.5 ENTER 1.75 ENTER 0 ENTER
1.25 ENTER 3.33 ENTER
X
2.5
1.75
0
1.25
3.33

21

CLEAR 3 X,T,,n
ENTER 2nd

X
11
15
23
44

Page 217

KEYSTROKES:

Y1
37
41
65
128

CLEAR 6.5 X,T,,n


ENTER 2nd

X
8
5
0
3
7
12

TblSet
TABLE 8 ENTER 5 ENTER 0 ENTER 3
ENTER 7 ENTER 12 ENTER

42 2nd

Maintain Your Skills

56. relation: {(4, 9), (3, 2), (1, 5), (4, 2)}
inverse: {(9, 4), (2, 3), (5, 1), (2, 4)}
57. relation: {(2, 7), (6, 4), (6, 1), (11, 8)}
inverse: {(7, 2), (4, 6), (1, 6), (8, 11)}
58. relation: {(3, 2), (2, 3), (3, 3), (4, 2)}
inverse: {(2, 3), (3, 2), (3, 3), (2, 4)}
59. To reflect the triangle over the y-axis, multiply
the x-coordinate of each vertex by 1.
(x, y) S (x, y)
X(6, 4) S X(6, 4)
Y(5, 0) S Y(5, 0)
Z(3, 3) S Z(3, 3)

The solution set is


{(11, 37), (15, 41) , (23, 65), (44, 128)}.
53.

Y1
4.26
3.21
.76
.99
3.902

The solution set is {(2.5, 4.26), (1.75, 3.21),


(0, 0.76), (1.25, 0.99), (3.33, 3.90)}.

TblSet
TABLE 11 ENTER 15 ENTER 23 ENTER 44
ENTER
2nd

CLEAR 1.4 X,T,,n

KEYSTROKES:

0.76 2nd

2/  2w
2

KEYSTROKES:

Y1
13.2
13.8
17.4
18.6
21.3

The solution set is {(0.4, 13.2), (0.6, 13.8),


(1.8, 17.4), (2.2, 18.6), (3.1, 21.3)}.

7/w
So, the sum of the length and width is 7, and the
product of the length and width is 12. Of the
dimensions given, this is true only for a 3  4
rectangle.
52.

12

TblSet
TABLE 0.4 ENTER 0.6 ENTER 1.8 ENTER
2.2 ENTER 3.1 ENTER
2nd

 3
x7
51. C; P  2/  2w
A  /w
14  2/  2w
12  /w
14
2

CLEAR 3 X,T,,n
ENTER 2nd

KEYSTROKES:

Z
Y

Y1
94
74.5
42
22.5
3.5
36

Z
O

The solution set is {(8, 94), (5, 74.5), (0, 42),


(3, 22.5), (7, 3.5), (12, 36)}.

147

Chapter 4

PQ249-6481F-04[122-168] 31/7/02 11:01 PM Page 148 Sahuja Ahuja_QXP_06:Desktop Folder:

67. Hypothesis: it is hot


Conclusion: we will go swimming
68. Hypothesis: you do your chores
Conclusion: you get an allowance
69. Hypothesis: 3n  7  17
Conclusion: n  8
70. Hypothesis: a b and b c
Conclusion: a c
71.
72.
a  15  20
r  9  12
a  15  15  20  15
r  9  9  12  9
a5
r  21
73.
74.
4  5n  6
3  8w  35
4  6  5n  6  6
3  8w  3  35  3
10  5n
8w  32

60. To rotate the quadrilateral 90 counterclockwise


about the origin, switch the coordinates of each
vertex and then multiply the new first coordinate
by 1.
(x, y) S (y, x)
Q(2, 2) S Q(2, 2)
R(3, 3) S R(3, 3)
S(1, 4) S S(4, 1)
T(4, 3) S T(3, 4)
y

R
Q

S x

61.

10
5

R
T

g
4

75.
g
4

6 ? 18
 45
15
?

11 ? 33
 34
12
?

11(34)  12(33)
374 396
The cross products are not equal, so
ratios do not form a proportion.

11
12

33
.
34

The

8(55)  22(20)
440  440
20

The cross products are equal, so 22  55. Since the


ratios are equal, they form a proportion.
64.

6 ? 3
4
8
?

6(4)  8(3)
24  24
6

The cross products are equal, so 8  4. Since the


ratios are equal, they form a proportion.
65.

3 ? 9
 25
5
?

3(25)  5(9)
75 45
The cross products are not equal, so
ratios do not form a proportion.
66.

3
5

25. The

26 ? 12
 15
35
?

26(15)  35(12)
390 420
The cross products are not equal, so
ratios do not form a proportion.

Chapter 4

26
35

76.

3

1 2  4(3)

3
m
5
5
m  3
5
m  3
5

2
2

2  5(2)

m  3  10
m  3  3  10  3
m7

4-5

Graphing Linear Equations

Page 221

Check for Understanding

1. The former will be a graph of four points, and the


latter will be a graph of a line.
2a. Sample answer: y  8 is a linear equation in the
form Ax  By  C, where A  0, B  1, and C  8.
2b. Sample answer: x  5 is a linear equation in
the form Ax  By  C, where A  1, B  0, and
C  5.
2c. Sample answer: x  y  0 is a linear equation in
the form Ax  By  C, where A  1, B  1, and
C  0.
3. Determine the point at which the graph intersects
the x-axis by letting y  0 and solving for x.
Likewise, determine the point at which the graph
intersects the y-axis by letting x  0 and solving
for y. Draw a line through the two points.
4. Since the term y2 has an exponent of 2, the
equation cannot be written in the form Ax  By 
C. Therefore, this is not a linear equation.
5. First rewrite the equation so that the variables
are on one side of the equation and a constant is
on the other side.
3y  2  0
3y  2  2  0  2
3y  2
The equation can be written as 0x  3y  2.
Therefore, it is a linear equation in standard form
where A  0, B  3, and C  2.

8 ? 20
 55
22
?

g
4
g
4

g  12

18

The cross products are equal, so 15  45. Since the


ratios are equal, they form a proportion.

63.

25

32

 8
w  4

2252

6(45)  15(18)
270  270

62.

8w
8

5n

 5
2  n

12

15. The

148

PQ249-6481F-04[122-168] 31/7/02 11:01 PM Page 149 Sahuja Ahuja_QXP_06:Desktop Folder:

6. To write the equation with integer coefficients,


multiply each term by 5.
3
x
5

10. Select five values for the domain and make a table.

135 2x  5125 2y  5152

3x  2y  25
The equation is now in standard form where A  3,
B  2, and C  25. This is a linear equation.
1

y
1
0
1
2
4

y  2x  8

(x, y)
(3, 1)
(3, 0)
(3, 1)
(3, 2)
(3, 4)

11. To find the x-intercept, let y  0.


y  3  x
0  3  x
0  3  3  x  3
3  x
1(3)  1(x)
3  x
The graph intersects the x-axis at (3, 0).
To find the y-intercept, let x  0.
y  3  x
y  3  (0)
y  3
The graph intersects the y-axis at (0, 3).
Plot these points and draw the line that connects
them.

x3

9. Solve the equation for y.


xy0
xyx0x
y  x
11y2  11x2
yx
Select five values for the domain and make a table.
y
2
1
0
2
4

x
2
1
0
2
4

(x, y)
(4, 0)
(3, 2)
(1, 6)
(0, 8)
(1, 10)

Graph the ordered pairs and draw a line through


the points.

y
0
2
6
8
10

Graph the ordered pairs and draw a line through


the points.

7. Since the term y has a variable in the denominator,


the equation cannot be written in the form Ax  By
 C. Therefore, this is not a linear equation.
8. The only value in the domain is 3. Since there is no
y in the equation, the value of y does not depend on
the value on x. Therefore, y can be any real number.
Select five values for the range and make a table.
x
3
3
3
3
3

2x  8
2(4)  8
2(3)  8
2(1)  8
2(0)  8
2(1)  8

x
4
3
1
0
1

 5y  5

y  3  x

(x, y)
(2, 2)
(1, 1)
(0, 0)
(2, 2)
(4, 4)

Graph the ordered pairs and draw a line through


the points.
y
xy0

149

Chapter 4

PQ249-6481F-04[122-168] 31/7/02 11:01 PM Page 150 Sahuja Ahuja_QXP_06:Desktop Folder:

12. To find the x-intercept, let y  0.


x  4y  10
x  4102  10
x  10
The graph intersects the x-axis at (10, 0).
To find the y-intercept, let x  0.
x  4y  10
0  4y  10
4y  10
4y
4

14. Select five values for the domain and make a table.
0.75m  2.25
0.75(0)  2.25
0.75(4)  2.25
0.75(8)  2.25
0.75(12)  2.25
0.75(16)  2.25

m
0
4
8
12
16

18
16
14
12
Cost
($) 10
8
6
4
2

x  4y  10

c  0.75m  2.25

x
0

12
4

x3
The graph intersects the x-axis at (3, 0).
To find the y-intercept, let x  0.
4x  3y  12
4(0)  3y  12
3y  12
3y
3

Pages 221223

12
3

y
4x  3y  12

Chapter 4

Practice and Apply

16. First rewrite the equation so that the variables


are on the same side of the equation.
3x  5y
3x  5y  5y  5y
3x  5y  0
The equation is now in standard form where A  3,
B  5, and C  0. This is a linear equation.
17. First rewrite the equation so that the variables
are on the same side of the equation.
6  y  2x
6  y  y  2x  y
6  2x  y
2x  y  6
The equation is now in standard form where A  2,
B  1, and C  6. This is a linear equation.
18. Since the term 6xy has two variables, the equation
cannot be written in the form Ax  By  C.
Therefore, this is not a linear equation.

y4
The graph intersects the y-axis at (0, 4).
Plot these points and draw the line that connects
them.

2 4 6 8 10 12 14 16
Miles

15. Use the graph to estimate the y-coordinate in the


ordered pair that contains 18 as the x-coordinate.
The ordered pair (18, 16) appears to be on the
line, so an 18-mile taxi ride will cost about $16.
To check this solution algebraically, substitute
m  18 into the original equation:
c  0.75m  2.25
c  0.751182  2.25
c  15.75
An 18-mile taxi ride will cost $15.75.

13. To find the x-intercept, let y  0.


4x  3y  12
4x  3(0)  12
4x  12


(m, c)
(0, 2.25)
(4, 5.25)
(8, 8.25)
(12, 11.25)
(16, 14.25)

Graph the ordered pairs and draw a line through


the points.

10
4

y  2.5
The graph intersects the y-axis at (0, 2.5).
Plot these points and draw the line that connects
them.

4x
4

c
2.25
5.25
8.25
11.25
14.25

150

PQ249-6481F-04[122-168] 31/7/02 11:01 PM Page 151 Sahuja Ahuja_QXP_06:Desktop Folder:

19. First rewrite the equation so that the variables


are on one side of the equation and a constant is
on the other side.
y50
y5505
y  5
The equation can be written as 0x  y  5.
Therefore, it is a linear equation in standard form
where A  0, B  1, and C  5.
20. First simplify. Then rewrite the equation so that
the variables are on the same side of the equation.
7y  2x  5x
7y  7x
7y  7y  7x  7y
0  7x  7y
7x  7y  0
Since the GCF of 7, 7, and 0 is not 1, divide
each side by 7.
7(x  y)  0
7(x  y2
7

25. First rewrite the equation so that the variables


are on one side of the equation and a constant is
on the other side.
3a  b  2  b
3a  b  2  b  b  b
3a  2  0
3a  2  2  0  2
3a  2
The equation can be written as 3a  0b  2.
Therefore, it is a linear equation in standard form
where A  3, B  0, and C  2.
26. The only value in the range is 1. Since there is
no x in the equation, the value of x does not
depend on y. Therefore, x can be any real number.
Select five values for the domain and make a
table.
x
3
1
0
2
4

7

xy0
The equation is now in standard form where A  1,
B  1, and C  0. This is a linear equation.
21. Since the term 4x2 has an exponent of 2, the
equation cannot be written in the form Ax  By 
C. Therefore, this is not a linear equation.
3
4
22. Since each of the terms x and y has a variable in
the denominator, the equation cannot be written
in the form Ax  By  C. Therefore, this is not a
linear equation.
23. To write the equation with integer coefficients,
multiply each term by 6.

x
2
x
2

 10 

(x, y)
(3, 1)
(1, 1)
(0, 1)
(2, 1)
(4, 1)

y
1
1
1
1
1

Graph the ordered pairs and draw a line through


the points.
y

y  1

2y
3

1 2  6(10)  61 3 2
2y

27. Select five values for the domain and make a


table.

3x  60  4y
Then rewrite the equation so that the variables
are on the same side of the equation.
3x  4y  60  4y  4y
3x  4y  60
The equation is now in standard form the A  3,
B  4, and C  60. This is a linear equation.
24. First rewrite the equation so that the variables
are on the same side of the equation.
7n  8m  4  2m
7n  8m  2m  4  2m  2m
7n  6m  4
6m  7n  4
1(6m  7n)  1(4)
6m  7n  4
The equation is now in standard form where A  6,
B  7, and C  4. This is a linear equation.

2x
2(2)
2(1)
2(0)
2(1)
2(2)

x
2
1
0
1
2

y
4
2
0
2
4

(x, y)
(2, 4)
(1, 2)
(0, 0)
(1, 2)
(2, 4)

Graph the ordered pairs and draw a line through


the points.
y
y  2x
O

151

Chapter 4

PQ249-6481F-04[122-168] 31/7/02 11:01 PM Page 152 Sahuja Ahuja_QXP_06:Desktop Folder:

Graph the ordered pairs and draw a line through


the points.

28. Select five values for the domain and make a


table.
5x
5  (1)
50
51
53
55

x
1
0
1
3
5

y
6
5
4
2
0

(x, y)
(1, 6)
(0, 5)
(1, 4)
(3, 2)
(5, 0)

y  4  3x
x

Graph the ordered pairs and draw a line through


the points.
y

31. Select five values for the domain and make a table.

2x  8
2(1)  8
2(0)  8
2(2)  8
2(4)  8
2(5)  8

y
10
8
4
0
2

yx6

32. Solve the equation for y.


x  3y

1
1
(x)  3 (3) y
3
1
xy
3

y  2x  8

Select five values for the domain and make a table.


1
x
3

30. Select five values for the domain and make a


table.
x
1
0
1
2
3

4  3x
4  3(1)
4  3(0)
4  3(1)
4  3(2)
4  3(3)

y
7
4
1
2
5

(x, y)
(1, 7)
(0, 4)
(1, 1)
(2, 2)
(3, 5)

(x, y)
(3, 1)

3

1
(3)
3

1

1

1
(1)
3

1
3

1
(0)
3

1
(1)
3

1
3

1
(3)
3

11, 13 2

(0, 0)

11, 13 2

(3, 1)

Graph the ordered pairs and draw a line through


the points.
y

x  3y
O

Chapter 4

(x, y)
(1, 7)
(0, 6)
(2, 4)
(4, 2)
(6, 0)

(x, y)
(1, 10)
(0, 8)
(2, 4)
(4, 0)
(5, 2)

Graph the ordered pairs and draw a line through


the points.

y
7
6
4
2
0

Graph the ordered pairs and draw a line through


the points.

29. Select five values for the domain and make a


table.
x
1
0
2
4
5

x6
1  6
06
26
46
66

x
1
0
2
4
6

y5x

152

PQ249-6481F-04[122-168] 31/7/02 11:01 PM Page 153 Sahuja Ahuja_QXP_06:Desktop Folder:

35. Solve the equation for y.


x  3y  9
x  3y  x  9  x
3y  9  x

33. Solve the equation for y.


x  4y  6
x  6  4y  6  6
x  6  4y
x  6
4
x  6
4

3y
3

4y
4

y

y

Select five values for the domain and make a table.


x  6
4

6

6  6
4

4

4  6
4

2

2  6
4

0  6
4

1
12

1  6
4

14

Select five values for the domain and make a table.

(x, y)

1
2

14, 2
1
2

(2, 1)

9  x
3

(x, y)

3

9  (3)
3

(3, 4)

2

9  (2)
3

2
33

9  0
3

9  1
3

23

9  3
3

(6, 0)

9  x
3
9  x
3

10, 2
11, 134 2
1
12

Graph the ordered pairs and draw a line through


the points.

12, 323 2

(0, 3)

11, 223 2

(3, 2)

Graph the ordered pairs and draw a line through


the points.

x  4y  6
x  3y  9
O

34. Solve the equation for y.


x  y  3
x  y  x  3  x
y  3  x
1(y)  1(3  x)
y3x
Select five values for the domain and make a
table.
3x
3  (3)
3  (1)
30
31
32

x
3
1
0
1
2

y
0
2
3
4
5

36. To find the x-intercept, let y  0.


4x  6y  8
4x  6(0)  8
4x  8
4x
4

4

x2
The graph intersects the x-axis at (2, 0).
To find the y-intercept, let x  0.
4x  6y  8
4(0)  6y  8
6y  8

(x, y)
(3, 0)
(1, 2)
(0, 3)
(1, 4)
(2, 5)

6y
6

6
4

y  3 or 13

Graph the ordered pairs and draw a line through


the points.

The graph intersects the y-axis at 0, 13 .


Plot these points and draw the line that connects
them.

x  y  3

4x  6y  8
O

x
O

153

Chapter 4

PQ249-6481F-04[122-168] 31/7/02 11:01 PM Page 154 Sahuja Ahuja_QXP_06:Desktop Folder:

37. To find the x-intercept, let y  0.


3x  2y  15
3x  2(0)  15
3x  15
3x
3

39. Solve the equation for y.


2.5x  5y  75
2.5x  5y  2.5x  75  2.5x
5y  75  2.5x
5y
5

15
3

x5
The graph intersects the x-axis at (5, 0).
To find the y-intercept, let x  0.
3x  2y  15
3(0)  2y  15
2y  15
2y
2

15

3x  2y  15

40. To find the x-intercept, let y  0.

38. To find the x-intercept, let y  0.


1.5x  y  4
1.5x  (0)  4
1.5x  4

1
2

1
2

xy4

x  102  4

 1.5

1
2
1
2

x4

1 2 x  2(4)

x8
The graph intersects the x-axis at (8, 0).
To find the y-intercept, let x  0.

The graph intersects the x-axis at 23, 0 .


To find the y-intercept, let x  0.
1.5x  y  4
1.5(0)  y  4
y4
The graph intersects the y-axis at (0, 4).
Plot these points and draw the line that connects
them.

1
2

xy4

1
(0)
2

y4

y4
The graph intersects the y-axis at (0, 4).
Plot these points and draw the line that connects
them.

y
1.5x  y  4
O

Chapter 4

(x, y)
(2, 16)
(0, 15)
(2, 14)
(6, 12)
(10, 10)

y
16
14
12
10
8 2.5x  5y  75
6
4
2
O
42
2 4 6 8 10 12x

y
16
15
14
12
10

Graph the ordered pairs and draw a line through


the points.

x  23

15  0.5x
15  0.5(2)
15  0.5(0)
15  0.5(2)
15  0.5(6)
15  0.5(10)

x
2
0
2
6
10

y  7.5
The graph intersects the y-axis at (0, 7.5).
Plot these points and draw the line that connects
them.

1.5x
1.5

75  2.5x
5

y  15  0.5x
Select five values for the domain and make a table.

 2

154

1
xy4
2

PQ249-6481F-04[122-168] 31/7/02 11:01 PM Page 155 Sahuja Ahuja_QXP_06:Desktop Folder:

41. To find the x-intercept, let y  0.

Plot these points and draw the line that connects


them.

2
3

x y1
2

x  3 (0)  1

x1
The graph intersects the x-axis at (1, 0).
To find the y-intercept, let x  0.

4x 3y
41
3

1

2

x3y1

2x

1

(0)  3 y  1
2

3 y  1

1 22

43. Solve the equation for y.

2 3 y  2 (1)

1 2x 

3y
1
4
3(0)
1
4

10
3

(x, y)

10
3

7
312

1
(0)
4

10
3

33

1
(2)
4

10
3

26

1
(4)
4

10
3

23

1
(6)
4

10
3

16

1
0

1
5
1
5

11, 3127 2
10, 313 2
12, 256 2
14, 213 2
16, 156 2

3
(1)
4




0
01
1 
4
(1)
3
4
3
1
13

Graph the ordered pairs and draw a line through


the points.

x  0.75
The graph intersects the x-axis at (0.75, 0).
To find the y-intercept, let x  0.
4x
3
4(0)
3

10

1
(1)
4

1

3 4
4 3

1
x
4

42. To find the x-intercept, let y  0.




1
1

y  4x  3
Select five values for the domain and make a
table.

x  3y  1

4x
3
4x
3
4x
3

y  3  3  4x  3  3

y  3  4x  3

y  1.5
The graph intersects the y-axis at (0, 1.5).
Plot these points and draw the line that connects
them.

3y
1
4
3y
1
4
3y
1
4
3y
1
4
3y
4
4 3
y
3 4

y  3  4x  3

44. To find the x-intercept, let y  0.


4x  7y  14
4x  7(0)  14
4x  14

1

12

4x
4

x

y
y

14
4
7
2

The x-intercept is

7
2

or 32.

To find the y-intercept, let x  0.


4x  7y  14
4(0)  7y  14
7y  14

The graph intersects the y-axis at 0,  13 .

7y
7

14

 7

y  2.
The y-intercept is 2.

155

Chapter 4

PQ249-6481F-04[122-168] 31/7/02 11:01 PM Page 156 Sahuja Ahuja_QXP_06:Desktop Folder:

45. Since the x-intercept is 3, the graph intersects the


x-axis at (3, 0). Since the y-intercept is 5, the
graph intersects the y-axis at (0, 5). Both points,
(3, 0) and (0, 5), lie on the line, so must make
Ax  By  C true.
Substituting (3, 0) for (x, y), we have A(3)  B(0) 
C, or 3A  C.
Substituting (0, 5) for (x, y) we have A(0)  B(5) 
C, or 5B  C.
Since 3A  C and 5B  C, we can conclude that
3A  5B.
Since A and B must be integers with a GCF of 1,
A  5 and B  3. This means C  15.
Therefore, the equation of the line with an
x-intercept of 3 and a y-intercept of 5 in standard
form is 5x  3y  15.
46.
2/  2w  P
2(2x)  2( y)  30
4x  2y  30
Since the GCF of 4, 2, and 30 is not 1, the
equation is not written in standard form. Divide
each side by the GCF.
2(2x  y)  30
2(2x  y)
2

49.

Distance (miles)

30
2

3
0.21

15
2

d  0.21t
t
2 4 6 8 10 12 14
Time (seconds)

0.21t
0.21

14.29  t
So, it will take about 14 s to hear the thunder
from a storm 3 mi away.
52. Select five values for w and make a table.
0.07w
0.07(100)
0.07(140)
0.07(220)
0.07(260)
0.07(300)

w
100
140
220
260
300

y
7
9.8
15.4
18.2
21

(w, y)
(100, 7)
(140, 9.8)
(220, 15.4)
(260, 18.2)
(300, 21)

Graph the ordered pairs in the table and draw a


line through the points.
24

Pints of Blood

20

16
12
8
0

Chapter 4

51. Use the graph to estimate the value of the


x-coordinate in the ordered pair that contains
3 as the y-coordinate. The ordered pair (14, 3)
appears to be on the line. To check this solution
algebraically, substitute d  3 into the original
equation:
d  0.21t
3  0.21t

5
4
3
2
1
0

x  7.5
The x-intercept of the graph is 7.5.
To find the y-intercept, let x  0.
2x  y  15
2(0)  y  15
y  15
The y-intercept of the graph is 15.
48. Since the x-intercept is 7.5, the graph crosses the
x-axis at (7.5, 0). Since the y-intercept is 15, the
graph crosses the y-axis at (0, 15). Plot these
points and draw the line that connects them.
18
16
14
12
10
8
6
4
2

(t, d)
(0, 0)
(2, 0.42)
(4, 0.84)
(6, 1.26)
(8, 1.68)
(10, 2.1)
(12, 2.52)
(14, 2.94)
(16, 3.36)

d
0
0.42
0.84
1.26
1.68
2.1
2.52
2.94
3.36

50. Graph the ordered pairs in the table and draw a


line through the points.

2x  y  15
47. To find the x-intercept, let y  0.
2x  y  15
2x  (0)  15
2x  15
2x
2

0.21t
0.21(0)
0.21(2)
0.21(4)
0.21(6)
0.21(8)
0.21(10)
0.21(12)
0.21(14)
0.21(16)

t
0
2
4
6
8
10
12
14
16

156

w
100 140 180 220 260 300 340
Weight (lb)

PQ249-6481F-04[122-168] 31/7/02 11:01 PM Page 157 Sahuja Ahuja_QXP_06:Desktop Folder:

58. You can graph an equation that represents how


many Calories and nutrients your diet should
contain. Since your diet is different every day, it is
easier to use the graph to determine your goal
instead of making calculations every day. Answers
should include the following.
Nutrition information labels provide facts
about how many grams of fat are in each
serving and/or how many Calories are from fat.
The number of grams of protein would equal
10% of the total number of Calories divided by
4 Calories per gram or p  0.025C.
59. A;
x
3x  5
y
(x, y)

53. Use the graph to estimate the value of the


x-coordinate in the ordered pair that contains 12 as
the y-coordinate. The ordered pair (170, 12) appears
to be on the line. To check this solution algebraically,
substitute y  12 into the original equation:
y  0.07w
12  0.07w
12
0.07

0.07w
0.07

171.43  w
So, a person weighing about 171 lb will have
12 pt of blood.
54. Select five values for d and make a table.
0.43d  14.7
0.43(0)  14.7
0.43(20)  14.7
0.43(60)  14.7
0.43(80)  14.7
0.43(100)  14.7

d
0
20
60
80
100

p
14.7
23.3
40.5
49.1
57.7

(d, p)
(0, 14.7)
(20, 23.3)
(60, 40.5)
(80, 49.1)
(100, 57.7)

2

(1, 2)

60. B; of the given points, only (2, 2) appears to lie on


the line.

Page 223
61.

Graph the ordered pairs in the table and draw a


line through the points.
p
100
80

x
3
1
2
5
8

Maintain Your Skills


x5
3  5
1  5
25
55
85

y
8
6
3
0
3

(x, y)
(3, 8)
(1, 6)
(2, 3)
(5, 0)
(8, 3)

The solution set is


{(3, 8), (1, 6), (2, 3), (5, 0), (8, 3)}.

60
40

62.

20
O

3(1)  5

20

40

60

80

100

55. Substitute d  400 into the equation:


p  0.43d  14.7
p  0.43(4002  14.7
p  186.7
At a depth of 400 ft, the pressure is 186.7 psi.
56. The pressure at sea level is 14.7 psi. The pressure
186.7
at 400 ft, 186.7 psi, is 14.7  12.7 times as great.
57. Solve the equation for y in terms of x.
2x  y  8
2x  y  2x  8  2x
y  8  2x
1(y)  1(8  2x)
y  2x  8
Substitute the value for x into the expression
2x  8. If the value of y is less than the value of
2x  8, then the point lies below the line. If the
value of y is greater than the value of 2x  8,
then the point lies above the line. If the value of
y is equal to the value of x, then the point lies on
the line. Sample answers:
(1, 5) Since 5 7 2(1)  8, the point (1, 5) lies
above the line.
(5, 1) Since 1 6 2(5)  8, the point (5, 1) lies
below the line.
(6, 4) Since 4  2(6)  8, the point (6, 4) lies on
the line.

x
3
1
2
5
8

2x  1
2(3)  1
2(1)  1
2(2)  1
2(5)  1
2(8)  1

y
5
1
5
11
17

(x, y)
(3, 5)
(1, 1)
(2, 5)
(5, 11)
(8, 17)

The solution set is


{(3, 5), (1, 1), (2, 5), (5, 11), (8, 17)}.
63. First solve the equation for y in terms of x.
3x  y  12
3x  y  3x  12  3x
y  12  3x
x
3
1
2
5
8

12  3x
12  3(3)
12  3(1)
12  3(2)
12  3(5)
12  3(8)

y
21
15
6
3
12

(x, y)
(3, 21)
(1, 15)
(2, 6)
(5, 3)
(8, 12)

The solution set is


{(3, 21), (1, 15), (2, 6), (5, 3), (8, 12)}.

157

Chapter 4

PQ249-6481F-04[122-168] 31/7/02 11:01 PM Page 158 Sahuja Ahuja_QXP_06:Desktop Folder:

64. First solve the equation for y in terms of x.


2 x  y  3
2 x  y  2x  3  2x
y  3  2x
1(y2  113  2x2
y  3  2x
3  2x

x
3
1
2
5
8

3
3
3
3
3







2(3)
2(1)
2(2)
2(5)
2(8)

y
3
1
7
13
19

67.

3x 
3x 

1
y
2

(3, 3)
(1, 1)
(2, 7)
(5, 13)
(8, 19)

3
4
3

5
1
2
1

The domain of this relation is {4, 3, 3}.


The range is {1, 1, 2, 5}.
68.

6

 3x  6  3x
1

1 2y  2(6  3x)

x
4
2
1
4

1
2

6x  12
6(3)  12
6(1)  12
6(2)  12
6(5)  12
6(8)  12

y
30
18
0
18
36

(x, y)
(3, 30)
(1, 18)
(2, 0)
(5, 18)
(8, 36)
69.

2x  3 y  4
1

2x  3 y  2x  4  2x

x
3
1
2
5
8

12

 4  2x
 3(4  2x)

y  12  6x

12  6x
12  6(3)
12  6(1)
12  6(2)
12  6(5)
12  6(8)

y
6
6
24
42
60

(x, y)
(3, 6)
(1, 6)
(2, 24)
(5, 42)
(8, 60)

4
2
1

0
3
4

x
1
3
1
3

y
4
0
1
5

1
3
1

4
0
1
5

The domain of the relation is {1, 1, 3}. The


range is {1, 0, 4, 5}.

The solution set is


{(3, 6), (1, 6), (2, 24), (5, 42), (8, 60)}.

Chapter 4

The domain of the relation is {1, 2, 4}. The


range is {3, 0, 4}.

The solution set is


{(3, 30), (1, 18), (2, 0), (5, 18), (8, 36)}.
66. First solve the equation for y in terms of x.

1
y
3
1
y
3

y
0
3
3
4

y  6x  12

x
3
1
2
5
8

2 y  6  3x

2

y
5
1
2
1

(x, y)

The solution set is


{(3, 3), (1, 1), (2, 7), (5, 13), (8, 19)}.
65. First solve the equation for y in terms of x.
1
y
2

x
3
4
3
3

158

PQ249-6481F-04[122-168] 31/7/02 11:01 PM Page 159 Sahuja Ahuja_QXP_06:Desktop Folder:

70.

x
4
2
4
3

74.

y
5
5
1
2
O

4
2
3

5
1
2

6(x  32  3x
6x  18  3x
6x  18  3x  3x  3x
3x  18  0
3x  18  18  0  18
3x  18
3x
3

6[ (6)  3]  3(6)
?

6(3)  18
18  18
75. The lowest value is 1, and the highest value is 40,
so use a scale that includes these values. Place an
X above each value for each occurrence.

 
    

      

 

The domain of the relation is {2, 3, 4}. The range


is {1, 2, 5}.
2(x  22  3x  (4x  52
2x  4  3x  4x  5
2x  4  x  5
2x  4  x  x  5  x
3x  4  5
3x  4  4  5  4
3x  9
3x
3

Check:

x3
2(x  2)  3x  (4x  5)
?

2(3  2)  3(3)  [4(3)  5]


?

2(1)  9  (12  5)
?

2  9  (7)
22
72.
3a  8  2a  4
3a  8  2a  2a  4  2a
a  8  4
a  8  8  4  8
a  12
Check:
3a  8  2a  4
?
3(12)  8  2(12)  4
?

36  8  24  4
28  28
73.
3n  12  5n  20
3n  12  3n  5n  20  3n
12  2n  20
12  20  2n  20  20
8  2n

Check:

8 10 12 14 16 18 20 22 24

40

76. Looking at the line plot, we can see that there are
12 Xs above the numbers between 7 and 16,
inclusive. So, 12 animals have average life spans
between 7 and 16 years, inclusive.
77. Since there are more Xs above 15 than above any
other number, we can easily see that an average
life span of 15 years occurs most frequently.
78. Since there are 4 Xs above numbers 20 and
greater, there are 4 animals with average life
spans of at least 20 years.
79. 19  5  4  19  20
 39
80. (25  4)  (22  13)  (25  4)  (4  1)
 21  3
7
81. 12  4  15  3  3  45
 48
82. 12(19  15)  3  8  12(4)  3  8
 48  24
 24
83. 6(43  22)  6(64  4)
 6(68)
 408
84. 7[43  2(4  3)]  7  2  7[43  2(4  3 )]  7  2

3

8
2

18
3

x  6
6(x  3)  3x

Check:

71.








2n
2

4n
3n  12  5n  20
?
3(4)  12  5(4)  20
?

12  12  20  20
00

159

7[64 2(7)]  7  2
7(64  14)  7  2
7(50)  7  2
350  7  2
50  2
52

Chapter 4

PQ249-6481F-04[122-168] 14/8/02 03:24 PM Page 160 Sahuja Ahuja_QXP_13:InProcess_AQ13:Sunita_14/8/02:

5. First solve the equation for y.


x  y  2
x  y  x  2  x
y  2  x

Pages 224225
Graphing Calculator
Investigation (Follow-Up of Lesson 4-5)
1.

KEYSTROKES:

CLEAR

X,T,,n

ZOOM 6

[10, 10] scl: 1 by [10, 10] scl: 1


[10, 10] scl: 1 by [10, 10] scl: 1

2.

KEYSTROKES:

CLEAR 2

KEYSTROKES:

CLEAR 4 X,T,,n

X,T,,n

ZOOM 6

ZOOM 6

6. First solve the equation for y.


x  4y  8
x  4y  x  8  x
4y  8  x
4y
4

y

8  x
4
x  8
4

[10, 10] scl: 1 by [10, 10] scl: 1

3.

KEYSTROKES:

CLEAR 6

5 X,T,,n

ZOOM 6
[10, 10] scl: 1 by [10, 10] scl: 1
KEYSTROKES:

8
7.

[10, 10] scl: 1 by [10, 10] scl: 1

CLEAR

X,T,,n

4 ZOOM 6

KEYSTROKES:

CLEAR 5 X,T,,n

ZOOM 6

4. First solve the equation for y.


2x  y  6
2x  y  2x  6  2x
y  6  2x

Since the origin and the x- and y-intercepts of the


graph are displayed, the graph is complete.

[10, 10] scl: 1 by [10, 10] scl: 1


[10, 10] scl: 1 by [10, 10] scl: 1
KEYSTROKES:

CLEAR 6

8.

KEYSTROKES:

CLEAR 10 X,T,,n

2 X,T,,n

Since the origin and the x- and y-intercepts of the


graph are displayed, the graph is complete.

ZOOM 6

[10, 10] scl: 1 by [10, 10] scl: 1

Chapter 4

ZOOM 6

160

PQ249-6481F-04[122-168] 31/7/02 11:02 PM Page 161 Sahuja Ahuja_QXP_06:Desktop Folder:

9.

KEYSTROKES:

CLEAR 3 X,T,,n

11. First solve the equation for y.


4x  2y  21
4x  2y  4x  21  4x
2y  21  4x

18

ZOOM 6
Since the y-intercept is outside of the standard
viewing window, the graph is not complete. Find
the y-intercept.
y  3x  18
y  3(0)  18
y  18

2y
2

CLEAR 10.5

KEYSTROKES:

2 X,T,,n

ZOOM 6

[10, 10] scl: 1 by [10, 10] scl: 1

One possible viewing window that will show a


complete graph is [2, 10] scl: 1 by [20, 6] scl: 2.

Since the y-intercept is outside of the standard


viewing window, the graph is not complete. Find
the y-intercept.
4x  2y  21
4102  2y  21
2y  21
2y
2

21
2

y  10.5
One possible viewing window that will show a
complete graph is [5, 10] scl: 1 by [5, 15] scl: 1.

[2, 10] scl: 1 by [20, 6] scl: 2

10. First solve the equation for y.


3x  y  12
3x  y  3x  12  3x
y  12  3x
11y2  112  3x2
y  3x  12

CLEAR 3 X,T,,n

21  4x
2

y  10.5  2x

[10, 10] scl: 1 by [10, 10] scl: 1

KEYSTROKES:

12

ZOOM 6

[5, 10] scl: 1 by [5, 15] scl: 1

[10, 10] scl: 1 by [10, 10] scl: 1

Since the y-intercept is outside of the standard


viewing window, the graph is not complete. Find
the y-intercept.
3x  y  12
3102  y  12
y  12
11y2  11122
y  12
One possible viewing window that will show a
complete graph is [2, 8] scl: 1 by [15, 5] scl: 1.

[2, 8] scl: 1 by [15, 5] scl: 1

161

Chapter 4

PQ249-6481F-04[122-168] 31/7/02 11:02 PM Page 162 Sahuja Ahuja_QXP_06:Desktop Folder:

4. The mapping represents a function since, for each


element of the domain, there is only one
corresponding element in the range. It does not
matter if two elements of the domain, 2 and 4,
are paired with the same element in the range.
5. The table represents a relation that is not a
function. The element 2 in the domain is paired
with both 1 and 4 in the range. If you are given
that x is 2, you cannot determine the value of y.
6. Since an element of the domain is paired with
more than one element in the range, the relation
is not a function. If you are given that x is 24, you
cannot determine the value of y since both 1 and
5 in the range are paired with x  24.
7. Graph the equation using the x- and y-intercepts.
For each value of x, a vertical line passes through
no more than one point on the graph. Thus, the
line represents a function.

12. First solve the equation for y.


3x  5y  45
3x  5y  3x  45  3x
5y  45  3x


45  3x
5

y

45  3x
5

5y
5

CLEAR

KEYSTROKES:

X,T,,n

45

5 ZOOM 6

[10, 10] scl: 1 by [10, 10] scl: 1

Since the x-intercept is outside of the standard


viewing window, the graph is not complete. Find
the x-intercept.
3x  5y  45
3x  5102  45
3x  45
3x
3

x  15
One possible viewing window that will show
a complete graph is [20, 4] scl: 2 by [10, 5]
scl: 1.

8. The vertical line x  1 intersects the graph at two


points, (1, 2) and (1, 1). Thus, the relation
graphed does not represent a function.
9. For each value of x, a vertical line passes through
no more than one point on the graph. Thus, the
relation graphed represents a function.
10. f(x)  4x  5
11. g(x)  x 2  1
f(2)  4(2)  5
g(1)  (1) 2  1
85
11
3
2

[20, 4] scl: 2 by [10, 5] scl: 1

13. See students work.


14. The complete graph is in the standard viewing
window if 10 b 10.
15. b is the y-intercept of the graph.

4-6

 45

12. f(x)  4x  5
f(c)  4(c)  5
 4c  5

g(x)  x 2  1

g(t)  4  [ (t) 2  1]  4
 t2  1  4
 t2  3
14. f(x)  4x  5
15.
f(x)  4x  5
f(3a2 )  4(3a2 )  5
f(x  5)  4(x  5)  5
 12a2  5
 4x  20  5
 4x  15
16. D;

Functions

Pages 228229

13.

Check for Understanding

1. y is not a function of x since 3 in the domain is


paired with 2 and 3 in the range. x is not a
function of y since 3 in the domain of the
inverse is paired with 4 and 3 in the range.
2. Sample answer: # x #  x  1
3. x  c, where c is any constant is a linear equation
that is not a function, since the value of c in the
domain is paired with every real number.

x**  2x  1
5**  2**  [2(5)  1]  [ 2(2)  1]
 (10  1)  (4  1)
93
6

Pages 229231

Practice and Apply

17. The mapping does not represent a function since


the element 3 in the domain is paired with two
elements in the range.

Chapter 4

162

PQ249-6481F-04[122-168] 31/7/02 11:02 PM Page 163 Sahuja Ahuja_QXP_06:Desktop Folder:

27. Graph the equation. For each value of x, a vertical


line passes through no more than one point on the
graph. Thus, the equation represents a function.

18. The mapping represents a function since, for each


element in the domain, there is only one
corresponding element in the range.
19. The table represents a relation that is a function
since each element in the domain is paired with
exactly one element in the range.
20. The table represents a relation that is not a
function. The element 3 in the domain is paired
with both 6 and 2 in the range.
21. For each value of x, a vertical line passes through
no more than one point on the graph. Thus, the
relation graphed represents a function.
22. For each value of x, a vertical line passes through
no more than one point on the graph. Thus, the
relation graphed represents a function.
23. Since each element of the domain is paired with
exactly one element of the range, the relation is a
function.
24. Since an element of the domain is paired with
more than one element in the range, the relation
is not a function. Both 5 and 7 in the range are
paired with x  4.
25. Graph the equation. For each value of x, a vertical
line passes through no more than one point on the
graph. Thus, the equation represents a function.
16
12
8
4

28. Graph the equation. For each value of x, a vertical


line passes through no more than one point on the
graph. Thus, the equation represents a function.
y

29. The line x  1 intersects the graph at (1, 1), (1, 2),
(1, 1.5), and many other points. Thus, the relation
graphed does not represent a function.
30. The vertical line x  2 intersects the graph at
two points, (2, 2) and (2, 2). Thus, the
relation graphed does not represent a function.
31. For each value of x, a vertical line passes through
no more than one point on the graph. Thus, the
relation graphed represents a function.
32. f(x)  3x  7
33. f(x)  3x  7
f(3)  3(3)  7
f(2)  3(2)  7
97
 6  7
 16
1

16 12 8 4 O 4 8 12 16 x
4
8
12
16

y  8
26. Graph the equation. The vertical line x  15
intersects the graph at more than one point.
Thus, the equation does not represent a function.
20
15
10
5

34. g(x)  x2  2x
g(5)  (5) 2  2(5)
 25  10
 15

35. g(x)  x2  2x
g(0)  (0) 2  2(0)
00
0

g(x)  x2  2x
g(3)  1  [ (3) 2  2(3) ]  1
 [ 9  (6) ]  1
 15  1
 16
37.
f(x)  3x  7
f(8)  5  [ 3(8)  7]  5
 (24  7)  5
 31  5
 26
36.

20 1510 5 O 5 10 15 20 x
5
10
15
20

x  15

38. g(x)  x2  2x
g(2c)  (2c) 2  2(2c)
 4c2  4c
40.
f(x)  3x  7
f(K  2)  3(K  2)  7
 3K  6  7
 3K  13

163

39. f(x)  3x  7
f(a2 )  3(a2 )  7
 3a2  7

Chapter 4

PQ249-6481F-04[122-168] 31/7/02 11:02 PM Page 164 Sahuja Ahuja_QXP_06:Desktop Folder:

f(x)  3x  7
f(2m  5)  3(2m  5)  7
 6m  15  7
 6m  8

42.

g(x)  x2  2x
3[g(x)  4]  3[ (x2  2x)  4]
 3(x2  2x  4)
 3x2  6x  12
f(x)  3x  7
2[f(x2 )  5]  2[ (3x2  7)  5]
 2(3x2  2)
 6x2  4
a; The cost is the same for 01 hour, excluding 0.
Then it jumps after 1 hour and remains the same
up to 2 hours. So the line is constant for x values
between 0 and 1, then it jumps at 1. This trend
continues at each hourly interval until 5 hours;
then the cost is constant. This trend is best
represented by graph a.
Since, for each value of x, a vertical line passes
through no more than one point on the graph, the
relation represents a function.
Since the equation t  77  0.005h is solved for t, h
is the independent variable and t is the dependent
variable. This means h represents the elements of
the domain. Therefore, in function notation, the
corresponding elements in the range are represented
by f(h). Thus, the original equation can be written in
function notation as f(h)  77  0.005h.
f(100)  77  0.005(100)
 77  0.5
 76.5
f(200)  77  0.005(200)
 77  1
 76
f(1000)  77  0.005(1000)
 77  5
 72
Using the function values determined in Exercise
46, make a table.

43.

44.

45.

46.

47.

h
100
200
1000
2000
5000

f(h)
76.5
76
72
67
52

48. It appears that the ordered pair (4000, 55)


represents a point that lies on the line. To check
this solution algebraically, substitute h  4000
into the function:
f(4000)  77  0.005(4000)
 77  20
 57
The temperature at 4000 ft is 57F.
49. Select five values for s and make a table.

Temperature (F)

Chapter 4

f(s)
72
152
232
312
392

(s, f(s))
(0, 72)
(100, 152)
(200, 232)
(300, 312)
(400, 392)

Graph the ordered pairs and draw the line that


connects them.
450
400
350
300
250
200
150
100
50

f (s )

f (s)  0.8s  72

100

200 300 400 s

Science Scores

50. It appears that (290, 308) is a point that lies on


the line. To check this solution algebraically,
substitute f(s)  308 into the function:
f(s)  0.8s  72
308  0.8s  72
308  72  0.8s  72  72
236  0.8s
236
0.8

0.8s
0.8

295  s
A science score of 295 corresponds to a math score
of 308.
51. Kristas math score is above the average because
the point at (260, 320) lies above the graph of the
line for f(s).
52. The set of ordered pairs {(1, 2), (3, 4), (5, 6)}
represents a function since each element of the
domain is paired with exactly one element of the
range. The inverse of this function, {(2, 1), (4, 3),
(6, 5)}, is also a function.
However, {(1, 2), (3, 2), (5, 6)} is a function, while
its inverse {(2, 1), (2, 3), (6, 5)} is not a function
since the element 2 in the domain is paired with
both 1 and 3 in the range.
It is sometimes true that the inverse of a function
is also a function.

(h, f(h))
(100, 76.5)
(200, 76)
(1000, 72)
(2000, 67)
(5000, 52)

Graph the ordered pairs and draw the line that


connects them.
90
80
70
60
50
40
30
20
10

0.8s  72
0.8(0)  72
0.8(100)  72
0.8(200)  72
0.8(300)  72
0.8(400)  72

s
0
100
200
300
400

Math Scores

41.

t
t  770.005h

1000 2000 3000 4000 h


Height (ft)

164

PQ249-6481F-04[122-168] 31/7/02 11:02 PM Page 165 Sahuja Ahuja_QXP_06:Desktop Folder:

Graph the ordered pairs and draw a line through


the points.

53. Functions can be used in meteorology to


determine if there is a relationship between
certain weather conditions. This can help to
predict future weather patterns. Answers should
include the following.
As barometric pressure decreases,
temperature increases. As barometric pressure
increases, temperature decreases.
The relation is not a function since there is more
than one temperature for a given barometric
pressure. However, there is still a pattern in the
data and the two variables are related.
54. A; f(x)  20  2x
f(7)  20  2(7)
 20  14
6
55. A;
I. If f(x)  2x, then f(3x)  2(3x)  6x and
3[f(x) ]  3(2x)  6x. So it is true that
f(3x)  3[f(x) ].
II. If f(x)  2x, then
f(x  3)  2(x  3)  2x  6 and
f(x)  3  2x  3. So it is not true that
f(x  3)  f(x)  3.

y  2x  4

58. To find the x-intercept, let y  0.


2x  5y  10
2x  5(0)  10
2x  10
2x
2

10
2

x5
The graph intersects the x-axis at (5, 0).
To find the y-intercept, let x  0.
2x  5y  10
2(0)  5y  10
5y  10
5y
5

III. If f(x)  2x, then f(x2 )  2(x2 )  2x2 and


[ f(x) ] 2  (2x) 2  4x2. So it is not true that
f(x2 )  [f(x) ] 2.
Thus, only statement I is true.

10
5

y2
The graph intersects the y-axis at (0, 2).
Plot these points and draw the line that connects
them.
y

Page 231

Maintain Your Skills

56. Select five values for the domain and make a table.
x3
3  3
1  3
03
13
23

x
3
1
0
1
2

y
0
2
3
4
5

2x  5y  10

(x, y)
(3, 0)
(1, 2)
(0, 3)
(1, 4)
(2, 5)

59.

Graph the ordered pairs and draw a line through


the points.
y
yx3

x
3

y
12

2

2

7

1

8

y  5x  3
12  5(3)  3
12  12
2  5(1)  3
2  2
7  5(2)  3
7  13
8  5(1)  3
8  8

True or False?
true
false
false
true

The solution set is {(3, 12), (1, 8)}.

57. Select five values for the domain and make a table.
x
0
1
2
3
4

2x  4
2(0)  4
2(1)  4
2(2)  4
2(3)  4
2(4)  4

y
4
2
0
2
4

(x, y)
(0, 4)
(1, 2)
(2, 0)
(3, 2)
(4, 4)

165

Chapter 4

PQ249-6481F-04[122-168] 31/7/02 11:02 PM Page 166 Sahuja Ahuja_QXP_06:Desktop Folder:

60.

x
3

y
0

1

1

y  2x  6
0  2(3)  6
0  12
4  2(1)  6
44
0  2(6)  6
0  18
1  2(5)  6
1  16

Page 231

True or False?

1.
false
true
false
false

2.

26
t

6.2(t)  0.75(26)
6.2t  19.5
6.2t
6.2

3
.
5

65. 12  16  12  (16)
 ( 016 0  012 0 )
 (16  12)
 4
66. 5  (8)  5  [(8) ]
 5  8
 ( 08 0  05 0 )
 (8  5)
3
67. 16  (4)  16  4
 20
68. 9  6  9  (6)
 ( 09 0  06 0 )
 (9  6)
 15
69.

3
4

888
5

8

y
5
1
4
10
16

(x, y)
(3, 5)
(1, 1)
(0, 4)
(2, 10)
(4, 16)

8  x
2

(x, y)

3

8  (3)

5.5

(3, 5.5)

1

8  (1)
2

4.5

(1, 4.5)

8  0
2

(0, 4)

8  2
2

(2, 3)

8  4
2

(4, 2)

The solution set is


{(3, 5.5), (1, 4.5), (0, 4), (2, 3), (4, 2)}.
4. Select five values for the domain and make a table.
x2
2  2
02
12
22
42

x
2
0
1
2
4
2

 36  16


y
4
2
1
0
2

(x, y)
(2, 4)
(0, 2)
(1, 1)
(2, 0)
(4, 2)

Graph the ordered pairs and draw a line through


the points.

70. 32  (13 )  32  13


7
46
1
56

x
O

yx2

Chapter 4

(x, y)
(3, 2)
(1, 4)
(0, 5)
(2, 7)
(4, 9)

8  x
2
8  x
2

Multiplicative Identity Property




y

n  1, since

3x  4
3(3)  4
3(1)  4
3(0)  4
3(2)  4
3(4)  4

x
3
1
0
2
4

2y
2

64. 5n  5
3
(1)
5

y
2
4
5
7
9

The solution set is


{(3, 5), (1, 1), (0, 4), (2, 10), (4, 16)}.
3. First solve the equation for y in terms of x.
x  2y  8
x  2y  x  8  x
2y  8  x

19.5
6.2

t  3.15
It will take Adam about 3.15 h, or approximately
3 h 9 min, to finish the marathon.
62. 16  n  16
Additive Identity Property
n  0, since 16  0  16.
63. 3.5  6  n  6
Reflexive Property of Equality
n  3.5, since 3.5  6  3.5  6.
3

x5
3  5
1  5
05
25
45

x
3
1
0
2
4

The solution set is


{(3, 2), (1, 4), (0, 5), (2, 7), (4, 9)}.

The solution set is {(1, 4)}.


61. Let t represent the number of hours it will take
Adam to finish the 26-mile marathon.
(45 min  0.75 h)
6.2
0.75

Practice Quiz 2

166

PQ249-6481F-04[122-168] 31/7/02 11:02 PM Page 167 Sahuja Ahuja_QXP_06:Desktop Folder:

5. To find the x-intercept, let y  0.


3x  2y  6
3x  2(0)  6
3x  6
3x
3

3. a2  a1  4 is a formula for a2 in terms of a1.


A formula for a3 in terms of a1 is a  a  4 
3
1
4 or a3  a1  2(4).
A formula for a4 in terms of a1 is a4  a1  4 
4  4 or a4  a1  3(4).
4. Each term can be found by adding the first term
a1 to the product of 4 and 1 less than the position
n of the term in the sequence.
an  a1  (n  1)4

3

x2
The graph intersects the x-axis at (2, 0).
To find the y-intercept, let x  0.
3x  2y  6
3(0)  2y  6
2y  6
2y
2

5. The 21st term in the sequence is a21. Substitute


n  21 and a1  7 into the equation from Exercise 4.
a  a  (n  1)4
n
1

2

a
21
a21
a21
a21

y3
The graph intersects the y-axis at (0, 3).
Plot these points and draw the line that connects
them.

 7  (21  1)4
 7  (20)4
 7  80
 87

The 21st term in the sequence is 87.

y
3x  2y  6

4-7
O

Page 236

Check for Understanding

1. Sample answer: If the first term is 2 and the


common difference is 10, add 10 to each term
of the sequence to get the next term of the
sequence.
2
8
18
28

6. Since each element of the domain is paired with


exactly one element of the range, this relation is a
function.
7. Since an element of the domain is paired with
more than one element in the range, the relation
is not a function. Both 5 and 1 in the range are
paired with x  2.
8. f(x)  3x  5
9. f(x)  3x  5
f(4)  3(4)  5
f(2a)  3(2a)  5
 12  5
 6a  5
 7
10.
f(x)  3x  5
f(x  2)  3(x  2)  5
 3x  6  5
 3x  11

Page 232

Arithmetic Sequences



10
10


10

Thus, one possible arithmetic sequence with a


common difference of 10 is 2, 8, 10, 28 .
2. If an  5n  2, then:
a1  5(1)  2
a2  5(2)  2
52
 10  2
7
 12
a3  5(3)  2
a4  5(4)  2
 15  2
 20  2
 17
 22
The first term in the sequence, a1, is 7.
7
12
17
22




5
5
5

The common difference, d, is 5.


3. Marisela is correct. To find the common
difference, subtract the first term from the second
term.
4. 24
16
8
0

Spreadsheet Investigation
(Preview of Lesson 4-7)

1. Step 1: Enter the initial value 7 in cell A1.


Step 2: Highlight the cells in column A. Under
the Edit menu, choose the Fill option
and then Series.
Step 3: Enter 4 as the Step value and 63 as the
Stop value, and click OK.
The spreadsheet column A is filled with the
numbers in the sequence:
7, 11, 15, 19, 23, 27, 31, 35, 39, 43, 47, 51, 55,
59, 63.
2. The last number in the sequence is in cell A15, so
there are 15 numbers in the sequence.




8
8
8

This is an arithmetic sequence because the


difference between terms is constant. The
common difference is 8.
5. 3
6
12
24




3
6
12

This is not an arithmetic sequence because the


difference between terms is not constant.

167

Chapter 4

PQ249-6481F-04[122-168] 31/7/02 11:02 PM Page 168 Sahuja Ahuja_QXP_06:Desktop Folder:

6. 7

14

21

28

12. 6





7
7
7
7
7
7

12

The common difference is 7. Add 7 to the last


term of the sequence and continue adding 7 until
the next three terms are found.
28
35
42
49

24

The first term is 6. The common difference is 6.


Use the formula for the nth term to write an
equation with a1  6 and d  6.
an  a1  (n  1)d
an  6  (n  1)6
an  6  6n  6
an  6n




7
7
7

The next three terms are 35, 42, 49.


7. 34
29
24
19
?
?
?






5
5
5
5
5
5

The common difference is 5. Add 5 to the last


term of the sequence and continue adding 5
until the next three terms are found.
19
14
9
4




5
5
5

The next three terms are 14, 9, 4.


8. Use the formula for the nth term of an arithmetic
sequence with a1  3, d  4, and n  8.
an  a1  (n  1)d
a8  3  (8  1)4
a8  3  28
a  31
8

6n

an

(n, an)

1
2
3
4
5

6(1)
6(2)
6(3)
6(4)
6(5)

6
12
18
24
30

(1, 6)
(2, 12)
(3, 18)
(4, 24)
(5, 30)

32
28
24
20
16
12
8
4

The 8th term of the arithmetic sequence is 31.


9. Use the formula for the nth term of an arithmetic
sequence with a1  10, d  5, and n  21.
a  a  (n  1)d
n
1
a21  10  (21  1) (5)
a21  10  (100)
a21  90

an

21

13. 12

1 2 3 4 5 6n

17

22

27




5
5
5

The first term is 12. The common difference is 5.


Use the formula for the nth term to write an
equation with a1  12 and d  5.
an  a1  (n  1)d
a  12  (n  1)5
n
an  12  5n  5
an  5n  7

The 21st term of the arithmetic sequence is 90.


10. 23
25
27
29




2
2
2

The first term is 23. The common difference is 2.


Use the formula for the nth term of an arithmetic
sequence with a1  23, d  2, and n  12.
an  a1  (n  1)d
a12  23  (12  1)2
a  23  22
12
a12  45

5n  7
5(1)  7
5(2)  7
5(3)  7
5(4)  7
5(5)  7

n
1
2
3
4
5

The 12th term of the arithmetic sequence is 45.


11. 27
19
11 3




8
8
8

32
28
24
20
16
12
8
4

The first term is 27. The common difference is 8.


Use the formula for the nth term of an arithmetic
sequence with a1  27, d  8, and n  17.
an  a1  (n  1)d
a17  27  (17  1)8
a17  27  128
a17  101

21

The 17th term of the arithmetic sequence is 101.

Chapter 4

18




6
6
6

168

an

O
1 2 3 4 5 6n

an
12
17
22
27
32

(n,
(1,
(2,
(3,
(4,
(5,

an)
12)
17)
22)
27)
32)

23. 66

14. For the first week, Latisha walks 20 min/day, so


a1  20. She must increase each week by
7 min/day, so the common difference, d, is 7.
We are to find the first value of n for which an is
more than 1 h, or 60 min.
20
27
34
41
48
55
62

Practice and Apply


4

78




4
4
4

a1
a2
a3
a4
a5
a6 a7
Since a7  62 is the first time an is more than 60,
Latisha will begin walking over an hour a day
during the seventh week of her exercise program.

15. 7

74

The common difference is 4. Add 4 to the last


term of the sequence and continue adding 4
until the next three terms are found.
78 82 86 90







7
7
7
7
7
7

Pages 236238

70






4
4
4
4 4 4

The next three terms are 82, 86, 90.


24. 31
22 13
4
? ?
?






9
9
9
9 9 9

The common difference is 9. Add 9 to the last


term of the sequence and continue adding 9 until
the next three terms are found.
4 5 14
23


9 9
9


1
1
1

The next three terms are 5, 14, 23.

This is an arithmetic sequence because the


difference between terms is constant. The
common difference is 1.
16. 10 12 15 18

25. 23

23

33


1
1
1
1
1
1
3




2
3
3

3

3

3

3

3

1
.
3

The common difference is Add 3 to the last term


1
of the sequence and continue adding 3 until the
next three terms are found.

This is not an arithmetic sequence because the


difference between terms is not constant.
17. 9 5 1 5

33



4
6
4

33

43


1
1
1
3

This is not an arithmetic sequence because the


difference between terms is not constant.
18. 15 11 7 3

3

3

The next three terms are 33, 4, 43.


26.



4
4
4

7
12

212

26


3
3
3
3
3
3
4

This is an arithmetic sequence because the


difference between terms is constant. The
common difference is 4.
19. 0.3 0.2 0.7 1.2

13

4

4

4

4

4

The common difference is 4. Add 4 to the last term


3
of the sequence and continue adding 4 until the
next three terms are found.



0.5 0.5 0.5

26

312

43

512




3
3
3

This is an arithmetic sequence because the


difference between terms is constant. The
common difference is 0.5.
20. 2.1 4.2 8.4 17.6

4

4

4

The next three terms are 312, 43, 512.


2.1 4.2 9.2

27. Use the formula for the nth term of an arithmetic


sequence with a1  5, d  5, and n  25.
an  a1  (n  1)d
a25  5  (25  1)5
a25  5  120
a25  125

This is not an arithmetic sequence because the


difference between terms is not constant.
21. 4 7
10 13
?
?
?



3
3
3 3
3
3

The common difference is 3. Add 3 to the last


term of the sequence and continue adding 3 until
the next three terms are found.
13 16 19 22

The 25th term of the arithmetic sequence is 125.


28. Use the formula for the nth term of an arithmetic
sequence with a1  8, d  3, and n  16.
an  a1  (n  1)d
a16  8  (16  1)3
a16  8  45
a16  53
The 16th term of the arithmetic sequence is 53.



3
3
3

The next three terms are 16, 19, 22.


22. 18
24 30 36 ?
?
?





6
6
6 6
6
6

The common difference is 6. Add 6 to the last


term of the sequence and continue adding 6 until
the next three terms are found.
36 42 48 54



6
6
6

The next three terms are 42, 48, 54.

169

Chapter 4

35. 0.5

29. Use the formula for the nth term of an arithmetic


sequence with a1  52, d  12, and n  102.
an  a1  (n  1)d
a102  52  (102  1)12
a102  52  1212
a102  1264
The 102nd term of the arithmetic sequence is
1264.
30. Use the formula for the nth term of an arithmetic
sequence with a1  34, d  15, and n  200.
an  a1  (n  1)d
a200  34  (200  1)15
a200  34  2985
a200  3019
The 200th term of the arithmetic sequence is
3019.
31. Use the formula for the nth term of an arithmetic
5
1
sequence with a  8, d  8, and n 22.

The first term is 5.3. The common difference is


0.6. Use the formula for the nth term of an
arithmetic sequence with a1  5.3, d  0.6, and
n  12.
an  a1  (n  1)d
a12  5.3  (12  1)(0.6)
a12  5.3  6.6
a12  11.9
The 12th term of the arithmetic sequence is 11.9.
37. 24
35
46
57

a22 
a22 

12

21
8



11 11
11

13
4
1
34

The first term is 24. The common difference is 11.


Use the formula for the nth term of an arithmetic
sequence with a1  24 and d  11 to find n when
an  200.
an  a1  (n  1)d
200  24  (n  1)11
200  24  11n  11
200  11n  13
200  13  11n  13  13
187  11n

The 22nd term of the arithmetic sequence is 34.


32. Use the formula for the nth term of an arithmetic
1
1
sequence with a  12, d  24, and n  39.
1

an  a1  (n  1)d
1
1
a39  12  (39  1) 24
a

39

1
12

1
852

1 2

a39  87

187
11

The 39th term of the arithmetic sequence is 87.


33. 9 7 5 3

11n
11

17  n
200 is the 17th term of the arithmetic sequence.
38. 30
22
14
6



2
2 2

The first term is 9. The common difference is 2.


Use the formula for the nth term of an arithmetic
sequence with a1  9, d  2, and n  18.
an  a1  (n  1)d
a18  9  (18  1)2
a18  9  34
a18  25
The 18th term of the arithmetic sequence is 25.
34. 7 3 1 5




8
8
8

The first term is 30. The common difference is 8.


Use the formula for the nth term of an arithmetic
sequence with a1  30 and d  8 to find n when
an  34.
an  a1  (n  1)d
34  30  (n  1)(8)
34  30  8n  8
34  38  8n
34  38  38  8n  38
72  8n


4
4 4

The first term is 7. The common difference is 4.


Use the formula for the nth term of an arithmetic
sequence with a1  7, d  4, and n  35.
an  a1  (n  1)d
a35  7  (35  1)4
a35  7  136
a35  129
The 35th term of the arithmetic sequence is 129.

Chapter 4


 
0.6 0.6 0.6

1.5

The first term is 0.5. The common difference is 0.5.


Use the formula for the nth term of an arithmetic
sequence with a1  0.5, d  0.5, and n  50.
an  a1  (n  1)d
a50  0.5  (50  1)(0.5)
a50  0.5  24.5
a50  25
The 50th term of the arithmetic sequence is 25.
36. 5.3
5.9
6.5
7.1

an  a1  (n  1)d
5
1
a22  8  (22  1) 8
a22  8 



0.5 0.5 0.5

72
8

8n
8

9n
34 is the 9th term of the arithmetic sequence.

170

39. 3 6 9 12

41. 2

The first term is 3. The common difference is


3. Use the formula for the nth term to write an
equation with a1  3 and d  3.
an  a1  (n  1)d
an  3  (n  1)(3)
an  3  3n  3
an  3n
n

3n

an

1
2
3
4
5

3(1)
3(2)
3(3)
3(4)
3(5)

3
6
9
12
15

2
2

2
4
6
8
10
12
14

40. 8

10

20

The first term is 2. The common difference is 6.


Use the formula for the nth term to write an
equation with a1  2 and d  6.
an  a1  (n  1)d
an  2  (n  1)6
an  2  6n  6
an  6n  4

(n, an)
(1, 3)
(2, 6)
(3, 9)
(4, 12)
(5, 15)

6n  4

an

(n, an)

1
2
3
4
5

6(1)  4
6(2)  4
6(3)  4
6(4)  4
6(5)  4

2
8
14
20
26

(1, 2)
(2, 8)
(3, 14)
(4, 20)
(5, 26)

an
O

14


6 6
6



3
3 3

26
24
22
20
18
16
14
12
10
8
6
4
2

6n

11


1 1
1

The first term is 8. The common difference is 1.


Use the formula for the nth term to write an
equation with a1  8 and d  1.

2

n7

an

(n, an)

1
2
3
4
5

17
27
37
47
57

8
9
10
11
12

(1, 8)
(2, 9)
(3, 10)
(4, 11)
(5, 12)

42. 18

16

6n

14

12




2
2
2

an  a1  (n  1)d
an  8  (n  1)1
an  8  n  1
an  n  7
n

an

The first term is 18. The common difference is 2.


Use the formula for the nth term to write an
equation with a1  18 and d  2.
an  a1  (n  1)d
an  18  (n  1)2
an  18  2n  2
an  2n  20

an

2n  20

an

1
2
3
4
5

2(1)  20
2(2)  20
2(3)  20
2(4)  20
2(5)  20

18
16
14
12
10

(n, an)
(1,
(2,
(3,
(4,
(5,

18)
16)
14)
12)
10)

an
2

6n

4
6
8
10
12
14
16
18

171

Chapter 4

43. y  4, 6, y, . . . is an arithmetic sequence only if


the difference between the second and first terms
is the same as the difference between the third
and second terms.
a2  a1  a3  a2
6  ( y  4)  y  6
6y4y6
2yy6
2yyy6y
2  2y  6
2  6  2y  6  6
8  2y
8
2

47. The number of seats in each row form an


arithmetic sequence. The last three terms in the
sequence are 60, 68, 76. There are seven rows, so
there are seven terms in the sequence. This
means a5  60, a6  68, and a7  76.
60
68
76



8
8

The common difference is 8. Use the formula for


the nth term to write an equation with d  8,
n  7, and an  76 to find a1.
an  a1  (n  1)d
76  a1  (7  1)8
76  a1  48
76  48  a1  48  48
28  a1
Use the formula for the nth term to write an
equation with a1  28 and d  8.
an  a1  (n  1)d
an  28  (n  1)8
an  28  8n  8
an  8n  20
The formula to find the number of seats in any
given row is an  8n  20.
48. Use the formula from Exercise 47 with n  1.
an  8n  20
a1  8(1)  20
a1  8  20
a1  28
There are 28 seats in the first row.
49.
n
8n  20
a

2y
2

4y
The sequence is arithmetic if y  4.
44. y  8, 4y  6, 3y, . . . is an arithmetic sequence
only if the difference between the second and first
terms is the same as the difference between the
third and second terms.
a2  a1  a3  a2
(4y  6)  ( y  8)  3y  (4y  6)
4y  6  y  8  3y  4y  6
3y  2  y  6
3y  2  y  y  6  y
4y  2  6
4y  2  2  6  2
4y  4
4y
4

4
4

y  1
The sequence is arithmetic if y  1.
45. 5 8 11 14

1
2
3
4
5
6
7


3 3
3

The first term is 5. The common difference is 3.


Use the formula for the nth term to write an
equation with a1  5 and d  3.
Pn  a1  (n  1)d
Pn  5  (n  1)3
Pn  5  3n  3
Pn  3n  2
The formula for the perimeter of a pattern
containing n trapezoids is Pn  3n  2.
46. Use the formula from Exercise 45 with n  12.
an  3n  2
a12  3(12)  2
a12  36  2
a12  38
The perimeter of the pattern containing
12 trapezoids is 38 units.

Chapter 4

8(1)  20
8(2)  20
8(3)  20
8(4)  20
8(5)  20
8(6)  20
8(7)  20

28
36
44
52
60
68
76

The total number of seats in the section is:


a1  a2  a3  a4  a5  a6  a7
 28  36  44  52  60  68  76
 364
If 368 tickets were sold for the orchestra section
having 364 seats, the section was oversold by
4 seats.
50. 9
13
17
21
25
29






4
4
4
4
4

The distances form an arithmetic sequence


because the difference between terms is constant.
The common difference is 4.
51. The first term is 9. The common difference is 4.
Use the formula for the nth term of an arithmetic
sequence with a1  9 and d  4.
an  a1  (n  1)d
an  9  (n  1)4
an  9  4n  4
an  4n  5

172

52. Use the formula from Exercise 51 with n  35.


an  4n  5
a35  4(35)  5
a35  140  5
a35  145
The ball will travel 145 cm during the 35th s.
53.
an

57. Between 29 and 344, the least multiple of 7 is 35


and the greatest multiple of 7 is 343. The common
difference between multiples of 7 is 7. Use the
formula for the nth term of an arithmetic
sequence with a1  35, an  343, and d  7 to
find the value of n.
an  a1  (n  1)d
343  35  (n  1)7
343  35  7n  7
343  7n  28
343  28  7n  28  28
315  7n

Distance Traveled (cm)

26
24
22
20
18
16
14
12
10
8
6
4
2
O

315
7

2
4
Time (s)

1500n  1000

1
2
3
4
5
6
7
8
9
10

1500(1)  1000
1500(2)  1000
1500(3)  1000
1500(4)  1000
1500(5)  1000
1500(6)  1000
1500(7)  1000
1500(8)  1000
1500(9)  1000
1500(10)  1000

an
2500
4000
5500
7000
8500
10,000
11,500
13,000
14,500
16,000

9
3


2x

3d
3

3d
Use the formula for the nth term with a1  2,
d  3, n  20.
an  a1  (n  1)d
a20  2  (20  1)3
a20  2  (19)3
a20  2  57
a20  59

The total won for ten correct answers would be:


a1  a2  a3  a4  a5  a6  a7  a8  a9  a10
 2500  4000  5500  7000  8500 
10,000  11,500  13,000  14,500  16,000
 92,500.
The contestant would win $92,500 for ten correct
answers.
56. 2x  5 4x  5 6x  5 8x  5


2x

7n
7

45  n
There are 45 multiples of 7 between 29 and 344.
58. By finding a pattern in a sequence of numbers,
scientists can predict results of large numbers
that they are not able to observe. Answers should
include the following.
The formula at  8.2t  1.9 represents the
altitude at of the probe after t seconds.
Replace t with 15 in the equation for at to find
that the altitude of the probe after 15 seconds
is 121.1 feet.
59. C;
Total amount saved  amount in savings now 
(weekly deposit)(number of weeks)
 350  (25)(12)
 350  300
 650
60. B;
Use the formula for the nth term of an arithmetic
sequence with a1  2, a4  11, and n  4 to find d.
an  a1  (n  1)d
11  2  (4  1)d
11  2  3d
11  2  2  3d  2
9  3d

54. Use the formula for the nth term to write an


equation with a1  2500 and d  1500. Then
find a10.
an  a1  (n  1)d
an  2500  (n  1)1500
an  2500  1500n  1500
an  1500n  1000
a10  1500(10)  1000
a10  15,000  1000
a10  16,000
The value of the 10th question is $16,000.
55.

Page 238

Maintain Your Skills

61. f(x)  3x  2
f(4)  3(4)  2
 12  2
 10


2x

63.

The expressions form an arithmetic sequence


because the difference between terms is constant.
The common difference is 2x.

173

62. g(x)  x2  5
 (3)2  5
95
4

f(x)  3x  2
2[f(6)]  2[3(6)  2]
 2(18  2)
 2(16)
 32
Chapter 4

3. Inductive reasoning; you are observing specific


pairs of terms and discovering a common
difference, and you conclude that the common
difference applies to the sequence in general.
4. Deductive reasoning; you are using the general
formula for the nth term and applying it to a
particular term of a particular series.
5a.
37
38
39
31 32 33 34 35 36
3
9 27 81 243 729 2187 6561 19,683

64. Since the term x2 has an exponent of 2, the


equation cannot be written in the form Ax  By 
C. Therefore, this is not a linear equation.
65. First rewrite the equation so that the variables
are on one side of the equation and a constant is
on the other side.
y  8  10  x
y  8  x  10  x  x
x  y  8  10
x  y  8  8  10  8
x  y  18
The equation is now in standard form where A  1,
B  1, and C  18. This is a linear equation.
66. First rewrite the equation so that the variables
are on the same side of the equation.
2y  y  2x  3
2y  y  y  2x  3  y
y  2x  3
y  2x  2x  3  2x
2x  y  3
1( 2x  y)  1(3)
2x  y  3
The equation is now in standard form where A  2,
B  1, and C  3. This is a linear equation.

5b. The ones digits in the numbers in the second


row of the table are: 3, 9, 7, 1, 3, 9, 7, 1, 3.
5c. The pattern repeats every fourth digit.
Therefore, digits 4, 8, 12, 16, and so on, will all
be the same. According to the pattern, all
powers in the first row of the table with
exponents divisible by 4 have 1 in the ones
place. Since the exponent of 3100 is 100, which is
divisible by 4, the value of 3100 will have a 1 in
the ones place.
Since the answer was obtained by observing a
pattern in order to make a conjecture, the answer
was reached by inductive reasoning.
6. Since the conclusion was based on a given rule, it
was reached by deductive reasoning.

hundred
times
x is
equal to nine.
67. Two
144244
3 minus
123 three
14424
43
14243
123


200

3x

The equation is 200  3x  9.


68. Rewrite the sentence so it is easier to translate.

4-8

Twice
plus
times
s is
identical
to thirteen.
123r 1
23 three
14424
43
14
42443
14243
2r

3s

13

Page 241

73.

75.
76.
77.
78.
79.
80.

1 21 2 
4
7

Point
H
J
K
L
M
N

Page 239

20
56

or

5
14

12

74. 5  32  5  2


x-Coordinate
2
3
4
3
3
5

y-Coordinate
2
0
2
4
5
1

35
2

or 172

Ordered
Pair
(2, 2)
(3, 0)
(4, 2)
(3, 4)
(3, 5)
(5, 1)

Page 243

Check for Understanding

1. Once you recognize a pattern, you can find a


general rule that can be written as an algebraic
expression.
2. Sample answer: 4, 8, 16, 32, 64, . . . ; each
successive term doubles.
3. Test the values of the domain in the equation. If
the resulting values match the range, the
equation is correct.

Reading Mathematics

1. Sample answer: Inductive reasoning uses


examples or past experience to make conclusions;
deductive reasoning uses rules to make
conclusions. Looking at a pattern of numbers to
decided the next number is an example of
inductive reasoning. Using the formula and the
length and width of a rectangle to find the area of
a rectangle is an example of deductive reasoning.
2. Deductive reasoning; he is applying a general rule
about mens heights to a specific case.

Chapter 4

Algebra Activity

When the string makes 1 loop around the


scissors, you end up with 3 pieces as a result
of the cut.
When the string makes 2 loops around the
scissors, you end up with 4 pieces as a result
of the cut.
1. The number of pieces is 2 more than the number
of loops; 3, 4, 5, 6, 7, .
2. n  2
3. n  2  20  2
 22

The equation is 2r  3s  13.


69. 7(3)  21
70. 11  15  165
2
12
71. 8(1.5)  12
72. 6 3  3 or 4
5
8

Writing Equations from Patterns

174

4. The pattern consists of squares with one corner of


each shaded. The corner that is shaded is rotated
in a clockwise direction. The next two figures in
the pattern are shown.

If x  3, then y  2(3) or 6. But


the y value for x  3 is 5. This is a
difference of 1. Try other values in the
domain to see if the same difference
occurs.

Check:

2
4
3


1 2
3
4

The difference between each pair of terms


increases by 1 for each successive pair. Continue
increasing each successive difference by 1. Add
5, 6, and 7.
1 2
4
7 11 16
22
29

9.

Temperature (C)

The next three terms are 16, 22, and 29.


6. 5
9
6
10
7
11



4 3
4
3
4

200
150
100
50

The difference between terms alternates between


4 and 3. Continue the alternating pattern. Add
3, 4, and 3.
5
9
6
10
7
11
8
12
9


4 3
4 3 4
3
4
3

x
y

1

1

1

1

x
1
35x 35
y
55

2
70
90

3
4
5
6
105 140 175 210
125 160 195 230

y is always
20 more
than 35x.

For each pair, the difference in the y values is the


same as the difference in the x values. This
suggests that y  x.
Check:
If x  4, then y  4. If x  1, then
y  1. For all values in the domain,
y is always equal to x.
Thus, y  x or f(x)  x describes this relation.
8. Make a table of ordered pairs for several points
on the graph.

x
y

1

The difference of the values for x is 1, and the


difference of the values for y is 35. This suggests
that y  35x.
Check:
If x  1, then y  35(1) or 35. But the y
value for x  1 is 55. This is a difference
of 20. Try other values in the domain to
see if the same difference occurs.





1
4
2
1

2


 
35 35 35 35 35

1

   
4
3
1
3
4
4
3
1
3
4

1

4
6
Depth (km)

   
10. Depth (km)
1
2
3
4
5
6
Temperature (C) 55 90 125 160 195 230

The next three terms are 8, 12, and 9.


7. Make a table of ordered pairs for several points
on the graph.
2

y is always 1
more than 2x.

250




1 2
3
4
5
6
7

4

1
2
3

Check y  2x  1.
If x  2, then y  2(2)  1 or 3.
If x  0, then y  2(0)  1 or 1.
If x  1, then y  2(1)  1 or 3.
Thus, y  2x  1 or f(x)  2x  1 describes this
relation.

The pattern repeats every fourth design.


Therefore, designs 4, 8, 12, 16, and so on, will all
be the same. So the 16th figure in the pattern will
be the same as the fourth square.
5. 1 2
4
7 11

1

0
0
1

3
6
5

x
2x
y

Check y  35x  20.


If x  2, then y  35(2)  20 or 90.
If x  5, then y  35(5)  20 or 195.
Thus, y  35x  20 or f(x)  35x  20 describes
this relation.
11. Use the function from Exercise 10 with x  10.
f(x)  35x  20
f(10)  35(10)  20
f(10)  350  20
f(10)  370
The temperature of a rock that is 10 km below
the surface is 370C.

1

  
3
2
0
1
5
3
1
3




2
4
2

When the difference of the x values is 1, the


difference of the y values is 2. When the
difference of the x values is 2, the difference of the
y values is 4. The difference in y values is twice
the difference of x values. This suggests that
y  2x.

175

Chapter 4

Pages 244245

16. 1

Practice and Apply

16



3
5
7

12. The pattern consists of triangles that are


alternately inverted (or rotated 180) with every
other pair shaded. The next two figures in the
pattern are shown.

The difference between each pair of terms


increases by 2 for each successive pair. Continue
increasing each successive difference by 2. Add
9, 11, and 13.
1
4
9
16
25
36
49






3
5
7
9
11
13

The next three terms are 25, 36, and 49.


17. 0
2
5
9
14
20





2
3
4
5
6

The pattern repeats every fourth design.


Therefore, designs 4, 8, 12, 16, and so on, will all
be the same. Since the 20th figure will be the
same as the fourth triangle, the 21st figure will
be the same as the first triangle.

The difference between each pair of terms


increases by 1 for each successive pair. Continue
increasing each successive difference by 1. Add
7, 8, and 9.
0
2
5
9
14
20
27
35
44






2
3 4
5
6
7
8
9

The next three terms are 27, 35, and 44.


18. a  1
a2
a3


1

13. The pattern consists of circles with one-eighth


shaded. The section that is shaded is the third
section in a clockwise direction from the
previously-shaded section. The next two figures in
the pattern are shown.


1

The difference between each pair of terms is


always 1. The sequence is arithmetic with a
common difference of 1. Each term is 1 more than
the term before it. Add 1, 1, and 1.
a1 a2 a3 a4 a5 a6


1


1


1


1


1

The next three terms are a  4, a  5, and a  6.


19. x  1
2x  1
3x  1


x

The pattern repeats every eighth design. Therefore,


designs 8, 16, 24, and so on, will all be the same.
Since 16 is the greatest number less than 21 that is
a multiple of 8, the 17th circle in the pattern will be
the same as the first circle.

The difference between each pair of terms is


always x. The sequence is arithmetic with a
common difference of x. Each term is x more than
the term before it. Add x, x, and x.
x  1 2x  1 3x  1 4x  1 5x  1 6x  1


x

17

14. 0

18

19

12

20


x


x


x


x

The next three terms are 4x  1, 5x  1, and


6x  1.
20. Make a table of ordered pairs for several points
on the graph.

21

20




2
4
6
8

The difference between each pair of terms


increases by 2 for each successive pair. Continue
increasing each successive difference by 2. Add
10, 12, and 14.
0
2
6
12
20
30
42
56

x
y








2
4
6
8
10
12
14

1

1

1

1

2

2

2

2





2
1
0
1
2
4
2
0
2
4





The difference of the x values is 1, and the


difference of the y values is 2. The difference in
y values is twice the difference of the opposite of
the x values. This suggests y  2x.
Check:
If x  2, then y  2(2) or 4.
If x  1, then y  2(1) or 2.
Thus, y  2x or f(x)  2x describes the
relation.

The next three terms are 30, 42, and 56.


15. 9
7
10
8
11
9
12







2
3
2
3
2
3

The difference between terms alternates between


2 and 3. Continue alternating the difference.
Add 2, 3, and 2.
9
7
10
8
11
9
12
10
13
11



2
3 2
3
2
3 2
3
2

The next three terms are 10, 13, and 11.

Chapter 4


x

176

Check y  x  6.
If x  2, then y  (2)  6 or 4.
If x  6, then y  (6)  6 or 0.
Thus, y  x  6 or f(x)  6  x describes this
relation.
24. Make a table of ordered pairs for several points
on the graph.

21. Make a table of ordered pairs for several points


on the graph.
2

1

2

1

When the difference of x values is 2, the


difference of the y values is 1. When the
difference of x values is 4, the difference of the
y values is 2. The difference in y values is one-half
1
the difference of x values. This suggests y  2 x.

x
y

If x  4, then y  2(4)  2.

Check:

Thus, y  2x or f(x)  2x describes this relation.


22. Make a table of ordered pairs for several points
on the graph.
x
y

2

3

1

2

3

1

  
4
2
1
2
2
0
3
4
  

2
0

1
3

2
4

x
y

2

2

2

2

2

4
4
2

6
6
0

3

6

y is always 6 more
3

than 2x.

If x  4, then y  2(4)  6 or 0.
3

Thus, y  2x  6 or f(x)  6  2x describes this


relation.
25. Make a table of ordered pairs for several points
on the graph.

y is always 2
more than x.

x
y

2

2

6

6


0
2 4
12 6 0


For each pair, the difference in the y values is


three times the opposite of the difference of the
x values. This suggests y  3x.
Check:
If x  2, then y  3(2) or 6. But the
y value for x  2 is 6. This is a difference
of 12. Try other values in the domain to
see if the same difference occurs.
x
3x
y

0
0
12

2
6
6

4
12
0

y is always 12
more than 3x.

Check y  3x  12.


If x  2, then y  3(2)  12 or 6.
If x  4, then y  3(4)  12 or 0.
Thus, y  3x  12 or f(x)  12  3x describes
this relation.

y is always 6
more than x.

2
2
4

3
2x

  
0
2
4
6
6
4
2
0
  

0
0
6

If x  2, then y  2(2)  6 or 3.

For each pair, the difference in the y values is the


opposite of the difference in x values. This
suggests y  x.
Check:
If x  2, then y  (2) or 2. But the
y value for x  2 is 4. This is a
difference of 6. Try other values in the
domain to see if the same difference
occurs.
x
x
y

If x  2, then y  2(2) or 3. But the


y value for x  2 is 3. This is a difference
of 6. Try other values in the domain to
see if the same difference occurs.

Check y  x  2.
If x  4, then y  (4)  2 or 2.
If x  1, then y  (1)  2 or 3.
Thus, y  x  2 or f(x)  x  2 describes this
relation.
23. Make a table of ordered pairs for several points
on the graph.
2

3

Check y  2x  6.

4
2

3

Check:

For each pair, the difference in y values is the


same as the difference in the x values. This
suggests y  x.
Check:
If x  4, then y  4. But the y value
for x  4 is 2. This is a difference of
2. Try other values in the domain to
see if the same difference occurs.
x
y

2

The difference of the x values is 2, and the


difference of the y values is 3. The difference in
3
the y values is 2 times the opposite of the
3
difference in the x values. This suggests y  2x.

If x  2, then y  2(2)  1.
1

2

 
0
2
4
6
3
0
 

4

x
y

2




4
2
2
4
2
1
1
2




177

Chapter 4

1

26. The pattern (3 red, 3 blue, 3 green) repeats every


ninth chain. Therefore, chains 9, 18, 27, and so
on, will all be the same. Since 45 is the greatest
number less than 50 that is a multiple of 9, the
46th chain will be the same as the first chain.
red
red
red
blue
blue
46
47
48
49
50
The 50th person will receive a blue flower chain.
27. 1
1
211
321
532
853
13  8  5
21  13  8
34  21  13
55  34  21
89  55  34
144  89  55
28. 3, 21, 144; every fourth term is divisible by 3.
5, 55; every fifth term is divisible by 5.
10

29.

a
p

10

10

10

10

9

9

9

9

9

32. n
f(n)

3

50
45
148

60
54
139

70
63
130

1
3
5

2
6
8

3
9
11

4
12
14

f(n) is always 2
more than 3n.

Check f(n)  3n  2.
If n  1, then f(1)  3(1)  2 or 5.
If n  4, then f(4)  3(4)  2 or 14.
Thus, f(n)  3n  2 represents this function.
33. Use the function from Exercise 32 with n  24.
f(n)  3n  2.
f(24)  3(24)  2
f(24)  72  2
f(24)  74
The perimeter of the arrangement if 24 pentagons
are used is 74 cm.
34. In scientific experiments you try to find a
relationship or develop a formula from observing
the results of your experiment. Answers should
include the following.
For every 11 cubic feet the volume of water
increases, the volume of ice increases 12 cubic
feet.
The container should have a volume of at least
108 cubic feet.
35. B;
3 4
6
9


1
2
3

p is always 193 more than 0.9a.


Check p  0.9a  193.
If a  30, then p  0.9 (30)  193 or 166.
If a  50, then p  0.9(50)  193 or 148.
Thus, p  0.9a  193 or f(a)  0.9a  193
describes this relation.
30. Use the function from Exercise 29 with a  10,
then with a  80.
f(a)  0.9a  193
f(a)  0.9a  193
f(10)  0.9(10)  193
f(80)  0.9(80)  193
f(10)  9  193
f(80)  72  193
f(10)  184
f(80)  121
A 10-year old should maintain a maximum heart
rate of 184 beats/min and an 80-year old should
maintain a maximum heart rate of 121 beats/min
in aerobic training.
31. Number of Pentagons
1 2 3 4

The difference between each pair of terms


increases by 1 for each successive pair. Continue
increasing each successive difference by 1. Add
4 and 5.
3 4
6
9
13
18



1
2
3 4
5

The next two terms are 13 and 18.


36. D;
Let n represent the number of pieces given to
each child. Then 5n represents the total number
of pieces given to 5 children. Since there were
4 pieces remaining, there were originally 5n  4
pieces. Thus, P  5n  4.
If you then had P  4 pieces to distribute and gave
n pieces to each of the 5 children, you would have 4
more pieces left than you had previously, or 8 pieces
remaining. However, that would mean you would
have enough pieces to give each child 1 additional
piece, leaving 3 pieces of candy remaining.

Perimeter of Arrangement (cm) 5 8 11 14

Chapter 4

3

n
3n
f(n)

40
36
157

3

For each pair, the difference in f(n) values is three


times the difference in the n values. This suggests
f(n)  3n.
Check:
If n  2, then f(n)  3(2) or 6. But the
f(n) value for n  2 is 8. This is a
difference of 2. Try other values in the
domain to see if the same difference
occurs.

The difference of the a values is 10, and the


difference of the p values is 9. The difference in
9
the p values is 10 of the opposite of the x values.
This suggests p  0.9a.
Check:
If a  20, then p  0.9(20) or 18.
But the y value for a  20 is 175. This
is a difference of 193. Try other values
in the domain to see if the same
difference occurs.
30
27
166

1

  


20
30
40
50
60
70
175
166
157
148
139
130






a
20
0.9a 18
p
175

1

  
1
2
3
4
5
8
11
14

178

Page 245
37. 1

7.
8.
9.
10.

Maintain Your Skills


4

10





3
3
3
3
3
3

The common difference is 3. Add 3 to the last


term of the sequence and continue adding 3 until
the next three terms are found.
10
13
16
19

Pages 246250




3
3
3

Lesson-by-Lesson Review

11. A(4, 2)
Start at the origin.
Move right 4 units and up 2 units.
Draw a dot and label it A.
(See coordinate plane after Exercise 16.)
12. B(1, 3)
Start at the origin.
Move left 1 unit and up 3 units.
Draw a dot and label it B.
(See coordinate plane after Exercise 16.)
13. C(0, 5)
Start at the origin.
Since the x-coordinate is 0, the point is on the
y-axis.
Move down 5 units.
Draw a dot and label it C.
(See coordinate plane after Exercise 16.)
14. D(3, 2)
Start at the origin.
Move left 3 units and down 2 units.
Draw a dot and label it D.
(See coordinate plane after Exercise 16.)
15. E(4, 0)
Start at the origin.
Move left 4 units.
Since the y-coordinate is 0, the point is on the
x-axis.
Draw a dot and label it E.
(See coordinate plane after Exercise 16.)
16. F(2, 1)
Start at the origin.
Move right 2 units and down 1 unit.
Draw a dot and label it F.
1116.
y

The next three terms are 13, 16, 19.


38. 9
5
1
3
?
?
?





4
4
4
4 4
4

The common difference is 4. Add 4 to the last


term of the sequence and continue adding 4
until the next three terms are found.
3 7 11 15




4
4
4

The next three terms are 7, 11, 15.


39. 25
19
13
7
?
?
?





6
6
6
6
6
6

The common difference is 6. Add 6 to the last


term of the sequence and continue adding 6 until
the next three terms are found.
7
1
5
11




6
6
6

The next three terms are 1, 5, 11.


40. 22
34
46
58
?
?
?





12
12
12
12 12 12

The common difference is 12. Add 12 to the last


term of the sequence and continue adding 12 until
the next three terms are found.
58
70
82
94




12
12
12

The next three terms are 70, 82, 94.


41. The vertical line x  2 intersects the graph at
two points, (2, 2) and (2, 3). Thus, the
relation graphed does not represent a function.
42. Let T  the height of Tulega Falls.
The height of
the height of
Angel
Falls
is 102
ft higher
than
Tulega
Falls.
14424
43 {
123 1
4424
43 14
424
43
3212
 102

3212  102  T
3212  102  102  T  102
3110  T
Tulega Falls is 3110 ft high.

c; linear function
a; domain
b; dilation
i; y-axis

B (1, 3)

A (4, 2)

E (4, 0)
x

Chapter 4 Study Guide and Review

F ( 2,1)
D (3, 2)

Page 246
1.
2.
3.
4.
5.
6.

C (0, 5)

Vocabulary and Concept Check

e; origin
g; relation
d; reflection
h; x-axis
k; y-coordinate
f; quadrants

179

Chapter 4

17. To reflect the triangle over the x-axis, multiply


the y-coordinate of each vertex by 1.
(x, y) S (x, y)
A(3, 3) S A(3, 3)
B(5, 4) S B(5, 4)
C(4, 3) S C(4, 3)
y

20. To rotate the trapezoid 90 counterclockwise about


the origin, switch the coordinates of each vertex
and then multiply the new first coordinate by 1.
(x, y) S (y, x)
M(2, 0) S M(0, 2)
N(4, 3) S N(3, 4)
O(6, 3) S O(3, 6)
P(8, 0) S P(0, 8)

P
O

A C

N
B

18. To translate the quadrilateral 3 units down, add


3 to the y-coordinate of each vertex.
(x, y) S (x, y  3)
P(2, 4) S P(2, 4  3) S P(2, 1)
Q(0, 6) S Q(0, 6  3) S Q(0, 3)
R(3, 3) S R(3, 3  3) S R(3, 0)
S(1, 4) S S(1, 4  3) S S(1, 7)

21.

Q y

x
2
3
3
4

y
6
2
0
6

2
3
4

2
0
6

P
R

Q
P

R
x

The domain of this relation is {2, 3, 4}. The


range is {2, 0, 6}.

S
22.

S
1

19. To dilate the parallelogram by a scale factor of 2,


1
multiply the coordinates of each vertex by 2.

112 x, 12 y2
1
1
G(2, 2) S G 1 2  2, 2  2 2 S G(1, 1)
1
1
H(6, 0) S H 1 2  6, 2  0 2 S H(3, 0)
1
1
I(6, 2) S I 1 2  6, 2  2 2 S I(3, 1)
1
1
J(2, 4) S J 1 2  2, 2  4 2 S J(1, 2)

x
1
3
6

y
0
0
2

(x, y) S

O
1
3
6

G
O

Chapter 4

G
H

The domain of this relation is {1, 3, 6}. The


range is {0, 2}.

H
x

180

23.

x
3
9
3
5

y
8
3
8
3

26.

12

3
3
5
9

2
3
4

5
1
2
3

x9
4  9
2  9
09
29
49

x
4
2
0
2
4

4

2

y
13
11
9
7
5

2
4
6
8
10
12
14

4x

4x  5
4(4)  5
4(2)  5
4(0)  5
4(2)  5
4(4)  5

x
4
2
0
2
4

y
11
3
5
13
21

(x, y)
(4, 11)
(2, 3)
(0, 5)
(2, 13)
(4, 21)

Graph the solution set


{(4, 11), (2, 3), (0, 5), (2, 13), (4, 21)}.

(x, y)
(4, 13)
(2, 11)
(0, 9)
(2, 7)
(4, 5)

21
18
15
12
9
6
3
4321

y
O

27. First solve the equation for y in terms of x.


4x  y  5
4x  y 4x  5  4x
y  5  4x
1(y)  1(5  4x)
y  4x  5

Graph the solution set


{(4, 13), (2, 11), (0, 9), (2, 7), (4, 5)}.
2

2

4

y
5
1
2
3
O

25.

4
4

x
2
3
4
2

(x, y)
(4, 12)
(2, 8)
(0, 4)
(2, 0)
(4, 4)

The domain of this relation is {3, 3, 5, 9}. The


range is {3, 8}.
24.

y
12
8
4
0
4

Graph the solution set


{(4, 12), (2, 8), (0, 4), (2, 0), (4, 4)}.

4  2x
4  2(4)
4  2(2)
4  2(0)
4  2(2)
4  2(4)

x
4
2
0
2
4

O
1 2 3 4x

6
9
12

4x

181

Chapter 4

28. First solve the equation for y in terms of x.


2x  y  8
2x  y  2x  8  2x
y  8  2x
x
4
2
0
2
4

8  2x

(x, y)

8  2(4)
8  2(2)
8  2(0)
8  2(2)
8  2(4)

16
12
8
4
0

(4, 16)
(2, 12)
(0, 8)
(2, 4)
(4, 0)

30. First solve the equation for y in terms of x.


4x  3y  0
4x  3y  4x  0  4x
3y  4x
3y
3

y

4

1
53

2

4(2)
3

 23

0

4(0)
3

4(2)
3

2
23

4(4)
3

53

12

51

2

8
6
4
2

4x

29. First solve the equation for y in terms of x.


3x  2y  9
3x  2y  3x  9  3x
2y  9  3x
2y
2

y

4

4

9  3(4)
2

1
102

2

9  3(2)
2

72

9  3(0)
2

42

9  3(2)
2

12

9  3(4)
2

12

(x, y)

1
1
1
1

14, 1012 2
12, 712 2
10, 412 2
12, 112 2
14, 112 2

514, 1012 2, 12, 712 2, 10, 412 2, 12, 112 214, 112 26.

2

21

12, 223 2
14, 513 2
26

4x

y  x  2

12
10
8
6
4
2

Chapter 4

(0, 0)

31. To find the x-intercept, let y  0.


y  x  2
0  x  2
0  x  x  2  x
x2
The graph intersects the x-axis at (2, 0).
To find the y-intercept, let x  0.
y  x  2
y  (0)  2
y2
The graph intersects the y-axis at (0, 2).
Plot these points and draw the line that connects
them.

Graph the solution set

2

4
6
8

9  3x
2
9  3x
2
9  3x
2

4

14, 513 2
12,223 2

Graph the solution set


1
2
2
1
4, 53 , 2, 23 , (0, 0), 2, 23 , 4, 53 .

4
4

21

(x, y)

4(4)
3

4x
3
4x
3

4x
3

Graph the solution set


{(4, 16), (2, 12), (0, 8), (2, 4), (4, 0)}.
16

182

34. To find the x-intercept, let y  0.


5x  2y  10
5x  2(0)  10
5x  10

32. To find the x-intercept, let y  0.


x  5y  4
x  5(0)  4
x4
The graph intersects the x-axis at (4, 0).
To find the y-intercept, let x  0.
x  5y  4
0  5y  4
5y  4
5y
5

5x
5

4
4

1 42

2y
2

The graph intersects the y-axis at 0, 5 .


Plot these points and draw the line that connects
them.

10
2

y5
The graph intersects the y-axis at (0, 5).
Plot these points and draw the line that connects
them.

x  5y  4

5x  2y  10

10
5

x2
The graph intersects the x-axis at (2, 0).
To find the y-intercept, let x  0
5x  2y  10
5(0)  2y  10
2y  10

5

y5

33. To find the x-intercept, let y  0.


2x  3y  6
2x  3(0)  6
2x  6
2x
2

35. To find the x-intercept, let y  0.


1
x
2

6
2

1
x
2

x3
The graph intersects the x-axis at (3, 0).
To find the y-intercept, let x  0.
2x  3y  6
2(0)  3y  6
3y  6
3y
3

 3(0)  3

1
x
2
1
x
2

3

1 2  2(3)

x6
The graph intersects the x-axis at (6, 0).
To find the y-intercept, let x  0.

 3

y  2
The graph intersects the y-axis at (2, 0).
Plot these points and draw the line that connects
them.

1
x
2

 3y  3

1
(0)
2

 3y  3

 3y  3

1
y
3
1
y
3

3

1 2  3(3)

y9
The graph intersects the y-axis at (0, 9).
Plot these points and draw the line that connects
them.
x

y
14
12
10
8
6
4
2

2x  3y  6

2
2

183

1
x  1y  3
2
3

O
2 4 6 8 10 12 14x

Chapter 4

36. To find the x-intercept, let y  0.


1
y3
1
03
1
3
1
2
3  3






1 
3(1) 

1
x
3
1
x
3
1
x
3
1
x
3
1
x
3
1
3 3x

2
3
2
3
2
3
2
3

1
3
1
3

2
3
2
3

g(x)  x2  x  1
g(2a)  (2a) 2  (2a)  1
 4a2  2a  1
46. 9
18
27
36
?

2
2







9
9
9

The next three terms are 45, 54, 63.


47. 6
11
16
21
?
?

The common difference is 5. Add 5 to the last


term of the sequence and continue adding 5 until
the next three terms are found.
21
26
31
36

y  3  3x  3

1
3

2

1




5
5
5

1x

1

The next three terms are 26, 31, 36.


48. 10
21
32
43
?
?

The common difference is 11. Add 11 to the last


term of the sequence and continue adding 11 until
the next three terms are found.
43
54
65
76




11
11
11

The next three terms are 54, 65, 76.


49. 14
12
10
8
?
?







2
2
2
2
2
2

40. g(x)  x  x  1
g(2)  22  2  1
421
21
3




2
2
2

The next three terms are 6, 4, 2.


50. 3
11
19
27
?


8

112 2  112 22  112 2  1


8

421




Chapter 4

1
4
3
4


8





8
8
8
8

The common difference is 8. Add 8 to the last


term of the sequence and continue adding 8
until the next three terms are found.
27
35
43
51

42. g(x)  x2  x  1
1

The common difference is 2. Add 2 to the last


term of the sequence and continue adding 2
until the next three terms are found.
8
6
4
2

g(x)  x2  x  1
g(1)  (1) 2  (1)  1
111
21
3
g







11
11
11
11
11
11

37. The mapping represents a function since, for each


element of the domain, there is only one
corresponding element in the range.
38. The table represents a relation that is not a
function. The element 1 in the domain is paired
with both 4 and 6 in the range.
39. Since each element of the domain is paired with
exactly one element of the range, the relation is a
function.

41.







5
5
5
5
5
5

2 2

The common difference is 9. Add 9 to the last


term of the sequence and continue adding 9 until
the next three terms are found.
36
45
54
63

3







9
9
9
9
9
9

y1
The graph intersects the y-axis at (0, 1).
Plot these points and draw the line that connects
them.

g(a)  x2  x  1
g(a  1)  (a  1) 2  (a  1)  1
 (a2  2a  1)  (a  1)  1
 a2  2a  1  a  1  1
 a2  a  1

45.

y  3  3(0)  3
y

44.

1 2

y  3  3x  3

1
3

g(x)  x2  x  1
g(5)  3  (52  5  1)  3
 (25  5  1)  3
 (20  1)  3
 21  3
 18

3

3  x
The graph intersects the x-axis at (3, 0).
To find the y-intercept, let x  0.

y

43.

1


8


8

The next three terms are 35, 43, 51.

184

51. 35

29

23

17







6
6
6
6
6
6

Move down 5 units.


Draw a dot and label it K.
M(3, 5)
Start at the origin.
Move right 3 units and down 5 units.
Draw a dot and label it M.
N(2, 3)
Start at the origin.
Move left 2 units and down 3 units.
Draw a dot and label it N.

The common difference is 6. Add 6 to the last


term of the sequence and continue adding 6 until
the next three terms are found.
17
11
5
1




6
6
6

The next three terms are 11, 5, 1.


52. Make a table of ordered pairs for several points
on the graph.
1

x
y

2

1

1

 
2
1
1
2
3
6
3
3
6
9

  
3

6

3

3

The difference in y values is three times the


difference of x values. This suggests that y  3x.
Check:
If x  2, then y  3(2) or 6.
If x  3, then y  3(3) or 9.
Thus, y  3x or f(x)  3x describes the relation.
53. Make a table of ordered pairs for several points
on the graph.
1

1


x
y

2
1

1


1
0


1

1


0
1

1

1
2

1


1

5. Since both coordinates are positive, P(25, 1) lies


in quadrant I.
6. To reflect the parallelogram over the y-axis,
multiply the x-coordinate of each vertex by 1.
(x, y) S (x, y)
H(2, 2) S H(2, 2)
I(4, 6) S I(4, 6)
J(5, 5) S J(5, 5)
K(3, 1) S K(3, 1)

3
4

1

The difference in y values is the opposite of the


difference of x values. This suggests that y  x.
Check:
If x  2, then y  (2)  2. But
the y value for x  2 is 1. This is a
difference of 1. Try other values in the
domain to see if the same difference
occurs.
2
2
1

1
1
0

0
0
1

1
1
2

2
2
3

3
3
4

K x

H
J

J


x
x
y


2
3

(3, 5)

(0, 5)

1

O
(2, 3)

y is always 1 less than x.

7. To translate the parallelogram up 2 units, add


2 to the y-coordinate of each vertex.
(x, y) S (x, y  2)
H(2, 2) S H(2, 2  2) S H(2, 0)
I(4, 6) S I(4, 6  2) S I(4, 4)
J(5, 5) S J(5, 5  2) S J(5, 3)
K(3, 1) S K(3, 1  2) S K(3, 1)

Check y  x  1.
If x  1, then y  (1)  1 or 0.
If x  2, then y  (2)  1 or 3.
Thus, y  x  1 or f(x)  x  1 describes this
relation.

Chapter 4 Practice Test

K
H
K

Page 251
1.
2.
3.
4.

b; rotation
c; translation
a; reflection
K(0, 5)
Start at the origin.
Since the x-coordinate is 0, the point is on the
y-axis.

I
I

8. relation: {(0, 1), (2, 4), (4, 5), (6, 10)}


inverse: {(1, 0), (4, 2), (5, 4), (10, 6)}
9. relation: {(1, 2), (2, 2), (3, 2)}
inverse: {(2, 1), (2, 2), (2, 3)}

185

Chapter 4

10. relation: {(1, 1), (0, 3), (1, 0), (4, 2)}
inverse: {(1, 1), (3, 0), (0, 1), (2, 4)}
11.
x
4x  10
y
(x, y)
2
4(2)  10
18
(2, 18)
1
4(1)  10
14
(1, 14)
0
4(0)  10
10
(0, 10)
2
4(2)  10
2
(2, 2)
4
4(4)  10
6
(4,6)

13. First solve the equation for y in terms of x.


1
x
2
1
x
2

2

y

2

Chapter 4

2
4
6
8
8
10
12
14
16

5

(x, y)
(2, 6)

6

1

1
(1)
2

5

52

1
(0)
2

5

5

1
(2)
2

5

4

(2, 4)

1
(4)
2

5

3

(4, 3)

y
16
13
10
4
2

11, 512 2

(0, 5)

14. To find the x-intercept, let y  0.


yx2
0x2
02x22
2  x
The graph intersects the x-axis at (2, 0).
To find the y-intercept, let x 0.
yx2
y02
y2
The graph intersects the y-axis at (0, 2).
Plot these points and draw the line that connects
them.

(x, y)
(2, 16)
(1, 13)
(0, 10)
(2, 4)
(4, 2)

y
O

5

5

Graph the solution set


{(2, 16), (1, 13), (0, 10), (2, 4), (4, 2)}.

4

1
(2)
2

12. First solve the equation for y in terms of x.


3x  y  10
3x  y  3x  10  3x
y  10  3x
1(y)  1(10  3x)
y  3x  10

Graph the solution set


1
{(2, 6), 1, 52 , (0, 5), (2, 4), (4, 3)}.

4x

3x  10
3(2)  10
3(1)  10
3(0)  10
3(2)  10
3(4)  10

2

4

x
2
1
0
2
4

1
x
2

1
x
2

1(y)  1 5  2x

12
10
8
6
4
2
4

 y  2x  5  2x
y  5  2x

Graph the solution set


{(2, 18), (1, 14), (0, 10), (2, 2), (4, 6)}.
18
16

y5

4x

y
yx2

186

15. To find the x-intercept, let y  0.


x  2y  1
x  2(0)  1
x  1
The graph intersects the x-axis at (1, 0).
To find the y-intercept, let x  0.
x  2y  1
0  2y  1
2y  1
2y
2

y

1
2
1
2

19. Graph the equation. For each value of x, a vertical


line passes through no more than one point on the
graph. Thus, the equation represents a function.
y

The graph intersects the y-axis at 0,  2 .


Plot these points and draw the line that connects
them.

g(x)  x2  4x  1

20.

g(2)  (2) 2  4(2)  1


 4  4(2)  1
 4  (8)  1
481
 12  1
 13
21. f(x)  2x  5

x
x  2y  1

112 2  2112 2  5
 1  5
4

g(x)  x2  4x  1
g(3a)  1  [ (3a) 2  4(3a)  1]  1
 [9a2  4(3a)  1]  1
 (9a2  12a  1)  1
 9a2  12a  2
23.
f(x)  2x  5
f(x  2)  2(x  2)  5
 2x  4  5
 2x  1
24. 16
24
32
40
22.

16. To find the x-intercept, let y  0.


3x  5  y
3x  5  0
3x  5
3x
3

 3
5

x  3

The graph intersects the x-axis at 3, 0 .


To find the y-intercept, let x  0.
3x  5  y
3(0)  5  y
05y
0y 5yy
y5
The graph intersects the y-axis at (0, 5).
Plot these points and draw the line that connects
them.




8
8
8

This is an arithmetic sequence because the


difference between terms is constant. The
common difference is 8.
25. 99
87
76
65




12
11
11

This is not an arithmetic sequence because the


difference between terms is not constant.
26. 5
17
29
41




12
12
12

This is an arithmetic sequence because the


difference between terms is constant. The
common difference is 12.
27. 5 10
15
20
25

3x  5  y





15
25
35
45

The difference between each pair of terms


alternates in sign and the absolute value of each
difference is increased by 10 for each successive
pair. Continue the pattern. Add 55, 65, and 75.
5 10
15
20
25
30
35
40

17. Since each element of the domain is paired with


exactly one element of the range, the relation is a
function.
18. The relation is not a function. The element 3 in
the domain is paired with both 1 and 2 in the
range.








15
25
35
45
55
65
75

The next three terms are 30, 35, and 40.

187

Chapter 4

11

3. B;
Let x represent the number of students who eat
five servings.

15






0
1
2
3
4

The difference between each pair of terms


increases by 1 for each successive pair. Continue
increasing each successive difference by 1. Add
5, 6, and 7.
5
5
6
8
11
15
20
26
33

2
5

5(x)  2(470)
5x  940









0
1
2
3
4
5
6
7

5x
5

1a 2 c, b 2 d 2  11  2(5) , 4 2 4 2
4 8
 1 2 , 22

 (2, 4)
The center of the circle is at (2, 4).
6. B;
The relation would not be a function if an element
in the domain were paired with more than one
element in the range. This would occur for x  2
or x  7.
7. D;

1. B;
Use the percent proportion. You know the percent,
2, and the base, 315. Let a represent the part.
p

 100
2

 100

x
4

100(a)  315(2)
100a  630
630
100

1
1

p
p

 100

50,800
2183

p  23.3
About 23% of the total distance remained.

Chapter 4

True or False?
true

y
3
1
?
3

2

For this relation to be linear, the differences of


the y values must be the same when the
differences of the x values are the same.
Therefore, the difference between 1 and ? must
be 2, and the difference between ? and 3 must
also be 2. Thus, the missing value must be 1.

 100

x
1
2
3
4

1

2183( p)  508(100)
2183p  50,800
2183p
2183

3x  4y  12
3(4)  4(0)  12
12  12

y
0

8. C;

a  6.3
There will be about 6 new students, so a total of
315  6, or 321, students will attend next year.
2. C;
The total distance to be traveled is 1675  508, or
2183, mi. Use the percent proportion. You know
the part, 508, and the base, 2183. Let p represent
the percent.
a
b
508
2183

3

x2
5. B;
The center is the midpoint of (1, 4) and (5, 4).
Let (a, b)  (1, 4) and (c, d)  (5, 4). The
midpoint is

Pages 252253

940
5

3x
3

Chapter 4 Standardized Test Practice

100a
100

x  188
You can expect 188 students to eat five servings of
fruits and vegetables daily.
4. B;
13x  2(5x  3)
13x  10x  6
13x  10x  10x  6  10x
3x  6

The next three terms are 20, 26, and 33.


29.
K  C  273
K  273  C  273  273
K  273  C
Since this equation is solved for C in terms of K,
K is the independent variable and C is the
dependent variable.
For five values of K and corresponding values of
C, see students work.
30. D;
f(x)  3x  2
f(8)  f(5)  [3(8)  2 ]  [3(5)  2]
 (24  2)  (15  2)
 22  (17)
 22  17
 39

a
b
a
315

 470

  

28. 5

188

14. original: $160


new: $120
160  120  40

3
3
2

x
2
1
4
6

y
5
2
1
3

  

  

9. C;
3

40
160

3

160(r)  40(100)
160r  4000

2

160r
160

The difference in y values is the opposite of the


difference in x values. This suggests y  x.
Check: If x  2, then y  (2)  2. But the y
value for x  2 is 5. This is a difference
of 3. Try other values in the domain to see
if the same difference occurs.
2
2
5

1
1
2

4
4
1

6
6
3

x
x
y

15
4

V
/h
V
/h

y is always 3
more than x.

/wh
/h

w

1800
(20) (6)
1800
120

w
w

15  w
The pool is 15 ft wide.
16. 1 triangle
2 triangles
3 toothpicks
5 toothpicks

w

4000
160

Substitute V  1800, h  6, and /  20.

4(6)  15(w)
24  15w
24
15
8
5

r  25
There was a 25% decrease in price.
15. Since the unknown quantity is width, solve the
formula for w.
V  /wh

Check y  x  3.
If x  1, then y  (1)  3  2.
If x  6, then y  (6)  3  3.
Thus, y  x  3 describes this relation.
10.

 100


2

15w
15

The number of toothpicks form an arithmetic


sequence since the number added each time is
constant. The first term is 3. The common
difference is 2. Use the formula for the nth term
to write an equation with a1  3 and d  2.
an  a1  (n  1)d
an  3  (n  1)2
an  3  2n  2
an  2n  1

w
8

So, the width is 5, or 1.6, cm.


25

11. P(winning a television)  2000


 0.0125
The probability of winning is 0.0125, or 1.25%.
12. Let n  the first integer.
Then n  3  the second integer, and
2n  1  the third integer.
n  (n  3)  (2n  1)  52
4n  4  52
4n  4  4  52  4
4n  48
4n
4

To find the number of toothpicks used to make


7 triangles, find a7.
a7  2(7)  1
a7  14  1
a7  15
So, 15 toothpicks will be used to make 7 triangles.
17. C;

48
4

n  12
n  3  12  3 or 15
2n  1  2(12)  1 or 25
The integers are 12, 15, and 25.
13. 5(x  2)  3(x  4)  10
5x  10  3x  12  10
2x  22  10
2x  22  22  10  22
2x  32
2x
2


2

3 triangles
7 toothpicks

42  16(2  5)  3
 42  16(7)  3
 16  16(7)  3
 1(7)  3
 73
 21

32
2

60  23  3  6
3

4  62
60  8  3  6
 64  62
60  24  6

2
36  6
 2
42
 2

 21

x  16

The two quantities are equal.

189

Chapter 4

18. B;

123 21158 2119 2


30 1
 1 24 21 9 2
5 1
 1 4 21 9 2

If the width must be a whole number, the


domain of w is {1, 2, 3, . . . . , 24}. For each of
these widths, determine the area of the garden.

21

w (ft)

24  w

 (ft)

24  1

23

23

24  2

22

44

24  3

21

63

24  4

20

80

24  5

19

95

24  6

18

108

24  7

17

119

24  8

16

128
135

 56

 36
5

134 21141 2
21 1
 1 4 21 14 2
8

10

27

Since 36  72 and 8  72, the quantity in


Column B is greater.
19. B;
6x  15  3x  75
6x  15  3x  3x  75  3x
9x  15  75
9x  15  15  75  15
9x  90
9x
9

90
9

x  10
3y  32  7y  74
3y  32  3y  7y  74  3y
32  4y  74
32  74  4y  74  74
42  4y
42
4

4y
4

10.5  y
The quantity in Column B is greater.
20. A;
f(x)  37  10x
f(10)  37  10(10)
 37  100
 63
g(x)  9x  7
g(15)  9(15)  7
 135  7
 142
The quantity in Column A is greater.
21a. P  2/  2w
Since the perimeter is 48 ft, 2/  2w  48.
21b. A  /w
Use the formula from part a to solve for one
variable in terms of the other.
2/  2w  48
2(/  w)  48
2(/  w)
2

24  9

15

10

24  10

14

140

11

24  11

13

143

12

24  12

12

144

13

24  13

11

143

14

24  14

10

140

15

24  15

135

16

24  16

128

17

24  17

119

18

24  18

108

19

24  19

95

20

24  20

80

21

24  21

63

22

24  22

44

23

24  23

23

24

24  24

The greatest possible area is 144 ft2.

48
2

/  w  24
/  w  w  24  w
/  24  w

Chapter 4

A  w (ft2)

190

GREATEST

Chapter 5
Page 255
1.

3.

5.

7.

Analyzing Linear Equations


2.

10.

a  b
c  d

a  b
c  d

y2  y1

8  4

8
12

 12  4

mx

3
4.

6.

8.

3
9.

6. Let (0, 0)  (x1, y1) and (5, 4)  (x2, y2).

Getting Started

2  2
 10  2
1
5
2
2  2
 8  2
8
1
 4
5
5  5
 15  5
15
1
3
9
9  3
33
3
3
1
2
10

4  4
 84
1
 2
7  7
7
 28  7
28
1
4
18  6
18
 12  6
12
3
2
1
 12
4
8

7. Let (2, 2)  (x1, y1) and (1, 2)  (x2, y2).


y2  y1

mx




y2  y1

mx

6  5




5
2
5
2
6
3










(12
(12
1
1

11.

a  b
c  d





2  1
4  0
3
4
3
4

a  b
c  d





8  (22
1  1
8  2
2
10
2

13.

a  b
c  d




 5
14.

a  b
c  d

1
2

y2  y1

mx

3  (32
4  7
3  3
3
0
3

 x1

5  5
2  3
0
5

0
10. Let (1, 3)  (x1, y1) and (1, 0)  (x2, y2).
y2  y1

mx

0

 x1

0  3
 (12
3
0

 1

3
2

79
1
 2

Since division by zero is undefined, the slope is


undefined.
11. Let (6, 2)  (x1, y1) and (r, 6)  (x2, y2).

1

 2
1

2

y2  y1

16. (3, 2)


19. (2, 2)

15. (1, 2)
18. (0, 3)

 x1

1  (42
9  7
3
2

9. Let (3, 5)  (x1, y1) and (2, 5)  (x2, y2).

2
12.

 x1

2  2
1  (22
4
1

 4
8. Let (7,4)  (x1, y1) and (9, 1)  (x2, y2).

84
1
4

 x1

4  0
5  0
4
5

mx

17. (2, 3)


20. (3, 0)

4
4

4(r  62  4
4r  24  4
4r  24  24  4  24
4r  20

Slope

5-1

Pages 259260

Check for Understanding

4r
4

1. Sample answer: Use (1,3) as (x1, y1) and


(3,5) as (x2, y2) in the slope formula.
2. See students work.
3. The difference in the x values is always 0, and
division by 0 is undefined.
4. Carlos; Allison switched the order of the
x-coordinates, resulting in an incorrect sign.
5. Let (1, 1)  (x1, y1) and (3, 4)  (x2, y2).
2




20
4

r5
12. Let (9, r)  (x1, y1) and (6, 3)  (x2, y2).
y2  y1

mx
1
3
1
3




 x1

3  r
6  9
3  r
3

1(32  3(3  r2
3  9  3r
3  9  9  3r  9
6  3r

y2  y1

mx

 x1

6  (22
r  6
4
r  6

 x1

4  1
3  1
3
2

6
3

3r
3

2r

191

Chapter 5

22. Let (3, 6)  (x1, y1) and (2, 4)  (x2, y2).

13. Use the formula for slope.


rise
run

y2  y1

change in quantity
change in time
55  52
1992  1990
3
or 1.5
2





mx




Over this 2-year period, the number of


subscribers increased by 3 million, for a rate of
change of 1.5 million per year.
14. Sample answer: 9294; steeper segment means
greater rate of change.

Pages 260262

23. Let (3, 4)  (x1, y1) and (5, 1)  (x2, y2).
y2  y1

mx

Practice and Apply

15. Let (2, 4)  (x1, y1) and (2, 1)  (x2, y2).
2




y2  y1

 x1

mx

1  (4)
2  (2)
3
4




16. Let (0, 3)  (x1, y1) and (3, 2)  (x2, y2).


y2  y1

mx

 x1

y2  y1

mx

1
3
1
3

Since division by zero is undefined, the slope is


undefined.
26. Let (2, 6)  (x1, y1) and (1, 3)  (x2, y2).

y2  y1
2




 x1

3  (12
3  (42
2
1

y2  y1

mx

 2
18. Let (3, 3)  (x1, y1) and (1, 3)  (x2, y2).

y2  y1

mx




y2  y1

mx

0
19. Let (2, 1)  (x1, y1) and (2, 3)  (x2, y2).

y2  y1
2




y2  y1

mx




y2  y1
 x1

29. Let (8, 3)  (x1, y1) and (6, 2)  (x2, y2).

7  3

92


y2  y1

mx

4
7

21. Let (5, 7)  (x1, y1) and (2, 3)  (x2, y2).

y2  y1

mx

 x1

3  7

 2  5



10
7
10
7

Chapter 5

 x1

6  9
 (32
3
4
3
4

 7

Since division by zero is undefined, the slope is


undefined.
20. Let (2, 3)  (x1, y1) and (9, 7)  (x2, y2).
2

 x1

3  3
8  (22
0
10

0
28. Let (3, 9)  (x1, y1) and (7, 6)  (x2, y2).

 x1

3  1
2  (22
2
0

mx

 x1

3  6
1  2
3
3

1
27. Let (2, 3)  (x1, y1) and (8, 3)  (x2, y2).

 x1

3  3
1  (32
0
4

mx

 x1

1  4
 (52
5
0

 5

17. Let (4, 1)  (x1, y1) and (3, 3)  (x2, y2).
mx

 x1

3  (1)
5  2
2
3
2
3

25. Let (5, 4)  (x1, y1) and (5, 1)  (x2, y2).

2  3

30


 x1

1  (4)
5  (3)
3
8

24. Let (2, 1)  (x1, y1) and (5, 3)  (x2, y2).

y2  y1

mx

 x1

4  6
2  (32
2
5
2
5

192

 x1

2  3
6  (82
1
2
1
2

30. Let (2, 0)  (x1, y1) and (1, 1)  (x2, y2).

37. Let (0, 0)  (x1, y1) and (r, s)  (x2, y2).

y2  y1

mx




y2  y1

mx

 x1




y2  y1

 x1

2  (12

 5.3  4.5



3
0.8
15
4

39.

rise
run

40.

rise
run




 x1

1  1
0.75  0.75
2
0

33. Let 22,12  (x , y ) and 2,


1 1
y2  y1

mx

1
2

 x1

 12
1

2  22

1
2

2  (x2,

y2 ).

y2  y 1

mx
1 

(x1, y1 ) and

1
2,

1  (x2, y2 ).

 x1

y2  y 1

1  14
1

mx

2  4

8

4

8

4

35. Sample answer: The rise in the picture is about


16 mm, and the run is about 22 mm.



rise
run
16
22
8
11

y2  y 1

mx
4
3
4
3

36. Sample answer: The rise in the picture is about


10 mm, and the run is about 30 mm.
slope 



 x1

r  (52
3  4
r  5
1

8(12  r  5
8  r  5
8  5  r  5  5.
13  r
43. Let (5, r)  (x1, y1) and (2, 3)  (x2, y2).

5

slope 

 x1

r  2
9  6
r  2
3

1(32  r  2
3  r  2
3  2  r  2  2
1  r
42. Let (4, 5)  (x1, y1) and (3, r)  (x2, y2).

change in quantity
change in time
5.15  4.25
1997  1991
0.9
6

1 

2
 3
3
1
Let 4, 14 
y2  y1
2

16
4

 0.15
Over this 6-year period, the minimum wage
increased by $0.90, for a rate of change of $0.15
per year.
41. Let (6, 2)  (x1, y1) and (9, r)  (x2, y2).

mx

 3

34.

Since division by zero is undefined, the slope is


undefined.

b  b
a  a
2b
0

4

y2  y1
2

0

Since division by zero is undefined, the slope is


undefined.

32. Let (0.75, 1)  (x1, y1) and (0.75, 1)  (x2, y2).
mx

s  0
r  0
s
if r
r

38. Let (a, b)  (x1, y1) and (a,  b)  (x2, y2).


y2  y1
mx x

31. Let (4.5, 1)  (x1, y1) and (5.3, 2)  (x2, y2).
mx

 x1

1  0
1  (22
1
3
1
3




 x1

3  r
2  5
3  r
3

4(32  3(3  r2
12  9  3r
12  9  9  3r  9
3  3r

rise
run
10
30
1
3

3
3

3r
3

1r

193

Chapter 5

48. Let (r, 5)  (x1, y1) and (2, r)  (x2, y2).

44. Let (2, 7)  (x1, y1) and (r, 3)  (x2, y2).


y2  y1

mx
4
3
4
3




y2  y1

mx

 x1

3  7
r  (22
4
r  2

2
9

212  r2  91r  52
4  2r  9r  45
4  2r  9r  9r  45  9r
4  7r  45
4  7r  4  45  4
7r  49

4(r  22  3(42
4r  8  12
4r  8  8  12  8
4r  20
4r
4

45. Let

20
4

7r
7

r  5

1
,
2

1
4

2  (x1, y1) and 1r, 54 2  (x2, y2).


y2  y1

 x1
5
1
4  (4 2
1
r2
2

r

4(r  2 2  1

1
2

4r  2  1
4r  2  2  1  2
4r  1
4r
4

1 r2  (x1, y1) and 11, 2  (x2, y2).


y2  y1
 x1

1
2

r

1
2

1
2

r

113 2  21
1
3

1
3

13
1
2

1
3
1
2

r

 1  2r

54.

 1  1  2r  1
2
3
2
3

2
1
3

r

47. Let (4, r)  (x1, y1) and (r, 2)  (x2, y2).


y2  y1
2

5
3

 x1

2  r
r  4

5(r  4)  3(2  r)
5r  20  6  3r
5r  20  3r  6  3r  3r
2r  20  6
2r  20  20  6  20
2r  14
2r
2

14
2

r7
Chapter 5

change in enrollment
change in time

11.3  12.4

 1990  1985
1.1
5

 0.22
Between 1985 and 1990, the rate of change was
0.22 million students per year.
55. The negative slope represents a decline in
enrollment.
56. See students work.

2r
2

mx

12 14 16 18 20
Age (years)

 2r


60

51. Karen grew the fastest in the two-year period


from age 12 to age 14. The line segment
representing this two-year period is the steepest
part of the graph.
52. There was no change in height in that two-year
period.
53. The steepest segment is between 90 and 95.
Thus, the rate of change was the greatest during
this five-year period.
The least steep segment is between 80 and 85.
Thus, the rate of change was the least during this
five-year period.

1
2

mx

62

r4

46. Let

66
64

58

4

2
,
3

49
7

68

1
Height (in.)

4

r7
49. (4, 5) is in Quadrant III and (4, 5) is in
Quadrant I. The segment connecting them goes
from lower left to upper right, which is a positive
slope.
50.
Karens Height

mx
4

 x1

r  5
2  r

194

Let (2, 2)  (x1, y1) and (1, 1)  (x2, y2).

57. In the picture, the stairs rise 7 in. for each 11 in.
of run.
slope 


y2  y1

mx

rise
run
7
11




Use the formula for slope with slope  11, rise 


8 feet 9 inches or 105 in. and run  r.

Let (4, 0)  (x1, y1) and (2, 2)  (x2, y2).


y2  y1

mx

rise
run
105
r

slope 
7
11

7(r)  11(105)
7r  1155
7r
7

 x1

1  (2)
1  (2)
1
3




1155
7

 x1

2  0
2  4
2
6
1
3

The slope is the same regardless of points chosen.


62. Let (2, 3)  (x1, y1) and (1, 1)  (x2, y2).

r  165
The total run would be 165 in. or 13 feet 9 inches.
58. Sample answer: Analysis of the slope of a roof
might help to determine the materials of which it
should be made and its functionality. Answers
should include the following.
To find the slope of the roof, find a vertical line
that passes through the peak of the roof and a
horizontal line that passes through the eave.
Find the distances from the intersection of
those two lines to the peak and to the eave.
Use those measures as the rise and run to
calculate the slope.
A roof that is steeper than one with a rise of 6
and a run of 12 would be one with a rise
greater than 6 and the same run. A roof with a
steeper slope appears taller than one with a
less steep slope.
59. D; Let (5,4)  (x1, y1) and (5, 10)  (x2, y2).

y2  y1

mx

 x1

1  3
 2
4
3
4
3

 1



Let (1, 1)  (x1, y1) and (4, 2)  (x2, y2).
y2  y1

mx

 x1

2  (1)
 (1)
1
3
1
3

 4



No, they do not. Slope of QR is 3 and slope of RS


1
is 3. If they lie on the same line, the slopes should
be the same.

y2  y1

mx




 x1

Page 262

10  (4)
5  5
6
0

1

63.

Since division by zero is undefined, the slope is


undefined.
60. B; Let (c, d)  (x1, y1) and (a, b)  (x2, y2).
2

 x1

b  d
a  c

61. Let (5, 3)  (x1, y1) and (2, 2)  (x2, y2).
y2  y1

mx

 x1

2  (3)
 (5)
1
3

 2


x
f(x)

1
5

1

2
10

1

3
15

1

4
20

5
25





5
5
5
5
The difference of the values for x is 1, and the
difference of the values for f(x) is 5. This suggests
that f(x)  5x. Check this equation.
Check:
If x  1, then f(x)  5(1) or 5.
If x  2, then f(x)  5(2) or 10.
If x  3, then f(x)  5(3) or 15.
The equation checks. We can write the equation
as f(x)  5x.

y2  y1

mx

Maintain Your Skills

Let (5, 3)  (x1, y1) and (4, 0)  (x2, y2).


y2  y1

mx





 x1

0  (3)
4  (5)
3
9
1
3

195

Chapter 5

1
2
1
2

  
64. x
2
1
1
2
4
f(x)
13
12
10
9
7

  
1
2
1
2
The difference in f(x) values is 1 times the
difference in x values. This suggests that f(x)  x.
Check this equation.
Check:
If x  2, then f(x)  (2) or 2.
But the f(x) value for x  2 is 13.
This is a difference of 11. Try some other value in
the domain to see if the same difference occurs.
2
2
13

x
x
f(x)

1
1
12

1
1
10

2
2
9

69. Solve the equation for y.


xy0
xyx0x
y  x
(1)(y)  (1)(x)
yx
Select five values for the domain and make a
table. Then graph the ordered pairs and draw a
line through the points.
x
3
1
0
2
4

4
4
7

f(x) is always 11 more than x. This pattern


suggests that 11 should be added to one side of
the equation in order to correctly describe the
relation. Check f(x)  x  11.
Check:
If x  2, then f(x)  (2)  11 or 13.
If x  1, then f(x)  (1)  11 or 12.
The equation checks. We can write the equation
as f(x)  11  x.
65. Graph the equation. No vertical line passes
through more than one point on the graph. Thus,
the line represents a function.
6
3

x y  0

a
b

 100

a
37.5

 100

40

100a  37.5(40)
100a  1500
100a
100

1500
100

a  15
Forty percent of 37.5 is 15.
71. 7(3)  21
72. (4)(2)  8
73. (9)(4)  36
74. (8)(3.7)  29.6

18

66. Graph the equation. The vertical line x  5 passes


through more than one point on the graph. Thus,
the line does not represent a function.

1 7 2113 2  247

75. 8

77. 6  3  1  2

x5

76.

114 2112 2 (14)  118 21142


7

81.
67. This set of ordered pairs represents a relation
that is not a function. The element 1 in the
domain is paired with both 0 and 4 in the range.
68. This set of ordered pairs represents a function
since, for each element of the domain, there is
only one corresponding element in the range.

83.

85.

196

3
8

78. 12  4  12  4
 48

18
2

10 8

1 3
80
2
 3 or 26 3
3
1
3 6
 6  41
4
18
 4
9
1
 2 or 42
7
18 8
18  8  1  7
144
4
 7 or 20 7
2
1
8 4
23  4  3  1
32
2
 3 or 103

 4 or 14

9

79. 10 

Chapter 5

70. The percent is 40, and the base is 37.5. Let a


represent the part.

12 9 6 3 O 3 6 9 12 x
3
6
9
y  15
12

(x, y)
(3, 3)
(1, 1)
(0, 0)
(2, 2)
(4, 4)

y
3
1
0
2
4

80.

1
2

 3  21
3

 2 or 1 2
82.

3
4

 6  46
3

 24
1

8
84.

3
8

 5  82
15

 16

Page 263

6. Step 1:

Reading Mathematics

Term
2a. slope

2b. intercept

2c. parallel

Everyday
Meaning
1. to diverge
from the vertical
or horizontal;
incline
2. to move on a
slant; ascend
or descend
to stop, deflect,
or interrupt the
progress or
intended
course of
of, relating to, or
carrying
out the
simultaneous
performance of
separate tasks

Write the slope as a ratio.


2

21
Step 2: Graph (0, 0).
Step 3: From the point (0, 0), move up
2 units and right 1 unit.
Draw a dot.
Step 4: Draw a line containing the points.

1. Sample answer: The mathematical meaning of


function is most closely related to the third
definition in the everyday meanings.
Mathematical
Meaning
the ratio of
the rise to the
run

the coordinate
at which a graph
intersects an
axis
7. Step 1:
lines that never
intersect;
nonvertical lines
that have
the same slope

Write the slope as a ratio.


3 

3
1

Step 2: Graph (0, 0).


Step 3: From the point (0, 0), move
down 3 units and right 1 unit.
Draw a dot.
Step 4: Draw a line containing the points.
y

Slope and Direct Variation

5-2

y  2x

y  3 x

Page 265

Graphing Calculator Investigation

1. All the graphs pass through the origin.


2. None of the graphs have the same slope.
3. Sample answer: y  5x; See students graphs.

4. Sample answer: y  2x; See students graphs.


5. This family of graphs has a y-intercept of 0. Their
slopes are all different.
6. As|m|increases the graph becomes more steep.

Page 267

8. Step 1:

1
2

Step 2: Graph (0, 0).


Step 3: From the point (0, 0), move
up 1 unit and right 2 units.
Draw a dot.
Step 4: Draw a line containing the points.

Check for Understanding

1. y  kx, where k is a constant of variation.


2. b; 4a  b means b varies directly as a, and 4 is
the constant of variation.
1

c; z  3 x means z varies directly as x, and


constant of variation.
3. They are equal.

1
3

is the

y  12 x

4. The constant of variation is 3.

y2  y1

mx

Write the slope as a ratio.

 x1

0  1
(3)
1
3

m0
m

The slope is 3.


5. The constant of variation is 1.
y2  y1

mx

 x1

2  0

m20
m1
The slope is 1.

197

Chapter 5

13. The graph of y  6x passes through the origin


with slope 6.

9. Find the value of k.


y  kx
27  k(6)
27
6
9
2

m1

k(6)
6

k
9

Therefore, y  2 x.

y  6x

Now find x when y  45.


9

y  2x
9

45  2 x
2
(45)
9

1 2

2 9
x
9 2

10  x
10. Find the value of k.
y  kx
10  k(9)
10
9
10
9

14. y  6x
y  6(30)
y  180
You will earn $180 if you work 30 hours.

k(9)
9

k

Therefore, y 

Pages 268270

10
x.
9

y2  y1

Now find x when y  9.


y
9
9
(9)
10
81
10

mx

10
x
9
10
x
9
9 10
x
10 9

x

m

1 2

y2  y1

mx

m

 x1

0  (4)
0  (1)

m4
The slope is 4.

k(14)
14

17. The constant of variation is 2.

k

y2  y1

mx

Therefore, y  2 x.

m

Now find y when x  20.

 x1

1  0
2  0
1
2

m

The slope is 2.

y  2x

y  2 (20)
y  10
12. Words:
Variables:

18. The constant of variation is 1.


y2  y1

mx

Your pay for 7.5 hours is $45.


Let k  pay rate.

m

Amount of Pay equals pay rate times time.

$45
Equation:
Solve for the rate.
45  k(7.5)

 x1

2  0
2  0

m  1
The slope is 1.

424
3
144424443 14243 14243 1
424
3 1

45
7.5

 x1

4  0
2  0

m2
The slope is 2.
16. The constant of variation is 4.

8.1  x
11. Find the value of k.
y  kx
7  k(14)
7
14
1
2

Practice and Apply

15. The constant of variation is 2.

7.5h

19. The constant of variation is 2.


y2  y1

mx

k(7.5)
7.5

m

6k
Therefore, the direct variation equation is y  6x.

m

 x1

3  0
2  0
3
2
3

The slope is 2.

Chapter 5

198

24. Write the slope as a ratio.

20. The constant of variation is 4.


y2  y1

mx

m
m

4 

 x1

4
1

Graph (0, 0). From the point (0, 0), move down
4 units and right 1 unit. Draw a dot. Draw a line
containing the points.

1  0
4  0
1
4

The slope is 4.


21. Write the slope as a ratio.

y  4 x

11
Graph (0, 0). From the point (0, 0), move up
1 unit and right 1 unit. Draw a dot. Draw a line
containing the points.

25. Write the slope as a ratio.


yx

1
4

Graph (0, 0). From the point (0, 0), move up


1 unit and right 4 units. Draw a dot. Draw a line
containing the points.
y
y  14 x

22. Write the slope as a ratio.


3

31

Graph (0, 0). From the point (0, 0), move up


3 units and right 1 unit. Draw a dot. Draw a line
containing the points.
y

26. Write the slope as a ratio.

y  3x

3
5

Graph (0, 0). From the point (0, 0), move up


3 units and right 5 units. Draw a dot. Draw a line
containing the points.

y  35 x

23. Write the slope as a ratio.


1 

1
1

Graph (0, 0). From the point (0, 0), move down
1 unit and right 1 unit. Draw a dot. Draw a line
containing the points.
y
y  x

27. Write the slope as a ratio.


5
2

Graph (0, 0). From the point (0, 0), move up


5 units and right 2 units. Draw a dot. Draw a line
containing the points.

y  52 x
x

199

Chapter 5

32. Write the slope as a ratio.

28. Write the slope as a ratio.

9
2

7
5

2 

Graph (0, 0). From the point (0, 0), move up


7 units and right 5 units. Draw a dot. Draw a line
containing the points.

Graph (0, 0). From the point (0, 0), move down
9 units and right 2 units. Draw a dot. Draw a line
containing the points.

y
x

y   92 x
y  75 x
O

29. Write the slope as a ratio.

33. Find the value of k.


y  kx
8  k(4)

1
5

Graph (0, 0). From the point (0, 0), move up


1 unit and right 5 units. Draw a dot. Draw a line
containing the points.

8
4

y  15 x
O

36
6

30. Write the slope as a ratio.


2

Graph (0, 0). From the point (0, 0), move down
2 units and right 3 units. Draw a dot. Draw a line
containing the points.
y

42
6

16
4

4
3

Graph (0, 0). From the point (0, 0), move down
4 units and right 3 units. Draw a dot. Draw a
line containing the points.

20
4

y   43 x

Chapter 5

k(4)
4

4x
4

5  x

4  k
Therefore, y  4x.
Now find x when y  20.
y  4x
20  4x

31. Write the slope as a ratio.


4

6x
6

7x
35. Find the value of k.
y  kx
16  k(4)

y   23 x

3 

k(6)
6

6k
Therefore, y  6x.
Now find x when y  42.
y  6x
42  6x

2
3

k(4)
4

2k
Therefore, y  2x.
Now find y when x  5.
y  2x
y  2(5)
y  10
34. Find the value of k.
y  kx
36  k(6)

3 

200

36. Find the value of k.


y  kx
18  k(6)
18
6

40. Find the value of k.


y  kx
6.6  k(9.9)
6.6
9.9
2
3

k(6)
6

3  k
Therefore, y  3x.
Now find x when y  6.
y  3x
6  3x
6
3

Now find y when x  6.6.


2

y  3 x

3x
3

y  3 (6.6)
y  4.4
41. Find the value of k.
y  kx

114 2
4 8
4
1
 1 1k  4 2
1 13 2
2

23  k

k

32
3

Therefore, y  3 x.

32
x.
3
1

Now find y when x  18.

y

y  3x
y  3 (24)
y  8
38. Find the value of k.
y  kx
12  k(15)


9k
Therefore, y  9x.
Now find x when y  12.
y  9x
12  9x

y  5x
21  5x
4

105
4

x

1 2

5 4
x
5

12
9
4
3

26.25  x
39. Find the value of k.
y  kx
2.5  k(0.5)


1 2
12

123 2
3
3
2
(6)  2 1 k  3 2
2

Now find x when y  21.

5
(21)
4

y

32 9
3 8

6k

Therefore, y  5x.

y

32
x
3
32 1
18
3

y  12
42. Find the value of k.
y  kx

k(15)
15

k

2.5
0.5

k

Therefore, y 

Now find y when x  24.

12
15
4
5

k
2

k(12)
12

k(9.9)
9.9

Therefore, y  3 x.

2  x
37. Find the value of k.
y  kx
4  k(12)
4
12
1
3

9x
9

x

43. The
the diameter
d.
circumference
C is 3.14 times
1444
442444
443 { 123 1
424
3 1444
424444
3
C

k(0.5)
0.5

 3.14

The direct variation equation is C  3.14d.


The graph of C  3.14d passes through the origin
with slope 3.14.

5k
Therefore, y  5x.
Now find y when x  20.
y  5x
y  5(20)
y  100

m

3.14
1

C  3.14 d

201

Chapter 5

48. Line 4 passes through the points (0, 0) and


(1, 25).

The perimeter P is 4 times the length of a side s.


3 144444424444443
44. 14444244443 { { 1424
P
4 
s
The direct variation equation is P  4s.
The graph of P  4s passes through the origin
with slope 4.

y2  y1

mx

m

 x1

25  0
1  0

m  25
The slope of line 4 is 25. Therefore, line 4
represents the sprinting speed of the elephant.
49. Line 2 passes through the points (0, 0) and
(1, 32).

m1
P

y2  y1

mx

P  4s

m

m  32
The slope of line 2 is 32. Therefore, line 2
represents the sprinting speed of the reindeer.
50. Line 1 passes through the points (0, 0) and
(1, 50).

is 0.99 times the number of yards n.


The
total cost C
444424444
3 { 123 1
424
3 1444442444443
45. 1
 0.99 
C
n

y2  y1

mx

The direct variation equation is C  0.99n.


The graph of C  0.99n passes through the
origin with slope 0.99.
m

m

y2  y1

mx

C  0.99 n

m

The total cost C is


14.49 times
the
number of pounds p.
{ 123 1
424
3 1
4444442444444
3
C
 14.49 
p

1
444424444
3

The direct variation equation is C  14.49p.


The graph of C  14.49p passes through the
origin with slope 14.49.

1442443 1
424
3 1442443 1
424
3 1442443

60

k

Solve for the constant of variation.
60  k(360)

14.49
1

60
360
1
6

40
30

k(360)
360

k

53. m  6 e

10

p
2

m  6 (138)

m  23
If you weigh 138 pounds on Earth, you would
weigh 23 pounds on the moon.

47. It also doubles. If x  k, and x is multiplied by 2,


y must also be multiplied by 2 to maintain the
value of k.

Chapter 5

360

Therefore, the direct variation equation is


1
m  6 e.

C  14.49 p

20

 x1

30  0
1  0

m  30
The slope of line 3 is 30. Therefore, line 3
represents the sprinting speed of the grizzly bear.
52. The weight on the moon is 60 pounds, and the
weight on Earth is 360 pounds.
Let k  constant of variation.
the weight
The weight
the constant
equals
times
on Earth.
on the moon
of variation

m

 x1

50  0
1  0

m  50
The slope of line 1 is 50. Therefore, line 1
represents the sprinting speed of the lion.
51. Line 3 passes through the points (0, 0) and
(1, 30).

0.99
1

46.

 x1

32  0
1  0

202

54. The age of a human is 3 years old, and the age of


a horse is 1 year old.
Let k  constant of variation.
The age of
the constant
the age of
equals
times
a human
of variation
a horse.

Page 270
63. m  x

m

y2  y1

3

k

1
Solve for the constant of variation.
3  k(1)
3k
Therefore, the direct variation equation is y  3x.
55. y  3x
16  3x


 x1

0  3
2  1

m  3

1442443 1
424
3 1442443 1
424
3 1
44244
3

16
3
1
53

Maintain Your Skills

y2  y1

64. m  x

m
m

 x1

2  (2)
2  2
4
0

Since division by zero is undefined, the slope is


undefined.
y2  y1

65. m  x

3x
3

m

x

3  1
2  (3)

2

m2

The equivalent horse age for a human who is


1
16 years old is 53 years old or 5 years 4 months.

2

2r
2

m
m

2
2

r  1
1

67.

1


x
y

0
1

1


1

1


2
9

y2  y1
2

 x1

3  7
r  1
4
r  1

21r  12  4
2r  2  4
2r  2  2  4  2
2r  2

56. The slope of the equation that relates time and


water use is the number of gallons used per
minute in the shower. Answers should include the
following.
y  2.5x
Less steep; the slope is less than the slope of
the graph on page 268.
57. D; The graph passes through the points (0, 0) and
(2, 1).
mx

y2  y1

mx

66.

 x1

1

3
13



4

5
21

4

 x1

1  0
2  0
1
2

The difference in the x values is 1, and the


difference in the y values is 4. Since 1  4  5, and
13  4  17, the numbers 5 and 17 are inserted.
1

Therefore, the equation is y  2 x.

x
y

58. C; The ordered pair (0, 0) is a solution of a direct


variation equation. Check (0, 0) in the equation
y  3x  1.
Check:
y  3x  1
?
0  3(0)  1
01
59. The calculator screen shows the graphs of
y  1x, y  2x and y  4x.
[10, 10] scl: 1 by [10, 10] scl: 1

0
1

1
5
2

68.

2


x
y

2
9
2


4

3
13


6
4

4
17
2

5
21
2

 
8
10
12
2
2


2

The difference in the x values is 2, and the


difference in the y values is 2. Since 4  2  6,
6  2  8, and 2  2  0, the numbers 8, 6, and 0
are inserted.
x
y

2
8

4
6

6
4

8
2

10
0

12
2

69. 15  1122  1|15|  |12|2


 115  122
3
70. 8  152  8  5
 13
71. 9  6  9  162
 (|9|  |6|)
 19  62
 15
72. 18  12  18  1122
 1|18|  |12|2
 118  122
 30

60. They all pass through (0, 0), but these have
negative slopes.
61. Sample answer: y  5x.
62. Sample answer: Find the absolute value of k in
each equation. The one with the greatest value of
|k| has the steeper slope.

203

Chapter 5

3x  y  8
3x  y  3x  8  3x
y  3x  8
74.
2x  y  7
2x  y  2x  7  2x
y  2x  7
75.
76.
4x  y  3
2y  4x  10
1
1
4x  3  y  3  3
(2y)
 2 (4x  10)
2
4x  3  y
y  2x  5
77.
9x  3y  12
9x  3y  9x  12  9x
3y  9x  12
73.

1
(3y)
3

78.

7. Write the slope as a ratio.


7 

Graph (0, 0). From the point (0, 0), move down
7 units and right 1 unit. Draw a dot. Draw a line
containing the points.
y
y  7 x

 3 (9x  12)
8. Write the slope as a ratio.
3
4

Graph (0, 0). From the point (0, 0), move up


3 units and right 4 units. Draw a dot. Draw a line
containing the points.

5  x
2
x  5
2

y

Page 270

y
y  34 x

Practice Quiz 1

y2  y1

1. m  x

y2  y1

2. m  x

 x1

8  (6)
(4)

m  2

m  19

m0

y2  y1

m
m

3  3

m  11  8

 x1

m  3 

3. m  x

y2  y1

4. m  x

 x1

9  8
5  (4)
1
9

m
m
y2  y1

mx

5.

2
2

9. Find the value of k.


y  kx
24  k182

 x1

11  1
7  0
10
7

24
8

2r
2

2

10
15
2
3

r4

y2  y1

mx

3
2
3
2




 x1

9  r
4  6
9  r
10

k(15)
15

k
2

y  3x
2

6  3x
3

2 (6)  2 3 x
9x

2r
2

24  r

Chapter 5

k182
8

Therefore, y  3x.
Now find x when y  6.

3(10)  2(9  r)
30  18  2r
30  18  18  2r  18
48  2r
48
2

3k
Therefore, y  3x.
Now find y when x  3.
y  3x
y  3(3)
y  9
10. Find the value of k.
y  kx
10  k(15)

 x1

5  (3)
r  5
2
r  5

2(r  5)  2
2r  10  2
2r  10  10  2  10
2r  8

6.

y  3x  4
x  2y  5
x  2y  x  5  x
2y  5  x
2y
2

7
1

204

Page 271

5-3

See students work.


Sample answer: It is a linear pattern.
It is the y-intercept.
See students work. Sample answer: 1.5
The slope represents the rate of change.
Add 2.5 units to the y coordinate of each data
point. Plot points at (0, 10.5), (1, 11.75), (2, 13),
(3, 14.5), and (4, 15.5).
Length (cm)

1.
2.
3.
4.
5.
6.

Algebra Activity
(Preview of Lesson 5-3)

16
14

Page 275

12
10
8

1 2 3 4 5
Number of Washers

y2  y1

mx

Length (cm)

7. The sample data has a distance of 8 cm for


0 washers and an approximate change of 1.25 cm
for each washer added. Therefore, the new graph
should have a point at (0, 8), then increase the
distance more than 1.25 cm for each washer
added. Sample answer: plot points at (0, 8),
(1, 10), (2, 12), (3, 14), and (4, 16).
16
14

m
m

12
10
8

1 2 3 4 5
Number of Washers

8. The sample data has a distance of 8 cm for


0 washers and an approximate change of 1.25 cm
for each washer added. Therefore, the new graph
should have a point at (0, 8), then increase the
distance less than 1.25 cm for each washer added.
Sample answer: plot points at (0, 8), (1, 9), (2, 10),
(3, 11), and (4, 12).

Length (cm)

12

 x1

3  (1)
2  0
4
2

m2
The slope is 2.
Step 2: The line crosses the y-axis at (0, 1).
So, the y-intercept is 1.
Step 3: Finally, write the equation.
y  mx  b
y  2x  112
y  2x  1
The equation of the line is y  2x 1.
7. Step 1: You know the coordinates of two points
on the line. Find the slope.
Let (x1, y1)  (0, 2) and (x2, y2)  (2, 1).

Check for Understanding

1. Sample answer: y  7x  2
2. Vertical lines have undefined slope. Horizontal
lines have a slope of 0.
3. The rate of change is described by the slope.
4. Replace m with 3 and b with 1.
y  mx  b
y  3x  1
5. Replace m with 4 and b with 2.
y  mx  b
y  4x  122
y  4x  2
6. Step 1: You know the coordinates of two points
on the line. Find the slope.
Let (x1, y1)  (0, 1) and (x2, y2)  (2, 3).

Slope-Intercept Form

y2  y1

mx

m
m
m

 x1

1  2
2  0
3
2
3
2
3

The slope is 2.

11
10
9

Step 2: The line crosses the y-axis at (0, 2). So,


the y-intercept is 2.
Step 3: Finally, write the equation.
y  mx  b

8
7
6

y  2x  2

The equation of the line is y  2x  2.

1 2 3 4 5
Number of Washers

205

Chapter 5

8. Step 1: The y-intercept is 3. So, graph (0, 3).


2
Step 2: The slope is 1. From (0, 3), move up
2 units and right 1 unit. Draw a dot.
Step 3: Draw a line containing the points.

12. The graph passes through (0, 50) with slope 5.


T
80
60

T  50  5w

y
40

y  2x  3

20
0

Pages 275277

y  3x  1

16. Replace m with


y  mx  b

10. Step 1: Solve for y to find the slope-intercept


form.
2x  y  5
2x  y  2x  5  2x
y  2x  5
Step 2: The y-intercept is 5.
So, graph (0, 5).
2
Step 3: The slope is 1 . From (0, 5), move down
2 units and right 1 unit. Draw a dot.
Step 4: Draw a line containing the points.

y  5 x  0
3

y  5 x
18. Replace m with 1 and b with 10.
y  mx  b
y  1x  10
y  x  10
19. Replace m with 0.5 and b with 7.5.
y  mx  b
y  0.5x  7.5
20. Find the slope.
y2  y1

mx
m

11. Words:

The amount you save increases $5


per week, so the rate of change is $5
per week. You start with $50.
Variables: T is the total amount. W is the
number of weeks.
Equation:
Total
rate of
number of weeks
equals
times
amount
change
from now

plus

 x1

4  1
1  0

m3
The line crosses the y-axis at (0, 1).
So, the y-intercept is 1.
Finally, write the equation.
y  mx  b
y  3x  1

amount
at start.

14243 1
424
3 14243 1
424
3 14444244443 1
424
3 14243

50

The equation is T  5w  50.

Chapter 5

and b with 3.

1
2

17. Replace m with 5 and b with 0.


y  mx  b

Practice and Apply

2x  y  5

y  2x  3

14. Replace m with 2 and b with 6.


y  mx  b
y  2x  162
y  2x  6
15. Replace m with 3 and b with 5.
y  mx  b
y  3x  152
y  3x  5

13. After 7 weeks, w  7.


T  5w  50
T  5172  50
T  85
So, the total amount saved after 7 weeks is $85.

9. Step 1: The y-intercept is 1. So, graph (0, 1).


3
Step 2: The slope is 1 . From (0, 1), move down
3 units and right 1 unit. Draw a dot.
Step 3: Draw a line containing the points.

206

So, the y-intercept is 2.


Finally, write the equation.
y  mx  b
y  0x  2
y2
26. The slope of a horizontal line is 0.
Since the line crosses the y-axis at (0, 5), the
y-intercept is 5. Replace m with 0 and b with
5.
y  mx  b
y  0x  (5)
y  5
27. Since the line passes through the origin, the
y-intercept is 0. Replace m with 3 and b with 0.
y  mx  b
y  3x  0
y  3x
28. The y-intercept is 1. So, graph (0, 1). The slope is
3
. From (0, 1), move up 3 units and right 1 unit.
1
Draw a dot. Draw a line containing the points.

21. Find the slope.


y2  y1

mx

m
m

 x1

1  (4)
2  0
3
2

The line crosses the y-axis at (0, 4). So, the


y-intercept is 4.
Finally, write the equation.
y  mx  b
3

y  2 x  (4)
3

y  2x  4
22. Find the slope.
y2  y1

mx

m
m

 x1

2  2
1  0
4
1

m  4
The line crosses the y-axis at (0, 2).
So, the y-intercept is 2.
Finally, write the equation.
y  mx  b
y  4x  2
23. Find the slope.

y
y  3x  1

y2  y1

mx

m
m
m

 x1

1  1
3  0
2
3
2
3

29. The y-intercept is 2. So, graph (0, 2). The slope
1
is 1. From (0, 2), move up 1 unit and right
1 unit. Draw a dot. Draw a line containing the
points.

The line crosses the y-axis at (0, 1).


So, the y-intercept is 1.
Finally, write the equation.
y  mx  b

yx2

y  3 x  1

24. Find the slope.

y2  y1

mx

m
m

 x1

3  0
2  0
3
2

30. The y-intercept is 1. So, graph (0, 1). The slope


4
is 1 . From (0, 1) move down 4 units and right
1 unit. Draw a dot. Draw a line containing the
points.

The line crosses the y-axis at (0, 0).


So, the y-intercept is 0.
Finally, write the equation.
y  mx  b

y  2x  0
3

y  2x
25. Find the slope.

y2  y1

mx

m
m

y  4x  1

 x1

2  2
2  0
0
2

m0
The line crosses the y-axis at (0, 2).

207

Chapter 5

35. Solve for y to find the slope-intercept form.


2x  y  3
2x  y  2x  3  2x
y  2x  3
1(y)  1(2x  3)
y  2x  3
The y-intercept is 3. So, graph (0, 3). The slope is
2
. From (0, 3) move up 2 units and right 1 unit.
1
Draw a dot. Draw a line containing the points.

31. The y-intercept is 2. So, graph (0, 2). The slope is


1
. From (0, 2) move down 1 unit and right 1 unit.
1
Draw a dot. Draw a line containing the points.
y

y  x  2

32. The y-intercept is 4. So, graph (0, 4). The slope is


1
. From (0, 4) move up 1 unit and right 2 units.
2
Draw a dot. Draw a line containing the points.

2x  y  3

36. Solve for y to find the slope-intercept form.


3y  2x  3

y  2x  4

3y
3
3y
3




y
33. The y-intercept is 3. So, graph (0,3). The slope
1
is 3 . From (0, 3) move down 1 unit and right
3 units. Draw a dot. Draw a line containing the
points.

2x  3
3
2x
3
3
3
2
x1
3

The y-intercept is 1. So, graph (0, 1). The slope is


2
. From (0, 1) move up 2 units and right 3 units.
3
Draw a dot. Draw a line containing the points.
y

O
3y  2x  3
1

y 3x  3

37. Solve for y to find the slope-intercept form.


2y  6x  4

34. Solve for y to find the slope-intercept form.


3x  y  2
3x  y  3x  2  3x
y  3x  2
The y-intercept is 2. So, graph (0, 2). The slope
3
is 1 . From (0, 2) move down 3 units and right
1 unit. Draw a dot. Draw a line containing the
points.

2y
2
2y
2




6x  4
2
6x
4
 2
2

y  3x  2
The y-intercept is 2. So, graph (0, 2). The slope
3
is 1 . From (0, 2) move down 3 units and right
1 unit. Draw a dot. Draw a line containing the
points.

y
O

y
x

O
3x  y  2
2y  6x  4

Chapter 5

208

41. The cost increases $25 per hour, so the rate of


change is $25 per hour. The initial cost is $50.
Let C  total cost.
Let h  number of hours used for repair.
number
of hours
Total
rate of
used for
initial
cost
equals
change
times
repair
plus
cost.
123 123 1
424
3 123 1
424
3 123 123

38. Solve for y to find the slope-intercept form.


2x  3y  6
2x  3y  2x  6  2x
3y  2x  6
3y
3
3y
3




y

2x  6
3
2x
6
3
3
2
3x  2

C

25

The equation is C  25h  50.

The y-intercept is 2. So, graph (0, 2). The slope is


2
. From (0, 2) move down 2 units and right
3
3 units. Draw a dot. Draw a line containing the
points.

y

The y-intercept is 1. So, graph (0, 1). The slope


4
is 3. From (0, 1) move up 4 units and right
3 units. Draw a dot. Draw a line containing the
points.

4x  3y  3

 6.

S

1

t

16
The equation is S  t  16.
46. The year 2005 is 14 years after 1991.
So, t  14.
S  t  16
S  14  16
S  30
So, the total sales will be $30 billion in 2005.

40. The cost increases $2 per hour, so the rate of


change is $2 per hour. The initial cost is $20.
Let C  total cost.
Let t  number of hours bicycle is rented.
number
Total
rate of
of hours
initial
cost
change
times
rental
plus
cost.
1
23 equals
123 1
424
3 12
3 of
1
424
31
23 123
t


1
2 t


T

2
h

15
The equation is T  2h  15.
44. They all have a y-intercept of 3.
45. The amount of sales increased $1 billion per year,
so the rate of change is $1 billion per year. In
1991 there were $16 billion in book sales.
S is the total sales. t is the number of years after
1991.
number
rate
of years
amount
Total
of
after
at
sales
equals
change
times
1991
plus
start.
123 123 1
424
3 123 1
424
3 123 1
424
3

C

2

The equation is C  2t  20.

2

43. The temperature will fall 2 each hour, so the rate


of change is 2 each hour. The temperature is
currently 15.
Let T  temperature.
Let h  number of hours passed during the night.
rate
number
temperTemperof
of hours
ature
ature
equals
change
times
passed
plus
at
start.
1
424
3 123 1
424
3 123 1
424
3 123 1
424
3

4x  3
3
4x
3
 3
3
4
x1
3

The equation is H 

39. Solve for y to find the slope-intercept form.


4x  3y  3
4x  3y  4x  3  4x
3y  4x  3


50

2x  3y  6

3y
3
3y
3

42. The candle decreases in height 2 in. per hour, so


1
the rate of change is 2 in. per hour. The candle
is 6 in. tall when lit.
Let H  height of the candle.
Let t  number of hours the candle burns.
number
Height
rate
of hours
height
of
of
candle
at
candle
equals
change
times
burns
plus
start.
123 123 1
424
3 123 1
424
3 123 123

20

209

Chapter 5

55a. Replace A with 2, B with 1, and C with 4.

47. The fatality rate decreased 0.12 each year, so the


rate of change is 0.12 each year. In 1966 there
were 5.5 fatalities per 100 million vehicle miles.
R is the fatality rate. t is the number of years
after 1966.
number
rate
of years
amount
Fatality
of
after
at
rate
equals
change
times
1966
plus
start.
14243 123 1
424
3 123 1
424
3 123 1
424
3

B  1
 2
The slope is 2.
C
B

4
1

 4
The y-intercept is 4.
55b. Replace A with 3, B with 4, and C with 12.

R
 0.12

t

5.5
The equation is R  0.12t  5.5.
48. The graph passes through (0, 5.5) with slope
0.12.

B  4
3

The slope is 4.


C
B

12
4

3
The y-intercept is 3.
55c. Replace A with 2, B with 3, and C with 9.
A

B  3

R  5.5  0.12t

3
2

The slope is 3.

C
B

49. The year 1999 is 33 years after 1966.


So, t  33.
R  0.12t  5.5
R  0.12(33)  5.5
R  1.54
So, the fatality rate was 1.54 fatalities per
100 million vehicle miles in 1999.
50. The y-intercept is the flat fee in an equation that
represents a price. Answers should include the
following.
The graph crosses the y-axis at 5.99.
Sample answer: A mechanic charges $25 plus
$40 per hour to work on your car.
51. D; Solve for y to find the slope-intercept form.
2x  y  5
2x  y  2x  5  2x
y  2x  5
(1)(y)  (1)(2x  5)
y  2x  5
The y-intercept is 5.
52. B; The y-intercept is 100, which indicates that
you already have $100. The slope is 5 which
indicates that you are adding or saving $5 for
each time frame.
53.
Ax  By  C
Ax  By  Ax  C  Ax
By  Ax  C
Ax  C
B
Ax
C
 B B
A
C
y  B x  B, where B  0.
A
C
A
slope of y  B x  B is B, where B
C
y-intercept is B.
By
B
By
B

54. The
The

Chapter 5

 3

 3
The y-intercept is 3.

Page 277

Maintain Your Skills

56. Find the value of k.


y  kx
45  k(60)
45
60
3
4

k(60)
60

k
3

Therefore, y  4x.
Now find x when y  8.
3

y  4x
3

8  4x
4
(8)
3
32
3
2
103

1 2

4 3
x
4

3

x
x

57. Find the value of k.


y  kx
15  k(4)
15
4
15
4

k(4)
4

k

Therefore, y 

15
x.
4

Now find y when x  10.


y
 0.

y
y

210

15
x
4
15
(10)
4
75
1
or 372
2

y2  y1

58. m  x

 x1
6  0
 (3)

m  4
6

y2  y1

59. m  x

m

Notice that the graph of y  4 is the same as


the graph of y  0, moved 4 units down. Also, the
graph of y  7 is the same as the graph of y  0,
moved 7 units up. All graphs have the same slope
and different intercepts.
Because they all have the same slope, this family
of graphs can be described as linear graphs with a
slope of 0.
2. Enter the equations in the Y  list and graph.

 x1

4  (1)
3  3
3
0

m  1

m

m  6

Since division by zero


is undefined, the slope
is undefined.

y2  y1

60. m  x

m
m

 x1

2  (5)
9  5
7
4

61. Write each number as a decimal.


2.5
3
4

 0.75

[10, 10] scl: 1 by [10, 10] scl: 1

0.5
7
8

The graph of y  x  1 has a slope of 1 and


a y-intercept of 1.
The graph of y  2x  1 has a slope of 2 and a
y-intercept of 1.
1
1
The graph of y  4 x  1 has a slope of 4 and a
y-intercept of 1.
These graphs have the same intercept and
different slopes. This family of graphs can be
described as linear graphs with a y-intercept of 1.
3. Enter the equations in the Y  list and graph.

 0.875

0.5 0.75 0.875 2.5


The numbers arranged in order from least to
3 7
greatest are 0.5, 4, 8, 2.5.
62. x 
x

15  9
2
6
2

63. 3(7)  2  b
21  2  b

x3
64. q  62  22
q  36  4
q  32

y2  y1

65. m  x

m
m

y2  y1

66. m  x

 x1

8  8
 5
0
7

y2  y1
2

m

m

m

Page 279

y  x 4

m  2
67. m  x

m  2
m0

 x1

2  2
1  (1)
4
2

m

y  2x  4

 x1

13  (1)
10  1
12
9
4
3

y  2x  4

[10, 10] scl: 1 by [10, 10] scl: 1

The graph of y  x  4 has a slope of 1 and a


y-intercept of 4.
The graph of y  2x  4 has a slope of 2 and a
y-intercept of 4.
The graph of y  2x  4 has a slope of 2 and a
y-intercept of 4.
These equations are similar in that they all have
positive slope. However, since the slopes are
different and the y-intercepts are different, these
graphs are not all in the same family.

Graphing Calculator Investigation


(Follow-Up of Lesson 5-3)

1. Enter the equations in the Y  list and graph.

[10, 10] scl: 1 by [10, 10] scl: 1

The graph of y  4 has a slope of 0 and a


y-intercept of 4.
The graph of y  0 has a slope of 0 and a
y-intercept of 0.
The graph of y  7 has a slope of 0 and a
y-intercept of 7.

211

Chapter 5

4. Enter the equations in the Y  list and graph.

Because they all have the same slope, this family


of graphs can be described as linear graphs with a
slope of 3.
7. Sample answer: The value of m determines the
steepness and direction of the graph. If the graph
has a positive slope, it slants upward from left to
right. A graph with a negative slope slants
downward from left to right. The greater the
absolute value of the slope, the steeper the line.
Lines with the same slope are parallel. The value
of b determines where the line crosses the y-axis.
Lines with the same value of b form a family of
lines that intersect at that intercept. The values
of b in a family of parallel lines determine how far
the lines are apart on the y-axis.
8. This class of functions has graphs that are lines
with slope 1. Their y-intercepts are all different.
9. See students graphs. The graph of y  0 x 0  c is
the same as the graph of y  0 x 0 translated
vertically c units. If c is positive, the translation
is up; if c is negative, the translation is down. The
graph of y  0 x  c 0 is the same as the graph of
y  0 x 0 , translated horizontally c units. If c is
positive, the translation is to the left; if c is
negative, the translation is to the right.

y 4x4

y 3x3

y 2x2

[10, 10] scl: 1 by [10, 10] scl: 1


1

The graph of y  2x  2 has a slope of 2 and a


y-intercept of 2.
1
1
The graph of y  3x  3 has a slope of 3 and a
y-intercept of 3.
1
1
The graph of y  4x  4 has a slope of 4 and a
y-intercept of 4.
These equations are similar in that they all have
positive slope. However, since the slopes are
different and the y-intercepts are different, these
graphs are not all in the same family.
5. Enter the equations in the Y  list and graph.
y  2x  2

y  2x  2

y 2x2
[10, 10] scl: 1 by [10, 10] scl: 1

The graph of y  2x  2 has a slope of 2


and a y-intercept of 2.
The graph of y  2x  2 has a slope of 2 and a
y-intercept of 2.
1

Writing Equations in SlopeIntercept Form

Page 283

Check for Understanding

1. When you have the slope and one point, you can
substitute these values in for x, y, and m to find b.
When you are given two points, you must first
find the slope and then use the first procedure.
2. Sample answer: y  2x  3
3. Sometimes; if the x- and y-intercepts are both
zero, you cannot write the equation of the graph.
4. Step 1: The line has slope 2. To find the
y-intercept, replace m with 2 and (x, y)
with (4, 2) in the slope-intercept form.
Then solve for b.
y  mx  b
2  2(4)  b
2  8  b
2  8  8  b  8
10  b
Step 2: Write the slope-intercept form using
m  2 and b  10.
y  mx  b
y  2x  (10)
y  2x  10
Therefore, the equation is y  2x  10.

The graph of y  2x  2 has a slope of 2 and a


y-intercept of 2.
These graphs have the same intercept and different
slopes. This family of graphs can be described as
linear graphs with a y-intercept of 2.
6. Enter the equations in the Y  list and graph.
y  3x  6
y  3x

y  3x  7
[10, 10] scl: 1 by [10, 10] scl: 1

The graph of y  3x has a slope of 3 and a


y-intercept of 0.
The graph of y  3x  6 has a slope of 3 and a
y-intercept of 6.
The graph of y  3x  7 has a slope of 3 and a
y-intercept of 7.
Notice that the graph of y  3x  6 is the same as
the graph of y  3x, moved 6 units up. Also, the
graph of y  3x  7 is the same as the graph of
y  3x, moved 7 units down. All graphs have the
same slope and different intercepts.

Chapter 5

5-4

212

5. Step 1: The line has slope 3. To find the


y-intercept, replace m with 3 and (x, y)
with (3, 7) in the slope-intercept form.
Then solve for b.
y  mx  b
7  3(3)  b
7  9  b
7  9  9  b  9
16  b
Step 2: Write the slope-intercept form using
m  3 and b  16.
y  mx  b
y  3x  16
Therefore, the equation is y  3x  16.
6. Step 1: The line has slope 1. To find the
y-intercept, replace m with 1 and (x, y)
with (3, 5) in the slope-intercept form.
Then solve for b.
y  mx  b
5  1(3)  b
53b
533b3
2b
Step 2: Write the slope-intercept form using
m  1 and b  2.
y  mx  b
y  1x  2
y  x  2
Therefore, the equation is y  x  2.
7. Step 1: Find the slope of the line containing the
points.
Let (x1, y1)  (5, 1) and (x2, y2)  (8, 2).

8. Step 1: Find the slope of the line containing the


points.
Let (x1, y1)  (6, 0) and (x2, y2)  (0, 4).
y2  y1

mx

m
m
m

Step 2: You know the slope and two points. The


point (0, 4) lies on the y-axis. Thus, the
y-intercept is 4.
Step 3: Write the slope-intercept form using
2
m  3 and b  4.
y  mx  b
2

y  3x  4
2

Therefore, the equation is y  3x  4.


9. Step 1: Find the slope of the line containing the
points.
Let (x1, y1)  (5, 2) and (x2, y2)  (7, 4).
y2  y1

mx

m
m
m

m
m

 x1

4  2
7  5
6
12
1
2

Step 2: You know the slope and two points.


Choose one point and find the
y-intercept. In this case, we choose (5, 2).
y  mx  b
1

2  2 (5)  b

y2  y1

mx

 x1

4  0
0  6
4
6
2
3

 x1

22b

2  1
8  5
3
3

222b2
1

2  b

m  1
Step 2: You know the slope and two points.
Choose one point and find the
y-intercept. In this case, we choose (5, 1).
y  mx  b
1  1(5)  b
1  5  b
1  5  5  b  5
6b
Step 3: Write the slope-intercept form using
m  1 and b  6.
y  mx  b
y  1x  6
y  x  6
Therefore, the equation is y  x  6.

Step 3:

Write the slope-intercept form using


1

m  2 and b  2.
y  mx  b
1

1 12

y  2x  2
1

y  2x  2

Therefore, the equation is y  2x  2.

213

Chapter 5

14. Find the y-intercept.


y  mx  b
4  5(5)  b
4  25  b
4  25  25  b  25
29  b
Write the slope-intercept form.
y  mx  b
y  5x  29
15. Find the y-intercept.
y  mx  b
0  2(3)  b
0  6  b
0  6  6  b  6
6b
Write the slope-intercept form.
y  mx  b
y  2x  6
16. Find the y-intercept.
y  mx  b

10. A; Read the Test Item


The table represents the ordered pairs
(5, 2) and (0, 7).
Solve the Test Item
Step 1: Find the slope of the line containing the
points.
Let (x1, y1)  (5, 2) and (x2, y2)  (0, 7).
y2  y1

mx

m

 x1

7  2
0  (5)
5

m5
m1
Step 2: You know the slope and two points. The
point (0, 7) lies on the y-axis. Thus, the
y-intercept is 7.
Step 3: Write the slope-intercept form using
m  1 and b  7.
y  mx  b
y  1x  7
yx7
Therefore, the equation is y  x  7.

Pages 284285

3  2 (5)  b
5

32b

Practice and Apply

11. Find the y-intercept.


y  mx  b
2  3(1)  b
23b
233b3
1  b
Write the slope-intercept form.
y  mx  b
y  3x  (1)
y  3x  1
12. Find the y-intercept.
y  mx  b
1  1(4)  b
1  4  b
1  4  4  b  4
3b
Write the slope-intercept form.
y  mx  b
y  1x  3
y  x  3
13. Find the y-intercept.
y  mx  b
2  3(5)  b
2  15  b
2  15  15  b  15
17  b
Write the slope-intercept form.
y  mx  b
y  3x  (17)
y  3x  17

Chapter 5

1
2

b

322b2
Write the slope-intercept form.
y  mx  b
1

y  2x  2
17. Find the y-intercept.
y  mx  b
2

1  3 (3)  b
1  2  b
1  2  2  b  2
3  b
Write the slope-intercept form.
y  mx  b
2

y  3x  (3)
2

y  3x  3
18. Find the y-intercept.
y  mx  b
5

5  3 (3)  b
5  5  b
5  5  5  b  5
10  b
Write the slope-intercept form.
y  mx  b
5

y  3x  (10)
5

y  3x  10

214

Write the slope-intercept form.


y  mx  b
y  2x  (8)
y  2x  8
23. Find the slope.

19. Find the slope.


y2  y1

mx

m
m

 x1

2  1
5  4
1
1

y2  y1

mx

m1
Find the y-intercept.
y  mx  b
1  1(4)  b
14b
144b4
3  b
Write the slope-intercept form.
y  mx  b
y  1x  (3)
yx3
20. Find the slope.

m
m

m  2
Find the y-intercept.
y  mx  b
3  2(1)  b
32b
322b2
1b
Write the slope-intercept form.
y  mx  b
y  2x  1
24. Find the slope.

y2  y1

mx

 x1

0  2

m20
m

y2  y1

2
2

mx

m  1
From the graph, we see that the y-intercept is 2.
Write the slope-intercept form.
y  mx  b
y  1x  2
y  x  2
21. Find the slope.

m
m

m
m

 x1

4  2
2  4
6
6

m1
Find the y-intercept.
y  mx  b
2  1(4)  b
24b
244b4
2  b
Write the slope-intercept form.
y  mx  b
y  1x  (2)
yx2
22. Find the slope.

y2  y1

mx

m
m

m
m

 x1

2  (2)
4  7
0
11

m0
Find the y-intercept.
y  mx  b
2  0(7)  b
2  0  b
2  b
Write the slope-intercept form.
y  mx  b
y  0x  (2)
y  2

y2  y1

mx

 x1

2  (2)
3  2
4
1

m4
Find the y-intercept.
y  mx  b
2  4(3)  b
2  12  b
2  12  12  b  12
10  b
Write the slope-intercept form.
y  mx  b
y  4x  (10)
y  4x  10
25. Find the slope.

y2  y1

mx

 x1

3  3
2  (1)
6
3

 x1

4  (2)
6  3
6
3

m2
Find the y-intercept.
y  mx  b
4  2(6)  b
4  12  b
4  12  12  b  12
8  b

215

Chapter 5

Find the y-intercept.


y  mx  b

26. Find the slope.


y2  y1

mx

m
m

 x1

1  16  b

m
m
m

30. Find the slope of the line containing the points


(3, 0) and (0, 5).
y2  y1

 x1

mx

m
m

y  3x  5

b
b

31. Find the slope of the line containing the points


(3, 0) and (0, 4).

1
2

y2  y1

mx

b

m

Write the slope-intercept form.


y  mx  b
1

m

y  2x  2

m

28. Find the slope.


2

m
m
m

 x1

6  7
0  5
1
5
1
5

y  3x  4
32. Find the slope of the line containing the points
(6, 0) and (0, 3).
y2  y1

The point (0, 6) lies on the y-axis. Thus, the


y-intercept is 6.
Write the slope-intercept form.
y  mx  b

mx

m
m

y  5x  6

m

29. Find the slope.


2

m

 x1

3

4
1
4 
1

m

4

154 2

y  2x  3

1
1
m  4

Chapter 5

 x1

3  0
0  6
3
6
1
2

The y-intercept is 3.
Write the slope-intercept form.
y  mx  b

y2  y1

mx

 x1

4  0
0  3
4
3
4
3

The y-intercept is 4.
Write the slope-intercept form.
y  mx  b

y2  y1

mx

 x1

5  0
0  (3)
5
3

The y-intercept is 5.
Write the slope-intercept form.
y  mx  b

1
1
2
1
2

11

y  4x  16

1  2 (1)  b
1

b

y  mx  b

Find the y-intercept.


y  mx  b

1
2
1
2

11
16

4  1
7  1
3
6
1
2

1

Write the slope-intercept form using m  4 and


11
b  16.

y2  y1
2

1  16  16  b  16

m0
The point (0, 5) lies on the y-axis. Thus, the
y-intercept is 5.
Write the slope-intercept form.
y  mx  b
y  0x  5
y5
27. Find the slope.
mx

1 52

1  4 4  b

5  5
3  0
0
3

216

PQ249-6481F-05[217-243] 7/31/02 10:04 PM Page 217

Write the slope-intercept form.


W  mt  b

33. Find the slope of the line containing the points


(2, 0) and (0, 2).
y2  y1

mx

m

W  20t  (274.7)

 x1

W  20t  274.7

2  0
0  2

37. W  20t  274.7

2

m  2

W  20 (2005)  274.7

m1
The y-intercept is 2.
Write the slope-intercept form.
y  mx  b
y  1x  (2)
yx2
34. Write an equation of the line that passes through
(1970, 23.2) and (1998, 26.7). Find the slope.

38.

y2  y1

mx

m
m
m

 x1

26.7  23.2
1998  1970
3.5
28
1
8

Find the intercept.


M  mt  b

39.

23.2  8 (1970)  b
23.2  246.25  b
23.2  246.25  246.25  b  246.25
223.05  b
Write the slope-intercept form.
M  mt  b

40.

M  8t  (223.05)
1

M  8t  223.05
1

35. M  8t  223.05
1

M  8 (2005)  223.05
M  27.575
The median age of men who marry for the first
time in 2005 should be about 27.6 years old.
36. Write an equation of the line that passes through
(1970, 20.8) and (1998, 25).
Find the slope.

41.

y2  y1

mx

y2  y1

mx

m

 x1

m

25  20.8

m  1998  1970
m
m

W  26.05
The median age of women who marry for the first
time in 2005 should be about 26.05 years old.
Write an equation of the line that passes through
(1995, 175,000) with slope 2000.
Find the y-intercept.
y  mx  b
175,000  2000(1995)  b
175,000  3,990,000  b
175,000  3,990,000  3,990,000  b  3,990,000
3,815,000  b
Write the slope-intercept form.
y  mx  b
y  2000x  (3,815,000)
y  2000x  3,815,000
y  2000x  3,815,000
y  2000(2010)  3,815,000
y  205,000
The population of Orlando, Florida, in 2010
should be 205,000.
Write an equation of the line that passes through
(3, 45) with slope 10.
Find the intercept.
C  mh  b
45  10(3)  b
45  30  b
45  30  30  b  30
15  b
Write the slope-intercept form.
C  mh  b
C  10h  15
Find the slope.

m

4.2
28
3
20

 x1

4  2
7  14
6
21
2
7

Find the y-intercept.


y  mx  b

Find the intercept.


W  mt  b

2  7 (14)  b
24b
244b4
2  b

20.8  20 (1970)  b
20.8  295.5  b
20.8  295.5  295.5  b  295.5
274.7  b

Write the slope intercept form.


y  mx  b
2

y  7 x  (2)
2

y  7x  2

217

Chapter 5

PQ249-6481F-05[217-243] 7/31/02 10:04 PM Page 218

42. The slope-intercept form of the line is


y

2
x
7

Page 285

 2.
2

The slope is 7.
43. To find the x-intercept, let y  0.
22

y  13x 
22

0  13x 
22

22

0  13x  13x 
22
13x
13
22
22 13x

2

x

282
13
282
13
282
13

282
 13
13
282
22  13
141
11

22

y  3x  2

 13x

282

The y-intercept is  13 .

2.

Therefore, line  intersects the x-axis at


282
 13

49. Solve for y.


xy6
xyx6x
y  x  6
The y-intercept is 6. So, graph (0, 6). The slope is
1
. From (0, 6), move down 1 unit and right
1
1 unit. Draw a dot. Draw a line connecting the
points.

, 02
1141
11

44. Find the slope of the line containing the points


( p, 0) and (0, q).
y2  y1

mx

 x1

The slope-intercept form of the line is


22
282
y  13 x  13 .

and the y-axis at 0,

Maintain Your Skills

48. The y-intercept is 2. So, graph (0, 2). The slope
3
is 1. From (0, 2), move up 3 units and right
1 unit. Draw a dot. Draw a line connecting the
points.

q  0

m0p
q

xy6

m  p
q

m  p
The y-intercept is q.
Write the slope-intercept form.
y  mx  b

y  p x  q, where p  0.

50. Solve for y.


x  2y  8
x  2y  x  8  x
2y  x  8

45. Answers should include the following.


Linear extrapolation is when you use a linear
equation to predict values that are outside of
the given points on the graph.
You can use the slope-intercept form of the
equation to find the y value for any requested
x value.
46. B; Find the y-intercept.
y  mx  b

2y
2
2y
2

1  3  b
2

1  3  3  b  3
5
3

x  8
2
x
8
2
2
1
2x 

y
4
The y-intercept is 4. So, graph (0, 4). The slope is
1
. From (0, 4), move down 1 unit and right
2
2 units. Draw a dot. Draw a line connecting the
points.

1  3 (2)  b
2

b

x  2y  8

Write the slope-intercept form.


y  mx  b
1

y  3x  3

47. B; Write an equation of the line that passes


through (0, 560) with slope 20.
y  mx  b
y  20x  560

Chapter 5

equals rate times number of beats.


Volume
51. 1
4243 123 123
123 144424443
2.5

r

b
The equation is V  2.5b.

218

PQ249-6481F-05[217-243] 7/31/02 10:04 PM Page 219

52.
53.
54.
55.

The domain for this relation is {0, 4, 9}.


The domain for this relation is {2, 0, 5}.
3  5.
You can use a calculator to find an approximation
16
for 3 .

7.

4  4.0
16
3

 5.33333333...

Therefore, 4 

16
.
3

4y  12  3(x  1)
4y  12  3x  3
4y  12  12  3x  3  12
4y  3x  9
4y  3x  3x  9  3x
3x  4y  9
9.
y  3  2.5(x  1)
2(y  3)  2(2.5)(x  1)
2y  6  5(x  1)
2y  6  5x  5
2y  6  6  5x  5  6
2y  5x  11
2y  5x  5x  11 5x
5x  2y  11
(1)(5x  2y)  (1)11
5x  2y  11
10.
y  6  2(x  2)
y  6  2x  4
y  6  6  2x  4  6
y  2x  10

 0.66666666...
3
4

1 32

4(y  3)  4 4 (x  1)

 0.75

Therefore,

y  3  4 (x  1)

8.

56. You can use a calculator to find an approximation


2
for 3.
3
4
2
3

y  5  4(x  2)
y  5  4x  8
y  5  5  4x  8  5
y  4x  13
y  4x  4x  13  4x
4x  y  13
(1)(4x  y)  (1)13
4x  y  13

7 3.

57. 4  7  4  (7)
 (|7||4|)
 (7  4)
 3
58. 5  12  5  (12)
 (|12||5|)
 (12  5)
 7
59. 2  (3)  2  3
5
60. 1  4  1  (4)
 (|1|  |4|)
 (1  4)
 5
61. 7  8  7  (8)
 (|7|  |8|)
 (7  8)
 15
62. 5  (2)  5  2
 (|5||2|)
 (5  2)
 3

11.

y  3  3 (x  6)
2

y  3  3 x  4
2

y  3  3  3 x  4  3
2

y  3 x  1
12.

y  2  2x  2
7

5-5

y  2  2 (x  4)
7

y  2  2  2x  2  2

Writing Equations in Point-Slope


Form

y  2x  2
13. Step 1:

First find the slope of AD.


y2  y1

Page 289

mx

Check for Understanding

1. They are the coordinates of a given point on the


graph of the equation.
2. Akira; (2, 6) and (1, 6) are both on the line, so
either could be substituted into point-slope form
to find a correct equation.
3. Sample answer: y  2  4(x  1); y  4x  6
4. y  y1  m(x  x1)
y  3  2(x  1)
5.
y  y1  m(x  x1)
y  (2)  3[x  (1)]
y  2  3(x  1)
6. y  y1  m(x  x1)
y  (2)  0(x  2)
y20

m
m

 x1

1  3
3  112
4
2

m2
Step 2: You can use either point for (x1, y1) in
the point-slope form. We chose (1, 3).
y  y1  m(x  x1)
y  3  2[x  (1)]
y  3  2(x  1) or y  1  2(x  3)

219

Chapter 5

PQ249-6481F-05[217-243] 7/31/02 10:04 PM Page 220

14.

y  5  2(x  6)
y  5  2x  12
y  5  5  2x  12  5
y  2x  7
y  2x  2x  7  2x
2x  y  7
32.
y  3  5(x  1)
y  3  5x  5
y  3  3  5x  5  3
y  5x  8
y  5x  5x  8  5x
5x  y  8

y  3  2(x  1)
y  3  2x  2
y  3  3  2x  2  3
y  2x  5
y  2x  2x  5  2x
2x  y  5
1(2x  y)  1(5)
2x  y  5

Pages 289291

31.

Practice and Apply

15. y  y1  m(x  x1)


y  8  2(x  3)
16. y  y1  m(x  x1)
17. y  y1  m(x  x1)
y  (3)  1[x  (4)]
y  4  3[x  (2)]
y3x4
y  4  3(x  2)
18. y  y1  m(x  x1)
19. y  y1  m(x  x1)
y  1  4[x  (6)]
y  6  0[x  (3)]
y  1  4(x  6)
y60
20. y  y  m(x  x )
21.
y  y  m(x  x )
1
1
1
1
2

33.

2(y  7)  2

y  3  4 (x  8)

34.

22. y  y  m(x  x )
23.
y  y1  m(x  x1 )
1
1
2
5
y  3  3 [x  (6) ]
y  (3)  8 (x  1)
y3

2
3 (x

y3

 6)

y  y1  m(x  x1)
y  (5)  0(x  9)
y50
25. y  y  m(x  x )
1
1

5
8 (x

y8
y8

26.

156 2 (x  4)

6y  6  5(x  4)
6y  6  5x  20
6y  6  6  5x  20  6
6y  5x  14
6y  5x  5x  14  5x
5x  6y  14
(1)(5x  6y)  (1) (14)
5x  6y  14

 1)

y  y1  m(x  x1 )
8
y  (4)  3 (x  1)
35.

y  4  3 (x  1)

y  2  5 (x  8)

1 22

5(y  2)  5 5 (x  8)

27. The slope of a horizontal line is zero.


y  y1  m(x  x1)
y  (9)  0(x  5)
y90
28. The slope of a horizontal line is zero.
y  y1  m(x  x1)
y  7  0(x  0)
y70
29.
y  13  4(x  2)
y  13  4x  8
y  13  13  4x  8  13
y  4x  5
y  4x  4x  5  4x
4x  y  5
(1)(4x  y)  (1)5
4x  y  5
30.
y  3  3(x  5)
y  3  3x  15
y  3  3  3x  15  3
y  3x  12
y  3x  3x  12  3x
3x  y  12
(1)(3x  y)  (1)12
3x  y  12

Chapter 5

y  1  6 (x  4)
6(y  1)  6

24.

7
[x  (4) ]
2
7
(x  4)
2

112 2 (x  2)

2y  14  x  2
2y  14  14  x  2  14
2y  x  12
2y  x  x  12  x
x  2y  12
(1)(x  2y)  (1) (12)
x  2y  12

y  (3)  4 (x  8)

y  1  3 (x  9)

y  7  2 (x  2)

5y  10  2(x  8)
5y  10  2x  16
5y  10  10  2x  16  10
5y  2x  26
5y  2x  2x  26  2x
2x  5y  26
36.

y  4  3 (x  12)

1 12

3(y  4)  3 3 (x  12)
3y  12  1(x  12)
3y  12  x  12
3y  12  12  x  12  12
3y  x
3y  x  x  x
x  3y  0
37.

y  2  3 (x  6)
3(y  2)  3

153 2 (x  6)

3y  6  5(x  6)
3y  6  5x  30
3y  6  6  5x  30  6
3y  5x  24
3y  5x  5x  24  5x
5x  3y  24
(1)(5x  3y)  (1)24
5x  3y  24

220

PQ249-6481F-05[217-243] 7/31/02 10:04 PM Page 221

38.

y  6  2 (x  4)
2(y  6)  2

39.

40.

41.

43.

44.

45.

132 2 (x  4)

2y  12  3(x  4)
2y  12  3x  12
2y  12  12  3x  12  12
2y  3x  24
2y  3x  3x  24  3x
3x  2y  24
(1)(3x  2y)  (1)(24)
3x  2y  24
y  6  1.3(x  7)
10(y  6)  10(1.3)(x  7)
10y  60  13(x  7)
10y  60  13x  91
10y  60  60  13x  91  60
10y  13x  151
10y  13x  13x  151  13x
13x  10y  151
(1)(13x  10y)  (1)151
13x  10y  151
y  2  2.5(x  1)
2(y  2)  2(2.5)(x  1)
2y  4  5(x  1)
2y  4  5x  5
2y  4  4  5x  5  4
2y  5x  9
2y  5x  5x  9  5x
5x  2y  9
y  2  3(x  1)
42.
y  5  6(x  1)
y  2  3x  3
y  5  6x  6
y  2  2  3x  3  2
y  5  5  6x  6  5
y  3x  1
y  6x  11
y  2  2(x  5)
y  2  2x  10
y  2  2  2x  10  2
y  2x  8
y  1  7(x  3)
y  1  7x  21
y  1  1  7x  21  1
y  7x  22

y  5  5  5 x  6  5
2

y  5 x  1
1

y

y
y

1
4

1
4
1
4




y
3

y5

52.

y
y

3
5

3
5
3
5

2
2x  3
2
2x  3 
1
2x  3
1
3 x  2
3
3x  2
3
3x  2 
7
3x  4
1
4 x  2

1
3

 4x  2

1
4

 4x  2  5
7

y  4x  5
53. Point-slope form:
y  y1  m(x  x1)
y  (3)  10(x  5)
y  3  10(x  5)
Slope-intercept form:
y  3  10(x  5)
y  3  10x  50
y  3  3  10x  50  3
y  10x  53
Standard form:
y  10x  53
y  10x  10x  53  10x
10x  y  53
(1)(10x  y)  (1)(53)
10x  y  53
Slope-intercept form:
54. Point-slope form:
3

y  y1  m(x  x1 )
3
y  (6)  2 (x  1)

y  1  3 (x  9)

y

y4

51.

y33

1
3

1
3
1
3

y

y  2x  1

y3

y

y3

y  3  2 x  3

50.

y  3  3  2x  2  3

47.

yx1

y

y22x22

y11

y2x2

49.

y  3  2 (x  4)

y1

y  5  5 x  6

y  3  2x  2

46.

y  5  5 1x  152

48.

y  6  2 (x  1)

y  6  2 (x  1)

2
x6
3
2
x61
3
2
x7
3
1
4 1x  22
1
1
4 x  2
1
1
4 x  2  3
1
7
4 x  2

15
2

y  6  2x  2
y  6  6  2x  2  6
y  2x 

Standard form:
3

y  2x 

13

15
2

2(y)  2 2 x 

15
2

2y  3x  15
2y  3x  3x  15  3x
3x  2y  15
(1)(3x  2y)  (1) (15)
3x  2y  15

221

Chapter 5

PQ249-6481F-05[217-243] 7/31/02 10:04 PM Page 222

55. The rate of change is $5 per week, so m  5.


Write the point-slope form using m  5 and
(x1, y1)  (12, 210).
y  y1  m(x  x1)
y  210  5(x  12)
56.
y  210  5(x  12)
y  210  5x  60
y  210  210  5x  60  210
y  5x  150
57. The flat fee is the y-intercept of the equation. The
y-intercept of y  5x  150 is 150. Therefore, the
flat fee for installation is $150.
58. The rate of change is 1500 each year, so
m  1500. Write the point-slope form using
m  1500 and (x1, y1)  (1996, 29,690).
y  y1  m(x  x1)
y  29,690  1500(x  1996)
59.
y  29,690  1500(x  1996)
y  29,690  1500x  2,994,000
y  29,690  29,690  1500x  2,994,000  29,690
y  1500x  2,964,310
60. y  1500x  2,964,310
y  1500(2005)  2,964,310
y  43,190
The number of movie screens in the United States
should be 43,190 in 2005.
61. RQ: First, find the slope.

Method 1

62. RQ: y  3  2 (x  1)

m

m

or

63. RQ:

Method 2
y  y1  m(x  x1 )
1
y  (1)  2 (x  3)

PS:

2
Method 2
y  y1  m(x  x1 )
y  3  2(x  1)

1
2

Method 2
y  y1  m(x  x1 )
1
y  1  2 [x  (3) ]

 x1

Chapter 5

or

11

y  2x  2

11

2y  x  5
2y  x  x  5  x
x  2y  5
x  2y  5
RS: y  2x  5
y  2x  2x  5  2x
2x  y  5
64. Sample answer: The point-slope form of the
equation is y  1  (x  9). Let x  10 and y  0.
The equation becomes 0  1  (10  9) or
1  1. Since the equation holds true, (10, 0) is a
point on the line passing through (9, 1) and (5, 5).
65. Answers should include the following.
Write the definition of the slope using (x, y) as
one point and (x1, y1) as the other. Then solve
the equation so that the ys are on one side and
the slope and xs are on the other.

y2  y1

m

y  2x  2

2(y)  2 2 x  2

RS: First, find the slope.


2

2y  x  5
2y  x  x  5  x
x  2y  5
x  2y  5
QP: y  2x  5
y  2x  2x  5  2x
2x  y  5

1  (3)
3  (1)

2(y)  2 2 x  2

y  1  2 (x  3)
mx

RS: y  3  2(x  1)
y  3  2x  2
y  3  3  2x  2  3
y  2x  5

Method 1
or
y  y1  m(x  x1 )
1
y  3  2 (x  1)

y  2x  2

 x1

1  3
 1

m  3

y  3  3  2x  2  3

y2  y1

mx

y  3  2x  2

y  1  2 (x  3)

Method 1
or
y  y1  m(x  x1 )
y  (1)  2(x  3)
y  1  2(x  3)
PS: First, find the slope.

 x1

or

PS: y  3  2 (x  1)

y2  y1
3  (1)
1  3

QP: y  1  2(x  3)
y  1  2x  6
y  1  1  2x  6  1
y  2x  5

QP: First, find the slope.


mx

y  2x  2

y  3  2 (x  1)

y  3  3  2x  2  3

or 2

Method 1
or
y  y1  m(x  x1 )
1
y  (3)  2 [x  (1) ]

y  3  2x  2

 x1

1  (3)
3  (1)

Method 2

y  y1  m(x  x1 )
y  y1  m(x  x1 )
y  (3)  2[x  (1) ] y  1  2[x  (3) ]
y  3  2(x  1)
y  1  2(x  3)

y2  y1

mx

or

2

222

PQ249-6481F-05[217-243] 7/31/02 10:04 PM Page 223

66. D; x  3y  15
x  3y  x  15  x
3y  x  15
3y
3

x  15
3
1
y  3x  5
1
Therefore, m  3 and

Page 291

the line passes through (0, 5).

67. Write the point-slope form. Then solve for y.


y  y1  m(x  x1)
y  (5)  m(x  2)
y  5  m(x  2)
y  5  mx  2m
y  5  5  mx  2m  5
y  mx  2m  5
68. (1, 3) and (0, 1):
First, find the slope.

y2  y1

mx

m

y2  y1

mx

 x1

1  3
(1)

m0

m

or 2

y2  y1
2

m

 x1

1  1
1  0

or 2

Write the point-slope form. Then solve for y.


y  y1  m(x  x1)
y  1  2(x  0)
y  1  2x
y  1  1  2x  1
y  2x  1
(1, 1) and (2, 3):
First, find the slope.

a5

y2  y1

mx

m

2
v
9

78.

 x1

3  (1)
2  1

 x1

6  (4)
0  2
10
2

m  5
The point (0, 6) lies on the y-axis. The y-intercept
is 6.
Write the slope-intercept form.
y  mx  b
y  5x  6
75. The line has slope 0. Find the y-intercept.
y  mx  b
1  0(1)  b
1  0  b
1  b
Write the slope-intercept form.
y  mx  b
y  0x  (1)
y  1
76.
77.
4a  5  15
7  3c  11
4a  5  5  15  5
7  3c  7  11  7
4a  20
3c  18
4a
20
3c
18

 3
4
4
3

Write the point-slope form. Then solve for y.


y  y1  m(x  x1)
y  1  2(x  0)
y  1  2x
y  1  1  2x  1
y  2x  1
(0, 1) and (1, 1):
First, find the slope.
mx

Maintain Your Skills

72. y  mx  b
y  2x  (5)
y  2x  5
73. Find the y-intercept.
y  mx  b
4  3(2)  b
4  6  b
4  6  6  b  6
10  b
Write the slope-intercept form.
y  mx  b
y  3x  10
74. Find the slope.

2
v
9

or 2

 6  6  14  6
2
v
9
9 2
v
2 9

Write the point-slope form. Then solve for y.


y  y1  m(x  x1)
y  (1)  2(x  1)
y  1  2(x  1)
y  1  2x  2
y  1  1  2x  2  1
y  2x  1
69. All of the equations are the same.
70. The equation will be y  2x  1; see students
work.
71. Regardless of which two points on a line you
select, the slope-intercept form of the equation
will always be the same.

c  6

 6  14
 20

1 2  92 (20)

v  90
79. (25  4) (22  13)  21 (4  1)
 21 3
7
2 # 1
1
80.
1

The reciprocal of 2 is 2.
81.

10
1

 10  1

The reciprocal of 10 is

1
.
10

82. 1  1  1
The reciprocal of 1 is 1.

223

Chapter 5

PQ249-6481F-05[217-243] 7/31/02 10:04 PM Page 224

83.

1
1

The reciprocal of 1 is
84.

2
3

5. The line parallel to y  x  5 has the same slope,


1. Replace m with 1, and (x1, y1) with (2, 3) in the
point-slope form.
y  y1  m(x  x1 )
y  3  1(x  2)
y3x2
y33x23
yx1
6. The line parallel to y  2x  1 has the same
slope, 2. Replace m with 2, and (x1, y1) with
(1, 3) in the point-slope form.
y  y1  m(x  x1 )
y  (3)  2(x  1)
y  3  2x  2
y  3  3  2x  2  3
y  2x  5
7. Write the given equation in slope-intercept form.
3x  y  4
3x  y  3x  4  3x
y  3x  4
The line parallel to y  3x  4 has the same
slope, 3. Replace m with 3, and (x1, y1) with
(2, 2) in the point-slope form.
y  y1  m(x  x1 )
y  2  3[x  (2) ]
y  2  3(x  2)
y  2  3x  6
y  2  2  3x  6  2
y  3x  8
8. Find the slope of each segment.

 1  1
3
2

or 1.

1

The reciprocal of

1 92

1
1

2
3

is 2.

85. 9  1  1
1

The reciprocal of 9 is 9.


86.

5
2

51

The reciprocal of
2

1 32

5
2

is 5.

87. 3  2  1
2

The reciprocal of 3 is 2.

Geometry: Parallel and


Perpendicular Lines

5-6
Page 293

Algebra Activity

1. Sample answer: B is 3 units right of the origin


and 6 units up. Therefore, B  (3, 6).
2. Sample answer: Find the slope of the line
containing (0, 0) and (3, 6).
y2  y1

mx

 x1

6  0

m  3  0 or 2
3. Sample answer: B is 6 units left of the origin and
3 units up. Therefore, B  (6, 3).
4. Sample answer: Find the slope of the line
containing (0, 0) and (6, 3).
y2  y1

mx

Slope of BD: m 

 x1

3  0
 0

m  6

6
7

5. See students work.


6. They are perpendicular.
7. The x- and y-coordinates are reversed, and the
x-coordinate is multiplied by 1.
8. They are opposite reciprocals.
9. Their product is 1.

1. The slope is
2
which is 3.

Step 2: The slope of the given line is 3. So, the


slope of the line perpendicular to this
1
line is the opposite reciprocal to 3 or 3.
Step 3: Use the point-slope form to find the
equation.
y  y  m(x  x )
1
1
y  1  3[ x  (3) ]
y  1  3(x  3)
y  1  3x  9
y  1  1  3x  9  1
y  3x  8

so find the opposite reciprocal of 2,


1

2. Sample answer: 2, 2
3. Parallel lines lie in the same plane and never
intersect. Perpendicular lines intersect at right
angles.
4. The line parallel to y  2x  4 has the same
slope, 2. The point (0, 1) lies on the y-axis, so
the y-intercept is 1. Replace m with 2, and b
with 1 in the slope-intercept form.
y  mx  b
y  2x  (1)
y  2x  1

Chapter 5

9. Step 1: The slope of y  3x  2 is 3.

Check for Understanding


3
,
2

123 2  1.

The line segments are not perpendicular because

or 2

Pages 295296

7  1
6
or 7
(2)
5  3
2
or 3
0  3

Slope of AC: m  5 

224

PQ249-6481F-05[217-243] 7/31/02 10:04 PM Page 225

10. Step 1:

15. The line parallel to y  x  6 has the same


slope, 1.
y  y1  m(x  x1 )
y  2  1[x  (3) ]
y2x3
y22x32
yx5
16. The line parallel to y  2x  1 has the same
slope, 2.
y  y1  m(x  x1 )
y  (1)  2(x  4)
y  1  2x  8
y  1  1  2x  8  1
y  2x  9

The slope of y  5x  4 is 5.
3
.
5

Step 2: The slope of the given line is So, the


slope of the line perpendicular to this
3
5
line is the opposite reciprocal of 5 or 3.
Step 3: Use the point-slope form to find the
equation.
y  y  m(x  x )
1
1
5

y  (2)  3 (x  6)
5

y  2  3 (x  6)
5

y  2  3x  10
5

y  2  2  3x  10  2
5

y  3x  8

17. The line parallel to y  2x  1 has the same


1
slope, 2.

11. Step 1:

Find the slope of the given line.


2x  y  5
2x  y  2x  5  2x
y  2x  5
Step 2: The slope of the given line is 2. So, the
slope of the line perpendicular to this
1
line is the opposite reciprocal of 2 or 2.

y  y1  m(x  x1 )
1
y  (4)  2 [ x  (5) ]
1

y  4  2 (x  5)
5

3
2

18. The line parallel to y  3x  1 has the same


2
slope, 3.

1
1

y  2  2x  1

y  y1  m(x  x1 )
2
y  3  3 (x  3)

y  2  2  2x  1  2
1

y  2x  3

y  3  3x  2

12. The slope of the given line is 3. So, the slope of


the line perpendicular to this line is the opposite
1
reciprocal of 3 or 3. Use the point-slope form to
find the equation.
y  y1  m(x  x1 )

y  3  3  3x  2  3
2

y  3x  1
1

19. The line parallel to y  3x  3 has the same


1
slope, 3.

y  5  3 (x  3)

y  y1  m(x  x1 )
1
y  (3)  3 [ x  (4) ]

y  5  3x  1
1

y  5  5  3x  1  5

Pages 296297

y  2x  2

y  (2)  2 (x  2)

y

y  4  4  2x  2  4

Step 3: Use the point-slope form to find the


equation.
y  y1  m(x  x1 )

1
3x

y  4  2x  2

y  3  3 (x  4)

6

13
3

y  3  3x  3
y  3  3  3x  3  3

Practice and Apply

y  3x 

13. The line parallel to y  x  2 has the same


slope, 1.
y  y1  m(x  x1 )
y  (7)  1(x  2)
y7x2
y77x27
yx9
14. The line parallel to y  2x  2 has the same
slope, 2.
y  y1  m(x  x1 )
y  (1)  2(x  2)
y  1  2x  4
y  1  1  2x  4  1
y  2x  5

20. The line parallel to y  2x  4 has the same


1
slope,  2 .
y  y1  m(x  x1 )
1
y  2  2 [ x  (1) ]
1

y  2  2 (x  1)
1

y  2  2x  2
y  2  2  2x  2  2
y  2 x  2

225

Chapter 5

PQ249-6481F-05[217-243] 7/31/02 10:04 PM Page 226

.25. The lines for x  3 and x  1 are parallel


because all vertical lines are parallel. The lines
2
2
for y  3x  2 and y  3x  3 are parallel because
they have the same slope. Thus, both pairs of
opposite sides are parallel, and the figure is a
parallelogram.
26. The line parallel to y  5x  3 has the same
slope, 5. Since the line passes through the origin,
the y-intercept is 0. Use the slope-intercept form.
y  mx  b
y  5x  0
y  5x
27. Write the given equation in slope-intercept form.
x  3y  8
x  3y  x  8  x
3y  x  8

21. Write the given equation in slope-intercept form.


2y  x  1
2y
2

y

x  1
2
1
1
x
2
2
1

The line parallel to y  2x  2 has the same


1
slope, 2.
y  y1  m(x  x1 )
1
y  0  2 [x  (3) ]
1

y  2 (x  3)
1

y  2x  2
22. Write the given equation in slope-intercept form.
3y  2x  6
3y
3

y

2x  6
3
2
3x  2

3y
3

y

The line parallel to y  3x  2 has the same


2
slope, 3.

y  3x  (6)

y  3x  6

2
3x

10
3

y  2  3x  3


28. The slope of the given line is 1. So, the slope of


the line perpendicular to this line is the opposite
reciprocal of 1, or 1. Use the point-slope form.
y  y  m(x  x )
1
1
y  0  1[x  (2) ]
y  (x  2)
y  x  2
29. The slope of the given line is 4. So, the slope of
the line perpendicular to this line is the opposite
1
reciprocal of 4, or 4. Use the point-slope form.

23. Write the given equation in slope-intercept form.


6x  y  4
6x  y  6x  4  6x
y  6x  4
The line parallel to y  6x  4 has the same
slope, 6.
y  y1  m(x  x1 )
y  3  6[ x  (2) ]
y  3  6(x  2)
y  3  6x  12
y  3  3  6x  12  3
y  6x  9
24. Write the given equation in slope-intercept form.
3x  4y  4
3x  4y  3x  4  3x
 4y  3x  4
4y
4

y

y  y1  m(x  x1 )
1
y  1  4 (x  1)

y

Chapter 5

y  y1  m(x  x1 )
1
y  1  3 [x  (3) ]
1

y  1  3 (x  3)
1

y  1  3x  1

y  2  4x  2


30. The slope of the given line is 3. So, the slope of
the line perpendicular to this line is the opposite
1
reciprocal of 3, or 3. Use the point-slope form.

y  y1  m(x  x1 )
3
y  2  4 (x  2)
3
2
1
2

y  4x  4

3
x
4
3
x
4

y  1  1  4x  4  1

The line parallel to y  4x  1 has the same


3
slope, 4.

y  1  4x  4

3x  4
4
3
x1
4

y  2  2  3x  3  2

y22

x  8
3
1
8
x3
3

The line parallel to y  3x  3 has the same


1
slope, 3. The y-intercept is 6. Use the slopeintercept form.
y  mx  b

y  y  m(x  x )
1
1
2
y  2  3 (x  2)

y

y  1  1  3x  1  1

2

y  3x  2

226

PQ249-6481F-05[217-243] 7/31/02 10:04 PM Page 227

31. The slope of the given line is 8. So, the slope of
the line perpendicular to this line is the opposite
1
reciprocal of 8, or 8. Use the point-slope form.
y  y1  m(x  x1 )

36. Find the slope of the given line.


3y  x  3
3y  x  x  3  x
3y  x  3
3y
3

y  5  8 (x  0)

y

y  5  8x
1

y  8x  5
1

32. The slope of the given line is 2. So, the slope of


the line perpendicular to this line is the opposite
1
reciprocal of 2, or 2. Use the point-slope form.
y  y1  m(x  x1 )
y  (3)  2(x  1)
y  3  2x  2
y  3  3  2x  2  3
y  2x  1
2
33. The slope of the given line is 3. So, the slope of
the line perpendicular to this line is the opposite
2
3
reciprocal of 3, or 2. Use the point-slope form.
y  y1  m(x  x1 )
3

y  7  2 (x  4)
3

y  7  2x  6
3

y  7  7  2x  6  7

y  5x  (1)

y  2x  13

y  5x  1

34. Find the slope of the given line.


3x  8y  4
3x  8y  3x  4  3x
8y  3x  4


y

38. Find the slope of the given line.


5x  7  3y
3y  5x  7
3y
3

 3x  4
8
3
1
8x  2

y

y

5x  7
3
5
7
x
3
3

The slope of the line perpendicular to this line is


5
3
the opposite reciprocal of 3, or 5. Use the pointslope form.
y  y  m(x  x )
1
1

The slope of the line perpendicular to this line is


3
8
the opposite reciprocal of 8, or 3. The point (0, 4)
lies on the y-axis. Thus, the y-intercept is 4. Use
the slope-intercept form.
y  mx  b
8
x
3

 x  3
3
1
3x  1

The slope of the line perpendicular to this line is


1
the opposite reciprocal of 3, or 3. Use the pointslope form.
y  y1  m(x  x1 )
y  (1)  3(x  6)
y  1  3x  18
y  1  1  3x  18  1
y  3x  19
37. Find the slope of the given line.
5x  y  3
5x  y  5x  3  5x
y  5x  3
(1)(y)  (1) (5x  3)
y  5x  3
The slope of the line perpendicular to this line is
1
the opposite reciprocal of 5, or 5. The point
(0, 1) lies on the y-axis. Thus, the y-intercept is
1. Use the slope-intercept form.
y  mx  b

y  5  5  8x  5

8y
8

y  (2)  5 (x  8)

4

y  2  5x 

35. Find the slope of the given line.


2x  5y  3
2x  5y  2x  3  2x
5y  2x  3
5y
5

y

y  2  2  5x 
y

3
5x

24
5
24
5
14
5

2

39. Find the slope of the given line.


3x  7  2x
3x  7  2x  2x  2x
x70
x7707
x  7
This line is vertical. A line perpendicular to a
vertical line is horizontal and has slope 0.
Use the point-slope form.
y  y  m(x  x )
1
1
y  (3)  0(x  3)
y30
y3303
y  3

2x  3
5
2
3
x
5
5

The slope of the line perpendicular to this line is


2
5
the opposite reciprocal of 5, or 2. Use the pointslope form.
y  y  m(x  x )
1
1
5

y  7  2 [x  (2) ]
5

y  7  2 (x  2)
5

y  7  2x  5
5

y  7  7  2x  5  7
5

y  2x  2

227

Chapter 5

PQ249-6481F-05[217-243] 7/31/02 10:05 PM Page 228

46. Write each equation in slope-intercept form.


y  ax  5
2y  (a  4)x  1

40. Find the slope of the given line.


3x  6y  2
3x  6y  3x  2  3x
6y  3x  2
6y
6

y

2y
2

y

3x  2
6
1
1
2x  3

2  10
3  9

a
(2)a 

or 2

2x
2

4x
The line perpendicular to y  2x  8 and passing
1
through its x-intercept has slope 2 and passes
through (4, 0). Use the point-slope form.
1

2x  14
3
2

y  3x 

Page 297

y

y  7  5[x  (4) ]
y  7  5(x  4)
y  y1  m(x  x1 )
52.

2x  2
3
2
2
x
3
3

y  (3)  2 [x  (1) ]

2
,
3

The slopes are both so the two graphs are


parallel.
44. The slope of y  5x is 5. The slope of
y  5x  18 is 5. The slopes are not the same,
and 5(5)  1. Therefore, the two graphs are
neither parallel nor perpendicular.
45. Find the slope of each segment.
3  (3)
(1)
1  1
3  (3)

Slope of AC : m  1 

or 3

Slope of BD : m 

or 3

y  3  2 (x  1)

The diagonals AC and BD are perpendicular


1
because 3 (3)  1.

Chapter 5

Maintain Your Skills

50. y  y1  m(x  x1 )
y  5  2(x  3)
51. y  y1  m(x  x1 )

3y  2x  2


3x  24
4
3
4x  6

49. C; The y-intercept of y  3x  4 is 4. The


y-intercept of y  3x  2 is 2. The slope of both
lines is 3. Therefore, to graph y  3x  2 change
the y-intercept from 4 to 2.

2x  3y  2x  2  2x
3y
3

y

14
3

The slope of the line perpendicular to this line is


3
4
the opposite reciprocal of 4 or 3.

42. Write each equation in slope-intercept form.


y  2x  11
y  2x  23
y  2x  2x  23  2x
y  2x  23
The slopes are both 2, so the two graphs are
parallel.
43. Write each equation in slope-intercept form.
3y  2x  14
2x  3y  2


4y
4

y  0  2 (x  4) or y  2x  2

3y
3

a  4
2
a  4
(2) 2

2a  a  4
2a  a  a  4  a
a4
47. If two equations have the same slope, then the
lines are parallel. Answers should include the
following.
Sample answer: y  5x  1; The graphs
have the same slope.
1
Sample answer: y  5x; The slopes are
negative reciprocals of each other.
48. D; Write the given equation in slope-intercept
form.
3x  4y  24
3x  4y  3x  24  3x
4y  3x  24

Use the point-slope form to write its equation.


y  10  2(x  9) or y  2x  8
To find the x-intercept, let y  0.
0  2x  8
0  8  2x  8  8
8  2x
8
2

(a  4)x  1
2
a  4
1
x
2
2

The lines are parallel if the slopes are equal.

The slope of the line perpendicular to this line is


1
the opposite reciprocal of 2, or 2. The y-intercept
is 2. Use the slope-intercept form.
y  mx  b
y  2x  (2)
y  2x  2
41. Find the slope of the line through (9, 10) and
(3, 2).
m

228

PQ249-6481F-05[217-243] 7/31/02 10:05 PM Page 229

58. Find the slope of the line.

53. Write an equation of the line that passes through


(10, 3.19) and (15, 4.29).
Find the slope.
m
m

1  5

m  8  5 or 2
Find the y-intercept.
y  mx  b
5  2(5)  b
5  10  b
5  10  10  b  10
15  b
Write the slope-intercept form.
y  mx  b
y  2x  15
59. Find the slope of the line.

4.29  3.19
15  10
1.1
5

m  0.22
Find the y-intercept.
y  mx  b
3.19  0.22(10)  b
3.19  2.2  b
3.19  2.2  2.2  b  2.2
0.99  b
Write the slope-intercept form.
y  mx  b
y  0.22x  0.99
Therefore, using m for the number of
minutes and C for the total cost, the equation is
C  0.22m  0.99.
54. y  0.22x  0.99
y  0.22(12)  0.99
y  3.63
The cost of a 12-minute call is $3.63.
55. Find the slope of the line.
m

3  (1)
3  5

9  9

m  4  6 or 0
Find the y-intercept.
y  mx  b
9  0(6)  b
9b
Write the slope-intercept form.
y  mx  b
y  0x  9
y9
60. Find the slope of the line.
2  4
(6)

m2

or 2

or

3
4

Find the y-intercept.


y  mx  b

Find the y-intercept.


y  mx  b

4  4 (6)  b

1  2 (5)  b

42b

422b2

1  2  b
5

1  2  2  b 
3
2

5
2

2  b

b

Write the slope-intercept form.


y  mx  b

Write the slope-intercept form.


y  mx  b
1

1 12

y  4x  2

y  2x  2

y  4x  2

56. Find the slope of the line.


0  2

m  8  0 or 4

Page 297

The point (0, 2) lies on the y-axis. The y-intercept


is 2.
Write the slope-intercept form.
y  mx  b
1

y  4x  2
57. Find the slope of the line.
m

4  1
3  2

Practice Quiz 2

1. Use the slope-intercept form.


y  mx  b
y  4x  (3)
y  4x  3
2. Find the y-intercept.
y  mx  b
3  2(1)  b
3  2  b
3  2  2  b  2
5  b
Write the slope-intercept form.
y  mx  b
y  2x  (5)
y  2x  5

or 5

Find the y-intercept.


y  mx  b
1  5(2)  b
1  10  b
1  10  10  b  10
11  b
Write the slope-intercept form.
y  mx  b
y  5x  11

229

Chapter 5

PQ249-6481F-05[217-243] 7/31/02 10:05 PM Page 230

2a. Sample answer:

3. Find the slope.


m

3  (2)
1  (1)

5
2

or

Find the y-intercept.


y  mx  b
5

3  2 (1)  b
5

32b
5

1
2

b

2b. Sample answer:


y

Write the slope-intercept form.


y  mx  b
5

322b2

y  2x  2
4. The line parallel to y  2x  2 has the same
slope, 2. Use the point-slope form.
y  y1  m(x  x1 )
y  3  2[x  (2) ]
y  3  2(x  2)
y  3  2x  4
y  3  3  2x  4  3
y  2x  7
5. Standard form:

2c. Sample answer:


y

y  4  2 (x  3)
2(y  4)  2

112 2 (x  3)

2y  8  1(x  3)
2y  8  x  3
2y  8  8  x  3  8
2y  x  11
2y  x  x  11  x
x  2y  11
(1)(x  2y)  (1)11
x  2y  11
Slope-intercept form:

3. Linear extrapolation predicts values outside the


range of the data. Linear interpolation predicts
values inside the range of the data.
4. The graph shows a positive correlation. As the
number of hours of study increases, the test score
increases.
5. The graph shows a negative correlation. As the
number of hours of TV increases, the number of
hours of exercise decreases.
6. Let the independent variable x be the air
temperature, and let the dependent variable y be
the body temperature.
The scatter plot seems to indicate that as the air
temperature increases, the body temperature
increases. There is a positive correlation between
the two variables.

y  4  2 (x  3)
1

11
2

y  4  2x  2
y  4  4  2x  2  4
y  2x 

Statistics: Scatter Plots


and Lines of Fit

Page 299

Body Temperature (C)

5-7

Algebra Activity

1. See students work.


3. See students work.

Pages 301302

2. See students work.

Check for Understanding

40
30
20
10
0

1. If the data points form a linear pattern such that


y increases as x increases, there is a positive
correlation. If the linear pattern shows that y
decreases as x increases, there is a negative
correlation.

Chapter 5

230

10
20
30 40
Air Temperature (C)

15. Use the equation y  2.15x  4285.45.


y  2.15x  4285.45
y  2.15(2002)  4285.45
y  18.85
The number of bushels of apples in storage in
2002 will be about 18.85 million.
16. Find the slope.

Body Temperature (C)

7. No one line will pass through all of the data


points. Draw a line that passes close to the points.
35
30
25
20
15

y2  y1

mx

10
0

m

15 20 25 30 35
Air Temperature (C)

m

Use the point-slope form.


y  y1  m(x  x1 )

8. The line of fit passes through the data points


(26.2, 25.6) and (31.2, 31.0).
Step 1: Find the slope.

y  6000  1200(x  5)
y  6000  1200x  6000
y  6000  6000  1200x  6000  6000
y  1200x  12,000
17. Use the equation y  1200x  12,000.
y  1200x  12,000
y  1200(7)  12,000
y  3600
The price of a 7-year-old car should be about $3600.
18.

y2  y1

mx

m
m

 x1

31.0  25.6
31.2  26.2
5.4
or 1.08
5

1 2 3 4 5 6 7 8
Number of Carbon Atoms

19. Draw a line that passes close to the points.


120
90
60
30
0
30
60
90
120

Practice and Apply

10. The graph shows a negative correlation. As the


year increases, the percentage of forms returned
decreases.
11. The graph shows no correlation.
12. The graph shows a positive correlation. As the
year increases, the number of electronic tax
returns increases.
13. The graph shows a positive correlation. As the
amount of sugar increases, the number of calories
increases.
14. Find the slope.

1 2 3 4 5 6 7 8
Number of Carbon Atoms

20. We use the data points (3, 42) and (6, 69).
Find the slope.
y2  y1

y2  y1

mx

mx

 x1

m

12.4  8.1

m  1999  1997
4.3
2

120
90
60
30
0
30
60
90
120

Boiling Point (C)

Pages 302305

Boiling Point (C)

Step 2: Use m  1.08 and the point-slope form


to write the equation. You can use either
data point. We chose (26.2, 25.6).
y  y1  m(x  x1 )
y  25.6  1.08(x  26.2)
y  25.6  1.08x  28.296
y  25.6  25.6  1.08x  28.296  25.6
y  1.08x  2.696
9. Use the equation y  1.08x  2.696, where x is
the air temperature and y is the body
temperature.
y  1.08x  2.696
y  1.08(40.2)  2.696
y  40.72
The body temperature would be about 40.1F.

m

 x1

6000  9600
5  2
3600
or 1200
3

m

or 2.15

 x1

69  (42)
6  3
111
or 37
3

Use the point-slope form.


y  y  m(x  x )
1
1
y  69  37(x  6)
y  69  37x  222
y  69  69  37x  222  69
y  37x  153

Use the point-slope form.


y  y1  m(x  x1 )
y  8.1  2.15(x  1997)
y  8.1  2.15x  4293.55
y  8.1  8.1  2.15x  4293.55  8.1
y  2.15x  4285.45

231

Chapter 5

PQ249-6481F-05[217-243] 7/31/02 10:05 PM Page 232

27. Use the equation y  0.5125x  4.5. The year


2005 is 25 years after 1980.
y  0.5125x  4.5
y  0.5125(25)  4.5
y  17.3125
The federal spending on space and other
technologies in 2005 will be about $17.3 billion.
28. Sample answer: The governments prediction is
$17.3  $14.3 or $3 billion less.
29.
800

251.4
37

Acres Burned (thousands)

21. Use the equation y  37x  153.


y  37x  153
y  37(1)  153
y  116
The boiling point for methane should be
about 116
C.
22. Use the equation y  37x  153.
y  37x  153
y  37(5)  153
y  32
The boiling point for pentane should be
about 32
C.
23. Use the equation y  37x  153.
y  37x  153
98.4  37x  153
98.4  153  37x  153  153
251.4  37x

Acres Burned (thousands)

Spending (billions of dollars)


Spending (billions of dollars)

10
20
30
Rainfall (in.)

800
600
400
200
0

10
20
30
Rainfall (in.)

31. We use the points (12.7, 340) and (17.5, 194). Find
the slope.
y2  y1

mx

79 8085909500
Year

m
m

12
10
8
6
4
0

80 85 90 95 00
Year

26. We use the points (0, 4.5) and (16, 12.7).


Find the slope.
y2  y1

mx

 x1

12.7  4.5
16  0
8.2
or 0.5125
16

The y-intercept is 4.5. Use the slope-intercept


form.
y  mx  b
y  0.5125x  4.5

Chapter 5

 x1

194  340
17.5  12.7
146
4.8

m  30.4
Use the point-slope form.
y  y  m(x  x )
1
1
y  194  30.4(x  17.5)
y  194  30.4x  532
y  194  194  30.4x  532  194
y  30.4x  726
32. Use the equation y  30.4x  726.
y  30.4x  726
y  30.4(8.25)  726
y  475.2
The number of acres burned in 2000 should be
about 475 thousand acres.
33. The data point lies beyond the main grouping of
data points. It can be ignored as an extreme
value.

14

m

200

30. Draw a line that passes close to the points.

25. Draw a line that passes close to the points.

m

400

37x
37

7x
The number of carbon atoms in heptane should be
about 7.
24. The scatter plot seems to indicate that as the year
increases, federal spending increases. There is a
positive correlation between the two variables.
18
16
14
12
10
8
6
4
2
0

600

232

PQ249-6481F-05[217-243] 7/31/02 10:05 PM Page 233

34.

48.

Incorrect Answers

20
16

49.

12
8

50.

4
0

4
8
12 16
Correct Answers

20

35. See students work. 36. See students work.


37. You can visualize a line to determine whether the
data has a positive or negative correlation.
Answers should include the following.
y
Height

51.

O
Age

38.
39.

40.
41.
43.

Write a linear equation for the line of fit. Then


substitute the persons height and solve for the
corresponding age.
D; Graph D shows a negative correlation. As the
value of x increases, the value of y decreases.
B; The equation y  2x  3 represents a line that
passes through the points (1, 5), (2, 7), and (4, 11),
and lies close to the point (3, 7). Each of the other
three equations represents a line that does not
pass as close to the four points.
Sample answer: Cities with greater north
latitudes have lower January temperatures.
See students work. 42. See students work.
See students work. 44. See students work.

Page 305

52.

y  y1  m(x  x1 )
y  (2)  3(x  1)
y  2  3(x  1)
y  y1  m(x  x1 )
y  (3)  1[ x  (3) ]
y3x3
To find the x-intercept, let
3x  4y  12
3x  4(0)  12
3x  12
x4
To find the y-intercept, let
3x  4y  12
3(0)  4y  12
4y  12
y3
To find the x-intercept, let
2x  5y  8
2x  5(0)  8
2x  8
x4
To find the y-intercept, let
2x  5y  8
2(0)  5y  8
5y  8
y  1.6
To find the x-intercept, let
y  3x  6
0  3x  6
3x  6
x  2
To find the y-intercept, let
y  3x  6
y  3(0)  6
y6

53.
12

r  7
4
r  7
4

y  0.

x  0.

y  0.

x  0.

y  0.

x  0.

r  2
6

2  121r 6 2 2

3(r  7)  2(r  2)
3r  21  2r  4
3r  21  2r  2r  4  2r
5r  21  4
5r  21  21  4  21
5r  25

Maintain Your Skills

45. The line parallel to y  4x  5 has the same


slope, 4. Use the point-slope form.
y  y1  m(x  x1 )
y  5  4[x  (2) ]
y  5  4(x  2)
y  5  4x  8
y  5  5  4x  8  5
y  4x  3
46. The slope of the given line is 2. So, the slope of
the line perpendicular to this line is the opposite
1
reciprocal of 2, or 2. The y-intercept is 0. Use the
slope-intercept form.
y  mx  b

5r
5

25
5

r  5
54.
3

n  (4)
3
n  (4)
3

7

2  3 (7)

n  (4)  21
n  (4)  (4)  21  (4)
n  25

y  2x  0
1

y  2x
47. y  y  m(x  x )
1
1
y  3  2[x  (2) ]
y  3  2(x  2)

233

Chapter 5

PQ249-6481F-05[217-243] 7/31/02 10:05 PM Page 234

55.
35

2x  1
5
2x  1
5

4x  5
7
4x  5
35 7

2 1

4. Enter the number of votes cast in 1996 in L1


and the number of votes cast in 2000 in L2.

STAT ENTER 14,447 ENTER


ENTER
19,458
28,674 ENTER 31,658 ENTER
32,739 ENTER 46,543 ENTER 49,186 ENTER

KEYSTROKES:

7(2x  1)  5(4x  5)
14x  7  20x  25
14x  7  20x  20x  25  20x
6x  7  25
6x  7  7  25  7
 6x  18
6x
6

69,208 ENTER 103,429 ENTER 103,574

ENTER

18
6

30,921 ENTER 38,545 ENTER 38,626 ENTER

x3

Page 307

52,457 ENTER 53,907 ENTER 80,787 ENTER


126,911 ENTER 123,466
Find the regression equation by selecting
LinReg (ax  b) on the STAT CALC menu.

Graphing Calculator Investigation


(Follow-Up of Lesson 5-7)

4 ENTER
The regression equation is about y  1.23x 
3414.80.
The data are already in Lists 1 and 2. Find
the median-fit equation by using Med-Med on
the STAT CALC menu.
KEYSTROKES:

1. Enter the number of years since 1985 in L1


and the number of bald eagle pairs in L2.

STAT ENTER 3 ENTER 5


ENTER 7 ENTER 9 ENTER 11 ENTER 14
ENTER
2500 ENTER 3000 ENTER 3700

KEYSTROKES:

Find the regression equation by selecting


LinReg(ax  b) on the STAT CALC menu.
4 ENTER
The regression equation is about
y  309.48x  1555.88.
The data are already in Lists 1 and 2. Find
the median-fit equation by using Med-Med on
the STAT CALC menu.

STAT

STAT

According to the median-fit equation, the number


of votes in that county in 2000 was about 23,882.
The estimates both show the number of votes
increasing from 1996 to 2000. But both are far
from the actual number.

Chapter 5 Study Guide and Review


Page 308

Vocabulary and Concept Check

1. direct variation
3. parallel
5. slope-intercept

Pages 308312

2. rise; run
4. point-slope
6. y-intercept

Lesson-by-Lesson Review

7. Let (1, 3)  (x1, y1) and (2, 6)  (x2, y2).


y2  y1
mx x
2

m

234

6  3
 1
9
or 3
3

m  2

Chapter 5

STAT

According to the regression equation, the number


of votes in that county in 2000 was about 24,753.
Graph the median-fit equation.
Find y when x  22,839 using value on the
CALC menu.
CALC 1 22,839 ENTER
KEYSTROKES: 2nd

3 ENTER
The median-fit equation is about
y  311.76x  1537.25.
2. The correlation coefficient of y  309.48x 
1555.88 is 0.9970385087. The data are very
nearly linear.
3. Graph the regression equation.
Find y when x  13 using value on the CALC
menu.
CALC 1 13 ENTER
KEYSTROKES: 2nd
According to the regression equation, the
number of bald eagle pairs was about 5579.
Graph the median-fit equation.
Find y when x  13 using value on the CALC
menu.
CALC 1 13 ENTER
KEYSTROKES: 2nd
According to the median-fit equation, the number
of bald eagle pairs was about 5590.
Both predictions are close to the prediction in
Example 4. But these predictions show fewer
pairs of eagles in 1998.
KEYSTROKES:

STAT

3 ENTER
The median-fit equation is about
y  1.24x  4454.74.
5. Graph the regression equation.
Find y when x  22,839 using value on the
CALC menu.
CALC 1 22,839 ENTER
KEYSTROKES: 2nd
KEYSTROKES:

ENTER 4500 ENTER 5000 ENTER 5800 ENTER

KEYSTROKES:

16,284 ENTER 19,281 ENTER

PQ249-6481F-05[217-243] 7/31/02 10:05 PM Page 235

8. Let (0, 5)  (x1, y1) and (6, 2)  (x2, y2).


y2  y1
mx x
2

14. Write the slope as a ratio.


4 

Graph (0, 0). From the point (0, 0), move down
4 units and right 1 unit. Draw a dot. Draw a line
containing the points.

2  5

m60
m

3
6

4
1

or 2

9. Let (6, 4)  (x1, y1) and (6, 2)  (x2, y2).

y2  y1

mx

m
m

 x1

2  4
6  (6)
6
0

y  4x

Since division by zero is undefined, the slope is


undefined.
10. Let (8, 3)  (x1, y1) and (2, 3)  (x2, y2).
y2  y1

mx

m
m

15. Write the slope as a ratio.

 x1

1
3

3  (3)
2  8
0
or 0
10

Graph (0, 0). From the point (0, 0), move up


1 unit and right 3 units. Draw a dot. Draw a line
containing the points.

11. Let (2.9, 4.7)  (x1, y1) and (0.5, 1.1)  (x2, y2).
y2  y1
mx x
2

y  13 x

1.1  4.7

m  0.5  2.9
3.6

m  2.4 or 1.5
12. Let

112, 12  (x1, y1) and 11, 23 2  (x2, y2).

y2  y1

mx

m
m

2
3

 x1

1
1

1  2
1
3
3
2

16. Write the slope as a ratio.


1

4 

or 9

1
4

Graph (0, 0). From the point (0, 0), move down
1 unit and right 4 units. Draw a dot. Draw a line
containing the points.

13. Write the slope as a ratio.


2

21

Graph (0, 0). From the point (0, 0), move up


2 units and right 1 unit. Draw a dot. Draw a line
containing the points.
y

y   14 x

y  2x
O

17. Write the slope as a ratio.


3
2

Graph (0, 0). From the point (0, 0), move up


3 units and right 2 units. Draw a dot. Draw a line
containing the points.
y
y  32 x
O

235

Chapter 5

PQ249-6481F-05[217-243] 7/31/02 10:05 PM Page 236

31. The y-intercept is 1. So, graph (0, 1). The slope is


2
2 or 1. From (0, 1) move up 2 units and right
1 unit. Draw a dot. Draw a line containing the
points.

18. Write the slope as a ratio.


4
3

4
3

Graph (0, 0). From the point (0, 0), move down
4 units and right 3 units. Draw a dot. Draw a line
containing the points.

y
y  2x  1
x

y   43 x

19. Find the value of k.


y  kx
6  k(9)
6
9
2
3

k(9)
9

15
2
15
2

k

Therefore, y 

2
3x.

21. Find the value of k.


y  kx
4  k(4)

k(2)
2

k

Therefore, y 

22. Find the value of k.


y  kx
6  k(18)

1  k
Therefore, y  x.

Therefore, y  3x.

k(4)
4

23. Find the value of k.


y  kx
10  k(5)
10
5

k(5)
5

2  k

y  x  5

15
x.
2

6
18
1
3

4
4

32. The y-intercept is 5. So, graph (0, 5). The slope is


1
1 or 1 . From (0, 5) move down 1 unit and right
1 unit. Draw a dot. Draw a line containing the
points.

20. Find the value of k.


y  kx
15  k(2)

k

33. The y-intercept is 3. So, graph (0, 3). The slope is


1
. From (0, 3) move up 1 unit and right 2 units.
2
Draw a dot. Draw a line containing the points.

24. Find the value of k.


y  kx
7  k(14)
7
14
1
2

k(18)
18

k(14)
14

y  1 x 3
2

k
1

25.

26.

27.

28.

Therefore, y  2x.
Therefore, y  2x.
Replace m with 3 and b with 2.
y  mx  b
y  3x  2
Replace m with 1 and b with 3.
y  mx  b
y  1x  (3)
yx3
Replace m with 0 and b with 4.
y  mx  b
y  0x  4
y4
1
Replace m with 3 and b with 2.
y  mx  b

34. The y-intercept is 1. So, graph (0, 1). The slope
4
4
is 3 or 3. From (0, 1) move up 4 units and left
3 units. Draw a dot. Draw a line containing the
points.
y

y   4x 1

y3x2

29. Replace m with 0.5 and b with 0.3.


y  mx  b
y  0.5x  (0.3)
y  0.5x  0.3
30. Replace m with 1.3 and b with 0.4.
y  mx  b
y  1.3x  0.4
Chapter 5

236

PQ249-6481F-05[217-243] 7/31/02 10:05 PM Page 237

39. Find the y-intercept.


y  mx  b

35. Solve for y.


5x  3y  3
5x  3y  5x  3  5x
3y  5x  3
3y
3

y

6  2(1)  b
1

62b

5x  3
3
5
x
1
3

11
2

b

622b2

The y-intercept is 1. So, graph (0, 1). The slope is


5
. From (0, 1) move up 5 units and right 3 units.
3
Draw a dot. Draw a line containing the points.

Write the slope-intercept form.


y  mx  b
1

y  2x 

11
2

40. Find the y-intercept.


y  mx  b
3

3  5(4)  b
5x  3y  3

12

3   5  b

12
5
3
5

3 

The y-intercept
3

1 2

y  5x  5

41. Find the slope.


y2  y1

1 2. The slope is

So, graph 0,

mx

9
2

m

3 or 1 . From
move down 3 units and right
1 unit. Draw a dot. Draw a line containing the
points.

m

 x1

12  2
1  (4)
10
or 2
5

Find the y-intercept.


y  mx  b
12  2(1)  b
12  2  b
12  2  2  b  2
10  b
Write the slope-intercept form.
y  mx  b
y  2x  10
42. Find the slope.

y
6x  2 y  9

1 32

9
is 2.
9
0, 2

b

y  5x  5

6x  9
2

y  3x  2

12
5

Write the slope-intercept form.


y  mx  b

36. Solve for y.


6x  2y  9
6x  2y  6x  9  6x
2y  6x  9
2y
2

12

 5  b 

y2  y1

37. Find the y-intercept.


y  mx  b
3  1(3)  b
3  3  b
3  3  3  b  3
6b
Write the slope-intercept form.
y  mx  b
y  1x  6
yx6
38. The point (0, 6) lies on the y-axis. The y-intercept is 6.
y  mx  b
y  2x  6

mx

 x1

5  0

m45
5

m  1 or 5
Find the y-intercept.
y  mx  b
0  5(5)  b
0  25  b
0  25  25  b  25
25  b
Write the slope-intercept form.
y  mx  b
y  5x  25

237

Chapter 5

PQ249-6481F-05[217-243] 7/31/02 10:06 PM Page 238

54. The line parallel to y  3x  2 has the same


slope, 3.
y  y1  m(x  x1 )
y  6  3(x  4)
y  6  3x  12
y  6  6  3x  12  6
y  3x  6
55. The line parallel to y  2x  4 has the same
slope, 2.
y  y1  m(x  x1 2
y  (62  2(x  62
y  6  2x  12
y  6  6  2x  12  6
y  2x  6
56. The line parallel to y  6x  1 has the same
slope, 6.
y  y  m(x  x )
1
1
y  2  6(x  1)
y  2  6x  6
y  2  2  6x  6  2
y  6x  8

43. Find the y-intercept.


y  mx  b
1  0(8)  b
1  b
Write the slope-intercept form.
y  mx  b
y  0x  (1)
y  1
44. Find the y intercept.
y  mx  b
6  0(4)  b
6b
Write the slope-intercept form.
y  mx  b
y  0x  6
y6
Therefore, the equation is y  6.
45. y  y  m(x  x )
46. y  y  m(x  x )
1
1
1
1
y  6  5(x  4)
y  4  2[ x  (1) ]
y  4  2(x  1)
47.
y  y1  m(x  x1 ) 48.
y  y1  m(x  x1 )
1

y  (4)  2 (x  1)

y  4  2 (x  1)

y  (3)  2 (x  5)
y  3  2 (x  5)
49.

57. The line parallel to y  12 x  2 has the same


5
slope, 12. The point (0, 4) lies on the y-axis. The
y-intercept is 4.
y  mx  b

5
5

y  y1  m(x  x1 )
1
y  (2)  3 x  4

1 2
1
y  2  31x  4 2

y  12 x  4
58. Solve the equation for y.
4x  y  7
4x  y  4x  7  4x
y  4x  7
(1)(y)  (1)(4x  7)
y  4x  7
The line parallel to y  4x  7 has the same
slope, 4.
y  y1  m(x  x1 )
y  (1)  4(x  2)
y  1  4x  8
y  1  1  4x  8  1
y  4x  9
59. Solve the equation for y.
3x  9y  1
3x  9y  3x  1  3x
9y  3x  1

y  y1  m(x  x1 )
y  (2)  0(x  4)
y20
51.
y  1  2(x  1)
y  1  2x  2
y  1  1  2x  2  1
y  2x  3
y  2x  2x  3  2x
2x  y  3
(1)(2x  y)  (1)3
2x  y  3
50.

52.

y  6  3(x  9)
3( y  6)  3

113 2(x  9)

3y  18  x  9
3y  18  18  x  9  18
3y  x  27
3y  x  x  27  x
x  3y  27
(1)(x  3y)  (1)(27)
x  3y  27
53.
y  4  1.5(x  4)
2( y  4)  2(1.5)(x  4)
2y  8  3(x  4)
2y  8  3x  12
2y  8  8  3x  12  8
2y  3x  20
2y  3x  3x  20  3x
3x  2y  20
(1)(3x  2y)  (1)(20)
3x  2y  20
Chapter 5

9y
9

y

3x  1
9
1
1
3x  9
1

The line parallel to y  3x  9 has the same


1
slope, 3.
y  y1  m(x  x1 )
1
y  0  3(x  3)
1

y  3x  1

238

PQ249-6481F-05[217-243] 7/31/02 10:06 PM Page 239

65. Find the slope of the given line.


5y  x  1

60. The slope of the given line is 4. So, the slope of


the line perpendicular to this line is the opposite
1
reciprocal of 4 or 4.
y  y1  m(x  x1 )

5y
5

y

y  3  4(x  1)
1

13
4

The slope of the line perpendicular to this line is


1
the opposite reciprocal of 5 or 5.
y  y1  m(x  x1 )
y  (5)  5(x  2)
y  5  5x  10
y  5  5  5x  10  5
y  5x  15

y  3  4x  4
y  3  3  4x  4  3
y  4x 

61. The slope of the given line is 2. So, the slope of
the line perpendicular to this line is the opposite
1
reciprocal of 2 or 2. The point (0,3) lies on the
y-axis. The y-intercept is 3.
y  mx  b

Weight (long tons)

66.

y  2x  (3)
1

y  2x  3
2

62. The slope of the given line is 5. So, the slope of


the line perpendicular to this line is the opposite
2
5
reciprocal of 5 or 2.
y  y  m(x  x )
1
1

Weight (long tons)

y  5  2x  5
5

y  5  5  2x  5  5
5

y  2x
y  2.5x
63. Find the slope of the given line.
2x  7y  1
2x  7y  2x  1  2x
7y  2x  1


y

60
50
40
30
20
10
10
0
0 30 35 40 45 50 55 60
Length (ft)

67. Draw a line that passes close to the points.

y  (5)  2(x  2)

7y
7

x  1
5
1
1
5x  5

60
50
40
30
20
10

68. We use points (40, 25) and (52, 45).


Find the slope.

2x  1
7
2
1
x7
7

y2  y1

mx

m

Use the point-slope form.


y  y  m(x  x )

y  0  2 [x  (4) ]

y  2(x  4)
7

y  25  3x 

64. Find the slope of the given line.


8x  3y  7
8x  3y  8x  7  8x
3y  8x  7
y

y  25  25  3x 
y

y  3x 
5

13
2

125
3

y  383

125
.
3

125
3

y  3(482 

y  5  8(x  4)
3

 25

69. Use the equation y  3x 

The slope of the line perpendicular to this line is


8
3
the opposite reciprocal of 3 or 8.
y  y  m(x  x )
1
1
3

5
x
3

200
3
200
3
125
3

Using W for weight and / for length, the equation is


5
125
W  3x  3 .

8x  7
3
8
7
x3
3

The weight of a 48-foot humpback whale is about


1
383 long tons.

y  5  8x  2
y  5  5  8x  2  5
y  8x 

y  25  3(x  40)

y  2x  14

 x1

45  25
 40
20
5
or 3
12

m  52

The slope of the line perpendicular to this line is


2
7
the opposite reciprocal of 7 or 2.
y  y  m(x  x )
1
1

3y
3

35 40 45 50 55 60
Length (ft)

239

Chapter 5

PQ249-6481F-05[217-243] 7/31/02 10:06 PM Page 240

70. Use the equation y  3x 


5

y  3x 

125
.
3

9. The y-intercept is 3. So, graph (0, 3). The slope is


2
2 or 1. From (0, 3) move up 2 units and right
1 unit. Draw a dot. Draw a line containing the
points.

125
3

y  3(12) 

125
3

y  213
The weight of a 12-foot humpback whale is about
2
213 long tons. The prediction is not accurate. A
negative weight is not reasonable.

y
y  2x  3

Chapter 5 Practice Test


Page 313
1. Vertical lines have no slope.
2. Sample answer:

10. Solve for y.


2x  3y  9
2x  3y  2x  9  2x
3y  2x  9

3y
3

y

2x  9
3
2
3x  3
2

The y-intercept of y  3x  3 is 3. So, graph


2
2
(0, 3). The slope is 3 or 3 . From (0, 3) move
down 2 units and right 3 units. Draw a dot. Draw
a line containing the points.

3. The slope represents the rate of change.


4. Let (5, 8)  (x1, y1) and (3, 7)  (x2, y2).

y2  y1

mx

m
m

 x1

2x  3y  9

78
3  5
1
1
or 8
8

5. Let (5, 2)  (x1, y1) and (3, 2)  (x2, y2).
y2  y1

mx

m
m

 x1

2  (2)
35
0
or 0
2

11. The temperature is expected to fall 2


each hour,
so the rate of change is 2
each hour. At
midnight the temperature is 16
F.

6. Let (6, 3)  (x1, y1) and (6, 4)  (x2, y2).


y2  y1
mx x
2

m
m

number of hours
temperature
rate of
Temperature
equals
change times after42444
midnight
plus at44244
midnight.
14
44244
43 1
424
3 14243 123 1444
43 123 14
43

6
9
2
3

8. The y-intercept is 1. So, graph (0, 1). The slope


3
is 3 or 1. From (0, 1) move up 3 units and right
1 unit. Draw a dot. Draw a line containing the points.

k(92

12
4

16

Therefore, y 

8
8

y  3x  1

2
x.
3

k(8)
8

1  k
Therefore, y  x.

240

k(4)
4

3  k

k

14. Find the value of k.


y  kx
8  k(8)

Chapter 5

2

Therefore, the equation is T  2h + 16.


12. Find the value of k.
13. Find the value of k.
y  kx
y  kx
6  k(9)
12  k(4)

Since division by zero is undefined, the slope is


undefined.
7. The rate is $9.95 per month. Therefore, the direct
variation equation is C  0.95m.

4  (3)
66
7
0

Therefore, y  3x.
15. y  mx  b
y  4x  3

PQ249-6481F-05[217-243] 7/31/02 10:06 PM Page 241

16. Find the slope.


m

3  (5)
8  (2)

or

21. The scatter plot seems to indicate that as the


number of dog years increases, the number of
human years increases. There is a positive
correlation between the two variables.

1
5

Find the y-intercept.


y  mx  b

3  5(8)  b
8

3  5  b
8

Human Years

3  5  b
23

5  b
Write the slope-intercept form.
y  mx  b
1

23

y  5x   5
1
x
5

23
5


y
17. Solve the given equation for y.
3x  7y  4
3x  7y  3x  4  3x
7y  3x  4


y

3x  4
7
3
4
7x  7
3

The line parallel to y  7x  7 has the same


3
slope, 7.
y  y1  m(x  x1 )
3
y  (2)  7(x  5)
3

y  2  7x 
3

y  2  2  7x 
3

y  7x 

15
7
15
7
1
7

2

y

4
6
Dog Years

y
50
45
40
35
30
25
20
15
10
5
0

4
6
Dog Years

23. We use the points (1, 15) and (7, 47).


Find the slope.

18. The line has slope 0.


y  y1  m(x  x1 )
y  (8)  0(x  5)
y80
y8808
y  8
19. Find the slope of the given line.
5x  3y  9
5x  3y  5x  9  5x
 3y  5x  9
3y
3

22. Draw a line that passes close to the points.

Human Years

7y
7

50
45
40
35
30
25
20
15
10
5
0
0

m

47  15
7  1

or

16
3

Use the point-slope form to write the equation.


y  y  m(x  x )
1

16
(x  1)
3
16
16
y  15  3 x  3
16
16
y  15  15  3 x  3  15
16
29
y 3x 3
16
29
Use the equation y  3 x  3 .
16
29
y 3x 3
16
29
y  3 (13)  3

y  15 

24.

5x  9
3
5
x
3
3

The slope of the line perpendicular to this line is


5
3
the opposite reciprocal of 3 or 5. The y-intercept
is 0.
y  mx  b

y  79
The number of human years comparable to 13 dog
years is 79.
4

25. B; The y-intercept of y  3x  3 is 3. Therefore,


the line represented by this equation passes
through (0, 3) not (0, 4).

y  5x  0
3

y  5x
20. y  y1  m(x  x1 )
y  3  2[ x  (4) ]
y  3  2(x  4)

241

Chapter 5

PQ249-6481F-05[217-243] 7/31/02 10:06 PM Page 242

Find the y-intercept.


y  mx  b
7  2(3)  b
76b
766b6
1b
Write the slope-intercept form.
y  mx  b
y  2x  1
9. D; The line parallel to y  3x  4 has the same
slope, 3. The slope of y  3x  5 is 3.
10. Let x  the lowest score.
Twice
the lowest score
minus
12
is
98.
14444244443
123
{
{
123
2x

12

98

Chapter 5 Standardized Test Practice


Pages 314315
1. B; The amount of the persons salary that is spent
is $(x  y). The fraction of the salary that is spent
is the ratio of the amount spent to the total salary
x  y
or x .
2. A; 2x  7y  2(5)  7(4)
 10  7(4)
 10  28
 38
3. D;
5x  6  10
5x  6  6  10  6
5x  4
5x
5

5

2x  12  98
2x  12  12  98  12
2x  110

x5
4. C; Mean age 



8(12  10(32  14(22  16(12  17(22


1  3  2  1  2

2x
2

116
9
8
129
8

To find the median age, order the numbers from


least to greatest.
8, 10, 10, 10, 14, 14, 16, 17, 17
The median age is 14.
The mode is 10.
Therefore, mean age  median age.
5. D; When the difference in the x values is 1, the
difference in the y values is 1. When the
difference in the x values is 2, the difference in
the y values is 2. This suggests that y  x.
Check this equation. If x  3, then y  (3) or
3. But the y value for x  3 is 4. This is a
difference of 1. Try some other values in the
domain to see if the same difference occurs.
2
2
3

0
0
1

2y
2

The y-intercept

4  6

m  0  (12
m

1
3
5
1 3 5
0 2 4

y
y

21
y

2  1

1 2

13.
14.

2 2  2(1)
y  2
7  4

7. A; m  4  2

15.
1

m  6 or 2
8. C; Find the slope.
21  7

16.

14
7

17.

m  10  3
m

Chapter 5

3x  3
2
3
3
2x  2
3
is 2 or 1.5.

12. Use the points (1, 6) and (0, 4) to find the


equation of the line.
Find the slope.

21

y

y is always 1 more than the opposite of x.


6. B; To find the y-intercept, let x  0.
x
3
0
3

110
2

x  55
The lowest score was 55.
11. Solve for y.
3x  2y  3
3x  2y  3x  3  3x
2y  3x  3

The mean age is 129.

x
x
y

or 2

242

2
1

or 2

The point (0, 4) lies on the y-axis.


The y-intercept is 4.
Write the slope-intercept form.
y  mx  b
y  2x  4
Use the equation y  2x + 4 with x  5.
y  2x  4
y  2(5)  4
y  6
The y-coordinate of the point (5, y) is 6.
The slope is 2.
A; 2(x  6)  2x  12
 2x  6  6
Therefore, 2(x  6) is 6 more than 2x  6.
D; Let x  3. The |x| 3 and |x  1|  4.
Let x  3. Then |x|  3 and |x  1| 2.
Therefore, the relationship cannot be determined.
C; Two nonvertical lines are parallel if they have
the some slope.
B; The slope of y  2x is 2. The slope of the
line perpendicular to y  2x is the opposite
1
reciprocal of 2 or 2.

PQ249-6481F-05[217-243] 7/31/02 10:06 PM Page 243

18c. Evaluate each equation when m  100.


For Plan 1: C  0.59m
C  0.59 (1002
C  59
For Plan 2: C  0.39m  10
C  0.39 (1002  10
C  49
For Plan 3: C  59.95
Your friend should enroll in Plan 2.
When m  100, the cost is least for Plan 2.

18a. For Plan 1:


Monthly
rate of
number of
monthly
cost
plus
fee.
equals change times minutes
14243 123 14243 123 14243 123 14243
.

C
0.59
m

0

The equation for Plan 1 is C  0.59m.


For Plan 2:
monthly
Monthly
rate of
number of
fee.
cost
plus
equals change times minutes
123 14243 123 14243 123 14243
.

C
0.39
m

10

14243

The equation for Plan 2 is C  0.39m  10.


For plan 3:
Monthly
rate of
number of
monthly
cost
plus
fee.
equals change times minutes
123 14243 123 14243 123 14243
.

C
0
m

59.95

14243

The equation for Plan 3 is C  59.95.


18b. The graph of C  0.59m passes through (0, 0)
with slope 0.59. The graph of C  0.39m  10
passes through (0, 10) with slope 0.39. The
graph of C  59.95 passes through (0, 59.95)
with slope 0.
C
60
50

C  59.95

40

C  0.59m

30

C  0.39m  10
20
10

20

40

60

80

100

243

Chapter 5

PQ249-6481F-06[244-272] 24/7/02 05:06 PM Page 244 Sahuja Ahuja_QXP_13:InProcess_AQ13:Sunita_23/24/7/02:PQ249-06-ReproPages:PQ249-06-Art:

Chapter 6
Page 317

Solving Linear Inequalities


Getting Started

t  31  84
t  31  31  84  31
t  53
b  17  23
2.
b  17  17  23  17
b  40
3.
18  27  f
18  27  27  f  27
9  f
1.

2(n  3) 

13.

d32

4.

14.

d3323
7

d  6 or 1 6

15.

3r  45  4r
3r  45  3r  4r  3r
45  r
6.
5m  7  4m  12
5m  7  4m  4m  12  4m
m  7  12
m  7  7  12  7
m  19
7.
3y  4  16
3y  4  4  16  4
3y  12
5.

3y
3

16.

17.

18.

12
3

y4
8. 2a  5  3a  4
5a4
5a545
a  1
(1)(a)  (1)(1)
a1
1
k
2

9.
1
k
2

 11

1 2  2(11)

21.

k  22
4.3b  1.8  8.25
4.3b  1.8  1.8  8.25  1.8
4.3b  6.45
4.3b
4.3

11.

20.

4474

2
10.

19.

47
1
k
2
1
k
2

6  2x
2

y3x
Select five values for the domain and make a
table.

6.45
4.3

x
2
0
1
3
5

16
4

s4

Chapter 6

n  1
2
n  1
2 2

2n  6  n  1
2n  6  n  n  1  n
n61
n6616
n7
8 is eight units from zero in the negative
direction.
|8|  8
20 is twenty units from zero in the positive
direction.
|20|  20
30 is thirty units from zero in the negative
direction.
|30|  30
1.5 is one and five-tenths units from zero in the
negative direction.
|1.5|  1.5
|14  7|  |7|
7 is seven units from zero in the positive
direction.
|14  7|  |7|  7
|1  16|  |15|
15 is fifteen units from zero in the negative
direction.
|1  16|  |15|  15
|2  3|  |1|
1 is one unit from zero in the negative
direction.
|2  3|  |1|  1
|7  10|  |3|
3 is three units from zero in the negative
direction.
|7  10|  |3|  3
Solve the equation for y.
2x  2y  6
2x  2y  2x  6  2x
2y  6  2x
2y
2

b  1.5
6s  12  2(s  2)
6s  12  2s  4
6s  12  2s  2s  4  2s
4s  12  4
4s  12  12  4  12
4s  16
4s
4

n3

12.

244

3
3
3
3
3

3x
 (2)
0
1
3
5

y
5
3
2
0
2

(x, y)
(2, 5)
(0, 3)
(1, 2)
(3, 0)
(5, 2)

PQ249-6481F-06[244-272] 24/7/02 05:06 PM Page 245 Sahuja Ahuja_QXP_13:InProcess_AQ13:Sunita_23/24/7/02:PQ249-06-ReproPages:PQ249-06-Art:

Graph the ordered pairs and draw a line through


the points.

Graph the ordered pairs and draw a line through


the points.
y

2x  2y  6

y  2x  3

24. The only value in the range is 4. Since there is


no x in the equation, the value of x does not
depend on y. Therefore, x can be any real number.
Select five values for the domain and make a
table.

22. Solve the equation for y.


x  3y  3
x  3y  x  3  x
3y  3  x
3y
3

3  x
3
1
1  3x

y

x
3
1
0
2
4

Select five values for the domain and make a


table.
1

1  3x

(x, y)

6

1  3 (6)

1

(6, 1)

4

1  3 (4)

3

3

1  3 (3)

13

1  3 (0)

1  3 (1)

14, 13 2

(x, y)
(3, 4)
(1, 4)
(0, 4)
(2, 4)
(4, 4)

y
4
4
4
4
4

Graph the ordered pairs and draw a line through


the points.
y

(3, 0)

(0, 1)

13, 1 13 2

y  4

Graph the ordered pairs and draw a line through


the points.
y

25. Solve the equation for y.

x  3y  3

x  2 y
O

(2)(x)  (2) 2 y

2x  y
Select five values for the domain and make a
table.
x
2
1
0
1
2

23. Select five values for the domain and make a


table.
x
1
0
1
2
3

2x  3
2(1)  3
2(0)  3
2(1)  3
2(2)  3
2(3)  3

y
5
3
1
1
3

(x, y)
(1, 5)
(0, 3)
(1, 1)
(2, 1)
(3, 3)

245

2x
2(2)
2(1)
2(0)
2(1)
2(2)

y
4
2
0
2
4

(x, y)
(2, 4)
(1, 2)
(0, 0)
(1, 2)
(2, 4)

Chapter 6

PQ249-6481F-06[244-272] 24/7/02 05:06 PM Page 246 Sahuja Ahuja_QXP_13:InProcess_AQ13:Sunita_23/24/7/02:PQ249-06-ReproPages:PQ249-06-Art:

The graph intersects the y-axis at (0, 5).


Plot these points and draw the line that connects
them.

Graph the ordered pairs and draw a line through


the points.
y

y
x   12 y

15  3(x  y )

26. To find the x-intercept, let y  0.


3x  6  2y
3x  6  2(0)
3x  6  0
3x  6  6  0  6
3x  6
3x
3

28. To find the x-intercept, let y  0.


2  x  2y
2  x  2 (0)
2x0
2xx0x
2x
The graph intersects the x-axis at (2, 0).
To find the y-intercept, let x  0.
2  x  2y
2  0  2y
2  2y

3

x2
The graph intersects the x-axis at (2, 0).
To find the y-intercept, let x  0.
3x  6  2y
3(0)  6  2y
6  2y
6
2

2
2

2y
2

1y
The graph intersects the y-axis at (0, 1).
Plot these points and draw the line that connects
them.

2y
2

3  y
The graph intersects the y-axis at (0, 3). Plot
these points and draw the line that connects
them.

y
2  x  2y
O

3x  6  2y

27. To find the x-intercept, let y  0.


15  3 (x  y)
15  3 (x  0)
15  3x
15
3

3x
3

Page 321

5x
The graph intersects the x-axis at (5, 0).
To find the y-intercept, let x  0.
15  3 (x  y)
15  3 (0  y)
15  3y
15
3

Check for Understanding

1. Sample answers: y  1  2, y  1  4,


y30
2.

3y
3

5y

Chapter 6

Solving Inequalities by Addition


and Subtraction

6-1

3
4
a4

3
4
a4

In both graphs, the line is darkened to the left. In


the graph of a  4, there is a circle at 4 to
indicate that 4 is not included in the graph. In the
graph of a  4, there is a dot at 4 to indicate that
4 is included in the graph.

246

PQ249-6481F-06[244-272] 24/7/02 05:06 PM Page 247 Sahuja Ahuja_QXP_13:InProcess_AQ13:Sunita_23/24/7/02:PQ249-06-ReproPages:PQ249-06-Art:

3. {b|b  5} is the set of all numbers b such that b


is greater than or equal to 5.
4.
m3 7 7
m33 7 73
m 7 4
The solution set is {m|m  4}. The graph must
have a heavy arrow pointing to the right to show
that the inequality includes all numbers greater
than 4. The graph must have a circle at 4 to show
that 4 is not included in the inequality. This is
represented by graph a.
5.
a4 6 2
a44 6 24
a 6 2
Check:
Substitute 2, a number less than 2,
and a number greater than 2.
Let a  2.
Let a  5.
Let a  6.
?
?
?
2  4 6 2
5  4 6 2
64 6 2
2 2
1 6 2
10 2
The solution set is {a | a  2}.

9.

5.2r  6.7  6.2r


5.2r  6.7  5.2r  6.2r  5.2r
6.7  r
6.7  r is the same as r  6.7.
Check:
Substitute 6.7, a number less than 6.7,
and a number greater than 6.7.
Let r  6.7.
?
5.2 (6.7)  6.7  6.2 (6.7)
?

34.84  6.7  41.54


41.54  41.54
Let r  1.
?
5.2 (1)  6.7  6.2 (1)
11.9  6.2
Let r  10.
?
5.2 (10)  6.7  6.2 (10)
?

52  6.7  62
58.7
62
The solution set is {r | r  6.7}.
0 1 2 3 4 5 6 7 8

8 765 4 321 0

6.

10.

9b4
94b44
5b
5  b is the same as b  5.
Check:
Substitute 5, a number less than 5,
and a number greater than 5.
Let b  5.
Let b  0.
Let b  10.
?
?
?
954
904
9  10  4
99
9 4
9  14
The solution set is {b | b  5}.

7p  6p  2
7p  6p  6p  2  6p
p  2
Check:
Substitute 2, a number less than 2,
and a number greater than 2.
Let p  2.
?
7 (2)  6(2)  2
?

14  12  2
14  14
Let p  5.
?
7 (5)  6 (5)  2
?

35  30  2
35  32
Let p  1.
?
7(1)  6(1)  2
7 4
The solution set is { p|p  2}.

0 1 2 3 4 5 6 7 8

7.

t75
t7757
t  12
Check:
Substitute 12, a number less than 12,
and a number greater than 12.
Let t  12.
Let t  2.
Let t  15.
?
?
?
12  7  5
275
15  7  5
55
5
5
85
The solution set is {t | t  12}.

432 1 0 1 2 3 4

11. Let n  the number.


decreased
A number
by 8
14
424
43

y  2.5 7 3.1
y  2.5  2.5 7 3.1  2.5
y 7 5.6
Check:
Substitute 5.6, a number less than 5.6,
and a number greater than 5.6.
Let y  5.6.
Let y  1.
Let y  8.
?

1
424
3

14.

123

n
8

14
n  8  14
n  8  8  14  8
n  22
Check:
Substitute 22, a number less than 22,
and a number greater than 22.
Let n  22.
Let n  10.
Let n  30.
?
?
?
22  8  14
10  8  14
30  8  14
22 14
14  14
2  14
The solution set is {n|n  22}.

0 2 4 6 8 10 12 14 16

8.

14
424
43

is at
most

5.6  2.5 7 3.1 1  2.5 7 3.1 8  2.5 7 3.1


3.1 3.1
1.5 3.1
5.5 7 3.1
The solution set is { y | y  5.6}.
0 1 2 3 4 5 6 7 8

247

Chapter 6

PQ249-6481F-06[244-272] 24/7/02 05:06 PM Page 248 Sahuja Ahuja_QXP_13:InProcess_AQ13:Sunita_23/24/7/02:PQ249-06-ReproPages:PQ249-06-Art:

12. Let n  the number.


A number

plus

is greater
7
than

14
424
43

123

14
424
43

2.
{

Exercises 2037 For checks, see students work.


20.
t  14  18
t  14  14  18  14
t4
The solution set is {t|t  4}.

n7
n77
n
Check:

7 2
7 27
7 5
Substitute 5, a number less than 5,
and a number greater than 5.
Let n  5.
Let n  10.
Let n  3.
?
?
?
5  7 7 2
10  7 7 2
37 7 2
2 2
3 2
10 7 2
The solution set is {n|n  5}.
13. Let g  the number of grams of fat Chapa can
have during the rest of the day.
2132  21  g  60
6  21  g  60
27  g  60
27  g  27  60  27
g  33
Chapa can have no more than 33 g of fat during
the rest of the day.

Pages 321323

0 1 2 3 4 5 6 7 8

21.

0 1 2 3 4 5 6 7 8

22.

16.

17.

18.

19.

Practice and Apply

s  5  1
s  5  5  1  5
s4
The solution set is {s|s  4}.
0 1 2 3 4 5 6 7 8

x  3  2
x  3  3  2  3
x1
The solution set is {x|x  1}. The corresponding
graph is d.
f;
x76
x7767
x  1
The solution set is {x|x  1}. The corresponding
graph is f.
a;
4x 7 3x  1
4x  3x 7 3x  1  3x
x 7 1
The solution set is {x|x  1}. The corresponding
graph is a.
c;
8x 6 9
8x8 6 98
x 6 1
The solution set is {x|x  1}. The corresponding
graph is c.
e;
5x6
56x66
1  x
1  x is the same as x  1.
The solution set is {x|x  1}. The corresponding
graph is e.
b;
x1 7 0
x11 7 01
x 7 1
The solution set is {x|x  1}. The corresponding
graph is b.

Chapter 6

n  7  3
n  7  7  3  7
n4
The solution set is {n|n  4}.
0 1 2 3 4 5 6 7 8

23.

14. d;

15.

d57
d5575
d2
The solution set is {d|d  2}.

24.

53g
533g3
2g
2  g is the same as g  2.
The solution set is { g|g  2}.
0 1 2 3 4 5 6 7 8

25.

48r
488r8
4  r
4  r is the same as r  4.
The solution set is {r|r  4}.
8 765 4 321 0

26.

3  q  7
3  7  q  7  7
4q
4  q is the same as q  4.
The solution set is {q|q  4}.
0 1 2 3 4 5 6 7 8

27.

2m1
21m11
3m
3  m is the same as m  3.
The solution set is {m|m  3}.
0 1 2 3 4 5 6 7 8

248

PQ249-6481F-06[244-272] 24/7/02 05:06 PM Page 249 Sahuja Ahuja_QXP_13:InProcess_AQ13:Sunita_23/24/7/02:PQ249-06-ReproPages:PQ249-06-Art:

28.

2y  8  y
2y  y  8  y  y
y  8
The solution set is { y|y  8}.

3f  3  2f
3f  2f  3  2f  2f
f  3
The solution set is { f|f  3}.

432 1 0 1 2 3 4
2

p39

37.

p3393
1

p  19

3b  2b  5
3b  2b  2b  5  2b
b  5
The solution set is {b|b  5}.

The solution set is

5 p|p  1 19 6.

432 1 0 1 2 3 4

d  5  17
d  5  5  17  5
d  12
38b.
d  5  17
d  5  3  17  3
d  8  20
38c.
d  5  17
d  5  10  17  10
d57
39a.
z  2  10
z  2  2  10  2
z  12
39b.
z  2  10
z  2  5  10  5
z75
39c.
z  2  10
z  2  6  10  6
z  4  16
Exercises 4045 For checks, see students work.
40. Let n  the number.
n  13  27
n  13  13  27  13
n  14
The solution set is {n|n  14}.
41. Let n  the number.
n  5  33
n  5  5  33  5
n  38
The solution set is {n|n  38}.
42. Let n  the number.
30  n  (8)
30  n  8
30  8  n  8  8
38  n
38  n is the same as n  38.
The solution set is {n|n  38}.
43. Let n  the number.
2n  n  14
2n  n  n  14  n
n  14
The solution set is {n|n  14}.
38a.

4w  3w  1
4w  3w  3w  1  3w
w1
The solution set is {w|w  1}.
432 1 0 1 2 3 4

432 1 0 1 2 3 4

33. a  (2)  3
a  2  3
a  2  2  3  2
a  5
The solution set is {a|a  5}.
8 765 4 321 0

0.23  h  (0.13)
0.23  h  0.13
0.23  0.13  h  0.13  0.13
0.36  h
0.36  h is the same as h  0.36.
The solution set is {h|h  0.36}.
432 1 0 1 2 3 4

35.

The solution set is a|a 7 8 .

32. v  (4)  3
v43
v4434
v  1
The solution set is {v|v  1}.

34.

4
1

8 765 4 321 0

31.

1
8
1
8

a 7 8

8 7654 32 1 0

30.

a44 7

8 765 4 321 0

29.

a4 7

36.

x  1.7  2.3
x  1.7  1.7  2.3  1.7
x  0.6
The solution set is {x|x  0.6}.
432 1 0 1 2 3 4

249

Chapter 6

PQ249-6481F-06[244-272] 24/7/02 05:06 PM Page 250 Sahuja Ahuja_QXP_13:InProcess_AQ13:Sunita_23/24/7/02:PQ249-06-ReproPages:PQ249-06-Art:

53a. If a  b, then a  c  b  c by the Addition


Property of Inequalities. If c  d, then b  c 
b  d by the Addition Property of Inequalities.
Since a  c  b  c and b  c  b  d, it follows
that a  c  b  d by the Transitive Property of
Inequalities (If x  y and y  z, then x  z.).
Therefore, the given statement is always true.
53b. If x  y is always true, then x  y is never true.
In Exercise 53, it was shown that, if a  b and
c  d, the statement a  c  b  d is always
true. Therefore, the statement a  c  b  d is
never true.
53c. If a  1, b  2, c  3, and d  5, then a  c 
1  3 or 2, but b  d  2  5 or 3. In this
case, a  c  b  d.
However, if a  1, b  2, c  3, and d  4, then
a  c  1  3 or 2, and b  d  2  4 or 2.
In this case, a  c  b  d. Therefore, the given
statement is sometimes true.
54. 225  p  200
55.
225  p  200
225  p  p  200  p
225  200  p
225  200  200  p  200
25  p
25  p is the same as p  25.
The solution set is {p|p  25}.
56. Inequalities can be used to compare the number
of schools participating in certain sports, to
compare the number of participating schools if
sports are added or discontinued in a certain
number of schools, and to determine how many
schools need to add a certain sport to surpass the
number participating in another sport. Answers
should include the following.
To find how many schools must add girls track
and field to surpass the current number of
schools participating in girls basketball, solve
16,526  14,587  x. More than 1939 schools
must add girls track and field.
57. C; x  1  13
x  1  1  13  1
x  14
is at
The
sum of a
58. A;
least
five.
number
and
6
144424443 123 123

n6
5

44. Let n  the number.


n  (7)  18
n  7  18
n  7  7  18  7
n  25
The solution set is {n|n  25}.
45. Let n  the number.
4n  3n  (2)
4n  3n  3n  (2)  3n
n  2
The solution set is {n|n  2}.
46. Let n  the number of pounds a Nile crocodile
might be expected to gain.
n  157  2200
n  157  157  2200  157
n  2043
The crocodile would be expected to gain no more
than 2043 lb.
47. Let s  the number of stars that cannot be seen
without a telescope.
s  1100  200,000,000,000
s  1100  1100  200,000,000,000  1100
s  199,999,998,900
There are at least 199,999,998,900 stars that
cannot be seen without a telescope.
48. Let n  the number of species of insects that are
not bees.
n  3500  600,000
n  3500  3500  600,000  3500
n  596,500
There are more than 596,500 species of insects
that are not bees.
49. Let m  the amount of money in the account.
m  1300  947  1500
m  2247  1500
m  2247  2247  1500  2247
m  3747
Mr. Hayashi should have at least $3747 in his
account before writing the checks.
50.
12  4  x
12  4  4  x  4
8 x
8  x is the same as x  8.
The value of x must be more than 8 in.
51. Let j  the cost of a pair of jeans.
18  14  j  65
32  j  65
32  j  32  65  32
j  33
The jeans must cost no more than $33.
52. Let w  the number of wins to meet the goal.
w  4  0.60(18)
w  4  10.8
w  4  4  10.8  4
w  6.8
The team must win at least 7 more games to meet
their goal.

Chapter 6

Page 323

Maintain Your Skills

59. Since a persons height is not related to his or her


grade on a math test, a scatter plot for the
relationship would show no correlation.
60. The line parallel to y  3x  2 has the same
slope, 3. Replace m with 3, and (x1, y1) with
(1, 3) in the point-slope form.
y  y1  m(x  x1)
y  (3)  3(x  1)
y  3  3x  3
y  3  3  3x  3  3
y  3x  6

250

PQ249-6481F-06[244-272] 24/7/02 05:06 PM Page 251 Sahuja Ahuja_QXP_13:InProcess_AQ13:Sunita_23/24/7/02:PQ249-06-ReproPages:PQ249-06-Art:

66.

61. Find the slope of the given line.


x  y  3
x  y  x  3  x
y  x  3
The slope of the given line is 1, so the slope of
the line parallel to y  x  3 has the same
slope, 1. Replace m with 1 and (x1, y1) with
(0, 4) in the point-slope form.
y  y1  m(x  x1)
y  4  1(x  0)
y  4  x
y  4  4  x  4
y  x  4
62. Find the slope of the given line.
2x  y  1
2x  y  2x  1  2x
y  2x  1
1(y)  1(2x  1)
y  2x  1
The slope of the given line is 2, so the slope of the
line parallel to y  2x  1 has the same slope, 2.
Replace m with 2 and (x1, y1) with (1, 2) in the
point-slope form.
y  y1  m(x  x1)
y  2  2[x  (1)]
y  2  2(x  1)
y  2  2x  2
y  2  2  2x  2  2
y  2x  4
63. 7
13
19
25

y
8
4
2

(x, y)
(1, 8)
(3, 4)
(5, 2)

2x  6
2(1)  6
2(3)  6
2(5)  6

y
8
0
4

(x, y)
(1, 8)
(3, 0)
(5, 4)

The solution set is {(1, 8), (3, 0), (5, 4)}.


69. 6g  42
6g
6

42
6

g7
t
9
t
9

70.
192
71.

 14

 19214

t  126

2
y
3
3 2
y
2 3

 14

1 21 2  132 214

y  21
72. 3m  435

The next two terms are 31 and 37.


64. 243
81
27
9

3m
3




162
54
18

435
3

m  145
1

The difference between each pair of terms is 3 the


1
difference of the previous pair. Continue taking 3
of each successive difference. Add 6, then
add 2.
9

7x
7  (1)
73
75

x
1
3
5

x
1
3
5






6
6
6
6
6

27

(x, y)
(1, 2)
(3, 6)
(5, 10)

The solution set is {(1, 8), (3, 4), (5, 2)}.


68. First solve the equation for y in terms of x.
2x  y  6
2x  y  2x  6  2x
y  6  2x
1(y)  1(6  2x)
y  2x  6

The difference between each pair of terms is


always 6. The sequence is arithmetic with a
common difference of 6. Add 6, then add 6.
7
13
19
25
31
37

81

y
2
6
10

The solution set is {(1, 2), (3, 6), (5, 10)}.


67.




6
6
6

243

2x
2(1)
2(3)
2(5)

x
1
3
5

73.

4
x
7
7 4
x
4 7

 28

1 21 2  174 228

x  49
74. 5.3g  11.13





162
54
18
6
2

5.3g
5.3

The next two terms are 3 and 1.


65. 3
6
12
24

11.13
5.3

g  2.1




3
6
12

75.
13.52

The difference between each pair of terms doubles


for each successive pair. Continue doubling each
successive difference. Add 24, then add 48.
3
6
12
24
48
96

76.

a
3.5
a
3.5

 13.52 7

a  24.5
8p  35
8p
8






3
6
12
24
48

7

p

The next two terms are 48 and 96.

251

35
8
35
8

or 4.375

Chapter 6

PQ249-6481F-06[244-272] 24/7/02 05:06 PM Page 252 Sahuja Ahuja_QXP_13:InProcess_AQ13:Sunita_23/24/7/02:PQ249-06-ReproPages:PQ249-06-Art:

Page 324

Step 4 Separate the tiles into 4 groups.

Algebra Activity
(Preview of Lesson 6-2)

1. 4x  12
Step 1 Write a  symbol and model the
inequality.

x

x

x
x

1

1

1

1

1

1

1

1

1

1

3  x or x  3

The solution set is {x|x  3}.


2. 2x  8
Step 1 Write a  symbol and model the
inequality.

Step 2 Since you do not want to solve for a


negative x-tile, eliminate the negative by
adding 4 positive x-tiles to each side.
Remove the zero pairs.
x

x

1

4x  12

x

1

x


x

x

2x  8

Step 2 Since you do not want to solve for a


negative x-tile, eliminate the negative
x-tiles by adding 2 positive x-tiles to each
side. Remove the zero pairs.

4x  4x  12  4x

Step 3 Add 12 negative 1-tiles to each side to


isolate the x-tiles. Remove the zero pairs.

x

x
1 1

1 1 1

1 1

1 1

1 1

1 1

1 1

1 1

1 1

1 1

1 1

1 1

1 1

x

x
2x  2x  8  2x
Step 3 Add 8 negative 1-tiles to each side to
isolate the x-tiles. Remove the zero pairs.

1 1 1

1 1 1

1 1 1

x
12  4x

252

1 1

1 1

1 1

1 1

1 1

1 1

1 1

1 1

1

1

1

1

1

1

1

1

8  2x

Chapter 6

PQ249-6481F-06[244-272] 24/7/02 05:06 PM Page 253 Sahuja Ahuja_QXP_13:InProcess_AQ13:Sunita_23/24/7/02:PQ249-06-ReproPages:PQ249-06-Art:

Step 4 Separate the tiles into 3 groups.

Step 4 Separate the tiles into 2 groups.


1

1

1

1


1

1

1

1

4  x or x  4
The solution set is {x|x  4}.
3. 3x  6
Step 1 Write a  symbol and model the
inequality. Use a self-adhesive note to
cover the equals sign on the equation
mat. Then write a  symbol on the note.
Model the inequality.

2  x or x  2

The solution set is {x|x  2}.


4. 5x  5
Step 1 Write a  symbol and model the
inequality.
x

1

x

1

x
x

x
x

1

1

1

1

1

1

x

1

x

1

1

1

x

1

1

1

1

1

1

x

1

x

1

Step 2 Since you do not want to solve for a


negative x-tile, eliminate the negative
x-tiles by adding 5 positive x-tiles to each
side. Remove the zero pairs.

Step 2 Since you do not want to solve for a


negative x-tile, eliminate the negative
x-tiles by adding 3 positive x-tiles to each
side. Remove the zero pairs.

x

5x  5

3x  6

x

1

x

1

x
x

x

x

3x  3x  6  3x

Step 3 Add 6 positive 1-tiles to each side to


isolate the x-tiles. Remove the zero pairs.

5x  5x  5  5x

1 1

1 1

1 1

1 1

1 1

1 1

1

1

1

1

1

1

6  3x

253

Chapter 6

PQ249-6481F-06[244-272] 24/7/02 05:06 PM Page 254 Sahuja Ahuja_QXP_13:InProcess_AQ13:Sunita_23/24/7/02:PQ249-06-ReproPages:PQ249-06-Art:

There are no negative x-tiles, so the variable


remains on the left and the symbol remains .

Step 3 Add 5 positive 1-tiles to each side to


isolate the x-tiles. Remove the zero pairs.

1

1

1

1

1

Pages 328329

1
1
1

2. Sample answer: Three fourths of a number is


greater than 9.
3. Ilonia is correct since when you divide each side
of an inequality by a negative number, you must
reverse the direction of the inequality symbol.
4. a; since the phrase is at least is translated as ,
statement a represents 7n  14.
5. c; since the phrase is less than is translated as ,
inequality c represents the statement.
6. 15g 7 75

5  5x

Step 4 Separate the tiles into 5 groups.


1

15g
15


1

7.
192

108
9

180

18
9

6 12
2 12

3b  9

132 2 123b2  132 2 (9)

3 113.52  9
2

3 1122  9 3 1182  9
2

9  9
8 9
The solution set is {b|b  13.5}.




x3

Chapter 6

Check:

Step 2 Since there is no need to eliminate x-tiles


or isolate x-tiles, the only step remaining
is to group the tiles.

6 1921122

b  13.5
Substitute 13.5, a number less than
13.5, and a number greater than 13.5.
Let b  13.5.
Let b  12.
Let b  18.

2x  6

6 12

6 12
6 12
9
12 12
20 12
The solution set is {t|t  108}.
8.

t
9
t
9

t 6 108
Check:
Substitute 108, a number less than
108, and a number greater than
108.
Let t  108.
Let t  180.
Let t  18.




75
15

15 152 7 75 15 1102 7 75


15 152 7 75
75 75
150 7 75
75 75
The solution set is { g|g  5}.

The solution set is {x|x  1}.


5. In Exercises 14, the coefficients of x are 4,
2, 3, and 5, respectively. Thus, all
coefficients are negative.
6. The symbols in the solutions point in the opposite
direction with relationship to the variable than
the symbols in the original problem.
7. 2x  6
Step 1 Model the inequality.
1

g 6 5
Check:
Substitute 5, a number less than 5,
and a number greater than 5.
Let g  5.
Let g  10.
Let g  5.

1  x or x  1

Check for Understanding

1. You could solve the inequality by multiplying each


1
side by 7 or by dividing each side by 7. In
either case, you must reverse the direction of the
inequality symbol.

x
x
x
x
x

Solving Inequalities by
Multiplication and Division

6-2

254

12  9

PQ249-6481F-06[244-272] 24/7/02 05:06 PM Page 255 Sahuja Ahuja_QXP_13:InProcess_AQ13:Sunita_23/24/7/02:PQ249-06-ReproPages:PQ249-06-Art:

9. 25f  9
25f
25

Examples 1934 For checks, see students work.


19. 6g  144

9
25

6g
6

f  0.36
Check:
Substitute 0.36, a number less than
0.36, and a number greater than 0.36.
Let f  0.36.
Let f  0.
Let f  1.
?
?
?
25 10.362  9
25 102  9
25 112  9
99
0
9
25  9
The solution set is {f|f  0.36}.
10. Let n  the number.

g  24
The solution set is {g|g  24}.
20. 7t 7 84
7t
7

The opposite of 4 times a number is more than 12.



4n
7
12

14d
14

4n 7 12
6

16z
16

23.

24.

 (2) 26

6x
6

7
 (5)7

b
10
b
(10) 10

5
 (10)5

b  50
The solution set is {b|b  50}.
r

7 6 7

25.

1 r2

(7) 7 7 (7) (7)

?
1
1602 
2

r 7 49
The solution set is {r|r  49}.

26
30  26
26.

11 7 9
a

(11) 11 6 (11)9

24
6

a 6 99
The solution set is {a|a  99}.

6
x 6 4

27.

Pages 329331

m
5
m
(5) 5

m  35
The solution set is {m|m  35}.

 26

1522  26
1402  26
2
26  26
20
26
The solution set is {n|n  52}.
12. B; 6x 6 24

64

 16

z4
The solution set is {z|z  4}.

n  52
Check:
Substitute 52, a number less than 52,
and a number greater than 52.
Let n  52.
Let n  40.
Let n  60.
1
2

84

 14

d  6
The solution set is {d|d  6}.
22. 16z  64

12
4

n 6 3
Check:
Substitute 3, a number less than 3,
and a number greater than 3.
Let n  3.
Let n  5.
Let n  1.
?
?
?
4 132 7 12
4 152 7 12
4 112 7 12
12 12
20 7 12
4 12
The solution set is {n|n  3}.
11. Let n  the number.
Half of a number
is at least 26.
123 123 14424
43 1442443 123
1
n


26
2
1
n
2
1
(2) 2 n

84
7

t 7 12
The solution set is {t|t  12}.
21. 14d  84

144424443 14444244443 1442443 123

4n
4

144
6

Practice and Apply

5
y
8
8 5
y
5 8

12

 15


185 2 (15)

y  24
The solution set is {y y  24} .

13. d; 5 n 7 10 can be translated One fifth of a


number is greater than ten.
14. a; 5n  10 can be translated Five times a number
is less than or equal to ten.
15. e; 5n  10 can be translated Five times a number
is greater than ten.
16. f; 5n  10 can be translated Negative five times
a number is less than ten.

28.

2
v
3
3 2
v
2 3

12

6 6

132 26

v 6 9
The solution set is {v v  9}.
29.

17. b; 5 n  10 can be translated One fifth of a


number is no less than ten.
18. c; 5n  10 can be translated Five times a number
is less than ten.

4q  33

143 2134q2  143 2 (33)

q  44
The solution set is {q q  44}.

255

Chapter 6

PQ249-6481F-06[244-272] 24/7/02 05:06 PM Page 256 Sahuja Ahuja_QXP_13:InProcess_AQ13:Sunita_23/24/7/02:PQ249-06-ReproPages:PQ249-06-Art:

30.

38c.

5 p 7 10

152 2125 p2 6 152 210

p 6 25
The solution set is {p p  25}.
31. 2.5w 6 6.8
2.5w
2.5

Exercises 3944 For checks, see students work.


39. Let n  the number.
7n 7 28

6.8
2.5

7n
7

w 7 2.72
The solution set is {w w 7 2.72}.
32. 0.8s 7 6.4
0.8s
0.8

7n
7

s 6 8
33.

3
14
7 3
15 14

1 21 2 6 1 21 2
7

c 6

1
10

24  3n

The solution set is c c 6


34.

4m
5
5 4m
4 5

3
15
5 3
4 15
1
4

1 21 2 6 1 21 2
6

m 6

1
10

The solution set is m m 6


35.
(8)

y
8
y
8

1 26
7

1
2

6.

1
4

(3)24  (3) 3n
72  n
72  n is the same as n  72.
The solution set is {n n  72}.
42. Let n  the number.

2
n
3
3 2
n
2 3

36.
(9)

0.25n
0.25

m3
The solution set is {m m  3}.
1

0.40n
0.40

2

a  3.5
37b.

37c.

2a  7
(2)2a  (2)7
4a  14

20w
20

46. Let b  the number of bags of mulch to be sold.


2.50b  2000
2.50b
2.50

2
4

4t 6 2
(2)4t 7 (2)(2)
8t 7 4

Chapter 6

2000
2.50

b  800
The band should sell at least 800 bags of mulch.

t 6 0.5
38b.

85
20
1
44

The width is less than 44 ft.

38a. 4t 6 2
6

w 6

2a  7
(3)2a  (3)7
6a  21
4t
4

45

 0.40

n  112.5
The solution set is {n n  112.5}.
45. Let w  the width of the rectangle.
A 6 85
/w 6 85
20w 6 85

37a. 2a  7
2a
2

90

 0.25

n  360
The solution set is {n n  360}.
44. Let n  the number.
0.40n  45

1
3

1 2  (9) 113 2

132 2 (15)

n 6 22.5
The solution set is {n n  22.5}.
43. Let n  the number.
0.25n  90

1
(8) 2

6 5 4 3 2 1

6 15

12

y 6 4
The solution set is {y y  4}.

m
9
m
9

14

 7

n  2
The solution set is {n n  2}.
41. Let n  the number.

The solution set is {s s  8}.


15c
7
7 15c
15 7

28
7

n 7 4
The solution set is {n n 7 4}.
40. Let n  the number.
7n  14

6.4
0.8

4t 6 2
(7)4t 7 (7) (2)
28t 7 14

256

PQ249-6481F-06[244-272] 24/7/02 05:06 PM Page 257 Sahuja Ahuja_QXP_13:InProcess_AQ13:Sunita_23/24/7/02:PQ249-06-ReproPages:PQ249-06-Art:

54. Let s  the number of signatures to be sought.


0.85s  6000

47. Let m  the number of minutes Juan can talk.


0.09m  2.50
0.09m
0.09

2.50

0.85s
0.85

 0.09

m  27. 7
Juan can talk to his friend for no more than
27 min.
48. Let p  the number of people attending the
reunion.
28.95p  4000
28.95p
28.95

4000

 28.95

56. B; 5  1
x

(5) 5  (5) (1)

38
2

x  5

r 6 6.0 (to the nearest tenth)


The garden can have a radius of up to about 6 ft.
50. Let d  the distance that can be traveled legally.
d
6 65
1
12

11 12 2 1d1

d 6

16

t 7 15

The solution set is

132 265

Page 331

5t 0 t 7 1615 6 .

Maintain Your Skills

Exercises 5860 For checks, see students work.


58.
s  7 6 12
s  7  7 6 12  7
s 6 19
The solution set is {s s  19}.

1
97 2

A person can legally travel no more than 972 mi.


51. Let n  the number of visits to the zoo in one
year (assuming two adults and two children go to
the zoo each visit).
144 6 [ 2(18)  2(8) ] n
144 6 52n

15 16 17 18 19 20 21 22 23

59.

52n
52

2.8 6 n (to the nearest tenth)


The yearly membership will be less expensive
than regular admission if the family (of two
adults and two children) visits the zoo at least
3 times during the year.
52a. Sample answer: Let a  2 and b  3.
It is true that a  b since 2  3. However, it is
not true that a2  b2 since 4 9.
52b. Sample answer: Let a  1, b  2, c  3, and
d  2.
It is true that a  b since 1  2. It is also true
that c  d since 3  2. However, it is not true
that ac  bd since ac  (1)(3) or 3, bd 
(2)(2) or 4, and 3 4.
53. Let s  the number of parking spaces in the lot.
0.20s  35
0.20s
0.20

14
15

187 2178t2 7 187 211415 2

1 12

8 t 6

57. C;

6 1 2 65

d 6

144
52

6000
0.85

s  7058.8 (to the nearest tenth)


The candidate should seek at least 7059
signatures on the petition.
55. Inequalities can be used to compare the heights of
walls. Answers should include the following.
If x represents the number of bricks and the
wall must be no higher than 4 ft or 48 in.,
then 3x  48.
To solve this inequality, divide each side by 3
and do not change the direction of the
inequality symbol. The wall must be no more
than 16 bricks high.

p  138.2 (to the nearest tenth)


At least 139 people must attend the reunion to
avoid a rental fee.
49. Let r  the radius of the garden.
C 6 38
2r 6 38
2r
2

g  3  4
g  3  3  4  3
g  7
The solution set is {g g  7}.
8 7 6 5 4 3 2 1

60.

7 7 n2
72 7 n22
5 7 n
5  n is the same as n  5.
The solution set is {n n  5}.
0

35

 0.20

s  175
The parking lot must have at least 175 spaces.

257

Chapter 6

PQ249-6481F-06[244-272] 24/7/02 05:06 PM Page 258 Sahuja Ahuja_QXP_13:InProcess_AQ13:Sunita_23/24/7/02:PQ249-06-ReproPages:PQ249-06-Art:

Choose (3, 3) and find the y-intercept of the line.


y  mx  b

61. Sample answer:


Since y increases as x increases, there is a
positive correlation between x and y in the graph
shown.

3  4 (3)  b
3

34b

9
4

b

344b4
1

Write the slope-intercept form using m  4 and


9
b  4.

y  mx  b

62. Find the slope. Let (x1, y1)  (1, 3) and (x2, y2) 
(2, 4).

h(x)  3x  2
h(4)  3(4)  2
 12  2
 10
67. h(x)  3x  2
h(w)  3(w)  2
 3w  2
65.

y2  y1

mx

y  4x  4

 x1

4  3
(1)

m2
1

m3
Choose (2, 4) and find the y-intercept of the line.
y  mx  b
1

4  3 (2)  b

69.

43b
2

10
3

b

24
4

71.

y  mx  b

m
m

1.6t
1.6

 x1

72.

m
m

x
3

12x
12

25
5

x  5
15

15

 12
5

x  4 or 1 4
73.

5x  3  32
5x  3  3  32  3
5x  35
5x
5

74.

4t  9  14
4t  9  9  14  9
4t  5

35
5

x7
75.

 x1

2  3
1  3
1
4
1
4

Chapter 6

15(x)  3 (x  5)
15x  3x  15
15x  3x  3x  15  3x
12x  15

y2  y1

m

7
5

w5

m0
Choose (5, 2) and find the y-intercept of the line.
y  mx  b
2  0(5)  b
2  b
Write the slope-intercept form using m  0 and
b  2.
y  mx  b
y  (0)x  (2)
y  2
The standard form of the equation is y  2.
64. Find the slope. Let (x1, y1)  (3, 3) and (x2, y2) 
(1, 2).
2

3.6

 1.6

t  2.25
w  2
5

5w
5

2  (2)
1  5
0
6

mx

t(1.6)  1.5(2.4)
1.6t  3.6

4x
4

y2  y1
2

2.4

 1.6

5(w  2)  5(7)
5w  10  35
5w  10  10  35  10
5w  25

10
3

63. Find the slope. Let (x1, y1)  (5, 2) and (x2, y2) 
(1, 2).
mx

t
1.5

70.

6x

Write the slope-intercept form using m  3 and


10
b 3.
1

8

3(8)  4(x)
24  4x

433b3

y  3x 

3
4

66. h(x)  3x  2
h(2)  3(2)  2
62
8
68.
h(x)  3x  2
h(r  6)  3(r  6)  2
 3r 18  2
 3r  16

6y  1  4y  23
6y  1  4y  4y  23  4y
2y  1  23
2y  1  1  23  1
2y  24
2y
2

24
2

y  12

258

4t
4

4
5

t  4 or 1.25

PQ249-6481F-06[244-272] 24/7/02 05:06 PM Page 259 Sahuja Ahuja_QXP_13:InProcess_AQ13:Sunita_23/24/7/02:PQ249-06-ReproPages:PQ249-06-Art:

14g  5
6

76.
6

Substitute 5, a number less than 5,


and a number greater than 5.
Let p  5.
Let p  8.
Let p  2.
?
?
?
4  5  9
4  8  9
4  2  9
44
41
4
7
The solution set is { p|p  5}.
Check:

9

114g6 5 2  6 (9)

14g  5  54
14g  5  5  54  5
14g  49
14g
14

49

 14

8 7 6 5 4 3 2 1

g  2 or 3.5
77.

5a  6  9a  (7a  18)
5a  6  9a  7a  18
5a  6  2a  18
5a  6  2a  2a  18  2a
3a  6  18
3a  6  6  18  6
3a  24
3a
3

78.

4.

24
3

a  8
2( p  4)  7 ( p  3)
2p  8  7p  21
2p  8  2p  7p  21  2p
8  5p  21
8  21  5p  21  21
29  5p
29
5
29
5

5.

5p
5

p

1.

Practice Quiz 1

7(2)  6(2)  1
14 11
The solution set is {g|g  1}.

7(5)  6(5)  1
35  31

4 3 2 1

6. 15z  105
15z
15

105
15

z7
Check:

Substitute 7, a number less than 7,


and a number greater than 7.
Let z  7.
Let z  0.
Let z  10.
?
?
?
15(7)  105
15(0)  105
15(10)  105
105  105
0
105
150  105
The solution set is {z|z  7}.

r  3  1
r  3  3  1  3
r  4
Check:
Substitute 4, a number less then 4,
and a number greater than 4.
Let r  4.
Let r  10.
Let r  1.
?
?
?
4  3  1
10  3  1
1  3  1
1  1
7  1
4 1
The solution set is {r|r  4}.
8 7 6 5 4 3 2 1

3.

7(1)  6(1)  1
7  7
Let g  2.

h  16 7 13
h  16  16 7 13  16
h 7 3
Check:
Substitute 3, a number less than 3,
and a number greater than 3.
Let h  3.
Let h  1.
Let h  7.
?
?
?
3  16 7 13 1  16 7 13 7  16 7 13
13 13
15 13
9 7 13
The solution set is {h|h  3}.
0

2.

7g  6g  1
7g  6g  6g  1  6g
g  1
Check:
Substitute 1, a number less than 1,
and a number greater than 1.
Let g  1.
Let g  5.

p   5 or 5.8

29

Page 331

3  a  5
3  5  a  5  5
2a
2  a is the same as a  2.
Check:
Substitute 2, a number less than 2,
and a number greater than 2.
Let a  2.
Let a  0.
Let a  10.
?
?
?
3 6 2  5
3 6 0  5
3 6 10  5
3 3
3 5
3 6 5
The solution set is {a|a  2}.

7.

v
5
v
(5) 5

6 7
6 (5)7

v 6 35
Check:
Substitute 35, a number less than 35,
and a number greater than 35.
Let v  35.
Let v  30.
Let v  70.
35
5

4p9
4  9  p  9 9
5  p
5  p is the same as p  5.

6 7

30
5

6 7

7 7
6 6 7
The solution set is {v|v  35}.

259

70
5

6 7

14 7

Chapter 6

PQ249-6481F-06[244-272] 24/7/02 05:06 PM Page 260 Sahuja Ahuja_QXP_13:InProcess_AQ13:Sunita_23/24/7/02:PQ249-06-ReproPages:PQ249-06-Art:

8.

7 q 7 15

173 2137q2 6 173 215

q 6 35
Substitute 35, a number less than
35, and a number greater than 35.
Let q  35.
Let q  70.
Let q  0.
?
?
?
3
3
3
7 (0) 7 15
7 (35) 7 15
7 (70) 7 15

Pages 334335

Check:

15 15
30 7 15
The solution set is {q|q  35}.
9. 156 6 12r
156
12

0 15

13 6 r
13  r is the same as r  13.
Check:
Substitute 13, a number less than
13, and a number greater than 13.
Let r  20.

Let r  10.

156 6 12(13)
156 156

4y
4

156 6 12(20) 156 6 12(10)


156 240
156 6 120

5w  2

152 2125w2  152 2112 2


5

Let w  0.

1 2  12

2 5
4
1
2

5

Let w  5.
1

5 (0)  2

 2

0 2
5

80  23 6 19
57 19

4  23 6 19
27 6 19
The solution set is {y|y  10.5}.

2  2

2
r
3

5.
2
r
3

 9  3

 9  9  3  9
2
r
3
3 2
r
2 3

12

29 GRAPH

Check:

A portion of a horizontal line is shown. It is part


of the graph of y  1.
2. For all values of x less than 2, the y value is 1.
For all values of x greater than or equal to 2, the
y value is 0. That is, y  1 if x  2, and otherwise
y  0.
3.
6x  9 6 4x  29
6x  9  4x 6 4x  29  4x
10x  9 6 29
10x  9  9 6 29  9
10x 6 20

 12


132 2 (12)

r  18
Substitute 18, a number less than
18, and a number greater than 18.

Let r  18.

Let r  24.

2
(18)
3

2
(24)
3

 9  3
?

12  9  3
3  3

 9  3
?

16  9  3
7
3

The solution set is {r|r  18}.

20
10

x 6 2
The solution set is {x|x  2}.
y  1 for those values of x for which the
inequality is true; y  0 for those values of x for
which the inequality is not true.
Chapter 6

5 (5)  2

1. KEYSTROKES: Y CLEAR 6 X,T,,n


TEST 5 4 X,T,,n
9 2nd

Graphing Calculator Investigation

10x
10

4(20)  23 6 19

4(1)23 6 19

The solution set is w w  4 .

Page 333

42  23 6 19
19 19
Let y  1.

Substitute 4, a number less than 4, and


5
a number greater than 4.

Let w  4.

42
4

4(10.5)  23 6 19

w4

Check:

y 7 10.5
Check:
Substitute 10.5, a number less than
10.5, and a number greater than
10.5.
Let y  10.5.
Let y  20.

The solution set is {r|r  13}.


10.

Check for Understanding

1. To solve both the equation and the inequality, you


first subtract 6 from each side and then divide
each side by 5. In the equation, the equal sign
does not change. In the inequality, the inequality
symbol is reversed because you divided by a
negative number.
2. Sample answer: 2x  4  2
3a. Distributive Property
3b. Add 12 to each side.
3c. Divide each side by 3.
4.
4y  23 6 19
4y  23  23 6 19  23
4y 6 42

12r
12

Let r  13.

Solving Multi-Step Inequalities

6-3

260

Let r  3.
2
(3)
3

 9  3
?

2  9  3
7  3

PQ249-6481F-06[244-272] 24/7/02 05:06 PM Page 261 Sahuja Ahuja_QXP_13:InProcess_AQ13:Sunita_23/24/7/02:PQ249-06-ReproPages:PQ249-06-Art:

6.

7b  11
7b  11  7b
11
11  13
24

7
7
7
7
7

9b  13
9b  13  7b
2b  13
2b  13  13
2b

24
2

2b
2

8.

12 7 b
12  b is the same as b  12.

8
2

Substitute 12, a number less than 12,


and a number greater than 12.
Let b  12.
?

7(12)  11 7 9(12)  13
?

3  5 (4)  3(4  1)  4 (2  4)
?

3  20  3 (5)  4 (2)
?

23  15  8
23  23
Let t  1.

7(10)  11 7 9(10)  13
?

70  11 7 90  13
81 7 77
Let b  20.

3  5 (1)  3(1  1)  4 (2  1)
?

3  5  3 (2)  4 (1)
?
864
8 2
Let t  5.

7(20)  11 7 9(20)  13
?

140  11 7 180  13
151 167
The solution set is {b|b  12}.

8
8

3  5 (5)  3 (5  1)  4 (2  5)

7 3(g  4)
7 3g  12
7 3g  12  5g
7 8g  12
7 8g  12  12
7 8g
7

2t
2

84  11 7 108  13
95 95
Let b  10.

5(g  4)
5g  20
5g  20  5g
20
20  12
8

4t
4  t is the same as t  4.
Check:
Substitute 4, a number less than 4,
and a number greater than 4.
Let t  4.

Check:

7.

3  5t  3 (t  1)  4 (2  t)
3  5t  3t  3  8  4t
3  5t  7t  5
3  5t  5t  7t  5  5t
3  2t  5
3  5  2t  5  5
8  2t

3  25  3 (6)  4 (3)
?

28  18  12
28  30
The solution set is {t|t  4}.
9. Let n  the number.

8g
8

two times is less three times


Seven minus a number than the number plus thirty-two.

1
424
3 1
424
3 1
4424
43 1
424
3

1 7 g
1  g is the same as g  1.
Check:
Substitute 1, a number less than 1,
and a number greater than 1.
Let g  1.

5(1  4) 7 3(1  4)
?

5(3) 7 3(5)
15 15
Let g  4.
?
5(4  4) 7 3(4  4)

2n

1
44424
43 123

7  2n
7  2n  2n
7
7  32
25

6
6
6
6
6

3n  32
3n  32  2n
5n  32
5n  32  32
5n

25
5

5n
5

3n

14
424
43

32

5 6 n
5  n is the same as n  5.
Check:
Substitute 5, a number less than 5,
and a number greater than 5.
Let n  5.
?
7  2(5) 6 3(5)  32

5(0) 7 3(8)
0 7 24
Let g  4.
?
5(4  4) 7 3(4  4)

7  10 6 15  32
17 17
Let n  10.

5(8) 7 3(0)
40 0

7  2(10) 6 3(10)  32

The solution set is {g|g  1}.

7  20 6 30  32
27 2
Let n  2.
?
7  2(2) 6 3(2)  32
?

7  4 6 6  32
3 6 38
The solution set is {n|n  5}.

261

Chapter 6

PQ249-6481F-06[244-272] 24/7/02 05:07 PM Page 262 Sahuja Ahuja_QXP_13:InProcess_AQ13:Sunita_23/24/7/02:PQ249-06-ReproPages:PQ249-06-Art:

10. Let s  the amount of sales.


22,000  0.05s 7 35,000
22,000  0.05s  22,000 7 35,000  22,000
0.05s 7 13,000
0.05s
0.05

2 

17.

2 

1 d2

(5) 5 7 (5)25
d 7 125
The solution set is {d|d  125}.
w
8

18.

11a.
11b.
12a.
12b.
12c.
13.

Subtract 7 from each side.


Multiply each side by 52.
Multiply each side by 3 and change  to .
Add 2m to each side.
Multiply each side by 1 and change  to .
Original inequality
4 (t  7)  2 (t  9)
Distributive
4t  28  2t  18
Property
4t  28  2t  2t  18  2t Subtract 2t from
each side.
Simplify.
2t  28  18
Add 28 to each side.
2t  28  28  18  28
Simplify.
2t  46


t  23
The solution set is {t|t  23}.
14.

5k  20 7 3k  12
5k  20  5k 7 3k  12  5k
20 7 8k  12
20  12 7 8k  12  12
8 7 8k
8
8

9q
9

3t
3

2a
2

Original
inequality
Distributive
Property
Add 5k to each
side.
Simplify.
Add 12 to each
side.
Simplify.

30
9
10
3

or 33

18

 2

a  9
The solution set is {a|a  9}.
21.
9r  15  24  10r
9r  15  9r  24  10r  9r
15  24  r
15  24  24  r  24
9  r
9  r is the same as r  9.
The solution set is {r|r  9}.
22.
13k  11 7 7k  37
13k  11  7k 7 7k  37  7k
6k  11 7 37
6k  11  11 7 37  11
6k 7 48

8k
8

6k
6

48
6

k 7 8
The solution set is {k|k  8}.
23.

9

2v  3
5
2v  3
(5) 5

7
 (5)7

2v  3  35
2v  3  3  35  3
2v  38
2v
2

64
8

38
2

v  19
The solution set is {v|v  19}.

f 6 8
The solution set is {f|f  8}.

Chapter 6

20. 8a  2  10a  20
2a  2  20
2a  2  2  20  2
2a  18

 3

7 (8)7

The solution set is q|q  33 .

t3
The solution set is {t|t  3}.
16.
5  8f 7 59
5  8f  5 7 59  5
8f 7 64
8f
8

7 7

q

Divide each
side by 8.
Simplify.
1 7 k
1  k is the same as k  1. The solution set is
{k|k  1}.
Exercises 1532 For checks, see students work.
15.
3t  6  3
3t  6  6  3  6
3t  9
7

 13  13 7 6  13

w 7 56
The solution set is {w|w  56}.
19. 7q  1  2q  29
9q  1  29
9q  1  1  29  1
9q  30

Divide each side by 2.


Simplify.

5(k  4) 7 3 (k  4)

 13 7 6

w
8
w
(8) 8

Practice and Apply

46
2

 2 6 23  2
d

w
8

2t
2

6 23

5 6 25

13,000
0.05

s 7 260,000
The salesperson would need more than $260,000
in sales to have an annual income greater than
$35,000.

Pages 335337

d
5

d
5

262

PQ249-6481F-06[244-272] 24/7/02 05:07 PM Page 263 Sahuja Ahuja_QXP_13:InProcess_AQ13:Sunita_23/24/7/02:PQ249-06-ReproPages:PQ249-06-Art:

24.

3a  8
2
3a  8
(2) 2

Since the inequality results in a statement that is


always true, the solution set is {b|b is a real
number.}.
31.
3.1v  1.4  1.3v  6.7
3.1v  1.4  1.3v  1.3v  6.7  1.3v
1.8v  1.4  6.7
1.8v  1.4  1.4  6.7  1.4
1.8v  8.1

6 10
6 (2)10

3a  8 6 20
3a  8  8 6 20  8
3a 6 12
3a
3

12
3

a 6 4
The solution set is {a|a  4}.
25.

3w  5
4
3w  5
(4) 4

1.8v
1.8

v  4.5
The solution set is {v|v  4.5}.
32. 0.3(d  2)  0.8d 7 4.4
0.3d  0.6  0.8d 7 4.4
0.5d  0.6 7 4.4
0.5d  0.6  0.6 7 4.4  0.6
0.5d 7 5

 2w
 (4)2w

3w  5  8w
3w  5  3w  8w  3w
5  5w
5
5

5w
5

0.5d
0.5

1w
1  w is the same as w  1.
The solution set is {w|w  1}.
26.

5b  8
3
5b  8
(3) 3

6 3b
6 (3)3b

4b
4

22
2

2 6 b

27.

28.

29.

30.

5
0.5

d 6 10
The solution set is {d|d  10}.
33. 4(y  1)  3(y  5)  3(y  1)
4y  4  3y  15  3y  3
y  19  3y  3
y  19  y  3y  3  y
19  2y  3
19  3  2y  3  3
22  2y

5b  8 6 9b
5b  8  5b 6 9b  5b
8 6 4b
8
4

8.1

 1.8

2y
2

11  y
11  y is the same as y  11.
The solution set is { y|y  11}.

2  b is the same as b  2.
The solution set is {b|b  2}.
7  3t  2(t  3)  2(1  t)
7  3t  2t  6  2  2t
7  3t  4t  8
7  3t  3t  4t  8  3t
7t8
78t88
1  t
1  t is the same as t  1.
The solution set is {t|t  1}.
5(2h  6)  7(h  7)  4h
10h  30  7h  49  4h
3h  79  4h
3h  79  3h  4h  3h
79  h
79  h is the same as h  79.
The solution set is {h|h  79}.
3y  4  2(y  3)  y
3y  4  2y  6  y
3y  4  3y  6
3y  4  3y  3y  6  3y
46
Since the inequality results in a false statement,
the solution set is the empty set .
3  3(b  2) 6 13  3(b  6)
3  3b  6 6 13  3b  18
3b  9 6 3b  31
3b  9  3b 6 3b  31  3b
9 6 31

10 11 12 13

34. 5(x  4)  2(x  6)  5(x  1)  1


5x  20  2x  12  5x  5  1
3x  8  5x  4
3x  8  3x  5x  4  3x
8  2x  4
8  4  2x  4  4
4  2x
4
2

2x
2

2x
2  x is the same as x  2.
The solution set is {x|x  2}.
4 3 2 1

Exercises 3538 For checks, see students work.


35. Let n  the number.
1
n
8
1
n
8

 5  30

 5  5  30  5
1
n
8
1
(8) 8n

 35
 (8)35

n  280
The solution set is {n|n  280}.

263

Chapter 6

PQ249-6481F-06[244-272] 24/7/02 05:07 PM Page 264 Sahuja Ahuja_QXP_13:InProcess_AQ13:Sunita_23/24/7/02:PQ249-06-ReproPages:PQ249-06-Art:

36. Let n  the number.


2
n
3
2
n
3

44.

 8 7 12

 8  8 7 12  8
2
n
3
3 2
n
2 3

12

132 24

n 7 6
The solution set is {n n 7 6}.
37. Let n  the number.
4n  9  n  21
4n  9  n  n  21  n
5n  9  21
5n  9  9  21  9
5n  30
5n
5

42.

91  95  88  s
4
274  s
4
274  s
(4) 4

 92

520w
520

Chapter 6

4.375

 1.4375

1040
520

w2
The union worker can strike for no more than
2 weeks.
49. Let w  the number of weeks the worker could
strike.
[600  0.04(600) ] (52  w)  (600)52
(600  24)(52  w)  31,200
624(52  w)  31,200
32,448  624w  31,200
32,448  624w  32,448  31,200  32,448
624w  1248

 92
 (4)92

274  s  368
274  s  274  368  274
s  94
To earn an A in math, Carmen must score at least
94 on the test.
43. Mercury is a solid until it reaches its melting
point at 38C.
5(F  32)
9

122

t  3.0
(to the nearest tenth)
Nicholas can order 3 or fewer toppings.
48. Let w  the number of weeks the worker could
strike.
[500  0.04(500) ] (52  w)  (500)52
(500  20)(52  w)  26,000
520(52  w)  26,000
27,040  520w  26,000
27,040  520w  27,040  26,000  27,040
520w  1040

105
3

 92

25
2
25
or
2

1.4375t
1.4375

a 6 35
The solution set is {a a 6 35}.
41.

Keith should stay on the diet for more than


1
122 weeks.
46. Sample answers: 2x  5  2x  3; 2x  5  2x  3
47. Let t  the number of toppings Nicholas can order.
7.50  1.25t  0.15(7.50  1.25t)  13.00
7.50  1.25t  1.125  0.1875t  13.00
1.4375t  8.625  13.00
1.4375t  8.625  8.625  13.00  8.625
1.4375t  4.375

2n
2

91  95  88  s
4

195 2 (38)

w 7

30
5

17 7 n
17  n is the same as n  17.
The solution set is {n n 6 17}.
39. 3a  15 6 90
40.
3a  15 6 90
3a  15  15 6 90  15
3a 6 105
3a
3

2w
2

n6
The solution is {n n  6}.
38. Let n  the number.
3(n  7) 7 5n  13
3n  21 7 5n  13
3n  21  3n 7 5n  13  3n
21 7 2n  13
21  13 7 2n  13  13
34 7 2n
34
2

12

6 38

F  32 6 68.4
F  32  32 6 68.4  32
F 6 36.4
Mercury is a solid for temperatures less than
36.4F.
45. Let w  the number of weeks Keith should stay
on the diet.
200  2w 6 175
200  2w  200 6 175  200
2w 6 25

7 4
7

5(F  32)
9
9 5(F  32)
5
9

624w
624

1248
624

w2
The number of weeks the worker can strike does
not change if the worker makes $600 per week.

6 38

264

PQ249-6481F-06[244-272] 24/7/02 05:07 PM Page 265 Sahuja Ahuja_QXP_13:InProcess_AQ13:Sunita_23/24/7/02:PQ249-06-ReproPages:PQ249-06-Art:

50. Let w  the number of weeks the worker could


strike.
150w  [500  0.04(500) ] (52  w)  (500)52
150w  (500  20)(52  w)  26,000
150w  520(52  w)  26,000
150w  27,040  520w  26,000
370w  27,040  26,000
370w  27,040  27,040  26,000 
27,040
370w  1040

370w
370

55. C;


1040
370

57.

8
2

CLEAR 13 X,T,,n
TEST
6 7 X,T,,n

KEYSTROKES:

37 GRAPH
y  1 for those values of x for which the
inequality is true. Since y  1 for values of x less
than or equal to 8, the solution set is {x|x  8}.
58.

CLEAR 2 (
5 3 (
TEST

KEYSTROKES:

2nd

X,T,,n
2

X,T,,n

GRAPH
2 )
y  1 for those values of x for which the
inequality is true. Since y  1 for values of x
greater than 3, the solution set is {x|x  3}.

Page 337

Maintain Your Skills

59. Let m  the number of miles Mrs. Ludlow can


drive.
0.12m  50

or 113
1

0.12m
0.12

The positive even integers less than 113 are 2, 4,


6, 8, and 10. So the sets of three consecutive
positive even integers whose sum is less than 40
are 2, 4, and 6; 4, 6, and 8; 6, 8, and 10; 8, 10, and
12; 10, 12, and 14.
53. Inequalities can be used to describe the
temperatures for which an element is a gas or a
solid. Answers should include the following.
The inequality for temperatures in degrees
Celsius for which bromine is a gas is
9
C
5

2t
2

11 2nd

n8
The positive odd integers less than or equal to 8
are 1, 3, 5, and 7. So the pairs of consecutive
positive odd integers whose sum is no greater
than 18 are 1 and 3, 3 and 5, 5 and 7, 7 and 9.
52. Let n  the first positive even integer.
Then n  2 and n  4 represent the next two
consecutive positive even integers, respectively.
n  (n  2)  (n  4) 6 40
3n  6 6 40
3n  6  6 6 40  6
3n 6 34
3n
34
6 3
3
34
3

8t  (6t  10)
8t  6t  10
2t  10
2t  10  2t
10
10  2
8

9 GRAPH
y  1 for those values of x for which the
inequality is true. Since y  1 for values of x less
than 2, the solution set is {x|x  2}.

16
2

n 6

6
6
6
6
6
6
6

t 6 4
The solution set is {t|t  4}.
CLEAR 3 X,T,,n
56. KEYSTROKES:
TEST
3 4 X,T,,n
7 2nd

w  2.8
(to the nearest tenth)
The union worker can strike for up to 2.8 weeks.
51. Let n  the first positive odd integer.
Then n  2 represents the next consecutive
positive odd integer.
n  1n  22  18
2n  2  18
2n  2  2  18  2
2n  16
2n
2

4t  2
4t  2
4t  2
4t  2  2t
2t  2
2t  2  2
2t

50

 0.12
2

m  4163
If Mrs. Ludlow is charged $0.12 per mile or any
part of one mile, she can travel up to 416 mi
without going over her budget.
Exercises 6062 For checks, see students work.
60.
d  13  22
d  13  13  22  13
d9
The solution set is {d|d  9}.

 32 7 138.

Sample answer: Scientists may use


inequalities to describe the temperatures for
which an element is a solid.

61.

y  5

54. D; To solve 9  13, first eliminate fractions by


multiplying both sides of the inequality by 9.

t5 6
t55 6
t 6
The solution
5

265

10 11 12 13

3
35
8
set is {t|t < 8}.
8

10 11 12 13

Chapter 6

PQ249-6481F-06[244-272] 24/7/02 05:07 PM Page 266 Sahuja Ahuja_QXP_13:InProcess_AQ13:Sunita_23/24/7/02:PQ249-06-ReproPages:PQ249-06-Art:

62.

4 7 y7
47 7 y77
3 7 y
3  y is the same as y  3.
The solution set is {y|y  3}.
8 7 6 5 4 3 2 1

69. First rewrite the equation so that the variables


are on one side of the equation.
4x  7  2y
4x  2y  7  2y  2y
4x  2y  7
The equation is now in standard form where A  4,
B  2, and C  7. This is a linear equation.
70. Since the term 2x2 has an exponent of 2, the
equation cannot be written in the form Ax  By 
C. Therefore, this is not a linear equation.
71. The equation can be written as x  0y  12.
Therefore, it is a linear equation in standard form
where A  1, B  0, and C  12.
72.
21x  22  3x  14x  52
2x  4  3x  4x  5
2x  4  x  5
2x  4  x  x  5  x
3x  4  5
3x  4  4  5  4
3x  9

y  y1  m(x  x1)
y  (3)  2(x  1)
y  3  2x  2
y  3  3  2x  2  3
y  2x  5
y  2x  2x  5  2x
2x  y  5
(1)(2x  y)  (1)(5)
2x  y  5
The standard form of the equation is 2x  y  5.
The point-slope form of the equation is
y  3  2(x  1).
64.
y  y  m(x  x )
1
1
63.

3x
3

y  (1)  3 x  (2)

x3
Check:
2(x  2)  3x  (4x  5)
?
2(3  2)  3(3)  [ 4(3)  5]
?
2(1)  9  (12  5)
?
297
22
The solution is 3.
73.
5t  7  t  3
5t  7  t  t  3  t
4t  7  3
4t  7  7  3  7
4t  10

y  1  3 (x  2)

1 22

3(y  1)  3 3 (x  2)
3y  3  2(x  2)
3y  3  2x  4
3y  3  3  2x  4  3
3y  2x  7
3y  2x  2x  7  2x
2x  3y  7
The standard form of the equation is 2x  3y  7.
The point-slope form of the equation is
2

y  1  3 (x  2).

4t
4

y  y1  m(x  x1)
y  6  0(x  3)
y60
y6606
y6
The standard form of the equation is y  6.
The point-slope form of the equation is y  6  0.
66. Let (3, 1)  (x1, y1) and (4, 6)  (x2, y2).
65.

Check:




321 0 1 2 3 4 5

 x1

75.
321 0 1 2 3 4 5

76.
654321 0 1 2

77.

y2  y1




321 0 1 2 3 4 5

 x1

3  (4)
1  (2)
7
3

78.
4321 0 1 2 3 4

79.

68. Let (0, 3)  (x1, y1) and ( 2, 5)  (x2, y2).

0 1 2 3 4 5 6 7 8

y2  y1

mx

80.

 x1

54321 0 1 2 3

5  3
 0
8
or 4
2

 2


Chapter 6

t  2.5
5t  7  t  3
?
5(2.5)  7  2.5  3

74.

67. Let (2, 4)  (x1, y1) and (1, 3)  (x2, y2).
2

10
4

6  (1)
4  3
5
or 5
1

mx

12.5  7  5.5
5.5  5.5
The solution is 2.5.

y2  y1

mx

3

81.
54321 0 1 2 3

266

PQ249-6481F-06[244-272] 24/7/02 05:07 PM Page 267 Sahuja Ahuja_QXP_13:InProcess_AQ13:Sunita_23/24/7/02:PQ249-06-ReproPages:PQ249-06-Art:

12. For a compound statement connected by the word


and to be true, both simple statements must be
true. Since it is not true that 0  3, the compound
statement is false.

82.
21 0 1 2 3 4 5 6

Page 338

Reading Mathematics

1. For a compound statement connected by the word


or to be true, at least one of the simple
statements must be true. Since it is true that a
hexagon has six sides, the compound statement is
true.
2. For a compound statement connected by the word
and to be true, both simple statements must be
true. It is false that a pentagon has six sides; it
has five. Thus, the compound statement is false.
3. For a compound statement connected by the word
and to be true, both simple statements must be
true. It is true that a pentagon has five sides. It is
also true that a hexagon has six sides. Thus, the
compound statement is true.
4. For a compound statement connected by the word
or to be true, at least one of the simple
statements must be true. It is true that an
octagon does not have seven sides; it has eight.
Thus, the compound statement is true.
5. For a compound statement connected by the word
or to be true, at least one of the simple
statements must be true. In this case, it is not
true that a pentagon has three sides; it has five.
It is also not true that an octagon has ten sides; it
has eight. Since neither simple statement is true,
the compound statement is false.
6. For a compound statement connected by the word
or to be true, at least one of the simple
statements must be true. Since it is true that a
square has four sides, the compound statement is
true.
7. For a compound statement connected by the word
or to be true, at least one of the simple
statements must be true. In this case, it is not
true that 5  4. It is also not true that 8  6.
Since neither simple statement is true, the
compound statement is false.
8. For a compound statement connected by the word
and to be true, both simple statements must be
true. Since it is not true that 1  0, the
compound statement is false.
9. For a compound statement connected by the word
and to be true, both simple statements must be
true. In this case, it is true that 4  0. It is also
true that 4  0. Thus, the compound statement
is true.
10. For a compound statement connected by the word
or to be true, at least one of the simple
statements must be true. Since it is true that
0  0, the compound statement is true.
11. For a compound statement connected by the word
or to be true, at least one of the simple
statements must be true. It is true that 1  4.
Thus, the compound statement is true.

Solving Compound Inequalities

6-4
Page 341

Check for Understanding

1. A compound inequality containing and is true if


and only if both inequalities are true. A compound
inequality containing or is true if and only if one
of the inequalities is true.
2. 7  t  12
3. Sample answer: x  2 and x  3
4. Graph a  6.
Graph a  2.
Find the intersection.
3

5. Graph y  12.
Graph y  9.
Find the union.
5

10

11

12

13

14

15

10

11

12

13

14

15

10

11

12

13

14

15

6. 3 6 x  1
7. x  1 or x  5
8.
and
w  3 6 11
6 6 w3
w  3  3 6 11  3
63 6 w33
3 6 w
w 6 8
The solution set is the intersection of the two
graphs.
Graph 3  w or w  3.
Graph w  8.
Find the intersection.
0

10

10

10

The solution set is 5w|3 6 w 6 86.

267

Chapter 6

PQ249-6481F-06[244-272] 24/7/02 05:07 PM Page 268 Sahuja Ahuja_QXP_13:InProcess_AQ13:Sunita_23/24/7/02:PQ249-06-ReproPages:PQ249-06-Art:

9.

n  7  5
n  7  7  5  7
n2

or

12. Let n  the number.


5 6 3n  7 6 17
First express 5  3n  7  17 using and. Then
solve each inequality.
and
3n  7 6 17
5 6 3n  7
3n  7  7 6 17  7
5  7 6 3n  7  7
12 6 3n
3n 6 24

n71
n7717
n8

The solution set is the union of the two graphs.


Graph n  2.
Graph n  8.
Find the union.

10.

10

10

10

12
3

12
3

20
4.5

z 6 4
The solution set is the union of the two graphs.
Graph z  4.
Graph z  1.
Find the union.
2

24
3

4.5x
4.5

4.5x
4.5

30

 4.5

Practice and Apply

14. Graph x  5.
Graph x  9.
Find the intersection.
0

10

10

10

15. Graph s  7.


Graph s  0.
Find the intersection.

The solution set is {x|4 6 x  1}.

Chapter 6

Pages 342343

Notice that the graph of z  4 contains every


point in the graph of z  1. So, the union is the
graph of z  4. The solution set is 5z|z 6 46.
11. 8 6 x  4  3
First express 8  x  4  3 using and. Then
solve each inequality.
and
x  4  3
8 6 x  4
x  4  4  3  4
8  4 6 x  4  4
4 6 x
x1
The solution set is the intersection of the two
graphs.
Graph 4  x or x  4.
Graph x  1.
Find the intersection.
7

3n
3

4.4  x
x  6.6
The lengths of the stretched spring will be
between about 4.44 and 6.67 in., inclusive, (to the
nearest hundredth).

3n
3

4 6 n
n 6 8
The solution set is the intersection of 4  n and
n  8.
The solution set is 5n|4 6 n 6 86.
13. If forces are between 20 and 30 lb, inclusive, then
20  F  30.
Since F  4.5x, 20  4.5x  30.
Express 20  4.5x  30 using and. Then solve
each inequality.
20  4.5x
and
4.5x  30

The solution set is {n|n  2 or n  8}.


or
z1
3z  1 6 13
3z  1  1 6 13  1
3z 6 12
3z
3

268

10 9

10 9

10 9

PQ249-6481F-06[244-272] 24/7/02 05:07 PM Page 269 Sahuja Ahuja_QXP_13:InProcess_AQ13:Sunita_23/24/7/02:PQ249-06-ReproPages:PQ249-06-Art:

16. Graph r  6.
Graph r  6.
Find the union.

28.

10

10

10

17. Graph m  4.


Graph m  6.
Find the union.
10

10

10

10

10

10

29.

18. First express 7  d  11 using and.


7 6 d and d 6 11
Graph 7 6 d or d 7 7.
Graph d  11.
Find the intersection.
2

10

11

12

10

11

12

10

11

12

30.

19. First express 1  g  3 using and.


1  g and g 6 3
Graph 1  g or g  1.
Graph g  3.
Find the intersection.
5

20.
21.
22.
23.
24.
25.
26.
27.

and
k  2  18
k  2 7 12
k  2  2  18  2
k  2  2 7 12  2
k 7 10
k  16
The solution set is the intersection of the two
graphs.
Graph k  10.
Graph k  16.
Find the intersection.
8

10

11

12

13

14

15

16

17

18

10

11

12

13

14

15

16

17

18

10

11

12

13

14

15

16

17

18

The solution set is 5k|10 6 k  166.


and
f  9  4
f83
f  9  9  4  9
f8838
f  5
f  13
The solution set is the intersection of the two
graphs.
Graph f  5.
Graph f  13.
Find the intersection.
14 13 12 11 10

14 13 12 11 10

14 13 12 11 10

The solution set is 5f|13  f  56.


or
d41
d4 7 3
d4414
d44 7 34
d 7 7
d5
The solution set is the union of the two graphs.
Graph d  7.
Graph d  5.
Find the union.

5
0

10

10

10

2  x  2
7 6 x 6 3
x  12 or x 7 15
x  7 or x  6
x  0 or x  4
x  2 or x 7 5
158  w  206
t  18 or t  22

The solution set is {d|d  5 or d 7 7}.

269

Chapter 6

PQ249-6481F-06[244-272] 24/7/02 05:07 PM Page 270 Sahuja Ahuja_QXP_13:InProcess_AQ13:Sunita_23/24/7/02:PQ249-06-ReproPages:PQ249-06-Art:

31.

or
h3 6 2
h  10 6 21
h33 6 23
h  10  10 6 21  10
h 6 11
h 6 1
The solution set is the union of the two graphs.
Graph h  11.
Graph h  1.
Find the union.
16 14 12 10

16 14 12 10

16 14 12 10

34.

3t
3

2x
2

2x
2

10

10

The solution set is 5x|3 6 x 6 96.


33. First express 4  2y  2  10 using and.
and
2y  2 6 10
4 6 2y  2
2y  2  2 6 10  2
4  2 6 2y  2  2
6 6 2y
2y 6 12
2y
2

36.

12
2

3 6 y
y 6 6
The solution set is the intersection of the two
graphs.
Graph 3  y or y  3.
Graph y  6.
Find the intersection.
0

10

10

10

3q
3

3q
3

18
3

The solution set is 5q|1 6 q 6 66.


or
x  4
1  x  3
(1) (x)  (1) (4)
1  x  1  3  1
x4
x4
The solution set is the union of the two graphs.
Graph x  4.
Graph x  4.
Find the union.
5

The solution set is 5x|x is a real number.6.

The solution set is 5y|3 6 y 6 66.


Chapter 6

2

1 6 q
q 6 6
The solution set is the intersection of the two
graphs.
Graph 1  q or q  1.
Graph q  6.
Find the intersection.

10

2y
2

2t
2

3
3

12
3

Since the graphs do not intersect, the solution set


is the empty set .
35.
and
5  3q 7 13
8 7 5  3q
5  3q  5 7 13  5
8  5 7 5  3q  5
3 7 3q
3q 7 18

18
2

6
2

2t  6  12
2t  6  6  12  6
2t  6

3 6 x
x 6 9
The solution set is the intersection of the two
graphs.
Graph 3  x or x  3.
Graph x  9.
Find the intersection.
0

and

t4
t3
The solution set is the intersection of the two
graphs.
Graph t  4.
Graph t  3.
Find the intersection.

Notice that the graph of h  1 contains every


point in the graph of h  11. So, the union is the
graph of h  1. The solution set is 5h|h 6 16.
32. First express 3  2x  3  15 using and.
and
2x  3 6 15
3 6 2x  3
2x  3  3 6 15  3
3  3 6 2x  3  3
6 6 2x
2x 6 18
6
2

3t  7  5
3t  7  7  5  7
3t  12

270

PQ249-6481F-06[244-272] 24/7/02 05:07 PM Page 271 Sahuja Ahuja_QXP_13:InProcess_AQ13:Sunita_23/24/7/02:PQ249-06-ReproPages:PQ249-06-Art:

3n  11  13

37.

3n  12

or

3n
3

3n  11  11  13  11
3n  2
3n
3

Graph g  3.
Graph 12  g or g  12.
Find the intersection.

12
3

n4

3

n

10

12

14

10

12

14

10

12

14

2
3

The solution set is the union of the two graphs.


2

Graph n  3.
Graph n  4.
Find the union.
0

10

10

Since the graphs do not intersect, the solution set


is the empty set .
40.
4c 6 2c  10
or
3c 6 12
2c 6 10
2c
2

10

2p  2  4p  8

4p  8  3p  3

and

4p  8  3p  3p  3  3p

2  2p  8

p  8  3

2  8  2p  8  8

p  8  8  3  8

6  2p
6
2

3p

10

10

10

3g  12  6  g
3g  12  g  6  g  g

and

6  g  3g  18

2g  12  6

6  2g  18

2g  12  12  6  12

6  18  2g  18  18
24  2g


6
0.5

3b  16  b 6 8  b  b
2b  16 6 8
2b  16  16 6 8  16
2b 6 24
6

24
2

b 6 12
The solution set is the union of the two graphs.
Graph b  12.
Graph b  12.
Find the union.

6  g  g  3g  18  g

24
2

2b
2

The solution set is 5p|3  p  56.


39. First express 3g  12  6  g  3g  18 using
and.

g  3

b 7 12

6
2

0.5b
0.5

c 7 4

10
2

The solution set is {c|c 6 5 or c 7 4}.


41. 0.5b 7 6
or
3b  16 6 8  b

The solution set is the intersection of the two


graphs.
Graph 3  p or p  3.
Graph p  5.
Find the intersection.

2g
2

12
3

p5

2p
2

2g  6

c 6 5
The solution set is the union of the two graphs.
Graph c  5.
Graph c  4.
Find the union.

Notice that the graph of n  4 contains every


2
point in the graph of n  3. So, the union is the
graph of n  4. The solution set is 5n|n  46.
38. First express 2p  2  4p  8  3p  3 using
and.
2p  2  2p  4p  8  2p

3c
3

4c  2c 6 2c  10  2c

18 16 14 12 10

18 16 14 12 10

18 16 14 12 10

The solution set is {b|b 6 12 or b 7 12}.

2g
2

12  g

The solution set is the intersection of the two


graphs.

271

Chapter 6

PQ249-6481F-06[244-272] 24/7/02 05:07 PM Page 272 Sahuja Ahuja_QXP_13:InProcess_AQ13:Sunita_23/24/7/02:PQ249-06-ReproPages:PQ249-06-Art:

47. Let p  the price of a color printer.


Then p  30 represents the price of a printer
after a $30 rebate.
If 175  p  260, then 175  p and p  260.
and
p  260
175  p
175  30  p  30
p  30  260  30
145  p  30
p  30  230
So, 145  p  30  230.
After the rebate, Luisana can expect to spend
between $145 and $230, inclusive.
48. Let s  the total amount of sales needed to earn a
prize in category D. Then s  70 represents the
amount of sales Rashid still needs after selling
$70 worth of chocolates.
If 121  s  180, then 121  s and s  180.
121  s
and
s  180
s  70  180  70
121  70  s  70
51  s  70
s  70  110
So, 51  s  70  110.
Rashid must have additional sales between $51
and $110, inclusive, to earn a category D prize.
49a. Graph x  5 or x  8.

42. Let n  the number.


5  n  8  14
First express 5  n  8  14 using and.
and
n  8  14
5n8
n  8  8  14  8
58n
13  n
n  22
The solution set is the intersection of 13  n and
n  22.
The solution set is 5n|13  n  226.
43. Let n  the number.
8 6 3n  4 6 10
First express 8  3n  4  10 using and.
and
3n  4 6 10
8 6 3n  4
8  4 6 3n  4  4
3n  4  4 6 10  4
12 6 3n
3n 6 6
12
3

3n
3

3n
3

6
3

4 6 n
n 6 2
The solution set is the intersection of 4  n and
n  2.
The solution set is 5n|4 6 n 6 26.
44. Let n  the number.
5n 7 35 or 5n 6 10
5n
5

35
5

5n
5

10
5

n 6 7
n 7 2
The solution is the union of n  7 and n  2.
The solution set is 5n|n 6 7 or n 7 26.
45. Let n  the number.
1

First express 0 6 2n  1 using and.


1

(2)0 6

1
(2) 2n

and

1
n
2
1
(2) 2n

10

11

12

10

11

12

Find the points not graphed. These values make


the given compound inequality false.
So, x  5 and x  8. This compound inequality
may also be expressed as 5  x  8.
49b. Graph x  6 and x  1.

0 6 2n  1
0 6 2n

1
 (2)1

0 6 n
n2
The solution set is the intersection of 0  n and
n  2.
The solution set is 5n|0 6 n  26.
46. Let h  the number of hours an adult sleeps.
Then 0.20 h represents the number of hours an
adult spends in REM sleep.
If 7  h  8 then 7  h and h  8.
and
h8
7h
(0.20)7  (0.20)h
(0.20)h  (0.20)8
1.4  0.20h
0.20h  1.6
So, 1.4  0.20h  1.6.
An adult spends between 1.4 and 1.6 h, inclusive,
in REM sleep.

Find the points not graphed. These values make


the given compound inequality false.
So, x  1 or x  6.
50. Let h  the number of hertz heard by humans.
20  h  20,000
Let d  the number of hertz heard by dogs.
15  d  50,000
51. Graph 20  h  20,000
Graph 15  h  50,000.

20

15

20,000

50,000

Find the union. Notice that the graph of


15  h  50,000 contains every point in the
graph of 20  h  20,000. So, the union is the
graph of 15  h  50,000.
Find the intersection. The two graphs intersect for
values of x between 20 and 20,000, inclusive. So,
the intersection is the graph of 20  h  20,000.
Chapter 6

272

PQ249-6481F-06[273-291] 24/7/02 05:13 PM Page 273 Sahuja Ahuja_QXP_13:InProcess_AQ13:Sunita_23/24/7/02:PQ249-06-ReproPages:PQ249-06-Art:

52. Of the number of hertz a dog can hear, humans


cannot hear sounds between 15 and 20 hertz,
including 15 hertz, or sounds between 20,000 and
50,000 hertz, including 50,000 hertz. So,
15  h  20 or 20,000  h  50,000.
53. Let a  the altitude in miles.
Sample answers:
troposphere: a  10
stratosphere: 10  a  30
mesosphere: 30  a  50
thermosphere: 50  a  400
exosphere: a  400
54. The tax table gives intervals of income and how
much a taxpayer with taxable income in each
interval must pay in taxes. Each interval can be
expressed as compound inequalities. Answers
should include the following.
The incomes are in $50 intervals.
41,100  x  41,150 represents the possible
incomes of a head of a household paying $7024
in taxes.
55. A; Let c  the number of cups of cooked tomatoes
c
made from ten lb of fresh tomatoes. Then 10
represents the number of cups of cooked tomatoes
made from one lb of fresh tomatoes.
If 10  c  15, then 10  c and c  15.
10 6 c
and
c 6 15
10
10

1 6

c
10
c
10

So, 1 6

c
10
c
10
c
10

6
6

15
10
3
2

Page 344

1.40r
1.40

18d
18

7v
7

61.

or 12

6 (13)13

183 2138b2 6 183 29

k(3)
3

k
8

Therefore, y  3x.
Use the direct variation equation to find x
when y  6.

y  3x
8

6  3x

138 2 6  138 2 83 x

y  1 for those values of x for which the


compound inequality is true.
Since y  1 for values of x that are either less
than 6 or greater than 1, the solution set is
{x|x  6 or x  1}.
CLEAR X,T,,n
57b. KEYSTROKES:
3

X,T,,n

6 13

8b 7 9

8
3
8
3

GRAPH

TEST

t
13
t
(13) 13

b 6 24
The solution set is {b|b  24}.
63. Write a direct variation equation that relates
x and y. Find the value of k.
y  kx
8  k(3)

56. B; First express 7  x  2  4 using and.


7 6 x  2
and
x2 6 4
7  2 6 x  2  2
x22 6 42
9 6 x
x 6 2
The solution set is 9  x and x  2, which can
be expressed as 9  x  2.
CLEAR X,T,,n
57a. KEYSTROKES:
4

2nd

91
7

t 6 169
The solution set is {t|t  169}.

Between 1 and
cups of cooked tomatoes are
made from one lb of fresh tomatoes.

X,T,,n

90

 18

v 7 13
The solution set is {v|v  13}.

62.

5 2 2nd
TEST
3 3
TEST
4 2nd

800,000
1.40

d5
The solution set is {d|d  5}.
60. 7v 6 91

6 12.

TEST

r  571,428.57 (to the nearest hundredth)


The school must raise at least $571,428.57 from
other sources.
Examples 5962 For checks, see students work.
59. 18d  90

1
12

2nd

Maintain Your Skills

58. Let r  the amount the school must raise from


other sources. The 0.40 r represents the amount
promised by the corporation.
r  0.40r  800,000
1.40r  800,000

9
4

x

Therefore, x  4 or 2.25 when y  6.

6 5 2nd
1
TEST
TEST 4 4 GRAPH
6 2nd

y  1 for those values of x for which the


compound inequality is true.
Since y  1 for values of x between 2 and 8,
inclusive, the solution set is {x|2  x  8}.

273

Chapter 6

PQ249-6481F-06[273-291] 24/7/02 05:13 PM Page 274 Sahuja Ahuja_QXP_13:InProcess_AQ13:Sunita_23/24/7/02:PQ249-06-ReproPages:PQ249-06-Art:

76. 3.5 is three and one-half units from zero in the


negative direction. |3.5|  3.5
77. |12  6|  |6|
6 is six units from zero in the positive direction.
|12  6|  |6|  6
78. |5  9|  |4|
4 is four units from zero in the negative
direction.
|5  9|  |4|  4
79. |20  21|  |1|
1 is one unit from zero in the negative direction.
|20  21|  |1|  1
80. |3  18|  |15|
15 is fifteen units from zero in the negative
direction.
|3  18|  |15|  15

64. Write a direct variation equation that relates x


and y. Find the value of k.
y  kx
2.5  k(0.5)
2.5
0.5

65.

66.

67.

68.

k(0.5)
0.5

5k
Therefore, y  5x.
Use the direct variation equation to find y
when x  20.
y  5x
y  5(20)
y  100
Therefore, y  100 when x  20.
The relation, as a set of ordered pairs, is {(6, 0),
(3, 5), (2, 2), (3, 3)}.
The domain of this relation is {3, 2, 6}.
The range is {2, 0, 3, 5}.
Exchange x and y in each ordered pair to write
the inverse relation. The inverse of this relation is
{(0, 6), (5, 3), (2, 2), (3,3)}.
The relation, as a set of ordered pairs, is {(5, 2),
(3, 1), (2, 2), (1, 7)}.
The domain of this relation is {3, 1, 2, 5}.
The range is {1, 2, 7}.
Exchange x and y in each ordered pair to write
the inverse relation. The inverse of this relation is
{(2, 5), (1, 3), (2, 2), (7, 1)}.
The relation, as a set of ordered pairs, is {(3, 4),
(3, 2), (2, 9), (5, 4), (5, 8), (7, 2)}.
The domain of this relation is {7, 2, 3, 5}.
The range is {2, 4, 8, 9}.
Exchange x and y in each ordered pair to write
the inverse relation. The inverse of this relation is
{(4, 3), (2, 3), (9, 2), (4, 5), (8, 5), (2, 7)}.
There are 6 possible outcomes, 4 are successes
and 2 are failures.
So, the odds of rolling a number greater than 2
4
2
are 2 or 1 or 2:1.

Page 344
1.

4b
4

1 22

10

Let b  10.
?
5  4(10) 7 23
?
5  40 7 23
35
23
The solution set is {b|b  7}.
1
n
2

 3  5

 3  3  5  3
1
n
2
1
(2) 2n

 8
 (2) (8)

n  16
Check:
Substitute 16, a number less than
16, and a number greater than 16.
Let n  16.
Let n  20.

1 2134 2  12 134 2

7 6
7

1
(16)
2

3
8

 3  5
?

1
(20)
2

8  3  5
5  5
Let n  0.

73. 7 is seven units from zero in the negative


direction. |7|  7
74. 10 is ten units from zero in the positive direction.
|10|  10
75. 1 is one unit from zero in the negative direction.
|1|  1

Chapter 6

1
n
2

2.

71. 100(4.7)  470

28
4

b 6 7
Substitute 7, a number less than 7,
and a number greater than 7.
Let b  7.
Let b  1.
?
?
5  4(7) 7 23
5  4(1) 7 23
?
?
5  28 7 23
5  4 7 23
23
23
1 7 23

70. 6 5  30 or 3
72. 12

Check:

69. There are 6 possible outcomes, 5 are successes


and 1 is a failure.
So, the odds of rolling a number that is not a 3
5
are 1 or 5:1.
5

Practice Quiz 2

5  4b 7 23
5  4b  5 7 23  5
4b 7 28

1
(0)
2

 3  5
?

0  3  5
3  5
The solution set is {n|n  16}.

274

 3  5
?

10  3  5
7 5

PQ249-6481F-06[273-291] 24/7/02 05:13 PM Page 275 Sahuja Ahuja_QXP_13:InProcess_AQ13:Sunita_23/24/7/02:PQ249-06-ReproPages:PQ249-06-Art:

3.

3(t  6)
3t  18
3t  18  18
3t

6
6
6
6

9
9
9  18
9

3t
3

9
3

t 6 3
Substitute 3, a number less than
and a number greater than 3.
Let t  3.
Let t  10.
?
?
3(10  6) 6
3(3  6) 6 9
?
?
3(3) 6 9
3(4) 6
99
12 6
Let t  4.
?
3(4  6) 6 9
?
3(10) 6 9
30  9
The solution set is {t|t  3}.
4.
9x  2 7 20
9x  2  2 7 20  2
9x 7 18
Check:

9x
9

6.

(3)a 6

3,

9
9
9

? 2(15)  15
3
? 30  15
6
3
? 45
6 3

? 2(18)  15
3
? 36  15
6
3
? 51
6 3

15 6

18 6

15

18

15

18

15  15

18 6 17

Let a  12.
? 2(12)  15
3
? 24  15
6
3
? 39
6 3

12 6
12
12

12  13
The solution set is {a|a  15}.
7.
x2 6 7
and
x2 7 5
x22 6 72
x22 7 52
x 6 9
x 7 3
The solution set is the intersection of the two
graphs.
Graph x  9.
Graph x  3.
Find the intersection.

18
9

2a  15
3
2a  15
(3) 3

3a 6 2a  15
3a  2a 6 2a  15  2a
a 6 15
Check:
Substitute 15, a number less than
15, and a number greater than 15.
Let a  15.
Let a  18.

x 7 2
Check:
Substitute 2, a number less than 2,
and a number greater than 2.
Let x  2.
Let x  3.
?
?
9(2)  2 7 20
9(3)  2 7 20
?
?
18  2 7 20
27  2 7 20
20
20
25
20
Let x  5.
?
9(5)  2 7 20
?
45  2 7 20
47 7 20
The solution set is {x|x  2}.
5.
2m  5  4m  1
2m  5  2m  4m  1  2m
5  2m  1
5  1  2m  1  1
6  2m
6
2

a 6

8.

2m
2

3m
3  m is the same as m  3.
Check:
Substitute 3, a number less than 3,
and a number greater than 3.
Let m  3.
Let m  1.
?
?
2(3)  5  4(3)  1
2(1)  5  4(1)  1
?
?
6  5  12  1
2541
11  11
7 3
Let m  6.
?
2(6)  5  4(6)  1
?
12  5  24  1
17  23
The solution set is {m|m  3}.

10

10

10

The solution set is {x|3  x  9}.


2b  5  1
or
b  4  4
2b  5  5  1  5
b  4  4  4  4
2b  6
b0
2b
2

6
2

b  3
The solution set is the union of the two graphs.
Graph b  3.
Graph b  0.
Find the union.
7

The solution set is {b|b  3 or b  0}.

275

Chapter 6

PQ249-6481F-06[273-291] 24/7/02 05:13 PM Page 276 Sahuja Ahuja_QXP_13:InProcess_AQ13:Sunita_23/24/7/02:PQ249-06-ReproPages:PQ249-06-Art:

9.

4m  5 7 7
4m  5  5 7 7  5
4m 7 12
4m
4

or

4m  5 6 9
4m  5  5 6 9  5
4m 6 4

12
4

4m
4

So, the absolute error is less than 6 sec when the


students estimated time is greater than 54 s and
less than 66 s.
5. See students work.

4
4

m 6 1

m 7 3

The solution set is the union of the two graphs.


Graph m  3.
Graph m  1.
Find the union.

6-5

Solving Open Sentences


Involving Absolute Value

Pages 348349

10.

The solution set is {m|m  1 or m  3}.


a4 6 1
or
a2 7 1
a44 6 14
a22 7 12
a 6 5
a 7 1
The solution set is the intersection of the two graphs.
Graph a  5.
Graph a  1.
Find the intersection.
3

5 432 1 0 1 2 3 4 5

3. Leslie is correct. You need to consider the case


when the value inside the absolute value symbols
is positive and the case when the value inside the
absolute value symbols is negative. So x  3  2
or x  3  2.
4. a; Write |k|  3 as k  3 and k  3. The
solution set is {k| 3  k  3}.
The graph of this solution set is graph a.
5. c; Write |x  4|  2 as x  4  2 or x  4  2.
Case 1:
Case 2:
x4 7 2
x  4 6 2
x44 7 24
x  4  4 6 2  4
x 7 6
x 6 2
The solution set is {x|x  2 or x  6}. The graph
of this solution set is graph c.
6. The difference between the guess and the actual
number, 832, is within 46. This statement can be
expressed as |g  832|  46, where g represents
Amandas guess.
7. Write |r  3|  10 as r  3  10 or r  3  10.
Case 1:
Case 2:
r  3  10
r  3  10
r  3  3  10  3
r  3  3  10  3
r7
r  13
The solution set is {13, 7}.

The solution set is {a|1  a  5}.

Page 347

Algebra Activity

1. See students work.


2. A negative error indicates that the time guessed
was less than 1 min. A positive error indicates
that the time guessed was more than 1 min.
3. If the absolute error is 6, then the error is either
6 or 6.
Let t  the students time. Then t  60 represents
the error.
So, either t  60  6 or t  60  6.
t  60  6
t  60  60  6  60
t  66

or

t  60  6
t  60  60  6  60
t  54

1412108642 0 2 4 6 8

If the absolute error is 6, the students estimated


time is 54 s or 66 s.
4. Let t  the students estimated time. If the error
is represented by t  60, then the absolute error
is represented by |t  60|. The absolute error is
less than 6 sec when |t  60|  6.
Write |t  60|  6 as t  60  6 and t  60  6.
Case 1:
Case 2:
t  60 6 6
t  60 7 6
t  60  60 6 6  60
t  60  60 7 6  60
t 6 66
t 7 54
Chapter 6

Check for Understanding

1. |x  2|  6 means that the distance between x


and 2 is greater than 6 units.
|x  2|  6 means that the distance between x
and 2 is less than 6 units.
The solution of |x  2|  6 includes all values
that are less than 4 or greater than 8. The
solution of |x  2|  6 includes all values that
are greater than 4 and less than 8.
2. Sample answer: |x|  2

8. Write |c  2|  6 as c  2  6 and c  2  6.


Case 1:
Case 2:
c2 6 6
c  2 7 6
c22 6 62
c  2  2 7 6  2
c 6 8
c 7 4
The solution set is {c|4  c  8}.
10864 2 0 2 4 6 8 10

276

PQ249-6481F-06[273-291] 24/7/02 05:13 PM Page 277 Sahuja Ahuja_QXP_13:InProcess_AQ13:Sunita_23/24/7/02:PQ249-06-ReproPages:PQ249-06-Art:

9. Write |10  w|  15 as 10  w  15 or
10  w  15.
Case 1:
Case 2:

Pages 349351

10  w 7 15

10  w 6 15

10  w  10 7 15  10

10  w  10 6 15  10

w 7 5

w 6 25

(1) (w) 6 (1)5

(1)(w) 6 (1)(25)

w 6 5

w 7 25

The solution set is {w|w  5 or w  25}.


15105 0 5 10 15 20 25 30 35

10. Write |2g  5|  7 as 2g  5  7 or 2g  5  7.


Case 1:
Case 2:
2g  5  7
2g  5  7
2g  5  5  7  5
2g  5  5  7  5
2g  2
2g  12
2g
2

2g
2

2

12
2

g1
g  6
The solution set is {g|g  6 or g  1}.
8 765 4 321 0 1 2

2x
2

11. Find the point that is the same distance from 2


as the distance from 4. The midpoint between 2
and 4 is 1.
3 units

3 units

17.

So, an open sentence is |x  1|  3.


12. Find the point that is the same distance from 4 as
the distance from 12. The midpoint between 4 and
12 is 8.
4 units

18.

4 units

19.

10

11

12

13

The distance from 8 to any point on the graph is


greater than 4 units.
So, an open sentence is |x  8|  4.
13. The difference between the actual diameter and
1.5 cm is within 0.001 cm. This statement can be
expressed as |d  1.5|  0.001, where d
represents the actual diameter.
Write |d  1.5|  0.001 as d  1.5  0.001 and
d  1.5  0.001.
Case 1:
Case 2:
d  1.5  0.001

d  1.5  0.001

d  1.5  1.5  0.001  1.5

d  1.5  1.5  0.001  1.5

d  1.501

Practice and Apply

14. c; Write |x  5|  3 as x  5  3 and x  5  3.


Case 1:
Case 2:
x53
x  5  3
x5535
x  5  5  3  5
x  2
x  8
The solution set is {x|8  x  2}. The graph of
this solution set is graph c.
15. f; Write |x  4|  4 as x  4  4 or x  4  4.
Case 1:
Case 2:
x4 7 4
x  4 6 4
x44 7 44
x  4  4 6 4  4
x 7 8
x 6 0
The solution set is {x|x  0 or x  8}. The graph
of this solution set is graph f.
16. a; write |2x  8|  6 as 2x  8  6 or
2x  8  6.
Case 1:
Case 2:
2x  8  6
2x  8  6
2x  8  8  6  8
2x  8  8  6  8
2x  14
2x  2

20.

21.

d  1.499

22.

The solution set is {d|1.499  d  1.501}.


The diameter of the bolts must be between 1.499
and 1.501 cm, inclusively.

277

14
2

2x
2

2

x7
x1
The solution set is {1, 7}. The graph of this
solution set is graph a.
b; Write |x  3|  1 as x  3  1 or x  3  1.
Case 1:
Case 2:
x  3  1
x31
x  3  3  1  3
x3313
x  4
x  2
The solution set is {x|x is a real number.}. The
graph of this solution set is graph b.
e; Write |x|  2 as x  2 and x  2.
The solution set is {x| 2  x  2}. The graph of
this solution set is graph e.
d; Write |8  x|  2 as 8  x  2 or 8  x  2.
Case 1:
Case 2:
8x2
8  x  2
8x828
8  x  8  2  8
x  6
x  10
(1)(x)  (1) (6)
(1) (x)  (1) (10)
x6
x  10
The solution set is {6, 10}. The graph of this
solution set is graph d.
The difference between the pH and 7.3 must be
within 0.002. This statement can be expressed as
|p  7.3|  0.002, where p represents the pH of
the eye solution.
The difference between the temperature and 38
should be within 1.5. This statement can be
expressed as |t  38|  1.5, where t represents
the temperature inside the refrigerator.
The difference between the score and 98 was
within 6. This statement can be expressed as
|s  98|  6, where s represents Ramonas
bowling score.

Chapter 6

PQ249-6481F-06[273-291] 24/7/02 05:13 PM Page 278 Sahuja Ahuja_QXP_13:InProcess_AQ13:Sunita_23/24/7/02:PQ249-06-ReproPages:PQ249-06-Art:

The solution set is {t| 10 6 t 6 6}.

23. The difference between the speed and 55 should


be within 3. This statement can be expressed as
|s  55|  3, where s represents the speed of the
car.
24. Write |x  5|  8 as x  5  8 or x  5  8.
Case 1:
Case 2:
x58
x  5  8
x5585
x  5  5  8  5
x  13
x  3
The solution set is {3, 13}.

10 9876 5 4 32 1 0

30. Write |v  3| 7 1 as v  3 7 1 or v  3 6 1.


Case 1:
Case 2:
v3 7 1
v  3 6 1
v33 7 13
v  3  3 6 1  3
v 7 2
v 6 4
The solution set is {v 0v 6 4 or v 7 2}.
8 7654 321 0 1 2

31. Write 0w  6 0  3 as w  6  3 or w  6  3.


Case 1:
Case 2:
w63
w  6  3
w6636
w  6  6  3  6
w9
w3
The solution set is {w 0w  3 or w  9}.

642 0 2 4 6 8 10 12 14

25. Write |b  9|  2 as b  9  2 or b  9  2.


Case 1:
Case 2:
b92
b  9  2
b9929
b  9  9  2  9
b  7
b  11
The solution set is {11, 7}.

0 1 2 3 4 5 6 7 8 9 10

1312111098 765 4 3

32. Write |3s  2| 7 7 as 3s  2 7 7 or


3s  2 6 7.
Case 1:
Case 2:
3s  2 7 7
3s  2 6 7
3s  2  2 7 7  2
3s  2  2 6 7  2
3s 7 9
3s 6 5

26. Write 02p  3 0  17 as 2p  3  17 or


2p  3  17.
Case 1:
Case 2:
2p  3  17
2p  3  17
2p  3  3  17  3
2p  3  3  17  3
2p  20
2p  14
2p
2

20
2

2p
2

p  10
The solution set is {7, 10}.

3s
3

14
2

20
5

c4
The solution set is {0.8, 4}.

3s
3

5
3

5 432 1 0 1 2 3 4 5

33. Write |3k  4|  8 as 3k  4  8 or 3k  4  8.


Case 1:
Case 2:
3k  4  8
3k  4  8
3k  4  4  8  4
3k  4  4  8  4
3k  4
3k  12

27. Write 05c  8 0  12 as 5c  8  12 or


5c  8  12.
Case 1:
Case 2:
5c  8  12
5c  8  12
5c  8  8  12  8
5c  8  8  12  8
5c  20
5c  4


9
3

s 7 3
s 6 13
The solution set is {s|s is a real number.}.

p  7

8 642 0 2 4 6 8 10 12

5c
5

5c
5

c

4
5
4
5

3k
3

3k
3

3
1

k  13

The solution set is k|k  4 or k 

or 0.8

12
3

k  4
1
13

6.

5 432 1 0 1 2 3 4 5
5 4321 0 1 2 3 4 5

34. Write |2n  1| 6 9 as 2n  1 6 9 and


2n  1 7 9.
Case 1:
Case 2:
2n  1 6 9
2n  1 7 9
2n  1  1 6 9  1
2n  1  1 7 9  1
2n 6 8
2n 7 10

28. Write 0z  2 0  5 as z  2  5 and z  2  5.


Case 1:
Case 2:
z25
z  2  5
z2252
z  2  2  5  2
z7
z  3
The solution set is {z|3  z  7}.

2n
2

Chapter 6

8
2

2n
2

10
2

n 6 4
n 7 5
The solution set is {n|5 6 n 6 4} .

32 1 0 1 2 3 4 5 6 7

29. Write |t  8| 6 2 as t  8 6 2 and t  8


Case 1:
Case 2:
t8 6 2
t8 7
t88 6 28
t88 7
t 6 6
t 7

7 2.

5 432 1 0 1 2 3 4 5

2
2  8
10

278

PQ249-6481F-06[273-291] 24/7/02 05:13 PM Page 279 Sahuja Ahuja_QXP_13:InProcess_AQ13:Sunita_23/24/7/02:PQ249-06-ReproPages:PQ249-06-Art:

35. Write |6r  8| 6 4 as 6r  8 6 4 and


6r  8 7 4.
Case 1:
Case 2:
6r  8 6 4
6r  8 7 4
6r  8  8 6 4  8
6r  8  8 7 4  8
6r 6 12
6r 7 4
6r
6

12
6

6r
6

r 6 2

39. Write

Case 1:

r 7

Case 2:

2  3x
5
2  3x
(5) 5

3x
3

The solution set is the empty set .

 (5)2

3x
3

x  23

36. Write |6  (3d  5)|  14 as 6  (3d  5)  14


and 6  (3d  5)  14.
Case 1:
Case 2:
6  (3d  5)  14
6  (3d  5)  14
6  3d  5  14
6  3d  5  14
11  3d  14
11  3d  14
11  3d  11  14  11 11  3d  11  14  11
3d  3
3d  25
3d
3

 3

d  1

The solution set is d|1  d 

1
83

6.

d

5h  2
6

Case 1:

or x  4 .

5 units

25
3
1
83

5h  2
6

So, an equation is |x| 5.


41. Find the point that is the same distance from 2
as the distance from 8. The midpoint between 2
and 8 is 3.
5 units

5 units

So, an equation is |x  3| 5.


42. Find the point that is the same distance from 3
as the distance from 3. The midpoint between 3
and 3 is 0.
3 units

 7 or

12
3

x4
2
23

5 units

0 2 4 6 8 10 12 14 16 18 20

`  7 as

40. Find the point that is the same distance from 5


as the distance from 5. The midpoint between 5
and 5 is 0.

1 0 1 2 3 4 5 6 7 8 9

5h  2
6

 (5) (2)

5 432 1 0 1 2 3 4 5

37. Write 08  (w  1) 0  9 as 8  (w  1)  9 and


8  (w  1)   9.
Case 1:
Case 2:
8  (w  1)  9
8  (w  1)  9
8w19
8  w  1  9
9w9
9  w  9
9w999
9  w  9  9  9
w  0
w  18
(1)(w)  (1)0
(1)(w)  (1)(18)
w0
w  18
The solution set is {w|0  w  18} .

38. Write `

The solution set is x 0x 

5 432 1 0 1 2 3 4 5

 2

2  3x  10
2  3x  2  10  2
3x  12

 3
2

3d
3

2  3x
5
2  3x
(5) 5

2

2  3x  10
2  3x  2  10  2
3x  8

4
6
2
3

0 2 5 3x 0  2 as 2 5 3x  2 or 2 5 3x  2.

3 units

 7.

Case 2:

5h  2
6
5h  2
(6) 6

5h  2
6
5h  2
(6) 6

7
 (6)7

5h  2  42
5h  2  2  42  2
5h  40
5h
5

The solution set is

5h
5

4
85,

h

The distance from 0 to any point on the graph is


less than or equal to 3 units. So, an inequality is
|x|  3.
43. Find the point that is the same distance from 7
as the distance from 1. The midpoint between 7
and 1 is 3.

 (6)(7)

5h  2  42
5h  2  2  42  2
5h  44

40
5

h8

 7

44
5
4
85

4 units

4 units

8 .

10864 2 0 2 4 6 8 10
8

The distance from 3 to any point on the graph is


less than 4 units. So, an inequality is
|x  (3)|  4, or |x  3|  4.

279

Chapter 6

PQ249-6481F-06[273-291] 24/7/02 05:13 PM Page 280 Sahuja Ahuja_QXP_13:InProcess_AQ13:Sunita_23/24/7/02:PQ249-06-ReproPages:PQ249-06-Art:

44. Find the point that is the same distance from 1


as the distance from 3. The midpoint between 1
and 3 is 1.
2 units

49. The difference between the temperature and 68


should be within 3. This statement can be
expressed as |t  68|  3, where t represents the
temperature in the house.
Write |t  68|  3 as t  68  3 and
t  68  3.
Case 1:
Case 2:
t  68  3
t  68  3
t  68  68  3  68
t  68  68  3  68
t  71
t  65
The solution set is {t|65  t  71}.
The temperature in the house should be between
65 F and 71 F, inclusive.
50. The difference between the percent of people who
say protection of the environment should have
priority and 52 is within 3. This statement can be
expressed as |p  52|  3, where p represents
the percent of people described.
Write |p  52|  3 as p  52  3 and
p  52  3.
Case 1:
Case 2:
p  52  3
p  52  3
p  52  52  3  52
p  52  52  3  52
p  55
p  49
The solution set is { p|49  p  55}.
The percent of people who say protection of the
environment should have priority is between 49%
and 55%, inclusive.
51. The difference between the pressure and 30
should be within 2. This statement can be
expressed as |p  30|  2, where p represents
the tire pressure in psi.
Write |p  30|  2 as p  30  2 and
p  30  2.
Case 1:
Case 2:
p  30  2
p  30  2
p  30  30  2  30
p  30  30  2  30
p  32
p  28
The solution set is { p|28  p  32}.
The tire pressure should be between 28 and
32 psi, inclusive.
52a. Since absolute value represents a distance, it
cannot be negative. So, there is no value of x
that will make |x  3| less than 5. Therefore,
|x  3| 5 is never true.
52b. Since absolute value represents a distance, it
cannot be negative. So, any value of x will make
|x  6| greater than 1. Therefore,
|x  6|  1 is always true.
52c. |x  2|  0 is true only if x  2. Therefore,
|x  2|  0 is sometimes true.

2 units

The distance from 1 to any point on the graph is


greater than 2 units. So, an inequality is
|x  1|  2.
45. Find the point that is the same distance from 12
as the distance from 8. The midpoint between
12 and 8 is 10.
2 units

15 14 13 12 11 10

2 units

The distance from 10 to any point on the graph


is greater than or equal to 2 units. So, an
inequality is |x  (10)|  2, or |x  10|  2.
46. The difference between the number of days and
280 should be within 14. This statement can be
expressed as |d  280|  14, where d represents
the number of days of the pregnancy.
47. Write |d  280|  14 as d  280  14 and
d  280  14.
Case 1:
d  280  14
d  280  280  14  280
d  294
Case 2:
d  280  14
d  280  280  14  280
d  266
The solution set is {d|266  d  294}.
The length of a full-term pregnancy should be
between 266 and 294 days, inclusive.
48. The difference between the pressure and 195
should be within 25. This statement can be
expressed as |p  195|  25, where p represents
the pressure in pounds per square inch.
Write |p  195|  25 as p  195  25 and
p  195  25.
Case 1:
p  195 6 25
p  195  195 6 25  195
p 6 220
Case 2:
p  195 7 25
p  195  195 7 25  195
p 7 170
The solution set is { p|170  p  220}.
The pressure of a fire extinguisher should be
between 170 and 220 psi.

Chapter 6

280

PQ249-6481F-06[273-291] 24/7/02 05:13 PM Page 281 Sahuja Ahuja_QXP_13:InProcess_AQ13:Sunita_23/24/7/02:PQ249-06-ReproPages:PQ249-06-Art:

56. Inequalities involving absolute value are used to


represent margin of error. Answers should include
the following.
The inequality representing the people who are
against the tax levy is |x  45|  3. To solve
this inequality, find the intersection of x  45
 3 and x  45  3. To solve these
inequalities, add 45 to each side of each
inequality. The solution set is {x|42  x  48}.
The votes for the tax levy can be between 44%
and 50%. The votes against the tax levy can be
between 42% and 48%. Depending on where the
actual votes are in each range, it could either
pass or fail.
57. B; Write |x  5| 2 as x  5  2 or x  5  2.
Case 1:
Case 2:
x52
x  5  2
x5525
x  5  5  2  5
x  3
x  7
The solution set is {7, 3}.
58. C; Write 6  |x|  6 as 6  |x| and |x|  6.
Case 1:
6  |x| is the same as |x|  6.
Write |x|  6 as x  6 or x  6.
The solution set is {x|x is a real number.}.
Case 2:
Write |x|  6 as x  6 and x  6.
The solution set is {x| 6  x  6}.
The final solution is the intersection of {x|x is a
real number.} and {x| 6  x  6}. Therefore, the
solution set is {x| 6  x  6}.
If 6  x  6, then 6  x.
6 6 x
(1)(6) 7 (1) (x)
6 7 x
6  x is the same as x  6.

53. The difference between the amount of sodium


chloride and 3.0 must be within 0.5. This
statement can be represented by |a  3.0|  0.5,
where a represents the amount of sodium chloride
to be added.
Write |a  3.0|  0.5 as a  3.0  0.5 and
a  3.0  0.5.
Case 1:
a  3.0  0.5
a  3.0  3.0  0.5  3.0
a  3.5
Case 2:
a  3.0  0.5
a  3.0  3.0  0.5  3.0
a  2.5
The solution set is {a|2.5  a  3.5}.
The amount of sodium chloride added must be
between 2.5 and 3.5 mm, inclusive.
54. The difference between the guess and 18,000
must be within 1500 without going over 18,000.
This statement can be expressed as the compound
inequality |p  18,000|  1500 and p  18,000,
where p represents the price guessed by Luis.
Write |p  18,000|  1500 as p  18,000  1500
and p  18,000  1500.
Case 1:
p  18,000  1500
p  18,000  18,000  1500  18,000
p  19,500
Case 2:
p  18,000  1500
p  18,000  18,000  1500  18,000
p  16,500
The solution set is the intersection of
16,500  p  19,500 and p  18,000. This set is
{ p|16,500  p  18,000}.
Luis will win the vehicle if his guess is between
$16,500 and $18,000, inclusive.
55a. Write x  3  1.2 as x  3  1.2 or x  3  1.2.
Case 1:
Case 2:
x  3  1.2
x  3  1.2
x  4.2
x  1.8
55b. Find the point that is the same distance from
1.8 as the distance from 4.2. The midpoint
between 1.8 and 4.2 is 3.
1.2 units

1.4

1.8 2.2

2.6

Page 351

1.2 units

3.4 3.8 4.2 4.6

Maintain Your Skills

59. Let t  number of beats per minute in Rafaels


target zone.
0.60(190) 6 t 6 0.80(190)
114 6 t 6 152
Rafaels target zone is between 114 and 152 beats
per min.
Exercises 6062 For checks, see students work.
60.
2m  7 7 17
2m  7  7 7 17  7
2m 7 10
2m
2

So, an equation is |x  3|  1.2.

10
2

m 7 5
The solution set is {m|m  5}.

281

Chapter 6

PQ249-6481F-06[273-291] 24/7/02 05:13 PM Page 282 Sahuja Ahuja_QXP_13:InProcess_AQ13:Sunita_23/24/7/02:PQ249-06-ReproPages:PQ249-06-Art:

61.

2  3x  2
2  3x  2  2  2
3x  4
3x
3

70. 13.2  6.1  13.2  (6.1)


 (|13.2|  |6.1|)
 (13.2  6.1)
 19.3
71. 4.7  (8.9)  4.7  8.9
 (|8.9|  |4.7|)
 (8.9  4.7)
 4.2
72. Distributive Property
73. Substitution Property
74. To find the x-intercept, let y  0.
y  3x  4
0  3x  4
0  4  3x  4  4
4  3x

 3
1

x  13

The solution set is x|x  13 .


2
w
3

62.
2
w
3

37

3373
2
w
3
3 2
w
2 3

12

 10


132 210

w  15
The solution set is {w|w  15}.
63. To express the equation in slope-intercept form,
solve the equation for y in terms of x.
2x  y  4
2x  y  2x  4  2x
y  2x  4
The equation is now of the form y  mx  b, so
the slope m is 2 and the y-intercept b is 4.
64. To express the equation in slope-intercept form,
solve the equation for y in terms of x.
2y  3x  4
2y  3x  3x  4  3x
2y  3x  4
2y
2

y

4
3
4
3

3x
3

x

The graph intersects the x-axis at 3, 0 .


To find the y-intercept, let x  0.
y  3x  4
y  3(0)  4
y4
The graph intersects the y-axis at (0, 4). Plot
these points and draw the line that connects
them.
y

3x  4
2
3x
4
2
2
3
x2
2

y  3x  4

y
The equation is now of the form y  mx  b, so
3
the slope m is 2 and the y-intercept b is 2.

65. To express the equation in slope-intercept form,


solve the equation for y in terms of x.
1
x
2

1
x
2
3
y
4

 4y  0
1

75. Select five values for the domain and make a


table. The only value in the range is 2. Since
there is no x in the equation, the value of x does
not depend on y. Therefore, x can be any real
number.

 2x  0  2x
3
y
4
4 3
y
3 4

12

 2x


143 2112x2

x
3
1
0
2
4

y  3x

This equation can be written as y  3x  0. The


equation is now of the form y  mx  b, so the
2
slope m is 3 and the y-intercept is 0.
66. I  prt
I
pt
I
pt

67.

prt
pt

r

ex  2y  3z
ex  2y  2y  3z  2y
ex  3z  2y
ex
e

x
68.

a  5
3
1 a  5
7
3
a  5
21

12

(x, y)
(3, 2)
(1, 2)
(0, 2)
(2, 2)
(4, 2)

Graph the ordered pairs and draw a line through


the points.

3z  2y
e
3z  2y
e

 7x


117 27x

x

69. 13  8  (|13|  |8|)


 (13  8)
 5
Chapter 6

y
2
2
2
2
2

y  2

282

PQ249-6481F-06[273-291] 24/7/02 05:13 PM Page 283 Sahuja Ahuja_QXP_13:InProcess_AQ13:Sunita_23/24/7/02:PQ249-06-ReproPages:PQ249-06-Art:

76. In order to find values for y more easily, solve the


equation for y.
xy3
xyx3x
y3x
Select five values for the domain and make a
table.
3x
3  (1)
30
31
32
33

x
1
0
1
2
3

The graph intersects the x-axis at (6, 0). To find


the y-intercept, let x  0.
2y  x  6
2y  (0)  6
2y  6
2y
2

6
2

y  3
The graph intersects the y-axis at (0, 3).
Plot these points and draw the line that connects
them.

(x, y)
(1, 4)
(0, 3)
(1, 2)
(2, 1)
(3, 0)

y
4
3
2
1
0

Graph the ordered pairs and draw a line through


the points.

2y  x  6

y
xy3

79. To find the x-intercept, let y  0.


2(x  y)  10
2(x  0)  10
2x  10

2x
2

77. To find the x-intercept, let y  0.


y  2x  1
(0)  2x  1
2x  1
2x
2

1
1

The graph intersects the x-axis at

1 2
1
,
2

10
2

x 5
The graph intersects the x-axis at (5, 0).
To find the y-intercept, let x  0.
2(x  y)  10
2(0  y)  10
2y  10

 2

x2

2y
2

0.

10
2

y 5
The graph intersects the y-axis at (0, 5).
Plot these points and draw the line that connects
them.

To find the y-intercept, let x = 0


y  2x  1
y  2(0)  1
y  1
The graph intersects the y-axis at (0, 1).
Plot these points and draw the line that connects
them.

y
2(x  y )  10

O
O

x
y  2x  1

6-6
78. To find the x-intercept, let y  0.
2y  x  6
2(0)  x  6
x  6
(1)(x)  (1)(6)
x6

Page 355

Graphing Inequalities
in Two Variables
Check for Understanding

1. The graph of y  x  2 is a line. The graph of


y  x  2 does not include the boundary
y  x  2, and it includes all ordered pairs in the
half-plane that contains the origin.

283

Chapter 6

PQ249-6481F-06[273-291] 24/7/02 05:13 PM Page 284 Sahuja Ahuja_QXP_13:InProcess_AQ13:Sunita_23/24/7/02:PQ249-06-ReproPages:PQ249-06-Art:

2. Sample answer: x  y
Step 1 Solve for y in terms of x.
xy
yx
Step 2 Graph y  x. Since the inequality
includes y values less than x, but not
equal to x, the boundary is not included
in the solution set. The boundary should
be drawn as a dashed line.
Step 3 Select a point in one of the half-planes
and test it. Try (1, 3).
yx
3  1 false
Since the statement is false, the half-plane
containing (1, 3) is not part of the solution. Shade
the other half-plane.

6. b; Since y  2x  2 means y  2x  2 or
y 2x  2, the boundary line is included in the
solution set. So, the boundary should be drawn as
a solid line. When (0, 0) is used as a test point in
the original inequality, the resulting statement is
0  2. Since this statement is false, the half-plane
containing the origin is not part of the solution.
So, the other half-plane should be shaded. This
describes graph b.
7. Step 1 The inequality y  4 is already solved
for y in terms of x.
Step 2 Graph y  4. Since y  4 means y  4 or
y  4, the boundary is included in the
solution set. The boundary should be
drawn as a solid line.
Step 3 Select a point in one of the half-planes
and test it. Try (0, 0).
y4
0  4 false
Since the statement is false, the half-plane
containing the origin is not part of the solution.
Shade the other half-plane.

y
xy

y4

3. If the test point results in a true statement, shade


the half-plane that contains the point. If the test
point results in a false statement, shade the other
half-plane.
4. Use a table to substitute the x and y values of
each ordered pair into the inequality.
x

1

2

yx1
0  1  1
00
231
24
521
53
1  2  1
1  1

8. Step 1 The inequality y  2x  3 is already


solved for y in terms of x.
Step 2 Graph y  2x  3. Since y  2x  3
means y 6 2x  3 or y  2x  3, the
boundary is included in the solution set.
The boundary should be drawn as a solid
line.
Step 3 Select a point in one of the half-planes
and test it. Try (0, 0).
y  2x  3
0  2(0)  3
0  3 false
Since the statement is false, the half-plane
containing the origin is not part of the solution.
Shade the other half-plane.

True or False
true
true
false
false

The ordered pairs {(1, 0), (3, 2)} are part of the
solution set.
5. Use a table to substitute the x and y values of
each ordered pair into the inequality.
x

1

1

2

y  2x
6  2(2)
64
1  2(0)
1  0
5  2(3)
56
2  2(1)
2  2

True or False

y
y  2x  3

true
false

false
false

The ordered pair {(2, 6)} is part of the solution set.

Chapter 6

284

PQ249-6481F-06[273-291] 24/7/02 05:13 PM Page 285 Sahuja Ahuja_QXP_13:InProcess_AQ13:Sunita_23/24/7/02:PQ249-06-ReproPages:PQ249-06-Art:

11. Step 1 Let x  the number of pizzas ordered.


Let y  the number of pitchers of soft
drink ordered.

9. Step 1 Since there is no y in the inequality,


solve the inequality for x.
4  2x 6 2
4  2x  4 6 2  4
2x 6 6
2x
2

the cost
the number
the
of one
of pitchers
The cost
number
pitcher
of soft
of one
of pizzas
of soft
drink
pizza times ordered plus drink times ordered
14243 123 14243 123 14243 123 1442443

6
2

7
x 7 3
Step 2 Graph x  3. Since the inequality
includes x values greater than 3, but not
equal to 3, the boundary is not included
in the solution set. The boundary should
be drawn as a dashed line.
Step 3 Select a point in one of the half-planes
and test it. Try (0, 0).
x 7 3
0 7 3 false
Since the statement is false, the half-plane
containing the origin is not part of the solution.
Shade the other half-plane.

Step 2

123 123

60

Solve for y in terms of x.


12x  3y  60
12x  3y  12x  60  12x
3y  60  12x


y

60  12x
3
60
12x

3
3

y  20  4x
Step 3 Since the open sentence includes the
equation, graph y  20  4x as a solid
line. Test a point in one of the halfplanes, for example (0, 0). Shade the
half-plane containing (0, 0) since
0  20  4(02 is true.
Step 4 Examine the solution.
Coach Riley cannot order a negative
number of pizzas or a negative
number of pitchers of soft drinks.
Therefore, the domain and range
contain only nonnegative numbers.
Coach Riley cannot order a fraction of
a pizza or a portion of a pitcher of soft
drink. Thus, only points in the
shaded half-plane whose x-and
y-coordinates are whole numbers are
possible solutions.

4  2x  2

10. Step 1

Solve for y in terms of x.


1y 7 x
1y1 7 x1
y 7 x  1
(1)(y) 6 (1)(x  1)
y 6 1x
Step 2 Graph y  1  x. Since the inequality
includes y values less than 1  x, but not
equal to 1  x, the boundary is not
included in the solution set. The
boundary should be drawn as a dashed
line.
Step 3 Select a point in one of the half-planes
and test it. Try (0, 0).
y 6 1x
0 6 10
0 6 1 true
Since the statement is true, the half-plane
containing the origin is part of the solution.
Shade the half-plane containing (0, 0).

y
24
20
16
12
8
4
21 O
4

1 2 3 4 5 6x

Pages 356357

Practice and Apply

12. Use a table to substitute the x and y values of


each ordered pair into the inequality.

1 y  x
O

3y
3

12

is
not
more
than $60 .

x
0

y
4

1

8

4

y  3  2x
4  3  2(02
43
3  3  2(12
35
8  3  2(62
8  9
5  3  2(42
5  11

True or False
false
true
false
true

The ordered pairs {(1, 3), (4, 5)} are part of the
solution set.

285

Chapter 6

PQ249-6481F-06[273-291] 24/7/02 05:13 PM Page 286 Sahuja Ahuja_QXP_13:InProcess_AQ13:Sunita_23/24/7/02:PQ249-06-ReproPages:PQ249-06-Art:

17. Use a table to substitute the x and y values of


each ordered pair into the inequality.

13. Use a table to substitute the x and y values of


each ordered pair into the inequality.
x
3

y
1

3

1
1
2
2
1
1
2
2

y  3x
6 3132
6 9
6 3132
6 9
6 3112
6 3
6 3112
6 3

True or False

x
1

y
1

false

1

true

1

true

2

false

y
7

13

10

6

2

x  y  11
5  7 6 11
12 6 11
13  10 6 11
3 6 11
4  4 6 11
8 6 11
6  122 6 11
8 6 11

True or False
false

x
6

y
4

true

1

true

3

true

The ordered pairs {( 13, 10), (4, 4), (6, 2)} are
part of the solution set.
15. Use a table to substitute the x and y values of
each ordered pair into the inequality.
x
3

y
2

2

4

2x  3y  6
2132  3122 7
0 7
2122  3142 7
8 7
2162  3122 7
6 7
2152  3112 7
7 7

false

x
2

y
4

true

1

5

false

7

true

The ordered pairs {( 2, 4), (5, 1)} are part of


the solution set.
16. Use a table to substitute the x and y values of
each ordered pair into the inequality.
x
5

y
1

2

4y  8  0
4112  8  0
12  0
4122  8  0
00
4152  8  0
12  0
4102  8  0
8  0

true
true
false

|x  3| y
06  3 0  4
34
01  3 0  8
48
03  3 0  2
62
05  3 0  7
27

True or False
false
false
true
false

|y  2|  x
04  2 0 6 2
2 6 2
05  2 0 6 1
3 6 1
07  2 0 6 6
5 6 6
00  2 0 6 0
2 6 0

True or False
false
false
true
false

The ordered pair {(6, 7)} is part of the solution


set.

True or False
false
true
true
false

The ordered pairs {(0, 2), (2, 5)} are part of the
solution set.

Chapter 6

false

The ordered pair {(3, 2)} is part of the solution


set.
19. Use a table to substitute the x and y values of
each ordered pair into the inequality.

True or False
6
6
6
6
6
6
6
6

True or False
7
7
7
7
7
7
7
7

The ordered pairs {(2, 1), (1, 1)} are part of the
solution set.
18. Use a table to substitute the x and y values of
each ordered pair into the inequality.

The ordered pairs {(1, 1), (1, 2)} are part of the
solution set.
14. Use a table to substitute the x and y values of
each ordered pair into the inequality.
x
5

3x  4y  7
3112  4112 6
7 6
3122  4112 6
2 6
3112  4112 6
1 6
3122  4142 6
10 6

286

PQ249-6481F-06[273-291] 24/7/02 05:13 PM Page 287 Sahuja Ahuja_QXP_13:InProcess_AQ13:Sunita_23/24/7/02:PQ249-06-ReproPages:PQ249-06-Art:

20. c;
Step 1

Step 2 Graph y  3  2x. Since the inequality


1
includes values of y greater than 3  2x,
1
but not equal to 3  2x, the boundary is
not included in the solution set. The
boundary should be drawn as a dashed
line.
Step 3 Select a point in one of the half-planes
and test it. Try (0, 0).

6  x
2
6  x
y 2
6  x
6  x
Graph y  2 . Since y  2 means
6  x
6  x
y 6 2 or y  2 , the boundary is
2y
2

Step 2

22. d;
1
Step 1 The inequality y 7 3  2x is already
solved for y in terms of x.

Solve for y in terms of x.


2y  x  6
2y  x  x  6  x
2y  6  x


included in the solution set. The


boundary should be drawn as a solid
line.
Step 3 Select a point in one of the half-planes
and test it. Try (0, 0).
y
0

y 7 3  2x
1

0 7 3  2 (0)
false
0 7 3
Since the statement is false, the half-plane
containing the origin is not part of the solution.
Shade the other half-plane. The graph described
is graph d.
23. b;
Step 1 Solve for y in terms of x.
4y  2x  16
4y  2x  2x  16  2x
4y  16  2x

6  x
2
6  0
2

true
03
Since the statement is true, the half-plane
containing the origin is part of the solution.
Shade the half-plane containing (0, 0). The graph
described is graph c.
21. a;
Step 1 Solve for y in terms of x.
1
x
2
1
x
2

4y
4

y 7 4
1

y
1

 y  2x 7 4  2x
y 7 4 

16  2x
4
1
4  2x

Step 2 Graph y  4  2x. Since y  4  2x

1
x
2

means y 7 4  2x or y  4  2x, the


boundary is included in the solution set.
The boundary should be drawn as a solid
line.
Step 3 Select a point in one of the half-planes
and test it. Try (0, 0).

(1)(y) 6 (1)(4  2x)


1

y 6 2x  4
1

Step 2 Graph y  2x  4. Since the inequality


1
includes values of y less than 2x  4, but
1
not equal to 2x  4, the boundary is not
included in the solution set. The
boundary should be drawn as a dashed
line.
Step 3 Select a point in one of the half-planes
and test it. Try (0, 0).

y  4  2x
1

0  4  2 (0)
04
false
Since the statement is false, the half-plane
containing the origin is not part of the solution.
Shade the other half-plane. The graph described
is graph b.
24. Substitute x  2 into the equation 2x  3y  5
to find the corresponding value of y.
2x  3y  5
2122  3y  5
4  3y  5
4  3y  4  5  4
3y  9

y 6 2x  4

0 6 2 102  4
1

false
0 6 4
Since the statement is false, the half-plane
containing the origin is not part of the solution.
Shade the other half-plane. The graph described
is graph a.

3y
3

3

y3
The point A (2, 3) is on the graph of
2x  3y  5.

287

Chapter 6

PQ249-6481F-06[273-291] 24/7/02 05:13 PM Page 288 Sahuja Ahuja_QXP_13:InProcess_AQ13:Sunita_23/24/7/02:PQ249-06-ReproPages:PQ249-06-Art:

25. Substitute x  0 into the equation 4x  3y  4 to


find the corresponding value of y.
4x  3y  4
4102  3y  4
3y  4
3y
3

10y
10

1
1
13

y 7 2x

1 7 2 112
1
1

1 7 2

true

Since the statement is true, the half-plane


containing (1, 1) is part of the solution. Shade the
half-plane containing (1, 1).
y
5x  10y  0

29. Step 1 The inequality y 6 x is already solved


for y in terms of x.
Step 2 Graph y  x. Since the inequality
includes y values less than x, but not
equal to x, the boundary is not included
in the solution set. The boundary should
be drawn as a dashed line.
Step 3 Select a point in one of the half-planes
and test it. Try (2, 0).
y 6 x
0 6 2 true
Since the statement is true, the half-plane
containing (2, 0) is part of the solution. Shade the
half-plane containing (2, 0).

y  3

27. Step 1 There is no y in the inequality. The


inequality x  2 is already solved for x.
Step 2 Graph x  2. Since x  2 means x 7 2 or
x  2, the boundary is included in the
solution set. The boundary should be
drawn as a solid line.
Step 3 Select a point in one of the half-planes
and test it. Try (0, 0).
x2
0  2 false
Since the statement is false, the half-plane
containing the origin is not part of the solution.
Shade the other half-plane.

y
yx

5x
10
1
2x

Step 2 Graph y  2x. Since the inequality


1
includes y values greater than 2x, but
1
not equal to 2x, the boundary line is
not included in the solution set. The
boundary should be drawn as a dashed
line.
Step 3 Select a point in one of the half-planes
and test it. Try (1, 1).

y 7

The point 0, 
is on the graph of
2x  3y  5. Since the graph of (0, 1) is above
1
the graph of 0,  13 , the point B (0, 1) is above
the graph of 4x  3y  4.
26. Step 1 The inequality y 6 3 is already solved
for y in terms of x.
Step 2 Graph y  3. Since the inequality
includes y values less than 3, but not
equal to 3, the boundary is not
included in the solution set. The
boundary should be drawn as a dashed
line.
Step 3 Select a point in one of the half-planes
and test it. Try (0, 0).
y 6 3
0 6 3 false
Since the statement is false, the half-plane
containing the origin is not part of the solution.
Shade the other half-plane.

x2

Chapter 6

Solve for y in terms of x.


5x  10y 7 0
5x  10y  5x 7 0  5x
10y 7 5x

4
3

y  13

28. Step 1

288

PQ249-6481F-06[273-291] 24/7/02 05:14 PM Page 289 Sahuja Ahuja_QXP_13:InProcess_AQ13:Sunita_23/24/7/02:PQ249-06-ReproPages:PQ249-06-Art:

30. Step 1

6  x
2
6  x
y 2
6  x
6  x
Graph y  2 . Since y  2 means
6  x
6  x
y 6 2 or y  2 , the boundary is
2y
2

Step 2

32. Step 1

Solve for y in terms of x.


2y  x  6
2y  x  x  6  x
2y  6  x

12  4x
3
12  4x
y
3
12  4x
12  4x
.
Since y 
3
3
12  4x
12  4x
or y 
, the
3
3

3y
3

0

Step 2 Graph y 
means y 7
boundary is included in the solution set.
The boundary should be drawn as a solid
line.
Step 3 Select a point in one of the half-planes
and test it. Try (0, 0).

included in the solution set. The


boundary should be drawn as a solid line.
Step 3 Select a point in one of the half-planes
and test it. Try (0, 0).
y

Solve for y in terms of x.


3y  4x  12
3y  4x  4x  12  4x
3y  12  4x

6  x
2
6  0
2

y
0

true
03
Since the statement is true, the half-plane
containing the origin is part of the solution.
Shade the half-plane containing (0, 0).

12  4x
3
12  4102
3

false
04
Since the statement is false, the half-plane
containing the origin is not part of the solution.
Shade the other half-plane.

y
2y  x  6

O
3y  4x  12

31. Step 1

Solve for y in
6x  3y
6x  3y  6x
3y
3y
3

terms of x.
7 9
7 9  6x
7 9  6x
7

33. Step 1 The inequality y  2x  4 is already


solved for y in terms of x.
Step 2 Graph y  2x  4. Since y  2x  4
means y 6 2x  4 or y  2x  4, the
boundary is included in the solution set.
The boundary should be drawn as a solid
line.
Step 3 Select a point in one of the half-planes
and test it. Try (0, 0).
y  2x  4
0  2(0)  4
false
0  4
Since the statement is false, the half-plane
containing the origin is not part of the solution.
Shade the other half-plane.

9  6x
3

y 7 3  2x
Step 2 Graph y  3  2x. Since the inequality
includes y values greater than 3  2x,
but not equal to 3  2x, the boundary is
not included in the solution set. The
boundary should be drawn as a dashed
line.
Step 3 Select a point in one of the half-planes
and test it. Try (0, 0).
y 7 3  2x
0 7 3  2102
false
0 7 3
Since the statement is false, the half-plane
containing the origin is not part of the solution.
Shade the other half-plane.

y
y  2x  4

6x  3y  9

289

Chapter 6

PQ249-6481F-06[273-291] 24/7/02 05:14 PM Page 290 Sahuja Ahuja_QXP_13:InProcess_AQ13:Sunita_23/24/7/02:PQ249-06-ReproPages:PQ249-06-Art:

34. Step 1

Solve for y in
8x  6y
8x  6y  8x
6y
6y
6

36. Step 1

terms of x.
6 10
6 10  8x
6 10  8x
7

y 7
y 7
4

3(x  2y)
3

2y
2

0 7

0 7

18
3

6  x
2
6  x
7
2
6  x
. Since
2

Step 2 Graph y 
the inequality
6  x
includes y values greater than 2 , but
6  x
not equal to 2 , the boundary is not
included in the solution set. The boundary
should be drawn as a dashed line.
Step 3 Select a point in one of the half-planes
and test it. Try (0, 0).

y 7 3x  3
4
102
3
5
3

x  2y 7 6
x  2y  x 7 6  x
2y 7 6  x

10  8x
6
8x  10
6
5
4
x
3
3
5
. Since
3

Step 2 Graph y  3x 
the inequality
5
4
includes y values greater than 3x  3,
5
4
but not equal to 3x  3, the boundary is
not included in the solution set. The
boundary should be drawn as a dashed
line.
Step 3 Select a point in one of the half-planes
and test it. Try (0, 0).
4

Solve for y in terms of x.


3(x  2y) 7 18

y 7
5
3

0 7

6  x
2
6  0
2

true
0 7 3
Since the statement is true, the half-plane
containing the origin is part of the solution.
Shade the half-plane containing (0, 0).

true
Since the statement is true, the half-plane
containing the origin is part of the solution.
Shade the half-plane containing (0, 0).

8x  6y  10
O

x
3(x  2y )  18

35. Step 1

Solve for y in terms of x.


3x  1  y
y  3x  1
Step 2 Graph y  3x  1. Since y  3x  1
means y 6 3x  1 or y  3x  1, the
boundary is included in the solution set.
The boundary should be drawn as a solid
line.
Step 3 Select a point in one of the half-planes
and test it. Try (0, 0).
y  3x  1
0  3(0)  1
0  1
false
Since the statement is false, the half-plane
containing the origin is not part of the solution.
Shade the other half-plane.
y

3x  1  y

Chapter 6

290

PQ249-6481F-06[273-291] 24/7/02 05:14 PM Page 291 Sahuja Ahuja_QXP_13:InProcess_AQ13:Sunita_23/24/7/02:PQ249-06-ReproPages:PQ249-06-Art:

37. Step 1

42. Let s  the number of tickets sold to a single


person. Let c  the number of tickets sold to
couples.

Solve for y in terms of x.


1
(2x
2
1
(2) 2 (2x

 y) 6 2
 y) 6 (2)2

The cost of
one ticket sold
to a single person

2x  y 6 4
2x  y  2x 6 4  2x
y 6 4  2x
Step 2 Graph y  4  2x. Since the inequality
includes y values less than 4  2x, but not
equal to 4  2x, the boundary is not
included in the solution set. The boundary
should be drawn as a dashed line.
Step 3 Select a point in one of the half-planes
and test it. Try (0, 0).
y 6 4  2x
0 6 4  2(0)
0 6 4
true
Since the statement is true, the half-plane
containing the origin is part of the solution.
Shade the half-plane containing (0, 0).

14444244443

the number of
tickets sold to a
single person

times
1
424
3


14
444244
443

plus

1
424
3

the number of
the cost of
tickets sold to
one ticket sold
times
couples
to a couple
is at least $1200
424
3 14
1444
4244443 1
444244
443 1
4
44244
4
3 1
424
3

c

1200
8

This situation is represented by the inequality


5s  8c  1200.
43. Step 1 Solve the inequality from Exercise 42 for
c in terms of s.
5s  8c  1200
5s  8c  5s  1200  5s
8c  1200  5s
8c
1200  5s

8
8
c  150 

Step 2

5
s
8

Graph c  150  8s. Since c  150  8s


5

1(
2x  y )  2
2

means c 7 150  8s or c  150  8s, the


boundary is included in the solution set.
The boundary should be drawn as a solid
line.
Step 3 Select a point in one of the half-planes
and test it. Try (0, 0).
5

c  150  8s
38. Let /  the length of the package. Let d  the
distance around the thickest part of the package.

0  150  8 (0)
false
0  150
Since the statement is false, the half-plane
containing the origin is not part of the solution.
Shade the other half-plane.
Step 4 Examine the solution.
It is not possible to sell a negative number of
tickets. Therefore, the domain and range
contain only nonnegative numbers.
It is not possible to sell a portion of a ticket.
Thus, only points in the shaded half-plane
whose x- and y-coordinates are whole numbers
are possible solutions.

the distance around is less


The length
the thickest part than or
of the
of the package
equal to 108 inches.
longest side plus

1442443 1
424
3 14
44442444
443 14243

1442443

108

This situation is represented by the inequality


/  d  108.
39. The solution set is limited to pairs of positive
numbers.
40. Let t  the number of televisions. Let m  the
number of microwaves.
The weight
the number
the weight
the number
of one
of
of one
of
television times televisions plus microwave times microwaves

1442443 1
424
3 1442443 1
424
3 1442443 1
424
3 1442443

77

is less
than or
equal to

4000 lb.

4000

55

160
140
120
100
80
60
40
20

14243 1442443

This situation is represented by the inequality


77t  55m  4000.
41. Substitute the t and m values of the ordered pair
(35, 25) into the inequality from Exercise 40.
77t  55m  4000
77(35)  55(25)  4000
2695  1375  4000
4070  4000 false
Since the ordered pair does not make the
inequality true, the truck will not be able to
deliver 35 televisions and 25 microwaves at once.
The total weight would be greater than 4000 lb.

5s  8c  1200

20

60
40

291

80

100 140 180 220 s


120 160 200 240

Chapter 6

PQ249-6481F-06[292-304] 31/7/02 21:38 Page 292 Sahuja Ahuja_QXP_09:Desktop Folder:Chandra_31-07-2002:249:

46. The amount of money spent in each category


must be less than or equal to the budgeted
amount. How much you spend on individual items
can vary. Answers should include the following.
The domain and range must be positive
integers.
Sample answers: Hannah could buy 5
cafeteria lunches and 3 restaurant lunches, 2
cafeteria lunches and 5 restaurant lunches, or
8 cafeteria lunches and 1 restaurant lunch.
47. D; y  2x 6 5
6  2152 6 5
6  10 6 5
4 6 5 false
48. B; Since 2x  y 7 1 does not include 2x  y  1,
the boundary should be drawn as a dashed line.
Test (2, 2).
2x  y 7 1
2(2)  2 7 1
6 7 1 true
The half-plane containing (2, 2) should be shaded.

44. Since the graph of the ordered pair (100, 125) is in


the shaded half-plane of the graph from Exercise 43,
the values s  100 and c  125 make the inequality
true. So, the committee would cover its expenses if
100 single tickets and 125 couple tickets are sold.
To check algebraically, substitute s  100 and
c  125 into the inequality from Exercise 42.
5s  8c  1200
5(100)  8(125)  1200
500  1000  1200
1500  1200 true
Since the ordered pair makes the inequality true,
the conclusion reached using the graph is correct.
45. First graph y  x  1. Since the inequality
includes y  x  1, the boundary should be drawn
as a solid line.
Test (0, 0).
001
false
0  1
Shade the half-plane that does not contain the origin.
y
y  x 1

Page 357
x

Now graph y  x. Since the inequality includes


y  x, the boundary should be drawn as a solid
line.
Test (1, 1).
true
1  1
Shade the half-plane that contains (1, 1).

2t
2

14
2

t  7

50. Write x  8 6 6 as x  8 6 6 and x  8


Case 1:
Case 2:
x8 6 6
x8 7
x88 6 68
x88 7
x 6 2
x 7
The solution set is {x 14 6 x 6 2}.

y  x

18 1614 12 10 8 6 4 2

7 6.
6
6  8
14

51. Write 2y  5  3 as 2y  5  3 or 2y  5  3.


Case 1:
Case 2:
2y  5  3
2y  5  3
2y  5  5  3  5
2y  5  5  3  5
2y  2
2y  8

To graph the intersection of the graphs of y  x  1


and y  x, graph both inequalities above on the
the same coordinate plane. The region where the
two shaded half-planes overlap represents the
intersection of the graphs. Only points in this
region will make both inequalities true.

2y
2

2
2

2y
2

8
2

y  1
y  4
The solution set is { y y  4 or y  1}.

yx1

8 7 6 5 4 3 2 1 0 1 2

y  x

Chapter 6

108 6 4 2 0 2 4 6 8 10

2t
2

2

t4
The solution set is {7, 4}.

Maintain Your Skills

49. Write 3  2t 0  11 as 3  2t  11 or
3  2t  11.
Case 1:
Case 2:
3  2t  11
3  2t  11
3  2t  3  11  3
3  2t  3  11  3
2t  8
2t  14

292

PQ249-6481F-06[292-304] 31/7/02 21:38 Page 293 Sahuja Ahuja_QXP_09:Desktop Folder:Chandra_31-07-2002:249:

52.

and
y  6 7 1
y2 6 4
y  6  6 7 1  6
y22 6 42
y 7 7
y 6 6
The solution set is the intersection of the two graphs.
Graph y 7 7.
Graph y 6 6.
Find the intersection.
10

56. Find the amount of change. Since the new


amount is greater than the original amount, the
percent of change is a percent of increase.
75  53  22
Find the percent using the original number, 53, as
the base.
22
53

22(100)  53(r)
2200  53r

10

10

10

10

10

 100

2200
53

53r
53

41.51  r
To the nearest whole percent, the percent of
increase is 42%.
57.

The solution set is {y 7 6 y 6 6}.


53.
or
m4 6 2
m2 7 1
m44 6 24
m22 7 12
m 6 2
m 7 3
The solution set is the union of the two graphs.
Graph m 6 2.
Graph m 7 3.
Find the union.

d  2
3
d  2
3

7

2  3(7)

d  2  21
d  2  2  21  2
d  23
58.
3n  6  15
3n  6  6  15  6
3n  21
3n
3

59.
5

n  7
35  20h  100
35  20h  35  100  35
20h  65

60.

64
4

61.

27c
9

62.

12a  14b
2

 27c  192
 3c
 112a  14b2  122

1 12

 112a  14b2 2

1 12

 6a  7b

200r
200

63.

1 12

18y  9
3

 118y  92  3

113 2
1
1
 18y 1 3 2  9 1 3 2
1
1
 18y 1 3 2  9 1 3 2
 118y  92

 6y  3

Page 358

 100

or 3.25

 12a 2  114b2 2

4211002  1001r2
4200  100r
4200
100

13
4

 64  4
 16

r
100

14  r
The percent of decrease is 14%.
55. Find the amount of change. Since the new
amount is greater than the original amount, the
percent of change is a percent of increase.
142  100  42
Find the percent using the original number, 100,
as the base.
42
100

65

 20

h

2811002  2001r2
2800  200r
2800
200

21
3

20h
20

The solution set is {m m 6 2 or m 7 3}.


54. Find the amount of change. Since the new
amount is less than the original amount, the
percent of change is a percent of decrease.
200  172  28
Find the percent using the original number, 200,
as the base.
28
200

Graphing Calculator Investigation


(Follow-Up of Lesson 6-6)

1. y  3x  1 is shaded below the line.


y  3x  1. y  3x  1 is shaded above the line.
y  3x  1.

100r
100

42  r
The percent of increase is 42%.

293

Chapter 6

PQ249-6481F-06[292-304] 31/7/02 21:38 Page 294 Sahuja Ahuja_QXP_09:Desktop Folder:Chandra_31-07-2002:249:

2.

KEYSTROKES:

4 , 10

2nd

3.
4.
5.
6.
7.
8.

DRAW 7 ( ) 2 X,T,,n
ENTRY

d;
a;
c;
g;
h;
b;

intersection
Addition Property of Inequalities
half-plane
Subtraction Property of Inequalities
union
Division Property of Inequalities

Pages 359362

2a. The lower boundary is y  2x  4. The upper


boundary is Ymax or 10.
2b. The coordinates of any point in the shaded
region are solutions of the inequality. Sample
answer: {(0, 4), (1, 7), (2, 6), (4.2, 1.5)}
3a. The number of
the number of
plus
student
tickets
adult tickets
1442443 123 1442443
x

y
is
at
least
10.
14243
123

10

25

10.

The cost
the number
the cost
of one
of
of one
student
student
adult
ticket
times
tickets
plus
ticket
14243 123 1442443 123 14243
4

x

8
the number
of
times 14
adult
tickets
no more than 1
$80.
123
424
43 is
1442443
23

23

11.

19

17

15

13

11

7

9

5

w  14  23
w  14  14  23  14
w  37
The solution set is {w w  37}.
35 36 37 38 39 40 41 42 43 44 45

12.

a6 7
a66 7
a 7
The solution

10
10  6
4
set is {a a 7 4}.

7 6 5 4 3 2 1 0 1 2 3

3c. KEYSTROKES: 2nd DRAW 7 ( ) X,T,,n


10 , ( ) 0.5 X,T,,n
10 )

13.

ENTRY

0.11  n  10.042
0.11  n  0.04
0.11  0.04  n  0.04  0.04
0.15  n
0.15  n is the same as n  0.15.
The solution set is {n n  0.15}.
2

14.
3d. The coordinates of any point in the shaded
region are solutions of the inequality. Sample
answer: {(8, 5), (10, 4), (14, 2), (20, 0)} where
(8, 5) represents the purchase of 8 student
tickets and 5 adult tickets, for example.
15.

Chapter 6 Study Guide and Review


Vocabulary and Concept Check

1. f; set-builder notation
2. e; Multiplication Property of Inequalities

1

2.3 6 g  12.12
2.3 6 g  2.1
2.3  2.1 6 g  2.1  2.1
0.2 6 g
0.2 6 g is the same as g 7 0.2.
The solution set is { g g 7 0.2}.
2

Chapter 6

21

r  7 7 5
r  7  7 7 5  7
r 7 12
The solution set is {r r 7 12}.
15


y

80
So, x  y  10 represents the total number of
tickets, and 4x  8y  80 represents the total
cost of the tickets.
3b. Rewrite x  y  10 as y  x  10. Rewrite
4x  8y  80 as y  0.5x  10. The lower
boundary is y  x  10. The upper boundary
is y  0.5x  10.

Page 359

Lesson-by-Lesson Review

Exercises 917 For checks, see students work.


9.
c  51 7 32
c  51  51 7 32  51
c 7 19
The solution set is {c c 7 19}.

1

7h  6h  1
7h  6h  6h  1  6h
h  1
The solution set is {h h  1}.
5 4 3 2 1 0 1 2 3 4 5

294

PQ249-6481F-06[292-304] 31/7/02 21:38 Page 295 Sahuja Ahuja_QXP_09:Desktop Folder:Chandra_31-07-2002:249:

16.

5b 7 4b  5
5b  4b 7 4b  5  4b
b 7 5
The solution set is {b b 7 5}.

Exercises 2735 For checks, see students work.


4h  7 7 15
27.
4h  7  7 7 15  7
4h 7 8
4b
4

2 1 0 1 2 3 4 5 6 7 8

h 6 2

17. Let n  the number.


21  n  (2)
21  n  2
21  2  n  2  2
23  n
23  n is the same as n  23.
The solution set is {n n  23}.
Exercises 1826 For checks, see students work.
18. 15v 7 60
15v
15

The solution set is {h h 6 2}.


28.

60
15

16
8

72

75

 15

z5
The solution set is {z z  5}.
21. 9m 6 99
7

99
9

b
12
b
(12) 12

d
13
d
(13) 13

3
 (12)3

7 5

56
8

6 (13)(5)

2
w
3
3 2
w
2 3

12

7 22
7

132 2 (22)

w 7 33
The solution set is {w w 7 33}.
25.

3
p
5
5 3
p
3 5

12

 15


153 2 (15)

33.

p  25
The solution set is {p p  25}.
26. Let n  the number.
0.80n  24
0.80n
0.80

72
8

8q
8

7 6 q
7 6 q is the same as q 7 7.
The solution set is 5q q 7 76.
7(g  8) 6 3(g  2)  4g
32.
7g  56 6 3g  6  4g
7g  56 6 7g  6
7g  56  7g 6 7g  6  7g
56 6 6
Since the inequality results in a false statement,
the solution set is the empty set .

d 6 65
The solution set is {d d 6 65}.
24.

8x
8

b 7 9
The solution set is {b b 7 9}.
5(q  122 6 3q  4
31.
5q  60 6 3q  4
5q  60  5q 6 3q  4  5q
60 6 8q  4
60  4 6 8q  4  4
56 6 8q

b  36
The solution set is {b b  36}.
23.

8b
8

m 7 11
The solution set is {m m 7 11}.
22.

 24
6

2 6 x
2 x is the same as x
2.
The solution set is {x x 7 2}.
15b  12 7 7b  60
30.
15b  12  7b 7 7b  60  7b
8b  12 7 60
8b  12  12 7 60  12
8b 7 72

r6
The solution set is {r r  6}.
20. 15z  75

9m
9

n 6 4
The solution set is 5n n 6 46.
5x  3 6 3x  19
29.
5x  3  5x 6 3x  19  5x
3 6 8x  19
3  19 6 8x  19  19
16 6 8x

 12

15z
15

5  6n 7 19
5  6n  5 7 19  5
6n 7 24
6n
6

v 7 4
The solution set is 5v v 7 46 .
19. 12r  72
12r
12

8
4

12

2(x  22

3
3 2(x  2)
2
3

4


132 24

x26
x2262
x4
The solution set is {x x  4}.

24

 0.80

n  30
The solution set is {n n  30}.

295

Chapter 6

PQ249-6481F-06[292-304] 31/7/02 21:38 Page 296 Sahuja Ahuja_QXP_09:Desktop Folder:Chandra_31-07-2002:249:

1  7n
5
1  7n
(52 5

34.

38.

7 10
7 5(102

or
3w  8 6 2
w  12 7 2  w
3w  8  8 6 2  8
w  12  w 7 2  w  w
3w 6 6
2w  12 7 2

1  7n 7 50
1  7n  1 7 50  1
7n 7 49
7n
7

3w
3

2
n
3

132 236

39.

The solution set is 5p|4 6 p 6 26.


37. First express 3 6 2k  1 6 5 using and.
3 6 2k  1
2k  1 6 5
and
3  1 6 2k  1  1
2k  1  1 6 5  1
2 6 2k
2k 6 6
6

10
2

 36

n  54
The solution set is 5n n  546.
36. First express 1 6 p  3 6 5 using and.
1 6 p  3
p3 6 5
and
1  3 6 p  3  3
p33 6 53
4 6 p
p 6 2
The solution set is the intersection of the two
graphs.
Graph 4 6 p or p 7 4.
Graph p 6 2.
Find the intersection.

2
2

2w 7 10

w 7 5
The solution set is the union of the two graphs.
Graph w 6 2. Graph w 7 5.
Find the union.

 27  27  9  27

12

2w  12  12 7 2  12
2w
2

49
7

 27  9
2
n
3
3 2
n
2 3

6
3

w 6 2

n 6 7
The solution set is {n n 6 7}.
35. Let n  the number.
2
n
3

2k
2

2k
2

1 6 k

40.

6
2

k 6 3

The solution set is the intersection of the two


graphs.
Graph 1 6 k or k 7 1.
Graph k 6 3.
Find the intersection.

The solution set is {w|w is a real number.}.


or
a38
a  5  21
a3383
a  5  5  21  5
a  11
a  16
The solution set is the union of the two graphs.
Graph a  11.
Graph a  16.
Find the union.
9

10

11

12

13

14

15

16

17

18

19

10

11

12

13

14

15

16

17

18

19

10

11

12

13

14

15

16

17

18

19

The solution set is {a a  11 or a  16}.


and
3m 6 5
m8 6 4
3m3 6 53
m88 6 48
m 6 4
m 6 2
(12(m2 7 (122
m 7 2
The solution set is the intersection of the two
graphs.
Graph m 6 4
Graph m 7 2.
Find the intersection.
5

Since the graphs do not intersect, the solution set


is the empty set .

The solution set is 5k|1 6 k 6 36.

Chapter 6

296

PQ249-6481F-06[292-304] 31/7/02 21:38 Page 297 Sahuja Ahuja_QXP_09:Desktop Folder:Chandra_31-07-2002:249:

41.

and
10  2y 7 12
7y 6 4y  9
10  2y  10 7 12  10 7y  4y 6 4y  9  4y
2y 7 2
3y 6 9
2y
2

3y
3

2
2

46. Write r  10 6 3 as r  10 6 3 and


r  10 7 3.
Case 1:
Case 2:
r  10 6 3
r  10 7 3
r  10  10 6 3  10 r  10  10 7 3  10
r 6 7
r 7 13
The solution set is 5r 13 6 r 6 76.

9
3

y 6 1
y 6 3
The solution set is the intersection of the two
graphs.
Graph y 6 1.
Graph y 6 3.
Find the intersection.

1514131211109 8 7 6 5

47. Write t  4  3 as t  4  3 and t  4  3.


Case 1:
Case 2:
t43
t  4  3
t434
t  4  4  3  4
t  1
t  7
The solution set is 5t 7  t  16.
9 8 7 6 5 4 3 2 1 0 1

48. Write 0 2x  5 0 6 4 as 2x  5 6 4 and


2x  5 7 4.
Case 1:
Case 2:
2x  5 6 4
2x  5 7 4
2x  5  5 6 4  5
2x  5  5 7 4  5
2x 6 1
2x 7 9

The solution set is 5y y 6 16.


42. Write w  8  12 as w  8  12 or
w  8  12.
Case 1:
Case 2:
w  8  12
w  8  12
w  8  8  12  8
w  8  8  12  8
w  20
w  4
The solution set is 54, 206.

2x
2

1
2
1
2

x 6

2x
2

x 7

9
2
9
2

or 42

The solution set is x 42 6 x 6 2 .

128 4 0 4 8 12 16 20 24 28

43. Write q  5  2 as q  5  2 or q  5  2.


Case 1:
Case 2:
q52
q  5  2
q5525
q  5  5  2  5
q  3
q  7
The solution set is 57, 36.

8 7 6 5 4 3 2 1 0 1 2

49. Write 0 3d  4 0 6 8 as 3d  4 6 8 and


3d  4 7 8.
Case 1:
Case 2:
3d  4 6 8
3d  4 7 8
3d  4  4 6 8  4
3d  4  4 7 8  4
3d 6 4
3d 7 12

9 8 7 6 5 4 3 2 1 0 1

44. Write h  5 7 7 as h  5 7 7 or h  5 6 7.


Case 1:
Case 2:
h5 7 7
h  5 6 7
h55 7 75
h  5  5 6 7  5
h 7 2
h 6 12
The solution set is 5h h 6 12 or h 7 26.

3d
3

d 6

4
3
4
3

3d
3
1

12
3

d 7 4

or 13

The solution set is d 4 6 d 6

1
13

6.

6 5 4 3 2 1 0 1 2 3 4

50. Use a table to substitute the x and y values of


each ordered pair into the inequality.

16141210 8 6 4 2 0 2 4

45. Write w  8  1 as w  8  1 or w  8  1.


Case 1:
Case 2:
w81
w  8  1
w8818
w  8  8  1  8
w  7
w  9
The solution set is {w w  9 or w  7}.

109 8 7 6 5 4 3 2 1 0

2

4

11

3x  2y < 9
3(12  2(32
9
3(32  2(22
13
3(22  2(72
8
3(42  2(112
10

6
6
6
6
6
6
6
6

True or False
9
9
9
9
9
9
9
9

false
false
true
false

The ordered pair 5(2, 726 is part of the solution


set.

297

Chapter 6

PQ249-6481F-06[292-304] 31/7/02 21:39 Page 298 Sahuja Ahuja_QXP_09:Desktop Folder:Chandra_31-07-2002:249:

51. Use a table to substitute the x and y values of


each ordered pair into the inequality.
x

5  y  4x

True or False

5

5  (52  4(22
10  8

true

1
2

574
2  2

false

1

5  6  4(12
1  4

true

3

20

5  20  4(32
15  12

false

112 2

54. Since the boundary is not included in the


solution, draw a dashed line.
Test the point (0, 0).
y  2x 6 3
0  2(0) 6 3
0 6 3 false
The half plane that does not contain (0, 0) should
be shaded.
y
y  2x  3

The ordered pairs {(2, 5), (1, 6)} are part of


the solution set.
52. Use a table to substitute the x and y values of
each ordered pair into the inequality.
x

4

15

1
y
2
1
(152
2
1
72

6x
 6  (42
 10

1
(12
2
1
2

1

1
(82
2

63

65

43
2

25

1
(252
2
1
122

 6  (22
8

True or False

55. Since the boundary is included in the solution,


draw a solid line.
Test the point (0, 0).
x  2y  4
0  2(0)  4
0  4 false
The half plane that does not contain (0, 0) should
be shaded.

true

true

false

false

x  2y  4
x

The ordered pairs {(4, 15), (5, 1)} are part of the
solution set.
53. Use a table to substitute the x and y values of
each ordered pair into the inequality.
x

2x  8  y

true

2(3) 6 8  6
6 6 2

true

4

2(42 6 8  0
8 6 8

false

3

2(32 6 8  6
6 6 2

false

10

True or False

2(5) 6 8  10
10 6 2

56. Since the boundary is included in the solution,


draw a solid line.
Test the point (0, 0).
y  5x  1
0  5(0)  1
01
true
The half plane that contains (0, 0) should be
shaded.
y

The ordered pairs 5(5, 102, (3, 626 are part of the
solution set.

y  5x  1

Chapter 6

298

PQ249-6481F-06[292-304] 31/7/02 21:39 Page 299 Sahuja Ahuja_QXP_09:Desktop Folder:Chandra_31-07-2002:249:

6.

57. Since the boundary is not included in the


solution, draw a dashed line.
Test the point (0, 0).
2x  3y 7 6
2(0)  3(0) 7 6
0 7 6 false
The half plane that does not contain (0, 0) should
be shaded.

9p 6 8p  18
9p  8p 6 8p  18  8p
p 6 18
Check:
Substitute 18, a number less than
18, and a number greater than 18.
Let p  18
Let p  20
?

9(20) 6 8(20)  18

180 6 160  18
180 6 178

9(18) 6 8(18)  18

?
?

162 6 144  18
162 162

Let p  10.
?

9(10) 6 8(10)  18
?

90 6 80  18
90 98
The solution set is 5p p 6 186.
7.
d  5 6 2d  14
d  5  d 6 2d  14  d
5 6 d  14
5  14 6 d  14  14
9 6 d
9 6 d is the same as d 7 9.
Check:
Substitute 9, a number less than 9,
and a number greater than 9.
Let d  9.
Let d  0.

O
2x  3y  6

Chapter 6 Practice Test


Page 363

1. 5t t  176
2.
6(a  52 6 2a  8
6a  30 6 2a  8
6a  30  2a 6
4a  30 6
4a  30  30 6
4a 6
4a
4

Original equation
Distributive
Property
2a  8  2a Subtract 2a from
each side.
Simplify.
8
Subtract 30 from
8  30
each side.
Simplify.
22
22
Divide each side
4
by 4.

23  17  6
23  23
Let g  10.

0  5 6 2(02  14

4 6 18  14
4 4

5 14

Let d  12.
?
12  5 6 2(12)  14
?

7 6 24  14
7 6 10
The solution set is 5d|d 7 96.

a 6 5.5
Simplify.
The solution set is 5a a 6 5.56.
3. Sample answer: 2 6 x 6 8 is a compound
inequality that is an intersection since it can be
written using and as 2 6 x and x 6 8.
Sample answer: x 6 2 or x 7 8 is a compound
inequality that is a union since it contains the
word or.
4. Both graphs have dots at 3 and 3. The graph of
x  3 is darkened between the two dots. The
graph of x  3 is darkened to the right of the
dot at 3 and to the left of the dot at 3.
5.
23  g  6
23  6  g  6  6
17  g
17  g is the same as g  17.
Check:
Substitute 17, a number less than
17, and a number greater than 17.
Let g  17.
Let g  20.
?

9  5 6 2(9)  14

8.

7
w
8
8 7
w
7 8

12

 21


187 2(21)

w  24
Check:
Substitute 24, a number less than
24, and a number greater than 24.
Let w  24.
Let w  40. Let w  16.
?
7
(242 
8

21

?
7
(402 
8

21  21

21

35 21

The solution set is 5w w  246.

?
7
(162 
8

21

14  21

9. 22b  99
22b
22

99

 22

b  4.5
Check:
Substitute 4.5, a number less than
4.5, and a number greater than 4.5.
Let b  4.5.
Let b  6.
?

22(4.5)  99
99  99

23  20  6
23  26

22(6)  99
132 99

Let b  1.
?

22(1)  99
22  99
The solution set is 5b b  4.56.

23  10  6
23 4
The solution set is {g g  17}.

299

Chapter 6

PQ249-6481F-06[292-304] 31/7/02 21:39 Page 300 Sahuja Ahuja_QXP_09:Desktop Folder:Chandra_31-07-2002:249:

10.

4m  11  8m  7
4m  11  4m  8m  7  4m
11  4m  7
11  7  4m  7  7
18  4m
18
4

13.

4m
4

0.14y
0.14

4.5  m
4.5  m is the same as m  4.5.
Check:
Substitute 4.5, a number less than
4.5, and a number greater than 4.5.
Let m  4.5.
?
4(4.52  11  8(4.52  7

0.3(16)  0.8(4  2)
?

4.8  0.8(6)
4.8  4.8
Let y  10.
?
0.3(10  4)  0.8[0.2(10)  2]

Let m  0.
?
4(02  11  8(02  7

0.3(6)  0.8(2  2)
?

1.8  0.8(4)
1.8  3.2
Let y  30.
?

0.3(30  4)  0.8[0.2(30)  2]
?

0.3(26)  0.8(6  2)

6
3

7.8  0.8(8)
7.8  6.4
The solution set is { y|y  20}.
14. Let p  the selling price of the house.

k 6 2
Check:
Substitute 2, a number less than 2,
and a number greater than 2.
Let k  2.
Let k  5.
?
?
3(2  2) 7 12
3(5  2) 7 12
?

The selling
price
minus 1
7%
times the
selling price 14243
is at least 14243
$110,000.
14243
123
23 123
1442443
p

0.07

p

110,000

3(4) 7 12
12  12
Let k  3.
?
3(3  2) 7 12

3(7) 7 12
21 7 12

p  0.07p  110,000
0.93p  110,000
0.93p
0.93

3(1) 7 12
3  12
The solution set is 5k|k 6 26.
f  5
3
f  5
(3) 3

6  r  3
6 r  6  3  6
r  3
r  3 means that the distance between r and
0 is 3 units. Since distance cannot be negative,
there is no real number that makes this
statement true. The solution set is the empty
set .
16. Write |d|
2 as d
2 or d 2.
The solution set is {d|d is a real number}.
15.

7 3
7 (3)(3)

7 9
7 9  5
7 4
Substitute 4, a number less than 4,
and a number greater than 4.
Let f  4.
Let f  10.
Let f  5.
?

7 3
?

7 3

10  5
3
15
3

7 3
?

7 3

3  3
5  3
The solution set is { f|f
 4}.

Chapter 6

110,000
0.93

The selling price must be at least $118,280.

f5
f55
f
Check:

4  5
3
9
3

p  118,280 1to the nearest dollar2

12.

Substitute 20, a number less than 20,


and a number greater than 20.

Let y  20.
?
0.3(20  4)  0.8[0.2(20)  2]

24  11  48  7
11 7
35  41
The solution set is {m m  4.5}.
3(k  2) 7 12
11.
3k  6 7 12
3k  6  6 7 12  6
3k 7 6
6

2.8

 0.14

y  20
Check:

18  11  36  7
29  29
Let m  6.
?
4(62  11  8(62  7

3k
3

0.3( y  4)  0.8(0.2y  2)
0.3y  1.2  0.16y  1.6
0.3y  1.2  0.16y  0.16y  1.6  0.16y
0.14y  1.2  1.6
0.14y  1.2  1.2  1.6  1.2
0.14y  2.8

5  5
3
0
3

7 3
?

7 3

0 7 3

300

PQ249-6481F-06[292-304] 31/7/02 21:39 Page 301 Sahuja Ahuja_QXP_09:Desktop Folder:Chandra_31-07-2002:249:

r3 7 2

17.

20. Write |2a  5| 6 7 as 2a  5 6 7


2a  5 7 7.
Case 1:
Case 2:
2a  5 6 7
2a  5
2a  5  5 6 7  5
2a  5  5
2a 6 12
2a

4r 6 12

and

4r
4

12
4

6
r33 7 23
r 6 3
r 7 1
The solution set is the intersection of the two
graphs.
Graph r
1.
Graph r 3.
Find the intersection.
5

2a
2

2a
2

12
2

3s
3

3n  2  1
3n  2  2  1  2
3n  3
3n
3

15
3

6
2

16
8

s3

3 21 0 1 2 3 4

5 6 7

22. Write |7  5z| 7 3 as 7  5z 7 3 or


7  5z 6 3.
Case 1:
Case 2:
7  5z 7 3
7  5z 6 3
7  5z  7 7 3  7
7  5z  7 6 3  7
5z 7 4
5z 6 10
5z
5

5z
5

4
5

23. Let n  the number.


1
n
4
1
(4) 4n

 3

 (4) (3)
n  12
Check:
Substitute 12, a number less than
12, and a number greater than 12.
Let n  12.
Let n  20.
Let n  4.

p 7 3
2 7 p
The solution set is the intersection of the two
graphs.
Graph p 7 3.
Graph p 6 2.
Find the intersection.

?
1
1122 
4

3

?
1
1202 
4

3

3  3
5 3
The solution set is {n|n  12}.

10
5

543 2 1 0 1 2 3 4 5

8p
8

z 6 0.8
z 7 2
The solution set is {z|z 6 0.8 or z 7 2}.

The solution set is {n|n  1 or n  5}.


19.
and
9  2p 7 3
13 7 8p  3
9  2p  9 7 3  9
13  3 7 8p  3  3
2p 7 6
16 7 8p
2p
2

9

 3

The solution set is s|s  13 or s  3 .

n5
n  1
The solution set is the union of the two graphs.
Graph n  5.
Graph n  1.
Find the union.
2

3s
3

5

 3

s  3 or 13

3
3

5 6 7

21. Write |7  3s|  2 as 7  3s  2 or 7  3s  2.


Case 1:
Case 2:
7  3s  2
7  3s  2
7  3s  7  2  7
7  3s  7  2  7
3s  5
3s  9

The solution set is {r|  1 r 3}.

3n
3

2
2

or
3n  2  17
3n  2  2  17  2
3n  15

7 7
7 7  5
7 2

a 6 6
a 7 1
The solution set is 5a|1 6 a 6 66.
3 21 0 1 2 3 4

18.

and

?
1
142 
4

3

1  3

The solution set is { p|3 6 p 6 2}.

301

Chapter 6

PQ249-6481F-06[292-304] 31/7/02 21:39 Page 302 Sahuja Ahuja_QXP_09:Desktop Folder:Chandra_31-07-2002:249:

24. Let n  the number.


14  3n 6 2
14  3n  14 6 2  14
3n 6 12
3n
3

The half plane that contains (0, 0) should be


shaded.
y

12
3

n 7 4
Substitute 4, a number less than 4,
and a number greater than 4.
Let n  4.
Let n  2.
Let n  6.
Check:

14  3(4) 6 2

14  3(2) 6 2

y  3x  2

14  3(6) 6 2

14  12 6 2
14  6 6 2
14  18 6 2
2 2
8 2
4 6 2
The solution set is 5n|n 7 46.
25. Let n  the number.
13 6 2n  5 6 21
First express 13 6 2n  5 6 21 using and.
and
13 6 2n  5
2n  5 6 21
13  5 6 2n  5  5
2n  5  5 6 21  5
18 6 2n
2n 6 26
18
2

2n
2

2n
2

28. Since the boundary is not included in the


solution, draw a dashed line.
Test the point (0, 0).
2x  3y 6 6
2(0)  3(0) 6 6
0 6 6 true
The half plane that contains (0, 0) should be
shaded.
y

26
2

9 6 n
n 6 13
The solution set is the intersection of 9 6 n and
n 6 13.
Check:
Substitute a number less than 9, 9, a
number between 9 and 13, 13, and a
number greater than 13.
Let n  2.
Let n  9.
?

13 6 2(2)  5 6 21
?

?
?

Let n  13.

13 6 2(13)  5 6 21
?

13 6 22  5 6 21
13 6 17 6 21

29. Since the boundary is not included in the


solution, draw a dashed line.
Test the point (0, 0).
x  2y 7 4
0  2102 7 4
0 7 4 false
The half plane that does not contain (0, 0) should
be shaded.

Let n  11.

13 6 2(11)  5 6 21

2x  3y  6

13 6 18  5 6 21
13 13 6 21

x
O

13 6 2(9)  5 6 21

13 6 4  5 6 21
13 1 6 21
?

13 6 26  5 6 21
13 6 21 21

Let n  15.
?

13 6 2(15)  5 6 21
?

13 6 30  5 6 21
13 6 25 21
The solution set is 5n|9 6 n 6 136.
26. Let m  the number of miles Megan drives.

x  2y  4

Then 15 represents the number of miles per gallon


Megans car gets.
If 18 6
18

m
6
15
m
6 15

21, then 18 6
and

115218 6 1152 15
m

m
15

and

m
15
m
1152 15

m
15

6 21.

30. B; Find the point that is the same distance from


3 as the distance from 7. The midpoint of 3
and 7 is 2.

6 21
6 115221

5 units

270 6 m
m 6 315
So, 270 6 m 6 315.
Megan can drive between 270 and 315 miles.
27. Since the boundary is included in the solution,
draw a solid line. Test the point (0, 0).
y  3x  2
0  3(0)  2
true
0  2
Chapter 6

5 units

All points on the graph are at least 5 units from 2.


So, an inequality is |x  2|  5.

302

PQ249-6481F-06[292-304] 31/7/02 21:39 Page 303 Sahuja Ahuja_QXP_09:Desktop Folder:Chandra_31-07-2002:249:

9. C; The graph is the intersection of x  2 and


x 6 3. The compound inequality
x  2 and x 6 3 can be expressed as
2  x 6 3.
10. There are 6 possible outcomes; 4 are successes
and 2 are failures.
So, the odds of rolling a number less than five are
4
2
or 1 or 2:1.
2

Chapter 6 Standardized Test Practice


Pages 364365
3

1. B; 9 7 3
1

3 7 3
2. D; (6) (7)  42
3. C;
V  r2h
5625  r2 1252
5625
25

11. Since 1 h  60 min, 54 miles per hour is the same


54 mi
as 54 miles per 60 minutes, or 60 min.
Let m  the number of minutes it will take to
travel 117 mi.

 r2

225  r2
15  r
The radius is 15 cm.
4. B; Let h  the number of hours worked.
80  65h  177.50
80  65h  80  177.50  80
65h  97.50
65h
65

54
60




54m
54

4(220  15)
5
412052

12
48

5
820
5

3

4
16

3

1
4


12

3



12

2
8

1200
48

The sum of the grades divided by


(78  82  75  x)

So, the inequality is

8
32

m

m  1
Step 2 You know the slope and two points.
Choose one point and find the y-intercept.
In this case, (2, 1) is used.
y  mx  b
1  1(2)  b
1  2  b
1  2  2  b  2
3b
Step 3 Write the slope-intercept form using
m  1 and b  3.
y  mx  b
y  1x  3
Therefore, the equation is y  x  3.

the number must be


of4244
grades
at least 80.
14
3 14243 123
4

1  4
(1)
3
3

m2



12
12

78  82  75  x
4

48r
48

y2  y1

The difference of x values is 3, and the difference


of y values is 12. The difference of y values is four
times the difference of x values. This suggests
y  4x.
Check:
If x  1, then y  4(1) or 4.
If x  8, then y  4(8) or 32.
Thus, y  4x describes this set of data.
8. B;
1444442444443 14
4244
3

mx x
2
1


5
20

 100

25  r
The percent of decrease is 25%.
13. Select two points on the line, for example (1, 4)
and (2, 1).
Step 1 Find the slope of the line containing the
points.
Let (x , y )  (1, 4) and (x , y )  (2, 1).
1 1
2 2

3

7020
54

12(100)  48(r)
1200  48r

 164
Camerons recommended maximum pulse rate is
164.
6. A; For the function y  2x  1, every ordered pair
that satisfies the equation can be written as
(x, 2x  1) . To translate the graph of this function
3 units up, add 3 to the y-coordinate of each point
on the graph.
P(x, 2x  1) S P(x, (2x  1)  3) S P(x, 2x  2)
So, the equation that represents the new line, or
image, is y  2x  2.
7. D;
x
y

m  130
It will take 130 min to travel 117 mi.
12. The percent of change is a percent of decrease
because the new amount is less than the original.
Find the change.
48  36  12
Find the percent using the original number, 48, as
the base.

97.50
65

117
m

54(m)  60(117)
54m  7020

h  1.5
The technician worked 1.5 h.
4(220  A)
5. B; P 
5
P

80

 80.

303

Chapter 6

PQ249-6481F-06[292-304] 31/7/02 21:39 Page 304 Sahuja Ahuja_QXP_09:Desktop Folder:Chandra_31-07-2002:249:

19. A; Since x 7 5 or x 6 7, the distance from 0 to x


is greater than 5 units.
Since 3 y 4, the distance from 0 to y is less
than 4 units.
A number |x| that is greater than 5 is greater
than a number |y| that is less than 4. So, the
quantity in Column A is greater.
20a.
10 ft

14. The line parallel to 3y  3x  1 has the same


slope. Rewrite the equation of the line in slopeintercept form, y  mx  b.
1
y
3

 3x  1

12

(3) 3y  (3) 3x  1

y  2x  3
The slope of the line is 2.
1

15. 2 (10x  8)  3(x  1)  15


5x  4  3x  3  15
2x  1  15
2x  1  1  15  1
2x  16
2x
2

16
2

10 ft

x8
The solution set is 5x|x  86.
16. Write |x  3| 7 5 as x  3 7 5 and x  3 6 5.
Case 1:
Case 2:
x3 7 5
x  3 6 5
x33 7 53
x  3  3 6 5  3
x 7 8
x 6 2
The solution set is {x|x 6 2 and x 7 8}.
17. Since the boundary is not included in the
solution, draw a dashed line.
Test the point (0, 0).
y 6 2x  4
0 6 2102  4
true
0 6 4
The half plane that contains (0, 0) should be shaded.

91 ft

Let /  the length of the house.


The length
cannot be the length decreased
the number
of the house greater than of the lot
by
10 ft times of edges.
1442443 1442443 1442443 1442443 123 123 1442443


91

10

So, the inequality for the possible lengths of the


house is /  91  20, or /  71. Therefore, the
length of the house is at most 71 ft.
20b. If A  the area of the house, then
2800  A  3200.
Since A  /w, then 2800  /w  3200.
If the house has the maximum possible length,
then /  71, so 2800  71w  3200.
First express 2800  71w  3200 using and.
Then solve each inequality.
and
2800  71w
71w  3200

y  2x  4

2800
71

18. B; Use a calculator to find an approximation for


168.
168  8.246211251...
Therefore, 168 6 9. So, the quantity in Column
B is greater.

Chapter 6

158 ft

10 ft

10 ft

71w
71

39  w

(to the nearest ft)

71w
71

3200
71

w  45

The solution set is 5w|39  w  456.


The width of the house can be between 39 and
45 ft, inclusive.

304

PQ249-6481F-07[305-325] 8/6/02 08:38 PM Page 305 Sahuja Ahuja_QXP_03:Desktop Folder:6/8/02:

Chapter 7
Page 367

Solving Systems of Linear Equations and Inequalities


Graph the ordered pairs and draw a line through
the points.

Getting Started

1. The only value in the range is 1. Select five


values for the domain and make a table.
x
3
1
0
2
4

y
y4x

(x, y)
(3, 1)
(1, 1)
(0, 1)
(2, 1)
(4, 1)

y
1
1
1
1
1

Graph the ordered pairs and draw a line through


the points.

4. To find the x-intercept, let y  0.


y  2x  3
0  2x  3
0  3  2x  3  3
3  2x

y
y1

3
2

2x
2(2)
2(1)
2(0)
2(1)
2(2)

y
4
2
0
2
4

2x
2

1.5  x
The graph intersects the x-axis at (1.5, 0).
To find the y-intercept, let x  0.
y  2x  3
y  2(0)  3
y3
The graph intersects the y-axis at (0, 3).
Plot these points and draw the line that connects
them.

2. Select five values for the domain and make a


table.
x
2
1
0
1
2

(x, y)
(2, 4)
(1, 2)
(0, 0)
(1, 2)
(2, 4)

Graph the ordered pairs and draw a line through


the points.

y  2x  3

y
y  2x

5. To find the x-intercept, let y  0.


y  5  2x
0  5  2x
0  2x  5  2x  2x
2x  5

2x
2

3. Select five values for the domain and make a


table.
x
1
0
2
4
5

4x
4  (1)
40
42
44
45

y
5
4
2
0
1

2

x  2.5
The graph intersects the x-axis at (2.5, 0).
To find the y-intercept, let x  0.
y  5  2x
y  5  2(0)
y5
The graph intersects the y-axis at (0, 5).

(x, y)
(1, 5)
(0, 4)
(2, 2)
(4, 0)
(5, 1)

305

Chapter 7

PQ249-6481F-07[305-325] 8/6/02 08:38 PM Page 306 Sahuja Ahuja_QXP_03:Desktop Folder:6/8/02:

Plot these points and draw the line that connects


them.

9.

7bc  d
10
7bc  d
(10) 10

 (10)12

7bc  d  120
7bc  d  d  120  d
7bc  120  d

y  5  2x

120  d
7c
120  d
b  7c
120  d
of b is 7c .

7bc
7c

 12

The value
Since division by 0 is
undefined, 7c  0 or c  0.
10.

6. To find the x-intercept, let y  0.


1

y  2x  2

7m  n
2m
7m  n
2m

0  2  2x  2  2
1

2  2x

y  2x  2
1

y  2 (0)  2
y2
The graph intersects the y-axis at (0, 2).
Plot these points and draw the line that connects
them.
y
1

4x  a  6x
4x  a  4x  6x  4x
a  2x
2x
2

x
a

The value of x is 2 .
8a  y  16
8a  y  y  16  y
8a  16  y

The

16  y
8
16  y
a 8
16  y
value of a is 8 .

Chapter 7

q

14. (8x  4y)  (8x  5y)  8x  4y  8x  5y


 8x  8x  4y  5y
 (8  8)x  (4  5)y
 0x  (1)y
y
15. 4(2x  3y)  (8x  y)
 4 # 2x  4 # 3y  8x  (y)
 8x  12y  8x  y
 8x  8x  12y  y
 (8  8)x  (12  1)y
 0x  13y
 13y
16. 3(x  4y)  (x  12y)  3  x  3  4y  x  12y
 3x  12y  x  12y
 3x  x  12y  12y
 (3  1)x  (12  12)y
 4x  0y
 4x
17. 2(x  2y)  (3x  4y)  2  x  2  2y  3x  4y
 2x  4y  3x  4y
 2x  3x  4y  4y
 (2  3)x  (4  4)y
 5x  0y
 5x

8a
8

2qm
2m

 16x  11x  3y  3y
 (16  11)x  (3  3)y
 27x  0y
 27x

y 2x2

8.

7m  n

4  x
The graph intersects the x-axis at (4, 0).
To find the y-intercept, let x  0.

 (q)2m

The value of q is 2m . Since division by 0 is


undefined, 2m  0 or m  0.
11. (3x  y)  (2x  y)  3x  y  2x  y
 3x  2x  y  y
 (3  2)x  (1  1)y
 1x  0y
x
12. (7x  2y)  (7x  4y)  7x  2y  7x  4y
 7x  7x  2y  4y
 (7  7)x  (2  4)y
 0x  (6)y
 6y
13. (16x  3y)  (11x  3y)  16x  3y  11x  3y

(2)(2)  (2) 2x

a
2
a
2

 2m

7m  n  2qm

0  2x  2

7.

7m  n
q
7m  n
(q) q

306

PQ249-6481F-07[305-325] 8/6/02 08:39 PM Page 307 Sahuja Ahuja_QXP_03:Desktop Folder:6/8/02:

18. 5(2x  y)  2(5x  3y)


 5  2x  5  y  2  5x  2  3y
 10x  5y  10x  6y
 10x  10x  5y  6y
 (10  10)x  (5  6)y
 0x  (11)y
 11y
19. 3(x  4y)  2(2x  6y)
 3  x  3  4y  2  2x  2  (6y)
 3x  12y  4x  (12)y
 3x  4x  12y  12y
 (3  4)x  (12  12)y
 7x  0y
 7x

Page 368

7-1

Pages 371372

y
(2, 3)

True or
False?

y  375  0.15x

The system of equations has one solution at


(2, 3) since the graphs intersect at (2, 3).
2. Always; if a system of linear equations has 2
solutions, the graphs are the same line and there
are infinitely many solutions.
3. Sample answer: The graphs of the equations
x  y  3 and 2x  2y  6 have a slope of 1.
Since the graphs of the equations coincide, there
are infinitely many solutions.

True or
False?

true

410  375  0.15(100)


410  390

false
true

100

410 410  400  0.1(100)


410  410

300

420 420  400  0.1(300)


420  430

false

420  375  0.15(300)


420  420

500

450 450  400  0.1(500)


450  450

true

450  375  0.15(500)


450  450

true

900

510 900  400  0.1(900)


900  490

false

510  375  0.15(900)


510  510

true

4. Since the graphs of y  x  4 and y  3 x  2


intersect, there is one solution.
1

Job Salaries

Series 1

450
400
350

y  2x

(0, 0)
Series 2

0
20
0
40
0
60
0
80
0
10
00

Total Weekly Salary

7. Since the graphs of x  y  4 and y  3x  4


intersect, there is one solution.
8.
y

550
500

5. Since the graphs of y  3 x  2 and y  3 x  2 are


parallel, there are no solutions.
6. Since the graphs of x  y  4 and y  x  4
coincide, there are infinitely many solutions.

Since only the ordered pair (500, 450) makes both


statements true, the correct choice is c.
4.

Spreadsheet Investigation
(Preview of Lesson 7-1)

y  400  0.1x

Check for Understanding

1. Sample answer:

1. y  400  0.1x
2. y  375  0.15x
3. Make a table. Substitute each ordered pair into
both equations.
x

Graphing Systems of Equations

y  x

Weekly Sales

The two lines intersect at (500, 450). If


Mr. Winter makes $500 in sales, he will make
$450 for either job.
5. Sample answer: Write and graph two linear
equations. Find the point where the graphs
intersect.

The graphs appear to intersect at the point with


coordinates (0, 0). Check this estimate by
replacing x with 0 and y with 0 in each equation.
Check:
y  x
y  2x
?
?
0  2(0)
0  0
00
00
There is one solution. It is (0, 0).

307

Chapter 7

PQ249-6481F-07[305-325] 8/6/02 08:39 PM Page 308 Sahuja Ahuja_QXP_03:Desktop Folder:6/8/02:

13.

9. y

xy8

xy2

xy2

(1, 3)

(5, 3)

y  4x  7
x

The graphs appear to intersect at the point with


coordinates (5, 3). Check this estimate by
replacing x with 5 and y with 3 in each equation.
Check:
xy8
xy2
?
?
538
532
88
22
There is one solution. It is (5, 3).
10.

The graphs appear to intersect at the point with


coordinates (1, 3). Check this estimate by
replacing x with 1 and y with 3 in each
equation.
Check:
xy2
y  4x  7
?
?
3  4(1)  7
1  3  2
?
22
3  4  7
33
There is one solution. It is (1, 3).
14. Let a  the price for an adult, and let
c  the price for a child.
Rodriguez family: 2a  3c  40.50
Wong family:
3a  c  38.00
Graph the equations 2a  3c  40.50 and
3a  c  38.

y
2x  4y  2

O
3x  6y  3

11.

Price of Child Ticket

The graphs of the equations coincide. Since every


point is a point of intersection, there are infinitely
many solutions to this system of equations.
y
xy4

3y  2x  9

(10.5, 6.5)
2a  3c  40.50

The graphs appear to intersect at the point with


coordinates (10.5, 6.5). Check this estimate by
replacing a with 10.5 and c with 6.5 in each
equation.
Check:
2a  3c  40.50
3a  c  38.00
?
?
2(10.5)  3(6.5)  40.50 3(10.5)  6.5  38.00
?
?
31.5  6.5  38.00
21  19.5  40.50
40.5  40.50
38  38.00
The price for an adult was $10.50, and the price
for a child was $6.50.

The graphs of the equations are parallel lines.


Since they do not intersect, there are no solutions
to this system of equations.
12.

3a  c  38

0 4 8 12 16 20 24 28 32 36 a
Price of Adult Ticket

O
xy1

c
36
32
28
24
20
16
12
8
4

(3, 1)
x

xy2

Pages 372374

The graphs appear to intersect at the point with


coordinates (3, 1). Check this estimate by
replacing x with 3 and y with 1 in each equation.
Check:
xy2
3y  2x  9
?
?
312
3(1)  2(3)  9
?
22
369
99
There is one solution. It is (3, 1).
Chapter 7

Practice and Apply

15. Since the graphs of x  3 and y  2x  1


intersect, there is one solution.
16. Since the graphs of y  x  2 and y  2x  4
intersect, there is one solution.
17. Since the graphs of y  x  2 and y  x  2
coincide, there are infinitely many solutions.
18. Since the graphs of y  2x  1 and y  2x  4 are
parallel, there are no solutions.

308

PQ249-6481F-07[305-325] 8/6/02 08:40 PM Page 309 Sahuja Ahuja_QXP_03:Desktop Folder:6/8/02:

19. Since the graphs of y  3x  6 and y  2x  4


intersect, there is one solution.
20. Since the graphs of 2y  4x  2 and y  2x  4
are parallel, there are no solutions.
21. Since the graphs of 2y  4x  2 and y  3x  6
intersect, there is one solution.
22. Since the graphs of 2y  4x  2 and y  2x  1
coincide, there are infinitely many solutions.
23.
y

26.

x
(2, 2)

y  x

The graphs appear to intersect at (2, 2). Check


in each equation.
y  x
y  2x  6
Check:
?
?
2  (2)
2  2(2)  6
?
2  2
2  4  6
2  2
There is one solution. It is (2, 2).

4x  y  2

y  6
(2, 6)

27.

The graphs appear to intersect at (2, 6). Check


in each equation.
y  6
4x  y  2
Check:
?
6  6
4(2)  (6)  2
?
8  (6)  2
22
There is one solution. It is (2, 6).
24.

y  2x  6

y  3x  4
y  3x  4 (0, 4)

The graphs appear to intersect at (0, 4). Check


in each equation.
y  3x  4
y  3x  4
Check:
?
?
4  3(0)  4
4  3(0)  4
4  4
4  4
There is one solution. It is (0, 4).

x2
3x  y  8

O
(2, 2)

28.
The graphs appear to intersect at (2, 2). Check
in each equation.
x2
3x  y  8
Check:
?
22
3(2)  (2)  8
?
628
88
There is one solution. It is (2, 2).
25.

y  2x  6
(3, 0)

y
(4, 2)

The graphs appear to intersect at (3, 0). Check


in each equation.
y  2x  6
y  x  3
Check:
?
?
0  (3)  3
0  2(3)  6
?
?
0  6  6
033
00
00
There is one solution. It is (3, 0).

y  1x
2

y
y  x  3

2x  y  10

The graphs appear to intersect at (4, 2). Check in


each equation.
Check:

y  2x
2

? 1
 2 (4)

22

2x  y  10
?

2(4)  2  10
?

8  2  10
10  10

There is one solution. It is (4, 2).

309

Chapter 7

PQ249-6481F-07[305-325] 8/6/02 08:42 PM Page 310 Sahuja Ahuja_QXP_03:Desktop Folder:6/8/02:

29.

33.

16

3x  y  6

12

(2, 0 )

5x  3y  6
(6, 8)

4
2x  y  4

x  2y  2

16 12 8

The graphs appear to intersect at (2, 0). Check in


each equation.
Check:
x  2y  2
3x  y  6
?
?
2  2(0)  2
3(2)  0  6
22
66
There is one solution. It is (2, 0).
30.

xy 2

O x

4

The graphs appear to intersect at (6, 8). Check


in each equation.
Check:
2x  y  4
5x  3y  6
?
?
2(6)  8  4
5(6)  3(8)  6
?
?
12  8  4
30  24  6
4  4
6  6
There is one solution. It is (6, 8).

34.

5x  8y  17

(2, 4)
(3, 4)

2y  x  10

x
O
O

The graphs appear to intersect at (2, 4). Check


in each equation.
Check:
xy2
2y  x  10
?
?
2  4  2
2(4)  (2)  10
?
22
8  2  10
10  10
There is one solution. It is (2, 4).
31.

35.

3x  2y  12

y
2y  6x  6

3x  2y  6

3x  y  3

The graphs of the equations are parallel lines.


Since they do not intersect, there are no solutions
to this system of equations.
y

2x  3y  4
O

The graphs of the equations coincide. Since every


point is a point of intersection, there are infinitely
many solutions to this system of equations.

4x  6y  8

The graphs of the equations coincide. Since every


point is a point of intersection, there are infinitely
many solutions to this system of equations.

Chapter 7

The graphs appear to intersect at (3, 4). Check in


each equation.
Check:
4x  3y  24
5x  8y  17
?
?
4(3)  3(4)  24
5(3)  8(4)  17
?
?
12  12  24
15  32  17
24  24
17  17
There is one solution. It is (3, 4).

32.

4x  3y  24

310

PQ249-6481F-07[305-325] 8/6/02 08:42 PM Page 311 Sahuja Ahuja_QXP_03:Desktop Folder:6/8/02:

36.

40.

12
10
8
6
4
2

3y  x  5

yx3

(1, 2)

2
2
4

The graphs appear to intersect at (1, 2). Check


in each equation.
Check:
yx3
3y  x  5
?
?
2  1  3
3(2)  (1)  5
?
22
615
55
There is one solution. It is (1, 2).
37.

x
O
2 4 6 8 10 12 14
1
1
x  3y  6
2

1
x
2

 3y  6
?

 3 (6)  6
?

426
66
There is one solution. It is (8, 6).

y  2x  2
? 1

6  2 (8)  2
?

642
66

41. Let   the length of the rectangle, and let


w  the width of the rectangle.
/  2w  1
2/  2w  40
Graph the equations /  2w  1 and
2/  2w  40.

yx4

Length (m)

The graphs appear to intersect at (1, 5).


Check in each equation.
Check:
2x  3y  17
yx4
?
?
2(1)  3(5)  17
5  1  4
?
2  (15)  17
5  5
17  17
There is one solution. It is (1, 5).
y

l
18
16
14
12
10
8
6
4
2
0

(7, 13)
2l  2w  40

l  2w  1
0 2 4 6 8 10 12 14 16 18 w
Width (m)

The graphs appear to intersect at (7, 13). Check


in each equation.
/  2w  1
2/  2w  40
Check:
?
?
2(13)  2(7)  40
13  2(7)  1
?
?
13  14  1
26  14  40
13  13
40  40
The length of the rectangle is 13 m, and the width
of the rectangle is 7 m.

(8, 6)

1
(8)
2

38.

Check:

(1, 5)

y  1x  2

The graphs appear to intersect at (8, 6). Check in


each equation.

y
2x  3y  17

3y  2x
2

y 3x5

The graphs of the equations are parallel lines.


Since they do not intersect, there are no solutions
to this system of equations.
39.
16
14
12
10
8
6
4
2
2

y
3

6  8y  x
2
1
x  4y  4
3

x
O
2 4 6 8 10 12 14

The graphs of the equations coincide. Since every


point is a point of intersection, there are infinitely
many solutions to this system of equations.

311

Chapter 7

PQ249-6481F-07[305-325] 8/6/02 08:43 PM Page 312 Sahuja Ahuja_QXP_03:Desktop Folder:6/8/02:

42. Graph the equations y  2x  6, 3x  2y  19,


and y  2.

3x  2y  19

y2

y
y  2x  6

10
8
6

Height (m)

14

Graph the equations h  10  15m and


h  150  20m.

(1, 8)

(2, 2)

(5, 2)
O

8 6 4 22

2 4 6 8x

The graphs of y  2x  6 and 3x  2y  19


appear to intersect at (1, 8). Check in each
equation.
Check:
y  2x  6
3x  2y  19
?
?
8  2(1)  6
3(1)  2(8)  19
?
?
826
3  16  19
88
19  19
The graphs of y  2x  6 and y  2 appear to
intersect at (2, 2). Check in each equation.
Check:
y2
y  2x  6
?
22
2  2(2)  6
?
2  4  6
22
The graphs of 3x  2y  19 and y  2 appear to
intersect at (5, 2). Check in each equation.
y2
3x  2y  19
Check:
?
22
3(5)  2(2)  19
?
15  4  19
19  19
Therefore, the coordinates of the vertices of the
triangle are (1, 8), (2, 2), and (5, 2).
43. The base of the triangle from Exercise 42 is the
segment joining the points with coordinates (2, 2)
and (5, 2). The length of this segment is 7 units,
so b  7. The height of the triangle is 6 units, the
length of the segment joining the points with
coordinates (1, 2) and (1, 8). Thus, h  6.
Substitute b  7 and h  6 in the formula for the

h  150  20m
h  10  15m
(4, 70)

0 1 2 3 4 5 6 7 8 9m
Minutes

Total Amount Saved

The graphs appear to intersect at (4, 70). Check


in each equation.
Check:
h  10  15m
h  150  20m
?
?
70  10  15(4)
70  150  20(4)
?
?
70  150  80
70  10  60
70  70
70  70
The balloons will be the same height in 4 min.
45. Substitute m  4 in each equation from
Exercise 44.
h  10  15m
h  150  20m
h  10  15(4)
h  150  20(4)
h  10  60
h  150  80
h  70
h  70
In 4 min, both balloons will be 70 m high.
46. Let a  the total amount saved, and let
w  the number of weeks.
Monica: a  25  5w
Michael: a  16  8w
Graph the equations a  25  5w and
a  16  8w.

area of a triangle, A  2bh.


1

A  2bh

a
45
40
35
30
25
20
15
10
5
0

a  25  5w
(3, 40)

a  16  8w

A  276
A  21
So, the area of the triangle is 21 units2.
44. Let h  the height of the balloon in meters, and
let m  the number of minutes.
Balloon 1: h  10  15m
Balloon 2: h  150  20m

Chapter 7

h
180
160
140
120
100
80
60
40
20
0

2
3
4
Number of Weeks

The graphs appear to intersect at the point with


coordinates (3, 40). Check this estimate by
replacing w with 3 and a with 40 in each
equation.
Check:
a  25  5w
a  16  8w
?
?
40  25  5(3)
40  16  8(3)
?
?
40  16  24
40  25  15
40  40
40  40
Monica and Michael will have saved the same
amount in 3 weeks.

312

PQ249-6481F-07[305-325] 8/6/02 08:43 PM Page 313 Sahuja Ahuja_QXP_03:Desktop Folder:6/8/02:

Substitute A  4 into the equation


2A  3B  20, and solve for B.
2A  3B  20
2(4)  3B  20
8  3B  20
8  3B  8  20  8
3B  12

Population
(millions of people)

47. Substitute w  3 in each equation from


Exercise 46.
a  25  5w
a  16  8w
a  25  5(3)
a  16  8(3)
a  25  15
a  16  24
a  40
a  40
Monica and Michael will each have saved $40.
48. For each of the years shown, the profit of the
Widget Company is greater than the profit of the
Gadget Company. For example, the profit of the
Widget Company in year 1 appears to be
$6,000,000. This is greater than the profit of the
Gadget Company in year 1, which appears to be
$3,000,000.
49. Since the slopes of the graphs for both companies
appear to be the same, the rates of growth appear
to be the same. Thus, neither company has a
greater rate of growth.
50. The graphs are parallel, so the lines will never
meet. Thus, there is no year when the profits of
the two companies will be equal if the profit
patterns continue.
51. Let t  the number of years since 1990, and let
p  the population of the Midwest in millions of
people.
p  60  0.4t
52. Let t  the number of years since 1990, and let
p  the population of the West in millions of
people.
p  53  1t or p  53  t
53.
p
80

3B
3

B  4
So, A  4 and B  4.
56. Graphs can show when the sales of one item are
greater than the sales of the other item and when
the sales of the items are equal. Answers should
include the following.
The sales of cassette singles equaled the sales
of CD singles in about 5 years or by the end of
1996.
The graph of each equation contains all of the
points whose coordinates satisfy the equation.
If a point is contained in both lines, then its
coordinates satisfy both equations.
1

57. B; Since the graphs of y  3 x  2 and


1
y  3 x  1 are parallel, the system of equations
has no solution.
58. B; Since the graphs of 4x  y  7 and 3x  y  0
intersect, there is one solution.
y

3x  y  0

4x  y  7

p  60  0.4t

60

12

 3

(1, 3)

(11.7, 64.7)

p  53  t

40

20

t
O

4
6
8
10
Years Since 1990

Page 374

12

54. The graphs appear to intersect at (11.7, 64.7).


Check in each equation.
Check:
p  60  0.4t
p  53  t
?
?
64.7  60  0.4(11.7) 64.7  53  11.7
?
64.7  60  4.68
64.7  64.7
?
64.7  64.68
64.7  64.7 (to the nearest tenth)
Since t  11.7, the populations will be the same
about 11.7 yr after 1990, or sometime in 2001.
55. Since (2, 3) is the solution of the system of
equations, replace x with 2 and y with 3 in each
equation.
Ax  y  5
Ax  By  20
A(2)  (3)  5
A(2)  B(3)  20
2A  3  5
2A  3B  20
2A  3  3  5  3
2A  8
2A
2

Maintain Your Skills

59. Use a table to substitute the x and y values of


each ordered pair into the inequality.
x
1

y
4

1

6

7

y  2x
4  2(1)
42
5  2(1)
5  2
6  2(5)
6  10
0  2(7)
0  14

True or False
false
false
true
false

The ordered pair {(5, 6)} is part of the solution


set of y  2x.

2

A4

313

Chapter 7

PQ249-6481F-07[305-325] 8/6/02 08:43 PM Page 314 Sahuja Ahuja_QXP_03:Desktop Folder:6/8/02:

60. Use a table to substitute the x and y values of


each ordered pair into the inequality.
x
4

y
2

3

y  8  3x
2  8  3(4)
2  20
0  8  3(3)
0  17
4  8  3(1)
45
8  8  3(1)
85

66.

True or False
true

4a
4

true

true

67.

false

7m  n
q
7m  n
(q) q

 10
 (q)10

7m  n  10q
7m  n
10
7m  n
10

10q
10

q

The value of q is
68.

5tz  s
2
5tz  s
(2) 2

7m  n
.
10

6
 (2)6

5tz  s  12
5tz  s  s  12  s
5tz  12  s
12  s
5t
12  s
z  5t
12  s
value of z is 5t .
5tz
5t

The
Since division by 0 is
undefined, 5t  0 or t  0.

Page 375

Graphing Calculator Investigation


(Follow-Up of Lesson 7-1)

1. Step 1 Each of the equations is already solved


for y.
y  3x  4
y  0.5x  6
Step 2 Enter these equations in the Y  list
and graph.
Step 3 Use the CALC menu to find the point of
intersection.
KEYSTROKES: 2nd [CALC] 5 ENTER

113 2 (x  3)

3y  6  x  3
3y  6  3  x  3  3
3y  3  x
3y  3  3y  x  3y
3  x  3y
The standard form of the equation is x  3y  3.
64.
y  4  6 (x  2)
y  4  6x  12
y  4  4  6x  12  4
y  6x  8
y  6x  6x  8  6x
6x  y  8
The standard form of the equation is 6x  y  8.
65.
12x  y  10x
12x  y  10x  10x  10x
2x  y  0
2x  y  y  0  y
2x  y
The value of y is 2x.
Chapter 7

The value of a is 4 or 4 b.

y  2  3 (x  3)
3( y  2)  3

 4

a  4

The ordered pairs {(4, 2), (3, 0), (1, 4)} are part
of the solution set of y  8  3x.
61. Let n  the number of ounces of perfume in a bottle.
The difference between the actual number and
2 is less than 0.05.
|n  2| 6 0.05
Write |n  2| 6 0.05 as n  2 6 0.05 and
n  2 7 0.05.
Case 1:
n  2 6 0.05
n  2  2 6 0.05  2
n 6 2.05
Case 2:
n  2 7 0.05
n  2  2 7 0.05  2
n 7 1.95
The solution set is {n|1.95  n  2.05}.
A bottle is accepted if it contains between 1.95
and 2.05 oz of perfume.
62.
y  1  4(x  5)
y  1  4x  20
y  1  1  4x  20  1
y  4x  19
y  4x  4x  19  4x
4x  y  19
(1)(4x  y)  (1) (19)
4x  y  19
The standard form of the equation is 4x  y  19.
63.

6a  b  2a
6a  b  2a  2a  2a
4a  b  0
4a  b  b  0  b
4a  b

ENTER ENTER
The solution is approximately (2.86, 4.57).
2. Step 1 Each of the equations is already solved
for y.
y  2x  5
y  0.2x  4
See Steps 2 and 3 in Exercise 1.
The solution is approximately (4.09, 3.18).

314

PQ249-6481F-07[305-325] 8/6/02 08:43 PM Page 315 Sahuja Ahuja_QXP_03:Desktop Folder:6/8/02:

8. Step 1 Solve each equation for y to enter them


into the calculator.
2x  3y  11
4x  y  6
2x  3y  2x  11  2x 4x  y  4x  6  4x
3y  11  2x
y  6  4x

3. Step 1 Solve each equation for y to enter them


into the calculator.
x  y  5.35
x  y  x  5.35  x
y  5.35  x
3x  y  3.75
3x  y  3x  3.75  3x
y  3.75  3x
(1)(y)  (1)(3.75  3x)
y  3.75  3x
See Steps 2 and 3 in Exercise 1.
The solution is approximately (2.28, 3.08).
4. Step 1 Solve each equation for y to enter them
into the calculator.
0.35x  y  1.12
0.35x  y  0.35x  1.12  0.35x
y  1.12  0.35x
(1)(y)  (1)(1.12  0.35x)
y  1.12  0.35x
2.25x  y  4.05
2.25x  y  2.25x  4.05  2.25x
y  4.05  2.25x
See Steps 2 and 3 in Exercise 1.
The solution is approximately (1.13, 1.51).
5. Step 1 Solve each equation for y to enter them
into the calculator.
1.5x  y  6.7
1.5x  y  1.5x  6.7  1.5x
y  6.7  1.5x
5.2x  y  4.1
5.2x  y  5.2x  4.1  5.2x
y  4.1  5.2x
(1)(y)  (1)(4.1  5.2x)
y  4.1  5.2x
See Steps 2 and 3 in Exercise 1.
The solution is approximately (1.61, 4.28).
6. Step 1 Solve each equation for y to enter them
into the calculator.
5.4x  y  1.8
5.4x  y  5.4x  1.8  5.4x
y  1.8  5.4x
(1)(y)  (1)(1.8  5.4x)
y  1.8  5.4x
6.2x  y  3.8
6.2x  y  6.2x  3.8  6.2x
y  3.8  6.2x
See Steps 2 and 3 in Exercise 1.
The solution is approximately (0.17, 2.73).
7. Step 1 Solve each equation for y to enter them
into the calculator.
5x  4y  26
4x  2y  53.3
5x  4y  5x  26  5x
4x  2y  4x  53.3  4x
4y  26  5x
2y  53.3  4x
4y
4

y

26  5x
4
26
5x
4  4

y  6.5  1.25x
See Steps 2 and 3 in Exercise 1.
The solution is (10.2, 6.25).

2y
2

y

3y
3

y

11  2x
3
11
2
 3x
3

See Steps 2 and 3 in Exercise 1.


The solution is (2.9, 5.6).
9. Step 1 Solve each equation for y to enter them
into the calculator.
0.22x  0.15y  0.30
0.22x  0.15y  0.22x  0.30  0.22x
0.15y  0.30  0.22x
0.15y
0.15

y
y

0.30  0.22x
0.15
0.30
0.22x
 0.15
0.15
22
2  15x

0.33x  y  6.22
0.33x  y  0.33x  6.22  0.33x
y  6.22  0.33x
See Steps 2 and 3 in Exercise 1.
The solution is approximately (2.35, 5.44).
10. Step 1 Solve each equation for y to enter them
into the calculator.
125x  200y  800
125x  200y  125x  800  125x
200y  800  125x
200y
200

y

800  125x
200
800
125x
200  200

y  4  0.625x
65x  20y  140
65x  20y  65x  140  65x
20y  140  65x
20y
20

y

140  65x
20
140
65x
 20  20

y  7  3.25x
See Steps 2 and 3 in Exercise 1.
The solution is approximately (1.14, 3.29).

53.3  4x
2
53.3
4x
 2
2

y  26.65  2x

315

Chapter 7

PQ249-6481F-07[305-325] 8/6/02 08:43 PM Page 316 Sahuja Ahuja_QXP_03:Desktop Folder:6/8/02:

Page 379

Substitution

7-2
Page 376

Algebra Activity

1. Step 1 Model the equation. Since y  x  4, use


1 positive x tile and 4 negative 1 tiles to
represent y.

3x

1 1

1 1

10y
10

3x  y  8

1 1

1 1

1 1

1 1

Separate the tiles into 4 groups.

4x
4

x3

3.
4.

5.

So, the value of x is 3.


Replace x with 3 in the equation and solve for y.
yx4
y34
y  1
The value of y is 1.
Since x  3 and y  1, the solution of the
system of equations is the ordered pair (3, 1).
On an equation mat, use algebra tiles to model
4x  3y  10 using 1 positive x tile and 1 positive
1 tile to represent each y. Use what you know
about equation mats to solve for x. Use the value of
x and y  x  1 to solve for y. The solution is (1, 2).
This method is called substitution since you
substitute a representation of y for y.

Chapter 7

12
4

x3
Substitute 3 for x in either equation to find the
value of y.
y4x
y43
y1
The solution is (3, 1).
6. Since x  1  4y, substitute 1  4y for x in the
first equation.
2x  7y  3
2(1  4y)  7y  3
2  8y  7y  3
2y3
2y232
y  1
(1) (y)  (1)1
y  1
Substitute 1 for y in either equation to find the
value of x.
x  1  4y
x  1  4(1)
x14
x5
The solution is (5, 1).

2.

or 1.5

Use x  2y to find the value of x.


x  2y
x  2(1.5)
x3
The solution is (3, 1.5).
5. Since y  3x  8, substitute 3x  8 for y in the
second equation.
y4x
3x  8  4  x
3x  8  8  4  x  8
3x  12  x
3x  x  12  x  x
4x  12

4x  12

Step 3

15

 10

y2

Step 2 Add 4 positive 1 tiles to each side to


isolate the x tiles. Remove zero pairs.

Check for Understanding

1. Substitution may result in a more accurate


solution than graphing.
2. Since the statement 4  2 is false, there are no
solutions of the system of equations. This means
that the graphs do not intersect, so they are
graphs of parallel lines.
3. Sample answer: y  x  3 and 2y  2x  6
4. Since x  2y, substitute 2y for x in the second
equation.
4x  2y  15
4(2y)  2y  15
8y  2y  15
10y  15

316

PQ249-6481F-07[305-325] 8/6/02 08:43 PM Page 317 Sahuja Ahuja_QXP_03:Desktop Folder:6/8/02:

Since both vehicles travel the same distance, t  c.


Use substitution to solve this system.

7. Since y  3x  2, substitute 3x  2 for y in the


first equation.
6x  2y  4
6x  2(3x  2)  4
6x  6x  4  4
4  4
The statement 4  4 is true. This means that
there are infinitely many solutions of the system
of equations.
8. Solve the second equation for x since the
coefficient of x is 1.
xy8
xyy8y
x8y
Find the value of y by substituting 8  y for x in
the first equation.
x  3y  12
(8  y)  3y  12
8  4y  12
8  4y  8  12  8
4y  4
4y
4

c
200
c
200
c
(152,600) 200

563c
563

17x
17

13 2

3x  5 5x  15
3x  3x  15
0  15
The statement 0  15 is false. This means that
there is no solution of the system of equations.
10. Let t  the distance traveled by the Thrust SSC
in miles, and let c  the distance traveled by the
car in miles. Use a table to organize the
information.

763

t
763

Car

200

c
200

11y
11

76,300
563

Practice and Apply

34

 17

44

 11

y4
Use x  4y to find the value of x.
x  4y
x  4(4)
x  16
The solution is (16, 4).
13. Since x  4y  5, substitute 4y  5 for x in the
second equation.
x  3y  2
4y  5  3y  2
4y  5  5  3y  2  5
4y  3y  7
4y  3y  3y  7  3y
y  7
Use x  4y  5 to find the value of x.
x  4y  5
x  4(7)  5
x  28  5
x  23
The solution is (23, 7).

Since the car had a head start of one-half hour,


c
200

x2
Use y  5x to find the value of y.
y  5x
y  5(2)
y  10
The solution is (2, 10).
12. Since x  4y, substitute 4y for x in the second
equation.
2x  3y  44
2(4y)  3y  44
8y  3y  44
11y  44

Thrust SSC

1 763c  12 2

11. Since y  5x, substitute 5x for y in the second


equation.
2x  3y  34
2x  3(5x)  34
2x  15x  34
17x  34

9. Since y  5 x, substitute 5 x for y in the second


equation.
3x  5y  15

Distance

 (152,600)

Pages 379381

y1
Substitute 1 for y in either equation to find the
value of x.
xy8
x18
x1181
x9
The solution is (9, 1).

Time

 763  2

c  135.5 (to the nearest tenth)


The Thrust SSC will pass the car at about
135.5 mi.

Rate

763c  200c  76,300


763c  200c  200c  76,300  200c
563c  76,300

4

 763  2

 763  2.

317

Chapter 7

PQ249-6481F-07[305-325] 8/6/02 08:43 PM Page 318 Sahuja Ahuja_QXP_03:Desktop Folder:6/8/02:

14. Since y  2x  3, substitute 2x  3 for y in the


second equation.
y  4x  1
2x  3  4x  1
2x  3  3  4x  1  3
2x  4x  4
2x  4x  4x  4  4x
2x  4
2x
2

18. Solve the second equation for y since the


coefficient of y is 1.
3x  y  9
3x  y  3x  9  3x
y  9  3x
Find the value of x by substituting 9  3x for y
in the first equation.
2x  y  4
2x  (9  3x)  4
2x  9  3x  4
9  x  4
9  x  9  4  9
x  13
(1) (x)  (1) (13)
x  13
Substitute 13 for x in either equation to find the
value of y.
3x  y  9
3(13)  y  9
39  y  9
39  y  39  9  39
y  30
The solution is (13, 30).
19. Solve the second equation for x since the
coefficient of x is 1.
x  3y  1
x  3y  3y  1  3y
x  1  3y
Find the value of y by substituting 1  3y for x in
the first equation.
3x  5y  11
3(1  3y)  5y  11
3  9y  5y  11
3  4y  11
3  4y  3  11  3
4y  8

4

 2

x2
Use y  2x  3 to find the value of y.
y  2x  3
y  2(2)  3
y43
y7
The solution is (2, 7).
15. Since c  d  1, substitute d  1 for c in the first
equation.
4c  3d  3
4(d  1)  3d  3
4d  4  3d  3
4d  4  4  3d  3  4
4d  3d  7
4d  3d  3d  7  3d
d7
Use c  d  1 to find the value of c.
cd1
c71
c6
The solution is (6, 7).
16. Since y  3x  13, substitute 3x  13 for y in the
first equation.
4x  5y  11
4x  5(3x  13)  11
4x  15x  65  11
19x  65  11
19x  65  65  11  65
19x  76
19x
19

4y
4

y2
Substitute 2 for y in either equation to find the
value of x.
x  3y  1
x  3(2)  1
x61
x6616
x7
The solution is (7, 2).

76

 19

x4
Use y  3x  13 to find the value of y.
y  3x  13
y  3(4)  13
y  12  13
y  1
The solution is (4, 1).
17. Solve the second equation for y since the
coefficient of y is l.
4x  y  11
4x  y  4x  11  4x
y  11  4x
Find the value of x by substituting 11  4x for y
in the first equation.
8x  2y  13
8x  2(11  4x)  13
8x  22  8x  13
22  13
The statement 22  13 is false. This means that
there is no solution of this system of equations.
Chapter 7

4

318

PQ249-6481F-07[305-325] 8/6/02 08:43 PM Page 319 Sahuja Ahuja_QXP_03:Desktop Folder:6/8/02:

Find the value of r by substituting 5  5r for s


in the second equation.
4r  5s  17
4r  5(5  5r)  17
4r  25  25r  17
21r  25  17
21r  25  25  17  25
21r  42

20. Solve the second equation for y since the


coefficient of y is 1.
3x  y  15
3x  y  3x  15  3x
y  15  3x
Find the value of x by substituting 15  3x for y
in the first equation.
2x  3y  1
2x  3(15  3x)  1
2x  45  9x  1
11x  45  1
11x  45  45  1  45
11x  44
11x
11

21r
21

r2
Substitute 2 for r in either equation to find the
value of s.
5r  s  5
5(2)  s  5
10  s  5
10  s  10  5  10
s  5
(1)(s)  (1) (5)
s5
The solution is (2, 5).
23. Solve the second equation for x since the
coefficient of x is 1.
x  2y  6
x  2y  2y  6  2y
x  6  2y
Find the value of y by substituting 6  2y for x in
the first equation.
3x  2y  12
3(6  2y)  2y  12
18  6y  2y  12
18  8y  12
18  8y  18  12  18
8y  6

44
11

x  4
Substitute 4 for x in either equation to find the
value of y.
3x  y  15
3(4)  y  15
12  y  15
12  y  12  15  12
y3
The solution is (4, 3).
21. Solve the first equation for c since the coefficient
of c is 1.
c  5d  2
c  5d  5d  2  5d
c  2  5d
Find the value of d by substituting 2  5d for c in
the second equation.
2c  d  4
2(2  5d )  d  4
4  10d  d  4
4  11d  4
4  11d  4  4  4
11d  0
11d
11

42

 21

8y
8

6

 8
3

y4
3

Substitute 4 for y in either equation to find the


value of x.
x  2y  6

 11

d0
Substitute 0 for d in either equation to find the
value of c.
c  5d  2
c  5(0)  2
c02
c2
The solution is (2, 0).
22. Solve the first equation for s since the coefficient
of s is 1.
5r  s  5
5r  s  5r  5  5r
s  5  5r
(1)(s)  (1)(5  5r)
s  5  5r

x2

134 2  6
3

x26
3

x2262
1

x  42

1 3

The solution is 42, 4 .

319

Chapter 7

PQ249-6481F-07[305-325] 8/6/02 08:43 PM Page 320 Sahuja Ahuja_QXP_03:Desktop Folder:6/8/02:

Find the value of x by substituting 104  2x for y


in the first equation.
0.5x  2y  17
0.5x  2(104  2x)  17
0.5x  208  4x  17
4.5x  208  17
4.5x  208  208  17  208
4.5x  225

24. Solve the first equation for x since the coefficient


of x is 1.
x  3y  0
x  3y  3y  0  3y
x  3y
Find the value of y by substituting 3y for x in the
second equation.
3x  y  7
3(3y)  y  7
9y  y  7
10y  7
10y
10

4.5x
4.5

x  50
Substitute 50 for x in either equation to find the
value of y.
2x  y  104
2(50)  y  104
100  y  104
100  y  100  104  100
y4
The solution is (50, 4).

 10
7

y  10
7

Substitute 10 for y in either equation to find the


value of x.
x  3y  0
x3

1107 2  0

21

21

x  10  10  0  10
21

x  10 or 210
The solution is

1 7
210, 10

1
x
2

2.

1
x
2

0.41x
0.41

 3  2x  1

 3  2x  2x  1  2x
3

25. Solve the first equation for y since the coefficient


of y is 1.
0.3x  y  0.5
0.3x  y  0.3x  0.5  0.3x
y  0.5  0.3x
Find the value of x by substituting 0.5  0.3x for y
in the second equation.
0.5x  0.3y  1.9
0.5x  0.3(0.5  0.3x)  1.9
0.5x  0.15  0.09x  1.9
0.41x  0.15  1.9
0.41x  0.15  0.15  1.9  0.15
0.41x  2.05

2x  3  1
3

2x  3  3  1  3
3

2x  4

123 2132x2  123 2142


8

x  3 or 23

Substitute 3 for x in either equation to find the


value of y.
y  2x  1
y2
y

2.05
0.41

y

x5
Substitute 5 for x in either equation to find the
value of y.
0.3x  y  0.5
0.3(5)  y  0.5
1.5  y  0.5
1.5  y  1.5  0.5  1.5
y2
The solution is (5, 2).
26. Solve for y in the second equation since the
coefficient of y is 1.
2x  y  104
2x  y  2x  104  2x
y  104  2x

Chapter 7

27. Since y  2x  3, substitute 2x  3 for y in the


second equation.
y  2x  1

21

x  10  0
21

225
4.5

183 2  1

16
3
13
3

1

or 43

The solution is 23, 43 .


1
y
2

28. Since x 
 3, substitute 2 y  3 for x in the
second equation.
2x  y  6

11

2 2y  3  y  6
y6y6
66
The statement 6  6 is true. This means that
there are infinitely many solutions of the system
of equations.

320

PQ249-6481F-07[305-325] 8/6/02 08:43 PM Page 321 Sahuja Ahuja_QXP_03:Desktop Folder:6/8/02:

The system of equations is a  b  500 and


0.25a  0.50b  0.34(500). Use substitution to
solve this system.
a  b  500
a  b  b  500  b
a  500  b

29. Let y  the length of the base of the triangle, and


let x  the length of each of the other sides. Since
the base is 4 in. longer than the length of one of
the other sides, y  x  4. Since the perimeter is
46 in., x  x  y  46, or 2x  y  46. Use
substitution to solve this system.
2x  y  46
2x  (x  4)  46
3x  4  46
3x  4  4  46  4
3x  42
3x
3

0.25a  0.50b  0.34(500)


0.25(500  b)  0.50b  0.34(500)
125  0.25b  0.50b  170
125  0.25b  170
125  0.25b  125  170  125
0.25b  45

42
3

x  14
yx4
y  14  4
y  18
The length of the base is 18 in., and the length of
each of the other sides is 14 in.
30. Let r  the number of lb of raisins, and let
s  the number of lb of sunflower seeds. Use a
table to organize the information.
Number of Cost per
Pounds
Pound
Raisins
r
1.50
Sunflower Seeds
s
4.00

0.25b
0.25

b  180
a  b  500
a  180  500
a  180  180  500  180
a  320
320 gal of 25% acid solution and 180 gal of 50%
acid solution should be used.
32. Let x  the measure of angle X, and let y  the
measure of angle Y. Since angles X and Y are
supplementary, x  y  180. Since the measure of
angle X is 24 degrees greater than the measure of
angle Y, x  y  24. Use substitution to solve this
system.
x  y  180
( y  24)  y  180
2y  24  180
2y  24  24  180  24
2y  156

Total
Cost
1.50r
4.00s

Since the mix will have three times the number of


lb of raisins as sunflower seeds, r  3s. Since the
total cost is $34.00, 1.50r  4.00s  34.00. Use
substitution to solve this system.
1.50r  4.00s  34.00
1.50(3s)  4.00s  34.00
4.50s  4.00s  34.00
8.50s  34.00
8.50s
8.50

45

 0.25

2y
2

34.00
8.50

156
2

y  78

s4

x  y  24
x  78  24
x  102
The measure of angle X is 102 degrees, or
mX  102.
The measure of angle Y is 78 degrees, or mY  78.
33. Let y  the number of World Series won by the
Yankees, and let r  the number of World Series
won by the Reds. Since the total number of games
won by both teams is 31, y  r  31. Since the
Yankees won 5.2 times as many World Series as the
Reds, y  5.2r. Use substitution to solve this system.
y  r  31
5.2r  r  31
6.2r  31

r  3s
r  3(4)
r  12
The club should purchase 4 lb of sunflower seeds
and 12 lb of raisins.
31. Let a  the number of gallons of 25% acid
solution, and let b  the number of gallons of 50%
solution. Use a table to organize the information.
25%
50%
34%
Solution Solution Solution
Total Gallons
a
b
500
Gallons of Acid 0.25a
0.50b
0.34 (500)

6.2r
6.2

31

 6.2

r5
y  5.2r
y  5.2(5)
y  26
The Yankees won 26 World Series, and the Reds
won 5 World Series.

321

Chapter 7

PQ249-6481F-07[305-325] 8/6/02 08:43 PM Page 322 Sahuja Ahuja_QXP_03:Desktop Folder:6/8/02:

34. Let p  the total price of the automobiles Shantel


sells, and let t  her total income. At the first
dealership, t  600  0.02p. At the second
dealership, t  1000  0.015p. Use substitution
to solve this system.
t  600  0.02p
1000  0.015p  600  0.02p
1000  0.015p  0.015p  600  0.02p  0.015p
1000  600  0.005p
1000  600  600  0.005p  600
400  0.005p
400
0.005

The number of tourists would be expected to be


the same about 23.3 yr after 2000, or some time
during the year 2023.
38. See students work.
39. Since y  3z  7, substitute 3z  7 for y in the
first equation.
2x  3y  z  17
2x  3(3z  7)  z  17
2x  9z  21  z  17
2x  10z  21  17
2x  10z  21  21  17  21
2x  10z  38
Next, use this equation and the third equation in
the given system to write a system of two
equations containing only x and z.
2x  10z  38
2x  z  2
Solve the second equation for z since the
coefficient of z is 1.
2x  z  2
2x  2  z  2  2
2x  2  z
Since z  2x  2, substitute 2x  2 for z in the
first equation.
2x  10z  38
2x  10(2x  2)  38
2x  20x  20  38
18x  20  38
18x  20  20  38  20
18x  18

0.005p
0.005

80,000  p
Shantel will make the same income from either
dealership if the total price of the automobiles she
sells is $80,000.
35. The second offer is better if she sells less than
$80,000. The first offer is better if she sells more
than $80,000. For example, suppose the total
price of the automobiles she sells is $50,000.
At the first dealership, her income will be 600 
0.02(50,000) or $1600; at the second dealership,
her income will be 1000  0.015(50,000) or $1750.
Thus, the second dealerships offer is better.
However, if the total price is $100,000, Shantels
income at the first dealership will be 600 
0.02(100,000) or $2600; her income at the second
dealership will be 1000  0.015(100,000) or
$2500. Thus, the first dealerships offer is better.
36. Let y  the number of years, and let h  the
height of the tree in inches. Since the blue spruce
grows 6 in. per yr and is 4 ft, or 48 in., tall,
h  6y  48. Since the hemlock grows 4 in. per yr
and is 6 ft, or 72 in., tall, h  4y  72. Use
substitution to solve this system.
h  6y  48
4y  72  6y  48
4y  72  4y  6y  48  4y
72  2y  48
72  48  2y  48  48
24  2y
24
2

18x
18

x  1
Use 2x  z  2 to find the value of z.
2x  z  2
2(1)  z  2
2  z  2
2  2  z  2  2
4  z
Use y  3z  7 to find the value of y.
y  3z  7
y  3(4)  7
y  12  7
y5
Since x  1, y  5 and z  4, the solution is
(1, 5, 4).

2y
2

12  y
The trees will be the same height in 12 yr.
37. Let y  the number of years since 2000, and let
t  the number of tourists in millions. Since the
number of tourists visiting South America and the
Caribbean was 40.3 million, and tourism is
increasing at a rate of 0.8 million per year,
t  40.3  0.8y. Since the number of tourists to
the Middle East was 17.0 million, and tourism is
increasing at a rate of 1.8 million tourists per year,
t  17.0  1.8y. Use substitution to solve this system.
t  40.3  0.8y
17.0  1.8y  40.3  0.8y
17.0  1.8y  0.8y  40.3  0.8y  0.8y
17.0  y  40.3
17.0  y  17.0  40.3  17.0
y  23.3

Chapter 7

18

 18

322

PQ249-6481F-07[305-325] 8/6/02 08:44 PM Page 323 Sahuja Ahuja_QXP_03:Desktop Folder:6/8/02:

40. When problems about technology involve a


system of equations, the problem can be solved by
substitution. Answers should include the
following.
To solve a system of equations using
substitution, solve one equation for one
unknown. Substitute this value for the
unknown in the other equation and solve the
equation. Use this number to find the other
unknown.
Since y  2.8x  170, substitute 2.8x  170
for y in the second equation.
y  14.4x  2
2.8x  170  14.4x  2
2.8x  170  2.8x  14.4x  2  2.8x
170  17.2x  2
170  2  17.2x  2  2
168  17.2x
9.8  x
The number of hours will be the same about
9.8 years after 1993. That represents the year
2002.
41. B; Solve the first equation for x since the
coefficient of x is 1.
x  4y  1
x  4y  4y  1  4y
x  1  4y
Since x  1  4y, the expression 1  4y could be
substituted for x in the second equation.
42. C; Solve the first equation for x since the
coefficient of x is 1.
x  3y  9
x  3y  3y  9  3y
x  9  3y
Find the value of y by substituting 9  3y for x
in the second equation.
5x  2y  7
5(9  3y)  2y  7
45  15y  2y  7
45  13y  7
45  13y  45  7  45
13y  52
13y
13

Page 381

Maintain Your Skills

43.

y
xy4

xy3
x
O

The graphs of the equations are parallel lines.


Since they do not intersect, there are no solutions
to this system of equations.
44.

x  2y  1

2x  y  5

(3, 1) x

The graphs appear to intersect at (3, 1). Check


in each equation.
x  2y  1
2x  y  5
Check:
?
?
3  2(1)  1 2(3)  (1)  5
?
?
3  (2)  1
6  (1)  5
11
55
There is one solution. It is (3, 1).
45.

2x  y  3
4x  2y  6
O

The graphs of the equations coincide. Since every


point is a point of intersection, there are infinitely
many solutions to this system of equations.

52

 13

y4
Substitute 4 for y in either equation to find the
value of x.
x  3y  9
x  3(4)  9
x  12  9
x  12  12  9  12
x3
So, the value of x is 3.

323

Chapter 7

PQ249-6481F-07[305-325] 8/6/02 08:44 PM Page 324 Sahuja Ahuja_QXP_03:Desktop Folder:6/8/02:

Step 3 Select a point in one of the half-planes


and test it. Try (0, 0).
y 7 6  2x
0 7 6  2(0)
false
0 7 6
Since the statement is false, the half-plane
containing the origin is not part of the solution.
Shade the other half-plane.

46. Step 1 The inequality is already solved for y in


terms of x.
Step 2 Graph y  5. Since the inequality
includes y values less than 5, but not
equal to 5, the boundary is not included
in the solution set. The boundary should
be drawn as a dashed line.
Step 3 Select a point in one of the half-planes
and test it. Try (0, 0).
y 6 5
0 6 5 false
Since the statement is false, the half-plane
containing the origin is not part of the solution.
Shade the other half-plane.

2x  y  6

y
x

49. Let d represent the number of pounds of denim


left.
number of pounds left 1
d number of pounds left

number of pairs of jeans 5
250 number of pairs of jeans
1(250)  5(d)
250  5(d)

y  5

250
5

47. Step 1 There is no y in the inequality. The


inequality is already solved for x.
Step 2 Graph x  4. Since x
4 means x 4 or
x  4, the boundary is included in the
solution set. The boundary should be
drawn as a solid line.
Step 3 Select a point in one of the half-planes
and test it. Try (0, 0).
x
4
0
4 false
Since the statement is false, the half-plane
containing the origin is not part of the solution.
Shade the other half-plane.

50.
51.
52.
53.

5d
5

50  d
There would be 50 lb of denim left.
6a  9a  (6  9)a
 3a
8t  4t  (8  4)t
 12t
7g  8g  (7  8)g
 15g
7d  (2d  b)  7d  2d  b
 5d  b

Page 381
1.

Practice Quiz 1
y

x4
xy3
O

x
xy1

Solve for y in terms of x.


2x  y 7 6
2x  y  2x 7 6  2x
y 7 6  2x
Step 2 Graph y  6  2x. Since the inequality
includes values greater than 6  2x, but
not equal to 6  2x, the boundary is not
included in the solution set. The
boundary should be drawn as a dashed
line.

(2, 1)

48. Step 1

Chapter 7

The graphs appear to intersect at (2, 1). Check in


each equation.
Check:
xy3
xy1
?
?
211
213
33
11
There is one solution. It is (2, 1).

324

PQ249-6481F-07[305-325] 8/6/02 08:44 PM Page 325 Sahuja Ahuja_QXP_03:Desktop Folder:6/8/02:

2.

The statement 2  2 is true. This means that


there are infinitely many solutions of the system
of equations.

y
3x  2y  6

7-3

Elimination Using Addition


and Subtraction

3x  2y  6

Pages 384385

The graphs of the equations are parallel lines.


Since they do not intersect, there are no solutions
to this system of equations.
3. Solve the first equation for y since the coefficient
of y is 1 (in both equations).
xy0
xyx0x
y  x
Find the value of x by substituting x for y in the
second equation.
3x  y  8
3x  (x)  8
2x  8
2x
2

Check for Understanding

1. Sample answer: 2a  b  5, a  b  4
Since the coefficients of the b terms are additive
inverses, this system can be solved by using
addition to eliminate one variable.
2. Subtraction can be used to eliminate one variable
for a system in which one variable has the same
coefficient in both equations.
3. Michaels method is correct since, in order to
eliminate the s terms, you must add the two
equations.
4. Since the coefficients of the y terms, 1 and 1,
are additive inverses, eliminate the y terms by
adding the equations.
x  y  14
() x  y  20
2x
 34

8
2

x  4
Substitute 4 for x in either equation to find the
value of y.
xy0
(4)  y  0
4  y  4  0  4
y4
The solution is (4, 4).
4. Solve the first equation for x since the coefficient
of x is 1.
x  2y  5
x  2y  2y  5  2y
x  5  2y
Find the value of y by substituting 5  2y for x in
the second equation.
3x  5y  8
3(5  2y)  5y  8
15  6y  5y  8
15  y  8
15  y  15  8  15
y  7
Substitute 7 for y in either equation to find the
value of x.
x  2y  5
x  2(7)  5
x  (14)  5
x  14  5
x  14  14  5  14
x  9
The solution is (9, 7).
5. Since y  2  x, substitute 2  x for y in the first
equation.
xy2
x  (2  x)  2
22

2x
2

34
2

x  17
Now substitute 17 for x in either equation to find
the value of y.
x  y  20
17  y  20
17  y  17  20  17
y3
The solution is (17, 3).
5. Since the coefficients of the b terms, 3 and 3,
are additive inverses, eliminate the b terms by
adding the equations.
2a  3b  11
8
() a  3b 
3a
 3
3a
3

3
3

a  1
Now substitute 1 for a in either equation to find
the value of y.
a  3b  8
1  3b  8
1  3b  1  8  1
3b  9
3b
3

3

b3
The solution is (1, 3).

325

Chapter 7

PQ249-6481F-07[326-345] 8/6/02 09:50 PM Page 326 Sahuja Ahuja_QXP_03:Desktop Folder:6/8/02:

9. Since the coefficients of the m terms, 4 and 4,


are the same, eliminate the m terms by
subtracting the equations.
4m  2n  6
() 4m  n  8
n  2
Now substitute 2 for n in either equation to find
the value of m.
4m  n  8
4m  (2)  8
4m  2  8
4m  2  2  8  2
4m  10

6. Since the coefficients of the x terms, 4 and 4, are


the same, eliminate the x terms by subtracting
the equations.
4x  y  9
() 4x  2y  10
y  1
(1)(y)  (1)1
y  1
Now substitute 1 for y in either equation to find
the value of x.
4x  y  9
4x  (1)  9
4x  1  9
4x  1  1  9  1
4x  8
4x
4

4m
4

6x
6

y  5
The solution is (0, 5).
8. Since the coefficients of the a terms, 2 and 2,
are additive inverses, eliminate the a terms by
adding the equations.
2a  4b  30
() 2a  2b  21.5
2b  8.5
8.5
2

b  4.25
Now substitute 4.25 for b in either equation to
find the value of a.
2a  4b  30
2a  4(4.25)  30
2a  17  30
2a  17  17  30  17
2a  13
2a
2

a

13
2
13
2

or 6.5

The solution is (6.5, 4.25).

Chapter 7

36
6

x6
Now substitute 6 for x in either equation to find
the value of y.
x  y  24
6  y  24
6  y  6  24  6
y  18
The numbers are 6 and 18.
11. D; Since the coefficients of the x terms, 2 and 2,
are the same, eliminate the x terms by
subtracting the equations.
2x  7y  17
() 2x  5y  11
2y  6
The value of 2y is 6.

10
2

10. Let x represent the first number and let y


represent the second number.
x  y  24
5x  y  12
Use elimination to solve the system.
x  y  24
()5x  y  12
6x
 36

4

2b
2

The solution is 22, 2 .

x0
Now substitute 0 for x in either equation to find
the value of y.
6x  2y  10
6(0)  2y  10
2y  10
2y
2

m  2 or 22

8
4

x  2
The solution is (2, 1) .
7. Since the coefficients of the y terms, 2 and 2, are
the same, eliminate the y terms by subtracting
the equations.
6x  2y  10
() 2x  2y  10
4x
0
4x
4

10

 4

326

PQ249-6481F-07[326-345] 8/6/02 09:50 PM Page 327 Sahuja Ahuja_QXP_03:Desktop Folder:6/8/02:

Pages 385386

15. Since the coefficients of the x terms, 4 and 4,


are additive inverses, eliminate the x terms by
adding the equations.
4x  2y 
8
() 4x  3y  10
y  2
(1)(y)  (1)(2)
y2
Now substitute 2 for y in either equation to find
the value of x.
4x  3y  10
4x  3(2)  10
4x  6  10
4x  6  6  10  6
4x  4

Practice and Apply

12. Since the coefficients of the y terms, 1 and 1,


are additive inverses, eliminate the y terms by
adding the equations.
x  y  3
() x  y  1
2x
 2
2x
2

2
2

x  1
Now substitute 1 for x in either equation to find
the value of y.
x  y  3
(1)  y  3
1  y  1  3  1
y  2
The solution is (1, 2) .
13. Since the coefficients of the t terms, 1 and 1, are
additive inverses, eliminate the t terms by adding
the equations.
st4
() s  t  2
2s
6
2s
2

4x
4

s3
Now substitute 3 for s in either equation to find
the value of t.
st4
(3)  t  4
3t343
t  1
1

 1

t  1
The solution is (3, 1) .
14. Since the coefficients of the n terms, 2 and 2,
are additive inverses, eliminate the n terms by
adding the equations.
3m  2n  13
() m  2n  7
4m
 20
4m
4

2n
2

2

n4
Now substitute 4 for n in either equation to find
the value of m.
2m  5n  6
2m  5(4)  6
2m  20  6
2m  20  20  6  20
2m  14

20
4

m5
Now substitute 5 for m in either equation to find
the value of n.
m  2n  7
5  2n  7
5  2n  5  7  5
2n  2
2n
2

4
4

x  1
The solution is (1, 2) .
16. Since the coefficients of the b terms, 1 and 1, are
the same, eliminate the b terms by subtracting
the equations.
3a  b  5
() 2a  b  10
a
 5
Now substitute 5 for a in either equation to find
the value of b.
2a  b  10
2(5)  b  10
10  b  10
10  b  10  10  10
b  20
The solution is (5, 20) .
17. Since the coefficients of the m terms, 2 and 2, are
the same, eliminate the m terms by subtracting
the equations.
2m  5n  6
() 2m  7n  14
2n  8

2

t
1

2m
2

14
2

m7
The solution is (7, 4).

2

n1
The solution is (5, 1).

327

Chapter 7

PQ249-6481F-07[326-345] 8/6/02 09:50 PM Page 328 Sahuja Ahuja_QXP_03:Desktop Folder:6/8/02:

18. Since the coefficients of the s terms, 5 and 5,


are the same, eliminate the s terms by
subtracting the equations.
3r  5s  35
() 2r  5s  30
r
 5
Now substitute 5 for r in either equation to find
the value of s.
3r  5s  35
3(5)  5s  35
15  5s  35
15  5s  15  35  15
5s  20
5s
5

21. Since the coefficients of the s terms, 6 and 6, are


the same, eliminate the s terms by subtracting
the equations.
6s  5t  1
() 6s  5t  11
10t  10
10t
10

20
5

6s
6

18
6

6y
6

26
13

4x
4

The

12
6

18
4
9
1
x  2 or 42
1
solution is 42, 2

2.

23. Since the coefficients of the b terms, 2 and 2,


are the same, eliminate the b terms by
subtracting the equations.
a  2b  5
() 3a  2b  9
2a
 4

 3

y  2
Now substitute 2 for y in either equation to find
the value of x.
3x  5y  16
3x  5(2)  16
3x  10  16
3x  10  10  16  10
3x  6
3x
3

y2
Now substitute 2 for y in either equation to find
the value of x.
4x  3y  12
4x  3(2)  12
4x  6  12
4x  6  6  12  6
4x  18

a  2
The solution is (2, 3) .
20. Since the coefficients of the x terms, 3 and 3,
are additive inverses, eliminate the x terms by
adding the equations.
3x  5y  16
() 3x  2y  10
3y  6
3y
3

6

s1
The solution is (1, 1) .
22. Since the coefficients of the x terms, 4 and 4, are
the same, eliminate the x terms by subtracting
the equations.
4x  3y  1
() 4x  3y  24
6y  12

b3
Now substitute 3 for b in either equation to find
the value of a.
13a  5b  11
13a  5(3)  11
13a  15  11
13a  15  15  11  15
13a  26
13a
13

10
10

t  1
Now substitute 1 for t in either equation to find
the value of s.
6s  5t  1
6s  5(1)  1
6s  5  1
6s  5  5  1  5
6s  6

s4
The solution is (5, 4) .
19. Since the coefficients of the a terms, 13 and 13,
are the same, eliminate the a terms by
subtracting the equations.
13a  5b  11
() 13a  11b 
7
6b  18
6b
6

2a
2

4

 2

a2
Now substitute 2 for a in either equation to find
the value of b.
a  2b  5
2  2b  5
2  2b  2  5  2
2b  3

3

x2
The solution is (2, 2) .

2b
2

 2
3

b  2 or 12

The solution is 2, 12 .

Chapter 7

328

PQ249-6481F-07[326-345] 8/6/02 09:50 PM Page 329 Sahuja Ahuja_QXP_03:Desktop Folder:6/8/02:

Now substitute 1.75 for x in either equation to


find the value of y.
1.44x  3.24y  5.58
1.4411.752  3.24y  5.58
2.52  3.24y  5.58
2.52  3.24y  2.52  5.58  2.52
3.24y  8.1

24. Since the coefficients of the y terms, 5 and 5, are


the same, eliminate the y terms by subtracting
the equations.
4x  5y  7
() 8x  5y  9
4x
 2
4x
4

2

 4

3.24y
3.24

x2
Now substitute
the value of y.
4x  5y  7
4

1
2

y  2.5
The solution is (1.75, 2.5).
27. Since the coefficients of the m terms, 7.2 and 7.2,
are the same, eliminate the m terms by
subtracting the equations.
7.2m  4.5n  129.06
() 7.2m  6.7n  136.54
2.2n  7.48

for x in either equation to find

112 2  5y  7

2  5y  7
2  5y  2  7  2
5y  5
5y
5

2.2n
2.2

5

y1
The solution is

1 12.

2
4
1
b  2
1
substitute 2

for b in either equation to find

1 12

()

8a  2  1
1

8a  2  2  1  2

3
1
c  5d
5
7
1
c  5d
5

 9

2c

 20
2c
2

8a  2

118 28a  118 232


3

a  16

The solution is

113.76
7.2

 11


20
2

c  10
Now substitute 10 for c in either equation to find
the value of d.

1163 , 12 2.

26. Since the coefficients of the y terms, 3.24 and


3.24, are additive inverses, eliminate the y terms
by adding the equations.
1.44x  3.24y  5.58
() 1.08x  3.24y  9.99
2.52x
 4.41
2.52x
2.52

m  15.8
The solution is (15.8, 3.4).
1
1
28. Since the coefficients of the d terms, 5 and 5, are
additive inverses, eliminate the d terms by
adding the equations.

8a  2  1
1

7.48
2.2

7.2m
7.2

Now
the value of a.
8a  b  1

n  3.4
Now substitute 3.4 for n in either equation to find
the value of m.
7.2m  4.5n  129.06
7.2m  4.513.42  129.06
7.2m  15.3  129.06
7.2m  15.3  15.3  129.06  15.3
7.2m  113.76

1
,
2

25. Since the coefficients of the a terms, 8 and 8, are


the same, eliminate the a terms by subtracting
the equations.
8a  b  1
() 8a  3b  3
4b  2
4b
4

8.1

 3.24

3
c
5

 5d  9

3
(10)
5

 5d  9

1
1

6  5d  9
1

6  5d  6  9  6

5d  3
1

(5) 5d  (5)3

4.41

 2.52

d  15
The solution is (10, 15).

x  1.75

329

Chapter 7

PQ249-6481F-07[326-345] 8/6/02 09:50 PM Page 330 Sahuja Ahuja_QXP_03:Desktop Folder:6/8/02:

32. Let x represent the first number and let y


represent the second number.
3x  y  18
2x  y  12
Use elimination to solve the system.
3x  y  18
() 2x  y  12
5x
 30

29. Since the coefficients of the y terms, 2 and 2,


are the same, eliminate the y terms by
subtracting the equations.
2
1
x  2y
3
5
1
() 6 x  2 y
1
6x
1
(6) 6x

 14
 18
 4

2  (6)(4)

5x
5

x  24
Now substitute 24 for x in either equation to find
the value of y.
2
x
3

 2y  14

2
(24)
3

 2y  14

1
1

16  2y  14
1

16  2y  16  14  16

2y  2
1

(2) 2y  (2)(2)


y4
The solution is (24, 4).
30. Let x represent the first number and y represent
the second number.
x  y  48
x  y  24
Use elimination to solve the system.
x  y  48
() x  y  24
2x
 72
2x
2

3x
3

72
2

2y
2

15
3

18
2

y9
The numbers are 5 and 9.

64
2

x  32
Now substitute 32 for x in either equation to find
the value of y.
x  y  51
32  y  51
32  y  32  51  32
y  19
The numbers are 32 and 19.

Chapter 7

x5
Now substitute 5 for x in either equation to find
the value of y.
x  2y  23
5  2y  23
5  2y  5  23  5
2y  18

x  36
Now substitute 36 for x in either equation to find
the value of y.
x  y  48
36  y  48
36  y  36  48  36
y  12
The numbers are 36 and 12.
31. Let x represent the first number and let y
represent the second number.
x  y  51
x  y  13
Use elimination to solve the system.
x  y  51
() x  y  13
2x
 64
2x
2

30
5

x6
Now substitute 6 for x in either equation to find
the value of y.
3x  y  18
3(6)  y  18
18  y  18
18  y  18  18  18
y0
The numbers are 6 and 0.
33. Let x represent the first number and let y
represent the second number.
x  2y  23
4x  2y  38
Use elimination to solve the system.
x  2y  23
() 4x  2y  38
3x
 15

330

PQ249-6481F-07[326-345] 8/6/02 09:50 PM Page 331 Sahuja Ahuja_QXP_03:Desktop Folder:6/8/02:

Rewrite t  d  0.467 so that the system can be


written in column form.
t  d  0.467
t  d  d  0.467  d
t  d  0.467
Use elimination to solve the system.
t  d  0.467
() t  d  12.867
2t
 13.334

34. Let u represent the number of motor vehicles


produced in the United States in millions, and let
j represent the number of motor vehicles
produced in Japan in millions.
uj2
u  j  22
Rewrite u  j  2 so that the system can be
written in column form.
uj2
ujj2j
uj2
Use elimination to solve the system.
uj 2
() u  j  22
2u
 24
2u
2

2t
2

24
2

y2  y1

mx

m
m

18
2

 x1

1.52  1.28
50  0
0.24
50

m  0.0048
The slope is 0.0048.
Choose (0, 1.28) and find the y-intercept of the line.
y  mx  b
1.28  0.0048(0)  b
1.28  b
Write the slope-intercept form using m  0.0048
and b  1.28.
y  mx  b
y  0.0048x  1.28
Therefore, the equation is y  0.0048x  1.28.

s9
Now substitute 9 for s in either equation to find
the value of a.
2a  5s  77
2a  5(9)  77
2a  45  77
2a  45  45  77  45
2a  32
2a
2

13.334
2

t  6.667
Now substitute 6.667 for t in either equation to
find the value of d.
t  d  0.467
6.667  d  0.467
6.667  0.467  d  0.467  0.467
6.200  d
In 1999, Troy Aikman earned $6.667 million and
Deion Sanders earned $6.200 million.
37. Chinas population in 2000, when x  0, is
1.28 billion people. This means (0, 1.28) is in the
solution set for the equation. Chinas population
in 2050, when x  50, is 1.52 billion people.
This means (50, 1.52) is in the solution set for
the equation. Let (x1, y1)  (0, 1.28) and
(x2, y2)  (50, 1.52).

u  12
Now substitute 12 for u in either equation to find
the value of j.
uj2
12  j  2
12  2  j  2  2
10  j
In 1999, the U.S. produced about 12 million motor
vehicles and Japan produced about 10 million
motor vehicles.
35. Let a represent the adult price and let s represent
the student price.
2a  5s  77
2a  7s  95
Use elimination to solve the system.
2a  5s  77
() 2a  7s  95
2s  18
2s
2

32
2

a  16
The adult price of the tour is $16 and the student
price of the tour is $9.
36. Let t represent the amount earned by
Troy Aikman in millions and let d represent the
amount earned by Deion Sanders in millions.
t  d  0.467
t  d  12.867

331

Chapter 7

PQ249-6481F-07[326-345] 8/6/02 09:50 PM Page 332 Sahuja Ahuja_QXP_03:Desktop Folder:6/8/02:

38. Indias population in 2000, when x  0, is


1.01 billion people. This means (0, 1.01) is in the
solution set for the equation. Indias population in
2050, when x  50, is 1.53 billion people. This
means (50, 1.53) is in the solution set for the
equation. Let (x1, y1)  (0, 1.01) and
(x2, y2)  (50, 1.53).

Use elimination to solve the system.


2A  B  15
() 2 A  B  9
4A
 24
4A
4

m
m

 x1

1.53  1.01
50  0
0.52
50

m  0.0104
The slope is 0.0104.
Choose (0, 1.01) and find the y-intercept of the line.
y  mx  b
1.01  0.0104(0)  b
1.01  b
Write the slope-intercept form using m  0.0104
and b  1.01.
y  mx  b
y  0.0104x  1.01
Therefore, the equation is y  0.0104x  1.01.
39. From Exercise 37, Chinas population is
represented by y  0.0048x  1.28. From
Exercise 38, Indias population is represented by
y  0.0104x  1.01. Use elimination to solve the
system.
y  0.0048x  1.28
() y  0.0104x  1.01
0  0.0056x  0.27
0  0.0056x  0.0056x  0.27  0.0056x
0.0056x  0.27
0.0056x
0.0056

24
4

A6
Now substitute 6 for A in either equation to find
the value of B.
2A  B  15
2(6)  B  15
12  B  15
12  B  12  15  12
B3
So, the value of A is 6 and the value of B is 3.
41. Elimination can be used to solve problems about
meteorology if the coefficients of one variable are
the same or are additive inverses. Answers should
include the following.
The two equations in the system of equations
are added or subtracted so that one of the
variables is eliminated. You then solve for the
remaining variable. This number is substituted
into one of the original equations, and that
equation is solved for the other variable.

Write the equations in


n  d  24
column form and add.
() n  d  12
Notice the d variable
2n
 36
is eliminated.

y2  y1

mx

2n
2

36
2

n  18
n  d  24
18  d  24
18  d  18  24  18

Simplify.
First equation
n  18
Subtract 18 from each
side.
Simplify.
d6
On the winter solstice, Seward, Alaska, has
18 hours of darkness and 6 hours of daylight.
42. B; Use elimination to solve the system.
2x  3y  9
() 3x  3y  12
x
3
(1)(x)  (1)3
x  3
Now substitute 3 for x in either equation to find
the value of y.
2x  3y  9
2(3)  3y  9
6  3y  9
6  3y  6  9  6
3y  3

0.27

 0.0056

x  48
Now substitute 48 for x in either equation to find
the value of y.
y  0.0048x  1.28
y  0.0048(48)  1.28
y  0.2304  1.28
y  1.5104
The populations of China and India are predicted
to be the same about 48 yr after 2000, or in 2048.
At that time, the predicted population is about
1.51 billion people.
40. Since the graphs intersect at (2, 1), (x, y)  (2, 1)
must make both equations true. Substitute 2 for x
and 1 for y in both equations to find the values of
A and B.
Ax  By  15
Ax  By  9
A(2)  B(1)  15
A(2)  B(1)  9
2A  B  15
2A  B  9

3y
3

3

 3

y1
The value of y is 1.

Chapter 7

Divide each side by 2.

332

PQ249-6481F-07[326-345] 8/6/02 09:50 PM Page 333 Sahuja Ahuja_QXP_03:Desktop Folder:6/8/02:

Since x  2y  5, substitute 2y  5 for x in the


second equation.
4y  3x  1
4y  3(2y  5)  1
4y  6y  15  1
2y  15  1
2y  15  15  1  15
2y  14

43. C; Use elimination to solve the system.


4x  2y  8
() 2x  2y  2
2x
6
2x
2

2

x3
Now substitute 3 for x in either equation to find
the value of y.
4x  2y  8
4(3)  2y  8
12  2y  8
12  2y  12  8  12
2y  4
2y
2

2y
2

y  7
Substitute 7 for y in either equation to find the
value of x.
2y  x  5
2(7)  x  5
14  x  5
14  x  14  5  14
x  9
(1)(x)  (1)(9)
x  9
The solution is (9, 7).

4
2

y  2
The solution is (3, 2).

Page 386

Maintain Your Skills

44. Since y  5x, substitute 5x for y in the second


equation.
x  2y  22
x  2(5x)  22
x  10x  22
11x  22
11x
11

47.

3x  y  1

xy3

22

x2
Use y  5x to find the value of y.
y  5x
y  5(2)
y  10
The solution is (2, 10).
45. Since x  2y  3, substitute 2y  3 for x in the
second equation.
3x  4y  1
3(2y  3)  4y  1
6y  9  4y  1
10y  9  1
10y  9  9  1  9
10y  10


 11

10y
10

14

 2

(1, 2)

The graphs appear to intersect at (1, 2). Check


in each equation.
Check:
xy3
3x  y  1
?
?
1  (2)  3
3(1)  (2)  1
?
?
123
3  (2)  1
33
11
There is one solution. It is (1, 2).
48.

y
3y  7  2x

10
10

y  1
Use x  2y  3 to find the value of x.
x  2y  3
x  2(1)  3
x  2  3
x1
The solution is (1, 1).
46. Solve the first equation for x since the coefficient
of x is 1.
2y  x  5
2y  x  x  5  x
2y  5  x
2y  5  5  x  5
2y  5  x

x
2x  3y  7

The graphs of the equations are parallel lines.


Since they do not intersect, there are no solutions
to this system of equations.

333

Chapter 7

PQ249-6481F-07[326-345] 8/6/02 09:50 PM Page 334 Sahuja Ahuja_QXP_03:Desktop Folder:6/8/02:

49.

4. Multiply the first equation by 4 so the coefficients


of the y terms are additive inverses. Then add the
equations.
Multiply by 4.
2x  y  6
8x  4y  24
3x  4y  2
() 3x  4y  2
11x
 22

y
12

4x  y  12

8
4

x  3  1y
4

11x
11

x2
Now substitute 2 for x in either equation to find
the value of y.
2x  y  6
2(2)  y  6
4y6
4y464
y  2
(1)(y)  (1)2
y  2
The solution is (2, 2).
5. Multiply the first equation by 3 so the
coefficients of the x terms are additive inverses.
Then add the equations.
Multiply by 3. 3x  15y  12
x  5y  4
3x  7y  10
() 3x  7y  10
22y  22

The graphs of the equations coincide. Since every


point is a point of intersection, there are infinitely
many solutions to this system of equations.
5

50. The line parallel to y  4x  3 has slope 4.


5
Replace m with 4, and (x1, y1 ) with (0, 0) in the
point-slope form.
y  y1  m(x  x1 )
5

y  0  4 (x  0)
5

y  4x
5

Therefore, the equation is y  4x.


51. 2(3x  4y)  2  3x  2  4y
 6x  8y
52. 6(2a  5b)  6  2a  6  5b
 12a  30b
53. 3(2m  3n)  (3)(2m)  (3)(3n)
 6m  (9n)
 6m  9n
54. 5(4t  2s)  (5)(4t)  (5)(2s)
 20t  (10s)
 20t  10s

7-4

22y
22

4x  7y  6
6x  5y  20

Check for Understanding

1. If one of the variables cannot be eliminated by


adding or subtracting the equations, you must
multiply one or both of the equations by numbers
so that a variable will be eliminated when the
equations are added or subtracted.
2. Sample answer: 3x  2y  5 and 4x  10y  6
If the first equation is multiplied by 5, the
coefficients of the y terms will be 10 and 10,
which are additive inverses. Thus, when the two
equations are added together, the y variable will
be eliminated.
3. Sample answer: (1) You could solve the first
equation for a and substitute the resulting
expression for a in the second equation. Then find
the value of b. Use this value for b and one of the
equations to find the value of a. (2) You could
multiply the first equation by 3 and add this new
equation to the second equation. This will
eliminate the b term. Find the value of a. Use this
value for a and one of the equations to find the
value of b. See students work for their
preferences and explanations.

Chapter 7

22

 22

y1
Now substitute 1 for y in either equation to find
the value of x.
x  5y  4
x  5(1)  4
x54
x5545
x  1
The solution is (1, 1).
6. Eliminate x.

Elimination Using Multiplication

Pages 390391

22

 11

Multiply by 3.
Multiply by 2.

12x  21y  18
() 12x  10y  40
11y  22
11y
11

22
11

y  2

Now substitute 2 for y in either equation to find


the value of x.
4x  7y  6
4x  7(2)  6
4x  14  6
4x  14  14  6  14
4x  20
4x
4

20
4

x5
The solution is (5, 2).

334

PQ249-6481F-07[326-345] 8/6/02 09:50 PM Page 335 Sahuja Ahuja_QXP_03:Desktop Folder:6/8/02:

7. Multiply the second equation by 2 so the


coefficients of the x terms are additive inverses.
Then add the equations.

Now substitute 2 for x in either equation to find


the value of y.
3x  7y  6
3(2)  7y  6
6  7y  6
6  7y  6  6  6
7y  0

4x  2y  10.5
4x  2y  10.5
2x  3y  10.75 Multiply by 2. () 4x  6y  21.5
4y  11
4y
4

11
4

7y
7

y  2.75

y0
The solution is (2, 0).
10. For an exact solution, an algebraic method is
best.
Since neither the coefficients of x nor the
coefficients of y are the same or additive
inverses, you cannot use elimination using
addition or subtraction.
Since the coefficient of y in the first equation
is 1, you can use the substitution method.
Since y  4x  11, substitute 4x  11 for y in the
second equation.
3x  2y  7
3x  2 (4x  11)  7
3x  8x  22  7
5x  22  7
5x  22  22  7  22
5x  15

Now substitute 2.75 for y in either equation to


find the value of x.
4x  2y  10.5
4x  2(2.75)  10.5
4x  5.5  10.5
4x  5.5  5.5  10.5  5.5
4x  5
4x
4

4

x  1.25
The solution is (1.25, 2.75).
8. For an exact solution, an algebraic method is
best.
Since neither the coefficients of x nor the
coefficients of y are the same or additive
inverses, you cannot use elimination using
addition or subtraction.
Since no coefficient of x or y is 1 or 1,
substitution is not the best method.
Since it is necessary to multiply each equation
by a different number, solve the system by
elimination using multiplication.
Eliminate x.

5x
5

y1

Now substitute 1 for y in either equation to find


the value of x.
4x  3y  19
4x  3(1)  19
4x  3  19
4x  3  3  19  3
4x  16


16
4

2x
2

x4
The solution is (4, 1).
9. For an exact solution, an algebraic method is
best.
Since the coefficients of the y terms, 7 and 7,
are additive inverses, solve by elimination
using addition.
3x  7y  6
() 2x  7y  4
5x
 10
5x
5

15

 5

x  3
Use y  4x  11 to find the value of y.
y  4x  11
y  4(3)  11
y  12  11
y  1
The solution is (3, 1).
11. For an exact solution, an algebraic method is
best.
Since the coefficients of the y terms, 2 and
2, are the same, you can eliminate the y
terms by subtracting the equations.
5x  2y  12
() 3x  2y  2
2x
 14

4x  3y  19 Multiply by 3.
12x  9y  57
3x  4y  8 Multiply by 4. () 12x  16y  32
25y  25
25y
25
 25
25

4x
4

 7

14
2

x7
Now substitute 7 for x in either equation to solve
for the value of y.
5x  2y  12
5 (7)  2y  12
35  2y  12
35  2y  35  12  35
2y  23
2y
2

23
2

y  11.5
The solution is (7, 11.5).

10
5

x2

335

Chapter 7

PQ249-6481F-07[326-345] 8/6/02 09:50 PM Page 336 Sahuja Ahuja_QXP_03:Desktop Folder:6/8/02:

12. Let t  the number of two-seat tables and f  the


number of four-seat tables.
t  f  17
2t  4f  56
Multiply the first equation by 2 so the
coefficients of the t terms are additive inverses.
Then add the equations.
t  f  17 Multiply by 2. 2t  (2f )  34
2t  4f  56
() 2t  4f  56
2f  22
2f
2

15. Multiply the first equation by 2 so the coefficients


of the y terms are additive inverses. Then add the
equations.
2x  y  5 Multiply by 2.
4x  2y  10
() 3x  2y  4
3x  2y  4
7x
 14
7x
7

22
2

4x  3y  12
x  2y  14

y  13
Now substitute 13 for y in either equation to
find the value of x.
xy4
x  (13)  4
x  13  4
x  13  13  4  13
x  9
The solution is (9, 13).
14. Multiply the first equation by 3 so the coefficients
of the y terms are additive inverses. Then add the
equations.
xy3
Multiply by 3.
3x  3y  9
() 2x  3y  16
2x  3y  16
5x
 25

23x
23

23
23

x  1
Now substitute 1 for x in either equation to find
the value of y.
5x  2y  15
5(1)  2y  15
5  2y  15
5  2y  5  15  5
2y  10

25
5

x5
Now substitute 5 for x in either equation to find
the value of y.
xy3
5y3
5y535
y  2
The solution is (5, 2).

Chapter 7

12

() 4x  8y  56


11y  44
11y
44
 11
11

Now substitute 4 for y in either equation to find


the value of x.
x  2y  14
x  2(4)  14
x  8  14
x  8  8  14  8
x6
The solution is (6, 4).
17. Multiply the first equation by 4 so the coefficients
of the y terms are additive inverses. Then add the
equations.
5x  2y  15 Multiply by 4.
20x  8y  60
() 3x  8y  37
3x  8y  37
23x
 23

26

4x  3y 
Multiply by 4.

y4

 2

5x
5

14
7

x2
Now substitute 2 for x in either equation to find
the value of y.
2x  y  5
2(2)  y  5
4y5
4y454
y1
The solution is (2, 1).
16. Multiply the second equation by 4 so the
coefficients of the x terms are additive inverses.
Then add the equations.

f  11
Now substitute 11 for f in either equation to find
the value of t.
t  f  17
t  11  17
t  11  11  17  11
t6
The owners should purchase 6 two-seat tables
and 11 four-seat tables.
13. Multiply the second equation by 5 so the
coefficients of the x terms are additive inverses.
Then add the equations.
5x  3y  6
5x  3y  6
x  y  4 Multiply by 3.
() 5x  5y  20
2y  26
2y
2

2y
2

10
2

y5
The solution is (1, 5).

336

PQ249-6481F-07[326-345] 8/6/02 09:50 PM Page 337 Sahuja Ahuja_QXP_03:Desktop Folder:6/8/02:

21. Multiply the second equation by 1.8 so the


coefficients of the x terms are additive inverses.
Then add the equations.

18. Multiply the second equation by 4 so the


coefficients of the x terms are additive inverses.
Then add the equations.
8x  3y  11
2x  5y  27 Multiply by 4.

8x  3y  11
() 8x  20y  108

1.8x  0.3y  14.4

1.8x  0.3y 

x  0.6y  2.8 Multiply by 1.8.

0.78y  9.36

17y  119
17y
17

119
17

0.78y
0.78

y  7

Now substitute 12 for y in either equation to find


the value of x.
x  0.6y  2.8
x  0.6(12)  2.8
x  7.2  2.8
x  7.2  7.2  2.8  7.2
x  10
The solution is (10, 12).
22. Multiply the first equation by 3 so the
coefficients of the x terms are additive inverses.
Then add the equations.

32
8

x  4
The solution is (4, 7).
19. Eliminate y.
4x  7y  10

Multiply by 2.
Multiply by 7.

3x  2y  7

0.4x  0.5y  2.5

5y
5

0.4x
0.4

 0.4

x5
The solution is (5, 1).
23. Multiply the second equation by 2 so the
coefficients of the y terms are additive inverses.
Then add the equations.

4
2

3x  2 y  10

Multiply by 5.
10x  15y  10
Multiply by 2. () 10x  8y  56
23y  46
23y
46
 23
23

5x  4 y  8

3x  2 y  10
Multiply by 2.

() 10x  2 y  16
 26

13x
13x
13

y  2

26

 13

x2
Now substitute 2 for x in either equation to find
the value of y.

Now substitute 2 for y in either equation to find


the value of x.
5x  4y  28
5x  4(2)  28
5x  8  28
5x  8  8  28  8
5x  20


5

 5

Now substitute 1 for y in either equation to find


the value of x.
0.4x  0.5y  2.5
0.4x  0.5(1)  2.5
0.4x  0.5  2.5
0.4x  0.5  0.5  2.5  0.5
0.4x  2

29
29

y  2
The solution is (1, 2).
20. Eliminate x.

5x
5

2.5

y1

 29
x  1

2x  3y  2
5x  4y  28

1.2x  3.5y 

5y  5

Now substitute 1 for x in either equation to find


the value of y.
3x  2y  7
3(1)  2y  7
3  2y  7
3  2y  3  7  3
2y  4


1.2x  1.5y  7.5


()

8x  14y  20
() 21x  14y  49
29x
29

2y
2

Multiply by 3.

1.2x  3.5y  2.5

29x

9.36

 0.78

y  12

Now substitute 7 for y in either equation to find


the value of x.
8x  3y  11
8x  3(7)  11
8x  (21)  11
8x  21  11
8x  21  21  11  21
8x  32
8x
8

14.4

() 1.8x  1.08y  5.04

3x  2y  10
1

3(2)  2y  10
1

6  2y  10
1

20
5

6  2y  6  10  6

x4
The solution is (4, 2).

2y  4
1

(2) 2y  (2)4


y  8
The solution is (2, 8).

337

Chapter 7

PQ249-6481F-07[326-345] 8/6/02 09:50 PM Page 338 Sahuja Ahuja_QXP_03:Desktop Folder:6/8/02:

24. Multiply the second equation by 2 so the


coefficients of the x terms are additive inverses.
Then add the equations.
2

2x  3 y  4
x

1
y
2

7

2x  3 y 
Multiply by 2.

Now substitute 4 for x in either equation to find


the value of y.
xy3
4y3
4y434
y  1
The numbers are 4 and 1.
27. For an exact solution, an algebraic method is
best.
Since neither the coefficients of x nor the
coefficients of y are the same or additive
inverses, you cannot use elimination using
addition or subtraction.
Since no coefficient of x or y is 1 or 1,
substitution is not the best method.
Since it is necessary to multiply at least one
equation by a number to eliminate a variable,
solve the system by elimination using
multiplication.
Multiply the first equation by 2 so the coefficients
of the y terms are additive inverses. Then add the
equations.
3x  4y  10 Multiply by 2.
6x  8y  20
5x  8y  2
() 5x  8y  2
11x
 22

() 2x  y  14

12

5
y
3

3 5
y
5 3

 10

135 21102

y  6

Now substitute 6 for y in either equation to find


the value of x.
1

x  2y  7
1

x  2 (6)  7
x  (3)  7
x37
x3373
x4
The solution is (4, 6).
25. Let x represent the first number and let y
represent the second number.
7x  3y  1
x  y  3
Multiply the second equation by 7 so the
coefficients of the x terms are additive inverses.
Then add the equations.
7x  3y  1
x  y  3

Multiply by 7.

11x
11

4y  20
20

 4

y  5

Now substitute 5 for y in either equation to find


the value of x.
x  y  3
x  (5)  3
x  5  3
x  5  5  3  5
x2
The numbers are 2 and 5.
26. Let x represent the first number and let y
represent the second number.
5x  2y  22
xy3
Multiply the second equation by 2 so the
coefficients of the y terms are additive inverses.
Then add the equations.
5x  2y  22
5x  2y  22
() 2x  2y  6
x  y  3 Multiply by 2.
7x
 28
7x
7

8y
8

8

y1
The solution is (2, 1).
28. For an exact solution, an algebraic method is
best.
Since the coefficients of the y terms, 8 and 8,
are additive inverses, solve by elimination
using addition.
9x  8y  42
() 4x  8y  16
13x
 26
13x
13

26

 13

x2
Now substitute 2 for x in either equation to find
the value of y.
9x  8y  42
9(2)  8y  42
18  8y  42
18  8y  18  42  18
8y  24

28
7

x4

8y
8

24

 8

y  3
The solution is (2, 3).
Chapter 7

22
11

x  2
Now substitute 2 for x in either equation to find
the value of y.
5x  8y  2
5(2)  8y  2
10  8y  2
10  8y  10  2  10
8y  8

7x  3y  1
() 7x  7y  21
4y
4

338

PQ249-6481F-07[326-345] 8/6/02 09:50 PM Page 339 Sahuja Ahuja_QXP_03:Desktop Folder:6/8/02:

32. For an exact solution, an algebraic method is


best.
Since the coefficient of y in the second
equation is 1, you can use the substitution
method.
Since y  x, substitute x for y in the first
equation.
4x  2y  14
4x  2(x)  14
4x  2x  14
2x  14

29. For an exact solution, an algebraic method is


best.
Since the coefficient of y in the first equation
is 1, you can use the substitution method.
Since y  3x, substitute 3x for y in the second
equation.
3x  4y  30
3x  4(3x)  30
3x  12x  30
15x  30
15x
15

30

 15

2x
2

x2
Use y  3x to find the value of y.
y  3x
y  3(2)
y6
The solution is (2, 6).
30. For an exact solution, an algebraic method is
best.
Since the coefficient of x in the first equation
is 1, you can use the substitution method.
Since x  4y  8, substitute 4y  8 for x in the
second equation.
2x  8y  3
2(4y  8)  8y  3
8y  16  8y  3
16  3
The statement 16  3 is false. This means that
there is no solution of the system of equations.
31. For an exact solution, an algebraic method is
best.
Since the coefficients of the y terms, 3 and 3,
are additive inverses, you can eliminate the y
terms by adding the equations.
2x  3y  12
() x  3y  12
3x
 24
3x
24

3
3

8y
8

4
4

8

y1
Now substitute 1 for y in either equation to find
the value of x.
xy2
x12
x1121
x3
The solution is (3, 1).
34. For an exact solution, an algebraic method is
best.
Since the coefficient of y in the first equation
is 1, you can use the substitution method.
Since y  2x  9, substitute 2x  9 for y in the
second equation.
2x  y  9
2x  (2x  9)  9
2x  2x  9  9
9  9
The statement 9  9 is true. This means that
there are infinitely many solutions of the system
of equations.

3

y3

14
2

x7
Use y  x to find the value of y.
yx
y7
The solution is (7, 7).
33. For an exact solution, an algebraic method is
best.
Since the coefficient of x in the first equation
is 1, you can use the substitution method.
Solve the first equation for x.
xy2
xyy2y
x2y
Since x  2  y, substitute 2  y for x in the
second equation.
5x  3y  18
5(2  y)  3y  18
10  5y  3y  18
10  8y  18
10  8y  10  18  10
8y  8

x8
Now substitute 8 for x in either equation to find
the value of y.
x  3y  12
8  3y  12
8  3y  8  12  8
3y  4
3y
3

1 42

The solution is 8, 3 .

339

Chapter 7

PQ249-6481F-07[326-345] 8/6/02 09:50 PM Page 340 Sahuja Ahuja_QXP_03:Desktop Folder:6/8/02:

38. For an exact solution, an algebraic method is


best.
Since neither the coefficients of x nor the
coefficients of y are the same or additive
inverses, you cannot use elimination using
addition or subtraction.
Since no coefficient x or y is 1 or 1,
substitution is not the best method.
Since it is necessary to multiply each equation
by a different number, solve the system by
elimination using multiplication.
Eliminate y.

35. For an exact solution, an algebraic method is


best.
Since the coefficients of the x terms, 6 and 6,
are the same, solve by elimination using
subtraction.
6x  y  9
() 6x  y  11
0  2
The statement 0  2 is false. This means that
there is no solution of the system of equations.
36. For an exact solution, an algebraic method is
best.
Since the coefficient of x in the first equation
is 1, you can use the substitution method.
Since x  8y, substitute 8y for x in the second
equation.
2x  3y  38
2(8y)  3y  38
16y  3y  38
19y  38
19y
19

1
x
2
3
x
2

 2 y  18
 4

2  (6)(4)

x  24
Now substitute 24 for x in either equation to find
the value of y.
 2 y  14

2
(24)
3

 2 y  14
1

16  2 y  14
1

16  2 y  16  14  16
2 y  2
1

(2) 2 y  (2)(2)
y4
The solution is (24, 4).

Chapter 7

 18


18
3

 2y  25

3
(6)
2

 2y  25

16
2

y
61
Now substitute 61 for y in either equation to find x.
x  y  701
x  61  701
x  61  61  701  61
x  640
So Bryant had 640 2-point field goals and 61
3-point field goals.

 2y  25

x  y  701 Multiply by 2.


2x  2y  1402
2x  3y  1463
() 2x  3y  1463

y  8
The solution is (6, 8).
39. Since Bryant made 475 free throws and each is
1 point, he scored 475 points in free throws.
Therefore, Bryant scored 1938  475, or 1463,
points in 2-point and 3-point field goals.
Let x  the number of 2-point field goals, and let
y  the number of 3-point field goals Bryant made.
x  y  701
2x  3y  1463
Eliminate x.

2
x
3

3
x
2

2y
2

 2 y  14

()

 2y 

9  2y  25
9  2y  9  25  9
2y  16

Since the coefficients of the y terms, 2 and 2 ,


are the same, solve by elimination using
subtraction.

(6)

 2y  25

3
x
2
3
x
2

x  6
Now substitute 6 for x in either equation to find
the value of y.

38

1
6x

Multiply by 3.

3x
3

y2
Use x  8y to find the value of x.
x  8y
x  8(2)
x  16
The solution is (16, 2).
37. For an exact solution, an algebraic method is
best.

2
x
3
5
() 6 x
1
6x

3x

 19

 3y  3

340

PQ249-6481F-07[326-345] 8/6/02 09:50 PM Page 341 Sahuja Ahuja_QXP_03:Desktop Folder:6/8/02:

42. Let x  the tens digit of the original number, and


let y  the ones digit of the original number. Then
the original number is represented by 10x  y,
and the number with the digits reversed is
represented by 10y  x. Since the sum of the
digits is 14, x  y  14. Since the number with
the digits reversed is 18 less than the original
number, 10y  x  (10x  y)  18.
Rewrite 10y  x  (10x  y)  18 so the system
can be written in column form.
10y  x  (10x  y)  18
10y  x  10x  10x  y  18  10x
10y  9x  y  18
10y  9x  y  y  18  y
9x  9y  18
Eliminate x.
x  y  14 Multiply by 9.
9x  9y  126
9x  9y  18
() 9x  9y  18
18y  108

40. Since (3, a) is the solution of the given system,


substituting 3 for x and a for y will make both
equations in the system true.
4x  5y  2
6x  2y  b
4(3)  5(a)  2
6(3)  2(a)  b
12  5a  2
18  2a  b
12  5a  12  2  12
5a  10
5a
5

10
5

a  2
Since a  2, substitute 2 for a in the second
equation to find b.
18  2a  b
18  2(2)  b
18  (4)  b
18  4  b
22  b
The values of a is 2 and the value of b is 22.
41. Let x  the tens digit of the students actual
score, and let y  the ones digit of the students
actual score. Then the actual score is given by
10x  y, and the accidentally-reversed score is
given by 10y  x. Since the actual score is 36
points greater than the accidentally-reversed
score, 10x  y  (10y  x)  36. Since the sum of
the digits of the score is 14, x  y  14.
Rewrite 10x  y  (10y  x)  36 so the system
can be written in column form.
10x  y  (10y  x)  36
10x  y  x  10y  x  36  x
9x  y  10y  36
9x  y  10y  10y  36  10y
9x  9y  36
Eliminate y.
x  y  14 Multiply by 9.
9x  9y  126
9x  9y  36
(  ) 9x  9y  36
18x
 162
18x
18

18y
18

108
18

y6
Now substitute 6 for y in either equation to find
the value of x.
x  y  14
x  6  14
x  6  6  14  6
x8
So, the tens digit of the original number is 8, and
the ones digit of the original number is 6. This
means the original number is 86.
43. Let r  the rate of the plane in still air, and let
w  the rate of the wind. Use the formula
rate  time  distance, or rt  d.
2

(4 h 40 min  43 h)

162
18

x9
Now substitute 9 for x in either equation to find
the value of y.
x  y  14
9  y  14
9  y  9  14  9
y5
So, the tens digit of the actual score was 9 and
the ones digit of the actual score was 5. This
means the students actual score was 95.

rt  d

Against the wind

rw

2
43

d
2100

With a wind that


is twice as fast

r  2w

2100

1 2 (r  w)  2100
2
43

4(r  2w)  2100

1423 2 (r  w)  2100 or 143 (r  w)  2100

4(r  2w)  2100


Use the distributative property to rewrite each
equation so the system is in column form.
14
r
3

14
w
3

 2100

4r  8w  2100
Eliminate w.
14
r
3

14
w
3

 2100

4r  8w  2100

Multiply by

12
.
7

8r  8w  3600

()4r  8w  2100
 5700

12r
12r
12

5700
12

r  475

The rate of the plane in still air is 475 mph.

341

Chapter 7

PQ249-6481F-07[326-345] 8/6/02 09:50 PM Page 342 Sahuja Ahuja_QXP_03:Desktop Folder:6/8/02:

Now substitute 6 for x in either equation to find


the value of y.
xy8
6y8
6y686
y2
The solution is (6, 2).
48. Since the coefficients of the s terms, 1 and 1, are
additive inverses, you can eliminate the s terms
by adding the equations.
2r  s  5
() r  s  1
3r
6

44. By having two equations that represent the time


constraints, a manager can determine the best
use of employee time. Answers should include the
following.
20c  10b  800 S
20c  10b  800
10c  30b  900 S
20c  60b  1800
50b  1000
50b
50

1000
50

b  20
20c  10b  800
20c  10(20)  800
20c  200  800
20c  200  200  800  200
20c  600
20c
20

3r
3

600
20

r2
Now substitute 2 for r in either equation to find
the value of s.
rs1
2s1
2s212
s  1
(1)(s)  (1)(1)
s1
The solution is (2, 1).
49. Since the coefficients of the x terms, 1 and 1, are
the same, you can eliminate the x terms by
subtracting the equations.
x  y  18
() x  2y  25
y  7
(1)(y)  (1)(7)
y7
Now substitute 7 for y in either equation to find
the value of x.
x  y  18
x  7  18
x  7  7  18  7
x  11
The solution is (11, 7).
50. Since x  3y, substitute 3y for x in the first
equation.
2x  3y  3
2(3y)  3y  3
6y  3y  3
3y  3

c  30
In order to make the most of the employee and
oven time, the manager should make
assignments to bake 30 batches of cookies and
20 loaves of bread.
45. A; Eliminate y.
5x  3y  12 Multiply by 5.
25x  15y  60
4x  5y  17 Multiply by 3. ()12x  15y  51
37x
 111
37x
37

111
37

x3
Now substitute 3 for x in either equation to find
the value of y.
5x  3y  12
5(3)  3y  12
15  3y  12
15  3y  15  12  15
3y  3
3y
3

3
3

y  1
The value of y is 1.
46. D; Multiply the first equation by 2 so the
coefficients of the x terms are additive inverses.
Then add the equations.
x  2y  1 Multiply by 2. 2x  4y  2
2x  4y  2
()2x  4y  2
0 0
The statement 0  0 is true. This means that
there are infinitely many solutions of the system
of equations.

Page 392

3y
3

Maintain Your Skills

12
2

x6

Chapter 7

 3

y  1
Use x  3y to find the value of x.
x  3y
x  3(1)
x3
The solution is (3, 1).

47. Since the coefficients of the y terms, 1 and 1,


are additive inverses, you can eliminate the y
terms by adding the equations.
xy8
()x  y  4
2x
 12
2x
2

3

342

PQ249-6481F-07[326-345] 8/6/02 09:51 PM Page 343 Sahuja Ahuja_QXP_03:Desktop Folder:6/8/02:

Since the statement is true, the half-plane


containing the origin is part of the solution.
Shade the half-plane containing (0, 0).

51. Solve the first equation for x since the coefficient


of x is 1.
xy0
xyy0y
x  y
Since x  y, substitute y for x in the second
equation.
3x  y  8
3(y)  y  8
3y  y  8
2y  8
2y
2

y
x

yx7

8

 2

y4
Substitute 4 for y in either equation to find the
value of x.
xy0
x40
x4404
x  4
The solution is (4, 4).
52. Solve the first equation for x since the coefficient
of x is 1.
x  2y  7
x  2y  2y  7  2y
x  7  2y
Since x  7  2y, substitute 7  2y for x in the
second equation.
3x  6y  21
3(7  2y)  6y  21
21  6y  6y  21
21  21
The statement 21  21 is true. This means
that there are infinitely many solutions of the
system of equations.
53. Let a  the amount of revenue above quota.
32,000  0.04a 7 45,000
32,000  0.04a  32,000 7 45,000  32,000
0.04a 7 13,000
0.04a
0.04

55. Step 1 Solve for y in terms of x.


x  3y  9
x  3y  x  9  x
3y  9  x
9  x
3
9
x
y33
1
y  3  3x
1
1
 3  3 x. Since y  3  3 x
1
1
7 3  3 x or y  3  3 x, the

3y
3

Step 2 Graph y

means y
boundary is included in the solution set.
The boundary should be drawn as a solid
line.
Step 3 Select a point in one of the half-planes
and test it. Try (0, 0).
1

y  3  3x
1

0  3  3 (0)
false
03
Since the statement is false, the half-plane
containing the origin is not part of the solution.
Shade the other half-plane.
8
6
4
2

13,000
0.04

42
2
4
6
8

a 7 325,000
The store must make more than $325,000 above
quota.
54. Step 1 The inequality is already solved for y in
terms of x.
Step 2 Graph y  x  7. Since y  x  7 means
y  x  7 or y  x  7, the boundary is
included in the solution set. The boundary
should be drawn as a solid line.
Step 3 Select a point in one of the half-planes
and test it. Try (0, 0).
yx7
007
true
0  7

343

y
x  3y  9
O
2 4 6 8 10

Chapter 7

PQ249-6481F-07[326-345] 8/6/02 09:51 PM Page 344 Sahuja Ahuja_QXP_03:Desktop Folder:6/8/02:

56. Step 1 Solve for y in terms of x.


y x
(1) (y)  (1)x
y  x
Step 2 Graph y  x. Since y  x means
y  x or y  x, the boundary is
included in the solution set. The
boundary should be drawn as a solid line.
Step 3 Select a point in one of the half-planes
and test it. Try (2, 1).
y  x
1  (2)
true
1  2
Since the statement is true, the half-plane
containing (2, 1) is part of the solution. Shade the
half-plane containing (2, 1).

Page 392

Practice Quiz 2

1. Since the coefficients of the y terms, 4 and 4,


are additive inverses, you can eliminate the y
terms by adding the equations.
5x  4y  2
() 3x  4y  14
8x
 16
8x
8

16
8

x2
Now substitute 2 for x in either equation to find
the value of y.
5x  4y  2
5(2)  4y  2
10  4y  2
10  4y  10  2  10
4y  8

4y
4

8
4

y  2
The solution is (2, 2).
2. Since the coefficients of the x terms, 2 and 2, are
the same, you can eliminate the x terms by
subtracting the equations.
2x  3y  13
() 2x  2y  2
5y  15

y  x

57. Step 1 Solve for y in terms of x.


3x  y  1
3x  y  3x  1  3x
y  3x  1
Step 2 Graph y  3x  1. Since y  3x  1 means
y  3x  1 or y  3x  1, the boundary is
included in the solution set. The boundary
should be drawn as a solid line.
Step 3 Select a point in one of the half-planes
and test it. Try (0, 0).
y  3x  1
0  3(0)  1
0  1
true
Since the statement is true, the half-plane
containing the origin is part of the solution.
Shade the half-plane containing (0, 0).

5y
5

15

 5

y  3
Now substitute 3 for y in either equation to find
the value of x.
2x  3y  13
2x  3(3)  13
2x  (9)  13
2x  9  13
2x  9  9  13  9
2x  4
2x
2

2

x2
The solution is (2, 3).
3. Eliminate y.
6x  2y  24 Multiply by 2.
12x  4y  48
3x  4y  27
() 3x  4y  27
15x
 75

y
3x  y  1

15x
15

x5
Now substitute 5 for x in either equation to find
the value of y.
6x  2y  24
6(5)  2y  24
30  2y  24
30  2y  30  24  30
2y  6

2y
2

6

 2

y3
The solution is (5, 3).

Chapter 7

75

 15

344

PQ249-6481F-07[326-345] 8/6/02 09:52 PM Page 345 Sahuja Ahuja_QXP_03:Desktop Folder:6/8/02:

5.

4. Eliminate x.
5x  2y  4 Multiply by 2.
10x  4y  8
10x  4y  9
() 10x  4y  9
0 1
The statement 0  1 is false. This means that
there is no solution of the system of equations.
5. Let p  the rate per minute for peak time, and
let n  the rate per minute for nonpeak time.
Kelseys charges:
45p  50n  27.75
Mitchs charges:
70p  30n  36
Eliminate n.

System of Equations
one variable has a
coefficient of 1 or 1
Substitution

opposite signs

 96.75
215 p
215

Elimination

Variables with
Same Coefficients

45p  50n  27.75 Multiply by 3.


135p  150n  83.25
70p  30n  36 Multiply by 5. () 350p  150n  180
215p

variables have
any coefficient

Variables with
Different
Coefficients

same signs

Add

Subtract

96.75

215

Multiply by a
Factor

p  0.45

Now substitute 0.45 for p in either equation to


find the value of n.
70p  30n  36
70(0.45)  30n  36
31.5  30n  36
31.5  30n  31.5  36  31.5
30n  4.5
30n
30

Graphing Systems of Inequalities

Page 395

Graphing Calculator Investigation

1. To use the SHADE feature in the DRAW menu,


you must enter the lower boundary of the region
to be shaded first. Since, when both inequalities
are solved for y, the lower boundary is the
inequality having  or , you must enter
y  2x  5 first.
2. KEYSTROKES: 2nd DRAW 7 ( ) 2 X,T,,n

4.5
30

n  0.15
The rate for peak time is $0.45 per min and the
rate for nonpeak time is $0.15 per min.

Page 393

7-5

Reading Mathematics

3 X,T,,n

ENTER

1. There are two types of systems of equations,


consistent and inconsistent. Consistent systems
have one or more solutions and inconsistent
systems have no solutions. If consistent systems
have one solution, they are called independent. If
consistent systems have infinite solutions, they
are called dependent.
2. Use substitution if an expression of one variable
is given or if the coefficient of a variable is
1.
Use elimination if both equations are written in
standard form.
Sample answers:
system to solve using
system to solve using
substitution
elimination
y  3x  3
4x  3y  9
5x  2y  6
6x  y  10
3. Multiply by a factor if neither variable has the
same or opposite coefficients in the two equations.
4. Add if one of the variables has opposite
coefficients that are additive inverses in the two
equations. Subtract if one of the variables has the
same coefficient in the two equations.

[10, 10] scl: 1 by [10, 10] scl: 1

3. Solve each inequality for y. Then enter the


function that is the lower boundary ( y  7  2x),
a comma, and the function that is the upper
boundary ( y 0.5x  2.5).
2 X,T,,n
4. KEYSTROKES: 2nd DRAW 7 7

,
)

5 X,T,,n

ENTER

[10, 10] scl: 1 by [10, 10] scl: 1

345

Chapter 7

PQ249-6481F-07[346_363] 8/6/02 09:42 PM Page 346 Sahuja Ahuja_QXP_03:Desktop Folder:6/8/02:

Pages 396397

4. The solution includes the ordered pairs in the


intersection of the graphs of x  5 and y  4. The
region is shaded. The graphs of x  5 and y  4
are boundaries of this region. The graph of x  5
is dashed and is not included in the graph of
x  5. The graph of y  4 is solid and is included
in the graph of y  4.

Check for Understanding

1. Sample answer:
y

y  2x
O

y
x5

y  2  x

y4

Because the regions have no points in common,


the system of inequalities graphed above,
y  2  x and y  2  x, has no solution.
2a. Since the point with coordinates (3, 1) lies in the
shaded region that represents the intersection of
the graphs of the inequalities, the ordered pair
(3, 1) does represent a solution of the system.
2b. Since the point with coordinates (1, 3) does
not lie in the shaded region that represents the
intersection of the graphs of the inequalities,
the ordered pair (1, 3) does not represent a
solution of the system.
2c. The point with coordinates (2, 3) lies on the
boundary of the region that represents the
intersection of the graphs of the inequalities.
Since the portion of the boundary on which (2, 3)
lies is drawn as a solid line, this point is
included in the solution set. So the ordered pair
(2, 3) does represent a solution of the system.
2d. Since the point with coordinates (4, 2) lies in
the shaded region that represents the
intersection of the graphs of the inequalities,
the ordered pair (4, 2) does represent a
solution of the system.
2e. The point with coordinates (3, 2) lies on the
boundary of the region that represents the
intersection of the graphs of the inequalities.
Since the portion of the boundary on which
(3, 2) lies is drawn as a dashed line, this point
is not included in the solution set. So the
ordered pair (3, 2) does not represent a
solution of the system.
2f. Since the point with coordinates (1, 4) does not
lie in the shaded region that represents the
intersection of the graphs of the inequalities, the
ordered pair (1, 4) does not represent a solution
of the system.
3. Kaylas solution is correct. The graph of
x  2y  2 is the line representing x  2y  2
and the region above it. Sonias solution indicates
that she shaded the region below this boundary.

Chapter 7

5. The solution includes the ordered pairs in the


intersection of the graphs of y  3 and
y  x  4. The region is shaded. The graphs of
y  3 and y  x  4 are boundaries of this
region. The graphs of both boundaries are dashed
and are not included in the solution.
y

y 3
O

y  x  4

6. The solution includes the ordered pairs in the


intersection of the graphs of y  x  3 and
y  x  3. The region is shaded. The graphs of
y  x  3 and y  x  3 are boundaries of this
region. The graphs of both boundaries are solid
and are included in the solution.
y
y  x  3
y  x 3
x
O

346

PQ249-6481F-07[346_363] 8/6/02 09:43 PM Page 347 Sahuja Ahuja_QXP_03:Desktop Folder:6/8/02:

10. Let x  the number of minutes Natasha spends


walking, and let y  the number of minutes she
spends jogging.
Since her total exercise time is at most 1 halfhour, or 30 min, x  y  30. Since her total
4
8
distance is at least 3 mi, 60 x  60 y  3.
The solution includes the ordered pairs in the
intersection of the graphs of x  y  30 and
4
8
x  60 y  3. The region is shaded. The graphs of
60
4
8
x  y  30 and 60 x  60 y  3 are boundaries of
this region. The graphs of both boundaries are
solid and are included in the solution.

7. The graphs of 2x  y  4 and y  2x  1 are


parallel lines. Because the two regions have no
points in common, the system of inequalities has
no solution.
y

2x  y  4

y  2x  1

Natashas Daily Exercise


y
Minutes Jogging

8. The solution includes the ordered pairs in the


intersection of the graphs of 2y  x 6 6 and
3x  y 7 4. The region is shaded. The graphs of
2y  x  6 and 3x  y  4 are boundaries of this
region. The graphs of both boundaries are dashed
and are not included in the solution.
y

40
30

x  y  30

20

4
x 8 y3
60
60

10

3x  y  4

2y  x  6

11. Sample answers: Natasha could walk 15 min and


jog 15 min, or walk 10 min and jog 20 min, or she
could walk 5 min and jog 25 min since the points
with coordinates (15, 15), (10, 20), and (5, 25) lie
in the region that represents the intersection of
the graphs of the inequalities.

x
O

9. The solution includes the ordered pairs in the


intersection of the graphs of x  2y  2,
3x  4y  12, and x  0. The region is shaded.
The graphs of x  2y  2, 3x  4y  12, and x  0
are boundaries of this region. The graphs of these
boundaries are solid and are included in the
solution.
y

10 20 30 40
Minutes Walking

Pages 397398
12.

Practice and Apply

y
x

y0

x0
3x  4y  12

x0

x
O

x  2y  2

13.

y
x  4
O

y  1

347

Chapter 7

PQ249-6481F-07[346_363] 8/6/02 09:44 PM Page 348 Sahuja Ahuja_QXP_03:Desktop Folder:6/8/02:

14.

20.

y  x 1
x

y x 3
y x 2

y  2

15.

21.

x 2
3x  y  6

x
2x  y  4

y x 5

16.

22.

x 3

x y 2

3x  4y  1

x  2y  7

x
O

17.

23.

y
xy 4

y  2x  1

y  x  1

2x  3y  12

18.

24.

y  x  3

2x  y  4

5x  2y  1

y  2x  1

19.

25.

y  x 1

x
x
2x  7y  4

Chapter 7

3x  2y  6

y  x 3

y x 3

348

PQ249-6481F-07[346_363] 8/6/02 09:44 PM Page 349 Sahuja Ahuja_QXP_03:Desktop Folder:6/8/02:

26.

The solution is the set of all ordered pairs whose


graphs are in the intersection of the graphs of
these inequalities. This region is shaded. Only the
portion of the region in the first quadrant is used
since x  0 and y  0.

x  3y  9
y
x 2

2x  y  9

Green Paint

x
4y  x

y
Dark Green

27. The equation of the graph of the solid boundary is


y  x. Since the shaded region that represents the
intersection of the graphs of the two inequalities
is below this boundary, this region is represented
by y  x. The equation of the graph of the dashed
boundary is y  x  3. Since the shaded region
that represents the intersection of the graphs of
the two inequalities is above this boundary, this
region is represented by y 7 x  3. So, the
system graphed is y  x
.
y 7 x3
28. The equation of the graph of the solid boundary is
2
y  3x  2. Since the shaded region that
represents the intersection of the graphs of the
two inequalities is below this boundary, this
2
region is represented by y  3x  2. The equation
of the graph of the dashed boundary is
y  x  3. Since the shaded region that
represents the intersection of the graphs of the
two inequalities is above this boundary, this
region is represented by y 7 x  3. So, the
2
system graphed is y  3x  2
.
y 7 x  3
29. Let x  the number of gal of light green dye to be
made. Then, for the light green dye, 4x represents
the number of units of yellow dye required and 1x
represents the number of units of blue dye
required.
Let y  the number of gal of dark green dye to be
made. Then, for the dark green dye, 1y represents
the number of units of yellow dye required and 6y
represents the number of units of blue dye
required.
Since the total number of units of yellow dye
cannot be greater than 32, 4x  1y  32. Since
the total number of units of blue dye cannot be
greater than 54, 1x  6y  54.
The following system of inequalities can be used
to represent the conditions of the problem.
x0
y0
4x  y  32
x  6y  54

30
4x  y  32

20

x  6y  54

10

x
O

10

20 30 40
Light Green

50

30. Sample answers: The painter could make 2 gal of


light green dye and 8 gal of dark green dye, or
6 gal of light green dye and 8 gal of dark green
dye, or 7 gal of light green dye and 4 gal of dark
green dye since the points with coordinates (2, 8),
(6, 8), and (7, 4) lie in the region that represents
the intersection of the graphs of the inequalities.
31. Let x  the level of LDL a teenager should have,
and let y  the level of HDL a teenager should
have.
The following system of inequalities can be used
to represent the conditions of the problem.
0  x  110
35  y  59
The solution is the set of all ordered pairs whose
graphs are in the intersection of the graphs of
these inequalities. This region is shaded. Only the
portion of the region in the first quadrant is used
since there can be only positive levels of LDL and
HDL.

Appropriate
Cholesterol Levels
y

HDL

60
40
20
O

20

40

60 80
LDL

100 120 x

32. Sample answer: x  4 and x  4

349

Chapter 7

PQ249-6481F-07[346_363] 8/6/02 09:44 PM Page 350 Sahuja Ahuja_QXP_03:Desktop Folder:6/8/02:

33. Let x  the number of desks made. Then, 2x


represents the number of hours of sanding
required and 1.5x represents the number of hours
of varnishing required for the desks.
Let y  the number of tables made. Then, 1.5y
represents the number of hours of sanding
required and 1y represents the number of hours
of varnishing required for the tables.
Since the total number of hours available each
week for sanding is 31, 2x  1.5y  31 . Since the
total number of hours available for varnishing
each week is 22, 1.5x  1y  22.
The following system of inequalities can be used
to represent the conditions of the problem.
x0
y0
2x  1.5y  31
1.5x  y  22
The solution is the set of all ordered pairs whose
graphs are in the intersection of the graphs of
these inequalities. This region is shaded. Only the
portion of the region in the first quadrant is used
since x  0 and y  0. However, since the number
of items produced are whole numbers, only wholenumber solutions make sense for this problem.

36. Use the SHADE feature in the DRAW menu with


y  x  4 as the lower boundary and y  x  9
as the upper boundary.
DRAW 7 ( ) X,T,,n
KEYSTROKES: 2nd

X,T,,n

ENTER

[10, 10] scl: 1 by [10, 10] scl: 1

37. Use the SHADE feature in the DRAW menu with


y  7x  15 as the lower boundary and
y  2x  10 as the upper boundary.
KEYSTROKES:

2nd

DRAW 7 7 X,T,,n
15 , 2 X,T,,n
10

ENTER

Furniture Manufacturing

Tables

20

1.5x  y  22

[10, 10] scl: 1 by [10, 10] scl: 1

38. Solve each inequality for y.


10

3x  y  6
3x  y  3x  6  3x
y  6  3x

2x  1.5y  31

10

20

(1)(y)  (1)(6  3x)

(1)(y)  (1)(1  x)

y  3x  6

Desks

y1x

Use the SHADE feature in the DRAW menu with


y  3x  6 as the lower boundary and y  1  x
as the upper boundary.
DRAW 7 3 X,T,,n
KEYSTROKES: 2nd

34. Sample answers: The company can make 8 desks


and 10 tables, or 6 desks and 12 tables, or 4 desks
and 14 tables since the points with coordinates
(8, 10), (6, 12), and (4, 14) lie in the region that
represents the intersection of the graphs of the
inequalities.
35. By graphing a system of equations, you can see
the appropriate range of Calories and fat intake.
Answers should include the following.
Two sample appropriate Calorie and fat
intakes are 2200 Calories and 60 g of fat and
2300 Calories and 65 g of fat since the points
with coordinates (2200, 60) and (2300, 65) lie
in the shaded region representing appropriate
eating habits.
The graph represents 2000  c  2400
and 60  f  75, where c represents the
number of Calories and f represents the
number of grams of fat consumed per day.

Chapter 7

x  y  1
x  y  x  1  x
y  1  x

X,T,,n

[10, 10] scl: 1 by [10, 10] scl: 1

350

ENTER

PQ249-6481F-07[346_363] 8/6/02 09:44 PM Page 351 Sahuja Ahuja_QXP_03:Desktop Folder:6/8/02:

42. Multiply the first equation by 3 so the coefficients


of the y terms are additive inverses. Then add the
equations.
5x  2y  3 Multiply by 3.
15x  6y  9
() 3x  6y  9
3x  6y  9
18x
 18

39. D; Use a table to substitute the x and y values of


each ordered pair into both inequalities.
x
3
0
1
0

x  2y  5

3  2(7)
11
5
0  2(5)
10
4
1  2(4)
7
2.5 0  2(2.5)
5
7

5
5
5
5
5
5
5
5

True or
False
true
true
true
false

3x  y  2
3(3)  7
16
3(0)  5
5
3(1)  4
7
3(0) 2.5
2.5

True or
False

 2
 2
 2
 2
 2
 2
 2
 2

true

18x
18

true

true

2y
2

Multiply by 2.

2x  3y  13

Multiply by 3.

6x  4y 

24

() 6x  9y  39
5y  15


15
5

y3

Now substitute 3 for y in either equation to find


the value of x.
3x  2y  12
3x  2(3)  12
3x  6  12
3x  6  6  12  6
3x  6
3x
3

 3

x  2
The solution is (2, 3).
44. Eliminate y.

11
11

y  1
Now substitute 1 for y in either equation to
find the value of x.
2x  3y  1
2x  3(1)  1
2x  3  1
2x  3  3  1  3
2x  4
2x
2

3x  2y  12

5y
5

Maintain Your Skills

 2

y  1
The solution is (1, 1).
43. Eliminate x.

41. Multiply the first equation by 2 so the


coefficients of the x terms are additive inverses.
Then add the equations.
2x  3y  1 Multiply by 2.
4x  6y  2
() 4x  5y  13
4x  5y  13
11y  11
11y
11

18
18

x  1
Now substitute 1 for x in either equation to find
the value of y.
5x  2y  3
5(1)  2y  3
5  2y  3
5  2y  5  3  5
2y  2

true

Since it does not make both inequalities true, the


ordered pair (0, 2.5) is not part of the solution set
of the system.
40. A; The equation of the graph of the solid
boundary is y  2x  2. Since the shaded region
representing the intersection of the graphs of the
two inequalities is below this boundary, this
region is represented by y  2x  2. The equation
of the graph of the dashed boundary is
y  x  1. Since the shaded region representing
the intersection of the graphs of the two
inequalities is above this boundary, this region is
represented by y 7 x  1. So, the system
graphed is y  2x  2 .
y 7 x  1

Page 398

6x  2y  4
5x  3y  2

Multiply by 3.
Multiply by 2.

18x  6y  12
() 10x  6y  4
8x
 16
8x
8

16
8

x2

Now substitute 2 for x in either equation to find


the value of y.
6x  2y  4
6(2)  2y  4
12  2y  4
12  2y  12  4  12
2y  8

2

x2
The solution is (2, 1).

2y
2

8

 2

y4
The solution is (2, 4).

351

Chapter 7

PQ249-6481F-07[346_363] 8/6/02 09:44 PM Page 352 Sahuja Ahuja_QXP_03:Desktop Folder:6/8/02:

48. Step 1 The line has slope 6. To find the


y-intercept, replace m with 6 and (x, y)
with (1, 0) in the slope-intercept form.
Then, solve for b.
y  mx  b
0  6(1)  b
0  6  b
0  6  6  b  6
6b
Step 2 Write the slope-intercept form using
m  6 and b  6. Therefore, the
equation is y  6x  6.

45. Since the coefficients of the y terms, 5 and 5,


are additive inverses, you can eliminate the y
terms by adding the equations.
2x  5y  13
() 3x  5y  18
5x
 5
5x
5

5
5

x  1
Now substitute 1 for x in either equation to find
the value of y.
2x  5y  13
2(1)  5y  13
2  5y  13
2  5y  2  13  2
5y  15
5y
5

49. Step 1 The line has slope 3. To find the


1
y-intercept, replace m with 3 and (x, y)
with (5, 2) in the slope-intercept form.
Then, solve for b.
y  mx  b

15
5

y3
The solution is (1, 3).
46. Since the coefficients of the x terms, 3 and 3, are
the same, you can eliminate the x terms by
subtracting the equations.
3x  y  6
() 3x  2y  15
3y  9
3y
3

2  3 (5)  b
5

2  3  b
5

11

3  b
Step 2 Write the slope-intercept form using
1
11
.
m  3 and b   3

9

 3

Therefore, the equation is y  3x 

y3
Now substitute 3 for y in either equation to find
the value of x.
3x  y  6
3x  3  6
3x  3  3  6  3
3x  9
3x
3

2  3  3  b  3

11
.
3

Chapter 7 Study Guide and Review


Page 399
1.
2.
3.
4.
5.
6.

3

x3
The solution is (3, 3).
47. Step 1 The line has slope 2. To find the
y- intercept, replace m with 2 and (x, y)
with (4, 1) in the slope-intercept form.
Then, solve for b.
y  mx  b
1  2(4)  b
1  8  b
1  8  8  b  8
9  b
Step 2 Write the slope-intercept form using
m  2 and b  9.
y  mx  b
y  2x  (9)
Therefore, the equation is y  2x  9.

Vocabulary and Concept Check

independent
inconsistent
dependent
parallel lines
infinitely many
consistent

Pages 399402
7.

y
8
6
4
2
2
2
4
6
8

Lesson-by-Lesson Review
x  y  11

(10, 1)
O
2 4 6 8 10 12 14 x

xy9

The graphs appear to intersect at (10, 1).


Check in each equation.
Check:
xy9
x  y  11
?
?
10  1  9
10  1  11
99
11  11
There is one solution. It is (10, 1).

Chapter 7

352

PQ249-6481F-07[346_363] 8/6/02 09:45 PM Page 353 Sahuja Ahuja_QXP_03:Desktop Folder:6/8/02:

8.
y  3x  8

Substitute 5 for n in either equation to find the


value of m.
mn8
m  (5)  8
m58
m5585
m3
The solution is (3, 5).
12. Since x  3  2y, substitute 3  2y for x in the
second equation.
2x  4y  6
2(3  2y)  4y  6
6  4y  4y  6
66
The statement 6  6 is true. This means that
there are infinitely many solutions of the system
of equations.
13. Solve the first equation for y since the coefficient
of y is 1.
3x  y  1
3x  y  y  1  y
3x  1  y
3x  1  1  y  1
3x  1  y
Since y  3x  1, substitute 3x  1 for y in the
second equation.
2x  4y  3
2x  4(3x  1)  3
2x  12x  4  3
14x  4  3
14x  4  4  3  4
14x  7

9x  2  3y

The graphs of the equations are parallel lines.


Since they do not intersect, there are no solutions
to this system of equations.
9.

6y  4x  8

2x  3y  4

The graphs of the equations coincide. Since every


point is a point of intersection, there are infinitely
many solutions to this system of equations.
10.

y
3x  y  8

O
(2, 2)

14x
14

3x  4  y

 14
1

x2
1

The graphs appear to intersect at (2, 2). Check


in each equation.
Check:
3x  y  8
3x  4  y
?
?
3(2)  (2)  8
3(2)  4  (2)
?
?
628
642
88
66
There is one solution. It is (2, 2).
11. Solve the second equation for m since the
coefficient of m is 1.
mn8
mnn8n
m8n
Since m  8  n, substitute 8  n for m in the
first equation.
2m  n  1
2(8  n)  n  1
16  2n  n  1
3n  16  1
3n  16  16  1  16
3n  15
3n
3

Substitute 2 for x in either equation to find the


value of y.
y  3x  1
y3
3

112 2  1

y21
1

y2
The solution is

112, 12 2.

15
3

n  5

353

Chapter 7

PQ249-6481F-07[346_363] 8/6/02 09:45 PM Page 354 Sahuja Ahuja_QXP_03:Desktop Folder:6/8/02:

14. Since n  4.5  3m, substitute 4.5  3m for n in


the first equation.
0.6m  0.2n  0.9
0.6m  0.2(4.5  3m)  0.9
0.6m  0.9  0.6m  0.9
1.2m  0.9  0.9
1.2m  0.9  0.9  0.9  0.9
1.2m  1.8
1.2m
1.2

Now substitute 4 for x in either equation to find


the value of y.
xy5
4y5
4y454
y1
The solution is (4, 1).
18. Rewrite the first equation so the system is in
column form.
3x  1  7y
3x  1  1  7y  1
3x  7y  1
3x  7y  7y  1  7y
3x  7y  1
Since the coefficients of the y terms, 7 and 7, are
the same, you can eliminate the y terms by
subtracting the equations.
3x  7y  1
() 6x  7y  0
3x
 1

1.8

 1.2

m  1.5
Use n  4.5  3m to find the value of n.
n  4.5  3m
n  4.5  3(1.5)
n  4.5  4.5
n0
The solution is (1.5, 0).
15. Since the coefficients of the x terms, 1 and 1, are
the same, you can eliminate the x terms by
subtracting the equations.
x  2y  6
() x  3y  4
5y  10
5y
5

3x
3

x3

10
5

Now substitute
the value of y.
6x  7y  0

y2
Now substitute 2 for y in either equation to find
the value of x.
x  2y  6
x  2(2)  6
x46
x4464
x2
The solution is (2, 2).
16. Since the coefficients of the n terms, 1 and 1,
are additive inverses, you can eliminate the n
terms by adding the equations.
2m  n  5
() 2m  n  3
4m
8
4m
4

for x in either equation to find

113 2  7y  0

2
7
2
y  7
1
solution is 3,
7y
7

The

7 .

19. Multiply the first equation by 2 so the


coefficients of the x terms are additive inverses.
Then add the equations.
x  5y  0 Multiply by 2.
2x  10y  0
2x  3y  7
() 2x  3y  7
7y  7

4

7y
7

7

y1
Now substitute 1 for y in either equation to find
the value of x.
x  5y  0
x  5(1)  0
x50
x5505
x5
The solution is (5, 1).

16
4

x4
Chapter 7

1
3

2  7y  0
2  7y  2  0  2
7y  2

m2
Now substitute 2 for m in either equation to find
the value of n.
2m  n  5
2(2)  n  5
4n5
4n454
n  1
(1)(n)  (1)1
n  1
The solution is (2, 1).
17. Since the coefficients of the y terms, 1 and 1,
are additive inverses, you can eliminate the y
terms by adding the equations.
3x  y  11
() x  y  5
4x
 16
4x
4

1

 3

354

PQ249-6481F-07[346_363] 8/6/02 09:45 PM Page 355 Sahuja Ahuja_QXP_03:Desktop Folder:6/8/02:

20. Multiply the first equation by 3 so the


coefficients of the x terms are additive inverses.
Then add the equations.
x  2y  5 Multiply by 3.
3x  6y  15
3x  5y  8
() 3x  5y 
8
y  7
Now substitute 7 for y in either equation to find
the value of x.
x  2y  5
x  2(7)  5
x  (14)  5
x  14  5
x  14  14  5  14
x  9
The solution is (9, 7) .
21. Multiply the second equation by 3 so the
coefficients of the y terms are additive inverses.
Then add the equations.
2x  3y  8
2x  3y  8
x  y  2 Multiply by 3.
() 3x  3y  6
5x
 14
5x
5

x
Now substitute
the value of y.
xy2
14
5
14
5

14
5

14
5
14
5

23. For an exact solution, an algebraic method is


best.
Since the coefficient of y in the first equation
is 1, you can use the substitution method.
Since y  2x, substitute 2x for y in the second
equation.
x  2y  8
x  2(2x)  8
x  4x  8
5x  8
5x
5

14
5

2
4

y  5

Use y  2x to find the value of y.


y  2x
y2
y

or 25

14
5

1 42
2

The solution is 25, 5 .


22. Multiply the first equation by 2 so the coefficients
of the x terms are additive inverses. Then add the
equations.
5x  8y  21 Multiply by 2. 10x  16y  42
10x  3y  15
() 10x  3y  15
19y  57
19y
19

or 35

31y
31

 31

y0
Now substitute 0 for y in either equation to find
the value of x.
9x  8y  7
9x  8(0)  7
9x  7

57

 19

y3
Now substitute 3 for y in either equation to find
the value of x.
5x  8y  21
5x  8(3)  21
5x  24  21
5x  24  24  21  24
5x  3
5x
5

16
5

24. For an exact solution, an algebraic method is


best.
Since neither the coefficients of x nor the
coefficients of y are the same or additive
inverses, you cannot use elimination using
addition or subtraction.
Since no coefficient of x or y is 1 or 1,
substitution is not the best method.
Since it is necessary to multiply at least one
equation by a number to eliminate a variable,
solve the system by elimination using
multiplication.
Multiply the first equation by 2 so the
coefficients of the x terms are additive inverses.
Then add the equations.
Multiply by 2. 18x  16y  14
9x  8y  7
18x  15y  14
() 18x  15y  14
31y 
0

for x in either equation to find

4 4

185 2

The solution is 15 , 35 .

(1)(y)  (1) 5
y5

x  5 or 15

y2

y

5

9x
9

9
7

x9
The solution is

179, 02.

3

 5
3

x5
The solution is

135, 32.

355

Chapter 7

PQ249-6481F-07[346_363] 8/6/02 09:45 PM Page 356 Sahuja Ahuja_QXP_03:Desktop Folder:6/8/02:

27. The solution includes the ordered pairs in the


intersection of the graphs of y  3x and x  2y 
21. The region is shaded. The graphs of y  3x
and x  2y  21 are boundaries of this region.
The graph of y  3x is dashed and is not included
in the graph of y  3x. The graph of x  2y  21
is solid and is included in the graph of x  2y  21.

25. For an exact solution, an algebraic method is


best.
Since the coefficient of x in the second
equation is 1, you can use the substitution
method.
Solve the second equation for x.
x  3y  y
x  3y  3y  y  3y
x  2y
Since x  2y, substitute 2y for x in the first
equation.
3x  5y  2x
3(2y)  5y  2(2y)
6y  5y  4y
y  4y
y  4y  4y  4y
3y  0
3y
3

y
2
12 8

2
4
6
8
x  2y  21 10
12
y  3x
14

4x

y
y  2x  1
x

y  x  1

29. The solution includes the ordered pairs in the


intersection of the graphs of 2x  y  9 and
x  11y  6. The region is shaded. The graphs
of 2x  y  9 and x  11y  6 are boundaries of
this region. The graphs of both boundaries are
dashed and are not included in the solution.
y
2x  y  9

5x
5

13  x
Substitute 13 for x in either equation to find the
value of y.
y  x  15
y  13  15
y  2
The solution is (13, 2).

Chapter 7

28. The solution includes the ordered pairs in the


intersection of the graphs of y  x  1 and
y  2x  1. The region is shaded. The graphs of
y  x  1 and y  2x  1 are boundaries of this
region. The graph of y  x  1 is dashed and is
not included in the graph of y  x  1. The
graph of y  2x  1 is solid and is included in the
graph of y  2x  1.

3

y0
Substitute 0 for y in either equation to find the
value of x.
x  2y
x  2(0)
x0
The solution is (0, 0).
26. For an exact solution, an algebraic method is
best.
Since the coefficient of y in the first equation
is 1, you can use the substitution method.
Solve the first equation for y.
2x  y  3x  15
2x  y  2x  3x  15  2x
y  x  15
Since y  x  15, substitute x  15 for y in the
second equation.
x  5  4y  2x
x  5  4(x  15)  2x
x  5  4x  60  2x
x  5  6x  60
x  5  x  6x  60  x
5  5x  60
5  60  5x  60  60
65  5x
65
5

4

x  11y  6

356

PQ249-6481F-07[346_363] 8/6/02 09:45 PM Page 357 Sahuja Ahuja_QXP_03:Desktop Folder:6/8/02:

6.

30. The solution includes the ordered pairs in the


intersection of the graphs of x  1 and y  x  3.
The region is shaded. The graphs of x  1 and
y  x  3 are boundaries of this region. The
graphs of both boundaries are solid and are
included in the solution.

2y  10  6x
3x  y  5

x1

The graphs of the equations coincide. Since every


point is a point of intersection, there are infinitely
many solutions to this system of equations.
7. Eliminate y.
2x  5y  16 Multiply by 2.
4x  10y  32
5x  2y  11 Multiply by 5. () 25x  10y  55
29x
 87

yx3

29x
29

Chapter 7 Practice Test

x3
Now substitute 3 for x in either equation to find
the value of y.
2x  5y  16
2(3)  5y  16
6  5y  16
6  5y  6  16  6
5y  10

Page 403
1. c; inconsistent
2. a; consistent
3. b; elimination
4.

y
O

5y
5

y  2x  7

The graphs appear to intersect at (5, 3). Check


in each equation.
Check:
yx2
y  2x  7
?
?
3  5  2
3  2(5)  7
?
3  3
3  10  7
3  3
There is one solution. It is (5, 3).
7
6
5
4
3
2
1
2
1

10
5

y2
The solution is (3, 2).
8. Solve the second equation for y.
y  4  2x
y  4  4  2x  4
y  2x  4
Since y  2x  4, substitute 2x  4 for y in
the first equation.
y  2x  1
(2x  4)  2x  1
4  1
The statement 4  1 is false. This means there
is no solution of the system of equation.
9. Multiply the first equation by 3 so the
coefficients of the y terms are additive inverses.
Then add the equations.
2x  y  4 Multiply by 3. 6x  3y  12
() 5x  3y  6
5x  3y  6
x
 6
(1) (x)  (1)6
x  6
Now substitute 6 for x in either equation to find
the value of y.
2x  y  4
2(6)  y  4
12  y  4
12  y  12  4  12
y8
The solution is (6, 8).

yx2
(5, 3)

5.

87

 29

y
x  2y  11
x  14  2y

x
O
2 4 6 8 10 12 14

The graphs of the equations are parallel lines.


Since they do not intersect, there are no solutions
to this system of equations.

357

Chapter 7

PQ249-6481F-07[346_363] 8/6/02 09:45 PM Page 358 Sahuja Ahuja_QXP_03:Desktop Folder:6/8/02:

Now substitute 2 for x in either equation to find


the value of y.
3x  y  11
3(2)  y  11
6  y  11
6  y  6  11  6
y  17
(1)(y)  (1)17
y  17
The solution is (2, 17).
14. Since the coefficients of the x terms, 3 and 3, are
the same, you can eliminate the x terms by
subtracting the equations.
3x  y  10
() 3x  2y  16
3y  6

10. Since y  7  x, substitute 7  x for y in the


second equation.
x  y  3
x  (7  x)  3
x  7  x  3
2x  7  3
2x  7  7  3  7
2x  4
2x
2

2

x2
Use y  7  x to find the value of y.
y7x
y72
y5
The solution is (2, 5).
11. Since x  2y  7, substitute 2y  7 for x in the
second equation.
y  3x  9
y  3(2y  7)  9
y  6y  21  9
5y  21  9
5y  21  21  9  21
5y  30
5y
5

3y
3

30
5

3x
3

3x
3

3y
3

3

3

 3

y1
The solution is (3, 1).

14
7

x  2

Chapter 7

12
3

x3
Now substitute 3 for x in either equation to find
the value of y.
5x  3y  12
5(3)  3y  12
15  3y  12
15  3y  15  12  15
3y  3

x6
Now substitute 6 for x in either equation to find
the value of y.
x  y  10
6  y  10
6  y  6  10  6
y4
The solution is (6, 4).
13. Multiply the first equation by 2 so the coefficients
of the y terms are additive inverses. Then add the
equations.
Multiply by 2.
3x  y  11
6x  2y  22
x  2y  36
() x  2y  36
7x
 14


x4
The solution is (4, 2).
15. Since the coefficients of the y terms, 3 and 3,
are additive inverses, you can eliminate the y
terms by adding the equations.
5x  3y  12
() 2x  3y  3
3x
 9

12
2

7x
7

6
3

y  2
Now substitute 2 for y in either equation to find
the value of x.
3x  y  10
3x  (2)  10
3x  2  10
3x  2  2  10  2
3x  12

y6
Use x  2y  7 to find the value of x.
x  2y  7
x  2(6)  7
x  12  7
x5
The solution is (5, 6).
12. Since the coefficients of the y terms, 1 and 1,
are additive inverses, you can eliminate the y
terms by adding the equations.
x  y  10
() x  y  2
2x
 12
2x
2

358

PQ249-6481F-07[346_363] 8/6/02 09:45 PM Page 359 Sahuja Ahuja_QXP_03:Desktop Folder:6/8/02:

19. Let t  the tens digit, and let u  the units


digit.
u  2t  1
u  t  10
Since u  2t  1, substitute 2t  1 for u in the
second equation.
u  t  10
(2t  1)  t  10
3t  1  10
3t  1  1  10  1
3t  9

16. Multiply the second equation by 2 so the


coefficients of the x terms are additive inverses.
Then add the equations.
2x  5y  12
2x  5y  12
x  6y  11 Multiply by 2. () 2x  12y  22
17y  34
17y
17

34

 17

y2
Now substitute 2 for y in either equation to find
the value of x.
x  6y  11
x  6(2)  11
x  12  11
x  12  12  11  12
x1
The solution is (1, 2).
17. Multiply the first equation by 3 so the
coefficients of the x terms are additive inverses.
Then add the equations.
x  y  6 Multiply by 3.
3x  3y  18
3x  3y  13
() 3x  3y  13
6y  5
6y
6

3t
3

t3
Use u  2t  1 to find the value of u.
u  2t  1
u  2(3)  1
u61
u7
The tens digit is 3 and the units digit is 7. So, the
number is 37.
20. Let /  the length of the rectangle and let
w  the width of the rectangle.
/w7
2/  2w  50
Solve the first equation for /.
/w7
/ww7w
/7w
Since /  7  w, substitute 7  w for / in the
second equation.
2/  2w  50
2(7  w)  2w  50
14  2w  2w  50
14  4w  50
14  4w  14  50  14
4w  36

5

 6
5

y6
Now substitute
the value of x.
xy6

5
6

for y in either equation to find

x66
5

x6666
1

x  56

The solution is 56 ,

5
6

2.

18. Multiply the first equation by 5 so the coefficients


of the y terms are additive inverses. Then add the
equations.
1

15x  3y  50

() 2x  3 y  35

3x  3 y  10 Multiply by 5.

4w
4

 85

17x
17x
17

36
4

w9
Substitute 9 for w in either equation to find the
value of /.
/7w
/79
/  16
The length of the rectangle is 16 cm and the
width of the rectangle is 9 cm.

2x  3 y  35

3

85

 17

x5
Now substitute 5 for x in either equation to find
the value of y.
1

3x  3 y  10
1

3(5)  3 y  10
1

15  3 y  10
1

15  3 y  15  10  15
1
y
3
1
(3) 3 y

 5
 (3)(5)

y  15
The solution is (5, 15).

359

Chapter 7

PQ249-6481F-07[346_363] 8/6/02 09:46 PM Page 360 Sahuja Ahuja_QXP_03:Desktop Folder:6/8/02:

Since x  10,000  y, substitute 10,000  y for x


in the second equation.
0.06x  0.08y  760
0.06(10,000  y)  0.08y  760
600  0.06y  0.08y  760
600  0.02y  760
600  0.02y  600  760  600
0.02y  160

21. The solution includes the ordered pairs in the


intersection of the graphs of y 7 4 and y 6 1.
The region is shaded. The graphs of y  4 and
y  1 are boundaries of this region. The graphs
of both boundaries are dashed and are not
included in the solution.
y

0.02y
0.02

y  8000
Substitute 8000 for y in either equation to find
the value of x.
x  10,000  y
x  10,000  8000
x  2000
$2000 was invested at 6% and $8000 was
invested at 8%.
25. D; For the system
y 7 2x  1
, both boundary lines should be
y 6 x  2
dashed. The region representing the intersection
of the graphs of these two inequalities is above
the graph of y  2x  1 and below the graph of
y  x  2. To check, test an ordered pair in this
region to verify that the coordinates satisfy both
inequalities. For example, check (3, 2).
Check:
y 7 2x  1
y 6 x  2
?
?
2 6 (3)  2
2 7 2(3)  1
?
?
2 6 3  2
2 7 6  1
2 7 5
2 6 1

y  4

y  1

22. The solution includes the ordered pairs in the


intersection of the graphs of y  3 and
y 7 x  2. The region is shaded. The graphs of
y  3 and y  x  2 are boundaries of this
region. The graph of y  3 is solid and is included
in the graph of y  3. The graph of y  x  2 is
dashed and is not included in the graph of
y 7 x  2.
y

y3

y  x  2
x

Chapter 7 Standardized Test Practice

23. The solution includes the ordered pairs in the


intersection of the graphs of x  2y and
2x  3y  7. The region is shaded. The graphs of
x  2y and 2x  3y  7 are boundaries of this
region. The graphs of both boundaries are solid
and are included in the solution.

Pages 404405
1. B;
4x  2(x  2)  8  0
4x  2x  4  8  0
2x  4  0
2x  4  4  0  4
2x  4

y
2x  3y  7

2x
2

x
O

2

x2
2. C;
Let p  the price of the CD before tax.
p  0.07p  17.11
1.07p  17.11

x  2y

24. Let x  the amount invested at 6% and let


y  the amount invested at 8%.
x  y  10,000
0.06x  0.08y  760
Solve the first equation for x.
x  y  10,000
x  y  y  10,000  y
x  10,000  y

1.07p
1.07

17.11
1.07

p  15.99 (to the nearest hundredth)


The price of the CD before tax was $15.99.
3. B;
f(x)  2x  3
f(3)  2(3)  3
63
3

f(x)  2x  3
f(4)  2(4)  3
83
5

The range is {3, 5, 7}.

Chapter 7

160

 0.02

360

f(x)  2x  3
f(5)  2(5)  3
 10  3
7

PQ249-6481F-07[346_363] 8/6/02 09:46 PM Page 361 Sahuja Ahuja_QXP_03:Desktop Folder:6/8/02:

4. D;

2

1

7. D;

2



Number of hours, x
Number of birds, y

1
6

3
14

4
18

Tamikas
her car
current
her
payment
balance plus deposit minus withdrawal is at least $200.
14243 123 14243 1
424
3 1442443 14
424
43 1
424
3

6
26

185



8
4
8

The difference in y values is four times the


difference of the x values. This suggests that
y  4x. Check this equation.
Check:
If x  1, then y  4(1) or 4. But the y
value for x  1 is 6. This is a difference
of 2. Try some other values in the
domain to see if the same difference
occurs.
1
4
6

3
12
14

4
16
18

6
24
26

y is always 2
more than 4x.

x
4x
y

This pattern suggests that 2 should be added to


one side of the equation in order to correctly
describe the relation. Check y  4x  2.
If x  3, then y  4(3)  2 or 14.
If x  6, then y  4(6)  2 or 26.
Thus, y  4x  2 correctly describes the relation.
5. C;
Step 1 The coordinates of two points on the line
are (3, 0) and (0,4). Find the slope. Let
(x1, y1)  (3, 0) and (x2, y2)  (0, 4).
1

4  0
 (3)
4
3

m0

The slope is 3.


Step 2 The line crosses the y-axis at (0, 4). So,
the y-intercept is 4.
Step 3 Write the equation.
y  mx  b

(3)y  (3) 3x  4

200

58
2

10
2

y5
The value of y is 5.
11. Use the formula V  / w h.

4
4

2y
2

y  3x  (4)
y  3x  4

230

s  29
Use j  s  6 to find the value of j.
js6
j  29  6
j  35
The jeans cost $35.
10. C;
Since the coefficients of the x terms, 3 and 3, are
the same, you can eliminate x by subtracting the
equations.
3x  4y  8
() 3x  2y  2
2y  10

y  y

m

2s
2

m  x2  x1
2

8. B;
Since the perimeter is 68 ft, 2/  2w  68.
Since the length is 4 more than twice the width,
/  2w  4.
9. C;
Let j  the cost of the jeans and let s  the cost of
the shirt. Since Ernestos total cost is $64, j  s 
64. Since the jeans cost $6 more than the shirt,
j  s  6.
Since j  s  6, substitute s  6 for j in the first
equation.
j  s  64
(s  6)  s  64
2s  6  64
2s  6  6  64  6
2s  58

2 1

Volume  242 8 9  22 8 6

 1764  120
 1644
The volume is 1644 ft3.
12. (5x  6)  (5x  6)  (5x  6)
 (5x  6)  204
20x  24  204
20x  24  24  204  24
20x  180

3y  4x  12
3y  4x  4x  12  4x
3y  4x  12
The equation is 3y  4x  12.
6. B;
A line parallel to the graph of y  3x  6 has the
same slope.
y  3x  6
y  3x  3x  6  3x
y  3x  6
The slope of the line is 3.

20x
20

180
20

x9
The value of x is 9.

361

Chapter 7

PQ249-6481F-07[346_363] 8/6/02 09:46 PM Page 362 Sahuja Ahuja_QXP_03:Desktop Folder:6/8/02:

13. To find the x-intercept, let y  0.


4x  3y  12
4x  3(0)  12
4x  12
4x
4

Column B: Let (2, 1)  (x1, y1) and


(5, 3)  (x2, y2).
y  y

m  x2  x1
2

5

12
4

x3
The x-intercept is 3.
14. Write the equation in slope-intercept form
y  mx  b.
4x  2y  5
4x  2y  4x  5  4x
2y  5  4x
2y
2

y
y
y

7
1

15. Multiply the first equation by 2 so the


coefficients of the y terms are additive inverses.
Then add the equations.
10x  2y  20
() 7x  2y  11
3x
 9
3x
3

9

 3

x3
Now substitute 3 for x in either equation to find
the value of y.
5x  y  10
5(3)  y  10
15  y  10
15  y  15  10  15
y  5
(1)(y)  (1) (5)
y5
The solution is (3, 5).
16. C;

3x  2y  19
5x  4y  17

Multiply by 2.
()

50

 10

6x  4y  38
5x  4y  17
11x
 55
11x
11

55

 11

x5
Now substitute 5 for x in either equation to find
the value of y.
5x  4y  17
5(5)  4y  17
25  4y  17
25  4y  25  17  25
4y  8

92  9 9
34  3 3 3 3
 81
 81
Since 34  81 and 92  81, the two quantities are
equal.
17. A;
Column A: Let (2, 4)  (x1, y1) and
(1, 3)  (x2, y2).

4y
4

8
4

y  2
Since the value of x, 5, is greater than the value
of y, 2, the quantity in Column A is greater.

y  y

m  x2  x1
1

3  4

 1  2
1

 3 or 3

Chapter 7

x5
Now substitute 5 for x in either equation to find
the value of y.
3x  y  13
3(5)  y  13
15  y  13
15  y  15  13  15
y  2
The value of y is 2. Since 0 is greater than 2,
the quantity in Column B is greater.
19. A;
Multiply the first equation by 2 so the coefficients
of the y terms are additive inverses. Then add the
equations.

1

10x
10

The slope m is 2 and the y-intercept b is 2.

Since 3  21 and 7  21, 3 is greater than 7. So, the


quantity in Column A is greater.
18. B;
Multiply the second equation by 3 so the
coefficients of the y terms are additive inverses.
Then add the equations.
x  3y  11
x  3y  11
3x  y  13 Multiply by 3. () 9x  3y  39
10x
 50

5  4x
2
5
4x
 2
2
5
2  2x
5
2x  2

5x  y  10 Multiply by 2.
7x  2y  11

3  1
 (2)

362

PQ249-6481F-07[346_363] 8/6/02 09:46 PM Page 363 Sahuja Ahuja_QXP_03:Desktop Folder:6/8/02:

20a. Let A  the number of adult tickets sold and let


C  the number of childrens tickets sold.

20b.

The number of
the number of
650.
adult tickets plus childrens 4443
tickets is
144424443 123 1444424
{ 123


7.5A  4.5C  3675


7.5(650  C)  4.5C  3675
4875  7.5C  4.5C  3675
4875  3C  3675
4875  3C  4875  3675  4875
3C  1200

 650

The
the
the
the
cost
number
cost
number
of an
of
of a
of
adult
adult
childrens
childrens
ticket
times tickets plus ticket
times tickets
is $3675.
1
424
3 123 14243 123 14
4244
3 123 14
4244
3 { 1
424
3
7.50

4.50

A  C  650
A  C  C  650  C
A  650  C

3C
3

3675

The equations are A  C  650 and


7.5 A  4.5 C  3675.

1200
3

C  400
A  650  C
A  650  400
A  250
250 adult tickets and 400 child tickets were
sold.

363

Chapter 7

PQ249-6481F-08[364-387] 26/9/02 10:13 PM Page 364 Sahuja Ahuja_QXP_06:Desktop Folder:Chandra:6481F-QuarkFile:

Chapter 8
Page 409

Polynomials
25

1. 2  2  2  2  2 
3. 5  5  52
5. a  a  a  a  a  a  a6
7.

1
2

2222 

112 25

34

2. 3  3  3  3 
4. x  x  x  x3
6. x  x  y  y  y  x2y3
8.

a
b

bddd 

1ab 221dc 23

9. 32  3  3
9
11. 52  5  5
 25
13. (6) 2  (6)(6)
 36

10. 43  4  4  4
 64
12. 104  10  10  10  10
 10,000
14. (3) 3  (3)(3) (3)
 27

15.

7
7
7
16. 8 2  8 8

17.

3. Poloma; when finding the product of powers with


the same base, keep the same base and add the
exponents. Do not multiply the bases.
4. No; 5  7d shows subtraction, as well as
multiplication.
4a
5. No; 3b shows division, as well as multiplication.
6. Yes; a single variable is a monomial.
7. x (x4 )(x6 )  x146
 x11
4
8. (4a b)(9a2b3 )  (4)(9) (a4  a2 ) (b  b3 )
 36 (a42 ) (b13 )
 36a6b4
3
2
3
32
9. [ (2 ) ]  [2 ] 3
 [26 ] 3
 263
 218 or 262,144
10. (3y5z) 2  32  (y5 ) 2  z2
 9  y52  z2
 9y10z2

Getting Started

123 24  23  23  23  23

16
 81
1
Area  2bh
1
 2 14  9

1 2

1 21 2

49
64

 79
 63
The area of the triangle is 63 yd2.
18. Area  r2
 (62 )
 36
 113.04
The area of the circle is 36 m2 or about
113.04 m2.
19. Volume  /wh
 734
 21  4
 84
The volume is 84 ft3.
20. Volume  /wh
 555
 25  5
 125
The volume is 125 cm3.

11. (4mn2 )(12m2n)  (4  12)(m  m2 ) (n2  n)


 48(m12 ) (n21 )
 48m3n3
3
4
3
3
12. (2v w ) (3vw ) 2
 [ (2) 3 (v3 ) 3 (w4 ) 3 ] [ (3) 2v2 (w3 ) 2 ]
 [ 8  v33  w43 ] [ 9  v2  w32 ]
 [8v9w12 ] [ 9v2w6 ]
 (8  9)(v9  v2 ) (w12  w6 )
 72(v92 ) (w126 )
 72v11w18
1

13. Area  2bh


1

 2 (5n3 )(2n2 )
1

 2 (5  2)(n3  n2 )
1

 2 (10)(n32 )
 5n5
1

14. Area  2bh

Page 413

Check for Understanding

 2 (3a4b)(4ab5 )

1a. Sample answer: n2(n5)  n2 + 5


 n7
2 5
1b. Sample answer: (n )  n2  5
 n10
1c. Sample answer: (nm2)5  n5(m2)5
 n5  m2  5
 n5m10
2
2
2
2a. No; (5m)  5  m
 25m2
2b. Yes; the power of a product is the product of the
powers.
2c. No; (3a)2  (3)2  a2
 9a2
7 3
2d. No; 2(c )  2  c7  3
 2c21

 2 (3  4)(a4  a) (b  b5 )
1

 2 (12)(a41 )(b15 )
 6a5b6

Pages 413415

19. No;

x
y2

shows division, not multiplication of

variables.
Chapter 8

Practice and Apply

15. Yes; 12 is a real number and therefore a monomial.


16. Yes; 4x3 is the product of a number and three
variables.
17. No; a  2b shows subtraction, not multiplication
of variables.
18. No; 4n  5m shows addition, not multiplication of
variables.

364

PQ249-6481F-08[364-387] 26/9/02 10:13 PM Page 365 Sahuja Ahuja_QXP_06:Desktop Folder:Chandra:6481F-QuarkFile:

37. (2ag2 ) 4 (3a2g3 ) 2  [ (24 ) (a4 ) (g2 ) 4 ][ (32 ) (a2 ) 2 (g3 ) 2 ]


 [ 16a4g24 ] [ 9a22g32 ]
 [ 16a4g8 ] [ 9a4g6 ]
 (16) (9) (a4  a4 ) (g8  g6 )
 144(a44 ) (g68 )
 144a8g14
2
3
3
3
38. (2m n ) (3m n) 4
 [ (23 )(m2 ) 3 (n3 ) 3 ] [ (34 ) (m3 ) 4 (n4 ) ]
 [8m23n33 ] [ 81m34n4 ]
 [8m6n9 ] [ 81m12n4 ]
 (8)(81) (m6  m12 ) (n9  n4 )
 648(m612 ) (n94 )
 648m18n13

20. Yes; 5abc14 is the product of a number, 5, and


several variables.
21. (ab4 )(ab2 )  (a  a)(b4  b2 )
 (a11 )(b42 )
 a2b6
5
4
2
22. ( p q )( p q)  ( p5  p2 )(q4  q)
 ( p52 )(q41 )
 p7q5
23. (7c3d4 ) (4cd3 )  (7  4)(c3  c) (d4  d3 )
 (28) (c31 ) (d43 )
 28c4d7
7k5 )(8jk8 )  (3)(8)( j7  j)(k5  k8 )
24. (3j
 (24)( j71 )(k58 )
 24j8k13
2
3
4
3
4
2
25. (5a b c )(6a b c )  (5  6) (a2  a3 )(b3  b4 ) (c4  c2 )
 (30)(a23 )(b34 )(c42 )
 30a5b7c6
5
3
4
6
3
26. (10xy z )(3x y z )  (10  3)(x  x4)(y5  y6)(z3  z3)
 (30)(x1  4)(y5  6)(z3  3)
 30x5y11z6
7
2
2
2
27. (9pq )  9  p  (q7 ) 2
 81  p2  q72
 81p2q14
28. (7b3c6 ) 3  73  (b3 ) 3  (c6 ) 3
 343  b33  c63
 343b9c18
2
4
2
29. [ (3 ) ]  [324 ] 2
 [38 ] 2
 382
 316 or 43,046,721
2
3
2
30. [ (4 ) ]  [423 ] 2
 [46 ] 2
 462
 412 or 16,777,216
31. (0.5x3 ) 2  (0.5) 2  (x3 ) 2
 0.25  x32
 0.25x6
5
3
32. (0.4h )  (0.4) 3  (h5 ) 3
 0.064  h53
 0.064h15

1 2

3
3
33. 4c 3  4 3  c3
27

 64 c3

34.

13 2

39. (8y3 ) (3x2y2 ) 8 xy4  (8) (3)

40.

1 2

 24

1 38 2 (x2  x) (y3  y2  y4 )

138 2 (x21) (y324)

 9x3y9

4
1
m 2 (49m) (17p) 34p5
7
4
1
 7 2 (m2 ) (49m) (17p) 34p5
16
1
 49 (49) (17) 34 (m2  m) ( p  p5 )
1
 (16) 2 (m21 ) ( p15 )

31 2
1 2

12

1 2

3 6

 8m p

41. (2b3 ) 4  3(2b4 ) 3  (2) 4 (b3 ) 4  3(2) 3 (b4 ) 3


 16b34  3(8)b43
 16b12  24b12
 (16  24)b12
 40b12
3
2
3
3
42. 2(5y )  (3y )  2(5) 2 (y3 ) 2  (3) 3 (y3 ) 3
 2(25) (y32 )  (27)(y33 )
 50y6  27y9
43. Area  /w
 (5f 4g3 ) (3fg2 )
 (5) (3) ( f 4  f ) ( g3  g2 )
 15( f 41 ) (g32 )
 15f 5g5
44. Area  s2
45. Area  r2
2 2
 (a b)
 (7x4 ) 2
2 2 2
 (a ) (b )
 [ 72 (x4 ) 2 ] 
22 2
a b
 (49x4 2 )
4 2
a b
 (49x8 )
3
46. Volume  s
47. Volume  /wh
 14k3 2 3
 (xy3 ) (y) (x2y)
3
3
3
 4 1k 2
 (x  x2 ) (y3  y  y)
33
 64k
 (x12 ) (y311 )
 64k9
 x3y5
48. Volume  r2h
 (2n) 2 (4n3 )
 (22  n2 ) (4n3 )
 (4n2 ) (4n3 )
 (4  4) (n2  n3 )
 16n23
 16 n5

145a222  145 22  (a2)2


16

 25  a22
16

 25 a4
35. (4cd) 2 (3d2 ) 3  [ 42  c2  d2 ] [ (3) 3  (d2 ) 3 ]
 [ 16c2d2 ] [27  d23 ]
 [ 16c2d2 ] [27d6 ]
 (16)(27)(c2 )(d2  d6 )
 432  c2  d26
 432c2d8
36. (2x5 ) 3 (5xy6 ) 2  [ (2) 3 (x5 ) 3 ] [ (5) 2 (x2 )(y6 ) 2 ]
 [ 8x53 ] [ 25x2y62 ]
 [ 8x15 ] [ 25x2y12 ]
 (8)(25)(x15  x2 )(y12 )
 200 x152y12
 200x17y12

365

Chapter 8

PQ249-6481F-08[364-387] 26/9/02 10:13 PM Page 366 Sahuja Ahuja_QXP_06:Desktop Folder:Chandra:6481F-QuarkFile:

49. 106  106  1066


 1012 or 1 trillion
6 4
50. (10 )  1064
 1024
51. For 1 km/min:
I  2s2
 2(1) 2 or 2
For 2 km/min:
I  2s2
 2122 2 or 8
For 4 km/min:
I  2s2
 2(4) 2 or 32
52. The collision impact quadruples, since 2(2s)2 is
4(2s2).
53. 212  210  21210
 222 or 4,194,304 ways
54. There is 1 way of answering all questions
correctly out of a total of 4,194,304 ways to
answer all questions.

62.

y
y  2x  1
x

yx2

63.

y
x  2

yx3
O

P(answers all correct)  4,194,304

64. 4x  5y  2
x  2y  6
Multiply the second equation by 4. Then add.
4x  5y  2
() 4x  8y  24
13y  26

55. False. If a  4, then (4)2  16 and 42  16.


56. True. (ambn)p  (am) p(bn) p Power of a Product
 ampbnp
Power of a Power
57. False. Let a  3, b  4, and n  2. Then (a  b)n
 (3  4)2 or 49 and an  bn  32  42 or 25.
58. Answers should include the following.

13y
13

80 feet

the ratio 320 feet which simplifies to a ratio of


1 to 4
If s is replaced by 2s in the formula for the
braking distance required for a car traveling
1
s miles per hour the result is 20 (2s) 2. Using
the Power of a Product and Power of a Power
1
Properties, this simplifies to 4  20s2 . This

y2
Substitute 2 for y in the second equation.
x  2y  6
x  2122  6
x46
x4464
x2
The solution is (2, 2).
65. 3x  4y  25
2x  3y  6
Multiply the first equation by 3 and the second
equation by 4. Then add.
9x  12y  75
() 8x  12y  24
17x
 51

means that doubling the speed of a car


multiplies the braking distance by 4.
59. D; 42  45  425
60. D; Volume  s3
7
4
 15x2 3
 53x3
 125x3

Page 415

Maintain Your Skills

61.

17x
17

51
17

x  3
Substitute 3 for x in the second equation.
2x  3y  6
2(3)  3y  6
6  3y  6
6  3y  6  6  6
3y  12

y  2x  2
O

y  x  1

26

 13

3y
3

12

 3

y  4
The solution is (3, 4).

Chapter 8

366

PQ249-6481F-08[364-387] 26/9/02 10:13 PM Page 367 Sahuja Ahuja_QXP_06:Desktop Folder:Chandra:6481F-QuarkFile:

66. x  y  20
0.4x  0.15y  4
Multiply the first
0.15x  0.15y
() 0.4x  0.15y
0.25x
0.25x
0.25

74. Let x be the number of hours.


Train
Northbound
Southbound

equation by 0.15. Then add.


 3
 4
 1


6

4

2

70x
70

6
6
6
6

4a  12 6 24
4a  12
4a  12 6 24
and
4a  12  12
4a  12  12 6 24  12
4a
4a 6 12

8
4

4a
4

4a
4

4

2

14
14
14  2
12

6
6
6
6

3h  2 6 2
3h  2
3h  2  2
3h

12
3

3h
3

75.

78.

81.

4 6 h
The solution set is the empty set, .
4

70.

2

2m  3 7 7
2m  3  3 7 7  3
2m 7 10
2m
2

or

2

h 6 0

3
15

3
3

 15
3

77.

10
5

80.

9
48

5
79.

14
36

82.

45
18









14
2
36
2
7
18
45
9
18
9
5
1
or 22
2




10
5
5
5
2
or 2
1
9
3
48
3
3
16

Algebra Activity
(Follow-Up of Lesson 8-1)

Surface

2m
2

0
3

2m  7 7 9
2m  7  7 7 9  7
2m 7 2

10
2

76.

Volume
Ratio

Original

2 by 5 by 3

62

30

4 by 10 by 6

248

240

248
62

810

558
62

2
2

Surface

Dimensions

Area

Volume

Area Ratio

Prism

m 7 5
The solution set is {m|m 1}.
4

6

Collect the data.


Method 1
By counting, we find that there are 62 squares.
Method 2
SA  2w/  2wh  2/h
 2(3) (5)  2(3) (2)  2(5) (2)
 30  12  20
 62
The surface area is 62 cm2.
V  /wh
 532
 30
The volume is 30 cm3.
1.

12
4

2
2

2
1
3
27
27
9
 9
9
9
3
 1 or 3
44
44
4
 32
4
32
11
3
 8 or 18
2
6

Page 416

3h  2 6 2
3h  2  2 6 2  2
3h 6 0
3h
3

32
1

and

245
70
7
or
2

The trains will be 245 miles apart in 32 hours.

2 6 a
a 6 3
The solution set is 5a|2 6 a 6 36 .

69.

x

4
4
4  12
8

68.

Distance (d = rt)
40x
30x

40x  30x  245


70x  245

1
0.25

x4
Substitute 4 for x in the first equation.
x  y  20
4  y  20
4  y  4  20  4
y  16
The solution is (4, 16).
4  h  3
4h5
67.
or
4  h  4  3  4
4h454
h  7
h54
h1
The solution set is {h|h  7 or h  1}.
8

Rate
40
30

6 by 15 by 9

558

New
New
1SASAofofOriginal
2 1V VofofOriginal
2

4
9

240
30
810
30

8
 27

2. Sample answer:

m 7 1

Surface

Volume
Ratio

Original

4 by 6 by 9

228

216

8 by 12 by 18

912

1728

912
228

4

12 by 18 by
27

2052

5832

2052
228

9

71. The figure has been enlarged. This is a dilation.


72. The figure has been moved around a point. This is
a rotation.
73. The figure has been flipped over a line. This is a
reflection.

Surface

Dimensions

Area

Volume

Area Ratio

Prism

New
New
1SASAofofOriginal
2 1V VofofOriginal
2

1728
216
5832
216

8
 27

3. The ratio of the surface areas is 4, and the ratio of


the volumes is 8.
4. The ratio of the surface areas is 9, and the ratio of
the volumes is 27.

367

Chapter 8

PQ249-6481F-08[364-387] 26/9/02 10:13 PM Page 368 Sahuja Ahuja_QXP_06:Desktop Folder:Chandra:6481F-QuarkFile:

5. The ratio of the surface areas is a2, and the ratio


of the volumes is a3.
6. Yes, the conjectures hold. If the ratio of the
dimensions of two cylinders is a, then the ratio of
the surface areas is a2 and the ratio of the
volumes is a3.

8. 132  132
1

10.

5pq
10p6q3

Dividing Monomials





Page 418
1.

Graphing Calculator Investigation

KEYSTROKES:

4 ENTER ; 2

1
4

1
8

11.

KEYSTROKES:

p q
15
10 21 p 21 q 2
7

1 16
1p
21q73 2
2
1 5
1p 21q4 2
2
1 1
(q4 )
2 p5

1 2

(cd2 ) 3
(c4d9 ) 2

2

c (d ) 3
(c4 ) 2 (d9 ) 2

3 6

1cc 21dd 2

6

8

18

 1c3182 21d61182 2
 c11d12

1
16

(4m3n5 ) 0
mn

13. C;

is the

 mn

Volume of cylinder
Volume of sphere

1 ENTER




The value of 51 is 0.2 or 5. The conjecture is


correct.

r2h
4
r3
3

 (r) 2 (2r)
4
(r) 3
3

4.

KEYSTROKES:

0 ENTER

0
0 ENTER
An error message appears.

KEYSTROKES:




Page 421

3 5

a b
ab2

a
1

b
1

Pages 421423
12

14.

7
72

5.

12c7z d 23  (2c(7z d))


3




Chapter 8

2 3

3 3 3

2 (c ) d
73 (z2 ) 3
9 3

8c d
343z6

4
42

3
3

3
2 3
 (1)r33
1 4
3 0
3
r or 2
2

x y
x2y7

Practice and Apply


13

 4122

7 3

8 12

 782
 76 or 117,649

6.

1 21rr 2

15.

3
37

 3137

 410 or 1,048,576

 a  b2
16.

 a3  b5  a1  b2
 a3(1)  b5(2)
 a2b3
3. Jamal; a factor is moved from the numerator of a
fraction to the denominator or vice versa only if
1
the exponent of the factor is negative; 4 4.
4.

4
r3
3

Check for Understanding

1. Sample answer: 9xy and 6xy2


(9xy)(6xy2 )  (9)(6)(x  x)(y  y2 )
 54x2y3
2.

2 r3

2
 a4b

The value of 50 is 1.
5.

g8
c5d3

c d

2ad. The numbers are reciprocals of one another.


1
5

 c8d18

12.
1
3. The fractional value of 51 is 5, since
reciprocal of 5 or 51.

8

1
2

5

Power 24 23 22 21 20 21 22 23 24


16

1c1 21d1 21g1 2


1 1 g
 1 c 21 d 21 1 2


 2p5

ENTER ; 2
2 ENTER ; 2
1 ENTER ; 2
0 ENTER ; 2
1 ENTER ; 2
2
ENTER ; 2
3 ENTER ; 2
4 ENTER

Value

c5
d3g8

1
169

8-2

9.

12

18.

1
y4

15b2a n 2
4





20.

2a3
10a8

3 9

17.

368

15b4n2 2
12a6 2 2

y z
yz2

1yy 21zz 2

 (y
 y2z7

19.

14x3my 2
7

5 3

25b8n2
4a12
3

13m7 2 4
14x5y3 2 4

34 1m7 2 4

 44 1x5 2 4 1y3 2 4
81m28

 256x20y12
21.

15b
45b5

11545 21bb 2
5




1 1
5 a5

3

1
5a5

1 2

31 ) (z92 )

52 1b4 2 2n2
22 1a6 2 2

a
12
10 21 a 2

1 38
(a
)
5
1 5
a
5

7. y0 1y5 21y9 2  y05192




1pp 21nn 2


 p3n

 (x82 ) (y127 )
 x6y5

y4

(p74 )(n32 )

1xx 21yy 2
8

p n
p4n2

 36 or 729

1
3
1
3

(b15 )

 b 4
1

1b1 2

1
3b4

PQ249-6481F-08[364-387] 26/9/02 10:13 PM Page 369 Sahuja Ahuja_QXP_06:Desktop Folder:Chandra:6481F-QuarkFile:

22. x3y0x7  x3(7)y0


 x 4 (1)

33.

p  4q  3
(p5q2 )  1

 x4

23. n2 ( p 4 )(n 5 )  n2(5)p 4


 n 3p 4



2

27.

2

2

2 3

3

52

1 21 21 21 2


30.

18x3y4z7
2x2yz

14

3

37.

161 21h1 21k1 2

12a3ab 2

19y0z4
3z16

2

17

32.

1 21 21 21 2

16


38. / 

1 2




9 3

27a c
8b9

Area
w
24x5y3
8x3y2
24 x5 y3
8 x3 y2

1 21 21 2

 3(x53 ) (y32 )
 3x2y
The length is 3x2y units.
Area
39. h  1
b
2

5  2r4
22r6
5  2 1 r4
1
22 r6




23 (a2 ) 3 (b3 ) (c1 ) 3


33 (a3 ) (b2 ) 3
2  3a6b  3c3
3  3a  3b6
23
1
a6 b  3 c3
1 3  3 a  3 b6
1
3

1 21 21 2
1 1
 1 25 21 4 2 (r46 )
1
 1 100 2 (r 2 )
1
1
 1 100 21 r 2

(2a2bc1 ) 3
(3ab2 ) 3

1 21 21 21 21 2
1 3
c
 1 2 21 1 2 (a6(3) ) (b 36 ) 1 1 2
1 27
c
 1 8 21 1 2 (a9 ) (b 9 ) 1 1 2
27c a
1
 8 1 1 21 b 2

19
(z12 )
3
19 1
 3 z12
19
 3z12
(5r  2 ) 2
5  2 (r  2 )  2
 22 (r3 ) 2
(2r3 ) 2

z
(y0 ) 1 z 2
119
3 2
19
 1 3 2 (1)(z416 )

b2
5n2z3

6k17
h3
18 x3 y4 z7
2 x2 y z

bc1 3

2

 9(x32 )(y41 )(z71 )


 9xy3z6

31.

51b2 n4
n2 z3
51 b2 n4 1
1
1 n2 z3

 6(h21 ) (k14(3) )
 6h3k17


51 (b2 ) 1 (n4 ) 1


(n2 ) 1 (z3 ) 1

1305 21hh 21kk 2


2

1


 4a4c4
30h  2k14
5hk  3

 27

4(a73 )(1)(c 4(8) )

29.

2

2 3 1

1 21 21 21 2
1 b
1
 1 5 21 1 2 (n4(2) ) 1 z 2
b
 1 5z 2 (n2 )
b
1
 1 5z 21 n 2


23

 33

28 a7 1 c  4
7 a3 b0 c  8

1b 4cc dd 2

2 3 1

 42

28.

3

35.

2 4 1

1

25

1

3

 16

132 23  32
3
1
 1 1 21 2 2
1 2
 1 3 21 1 2

2 5

2 4

3

2

28a7c  4
7a3b0c  8

34.

 125

145 2 2  45
4
1
 1 1 21 5 2
1 5
 1 4 21 1 2

p
q

11q 2

1r t t 2  1
5b n
(5b n )
36. 1 n z 2  (n z )

 36

26.

25. 5 3  53

1 21 2

p

1
n3p4

24. 6 2  62

p  4q  3
(p5 )  1 (q2 ) 1
p  4q  3
p  5q  2
p4 q3
p5 q2

 ( p4(5) ) (q3(2) )
 p(q1 )

1n1 21p1 2
3

1
100r2

100 a3b
1
(20a2 )
2
100a3b

1 21 21 2
10a2

100 a3 b
10 a2 1

 10 (a32 ) (b)
 10ab
The height is 10ab units.

369

Chapter 8

PQ249-6481F-08[364-387] 26/9/02 10:13 PM Page 370 Sahuja Ahuja_QXP_06:Desktop Folder:Chandra:6481F-QuarkFile:

40.

heavy traffic
normal conversation

10  3

51. You can compare pH levels by finding the ratio of


one pH level to another written in terms of the
1
concentration c of hydrogen ions, c  10 pH.
Answers should include the following.
Sample answer: To compare a pH of 8 with a
pH of 9 requires simplifying the quotient of
powers,

 10  6

1 2

 103(6)
 103 or 1000
The sound from heavy traffic is 103 or 1000 times
more intense than normal conversation.
41. 10,000 noisy kitchen  104  10 2
 104(2)
 102
The sound of a jet plane (30 m away) is 10,000
times more intense than a noisy kitchen.
42.

whisper
normal conversation

1101 2
1101 2

 109(6)
 10 3

The intensity of a whisper is


that of normal
conversation.
43. If you toss a coin n times, the probability of
1
getting n heads is 2 n.

12 2

10 4  104 or
1

1
10,000

1
105

to

1
104

cm or

Page 423

1
100,000

55.
to

1
109

cm or

1
10,000,000

to
56.

an3

47.

48. (54x3 )(52x1 )  5(4x3)(2x1)
 56x2
c x7
c x4

50.

58.
59.

 c(x7)(x4)
 c x7x4
 c11

3b2n9
b3(n3)

131 21bb

2n9

3(n3)

60.

(2n9)3(n3)

)
 3(b
 3(b2n93n9 )
 3bn



Maintain Your Skills

 (m3  m) (n  n2 )
 (m31) (n12 )
 m4n3
4y3 )(4x4y)  (3  4) (x4  x4 ) (y3  y)
(3x
 12(x44) (y31 )
 12x8y4
3 2 4
3 4 2 4
(a x )  (a ) (x )
57. 13cd5 2 2  32 1c2 21d5 2 2
12
8
a x
 9c2d10
3 123 2 2 4 2  3 26 4 2
 212 or 4096
3 (2b3 ) 2  (3) 3 (a3 ) (b3 ) (22 ) (b3 ) 2
(3ab)
 (27) (a3 ) (b3 ) (4) (b6 )
 108a3 (b36 )
 108a3b9
The following system of inequalities can be used
to represent the conditions of the problem.
x0
y0
147x  219y  1200
xy8

54. (m

46. 107  107 or

49.

22  3

3n)(mn2 )

cm.

an (a3 )

Negative Exponent Property

25

1
100,000
1
10,000

1
10,000,000
1
1
109  109 or 1,000,000,000
1
The range of x-rays is 107
1
cm.
1,000,000,000

 25(5)
 210
53. Since each number is obtained by dividing the
previous number by 3, 31  3 and 30  1.

The range of visible light is


to

1
10

 25

 2n
1

1 2

1

52. A; 22  23  22  (3)

1
2n

45. 10 5  105 or

1101 2

89

22  23

12

 2n

1101 2

 10
Thus, a pH of 8 is ten times more acidic than a
pH of 9.

1
1000
1
1000




10  9

44.

 10  6

 103 or

1 n

131 21b1 2
n

3
bn

y
xy8

147x  219y  1200


O

Chapter 8

370

PQ249-6481F-08[364-387] 26/9/02 10:13 PM Page 371 Sahuja Ahuja_QXP_06:Desktop Folder:Chandra:6481F-QuarkFile:

68. To find the x-intercept, let y  0.


2x  7  3y
2x  7  3(0)
2x  7

61. Sample answers: 3 oz of mozzarella, 4 oz of Swiss;


4 oz of mozzarella, 3 oz of Swiss; 5 oz of
mozzarella, 3 oz of Swiss
62. y  mx  b
63. y  mx  b
y  1x  (4)
y  2x  3
yx4
64. y  mx  b
65. y  mx  b
1

y

To find the y-intercept, let x  0.


2x  7  3y
2(0)  7  3y
0  7  3y
0  3y  7  3y  3y
3y  7

y  2x  2

1

66. To find the x-intercept, let y  0.


2y  x  10
2102  x  10
0  x  10
0  10  x  10  10
10  x
To find the y-intercept, let x  0.
2y  x  10
2y  0  10
2y  10
2y
2

2

x  2 or 32

y  3x  (1)
1
3x

2x
2

3y
3

3
7

y  3 or 23
y

(0, 2 13)
2x  7  3y

10
2

y5

(3 12 , 0)

69. 1121  11 since both


112  121 and (11) 2  121.

(0, 5)

2y  x  10

(10, 0)

70. 13.24  1.8 since 1.82  3.24.


71. 152  7.21 since
(7.21) 2  52.
72. 102 103  1023
 105
73. 108 105  10(8)  (5)
 1013
6
9
74. 10 10  10(6)  9
 103
8
1
75. 10 10  108  (1)
 107
4 104  104  (4)
76. 10
 100 or 1
77. 1012 10  10(12)  1
 1011

67. To find the x-intercept, let y  0.


4x  y  12
4x  0  12
4x  12
4x
4

12
4

x3
To find the y-intercept, let x  0.
4x  y  12
4102  y  12
 y  12
(1)(y)  (1)(12)
y  12
y

Page 424

(3, 0)
8

4

8x

4
8
12

Reading Mathematics

1. Sample answer: triangle; a three-sided polygon.


2. See students work.
3a. precisely half of
3b. six
3c. eight

4x  y  12
(0, 12)

371

Chapter 8

PQ249-6481F-08[364-387] 26/9/02 10:13 PM Page 372 Sahuja Ahuja_QXP_06:Desktop Folder:Chandra:6481F-QuarkFile:

8-3

Pages 428430

Scientific Notation

Page 428

18.
19.
20.
21.
22.
23.
24.
25.
26.
27.
28.

Check for Understanding

1. When numbers between 0 and 1 are written in


scientific notation, the exponent is negative. If the
number is not between 0 and 1, use a positive
exponent.
2. 65.2 103 is not written in scientific notation.
The number 65.2 is greater than 10.
3. Sample answer: 6.5 million; 6,500,000; 6.5 106
4. 2 108  0.00000002
5. 4.59 103  4590
6. 7.183 1014  718,300,000,000,000
7. 3.6 105  0.000036
8. 56,700,000  5.67 107
9. 0.00567  5.67 103
10. 0.00000000004  4 1011
11. 3,002,000,000,000,000  3.002 1015
12. 15.3 102 2 14.1 105 2  15.3 4.121102 105 2
 21.73 107
 12.173 101 2 1107 2
 2.173 1101 107 2
 2.173 108
or 217,300,000
5
3
13. 12 10 2 19.4 10 2  12 9.421105 103 2
 18.8 108
 11.88 101 2 1108 2
 1.88 1101 108 2
 1.88 107
or 0.000000188
14.

1.5 102
2.5 1012

29.
30.
31.
32.
33.
34.
35.
36.

37.

38.

10
11.5
2.5 21 10 2

39.

12

 0.6 1010
 16 101 2 1010
 6 1101 1010 2
 6 1011
or 0.00000000006
15.

1.25 104
2.5 106

40.

10
11.25
2.5 21 10 2

41.

6

 0.5 1010
 15 101 2 1010
 5 1101 1010 2
 5 109
or 5,000,000,000
16. 1,650,000,000; 1.65 109; 1,540,000,000,000;
1.54 1012
17. Average 


42.
43.
44.

1.54 1012
1.65 109
1.54 1012
1.65 109

1 21 2

 0.933 103
 19.33 101 2 103
 9.33 1101 103 2
 9.33 102
 933. 33
An average of $933.33 is charged per credit card.

Chapter 8

45.

372

106

Practice and Apply

5
 0.000005
6.1 109  0.0000000061
7.9 104  79,000
8 107  80,000,000
1.243 107  0.0000001243
2.99 101  0.299
4.782 1013  47,820,000,000,000
6.89 100  6.89
2 1011  200,000,000,000
2.389 105  238,900
1.67265 1027
 0.00000000000000000000000000167265
9.1095 1031
 0.00000000000000000000000000000091095
50,400,000,000  5.04 1010
34,402,000  3.4402 107
0.000002  2 106
0.00090465  9.0465 104
25.8  2.58 10
380.7  3.807 102
622 106  16.22 102 2 106
 6.22 1102 106 2
 6.22 108
11
87.3 10  18.73 102 1011
 8.73 110 1011 2
 8.73 1012
4
0.5 10  15 101 2 104
 5 1101 10 4 2
 5 105
0.0081 103  (8.1 103 ) 103
 8.1 (103 103 )
 8.1 106
7
94 10  19.4 102 107
 9.4 110 107 2
 9.4 106
12
0.001 10  (1 103 ) 1012
 1 1103 1012 2
 1 109
10 billion tons  10,000,000,000 tons
 1 1010 tons
602,214,299,000,000,000,000,000
 6.02214299 1023
18.9 104 214 103 2  18.9 421104 103 2
 35.6 107
 13.56 102 107
 3.56 110 107 2
 3.56 108
or 356,000,000
(3 106 )(5.7 102 )  (3 5.7)(106 102 )
 17.1 108
 (1.71 10) 108
 1.71 (10 108 )
 1.71 109
or 1,710,000,000

PQ249-6481F-08[364-387] 26/9/02 10:13 PM Page 373 Sahuja Ahuja_QXP_06:Desktop Folder:Chandra:6481F-QuarkFile:

46. (5 102 )(8.6 103 )  (5 8.6)(102 10 3 )


 43 105
 (4.3 10) 105
 4.3 (10 105 )
 4.3 104
or 0.00043
47. (1.2 105 ) (1.2 103 )  (1.2 1.2) (105 10 3 )
 1.44 108
or 0.0000000144
48. (3.5 107 ) (6.1 108 )  (3.5 6.1) (107 108 )
 21.35 101
 (2.135 10) 101
 2.135 (10 101 )
 2.135 100
or 2.135
49. (2.8 102 ) (9.1 106 )  (2.8 9.1) (102 106 )
 25.48 104
 (2.548 10) 104
 2.548 (10 104 )
 2.548 105
or 254,800
50.

7.2 109
4.8 104

1 21 2
7.2
4.8

109
104

51.

7.2 103
1.8 107

 1.5
or 150,000
52.

21

5.745 trillion
283.9 million

3.162 104
5.1
102

58.

53.

21

55.

2.795 108
4.3 104

4.65 101
5 105

1 21 2

Rodriguezs salary
Fosters salary

25.2 million

 2.04 million

 4 10
or 0.0004

10
125.2
2.04 21 10 2
6
6

 12.35
Rodriguezs salary in 2000 was about 12 times
Fosters salary in 1982.
59. There are 365 24 60 60 or 31,536,000 or
3.1536 107 seconds per year.
(4.4 106 )(3.1536 107 )
 (4.4 3.1536) (106 107 )

 13.87584 1013
 1.387584 1014
The sun burns about 1.4 1014 or 140 trillion tons
of hydrogen per year.
60a. always
(a 10n ) p  ap (10n ) p
Product of Powers
 ap 10np
Power of a Product
p
np
60b. Sometimes; a 10 is only in scientific
notation if 1  ap  10. Counterexample:
(5 103)2  52 106 or 25 106, but 25 106
is not in scientific notation since 25 is greater
than 10.

105

10
12.795
4.3 21 10 2
8
4

 0.65 104
 (6.5 10 1 ) 10 4
 6.5 (101 10 4 )
 6.5 105
or 0.000065

12

25.2 106

 0.23
 (2.3 101 ) 105
 2.3 (101 105 )
 2.3 106
or 0.0000023
54.

10
15.745
283.9 21 10 2

 2.04 106

7.2 103
1.8 107

106

1.035 102
4.5
103

5.745 1012
283.9 106

 0.020 106
or 20,236
In April 2001, each persons share of the debt was
about $20,236.

 0.62
 (6.2 101 ) 106
 6.2 (101 106 )
 6.2 107
or 0.00000062
1.035 102
4.5 103




4

105

3.162 104
5.1 102

56. Multiply the daily growth rate by the number of


days. Since we are considering 10 years, there are
365 10 days.
13.3 104 21365 102  13.3 36521104 102
 1204.5 103
 11.2045 103 2 103
 1.2045 (103 103 )
 1.2045 100
or 1.2045
After 10 years, the hair would be about
1.2 meters long.
57. Divide the national debt by the number of people.

10
14.65
5 21 10 2
1
5

 0.93 106
 19.3 101 2 106
 9.3 (101 106 )
 9.3 107
or 0.00000093

373

Chapter 8

PQ249-6481F-08[364-387] 26/9/02 10:13 PM Page 374 Sahuja Ahuja_QXP_06:Desktop Folder:Chandra:6481F-QuarkFile:

61. Astronomers work with very large numbers such


as the masses of planets. Scientific notation
allows them to more easily perform calculations
with these numbers. Answers should include the
following.

Planet

69.

4,870,000,000,000,000,000,000,000

Earth

5,970,000,000,000,000,000,000,000

Mars


70.

1,900,000,000,000,000,000,000,000,000

Saturn

569,000,000,000,000,000,000,000,000

Uranus

86,800,000,000,000,000,000,000,000

Neptune

102,000,000,000,000,000,000,000,000

Pluto

4.5 10

73. Yes;
74.

75.

KEYSTROKES:

ENTER

5.5 10

 1.09

Page 430
68.

49a4b7c2
7ab4c3

4

6 10
7

ENTER

103

Maintain Your Skills

1 21 21 21 2
49 a4 b7 c2
7
a b4 c3

 7(a41 )(b74 )(c23 )


 7a3b3c1
 7a3b3

Chapter 8

7a3b3
c

16

and v2.

17
17  3
14
14}
14

7
7
7
7

14

1
3

12

10

9
99
18
186
18

20

22

36

34

11c 2

374

32

30

77. 5b  5122
78. c2  9  32  9
 5142
99
 20
0
79. b3  3ac  122 3  3152132
 8  45
 37
80. a2  2a  1  52  2(5)  1
 25  10  1
 34
81. 2b4  5b3  b  2(2) 4  5(2) 3  (2)
 2(16)  5(8)  2
 32  40  2
 10
82. 3.2c3  0.5c2  5.2c  3.2(3) 3  0.5(3) 2  5.2(3)
 3.21272  0.5192  5.2132
 86.4  4.5  15.6
 75.3

131 2

x  11  23
x  11  11  23  11
x  34
(1)(x)  (1)34
x  34
{x|x  34}

6
6
6
6

9  d
9  d  9
d
5d|d

38

3.15 10

is the product of

m3
m33
m
{m|m

12

76.

1.03 103
67.

v2
3

18

11 1.2 10

4.095 10

3

64(n14(6)

106

KEYSTROKES:

14

6

72. No; n shows division, not multiplication of


variables.

5 ENTER
66.

1641 21nn 2131 2

 64(n20 )(27)
 1728n20
71. No; 3a  4b shows addition, not multiplication of
variables.

7.83 107

8.52

82 (n7 ) 2

9 1.74 10

7.1 10

4n5
p5

1 1p 2

 33 (n2 ) 3

2 ENTER

KEYSTROKES:

5

64n14

12,700,000,000,000,000,000,000

KEYSTROKES:

2

 33n6

Scientific notation allows you to fit numbers


such as these into a smaller table. It allows
you to compare large values quickly by
comparing the powers of 10 instead of counting
zeros to find place value. For computation,
scientific notation allows you to work with
fewer place values and to express your
answers in a compact form.
62. C; 360 104  3.6 102 104
 3.6 102
63. (25 billion) (270 million)
 (25 109)(270 106)
 (25 270)(109 106)
 6750 1015
 6.75 103 1015
 6.75 1018
There are about 6.75 1018 hemoglobin
molecules in the human body.

65.

(8n7 ) 2
(3n2 ) 3

642,000,000,000,000,000,000,000

Jupiter

141 21nn 21p1 2

 4n5

330,000,000,000,000,000,000,000

Venus

 (4)(n3(2 ) (p5 )

Mass (kg)

Mercury

64.

4n3p5
n2

PQ249-6481F-08[364-387] 26/9/02 10:14 PM Page 375 Sahuja Ahuja_QXP_06:Desktop Folder:Chandra:6481F-QuarkFile:

Page 430

2.

Practice Quiz 1

1. n 1n 2 1n2  n
 n8
3
2. 4ad(3a d)  (4  3)(a  a3 )(d  d)
 12(a13 )(d11 )
 12a4d2
3. (2w3z4 ) 3 (4wz3 ) 2
(2) 3 (w3 ) 3 (z4 ) 3 (4) 2 (w2 )(z3 ) 2
 8(w9 )(z12 ) (16) (w2 ) (z6 )
 128(w92 )(z126 )
 128w11z18
3

4.

25p10
15p3

341

1 21 2
5
 1 3 2 1 p103 2
5
 1 3 2 (p7 )



6.

p10
p3

25
15

5.

6k3 2
7np4

5.
6.
7.
8.
9.

(6k3 ) 2
(7np4 ) 2

62 1k3 2 2

 72n2 1p4 2 2
36k6

 49n2p8

2

 36z10
36z10
y4

10

x

x
1

3x  2x
x2  x  4
2x2  3x  1
x2  2x  3
Sample answer: x2, x, and 1 represent the areas of
tiles.

Polynomials

1y1 2
4

5.

6.

10
19.2
2.3 21 10 2
3
5

 4 102
or 0.04


7.

10
13.6
1.2 21 10 2
7

8.

2

 3 10
or 3,000,000,000

9.

Algebra Activity
(Preview of Lesson 8-4)

10.

1.
x

Check for Understanding

1. Sample answer: 8
2. A negative exponent indicates division, and an
expression involving division by a variable is not
a monomial. If one of the terms of an expression
is not a monomial, then the expression is not a
polynomial.
3a. True; a binomial is a polynomial with two terms.
3b. False; 3x  5 is a polynomial but since it has
two terms it is not a monomial.
3c. True; a monomial is a polynomial with one term.
4. 5x  3xy  2x  7x  13xy2
This expression is the sum of two monomials. Yes,
it is a polynomial; it is a binomial.

Page 434

10

7. 16.4 103 217 102 2  16.4 721103 102 2


 44.8 105
 14.48 102 105
 4.48 110 105 2
 4.48 106
or 4,480,000
8. (4 102 ) (15 106 )  (4 15)(102 106 )
 60 104
 (6.0 10) 104
 6.0 (10 104 )
 6.0 103
or 0.006

x

8-4

 41y4 21921z10 2

Page 431

3.6 107
1.2 102

4(1) (y2 )

141 21yy 2131 21z 1 2


3 z
 41y26 2 1 1 21 1 2

10.

 32 (y3 ) 2 (z5 ) 2

9.2 10
2.3 105

4.

4y2

9.

3.

 32y6z10

1 1 1 1

5p7
3

4x0y2
(3y3z5 ) 2

11.

12.

375

2z
5

 5z

This expression is a monomial. Yes, it is a


polynomial.
9a2  7a  5  9a2  7a  152
This expression is the sum of three monomials.
Yes, it is a polynomial; it is a trinomial.
The polynomial 1 has only one term, whose
degree is 0. Thus, the degree of 1 is 0.
The polynomial 3x  2 has two terms, 3x and 2,
whose degrees are 1 and 0, respectively. Thus, the
degree of 3x  2 is 1, the greater of 1 and 0.
The polynomial 2x2y3  6x4 has two terms, 2x2y3
and 6x4, whose degrees are 5 and 4, respectively.
Thus, the degree of 2x2y3  6x4 is 5, the greater
of 5 and 4.
6x3  12  5x  6x3  12x0  5x1
 12  5x  6x3
2
3
2
7a x  4x  2ax5  2a
 7a2x3  4x2  2ax5  2ax0
 2a  4x2  7a2x3  2ax5
2c5  9cx2  3x  2c5x0  9cx2  3x1
 9cx2  3x  2c5
Chapter 8

PQ249-6481F-08[364-387] 26/9/02 10:14 PM Page 376 Sahuja Ahuja_QXP_06:Desktop Folder:Chandra:6481F-QuarkFile:

13. y3  x3  3x2y  3xy2  y3x0  x3  3x2y  3xy2


 x3  3x2y  3xy2  y3
14. Since the diameter of each semicircle is 2d, the
radius is half of 2d or d.
Area of shaded region
 area of rectangle  area of semicircles

27. The polynomial 4ab has only one term, whose


degree is 2. Thus, the degree of 4ab is 2.
28. The polynomial 13 has only one term, whose
degree is 0. Thus, the degree of 13 is 0.
29. The polynomial c4  7c2 has two terms, c4 and
7c2, whose degrees are 4 and 2, respectively.
Thus, the degree of c4  7c2 is 4, the greater of 4
and 2.
30. The polynomial 6n3  n2p2 has two terms, 6n3
and n2p2, whose degrees are 3 and 4,
respectively. Thus, the degree of 6n3  n2p2 is 4,
the greater of 3 and 4.
31. The polynomial 15  8ag has two terms, 15 and
8ag, whose degrees are 0 and 2, respectively.
Thus, the degree of 15  8ag is 2, the greater of
0 and 2.
32. The polynomial 3a2b3c4  18a5c has two terms,
3a2b3c4 and 18a5c, whose degrees are 9 and 6,
respectively. Thus, the degree of 3a2b3c4  18a5c
is 9, the greater of 9 and 6.
33. The polynomial 2x3  4y  7xy has three terms
2x3, 4y, and 7xywhose degrees are 3, 1, and 2,
respectively. Thus, the degree of 2x3  4y  7xy is
3, the greatest of 3, 1, and 2.
34. The polynomial 3z5  2x2y3z  4x2z has three
terms3z5, 2x2y3z, and 4x2zwhose degrees
are 5, 6, and 3, respectively. Thus, the degree of
3z5  2x2y3z  4x2z is 6, the greatest of 5, 6,
and 3.
35. The polynomial 7  d5  b2c2d3  b6 has four
terms7, d5, b2c2d3, and b6whose degrees are
0, 5, 7, and 6, respectively. Thus, the degree of
7  d5  b2c2d3  b6 is 7, the greatest of 0, 5, 7,
and 6.
36. The polynomial 11r2t4  2s4t5  24 has three
terms11r2t4, 2s4t5, and 24whose degrees are
6, 9, and 0, respectively. Thus, the degree of
11r2t4  2s4t5  24 is 9, the greatest of 6, 9,
and 0.

11 2
1
 c12d2  2 1 2d2 2
 lw  2 2r2
 2cd  d2

Pages 434436

Practice and Apply

15. 14
This expression is a monomial. Yes, it is a
polynomial.
16.

6m2
p

 p3

The expression

6m2
p

is not a monomial. No, this is

not a polynomial.
17. 7b  3.2c  8b  15b  13.2c2
This expression is the sum of two monomials. Yes,
it is a polynomial; it is a binomial.
18. 3x2  x  2  3x2  x  122
1

This expression is the sum of three monomials.


Yes, it is a polynomial; it is a trinomial.
19. 6gh2  4g2h  g  6gh2  14g2h2  g
This expression is the sum of three monomials.
Yes, it is a polynomial; it is a trinomial.
5

20. 4  2a  a2
5

The expression a2 is not a monomial. No, this is


not a polynomial.
21. Area of shaded region
 area of rectangle  area of triangle
1

 bh  2bh
1

 2bh or 0.5bh

37. 2x  3x2  1  2x1  3x2  1x0


 1  2x  3x2
3
5
38. 9x  7  3x  9x3  7x0  3x5
 7  9x3  3x5
2
3
3
2
39. c x  c x  8c  c2x3  c3x2  8cx0
 8c  c3x2  c2x3
40. x3  4a  5a2x6  x3  4ax0  5a2x6
 4a  x3  5a2x6
5
2
41. 4  3ax  2ax  5a7
 4x0  3ax5  2ax2  5a7x0
 4  5a7  2ax2  3ax5
42. 10x3y2  3x9y  5y4  2x2
 10x3y2  3x9y  5x0y4  2x2
 5y4  2x2  10x3y2  3x9y
43. 3xy2  4x3  x2y  6y
 3x1y2  4x3  x2y  6x0y
 6y  3xy2  x2y  4x3
44. 8a5x  2ax4  5  a2x2
 8a5x1  2ax4  5x0  a2x2
 5  8a5x  a2x2  2ax4

22. Area of shaded region


 area of rectangle  area of squares
 ab  41x2 2
 ab  4x2
23. Area of shaded region
 area of triangle  area of circle
1

 2xy  r2
24. Since the radius of the circle is r, the diameter is
2r. Since the base of the triangle is a diameter,
the base is 2r.
Area of shaded region
 area of circle  area of triangle
1

 r2  2 (2r)(r)
 r2  r2
25. The polynomial 5x3 has only one term, whose
degree is 3. Thus, the degree of 5x3 is 3.
26. The polynomial 9y has only one term, whose
degree is 1. Thus, the degree of 9y is 1.

Chapter 8

376

PQ249-6481F-08[364-387] 26/9/02 10:14 PM Page 377 Sahuja Ahuja_QXP_06:Desktop Folder:Chandra:6481F-QuarkFile:

45. 5  x5  3x3  5x0  x5  3x3


 x5  3x3  5
2
46. 2x  1  6x  2x1  1x0  6x2
 6x2  2x  1
3x2  5a  2a2x3  4a3x2  5ax0  2a2x3
47. 4a
 2a2x3  4a3x2  5a
2
2
2
48. b  x  2xb  b x0  x2  2x1b
 x2  2xb  b2
49. c2  cx3  5c3x2  11x
 c2x0  cx3  5c3x2  11x1
 cx3  5c3x2  11x  c2
50. 9x2  3  4ax3  2a2x
 9x2  3x0  4ax3  2a2x1
 4ax3  9x2  2a2x  3
51. 8x  9x2y  7y2  2x4
 8x1  9x2y  7x0y2  2x4
 2x4  9x2y  8x  7y2
52. 4x3y  3xy4  x2y3  y4
 4x3y  3x1y4  x2y3  x0y4
 4x3y  x2y3  3xy4  y4
53. Multiply the number of each type of coin by its
value, and add.
0.25q  0.10d  0.05n
54. t 23; For t 23, the model predicts a negative
number of quadruplet births.
55. Volume

58. A polynomial model of a set of data can be used to


predict future trends in data. Answers should
include the following.

1 4

 r2h  3r3
56. Volume  r2h 

123 2r3
2

 (4)(6)  3(8)
88

3

19
19
22
23.5
25
34

Actual
Data Values
19
19
22
24
26
36

Maintain Your Skills

61. 12,300,000  1.23 107


62. 0.00345  3.45 103
63. 12 106  1.2 10 106
 1.2 107
64. 0.77 1010  7.7 101 1010
 7.7 1011

0
1
2
3
4
5

Page 436

 (2) 2 (6)  3r(2) 3


 24 

The polynomial function models the data


exactly for the first 3 values of t, and then
closely for the next 3 values.
Someone might point to this model as evidence
that the time people spend playing video
games is on the rise. This model may assist
video game manufacturers in predicting
production needs.
59. B; 3x3  2x2  x  1  3(1)3 + 2(1)2  (1)  1
 3(1)  2(1)  0
 3  2
 1
60. C; The degree of 5x2y3 is 5. The degree of 3x3y2 is
5. So the two quantities are equal.

 volume of cylinder  2 (volume of sphere)


 r2h  2 3 r3

65. a0b2c1  1

16

3

1b1 2 11c 2
2

 b2c

or about 92.15

66.

5n5
n8

1nn 2
5
8

 5(n58 )
 5n3

The volume of the container is about 92.15 in3.


57. True; for the degree of a binomial to be zero, the
highest degree of both terms would need to be
zero. The terms would be like terms. With these
like terms combined, the expression is not a
binomial, but a monomial. Therefore, the degree
of a binomial can never be zero. Only a monomial
can have a degree of zero.

67.

68.

4x3y2
3z


2

(4x3y2 ) 2
(3z) 2
42 (x3 ) 2 (y2 ) 2
32z2
16x6y4
9z2

y5m8
y3m7




(y) m
y3m7



1 2

5 1
1 n3
5
n3

1yy 2 1mm 2
5

7

 (y53 ) (m8(7) )
 y2m15
69. The graph represents a relation that is not a
function. The element 1 in the domain is paired
with both 3 and 4 in the range.
70. The table represents a function since, for each
element in the domain, there is only one
corresponding element in the range.

377

Chapter 8

PQ249-6481F-08[364-387] 26/9/02 10:14 PM Page 378 Sahuja Ahuja_QXP_06:Desktop Folder:Chandra:6481F-QuarkFile:

3. 4x2  x
5x  2

71. There are 26 black cards and 52 total cards.


P(black) 

26
52

or

1
2

72. 3n  5n  (3  5)n
 8n
2
73. 9a  3a  2a2  9a2  2a2  3a
 (9  2)a2  3a
 7a2  3a
74. The expression 12x2  8x  6 is simplified since
it has no like terms or parentheses.
75. 3a  5b  4a  7b  (3a  4a)  (5b  7b)
 (3  4)a  (5  7)b
 a  2b
76. 4x  3y  6  7x  8  10y
 (4x  7x)  (3y  10y)  (6  8)
 (4  7)x  (3  10)y  2
 11x  7y  2

Page 438

x 2

x 2

x 2

x 2

x
1 1

4x 2

 2

6x

(4x2  x)  (5x  2)  4x2  6x  2


4. 3x2  4x  2
remove x2  5x  5

Algebra Activity
(Preview of Lesson 8-5)

1. 5x2  3x  4
2x2  4x  1

x2
x2

1

1

1

1

1

x2

x2

x2

x2

x2

x

x2

1

1

x

x

x

x

x2

x

x

x

x

9x

x 2

x2

x2

x2

x2

5x 2

x x

 2x

x 2

x 2

11

x2

3x 2

(2x2  5)  (3x2  2x  6)  5x2  2x  11

x2

(3x2  4x  2)  (x2  5x  5)  2x2  9x  7


5. x2  7x
remove 2x2  3x

3

(5x2  3x  4)  (2x2  4x  1)  7x2  x  3


2. 2x2  5
3x2  2x  6

x2

x

1
2x 2

7x 2

x2

1

Chapter 8

4x

(x2  7x)  (2x2  3x)  3x2  4x

378

PQ249-6481F-08[364-387] 26/9/02 10:14 PM Page 379 Sahuja Ahuja_QXP_06:Desktop Folder:Chandra:6481F-QuarkFile:

6. 8x  4
remove 6x2  x  3

x2

x 2

x2

x 2

x2

x 2

x2

x 2

x2

x 2

x 2

x2

1

1

1

7x

(8x  4)  (6x2  x  3)  6x2  7x  7


7. Method from Activity 2:
You need to add zero pairs so that you can remove
2 x tiles and 3 1 tiles.

x

x

1

1

x x x

2x 2  5x  2

Method from Activity 3:


You remove all zero pairs to find the difference in
simplest form.

x x x

Pages 441443

Opposite of 2x  3

Check for Understanding

1. The powers of x and y are not the same.


2. Sample answer: 6x2  4x  7 and 4x2  3x  4
3. Kendra; Esteban added the additive inverses of
both polynomials when he should have added the
opposite of the polynomial being subtracted.
4. (4p2  5p)  (2p2  p)
 [4p2  (2p2 ) ]  (5p  p)
 2p2  6p
5. (5y2  3y  8)  (4y2  9)
 (5y2  4y2 )  (3y)  [ 8  (9) ]
 9y2  3y  1
6. (8cd  3d  4c)  (6  2cd)
 (8cd  2cd)  (3d)  4c  (6)
 10cd  3d  4c  6
7. (6a2  7a  9)  (5a2  a  10)
 (6a2  7a  9)  (5a2  a  10)
 (6a2  5a2 )  [ 7a  (a) ] (9  10)
 11a2  6a  1
8. (g3  2g2  5g  6)  (g2  2g)
 (g3  2g2  5g  6)  (g2  2g)
 g3  [2g2  (g2 ) ]  [ 5g  (2g) ]  6
 g3  3g2  3g  6
9. (3ax2  5x  3a)  (6a  8a2x  4x)
 (3ax2  5x  3a)  (6a  8a2x  4x)
 3ax2  [ 5x  (4x) ]  [ 3a  (6a) ]
 8a2x
 3ax2  9x  9a  8a2x
10. You can find a model for T by adding the
polynomials for F and M.
T  (1247n  126,971)  (1252n  120,741)
 (1247n  1252n)  (126,971  120,741)
 2499n  247,712
11. The year 2010 is 2010 1990 or 20 years after
1990.
T  2499n  247,712
 2499(20)  247,712
 297,692
If this trend continues, the population in 2010
would be about 297,692 thousand or 297,692,000.

Page 441

6x 2

Adding and Subtracting


Polynomials

8-5

Practice and Apply

12. (6n  4)  (2n2  9)


 [6n2  (2n2 ) ]  (4  9)
 4n2  5
13. (9z  3z2 )  (4z  7z2 )
 (9z  4z)  [3z2  (7z2 ) ]
 13z  10z2
14. (3  a2  2a)  (a2  8a  5)
 (a2  a2 )  [ 2a  (8a) ]  (3  5)
 2a2  6a  8

x x
1 1 1

2x 2  5x  2

379

Chapter 8

PQ249-6481F-08[364-387] 26/9/02 10:14 PM Page 380 Sahuja Ahuja_QXP_06:Desktop Folder:Chandra:6481F-QuarkFile:

15. (3n2  8  2n)  (5n  13  n2 )


 (3n2  n2 )  (2n  5n)  (8  13)
 2n2  7n  5
16. (x  5)  (2y  4x  2)
 (x  4x)  2y  [5  (2) ]
 5x  2y  3
17. (2b3  4b  b2 )  (9b2  3b3 )
 (2b3  3b3 )  [b2  (9b2 ) ]  (4b)
 5b3  8b2  4b
18. (11  4d2 )  (3  6d2 )
 (11  4d2 )  (3  6d2 )
 (4d2  6d2 )  [11  (3) ]
 10d2  8
19. (4g3  5g)  (2g3  4g)
 (4g3  5g)  (2g3  4g)
 [4g3  (2g3 ) ]  [5g  (4g) ]
 2g3  9g
20. (4y3  y  10)  (4y3  3y2  7)
 (4y3  y  10)  (4y3  3y2  7)
 [4y3  (4y3 ) ]  (3y2 )  (y)  (10  7)
 8y3  3y2  y  17
21. (4x  5xy  3y)  (3y  6x  8xy)
 (4x  5xy  3y)  (3y  6x  8xy)
 [4x  (6x) ]  [ 5xy  (8xy) ] 
[3y  (3y) ]
 2x  3xy
22. (3x2  8x  4)  (5x2  4)
 (3x2  8x  4)  (5x2  4)
 [3x2  (5x2 ) ]  8x  (4  4)
 2x2  8x  8
23. (5ab2  3ab)  (2ab2  4  8ab)
 (5ab2  3ab)  (2ab2  4  8ab)
 [5ab2  (2ab2 ) ]  (3ab  8ab)  (4)
 3ab2  11ab  4
3
24. (x  7x  4x2  2)  (2x2  9x  4)
 (x3  7x  4x2  2)  (2x2  9x  4)

29. (9x3  3x  13)  (6x2  5x)


 (2x3  x2  8x  4)
 (9x3  3x  13)  (6x2  5x)
 (2x3  x2  8x  4)
 (9x3  2x3 )  [6x2  (x2 ) ]
 [3x  5x  (8x) ]  (13  4)
 11x3  7x2  9
30. The measure of the third side is the perimeter
minus the measures of the other two sides.
(7x  3y)  (x  2y)  (2x  3y)
 (7x  3y)  (x  2y)  (2x  3y)
 [7x  (x)  (2x) ]  [ 3y  2y  (3y) ]
 4x  2y
The measure of the third side is 4x  2y.
31. The measure of the third side is the perimeter
minus the measures of the other two sides.
(10x2  5x  16)  (4x3  3)  (10x  7)
 (10x2  5x  16)  (4x2  3)  (10x  7)
 [10x2  (4x2 ) ]  [ 5x  (10x) ] 
[16  3  (7) ]
 6x2  15x  12
The measure of the third side is 6x2  15x  12.
32. You can find a model for D by subtracting the
polynomial for I from the polynomial for T.
D  (160.3n2  26n  24,226) 
(161.6n2  20n  23,326)
 (160.3n2  26n  24,226) 
(161.6n2  20n  23,326)
 [160.3n2  (161.6n2 ) ]  (26n  20n) 
[24,226  (23,326) ]
 1.3n2  6n  900
33. The year 2010 is 2010  1990 or 20 years after
1990.
D  1.3n2  6n  900
 1.3(20) 2  6(20)  900
 260
If this trend continues, there will be 260 outdoor
movie screens in 2010.
34. The result is always the original number with its
digits swapped.
35. Original number  10x  y; show that the new
number will always be represented by 10y  x.
new number  9(y  x)  (10x  y)
 9y  9x  10x  y
 10y  x
36. The length is 60  x  x or 60  2x inches.
37. The width is 40  x  x or 40  2x inches.
38. Girth  2(width)  2(height)
 2(40  2x)  2(x)
 80  4x  2x
 80  2x
The girth is 80  2x inches.

 x3  [4x2  (2x2)] (7x  9x)  [2  (4)]

25.

26.

27.

28.

 x3  2x2  2x  6
(5x2  3a2  5x)  (2x2  5ax  7x)
 (5x2  3a2  5x)  (2x2  5ax  7x)
 [5x2  (2x2 ) ]  [5x  (7x) ]
 5ax  3a2
 3x2  12x  5ax  3a2
(3a  2b  7c)  (6b  4a  9c)
 (7c  3a  2b)
 [3a  (4a)  (3a) ]  [2b  6b  (2b) ]
 [7c  9c  (7c) ]
 4a  6b  5c
(5x2  3)  (x2  x  11)  (2x2  5x  7)
 (5x2  x2  2x2 )  [x  (5x) ]
 (3  11  7)
 8x2  6x  15
(3y2  8)  (5y  9)  ( y2  6y  4)
 [3y2  8)  (5y  9)  (y2  6y  4)
 [ 3y2  (y2 ) ]  [ 5y  (6y) ]  (8  9  4)

 2y2  y  5

Chapter 8

380

PQ249-6481F-08[364-387] 26/9/02 10:14 PM Page 381 Sahuja Ahuja_QXP_06:Desktop Folder:Chandra:6481F-QuarkFile:

39. The sum of the length and the girth must not
exceed 108 inches.
(60  2x)  (80  2x)  108
(60  80)  [ (2x)  (2x) ]  108
140  4x  108
140  4x  140  108  140
4x  32
4x
4

Page 443

32
4

x8
The least possible value of x is 8 inches.
40. 19 inches; For integral values of x greater than
19, the width of the box, 40  2x, would be
negative or zero.
41. The next integer greater than x is 1 greater, so
x + 1.
42. The sum of two consecutive integers x and x  1
is x  (x  1) or 2x  1. When 2x  1 is divided by
2x  1
2x
1
1
2; the quotient is 2  2  2 or x  2.

6a
2

4b
2

60
50
40
30
20
10

wpm

The quotient is not an integer, so 2x  1 is not


divisible by 2, and hence, not even. Thus, the sum
of two consecutive integers is odd.
43. 2, 3, and 4 are three consecutive integers whose
sum, 2  3  4 or 9, is not even.
Let x, x  1, x  2, and x  3 be four consecutive
integers. Their sum is x  (x  1)  (x  2) 
(x  3) or 4x  6. When 4x  6 is divided by 2, the
4x  6
4x
6
quotient is 2  2  2 or 2x  3,
which is an integer. Thus, 4x  6 is divisible by 2,
so that the sum of four consecutive integers is
always even.
4 is the least number.
44. In order to find the sum of the video games sales
and the traditional toy sales, you must add the
two polynomial models V and R, which represent
each of these sales from 1996 to 1999.
T  0.45t3  1.85t2  4.4t  22.6
If a person was looking to invest in a toy
company, they might want to look at the trend
in toy sales over the last several years and try
to predict toy sales for the future.
45. A; subtract the width from the perimeter twice.
(16a  2b)  (5a  b)  (5a  b)
 (16a  2b)  (5a  b)  (5a  b)
 [16a  (5a)  (5a) ]  (2b  b  b)
 6a  4b
This is twice the length, so divide by 2.
6a  4b
2

Maintain Your Skills

47. The polynomial 15t3y2 has only one term, whose


degree is 5. Thus, the degree of 15t3y2 is 5.
48. The polynomial 24 has only one term, whose
degree is 0. Thus, the degree of 24 is 0.
49. The polynomial m2  n3 has two terms, m2 and
n3, whose degrees are 2 and 3, respectively. Thus,
the degree of m2  n3 is 3, the greater of 2 and 3.
50. The polynomial 4x2y3z  5x3z has two terms,
4x2y3z and 5x3z, whose degrees are 6 and 4,
respectively. Thus, the degree of 4x2y3z  5x3z is
6, the greater of 6 and 4.
51. 8 106  8,000,000
52. 2.9 105  290,000
53. 5 104  0.0005
54. 4.8 107  0.00000048
5556.

1 2 3 4 5 6 7 8 9 10
Weeks

57. Sample answer: We use the points (4, 33) and


(7, 45).
y2  y1

mx




58.

59.
60.

or 3a  2b

61.

The length is 3a  2b.


62.

46. D; (a2  2ab  b2 )  (a2  3ab  b2 )


 (a2  2ab  b2 )  (a2  3ab  b2 )
 [a2  (a2 ) ]  [ (2ab)  3ab]
 [b2  (b2 ) ]
 ab
This expression is also equivalent to 36 22 or
14. Thus, ab  14.

 x1

45  33
7  4
12
or 4
3

Use the point-slope form.


y  y1  m(x  x1 )
y  33  4(x  4)
y  33  4x  16
y  33  33  4x  16  33
y  4x  17
y  4x  17
 4(12)  17
 48  17
 65
After 12 weeks, a students keyboarding speed
should be about 65 wpm.
No; there is a limit to how fast one can keyboard.
The domain is {2, 0, 6}.
The range is {5, 2, 3}.
The domain is {4, 1, 5}.
The range is {2,3, 0, 1}.
Let x represent the length.
1
87

8
x

1(x)  87(8)
x  696
The real locomotive is 692 inches or 58 feet long.
63. 6(3x  8)  6(3x)  6(8)
 18x  48
64. 2(b  9)  2(b)  (2) (9)
 2b  18

381

Chapter 8

PQ249-6481F-08[364-387] 26/9/02 10:14 PM Page 382 Sahuja Ahuja_QXP_06:Desktop Folder:Chandra:6481F-QuarkFile:

65. 7(5p  4q)  7(5p)  (7)(4q)


 35p  28q
66. 9(3a  5b  c)  9(3a)  9(5b)  9(c)
 27a  45b  9c
67. 8(x2  3x  4)  8(x2 )  8(3x)  8(4)
 8x2  24x  32
68. 3(2a2  5a  7)
 3(2a2 )  (3)(5a)  (3)(7)
 6a2  15a  21

8-6

2(w  1)  w  7  4w
2(w)  (2)(1)  w  7  4w
2w  2  w  7  4w
w  2  7  4w
3w  2  7
3w  9
w3
x(x  2)  3x  x(x  4)  5
11.
x(x)  x(2)  3x  x(x)  x(4)  5
x2  2x  3x  x2  4x  5
x2  x  x2  4x  5
x  4x  5
3x  5
5
x3

10.

Multiplying a Polynomial by a
Monomial

Page 446

12. Subtract x from 10,000 to find the amount in the


CD. The expression is 10,000  x.
13. The total is the sum of the original investment
and the interest earned.
T  10,000  0.04x  0.07(10,000  x)
T  10,000  0.04x  0.07(10,000)  0.07(x)
T  10,000  0.04x  700  0.07x
T  10,700  0.03x
14. T  10,700  0.03x
 10,700  0.03(3000)
 10,700  90
 10,610
If she puts $3000 in savings, she will have $10,610.

Check for Understanding

1. Distributive Property; Product of Powers Property


2. The three monomials that make up the trinomial
are similar to the three digits that make up the
3-digit number. The single monomial is similar to
a 1-digit number. With each procedure you are
performing multiplications. The difference is that
polynomial multiplication involves variables and
the resulting product is often the sum of two or
more monomials while numerical multiplication
results in a single number.
3. Sample answer: 4x and x2  2x  3;
4x(x2  2x  3)
 4x(x2 )  4x(2x)  4x(3)
 4x3  8x2  12x
4. 3y(5y  2)  3y(5y)  (3y)(2)
 15y2  (6y)
 15y2  6y
2
3
2
5. 9b (2b  3b  b  8)
 9b2 (2b3 )  9b2 (3b2 )  9b2 (b)  9b2 (8)
 18b5  27b4  9b3  72b2
6. 2x(4a4  3ax  6x2 )
 2x(4a4 )  2x(3ax)  2x(6x2 )
 8a4x  6ax2  12x3
7. 4xy(5x2  12xy  7y2 )
 4xy(5x2 )  (4xy)(12xy)  (4xy)(7y2 )
 20x3y  (48x2y2 )  (28xy3 )
 20x3y  48x2y2  28xy3
8. t(5t  9)  2t  t(5t)  t(9)  2t
 5t2  9t  2t
 5t2  11t
3
2
9. 5n(4n  6n  2n  3)  4(n2  7n)
 5n(4n3 )  5n(6n2 )  5n(2n)  5n(3)
 (4)(n2 )  (4)(7n)
 20n4  30n3  10n2  15n  (4n2 )  (28n)
 20n4  30n3  [ (10n2 )  (4n2 ) ]
 [15n  (28n) ]
 20n4  30n3  (14n2 )  (13n)
 20n4  30n3  14n2  13n

Chapter 8

Pages 446448

Practice and Apply

15. r(5r  r2 )  r(5r)  r(r2 )


 5r2  r3
3
16. w(2w  9w2 )  w(2w3 )  w(9w2 )
 2w4  9w3
17. 4x(8  3x)  4x(8)  (4x) (3x)
 32x  12x2
2
18. 5y(2y  7y)  5y (2y2 )  5y(7y)
 10y3  35y2
3
19. 7ag (g  2ag)  7ag (g3 )  7ag (2ag)
 7ag4  14a2g2
2
20. 3np(n  2p)  3np(n2 )  (3np) (2p)
 3n3p  (6np2 )
 3n3p  6np2
2
2
21. 2b (3b  4b  9)
 2b2 (3b2 )  (2b2 ) (4b)  (2b2 ) (9)
 6b4  (8b3 )  (18b2 )
 6b4  8b3  18b2
22. 6x3 (5  3x  11x2 )
 6x3 (5)  6x3 (3x)  6x3 (11x2 )
 30x3  18x4  66x5
23. 8x2y(5x  2y2  3)
 8x2y(5x)  8x2y(2y2 )  8x2y(3)
 40x3y  16x2y3  24x2y
24. cd2 (3d  2c2d  4c)
 cd2 (3d)  (cd2 ) (2c2d)  (cd2 ) (4c)
 3cd3  (2c3d3 )  (4c2d2 )
 3cd3  2c3d3  4c2d2

382

PQ249-6481F-08[364-387] 26/9/02 10:14 PM Page 383 Sahuja Ahuja_QXP_06:Desktop Folder:Chandra:6481F-QuarkFile:

25. 4 hk2 (20k2  5h  8)


3

35. 3c2 (2c  7)  4c(3c2  c  5)  2(c2  4)

 3c2 (2c)  (3c2 ) (7)  4c(3c2 )  4c(c)

 4 hk2 (20k2 )  4 hk2 (5h)  4 hk2 (8)

15
 15hk4   4 h2k2  (6hk2 )

 15hk4 

15 2 2
h k
4

 4c(5)  2(c2 )  2(4)


 6c3  (21c2 )  12c3  4c2  20c  2c2  8
 (6c3  12c3 )  (21c2  4c2  2c2 )  20c  8

 6hk2

 6c3  23c2  20c  8

26. 3 a2b(6a3  4ab  9b2 )





2 2
2
a b(6a3 )  3 a2b(4ab)
3
8
4a5b  3 a3b2  6a2b3

36. 4x2 (x  2)  3x(5x2  2x  6)  5(3x2  4x)


 4x2 (x)  4x2 (2)  3x(5x2 )  3x(2x)  3x(6)
 (5) (3x2 )  (5) (4x)
 4x3  8x2  15x3  6x2  18x  (15x2 )
 (20x)
 (4x3  15x3 )  (8x2  6x2  15x2 )
 (18x  20x)
 19x3  x2  2x
37. area of shaded region  area of large rectangle 
area of small rectangle
 4x(3x  2)  2x(3x)
 4x(3x)  4x(2)  2x(3x)
 12x2  8x  6x2
 (12x2  6x2 )  8x
 6x2  8x
38. area of shaded region  area of large rectangle 
area of small rectangle
 5p(3p  4)6(2p  1)
 5p(3p)  5p(4)  (6) (2p)  (6) (1)
 15p2  20p  (12p)  (6)
 15p2  (20p  12p)  6
 15p2  8p  6
39.
2(4x  7)  5(2x  9)  5
2(4x)  2(7)  5(2x)  5(9)  5
8x  14  10x  45  5
8x  14  10x  50
18x  14  50
18x  36
x  2
40.
2(5a  12)  6(2a  3)  2
2(5a)  2(12)  6(2a)  (6) (3)  2
10a  24  12a  (18)  2
10a  24  12a  18  2
10a  24  12a  20
22a  24  20
22a  44
a2
41.
4(3p  9)  5  3(12p  5)
4(3p)  4(9)  5  3(12p)  (3) (5)
12p  36  5  36p  (15)
12p  31  36p  15
48p  31  15
48p  16

2 2
a b(9b2 )
3

27. 5a3b(2b  5ab  b2  a3 )


 5a3b(2b)  (5a3b)(5ab)  (5a3b)(b2 )
 (5a3b) (a3 )
 10a3b2  (25a4b2 )  (5a3b3 )  (5a6b)
 10a3b2  25a4b2  5a3b3  5a6b
28. 4p2q2 (2p2  q2  9p3  3q)
 4p2q2 (2p2 )  4p2q2 (q2 )  4p2q2 (9p3 )  4p2q2 (3q)
 8p4q2  4p2q4  36p5q2  12p2q3

29. d(2d  4)  15d  d(2d)  d(4)  15d


 2d2  4d  15d
 2d2  (4d  15d)
 2d2  19d
30. x(4x2  2x)  5x3  x(4x2 )  (x)(2x)  5x3
 4x3  (2x2 )  5x3
 4x3  2x2  5x3
 (4x3  5x3 )  2x2
 9x3  2x2
2
31. 3w(6w  4)  2(w  3w  5)
 3w(6w)  3w(4)  2(w2 )  2(3w)  2(5)
 18w2  12w  2w2  6w  10
 (18w2  2w2 )  (12w  6w)  10
 20w2  18w  10
32. 5n(2n3  n2  8)  n(4  n)
 5n(2n3 )  5n(n2 )  5n(8)  n(4)  n(n)
 10n4  5n3  40n  4n  n2
 10n4  5n3  n2  (40n  4n)
 10n4  5n3  n2  44n
33. 10(4m3  3m  2)  2m(3m2  7m  1)
 10(4m3 )  10(3m)  10(2)  (2m)(3m2 )
 (2m)(7m)  (2m)(1)
 40m3  30m  20  6m3  (14m2 )
 (2m)
 40m3  30m  20  6m3  14m2  2m
 (40m3  6m3 )  14m2  (30m  2m)  20
 46m3  14m2  32m  20
34. 4y(y2  8y  6)  3(2y3  5y2  2)
 4y(y2 )  4y(8y)  4y(6)  (3)(2y3 )
 (3)(5y2 )  (3)(2)
 4y3  32y2  24y  (6y3 )  (15y2 )  (6)
 4y3  32y2  24y  6y3  15y2  6
 (4y3  6y3 )  (32y2  15y2 )  24y  6
 2y3  17y2  24y  6

p
42.

16
48

or 3

7(8w  3)  13  2(6w  7)
7(8w)  7(3)  13  2(6w)  2(7)
56w  21  13  12w  14
56w  (8)  12w  14
44w  8  14
44w  22
22

w  44 or 2

383

Chapter 8

PQ249-6481F-08[364-387] 26/9/02 10:14 PM Page 384 Sahuja Ahuja_QXP_06:Desktop Folder:Chandra:6481F-QuarkFile:

d(d  1)  4d  d(d  8)
d(d)  d(1)  4d  d(d)  d(8)
d2  d  4d  d2  8d
d2  3d  d2  8d
3d  8d
11d  0
d0
c(c  3)  c(c  4)  9c  16
44.
c(c)  c(3)  (c)(c)  (c) (4)  9c  16
c2  3c  c2  4c  9c  16
(c2  c2 )  (3c  4c)  9c  16
7c  9c  16
2c  16
c8
y(y  12)  8y  14  y(y  4)
45.
y(y)  y(12)  8y  14  y(y)  y(4)
y2  12y  8y  14  y2  4y
y2  4y  14  y2  4y
4y  14  4y
8y  14

51. The increased length is 5x  12.


Area  4x(5x  12)
 4x(5x)  4x(12)
 20x2  48x
52. Since 1 mile is at the $2.75 rate, m  1 miles are
at the $1.25 rate for each taxi. So the cost for
each taxi is 2.75(1)  1.25(m  1) or 2.75 
1.25(m  1). Since there are t taxis, we multiply
the cost of each taxi by t.
t [ 2.75  1.25(m  1) ]  t(2.75  1.25m  1.25)
 t(1.50  1.25m)
 1.50t  1.25mt
The total cost to transport the group is
1.50t  1.25mt dollars.
53. The next odd integer is 2 greater than x, so x  2
is the next odd integer.
54. x(x  2)  x(x)  x(2)
 x2  2x
55. Let x and y be integers. Then 2x and 2y are even
numbers, and (2x)(2y)  4xy. 4xy is divisible by 2
since one of its factors, 4, is divisible by 2.
Therefore 4xy is an even number.
56. 2x  1 or 2x  1
57. Let x and y be integers. Then 2x is an even
number and 2y  1 is an odd number. Their
product, 2x(2y  1), is always even since one of its
factors is 2.
58. Since a represents the number of apples, 10-a
represents the number of oranges.
T  0.25a  0.20(10  a)
T  0.25a  0.20(10)  0.20(a)
T  0.25a  2  0.20a
T  2  0.05a
59. T  2  0.05a
 2  0.05(4)
 2  0.20
 2.20
With 4 apples, the total cost is $2.20.
60. T  p  0.30p  0.01n(p  0.30p)
T  p  0.30p  0.01n(p)  (0.01n)(0.30p)
T  p  0.30p  0.01np  0.003np
T  0.7p  0.007np
61. Replace p with 200 and n with 10.
T  0.7(200)  0.007(10)(200)
 140  14
 126
The discounted cost is $126.
62. Distance apart at start
 circumference of outer semicircle 
circumference of inner semicircle

43.

y
46.

14
8

or 4

k(k  7)  10  2k  k(k  6)
k(k)  k(7)  10  2k  k(k)  k(6)
k2  7k  10  2k  k2  6k
k2  7k  10  k2  8k
7k  10  8k
15k  10  0
15k  10
10

k  15 or 3
47. 2n(n  4)  18  n(n  5)  n(n  2)  7
2n(n)  2n(4)  18  n(n)  n(5)  n(n)  n(2)  7
2n2  8n  18  n2  5n  n2  2n  7
2n2  8n  18  2n2  3n  7
8n  18  3n  7
5n  18  7
5n  25
n  5
48. 3g(g  4)  2g(g  7)  g(g  6)  28
3g(g)  3g(4)  (2g)(g)  (2g)(7)
 g(g)  g(6)  28
3g2  12g  2g2  14g  g2  6g  28
g2  2g  g2  6g  28
2g  6g  28
4g  28
g7
49. Since x represents the amount put into a savings
account, 6000  x represents the amount used to buy
a certificate of deposit. The total amount is the sum
of the original investment and the interest earned.
T  6000  0.03x  0.06(6000  x)
T  6000  0.03x  0.06(6000)  .06(x)
T  6000  0.03x  360  0.06x
T  6360  0.03x or T  0.03x  6360
50.
T  6360  0.03x
6315  6360  0.03x
45  0.03x
1500  x
6000  x  6000  1500 or 4500
Savings account: $1500; certificate of deposit: $4500
Chapter 8

 2 [2(x  2.5) ]  2 (2x)


 (x  2.5)  x
 x  2.5  x
 2.5
The runners should be 2.5 or about 7.9 feet apart.

384

PQ249-6481F-08[364-387] 26/9/02 10:14 PM Page 385 Sahuja Ahuja_QXP_06:Desktop Folder:Chandra:6481F-QuarkFile:

70. 4x2  10ab  6  4x2  (10ab)  6


This expression is the sum of three monomials.
Yes, it is a polynomial; it is a trinomial.
71. 4c  ab  c  3c  ab
This expression is the sum of two monomials. Yes,
it is a polynomial; it is a binomial.

63. Answers should include the following.


The product of a monomial and a polynomial
can be modeled using an area model. The area
of the figure shown at the beginning of the
lesson is the product of its length 2x and width
(x  3). This product is 2x(x  3), which when
the Distributive Property is applied, becomes
2x(x)  2x(3) or 2x2  6x. This is the same
result obtained when the areas of the algebra
tiles are added together.
Sample answer: (3x)(2x  1)
(3x)(2x  1)  (3x)(2x)  (3x)(1)
 6x2  3x
1
x
x

72.

 y2
7

The expression y is not a monomial. No, this is


not a polynomial.
73.

n2
3

 3 n2

This expression is a monomial. Yes, it is a


polynomial.
74. Let n be the number.
6  10n  9n
6  10n 10n  9n  10n
6  n
(1)6 (1)(n)
6 n
The solution set is {n|n  6}.
75. Let n be the number.
9n  4  7  13n
9n  4  13n  7  13n  13n
22n  4  7
22n  4  4  7  4
22n  3

64. B; [ (3x2  2x  4)  (x2  5x  2) ] (x  2)


 [3x2  2x  4  (x2 )  (5x)  2] (x  2)
 [2x2  7x  6](x  2)
 2x2 (x  2)  (7x)(x  2)  6(x  2)
 2x2 (x)  2x2 (2)  (7x) (x)  (7x)(2)
 6(x)  6(2)
 2x3  4x2  7x2  14x  6x  12
 2x3  3x2  8x  12
65. A; Let m represent the number of minutes over
30 minutes and T represent the total charges.
T  70  4m
122  70  4m
52  4m
13  m
The total time is 13  30 or 43 minutes.

Page 449

7
y

22n
22

 22
3

n  22

The solution set is n|n  22 .


76. Find the slope.
y2  y1

mx




 x1

4  (8)
1  (3)
12
or 3
4

Find the y-intercept.


y  mx  b
8  3(3)  b
8  9  b
1b
Write the slope-intercept form.
y  mx  b
y  3x  1
77. Find the slope.

Maintain Your Skills

66. (4x2  5x)  (7x2  x)


 [4x2  (7x2 ) ]  (5x  x)
 3x2  6x
67. (3y2  5y  6)  (7y2  9)
 (3y2  5y  6)  (7y2  9)
 (3y2  7y2 )  5y  (6  9)
 4y2  5y  3
68. (5b  7ab  8a)  (5ab  4a)
 (5b  7ab  8a)  (5ab  4a)
 5b  (7ab  5ab)  (8a  4a)
 5b  12ab  12a
69. (6p3  3p2  7)  (p3  6p2  2p)
 (6p3  p3 )  (3p2  6p2 )  2p  7
 7p3  3p2  2p  7

y2  y1

mx




 x1

7  5
2  (4)
12
or 2
6

Find the y-intercept.


y  mx  b
5  2(4)  b
58b
3  b
Write the slope-intercept form.
y  mx  b
y  2x  3

385

Chapter 8

PQ249-6481F-08[364-387] 26/9/02 10:14 PM Page 386 Sahuja Ahuja_QXP_06:Desktop Folder:Chandra:6481F-QuarkFile:

78. Find the slope.

Page 449

y2  y1

mx




 x1

2  (1)
3  3
3
1
or  2
6

Find the y-intercept.


y  mx  b
1
1  2 (3)  b
3

1  2  b
1
2

b

Write the slope-intercept form.


y  mx  b
1

y  2 x  2

5. 4x2  9x  12  5x3  4x2  9x1  12x0  5x3


 5x3  4x2  9x  12
 12  9x  4x2  5x3
6. 2xy4  x3y5  5x5y  13x2
 2x1y4  x3y5  5x5y  13x2
 5x5y  x3y5  13x2  2xy4
 2xy4  13x2  x3y5  5x5y
7. (7n2  4n  10)  (3n2  8)
 (7n2  3n2 )  4n  (10  8)
 10n2  4n  2
8. (3g3  5g)  (2g3  5g2  3g  1)
 (3g3  5g)  (2g3  5g2  3g  1)
 [3g3  (2g3 ) ]  (5g2 )  (5g  3g)  1
 g3  5g2  2g  1
9. 5a2 (3a3b  2a2b2  6ab3 )
 5a2 (3a3b)  5a2 (2a2b2 )  5a2 (6ab3 )
 15a5b  10a4b2  30a3b3
10. 7x2y(5x2  3xy  y)
 7x2y(5x2 )  7x2y(3xy)  7x2y(y)
 35x4y  21x3y2  7x2y2

79. Let x be the amount of money Kristen had


originally.
1
x
5

gasoline:
haircut:
lunch: 7
1

1
2

1x  15 x2 or 21 145 x2 or 25 x

x  5 x  5 x  7  13
2
x
5

 7  13

2
x
5
5 2
x
2 5

 20

1 2  52 (20)

x  50
Kristen originally had $50.
80. Stem
3
4
5
6
81. Stem
1
2
3
4

0
5
1
2

Leaf
2 3
7 7 8 9
3 5 6 7
8
6|2  62

0
0
0
3

Leaf
4 5 8 8 8
0 1 1 2
4
4
3|4  34

Practice Quiz 2

1. The polynomial 5x4 has only one term,


whose degree is 4. Thus, the degree of the
polynomial is 4.
2. The polynomial 9n3p4 has only one term,
whose degree is 7. Thus, the degree of the
polynomial is 7.
3. The polynomial 7a2  2ab2 has two terms,
7a2 and 2ab2, whose degrees are 2 and 3,
respectively. Thus, the degree of 7a2  2ab2 is 3,
the greater of 2 and 3.
4. The polynomial 6  8x2y2  5y3 has three
terms, 6, 8x2y2, and 5y3, whose degrees are 0,
4, and 3, respectively. Thus, the degree of
6  8x2y2  5y3 is 4, the greatest of 0, 4, and 3.

Page 451 Algebra Activity


(Preview of Lesson 8-7)
1.

82. (a)(a)  a2

x2

83. 2x(3x2 )  (2  3)(x  x2 )


 6x3
2
84. 3y (8y2 )  (3  8) (y2  y2 )
 24y4
85. 4y(3y)  4y(6)  (4  3)(y  y)  (4  6)y
 12y2  24y
86. 5n(2n2 )  (5n) (8n)  (5n)(4)
 (5  2)(n  n2 )  (5  8)(n  n)  (5  4)n
 10n3  40n2  20n
87. 3p2 (6p2 )  3p2 (8p)  3p2 (12)
 (3  6)(p2  p2 )  (3  8)(p2  p)  (3  12)p2
 18p4  24p3  36p2

Chapter 8

x3

x2

x x

1
1
1

x
x
x

1
1
1

1
1
1

(x  2)(x  3)  x2  5x  6

386

PQ249-6481F-08[364-387] 26/9/02 10:14 PM Page 387 Sahuja Ahuja_QXP_06:Desktop Folder:Chandra:6481F-QuarkFile:

2.

5.

x2

x2

x

x  3 1

x
x

1
1
1

1
1

x

1 1

x1
x 1

x2

x x

x2

x x

1
1
1

x

x

x

2x  3

(x  2)(2x  3)  2x2  7x  6

(x  1)(x  3)  x2  4x  3
6.

3.

x3

x
x1
x
1

x
x2

1
1

x2
x
x

x
2x  4

1
1

4.

x1
x
1

2x  1

x2

x2

(x  1) (2x  1) 

2x2

x2

x x x

x2

x x x

1
1
1

x

1 1 1

x

1 1 1

x

1 1 1

1

x

1 1 1

(x  3)(2x  4)  2x2  2x  12
7. By the Distributive Property,
(x  3)(x  4)  x(x  4)  3(x  4). The top row
represents x(x  4) or x2  4x. The bottom row
represents 3(x  4) or 3x  12.

(x  1)(x  2)  x2  x  2

8-7
Page 455

Polynomials
Check for Understanding
x3

1.

 3x  1

x x x

x x x

2x  1

x

1 1 1

2a. (3x  4)(2x  5)


 3x(2x  5)  4(2x  5)
 6x2  15x  8x  20
 6x2  7x  20
2b. (3x  4)(2x  5)
 3x(2x)  3x(5)  4(2x)  4(5)
 6x2  15x  8x  20
 6x2  7x  20

387

Chapter 8

3x  4
()2x  5
15x  20

2c.

6x2 
6x2 
2d.

8. (9p  1) (3p  2)
 (9p) (3p)  (9p) (2)  (1) (3p)  (1) (2)
 27p2  18p  3p  2
 27p2  21p  2

8x

9. (2g  7)(5g  8)
 (2g)(5g)  (2g)(8)  (7)(5g)  (7)(8)
 10g2  16g  35g  56
 10g2  19g  56
10. (3b  2c)(6b  5c)
 (3b)(6b)  (3b)(5c)  (2c) (6b)  (2c)(5c)
 18b2  15bc  12bc  10c2
 18b2  3bc  10c2
11. (3k  5) (2k2  4k  3)
 3k(2k2  4k  3)  5(2k2  4k  3)
 6k3  12k2  9k  10k2  20k  15
 6k3  2k2  29k  15

7x  20
3x  4

2x  5

x x x x

x x x x

x
x
x
x
x

x
x
x
x
x

1 1 1 1
1 1 1 1
1 1 1 1
1 1 1 1
1 1 1 1

x
x
x
x
x

12. A  2 bh
1

 2 (2x  3)(3x  1)
1

 2 [ (2x)(3x)  (2x)(1)  3(3x)  3(1) ]


1

 2 [6x2  2x  9x  3) ]
1

x
x
x
x
x
x
x

1 1 1 1
1 1 1 1
1 1 1 1
1 1 1 1
1 1 1 1

x
x
x
x
x
x
x
x

 2 [6x2  7x  3]

x
x
x
x
x
x
x
x

Pages 455457

or 3x2  2 x  2

Practice and Apply

13. (b  8)(b  2)
 (b)(b)  (b)(2)  (8) (b)  (8)(2)
 b2  2b  8b  16
 b2  10b  16
14. (n  6)(n  7)
 (n)(n)  (n)(7)  (6)(n)  (6) (7)
 n2  7n  6n  42
 n2  13n  42
15. (x  4)(x  9)
 (x)(x)  (x)(9)  (4)(x)  (4)(9)
 x2  9x  4x  36
 x2  13x  36
16. (a  3)(a  5)
 (a)(a)  (a)(5)  (3)(a)  (3)(5)

3. See students work.


4. ( y  4) ( y  3)
 ( y) ( y)  ( y)(3)  (4)( y)  (4)(3)
 y2  3y  4y  12
 y2  7y  12
5. (x  2) (x  6)
 (x) (x)  (x)(6)  (2)(x)  (2)(6)
 x2  6x  2x  12
 x2  4x  12
6. (a  8) (a  5)
 (a) (a)  (a)(5)  (8)(a)  (8)(5)
 a2  5a  8a  40
 a2  3a  40
7. (4h  5) (h  7)
 (4h) (h)  (4h)(7)  (5)(h)  5(7)
 4h2  28h  5h  35
 4h2  33h  35

Chapter 8

6x2  7x  3
2

 a2  5a  3a  15
 a2  8a  15
17. (y  4)(y  8)
 (y)(y)  (y)(8)  (4)(y)  (4)(8)
 y2  8y  4y  32
 y2  4y  32
18. (p  2)(p  10)
 (p)(p)  (p)(10)  (2)(p)  (2)(10)
 p2  10p  2p  20
 p2  8p  20
19. (2w  5)(w  7)
 (2w)(w)  (2w)(7)  (5)(w)  (5)(7)
 2w2  14w  5w  35
 2w2  9w  35

388

20. (k  12) (3k  2)


 (k)(3k)  (k)(2)  (12)(3k)  (12)(2)
 3k2  2k  36k  24
 3k2  34k  24
21. (8d  3) (5d  2)
 (8d)(5d)  (8d)(2)  (3)(5d)  (3)(2)
 40d2  16d  15d  6
 40d2  31d  6
22. (4g  3) (9g  6)
 (4g)(9g)  (4g)(6)  (3)(9g)  (3)(6)
 36g2  24g  27g  18
 36g2  51g  18
23. (7x  4) (5x  1)
 (7x)(5x)  (7x)(1)  (4)(5x)  (4)(1)
 35x2  7x  20x  4
 35x2  27x  4
24. (6a  5)(3a  8)
 (6a)(3a)  (6a)(8)  (5) (3a)  (5)(8)
 18a2  48a  15a  40
 18a2  63a  40
25. (2n  3)(2n  3)
 (2n)(2n)  (2n)(3)  (3)(2n)  (3)(3)
 4n2  6n  6n  9
 4n2  12n  9
26. (5m  6) (5m  6)

33. (2x  5)(3x2  4x  1)


 2x(3x2  4x  1)  5(3x2  4x  1)
 6x3  8x2  2x  15x2  20x  5
 6x3  23x2  22x  5
34. (3k  4) (7k2  2k  9)
 3k(7k2  2k  9)  4(7k2  2k  9)
 21k3  6k2  27k  28k2  8k  36
 21k3  34k2  19k  36
35. 1n2  3n  221n2  5n  42
 n2 (n2  5n  4)  3n(n2  5n  4)
 21n2  5n  42
 n4  5n3  4n2  3n3  15n2  12n  2n2
 10n  8
 n4  2n3  17n2  22n  8
36. (y2  7y  1)(y2  6y  5)
 y2 (y2  6y  5)  7y(y2  6y  5) 
1(y2  6y  5)
 y4  6y3  5y2  7y3  42y2  35y  y2 
6y  5
 y4  y3  38y2  41y  5
37. (4a2  3a  7) (2a2  a  8)
 4a2 (2a2  a  8)  3a(2a2  a  8) 
7(2a2  a  8)
 8a4  4a3  32a2  6a3  3a2 
24a  14a2  7a  56
 8a4  2a3  15a2  31a  56
38. 16x2  5x  2213x2  2x  42
 6x2 (3x2  2x  4)  5x(3x2  2x  4) 
2(3x2  2x  4)
 18x4  12x3  24x2  15x3  10x2  20x 
6x2  4x  8
 18x4  3x3  20x2  16x  8
39. A  /w
 1x  4212x  52
 1x212x2  1x2152  412x2  4152
 2x2  5x  8x  20
 2x2  3x  20
The area is 2x2  3x  20 square units.

 (5m) (5m)  (5m) (6)  (6) (5m)  (6) (6)


 25m2  30m  30m  3b
 25m2  60m  36

27. (10r  4) (10r  4)


 (10r)(10r)  (10r)(4)  (4)(10r)  (4)(4)
 100r2  40r  40r  16
 100r2  16
28. (7t  5) (7t  5)
 (7t)(7t)  (7t)(5)  (5) (7t)  (5)(5)
 49t2  35t  35t  25
 49t2  25
29. (8x  2y)(5x  4y)
 (8x)(5x)  (8x)(4y)  (2y)(5x)  (2y)(4y)
 40x2  32xy  10xy  8y2
 40x2  22xy  8y2
30. (11a  6b) (2a  3b)

40. A  2bh

 2 14x  3213x  22
1

 2 3 14x213x2  14x2122  13213x2  132122 4

 (11a) (2a)  (11a) (3b)  (6b) (2a)  (6b) (3b)

 22a2  33ab  12ab  18b2


 22a2  21ab  18b2

 2 312x2  8x  9x  64
1

31. ( p  4)( p2  2p  7)
 p( p2  2p  7)  4(p2  2p  7)

 2 [12x2  17x  6]
 6x2 

 p3  2p2 7p  4p2  8p  28

 p3  6p2  p  28

17
x
2

The area is

3

6x2

17
x
2

 3 square units.

32. (a  3)(a  8a  5)
 a(a2  8a  5)  3(a2  8a  5)
 a3  8a2  5a  3a2  24a  15
 a311a2  29a  15

389

Chapter 8

50. A  /w
 (x  2)(x  4)
 1x21x2  1x2142  1221x2  122142
 x2  4x  2x  8
 x2  2x  8
The area of the office is x2  2x  8 square feet.
51. Find the area of the square office.
A  s2
 92 or 81
Find the area of the new office.
A  x2  2x  8
 (9) 2  2(9)  8
 81  18  8
 91
The area of the new office is 91  81 or 10 square
feet bigger.
52a. Sample answer:
(30  5)(10  9)
 13021102  30192  51102  5192
 300  270  50  45
 665
52b. Sample answer:
(60  7)(100  2)
 6011002  60122  711002  7122
 6000  120  700  14
 6834
52c. Sample answer:

41. A  2h(b1  b2 )
1

 2 (5x  8) [ (x  7)  (2x  1) ]
1

 2 (5x  8) (3x  6)

 2 3 15x213x2  15x2162  182 13x2  182 162 4


1

 2 3 15x2  30x  24x  484


1

 2 3 15x2  6x  484
1

15 2
x
2

 3x  24

The area is

15 2
x
2

 3x  24 square units.

42. A  r
 13x  42 2
 13x  42 13x  42
 3 13x2 13x2  13x2 142  14213x2  142 142 4
  39x2  12x  12x  16 4
 19x2  24x  162
The area is (9x2  24x  16) square units.
43. V  Bh
 3 12a  22 1a  52 4 1a  12
 3 12a21a2  12a2152  1221a2  122152 4 1a  12
 32a2  10a  2a  104 1a  12
 12a2  8a  1021a  12
 a12a2  8a  102  112a2  8a  102
 2a3  8a2  10a  2a2  8a  10
 2a3  10a2  2a  10
The volume is 2a3  10a2  2a  10 cubic units.
44. V  Bh
 3 13y2 13y212y2 162 4 17y  32
 19y2  12y217y  32
 19y2 2 17y2  19y2 2132  112y2 17y2  112y2132
 63y3  27y2  84y2  36y
 63y3  57y2  36y
The volume is 63y3  57y2  36y cubic units.
45. Since a is the first integer, the second and third
integers are a  1 and a  2.
a(a  1) (a  2)
 a [ (a) (a)  (a)(2)  (1) (a)  (1) (2) ]
 a [ a2  2a  a  2]
 a [a2  3a  2]
 a3  3a2  2a
46. Sample answer: a  1; 1(2)(3)  6
47. Sample answer: 6; the result is the same as the
product in Exercise 46.
a3  3a2  2a  13  3(1) 2  2(1)
132
6
48. A  /w
 (5y  6) (2y  10)
 (5y) (2y)  (5y)(10)  (6)(2y)  (6)(10)
 10y2  50y  12y  60
 10y2  38y  60
The area is 10y2  38y  60 square feet.
49. Let x represent the length of a side of the square
office. Then the new office has dimensions x  2
and x  4.

Chapter 8

18  12 216  34 2
3
1
1 3
 8(6)  8 1 4 2  2 (6)  2 1 4 2
3

 48  6  3  8
3

 578
52d. Sample answer:

112  35 2110  23 2

 121102  12

123 2  35 1102  35 123 2


2

 120  8  6  5
2

 1345
53. Outer length: w  10 outer width: w  6
Area of concrete
 total area  area of pool.
 (w  10)(w  6)  w(w  4)
 1w21w2  1w2162  11021w2  1102162
 w(w)  w(4)
 w2  6w  10w  60  w2  4w
 12w  60
The area of the concrete is 300 square feet. Find w.
12w  60  300
12w  240
w  20
The pool should be 20 ft by 24 ft.
54. Sometimes; the product of x  1 and x2  2x  3
is x3  3x2  5x  3, which has 4 terms; the
product of y  1 and x3  2x2  3x is x3y  2x2y 
3xy  x3  2x2  3x, which has 6 terms.

390

f(x)  2x  5
f(4)  2(4)  5
 8  5
 13
67.
g1x2  x2  3x
g122  7  3 122 2  3122 4  7
 14  62  7
5
v
at
69.
68.
f(x)  2x  5
v
f(a  3)  2(a  3)  5
(t)a  (t) t
 2a  6  5
at  v
 2a  1
at
v
a
a

55. Multiplying binomials and two-digit numbers each


involve the use of the Distributive Property twice.
Each procedure involves four multiplications and
the addition of like terms. Answers should include
the following.
24  36  (4  20) (6  30)
 (4  20)6  (4  20)30
 (24  120)  (120  600)
 144  720
 864
The like terms in vertical two-digit
multiplication are digits with the same place
value.
56. C; (x  2)(x  4)  (x  4)(x  2)

66.

ta

 [ (x) (x)  (x) (4)  (2) (x)  (2) (4) ]  [ (x) (x) 

(x) (2)  (4) (x)  (4)(2) ]


 [x2  4x  2x  8][x2  2x  4x  8]
 [ x2  2x  8][x2  2x  8 ]
 [x2  2x  8]  [x2  2x  8]
 4x

70.

by
b

y

57. B; (x  y)(x2  xy  y2 )
 x(x2  xy  y2 )  y(x2  xy  y2 )
 x3  x2y  xy2  x2y  xy2  y3
 x3  y3

Page 457

ax  by  2cz
ax  by  ax  2cz  ax
by  2cz  ax

y
71.

4x  3y  7
4x  3y  4x  7  4x
3y  4x  7
3y
3

Maintain Your Skills

y

58. 3d(4d2  8d  15)


 (3d)(4d2 )  (3d)(8d)  (3d)(15)
 12d3  24d2  45d

72.

2cz  ax
b
2cz  ax
b
ax  2cz
b

(6a) 2

4x  7
3
4
7
x
3
3

62 


a2
2
 36a

73. (7x) 2  72  x2
 49x2

74. (9b) 2  92  b2
 81b2

59. 4y(7y2  4y  3)
 (4y)(7y2 )  (4y)(4y)  (4y)(3)
 28y3  16y2  12y

75. (4y2 ) 2  42 (y2 ) 2


 16y4

76. 12v3 2 2  22 1v3 2 2


 4v6

2 (5m2

 7m  8)
60. 2m
 (2m2 )(5m2 )  (2m2 )(7m)  (2m2 )(8)
 10m4  14m3  16m2
61. 3x(2x  4)  6(5x2  2x  7)
 6x2  12x  30x2  12x  42
 36x2  42
62. 4a(5a2  2a  7) 3(2a2  6a  9)
 20a3  8a2  28a  6a2  18a  27
 20a3  2a2  10a  27
63. The measure of the third angle is 180 minus the
measures of the other two angles.
180  (2x  1)  (5x  2)
 180  (2x  1)  (5x  2)
 181  7x
The measure of the third angle is (181  7x).
64. 1st angle  2x  1
 2(15)  1 or 31
2nd angle  5x  2
 5(15)  2 or 73
3rd angle  181  7x
 181  7(15) or 76
The angles measure 31, 73, and 76.
65. The lines intersect at the point (6, 3). Thus, the
system has one solution, (6, 3).

8-8
Page 461

77. 13g4 2 2  32 1g4 2 2


 9g8

Special Products
Check for Understanding

1. The patterns are the same except for their middle


terms. The middle terms have different signs.
2. The square of a difference is (a  b)2, which
equals a2  2ab  b2. The difference of squares is
the product of a  b and a  b or a2  b2.
3.
x3

x
x3

x
x
x

x x x
1
1
1

1
1
1

1
1
1

4. Sample answer: x  1 and x  1


(x  1)(x  1)  x2  12
 x2  1

391

Chapter 8

5. 1a  62 2  a2  21a2 162  62
 a2  12a  36
6. 14n  3214n  32  14n  32 2
 (4n) 2  2(4n)(3)  32
 16n2  24n  9
7. (8x  5) (8x  5)  (8x) 2  52
 64x2  25
8. (3a  7b) (3a  7b)  (3a) 2  (7b) 2
 9a2  49b2
2
2
2
2
9. (x  6y)  (x )  2(x2 )(6y)  (6y) 2
 x4  12x2y  36y2
10. (9  p) 2  92  2(9)(p)  p2
 81  18p  p2
11. The genetic makeup of the purebred golden can
be modeled by 1.0 G. The genetic makeup of the
purebred cinnamon can be modeled by 1.0 g.
Their offspring can be modeled by the product of
1.0 G and 1.0 g or 1.0 Gg.
12. 0%; All pups will be golden since only Gg
combinations are possible and the golden gene G
is dominant.

27. (2x  9y) 2  (2x) 2  2(2x)(9y)  (9y) 2


 4x2  36xy  81y2
2
28. 13n  10p2  13n2 2  213n2110p2  110p2 2
 9n2  60np  100p2
29. 15w  14215w  142  15w2 2  142
 25w2  196
30. (4d  13)(4d  13)  (4d) 2  132
 16d2  169
3
2
3
2
31. 1x  4y2  1x 2  21x3 214y2  14y2 2
 x6  8x3y  16y2
2
2
2
32. 13a  b 2  13a2 2 2  213a2 21b2 2  1b2 2 2
 9a4  6a2b2  b4
2
3
33. 18a  9b 218a2  9b3 2  18a2 2 2  19b3 2 2
 64a4  81b6
4
4
34. (5x  y)(5x  y)  (5x4 ) 2  y2
 25x8  y2
35.

123x  622  123x22  2123x2 162  62


4

 9x2  8x  36

36.

145x  1022  145x22  2145x2 1102  102


16

 25x2  16x  100

Page 462

37. (2n  1)(2n  1)(n  5)


 [ (2n) 2  12 ] (n  5)
 (4n2  1)(n  5)
 (4n2 ) (n)  (4n2 )(5)  (1) (n)  (1)(5)
 4n3  20n2  n  5
38. ( p  3)( p  4)( p  3)( p  4)
 [ ( p  3)( p  3) ] [ ( p  4) ( p  4) ]
 [p2  32 ] [p2  42 ]
 [p2  9] [ p2  16]
 (p2 )(p2 )  (p2 )(16)  (9)(p2 )  (9) (16)
 p4  16p2  9p2  144
 p4  25p2  144
39. Since Pams genes are Bb, her makeup can be
modeled by 0.5 B  0.5 b. Since Bob has blue
eyes, his genes are bb and can be modeled by
1.0 b. Their childrens genes can be modeled by
the product of 0.5 B  0.5 b and 1.0 b.

Practice and Apply

13. 1y  42 2  y2  21y2142  42
 y2  8y  16
14. 1k  821k  82  1k  82 2
 k2  21k2 182  82
 k2  16k  64
15. 1a  52 1a  52  1a  52 2
 a2  21a2 152  52
 a2  10a  25
2
2
16. 1n  122  n  21n2 1122  122
 n2  24n  144
17. 1b  721b  72  b2  72
 b2  49
18. (c  2) (c  2)  c2  22
 c2  4
2
19. 12g  52  12g2 2  212g2152  52
 4g2  20g  25
20. 19x  32 2  19x2 2  219x2 132  32
 81x2  54x  9

(0.5 B  0.5 b) (1.0 b)  (0.5 B) (1.0 b)  (0.5 b) (1.0 b)


 0.5 Bb  0.5 bb

40. Since only bb will be blue eyes, we see that 0.5 or


1
of their children should have blue eyes. The
2
1
probability is 2.
41. Sample answer:
We pick 2.
Square the number: 22  4
Add twice the original: 4  2(2)  8
Add 1: 8  1  9
Take square root: 19  3
Subtract original number: 3  2  1
Yes, the result is 1.
42. Square the number: a2
Add twice the original: a2  2a
Add 1: a2  2a  1

21. 17  4y2 2  72  2172 14y2  14y2 2


 49  56y  16y2
22. 14  6h2 2  42  214216h2  16h2 2
 16  48h  36h2
23. (11r  8) (11r  8)  (11r) 2  (8) 2
 121r2  64
24. (12p  3) (12p  3)  (12p) 2  32
 144p2  9
25. 1a  5b2 2  a2  21a2 15b2  15b2 2
 a2  10ab  25b2
2
26. 1m  7n2  m2  21m2 17n2  17n2 2
 m2  14mn  49n2

Chapter 8

392

Sample answer: (10x  11)(10x  11)


(10x  11)(10x  11)
 (10x  11) 2
 (10x) 2  2(10x) (11)  112
 100x2  220x  121
49. C; (a  b) 2  a2  2ab  b2
 (a2  b2 )  2(ab)
 40  2(12)
 40  24
 16
50. B; Since (x  y)(x  y)  x2  y2, we have
x2  y2  (x  y)(x  y)
 (10)(20)
 200.
51a. (a  b)(a  b) (a  b)
 (a  b)(a2  2ab  b2 )  b(a2  2ab  b2 )
 a(a2  2ab  b2 )  b(a2  2ab2  b2 )
 a3  2a2b  ab2  ba2  2ab2  b3
 a3  3a2b  3ab2  b3
51b. (a  b) 3  a3  3a2b  3ab2  b3
(x  2) 3  x3  3x2 (2)  3x(2) 2  23
 x3  6x2  12x  8
51c.

43. (a  1) 2  a2  2(a)(1)  12
 a2  2a  1
2  2a  1 is the square of a  1.
Thus, a
44. Take square root: 21a  12 2  a  1
Subtract original number: (a  1)  a  1
The result is 1.
45. Since each seating level is about 1 meter wide,
the second and third levels have radii s  2 and
s  3 meters, respectively.
46. A  (s  3) 2  (s  2) 2
 1s2  2132 1s2  32 2  1s2  21221s2  22 2
 (s2  6s  9)  (s2  4s  4)
 1s2  6s  9  s2  4s  42
 12s  52
 6.3s  15.7
The area is about 6.3s  15.7 square meters.
1

47. Area of a trapezoid  2 (height) (base 1  base 2)


A1 
A2 

1
1a
2
1
1a
2

 b2 1a  b2
 b2 1a  b2
a

ab
a

b
b

b
b
a

ab

3 12 1a  b2 1a  b2 4  3 12 1a  b21a  b2 4

Total area of shaded region




 1a  b2 1a  b2
a

ab

Page 463

ab

ab

a
ab

 20y2  29y  6
55. (3n  5)(8n  5)
 3n(8n)  3n(5)  (5)(8n)  (5) (5)
 24n2  15n  40n  25
 24n2  25n  25

Area of rectangle  (a  b)(a  b)


a

A1

ab

a
b

Maintain Your Skills

52. 1x  221x  72  1x21x2  1x2172  1221x2  122172


 x2  7x  2x  14
 x2  9x  14
53. 1c  921c  32  1c21c2  1c2132  1921c2  192132
 c2  3c  9c  27
 c2  6c  27
54. (4y  1)(5y  6)
 (4y)(5y)  (4y)(6)  (1)(5y)  (1) (6)
 20y2  24y  5y  6

A2

56. (x  2)(3x2  5x  4)
 x(3x2  5x  4)  2(3x2  5x  4)
 3x3  5x2  4x  6x2  10x  8
 3x3  11x2  14x  8

b
ab

48. The product of two binomials is also a binomial


when the two binomials are the sum and the
difference of the same two terms. Answers should
include the following.
Sample answer: 12x  132 12x  132
12x  13212x  132  12x2 2  132
 4x2  169

57. (2k  5) (2k2  8k  7)


 2k(2k2  8k  7)  5(2k2  8k  7)
 4k3  16k2  14k  10k2  40k  35
 4k3  6k2  26k  35

393

Chapter 8

58. 6(x  2)  4  5(3x  4)


6x  12  4  15x  20
6x  16  15x  20
9x  16  20
9x  36
x4
59. 3(3a  8)  2a  4(2a  1)
9a  24  2a  8a  4
7a  24  8a  4
15a  24  4
15a  20
20
4
a  15 or 3

Find y.
2x  3y  4
2152  3y  4
10  3y  4
3y  6
y  2
The solution is (5, 2).
65. Solve for y.
5x  5y  35
5y  5x  35
y  x  7
The slope of the given line is 1. The slope of any
line perpendicular to it is 1.
y  y1  m1x  x1 2

60. p( p  2)  3p  p( p  3)
p2  2p  3p  p2  3p
p2  5p  p2  3p
5p  3p
8p  0
p0
61. y(y  4)  2y  y(y  12)  7
y2  4y  2y  y2  12y  7
y2  2y  y2  12y  7
2y  12y  7
14y  7
y

7
14

or

y  2  11x  1322
y2x3
yx5
66. Solve for y.
2x  5y  3
5y  2x  3
2

The slope of the given line is 5. The slope of any


5
line perpendicular to it is 2.

1
2

y  y1  m1x  x1 2
5
y  7  2 (x  (2))

62. Add the equations.


3
x
4
3
x
4
6
x
4

1
y
5
1
y
5

y  5x  5

5

 5

y  7  2 (x  2)

0

y  7  2 x  5

x0
Find y.

y  2 x  2

3
x
4

3
102
4




1
y
5
1
y
5
1
y
5

5
5

67. Solve for y.


5x  y  2
y  5x  2
The slope of the given line is 5. The slope of any
1
line perpendicular to it is 5. The y-intercept is 6.
y  mx  b

5
5
5

y  25
The solution is (0, 25).
63. Multiply the first equation by 3. Then add.
6x  3y  30
5x  3y  3
11x
 33
x3
Find y.
2x  y  10
2132  y  10
6  y  10
y  4
y  4
The solution is (3, 4).
64. Rewrite the equations.
2x  3y  4
x  3y  11
Multiply the second equation by 1. Then add.
2x  3y  4
x  3y  11
3x
 15
x5
Chapter 8

y  5x  6

68. a  a  1n  12d
n
1
a18  3  118  12142
 3  17142
 3  68
 71
69. a  a  (n  1)d
n
1
a  5  112  12162
12
 5  1112162
 5  66
 61
70. b

394

Chapter 8 Study Guide and Review


Page 464
1.
3.
5.
7.
9.

25.

16a3b2x4y
48a4bxy3

2.
4.
6.
8.
10.

26.

5 8

(a) b
a5b2




Pages 464468
3

11. y  y  y  y
 y7
12. (3ab) (4a2b3 )  [ 3  (4) ] (a  a2 )(b  b3 )
 12a3b4
2
3
4
13. (4a x)(5a x )  [ (4)  (5) ] (a2  a3 )(x  x4 )
 20a5x5
2
3
3
2
3
14. (4a b)  4 (a ) b3
 64a6b3
2
15. 13xy2 14x2 3  132 2x2y2 142 3x 3
 9  x2  y2  64  x3
 (9  64) (x2  x3 )(y2 )
 576x5y2
2
4
16. 12c d2 13c2 2 3  122 4 1c2 2 4d4 132 3 1c2 2 3
 16c8d4 1272c6
 [16  (27) ] (c8  c6 )(d4 )
 432c14d4

27.

13bc
4d 2

2 3

28.

(3bc )
(4d) 3

(3) 3b3 (c2 ) 3


(4) 3 (d) 3

36.

23.

27b2
14b3

z3
x2




27b3c6
64d3
27 b  2
14 b3
27 2132
1b
2
14

1 21 2

 14 1b2
27


24.

(3a3bc2 ) 2
18a2b3c4









42 (a1 ) 2
22 (a4 ) 2

42a2
4a8

144 21aa 2
2

2
6

141 21a1 2
3

1
64a6

135x5xy y 20  1
2

29. 2.4  105  240,000

2 6

8.4  106
1.4  109

10
18.4
1.4 21 10 2
6
9

 6  10 6192
 6  103
or 6000
37. (3  102 )(5.6  10 8 )  (3  5.6)(102  10 8 )
 16.8  10 6
 1.68  10  10 6
 1.68  10 5
or 0.0000168
38. The polynomial n  2p2 has two terms, n and
2p2, whose degrees are 1 and 2, respectively.
Thus, the degree of n  2p2 is 2, the greater of
1 and 2.
39. The polynomial 29n2  17n2t2 has two terms,
29n2 and 17n2t2, whose degrees are 2 and 4,
respectively. Thus, the degree of 29n2  17n2t2 is
4, the greater of 2 and 4.


1
z3
22. x 2y0z3  x2  1  1

3.14  10 4  0.000314
4.88  109  4,880,000,000
0.00000187  1.87  10 6
796  103  7.96  102  103
 7.96  105
34. 0.0343  102  3.43  102  102
 3.43  10 4
5
35. 12  10 213  106 2  12  321105  106 2
 6  1011
or 600,000,000,000
30.
31.
32.
33.

2 3

(4a1 ) 2
(2a4 ) 2

18. 15a2 2 3  71a6 2  152 3 1a2 2 3  7a6


 125a6  7a6
 132a6
2
2
3
4
3
19. 3 13 2 4  33 4
 312 or 531,441
21.

1 21 21 21 2

1 21 2

bx3
3ay2


 4 3a 6

 6a

(4 21 )(a28 )

 2m4n8

13y2 0
6a

a5b8
a5b2
a5 b8
 a5 b2

17. 2 1m2n4 2 2  2 1m2 2 2 1n4 2 2

20.

 (a55 )(b82 )
 (a0 )(b6 )
 (1) (b6 )
 b6

Lesson-by-Lesson Review
331

Power of a Power
monomial
zero exponent
FOIL method
Product of Powers

a b x
y
116
48 21 a 21 b 21 x 21 y 2

1 34
1a
21b21 21x41 21y13 2
3
1 1 1 3 2
a b x y
3
1 1 b x3 1
3 a 1 1 y2

Vocabulary and Concept Check

negative exponent
Quotient of Powers
trinomial
polynomial
binomial

27b
14

32 (a3 ) 2b2 (c2 ) 2


18a2b3c4
9a6b2c4
18a2b3c4
9 a6 b2 c4
18 a2 b3 c4
1
(a62 )(b23 )(c44 )
2
1 4 1 0
a b c
2
1 a4 1
(1)
2 1 b
a4
2b

1 21 21 21 2
1 21 2

395

Chapter 8

55. 2x (x  y2  5)  5y2 (3x  2)


 2x(x)  2x(y2 )  2x(5)  5y2 (3x)
 5y2 (2)
 2x2  2xy2  10x  15xy2  10y2
 2x2  17xy2  10x  10y2
56. m(2m  5)  m  2m(m  6)  16
2m2  5m  m  2m2  12m  16
2m2  4m  2m2  12m  16
4m  12m  16
8m  16
m2
57. 213w  w2 2  6  2w1w  42  10
6w  2w2  6  2w2  8w  10
6w  6  8w  10
14w  6  10
14w  16

40. The polynomial 4xy  9x3z2  17rs3 has three


terms, 4xy, 9x3z2, and 17rs3, whose degrees are 2,
5, and 4, respectively. Thus, the degree of
4xy  9x3z2  17rs3 is 5, the greatest of 2, 5, and 4.
41. The polynomial 6x5y  2y4  4  8y2 has four
terms, 6x5y, 2y4, 4, and 8y2, whose degrees
are 6, 4, 0, and 2, respectively. Thus, the degree of
6x5y  2y4  4  8y2 is 6, the greatest of 6, 4, 0,
and 2.
42. The polynomial 3ab3  5a2b2  4ab has three
terms, 3ab3, 5a2b2, and 4ab, whose degrees are
4, 4, and 2, respectively. Thus, the degree of
3ab3  5a2b2  4ab is 4, the greatest of 4, 4, and 2.
43. The polynomial 19m3n4  21m5n has two terms,
19m3n4 and 21m5n, whose degrees are 7 and 6,
respectively. Thus, the degree of 19m3n4  21m5n
is 7, the greater of 7 and 6.
44. 3x4  x  x2  5  3x4  x1  x2  5x0
 3x4  x2  x  5
2
3
45. 2x y  27  4x4  xy  5x3y2
 2x2y3  27x0  4x4  x1y  5x3y2
 4x4  5x3y2  2x2y3  xy  27
46. (2x2  5x  7)  (3x3  x2  2)
 12x2  5x  72  13x3  x2  22
 3x3  12x2  x2 2  5x  17  22
 3x3  x2  5x  5
2
47. (x  6xy  7y2 )  (3x2  xy  y2 )
 1x2  3x2 2  16xy  xy2  17y2  y2 2
 4x2  5xy  6y2
48. (7z2  4)  (3z2  2z  6)
 17z2  42  13z2  2z  62
 (7z2  3z2 )  2z  (4  6)
 4z2  2z  10
49. (13m4  7m  10)  (8m4  3m  9)
 113m4  8m4 2  17m  3m2  110  92
 21m4  10m  1
50. (11m2n2  4mn  6)  (5m2n2  6mn  17)
 (11m2n2  5m2n2 )  (4mn  6mn)
 (6  17)
 16m2n2  10mn  11
51. 15p2  3p  492  12p2  5p  242
 15p2  3p  492  12p2  5p  242
 15p2  2p2 2  13p  5p2  149  242
 7p2  2p  25
52. b(4b  1)  10b
 b(4b)  b(1)  10b
 4b2  b  10b
 4b2  9b
53. x13x  52  71x2  2x  92
 x13x2  x152  71x2 2  712x2  7192
 3x2  5x  7x2  14x  63
 10x2  19x  63
54. 8y(11y2  2y  13)  9(3y3  7y  2)
 8y(11y2 )  8y(2y)  8y(13)  9(3y3 )
 917y2  9122
 88y3  16y2  104y  27y3  63y  18
 61y3  16y2  167y  18

Chapter 8

16

w  14 or 7 or 17

58. 1r  321r  72  1r21r2  1r2172  1321r2  132172


 r2  7r  3r  21
 r2  4r  21
59. (4a  3)(a  4)
 (4a)(a)  (4a)(4)  (3) (a)  (3)(4)
 4a2  16a  3a  12
 4a2  13a  12
60. (3x  0.25) (6x  0.5)
 (3x) (6x)  (3x) (0.5)  0.25(6x)  (0.25) (0.5)
 18x2  1.5x 1.5x  0.125
 18x2  0.125

61. (5r  7s)(4r  3s)


 (5r)(4r)  (5r)(3s)  (7s)(4r)  (7s) (3s)
 20r2  15rs  28rs  21s2
 20r2  13rs  21s2
62. (2k  1) (k2  7k  9)
 2k(k2  7k  9)  1(k2  7k  9)
 2k3  14k2  18k  k2  7k  9
 2k3  15k2  11k  9
63. (4p  3)(3p2  p  2)
 4p(3p2  p  2)  3(3p2  p  2)
 12p3  4p2  8p  9p2  3p  6
 12p3  13p2  11p  6
64. 1x  621x  62  x2  62
 x2  36
2
65. 14x  72  14x2 2  214x2172  72
 16x2  56x  49
2
66. 18x  52  18x2 2  218x2152  52
 64x2  80x  25
67. 15x  3y215x  3y2  15x2 2  13y2 2
 25x2  9y2
2
2
68. (6a  5b)  (6a)  2(6a) (5b)  (5b) 2
 36a2  60ab  25b2
2
69. (3m  4n)  (3m) 2  2(3m) (4n)  (4n) 2
 9m2  24mn  16n2

396

16. 13  103 212  104 2  13  221103  104 2


 6  107
or 60,000,000

Chapter 8 Practice Test


Page 469
1. (42 ) (43 )  (4  4)(4  4  4)
 44444
 45 and 165  45.
2.

1
5

17.

 51

5. (12abc)(4a2b4 )  (12  4)(a  a2 )(b  b4 )c


 48a3b5c

135m22  135 22m2

7. 13a2 4 1a5b2 2  132 4 1a2 4 1a5 2 2 1b2 2


 81a4  a10  b2
 81(a4  a10 )b2
 81a14b2

1 21 2
1m1 21n1 2
2

n2

 m2
9a2bc2
63a4bc

1639 21aa 21bb 21cc 2


1
 1 7 2 (a24 )(b11 )(c21 )
2

11.

48a2bc5
(3ab3c2 ) 2

117 21a1 2 (1) 11c 2


2








9
5

23. (x3  3x2y  4xy2  y3 )  (7x3  x2y 


9xy2  y3 )
 (x3  3x2y  4xy2  y3 )  (7x3  x2y 
9xy2  y3 )
 (x3  7x3 )  (3x2y  x2y)  (4xy2  9xy2 )
(y3  y3 )
 6x3  4x2y  13xy2
24. To find the measure of the third side, subtract the
measures of the two sides given from the
perimeter.
(11x2  29x  10)  (x2  7x  9) 
(5x2  13x  24)
 (11x2  29x  10)  (x2  7x  9) 
(5x2  13x  24)
 (11x2  x2  5x2 )  (29x  7x  13x) 
(10  9  24)
 5x2  23x  23
25. 1h  52 2  h2  21h2152  52
 h2  10h  25

c
7a2

10
12.85
1.86 21 10 2

22. 15a  3a2  7a3 2  12a  8a2  42


 17a3 2  13a2  8a2 2  15a  2a2  4
 7a3  5a2  7a  4

5xy  7  2y4  x2y3


 5xy1  7y0  2y4  x2y3
 2y4  x2y3  5xy  7

 7 a 2b0c


2y2  8y4  9y  2y2  8y4  9y1


 8y4  2y2  9y
21. The polynomial 5xy  7  2y4  x2y3 has four
terms, 5xy, 7, 2y4, and x2y3, whose degrees are
2, 0, 4, and 5, respectively. Thus, the degree of
5xy  7  2y4  x2y3 is 5, the greatest of 2, 0, 4,
and 5.

m n4
m3 n2

 (m13 )(n42 )
 m 2n2

10.

3

4(3)

 (1.53)(109 5 )
 1.53  104
It will take about 1.53  104 seconds or 4.25
hours.
20. The polynomial 2y2  8y4  9y has three terms,
2y2, 8y4, and 9y, whose degrees are 2, 4, and 1,
respectively. Thus, the degree of 2y2  8y4  9y is
4, the greatest of 2, 4, and 1.

8. (5a2 ) (6b3 ) 2  (5)(a2 )(6) 2 (b3 ) 2


 (5)(a2 )(36)(b6 )
 (5  36) (a2 )(b6 )
 180a2b6

2.85  109
1.86  105

4

19. To find the time, divide the distance by the rate


d
tr .

 25m2

9.

10
114.72
3.2 21 10 2

18. 115  10 7 213.1  104 2  115  3.12110 7  104 2


 46.5  10 3
 4.65  10  10 3
 4.65  10 2
or 0.0465

4. (a2b4 ) (a3b5 )  (a2  a3 )(b4  b5 )


 a5b9

mn4
m3n2

 (4.6)(10
 4.6  10 1
or 0.46

3. Sample answer: A monomial is a number, variable,


or product of numbers and variables; 6x2.

6.

14.72  10  4
3.2  10  3

48a2bc5
(3) 2 (a) 2 (b3 ) 2 (c2 ) 2
48a2bc5
9a2b6c4
48 a2 b c5
9 a2 b6 c4
16 22 16 54
a
b
c
3
16 0 5
a b c
3
16
1 c
(1) b5 1
3
16c
3b5

1 21 21 21 2
1 2 1 21 2

12. 46,300  4.63  104


13. 0.003892  3.892  10 3
14. 284  103  2.84  102  103
 2.84  105
9
15. 52.8  10
 5.28  10  10 9
 5.28  10 8

397

Chapter 8

26. (4x  y) (4x  y)  (4x) 2  y2


 16x2  y2

6. A; The cost of 4 sets of table and chairs plus the


cost of b bookcases must be less than or equal to
$7500.
415502  125b  7500
7. B; Let s  amount Sophia spent,
and a  amount Allie spent.
Write a system of equations.
s  a  122
s  2a  25
Solve the system by substitution.
2a  25  a  122
3a  25  122
3a  147
a  49
Allie spent $49.
8. D; (2x3 )(4x4 )  (2  4)(x3  x4 )
 8x7
9. B; 0.00037  3.7  104
10. A; 13x2  4x  52  1x2  2x  12
 13x2  4x  52  1x2  2x  12
 13x2  1x2 22  14x  2x2  15  1122
 2x2  2x  4
11. a  a  (n  1)d
n
1
a15  20  (15  1)(9)
 20  (14)(9)
 20  126
 106
12. Each y-value is 4 times the corresponding
x-value.
f (x)  4x or y  4x
13. To find the y-intercept, let x  0.
3x  2y  8  0
3(0)  2y  8  0
2y  8  0
2y  8
y4
14. Graph 3x  y  2. Since the boundary is included
in the solution set, draw a solid line.
Test (0, 0).
3x  y  2
3(0)  0  2
0  2 true
Shade the half-plane that contains (0, 0).

27. 3x2y3 (2x  xy2 )  3x2y3 (2x)  3x2y3 (xy2 )


 6x3y3  3x3y5
28. 12a2b  b2 2 2  12a2b2 2  212a2b2 1b2 2  1b2 2 2
 4a4b2  4a2b3  b4
29. (4m  3n)(2m  5n)

 (4m)(2m)  (4m) (5n)  (3n) (2m)  (3n) (5n)


 8m2  20mn  6mn  15n2
 8m2  14mn  15n2

30. (2c  5) (3c2  4c  2)


 2c(3c2  4c  2)  5(3c2  4c  2)
 6c3  8c2  4c  15c2  20c  10
 6c3  7c2  16c  10
31. 2x (x  3)  2(x2  7)  2
2x2  6x  2x2  14  2
2x2  6x  2x2  12
6x  12
x2
32. 3a1a2  52  11  a13a2  42
3a3  15a  11  3a3  4a
15a  11  4a
11a  11  0
11a  11
a1
33. C; 31x  y2 2  31x2  2xy  y2 2
 3182
 24

Chapter 8 Standardized Test Practice


Pages 470471
1. A; Mean of first 5 games: 72
Median of first 5 games: 70
Mode of first 5 games: 70
Mean of 6 games: 65
Median of 6 games: 70
Mode of 6 games: 70
Mean changes the most.
2. D;

2100
60

5600
x

3. D; r 

20,750  20,542
4

2100x  336,000
x  160
4. A; From the graph, we see that the y-intercept is
1
1 and the slope is 5. Thus, the equation is
1
y  5x  1.

3x  y  2

5. B; Find the y-intercept.


y  mx  b
4  2112  b
42b
2b
Write the equation.
y  mx  b
y  2x  2

Chapter 8

15. P  Q
 (3x2  2x  1)  (x2  2x  2)
 (3x2  (x2 ))  (2x  2x)  (1  (2))
 2x2  3

398

24a. A  /w
 (3m  3)(m  1)
 (3m)(m)  (3m)(1)  (3)(m)  (3) (1)
 3m2  3m  3m  3
 3m2  3
24b. A  /w
 (3m  3)(m  4)
 (3m)(m)  (3m)(4)  (3)(m)  (3) (4)
 3m2  12m  3m  12
 3m2  9m  12
24c. V  /wh
 (3m  3)(m  1) (m  4)
 [ (3m)(m)  (3m)(1)  (3)m
 (3)(1) ] (m  4)
 (3m2  3m  3m  3)(m  4)
 (3m2  3)(m  4)

16. (x2  1)(x  3)


 (x2 )(x)  (x2 ) (3)  (1)(x)  (1) (3)
 x3  3x2  x  3
17. A; The x-coordinate of A is 6. The y-coordinate of
B is 5.
The quantity in column A is greater.
6(x  1)
3
18. A; 4x  10  20
8
4x  30
6(x  1)  24
x

30
4

The quantity in
19. B; x  3y  2
x  3y  0
2x
2
x1
The quantity in
20. C;

2b3c2
4bc

15
2

6x  6  24
6x  18
x  3
column A is greater.
3x  8y  6
x  8y  2
4x
8
x2
column B is greater.

or

124 21bb 21cc 2


1
 1 2 2 (b31 )(c21 )


3

1
 2 b4c

 (3m 2 ) (m )  (3m 2 ) (4)  (3) (m )  (3) (4)

 3m3  12m2  3m  12
24d. V  3m3  12m2  3m  12
 3(2) 3  12(2) 2  3(2)  12
 3(8)  12(4)  6  12
 24  48  18
 54
The volume is 54 cm3.

10b4
20b8c1

11020 21bb 21c1 2


1
 1 2 2 (b48 )c

4
8

1

1
 2 b4c

The two quantities are equal.


21. A; 5.01  102
50.1  104
 0.0501
 0.00501
The quantity in column A is greater.
22. C; The degree of
The degree of
x2  5  6x  13x3 10  y  2y2  4y3
is 3.
is 3.
The two quantities are equal.
23. B; (m  n) 2  m2  2mn  n2
 (m2  n2 )  2(mn)
 10  2(6)
 10  12
 2
(m  n) 2  m2  2mn  n2
 (m2  n2 )  2(mn)
 10  2(3)
 10  6
 16
The quantity in column B is greater.

399

Chapter 8

Chapter 9
Page 473

Factoring
3. Sample answer: 5x2 and 10x3
5x2  5  x  x
10x3  2  5  x  x  x
GCF: 5  x  x or 5x2
4. List all pairs of numbers whose product is 8.
18
24
factors: 1, 2, 4, 8
composite
5. List all pairs of numbers whose product is 17.
1  17
factors: 1, 17
prime
6. List all pairs of numbers whose product is 112.
2  56
4  28
1  112
7  16
8  14
factors: 1, 2, 4, 7, 8, 14, 16, 28, 56, 112
composite
7. 45  3  15

Getting Started

1. 3(4  x)  3  4  3  x
 12  3x
2. a(a  5)  a  a  a  5
 a2  5a
2
3. 7(n  3n  1)  7(n2 )  (7)(3n)  (7)(1)
 7n2  21n  7
4. 6y(3y  5y2  y3 )  6y(3y)  6y(5y2 )  6y(y3 )
 18y2  30y3  6y4
5. (x  4) (x  7)  (x)(x)  (x)(7)  (4)(x)  (4)(7)
 x2  7x  4x  28
 x2  11x  28
6. (3n  4)(n  5)
 (3n)(n)  (3n)(5)  (4)(n)  (4)(5)
 3n2  15n  4n  20
 3n2  11n  20
7. (6a  2b)(9a  b)
 (6a)(9a)  (6a)(b)  (2b)(9a)  (2b)(b)
 54a2  6ab  18ab  2b2
 54a2  12ab  2b2
8. (x  8y)(2x  12y)

8.

 (x)(2x)  (x) (12y)  (8y) (2x)  (8y) (12y)


 2x2  12xy  16xy  96y2
 2x2  4xy  96y2

9.

9. (a  b) 2  a2  2ab  b2
(y  9) 2  y2  2(y) (9)  92
 y2  18y  81
10. (a  b) 2  a2  2ab  b2
(3a  2) 2  (3a) 2  2(3a)(2)  22
 9a2  12a  4
11. (a  b) (a  b)  a2  b2
(n  5) (n  5)  n2  52
 n2  25
12.
(a  b) (a  b)  a2  b2
(6p  7q) (6p  7q)  (6p) 2(7q) 2
 36p2  49q2
13. 1121 is the positive square root of 121.
112  121 S 1121  11

10.
11.
12.

13.

14.

14. 10.0064 is the positive square root of 0.0064.


(0.08) 2  0.0064 S 10.0064  0.08
15.

16.

25

3 36 is the positive square root of

156 22  2536 S 3 2536  56

8
98

is the positive square root of

127 22  494  988 S 3 988  27

9-1

25
.
36

15.
8
98

or

4
.
49

Factors and Greatest Common


Factors

Page 477

16.

Check for Understanding

1. False; 2 is a prime number that is even.


2. Two numbers are relatively prime if their GCF is 1.
Chapter 9

400

 3  3  5 or 32  5
32  1  32
 1  2  16
 1  2  2  8
 1  2  2  2  4
 1  2  2  2  2  2 or 1  25
150  1  150
 1  2  75
 1  2  3  25
 1  2  3  5  5 or 1  2  3  52
2  22pp
4p
39b3c2  3  13  b  b  b  c  c
100x3yz2  1  100  x  x  x  y  z  z
 1  2  50  x  x  x  y  z  z
 1  2  2  25  x  x  x  y  z  z
 1  2  2  5  5  x  x  x  y  z  z
Factor each monomial and circle the common
prime factors.
10  2  5
15  3  5
GCF: 5
Factor each monomial and circle the common
prime factors.
18xy  2  3  3  x  y
36y2  2  2  3  3  y  y
GCF: 2  3  3  y or 18y
Factor each monomial and circle the common
prime factors.
54  2  3  3  3
63  3  3  7
180  2  2  3  3  5
GCF: 3  3 or 9
Factor each monomial and circle the common
prime factors.
25n  5  5  n
21m  3  7  m
GCF: 1

factors: 1, 2, 3, 6, 7, 9, 14, 18, 21, 42, 63, 126


composite
27. List all pairs of numbers whose product is 304.
2  152
4  76
1  304
16  19
8  38
factors: 1, 2, 4, 8, 16, 19, 38, 76, 152, 304
composite
28. List all pairs of whole numbers whose product is
96. These are the possible dimensions for the
rectangle. Find the perimeter of the rectangle
associated with each pair and identify the least
perimeter among them.

17. Factor each monomial and circle the common


prime factors.
12a2b  2  2  3  a  a  b
90a2b2c  2  3  3  5  a  a  b  b  c
GCF: 2  3  a  a  b or 6a2b
18. Factor each monomial and circle the common
prime factors.
15r2  3  5  r  r
35s2  5  7  s  s
70rs  2  5  7  r  s
GCF: 5
19. List all pairs of numbers whose product is 120
and both numbers are at least 5.
6  20
5  24
8  15 10  12
Since either number in each pair can be the
number of rows and the other number the
number of plants, Ashley can arrange the plants
in any of the following ways.
5 rows of 24 plants
6 rows of 20 plants
8 rows of 15 plants
10 rows of 12 plants
12 rows of 10 plants
15 rows of 8 plants
20 rows of 6 plants
24 rows of 5 plants

Pages 477479

A  lw
96  96  1
96  48  2
96  32  3
96  24  4
96  16  6
96  12  8

The minimum perimeter is 40 mm.


29. Identify the greatest perimeter in the chart in
Exercise 28. The maximum perimeter is 194 mm.
30. Find the GCF of 18 and 24. Factor each number
and circle the common prime factors.
18  2  3  3
24  2  2  2  3
GCF: 2  3 or 6
Each cellophane package should contain
6 cookies.
31. Since 18  3  6 and 24  4  6, 3 cellophane
packages will go in each box of 18 cookies, and
4 cellophane packages will go in each box of
24 cookies.
32. 39  3  13
33. 98  1  98
 1  2  49
 1  2  7  7 or 1  2  72
34. 117  3  39
 3  3  13 or 32  13
35. 102  2  51
 2  3  17
36. 115  1  115
 1  5  23
37. 180  2  90
 2  2  45
 2  2  3  15
 2  2  3  3  5 or 22  32  5
38. 360  2  180
 2  2  90
 2  2  2  45
 2  2  2  3  15
 2  2  2  3  3  5 or 23  32  5
39. 462  1  462
 1  2  231
 1  2  3  77
 1  2  3  7  11

Practice and Apply

20. List all pairs of numbers whose product


1  19
factors: 1, 19
prime
21. List all pairs of numbers whose product
1  25
55
factors: 1, 5, 25
composite
22. List all pairs of numbers whose product
2  40
4  20
1  80
8  10
5  16
factors: 1, 2, 4, 5, 8, 10, 16, 20, 40, 80
composite
23. List all pairs of numbers whose product
1  61
factors: 1, 61
prime
24. List all pairs of numbers whose product
1  91
7  13
factors: 1, 7, 13, 91
composite
25. List all pairs of numbers whose product
1  119
7  17
factors: 1, 7, 17, 119
composite
26. List all pairs of numbers whose product
2  63
3  42
1  126
7  18
9  14
6  21

2l  2w  P
2(96)  2(1)  194
2(48)  2(2)  100
2(32)  2(3)  70
2(24)  2(4)  56
2(16)  2(6)  44
2(12)  2(8)  40

is 19.

is 25.

is 80.

is 61.

is 91.

is 119.

is 126.

401

Chapter 9

40. 66d4  2  33  d  d  d  d
 2  3  11  d  d  d  d
2 2
41. 85x y  5  17  x  x  y  y
42. 49a3b2  7  7  a  a  a  b  b
43. 50gh  2  25  g  h
 255gh
44. 128pq2  2  64  p  q  q
 2  2  32  p  q  q
 2  2  2  16  p  q  q
 22228pqq
 222224pqq
 2222222pqq
45. 243n3m  3  81  n  n  n  m
 3  3  27  n  n  n  m
 3339nnnm
 33333nnnm
46. 183xyz3  1  183  x  y  z  z  z
 1  3  61  x  y  z  z  z
2bc2  1  169  a  a  b  c  c
47. 169a
 1  13  13  a  a  b  c  c
48. Factor each monomial and circle the common
prime factors.
27  3  3  3
72  2  2  2  3  3
GCF: 3  3 or 9
49. Factor each monomial and circle the common
prime factors.
18  2  3  3
35  5  7
GCF: 1
50. Factor each monomial and circle the common
prime factors.
32  2  2  2  2  2
48  2  2  2  2  3
GCF: 2  2  2  2 or 16
51. Factor each monomial and circle the common
prime factors.
84  2  2  3  7
70  2  5  7
GCF: 2  7 or 14
52. Factor each monomial and circle the common
prime factors.
16  2  2  2  2
20  2  2  5
64  2  2  2  2  2  2
GCF: 2  2 or 4
53. Factor each monomial and circle the common
prime factors.
42  2  3  7
63  3  3  7
105  3  5  7
GCF: 3  7 or 21
54. Factor each monomial and circle the common
prime factors.
15a  3  5  a
28b2  2  2  7  b  b
GCF: 1

Chapter 9

55. Factor each monomial and circle the common


prime factors.
24d2  2  2  2  3  d  d
30c2d  2  3  5  c  c  d
GCF: 2  3  d or 6d
56. Factor each monomial and circle the common
prime factors.
20gh  2  2  5  g  h
36g2h2  2  2  3  3  g  g  h  h
GCF: 2  2  g  h or 4gh
57. Factor each monomial and circle the common
prime factors.
21p2q  3  7  p  p  q
32r2t  2  2  2  2  2  r  r  t
GCF: 1
58. Factor each monomial and circle the common
prime factors.
18x  2  3  3  x
30xy  2  3  5  x  y
54y  2  3  3  3  y
GCF: 2  3 or 6
59. Factor each monomial and circle the common
prime factors.
28a2  2  2  7  a  a
63a3b2  3  3  7  a  a  a  b  b
91b3  7  13  b  b  b
GCF: 7
60. Factor each monomial and circle the common
prime factors.
14m2n2  2  7  m  m  n  n
18mn  2  3  3  m  n
2m2n3  2  m  m  n  n  n
GCF: 2  m  n or 2mn
61. Factor each monomial and circle the common
prime factors.
80a2b  2  2  2  2  5  a  a  b
96a2b3  2  2  2  2  2  3  a  a  b
bb
128a2b2  2  2  2  2  2  2  2  a  a
 b b
GCF: 2  2  2  2  a  a  b or 16a2b
62. The next five pairs of twin primes are: 5 and 7,
11 and 13, 17 and 19, 29 and 31, and 41 and 43.
63. Find the GCF of 75 and 90. Factor each number
and circle the common prime factors.
75  3  5  5
90  2  3  3  5
GCF: 3  5 or 15
The maximum number of rows is 15.
64. Since there is a total of 75  90 or 165 members
in 15 rows and 165  15  11, there will be
11 members in each row.

402

65. The next prime number after 2 is 3, so let p  3.


2p  1  23  1
81
7
Since 7 is prime, the second Mersenne prime is 7.
The next prime number after 3 is 5, so let p  5.
2p  1  25  1
 32  1
 31
Since 31 is prime, the third Mersenne prime is 31.
66. No; the first Mersenne prime is 3, so the formula
does not generate the first prime number, 2.
67. The area of the triangle is 20 sq cm.

70. D; List all pairs of numbers whose product is 120.


2  60
3  40
1  120
5  24
6  20
4  30
10  12
8  15
factors: 1, 2, 3, 4, 5, 6, 8, 10, 12, 15, 20, 24, 30, 40,
60, 120
Thus, it is true that 120 has at least eight factors.
Any number that has at least eight factors will
have at least four factorizations into two numbers.
71. A; Find the prime factorization of each number.
53  53
74  2  37
99  3  3  11
117  3  3  13
Thus,   53,   37,   11,
53
74
99

A  2bh
1

20  2bh
2(20)  2

1
bh
2

and 
 13. 53 has the greatest value.
117

40  bh
List all pairs of whole numbers whose product is
40. These are the possible dimensions for the
triangle.
2  20
1  40
58
4  10
Since either number in each pair can be the base
and the other number the height, the triangle can
have any of the following whole-number
dimensions.
base: 1 cm, height: 40 cm
base: 2 cm, height: 20 cm
base: 4 cm, height: 10 cm
base: 5 cm, height: 8 cm
base: 8 cm, height: 5 cm
base: 10 cm, height: 4 cm
base: 20 cm, height: 2 cm
base: 40 cm, height: 1 cm
68a. False; 6 is a factor of 3  4 or 12, but 6 is not a
factor of 3 or 4.
68b. True; since 6 is a factor of ab, the prime
factorization of ab must contain a factor of 3
since 3 is a factor of 6. Thus, 3 must be a factor
of either a or b.
68c. False; 6 is a factor of 3  1082 or 3246, but 3 is
not a factor of 1082.
69. Scientists listening to radio signals would suspect
that a modulated signal beginning with prime
numbers would indicate a message from an
extraterrestrial. Answer should include the
following.
2, 3, 5, 7, 11, 13, 17, 19, 23, 29, 31, 37, 41, 43,
47, 53, 59, 61, 67, 71, 79, 83, 89, 97, 101, 103,
107, 109, 113; See students work.
Sample answer: It is unlikely that any natural
phenomenon would produce such an artificial
and specifically mathematical pattern.

Page 479

Maintain Your Skills

72. (a  b) 2  a2  2ab  b2
(2x  1) 2  (2x) 2  2(2x)(1)  12
 4x2  4x  1
73.
(a  b)(a  b)  a2  b2
(3a  5)(3a  5)  (3a) 2  52
 9a2  25
74. (7p2  4)(7p2  4)  (7p2  4) 2
(a  b) 2  a2  2ab  b2
(7p2  4) 2  (7p2 ) 2  2(7p2 )(4)  42
 49p4  56p2  16
75. (6r  7)(2r  5)
 (6r)(2r)  (6r)(5)  (7)(2r)  7(5)
 12r2  30r  14r  35
 12r2  16r  35
76. (10h  k)(2h  5k)
 (10h)(2h)  (10h)(5k)  (k)(2h)  (k) (5k)
 20h2  50hk  2hk  5k2
 20h2  52hk  5k2
77. (b  4)(b2  3b  18)
 b(b2  3b  18)  4(b2  3b  18)
 b3  3b2  18b  4b2  12b  72
 b3  7b2  6b  72
78. Use the slope formula with (x1, y1)  (1, 2),
(x2, y2)  (2, r) and m  3.
y2  y 1

mx
3
3
1

 x1

r  2
2  1
r  2
3

3(3)  1(r  2)
9  r  2
9  2  r  2  2
7  r

403

Chapter 9

2. Model 6x  8.

79. Use the slope formula with (x1, y1)  (5, 9),
3
(x2, y2)  (r, 6) and m  5.
y2  y1

mx

3
5
3
5




 x1

1 1 1 1

6  9
r  (5)
3
r  5

7900
125

84.
85.
86.

x
x

x
x

1 1 1 1
1 1 1 1

x
x

25

 100

The rectangle has a width of 2 and a length of


3x  4. So, 6x  8  2(3x  4).
3. Model 5x2  2x.

125x
125

63.2  x
The wholesale price was $63.20.
5(2x  8)  5(2x)  5(8)
 10x  40
a(3a  1)  a(3a)  a(1)
 3a2  a
2g(3g  4)  2g(3g)  (2g)(4)
 6g2  8g
4y(3y  6)  (4y) (3y)  (4y)(6)
 12y2  24y
7b  7c  7(b  c)
2x  3x  (2  3)x

Page 480

3x  4

(79  x) (100)  (x)(25)


7900  100x  25x
7900  100x  100x  25x  100x
7900  125x

83.

 3

79  x
x

82.

Arrange the tiles into a rectangle.

r0
80. Let x  the wholesale price. Then 79  x is the
amount of change. The percent of increase is 25%.

81.

1 1 1 1

3(r  5)  5(3)
3r  15  15
3r  15  15  15  15
3r  0
3r
3

x2

x2

x2

x2

x2

Arrange the tiles into a rectangle.


5x  2

x2

x2

x2

x2

x2

x x

The rectangle has a width of x and a length of


5x  2. So, 5x2  2x  x(5x  2).
4. Model 9  3x.

Algebra Activity
(Preview of Lesson 9-2)

1. Model 2x  10.

x

x

x

Arrange the tiles into a rectangle.

Arrange the tiles into a rectangle.

x5
2

x
x

1
1

1
1

3x
1
1

1
1

1
1

The rectangle has a width of 2 and a length of


x  5. So, 2x  10  2(x  5).

Chapter 9

x
x
x

1
1
1

1
1
1

1
1
1

The rectangle has a width of 3 and a length of


3  x. So, 9  3x  3(3  x).

404

5. Yes;

4. The GCF of 9x2 and 36x is 9x.


9x2  36x  9x(x)  9x(4)
 9x(x  4)
5. The GCF of 16xz and 40xz2 is 8xz.
16xz  40xz2  8xz(2)  8xz(5z)
 8xz(2  5z)
6. The GCF of 24m2np2 and 36m2n2p is 12m2np.
24m2np2  36m2n2p  12m2np(2p)  12m2np(3n)
 12m2np(2p  3n)
7. The GCF of 2a3b2, 8ab, and 16a2b3 is 2ab.

2x  5
2

x
x

x
x

1 1 1 1 1
1 1 1 1 1

6. No;

2a3b2  8ab  16a2b3  2ab(a2b)  2ab(4)  2ab(8ab2 )


 2ab(a2b  4  8ab2 )

1 1 1
1 1
1 1

x
x
x

8. 5y2  15y  4y  12  (5y2  15y)  (4y  12)


 5y(y  3)  4(y  3)
 (y  3)(5y  4)
9. 5c  10c2  2d  4cd  (5c  10c2 )  (2d  4cd)
 5c(1  2c)  2d(1  2c)
 (1  2c)(5c  2d)
10. h(h  5)  0
h  0 or h  5  0
h  5
The solution set is {0, 5}.
Check:
h(h  5)  0
h(h  5)  0
?
?
5(5  5)  0
0(0  5)  0
?
?
5(0)  0
0(5)  0
00
00
11. (n  4)(n  2)  0
n  4  0 or n  2  0
n4
n  2
The solution set is {4, 2}.
Check:
(n  4)(n  2)  0
(n  4)(n  2)  0
?
?
(2  4)(2  2)  0
(4  4)(4  2)  0
?
?
6(0)  0
0(6)  0
00
00
12.
5m  3m2
5m  3m2  0
m(5  3m)  0
m  0 or 5  3m  0
3m  5

7. Yes;

x2
x

x x

8. No;

9. Sample answer: Binomials can be factored if they


can be represented by a rectangle. Examples:
2x  2 can be factored and 2x  1 cannot be
factored.

9-2
Page 484

Factoring Using the Distributive


Property

5 56

Check for Understanding

m3

The solution set is 0, 3 .

1. Sample answers:
4x2  12x  4(x2 )  4(3x)
 4(x2  3x)
2  12x  x(4x)  x(12)
4x
 x(4x  12)
4x2  12x  4x(x)  4x(3)
 4x(x  3)
4x(x  3) is the completely factored form because
4x is the GCF of 4x2 and 12x.
2. Sample answer: The equation x2  6x  7  0
can be solved using the Zero Product Property
because x2  6x  7 can be factored to
(x  1)(x  7).
3. The equation has two solutions, 4 and 2.
Dividing each side by x  2 would eliminate 2 as
a solution.

Check:

5m  3m2
?

5(0)  3(0) 2
00

5m  3m2
5

153 2  3153 22
25 ?
25
 31 9 2
3
?

25
3

25
3

13. Since the height is expressed in units of feet


above sea level, the flare is at height 0 ft when it
returns to the sea.
14. h  100t  16t2
0  100t  16t2
0  4t(25  4t)
4t  0 or 25  4t  0
t0
4t  25
25
t 4
t  6.25

405

Chapter 9

15. The flare returns to the sea in 6.25 s. The answer


0 is not reasonable since it represents the time at
which the flare is launched.

37. 2my  7x  7m  2xy  2my  7m  7x  2xy

Pages 484486

38. 8ax  6x  12a  9  (8ax  6x)  (12a  9)


 2x(4a  3)  3(4a  3)
 (4a  3)(2x  3)






Practice and Apply

16. 5x  30y  5(x)  5(6y)


 5(x  6y)
17. 16a  4b  4(4a)  4(b)
 4(4a  b)
5
18. a b  a  a(a4b)  a(1)
 a(a4b  1)
3 2
19. x y  x  x(x2y2 )  x(1)
 x(x2y2  1)
20. 21cd  3d  3d(7c)  3d(1)
 3d(7c  1)
21. 14gh  18h  2h(7g)  2h(9)
 2h(7g  9)
22. 15a2y  30ay  15ay(a)  15ay(2)
 15ay(a  2)
23. 8bc2  24bc  8bc(c)  8bc(3)
 8bc(c  3)
24. 12x2y2z  40xy3z2  4xy2z(3x)  4xy2z(10yz)
 4xy2z(3x  10yz)
25. 18a2bc2  48abc3  6abc2 (3a)  6abc2 (8c)
 6abc2 (3a  8c)
2b2  a3b3  a(1)  a(ab2 )  a(a2b3 )
26. a  a
 a(1  ab2  a2b3 )
2y2  25xy  x  x(15xy2 )  x(25y)  x(1)
27. 15x
 x(15xy2  25y  1)
28. 12ax3  20bx2  32cx  4x(3ax2 )  4x(5bx)  4x(8c)

39. 10x2  14xy  15x  21y


 (10x2  14xy)  (15x  21y)
 2x(5x  7y)  3(5x  7y)
 (5x  7y)(2x  3)
1

29. 3p

9pq2

 2n(n  3)
41. Replace n with 10 in the polynomial.
1
n(n
2

 3)  2 (10)(5  3)

 5(7) or 35
A decagon has 35 diagonals.
1

42. g  2n2  2n
1

 2n(n)  2n(1)
1

 2n(n  1)
43. First find the number of games needed for
7 teams to play each other once.
1

g  2 (7)(7  1)
1

 2 (7)(6)
1

 2 (42)
 21
Thus, the number of games needed for 7 teams to
play each other 3 times is 3(21) or 63 games.
44. area of shaded  area of outer  area of inner
region
rectangle
rectangle
 (a  4)(b  4)  ab
 ab  a(4)  4b  16  ab
 4a  4b  16
 4(a)  4(b)  4(4)
 4(a  b  4)
45. area of shaded  area of the  2  area of a
region
rectangle
circle
 2r(4r)  2(r2 )
 8r2  2r2
 2r2 (4)  2r2 ()
 2r2 (4  )
46. The length of one side of the square is
1
(12x  20y) or 3x  5y in.
4

 36pq  3pq(p2 )  3pq(3q)  3pq(12)


 3pq(p2  3q  12)

30. x2  2x  3x  6  (x2  2x)  (3x  6)


 x(x  2)  3(x  2)
 (x  2)(x  3)
31. x2  5x  7x  35  (x2  5x)  (7x  35)
 x(x  5)  7(x  5)
 (x  5)(x  7)
32. 4x2  14x  6x  21  (4x2  14x)  (6x  21)
 2x(2x  7)  3(2x  7)
 (2x  7) (2x  3)
33. 12y2  9y  8y  6  (12y2  9y)  (8y  6)
 3y(4y  3)  2(4y  3)
 (4y  3)(3y  2)
34. 6a2  15a  8a  20  (6a2  15a)  (8a  20)
 3a(2a  5)  4(2a  5)
 (2a  5) (3a  4)
2  30x  3x  5  (18x2  30x)  (3x  5)
35. 18x
 6x(3x  5)  (3x  5)
 (3x  5)(6x  1)
36. 4ax  3ay  4bx  3by  (4ax  3ay)  (4bx  3by)

A  (3x  5y) 2
 (3x) 2  2(3x)(5y)  (5y) 2
 9x2  30xy  25y2
The area of the square is 9x2  30xy  25y2 in2.

 a(4x  3y)  b(4x  3y)


 (4x  3y)(a  b)

Chapter 9

40. 2n2  2n  2n(n)  2n(3)

 4x(3ax2  5bx  8c)


3q

(2my  7m)  (7x  2xy)


m(2y  7)  x(7  2y)
m(2y  7)  x(2y  7)
(2y  7) (m  x)

406

47. The length of one side of the square is


1
(36a  16b) or 9a  4b cm.
4

57.

A  (9a  4b) 2
 (9a) 2  2(9a)(4b)  (4b) 2
 81a2  72ab  16b2
The area of the square is 81a2  72ab  16b2 cm2.
Exercises 4859 For checks, see students work.
48. x(x  24)  0
x  0 or x  24  0
x  24
{0, 24}
49. a(a  16)  0
a  0 or a  16  0
a  16
{16, 0}
50. (q  4)(3q  15)  0
or 3q  15  0
q40
q  4
3q  15
q5
{4, 5}
51. (3y  9)( y  7)  0
or y  7  0
3y  9  0
3y  9
y7
y  3
{3, 7}
52. (2b  3) (3b  8)  0
2b  3  0 or 3b  8  0
2b  3
3b  8

58.

59.

532, 83 6

x7

50, 67 6

6x2  4x
6x  4x  0
2x(3x  2)  0
2x  0
x0

or

3x  2  0
3x  2
2

x  3

523, 06

20x2  15x
20x  15x  0
5x(4x  3)  0
5x  0
x0

or

4x  3  0
4x  3
3

x  4

60. Let h  0 and solve for t.


h  20t  16t2
0  20t  16t2
0  4t(5  4t)
4t  0 or 5  4t  0
t0
4t  5

t4

or

t  1.25
The dolphin leaves the water at 0 seconds and
returns to the water at 1.25 seconds. The dolphin
is in the air for 1.25 s.
61. Let h  2 and solve for t.
h  2  45t  16t2
2  2  45t  16t2
0  45t  16t2
0  16t(2.8125  t)
16t  0 or 2.8125  t  0
t  2.8125
t0
t  2.8125
Malik hits the ball at 0 seconds, and the catcher
catches it at about 2.8 seconds. The ball is in the
air about 2.8 s before it is caught.

3n  7  0
3n  7

n  4

n3

54. 3z2  12z  0


3z(z  4)  0
3z  0 or z  4  0
z0
z  4
{4, 0}
55. 7d2  35d  0
7d(d  5)  0
7d  0 or d  5  0
d0
d5
{0, 5}
56.
2x2  5x
2  5x  0
2x
x(2x  5)  0
or 2x  5  0
x0
2x  5

62. axy  axby  aybx  bxy


 (axy  axby )  (aybx  bxy )
 (axay  axby )  (aybx  bxby )
 ax (ay  by )  bx (ay  by )
 (ay  by )(ax  bx )

50, 52 6

7x  6  0
7x  6

534, 06

b3

53. (4n  5)(3n  7)  0


4n  5  0
4n  5

554, 73 6

or

b2

7x2  6x
7x  6x  0
x(7x  6)  0
x0
2

x2

407

Chapter 9

63. Answers should include the following.


Let h  0 in the equation h  151t  16t2. To
solve 0  151t  16t2, factor the right-hand side
as t(151  16t). Then, since t(151  16t)  0,
either t  0 or 151  16t  0. Solving each
equation for t, we find that t  0 or t  9.44.
The solution t  0 represents the point at
which the ball was initially thrown into the
air. The solution t  9.44 represents how long
it took after the ball was thrown for it to
return to the same height at which it was
thrown.
64. A; Since there are 3 feet in one yard, the number
of feet in x yards is 3x. The number of feet in
y feet is y. Since there are 12 inches per foot, the
z
number of feet in z inches is 12.
65. A; First determine the negative solution of the
equation in Column A.
(a  2) (a  5)  0
a  2  0 or a  5  0
a2
a  5
The solution set is {5, 2}. The negative solution
is 5.
Next determine the negative solution of the
equation in Column B.
(b  6) (b  1)  0
b60
or b  1  0
b  6
b1
The solution set is {6, 1}. The negative solution
is 6. 5  6, so the quantity in Column A is
greater.

Page 486

73.

s4
s7




74.

11812 21xx 21yy 2


3

1

3 32
(x
)(y14 )
2
3
(x)(y5 )
2
3
1
(x) y5
2
3x
2y5

34p7q2r5
17(p3qr1 ) 2

1 2

34p7q2r5

 17(p3 ) 2 (q) 2 (r1 ) 2


34p7q2r5

 17p6q2r2


13417 21pp 21qq 21rr 2


7

5
2

)(q
)(r5(2) )
 2(p
0
3
 2(p)(q )(r )
 2(p)(1)(r3 )
76

22

2p
r3

75. Let x represent the number of shares that


Michael can purchase. The total purchase price
cannot exceed 60% of last years dividend.
14x  0.60(885)
14x  531
x

531
14

or about 37.9

Michael can purchase up to 37 shares.


76. (n  8)(n  3)  (n)(n)  (n)(3)  (8)(n)  8(3)
 n2  3n  8n  24
 n2  11n  24
77. (x  4)(x  5)
 (x)(x)  (x)(5)  (4)(x)  (4)(5)
 x2  5x  4x  20
 x2  9x  20
78. (b  10) (b  7)
 (b)(b)  (b)(7)  (10)(b)  (10)(7)
 b2  7b  10b  70
 b2  3b  70
79. (3a  1)(6a  4)
 (3a)(6a)  (3a)(4)  (1)(6a)  (1)(4)

Maintain Your Skills

 18a2  12a  6a  4
 18a2  6a  4
80. (5p  2)(9p  3)
 (5p)(9p)  (5p)(3)  (2) (9p)  (2)(3)
 45p2  15p  18p  6
 45p2  33p  6
81. (2y  5)(4y  3)
 (2y)(4y)  (2y)(3)  (5)(4y)  (5)(3)
 8y2  6y  20y  15
 8y2  14y  15

Page 486

Practice Quiz 1

1. List all pairs of numbers whose product is 225.


1  225
3  75
5  45
9  25
15  15
factors: 1, 3, 5, 9, 15, 25, 45, 75, 225
composite

 s4(7)
 s11

Chapter 9




66. List all pairs of numbers whose product is 123.


1  123
3  41
factors: 1, 3, 41, 123
composite
67. List all pairs of numbers whose product is 300.
1  300
2  150
3  100
4  75
5  60
6  50
10  30
12  25
15  20
factors: 1, 2, 3, 4, 5, 6, 10, 12, 15, 20, 25, 30, 50,
60, 75, 100, 150, 300
composite
68. List all pairs of numbers whose product is 67.
1  67
factors: 1, 67
prime
69. (4s3  3) 2  (4s3 ) 2  2(4s3 )(3)  (3) 2
 16s6  24s3  9
70. (2p  5q) (2p  5q)  (2p) 2  (5q) 2
 4p2  25q2
71. (3k  8) (3k  8)  (3k  8) 2
 (3k) 2  2(3k)(8)  82
 9k2  48k  64
72.

18x3y1
12x2y4

408

2. 320  1  320
 1  2  160
 1  2  2  80
 1  2  2  2  40
 1  2  2  2  2  20
 1  2  2  2  2  2  10
 1  2  2  2  2  2  2  5 or 1  26  5
2 3
3. 78a bc  2  39  a  a  b  c  c  c
 2  3  13  a  a  b  c  c  c
4. 54x3  2  3  3  3  x  x  x
42x2y  2  3  7  x  x  y
30xy2  2  3  5  x  y  y
GCF: 2  3  x or 6x
5. 4xy2  xy  xy(4x)  xy(1)
 xy(4x  1)
2
6. 32a b  40b3  8a2b2  8b(4a2 )  8b(5b2 )  8b(a2b)

Page 488

Algebra Activity
(Preview of Lesson 9-3)

1. Model x2  4x  3.

x2

Place the x2 tile at the corner. Arrange the 1 tiles


into a rectangular array.

 8b(4a2  5b2  a2b)

7. 6py  16p  15y  40  (6py  16p)  (15y  40)

x2

 2p(3y  8)  5(3y  8)
 (3y  8) (2p  5)

8. (8n  5)(n  4)  0
8n  5  0
8n  5

or

n40
n4

n  8

558, 46

Complete the rectangle with the x tiles.

x3

Check:

(8n  5) (n  4)  0

3 8158 2  5 4158  42  0
?
5
(5  5) 1 8  4 2  0
37 ?
(0) 1  8 2  0
?

(8n  5) (n  4)  0
?

[ 8(4)  5] (4  4)  0

x1

(37)(0)  0
00

x2

x x x

00

9. 9x  27x  0
9x(x  3)  0
9x  0 or
x0
{0, 3}
Check:
9x(x  3)  0

9x(x  3)  0

9(0) (0  3)  0

9(3)(3  3)  0

0(3)  0
00
10.
10x2  3x
10x2  3x  0
x(10x  3)  0
x0

The rectangle has a width of x  1 and a length of


x  3. Therefore, x2  4x  3  (x  1)(x  3).
2. Model x2  5x  4.

x30
x3

x2

27(0)  0
00

or

10x  3  0
10x  3
3

Check:

10x2  3x

10x2  3x

10(0) 2  3(0)

1103 22  31103 2
? 9
9
10 1 100 2  10

10

9
10

9
10

Place the x2 tile at the corner. Arrange the 1 tiles


into a rectangular array.

x  10

5103 , 06

x2

10(0)  0
00

409

Chapter 9

Complete the rectangle with the x tiles.

4. Model x2  3x  2.

x4

x x x x

x2

x1

x2
x

x x x
1

The rectangle has a width of x  1 and a length of


x  4. Therefore, x2  5x  4  (x  1)(x  4).
3. Model x2  x  6.

Place the x2 tile at the corner. Arrange the 1 tiles


into a rectangular array.

x2
x

x2

1 1 1
1

1 1 1

Complete the rectangle with the x tiles.

x2

Place the x2 tile at the corner. Arrange the 1 tiles


into a rectangular array.

x1
x2

x2

x x

x

1 1

1 1 1
1 1 1
The rectangle has a width of x  1 and a length of
x  2. Therefore, x2  3x  2  (x  1)(x  2).
5. Model x2  7x  12.

Complete the rectangle with the x tile and two


zero-pairs of x tiles

x2

x3

x2

x x x

x
x

1 1 1
1 1 1

x2
Place the x2 tile at the corner. Arrange the 1 tiles
into a rectangular array.

The rectangle has a width of x  2 and a length of


x  3. Therefore, x2  x  6  (x  2)(x  3).

Chapter 9

x2

410

1
1

1
1

1
1

1
1

Complete the rectangle with the x tiles.

7. Model x2  x  2.

x4

x x x x

x2

x

x2

x3

x
x
x

1
1

1 1 1 1
1 1 1 1
1 1 1 1

Place the x2 tile at the corner. Arrange the 1 tiles


into a rectangular array.

The rectangle has a width of x  3 and a length of


x  4. Therefore, x2  7x  12  (x  3)(x  4).
6. Model x2  4x  4.

x2

1
1

x x x x

x2

Complete the rectangle with the x tile and a zero


pair of x tiles.

x1
x2

Place the
tile at the corner. Arrange the 1 tiles
into a rectangular array.

x2

x
x

1
1

x2

x2
1
1

1
1

The rectangle has a width of x  2 and a length of


x  1. Therefore, x2  x  2  (x  2)(x  1).

Complete the rectangle with the x tiles.

x2

x2

x x

x
x

1 1
1 1

x2

The rectangle has a width of x  2 and a length of


x  2. Therefore, x2  4x  4  (x  2)(x  2).

411

Chapter 9

8. Model x2  6x  8.

x2

x x x x x x

3.
4.

Place the x2 tile at the corner. Arrange the 1 tiles


into a rectangular array.

x2
5.
1 1 1 1
1 1 1 1

Complete the rectangle with the x tiles.


6.

x4

x2

x x x x

x
x

1 1 1 1
1 1 1 1

x2

The rectangle has a width of x  2 and a length of


x  4. Therefore, x2  6x  8  (x  2)(x  4).

9-3

Factoring Trinomials x 2  bx  c

Pages 492493

7.

Check for Understanding

1. In this trinomial, b  6 and c  9. This means


that m  n is positive and mn is positive. Only
two positive numbers have both a positive sum
and product. Therefore, negative factors of 9 need
not be considered.
2. Sample answer: Factor x2  14x  40  0.
Since b  14 and c  40, make a list of negative
factors of 40, and look for the pair whose sum is 14.
Factors of 40
Sum of Factors
1, 40
41
2, 20
22
4, 10
14
5, 8
13

Chapter 9

412

The correct factors are 4 and 10.


x2  14x  40  0
(x  4)(x  10)  0
x  4  0 or x  10  0
x4
x  10
The solution set is {4, 10}.
Aleta; to use the Zero Product Property, one side
of the equation must equal zero.
Since b  11 and c  24, make a list of positive
factors of 24, and find the pair whose sum is 11.
Factors of 24
Sum of Factors
1, 24
25
2, 12
14
3, 8
11
4, 6
10
The correct factors are 3 and 8.
x2  11x  24  (x  m)(x  n)
 (x  3)(x  8)
Since b  3 and c  2, make a list of negative
factors of 2, and find the pair whose sum is 3.
Factors of 2
Sum of Factors
1, 2
3
The correct factors are 1 and 2.
c2  3c  2  (c  m)(c  n)
 (c  1)(c  2)
Make a list of factors of 48, and find the pair
whose sum is 13.
Factors of 48
Sum of Factors
1, 48
47
1, 48
47
2, 24
22
2, 24
22
3, 16
13
3, 16
13
4, 12
8
4, 12
8
6, 8
2
6,
8
2
The correct factors are 3 and 16.
n2  13n  48  (n  m)(n  p)
 (n  3)(n  16)
Make a list of factors of 35, and find the pair
whose sum is 2.
Factors of 35
Sum of Factors
1, 35
34
1, 35
34
5, 7
2
5,
7
2
The correct factors are 5 and 7.
p2  2p  35  ( p  m)( p  n)
 ( p  5)( p  7)

8. 72  27a  a2  a2  27a  72
Make a list of positive factors of 72, and find the
pair whose sum is 27.
Factors of 72
Sum of Factors
1, 72
73
2, 36
38
3, 24
27
4, 18
22
6, 12
18
8, 9
17
The correct factors are 3 and 24.
72  27a  a2  a2  27a  72
 (a  m)(a  n)
 (a  3)(a  24)
9. x2  4xy  3y2  x2  (4y)x  3y2
Make a list of negative factors of 3y2, and find the
pair whose sum is 4y.
Factors of 3y2
Sum of Factors
y, 3y
4y
y2, 3
y2  3
1, 3y2
1  3y2
The correct factors are y and 3y.
x2  4xy  3y2  x2  (4y)x  3y2
 (x  m)(x  n)
 (x  y)(x  3y)
10.
n2  7n  6  0
(n  1)(n  6)  0
or n  6  0
n10
n  1
n  6
The solution set is {6, 1}.
Check:
n2  7n  6  0
?
2  7(1)  6 
0
(1)
?
1760
00
n2  7n  6  0
?
(6) 2  7(6)  6  0

The solution set is {2, 21}.


Check:
p2  19p  42  0
2

(2)  19(2)  42  0
?

4  38  42  0
00

p2  19p  42  0
2

(21)  19(21)  42  0
?

441  399  42  0
00

y2  9  10y
y  9  10y  0
y2  10y  9  0
( y  1)( y  9)  0
or y  9  0
y10
y  1
y  9
The solution set is {9, 1}.
Check:
y2  9  10y
y2  9  10y
?
?
2
(9)  9  10(9)
(1) 2  9  10(1)
?
?
81  9  90
1  9  10
90  90
10  10
14.
9x  x2  22
9x  x2  22  0
x2  9x  22  0
(x  2)(x  11)  0
x  2  0 or x  11  0
x2
x  11
The solution set is {11, 2}.
Check:
9x  x2  22
9x  x2  22
?
?
2
9(11)  (11)  22
9(2)  (2) 2  22
?
?
99  121  22
18  4  22
22  22
22  22
15.
d2  3d  70
d2  3d  70  0
(d  7)(d  10)  0
or d  10  0
d70
d  7
d  10
The solution set is {7, 10}.
Check:
d2  3d  70
d2  3d  70
?
?
2
2
(7)  3(7)  70
(10)  3(10)  70
?
?
49  21  70
100  30  70
70  70
70  70
16. Let n be the first integer. Then n  1 is the
second integer.
n(n  1)  156
n2  n  156
2  n  156  0
n
(n  12)(n  13)  0
n  12  0
or n  13  0
n  12
n  13
When n  12, n  1  13.
When n  13, n  1  12.
The two consecutive integers are either 12 and 13
or 13 and 12.

13.

36  42  6  0
00
11. a2  5a  36  0
(a  4)(a  9)  0
a  4  0 or a  9  0
a4
a  9
The solution set is {9, 4}.
Check:
a2  5a  36  0
a2  5a  36  0
?
?
2
2
(9)  5(9)  36  0 (4)  5(4)  36  0
?
?
81  45  36  0
16  20  36  0
00
00
12. p2  19p  42  0
( p  2)( p  21)  0
or p  21  0
p20
p  2
p  21

413

Chapter 9

Pages 493494

31. 72  6w  w2  w2  6w  72
Among all pairs of factors of 72, choose 6 and
12, the pair of factors whose sum is 6.
72  6w  w2  w2  6w  72
 (w  m)(w  n)
 (w  6)(w  12)
32. 30  13x  x2  x2  13x  30
Among all pairs of factors of 3, choose 2 and
15, the pair of factors whose sum is 13.
30  13x  x2  x2  13x  30
 (x  2)(x  15)
33. a2  5ab  4b2  a2  (5b)a  4b2
Among all pairs of positive factors of 4b2, choose
b and 4b, the pair of factors whose sum is 5b.
a2  5ab  4b2  a2  (5b)a  4b2
 (a  m)(a  n)
 (a  b)(a  4b)
34. x2  13xy  36y2  x2  (13y)x  36y2
Among all pairs of negative factors of 36y2, choose
4y and 9y, the pair of factors whose sum is
13y.
x2  13xy  36y2  x2  (13y)x  36y2
 (x  m)(x  n)
 (x  4y) (x  9y)
35. Factor the expression given for the area.
area  x2  24x  81
 (x  27)(x  3)
Assuming that the factors represent the length
and width, find the perimeter of the rectangle.
perimeter  2(x  27)  2(x  3)
 2x  54  2x  6
 4x  48
36. Factor the expression given for the area.
area  x2  13x  90
 (x  18)(x  5)
Assuming that the factors represent the length
and width, find the perimeter of the rectangle.
perimeter  2(x  18)  2(x  5)
 2x  36  2x  10
 4x  26
Exercises 3753 For checks, see students work.
37. x2  16x  28  0
(x  2)(x  14)  0
x20
or x  14  0
x  2
x  14
{14, 2}
38. b2  20b  36  0
(b  2)(b  18)  0
b20
or b  18  0
b  2
b  18
{18, 2}
39. y2  4y  12  0
( y  2)( y  6)  0
y  2  0 or y  6  0
y2
y  6
{6, 2}

Practice and Apply

17. Among all pairs of positive factors of 15, choose


3 and 5, the pair of factors whose sum is 8.
a2  8a  15  (a  m)(a  n)
 (a  3)(a  5)
18. Among all pairs of positive factors of 27, choose
3 and 9, the pair of factors whose sum is 12.
x2  12x  27  (x  m)(x  n)
 (x  3)(x  9)
19. Among all pairs of positive factors of 35, choose
5 and 7, the pair of factors whose sum is 12.
c2  12c  35  (c  m)(c  n)
 (c  5) (c  7)
20. Among all pairs of positive factors of 30, choose
3 and 10, the pair of factors whose sum is 13.
y2  13y  30  ( y  m)( y  n)
 ( y  3)( y  10)
21. Among all pairs of negative factors of 21, choose
1 and 21, the pair of factors whose sum is 22.
m2  22m  21  (m  p)(m  n)
 (m  1)(m  21)
22. Among all pairs of negative factors of 10, choose
2 and 5, the pair of factors whose sum is 7.
d2  7d  10  (d  m)(d  n)
 (d  2)(d  5)
23. Among all pairs of negative factors of 72, choose
8 and 9, the pair of factors whose sum is 17.
p2  17p  72  ( p  m)( p  n)
 ( p  8)( p  9)
24. Among all pairs of negative factors of 60, choose
4 and 15, the pair of factors whose sum is 19.
g2  19g  60  ( g  m)( g  n)
 ( g  4)( g  15)
25. Among all pairs of factors of 7, choose 1 and 7,
the pair of factors whose sum is 6.
x2  6x  7  (x  m)(x  n)
 (x  1)(x  7)
26. Among all pairs of factors of 20, choose 4 and
5, the pair of factors whose sum is 1.
b2  b  20  (b  m)(b  n)
 (b  4)(b  5)
27. Among all pairs of factors of 40, choose 5 and
8, the pair of factors whose sum is 3.
h2  3h  40  (h  m)(h  n)
 (h  5)(h  8)
28. Among all pairs of factors of 54, choose 6 and
9, the pair of factors whose sum is 3.
n2  3n  54  (n  m)(n  p)
 (n  6)(n  9)
29. Among all pairs of factors of 42, choose 6 and
7, the pair of factors whose sum is 1.
y2  y  42  ( y  m)( y  n)
 ( y  6)( y  7)
30. Among all pairs of factors of 40, choose 2 and
20, the pair of factors whose sum is 18.
z2  18z  40  (z  m)(z  n)
 (z  2)(z  20)

Chapter 9

414

40.

41.

42.

43.

44.

45.

46.

47.

48.

49.

d2  2d  8  0
(d  2)(d  4)  0
d  2  0 or d  4  0
d2
d  4
{4, 2}
a2  3a  28  0
(a  7)(a  4)  0
a  7  0 or a  4  0
a7
a  4
{4, 7}
g2  4g  45  0
( g  9)( g  5)  0
g  9  0 or g  5  0
g9
g  5
{5, 9}
m2  19m  48  0
(m  3)(m  16)  0
m  3  0 or m  16  0
m3
m  16
{3, 16}
n2  22n  72  0
(n  4)(n  18)  0
n  4  0 or n  18  0
n4
n  18
{4, 18}
z2  18  7z
2
z  7z  18  0
(z  2)(z  9)  0
z  2  0 or z  9  0
z2
z  9
{9, 2}
h2  15  16h
2
h  16h  15  0
(h  15) (h  1)  0
h  15  0
or h  1  0
h  15
h  1
{15,  1}
24  k2  10k
2
k  10k  24  0
(k  4)(k  6)  0
k  4  0 or k  6  0
k4
k6
{4, 6}
x2  20  x
2
x  x  20  0
(x  4)(x  5)  0
x40
or x  5  0
x  4
x5
{4, 5}
c2  50  23c
2  23c  50  0
c
(c  2)(c  25)  0
c  2  0 or c  25  0
c2
c  25
{25, 2}

50.

y2  29y  54
 29y  54  0
( y  2)( y  27)  0
y  2  0 or y  27  0
y2
y  27
{2, 27}
14p  p2  51
2
p  14p  51  0
( p  3)( p  17)  0
p  3  0 or p  17  0
p3
p  17
{17, 3}
x2  2x  6  74
2
x  2x  6  74  0
x2  2x  80  0
(x  8)(x  10)  0
or x  10  0
x80
x  8
x  10
{8, 10}
x2  x  56  17x
2  x  56  17x  0
x
x2  18x  56  0
(x  4)(x  14)  0
x  4  0 or x  14  0
x4
x  14
{4, 14}
To find the number of Justices on the Supreme
Court, replace h with 36 in the equation and solve
for n.
y2

51.

52.

53.

54.

h
36 

n2  n
2
n2  n
2

72  n2  n
0  n2  n  72
0  (n  9)(n  8)
n  9  0 or n  8  0
n9
n  8
Only 9 is a valid solution since the number of
Justices cannot be negative. Thus there are
9 Justices on the Supreme Court.
55. Let n be the first integer. Let n  2 be the second
integer.
n(n  2)  168
n2  2n  168
2  2n  168  0
n
(n  12)(n  14)  0
n  12  0
or n  14  0
n  12
n  14
When n  12, n  2  14.
When n  14, n  2  12.
The two consecutive even integers are either
12 and 14 or 14 and 12.

415

Chapter 9

62. Replace the area with 8160 and solve.


area  w(w  52)
8160  w(w  52)
8160  w2  52w
0  w2  52w  8160
0  (w  68)(w  120)
or w  68  0
w  120  0
w  120
w  68
Only 68 is a valid solution since dimensions
cannot be negative.
Now find the length of the field.
length  w  52
 68  52
 120
The dimensions of the field are 120 m by 68 m.
63. Answers should include the following.
You would use a guess-and-check process,
listing the factors of 54, checking to see which
pairs added to 15.
To factor a trinomial of the form x2  ax  c,
you also use a guess-and-check process, list
the factors of c, and check to see which ones
add to a.
64. C; Of the pairs of factors of 42 shown in the
choices, only 3 and 14 have a sum of 17.
So, the answer must be C.
65. p2  13p  30  0
( p  2)( p  15)  0
or p  15  0
p20
p  2
p  15
The positive solution is 15.
66. No; the graphs of y1  x2  14x  48 and
y2  (x  6)(x  8) do not coincide. The correct
factorization is x2  14x  48  (x  6)(x  8).
67. Yes; the graphs of y1  x2  16x  105 and
y2  (x  5)(x  21) coincide.
68. No; the graphs of y1  x2  25x  66 and
y2  (x  33)(x  2) do not coincide. The correct
factorization is x2  25x  66  (x  22)(x  3).
69. No; the graphs of y1  x2  11x  210 and
y2  (x  10)(x  21) do not coincide. The correct
factorization is x2  11x  210  (x  10)(x  21).

56. Write an equation for the area of the triangle and


solve for h.
1

area  2 base  height


1

40  2 (2h  6)(h)

57.

58.

59.

60.

40  h2  3h
0  h2  3h  40
0  (h  5) (h  8)
h  5  0 or h  8  0
h5
h  8
Only 5 is a valid solution since the height of the
triangle cannot be negative. Thus, the height of
the triangle is 5 cm.
List all pairs of factors of 19 and the sums of
the pairs.
Factors of 19
Sum of Factors
1, 19
18
1, 19
18
k can be 18 or 18.
List all pairs of factors of 14 and the sums of the
pairs.
Factors of 14
Sum of Factors
1, 14
15
1, 14
15
2,
7
9
2, 7
9
k can be 15, 9, 9, or 15.
Because k is positive, m and n must have the
same sign. Because b  8, m and n must be
negative.
List all pairs of negative integers whose sum is
8, and list the products of the pairs.
Addends of 8 Product of Addends
1, 7
7
2, 6
12
3, 5
15
4, 4
16
k can be 7, 12, 15, or 16.
Because k is positive, m and n must have the
same sign. Because b  5, m and n must be
negative.
List all pairs of negative integers whose sum is
5, and list the products of the pairs.

Addends of 5 Product of Addends


1, 4
4
2, 3
6
k can be either 4 or 6.
61. area  length  width
 (w  52) (w)
 w(w  52)
The area of the rugby field is [w(w  52)] m2.

Page 494

Maintain Your Skills

Exercises 7072 For checks, see students work.


70. (x  3)(2x  5)  0
or 2x  5  0
x30
x  3
2x  5
5

x2

53, 52 6

71. b(7b  4)  0
b  0 or 7b  4  0
7b  4

50, 47 6
Chapter 9

416

b7

72.

5y2  9y
 9y  0
y(5y  9)  0
y  0 or 5y  9  0
5y  9

83. 4g2  2g  6g  3  (4g2  2g)  (6g  3)


 2g(2g  1)  3(2g  1)
 2g(2g  1)  3(1) (2g  1)
 2g(2g  1)  (3) (2g  1)
 (2g  1)(2g  3)

5y2

y  5

595, 06

9-4

73. Factor each monomial and circle the common


prime factors.
24  2  2  2  3
36  2  2  3  3
72  2  2  2  3  3
GCF: 2  2  3 or 12
74. Factor each monomial and circle the common
prime factors.
9p2q5  3  3  p  p  q  q  q  q  q
21p3q3  3  7  p  p  p  q  q  q
GCF: 3  p  p  q  q  q or 3p2q3
75. Factor each monomial and circle the common
prime factors.
30x4y5  2  3  5  x  x  x  x  y  y  y  y  y
20x2y7  2  2  5  x  x  y  y  y  y  y  y  y
75x3y4  3  5  5  x  x  x  y  y  y  y
GCF: 5  x  x  y  y  y  y or 5x2y4
76. Find the change.
28.2  1.54  26.66
Find the percent using the original number, 1.54,
as the base.
26.66
1.54

Pages 498499

 100

77.

78.

79.

80.

81.

Check for Understanding

1. m and n are the factors of ac whose sum is b.


2. Sample answer: 2x2  9x  7. ac  2(7)  14 and
b  9.
2x2  9x  7  2x2  2x  7x  7
 2x(x  1)  7(x  1)
 (x  1)(2x  7)
3. Craig; when factoring a trinomial of the form
ax2  bx  c, where a 1, you must find the
factors of ac, not of c.
4. 3a2  8a  4
Since ac  12 and b  8, make a list of positive
factors of 12 and find the pair whose sum is 8.
Factors of 12
Sum of Factors
1, 12
13
2, 6
8
3, 4
7
The correct factors are 2 and 6.
3a2  8a  4  3a2  ma  na  4
 3a2  2a  6a  4
 (3a2  2a)  (6a  4)
 a(3a  2)  2(3a  2)
 (3a  2)(a  2)
5. 2a2  11a  7
Since ac  14 and b  11, make a list of
negative factors of 14 and find the pair whose
sum is 11.
Factors of 14
Sum of Factors
1, 14
15
2, 7
9
There are no factors whose sum is 11.
So, 2a2  11a  7 is prime.

26.66(100)  1.54(r)
2666  1.54r
2666
1.54

Factoring Trinomials:
ax 2  bx  c

1.54r
1.54

1731  r
The percent increase is about 1731%.
1.54  17.31(1.54)  1(1.54)  17.31(1.54)
 (1  17.31) (1.54)
or 18.31(1.54)
3y2  2y  9y  6  (3y2  2y)  (9y  6)
 y(3y  2)  3(3y  2)
 (3y  2)(y  3)
3a2  2a  12a  8  (3a2  2a)  (12a  8)
 a(3a  2)  4(3a  2)
 (3a  2)(a  4)
4x2  3x  8x  6  (4x2  3x)  (8x  6)
 x(4x  3)  2(4x  3)
 (4x  3)(x  2)
2p2  6p  7p  21  (2p2  6p)  (7p  21)
 2p(p  3)  7(p  3)
 (p  3) (2p  7)

6. 2p2  14p  24
First factor out the GCF, 2.
2p2  14p  24  2( p2  7p  12)
Now factor p2  7p  12. Since the lead
coefficient is 1, find two factors of 12 whose sum
is 7. The correct factors are 3 and 4.
So, p2  7p  12  ( p  3)( p  4).
Thus, 2p2  14p  24  2( p  3)( p  4).

82. 3b2  7b  12b  28  (3b 2  7b)  (12b  28)







b(3b  7)  4(3b  7)
b(3b  7)  4(1) (3b  7)
b(3b  7)  (4) (3b  7)
(3b  7) (b  4)

417

Chapter 9

7. 2x2  13x  20
Since ac  40 and b  13, make a list of positive
factors of 40 find the pair whose sum is 13.
Factors of 40
Sum of Factors
1, 40
41
2, 20
22
4, 10
14
5, 8
13
The correct factors are 5 and 8.
2x2  13x  20  2x2  mx  nx  20
 2x2  5x  8x  20
 (2x2  5x)  (8x  20)
 x(2x  5)  4(2x  5)
 (2x  5)(x  4)
8. 6x2  15x  9
First factor out the GCF, 3.
6x2  15x  9  3(2x2  5x  3)
Now factor 2x2  5x  3. Since ac  6 and
b  5, make a list of the factors of 6 and find
the pair whose sum is 5.
Factors of 6
Sum of Factors
1, 6
5
1, 6
5
2, 3
1
2, 3
1
The correct factors are 1 and 6.
2x2  5x  3  2x2  mx  nx  3
 2x2  x  6x  3
 (2x2  x)  (6x  3)
 x(2x  1)  3(2x  1)
 (2x  1)(x  3)
Thus, 6x2  15x  9  3(2x  1)(x  3).

10.

x  3
Check:

3x2  11x  6  0

1 2

1 2

3x2  11x  6  0

2
2
3 3 2  11 3 #  6  0

3(3) 2  11(3)  6  0

149 2  223  6  0
4
3

22
3

18

3 

18 ?

3
18 ?
3

3(9)  33  6  0
?

27  33  6  0

0

6  6  0

00

11.

00

10p  19p  7  0
(5p  7)(2p  1)  0
5p  7  0 or 2p  1  0
5p  7
2p  1
7
1
p5
p2
The solution set is

512, 75 6.

Check:
10p2  19p  7  0
10

19
2

14

Sum of Factors
139
139
68
68
31
31
23
23
13
13
4
4

175 22  19175 2  7  0
?
49
133
10 1 25 2  5  7  0

2 
12.

10p2  19p  7  0

112 22  19112 2  7  0
?
1
19
10 1 4 2  2  7  0
5
2

9. Factor
 4n  35.
Since ac  140 and b  4, make a list of the
pairs of factors of 140 and find the pair whose
sum is 4.

14 ?

2
14 ?
2

10

98
5

133
5

35

0

5 

00
6n2  7n  20
6n2  7n  20  0
(3n  4)(2n  5)  0
3n  4  0 or 2n  5  0
3n  4
2n  5
4

n3

35 ?

5
35 ?
5

00

n  2
5 4

Check:
6n2  7n  20

1 2

1 2

20
20

20

1254 2  352  20
75
2

35 ?

2
40 ?

2

20  20

418

6n2  7n  20

143 22  7143 2  20
16
28 ?
6 1 9 2  3  20

5
5 ?
6 2 2  7 2  20

32
3

0

The solution set is 2, 3 .

The correct factors are 10 and 14.


4n2  4n  35  4n2  mn  pn  35
 4n2  10n  14n  35
 (4n2  10n)  (14n  35)
 2n(2n  5)  7(2n  5)
 (2n  5) (2n  7)

Chapter 9

The solution set is 3, 3 .

4n2

Factors of 140
1, 140
1, 140
2, 70
2,
70
4, 35
4,
35
5, 28
5,
28
7, 20
7,
20
10, 14
10,
14

3x2  11x  6  0
(3x  2)(x  3)  0
or x  3  0
3x  2  0
3x  2
x  3

28 ?

3
60 ?

3

20

20  20

21. Among all pairs of factors of 40, choose 5 and


8, the pair of factors whose sum is 3.
2x2  3x  20  2x2  5x  8x  20
 (2x2  5x)  (8x  20)
 x(2x  5)  4(2x  5)
 (2x  5)(x  4)
22. Among all pairs of negative factors of 70, choose
10 and 7, the pair of factors whose sum is 17.
5c2  17c  14  5c2  10c  7c  14
 (5c2  10c)  (7c  14)
 5c(c  2)  7(c  2)
 (c  2)(5c  7)
23. Among all negative factors of 48, there are no
pairs whose sum is 25.
Thus, 3p2  25p  16 is prime.
24. Among all pairs of factors of 72, choose 12 and
6, the pair of factors whose sum is 6.
8y2  6y  9  8y2  12y  6y  9
 (8y2  12y)  (6y  9)
 4y(2y  3)  3(2y  3)
 (2y  3)(4y  3)
25. Among all pairs of factors of 60, choose 15 and
4, the pair of factors whose sum is 11.
10n2  11n  6  10n2  15n  4n  6
 (10n2  15n)  (4n  6)
 5n(2n  3)  2(2n  3)
 (2n  3)(5n  2)
26. Among all pairs of factors of 270, choose 10
and 27, the pair whose sum is 17.
15z2  17z  18  15z2  10z  27z  18
 (15z2  10z)  (27z  18)
 5z(3z  2)  9(3z  2)
 (3z  2) (5z  9)
27. Among all pairs of factors of 168, choose 8 and
21, the pair whose sum is 13.
14x2  13x  12  14x2  8x  21x  12
 (14x2  8x)  (21x  12)
 2x(7x  4)  3(7x  4)
 (7x  4) (2x  3)
28. First factor out the GCF, 2.
6r2  14r  12  2(3r2  7r  6)
Now factor 3r2  7r  6. Among all pairs of factors
of 18, choose 9 and 2, the pair whose sum is 7.
3r2  7r  6  3r2  9r  2r  6
 3r(r  3)  2(r  3)
 (r  3)(3r  2)
Thus, 6r2  14r  12  2(r  3)(3r  2) .
29. First factor out the GCF, 5.
30x2  25x  30  5(6x2  5x  6)
Now factor 6x2  5x  6. Among all pairs of factors
of 36, choose 9 and 4, the pair whose sum is 5.
6x2  5x  6  6x2  9x  4x  6
 3x(2x  3)  2(2x  3)
 (2x  3)(3x  2)
Thus, 30x2  25x  30  5(2x  3)(3x  2).

13. Use the model for vertical motion with h  0,


v  8, and s  8.
h  16t2  vt  s
0  16t2  8t  8
0  8(2t2  t  1)
0  8(2t  1) (t  1)
2t  1  0
or t  1  0
2t  1
t1
1

t  2
1

Only 1 is a valid solution since 2 represents a


time before the vault was made. Thus the
gymnasts feet reach the mat in 1 s.

Pages 499500

Practice and Apply

14. Among all positive pairs of factors of 10, choose


2 and 5, the pair of factors whose sum is 7.
2x2  7x  5  2x2  2x  5x  5
 (2x2  2x)  (5x  5)
 2x(x  1)  5(x  1)
 (x  1)(2x  5)
15. Among all pairs of positive factors of 6, choose
2 and 3, the pair of factors whose sum is 5.
3x2  5x  2  3x2  2x  3x  2
 (3x2  2x)  (3x  2)
 x(3x  2)  1(3x  2)
 (3x  2)(x  1)
16. Among all pairs of factors of 36, choose 4 and
9, the pair of factors whose sum is 5.
6p2  5p  6  6p2  4p  9p  6
 2p(3p  2)  3(3p  2)
 (3p  2)(2p  3)
17. Among all pairs of factors of 40, choose 4 and
10, the pair of factors whose sum is 6.
5d2  6d  8  5d2  4d  10d  8
 d(5d  4)  2(5d  4)
 (5d  4)(d  2)
18. Among all pairs of negative factors of 72, there
are no pairs whose sum is 19.
Thus, 8k2  19k  9 is prime.
19. Among all pairs of negative factors of 36, choose
6 and 6, the pair of factors whose sum is 12.
9g2  12g  4  9g2  6g  6g  4
 (9g2  6g)  (6g  4)
 3g(3g  2)  2(3g  2)
 3g(3g  2)  2(1)(3g  2)
 3g(3g  2)  2(3g  2)
 (3g  2) (3g  2) or (3g  2) 2
20. Among all pairs of factors of 36, choose 3 and
12, the pair of factors whose sum is 9.
2a2  9a  18  2a2  12a  3a  18
 (2a2  12a)  (3a  18)
 2a(a  6)  3(a  6)
 (a  6)(2a  3)

419

Chapter 9

30. 9x2  30xy  25y2  9x2  (30y)x  (25y2 )


Among all pairs of positive factors of 225y2, choose
15y and 15y, the pair of factors whose sum is 30y.
9x2  30xy  25y2  9x2  (30y)x  25y2
 9x2  15yx  15yx  25y2
 (9x2  15yx)  (15yx  25y2 )
 3x(3x  5y)  5y(3x  5y)
 (3x  5y)(3x  5y) or
(3x  5y) 2
2
2
31. 36a  9ab  10b  36a2  (9b)a  (10b2 )
Among all pairs of positive factors of 360b2, choose
15b and 24b, the pair of factors whose sum is 9b.
36a2  9ab  10b2
 36a2  9ba  10b2
 36a2  15ba  24ba  10b2
 (36a2  15ba)  (24ba  10b2 )
 3a(12a  5b)  2b(12a  5b)
 (12a  5b) (3a  2b)
32. List all factors of 24 and the sums of the pairs of
factors.
Factors of 24
Sum of Factors
1, 24
25
1, 24
25
2, 12
14
2, 12
14
3,
8
11
3, 8
11
4,
6
10
4, 6
10
The possible values for k are
25,
14,
11,
and
10.
33. List all factors of 30 and the sums of the pairs of
factors.
Factors of 30
Sum of Factors
1, 30
31
1, 30
31
2, 15
17
2, 15
17
3, 10
13
3, 10
13
5,
6
11
5, 6
11
The possible values for k are
31,
17,
13,
and
11.
34. Because ac  2k is positive and b  12 is positive,
m and n must both be positive and mn must be
even. List all pairs of positive numbers whose
sum is 12 and list the products of the pairs.
Possible values for 2k will be the even products.

Exercises 3548 For checks, see students work.


35. 5x2  27x  10  0
(5x  2)(x  5)  0
or x  5  0
5x  2  0
5x  2
x  5

36.

55, 25 6

3x2  5x  12  0
(3x  4)(x  3)  0
or x  3  0
3x  4  0
3x  4
x3
4

x  3

37.

543, 36

24x2  11x  3  3x
24x2  14x  3  0
(4x  3)(6x  1)  0
4x  3  0 or 6x  1  0
4x  3
6x  1
3

x4

38.

516, 34 6

x  6

17x2  11x  2  2x2


15x2  11x  2  0
(5x  2)(3x  1)  0
5x  2  0 or 3x  1  0
5x  2
3x  1

513, 25 6

x5

x3

39. 14n2  25n  25


14n2  25n  25  0
(7n  5)(2n  5)  0
or
7n  5  0
7n  5
5
n  7

40.

2n  5  0
2n  5
5
n2

557, 52 6

12a2  13a  35
12a  13a  35  0
(4a  5)(3a  7)  0
or 3a  7  0
4a  5  0
4a  5
3a  7
2

a  4

41.

554, 73 6

x  3

523, 36
420

a3

6x2  14x  12
6x  14x  12  0
2(3x2  7x  6)  0
3x2  7x  6  0
(3x  2)(x  3)  0
or x  3  0
3x  2  0
3x  2
x3
2

Addends of 12
Product of Addends
1, 11
11
2, 10
20
(even)
3, 9
27
4, 8
32
(even)
5, 7
35
6, 6
36
(even)
The possible values of 2k are 20, 32, and 36.
Thus, the possible values of k are 10, 16, and 18.

Chapter 9

x  5

42.

21x2  6  15x
21x  15x  6  0
3(7x2  5x  2)  0
7x2  5x  2  0
(7x  2)(x  1)  0
or x  1  0
7x  2  0
7x  2
x1

48.

x  7

43.

527, 16

44.

512, 23 6

24x2  46x  18
24x  46x  18  0
2(12x2  23x  9)  0
12x2  23x  9  0
(3x  1) (4x  9)  0
3x  1  0
or 4x  9  0
3x  1
4x  9
2

45.

513, 94 6
12

x2
12

x2
12

2x
3

2x
3

x2

x4

40

 4  12(0)

t2

 8x  48  0
(x  12) (x  4)  0
x  12  0
or x  4  0
x  12
x  4
{4, 12}
46.

t2  6 

The negative value represents a time before the


diver leaps. The diver will enter the water after
5
or 2.5 seconds.
2
52. Use the model for vertical motion with h  30,
v  56, and s  6.
h  16t2  vt  s
30  16t2  56t  6
0  16t2  56t  24
0  8(2t2  7t  3)
0  8(2t  1)(t  3)
2t  1  0 or t  3  0
2t  1
t3
1
t2

35
6

t
6 t2  6  6

1356 2

6t2  t  35
6t  t  35  0
(3t  7) (2t  5)  0
or 2t  5  0
3t  7  0
3t  7
2t  5
7
5
t  3
t2
2

47.

573, 52 6

The first time, t  2, represents how long it takes


the hook to reach a height of 30 feet on its way
up. The second time, t  3, represents the time
the hook anchors on the ledge on its way down.
Thus, the hook is in the air for 3 seconds.

(3y  2) (y  3)  y  14
3y2  9y  2y  6  y  14
3y2  11y  6  y  14
3y2  10y  8  0
(3y  2) ( y  4)  0
3y  2  0 or y  4  0
3y  2
y  4

54, 23 6

a2

49. Write an equation for the area of the smaller


rectangle and solve for x.
(9  2x)(7  2x)  35
63  18x  14x  4x2  35
4x2  32x  63  35
4x2  32x  28  0
4(x2  8x  7)  0
4(x  7)(x  1)  0
x  7  0 or x  1  0
x7
x1
If x  7, the dimensions of the smaller rectangle
would be negative. Thus, the width of each strip
is 1 inch.
50. The dimensions of the new rectangle are (7  2) in.
by (9  2) in. or 5 in. by 7 in.
51. Use the model for vertical motion with h  0,
v  8, and s  80.
h  16t2  vt  s
0  16t2  8t  80
0  8(2t2  t  10)
0  8(2t  5) (t  2)
2t  5  0 or t  2  0
2t  5
t  2

x2

x  3

a2

512, 72 6

24x2  30x  8  2x


24x2  28x  8  0
4(6x2  7x  2)  0
6x2  7x  2  0
(3x  2)(2x  1)  0
3x  2  0 or 2x  1  0
3x  2
2x  1
x3

(4a  1)(a  2)  7a  5
4a2  8a  a  2  7a  5
4a2  9a  2  7a  5
4a2  16a  7  0
(2a  7)(2a  1)  0
2a  7  0 or 2a  1  0
2a  7
2a  1

y3

421

Chapter 9

PQ249-6481F-09[422-437] 26/9/02 6:14 PM Page 422 Sahuja Ahuja_QXP_06:Desktop Folder:Chandra:Algebra_FNL_Delivery:

60. (2k  9) (3k  2)  0


or
2k  9  0
2k  9

53. Answers should include the following.


2x  3 by x  2
With algebra tiles, you can try various ways to
make a rectangle with the necessary tiles.
Once you make the rectangle, however, the
dimensions of the rectangle are the factors of
the polynomial. In a way, you have to go
through the guess-and-check process whether
you are factoring algebraically or
geometrically (using algebra tiles.)
Guess: (2x  1)(x  3)
The guess is incorrect because one more x tile is
needed to complete the rectangle.

k  2

592, 23 6

3k  2  0
3k  2
2

k  3

61. 12u  u2
0  u2  12u
0  u(u  12)
u  0 or u  12  0
u  12
{0, 12}
62. Two points are (2, 83) and (5, 185). Find the slope.
y2  y1

x x x

mx

x x

x x x

1
1

x
x
x

1
1

1
1

x x
1
1
1

1
1
1

54. D;
2p2  p  3  0
(2p  3)(p  1)  0
2p  3  0
2p  3

or

p10
p  1

63.
64.
65.
66.
67.
68.
69.
70.

p2
55. B; Solve the equation for h  338.
h  16t2  48t  402
338  16t2  48t  402
0  16t2  48t  64
0  16(t2  3t  4)
0  16(t  1)(t  4)
t10
or t  4  0
t  1
t4
Choose the positive solution. The ball will be
338 feet from the ground after 4 seconds.

Page 500

Chapter 9

Find the y-intercept.


y  mx  b
83  34(2)  b
83  68  b
15  b
Write the equation.
y  mx  b
y  34x  15
42  16 S 116  4
72  49 S 149  7
62  36 S 136  6
52  25 S 125  5
102  100 S 1100  10
112  121 S 1121  11
132  169 S 1169  13
152  225 S 1225  15

Page 500

Practice Quiz 2

1. Among all pairs of factors of 72, choose 4 and


18, the pair of factors whose sum is 14.
x2  14x  72  (x  4)(x  18)
2. Among all pairs of factors of 280, choose 20
and 14, the pair whose sum is 6.
8p2  6p  35  8p2  20p  14p  35
 (8p2  20p)  (14p  35)
 4p(2p  5)  7(2p  5)
 (2p  5) (4p  7)
3. Among all pairs of negative factors of 80, choose
4 and 20, the pair of factors whose sum
is 24.
16a2  24a  5  16a2  4a  20a  5
 (16a2  4a)  (20a  5)
 4a(4a  1)  5(4a  1)
 (4a  1) (4a  5)
4. Among all pairs of negative factors of 52, choose
4 and 13, the pair of factors whose sum
is 17.
n2  17n  52  (n  4)(n  13)

Maintain Your Skills

56. Among all pairs of factors of 21, choose 3 and


7, the pair of factors whose sum is 4.
a2  4a  21  (a  m)(a  n)
 (a  3)(a  7)
57. Among all pairs of positive factors of 2, there is no
pair of factors whose sum is 2. Thus, t2  2t  2
is prime.
58. Among all pairs of factors of 44, choose 4 and 11,
the pair of factors whose sum is 15.
d2  15d  44  (d  m)(d  n)
 (d  4)(d  11)
Exercises 5961 For checks, see students work.
59. (y  4)(5y  7)  0
y  4  0 or 5y  7  0
y4
5y  7

575, 46

 x1

185  83
5  2
102
or 34
3

y  5

422

PQ249-6481F-09[422-437] 26/9/02 6:14 PM Page 423 Sahuja Ahuja_QXP_06:Desktop Folder:Chandra:Algebra_FNL_Delivery:

5. First factor out the GCF, 2.


24c2  62c  18  2(12c2  31c  9)
Now factor 12c2  31c  9. Among all pairs of
positive factors of 108, choose 4 and 27, the pair
whose sum is 31.
12c2  31c  9  12c2  4c  27c  9
 4c(3c  1)  9(3c  1)
 (3c  1)(4c  9)
Thus, 24c2  62c  18  2(3c  1)(4c  9).
6. First factor out the GCF, 3.
3y2  33y  54  3(y2  11y  18)
Now factor y2  11y 18. Since the lead
coefficient is 1, find two factors of 18 whose sum
is 11. The correct factors are 2 and 9.
So, y2  11y  18  (y  2)( y  9).
Thus, 3y2  33y  54  3(y  2)( y  9).
7. b2  14b  32  0
(b  2)(b  16)  0
b  2  0 or b  16  0
b2
b  16
{16, 2}
Check:
b2  14b  32  0
?

256  224  32  0

4  28  32  0

32  32  0

32  32  0

Check:
6a2  25a  14
?
2
2
6 3 2  25 3  14

12
12
4 ? 50
42
619 2  3  3
8
3

Page 501

y  4

y3

Check:

1 2

12

1169 2  214  484  0


27
4

21
4

48 ?

4

00

147
2

Algebra Activity

12y2  7y  12  0

143 22  7143 2  12  0
16
28
36 ?
12 1 9 2  3  3  0
?

12

64
3

28
3

36 ?

3

Check for Understanding

1. The binomial is the difference of two terms, each


of which is a perfect square.
2. Sample answer: x2  25  x2  52
 (x  5)(x  5)
3. Yes, you can use the differences of squares pattern
to factor 3n2  48 after you factor out the GCF.
3n2  48  3(n2  16)
 3(n2  42 )
 3(n  4) (n  4)
4. Manuel; 4x2  y2 is not the difference of squares.
5. n2  81  n2  92
 (n  9) (n  9)
6. 4  9a2  22  (3a) 2
 (2  3a) (2  3a)
7. 2x5  98x3  2x3 (x2  49)
 2x3 (x2  72 )
 2x3 (x  7) (x  7)
4  2y4  2(16x4  y4 )
8. 32x
 2[ (4x2 ) 2  (y2 ) 2 ]
 2(4x2  y2 ) (4x2  y2 )
 2(4x2  y2 ) [ (2x) 2  y2 ]
 2(4x2  y2 ) (2x  y) (2x  y)
2  27 is not the difference of perfect squares,
9. 4t
and 4t2 and 27 have no common factors. Thus
4t2  27 is prime.
10. x3  3x2  9x  27  (x3  3x2 )  (9x  27)
 x2 (x  3)  9(x  3)
 (x  3) (x2  9)
 (x  3) (x2  32 )
 (x  3) (x  3) (x  3)
 (x  3) (x  3) (x  3)
or (x  3) (x  3) 2

x2  45  18x
x  45  18x  0
x2  18x  45  0
(x  3)(x  15)  0
x  3  0 or x  15  0
x3
x  15
{3, 15}
Check:
x2  45  18x
x2  45  18x
?
?
2
(15) 2  45  18(15)
(3)  45  18(3)
?
?
225  45  270
9  45  54
54  54
270  270
9. 12y2  7y  12  0
(4y  3)(3y  4)  0
or 3y  4  0
4y  3  0
4y  3
3y  4

1 2

147
2

Factoring Differences of Squares

Pages 504505

00

3
3
12 4 2  7 4  12  0

12
12
49 ? 175
28
61 4 2  2  2

6a2  25a  14
?
7
7
6 2 2  25 2  14

1. (a  b)(a  b)
2. Since a2  b2 and (a  b)(a  b) describe the
same area, a2  b2  (a  b)(a  b).

8.

3

9-5

00

12y2  7y  12  0

a2

534, 43 6

a3

523, 72 6

(2)  14(2)  32  0

6a2  25a  14
6a  25a  14  0
(3a  2)(2a  7)  0
3a  2  0 or 2a  7  0
3a  2
2a  7
2

b2  14b  32  0
2

(16)  (14)(16)  32  0
?

10.

00

423

Chapter 9

PQ249-6481F-09[422-437] 26/9/02 6:14 PM Page 424 Sahuja Ahuja_QXP_06:Desktop Folder:Chandra:Algebra_FNL_Delivery:

11.

4y2  25
4y  25  0
(2y) 2  52  0
(2y  5)(2y  5)  0
or
2y  5  0
2y  5

Check:
121a  49a3

2y  5  0
2y  5

00
121a 

121

4y2  25

1 2 25
4 1 4 2  25

152 22  25
25 ?
4 1 4 2  25

?
5
2 2 
25 ?

1331
7

1 1

17  68k2  0

1 2
1 ?
17  68 1 4 2  0

112 22  0
1 ?
17  68 1 4 2  0

13.

Pages 505506

17  17  0
00

x2 

x60

x60

or
1

x  6

1 1

x6

The solution set is 6, 6 .


Check:
1

x2  36  0

x2  36  0

116 22  361  0

116 22  361  0

1
36

1
36

 36  0
00

14.

 36  0

00
121a  49a3

49a3  121a  0
a(49a2  121)  0
2

11

a  7
11

The solution set is  7 , 0,

Chapter 9

11
7

 72

1 2  2(72)
1 2
x
2

Practice and Apply

26. 8d2  18  2(4d2  9)


 2[ (2d) 2  32 ]
 2(2d  3) (2d  3)

a[ (7a)  11 ]  0
a(7a  11) (7a  11)  0
or
a  0 or 7a  11  0
a0
7a  11

16. x2  49  x2  72
 (x  7)(x  7)
17. n2  36  n2  62
 (n  6)(n  6)
18. 81  16k2 is prime.
19. 25  4p2  52  (2p) 2
 (5  2p) (5  2p)
20. 16  49h2  49h2  16
 (7h) 2  42
 (7h  4) (7h  4)
21. 9r2  121  121  9r2
 112  (3r) 2
 (11  3r) (11  3r)
22. 100c2  d2  (10c) 2  d2
 (10c  d) (10c  d)
23. 9x2  10y2 prime
24. 144a2  49b2  (12a) 2  (7b) 2
 (12a  7b) (12a  7b)
2
2
25. 169y  36z  (13y) 2  (6z) 2
 (13y  6z) (13y  6z)

x2  36  0

116 22  0
1x  16 21x  16 2  0

1331
7

17  17  0
00

x2  144
x  144  0
(x  12)(x  12)  0
or x  12  0
x  12  0
x  12
x  12
Since length cannot be negative, the only reasonable
solution is 12. The square is 12 in. by 12 in.

17  68

17  68k2  0

?
1
17  68 2 2  0

1331
7



The solution set is 2, 2 .


Check:

49a3

1 2
x
2

k2

15. The area of the square is x  x or x2 square inches,


1
1
and the area of the triangle is 2  x  x or 2x2
square inches. Thus, the area of the shaded
1
1
region is x2  2x2 or 2x2 square inches.

25  25
25  25
20
17  68k
12.
17(1  4k2 )  0
17 [12  (2k) 2 ]  0
17(1  2k)(1  2k)  0
or 1  2k  0
1  2k  0
2k  1
2k  1
k  2

1 2
1331
49 1 49  7 2

1117 2  491117 23
1331 ?
1331
 49 1 49  7 2
7

The solution set is 2, 2 .


Check:
4y2  25

1331 ?
 7 
1331 ?

0  49(0)

y2

11 ?
11
121  7  49  7 3

5 5

121(0)  49(0) 3

y  2

121a  49a3

7a  11  0
7a  11
a

27. 3x2  75  3(x2  25)


 3(x2  52 )
 3(x  5) (x  5)

11
7

28. 8z2  64  8(z2  8)


29. 4g2  50  2(2g2  25)

424

PQ249-6481F-09[422-437] 26/9/02 6:15 PM Page 425 Sahuja Ahuja_QXP_06:Desktop Folder:Chandra:Algebra_FNL_Delivery:

30. 18a4  72a2  18a2 (a2  4)


 18a2 (a2  22 )
 18a2 (a  2) (a  2)

39.

31. 20x3  45xy2  5x(4x2  9y2 )


 5x[ (2x) 2  (3y) 2 ]
 5x(2x  3y)(2x  3y)
32. n3  5n2  4n  20
 (n3  5n2 )  (4n  20)
 n2 (n  5)  (4)(n  5)
 (n  5)(n2  4)
 (n  5)(n2  22 )
 (n  5)(n  2)(n  2)
33. (a  b) 2  c2  [ (a  b)  c ] [ (a  b)  c ]
 (a  b  c) (a  b  c)
Exercises 3445 For checks, see students work.
25x2  36
34.
2  36  0
25x
(5x) 2  62  0
(5x  6)(5x  6)  0
or 5x  6  0
5x  6  0
5x  6
5x  6
x

5 6

6
5

x

36.

37.

5 83 6

40.

41.

6
5

2  3n  0
3n  2

43.

n3

44.

5  2a  0
2a  5

5 27 6

5 186

630

or

 6

3  6

r  18

r  18

1 2
x  25
4
1 2
x  52
2

0

0
1 2
112x  52112x  52  0
50

1
x
2

or

50
1
x
2

 5

5

x  10

12d3  147d  0
3d(4d2  49)  0
3d(2d  7) (2d  7)  0
3d  0 or 2d  7  0
d0
2d  7

572, 0, 72 6

n  3

2d  7  0
2d  7
7

d  2

18n3  50n  0
2n(9n2  25)  0
2n(3n  5) (3n  5)  0
2n  0 or 3n  5  0
n0
3n  5

553, 0, 53 6

or

d2

or

3n  5  0
3n  5
5

n3

x3  4x  12  3x2
x  4x  12  3x2  0
x3  3x2  4x  12  0
3
(x  3x2 )  (4x  12)  0
x2 (x  3)  4(x  3)  0
(x  3) (x2  4)  0
(x  3) (x  2) (x  2)  0
or x  2  0
or x  2  0
x30
x  2
x2
x  3
{3, 2, 2}
3

a2

w2 

w  7

13r 22  0
16  3r 2 16  3r 2  0

{10}

or

36  9r2  0

r
3

w2  49  0

5 109 6

42.

w70

p  10

x  10

127 22  0
1w  27 21w  27 2  0

p0
9

1
x
2

a  2
38.

9
10

p  10

1
x
2

n  3

50  8a2  0
2(25  4a2 )  0
2 [52  (2a) 2 ]  0
2(5  2a)(5  2a)  0
or
5  2a  0
2a  5

or

9
10

630

5 23 6

p0

62 

y3

12  27n2  0
3(4  9n2 )  0
3 [22  (3n) 2 ]  0
3(2  3n)(2  3n)  0
or
2  3n  0
3n  2

1 2  p2  0
1109  p21109  p2  0
p

9y2  64
9y2  64  0
(3y) 2  82  0
(3y  8)(3y  8)  0
or 3y  8  0
3y  8  0
3y  8
3y  8
y  3

 p2  0

9
10

6
5

35.

81
100
9 2
10

w70
2

w7

425

Chapter 9

PQ249-6481F-09[422-437] 26/9/02 6:15 PM Page 426 Sahuja Ahuja_QXP_06:Desktop Folder:Chandra:Algebra_FNL_Delivery:

45.

36x  16x3  9x2  4x4


 9x2  36x  0
4x 
3
2
x(4x  16x  9x  36)  0
x [ (4x3  16x2 )  (9x  36) ]  0
x [4x2 (x  4)  9(x  4) ]  0
x [ (x  4)(4x2  9) ]  0
x(x  4) (2x  3)(2x  3)  0
x  0 or x  4  0 or 2x  3  0 or 2x  3  0
x4
2x  3
2x  3
4

Since length cannot be negative, the only


reasonable solution is 3. The box is 3 in. by 12 in.
by 2 in.
51. The flaw is in line 5. Since a  b, a  b  0.
Therefore, dividing by a  b is dividing by zero,
which is undefined.
52. Answers should include the following:
1 foot
To find the hang time of a student athlete who
attains a maximum height of 1 foot, solve the
equation 4t2  1  0. You can factor the left
side using the difference of squares pattern
since 4t2 is the square of 2t and 1 is the
square of 1. Thus the equation becomes
(2t  1)(2t  1)  0. Using the Zero Product
Property, each factor can be set equal to zero,
1
1
resulting in two solutions, t  2 and t  2.
Since time cannot be negative, the hang time
1
is 2 second.

16x3

x  2

532, 0, 32, 46

x2

46. Use factoring by grouping.


a2  b2  a2  ab  ab  b2
 (a2  ab)  (ab  b2 )
 a(a  b)  b(a  b)
 (a  b)(a  b)
47. Replace b with 3600 in the equation and solve for c.
900c2  b
900c2  3600
2
900c  3600  0
900(c2  4)  0
900(c  2) (c  2)  0
or c  2  0
c20
c  2
c2
Since circumference cannot be negative, the only
reasonable solution is 2 in.
1

53. A; 25b2  1  (5b) 2  1


 (5b  1) (5b  1)
 (5b  1) (5b  1)
54. Let x represent the length of a side of the larger
square. Then the perimeter of the larger square is
4x in., and the perimeter of the smaller square is
68  4x in. Thus, the length of a side of the
1
smaller square is 4 (68  4x) or 17  x in.
Write an equation for the area between the two
squares.
x2  (17  x) 2  17
2
2
x  (17  2(17)(x)  x2 )  17
x2  (289  34x  x2 )  17
x2  289  34x  x2  17
34x  289  17
34x  306
x9
The length of a side of the larger square is 9 in.

48. p  2dv2  2dv2


1

 2d v21  v22
1
 2d(v  v )(v  v )
1
2
1
2
49. Replace d with 54 in the equation and solve for s.

24

1 2
s
24
1 2
s
24
1 2
s
24

d
 54

2  24(54)

s  1296
s2  1296  0
s2  362  0
(s  36) (s  36)  0
or s  36  0
s  36  0
s  36
s  36
Since speed cannot be negative, the only
reasonable answer is 36. The car that left the
54-foot skid marks was traveling approximately
36 mph when the brakes were applied.
50. The volume of the box is x(x  9)(x  1).
x(x  9) (x  1)  72
x(x2  x  9x  9)  72
x(x2  8x  9)  72
3  8x2  9x  72  0
x
x2 (x  8)  9(x  8)  0
(x  8)(x2  9)  0
(x  8)(x  3) (x  3)  0
or x  3  0
or x  3  0
x80
x  8
x  3
x3

Chapter 9

Page 506

Maintain Your Skills

55. Among all pairs of positive factors of 14, there are


no pairs whose sum is 5. Thus, 2n2  5n  7 is
prime.
56. Among all pairs of negative factors of 24, choose
3 and 8, the pair whose sum is 11.
6x2  11x  4  6x2  3x  8x  4
 3x(2x  1)  4(2x  1)
 (2x  1) (3x  4)
57. Among all pairs of factors of 210, choose 6 and
35, the pair whose sum is 29.
21p2  29p  10  21p2  6p  35p  10
 3p(7p  2)  5(7p  2)
 (7p  2) (3p  5)
Exercises 5860 For checks, see students work.
58. y2  18y  32  0
( y  2)( y  16)  0
or y  16  0
y20
y  2
y  16
{16, 2}

426

PQ249-6481F-09[422-437] 26/9/02 6:15 PM Page 427 Sahuja Ahuja_QXP_06:Desktop Folder:Chandra:Algebra_FNL_Delivery:

k2  8k  15
k  8k  15  0
(k  3)(k  5)  0
k  3  0 or k  5  0
k3
k5
{3, 5}
60.
b2  8  2b
2  8  2b  0
b
b2  2b  8  0
(b  2)(b  4)  0
or b  4  0
b20
b4
b  2
{2, 4}
61. Let x represent Amys score on the fourth biology
test. Her average must be between 88 and 92,
inclusive.

Page 507

59.

88 
88 
88 
4(88) 

88  90  91  x
 92
4
269  x
 92
4
269  x
269  x
and
4
4
269  x
269  x
4
4
4
4

9-6
 92

Page 512

2  4(92)

63. 5  10r 7 2
10r 7 7
7
10

6
10

8
10

12
10

5.
6.
7.
8.

14
10

64. 13x  3 6 23
13x 6 26
x 6 2
6

4

2

Check for Understanding

x3  5x2  4x  20  x2 (x  5)  4(x  5)
 (x  5) (x2  4)
 (x  5) (x  2) (x  2)
4. Yes;
The first term is a perfect square: y2.
The last term is a perfect square: 42.
The middle term is 2(y)(4) or 8y.

1 2 3 4 5 6 7 8 9 10 11 12

4
10

Perfect Squares and Factoring

1. Determine if the first term is a perfect square.


Then determine if the last term is a perfect
square. Finally, check to see if the middle term is
equal to twice the product of the square roots of
the first and last terms.
2. Never; (a  b)2  a2  2ab  b2
3. Sample answer: x3  5x2  4x  20

269  x  368
352  269  x
x  99
83  x
83  x  99
Her test score must be between 83 and 99, inclusive.
Exercises 6264 For checks, see students work.
62. 6  3d  12
18  3d
6d

r 7

Reading Mathematics

1. (1) explains how to factor a perfect square


trinomial; (2) summarizes methods used to factor
polynomials; (3) explains how to solve equations
involving perfect squares using the Square Root
Property
2. GCF, perfect square trinomial; x2  bx  c,
ax2  bx  c
3. a greatest common factor
4. For any number n, where n is positive, the square
of x equals n, then x equals plus or minus the
square root of n. This property can be applied to
the equation (a  4)2  49 since the variable
x  a  4 and n  49 in the equation x2  n.

9.

65. (x  1)(x  1)  (x  1) 2
 x2  2x(1)  12
 x2  2x  1
66. (x  6)(x  6)  (x  6) 2
 x2  2x(6)  62
 x2  12x  36
67. (x  8) 2  x2  2x(8)  82
 x2  16x  64

10.

y2  8y  16  y2  2(y) (4)  42
 (y  4) 2
No; the last term, 10, is not a perfect square.
2x2  18  2(x2  9)
c2  5c  6  (c  3) (c  2)
5a3  80a  5a(a2  16)
 5a(a  4) (a  4)
2
8x  18x  35  8x2  28x  10x  35
 4x(2x  7)  5(2x  7)
 (2x  7) (4x  5)
There is no pair of factors of 36 whose sum is 12.
9g2  12g  4 is prime.

11. 3m3  2m2n  12m  8n


 m2 (3m  2n)  4(3m  2n)
 (3m  2n) (m2  4)
 (3m  2n) (m  2) (m  2)

68. (3x  4) (3x  4)  (3x  4) 2


 (3x) 2  2(3x)(4)  42
 9x2  24x  16
2
2
69. (5x  2)  (5x)  2(5x)(2)  22
 25x2  20x  4
70. (7x  3) 2  (7x) 2  2(7x)(3)  32
 49x2  42x  9

427

Chapter 9

PQ249-6481F-09[422-437] 26/9/02 6:15 PM Page 428 Sahuja Ahuja_QXP_06:Desktop Folder:Chandra:Algebra_FNL_Delivery:

12.

4y2  24y  36  0
4(y2  6y  9)  0
2  2(y)(3)  32 ]  0
4[ y
4(y  3) 2  0
y30
y  3
{3}

Ignore the negative solution. It took about


3.35 seconds for the objects to hit the ground.

Pages 512514

4y2  24y  36  0

Check:

4(3) 2  24(3)  36  0
?

36  72  36  0
00
13.
3n2  48
3n2  48  0
3(n2  16)  0
3(n  4)(n  4)  0
or n  4  0
n40
n4
n  4
{4, 4}
Check:
3n2  48

3n2  48

?
2

48
3(4) 2  48
3(4)
48  48
48  48
14.
a2  6a  9  16
a2  2(a) (3)  32  16
(a  3) 2  16
a  3  116
a  3  4
a3  4
a  3  4 or a  3  4
 1
7
{1, 7}
Check:
a2  6a  9  16
?

(1) 2  6(1)  9  16
16  16
a2  6a  9  16
?
72  6(7)  9  16
16  16
2
15. (m  5)  13
m  5  113
m  5  113
{5  113} or {1.4, 8.6}
(m  5) 2  13
?
(5  113  5) 2  13
?
( 113) 2  13
13  13
(m  5) 2  13
?
(5  113  5) 2  13
?
(113) 2  13
13  13
16. Let h  0 and h0  180 and solve for t.
h  16t2  h0

x
1
2
3
4
5

Check:

Area
16
1
36
121
256

Perimeter
16
4
24
44
64

If the pattern in the table continues, the perimeter


will continue to get larger for increasing positive
integer values of x. The least perimeter is
4 meters.
25. 4k2  100  4(k2  25)
 4(k  5) (k  5)
26. 9x2  3x  20  9x2  15x  12x  20
 3x(3x  5)  4(3x  5)
 (3x  5) (3x  4)
27. x2 + 6x  9 is prime.

0  16t2  180
16t2  180
t2  11.25
t  111.25
t  3.35

Chapter 9

Practice and Apply

17. No; the middle term is not 2(x)(9).


18. Yes;
The first term is a perfect square: a2.
The third term is a perfect square: 122.
The middle term is 2(a)(12).
a2  24a  144  a2  2(a) (12)  122
 (a  12) 2
19. Yes;
The first term is a perfect square: (2y)2.
The third term is a perfect square: 112.
The middle term is 2(2y)(11).
4y2  44y  121  (2y) 2  2(2y)(11)  112
 (2y  11) 2
20. No; the first term, 2c2, is not a perfect square.
21. 9n2  49  42n  9n2  42n  49
Yes;
The first term is a perfect square: (3n)2.
The third term is a perfect square: 72.
The middle term is 2(3n)(7).
9n2  42n  49  (3n) 2  2(3n) (7)  72
 (3n  7) 2
22. Yes;
The first term is a perfect square: (5a)2.
The third term is a perfect square: (12b)2.
The middle term is 2(5a)(12b).
25a2  120ab  144b2
 (5a) 2  2(5a)(12b)  (12b) 2
 (5a  12b) 2
23. To find the radius of the circle, write the
expression for the area in the form r2.
(16x2  80x  100)
  [ (4x) 2  2(4x) (10)  102 ]
 (4x  10) 2
The radius is 4x  10 inches, so the diameter is
2(4x  10) or 8x  20 inches.
24. Make a table.

428

PQ249-6481F-09[422-437] 26/9/02 6:15 PM Page 429 Sahuja Ahuja_QXP_06:Desktop Folder:Chandra:Algebra_FNL_Delivery:

28. 50g2  40g  8  2(25g2  20g  4)


 2[ (5g) 2  2(5g)(2)  22 ]
 2(5g  2) 2
3
2
29. 9t  66t  48t  3t(3t2  22t  16)
 3t(3t2  2t  24t  16)
 3t [t(3t  2)  8(3t  2) ]
 3t(3t  2)(t  8)
30. 4a2  36b2  4(a2  9b2 )
 4[ a2  (3b) 2 ]
 4(a  3b) (a  3b)
31.

32.
33.

34.

35.

36.
37.

38.
39.

40.

20n2

The length s must be positive. The expression


7
4x  7 is positive when x 7 4 and 7  4x is
7
positive when x 6 4. Write two expressions for
the area of the rectangle.
7

If x 7 4, s  4x  7.

11 2

A  (s  3) 2 s

31

 [ (4x  7)  3] 2 (4x  7)
 (4x  4)

1
(4x
2

 7)

 (2x  2) (4x  7)
 8x2  14x  8x  14
 8x2  22x  14 in2

2(10n2

 34n  6 
 17n  3)
 2(10n2  15n  2n  3)
 2[5n(2n  3)  (2n  3) ]
 2(2n  3)(5n  1)
5y2  90  5(y2  18)
24x3  78x2  45x  3x(8x2  26x  15)
 3x(8x2  6x  20x  15)
 3x [2x(4x  3)  5(4x  3) ]
 3x(4x  3)(2x  5)
18y2  48y  32  2(9y2  24y  16)
 2[ (3y) 2  2(3y)(4)  42 ]
 2(3y  4) 2
2  75  27g2  90g  75
90g  27g
 3(9g2  30g  25)
 3[ (3g) 2  2(3g)(5)  52 ]
 3(3g  5) 2
2
45c  32cd  c(45c  32d)
4a3  3a2b2  8a  6b2
 a2 (4a  3b2 )  2(4a  3b2 )
 (4a  3b2 )(a2  2)
5a2  7a  6b2  4b is prime.
x2y2  y2  z2  x2z2
 y2 (x2  1)  z2 (1  x2 )
 y2 (x2  1)  z2 (x2  1)
 (x2  1)(y2  z2 )
 (x  1)(x  1)(y2  z2 )
4m4n  6m3n  16m2n2  24mn2
 2mn(2m3  3m2  8mn  12n)
 2mn [m2 (2m  3)  4n(2m  3) ]

If x 6 4, s  7  4x.

11 2

A  (s  3) 2 s

31

 [ (7  4x)  3] 2 (7  4x)
 (10  4x)

1
(7
2

 4x)

 (5  2x) (7  4x)
 35  20x  14x  8x2
 8x2  34x  35 in2
Exercises 4354 For checks, see students work.
43.
3x2  24x  48  0
3(x2  8x  16)  0
2
3[x  2(x) (4)  42 ]  0
3(x  4) 2  0
x40
x  4
{4}
44.
7r2  70r  175
2  70r  175  0
7r
7(r2  10r  25)  0
7[r2  2(r) (5)  52 ]  0
7(r  5) 2  0
r50
r5
{5}
45.
49a2  16  56a
2
49a  56a  16  0
(7a) 2  2(7a) (4)  42  0
(7a  4) 2  0
7a  4  0
7a  4

 2mn(2m  3) (m2  4n)


41. Factor the polynomial.
x3y  63y2  7x2  9xy3
 x3y  9xy3  7x2  63y2
 xy(x2  9y2 )  7(x2  9y2 )
 (x2  9y2 )(xy  7)
 (x  3y)(x  3y)(xy  7)
The dimensions are x  3y m, x  3y m, and
xy  7 m.
42. Express the polynomial for the area of the square
in the form s2.
16x2  56x  49  (4x) 2  2(4x)(7)  72
 (4x  7) 2
or
16x2  56x  49  49  56x  16x2
 72  2(7)(4x)  (4x) 2
 (7  4x) 2

46.

547 6

18y2  24y  8  0
2(9y2  12y  4)  0
2  2(3y) (2)  22 ]  0
2[ (3y)
2(3y  2) 2  0
3y  2  0
3y  2

523 6
429

a7

y  3

Chapter 9

PQ249-6481F-09[422-437] 26/9/02 6:15 PM Page 430 Sahuja Ahuja_QXP_06:Desktop Folder:Chandra:Algebra_FNL_Delivery:

y2  3y  9  0

47.

y2  2(y)

55. B  16 (D2  8D  16)

113 2  113 22  0
1y  13 22  0

B  16 [D2  2(D)(4)  42 ]
L

B  16 (D  4) 2
56. Replace B with 256 and L with 16 and solve
for D.

y30

56

y3

16

256  16 (D  4) 2

1
3

48.

a2  5a  25  0

a2  2(a)

125 2  125 22  0
1a  25 22  0
2

a50

525 6

a  5

z2  2z  1  16
z  2(z)(1)  12  16
(z  1) 2  16
z  1  116
z  1  4
z  1  4
z  1  4 or
3
{5, 3}
50.
x2  10x  25  81
2  2(x)(5)  52  81
x
(x  5) 2  81
x  5  181
x  5  9
x  5  9
x  5  9 or
4
{14, 4}
51. (y  8) 2  7
y  8  17
49.

57.

z  1  4
 5

58.

x  5  9
 14

59.

y  8  17
{8  17} or {5.4, 10.6}
52. (w  3) 2  2
w  3  12
w  3  12
{3  12} or {4.4, 1.6}
53.
p2  2p  1  6
2  2(p) (1)  12  6
p
(p  1) 2  6
p  1  16
p  1  16
{1  16} or {3.4, 1.4}
54.
x2  12x  36  11
2  2(x)(6)  62  11
x
(x  6) 2  11
x  6  111
x  6  111
{6  111} or {2.7, 9.3}

Chapter 9

60.

256  (D  4) 2
1256  D  4
16  D  4
4  16  D
D  4  16 or D  4  16
 20
 12
Since diameter must be positive, 20 is the only
reasonable solution. The logs should be 20 inches
in diameter.
Let h  0 and t  3. Solve for s.
h  16t2  s
0  16(3) 2  s
0  16(9)  s
0  144  s
144  s
The starting height of the car should be at least
144 feet.
Let h  0 and s  160. Solve for t.
h  16t2  s
0  16t2  160
16t2  160
t2  10
t  110 or 3.16
Ignore the negative solution. The riders will be in
free fall about 3.16 seconds.
h  16t2  vt  s
70  16t2  64t  6
0  16t2  64t  64
0  16(t2  4t  4)
0  16[ t2  2(t)(2)  22 ]
0  16(t  2) 2
0t2
2t
Yes; the acrobat will reach a height of 70 feet
after 2 seconds.
x2  kx  64  x2  kx  82
This will be a perfect square trinomial if
kx  2(x)(8) or k  16x. Thus, k can be
16 or 16.

61. 4x2  kx  1  (2x) 2  kx  12


This will be a perfect square trinomial if
kx  2(2x)(1) or kx  4x. Thus, k can be
4 or 4.
62. 25x2  kx  49  (5x) 2  kx  72
This will be a perfect square trinomial if
kx  2(5x)(7) or kx  70x. Thus, k can be
70 or 70.
63. x2  8x  k  x2  2(x) (4)  k
This would be a perfect square trinomial if k  42
or 16.

430

PQ249-6481F-09[422-437] 26/9/02 6:15 PM Page 431 Sahuja Ahuja_QXP_06:Desktop Folder:Chandra:Algebra_FNL_Delivery:

64. x2  18x  k  x2  2(x)(9)  k


This would be a perfect square trinomial if
k  92 or 81.
65. x2  20x  k  x2  2(x)(10)  k
This would be a perfect square trinomial if
k  102 or 100.
66. Answers should include the following.
The length of each side of the pavilion is
8  x  x or 8  2x feet. Thus, the area of the
pavilion is (8  2x)2 square feet. This area
includes the 80 square feet of bricks and the
82 or 64-square-foot piece of art, for a total
area of 144 square feet. These two
representations of the area of the pavilion
must be equal, so we can write the equation
(8  2x)2  144.
(8  2x) 2  144
Original
equation
Square Root
8  2x  12
Property
8  2x  12 or 8  2x  12 Separate into
two equations.
2x  4
2x  20 Solve each
equation.
x2
x  10
Since length cannot be negative, the border
should be 2 feet wide.
67. C; Solve the equation.
205  16t2
205
16

71.

72.

73.

74.

597, 97 6

8k2  22k  6  0
2(4k2  11k  3)  0
2(4k2  12k  k  3)  0
2[4k(k  3)  (k  3) ]  0
2(k  3) (4k  1)  0
or 4k  1  0
k30
k  3
4k  1
1

k4

53, 14 6

12w2  23w  5
12w  23w  5  0
12w2  20w  3w  5  0
4w(3w  5)  (3w  5)  0
(3w  5) (4w  1)  0
or 4w  1  0
3w  5  0
3w  5
4w  1
5

553, 14 6

w  4

6z2  7  17z
6z  17z  7  0
6z2  14z  3z  7  0
2z(3z  7)  (3z  7)  0
(3z  7) (2z  1)  0
3z  7  0 or 2z  1  0
3z  7
2z  1

512, 73 6

68. D; Since
represents the
 2ab 
principal square root of a2  2ab  b2, its value
cannot be negative. Thus, a  b  0 or a  b.

4
3

y  4  2x  2
y  2x  2  4
y  2x  2
2

76. The slope of y  3x  7 is 3. The slope of any


3
line perpendicular to it is 2.
y  y1  m(x  x1 )
3
y  7  2 [ x  (4) ]

3x  4  0
3x  4
x

z2

y  y1  m(x  x1 )
1
y  4  2 (x  1)

Maintain Your Skills

9x2  16  0
(3x  4)(3x  4)  0
or
3x  4  0
3x  4

z3

75. The slope of y  2x  1 is 2. The slope of any line


1
perpendicular to it is 2.

Exercises 6974 For checks, see students work.


69.
s2  25
2  25  0
s
(s  5)(s  5)  0
or s  5  0
s50
s  5
s5
{ 5, 5}

543, 43 6

m7

b2

x

m  7

w  3

t2

2a2

70.

7m  9  0
7m  9

12.8125  t2
112.8125  t
3.6  t
Ignore the negative solution; it will take about
3.6 seconds.

Page 514

49m2  81
49m  81  0
(7m  9)(7m  9)  0
or
7m  9  0
7m  9
2

y  7  2 (x  4)

4
3

y  7  2x  6
3

y  2x  6  7
3

y  2 x  13

431

Chapter 9

PQ249-6481F-09[422-437] 26/9/02 6:15 PM Page 432 Sahuja Ahuja_QXP_06:Desktop Folder:Chandra:Algebra_FNL_Delivery:

77. Let x represent the horizontal change.


slope 
1
2

21. 20n2  2  2  5  n  n
25np5  5  5  n  p  p  p  p  p
GCF: 5n
22. 60x2y2  2  2  3  5  x  x  y  y
35xz3  5  7  x  z  z  z
GCF: 5x
23. 13x  26y  13(x)  13(2y)
 13(x  2y)
2
2
24. 24a b  18ab  6ab(4ab)  6ab(3)
 6ab(4ab  3)
25. 26ab  18ac  32a2  2a(13b)  2a(9c)  2a(16a)

vertical change
horizontal change
1015
x

1(x)  2(1015)
x  2030
The horizontal change is 2030 feet.
78. 17
13
9
5




4
4
4

Add 4 three more times. The next three terms


are 1, 3, and 7.
79. 5 4.5
4 3.5

 2a(13b  9c  16a)




0.5
0.5
0.5

26. a  4ac  ab  4bc  (a2  4ac)  (ab  4bc)


 a(a  4c)  b(a  4c)
 (a  4c) (a  b)
27. 4rs  12ps  2mr  6mp
 2(2rs  6ps  mr  3mp)
 2[ (2rs  6ps)  (mr  3mp) ]
 2[2s(r  3p)  m(r  3p) ]
 2(r  3p)(2s  m)
28. 24am  9an  40bm  15bn
 (24am  9an)  (40bm  15bn)
 3a(8m  3n)  5b(8m  3n)
 (8m  3n)(3a  5b)
Exercises 2931 For checks, see students work.
29. x(2x  5)  0
x  0 or 2x  5  0
2x  5

Add 0.5 three more times. The next three terms


are 3, 2.5, and 2.
80. 45
54
63
72




9
9
9

Add 9 three more times. The next three terms are


81, 90 and 99.

Chapter 9 Study Guide and Review


Page 515
1.
2.
3.
4.
5.
6.
7.
8.
9.
10.

false;
true
false;
true
false;
false;
true
false;
true
true

Vocabulary and Concept Check


composite
sample answer: 64
24  3
difference of squares

50, 52 6

30. (3n  8)(2n  6)  0


3n  8  0
3n  8

sample answer: x2  2x  2

Pages 515518

Lesson-by-Lesson Review

11. 28  2  14
12. 33  3  11
 2  2  7 or 22  7
13. 150  2  75
14. 301  7  43
 2  3  25
 2  3  5  5 or 2  3  52
15. 83  1  83
16. 378  1  378
 1  2  189
 1  2  3  63
 1  2  3  3  21
 1  2  3  3  3  7
or 1  2  33  7
17. 35  5  7
18. 12  2  2  3
18  2  3  3
30  2  3  5
GCF: 5
40  2  2  2  5
GCF: 2
19. 12ab  2  2  3  a  b
4a2b2  2  2  a  a  b  b
GCF: 2  2  a  b or 4ab
20. 16mrt  2  2  2  2  m  r  t
30mr2  2  3  5  m  r  r
GCF  2mr
Chapter 9

x2

31.

8
3,

or

2n  6  0
2n  6

n  3

4x2  7x
 7x  0
x(4x  7)  0
x  0 or

n3

4x2

4x  7  0
4x  7
7

574, 06

x  4

32. Find a pair of factors of 12 whose sum is 7:


3 and 4.
y2  7y  12  (y  3)(y  4)
33. Find a pair of factors of 36 whose sum is 9:
12 and 3.
x2  9x  36  (x  12)(x  3)
34. Find a pair of factors of 6 whose sum is 5:
6 and 1.
b2  5b  6  (b  6)(b  1)
35. 18  9r  r2  r2  9r  18
Find a pair of factors of 18 whose sum is 9:
3 and 6.
18  9r  r2  (r  3)(r  6)

432

PQ249-6481F-09[422-437] 26/9/02 6:15 PM Page 433 Sahuja Ahuja_QXP_06:Desktop Folder:Chandra:Algebra_FNL_Delivery:

36. Find a pair of factors of 40x2 whose sum is 6x:


10x and 4x.
a2  6ax  40x2  (a  10x)(a  4x)
37. Find a pair of factors of 32n2 whose sum is 4n:
4n and 8n.
m2  4mn  32n2  (m  4n)(m  8n)
Exercises 3840 For checks, see students work.
38. y2  13y  40  0
( y  8)( y  5)  0
or y  5  0
y80
y  8
y  5
{8, 5}
39.
x2  5x  66  0
(x  11) (x  6)  0
or x  6  0
x  11  0
x  11
x  6
{6, 11}
40.
m2  m  12  0
(m  4)(m  3)  0
m  4  0 or m  3  0
m4
m  3
{3, 4}
41. There are no pairs of factors of 6 whose sum
is 9. 2a2  9a  3 is prime.
42. Find a pair of factors of 48 whose sum is 13:
3 and 16.

48.

49.

a3

52, 73 6

40x2  2x  24
40x  2x  24  0
2(20x2  x  12)  0
2(20x2  16x  15x  12)  0
2[4x(5x  4)  3(5x  4) ]  0
2(5x  4) (4x  3)  0
or 4x  3  0
5x  4  0
5x  4
4x  3
2

x  5

545, 34 6

x4

50. 2y3  128y  2y( y2  64)


 2y( y  8) ( y  8)
51. 9b2  20 is prime.
1

112n22  134r22
1
3
1
3
 1 2n  4r 21 2n  4r 2

52. 4n2  16r2 

Exercises 5355 For checks, see students work.


53.
b2  16  0
(b  4)(b  4)  0
b40
or b  4  0
b  4
b4
{4, 4}
54.
25  9y2  0
(5  3y)(5  3y)  0
5  3y  0
or 5  3y  0
3y  5
5  3y

2m2  13m  24  2m2  3m  16m  24


 (2m2  3m)  (16m  24)
 m(2m  3)  8(2m  3)
 (2m  3)(m  8)
43. Find a pair of factors of 100 whose sum is 20:
10 and 10.
25r2  20r  4  25r2  10r  10r  4
 (25r2  10r)  (10r  4)
 5r(5r  2)  2(5r  2)
 (5r  2)(5r  2)
44. There are no pairs of factors of 18 whose sum is 7.
6z2  7z  3 is prime.
45. Find a pair of factors of 72 whose sum is 17: 8 and 9.
12b2  17b  6  12b2  8b  9b  6
 (12b2  8b)  (9b  6)
 4b(3b  2)  3(3b  2)
 (3b  2)(4b  3)
46. 3n2  6n  45  3(n2  2n  15)
Now factor n2  2n  15. Find a pair of factors of
15 whose sum is 2: 5 and 3.
n2  2n  15  (n  5)(n  3)
Thus, 3n2  6n  45  3(n  5)(n  3).
Exercises 4749 For checks, see students work.
47.
2r2  3r  20  0
2  8r  5r  20  0
2r
2r(r  4)  5(r  4)  0
(r  4)(2r  5)  0
r  4  0 or 2r  5  0
r4
2r  5

552, 46

3a2  13a  14  0
3a  6a  7a  14  0
3a(a  2)  7(a  2)  0
(a  2) (3a  7)  0
a  2  0 or 3a  7  0
a2
3a  7
2

55.

5
3

y  3
5 5
3, 3

y

16a2  81  0
(4a  9)(4a  9)  0
or 4a  9  0
4a  9  0
4a  9
4a  9

a  4

594, 94 6

a4

56. a2  18a  81  a2  2(a) (9)  92


 (a  9) 2
2
57. 9k  12k  4  (3k) 2  2(3k) (2)  22
 (3k  2) 2
2
58. 4  28r  49r  22  2(2) (7r)  (7r) 2
 (2  7r) 2
2
59. 32n  80n  50  2(16n2  40n  25)
 2[ (4n) 2  2(4n) (5)  52 ]
 2(4n  5) 2

r  2

433

Chapter 9

PQ249-6481F-09[422-437] 26/9/02 6:15 PM Page 434 Sahuja Ahuja_QXP_06:Desktop Folder:Chandra:Algebra_FNL_Delivery:

10. 25y2  49w2  (5y) 2  (7w) 2


 (5y  7w) (5y  7w)
2
11. t  16t  64  t2  2(t) (8)  82
 (t  8) 2
2
12. x  14x  24
Find two numbers whose product is 24 and whose
sum is 14: 12 and 2.
x2  14x  24  (x  12)(x  2)
13. 28m2  18m  2m(14m  9)
14. a2  11ab  18b2
Find two numbers whose product is 18b2 and
whose sum is 11b: 2b and 9b.
a2  11ab  18b2  (a  2b)(a  9b)
15. 12x2  23x  24  12x2  9x  32x  24
 3x(4x  3)  8(4x  3)
 (4x  3) (3x  8)
16. 2h2  3h  18 is prime.
17. 6x3  15x2  9x  3x(2x2  5x  3)
 3x(2x2  6x  x  3)
 3x[ 2x(x  3)  (x  3) ]
 3x(x  3) (2x  1)
18. 64p2  63p  16 is prime.
19. 2d2  d  1  2d2  2d  d  1
 2d(d  1)  (d  1)
 (d  1) (2d  1)
20. 36a2b3  45ab4  9ab3(4a  5b)
21. 36m2  60mn  25n2  (6m) 2  2(6m) (5n)  (5n) 2

Exercises 6063 For checks, see students work.


60.
6b3  24b2  24b  0
6b(b2  4b  4)  0
2
6b(b  2(b) (2)  22 )  0
6b(b  2) 2  0
6b  0 or b  2  0
b0
b2
{0, 2}
61.
49m2  126m  81  0
(7m) 2  2(7m)(9)  92  0
(7m  9) 2  0
7m  9  0
7m  9
9

m7

597 6

62. (c  9) 2  144
c  9  1144
c  9  12
c  9  12
c  9  12 or c  9  12
c  21
c  3
{3, 21}
63. 144b2  36
36

b2  144
1

b2  4
1

b  2

512 6

 (6m  5n) 2

22. a2  4  a2  22
 (a  2)(a  2)
23. 4my  20m  3py  15p  4m( y  5)  3p( y  5)

Chapter 9 Practice Test

 (y  5)(4m  3p)
2

24. 15a b  5a  10a  5a(3ab  a  2)


25. 6y2  5y  6  6y2  9y  4y  6
 3y(2y  3)  2(2y  3)
 (2y  3) (3y  2)
26. 4s2  100t2  4(s2  25t2 )
 4(s  5t) (s  5t)
3  4x2  9x  36  x2 (x  4)  9(x  4)
27. x
 (x  4) (x2  9)
 (x  4) (x  3) (x  3)
28. area of shaded  area of large  area of small
region
rectangle
rectangle
 (x  6) (y  6)  xy
 xy  6x  6y  36  xy
 6x  6y  36
 6(x  y  6)
29. area of shaded  area of square  area of circles
region
 (4r) 2  4(r2 )
 16r2  4r2
 4r2 (4  )

Page 519
1. Sample answer: 7; Its only factors are 1 and itself.
2. Sample answer: n2  100
n2  100  n2  102
 (n  10)(n  10)
3. Check for a GCF other than 1 and factor it out.
4. 63  3  21
 3  3  7 or 32  7
5. 81  3  27
6. 210  1  210
 339
 1  2  105
 3  3  3  3 or 34
 1  2  3  35
 1  2  3  5  7
7. 48  2  2  2  2  3
64  2  2  2  2  2  2
GCF: 2  2  2  2 or 16
8. 28  2  2  7
75  3  5  5
GCF: 1; 28 and 75 are relatively prime.
9. 18a2b2  2  3  3  a  a  b  b
28a3b2  2  2  7  a  a  a  b  b
GCF: 2  a  a  b  b or 2a2b2

Chapter 9

434

PQ249-6481F-09[422-437] 26/9/02 6:15 PM Page 435 Sahuja Ahuja_QXP_06:Desktop Folder:Chandra:Algebra_FNL_Delivery:

38. Let x represent the amount of the increase. Write


an equation for the area of the new rectangle.
(4  x) (7  x)  28  26
28  4x  7x  x2  54
x2  11x  28  54
x2  11x  26  0
(x  13) (x  2)  0
or x  2  0
x  13  0
x  13
x2
Ignore the negative solution. Thus, 2 inches are
added to each dimension of the rectangle. The
new rectangle is 6 in. by 9 in.
39. Let x represent the width of the sidewalk. Write
an equation for the area of the remaining lawn.
(24  2x) (32  2x)  425
768  48x  64x  4x2  425
4x2  112x  768  425
4x2  112x  343  0
4x2  14x  98x  343  0
2x(2x  7)  49(2x  7)  0
(2x  7) (2x  49)  0
or 2x  49  0
2x  7  0
2x  7
2x  49

Exercises 3037 For checks, see students work.


30. (4x  3)(3x  2)  0
4x  3  0 or 3x  2  0
4x  3
3x  2
3

x4

523, 34 6

x  3

31. 18s2  72s  0


18s(s  4)  0
18s  0 or s  4  0
s0
s  4
{4, 0}
4x2  36
32.
2  36  0
4x
4(x2  9)  0
4(x  3)(x  3)  0
or x  3  0
x30
x  3
x3
{3, 3}
t2  25  10t
33.
2  10t  25  0
t
t2  2(t)(5)  52  0
(t  5) 2  0
t50
t5
{5}
34.
a2  9a  52  0
(a  13) (a  4)  0
or a  4  0
(a  13)  0
a  13
a  4
{4, 13}
35. x3  5x2  66x  0
x(x2  5x  66)  0
x(x  11)(x  6)  0
or x  6  0
x  0 or x  11  0
x  11
x  6
{6, 0, 11}

x  2 or 3.5

2x2  9x  5
2x  9x  5  0
2x2  10x  x  5  0
2x(x  5)  (x  5)  0
(x  5)(2x  1)  0
x  5  0 or 2x  1  0
x5
2x  1

37.

 98

1 2  2(98)

Chapter 9 Standardized Test Practice


Pages 520521

x  2

1. A; The line passes through the points (5, 0) and


(0, 3).
3  0
(5)

m0

3b2  6  11b
2  11b  6  0
3b
3b2  9b  2b  6  0
3b(b  3)  2(b  3)  0
(b  3)(3b  2)  0
b  3  0 or 3b  2  0
b3
3b  2

523, 36

1 2
x
2
1 2
x
2

x2  196
x  1196
x  14
Ignore the negative solution. Thus, the square is
14 meters by 14 meters.

or 24.5

x2  2x2  98

1
2,

49
2

The walk cannot be 24.5 feet wide since this is


wider than the original lawn. Thus, the sidewalk
is 3.5 feet wide.
40. Write an equation for the shaded part of the
square.

36.

x

or 5
3

The equation is y  5x  3.


2. D; Use (12, 176) and (18, 224) to find the slope
(price per ticket).
m


b3

224  176
18  12
48
or 8
6

Since the band initially had $80, the intercept


is 80. The equation is a  8t  80.

435

Chapter 9

PQ249-6481F-09[422-437] 26/9/02 6:15 PM Page 436 Sahuja Ahuja_QXP_06:Desktop Folder:Chandra:Algebra_FNL_Delivery:

Shade the half plane that contains (0, 0).

3. A; The slope of the boundary line is 3; this


eliminates answer choices C and D. Test (0, 0) in
answer choice B.

y  3x  1
0

1
(0)
3

xy3

1
O

0  1 false
Since (0, 0) is part of the shaded region but not a
solution of B, we can eliminate B. This leaves
only answer choice A.
4. D; The total was 258.
n  f  258
Twice as many bags were sold today as last
Friday.
n  2f
5. B; 5.387 103  0.005387
6. C;

16x8
8x4

12. Let a represent the number of adult tickets sold


and c represent the number of child tickets.
Write an equation for the total number of tickets.
a  c  145
Write an equation for the total receipts.
7.50a  4c  790
Solve the first equation for c.
a  c  145
c  145  a
Substitute 145  a for c in the second equation.
7.50a  4c  790
7.50a  4(145  a)  790
7.50a  580  4a  790
3.5a  580  790
3.5a  210
a  60
The theater sold 60 adult tickets.
13. 3x  y  8
4x  2y  14
Multiply the first equation by 2. Then add.
6x  2y  16
() 4x  2y  14
10x
 30
x3
Use 3x  y  8 to find y.
3x  y  8
3(3)  y  8
9y8
y  1
The solution is (3, 1).
14. (x  t)x  (x  t)y  (x  t) (x  y)
15. Let n be the first odd integer. Then n  2 is the
next odd integer.
n(n  2)  195
n2  2n  195
2  2n  195  0
n
(n  15)(n  13)  0
or n  13  0
n  15  0
n  15
n  13
When n  15, n  2  15  2 or 13.
When n  13, n  2  13  2 or 15. The integers
are 15 and 13 or 13 and 15.

1168 21xx 2
8
4

84

7.

 2x
 2x4
3x2  48  0
A;
3(x2  16)  0
3(x  4)(x  4)  0
or x  4  0
x40
x  4
x4
2  3x  8  6x  6
D;
x
x2  9x  14  0
(x  2)(x  7)  0
x  2  0 or x  7  0
x2
x7
B; Factor the expression for the area.
12x2  21x  6  3(4x2  7x  2)
 3(4x2  8x  x  2)
 3 [4x(x  2)  (x  2) ]
 3(x  2)(4x  1)
The width is 3x  6 or 3(x  2). Thus, the length
is 4x  1.
6 0x  2 0  18

8.

9.

10.

1
[ 6 0x
6

 2 0 ]  6 (18)
1

0x  2 0  3
x  2  3 or x  2  3
x5
x  1
Both 5 and 1 make the equation true.
11. x  y  3
Graph the boundary as a solid line since the
inequality includes equal. Test the point (0, 0).
xy3
003
0  3 true

Chapter 9

436

PQ249-6481F-09[422-437] 26/9/02 6:15 PM Page 437 Sahuja Ahuja_QXP_06:Desktop Folder:Chandra:Algebra_FNL_Delivery:

16.

21. B; 3x  y  5
x  3y  6

2x2  5x  12  0
2x  8x  3x  12  0
2x(x  4)  3(x  4)  0
(x  4)(2x  3)  0
x40
or 2x  3  0
x  4
2x  3
2

Solve the second equation for x.


x  3y  6
x  3y  6
Substitute 3y  6 for x in the first equation.
3x  y  5
3(3y  6)  y  5
9y  18  y  5
8y  18  5
8y  13

x2

54, 32 6

17. 2x2  7x  3  2x2  6x  x  3


 2x(x  3)  (x  3)
 (x  3)(2x  1)
18. C; 0x 0  0y 0  015 0  07 0
 15  7
8
0x  y 0  015  (7) 0
 015  7 0
 08 0
8
The two quantities are equal.

13

y  8

2a  3b  3
a  4b  24
Solve the second equation for a.
a  4b  24
a  24  4b
Substitute 24  4b for a in the first equation.
2a  3b  3
2(24  4b)  3b  3
48  8b  3b  3
48  11b  3
11b  51

19. B; 3x  27  39
2
x
3
3 2
x
2 3

 66

1 2  32 (66)

51

b  11

x  99

3
y
4

51

 55  20
3
y
4
4 3
y
3 4

 75

13

22. C; The GCF of 2x3, 6x2, and 8x is 2x.


The GCF of 18x3, 14x2, and 4x is 2x.
The two quantities are equal.
23a. Since the area is 28 square yards, the width is 3
yards less than the length, so the width is
L  3. Thus, L(L  3)  28.
23b. Write an equation for the area and solve for L.
L(L  3)  28
L2  3L  28  0
(L  7)(L  4)  0
L  7  0 or L  4  0
L7
L  4
Ignore the negative solution. The length is
7 yards.
23c. Find the perimeter. Since the length is 7 yd, the
width is 7  3 or 4 yd.
P  2(7)  2(4) or 22

y  100
The quantity in Column B is greater.
20. A; The line appears to pass through (0, 2) and
(2, 1).
1  (2)
2  0

51
11

Since 11 7  8 , the quantity in Column B is


greater.

1 2  43 (75)

m

or

or 2
2

A line perpendicular to it has slope 3.


y  y1  m(x  x1 )
2
y  (4)  3 (x  6)
2

y  4  3x  4
2

y  3x
This line passes through (0, 0). Its x-intercept
is 0. Since the x-intercept of the given line is
between 1 and 2, the quantity in Column A is
greater.

He will need 22 yd of fencing.

437

Chapter 9

PQ249-6481F-10[438-457] 26/9/02 6:18 PM Page 438 Sahuja Ahuja_QXP_06:Desktop Folder:Chandra:Algebra_FNL_Delivery:

Chapter 10
Page 523

Quadratic and Exponential Functions


6. Sample answer:

Getting Started

1. Sample answer:
x
6
4
2
0
2

y
1
1
3
5
7

x
5
0

y
yx5

y
0
2

5y  10  2x

7. Sample answer:
2. Sample answer:
x
0
1
2
3

x
6
4
2
0
2

y
3
1
1
3

y
0
1
2
3
4

x  2y  6

y  2x  3

8. Sample answer:
3. Sample answer:
x
4
2
0
2
4

y
1
0
1
2
3

x
1
1
3

y
y  0.5x  1

4. Sample answer:

49  72
14a  2(a) (7)
a2  14a  49  (a  7) 2
11. No; 81 is not a perfect square.
12. No; 12 is not a perfect square.
13. Yes;
9b2  (3b) 2

y  3x  2

5. Sample answer:
y
4
2
0

1  12
6b  2(3b) (1)
9b2  6b  1  (3b  1) 2
14. No; 6x2 is not a perfect square.
15. Yes;
4p2  (2p) 2

9  32
12p  2(2p) (3)
4p2  12p  9  (2p  3) 2

2x  3y  12

Chapter 10

t2  (t) 2

36  62
12  2(t) (6)
t2  12t  36  (t  6) 2
10. Yes;
a2  (a) 2

y
4
1
2

x
0
3
6

3x  2y  9

9. Yes;
x
2
1
0

y
6
3
0

438

PQ249-6481F-10[438-457] 26/9/02 6:18 PM Page 439 Sahuja Ahuja_QXP_06:Desktop Folder:Chandra:Algebra_FNL_Delivery:

16. Yes;

The lowest point on the graph is (3, 1). The


vertex is (3, 1).
2. The fold line is a vertical line on which each point
has x-coordinate 3. The equation is x  3.
3. The vertex, (3, 1), lies on the fold line.
4. Sample answer: A parabola is symmetrical. The
vertex of the parabola lies on the line that divides
the parabola into two matching halves.

16s2  (4s) 2
2

93
24s  2(4s)(3)
16s2  24s  9  (4s  3) 2
17. 5
9
13 17


4
4
4

Add 4 three more times. The next three terms are


21, 25, and 29.
18. 12
5 2 9


7
7
7

Page 528

Add 7 three more times. The next three terms


are 16, 23, and 30.
19. 4 1
2
5

Check for Understanding

1. Both types of parabolas are U shaped. A parabola


with a maximum opens downward, and its
corresponding equation has a negative coefficient
for the x2 term. A parabola with a minimum
opens upward, and its corresponding equation has
a positive coefficient for the x2 term.
2. Sample answer:


3
3
3

Add 3 three more times. The next three terms are


8, 11, and 14.
20. 24
32 40 48


8
8
8

Add 8 three more times. The next three terms are


56, 64, and 72.
21. 1 6 11 16


5
5
5

Add 5 three more times. The next three terms


are 21, 26, and 31.
22. 27 20 13 6


7
7
7

3. Sample answer: If you locate several points of the


graph on one side of the axis of symmetry, you
can locate the corresponding points on the other
side of the axis of symmetry to help graph the
equation.
4. y  x2  5
Sample answer:

Add 7 three more times. The next three terms are


1, 8, and 15.
23. 5.3
6.0 6.7 7.4


0.7 0.7 0.7

Add 0.7 three more times. The next three terms


are 8.1, 8.8, and 9.5.
24. 9.1 8.8 8.5 8.2


0.3 0.3 0.3

x
3
2
1
0
1
2
3

Add 0.3 three more times. The next three terms


are 7.9, 7.6, and 7.3.

10-1 Graphing Quadratic Functions


Page 525

y  x2  5

Algebra Activity

1. y  x2  6x  8
Sample answer:
x
5
4
3
2
1

y
4
1
4
5
4
1
4

y
3
0
1
0
3

5. y  x2  4x  5
Sample answer:
y

x
1
0
1
2
3
4
5

y  x 2  6x  8
O

439

y
0
5
8
9
8
5
0

y
y  x 2  4x  5

Chapter 10

PQ249-6481F-10[438-457] 26/9/02 6:18 PM Page 440 Sahuja Ahuja_QXP_06:Desktop Folder:Chandra:Algebra_FNL_Delivery:

The equation of the axis of symmetry is x  2.


y  x2  4x  3
y  (2) 2  4(2)  3
y  4  8  3
y1
The vertex is at (2, 1).
Since the coefficient of the x2 term is negative, the
vertex is a maximum.

6. In y  x2  4x  9, a  1 and b  4.
x
x

b
2a
4
2(1)

or 2

The equation of the axis of symmetry is x  2.


y  x2  4x  9
y  (2) 2  4(2)  9
y489
y  13
The vertex is at (2, 13).
Since the coefficient of the x2 term is positive, the
vertex is a minimum.
2

9. B; Since all four graphs have the same axis of


symmetry, x  0, you cannot eliminate any
graphs using the axis of symmetry. Since the
coefficient of the x2 term is negative, the graph
opens downward. Eliminate choices A and C.
Let x  0.

7. In y  x2  5x  6, a  1 and b  5.
b

x  2a

5
2(1)

or

5
2

y  2 x2  1

or 2.5

y  2 (0) 2  1 or 1

The equation of the axis of symmetry is x  2.5.


y  x2  5x  6
y  (2.5) 2  5(2.5)  6
y  6.25  12.5  6
y  12.25
The vertex is at (2.5, 12.25).
Since the coefficient of the x2 term is negative, the
vertex is a maximum.
14
12
10
8
6
4
2
21
2

O
x
654321
2 1 2
4
6
8
10
12
14
y  x 2  4x  9

x

y  (x  2)2  1

The graph passes through the point (0, 1).


Eliminate choice D. The answer is B.

Pages 528530

Practice and Apply

10. y  x2  3
Sample answer:
x
2
1
0
1
2

y  x 2  5x  6

y
1
2
3
2
1

O
1 2 3 4 5 6x

y  x2  3

11. y  x2  7
Sample answer:

8. y  (x  2) 2  1
 (x2  4x  4)  1
 x2  4x  4  1
 x2  4x  3
In y  x2  4x  3, a  1 and b  4.

x
2
1
0
1
2

x  2a
4

x  2(1) or 2

y
3
6
7
6
3

y  x 2  7

Chapter 10

440

PQ249-6481F-10[438-457] 26/9/02 6:18 PM Page 441 Sahuja Ahuja_QXP_06:Desktop Folder:Chandra:Algebra_FNL_Delivery:

12. y  x2  2x  8
Sample answer:
x
1
0
1
2
3

17. In y  4x2  5x  16, a  4 and b  5.


b

x  2a
y

y
5
8
9
8
5

(5)

x  8 is the equation of the axis of symmetry.


18. In y  4x2, a  4 and b  0.
b

x  2a
0

x  2(4) or 0
x  0 is the equation of the axis of symmetry.
y  4x2
y  4(0) 2
y0
The vertex is at (0, 0).
Since the coefficient of the x2 term is positive, the
vertex is a minimum.

y  x 2  2x  8

13. y  x2  4x  3
Sample answer:
x
0
1
2
3
4

5
8

x   2(4) or

y
3
0
1
0
3

y
x

y  x 2  4x  3
y  4x 2

14. y  3x2  6x  4
Sample answer:
x
3
2
1
0
1

y
5
4
7
4
5

y  3x 2  6x  4

19. In y  2x2, a  2 and b  0.


b

x  2a
0

x  2(2) or 0

x  0 is the equation of the axis of symmetry.


y  2x2
y  2(0) 2
y0
The vertex is at (0, 0).
Since the coefficient of the x2 term is negative, the
vertex is a maximum.

15. y  3x2  6x  1
Sample answer:
x
1
0
1
2
3

y
8
1
4
1
8

y
O

x
y  2x
O

y  3x 2  6x  1

16. In y  3x2  2x  5, a  3 and b  2.


b

x  2a
2

x  2(3) or

1
3

x  3 is the equation of the axis of symmetry.

441

Chapter 10

PQ249-6481F-10[438-457] 26/9/02 6:18 PM Page 442 Sahuja Ahuja_QXP_06:Desktop Folder:Chandra:Algebra_FNL_Delivery:

20. In y  x2  2, a  1 and b  0.
x
x

b
2a
0
2(1)

y
y  x 2  2x  3

or 0

x  0 is the equation of the axis of symmetry.


y  x2  2
y  (0) 2  2
y2
The vertex is at (0, 2).
Since the coefficient of the x2 term is positive, the
vertex is a minimum.

23. In y  x2  6x  15, a  1 and b  6.


b

x  2a

(6)

x  2(1) or 3
x  3 is the equation of the axis of symmetry.
y  x2  6x  15
y  (3) 2  6(3)  15
y  9  18  15
y  24
The vertex is at (3, 24).
Since the coefficient of the x2 term is negative, the
vertex is a maximum.

y  x2 2
x

21. In y  x2  5, a  1 and b  0.


b

x  2a

y  x 2  6x  15
0

x  2(1) or 0
x  0 is the equation of the axis of symmetry.
y  x2  5
y  (0) 2  5
y5
The vertex is at (0, 5).
Since the coefficient of the x2 term is negative, the
vertex is a maximum.
y  x 2  5

12 8

4 4

O
4x

x  2a
x

(14)
2(1)

or 7

x  7 is the equation of the axis of symmetry.


y  x2  14x  13
y  (7) 2  14(7)  13
y  49  98  13
y  36
The vertex is at (7, 36).
Since the coefficient of the x2 term is positive, the
vertex is a minimum.

22. In y  x2  2x  3, a  1 and b  2.


b

x  2a

y
5

x  2(1) or 1

O
25
10
15
20
25
30
35

x  1 is the equation of the axis of symmetry.


y  x2  2x  3
y  (1) 2  2(1)  3
y  1  2  3
y4
The vertex is at (1, 4).
Since the coefficient of the x2 term is negative, the
vertex is a maximum.

Chapter 10

28
24
20
16
12
8
4

24. In y  x2  14x  13, a  1 and b  14.

442

2 4 6 8 10 12 14 x

y  x 2  14x  13

PQ249-6481F-10[438-457] 26/9/02 6:19 PM Page 443 Sahuja Ahuja_QXP_06:Desktop Folder:Chandra:Algebra_FNL_Delivery:

25. In y  x2  2x  18, a  1 and b  2.


x
x

b
2a
2
2(1)

Since the coefficient of the x2 term is positive, the


vertex is a minimum.
y

or 1

x  1 is the equation of the axis of symmetry.


y  x2  2x  18
y  (1) 2  2(1)  18
y  1  2  18
y  17
The vertex is at (1, 17).
Since the coefficient of the x2 term is positive, the
vertex is a minimum.

y  x 2  2x  18

35
30
25
20
15
10
5

654321
5

y  3x 2  6x  4

28. In y  5  16x  2x2, a  2 and b  16.

x  2a
16

x  2(2) or 4
x  4 is the equation of the axis of symmetry.
y  5  16x  2x2
y  5  16(4)  2(4) 2
y  5  64  32
y  37
The vertex is at (4, 37).
Since the coefficient of the x2 term is negative, the
vertex is a maximum.

O
1 2x

26. In y  2x2  12x 11, a  2 and b  12.


b

x  2a
12

x  2(2) or 3

35
30
25
20
15
10
5

x  3 is the equation of the axis of symmetry.


y  2x2  12x  11
y  2(3) 2  12(3)  11
y  18  36  11
y  29
The vertex is at (3, 29).
Since the coefficient of the x2 term is positive, the
vertex is a minimum.

1
5

y  5  16x  2x 2

O
1 2 3 4 5 6 7 8 9x

29. In y  9  8x  2x2, a  2 and b  8.

y
3
O
x
7654321
3 1
6
9
12
15
18
21
24
27
30
2
y  2x  12x  11

x  2a
(8)

x   2(2) or 2
x  2 is the equation of the axis of symmetry.
y  9  8x  2x2
y  9  8(2)  2(22 )
y  9  16  8
y1
The vertex is at (2, 1).
Since the coefficient of the x2 term is positive, the
vertex is a minimum.
y

27. In y  3x2  6x  4, a  3 and b  6.


b

x  2a
(6)

x   2(3) or 1
x  1 is the equation of the axis of symmetry.
y  3x2  6x  4
y  3(1) 2  6(1)  4
y364
y1
The vertex is at (1, 1).

y  9  8x  2x2
O

443

Chapter 10

PQ249-6481F-10[438-457] 26/9/02 6:19 PM Page 444 Sahuja Ahuja_QXP_06:Desktop Folder:Chandra:Algebra_FNL_Delivery:

30. y  3(x  1) 2  20
 3(x2  2x  1)  20
 3x2  6x  3  20
 3x2  6x  17
In y  3x2  6x  17, a  3 and b  6.
x
x

b
2a
6
2(3)

32.

y  2  x2  10x  25
y  2  2  x2  10x  25  2
y  x2  10x  23
2
In y  x  10x  23, a  1 and b  10.
b

x  2a
x

or 1

(10)
2(1)

or 5

x  5 is the equation of the axis of symmetry.


y  x2  10x  23
y  52  10(5)  23
y  25  50  23
y  2
The vertex is at (5, 2).
Since the coefficient of x2 is positive, the vertex is
a minimum.

x  1 is the equation of the axis of symmetry.


y  3x2  6x  17
y  3(1) 2  6(1)  17
y  3  6  17
y  20
The vertex is at (1, 20).
Since the coefficient of the x2 term is positive, the
vertex is a minimum.

y
4
O
x
654321
4 1 2
8
12
16
20
y  3(x  1)2 20 24
28

y  2  x 2  10x  25

33.

31. y  2(x  4) 2  3
y  2(x2  8x  16)  3
y  2x2  16x  32  3
y  2x2  16x  35
In y  2x2  16x  35, a  2 and b  16.

y  1  3x2  12x  12
y  1  1  3x2  12x  12  1
y  3x2  12x  11
In y  3x2  12x  11, a  3 and b  12.
b

x  2a
12

x  2(3) or 2

x  2a

x  2 is the equation of the axis of symmetry.


y  3x2  12x  11
y  3(2) 2  12(2)  11
y  12  24  11
y  1
The vertex is at (2, 1).
Since the coefficient of the x2 term is positive, the
vertex is a minimum.

16

x  2(2) or 4
x  4 is the equation of the axis of symmetry.
y  2x2  16x  35
y  2(4) 2  16(4)  35
y  32  64  35
y  3
The vertex is at (4, 3).
Since the coefficient of the x2 term is negative, the
vertex is a maximum.

y
x

O
2

y  2(x  4)  3
O

y  1  3x 2 12x  12

Chapter 10

444

PQ249-6481F-10[438-457] 26/9/02 6:19 PM Page 445 Sahuja Ahuja_QXP_06:Desktop Folder:Chandra:Algebra_FNL_Delivery:

Since the coefficient of the x2 term is positive, the


vertex is a minimum.

y  5  3 (x  2) 2

34.

y  5  5  3 (x2  4x  4)  5
1

19
,
3

19
3
1
3

y  3x2  3x  3  5
y  3x2  3x 
1

In y  3x2  3x 

and b  3.

x  2a
x

4
3
 1
23

12

or

4
3

3
2

or 2

y  1  23 (x  1)2

36. If the vertex of the parabola is (4, 3), the axis


of symmetry is x  4. The point (11, 0) is
7 units from the axis of symmetry. The other
x-intercept must be (3, 0), which is 7 units on the
other side of the axis of symmetry.
37. The equation of the axis of symmetry is:

x  2 is the equation of the axis of symmetry.


y
y
y
y

1 2
4
19
x  3x  3
3
1
4
(2) 2  3 (2)
3
4
8
19
3 3
3
15
or 5
3

19
3

6  4
2
2
2

x

The vertex is at (2, 5).


Since the coefficient of the x2 term is positive, the
vertex is a minimum.

x

x  1
38. 2 m; parabolas are symmetric.

39. In h  16t2  32t  3, a  16 and b  32.


b

t  2a
32

t  2(16)

y  5  13 (x  2)2

t  1 is the equation of the axis of symmetry


h  16(1) 2  32  3
h  19
The vertex is at (1, 19).
The maximum height of the weight is 19 feet.
40. No, the prize will not be won since the maximum
height of 19 feet is less than the 20 feet required
to win.
41. A  x(20  x)
A  20x  x2
A  x2  20x
42. In A  x2  20x, a  1 and b  20.

y  1  3 (x  1) 2

35.

y  1  1  3 (x2  2x  1)  1
2

y  3x2  3x  3  1
y  3x2  3x  3
2

In y  3x2  3x  3, a  3 and b  3.
b

x  2a
x

4
3
 2
23

12

x  2a
4

 3  4 or 1

20

x  2(1) or 10
The x-coordinate of the vertex is 10. Since the
coefficient of the x2 term is negative, the vertex is
a maximum. The greatest area will result when
x is 10 m.
43. A  x2  20x
A  102  20(10)
A  100  200
A  100
The second coordinate of the vertex, which is a
maximum, is 100. Thus, the maximum possible
area for the pen is 100 m2.
44. In h  0.00635x2  4.0005x  0.07875,

x  1 is the equation of the axis of symmetry.


2

y  3x2  3x  3
2

y  3 (1) 2  3 (1)  3
2

y333
3

y  3 or 1
The vertex is at (1, 1).

a  0.00635 and b  4.0005.


b

x   2a
4.0005

x   2 (0.00635 ) or 315
x  315 is the equation of the axis of symmetry.

445

Chapter 10

PQ249-6481F-10[438-457] 26/9/02 6:19 PM Page 446 Sahuja Ahuja_QXP_06:Desktop Folder:Chandra:Algebra_FNL_Delivery:

51. In order to coordinate a firework with recorded


music, you must know when and how high it will
explode. Answers should include following.
The rocket will explode when it reaches the
39.2
vertex or when t  2(4.9) , which is
4 seconds.
The height of the rocket when it explodes is
the height when t  4. Therefore,
h  4.9(42)  39.2(4)  1.6 or 80 meters.
52. A; Since the parabola opens upward, the
coefficient of the x2 term is positive. Eliminate
choices B and D. The y-intercept of choice C is
y  02  4(0)  5 or 5. The y-intercept of the
graph is 5. Eliminate choice C. The answer is A.
53. D; y  x  5 is not a quadratic equation because
there is no x2 term.
54. y  x2  10x  25
Step 1 Graph the equation on the screen.

45. The ends of the arch are the x-intercepts of the


graph of the equation. Since the axis of symmetry
is an equal distance from both x-intercepts, the
axis is 315 feet from each end. Thus, the ends are
315  315 or 630 feet apart.
46. To find the maximum height of the arch, find the
second coordinate of the vertex by substituting
315 for x in the equation for the height.
h  0.00635x2  4.0005x  0.07875
h  0.00635(315) 2  4.0005(315)  0.07875
h  630
The maximum height of the arch is 630 ft.
47. To avoid confusion, we replace a with A in the
equation given.
In A  0.003x2  0.115x  21.3, a  0.003 and
b  0.115.
b

x  2a
(0.115)

x   2(0.003) or about 19

KEYSTROKES:

The x-coordinate of the vertex is about 19. Since


the coefficient of the x2 term is positive, the
vertex is a minimum. The minimum age occurs
about 19 years after 1940 or in 1959.
48. Substitute 19 for x in the equation.
A  0.003x2  0.115x  21.3
A  0.003(19) 2  0.115(19)  21.3
A  20
In 1959, the average age of brides was about
20 years old.
49. Step 1 Graph the equation on the screen.

X,T,,n

X,T,,n

10

25 GRAPH

Step 2

Approximate the minimum.


KEYSTROKES: 2nd [CALC] 3
Use the
and
keys to set the left and right
bounds and to guess the minimum.
The vertex is a minimum with ordered pair (5, 0).

0.003 X,T,,n

KEYSTROKES:

0.115 X,T,,n
21.3 GRAPH
Step 2 Approximate the minimum.
KEYSTROKES: 2nd [CALC] 3

55. y  x2  4x  3
Step 1 Graph the equation on the screen.

Use the
and
keys to set the left and right
bounds and to guess the minimum.
The minimum is about (19.2, 20.2).

KEYSTROKES:

X,T,,n

( )

X,T,,n

3 GRAPH

Step 2

Approximate the maximum.


KEYSTROKES: 2nd [CALC] 4
Use the
and
keys to set the left and right
bounds and to guess the maximum.
The vertex is a maximum with ordered pair (2, 7).
50. Sample answer: y  4x2  3x  5.
A quadratic equation y  ax2  bx  c with
3
x  8 as its axis of symmetry must satisfy
b

2a  8.
3

2a  8 or 2(4)
Choose b  3 and a  4. Then c can be any
number, such as 5.

Chapter 10

446

PQ249-6481F-10[438-457] 26/9/02 6:19 PM Page 447 Sahuja Ahuja_QXP_06:Desktop Folder:Chandra:Algebra_FNL_Delivery:

56. y  2x2  8x  1
Step 1 Graph the equation on the screen.
KEYSTROKES:

X,T,,n
Step 2

59. y  0.5x2  2x  3
Step 1 Graph the equation on the screen.

( ) 2 X,T,,n
1 GRAPH

KEYSTROKES:

X,T,,n

Approximate the maximum.


KEYSTROKES: 2nd [CALC] 4

2nd [CALC] 4

Use the
and
keys to set the left and right
bounds and to guess the maximum.
The vertex is a maximum with ordered pair (2, 5).

Use the
and
keys to set the left and right
bounds and to guess the maximum.
The vertex is a maximum with ordered pair (2, 7).

57. y  2x2  40x  214


Step 1 Graph the equation on the screen.
KEYSTROKES:

2 X,T,,n

Page 530

40

214 GRAPH
Step 2 Approximate the minimum.
KEYSTROKES: 2nd [CALC] 3
Use the
and
keys to sset the left and
right bounds and to guess the minimum.
The vertex is a minimum with ordered pair (10, 14).

65. 1  16g2  12  (4g) 2


 (1  4g) (1  4g)
66. (13x  9y)  11y  13x  (9y  11y)
 13x  20y
67. (7p2  p  7)  (p2  11)
 (7p2  p  7)  (p2  11)
 (7p2  p2 )  p  (7  11)
 6p2  p  18
68. Let m represent the cost for a member and n the
cost for a nonmember. Solve the following system
of equations.
3m  3n  180
5m  3n  210
Multiply the first equation by 1. Then add.
3m  3n  180
(  ) 5m  3n  210
2m
 30
m  15
Substitute 15 for m in the first equation and solve
for n.
3m  3n  180
3(15)  3n  180
45  3n  180
3n  135
n  45
An aerobics class costs $15 for members and $45
for nonmembers.

58. y  0.25x2  4x  2
Step 1 Graph the equation on the screen.
0.25 X,T,,n

Maintain Your Skills

60. There are no factors of 9 whose sum is 6. Thus,


x2  6x  9 is prime.
61. a2  22a  121  a2  2(a) (11)  112
 (a  11) 2
2
62. 4m  4m  1  (2m) 2  2(2m) (1)  12
 (2m  1) 2
2
2
63. 4q  9  (2q)  32
 (2q  3) (2q  3)
2
64. 2a  25  2a2  0a  25
There are no factors of 25 whose sum is 0. Thus,
2a2  25 is prime.

X,T,,n

KEYSTROKES:

Step 2

Approximate the maximum.

KEYSTROKES:

( ) 0.5 X,T,,n
3 GRAPH

X,T,,n

2 GRAPH
Step 2 Approximate the minimum.
KEYSTROKES: 2nd [CALC] 3
Use the
and
keys to set the left and right
bounds and to guess the minimum.
The vertex is a minimum with ordered pair
(8, 18).

447

Chapter 10

PQ249-6481F-10[438-457] 26/9/02 6:19 PM Page 448 Sahuja Ahuja_QXP_06:Desktop Folder:Chandra:Algebra_FNL_Delivery:

Exercises 6971 For checks, see students work.


69. 12b 7 144
70. 5w 7 125
12b
12

144
12

5w
5

b 7 12
{b|b 7 12}
71.

r

125
5

72. y  y  m(x  x )
1
1
y  13  4(x  2)

12

y  13  4x  8

8
9
8
9

5r|r  6

y  4x  5

y  y1  m(x  x1 )
y  (7)  0[x  (2) ]
y70
y  7
74. y  y  m(x  x )
1
1
73.

Graphing Calculator Investigation


(Follow-Up of Lesson 10-1)

1. y  x2
y  3x2
y  6x2

w 6 25
{w|w 6 25}

3r
2
3
4
4 3r
4 2
33
3 4

1 2

Page 532

All the graphs open downward from the origin.


y  3x2 is narrower than y  x2, and y  6x2
is the narrowest.
2. y  x2
y  0.6x2
y  0.4x2

y  6  2 [x  (4) ]
3

y  6  2x  6
3

y  2x  12
75. To find the x-intercept, let y  0.
3x  4y  24
3x  4(0)  24
3x  24
x8
76. To find the x-intercept, let y  0.
2x  5y  14
2x  5(0)  14
2x  14
x7
77. To find the x-intercept, let y  0.
2x  4y  7
2x  4(0)  7
2x  7

All the graphs open downward from the origin.


y  0.6x2 is wider than y  x2, and y  0.4x2
is the widest.
3. y  x2
y  (x  5) 2
y  (x  4) 2

x  2 or 3.5
78. To find the x-intercept, let y  0.
7y  6x  42
7(0)  6x  42
6x  42
x7
79. To find the x-intercept, let y  0.
2y  4x  10
2(0)  4x  10
4x  10
10

All the graphs open downward, have the same


shape, and have vertices along the x-axis.
However, each vertex is different.
4. y  x2
y  x2  7
y  x2  5

x   4 or 2 or 2.5
80. To find the x-intercept, let y  0.
3x  7y  9  0
3x  7(0)  9  0
3x  9  0
3x  9
x  3

Chapter 10

All the graphs open downward, have the same


shape, and have vertices along the y-axis.
However, each vertex is different.

448

PQ249-6481F-10[438-457] 26/9/02 6:19 PM Page 449 Sahuja Ahuja_QXP_06:Desktop Folder:Chandra:Algebra_FNL_Delivery:

5. y  0.1x2
Sample answer: The graph will have a vertex at
the origin, open downward, and be wider than
y  x2.
x
2
1
0
1
2

y
0.4
0.1
0
0.1
0.4

9. If |a| 7 1, the graph is narrower than the graph


of y  x2. If 0 6 |a| 6 1, the graph is wider than
the graph of y  x2. If a  0, it opens downward.
If a  0, it opens upward.
10. The graph has the same shape as y  x2, but is
shifted h units (left if h  0, right if h  0).
11. The graph has the same shape as y  x2, but is
shifted k units (up if k  0, down if k  0).
12. The graph has the same shape as y  x2, but is
shifted h units left or right and k units up or
down as prescribed in Exercises 10 and 11.

y
x

O
y   0.1 x

6. y  (x 
Sample answer: The graph will open upward and
have the same shape as y  x2, but the vertex
will be at (1, 0).
x
3
2
1
0
1

Pages 535536

7x2  2x  8
7x  2x  8  8  8
7x2  2x  8  0
Replace zero with f(x).
f(x)  7x2  2x  8
3. Sample answer:
2

y  (x  1)2
x

7. y  4x2
Sample answer: The graph will open upward,
have a vertex at the origin, and be narrower than
y  x2.
x
2
1
0
1
2

y
16
4
0
4
16

The graph has only one x-intercept. Thus, the


related quadratic equation has only one distinct
solution.
4. Graph f(x)  x2  7x  6.

y  4x 2
x

Sample answer:

8. y  x2  6

x
1
2
3
4
5
6

Sample answer: The graph will open upward and


have the same shape as y  x2, but its vertex will
be at (0, 6).
y
2
5
6
5
2

x
2
1
0
1
2

Check for Understanding

1. The x-intercepts of the graph are 3 and 1.


Thus, the roots of the equation are 3 and 1.
2. First rewrite the equation so one side is equal to
zero.

y
4
1
0
1
4

Solving Quadratic Equations


by Graphing

10-2

1) 2

f(x)
0
4
6
6
4
0

f (x )
x

f (x)  x 2  7x  6

The x-intercepts of the graph are 1 and 6. Thus,


the solutions of the equation are 1 and 6.

y  x2 6

449

Chapter 10

PQ249-6481F-10[438-457] 26/9/02 6:19 PM Page 450 Sahuja Ahuja_QXP_06:Desktop Folder:Chandra:Algebra_FNL_Delivery:

5. Graph f(a)  a2  10a  25.


Sample answer:
a
3
4
5
6
7

9. First rewrite the equation so that one side is


equal to zero.
w2  3w  5
2  3w  5  5  5
w
w2  3w  5  0
Graph f(w)  w2  3w  5.
Sample answer:

f (a )

f(a)
4
1
0
1
4

w
2
1
1
2
4
5

a
O

f (a)  a 2  10a  25

The graph has one a-intercept, 5. Thus, the


solution of the equation is 5.
6. Graph f(c)  c2  3.
Sample answer:
c
2
1
0
1
2

The w-intercepts of the graph are between


2 and 1 and between 4 and 5. So one root is
between 2 and 1, and the other root is
between 4 and 5.
10. Let m and n represent the two numbers.
The sum of the numbers is 4.
mn4
The product of the numbers is 12.
mn  12
Solve the first equation for m.
mn4
m4n
Replace m with 4  n in the second equation.
mn  12
(4  n)n  12
4n  n2  12
2
n  4n  12  0

f (c)  c 2  3
c

7. Graph f(t)  t2  9t  5.
Sample answer:
f (t )  t 2  9t  5

6
4
2

98 7654 321


2
4
6
8
10
12
14
16

f (t )

Graph y  x2  4x  12.

The t-intercepts of the graph are between 9 and


8 and between 1 and 0. So one root is between
9 and 8, and the other is between 1 and 0.
8. Graph f(x)  x2  16.
Sample answer:
x
4
2
0
2
4

f(x)
0
12
16
12
0

[10, 10] scl: 1 by [10, 20] scl: 2

The x-intercepts are 2 and 6.


Let n  2. Then m  4  (2) or 6.
Let n  6. Then m  4  6 or 2.
The numbers are 2 and 6.

f (x )
O
1 2 3 4x

4 321
2
4
f (x)6
 x 2  16
8
10
12
14
16

The x-intercepts of the graph are 4 and 4. Thus,


the roots of the equation are 4 and 4.
Chapter 10

f (w)  w 2  3w  5

The graph has no c-intercept. Thus, the equation


has no real number solutions: .

f(t)
5
3
15
15
3
5

f (w )

f (c )

f(c)
7
4
3
4
7
O

t
9
8
5
4
1
0

f(w)
5
1
7
7
1
5

450

PQ249-6481F-10[438-457] 26/9/02 6:20 PM Page 451 Sahuja Ahuja_QXP_06:Desktop Folder:Chandra:Algebra_FNL_Delivery:

Pages 536538

15. Graph f(x)  x2  2x  5  0.


Sample answer:

Practice and Apply


2

11. Graph f(c)  c  5c  24.


Sample answer:
c
3
0
3
6
7
8

x
3
2
1
0
1

f (c )
5
O
4 2
2 4 6 8 10 12 c
5
10
15
20
25
30
2
35 f (c)  c  5c  24

f(c)
0
24
30
18
10
0

f(n)
22
9
6
13
30

35
30
25
20
15
10
5
2 1
5

r
6
4
2
0
2

1 2 3 4 5 6n

f (r )

7654 321 O 1 2 3 r
2
4
6
8
10
12
14
16
f (r)  r 2  4r  12
18

The r-intercepts of the graph are 6 and 2. Thus,


the solutions of the equation are 6 and 2.

13. Graph f(x)  x  6x  9.


Sample answer:

17.

f (x )

f(x)
4
1
0
1
4

(2, 0)

(6, 0)
(4, 2)

f (x )  x 2  6x  9

18.

The graph has one x-intercept, 3. Thus, the


solution of the equation is 3.
14. Graph f(b)  b2  12b  36.
Sample answer:
b
4
5
6
7
8

f(r)
0
12
16
12
0

f (n)  5n 2  2n  6

The graph has no n-intercepts. Thus, the equation


has no real number solutions: .

x
1
2
3
4
5

The graph has no x-intercept. Thus, the equation


has no real number solutions: .
16. Graph f(r)  r2  4r  12.
Sample answer:

f (n )

f (x)  x 2  2x  5

The c-intercepts of the graph are 3 and 8. Thus,


the solutions of the equation are 3 and 8.
12. Graph f(n)  5n2  2n  6.
Sample answer:
n
2
1
0
1
2

f (x )

f(x)
8
5
4
5
8

(3, 4)

f (b )

f(b)
4
1
0
1
4

(6, 0)
O

f (b )  b 2  12b  36

(0, 0)
x

The graph has one b-intercept, 6. Thus, the


solution of the equation is 6.

451

Chapter 10

PQ249-6481F-10[438-457] 26/9/02 6:20 PM Page 452 Sahuja Ahuja_QXP_06:Desktop Folder:Chandra:Algebra_FNL_Delivery:

21. Graph f(a)  a2  12.


Sample answer:

19. Let m and n represent the numbers.


mn9
mn  20
Solve the first equation for m.
mn9
m9n
Substitute 9  n for m in the second equation.
mn  20
(9  n)n  20
9n  n2  20
2  9n  20  0
n
Graph y  x2  9x  20.

a
4
3
2
0
2
3
4

f(a)
4
3
8
12
8
3
4

f (a )
4

2

4a

O
4
8

12
f (a)  a 2  12

The a-intercepts are between 4 and 3 and


between 3 and 4. So, one root is between 4 and
3, and the other root is between 3 and 4.
22. Graph f(n)  n2  7.
Sample answer:
n
3
2
1
0
1
2
3

[2, 8] scl: 1 by [2, 2] scl: 1

The x-intercepts are 4 and 5.


Let n  4. Then m  9  4 or 5.
Let n  5. Then m  9  5 or 4.
The numbers are 4 and 5.
20. Let m and n represent the numbers.
mn5
mn  24
Solve the first equation for m.
mn5
m5n
Substitute 5  n for m in the second equation.
mn  24
(5  n)n  24
5n  n2  24
2
n  5n  24  0
Graph y  x2  5x  24.

f (n )

f(n)
2
3
6
7
6
3
2

f (n)  n 2  7

The n-intercepts of the graph are between 3 and


2 and between 2 and 3. So, one root is between
3 and 2, and the other root is between 2 and 3.
23. Graph f(c)  2c2  20c  32.
Sample answer:
c
8
6
4
2

f(c)
0
16
16
0

3
12 8

4

3
6
9
12
15
18
f (c)  2c 2  20c  3221

1 f (c )
O
c

The c-intercepts of the graph are 8 and 2. So,


the roots of the equation are 8 and 2.

[10, 10] scl: 1 by [10, 40] scl: 10

24. Graph f(s)  3s2  9s  12.


Sample answer:

The x-intercepts are 3 and 8.


Let n  3. Then m  5  (3) or 8.
Let n  8. Then m  5  8 or 3.
The numbers are 3 and 8.

s
4
3
2
1
0
1

f(s)
0
12
18
18
12
0

3
6

4

f (s )
O

2

3
6
9
12
15
18
f (s)  3s2  9s  1221

The s-intercepts of the graph are 4 and 1.


So, the roots of the equation are 4 and 1.

Chapter 10

452

2s

PQ249-6481F-10[438-457] 26/9/02 6:20 PM Page 453 Sahuja Ahuja_QXP_06:Desktop Folder:Chandra:Algebra_FNL_Delivery:

25. Graph f(x)  x2  6x  6.


Sample answer:
x
5
4
3
2
1

28. Rewrite the equation.


x2  6x  7
2
x  6x  7  7  7
x2  6x  7  0
Graph f(x)  x2  6x  7.
Sample answer:

f (x )

f(x)
1
2
3
2
1

x
5
4
3
2
1

f (x)  x 2  6x  6

The x-intercepts of the graph are between 5


and 4 and between 2 and 1. So, one root is
between 5 and 4, and the other root is
between 2 and 1.
26. Graph f(y)  y2  4y  1.
Sample answer:
y
0
1
2
3
4

f(y)
1
2
3
2
1

f (x )

f(x)
2
1
2
1
2

f (x)  x 2  6x  7

The x-intercepts are between 5 and 4 and


between 2 and 1. So, one root of the equation
is between 5 and 4, and the other root is
between 2 and 1.
29. Rewrite the equation.

f (y )

m2  10m  21
m  10m  21  21  21
m2  10m  21  0
Graph f(m)  m2  10m  21.
Sample answer:
2

f (y)  y 2  4y  1

m
3
4
5
6
7

The y-intercepts of the graph are between 0 and


1 and between 3 and 4. So, one root is between
0 and 1, and the other root is between 3 and 4.
27. Rewrite the equation.
a2  8a  4
a  8a  4  4  4
a2  8a  4  0

f(m)
0
3
4
3
0

f (m )

f (m)  m 2  10m  21

Graph f(a)  a2  8a  4.
Sample answer:
a
1
0
2
4
6
8
9

f(a)
5
4
16
20
16
4
5

The m-intercepts of the graph are 3 and 7.


So, the roots of the equation are 3 and 7.
30. Rewrite the equation.
p2  16  8p
2  16  8p  8p  8p
p
p2  8p  16  0
Graph f(p)  p2  8p  16.
Sample answer:

( )
3 f a
O
42
2 4 6 8 10 12a
3
6
9
12
15
18
21
f (a)  a 2  8a  4

p
2
3
4
5
6

The a-intercepts of the graph are between 1 and


0 and between 8 and 9. So, one root of the
equation is between 1 and 0, and the other root
is between 8 and 9.

f(p)
4
1
0
1
4

f (p )

f (p)  p 2  8p  16

The graph has only one p-intercept, 4. So, the root


of the equation is 4.

453

Chapter 10

PQ249-6481F-10[438-457] 26/9/02 6:20 PM Page 454 Sahuja Ahuja_QXP_06:Desktop Folder:Chandra:Algebra_FNL_Delivery:

34. Sample answer:

31. Rewrite the equation.


12n2  26n  30
2
12n  26n  30  30  30
12n2  26n  30  0
Graph f(n)  12n2  26n  30.
Sample answer:
n
1
0
1
2
3

( )
20 f n
10
O
21
1 2 3 4 5 6n
10
20
30
40
50
f (n)  12n 2  26n  30
60

f(n)
8
30
44
34
0

(3, 5)

35. f(x)  x2  4x  12


Sample answer:
x
6
4
2
0
2

One n-intercept of the graph is 3, and the other


n-intercept is between 1 and 0. So, one root of
the equation is 3, and the other root is between
1 and 0.
32. Rewrite the equation.
4x2  35  4x
2  35  4x  4x  4x
4x
4x2  4x  35  0
Graph f(x)  4x2  4x  35.
Sample answer:
x
4
3
1
1
2
3

( )
f (x)  x 2  4x  12 f x
15
12
9
6
3
O
765 4 321
1 2 3x
3
6

38. A  3 bh
2

 3 (8) (16)


256
3

or 85 3
1

The area to be painted is about 85 3 square feet.


39. Multiply the area under one arch by 12 to find
the area under 12 arches.

1 12

The x-intercepts of the graph are between 4


and 3 and between 2 and 3. So, one root of the
equation is between 4 and 3, and the other
root is between 2 and 3.
33. Sample answer:

1256 2

12 853  12 3 or 1024 ft2


Since there are two coats, multiply the area under
the 12 arches by 2.
2(1024)  2048 ft2
Each gallon covers 200 ft2, so

2048
200

(1, 6)

f(x)
0
12
16
12
0

The x-intercepts are 6 and 2.


36. The length of the segment is the distance between
the x-intercepts, which is |2  (6)| on 8 feet.
37. The highest point on the arch is at the vertex of
the parabola, which is at (2, 16). Thus, the
height of the arch is 16 feet.

( )
5 f x
O
4321
1 2 3 4x
5
10
15
20
25
30
35 f (x)  4x 2  4x  35

f(x)
13
11
35
27
11
13

256

 25 or 10 25 gallons.
Round up to 11 gallons and find the total cost.
11(27)  297 dollars
The paint for the walls would cost $297.
40. Graph y  0.005x2  0.22x and use the Zero
feature to find the relevant x-intercept.

[0, 50] scl: 10 by [0, 5] scl: 1

The x-intercept is about 44. The ball travels a


horizontal distance of 44 yd.

Chapter 10

454

PQ249-6481F-10[438-457] 26/9/02 6:20 PM Page 455 Sahuja Ahuja_QXP_06:Desktop Folder:Chandra:Algebra_FNL_Delivery:

41. Graph y  16x2  30x  1000 and use the Zero


feature to find the relevant x-intercept.

48. Since quadratic functions can be used to model a


golf ball after it is hit, solving the related
quadratic equation will determine where the ball
hits the ground. Answers should include the
following.
In the golf problem, one intercept represents
the balls original location and the other
intercept represents where the ball hits the
ground.
Using the quadratic function y  0.0015x2 
0.3x, the ball will hit the ground 200 yd from
the starting point.
49. C; The graph in answer choice C has no
x-intercepts.
50. A; The x-intercepts of the graph are 2 and 2.
Thus, 2 and 2 are the roots of the equation.
51. Graph y  x3  x2  4x  4.

[0, 10] scl: 1 by [0, 1100] scl: 100

42.
43.

44.

45.

The x-intercept is about 9. It took about 9 seconds


for the ball to hit the ground.
Yes; there are almost 9  3  1 or 5 seconds to
warn them.
The total area to be mowed is 500  400 or
200,000 ft2. Since each will mow half, each mows
1
 200,000 or 100,000 ft2.
2
A  w
100,000  (500  2x)(400  2x)
100,000  200,000  1000x  800x  4x2
100,000  200,000  1800x  4x2
0  100,000  1800x  4x2
or
4x2  1800x  100,000  0
Graph y  4x2  1800x  100,000 and find the
relevant x-intercept.

[5, 5] scl: 1 by [10, 10] scl: 1

The x-intercepts of the graph are 2, 1, and 2.


Thus, the solutions of the equation are 2, 1,
and 2.
52. Graph y  2x3  11x2  13x  4.

[0, 70] scl: 5 by [0, 100,000] scl: 10,000

The x-intercept is about 65. Kirk should mow a


width of about 65 ft.

[1, 5] scl: 1 by [10, 1] scl: 1

46. Kirk must mow a width of 65 feet. Since the


mower cuts a width of 5 feet each time, Kirk
65
should go around the field 5 or 13 times.

The x-intercepts of the graph are 2, 1, and 4.


1
Thus, the solutions of the equation are 2 , 1, and 4.

x3  2x2  3x

47. The x-intercepts of the graph of f(x) 


x  5
are the values of x that satisfy the following
equation.
x3  2x2  3x
x  5

0

The value of a fraction is 0 only when the


numerator is 0, so we can solve x3  2x2  3x  0
to find the x-intercepts.
x3  2x2  3x  0
x(x2  2x  3)  0
x(x  3)(x  1)  0
or x  1  0
x  0 or x  3  0
x  3
x1
The x-intercepts are 3, 0, and 1.

455

Chapter 10

PQ249-6481F-10[438-457] 26/9/02 6:20 PM Page 456 Sahuja Ahuja_QXP_06:Desktop Folder:Chandra:Algebra_FNL_Delivery:

Page 538

Since the coefficient of the x2 term is positive, the


vertex is a minimum.

Maintain Your Skills


2

53. In y  x  6x  9, a  1 and b  6.
b

x  2a

x  2(1) or 3

x  3 is the equation of the axis of symmetry.


y  x2  6x  9
y  (3) 2  6(3)  9
y  9  18  9 or 0
The vertex is at (3, 0).
Since the coefficient of the x2 term is positive, the
vertex is a minimum.

12

4
8
12

y  0.5x 2  6x  5

Exercises 5658 For checks, see students work.

m2  24m  144
m  24m  144  0
m2  2(m)(12)  122  0
(m  12) 2  0
m  12  0
m  12
2
57.
7r  70r  175
56.

y  x 2  6x  9

1
(7r2 )
7

r2  10r  25
r  10r  25  0
r2  2(r)(5)  52  0
(r  5) 2  0
r50
r5
58.
4d2  9  12d
4d2  12d  9  0
2  2(2d)(3)  32  0
(2d)
(2d  3) 2  0
2d  3  0
2d  3

54. In y  x2  4x  3, a  1 and b  4.

x  2a
4

x  2(1) or 2
x  2 is the equation of the axis of symmetry.
y  x2  4x  3
y  (2) 2  4(2)  3
y  4  8  3 or 1
The vertex is at (2, 1).
Since the coefficient of the x2 term is negative, the
vertex is a maximum.

y
y  x 2  4x  3
O

 7 (70r  175)

59.
x

10m4
30m

d  2 or 1.5

11030 21mm 2
4

 3m41

m3
3

 3m3 or
60.

22a2b5c7
11abc2

22 a b c
111
21 a 21 b 21c 2
2

7
2

21 51 72

55. In y 
x
x

0.5x2

b
2a
(6)
2(0.5)

 2a
b
 2ab4c5

 6x  5, a  0.5 and b  6.


61.

9m3n5
27m2n5y4




or 6

x  6 is the equation of the axis of symmetry.


y  0.5(6) 2  6(6)  5
y  18  36  5 or 13
The vertex is at (6, 13).

m
n
1
19
27 21 m 21 n 21 y 2
3

2

4

1
3m3(2)n55y4
1
3m5n0y4
m5y4
 3

62. Let n be the number of books.


30  1.5n
55 and 30  1.5n  60
1.5n
25
1.5n  30
3n
50
n  20
n

50
3

or 163

Since the number of books must be a whole


number, there must be at least 17 but no more
than 20 books in the crate.

Chapter 10

456

PQ249-6481F-10[438-457] 26/9/02 6:20 PM Page 457 Sahuja Ahuja_QXP_06:Desktop Folder:Chandra:Algebra_FNL_Delivery:

Square the result.


(6)2  36
Thus, c  36.
7. t2  5t  c

63. Yes;
a2  14a  49  a2  2(a)(7)  72
 (a  7) 2
64. Yes;
m2  10m  25  m2  2(m)(5)  52
 (m  5) 2
65. No; 64 is not a perfect square.
66. Yes;
4y2  12y  9  (2y) 2  2(2y)(3)  32
 (2y  3) 2
67. No; 4 is not a perfect square.
68. Yes;
25x2  10x  1  (5x) 2  2(5x)(1)  12
 (5x  1) 2

Find
1
(5)
2

152 2

2.

3.
4.

5.

x2

x x

1
(12)
2

25
.
4
1
2

of

1
2

of

Add 9 to each side.


c2  6c  9  7  9
(c  3) 2  16
c  3  4
c  3  3  4  3
c3 4
c  3  4 or c  3  4
7
 1
The solutions are 1, 7.
9. x2  7x  12
The x2 and x terms are already isolated. Find
7 and square the result.

1
1

172 22  494

Add

49
4

to each side.
49
4
7 2
2

x  7x 

 12 

1x  2

49
4

4

x  2  2
7

x  2  2  2  2
7

x  2
7

x  2  2
6

 2 or 3
10.

or

1
2
7

x  2  2
8

 2 or 4

The solutions are 4, 3.


v2  14v  9  6
2  14v  9  9  6  9
v
v2  14v  15
Find

1 2

m  7  120
m  7  7  120  7
m  7 120
m  7  120 or m  7  120
m  11.5
m  2.5
The solutions are 11.5, 2.5.
6. a2  12a  c
1
2

25
4

162 22  (3)2 or 9

The area is x2  4x  4.
Graphing f(x)  x2  5x  7 would not result in
an exact answer, and x2  5x  7 cannot be
factored.
Divide each side by 5.
b2  6b  9  25
(b  3) 2  25
2(b  3) 2  125
0b  3 0  5
b  3  5
b  3  3  5  3
b3 5
b  3  5 or b  3  5
b  2
b8
The solutions are 2, 8.
m2  14m  49  20
(m  7) 2  20
2(m  7) 2  120
0m  7 0  120

Find

8. c2  6c  7
The c2 and c terms are already isolated. Find
6 and square the result.

Check for Understanding


1
1

2

Thus, c 

1. Sample answer:
x
x

of 5.

Square the result.

10-3 Solving Quadratic Equations


by Completing the Square
Page 542

1
2

14 2
2

1
2

of 14 and square the result.

 72 or 49

Add 49 to each side.


v2  14v  49  15  49
(v  7) 2  64
v  7  8
v  7  7  8  7
v  7 8
v  7  8 or v  7  8
1
 15
The solutions are 15, 1.

of 12.
 6

457

Chapter 10

PQ249-6481F-10[458-483] 26/9/02 6:22 PM Page 458 Sahuja Ahuja_QXP_06:Desktop Folder:Chandra:Algebra_FNL_Delivery:

11. r2  4r  2
The r2 and r terms are already isolated. Find
4 and square the result.

142 22  (2)2 or 4

1
2

14. Let s represent the length of a side of the square.


The area of the square is then s2. When the
length is increased by 6 inches and the width by
4 inches, the area of the resulting rectangle is
(s  6)(s  4). The area of the rectangle is twice
the area of the square.

of

Add 4 to each side.


r2  4r  4  2  4
(r  2) 2  6
r  2  16
r  2  2  16  2
r  2  16
r  2  16 or r  2  16
 4.4
 0.4
The solutions are 0.4, 4.4.
12.
a2  24a  9  0
2  24a  9  9  0  9
a
a2  24a  9
Find

1 2

1
2

24 2
2

(s  6)(s  4)  2s2
 4s  6s  24  2s2
s2  10s  24  2s2
2  10s  24  s2  10s  2s2  s2  10s
s
24  s2  10s
s2

Find

Add 25 to each side and reverse the sides.


s2  10s  25  24  25
(s  5) 2  49
s  5  7
s  5  5  7  5
s5  7
s  5  7 or s  5  7
 12
 2
Since we are looking for a length, ignore the
negative number. The length of a side of the
square is 12 in.

 (12) 2 or 144

Add 144 to each side.


a2  24a  144  9  144
(a  12) 2  135
a  12  1135
a  12  12  1135  12
a  12  1135
a  12  1135 or a  12  1135
 23.6
 0.4
The solutions are 0.4, 23.6.
13. 2p2  5p  8  7
Since the coefficient of the p2 term is not 1, first
divide each side by 2, the coefficient of the p2
term.

3 1 24

1 2

Pages 542543
2

17

result.

17

p  4   3 16
5

p  4  4   3 16  4
5

p4 
5

p4
 2.3

Chapter 10

17

3 16

or

17

3 16
5

p4

Practice and Apply

15. b  4b  4  16
(b  2) 2  16
2(b  2) 2  116
0b  2 0  4
b  2  4
b  2  2  4  2
b2  4
b  2  4 or b  2  4
 2
6
The solutions are 2, 6.
16. t2  2t  1  25
(t  1) 2  25
2(t  1) 2  125
0t  1 0  5
t  1  5
t  1  1  5  1
t  1  5
t  1  5
t  1  5
 6
4
The solutions are 6, 4.
17. g2  8g  16  2
(g  4) 2  2
2(g  4) 2  12
0g  4 0  12
g  4  12
g  4  4  12  4
g  4  12
g  4  12 or g  4  12
 2.6
 5.4
The solutions are 2.6, 5.4.

5 2
5 2
25
2
 4 or 16
25
Add 16 to each side.
5
25
1
25
p2  2p  16  2  16
5 2
17
p  4  16
1
2

of 10 and square the result.

 (5) 2 or 25
110
2 2

of 24 and square the result.

2p2  5p  8
7
2
2
5
7
p2  2p  4  2
5
7
p2  2p  4  4  2  4
5
1
p2  2p  2
1
5
Find 2 of 2 and square the

1
2

17

3 16

 0.2

458

PQ249-6481F-10[458-483] 26/9/02 6:23 PM Page 459 Sahuja Ahuja_QXP_06:Desktop Folder:Chandra:Algebra_FNL_Delivery:

18. y2  12y  36  5
(y  6) 2  5
2(y  6) 2  15
0y  6 0  15
y  6  15
y  6  6  15  6
y  6  15
y  6  15 or y  6  15
 3.8
 8.2
The solutions are 3.8, 8.2.
19. w2  16w  64  18
(w  8) 2  18
2(w  8) 2  118
0w  8 0  118
w  8  118
w  8  8   118  8
w  8  118
w  8  118 or w  8  118
 12.2
 3.8
The solutions are 12.2, 3.8.
20. a2  18a  81  90
(a  9) 2  90
2(a  9) 2  190
0a  9 0  190
a  9  190
a  9  9  190  9

26. k2  11k  c
Find
c

c

1
of 16
2
16 2
2

1 2

Square

12

c 2
2

c

1
of 10
2
10 2
2

1 2

c
4

c
2

c

1
of
2
22 2
2

1 2

c
25.

1
of
2
34 2
2

1 2

and set it equal to 81. Then solve for c.

 81

1c4 2  4(81)
2

Square

12

c 2
2
2

c
4

c
2

and set it equal to 144. Then solve for c.

 144
 144

1 2  4(144)
c2
4

c2  576
c  24
29.
s2  4s  12  0
2
s  4s  12  12  0  12
s2  4s  12
2
s  4s  4  12  4
(s  2) 2  16
s  2  4
s  2  2  4  2
s2  4
s  2  4 or s  2  4
 2
6
The solutions are 2, 6.
d2  3d  10  0
30.
d2  3d  10  10  0  10
d2  3d  10

and square the result.

d2  3d  4  10  4

1d  32 22  494

and square the result.

d  2  2
3

d  2  2  2  2

22 and square the result.

d  2 
3

d  2  2

 112 or 121
24. a2  34a  c
Find

121
4

 81

 (5) 2 or 25
23. w2  22w  c
Find

or

c2  324
c  18
28. x2  cx  144

 82 or 64
22. y2  10y  c
Find

1 2

11 and square the result.

27. x2  cx  81

a  9  190
a  9  190 or a  9  190
 18.5
 0.5
The solutions are 18.5, 0.5.
21. s2  16s  c
Find

1
of
2
11 2
2

10

  2 or 5

or

7
2

d  2  2
4

 2 or 2

The solutions are 5, 2.

34 and square the result.

 172 or 289
 7p  c

p2

Find
c

1
of
2
7 2
2

1 2

7 and square the result.


or

49
4

459

Chapter 10

PQ249-6481F-10[458-483] 26/9/02 6:23 PM Page 460 Sahuja Ahuja_QXP_06:Desktop Folder:Chandra:Algebra_FNL_Delivery:

31.

y2  19y  4  70
y  19y  4  4  70  4
y2  19y  66

d2  8d  7  0
d  8d  7  7  0  7
d2  8d  7
2  8d  16  7  16
d
(d  4) 2  9
d  4  3
d  4  4  3  4
d4  3
d  4  3 or d  4  3
1
7
The solutions are 1, 7.
37.
s2  10s  23
2  10s  25  23  25
s
(s  5) 2  48
s  5   148
s  5  5   148  5
s  5  148
s  5  148 or s  5  148
 1.9
 11.9
The solutions are 1.9, 11.9.
38.
m2  8m  4
2  8m  16  4  16
m
(m  4) 2  20
m  4  120
m  4  4  120  4
m  4  120
m  4  120 or m  4  120
 0.5
 8.5
The solutions are 0.5, 8.5.
39.
9r2  49  42r
9r2  49  42r  42r  42r
9r2  42r  49  0
(3r  7) 2  0
3r  7  0
3r  7  7  0  7
3r  7
36.

361
4
19 2
y 2
19
y 2
19
19
 2  2

y2  19y 

y


 66 

361
4

625
4
25
 2
25
19
 2  2
19
25
y 2  2
19
25
or y  2  2
44
 2 or 22

19
25
 2
2
6
2 or 3

The solutions are 3, 22.


d2  20d  11  200
2  20d  11  11  200  11
d
d2  20d  189
2  20d  100  189  100
d
(d  10) 2  289
d  10  17
d  10  10  17  10
d  10  17
d  10  17 or d  10  17
 27
7
The solutions are 27, 7.
33.
a2  5a  4

32.

25
4
5
a22
5
a2
5
5
22

a2  5a 

25
4

 4 
9

4
3

 2
3

 2  2
5

3
2
5
2
8
2

a2 
5

a22
2

 2 or 1

or

2
or 4

Chapter 10

r  3 or 23

The solutions are 1, 4.


34.
p2  4p  21
2
p  4p  4  21  4
(p  2) 2  25
p  2  5
p  2  2  5  2
p2  5
p  2  5 or p  2  5
 3
7
The solutions are 3, 7.
35.
x2  4x  3  0
2
x  4x  3  3  0  3
x2  4x  3
2
x  4x  4  3  4
(x  2) 2  1
x  2  1
x  2  2  1  2
x  2  1
x  2  1 or x  2  1
 3
 1
The solutions are 3, 1.

The solution is
40.

7
3

or 23.

4h2  25  20h
 25  20h  20h  20h
4h2  20h  25  0
(2h  5) 2  0
2h  5  0
2h  5  5  0  5
2h  5
4h2

h2
The solution is

460

5
2

or 22.

PQ249-6481F-10[458-483] 26/9/02 6:23 PM Page 461 Sahuja Ahuja_QXP_06:Desktop Folder:Chandra:Algebra_FNL_Delivery:

41. 0.3t2  0.1t  0.2


2

0.3t  0.1t
0.3
1
t2  3t
1
1
t2  3t  36
1
t62

 0.3

44. 9w2  12w  1  0


9w2  12w  1
9
4
1
w2  3 w  9
4
1
1
w2  3 w  9  9
4
w2  3w
4
4
w2  3 w  9
2
w32

0.2
2

3



2
1
 36
3
25
36

t  6  6
1

t  6  6  6  6
1

t  6 
1

t  6  6


6
6

5
6
1

or 1

4
6

or

v

v

5
2

5
2
5
2

w3

 5v

45.

 5v  5v

43.

0
0

0
5
2

1
22

5
2

1
22.

or

39
2

w3

or

39

0

2  2(0)

d2  2d  6  0

11

11

d  4  4  4  4
5

11
4
5
11
4 4
16
 4 or 4

d4 
5

11
4

d4

7
5

6
4

or

or

3
2
3

7

5
12
5

The solutions are 2, 4.

46.
3

12
5

x  1  1  3

12
5

1

x  1 

35

or

25

d  4  4

0
5

0

25

1d  54 22  121
16

x  1  3

12

39

0

 2x  1 

35

d2  2d  16  6  16

x  1 

x2  2x  5
(x 

d2  2 d  6

5
2

1) 2

1 2
5
d  4d  3
2
1 2
5
d  4d  3
2

 10x  7  0

x2

0

5x2  10x  7
5
7
x2  2x  5
7
7
x2  2x  5  5

9

d2  2 d  6  6  0  6

v  or

5x2

 0.1
 1.4
The solutions are 0.1, 1.4.

2v

 0.4

The solution is

99

42. 0.4v2  2.5  2v

9

w3 

09

w  3  3  3 9  3

2
3

The solutions are 1, 3.


0.4v2  2.5
0.4
25
v2  4
25
v2  4  5v
25
v2  5v  4
5
v22

0

w  3  3 9

t  6  6

or

9

0

2  3(0)

f 2  2f  2  0
7

f 2  2f  2  2  0  2

12

x  1 

1 2
7
1
f  6f  2
3
1 2
7
1
f  6f  2
3

3
2
7
49
3
49
f 2  2f  16  2  16
7
25
f  4 2  16
7
5
f  4  4
7
7
5
7
f  4  4  4  4
7
5
f4  4
7
5
7
5
f  4  4 or f  4  4
2
1
12
 4 or 2
 4 or 3
1
The solutions are 2, 3.
7

f 2  2f 

12

35

 2.5
 0.5
The solutions are 2.5, 0.5.

461

Chapter 10

PQ249-6481F-10[458-483] 26/9/02 6:23 PM Page 462 Sahuja Ahuja_QXP_06:Desktop Folder:Chandra:Algebra_FNL_Delivery:

x2  4x  c  0
x  4x  c  c  0  c
x2  4x  c
2  4x  4  c  4
x
(x  2) 2  4  c
x  2   14  c
x  2  2   14  c  2
x  2  14  c
2
48.
x  6x  c  0
x2  6x  c  c  0  c
x2  6x  c
2
x  6x  9  c  9
(x  3) 2  9  c
x  3   19  c
x  3  3   19  c  3
x  3  19  c
49. area of garden  9  6 or 54
area of path
 area of large rectangle  area of garden
 (2x  9)(2x  6)  54
 4x2  12x  18x  54  54
 4x2  30x
Since these areas are equal, set the expressions
equal to each other and solve for x.
4x2  30x  54

x2  4x  12  0
x  4x  12  12  0  12
x2  4x  12
2  4x  4  12  4
x
(x  2) 2  8
There are no real solutions since the square of a
number cannot be negative.
52. The dimensions of the photograph are 12  4x
and 12  2x. The area of the photograph is
54 square inches.
(12  4x)(12  2x)  54
144  24x  48x  8x2  54
8x2  72x  144  54
51.

47.

4x2  30x
4
15
x2  2 x
15
225
x2  2 x  16
15
x 4 2
15
x 4
15
15
x 4  4

x


21
4

8x2  72x  144


8







or

x2  9x  18  18 
x2  9x 

1x

x
9

x23

or

x  2 or 1.5

15

3
3
2

or

y  0.059x2  7.423x  362.1


300  0.059x2  7.423x  362.1
0.059x2  7.423x  362.1
0.059

9
 3
9

 3  2

x23
x

15
2

x2  8x  35

5084.75  x2  125.81x  6137.29


1052.54  x2  125.81x
2904.50  x2  125.81x  3957.04
2904.50  (x  62.91) 2

x2

or 7.5

Original equation

 8x  16  35  16

Since

182 22  16, add 16 to each side.

Factor x2  8x  16.

(x  4)  51
x  4   151

Take the square root of each side.

x  4  4  151  4

 12904.50  x  62.91
62.91  12904.50  x
x  62.91  12904.50 or x  62.91  12904.50
9
 117
or in 1909
or in 2017

Subtract 4 from each side.

x  4  151

Simplify.

x  4  151 or x  4  151
x  11.14
x  3.14
The solutions are 11.14, 3.14.
54. C; 225 is not the square of any real number.
55. A; x2  5x  14

In the year 2017, an average American will


consume 300 pounds of bread and cereal per year.

x2  5x 

25
4

 14 

5
81
A; x  2 2  4

Chapter 10

Use x  1.5 because the solution must satisfy


12  2x  0. (The width must be positive.)
Then the width of the photograph is 12  2(1.5) or
9 inches, and the height is 12  4(1.5) or 6 inches.
53. Al-Khwarizmi used squares to geometrically
represent quadratic equations. Answers should
include the following.
Al-Khwarizmi represented x2 by a square
whose sides were each x units long. To this
square, he added 4 rectangles with length
8
x units long and width 4 or 2 units long. This
area represents 35. To make this a square,
four 4  4 squares must by added.
To solve x2  8x  35 by completing the
square, use the following steps.

21
4

Ignore the negative number. The path is


1.5 m wide.
50. Replace y with 300 and solve for x.

 4 or 2

300
0.059

81
4
9 2
2
9
x2
9
9
2
2

x2  9x 

54
8
27
4
27
 18
4
45
4
45
81
 4
4

x2  3

x  4 
6

or 9

x2  9x  18 

54
4
27
2
27
225
 16
2
441
16
21
4
21
15
4  4
15
21
4  4

x
15
4
36
4

462

25
4

PQ249-6481F-10[458-483] 26/9/02 6:23 PM Page 463 Sahuja Ahuja_QXP_06:Desktop Folder:Chandra:Algebra_FNL_Delivery:

Page 544

60. y  x2  3x  10
Sample answer:

Maintain Your Skills


2

56. Graph f(x)  x  7x  12.


Sample answer:
x
5
4
3
2
1

f(x)
2
0
0
2
6

x
2
0
1
2
3
5

f (x )

y
0
10
12
12
10
0

2

x
4
2
0
2
4

x
0
1
2
3
4

 16.
f (x )

f(x)
0
12
16
12
0

4

2

4x

4
8

8
12

y  x 2  3x  10

x2

x
y  x 2  3x  4

f(x)
9
6
5
6
9

GCF: a  b  b or ab2
63. 32m2n3  2  2  2  2  2  m  m 
n nn
m  m 
n
8m2n  2  2  2 
m  m  m n n
56m3n2  2  2  2  7 

 2x  6.
f (x )

GCF: 2  2  2  m  m  n or 8m2n
64. y  2x
xy9
Substitute 2x for y in the second equation.
xy9
x  2x  9
3x  9

f (x )  x 2  2x  6

3x
3

The graph has no x-intercepts. Thus, the equation


has no real number solutions: .

y
32
20
16
20
32

32

24
16
8
4

2

y  4x 2  16

3

x3
Use y  2x to find the value of y.
y  2x
y  2(3) or 6
The solution is (3, 6).
65. x  y  3
2x  3y  5
Substitute y  3 for x in the second equation.
2x  3y  5
2(y  3)  3y  5
2y  6  3y  5
y  6  5
y  6  6  5  6
y  1
(1)(y)  (1) (1)
y1
Use x  y  3 to find the value of x.
xy3
x  1  3 or 4
The solution is (4, 1).

59. y  4x2  16
Sample answer:
x
2
1
0
1
2

62. 14a2b3  2  7 
a a
b  b b
20a3b2c  2  2  5  a  a  a  b  b  c
35ab3c2  5  7 
a 
b  b bcc

f (x )  x 2  16

The x-intercepts of the graph are 4 and 4.


Thus, the solutions of the equation are 4 and 4.
x
1
0
1
2
3

y
4
2
2
4
8

12
16

58. Graph f(x) 

61. y  x2  3x  4
Sample answer:

The x-intercepts of the graph are 4 and 3.


Thus, the solutions of the equation are 4 and 3.
57. Graph f(x) 

4

f (x )  x 2  7x  12

x2

4x

463

Chapter 10

PQ249-6481F-10[458-483] 26/9/02 6:23 PM Page 464 Sahuja Ahuja_QXP_06:Desktop Folder:Chandra:Algebra_FNL_Delivery:

66. x  2y  3
3x  y  23
Solve the first equation for x.
x  2y  3
x  2y  2y  3  2y
x  2y  3
Substitute 2y  3 for x in the second equation.
3x  y  23
3(2y  3)  y  23
6y  9  y  23
7y  9  23
7y  9  9  23  9
7y  14
7y
7

72. 2b2  4ac  2(2) 2  4(1) (15)


 14  60
 164
8
73. 2b2  4ac  272  4(2) (3)
 149  24
 125
5
74. 2b2  4ac  252  4(1) (2)
 125  8
 133
 5.7
75. 2b2  4ac  272  4(2) (5)

14
7

 149  40
 189
 9.4

y2
Use x  2y  3 to find the value of x.
x  2y  3
x  2(2)  3 or 7
The solution is (7, 2).
67. 3 6 x 6 1
68. x
2 or x 7 1
69.
5x  3y  7
5x  3y  5x  7  5x
3y  5x  7
3y
3

y

Page 544
b

x  2a
(1)

x   2(1)
1

x  2 or 0.5

5x  7
3
5
7
x
3
3

x  0.5 is the equation of the axis of symmetry.


y  x2  x  6
y  (0.5) 2  (0.5)  6
y  0.25  0.5  6
y  6.25
The vertex is at (0.5, 6.25).
The vertex is a minimum since the coefficient of
the x2 term is positive.

The slope of any line perpendicular to this line is


3
5
5, the opposite of the reciprocal of 3.
Use the point-slope form.
y  y  m(x  x )
1

y  (2)  5 (x  8)
3

y  2  5 x 
3

y  2  2  5 x 
3

y  5 x 

24
5
24
5
14
5

Practice Quiz 1

1. In y  x2  x  6, a  1 and b  1.

2

70. We use the points (2, 0) and (0, 2). Find the
slope.
y2  y1

mx




 x1

2  0
0  2
2
or 1
2

y  x2  x  6

Use the slope-intercept form.


y  mx  b
y  x  (2)
yx2
71. We use the points (1, 2) and (0, 0). Find the
slope.
y2  y1

mx

 x1

0  2
(1)
2
or 2
1

0


Use the slope-intercept form.


y  mx  b
y  2x  0
y  2x

Chapter 10

464

PQ249-6481F-10[458-483] 26/9/02 6:23 PM Page 465 Sahuja Ahuja_QXP_06:Desktop Folder:Chandra:Algebra_FNL_Delivery:

2. In y  2x2  3, a  2 and b  0.

5. Graph f(x)  x2  2x  1.
Sample answer:

x  2a

x
1
0
1
2
3

x  2(2) or 0
x  0 is the equation of the axis of symmetry.
y  2x2  3
y  2(0) 2  3
y  0  3 or 3
The vertex is at (0, 3).
The vertex is a minimum since the coefficient of
the x2 term is positive.
21
18
15
12
9
6
3

The x-intercepts lie between 1 and 0 and


between 2 and 3. So, one root is between 1
and 0, and the other is between 2 and 3.
6. Graph f(x)  x2  5x  6.
Sample answer:

x
1
0
2
4
6

3. In y  3x2  6x  5, a  3 and b  6.


b

x  2a
(6)

x  2(3)
6

x  6 or 1
x  1 is the equation of the axis of symmetry.
y  3x2  6x  5
y  3(1) 2  6(1)  5
 3  6  5
8
The vertex is at (1, 8).
The vertex is a maximum since the coefficient of
the x2 term is negative.
y  3x 2  6x  5

4. Graph f(x)  x2  6x  10.


Sample answer:
f(x)
5
2
1
2
5

f (x )

f (x )  x 2  6x  10

f(x)
0
6
12
10
0

( )
6 f x
4
2
O
422 2 4 6 8 10 12 x
4
6
8
10
12
14

f (x )  x 2  5x  6

The x-intercepts are 1 and 6. Thus, the


solutions of the equation are 1 and 6.
s2  8s  15
7.
2  8s  16  15  16
s
(s  4) 2  1
s  4  1
s  4  4  1  4
s  4  1
s  4  1 or s  4  1
 5
 3
The solutions are 5, 3.
8.
a2  10a  24
2  10a  25  24  25
a
(a  5) 2  1
a  5  1
a  5  5  1  5
a5  1
a  5  1 or a  5  1
4
6
The solutions are 4, 6.
9. y2  14y  49  5
(y  7) 2  5

x
5
4
3
2
1

f (x )

f (x )  x 2 2x  1

y  2x 2  3
O
x
4321
3 1 2 3 4

f(x)
2
1
2
1
2

y  7  15
y  7  7  15  7
y  715
y  7  15 or y  7  15
 4.8
 9.2
The solutions are 4.8, 9.2.

The graph has no x-intercepts. The equation has


no real number solutions: .

465

Chapter 10

PQ249-6481F-10[458-483] 26/9/02 6:23 PM Page 466 Sahuja Ahuja_QXP_06:Desktop Folder:Chandra:Algebra_FNL_Delivery:

2b2  b  7  14

10.

2b2  b  7
2
1
7
2
b  2b  2
1
7
7
b2  2b  2  2
1
b2  2b
1
1
b2  2b  16
1 2
b4
1
b4

5. y  x2  6x  1
y  (x2  6x  9)  1  9
y  (x  3) 2  10
The vertex is (3, 10) .

14
2

7
7

72





21
2
21
1
 16
2
169
16
13
4

13

y  x 2  6x  1 4
4

6

2

4
8

b  4  4  4  4
1

b4 
1

13
4

12
4

or 3

b4


or

b


13
4
1
13
 4
4
14
7
or 2
4

Solving Quadratic Equations


by Using the Quadratic Formula

10-4

The solutions are 3, 3.5.

Page 550
Page 545

Check for Understanding

1. Sample answer: (1) Factor x2  2x  15 as


(x  3)(x  5). Then according to the Zero Product
Property, either x  3  0 or x  5  0. Solving
these equations, x  3 or x  5. (2) Rewrite the
equation as x2  2x  15. Then add 1 to each side
of the equation to complete the square on the left
side. Then (x  1)2  16. Taking the square root
of each side, x  1  4. Therefore, x  1  4 and
x  3 or x  5. (3) Use the Quadratic Formula.

Graphing Calculator Investigation


(Follow-Up of Lesson 10-3)

1. y  a(x  h) 2  k
The vertex is (h, k) .
2. y  x2  2x  3
y  (x2  2x  1 )  3  1
y  (x  1 ) 2  4
3. y  x2  2x  7
y  (x2  2x  1)  7  1
y  (x  1) 2  8
The vertex is (1, 8) .

Therefore, x 

2  2(2) 2  4(1) (15)


2(1)

2  164
.
2

y  x 2  2x  7

x

4. y  x  4x  8
y  (x2  4x  4)  8  4
y  (x  2) 2  4
The vertex is (2, 4).
y

b  2b2  4ac
2a

7  272  4(1) (6)


2(1)

7  149  24
2

7  125
2

7  5
2
7  5
2

x

or

x

7  5
2

 6
 1
The solutions are 6, 1.
y  x 2  4x  8
O

Chapter 10

or

x
Simplifying the expression, x  3
or x  5. See students preferences.
2. Sample answer: x2  x  5  0. See students
work. Any quadratic equation for which the
discriminant is negative is correct.
3. Juanita; you must first write the equation in the
form ax2  bx  c  0 to determine the values of
a, b, and c. Therefore, the value of c is 2, not 2.
4. x2  7x  6  0

y
O

2x

466

PQ249-6481F-10[458-483] 26/9/02 6:23 PM Page 467 Sahuja Ahuja_QXP_06:Desktop Folder:Chandra:Algebra_FNL_Delivery:

5.

t2  11t  12
t  11t  12  12  12
t2  11t  12  0

t




t

w

11  13
2

10  2102  4(1) (12)


2(1)

10  1100  48
2

10  152
2

r

10  152
2

or

r

5  252  4(3) (11)


2(3)

5  125  132
6

5  1107
6

10  152
2

4x2  2x  17
4x  2x  17  17  17
4x2  2x  17  0
b  2b2  4ac
2a

2  222  4(4) (17)


2(4)

2  14  272
8

2  1276
8

x

2  1276
8

or

x

 15w  2  0

b  2b2  4ac
2a

15  1225  200
50

15  125
50
15  5
50
15  5
or
50
10
1
or 5
50

15  5
50
20
2
or 5
50

w


The solutions are

x

1 2
, .
5 5

10. m2  5m  6  0
b2  4ac  52  4(1) (6)
 25  24
 49
Since the discriminant is positive, the equation
has 2 real roots.
11. s2  8s  16  0
b2  4ac  82  4(1) (16)
 64  64
0
Since the discriminant is 0, the equation has
1 real root.
12.
2z2  z  50
2
2z  z  50  50  50
2z2  z  50  0
b2  4ac  12  4(2) (50)
 1  400
 399
Since the discriminant is negative, the equation
has no real roots.
13. Write an expression for the volume of the pan.
V  /wh
 (x  4) (x  4) (2)
 (x2  8x  16)(2)
 2x2  16x  32
The volume is to be 441 cubic centimeters.

; There are no real solutions because the


discriminant is negative.
8.

b  2b2  4ac
2a

(15)  2(15) 2  4(25) (2)


2(25)

w

 8.6
 1.4
The solutions are 8.6, 1.4.
7. 3v2  5v  11  0
v

b  2b2  4ac
2a

w2  5w  25  0

25w2

 12
1
The solutions are 12, 1.
6. r2  10r  12  0
r

25 w2  5w  25  25  0

11  2112  4(1) (12)


2(1)
11  1121  48
2

t

w2  25  5w  5w  5w

b  2b2  4ac
2a

11  1169
2
11  13
2
11  13
or
2

w2  25  5w

9.

2  1276
8

2x2  16x  32  441


2x2  16x  409  0

 2.3
 1.8
The solutions are 2.3, 1.8.

x

(16)  2(16) 2  4(2) (409)


2(2)

16  13528
4
16  13528

4


x

or

x

16  13528
4

 10.8
 18.8
Ignore the negative number. The original sheet
should be about 18.8 cm by 18.8 cm.

467

Chapter 10

PQ249-6481F-10[458-483] 26/9/02 6:23 PM Page 468 Sahuja Ahuja_QXP_06:Desktop Folder:Chandra:Algebra_FNL_Delivery:

Pages 550552

19. r2  25  0

Practice and Apply

14. x2  3x  18  0
x

r

b  2b2  4ac
2a

3  232  4(1) (18)


2(1)

3  181
2
3  9
2
3  9
or
2


x

x

b  2b2  4ac
2a

0  10  4(1) (25)
2(1)

 1100
2

; There are no real solutions because the


discriminant is negative.
20.
2x2  98  28x
2  98  28x  28x  28x
2x
2x2  28x  98  0

3  9
2

 6
3
The solutions are 6, 3.
15. v2  12v  20  0

x

b  2b2  4ac
2a

b  2b2  4ac
2a

(28)  2(28) 2  4(2) (98)


2(2)

12  2122  4(1) (20)


2(1)

12  164
2
12  8
2
12  8
or
2

28  10
4
28
or 7
4

v


v

v

21.

12  8
2

 10
 2
The solutions are 10, 2.
16. 3t2  7t  20  0
t

(7)  2(7) 2  4(3) (20)


2(3)

7  1289
6
7  17
6
7  17
6
10
2
 6 or 13


t


s

b  2b2  4ac
2a

or t 


7  17
6
24
or 4
6

17. 5y  y  4  0




1  181
10
1  9
10
1  9
or
10
4
5

1  9
10

1

1  1113
4
1  1113
4

r

or

1  1113
4

b  2b2  4ac
2a

(7)  2(7) 2  4(2) (3)


2(2)

7  173
4
7  173
4

or

n

7  173
4

 0.4
 3.9
The solutions are 0.4, 3.9.
24.
5v2  7v  1
2  7v  1  1  1
5v
5v2  7v  1  0

18. x2  25  0

1  212  4(2) (14)


2(2)

n

n
y

b  2b2  4ac
2a

 2.9
 2.4
The solutions are 2.9, 2.4.
23. 2n2  7n  3  0

The solutions are 5, 1.


x

40  10
8
40
or
5
8

r

b  2b2  4ac
2a
(1)  2(1) 2  4(5) (4)
2(5)

r

y

(40)  2(40) 2  4(4) (100)


2(4)

The solution set is {5}.


22. 2r2  r  14  0

b  2b2  4ac
2a




The solutions are 13, 4.


y

The solution set is {7}.


4s2  100  40s
2
4s  100  40s  40s  40s
4s2  40s  100  0

b  2b2  4ac
2a
0  202  4(1) (25)
2(1)
 1100
2
10
2

v

 5
The solutions are 5, 5.

b  2b2  4ac
2a

(7)  2(7) 2  4(5) (1)


2(5)

7  169
10

v

7  169
10

or v 

7  169
10

 0.1
 1.5
The solutions are 0.1, 1.5.
Chapter 10

468

PQ249-6481F-10[458-483] 26/9/02 6:23 PM Page 469 Sahuja Ahuja_QXP_06:Desktop Folder:Chandra:Algebra_FNL_Delivery:

25.

11z2  z  3
11z  z  3  3  3
11z2  z  3  0

29.

z

b  2b2  4ac
2a

x

(1)  2(1) 2  4(11) (3)


2(11)

1  1133
22

z

1  1133
22

z

or



x

1  1133
22




7  125
4
7  5
4
7  5
or
4


w

27.

7  5
4
1
 2
1
3, 2.

2y2  4y  2  0
5

8y2  5y  2  0
b  2b2  4ac
2a

5  189
16

y

5  189
16

y

or

5  189
16

 0.3
 0.9
The solutions are 0.3, 0.9.
1 2
v
2

31.

v4

1 2
3
v v4
2
1 2
3
v v4
2
1 2
3
v v4
2

44
0

2  4(0)

2v  4v  3  0
g

or

The solutions are

v

2  38
48
40
5
or 6
48

3 5
, .
4 6

1.34d2  1.1d  1.02


1.34d  1.1d  1.02  1.02  1.02
1.34d2  1.1d  1.02  0

b  2b2  4ac
2a

(4)  2(4) 2  4(2) (3)


2(2)

4  140
4

v

(5)  2(5) 2  4(8) (2)


2(8)

b  2b  4ac
2a

d

2  11444
48
2  38
48
2  38
48
36
3
or 4
48


28.

(2)  2(2) 2  4(24) (15)


2(24)

g

4 2y2  4y  2  4(0)

0.7  1.7
4
1
4

2y2  4y  2  2  2

2(12g2  g)  15
24g2  2g  15
2  2g  15  15  15
24g
24g2  2g  15  0
g

w

The solutions are

x

2y2  4y  2

y

 3

0.7  12.89
4
0.7  1.7
4
0.7  1.7
or
4
2.4
4

30.

b  2b2  4ac
2a
7  272  4(2) (3)
2(2)

0.7  2(0.7) 2  4(2) (0.3)


2(2)



 0.6
 0.3
The solutions are 0.3, 0.6.

 0.5
 0.6
The solutions are 0.5, 0.6.
26.
2w2  (7w  3)
2  (7w  3)  (7w  3)  (7w  3)
2w
2w2  7w  3  0
w

2x2  0.7x  0.3


2x  0.7x  0.3  0.3  0.3
2x2  0.7x  0.3  0
2

4  140
4

v

4  140
4

 0.6
 2.6
The solutions are 0.6, 2.6.

b  2b  4ac
2a
(1.1) ; 2(1.1) 2  4(1.34) (1.02)
2(1.34)
1.1  14.2572


2.68
; There are no real solutions because the
discriminant is negative.

469

Chapter 10

PQ249-6481F-10[458-483] 26/9/02 6:23 PM Page 470 Sahuja Ahuja_QXP_06:Desktop Folder:Chandra:Algebra_FNL_Delivery:

34. Let n be the first odd integer. Then n  2 is the


next odd integer. Write an equation for the
product and solve.
n(n  2)  255
n2  2n  255
2
n  2n  255  0

32. Write an equation for the perimeter.


2/  2w  60
Solve for .
2/  2w  60
2/  60  2w
2/
2

60  2w
2

n

/  30  w
Thus, the length and width can be expressed as
30  w and w, respectively. Write an equation for
the area and solve.
(30  w)(w)  221
30w  w2  221
2
w  30w  221  0
w


w



n

w

30  4
2

n


n


w

4  11024
4
4  32
4
4  32
or
4

n

4  32
4

36. f(x)  4x2  9x  4


To find the x-intercepts, set f(x)  0 and solve for x.
4x2  9x  4  0
x

21  2212  4(1) (80)


2(1)

w

4  242  4(2) (126)


2(2)

 9
7
When n  9, n  2  9  2 or 7.
When n  7, n  2  7  2 or 9.
The numbers are 9 and 7 or 7 and 9.

42  2w
2

21  1121
2
21  11
2
21  11
2

2  32
2

/  21  w
Thus, the length and width can be expressed as
21  w and w, respectively. Write an equation for
the area and solve.
(21  w)(w)  80
21w  w2  80
2
w  21w  80  0
w

n

n2  (n  2) 2  130
n  (n2  4n  4)  130
2n2  4n  4  130
2n2  4n  126  0

 17
 13
When w  17,   30  17 or 13.
When w  13,   30  13 or 17.
The rectangle is 13 in. by 17 in.
33. Write an equation for the perimeter.
2/  2w  42
Solve for .
2/  2w  42
2/  42  2w
2/
2

2  11024
2
2  32
2
2  32
or
2

 17
 15
When n  17, n  2  17  2 or 15.
When n  15, n  2  15  2 or 17.
The numbers are 17 and 15 or 15 and 17.
35. Let n be the first odd integer. Then n  2 is the
next odd integer. Write an equation for the sum of
their squares.

30  2302  4(1) (221)


2(1)
30  116
2
30  4
2
30  4
or
2

2  222  4(1) (255)


2(1)

(9)  2(9) 2  4(4) (4)


2(4)

9  117
8

x

21  11
2

b  2b2  4ac
2a

9  117
8

or x 

9  117
8

 0.6
 1.6
The x-intercepts are about 0.6 and about 1.6.

 16
5
When w  16,   21  16 or 5.
When w  5,   21  5 or 16.
The rectangle is 5 cm by 16 cm.

37. f(x)  13x2  16x  4


To find the x-intercepts, set f(x)  0 and solve for x.
13x2  16x  4  0
x

b  2b2  4ac
2a

(16)  2(16) 2  4(13) (4)


2(13)

16  1464
26

x

16  1464
26

or

x

16  1464
26

 0.2
 1.4
The x-intercepts are about 0.2 and about 1.4.

Chapter 10

470

PQ249-6481F-10[458-483] 26/9/02 6:23 PM Page 471 Sahuja Ahuja_QXP_06:Desktop Folder:Chandra:Algebra_FNL_Delivery:

38. x2  3x  4  0
b2  4ac  (3) 2  4(1) (4)
 9  16
 25
Since the discriminant is positive, the equation
has 2 real roots.
39. y2  3y  1  0
b2  4ac  32  4(1)(1)
94
5
Since the discriminant is positive, the equation
has 2 real roots.

45. f(x)  x2  4x  7
The number of x-intercepts is equal to the
number of roots of the equation x2  4x  7  0.
b2  4ac  42  4(1) (7)
 16  28
 12
Since the discriminant is negative, the graph of
the function has 0 x-intercepts.
46. h(t)  16t2  35t  5
To find how many chances Jorge will have, set
h(t)  25 and solve for t.
25  16t2  35t  5
25  25  16t2  35t  5  25
0  16t2  35t  20
Check the discriminant.

4p2  10p  6.25


4p2  10p  6.25  6.25  6.25
4p2  10p  6.25  0
b2  4ac  102  4(4)(6.25)
 100  100
0
Since the discriminant is 0, the equation has
1 real root.
41.
1.5m2  m  3.5
2
1.5m  m  3.5  3.5  3.5
1.5m2  m  3.5  0
b2  4ac  12  4(1.5)(3.5)
 1  21
 20
Since the discriminant is negative, the equation
has no real roots.
40.

2r2 

42.
1

1
r
2

2
3

2r2  2r  3 
2

2r 

b2  4ac 




1
r
2
1
r
2




2
3
2
3

1 2

0  16t2  35t  5
t


 2r  3

4(2)

4 2
n
3
4 2
n
3
4 2
n
3

b2

123 2


t

t

35  11545
32

96  2962  4(16) (96)


2(16)
96  13072
32
96  13072
32

or

t

96  13072
32

 4.7
 1.3
The distance, s, is 96 feet when t is about
1.3 seconds and again when t is about 4.7 seconds.

 4n  3  3  3

 4ac  42  4

35  11545
32

t

 4n  3

 4n  3  0

35  2352  4(16) (5)


2(16)

 2.3
 0.1
Ignore the negative number. The camera will hit
the ground after about 2.3 seconds.
48. Replace s with 96 and solve for t.
s  96t  16t2
96  96t  16t2
0  16t2  96t  96

Since the discriminant is negative, the equation


has no real roots.
43.

b  2b2  4ac
2a

35  11545
32


t

0

1 2

2
1
16
 3
4
3
64
 12
12
61
12

b2  4ac  (35) 2  4(16)(20)


 1225  1280
 55
Since the discriminant is negative, there are no
real solutions to the equation. Jorge has no
chances to catch the camera.
47. Set h(t)  0 and solve for t.

143 2 (3)

 16  16
0
Since the discriminant is 0, the equation has
1 real root.
44. f(x)  7x2  3x  1
The number of x-intercepts is equal to the
number of roots of 7x2  3x  1  0.
b2  4ac  (3) 2  4(7)(1)
 9  28
 37
Since the discriminant is positive, the graph of
the function has 2 x-intercepts.

471

Chapter 10

PQ249-6481F-10[458-483] 26/9/02 6:23 PM Page 472 Sahuja Ahuja_QXP_06:Desktop Folder:Chandra:Algebra_FNL_Delivery:

49. Let L  20, D  6, and H  10. Solve for v.


4v2

 5v  2 

4v2

 5v  2 

54. If a population trend can be modeled by a


quadratic function, the Quadratic Formula can be
used to solve when the function equals a
particular value. Answers should include the
following.

15  0.0055t2  0.0796t  5.2810


Original equation
15  15  0.0055t2  0.0796t  5.2810  15
Subtract 15 from each side.
0  0.0055t2  0.0796t  9.7190
Simplify.

1200 HD
L
1200(10) (6)
20

4v  5v  2  3600
4v2  5v  2  3600  3600  3600
4v2  5v  3602  0
v

v

5  157,657
8

b  2b2  4ac
2a

5  252  4(4) (3602)


2(4)

5  157,657
8

v

t

5  157,657
8

t

 30.6
 29.4
Ignore the negative number. The water is flowing
at a rate of about 29.4 feet per second.
50. The equation 0  ax2  10x  3 will have 2 real
solutions if the discriminant is positive.
b2  4ac 7 0
2
10  4a(3) 7 0
100  12a 7 0
100  12a  100 7 0  100
12a 7 100
12a
12

a 6

100
12
25
or
3

x

1.87  2(1.87) 2  4(0.048) (4)


2(0.048)

1.87  14.264
0.096

or

x

56. C;

x

1.87  14.264
0.096

1.87  2(1.87) 2  4(0.048) (154)


2(0.048)
1.87  133.0649
0.096
1.87  133.0649
0.096

or

x

1.87  133.0649
0.096

 79.4
 40.4
We would expect the death rate to be 0 per
100,000 79 years after 1970 or about 2049.
Sample answer: No; the death rate from cancer
will never be 0 unless a cure is found. If and
when a cure will be found cannot be predicted.

Chapter 10

t

0.0796  10.22015416
0.011

or t 

b  2b2  4ac
2a

Simplify.
0.0796  10.22015416
0.011

5  252  4(2) (1)


2(2)

5  117
4

Maintain Your Skills


2

x  8x  7
x2  8x  16  7  16
(x  4) 2  9
x  4  3
x  4  4  3  4
x4  3
x  4  3 or x  4  3
1
7
The solutions are 1, 7.
a2  2a  5  20
58.
a2  2a  5  5  20  5
a2  2a  15
2  2a  1  15  1
a
(a  1) 2  16
a  1  4
a  1  1  4  1
a  1  4
a  1  4 or a  1  4
 5
3
The solutions are 5, 3.

57.

0  0.048x2  1.87x  154

0.0796  10.22015416
0.011

Page 552

 41.0
 2.0
When y  150, x  2.0 or x  41.0.
52. Ignoring the negative solution, we would expect
the death rate to be 150 per 100,000 in 1970  41
or 2011.
53. Solve for x when y  0.
x

t

b2  4ac  (5) 2  4(8) (1)


 7

83

b  2b2  4ac
2a

1.87  14.264
0.096

(0.0796)  2(0.0796)  4(0.0055) (9.7190)


2(0.0055)

t  35.42
t  49.89
Graphing the related function would not give
precise solutions. The quadratic equation
cannot be factored and completing the square
would involve difficult computations.
55. A; the discriminant is negative.

150  0.048x2  1.87x  154


150  150  0.048x2  1.87x  154  150
0  0.048x2  1.87x  4
x

Quadratic Formula
2

a  0.0055, b  0.0796, and c  9.7190.

The graph of the function has two x-intercepts


1
when a 6 83.
51.

b  2b2  4ac
2a

472

PQ249-6481F-10[458-483] 26/9/02 6:23 PM Page 473 Sahuja Ahuja_QXP_06:Desktop Folder:Chandra:Algebra_FNL_Delivery:

59.

n2  12n  5
n  12n  36  5  36
(n  6) 2  41
n  6  141
n  6  6   141  6
n  6  141
n  6  141 or n  6  141
 0.4
 12.4
The solutions are 0.4, 12.4.

63. 15xy3  y4  y3 (15x)  y3 (y)


 y3 (15x  y)
64. 2ax  6xc  ba  3bc  (2ax  6xc)  (ba  3bc)
 2x(a  3c)  b(a  3c)
 (a  3c) (2x  b)

60.

65. 0.000000000000000000001672  1.672  1021


66. x
2
y4 5
y

x2  x  6
2
x x666
x2  x  6  0
Graph f(x)  x2  x  6.
Sample answer:
x
2
1
0
1
2
3

x2

y45

f(x)
0
4
6
6
4
0

f (x )

67. x  y 7 2
xy
2
y

f (x )  x 2  x  6

xy2

The x-intercepts of the graph are 2 and 3. The


solutions of the equation are 2 and 3.
61.
2x2  x  2
2
2x  x  2  2  2
2x2  x  2  0
Graph f(x)  2x2  x  2.
Sample answer:
f (x )
x
f(x)
2
4
1
1
0
2
1
1
2
8
x

xy2

68. y 7 x
y
x4
y
yx4

yx

f (x )  2x 2  x  2

The x-intercepts of the graph lie between 2 and


1 and between 0 and 1. So, one root is between
2 and 1, and the other root is between 0 and 1.

Exercises 6971 For checks, see students work.


69.
2m  7 7 17
2m  7  7 7 17  7
2m 7 10
2m
10
7 2
2
m 7 5
{m 0m 7 5}
70.
2  3x 2
2  3x  2 2  2
3x 4
3x
4

3
3

62. Graph f(x)  x2  3x  6.


x
2
1
0
2
4
5

f(x)
4
2
6
8
2
4

f (x )

f (x )  x 2  3x  6

5x 0x
43 6

The x-intercepts of the graph lie between 2 and


1 and between 4 and 5. So, one root is between
2 and 1, and the other root is between 4 and 5.

473

x
3

Chapter 10

PQ249-6481F-10[458-483] 26/9/02 6:23 PM Page 474 Sahuja Ahuja_QXP_06:Desktop Folder:Chandra:Algebra_FNL_Delivery:

71.

72. c(ax )  1(24 )


20 8  7k
20  8 8  7k  8
 1(16) or 16
28 7k
28
7k
7
7
4 k
{k 0k
4}

73. c(ax )  3(72 )


 3(49) or 147

3. Solve the first equation for y.


1.8x  y  3.6
y  3.6  1.8x
Enter y  3.6  1.8x as Y .
1

Enter y  x2  3x  1 as Y2.
Graph.
Approximate the first intersection point.
KEYSTROKES: 2nd [CALC] 5 ENTER ENTER

74. c(ax )  2(53 )


 2(125) or 250

ENTER

Page 553

Graphing Calculator Investigation


(Follow-Up of Lesson 10-4)

1. Enter y  2(2x  3) as Y .
1

Enter y  x2  2x  3 as Y2.
Then graph.
Approximate the intersection point.
KEYSTROKES: 2nd [CALC] 5 ENTER

[10, 10] scl: 1 by [10, 10] scl: 1

ENTER

Use the TRACE feature and right arrow to move


the cursor near the other intersection point. Use
the intersect feature again to approximate the
other intersection point.

ENTER

[5, 1] scl: 1 by [1, 15] scl: 1

The solution is (3, 6).


2. Solve the first equation for y.
y50
y5
Enter y  5 as Y1.

The solutions are approximately (1.6, 6.5) and


(2.8, 1.5).
4. Enter y  1.4x  2.88 as Y .
1

Enter y  x2  0.4x  3.14 as Y .


2
Graph.
Approximate the first intersection point.
KEYSTROKES: 2nd [CALC] 5 ENTER ENTER

Enter y  x2 as Y .
2
Graph.

ENTER

[10, 10] scl: 1 by [10, 10] scl: 1

The graphs do not intersect. Thus, the system has


no solution.

[10, 10] scl: 1 by [10, 10] scl: 1

Use the TRACE feature and left arrow to move


the cursor near the other intersection point. Use
the intersect feature again.

The solutions are approximately (1.9, 0.2) and


(0.1, 3.1).

Chapter 10

474

PQ249-6481F-10[458-483] 26/9/02 6:24 PM Page 475 Sahuja Ahuja_QXP_06:Desktop Folder:Chandra:Algebra_FNL_Delivery:

5. Enter y  x2  3.5x  2.2 as Y1.

Enter y  2x  5.3625 as Y2.


Graph.
Approximate the intersection point.
KEYSTROKES: 2nd [CALC] 5 ENTER

The graphs are the same shape. The graph of


y  2x  3 is the graph of y  2x translated
3 units up. The graph of y  2x  4 is the graph
of y  2x translated 4 units down.
2. Enter y  2x as Y .
1

ENTER

Enter y  2x5 as Y .
2

ENTER

Enter y  2x4 as Y .
3
Then graph.

[1, 5] scl: 1 by [5, 5] scl: 1

The solution is approximately (2.8, 0.1).


6. Enter y  0.35x  1.648 as Y1.

The graphs are the same shape. The graph of


y  2x  5 is the graph of y  2x translated 5 units
to the left. The graph of y  2x  4 is the graph of
y  2x translated 4 units to the right.
3. Enter y  2x as Y .
1
Enter y  3x as Y2.
Enter y  5x as Y3.
Then graph.

Enter y  0.2x2  0.28x  1.01 as Y .


2
Graph.
Approximate the first intersection point.
KEYSTROKES: 2nd [CALC] 5 ENTER ENTER

ENTER

[10, 10] scl: 1 by [10, 10] scl: 1

Use the TRACE feature and left arrow to move


the cursor near the other intersection point. Use
the intersect feature again.

All of the graphs cross the y-axis at 1. The graph


of y  3x is steeper than the graph of y  2x, and
the graph of y  5x is steeper yet.
4. Enter y  3  2x as Y .
1
Enter y  3(2x  1) as Y .
2
Enter y  3(2x  1) as Y3.
Then graph.

The solutions are approximately (3.8, 3.0) and


(3.5, 0.4).

10-5
Page 556

Exponential Functions
The graphs are the same shape. The graph of
y  3(2x  1) is the graph of y  3(2x) translated
3 units down. The graph of y  3(2x  1) is the
graph of y  3(2x) translated 3 units up.

Graphing Calculator Investigation

1. Enter y  2x as Y .
1
Enter y  2x  3 as Y .
2
Enter y  2x  4 as Y .
3
Then graph.

475

Chapter 10

PQ249-6481F-10[458-483] 26/9/02 6:24 PM Page 476 Sahuja Ahuja_QXP_06:Desktop Folder:Chandra:Algebra_FNL_Delivery:

Pages 557558

6. y  9x
Sample answer:

Check for Understanding

1. Never; there is no value of x for which ax  0.


2. Sample answer: y  2x
The y-intercept of the graph is 1. The graph
increases quickly for x  0.
y

9x

2

92

1
81

1

91

1
9

0
1
2

90
91
92

80
60
40

1
9
81
4

y  2x

3. Kiski; the graph of y 


4. y  3x
Sample answer:
x

3x

2

32

1
9

1

31

1
3

30

0
1
2
3

31
32
33

113 2x decreases as x increases.

35
30
25
20
15
10
5

1
3
9
27

4 3 2 1O
5

12

1 2 3 4x

2
1
0
1
2

12
114 22
114 21
114 20
114 21
114 22

16
4

()

y  14

1
1
4

56
48
40
32
24
16
8

4 3 2 1O
8

2
9

1

231

2
3

4x

230
231
2 32

56
48
40
32
24
16
8

2
6
18

4 3 2 1O
8

4(5x  10)

y  2 3x
1 2 3 4x

2

4(52

1

4(51  10)

395

4(50  10)
4(51  10)
4(52  10)

36
20
60

0
1
2

40

21

 10) 3925
1

20

1 2 3 4x
4

1
16

2

4x

20

The y-intercept is 1.

114 21.7  0.1

Chapter 10

2

232

y  9x

The y-intercept is 2.
8. y  4(5x  10)
Sample answer:

Sample answer:
1 x
4

2 3x

y  3x

1 x
4

0
1
2

The y-intercept is 1.
31.2  3.7
5. y 

2

20

The y-intercept is 1.
90.8  5.8
7. y  2  3x
Sample answer:

40

y  4(5x  10)

The y-intercept is 36.


9. Yes; the domain values are at regular intervals,
and the range values have a common ratio 6.
10. No; the domain values are at regular intervals,
and the range values have a common difference 4.
11. 264  1.84  1019 grains.

476

PQ249-6481F-10[458-483] 26/9/02 6:24 PM Page 477 Sahuja Ahuja_QXP_06:Desktop Folder:Chandra:Algebra_FNL_Delivery:

12. Divide the number of grains by the number of


grains per pound to find the number of pounds.
1.84  1019
2.4  104

10
11.84
2.4 2 1 10 2

16. y 

Sample answer:

19
4

1015

x
2
1
0

1014

 0.767 
or 7.67 
Now divide the number of pounds by 2000 of
2  103 to find the number of tons.
7.67  1014
2  103

115 2x

10
17.67
2 2 1 10 2

y
25
5
1

40
30

14
3

 3.84  1011
The man will receive about 3.84  1011 tons of
rice the last day.

1
5

1
25

20
x

(1)

y 5
4

2

10
O

4x

The y-intercept is 1.

Pages 558560

115 20.5  0.4

Practice and Apply

13. y  5x
Sample answer:
x

17. y  6x
Sample answer:
y

2

1
25

1

1
5

0
1
2

40
30
20

1
5
25
4

2

10

y  5x

2
1

1
10

40

10
2

y  10x
2

2

2

1
64

y  6x

1

1
8

80

4x

60
40
20
4

2

y  8x
2

4x

The y-intercept is 1.
80.8  5.3
19. y  5(2x )
Sample answer:

1101 2x

y
100
10
1

40

30
20

1
10
x

(1)

1
100

20

4x

Sample answer:

40

1
6
36

0 1
1 8
2 64

The y-intercept is 1.
100.3  2.0

60

4

20

4

x
2
1
0

1
6

4x

30

0
1
1
10
2 100

15. y 

1

80

The y-intercept is 1.
60.3  1.7
18. y  8x
Sample answer:

y
1
100

2

1
36

0
1
2

The y-intercept is 1.
51.1  5.9
14. y  10x
Sample answer:
x

y  10
4

2

2

1
14

1

22

0
1
2

10
O

4x

40
30

20

5
10
20

10
4

The y-intercept is 1.

1101 21.3  20.0

2

y  5(2x)
2

4x

The y-intercept is 5.

477

Chapter 10

PQ249-6481F-10[458-483] 26/9/02 6:24 PM Page 478 Sahuja Ahuja_QXP_06:Desktop Folder:Chandra:Algebra_FNL_Delivery:

20. y  3(5x )
Sample answer:
x

40

3
25

2

30

3
5

1
0
1
2

24. y  5(2x )  4
Sample answer:

20

3
15
75
4

2

2

8
69

30

10

2

1
44

2

4x

10

16

5
6
8

4
2

4x

1

3

0
1
2

1
5
17

y  2(3x)  1
O

The y-intercept is 1.

Chapter 10

4x

23

30

20

4
8
20

10

y  2(3x  1)
2

4x

4x

2

1
144

1

132

20

12
9
3

10
4

2

O
10

y  3(2x  5)

The y-intercept is 12.


27. No; the domain values are at regular intervals,
and the range values have a common difference 3.
28. Yes; the domain values are at regular intervals,
and the range values have a common ratio 0.5.
29. Yes; the domain values are at regular intervals,
and the range values have a common ratio 0.75.
30. No; the domain values are at regular intervals,
but the range values do not have a positive
common ratio.
31. No; the domain values are at regular intervals,
but the range values do not change.
32. Yes; the domain values are at regular intervals,
and the range values have a common ratio 0.5.

2

1

0
1
2

y  2x  4

y
7
9

2

40

4

The y-intercept is 5.
23. y  2(3x )  1
Sample answer:
x

2

2
29

y3 7

4

y  5(2x)  4

12

42

0
1
2

10

The y-intercept is 4.
26. y  3(2x  5)
Sample answer:

1

20

9
14
24

0
1
2

The y-intercept is 6.


22. y  2x  4
Sample answer:
x

30

4

20

4

62

The y-intercept is 9.
25. y  2(3x  1)
Sample answer:

1 63
0 6
1 4
2
2
3 20

1

40

4x

The y-intercept is 3.
21. y  3x  7
Sample answer:
x

2

1
54

0
1
2

y  3(5x )

10

478

PQ249-6481F-10[458-483] 26/9/02 6:24 PM Page 479 Sahuja Ahuja_QXP_06:Desktop Folder:Chandra:Algebra_FNL_Delivery:

33. T(x)  12(1.12)x


In 2005, x  10.
T(10)  12(1.12)10
 37.27
In 2005, the sales are expected to be about
$37.27 million.
In 2006, x 11.
T(11)  12(1.12)11
 41.74
In 2006, the sales are expected to be about
$41.74 million.
In 2007, x  12.
T(12)  12(1.12)12
 46.75
In 2007, the sales are expected to be about
$46.75 million.
34. T(x)  12(1.12)x
Sample answer:
x
0
1
2
3

T(x)
12
13.44
15.05
16.86

16

40. D(x)  20(1.1)x


Week
1
2
3
4

41. Continue to find D(x) for integers greater than 4.


A partial table is shown.
Week
9
10

42. Graph y  5x and y 

2

graph of y 

T (x )

4x

The y-intercept is 12.


35. The y-intercept represents the sales 0 years after
1995. There were $12 million in sales in 1995.
36. There are four 15-minute time periods in
one hour. Thus, the process will be completed
four times. Each time the number of bacteria
doubles. Thus, there are 100 (24) or 1600 bacteria
after one hour.
1

37. Since 3 of the schools remain after each round, we


1
multiply by 3 for each round.
y  729

115 2x is the graph of y  5x reflected

around the y-axis.


43. Graph y  5x and y  5x  2 in the same
coordinate plane or viewing rectangle. You will
see that the graph of y  5x  2 is the graph of
y  5x translated 2 units up.
44. Graph y  5x and y  5x  4 in the same
coordinate plane or viewing rectangle. You will
see that the graph of y  5x  4 is the graph of
y  5x translated 4 units down.
45. If the number of items on each level of a piece of
art is a given number times the number of items
on the previous level, an exponential function can
be used to describe the situation. Answers should
include the following.
For the carving of the pliers, y  2x.
For this situation, x is an integer between 0
and 8, inclusive. The values of y are 1, 2, 4, 8,
16, 32, 64, 128, and 256.

T (x )  12(1.12)x

115 2x in the same coordinate

plane or viewing rectangle. You will see that the

4
4

Distance(miles)
about 47.2
about 51.9

The runner will exceed 50 miles for the first time


in the 10th week.

12
8

Distance (miles)
22
24.2
26.62
29.282

113 2x

38. Let x  3.

113 23
1
 729 1 27 2

y  729

y  2x

 27
There are 27 schools after 3 rounds.
39. Consider y for various values of x. In particular,
let x  6.

46. B; f(x)  6x is an exponential function since the


variable is the exponent.
47. A; graph y  2x and y  6x in the same coordinate
plane or viewing rectangle. You will see that the
graph of y  6x is steeper than the graph of y  2x.

113 26
1
 729 1 729 2

y  729

1
After 6 rounds, there will be only one school
remaining. This school is the winner.

479

Chapter 10

PQ249-6481F-10[458-483] 26/9/02 6:24 PM Page 480 Sahuja Ahuja_QXP_06:Desktop Folder:Chandra:Algebra_FNL_Delivery:

Page 560

Maintain Your Skills

x

b  2b2  4ac
2a

(9)  2(9) 2  4(1) (36)


2(1)

9  1225
2
9  15
2
9  15
or
2


x

x

9  15
2

 3
 12
The solutions are 3, 12.
49. 2t2  3t  1  0
t

54.

b  2b2  4ac
2a

3  232  4(2) (1)


2(2)

3  117
4

t

3  117
4

t

or

55.

56.

3  117
4

 1.8
 0.3
The solutions are 1.8, 0.3.
50.

57.

5y2  3  y
23yyy
5y
5y2  y  3  0
y

b  2b2  4ac
2a

(1)  2(1) 2  4(5) (3)


2(5)

1  159
10

; Since the discriminant is negative, there are


no real solutions.
x2  7x  10

51.

49
4
7 2
2
7
x2
7
7
2
2

x2  7x 

1x

x

 10 
9
3

 2
3

 2  2
7

49
4

4

x2 
x22

or

x

3
2
7
2

2

58.

2
5
The solutions are 2, 5.
52.

t2  6t  3  0

53.

48. x2  9x  36  0

a2  12a  3
a  12a  36  3  36
(a  6) 2  39
a  6  ; 139
2

60.

t  6t  3  3  0  3
t2  6t  3
t2  6t  9  3  9
(t  3) 2  6
t  3  ; 16
t  3  3  ; 16  3
t  3  16
t  3  16 or t  3  16
 5.4
 0.6
The solutions are 5.4, 0.6.
Two numbers whose product is 40 and whose sum
is 14 are 10 and 4.
m2  14m  40  (m  10)(m  4)
Among all pairs of integers whose product is 35,
there are no pairs whose sum is 2. Thus,
t2  2t  35 is prime.
A pair of numbers whose product is 24 and
whose sum is 5 and 8 and 3.
z2  5z  24  (z  8)(z  3)
Let n be the first number.
Let m be the second number.
Solve the following system.
3n  2m
2n  m  3
Solve the second equation for m.
2n  m  3
2n  3  m
Substitute 2n  3 for m in the first equation and
solve for m.
3n  2m
3n  2(2n  3)
3n  4n  6
3n  4n  6
n  6
n6
Use m  2n 3 to find m.
m  2n  3
 2(6)  3 or 9
The numbers are 6 and 9.
59.
x7 7 2
10 x  8
x77 7 27
10  8 x  8  8
x 7 5
2 x
{x|x 7 5}
{x|x
2}
y  7 6 12
y  7  7 6 12  7
y 6 5
{y|y 6 5}

a  6  6  ; 139  6

1
61. p(1  r) t  5 1  2 2

a  6  139
a  6  139 or a  6  139
 0.2
 12.2
The solutions are 0.2, 12.2.

 5(1.5)
 5(2.25)
 11.25

1
62. p(1  r) t  300 1  4 3

300(1.25) 3


 300(1.953125)
 585.9375

Chapter 10

480

PQ249-6481F-10[458-483] 26/9/02 6:24 PM Page 481 Sahuja Ahuja_QXP_06:Desktop Folder:Chandra:Algebra_FNL_Delivery:

63. p(1  r) t  100(1  0.2) 2


 100(1.2) 2
 100(1.44)
 144

5. y  5x  4
Sample answer:

64. p(1  r) t  6(1  0.5) 3


 6(1.5) 3
 6(3.375)
 20.25

Page 560
1.

2

24
325

1

35

3
1
21

0
1
2

y  5x  4

Practice Quiz 2

x2  2x  35
2  2x  35  35  35
x
x2  2x  35  0
x

The y-intercept is 3.

b  2b2  4ac
2a

2  222  4(1) (35)


2(1)

2  1144
2
2  12
2
2  12
x
2


2  12

10-6 Growth and Decay


Page 563

or
x
2
 7
5
The solutions are 7, 5.
2. 2n2  3n  5  0
n

b  2b2  4ac
2a

(3)  2(3) 2  4(2) (5)


2(2)

3  131
4

Check for Understanding

1. Exponential growth is an increase by the same


percent over a period of time, while exponential
decay is a decrease by the same percent over a
period of time.
2. Determine the amount of the investment if $500
is invested at an interest rate of 7% compounded
quarterly for 6 years.
3. Sample answer:
y

; Since the discriminant is negative, there are


no real solutions.
3.

2v2  4v  1
 4v  1  1  1
2v2  4v  1  0
2v2

v

v

4  124
4

b  2b2  4ac
2a

(4)  2(4) 2  4(2) (1)


2(2)

4  124
4

or

v

r) t

4. I  C(1 
I  37,060(1  0.005) t
I  37,060(1.005) t
5. First find t.
t  2009  1979 or 30
I  37,060 (1.005) 30
 43,041
The median household income in 2009 is expected
to be $43,041.

4  124
4

 0.2
 2.2
The solutions are 0.2, 2.2.
4. y  0.5(4x )
Sample answer:
x
2
1
0
1
2

y
0.01325
0.125
0.5
2
8

r
6. A  P 1  n nt

A  400 1 

A  400(1.018125) 28
A  661.44
The investment will be about $661.44.

y  0.5(4x)
O

0.0725 47
4

The y-intercept is 0.5.

481

Chapter 10

PQ249-6481F-10[458-483] 26/9/02 6:24 PM Page 482 Sahuja Ahuja_QXP_06:Desktop Folder:Chandra:Algebra_FNL_Delivery:

16. y  C(1  r) t
y  2,405,000(1  0.011) t
y  2,405,000(0.989) t
In 2015, t  2015  2000 or 15.
y  2,405,000(0.989) 15
y  2,037,321
In 2015, the population of Latvia will be about
2,037,321.

7. Write an equation.
y  C(1  r) t
y  1,821,000(1  0.002) t
y  1,821,000(0.998) t
Find t.
t  2010  1995 or 15
y  1,821,000(0.998) 15
y  1,767,128
The population of West Virginia will be about
1,767,128 in 2010.
8. y  C(1  r) t
y  16,000(1  0.18) 8

17. y  C(1  r) t
y  2,976,400,000(1  0.186) t
y  2,976,400,000(0.814) t
In 2009, t  2009  1994 or 15.
y  2,976,400,000(0.814) 15
y  135,849,289
In 2009, the sales of cassettes will be about
$135,849,289.
18. y  C(1  r) t
y  71,601(1  0.0563) t
y  71,601(1.0563) t
In 2020, t  2020  1920 or 100.
y  71,601(1.0563) 100
y  17,125,650
There will be about 17,125,650 visitors in 2020.
19. y  C(1  r) t
y  25,000(1  0.10) 8
y  25,000(0.9) 8
y  10,761.68
After 8 years, the equipment will be worth about
$10,761.68.
20. y  C(1  r) t
y  23,000(1  0.12) 5
y  23,000(0.88) 5
y  12,137.83
In 5 years, the value of the car will be about
$12,137.83.
21. In 2010, t  2010  1900 or 110.
P  3.86(1.013) 110
P  15.98
In 2010, about 15.98% of the population will be 65
or older.

y  16,000(0.82) 8
y  3270.63
After 8 years, the value of the car will be about
$3270.63.

Pages 563565

Practice and Apply

9. y  C(1  r) t
y  18.9(1  0.19) t
y  18.9(1.19) t
10. In 2015, t  2015  1980 or 35.
y  18.9(1.19) 35
y  8329.24
In 2015, there will be about 8329.24 computers.
11. W  C(1  r) t
W  43.2(1  0.06) t
W  43.2(1.06) t
12. In 2007, t  2007  1997 or 10.
W  43.2(1.06) 10
W  77.36
In 2007, about 77.36 million people will be using
free weights.
13. Write an equation.
y  C(1  r) t
y  100,350,000(1  0.017) t
y  100,350,000(1.017) t
In 2012, t  2012  2000 or 12.
y  100,350,000(1.017) 12
y  122,848,204
In 2012, the population of Mexico will be about
122,848,204.

14. A  P 1 

r nt
t

A  500 1 

22.

A  250 1

r nt
t
0.103
 4 440

A  14,607.78
The amount will be about $14,607.78.

Chapter 10

P
19.91
20.16

The percentage of the population 65 or older will


first be over 20% when t  128 or about 2028.
23. This equation represents growth since 1.026 7 1.
1  r  1.026
r  0.026
The annual rate of change is 2.6%.
24. This equation represents decay, since 0.761 6 1.
1  r  0.761
r  0.239
r  0.239
The annual rate of change is 2.39%.

0.0575 1225
12

A  2097.86
The amount will be about $2097.86.
15. A  P 1 

t
127
128

482

PQ249-6481F-10[458-483] 26/9/02 6:24 PM Page 483 Sahuja Ahuja_QXP_06:Desktop Folder:Chandra:Algebra_FNL_Delivery:

5730

34. y  2x  5
Sample answer:
x
y

25. y  256(0.5) 5730


y  256(0.5) 1
y  128
There are 128 grams of Carbon-14 remaining.
1000

26. y  256(0.5) 5730


y  226.83
There are about 226.83 grams of Carbon-14
remaining.

2

3
44

1

1
42

0
1
2

10,000

27. y  256(0.5) 5730


y  76.36
There will about 76.36 grams of Carbon-14
remaining.
28.

t
17,190

The organism lived 17,190 years ago.


29. See students work.
30. If the sales are growing by the same percent each
year, an exponential equation can be used to
model sales and predict future sales. Answers
should include the following.
The equation states that the new value equals
the amount in the year 1994, or $1698 times
the sum of 1 plus 4.6% raised to the power,
that is equal to the number of years since
1994.
According to the equation, the average family
will spend about $3486.94 for restaurant
meals in 2010.
31. C; In the equation y  35(1.05x ), the exponent is
the variable and the base is greater than 1.

33. y 

12

2
1

223

20

y  4(3x  6)

b  2b2  4ac
2a
(9)  2(9) 2  4(1) (10)
2(1)

9  11
2

 1
 10
The solutions are 1, 10.
2t2  4t  3
37.
2t2  4t  3  3  3
2t2  4t  3  0

16

12

t

8
4
2

4x

9  1121
2
9  11
 2
9  11
or m  2

t

4

Maintain Your Skills

1
64

2

10

m

1
8

4

20
12
12

Sample answer:

10

m

y
64
8
1

The y-intercept is 20.


36. m2  9m  10  0

1 x
8

x
2
1
0

5
239

0
1
2

0.0825 4.4
4

A  6931.53

Page 565

4
3
1

r nt

A  5000 1 

y  2x  5

The y-intercept is 4.


35. y  4(3x  6)
Sample answer:
x
y

y
32

32. D; A  P 1  n

(4)  2(4) 2  4(2) (3)


2(2)

4  140
4

or

t

4  140
4

 0.6
 2.6
The solutions are 0.6, 2.6.
38. 7x2  3x  1  0

(1)

y 8

4  140
4

b  2b2  4ac
2a

x

4x

The y-intercept is 1.

3  232  4(7) (1)


2(7)
3  119
14

; There are no real solutions since the


discriminant is negative.
39. m7 (m3b2 )  (m7m3 )b2
 m10b2

483

Chapter 10

40. 3(ax3y) 2  3a2 (x3 ) 2y2


 3a2x6y2

4a. Angela bought a car for $18,500. If the rate of


depreciation is 11%, find the value of the car in
4 years.

41. (0.3x3y2 ) 2  0.32 (x3 ) 2 (y2 ) 2


 0.09x6y4
42. |7x  2|  2
; There are no solutions since absolute value is
never negative.
43. |3x  3|  0
3x  3  0
3x  3
x1
The solution is 1.
44.
t  4  3
or
t  4  3
t43
t  4  4  3  4
t4434
t  1
t  7
{t|t  7 or t  1}
45. slope 


4b. y  C(1  r) t; 18,500(1  0.11) 4 or about


$11,607.31
5a. The population of Centerville is increasing at an
average annual rate of 3.5%. If its current
population is 12,500, predict its population in
5 years.
5b. y  C(1  r) t; y  12,500(1  0.035) 5 or about
14,846 people

Geometric Sequences

10-7
Page 569

2
1

 0.24
Yes, the hill meets the requirements since
0.24 6 0.33.
46. 8
11 14 17
?

2

The common ratio is 2.


16


3
3
3
3

2
1

Add 1.1 three more times. The next three terms


are 5.9, 7.0, and 8.1.

Reading Mathematics

 2

O
8

16

r nt

2. A  P 1  n ; since the final amount is greater


than the initial amount, P is multiplied by a
number greater than 1. Also, the annual rate is
divided by the number of times it is compounded
per year. Time is equal to nt because this is the
total number of times that the interest is
compounded over the course of t years.
3a. Suppose that $2500 is invested at an annual
rate of 6%. If the interest is compounded
quarterly, find the value of the account after
5 years.
$3367.14

Chapter 10

; A  2500 1 

0.06 4(5)
4

an

1. y  C(1  r) t; Since the final amount is less than


the initial amount, the initial amount is
multiplied by a number less than 1. If an amount
is decreased by r percent, then 1  r percent will
remain.

The common ratio is 2.


16

r nt

2. 1, 2, 4, 8, 16, . . .


Divide the second term by the first.


1.1 1.1 1.1 1.1

16

Add 3 three more times. The next three terms


are 5, 8, and 11.
48. 1.5
2.6 3.7 4.8 ?

3b. A  P 1  n

8


3
3
3
3

an

Add 3 three more times. The next three terms are


20, 23, and 26.
47. 7
4
1 2
?

Page 566

Algebra Activity

1. 1, 2, 4, 8, 16, . . .
Divide the second term by the first.

rise
run
60
250

or about

484

7. Graphs 1, 3, and 5those with positive common


factorsappear to be similar to an exponential
function.
8. Both types of graphs show rapid change. If r 7 0,
the graph of the geometric sequence is similar to an
exponential function. If r 6 0, the graph of the
geometric sequence has points above and below the
n-axis and is not similar to an exponential function.

3. 81, 27, 9, 3, 1, . . .
Divide the second term by the first.
27
81

3
1

The common ratio is 3.


80

an

40
O

9. The two values c(ax ) and a rn1 are very similar.


1
Both have a number multiplied by another
number that is raised to a power.

40

Page 570

80

4. 81, 27, 9, 3, 1, . . .


Divide the second term by the first.
27
81

 3
1

The common ratio is 3.


80

an

40
O

40
80

5. 0.2, 1, 5, 25, 125, . . .


Divide the second term by the first.
1
0.2

5

The common ratio is 5.


100


3
3
3

an

Yes; the common ratio is 3.


5. 56 28 14 7

50
O


1
1
1
 1 2 2  1 2 2  1 2 2

6. 25

15

10

No; the difference between successive terms is


constant. This sequence is arithmetic, not
geometric.
7. 5, 20, 80, 320, . . .
Divide the second term by the first.

6. 0.2, 1,5, 25,125, . . .


Divide the second term by the first.
 5

The common ratio is 5.

20
5

an

4

The common ratio is 4.


Multiply by 4 three more times. The next three
terms are 1280, 5120, and 20,480.
8. 176, 88, 44, 22, . . .
Divide the second term by the first.

50
O

20


5
5
5

100

100

Yes, the common ratio is 2.

50

1
0.2

Check for Understanding

1. Both arithmetic sequences and geometric


sequences are lists of related numbers. In an
arithmetic sequence, each term is found by
adding the previous term to a constant called the
common difference. In a geometric sequence, each
term is found by multiplying the previous term by
a constant called the common ratio.
2. If a common ratio equals 0, all of the terms except
possibly the first term will equal 0, since any
number times 0 equals 0. If a common ratio
equals 1, all of the terms will equal the first term,
since 1 is the multiplicative identity.
3. Sample answer:
1, 4, 9, 16, 25, 36, . . .
The difference between the first and second terms
is 4  1 or 3, and the difference between the
second and third terms is 9  4 or 5. Since these
are not equal, the sequence is not arithmetic. The
4
9
corresponding ratios are 1 or 4 and 4. Since these
are not equal, the sequence is not geometric.
4. 5 15 45 135

50

88
176

100

 2 or 0.5

The common ratio is 0.5.


Multiply by 0.5 three more times. The next
three terms are 11, 5.5, and 2.75.

485

Chapter 10

9. 8, 12, 18, 27, . . .


Divide the second term by the first.
12
8

18. 7

The common ratio is 1.5.


Multiply by 1.5 three more times. The next
three terms are 40.5, 60.75, and 91.25.

No; the difference between consecutive terms is


constant. This sequence is arithmetic, not
geometric.
20. 640 160 40 10

a6  (1) 25
a6  (1) 32
a6  32


1
1
1
40

12. a  a rn1
n
1
a7  4 (3) 71
a  4
7

21. 36

3

Yes; the common ratio is 4.5.


25. 1, 4, 16, 64, . . .
Divide the second term by the first.
4
1

r

114 2  12 or

6
1

4r2
4

25  r2
5  r
The geometric mean is (4)(5)  20 or
(4) (5)  20.

512
1024

112
224

 0.5

Multiply by 0.5 three more times. The next three


terms are 14, 7, and 3.5.
29. 80, 20, 5, 1.25, . . .
Divide the second term by the first.

The perimeter of the smallest triangle is 7.5 cm.

20
80

 0.25

Multiply by 0.25 three more times. The next


three terms are 0.3125, 0.078125, and
0.01953125.

Practice and Apply


24


3
3
3

Yes; the common ratio is 3.

Chapter 10

 0.5

Multiply by 0.5 three more times. The next three


terms are 64, 32, and 16.
28. 224, 112, 56, 28, . . .
Divide the second term by the first.

Perimeter
120cm
120(0.5) or 60 cm
60(0.5) or 30 cm
30(0.5) or 15 cm
15(0.5) or 7.5 cm

Pages 571572

6

Multiply by 6 three more times. The next three


terms are 1296, 7776 and 46,656.
27. 1024, 512, 256, 128, . . .
Divide the second term by the first.

100  (4) r2

Triangle
Largest
Next largest
Next largest
Next largest
Smallest

 4

Multiply by 4 three more times. The next three


terms are 256, 1024, and 4096.
26. 1, 6, 36, 216, . . .
Divide the second term by the first.

a3  a1 r31

18

21

3

90
405 1822.5

(4.5) (4.5) (4.5)

 r2

3

23. 20

48r2
48

1 2

17. 2

63



1
1

The geometric mean is 48


1
48 4  12.

16.

16

Yes; the common ratio is 3.

7r2
7

189


1

3  48 r2

100
4

25

22. 567

4  r2
2  r
The geometric mean is 7(2)  14 or 7(2)  14.
14. a3  a1 r31

15.

16

1
.
40

No; the ratios are not the same.

25

36

13. a3  a1 r31
28  7 r2

3
48
1
16
1
4

40


25
16
9

(3) 6

a  2916

40

Yes; the common ratio is

a  4 729
7

28
7

37


3
3
3

11. a  a rn1
n
1
a6  (1) 261

a5  3 44
a5  3 256
a5  768

27

No; the difference between consecutive terms is


constant. This sequence is arithmetic, not
geometric.
19. 19 16 13 10

 2 or 1.5

10. a  a rn1
n
1
a5  3 451

17


10 10 10

486

30. 10,000, 200, 4, 0.08, . . .


Divide the second term by the first.
200
10,000

41. a  a rn1
n
1
a10  300 (0.5) 101

 0.02

a10  300 (0.5) 9


a10  300 0.001953125 or 0.5859375

Multiply by 0.02 three more times. The next


three terms are 0.0016, 0.000032, and
0.00000064.
1 1 2

42. a  a rn1
n
1
a6  14 (1.5) 61

31. 2, 3, 9, 27, . . .

a6  14 (1.5) 5
a  14 7.59375 or 106.3125

Divide the second term by the first.


1
3
1
2

43. a  a r31
3
1

3

20  5 r2

2
3

Multiply by
terms are

20
5

three more times. The next three

8 16
,
,
81 243

and

5r2
5

4  r2
2  r
The geometric mean is 5(2)  10 or 5(2)  10.

32
.
729

3 1 1 2

32. 4, 2, 3, 9, . . .
Divide the second term by the first.
1
2
3
4

44. a  a r31
3
1
54  6 r2

3

54
6

Multiply by

2
3

16

terms are 27, 81, and 243.


33. The area of the original rectangle is 6 8 or
48 square inches. Multiply by 0.5 four times to
find the areas of the next four rectangles. The
areas of the five rectangles are 48 in2, 24 in2,
12 in2, 6 in2, and 3 in2.
34. Multiply the measurement of the first angle by
0.5 three times to find the measurements of the
other three angles. The measurements are 160
,
80
, 40
, and 20
.
35. a  a rn1
n
1
a7  5 271

45.

a3  a1 r31
225  (9) r2
225
9

9r2
9

25  r2
5  r
The geometric mean is (9) 5  45 or
(9) (5)  45.
46.

36. a  a rn1
n
1
a5  4 351

a7  5 26
a7  5 64 or 320

6r2
6

9  r2
3  r
The geometric mean is 6(3)  18 or 6(3)  18.

three more times. The next three


8

a3  a1 r31
80  (5) r2
80
5

a5  4 34
a5  4 81 or 324

5r2
5

16  r2
4  r
The geometric mean is (5)4  20 or
(5)(4)  20.

37. a  a rn1
n
1
a4  (2) (5) 41

a4  (2) (5) 3
a4  (2) (125) or 250

47. a  a r31
3
1
8  128 r2

38. a  a rn1
n
1
a6  3 (4) 61

8
128
1
16
1
4

(4) 5

a6  3
a6  3 (1024) or 3072

39. a  a rn1
n
1
a3  (8) 631

128r2
128

 r2
r

1 2  32.

The geometric mean is 128


128

1
4

114 2  32 or

48. a  a r31
3
1
5  180 r2

a3  (8) 62
a3  (8) 36 or 288

5
180
1
36
1
6

40. a  a rn1
n
1
a8  (10) 281
27

a8  (10)
a8  (10) 128 or 1280

180r2
180

 r2
r

1 12

The geometric mean is 180


180 6  30.

487

116 2  30 or
Chapter 10

49.

a3  a1 r31
98  (2) r2
98
2

58. Multiply 848 by 0.75 six times to find the level on


the next six days. The levels during the first week
are 848, 636, 477, 357.75, 268.3125, 201.234375,
and 150.9257813 parts per million.

2r2
2

49  r2
7  r
The geometric mean is (2)(7)  14 or
(2) (7)  14.
50.

59. a  848(0.75) 14 or about 15.1


15
a16  848(0.75) 15 or about 11.3
The lake will be safe in 16 days.
60. Sample answer: Although the amount of pesticide
will become very small, 75% of something will
always be something. Therefore, there will always
be pesticides according to this model.

a3  a1 r31
384  (6) r2
384
6

6r2
6

61. Always; if each term of a geometric sequence


whose nth term is a rn1 is multiplied by a

64  r2
8  r
The geometric mean is (6)(8)  48 or
(6) (8)  48.
51.

nonzero real number c, the nth term of the new


sequence is c(a rn1 ) or (c a )rn1, which is a
1
1
geometric sequence.

a3  a1 r31

62. Never; if the same nonzero real number is added


to each term of a geometric sequence, the
resulting sequence will have an nth term of the
a1rn1  b, which is not in the form of a
geometric sequence.
3
63. Since the distance of each bounce is 4 times the
distance of the last bounce, the list of the
distances from the stopping place is a geometric
sequence. Answers should include the following.

1.75  7 r2
1.75
7

7r2
7

0.25  r2
0.5  r
The geometric mean is 7(0.5)  3.5 or
7(0.5)  3.5.
52.

a3  a1 r31
0.75  3 r2
0.75
3

To find the 10th term, multiply the first term


3
80 by 4 to the 9th power.
The 17th bounce will be the first bounce less
than 1 ft from the resting place.

3r2
3

0.25  r2
0.5  r
The geometric mean is 3(0.5)  1.5 or
3(0.5)  1.5.
53.

100

64. C; The common ratio is 40 or 2.5. To find the next


term, multiply 625 by 2.5. The next term is
1562.5.

a3  a1 r31
3
20
5 3
3 20
1
4
1
2

 5 r2

49

1 2 1 2

The geometric mean is


54.

a3  a1 r31
2
45
5 2
2 45
1
9
1
3

1 2

66. The terms are getting closer to 0.


67. The limit is 0.

 r2
r

1 2  103 or 35 112 2  103 .

68. If 0 6 r 6 1, the nth term will approach 0. If


r 7 1, the nth term will approach infinity.

3 1
5 2

 5 r2

1 2

Page 572

5 2 2
r
5

2


Maintain Your Skills

r
69. A  P 1  n n  t

r2

r

65. The common ratio is 343 or 7. To find the next


1
1
term, multiply 1 by 7. The next term is 7.

5 3 2
r
3 5

A  1500 1 

1 2  152 or 25 113 2  152 .

0.065 12  3
12

55. Multiply 10 by 0.6 three times. The heights of


three rebounds are 6 m, 3.6 m, and 2.16 m.
56. Multiply 10 by 2 five times to find the next
five scores. The first six scores are 10, 20, 40, 80,
160, and 320.

 1822.01
The value of the investment will be about
$1822.01.
70. No; the domain values are at regular intervals,
but the range values have a common difference 2.
71. Yes; the domain values are at regular intervals,
and the range values have a common factor 3.

57. a  10 216 or 655,360


17
a18  10 217 or 1,310,720

72. 7a2  22a  3  7a2  a  21a  3


 a(7a  1)  3(7a  1)
 (7a  1) (a  3)

The geometric mean is

2 1
5 3

The score will be greater than a million when


18 questions are answered correctly.

Chapter 10

488

73. 2x2  5x  12  2x2  3x  8x  12


 x(2x  3)  4(2x  3)
 (2x  3)(x  4)

Richter Number
(x)

74. 3c2  3c  5 is prime.

Page 573

Algebra Activity:
(Follow-Up of Lesson 10-7)

1. The graph begins by increasing slowly and then


increases rapidly for the last few values.
1,000,000
900,000
800,000
700,000
600,000
500,000
400,000
300,000
200,000
100,000
0

0.00017

0.006

0.179

0.173

4.821

0.00583

179

174

5643

5464

179,100

173,357

The regression equation is y  5.54  10 6 (31.7x ) .

Chapter 10 Study Guide and Review


Page 574
1.
2.
3.
4.
5.
6.
7.
8.
9.
10.

0 1 2 3 4 5 6 7x

y
200,000
180,000
160,000
140,000
120,000

Vocabulary and Concept Check

d; exponential growth equation


g; quadratic function
i; symmetry
a; equation of axis of symmetry
c; exponential function
i; vertex
b; exponential decay equation
h; roots
f; Quadratic Formula
e; parabola

Pages 574578

Lesson-by-Lesson Review

11. In y  x2  2x, a  1 and b  2.

100,000
80,000

x
x

60,000
40,000

0 1 2 3 4 5 6 7 x

The graph begins increasing slowly and then


increases rapidly for the last few values. Use the
slope formula to find the values for the table.
0.006  0.00017
2  1
0.179  0.006
3  2
5  0.179
4  3
179  5
5  4
5643  179
6  5
179000  5643
7  6

b
2a
2
2(1)

or 1

x  1 is the equation of the axis of


symmetry.
y  (1) 2  2(1)
12
 1
The vertex is (1, 1).
Since the coefficient of the x2 term is positive, the
vertex is a minimum.

20,000
0

Rate of Change
(slope)

There is no constant value that can be multiplied


by each rate of change to obtain the next value.

2. No.
3. No; the rate of change between any two points is
always a different value.
4. To move from one rate of change to the next, you
multiply by 10.
5. The regression equation is y  0.1(10x ) .
6.

Energy Released
(y)

 0.00583

 0.173
 4.821
 174
 5464

 173,357

y  x  2x

489

Chapter 10

12. In y  3x2  4, a  3 and b  0.


x
x

b
2a
0
2(3)

Since the coefficient of the x2 term is positive, the


vertex is a minimum.

or 0

8
4

x  0 is the equation of the axis of symmetry.


y  3(0)  4
4
The vertex is (0, 4).
Since the coefficient of the x2 term is negative, the
vertex is a maximum.

6 5 4 3 2 1O
4
8
12
16
20
24
y  3x 2  6x  17

x
x

13. In y  x2  3x  4, a  1 and b  3.


x

The

12

x  2 is the equation of the axis of symmetry.


y

or 0

y  2x 2  1 y

or 2

b
2a
0
2(2)

x  0 is the equation of the axis of symmetry.


y  2(0) 2  1
1
The vertex is (0, 1).
Since the coefficient of the x2 term is negative, the
vertex is a maximum.

b
2a
(3)
2(1)

1 2x

15. In y  2x2  1, a  2 and b  0.

y  3x 2  4

x

12

3 2
3
 2 32 4
9
9
424
25
 4
3 25
vertex is 2, 4 .

Since the coefficient of the x2 term is positive, the


vertex is a minimum.

16. In y  x2  3x, a  1 and b  3.


x

x

b
2a
(3)
2(1)

or 2

x  2 is the equation of the axis of symmetry.


y



y  x 2  3x  4

132 22  3132 2

9
4
9
4

2

The vertex is
14. In y 

3x2

 6x  17, a  3 and b  6.

x

b
2a
6
2(3)

or 1

x

132, 94 2.

Since the coefficient of the x2 term is negative, the


vertex is a maximum.
y

y  x 2 3x

x  1 is the equation of the axis of symmetry.


y  3(1) 2  6(1)  17
 3  6  17
 20
The vertex is (1, 20).

Chapter 10

490

17. Graph f(x)  x2  x  12.


Sample answer:
x
3
1
1
3
4

f(x)
0
10
12
6
0

21.

2
4 3 2 1O
2
4
6
8
10
12
14

f (x )
1 2 3 4x

x
3
4
5
6
7

f (x )  x 2  x  12

The solutions are 3 and 4, the x-intercepts of


the graph.

22.

6x2  13x  15
 13x  15  15  15
6x2  13x  15  0
Graph f(x)  6x2  13x  15.
Sample answer:

x
1
0
1
2
3

The solution is 3, the x-intercept of the graph.


19. Graph f(x)  x2  4x  3.
Sample answer:
f(x)
2
3
7
3
2

f(x)
4
15
22
17
0

f (x )

1O 1 2 3 4 5 6 7 x
4
8
12
16
20
24
2
28 f (x )  6x  13x  15

f (x )
x

One root is 3, and the other root is between 1


and 0.
23.

3x2  4  0
3x2  4  4  0  4
3x2  4
4

x2  3

f (x )  x 2  4x  3

One root is between 5 and 4, and the other


root is between 0 and 1.

x  3 3
x  1.2
The solution set is {1.2, 1.2}.

20. Graph f(x)  2x2  5x  4.


Sample answer:
x
1
0
1
2
3

6x2

f (x )  x 2  6x  9

x
5
4
2
0
1

f (x )

The solutions are 3 and 7, the x-intercepts of the


graph.

f (x )

f(x)
4
1
0
1
4

f(x)
0
3
4
3
0

f (x )  x 2  10x  21

18. Graph f(x)  x2  6x  9.


Sample answer:
x
5
4
3
2
1

x2  10x  21
x  10x  21  21  21
x2  10x  21  0
Graph f(x)  x2  10x  21.
Sample answer:
2

f(x)
11
4
1
2
7

24.

f (x )  2x 2  5x  4
O

x2  16x  32  0
x  16x  32  32  0  32
x2  16x  32
2  16x  64  32  64
x
(x  8) 2  32
x  8  132
x  8  8  132  8
x  8  132
x  8  132 or x  8  132
x  2.3
x  13.7
The solution set is {2.3, 13.7}.
2

f (x )

; The graph has no x-intercepts. Therefore, the


equation has no real solutions.

491

Chapter 10

m2  7m  5

25.

m2 

49
7m  4
7 2
m2
7
m2

29.

5


69
4

x

69

 3 4

69

m  2  2  3 4  2
m
m

7
2

69
34

7
2

or

69
34

7
2

m


69
34

x

4a2  16a  15  0
1
(4a2
4

 16a  15) 
a2  4a 

a2  4a 

15
4

15
4
15
4

30. r2  10r  9  0

1
(0)
4

r

0

0

15
4

15

a2  4a   4
a2

 4a  4 
2) 2

(a 

a2
a22
a
a  2 

1
2

15
4  4
1
4
1
2
1
2  2
1
2  2

r

31.

 2 or 1.5

1 2
y  2y  1
2
1 2
y  2y  1
2

0

2  2(0)

p

y2  4y  2  0
2  4y  2  2  0  2
y
y2  4y  2
2  4y  4  2  4
y
(y  2) 2  6
y  2  16
y  2  2  16  2
y  2  16

32.

5
4

1n  32 22  1

4

n  2  1
3

n  2  2  1  2
n  1.5  1
n  1.5  1 or n  1.5  1
 0.5
 2.5
The solution set is {0.5, 2.5}.
Chapter 10

8  282  4(2) (3)


2(2)

8  140
4
8  140
4

or

y

8  140
4

 3.6
 0.4
The solution set is {3.6, 0.4}.

n2  3n  4
9

b  2b2  4ac
2a

y

n2  3n  4  4  0  4
n2  3n  4 

2y2  3  8y
 3  8y  8y  8y
2y2  8y  3  0
y

5
5

4  242  4(4) (15)


2(4)
4  1256
8
4  16
8
4  16
4  16
or p 
8
8

2y2

n2  3n  4  0
5

b  2b2  4ac
2a

 2.5
 1.5
The solution set is {2.5, 1.5}.

y  2  16 or y  2  16
 4.4
 0.4
The solution set is {4.4, 0.4}.
28.

4p2  4p  15
4p  4p  15  15  15
4p2  4p  15  0
p

The solution set is {2.5, 1.5}.


27.

10  2102  4(1) (9)


2(1)
10  164
2
10  8
2
10  8
10  8
or r 
2
2

 2 or 2.5

b  2b2  4ac
2a

 9
 1
The solution set is {9, 1}.

a  2  2

or

b  2b2  4ac
2a
(8)  2(8) 2  4(1) (20)
2(1)
8  1144
2
8  12
2
8  12
8  12
or x  2
2

 2
 10
The solution set is {2, 10}.

 0.7
 7.7
The solution set is {0.7, 7.7}.
26.

x2  8x  20
x  8x  20  20  20
x2  8x  20  0
2

49
4

492

33.

2d2  8d  3  3
2d  8d  3  3  3  3
2d2  8d  0
d

8  282  4 (2) (0)


2(2)

8  164
4
8  8
4
8  8
or
4


d

d

5  1613
42

or

 0.7

 5500(1.004375) 180
 12,067.68
The final amount is $12,067.68.

a

5  1613
42

14
12
10
8
6
4
2

3 2 1O
2

43. an  a1 rn1
2 3
a4  7 3

12
8
 7 1 27 2

1 2 3 4 5x

y  3x  2

4
2
1

1
2

243
81

a3  a1 r31

45.

20  5r2
1
(20)
5

 r2

4  r2
2  r
The geometric mean is 5(2)  10 or 5(2)  10.

1 2x

a3  a1 r31

46.

48  12r2
1
(48)
12

Sample answer:

1
0
1

1 2
1
 243 1 81 2
3

y
14
12
10
8
6
4
2

12

44. an  a1 rn1
1
a3  243 3 4

56

1 x
2

2

0.0975 365 40


365

 27

1
3
9
27

42. an  a1 rn1
a5  2(24 )
 2(16)
 32

y  3x  6

1
3

0.075 12  25
12

 500(1.000267123) 14,600
 $24,688.36
The final amount is $24,688.36.

O
The y-intercept is 9. 6 5 4 3 2 12

37. y  2

A  500 1 

36. y  3x2
Sample answer:
x
y
2
1
0
1

r
41. A  P 1  n nt

 0.5

1
69
1
63

3

r nt

 15,000(1.00625) 300
 97,243.21
The final amount is $97,243.21.

The solution set is {0.7, 0.5}.


35. y  3x  6
Sample answer:
x
y

0
7
1
9
2 15
The y-intercept is 7.

0.0525 12 15
12

A  15,000 1 

1

40. A  P 1  n

5  252  4(21) (7)


2(21)

2

A  5500 1 
8  8
4

b  2b2  4ac
2a

5  1613
42

32

r
39. A  P 1  n nt

34. 21a2  5a  7  0

a

0.08 4 8
4

 2000(1.02)
 3769.08
The final amount is $3769.08.

 4
0
The solution set is {4, 0}.
a

A  2000 1 

b  2b2  4ac
2a

r
38. A  P 1  n nt

1
(12r2 )
12

4  r2
2  r
The geometric mean is 12(2)  24 or
12(2)  24.

47. a  a r31
3
1
1
4
1
2

( 12 )x

y2

 1r2
r

The geometric mean is 1

The y-intercept is 2.

493

112 2  12 or 1112 2  12.


Chapter 10

6. In y  2x2  3, a  2 and b  0.

Chapter 10 Practice Test

x

Page 579
1.
2.
3.
4.

c; Quadratic Formula
b; exponential growth equation
a; exponential decay equation
In y  x2  4x  13, a  1 and b  4.
x


b
2a
(4)
2(1)

b
2a
0
2(2)

or 0

x  0 is the equation of the axis of symmetry.


y  2(0) 2  3
3
The vertex is (0, 3).
Since the coefficient of the x2 term is positive, the
vertex is a minimum.

or 1

x  2 is the equation of the axis of symmetry.


y  22  4(2)  13
 4  8  13
9
The vertex is (2, 9).
Since the coefficient of the x2 term is positive, the
vertex is a minimum.

y  2x 2  3
O

14
12
10
8
6
4
2

2
7. y  1(x  2)  1
y  (x2  4x  4)  1
y  x2  4x  4  1
y  x2  4x  3
a  1 and b  4

y  x 2  4x  13

2 1O
2

1 2 3 4 5 6x

x


5. In y  3x2  6x  4, a  3 and b  6.


x


b
2a
(6)
2(3)

x  1 is the equation of the axis of symmetry.


y  3(1) 2  6(1)  4
 3  6  4
7
The vertex is (1, 7).
Since the coefficient of the x2 term is negative, the
vertex is a maximum.
y  3x  6x  4

Chapter 10

or 2

2
y y  1(x  2)  1

b
2a
4
2(1)

x  2 is the equation of the axis of symmetry.


y  1(x  2) 2  1
 1(0) 2  1
1
The vertex is (2, 1).
Since the coefficient of the x2 term is negative, the
vertex is a maximum.

or 1

494

12. x2  7x  6  0
Sample answer: Solve using the Quadratic
Formula.

8. Graph f(x)  x2  2x  2.
Sample answer:
x

f(x)

1

x

0
1
2
3

2
1
2
5

7  272  4(1) (6)


2(1)

7  125
2
7  5
2
7  5
or
2

f (x )

f (x )  x 2 2x  2


x

13. 2x2  5x  12  0
Sample answer: Solve using the Quadratic
Formula.

x2  6x  7
2
x  6x  7  7  7
x2  6x  7  0
Graph f(x)  x2  6x  7.
Sample answer:
x

x

f(x)

f (x )

(5)  2(5) 2  4(2) (12)


2(2)

5  1121
4
5  11
4
5  11
or
4
1
12

4

1

3

2

x

2

1

1


x

14

13
12
11
10

1
0
1
4

8

4

12

7  2(7) 2  4(6) (20)


2(6)

7  1529
12
7  23
12
7  23
or
12

f(x)

4

22

3
2
7
8

0
50
0
22

O x

The solutions are 2.5, 1.3.


15. 3k2  2k  5
Sample answer: Solve using the Quadratic
Formula.
3k2  2k  5  0
k

f (x )
20
4

b  2b2  4ac
2a

2  222  4(3) (5)


2(3)

2  164
6
2  8
6
2  8
or
6
2
13

k

20
40

7  23

n  12
 1.3

n
 2.5

The solution is 12, the x-intercept of the graph.


2x2  8x  42
11.
2  8x  42  42  42
2x
2x2  8x  42  0
Graph f(x)  2x2  8x  42.
Sample answer:
x

b  2b2  4ac
2a

n

4
f (x )  x  24x  144
12

4

f (x )

16

5  11
4

14. 6n2  7n  20
Sample answer: Solve using the Quadratic
Formula.
6n2  7n  20  20  20
6n2  7n  20  0

One root is between 5 and 4, and the other


root is between 2 and 1.
10. Graph f(x)  x2  24x  144.
Sample answer:
f(x)

x

The solutions are 112, 4.

f (x )  x 2  6x  7

b  2b2  4ac
2a

5

7  5

x 2
x
 6
 1
The solutions are 6, 1.

; The graph has no x-intercepts. Thus, the


equation has no real number solutions.
9.

b  2b2  4ac
2a

k

2  8
6

1
2

f (x)  2x 2  8x  42

The solutions are 13, 1.

The solutions are 3 and 7, the x-intercepts of


the graph.

495

Chapter 10

20. 7m2  m  5
Sample answer: Solve using the Quadratic
Formula.

3
2
16. y2  5y  25  0

Sample answer: Solve by completing the square.


3

y2  5y  25  25  0  25

7m2  m  5  0

2
 25
2
9
 25  100
1
 100
1
 10
1
3
 10  10
3
1
y  10  10
3
1
3
1
y  10  10 or y  10  10
1
2
5
5
1 2
The solutions are 5, 5.
3
y  5y
3
9
y2  5y  100
3
y  10 2
3
y  10
3
3
y  10  10

1  1141
14

21. y 

1  1141
14

2

1

1
2

14  1256
6
14  16
6
14  16
or
6
1
3

m

y

1
2
1
4

x

14  16
6

 5

2
1
0
1

14
12
10
8
6
4
2

1
2

1
2
4
8

b  2b  4ac
2a
(13)  2(13) 2  4(1) (32)
2(1)

13  1297
2

23. y 

or

3x2

 4x  8  0

x

b  2b2  4ac
2a

z

4  242  4(3) (8)


2(3)

4  2112
6

1 2 3 4x

113 2x  3

or

x

x
y
2
6
1
0
0 2
2

1 2 3

2 2 9

x
y 1 3

( 3)

The y-intercept is 2.


24. an  a1 # rn1
25. an  a1 # rn1
5
a6  12(2 )
a4  20(33 )
 12(32)
 20(27)
 384
 540

4  2112
6

 1.1

The solutions are 2.4, 1.1.


Chapter 10

y  4 2x

Sample answer:
13  1297
2

19. 3x2  4x  8
Sample answer: Solve using the Quadratic
Formula.

 2.4

1 2 3x

The y-intercept is 4.

 2.1
 15.1
The solutions are 2.1, 15.1.

4  2112
6

4 3 2 1O
2

x

1
2

5 4 3 2 1O
2

z2  13z  32  0

z

( )x

3

The solutions are 5,


18. 72  13z  32
Sample answer: Solve using the Quadratic
Formula.

13  1297
2

14
12
10
8
6
4
2

The y-intercept is 1.
22. y  4 2x
Sample answer:

1
.
3

z

1  1141
14

112 2x

or

Sample answer:

b  2b  4ac
2a
(14)  2(14)  4(3) (5)
2(3)

x

(1)  2(1) 2  4(7) (5)


2(7)

 0.8
 0.9
The solutions are 0.8, 0.9.

17. 3x  5  14x


Sample answer: Solve using the Quadratic
Formula.
3x2  5  14x  14x  14x
3x2  14x  5  0

m

x

b  2b2  4ac
2a

m

496

a3  a1 r31

26.

4. D; Let x represent the number of grapefruit. Then


2x represents the number of apples.
0.20(2x)  0.25x  1.95
0.4x  0.25x  1.95
0.65x  1.95

63  7r2
1
(63)
7

 7 (7r2 )

9  r2
3  r
The geometric mean is 7(3)  21 or 7(3)  21.

0.65x
0.65

x3
2x  6
The shopper bought 6 apples.
5. D; Area  (2x  3) (2x  6)
 4x2  12x  6x  18
 4x2  6x  18
2
6. B; x  x  12  0
(x  4)(x  3)  0
x  4  0 or x  3  0
x  4 or x  3
7. B; Substitution of the x-values into y  x2  9
yields the corresponding y-values.

a3  a1 r31

27.

12  3 r2
3(12)  3
36  r2
6  r

113 r22
1

The geometric mean is 3 (6)  2 or


1
3 (6)  2.
28. y  C(1  r) t
 17,369(1  0.16) 2
 12,255.57
14,458  12,255.57  2202.43
The car will be worth $2202.43 less than the
buyout price of the lease.

r
29. A  P 1  n nt

 1500 1 

8. B; y  x2  2x  3
x


b
2a
(2)
2(1)

or 1

y  1  2(1)3
 4
The vertex is at (1, 4). This is the only equation
with the correct vertex.

0.06 4.10
4

 1500(1.015) 40
 2721.03
The total amount will be $2721.03.

9. B; 2x2  8x  6  0
2(x2  4x  3)  0
2(x  3)(x  1)  0
or
x10
x30
x3303
x1101
x  3
x  1
The graph intersects the x-axis at (3, 0) and
(1, 0).
10. Let x be the amount earned during the 4th week.

12

30. A; The common ratio is 4 or 3. Multiply 108 by


3 to find the next term. The next term is 324.

Chapter 10 Standardized Test Practice


Pages 580581
1. B; Translating the line down two units does not
change the slope of the line, but it does change
the y-intercept by 2 units.

x  18.50  23.00  15.00


4
x  56.5
4
x  56.5
4
4

2. C; a  kb
Find k.
21  k(6)
21
6
7
2

1.95

 0.65

 18
 18

2  4(18)

x  56.5  72
x  56.5  56.5  72  56.5
x  15.50
She should earn at least $15.50 during the 4th week.
11. 8x  4y  9  0
4y  8x  9

k
k
7

a  2b
Now find a when b  28.

y

a  2 (28) or 98

y

3. A; Since the slope of the graphed line is negative,


the answer must be A or B. From inspection of the
graph, the y-intercept must be less than 5. So,
by elimination, A is the answer.

8x  9
4
9
2x  4

The slope of this line is 2. The slope of a line


1
perpendicular to it is 2.
y  y1  m(x  x1 )
1
y  3  2 (x  2)
1

y  3  2x  1
1

y  2x  4

497

Chapter 10

12. 5a  4b  25
3a  8b  41
Multiply the first equation by 2. Then add.
10a  8b  50
3a  8b  41
13a
 91
a7
Use 5a  4b  25 to find b.
5(7)  4b  25
35  4b  25
4b  10
5
1
b  2 or 22

or

4(x  1) 2  4  0
1

4(x  1) 2  4
1

4(x  1) 2
4

4
4
1

(x  1) 2  16
1

x  1  4
1

x  1  1  4  1
x1 
1

x14

1
4

x14

or

4

The solution set is e 4, 14 f .


3

20e. y  4x2  8x 

14  1388
2

 4  8 

15
4

 4(12 )  8(1) 

 16.85
 2.85
Ignore the negative value. Each side was
increased by 2.85 inches.
16. B; By inspection, the median is 40. The values
greater than 40 are clumped near 40, whereas the
values less than 40 are spread out further from
the median. These low values will bring the
average below 40.

15
4

15
4

4

1 12

The minimum point is located at 1, 4 .


y

20f.
O

(1, 14 )
x

3
0, 4

17. A; The solution of 6p  12 is 2. The solution of


1
1
10q 5 is 2. 2 7 2

y  4x 2  8x  15
4

18. B; 5.3  103  5300 6 53,000


19. C; All even-numbered terms of the first sequence
are the same as the even-numbered terms in the
second sequence.

Chapter 10

4(x  1) 2  4  4  0  4

m

15
4

a  4 and b  8

 14

14  1388
2

4x2  8x  4  4

The equation of the axis of symmetry is x  1.


20c. Since the coefficient of the x2 term is negative,
the parabola opens downward.
20d. Solve the equation, using the right-side
expression.

4(x2  2x  1)  4

Equation for the axis of symmetry


of a parabola

b  2b  4ac
2a
14  214  4(1) (48)
2(1)

4(x  1) 2  4

 4x2  8x 

x  2(4) or 1

m

15
4

20b. x  2a

b2  4ac  (11) 2  4(6)(4)


 25
Since the discriminant is positive, the equation
has two real solutions.
15. Let m be the amount by which each side is
increased.
(m  8) (m  6)  2(8.6)
m2  6m  8m  48  96
m2  14m  48  96
m2  14m  48  0

14  1388
2

4x2  8x 

4x2  8x 

13. x2  4x  5  (x2  4x  4)  5  4
 (x  2) 2  9
h  2 and k  9
14. y  6x2  11x  4. Find the number of real
solutions to 6x2  11x  4  0.
Find the value of the discriminant.

m

4x2  8x 

15 ?

4
15 ?

4
15 ?

4

20a. 4x2  8x 

498

1
0, 1 4

PQ249-6481F-11[499-520] 26/9/02 6:29 PM Page 499 Sahuja Ahuja_QXP_06:Desktop Folder:Chandra:Algebra_FNL_Delivery:

Chapter 11
Page 585

Radical Expressions and Triangles


3. Sample answer: 212  313 and 212  313;
(212  313) (212  313)
 (212) 2  (313) 2
 8  27
 19
4. 120  12  2  5
5. 12  18  116
4
 222  15
 215

Getting Started

1. 125  5
2. 180  8.94
3. 156  7.48
4. 1324  18
5. 3a  7b  2a  (3a  2a)  7b
 a  7b
6. 14x  6y  2y  14x  (6y  2y)
 14x  4y
7. (10c  5d)  (6c  5d)  (10c  6c)  (5d  5d)
 16c

6. 3110  4110  (3  4) ( 2102 )


 (12) (10)
 120
2 2
7. 254a b  22  33  a2  b2
 12  232  13  2a2  2b2
 12  3  13  0a 0  0b 0
 3 0ab 0 16

8. (21m  15n)  (9n  4m)


 (21m  15n)  (9n  4m)
 (21m  4m)  (15n  9n)
 25m  6n
9. x(x  5)  0
x  0 or x  5  0
x5
{0, 5}

8. 260x5y6  222  3  5  x5  y6
 222  13  15  2x4  1x  2y6
 2  13  15  x2  1x  0y3 0
 2x2 0y3 0 115x

10. x2  10x  24  0
(x  4)(x  6)  0
x  4  0 or x  6  0
x  4
or x  6
{6, 4}
11.

12.

9.

x2  6x  27  0
(x  9)(x  3)  0
x  9  0 or x  3  0
x9
x  3
{3, 9}

11.

2x2  x  1  2
2x2  x  1  0
(2x  1)(x  1)  0
2x  1  0 or x  1  0
2x  1
x  1

13.

12.

14.

8  3  2  12
24  24
yes
8 ? 12
 16
10
?

15.

10  12  8  16
120  128
no

4 16
6

13
110

2 16
3

130
10

8
3  12

 16

10.

3

12

3 10  110
110

 110

3  12
12

3 

24  8 12
9  2

24  8 12
7

2 15
4  18

4 ? 16
 25
5
?

5  16  4  25
80  100
no
16.

13

4
16

8(3  12)
( 12) 2

2 ? 8
 12
3
?

16

 32 

x2

512, 16

4
16

6 ? 3
 15
30
?

6  15  30  3
90  90
yes

2 15
18

 4 

4  18
18

 4 

2 15(4  18)
(4) 2  ( 18) 2

8 15  2 140
16  8

8 15  2 222  2  5
8

8 15  4 110
8

2 15  110
2

13. A  s2
 (217 ) 2
 22 ( 17 ) 2
 4(7)
 28 ft2

11-1 Simplifying Radical Expressions

14. P  2 3 32
/

Pages 589590

 2 3 32

Check for Understanding

 2 3 4

1. Both x4 and x2 are positive even if x is a negative


number.
2.

1
1a

1a
1a

1 11
14 2
1
 2 1 2 2
 2

1a
a


 3.14 s

499

Chapter 11

PQ249-6481F-11[499-520] 26/9/02 6:29 PM Page 500 Sahuja Ahuja_QXP_06:Desktop Folder:Chandra:Algebra_FNL_Delivery:

Pages 590592

Practice and Apply


2

29.

15. 118  22  3
 12  232
 12  3
 312

16. 124  22  3
 222  12  13
 216

17. 180  224  5


 224  15
 22 15
 415

18. 175  252  3


 252  13
 513

19. 15  16  130
20. 13  18  124
 223  3
 222  12  13
 2 16
21. 7130  216  (7  2)( 130  16)
 141180
 14222  32  5
 14222  232  15
 14  2  3  15
 8415

31.

1t

3 8  18
1t
18

 18

1t  222  2
8

1t  222  12
8
2 12t
8

12t
4

5c5

25c5

12
13

12
13

16
3

28.

13

 13

34.

15  2c4  1c
14  1d4  1d

23x3
14y5

c2 15c

23  x2  x
14y4  y

1d

c2 15cd
2 0 d3 0
18
18
 6  12
6  12

x 13x
x 13x


6  12
12

6 

18(6  12)
62  ( 12) 2

18(6  12)
36  2

18(6  12)
34

9(6  12)
17

54  9 12
17

10
17  12

2 15
 18

 4

4  18
18

 4 

2 15 (4  18)
(4) 2  ( 18) 2

8 15  2 140
16  8

8 15  2 22 3  5
8

8 15  2 22 2  12  15
8

8 15  4 110
8

2 15  110
2
17  12
12

3 5  3 4  3 20

10( 17  12)
7  2

18

 17 

3 10

10( 17  12)
5

19
110

2( 17  12)
1

19
110

110

 217  212

 110
36.

500

2
13  16

1y

 2y2 1y  1y

2 15
4  18

3x3

 2y2 1y

10( 17  12)
( 17) 2  ( 12) 2

3 110
10

3 12x2y6
3 4y5

9x5y




10
17  12

35.

3 13
0p 0

25  c4  c
14  d4  d

33.

232  13
0p 0

29x5y
112x2y6

c2 15cd

Chapter 11

32.

 2d2 1d2

26. 272x3y4z5
 223  32  x3  y4  z5
 222  12  232  2x2  1x  2y4  2z4  1z
 2  12  3  0x 0  1x  y2  z2  1z
 6 0x 0 y2z2 12xz
2

18

3 4d5  14d5

25. 2147x6y7  272  3  x6  y7


 272  13  2x6  2y6  1y
 7  13  0x3 0  0y3 0  1y
 7 0x3y3 0 13y

233
0p 0

c2 15c

24. 250m3n5  252  2  m3  n5


 252  12  2m2  1m  2n4  1n
 5  12  0m 0  1m  n2  1n
 5n2 0m 0 12mn

127
1p2

27

 2d2 1d  1d

23. 240a4  223  5  a4


 222  12  15  2a4
 2  12  15  a2
 2a2 110

37  33  33

3 p2 

 2d2 1d

22. 213  5127  (2  5)( 13  127)


 10( 181 )
 10(9)
 90

27.

30.

13  16
16

2
13  16

2( 13  16)
( 13) 2  ( 16) 2

2( 13  16)
3  6

2 13  2 16
3

2 16  2 13
3

 13 

x 13xy
2 0 y3 0

PQ249-6481F-11[499-520] 26/9/02 6:29 PM Page 501 Sahuja Ahuja_QXP_06:Desktop Folder:Chandra:Algebra_FNL_Delivery:

37.

4
4  3 13

4

4
 3 13

46. s  130fd
 130(0.8)d
 124d
 223  3  d
 222  12  13  1d
 216d
47. wet road: s  312d
 312(110)
 31220
 44.5 mph
dry road: s  216d
 216(110)
 21660
 about 51.4 mph

4  3 13
 3 13

4

4(4  3 13)
(3 13) 2

 42 

38.

4(4  3 13)
16  27

16  12 13
11

16  12 13
11

3 17
5 13  3 15

3 17

5 13  3 15

 5 13  3 15  5 13  3 15


3 17(5 13  3 15)
(5 13) 2  (3 15) 2

15 121  9 135
75  45

15 121  9 135
30

5 121  3 135
10

48. s  2 (a  b  c)

39. A  /w
 (4110)(315)
 12150
 12252  2
 12252  12
 6012 or about 84.9 cm2

40. A  /w

41. A  s2
s  1A
s  172
 223  32
 222  22  232
 2  12  3

42.

 2 (13  10  7)

 2 (30)

38  32

3 16

 15
49. A  1s(s  a) (s  b) (s  c)
 115(15  13)(15  10)(15  7)
 115(2)(5)(8)
 11200
 224  3  52
 224  13  252
 22  13  5
 2013 or about 34.6 ft2

a2

2a2
116
a
 4 m2
1
E  2mv2

2E  mv2
2E
m

 v2

v

2E

3m

50.

 a(a  1

 612 in.
43. v 

1
(a  1  1a)

(a  1  1a)
1a)

 (a  1 

a  1  1a
 1a)  1(a  1  1a )  1a(a  1  1a)
a  1  1a
 1  1a  a 1a  1a  a

 a2  a  a 1a  a

2(54)
0.6

 1180
 222  32  5
 222  232  15
 2  3  15  615 or about 13.4 m/s
44. Escape Velocity 

2GM
R

2(6.7  1020 ) (7.4  1022 )


(1.7  103 )

2(6.7  7.4) (1020  1022 )


(1.7  103 )

(99.16) (102 )
(1.7) (103 )

31

a  1  1a
a2  3a  1

51. A lot of formulas and calculations that are used in


space exploration contain radical expressions.
Answers should include the following.
To determine the escape velocity of a planet, you
would need to know its mass and radius. It
would be very important to know the escape
velocity of a planet before you landed on it so
you would know if you had enough fuel and
velocity to launch from it to get back into space.
The astronomical body with the smaller radius
would have a greater escape velocity. As the
radius decreases, the escape velocity increases.
52. C; surface area  96a2

21 2

99.16 102
1.7
103

 258.3  101
 15.83
 2.4 km/s
The Moon has a much lower escape velocity than
Earth.
45. s  130fd
 130(0.6)d
 118d
 22  32  d
 12  232  1d
 312d

area of one face 

96a2
6

 16a2
length of each edge  4a
Volume  s3
 (4a) 3
 64a3
53. B; x  81b2
1x  281b2
 181  2b2
 9b

501

Chapter 11

PQ249-6481F-11[499-520] 26/9/02 6:29 PM Page 502 Sahuja Ahuja_QXP_06:Desktop Folder:Chandra:Algebra_FNL_Delivery:

54.

63. 1, 2, 4, 8

y  91.4  (91.4  t) [0.478  0.301( 1x  0.02) ]


9  91.4  (91.4  12) [0.478  0.301( 1x  0.02) ]
9  91.4  79.4[0.478  0.3011x  0.00602]
9  91.4  79.4[0.47198  0.3011x]
9  91.4  37.475212  23.8994 1x
9  53.924788  23.8994 1x
62.924788  23.89941x
2.632902416  1x
7 mph  x

2
1

The common factor is 2.


1, 2, 4, 8 16 32

The next three terms are 16, 32, and 64.


64. 384, 192, 96, 48
192
384

 91.4  (91.4  12) [ 0.478  0.301( 14  0.02) ]


 91.4  (91.4  12) [ 0.478  0.301(2  0.02) ]
 91.4  (79.4) [ 0.478  0.301(1.98) ]
 91.4  (79.4) [ 0.478  0.59598]
 91.4  (79.4) [ 1.07398]
 91.4  85.274012
 6.125988
 6F
1

5
2

x
58. x  x

5
1
2

57.

59.

 x2
 2x3
 2x2  1x
 x 1x

The next three terms are 24, 12, and 6.


1 2

65. 9, 3, 4, 24
2
3

96

The common factor is 6.


1
2

1 2
, ,
9 3

1a
a

1
3
a1
a
a a3

1 2

4, 24

3
4

The common factor is 4.

34
1

a3

3, 4,

3 3
,
16 64

0y3 0 

4

1
3 13

10
50

0y3 0  3  2

1
2

y 
3

23

13

3
4096

 5  0.2

69. y  75(0.875) t
 75(0.875) 15
y  10.1
So the coffee is 95  10.1  84.9C
6  (5)  30
70. 6x2  7x  5

y 
13  13
13
3

1
61. s2t 2 8 2s5t4  (s16t4 ) 2s4  s  t4

 (s16t4 ) 2s4  1s  2t4


 s16  t4  s2  1s  t2
 (s16  s2 )(t4  t2 ) 1s
 s18t6 1s

Factors of 30
10, 3

Sum of Factors
7

6x2  7x  5  6x2  mx  nx  (5)


 6x2  10x  (3)x  5
 2x(3x  5)  (1) (3x  5)
 (3x  5) (2x  1)

62. 2, 6, 18, 54
3

The common factor is 3.


2, 6, 18, 54 162
486 1458




3
3
3

The next three terms are 162, 486, and 1458.

Chapter 11

and

The next three terms are 0.08, 0.016, and 0.032.

6
2

3
3
,
,
256 1024

68. y  art
y  1000(212 )
 4,096,000

2

y 
13

4




 0.2
 0.2  0.2

4

The common factor is 0.2.


50, 10, 2, 0.4 0.08 0.016 0.032

32

y

3
4096

67. 50, 10, 2, 0.4


1

313 2
1

3
1024

The next three terms are

0y3 0  3

3
256




1
1
1

0y3 0 

5184

a2

 a6


864

66. 3, 4, 16, 64

a1 3

60.

144




6
6
6

The next three terms are 144, 864, and 5184.

a2

 a2  3
0y3 0


 
 0.5  0.5  0.5

1
2

 0.5

The common factor is 0.5.


384, 192, 96, 48
24
12

1x 24  x2

64

 

 (2)  (2)  (2)

55. y  91.4  (91.4  t) [ 0.478  0.301( 1x  0.02) ]

56. x 2  x 2  x 2  2
x

 2

502

PQ249-6481F-11[499-520] 26/9/02 6:29 PM Page 503 Sahuja Ahuja_QXP_06:Desktop Folder:Chandra:Algebra_FNL_Delivery:

71. 35x2  43x  12

78. 3a  2b  11
(3, 10); 3(3)  2(10)  11
9  20  11
11  11, true
(4, 1); 3(4)  2(1)  11
12  2  11
14  11, false
(2, 2.5); 3(2)  2(2.5)  11
6  5  11
11  11, true
(3, 2); 3(3)  2(2)  11
9  4  11
5  11, false
{(3, 10), (2, 2.5)}

35  12  420

Factors of 420
28, 15

Sum of Factors (43)


43

35x2  43x  12  35x2  mx  nx  12


 35x2  (28)x  (15)x  12
 (35x2  28x)  (15x  12)
 7x(5x  4)  3(5x  4)
 (5x  4) (7x  3)
72. 5x2  3x  31 prime
73. 3x2  6x  105
3  (105)  315
Factors of 315
21, 15

Sum of Factors (6)


6

3x2  6x  105  3x2  mx  nx  105


 3x2  (21)x  15x  105
 (3x2  21x)  (15x  105)
 3x(x  7)  15(x  7)
 (x  7)(3x  15)
 3(x  7)(x  5)
74. 4x2  12x  15 prime
75. 8x2  10x  3
8  3  24
Factors of 24
6, 4
2

79. 5  2 x  2y
3

(0, 1); 5  2 (0)  2(1)


502
5  2, false
3

(8, 2); 5  2 (8)  2(2)

14, 12 2;

Sum of Factors (10)


10

5  12  4
7  4, false
5  6  1
1  1, true

8x2

 mx  nx  3
8x  12x  15 
 8x2  (6)x  (4x)  3
 (8x2  6x)  (4x  3)
 2x(4x  3)  1(4x  3)
 (4x  3)(2x  1)
76. y  3x  2
(1, 5); 5  3(1)  2
532
5  5, true
(2, 6); 6  3(2)  2
662
6  8, false
(2, 2); 2  3(2)  2
2  6  2
2  4, false
(4, 10); 10  3(4)  2
10  12  2
10  10, true
{(1, 5), (4, 10)}
77. 5x  2y  10
(3, 5); 5(3)  2(5)  10
15  10  10
25  10, false
(2, 0); 5(2)  2(0)  10
10  0  10
10  10, true
(4, 2); 5(4)  2(2)  10
20  4  10
24  10, false
(1, 2.5); 5(1)  2(2.5)  10
5  5  10
10  10, true
{(2, 0), (1, 2.5)}

1 12

5  2 (4)  2 2

(2, 1); 5  2 (2)  2(1)


532
2  2, true

514, 12 2, (2, 1) 6

80. 40  5d
40
5

5d
5

20.4
3.4

8  d
82.
(11)

81. 20.4  3.4y




3.4y
3.4

6y

h
11
h
11

 25

1 2  (11)(25)
h  275

83.

65  29
29(65)  29

129r 2

1885  r
84. (x  3)(x  2)  x(x)  x(2)  3(x)  3(2)
 x2  2x  3x  6
 x2  x  6
85. (a  2)(a  5)  a(a)  a(5)  2(a)  2(5)
 a2  5a  2a  10
 a2  7a  10
86. (2t  1)(t  6)  2t(t)  2t(6)  1(t)  1(6)
 2t2  12t  t  6
 2t2  11t  6
87. (4x  3)(x  1)  4x(x)  4x(1)  3(x)  3(1)
 4x2  4x  3x  3
 4x2  x  3

503

Chapter 11

PQ249-6481F-11[499-520] 26/9/02 6:29 PM Page 504 Sahuja Ahuja_QXP_06:Desktop Folder:Chandra:Algebra_FNL_Delivery:

12. P  4s
 4(4  316)
 4(4)  4(316)
 16  1216 ft

88. (5x  3y) (3x  y)


 5x(3x)  5x(y)  3y(3x)  3y(y)
 15x2  5xy  9xy  3y2
 15x2  4xy  3y2
89. (3a  2b)(4a  7b)
 3a(4a)  3a(7b)  2b(4a)  2b(7b)
 12a2  21ab  8ab  14b2

The perimeter is 16  1216 ft.


A  s2
 (4  316)(4  316)
 4(4)  4(316)  (316) (4)  (316) (316)
 16  1216  1216  9136
 16  2416  9(6)
 16  2416  54
 70  2416 ft2
The area is 70  2416 ft2.
V  1PR
13.
100-watt: V  1100(110)
 1100  1110
 101110
 104.88 volts
75-watt: V  175(110)
 125  3  110
 252  13  1110
 51330
 90.83
The 100-watt bulb takes 101110  51330 or
about 14.05 volts more than a 75-watt bulb.

 12a2  13ab  14b2

11-2

Operations with Radical


Expressions

Page 595

Check for Understanding

1. to determine if there are any like radicands


2. The Distributive Property allows you to add like
terms. Radicals with like radicands can be added
or subtracted.
3. Sample answer: ( 12  13) 2  2  216  3
or 5  2 16
4. 413  713  (4  7) 13
 1113
5. 216  716  (2  7) 16
 516
6. 515  3120  515  3222  5
 515  3 ( 222  15)
 515  3 (215)
 515  615
 (5  6) 15
 15

Pages 595597

7. 213  112  213  222  3


 213  222  13
 213  213
 (2  2) 13
 413
8. 315  516  3120
 315  516  3222  5
 315  516  3 ( 222  15)
 315  516  3 (215)
 315  516  615
 (3  6) 15  516
 915  516
9. 813  13  19  813  13  3
 (8  1) 13  3
 913  3
10. 12( 18  413)  ( 12 )( 18)  ( 12) (413)
 116  416
 4  416

20. 813  212  312  513


 813  513  212  312
 (8  5) 13  (2  3) 12
 1313  12
21. 416  117  612  4117
 416  612  117  4117
 416  612  (1  4) 117
 416  612  5117
22. 118  112  18
 19  2  14  3  24  2
 232  2  222  3  222  2
 312  213  212
 312  212  213
 (3  2) 12  213
 512  213

11. (4  15)(3  15)


 4(3)  4 15  ( 15)(3)  ( 15) ( 15)
 12  415  315  125
 12  (4  3) 15  5
 17  715

Chapter 11

Practice and Apply

14. 815  315  (8  3) 15


 1115
15. 316  1016  (3  10) 16
 1316
16. 2115  6115  3115  (2  6  3) 115
 7115
17. 5119  6119  11119  (5  6  11) 119
 0119
0
18. 161x  21x  (16  2) 1x
 181x
19. 315b  415b  1115b  (3  4  11) 15b
 1015b

504

PQ249-6481F-11[499-520] 26/9/02 6:29 PM Page 505 Sahuja Ahuja_QXP_06:Desktop Folder:Chandra:Algebra_FNL_Delivery:

23. 16  213  112  16  213  14  3


 16  213  222  3
 16  213  213
 16  (2  2) 13
 16  413
24. 317  2128  317  214  7
 317  2222  7
 317  2(217)
 317  417
 (3  4) 17
 17
25. 2150  3132  2125  2  3116  2
 2252  2  3242  2
 2(512)  3(412)
 1012  1212
 (10  12) 12
 212
26. 12 

30. 16( 13  512)  ( 16) ( 13)  ( 16) (512)


 118  5112
 19  2  514  3
 232  2  5222  3
 312  5(213)
 312  1013
31. 15(2110  312)  ( 15) (2110)  ( 15) (312)
 2150  3110
 2125  2  3110
 2252  2  3110
 2(512)  3110
 1012  3110
32. (3  15)(3  15)  32  ( 15) 2
95
4
33. (7  110) 2  (7) 2  2(7) ( 110)  (110) 2
 49  14110  10
 59  14110
34. ( 16  18)  ( 124  12)

11

3 2  12  12
 12 

1
12

 12 

1
12

12
1

2 12
2
3 12
2


27. 110 

 ( 16)( 124)  ( 16)( 12)  ( 18) ( 124)  ( 18)( 12)


 1144  112  1192  116
 12  14  3  164  3  4

12
12

 16  222  3  282  3
 16  213  813
 16  (2  8) 13
 16  1013

12
2

12
2

35.

3 5  110 

22
15

 110 

12
15

 110 

110
5

5 110
5

110
5

4 110
5

15
15

 252  7  222  7
 517  217
 (5  2) 17
 317

36. (2110  3115) (313  212)


 (2110) (313)  (2110) (212)
 (3115) (313)  (3115) (212)
 6130  4120  9145  6130
 414  5  919  5
 4222  5  9232  5
 4(215)  9(315)
 815  2715
 1915
37. (512  315) (2110  3)
 (512) (2110)  (512) (3)
 (315) (2110)  (315) (3)
 10120  1512  6150  915
 1014  5  1512  6125  2  915
 10222  5  1512  6252  2  915
 10(215)  1512  6(512)  915
 2015  1512  3012  915
 (2015  915)  (1512  3012)
 1115  1512

1 11
13 2
1
2
 313  23  5  3 1 13 2
1
13
 313  315  3 1 13  13 2
13
 313  315  3 1 3 2

28. 313  145  3 3 3  313  19  5  3

 313  315  13
 313  13  315
 (3  1) 13  315
 413  315

11
 3 14  7  10 1 17 2
29. 6 3 74  3128  10 3 17  6 1 17
14 2

1 172 2  3 222  7  101 171 2


6 17
1
17
 2  3(217)  10 1 17  17 2
6 17
17
 2  6 17  10 1 7 2
6

6 17
2

42 17
14

(42  84  20) 17
14

106 17
14

53 17
7

6 17
1

( 15  12) ( 114  135)


 ( 15)( 114)  ( 15)( 135)  ( 12)( 114)  ( 12)( 135)
 170  1175  128  170
 125  7  14  7

84 17
14

10 17
7

20 17
14

505

Chapter 11

PQ249-6481F-11[499-520] 26/9/02 6:29 PM Page 506 Sahuja Ahuja_QXP_06:Desktop Folder:Chandra:Algebra_FNL_Delivery:

38. P  2/  2w
 2(817  415)  2(517  315)
 1617  815  1017  615
 (1617  1017)  (815  615)
 (16  10) 17  (8  6) 15
 2617  215

45. V  12gd
from 25 feet: V  12(32)(25)
 11600
 40 ft/s
from 100 feet: V  12(32)(100)
 16400
 80 ft/s
46. The velocity doubled.
47. The velocity should be 19 or 3 times the velocity
of an object falling 25 feet; 3  40  120 ft/s,
12(32)(225)  120 ft/s.
48. 10201P (1  0.01 1P)
 1020155 (1  0.01 155)
 1020(7.42) [1  0.01(7.42) ]
 7564.52[ 1  0.074]
 7564.52[ 0.93]
 7003.5
The pumping station must supply about
7003.5 gal/min.
49. Sample answer: a  4, b  9; 14  9  14  19
50. a  0 or b  0 or both
51. The distance a person can see is related to the

The perimeter is 2617  215 in.


39. P  2/  2w
213  4111  6  2(2111  1)  2w
213  4111  6  4111  2  2w
213  4  2w
13  2  w
The width is 13  2 cm.
1

40. A  2 d d
1 2
1

 2 (3 16)(5 14)
1

 2 (3 16) (5) (2)


 1516
The area is 15 16 cm2.
41. d 

3h

32

Sears Tower: d 

3(1450)
2

4350
2

height of the person using d 

500

should include the following.


You can find how far each lifeguard can see
from the height of the lifeguard tower. Each
tower should have some overlap to cover the
entire beach area.
On early ships, a lookout position (crows nest)
was situated high on the foremast. Sailors
could see farther from this position than from
the ships deck.
52. C; 917  2128  917  214  7
 917  2222  7
 917  2(217)
 917  417
 (9  4) 17
 517
53. D; 13(4  112) 2
 13(42  2(4)( 112)  ( 112) 2 )
 13(16  8112  12)
 13(28  8112)
 ( 13)(28)  ( 13) (8112)
 2813  8136
 2813  8(6)
 2813  48

100


Page 597

 12175
 125  87
 252  87
 5187
Empire State Building: d 

3(1250)
2

3750
2

 11875
 1625  3
 2252  3
 2513
5287  25 23  5(9.33)  25(1.73)
 46.65  43.30
 3.34 mi
A person can see about 3.34 mi farther.
42. Approximately 1000 feet; Solve
3(1250)
2

3h

 3 2  4.57; may use guess and test,


graphical, or analytical methods.

43. r 

3 5

r

3 5

1100
1
10
1

3h

3 2 . Answers

 5.64
 6 in.
The radius of the pipe is about 6 in.
44. No, each pipe would need to carry 500 gallons per
minute, so the pipes would need a radius greater
than 5.6 in.
Chapter 11

Maintain Your Skills

54. 140  14  10
 222  110
 2110

55. 1128  164  2


 282  12
 812

56. 2196x2y3  2142  x2  y2  y


 2142  2x2  2y2  1y
 14|x|y1y

506

PQ249-6481F-11[499-520] 26/9/02 6:29 PM Page 507 Sahuja Ahuja_QXP_06:Desktop Folder:Chandra:Algebra_FNL_Delivery:

57.

150
18

150
18

1400
8
20
8
5
2




18

 18

58.

225c4d
18c2

2225c4d
118c2

1225  2c4  1d
118  1c2







59.

63a
128a3b2

163
1128a2b2

19  7
164  2  a2  b2

232  17
182  12  1a2  1b2

15 0 c2 0 1d
19  2  c

15 0 c2 0 1d
3c 12
5 0 c 0 1d
12
5 0 c 0 1d
12

5 0 c 0 12d
2

65.

12
12

 4(4) 5
 4(1024)
 4096

48(0) 3  75(0)  0
000
or

or

154 23  751 54 2  0


125
5 ?
48 1 64 2  75 1 4 2  0
3 125
5 ?
48 1 64 2  75 1 4 2  0
4

375
4

63. 81  49y
0  49y2  81
0  (7y  9)(7y  9)
7y  9  0 or 7y  9  0
7y  9
7y  9
9

66.

y  7

197 22
?
81
81  49 1 49 2
?

81  49

or

1 2
81
81  49 1 49 2

?
9
81  49 7 2

81
49  49

81 

Check:

81
49  49
1

81  81

81  81

36

q2  121  0

q  11

5
116 6

or

or

q  11  0
q

6
11

507

375
4

0

5x  80x  240  15x2


5x  15x  80x  240  0
(5x3  15x2 )  (80x  240)  0
5x2 (x  3)  80(x  3)  0
(x  3) (5x2  80)  0
or 5x2  80  0
x30
5x2  80
x  3 or
x2  16
x 
4

54,3, 46

y7

48

 2(0.8) 7
 2(0.2097152)
 0.4194304

q  11  0

n  4

1 2
?
48 1
2  75154 2  0
3
125
5 ?
48 1  64 2  75 1 4 2  0
375
375
 4  1 4 2  0

 7(93 )
 7(729)
 5103

1q  116 21q  116 2  0

4n  5  0
4n  5

or

62. an  a1 (rn1 )
a8  2(0.8) 81

64.

36

 121  0

125
 64

61. a  a (rn1 )
n
1
a4  7(9) 41

81 

36
121

1 2

3 114

36

 121  0

5
5 ?
48 4 3  75 4  0

12

 16|ab|

Check:

n4

Check:

 8|ab| 12  12

5
97 6

or

6
11 2

554, 0, 54 6

3 17

60. a  a (rn1 )
n
1
a6  4(4) 61

36

 121  0

48n3  75n  0
3n(16n2  25)  0
3n(4n  5) (4n  5)  0
3n  0 or 4n  5  0
n0
4n  5

 8|ab| 12
3 17

36
121

63
128a2b2

36
0
1116 22  121

Check:

5(4) 3  80(4)  240  15(4) 2


?
320  (320)  240  (240)
00
or
?
5(3) 3  80(3)  240  15(3) 2
?
135  (240)  240  (135)
105  105
or
?
5(4) 3  80(4)  240  15(4) 2
?
320  320  240  240
00

Chapter 11

PQ249-6481F-11[499-520] 26/9/02 6:29 PM Page 508 Sahuja Ahuja_QXP_06:Desktop Folder:Chandra:Algebra_FNL_Delivery:

67. 8n 5
8n
8

3.

5
8
5
8

158 2 5
?

5 5

68.

6 14

45

5
8

168 2 5
?

6 5

6.

6 14  9

w 6 126
Let w  126
Check:
126
9

Let w  125

125
9

6 14

14 14

69.

Let n  8 .

148 2 5

The solution set is n|n


w
9
w
9 9

Let n  8 .

Let n  8 .

Check:

7k
2
2 7k

7 2

7
7

k 7
Check:

Let w  127

127
9

6 14

13.88 6 14

Page 600

6 14

14.11 14

Let k 

Let k 

75

? 21
10

75
2
3

? 21
5

75 7
21
5

21

 10

21
5

2
1

75

? 2
1

2
2

21

 10

2
1

? 21
5

75 7
14
5

? 21
10

? 2
1

3. Sample answer: 1x  1  8
( 1x  1) 2  82
x  1  64
x  63
4. Alex; the square of 1x  5 is x  5.

4
.
5

75

? 21
10

2
1

Let k 

75

2
.
5

21
5

75

? 2
1

21

 10

21
5

71. (x  2) 2  (x 2 )  2(x)(2)  (2) 2


 x2  4x  4
2
72. (x  5)  (x) 2  2(x)(5)  (5) 2
 x2  10x  25
2
73. (x  6)  (x) 2  2(x)(6)  (6) 2
 x2  12x  36
2  (3x) 2  2(3x)(1)  (1) 2
74. (3x  1)
 9x2  6x  1
75. (2x  3) 2  (2x) 2  2(2x)(3)  (3) 2
 4x2  12x  9
2  (4x) 2  2(4x)(7)  (7) 2
76. (4x  7)
 16x2  56x  49

12b  8
( 12b) 2  (8) 2
2b  64

12  32  8
?
164  8
8  8

Check:

64
2

b  32
Since 32 does not satisfy the original equation,
there is no solution.
7.

17x  7
( 17x) 2  72
7x  49
x7
The solution is 7.

8.

13a  6
Check:
( 13a) 2  (6) 2
3a  36
a  12
The solution is 12.

9. 18s  1  5
18s  4
( 18s ) 2  (4) 2
8s  16
s2
The solution is 2.

Graphing Calculator Investigation

1.

2. (10, 5)

Chapter 11

6.

2b
2

Radical Equations

Page 600

125  5
55

70. There are 6  6  6  216 possible outcomes in the


sample space. Only one outcome has each roll a 1.
1
Therefore, the probability is 216.

11-3

1x  5
Check:
( 1x) 2  52
x  25
The solution is 25.

75 7
28
5

5.

? 21
5

Check for Understanding

1. Isolate the radical on one side of the equation.


Square each side of the equation and simplify.
Then check for extraneous solutions.
2. The solution may not satisfy the original
equation.

21
10
2 21

7 10
3
5
3
.
5

13x  5  x  5
( 13x  5) 2  (x  5) 2
3x  5  (x) 2  2(x) (5)  (5) 2
3x  5  x2  10x  25
0  x2  13x  30
0  (x  3) (x  10)
x  3  0 or x  10  0
x  3 or
x  10
x  10; the solution is the same as the solution
from the graph. However, when solving
algebraically, you have to check that x  3 is an
extraneous solution.

508

Check:

Check:

17  7  7
?
149  7
77

1(3) (12)  6
?
236  6
66

18(2)  1  5
?
116  1  5
?
415
55

PQ249-6481F-11[499-520] 26/9/02 6:30 PM Page 509 Sahuja Ahuja_QXP_06:Desktop Folder:Chandra:Algebra_FNL_Delivery:

Check:

s  3.11d
240  3.11d
77.42  1d
(77.42) 2  ( 1d ) 2
5,993.76  d
The depth of the water is about 5994 m.
16.
s  3.11d
10,000 m: s  3.1110,000
 3.1(100)
 310
20 m: s  3.1120
 13.9
310  13.9  296
The wave loses about 296 m/s.

17x  18  9
Check:
( 17x  18) 2  (9) 2
7x  18  81
7x  63
x9
The solution is 9.
11. 15x  1  2  6
15x  1  4
( 15x  1) 2  (4) 2
5x  1  16
5x  15
x3

15.

17(9)  18  9
?
163  18  9
?
281  9
99

10.

15(3)  1  2  6
?

115  1  2  6
?

116  2  6

Pages 601603

426
66
12.

17.

The solution is 3.
16x  8  x  4
( 16x  8) 2  (x  4) 2
6x  8  (x) 2  2(x) (4)  (4) 2
6x  8  x2  8x  16
0  x2  14x  24
0  (x  12)(x  2)
x  12  0 or x  2  0
x  12
x  2
Check:
16x  8  x  4

1100  10
10  10
1k  4
18.
( 1k) 2  (4) 2
k  16
k  16
Check:

Check:

1(16)  4
?

116  4

16x  8  x  4

44

16(2)  8  2  4

16(12)  8  12  4
?

172  8  12  4

19.

112  8  6

164  8
220  6
88
Since 2 does not satisfy the original equation,
12 is the only solution.
13. 4  1x  2  x
1x  2  x  4
( 1x  2) 2  (x  4) 2
x  2  x2  2(x)(4)  (4) 2
x  2  x2  8x  16
0  x2  9x  18
0  (x  3)(x  6)
x  3  0 or x  6  0
x  3 or
x6
Check:

Practice and Apply

1a  10
# ( 1a) 2  102
a  100

4  13  2  3

4  16  2  6

4  11  3

4  14  6

512  1x
(512) 2  ( 1x) 2
25(2)  x
50  x
Check:

512  150
?

512  125  2
512  512
317  1y
20.
(317 ) 2  ( 1y ) 2
9(7)  y
63  y
63  y
Check:

317  1(63)
?

317  163
?

317  19  7
317  317
21. 314a  2  10
314a  12
14a  4
( 14a ) 2  (4) 2
4a  16
a4

426
413
53
66
Since 3 does not satisfy the original equation, 6 is
the only solution.
14. s  3.11d
s  3.1110
 3.1(3.16)
 9.8
The speed of the tsunami is about 9.8 m/s.

Check:

314(4)  2  10
?

3116  2  10
?

3(4)  2  10
?

12  2  10
10  10

509

Chapter 11

PQ249-6481F-11[499-520] 26/9/02 6:30 PM Page 510 Sahuja Ahuja_QXP_06:Desktop Folder:Chandra:Algebra_FNL_Delivery:

22. 3  51n  18
51n  15
1n  3
( 1n ) 2  (3) 2
n9
Check:

18  1  3  0
?

19  3  0
?

330
00

3  519  18

28. 13r  5  7  3
13r  5  4
( 13r  5) 2  (4) 2
3r  5  16
3r  21
r7

3  5(3)  18
?

3  15  18
18  18
23.
1x  3  5
( 1x  3 ) 2  (5) 2
x  3  25
x  22
Check:

14(2)  1  3  0

Check:

13(7)  5  7  3

Check:

121  5  7  3

116  7  3

122  3  5

125  5
5  5

473
11  3

No solution
24.
1x  5  2 16
( 1x  5 ) 2  (216) 2
x  5  4(6)
x  5  24
x  29
Check:

No solution
29.

4x

35

93
4x

1 3 4x5 22  (12)2

129  5  216

4x
5

124  216

93

720
5

93

?
?

1144  9  3
?
12  9  3
33

13(5)  12  313

4t

30. 5 3 3  2  0
4t

53 3  2

127  313
?

19  3  313
313  313

4t

33

5

1 3 4t3 22  125 22

12c  4  8
( 12c  4) 2  (8) 2
2c  4  64
2c  68
c  34

4t
3

 25
12

4t  25
3

t  25

12(34)  4  8
?

168  4  8

Check:

164  8
88

3
425
?
20
5B
3
12
25

5B  2  0
3

27. 14b  1  3  0
14b  1  3
( 14b  1) 2  (3) 2
4b  1  9
4b  8
b2

Chapter 11

4(180)
5

Check:

Check:

115  12  313

26.

 144

4x  720
x  180

14  6  216
216  216
25.
13x  12  313
( 13x  12 ) 2  (313) 2
3x  12  9(3)
3x  12  27
3x  15
x5
Check:

 12

35

5 3 25  2  0
5

125 2  2  0
?
?

220
00

510

PQ249-6481F-11[499-520] 26/9/02 6:30 PM Page 511 Sahuja Ahuja_QXP_06:Desktop Folder:Chandra:Algebra_FNL_Delivery:

31.

2x2  9x  14  x  4
( 2x2  9x  14 ) 2  (x  4) 2
x2  9x  14  x2  2(x)(4)  42
x2  9x  14  x2  8x  16
9x  14  8x  16
x  14  16
x2
Check:

36.

222  9(2)  14  2  4

Check:

y  2  2y2  5y  4
(y  2) 2  ( 2y2  5y  4 ) 2
2
y  2(y)(2)  22  y2  5y  4
y2  4y  4  y2  5y  4
4y  4  5y  4
y  4  4
y  0
y0

Check:

0  2  202  5(0)  4
2  10  0  4

38.

157  7  8
?

 81

181  9
99
x  16  x
35.
(x) 2  ( 16  x) 2
x2  6  x
2
x x60
(x  3)(x  2)  0
or x  2  0
x30
x  3 or
x2
?
?

128  3(4)  4

128  12  4

x  486

3  16  (3)

19  3
33

116  4
149  7
7  7
44
Since 7 does not satisfy the original equation,
4 is the only solution.
39.
1x  1  x  1
( 1x  1 ) 2  (x  1) 2
x  1  x2  2(x) (1)  (1) 2
x  1  x2  2x  1
0  x2  3x
0  x(x  3)
x  0 or x  3  0
x3
?
?
Check:
10  1  0  1 13  1  3  1

1 3 6x 22  92

Check:

128  3(7)  7
128  21  7

486 ?

6
?

115  6  3

128  3x  x
( 128  3x ) 2  (x) 2
28  3x  x2
0  x2  3x  28
0  (x  7) (x  4)
x  7  0 or x  4  0
x  7
x4
Check:

36  9

164  8
88
34. Let x  the number.

Check:

15(3)  6  3

14  2
22

2  14
22
33. Let x  the number.
1x  7  8
( 1x  7 ) 2  82
x  7  64
x  57

x
6

15(2)  6  2
110  6  2

Check:

4  14  20

4  115
5  125
55
Since 4 does not satisfy the original equation,
5 is the only solution.
15x  6  x
37.
( 15x  6 ) 2  (x) 2
5x  6  x2
0  x2  5x  6
0  (x  2) (x  3)
x  2  0 or x  3  0
x3
x  2 or

136  6
66

Check:

5  15  20
?

14  18  14  6

32.

x  1x  20
(x) 2  ( 1x  20) 2
x2  x  20
2  x  20  0
x
(x  5)(x  4)  0
x  5  0 or x  4  0
x5
x  4

11  1
14  2
1  1
22
Since 0 does not satisfy the original equation,
3 is the only solution.

2  16  2
?

2  14
3  19
3  3
22
Since 3 does not satisfy the original equation,
2 is the only solution.

511

Chapter 11

PQ249-6481F-11[499-520] 26/9/02 6:30 PM Page 512 Sahuja Ahuja_QXP_06:Desktop Folder:Chandra:Algebra_FNL_Delivery:

40.

11  2b  1  b
( 11  2b ) 2  (1  b) 2
1  2b  (1) 2  2(1)(b)  (b) 2
1  2b  1  2b  b2
0  b2  4b
0  b(b  4)
b  0 or b  4  0
b  4
Check:

Check:

11  8  3

?
?

19  3
11  1
11
3  3
Since 4 does not satisfy the original equation,
0 is the only solution.
41. 4  1m  2  m
1m  2  m  4
( 1m  2 ) 2  (m  4) 2
m  2  m2  2(m)(4)  (4) 2
m  2  m2  8m  16
0  m2  9m  18
0  (m  3)(m  6)
m  3  0 or m  6  0
m3
m6
Check:

4  16  2  6

4  14  6

4  13  2  3
4  11  3

16  3(25)  25  16
?

16  75  9

19  8  1
?

16  3(10)  10  16
?

16  30  6
?

136  6
66

Since 25 does not satisfy the original equation,


10 is the only solution.
45.

?
?

22r2  121  r
( 22r2  121 ) 2  r2
2r2  121  r2
r2  121  0
(r  11)(r  11)  0
or r  11  0
r  11  0
r  11 or
r  11
Check:
?

22(11) 2  121  11

12(121)  121  11

22(112 )  121  11
12(121)  121  11

?
?

1121  11
1121  11
11  11
11  11
Since 11 does not satisfy the original equation,
11 is the only solution.
46.

13(4)  8  4  2
?

112  8  2
?

14  2
11  1
11
22
43. x  16  x  4
16  x  4  x
( 16  x ) 2  (4  x) 2
6  x  42  2(4)(x)  (x) 2
6  x  16  8x  x2
0  x2  7x  10
0  (x  2)(x  5)
x  2  0 or x  5  0
x5
x2

Chapter 11

181  9
9  9

426
413
53
66
Since 3 does not satisfy the original equation, 6 is
the only solution.
42.
13d  8  d  2
( 13d  8 ) 2  (d  2) 2
3d  8  d2  2(d)(2)  (2) 2
3d  8  d2  4d  4
0  d2  7d  12
0  (d  3)(d  4)
d  3  0 or d  4  0
d3
d4
Check:
13(3)  8  3  2

5  11  4

514
224
44
64
Since 5 does not satisfy the original equation, 2 is
the only solution.
44.
16  3x  x  16
( 16  3x ) 2  (x  16) 2
6  3x  x2  2(x)(16)  162
6  3x  x2  32x  256
0  x2  35x  250
0  (x  25)(x  10)
or x  10  0
x  25  0
x  25
x  10
Check:

11  2(4)  1  4

11  0  1

5  16  5  4

2  14  4

11  2(0)  1  0

2  16  2  4

25p2  7  2p
( 25p2  7 ) 2  (2p) 2
5p2  7  4p2
p2  7  0
p2  7
p 
17
Check:
?

25( 17) 2  7  2 17

25(17) 2  7  2 (17)

15(7)  7  217

135  7  217

15(7)  7  2 17
135  7  2 17
?

128  2 17
2 17  217

?
?

128  217
217  217

Since 17 does not satisfy the original equation,


17 is the only solution.
47.

512

2(x  5) 2  x  5
( 2(x  5) 2 ) 2  (x  5) 2
(x  5) 2  (x  5) 2
Sometimes; only numbers that make (x  5)
non-negative.

PQ249-6481F-11[499-520] 26/9/02 6:30 PM Page 513 Sahuja Ahuja_QXP_06:Desktop Folder:Chandra:Algebra_FNL_Delivery:

L  1kP
42  10.1669P
(42) 2  ( 10.1669P ) 2
1764  0.1669P
10,569  P
The maximum take off weight is 10,569 lb.
49.
L  1kP
232  1k(870,000)
(232) 2  ( 1870,000k ) 2
53,824  870,000k
0.0619  k
50. A  r2
48.

A


 r2

A


 2r2

56. P  2 3 32
/

t  2 3 32
/

t
2

t2
42

r

52. r 

96


r

32t2
2

r  196
r  116  6
r  416 or about 9.8 m

A
48


r  148
r  116  3
r  413 or about 6.9 m

53. It increases by a factor of 12.


54.

1
3

 2 3 32
/

3 32
/

131 22  1 3 32/ 22
1
92
32
92

 32
/

/

/  0.36 ft
55. P  2 3 32
/

1  2 3 32
/

121 22  1 3 32/ 22
1
42

 32

32
42

/

/  0.81 ft

2  2 3 32
/

2
2

1
2
32
2

 32

2t
2

t2
2

 32

3 32
/

1t 22  1 3 32/ 22
/

32t2
2

8t2
2

8t2
2

24t2
2

58.

V  201t  273
340  201t  273
17  1t  273
(17) 2  ( 1t  273 ) 2
289  t  273
t  16C

1h  9  1h  13
( 1h  9  1h ) 2  ( 13 ) 2
( 1h  9 ) 2  2( 1h  9 ) (1h )  (1h ) 2  3
h  9  21h(h  9)  h  3
2h  9  2 1h(h  9)  3
21h(h  9)  6  2h
1h(h  9)  3  h
( 1h(h  9) ) 2  (3  h) 2
h(h  9)  32  2(3) (h)  (h) 2
h2  9h  9  6h  h2
3h  9
h3
Check:
13  9  13  13
112  13  13
213  13  13
13  13
61. You can determine the time it takes an object to
fall from a given height using a radical equation.
Answers should include the following.
It would take a skydiver approximately
42 seconds to fall 10,000 feet. Using the
equation, it would take 25 seconds. The time
is different in the two calculations because air
resistance slows the skydiver.
A skydiver can increase the speed of his fall by
lowering air resistance. This can be done by
pulling his arms and legs close to his body. A
skydiver can decrease his speed by holding his
arms and legs out, which increases the air
resistance.
62. A; 1x  3  6
1y  3  6
( 1x  3) 2  (6) 2
1y  3
( 1y ) 2  (3) 2
x  3  36
x  33
y9

P  2 3 32
2
3

32t2
42

V  201t  273
356  201t  273
17.8  1t  273
(17.8) 2  ( 1t  273 ) 2
316.84  t  273
t  43.84C
59. V  201t  273
V  2010  273
 201273
 330.45
V 6 330.45 m/s
60.

3
3

 32

/

57.

3

3 32

12t 22  1 3 32/ 22

3  r
51. r 

2t  2 3 32

3 32
/

11 22  1 3 32/ 22
/

/

/  3.24 ft
3.24  0.81  2.43 ft longer

513

Chapter 11

PQ249-6481F-11[499-520] 26/9/02 6:30 PM Page 514 Sahuja Ahuja_QXP_06:Desktop Folder:Chandra:Algebra_FNL_Delivery:

76. d2 50d  225


No; 50d  2(d)(15)
77. 4n2  28n  49
Yes; 28n  2(2n)(7)
(2n 7)2
78. 16b2  56bc  49c2
Yes; 56bc  2(4b)(7c)
(4b  7c)2
79. (r  3)(r  4)  r(r)  r(4)  3(r)  3(4)
 r2  4r  3r  12
 r2  r  12
80. (3z  7)(2z  10)
 3z(2z)  3z(10)  7(2z)  7(10)
 6z2  30z  14z  70
 6z2  44z  70

2
2(a  1) 2
63. C; ( 1a  1)
a1
a1
64. Clear the Y  list. Enter the left side of the
equation as Y1. Enter the right side of the
equation as Y2. Use the intersect feature on the
CALC menu to find the x value of the point of
intersection is 8.
65. Clear the Y  list. Enter the left side of the
equation as Y1. Enter the right side of the
equation as Y2. Use the intersect feature on the
CALC menu to find the x value of the point of
intersection is 11.
66. Clear the Y  list. Enter the left side of the
equation as Y1. Enter the right side of the
equation as Y2. Use the intersect feature on the
CALC menu to find the x value of the point of
intersection is 2.
67. Clear the Y  list. Enter the left side of the
equation as Y1. Enter the right side of the
equation as Y2. Use the intersect feature on the
CALC menu to find the x value of the point of
intersection is 15.08.
68. Clear the Y  list. Enter the left side of the
equation as Y1. Enter the right side of the
equation as Y2. Use the intersect feature on the
CALC menu to find the x value of the point of
intersection is 1.70.
69. Clear the Y  list. Enter the left side of the
equation as Y1. Enter the right side of the
equation as Y2. Use the intersect feature on the
CALC menu, or the fact that the graph has no
points of intersection, to determine that the
equation has no solution.

Page 603

81.

 2p(3p2 )  2p(4p)  2p(9)  5(3p2 )  5(4p)  5(9)


 6p3  8p2  18p  15p2  20p  45
 6p3  7p2  2p  45
9

82. F  5C  32
9

83.

Maintain Your Skills

21
110  13

21( 110  13)


( 110) 2  ( 13) 2

21( 110  13)


10  3

21( 110  13)


7

 110 

y  2x  7

89. 2a2  b2  282  122


 164  144
 1208
 116  13
 4113

 3( 110  13 )

Chapter 11

 72  32
 104F

88. 2a2  b2  212  12


 11  1
 12

110  13
13

21
110  13

 63  32
 95F
95  F  104

87. 2a2  b2  2(24) 2  (7) 2


 1576  49
 1625
 25

74. 16  110  160


 14  15
 2115

F  5 (40)  32

86. 2a2  b2  2(3) 2  (4) 2


 19  16
 125
5

 3 12  512  3(412 )
 3 12  512  1212
 (3  5  12) 12
 4 12

75.

F  5 (35)  32

7y  14x  3
14x  7y  3
14x  7y  3
84. y  3  2(x  6)
y  3  2x  12
y  2x  15
2x  y  15
85.
y  2  7.5(x  3)
y  2  7.5x  22.5
y  7.5x  24.5
7.5x  y  24.5
15x  2y  49

70. 516  1216  (5  12) 16


 1716
71. 112  6127  14  3  619  3
 213  6(313 )
 213  1813
 2013
72. 118  512  3132  19  2  512  3116  2

73. 1192  164  3


 813

(2p  5)(3p2  4p  9)

514

PQ249-6481F-11[499-520] 26/9/02 6:30 PM Page 515 Sahuja Ahuja_QXP_06:Desktop Folder:Chandra:Algebra_FNL_Delivery:

Page 603

Practice Quiz 1

1. 148  116  3
 413
3.

3
2  110

2

3
 110

10.

2. 13  16  118
 19  2
 312
2  110
 110

2

3(2  110)
(2) 2  ( 110) 2

3(2  110)
4  10

3(2  110)
6

2  110
2

x2
Check:

?
?

14  2
2  2

?
?

110  1  10  7
?

19  3
33

Since 2 does not satisfy the original equation, 5


is the only solution.

Page 604

Graphing Calculator Investigation


(Follow-Up of Lesson 11-3)

1. y  1x  1

[10, 10] scl: 1 by [10, 10] scl: 1

The domain is {x|x 0}; the graph is shifted up


1 unit.
2. y  1x  3

23x2  32  x
( 23x2  32) 2  x2
3x2  32  x2
2x2  32  0
2x2  32
x2  16
x 
4
Check:

3 2 1 2 2  1  2 1 2 2  7 12(5)  1  2(5)  7
15  1  5  7

4. 615  3111  515  615  515  3111


 (6  5) 15  3111
 1115  3111
5. 213  9112  213  9( 14.3)
 213  9(213)
 213  1813
 (2  18) 13
 2013
2
6. (3  16 )  (3) 2  2(3)(16 )  (16 ) 2
 9  616  6
 15  616
7. A  s2
 (2  17) 2
 22  2(2)( 17)  ( 17) 2
 4  417  7
 11  417 or 21.6 cm2
115  x  4
8.
( 115  x) 2  42
15  x  16
x  1
x  1
115  (1)  4
Check:
116  4
4  4
9.

12x  1  2x  7
( 12x  1) 2  (2x  7) 2
(2x  1)  (2x) 2  2(2x) (7)  (7) 2
2x  1  4x2  28x  49
0  4x2  30x  50
0  2(2x2  15x  25)
0  2(2x  5) (x  5)
2x  5  0 or x  5  0
x5
2x  5

[10, 10] scl: 1 by [10, 10] scl: 1

The domain is {x|x 0} ; the graph is shifted


down 3 units.
3. y  1x  2
?

23(4) 2  32  4

13(16)  32  4

148  32  4

23(4) 2  32  4
13(16)  32  4
148  32  4
?

116  4
44

?
?
?
?

116  4
4  4

Since 4 does not satisfy the original equation,


4 is the only solution.

[10, 10] scl: 1 by [10, 10] scl: 1

The domain is {x|x 2} ; the graph is shifted


left 2 units.

515

Chapter 11

PQ249-6481F-11[499-520] 26/9/02 6:30 PM Page 516 Sahuja Ahuja_QXP_06:Desktop Folder:Chandra:Algebra_FNL_Delivery:

4. y  1x  5

9. y  12x  5  4

[5, 15] scl: 1 by [10, 10] scl: 1

[10, 10] scl: 1 by [10, 10] scl: 1

The domain is {x|x 5} ; the graph is shifted


right 5 units.

The domain is {x|x 2.5}; the graph is shifted


left 2.5 units, down 4 units.
10. No, you must consider the graph of y  1x and
the graph of y  1x. This graph fails the
vertical line test. For every value of x 7 0, there
are two values for y.

5. y  1x

11. No; the equation y 


21  x2 is not a
function since there are both positive and
negative values for y for each value of x.
12. Enter y  |x|  21  x2 as Y1 and
y  |x|  21  x2 as Y2.

[10, 10] scl: 1 by [10, 10] scl: 1

The domain is {x|x  0} ; the graph is reflected


across y-axis.

1 X,T,,n

KEYSTROKES:

6. y  13x

1 X,T,,n

ENTER
[1 ] 1

X,T,,n

2nd [ 1 ] 1

X,T,,n

2nd

GRAPH

The graph is shaped like a heart.

11-4 The Pythagorian Theorem

[10, 10] scl: 1 by [10, 10] scl: 1

The domain is {x|x 0} ; the graph is expanded.

Pages 607608

7. y  1x

Check for Understanding

1.
hypotenuse
leg

leg

2. Compare the lengths of the sides. The hypotenuse


is the longest side, which is always the side
opposite the right angle.
3. s2  s2  d2
2s2  d2
22s2  2d2
22s2  d or d  s 12
2
4. c  a2  b2
c2  142  122
c2  196  144
c2  340
c 
1340
c  18.44
The length of the hypotenuse is 18.44 units.
c2  a2  b2
5.
412  a2  402
1681  a2  1600
81  a2

181  a
9a
The length of the leg is 9 units.

[10, 10] scl: 1 by [10, 10] scl: 1

The domain is {x|x 0} ; the graph is reflected


across x-axis.
8. y  11  x  6

[10, 10] scl: 1 by [5, 15] scl: 1

The domain is {x|x  1} ; the graph is reflected


across y-axis, shifted right 1 unit, up 6 units.

Chapter 11

516

PQ249-6481F-11[499-520] 26/9/02 6:30 PM Page 517 Sahuja Ahuja_QXP_06:Desktop Folder:Chandra:Algebra_FNL_Delivery:

6. c2  a2  b2
c2  102  242
c2  100  576
c2  676
c2 
1676
c  26
The length of the hypotenuse is 26 units.
c2  a2  b2
7.
612  112  b2
3721  121  b2
3600  b2

13600  b
60  b
The length of the leg is 60 units.
c2  a2  b2
8.
( 1233) 2  a2  132
233  a2  169
64  a2

164  a
8a
The length of the leg is 8 units.
9. c2  a2  b2
c2  72  42
c2  49  16
c2  65
c 
165
c  8.06
The length of the hypotenuse is about 8.06 units.
10. No; 42  62  92.
11. Yes; 162  302  342.
12. A; c2  a2  b2
82  a2  62
64  a2  36
28  a2

128  a
217  a
The length of the leg is 217 units.
A
A

14. c2  a2  b2
c2  72  92
c2  49  81
c2  130
c 
1130
c  11.40
The length of the hypotenuse is about 11.40 units.
15. c2  a2  b2
c2  282  452
c2  784  2025
c2  2809
c 
12809
c  53
The length of the hypotenuse is 53 units.
c2  a2  b2
16.
142  52  b2
196  25  b2
171  b2

1171  b
13.08  b
The length of the leg is about 13.08 units.
c2  a2  b2
17.
1802  a2  1752
32,400  a2  30,625
1775  a2

11775  a
42.13  a
The length of the leg is about 42.13 units.
18.
c2  a2  b2
1012  992  b2
10,201  9,801  b2
400  b2

1400  b
20  b
The length of the leg is 20 units.
19. c2  a2  b2
c2  162  632
c2  256  3,969
c2  4,225
c 
14225
c  65
The length of the hypotenuse is about 65 units.
20.
c2  a2  b2
342  162  b2
1156  256  b2
900  b2

1900  b
30  b
The length of the leg is 30 units.
21. c2  a2  b2
c2  ( 1112 ) 2  32
c2  112  9
c2  121
c 
1121
c  11
The length of the hypotenuse is 11 units.

bh
2
6  2 17
2

 617 units2

Pages 608610
13.

Practice and Apply

c2  a2  b2
152  a2  52
225  a2  25
200  a2

1200  a
14.14  a
The length of the leg is about 14.14 units.

517

Chapter 11

PQ249-6481F-11[499-520] 26/9/02 6:30 PM Page 518 Sahuja Ahuja_QXP_06:Desktop Folder:Chandra:Algebra_FNL_Delivery:

22. c2  a2  b2
c2  ( 115) 2  ( 110) 2
c2  15  10
c2  25
c 
125
c5
The length of the hypotenuse is 5 units.
c2  a2  b2
23.
142  92  b2
196  81  b2
115  b2

1115  b
10.72  b
The length of the leg is about 10.72 units.
24. c2  a2  b2
c2  62  32
c2  36  9
c2  45
c 
145
c  6.71
The length of the hypotenuse is about 6.71 units.
c2  a2  b2
25.
122  a2  ( 177) 2
144  a2  77
67  a2

167  a
8.19  a
The length of the leg is about 8.19 units.
26. c2  a2  b2
c2  42  ( 111) 2
c2  16  11
c2  27
c 
127
c  5.20
The length of the hypotenuse is about 5.20 units.
27. c2  a2  b2
c2  ( 1225) 2  ( 128) 2
c2  225  28
c2  253
c 
1253
c  15.91
The length of the hypotenuse is about 15.91 units.
28.
c2  a2  b2
( 1155) 2  ( 131) 2  b2
155  31  b2
124  b2

1124  b
11.14  b
The length of the leg is about 11.14 units.
29. c2  a2  b2
c2  (8x) 2  (15x) 2
c2  64x2  225x2
c2  289x2
c 
2289x2
c  17x
The length of the hypotenuse is about 17x units.

Chapter 11

30.

31.
32.
33.
34.
35.
36.
37.

38.

39.

518

c2  a2  b2
(7x) 2  a2  (3x) 2
49x2  a2  9x2
40x2  a2

240x2  a

140x  a
6.32x  a
The length of the leg is about 6.32x units.
Yes; 302  402  502.
No; 62  122  182.
No; 242  302  362.
Yes; 452  602  752.
Yes; 152  ( 131) 2  162.
Yes; 42  72  ( 165) 2.
Use the formula for the area of a square to find
the length of a side.
A  s2
162  s2

1162  s
1162  s
Use the Pythagorean Theorem to find the
diagonal.
d 2  s2  s2
d2  ( 1162) 2  ( 1162) 2
d2  162  162
d2  324
d 
1324
d  18
The length of the diagonal is 18 ft.
Let x  length of one leg.
x  5  length of second leg.
c2  a2  b2
252  x2  (x  5) 2
625  x2  x2  2(x) (5)  52
625  2x2  10x  25
0  2x2  10x  600
0  2(x2  5x  300)
0  2(x  20)(x  15)
x  20  0 or x  15  0
x  20
x  15
The first leg is 15 cm; the second leg is 20 cm.
The diagonal of the cube is the hypotenuse of a
triangle with one leg being the diagonal of a face
and the other leg being a side.
Diagonal of face: d2  s2  s2
d2  42  42
d2  16  16
d2  32
d  132
Diagonal of cube: d2  ( 132) 2  42
d2  32  16
d2  48
d  148
d  116.3
d  413 or 6.93 in.

PQ249-6481F-11[499-520] 26/9/02 6:30 PM Page 519 Sahuja Ahuja_QXP_06:Desktop Folder:Chandra:Algebra_FNL_Delivery:

40. Let x the length of the shorter leg.


144
x

41.

42.

43.

44.
45.

46.

47.

49. Use the Pythagorean Theorem to calculate the


length of the vertical leg of the right triangle
whose hypotenuse is 180 ft and horizontal leg is
130 ft.
c2  a2  b2
1802  a2  1302
32400  a2  16900
15500  a2

115500  a
124.5  a
The maximum height reached is the sum of the
three vertical sections.
80 ft  124.5 ft  120 ft  324.5 ft
This value of 324.5 ft is the vertical leg of the
right triangle whose hypotenuse is 381.2 ft. The
Pythagorean Theorem can be used to determine
the horizontal leg is about 200 ft.
The plateau at the top can be calculated by
considering the corresponding segment of equal
length at the bottom.
750 ft  (100 ft  130 ft  50 ft  200 ft)  270 ft
The Pythagorean Theorem can be used to show
that the right triangle with legs of 50 ft and 80 ft
has a hypotenuse of about 94.3 ft. The
Pythagorean Theorem can also be used to show
that the right triangle with legs of 100 ft and
120 ft has a hypotenuse of about 156.2 ft.
The total distance traveled is about 381.2 ft  270
ft  94.3 ft  180 ft  156.2 ft  1081.7 ft
The maximum height is about 324.5 ft.
50. Engineers can use the Pythagorean Theorem to
find the total length of the track, which
determines how much material and land area
they need to build the attraction. Answers should
include the following.
A tall hill requires more track length both
going uphill and downhill, which will add to
the total length of the tracks. Tall, steep hills
will increase the speed of the roller coaster. So
a coaster with a tall, steep first hill will have
more speed and a longer track length.
The steepness of the hill and speed are limited
for safety and to keep the cars on the track.
51. C; Use the Pythagorean Theorem to find x.
c2  a2  b2
152  x2  (2x) 2
225  x2  4x2
225  5x2
45  x2
145  x
Find the area of the triangle with b  145 and
h  2145.

8
5

8x  720
x  90
The shorter leg is 90 m.
c2  a2  b2
1442  902  b2
20,736  8100  b2
12,636  b2

112,636  b
112.41  b
The longer leg is about 112.41 m.
c2  a2  b2
c2  2082  3602
c2  43,264  129,600
c2  172,864
c  1172,864
c  415.8
It will travel about 415.8 ft.
c2  a2  b2
c2  442  2082
c2  1936  43,264
c2  45,200
c  145,200
c  212.6
It will travel about 212.6 ft.
The roller coaster makes a total horizontal advance
of 404 feet, reaches a vertical height of 208 feet,
and travels a total track length of about 628.4 feet.
See students work.
c2  a2  b2
c2  1002  602
c2  10,000  3600
c2  13,600
c 
113,600
c  116.6
The longest edge is about 116.6 ft.
c2  a2  b2
c2  52  122
c2  25  144
c2  169
c 
1169
c  13
The missing length is 13 ft.
The garage roof is made up of two 30 by 15
rectangles. The 15 ft dimension was obtained by
adding the 2 foot overhang to the 13 ft calculated
in the previous problem.
A  /w
A  30  15
A  450 ft2
The total area is 900 ft2.

48. The area of the largest semicircle is


The sum of the other two areas is


4

c2
4

bh
2
( 145) (2 145)
2

 4 c2.

 ( 145 ) ( 145 )
 45 units2

1a2  b22.

A

Using the Pythagorean Theorem, c  a  b , we


can show that the sum of the two small areas is
equal to the area of the largest semicircle.

519

Chapter 11

PQ249-6481F-11[499-520] 26/9/02 6:30 PM Page 520 Sahuja Ahuja_QXP_06:Desktop Folder:Chandra:Algebra_FNL_Delivery:

52. B; Find the length of the side by using the


Pythagorean Theorem.
d2  s2  s2
102  s2  s2
100  2s2
50  s2
150  s

61.

26a4b7c5
13a2b4c3

a b c
126
13 21 a 21 b 21 c 2
4

5

 2(a42 )(b74 ) (c53 )


 2a2b3c8
a2
1
2a2b3
c8
2

 1


b3
1

 c8

62. Let p  air speed of the plane


and w  wind speed.

Find the perimeter of the square with s  150.


p  4s
 4150
 4125  2
 4(512)
 2012 cm

Rate

Time (hr)

With

pw

Against

pw

2
3
3
4

Distance (m)
300
300

Write two equations based on the fact that the


product of the rate and the time is the distance.

Page 610
53.

Maintain Your Skills

1y  12
( 1y) 2  122
y  144

31s  126
1s  42
( 1s) 2  422
s  1764
55. 412v  1  3  17
412v  1  20
12v  1  5
( 12v  1) 2  52
2v  1  25
2v  24
v  12
Check:

1144  12
12  12

Check:

54.

2
(p
3
3
(p
4

311764  126
?
3(42)  126
126  126

63. 2(6  3) 2  (8  4) 2  232  42


 19  16
 125
5
64. 2(10  4) 2  (13  5) 2  262  82
 136  64
 1100
 10

412(12)  1  3  17
?

4 124  1  3  17

65. 2(5  3) 2  (2  9) 2  222  (7) 2


 14  49
 153

4125  3  17
?
20  3  17
17  17

66. 2(9  5) 2  (7  3) 2  2(14) 2  42


 1196  16
 1212
 14  53
 2153

56. 172  136  2


 262  12
 612
57. 71z  101z  (7  10) 1z
 3 1z
58.

59.

58
53

3
7

 121 

13
17

13
17

121
7

121
1

121
7

7 121
7

(1  7) 121
7

8 121
7

67. 2(4  5) 2  (4  3) 2  2(9) 2  (7) 2


 181  49
 1130

 121
17

 17 

121
1

68. 2(20  5) 2  (2  6) 2  2152  (8) 2


 1225  64
 1289
 17

11-5

 583
 55 or 3125

The Distance Formula

Pages 612613

1
60. d7  d7

Chapter 11

 w)  300

Simplify each equation and solve the system.


p  w  450
p  w  400
2p
 850
p  425 mph
The air speed of the plane is 425 mph.

Check:

 w)  300

Check for Understanding

1. The values that are subtracted are squared before


being added and the square of a negative number is
always positive. The sum of two positive numbers
is positive, so the distance will never be negative

520

PQ249-6481F-11[521-540] 26/9/02 8:15 PM Page 521 Sahuja Ahuja_QXP_06:Desktop Folder:Chandra:Algebra_FNL_Delivery:

2. See students graph; the distance from A to B


equals the distance from B to A. Using the
Distance Formula, the solution is the same no
matter which ordered pair is used first.
3. See students diagrams; there are exactly two
points that lie on the line y  3 that are 10 units
from the point (7, 5).
4. d  2(x  x
2
1

)2

 (y2  y1

10.

)2

 2(11  5)  [7  (1) ] 2
 262  82
 136  64
 1100
 10
 2(2  3) 2  (5  7) 2
 2(5) 2  (12) 2
 125  144
 1169
 13
6. d  2(x  x ) 2  (y  y ) 2
2
1
2
1
 2(5  2) 2  (1  2) 2
 232  (3) 2
 19  9
 118
 312 or about 4.24
7. d  2(x  x ) 2  (y  y ) 2
2
1
2
1

8.

2(6  (3)) 2  [ 4  (5) ] 2


2(3) 2  (1) 2
19  1
110 or about 3.16

d  2(x2  x1 ) 2  (y2  y1 ) 2
10  2(a  3) 2  [7  (1) ] 2
10  2a2  6a  9  82
10  2a2  6a  9  64
102  ( 2a2  6a  73 ) 2
100  a2  6a  73
0  a2  6a  27
0  (a  3)(a  9)
a30
or a  9  0
a  3
a9

9.

AB 




2(5  (3) ) 2  (2  4) 2
282  (2) 2
164  4
168

BC 




2(1  5) 2  (5  2) 2
2(6) 2  (7) 2
136  49
185

AC  2(1  (3) ) 2  (5  4) 2


 222  (9) 2
 14  81
 185
Yes, BC  AC.
11. The quarterback is located at (40, 10). The top
receiver is located at (20, 25). The bottom receiver
is located at (15, 5).
Distance from quarterback to top receiver:
d  2(40  20) 2  (10  25) 2
d  2202  (15) 2
d  1400  225
d  1625
d  25 yd
Distance from quarterback to bottom receiver:
d  2(40  15) 2  (10  5) 2
d  2252  52
d  1625  25
d  1650
d  125  26
d  5126 or 25.5 yd
12. The receivers are located at (20, 25) and (15, 5).
d  2(20  15) 2  (25  5) 2
d  252  202
d  125  400
d  1425
d  125  17
d  5117 or 20.6 yd

5. d  2(x  x ) 2  (y  y ) 2
2
1
2
1






d  2(x2  x1 ) 2  (y2  y1 ) 2

Pages 613615

Practice and Apply

13. d  2(x  x ) 2  (y  y ) 2
2
1
2
1

d  2(x2  x1 ) 2  (y2  y1 ) 2

 2(8  12) 2  (3  3) 2
 2(20) 2  02
 1400
 20

1145  2(1  10) 2  (6  a) 2


1145  2(9) 2  36  12a  a2
( 1145) 2  ( 281  36  12a  a2 ) 2
145  117  12a  a2
0  a2  12a  28
0  (a  14)(a  2)
or a  2  0
a  14  0
a  14 or
a2

14. d  2(x  x ) 2  (y  y ) 2
2
1
2
1
 2(5  0) 2  (12  0) 2
 252  122
 125  144
 1169
 13

521

Chapter 11

PQ249-6481F-11[521-540] 26/9/02 8:15 PM Page 522 Sahuja Ahuja_QXP_06:Desktop Folder:Chandra:Algebra_FNL_Delivery:

15. d  2(x  x ) 2  (y  y ) 2
2
1
2
1

22. d  2(x2  x1 ) 2  (y2  y1 ) 2

1
15 2
 3 (2) 2  1 4 2

 2(3  6) 2  (4  8) 2
 2(3) 2  (4) 2
 19  16
 125
5
16. d  2(x  x ) 2  (y  y ) 2
2
1
2
1
 2(4  (4)) 2  (17  2) 2
 282  152
 164  225
 1289
 17
17. d  2(x  x
2
1

)2

 (y2  y1

34 

10
3

183 22

289

3 12  5 2

4 2

3 15 2

6 2

36

3 25  4

112 2

3 12  (1) 4

169

3 100

 1.30
24. d  2(x2  x1 ) 2  (y2  y1 ) 2

3 (4  3)

31

3 1  49

3 49

174
7

157 2


2

127  37 2

25

74

 1.23
25. d  2(x2  x1 ) 2  (y2  y1 ) 2
 2(615  415) 2  (1  7) 2
 2(215) 2  (6) 2
 120  36
 156
 2114
 7.48
26. d  2(x2  x1 ) 2  (y2  y1 ) 2
 2(712  512) 2  (10  8) 2
 2(212) 2  (2) 2
 18  4
 112
 213
 3.46

123  222

27.

d  2(x2  x1 ) 2  (y2  y1 ) 2
5  2(a  4) 2  (3  7) 2
5  2a2  8a  16  (4) 2
5  2a2  8a  32
(5) 2  ( 2a2  8a  32 ) 2
25  a2  8a  32
0  a2  8a  7
0  (a  1)(a  7)
a  1  0 or a  7  0
a1
a7

64
9

100
9

 3.33

Chapter 11

225
16

13

21. d  2(x2  x1 ) 2  (y2  y1 ) 2

32

17
4

1 2

 44

 10

 2(10  2) 2  (4  7) 2
 282  (11) 2
 164  121
 1185
 13.60

20. d  2(x2  x1 ) 2  (y2  y1 ) 2

3 16

 2(3  (8)) 2  [ 8  (4) ] 2


 252  (4) 2
 125  16
 141
 6.40

 2(3  9) 2  [ 6  (2) ] 2
 2(6) 2  (4) 2
 136  16
 152
 14  13
 2113
 7.21

3 (6  4)

34 

18. d  2(x2  x1 ) 2  (y2  y1 ) 2

23. d  2(x2  x1 ) 2  (y2  y1 ) 2

)2

19. d  2(x2  x1 )  (y2  y1 )

3 (3  5)

 4.25

 2(5  (3)) 2  (4  8) 2
 282  (4) 2
 164  16
 180
 116  5
 415
 8.94

522

PQ249-6481F-11[521-540] 26/9/02 8:15 PM Page 523 Sahuja Ahuja_QXP_06:Desktop Folder:Chandra:Algebra_FNL_Delivery:

28.

d  2(x  x ) 2  (y  y ) 2
2
1
2
1

AC  2(5  7) 2  [ 6  (4) ] 2
 2(2) 2  (2) 2
 14  4
 18
The triangle has two sides that are equal in
lengthAB  BC  10.

17  2(4  (4)) 2  (2  a) 2
17  282  4  4a  a2
17  268  4a  a2
2
(17) 2  ( 268  4a  a2 )
2
289  68  4a  a
0  221  4a  a2
0  a2  4a  221
0  (a  17)(a  13)
a  17 or a  13
29.

34.

d  2(x2  x1 ) 2  (y2  y1 ) 2
110  2(6  5) 2  (1  a) 2
110  212  1  2a  a2
( 110 ) 2  ( 21  1  2a  a2 ) 2
10  a2  2a  2
0  a2  2a  8
0  (a  4)(a  2)
a  4 or a  2

30.

35.

BD 




2(0  10) 2  (5  6) 2
2(10) 2  (1) 2
1100  1
1101

d  2(x2  x1 ) 2  (y2  y1 ) 2

PM  2(x  2) 2  (2  5) 2
 2x2  4x  4  (7) 2
 2x2  4x  53
PL  PM
2x2  8x  17  2x2  4x  53
( 2x2  8x  17 ) 2  ( 2x2  4x  53 ) 2
x2  8x  17  x2  4x  53
8x  17  4x  53
12x  36
x3

d  2(x2  x1 ) 2  (y2  y1 ) 2

36.

d  2(x2  x1 ) 2  (y2  y1 ) 2
QR 




d  2(x2  x1 ) 2  (y2  y1 ) 2
1340  2(a  20) 2  (9  5) 2
1340  2a2  40a  400  42
( 1340 ) 2  ( 2a2  40a  400  16 ) 2
340  a2  40a  416
0  a2  40a  76
0  (a  2)(a  38)
a20
or a  38  0
a2
a  38

33.

2[9  (2) ] 2  (8  2) 2
2112  62
1121  36
1157

PL  2(x  (4) ) 2  [ 2  (3) ] 2


 2x2  8x  16  1
 2x2  8x  17

d  2(x2  x1 ) 2  (y2  y1 ) 2

1130  2(3  6) 2  [ a  (3) ] 2


1130  2(9) 2  a2  6a  9
( 1130 ) 2  ( 281  a2  6a  9 ) 2
130  a2  6a  90
0  a2  6a  40
0  (a  10)(a  4)
a  10 or a  4
32.

AC 




1157  1101; The trapezoid is not isosceles.

129  2(7  a) 2  (3  5) 2
129  249  14a  a2  (2) 2
( 129 ) 2  ( 249  14a  a2  4 ) 2
29  a2  14a  53
0  a2  14a  24
0  (a  2)(a  12)
a  2 or a  12
31.

d  2(x2  x1 ) 2  (y2  y1 ) 2

2(3  1) 2  (1  7) 2
222  (6) 2
14  36
140

ST  2(7  9) 2  (d  3) 2
 2(2) 2  d2  6d  9
 2d2  6d  13
QR  ST
140  2d2  6d  13
( 140) 2  ( 2d2  6d  13 ) 2
40  d2  6d  13
0  d2  6d  27
0  (d  9) (d  3)
d  9  0 or d  3  0
d9
d  3
The distance formula demonstrates that d  3
does not make TQ  140. Therefore, d  9.

d  2(x2  x1 ) 2  (y2  y1 ) 2
AB  2(1  7) 2  [2  (4) ] 2
 2(8) 2  (6) 2
 164  36
 1100
 10
BC  2[5  (1) ] 2  (6  2) 2
 2(6) 2  (8) 2
 136  64
 1100
 10

523

Chapter 11

PQ249-6481F-11[521-540] 26/9/02 8:15 PM Page 524 Sahuja Ahuja_QXP_06:Desktop Folder:Chandra:Algebra_FNL_Delivery:

43. Compare the slopes of the two potential legs to


determine whether the slopes are negative
reciprocals of each other. You can also compute
the lengths of the three sides and determine
whether the square of the longest side length is
equal to the sum of the squares of the other two
side lengths. Neither test holds true in this case
because the triangle is not a right triangle.
44. You can determine the distance between two
points by forming a right triangle. Drawing a line
through each point parallel to the axes forms the
legs of the triangle. The hypotenuse of this triangle
is the distance between the two points. You can
find the lengths of each leg by subtracting the
corresponding x- and y-coordinates, then use the
Pythagorean Theorem. Answers should include
the following.
You can draw lines parallel to the axes
through the two points that will intersect at
another point forming a right triangle. The
length of a leg of a triangle is the difference in
the x- or y-coordinates. The length of the
hypotenuse is the distance between the points.
Using the Pythagorean Theorem to solve for
the hypotenuse, you have the Distance
Formula.
The points are on a vertical line so you can
calculate distance by determining the absolute
difference between the y-coordinates.

37. d  2(x  x ) 2  (y  y ) 2
2
1
2
1
 2(254  132) 2  (105  428) 2
 2(122) 2  (323) 2
 114,884  104,329
 1119,213
 345.27 units
Since each unit is equal to 0.316 mile, the
distance is 345.27(0.316)  109 mi.
38.

c2  a2  b2

114  18 2
9
3 2
c2  64  1 8 2
c2 

138 2
9

c2  64  64
18

c2  64
18

2c2   3 64
c  0.53 mi
39.

0.53
3

 0.18 hr

(0.18)(60)  10.6 minutes


Yes; it will take her about 10.6 minutes to walk
between the two buildings.
40. Duluth, (44, 116); St. Cloud, (46, 39); Eau Claire,
(71, 8); Rochester, (27, 58)
41. d  2(x2  x1 ) 2  (y2  y1 ) 2
Minneapolis St. Cloud:

45. B; d  2(x  x ) 2  (y  y ) 2
2
1
2
1

2(46  (7)) 2  (39  3) 2


2(39) 2  (3) 2
11521  1296
12817
 53 mi

 2(2  6) 2  (4  11) 2


 2(8) 2  (15) 2
 164  225
 1289
 17 units

St. Paul Rochester: 2(27  0) 2  (58  0) 2


2272  (58) 2
1729  3364
14093
 64 mi
Minneapolis Eau Claire:

46. B; d  2(x  x ) 2  (y  y ) 2
2
1
2
1
AB  2(3  3) 2  (4  7) 2
 2(6) 2  (3) 2
 136  9
 145
P  4s
 4145
 419  5
 1215 units

2(71  (7)) 2  (8  3) 2


2(78) 2  (11) 2
16084  121
16205
 79 mi
Duluth St. Cloud:

Page 615

2(46  44) 2  (39  116) 2


2(90) 2  (77) 2
18100  5929
114,029
 118 mi
42. all cities except Duluth

Chapter 11

47.

524

Maintain Your Skills

c2  a2  b2
c2  72  242
c2  49  576
c2  625
2c2  1625
c  25
The length of the hypotenuse is 25 units.

PQ249-6481F-11[521-540] 26/9/02 8:15 PM Page 525 Sahuja Ahuja_QXP_06:Desktop Folder:Chandra:Algebra_FNL_Delivery:

48.

c2  a2  b2
342  a2  302
1156  a2  900
256  a2
1256  2a2
16  a
The length of the leg is 16 units.

49.

c2  a2  b2
( 116 ) 2  ( 17 ) 2  b2
16  7  b2
9  b2
19  2b2
3  b
The length of the leg is 3 units.

50.

53.

?
?

16  2  8  6
?

14  8  6

1500  29  20  3

149  7
77

1529  23
23  23

The solution set is {2, 10}.


54. Asia, 1,113,000,000,000; Europe, 1,016,000,000,000;
U.S./Canada, 884,000,000,000; Latin America,
241,000,000,000; Middle East, 101,200,000,000;
Africa, 56,100,000,000.
55. Asia, 1.113  1012; Europe, 1.016  1012;
U.S./Canada, 8.84  1011; Latin America,
2.41  1011; Middle East, 1.012  1011; Africa,
5.61  1010.
56. $8.72  1011 or $872 billion
57.
8 m1
9 m
{m 0m
9}
2

58.

10

3 7 10  k
7 7 k
{k 0k 6 7}
8 7 6 5 4 3 2 1 0

59.

111  2  8  11
?

19  8  11
?

286
3  8  11
10  6
11  11
Since 6 does not satisfy the original equation,
11 is the only solution.
1r  5  r  1
52.
( 1r  5) 2  (r  1) 2
r  5  r2  2r  1
0  r2  3r  4
0  (r  4)(r  1)
r  4  0 or r  1  0
r4
r  1
Check:

120  29  4  3

51. 1p  2  8  p
1p  2  p  8
( 1p  2 ) 2  (p  8) 2
p  2  p2  16p  64
0  p2  17p  66
0  ( p  6) ( p  11)
p  6  0 or p  11  0
p6
p  11
Check:

25(2) 2  29  2(2)  3 25(10) 2  29  2(10)  3

c2  a2  b2
c2  ( 113 ) 2  ( 150 ) 2
c2  13  50
c2  63
2c2  163
c  163
c  19  7
c  317
The length of the hypotenuse is 317 or about
7.94 units.

25t2  29  2t  3
( 25t2  29 ) 2  (2t  3) 2
5t2  29  4t2  12t  9
2  12t  20  0
t
(t  2)(t  10)  0
t  2  0 or t  10  0
t2
t  10
Check:

3x 2x  3
x 3
{x 0x 3}
8 7 6 5 4 3 2 1 0

60. v  (4) 7 6
v4 7 6
v 7 2
{v 0v 7 2}
4 3 2 1 0

10

61. r  5.2
3.9
r
9.1
{r 0r
9.1}

14  5  4  1 11  5  1  1

19  3
14  2
33
2  2
Since 1 does not satisfy the original equation,
4 is the only solution.

62. s 

3
1
6

s
s

2
3
3
6
1
2

5s 0s 12 6

4 3 2 1 0

525

Chapter 11

PQ249-6481F-11[521-540] 26/9/02 8:15 PM Page 526 Sahuja Ahuja_QXP_06:Desktop Folder:Chandra:Algebra_FNL_Delivery:

63.

x
4

2

64.

2x  4(3)
2x  12
x6
65.

6
9

8
x

x  2
7

5
2

8.

5x  2(20)
5x  40
x  8
66.

6x  9(8)
6x  72
x  12
67.

20
x

10
12

2
3

7(x  2)  3(7)
7x  14  21
7x  7
x1

x

9.

7

c

 18

68.

BC

 EF

7c  30
AC
DF
b
5

12x  10(18)
12x  180
x  15

7

AB
DE
c
6




b

2(x  4)  3(6)
2x  8  18
2x  10
x5

AC

 DF


15
e

10e  90

30
7

e9

BC
EF
5
7

AB
DE
10
6

BC

 EF


17
d

10d  102
d  10.2

7b  25

6
 4

AB
DE
10
6

25
7

10. Let x  the height of the school building.


10
26

25
x

10x  650
x  65 feet
The school building is 65 feet high.

11-6 Similar Triangles


Pages 618619

Pages 619620

Check for Understanding

1. If the measures of the angles of one triangle equal


the measures of the corresponding angles of
another triangle, and the lengths of the sides are
proportional, then the two triangles are similar.
2. Sample answer: ^ ABC  ^DEF

AB
DE
15
5

AC

 DF
b

9

7.

AB
DE
9
6

KL
NO
m
4

19.

5a  105
a  21

Chapter 11

KL
NO
11
6

KL
NO
11
6

18.




LM
OP
K
5




55
6
KM
NP
/
4

/


BC

18
d

9d  108
d  12

526

44
6
22
3

LM
OP
24
16




KM
NP
30
o

24o  480
o  20

LM
OP
9
6

6/  44

 EF


K

 10

AB
DE
9
6

a
7

KM
NP
/
8

6K  55

AC

AB
DE
15
5

KL
NO
15
p

6m  36
m6

 DF

6b  90
b  15

BC

LM
OP
9
6

F
corresponding sides
AB and DE
BC and EF

5b  135
b  27
 EF

17.

6/  72
/  12

C and  F
AC and DF
3. Consuela; the arcs indicate which angles
correspond. The vertices of the triangles are
written in order to show the corresponding parts.
4. No; the angle measures are not equal.
5. Yes; the angle measures are equal.
6.

Yes; the angle measures are equal.


No; the angle measures are not equal.
No; the angle measures are not equal.
Yes; the angle measures are equal.
No; the angle measures are not equal.
Yes; the angle measures are equal.

D
corresponding angles
 A and  D
 B and  E

Practice and Apply

11.
12.
13.
14.
15.
16.




LM
OP
24
16

24p  240
p  10
20.

KL
NO
12
p




KM
NP
13
7

13p  84
84

p  13
LM
OP
16
n




KM
NP
13
7

13n  112
n

112
13

PQ249-6481F-11[521-540] 26/9/02 8:15 PM Page 527 Sahuja Ahuja_QXP_06:Desktop Folder:Chandra:Algebra_FNL_Delivery:

21.

KL
NO

LM
OP

1.25
2.5

K
6

22.

2.5K  7.5
K3
KL
NO
1.25
2.5




KL
NO
m
2.7




KM
NP
4
o




LM
OP

7.5
5

10.5
n

KL
NO
7.5
5

33. Yes; all circles are similar because they have the
same shape.
34. 2:1; Let the first circle have radius r and the
larger have radius 2r. The circumference of the
first is 2r and the other has circumference
2(2r)  4r.
35. 4:1; The area of the first is r2 and the area of
the other is (2r)2  4r2.
36. The size of an object on the film of a camera can
be related to its actual size using similar triangles.
Answers should include the following.
Moving the lens closer to the object (and
farther from the film) makes the object appear
larger.
Taking a picture of a building; you would need
to be a great distance away to fit the entire
building in the picture.
37. D; We do not have enough information to
determine if the triangles are similar.
38. A; AB AC and AC DC. Therefore, AB DC.




KM
NP
15
o

7.5o  75
o  10

KM
NP
4.4
3.3

24.

3.3m  11.88
m  3.6
LM
QP
K
2.1

7.5n  52.5
n7

1.25o  10
o8
23.

KL
NO

KL
NO
5
2.5




LM
QP
12.6
n

5n  31.5
n  6.3

KM
NP
4.4
3.3

KL
NO
5
2.5




KM
NP
/
8.1

3.3K  9.24
2.5/  40.5
K  2.8
/  16.2
25. Always; if the measures of the sides form equal
ratios, the triangles are similar, and the measures
of their corresponding angles are equal.
26. Let x  the distance the man is from the camera.
x
3

Page 621

 1.5

 2(2  1) 2  (4  8) 2
 2(3) 2  (4) 2
 19  16
 125
5

1.5x  6
x4m
27. Let x  height on the model.
1
12

 40

40. d  2(x  x ) 2  (y  y ) 2
2
1
2
1

12x  40
1

 2(12  6) 2  [ 5  (3) ] 2
 2(6) 2  (8) 2
 136  64
 1100
 10

x  33 in.
28. Let x  distance from pocket B.
84
x
84
x

 42 

Maintain Your Skills

39. d  2(x  x ) 2  (y  y ) 2
2
1
2
1

28
(10  x)

28

 32  x

28x  2688  84x


112x  2688
x  24 in.

41. d  2(x  x ) 2  (y  y ) 2
2
1
2
1
 2(3  4) 2  (12  7) 2
 2(1) 2  (5) 2
 11  25
 126
 5.1

29.

42. d  2(x  x ) 2  (y  y ) 2
2
1
2
1
 2(6  1) 2  (716  516) 2
 252  (216) 2
 125  24
 149
7
43. Yes; 252  602  652.

8 pieces
30. twice as big; 4 by 4 by 5 in.
31. Let x  height of the building.
x
6

80
9

44. No; 202  252  352.

9x  480

45. Yes; 492  1682  1752.

x  533

46. No; 72  92  122.

The building is about 53 feet.


32. Vihos eyes are 6 feet off the ground, Viho and the
building each create right angles with the ground,
and the two angles with the ground at P have
equal measure.

47. 3x2  7x  1
48. 5x3  2x2  4x  7
49. 3x2  6x  3
50. x7  abx2  bcx  34

527

Chapter 11

PQ249-6481F-11[521-540] 26/9/02 8:15 PM Page 528 Sahuja Ahuja_QXP_06:Desktop Folder:Chandra:Algebra_FNL_Delivery:

2x  y  4
() x  y  5
3x
9
x3
Substitute 3 for x into either equation.
3y5
y  2
y  2
The solution is (3, 2).
52.
3x  2y  13  2
6x  4y  26
() 2x  5y  5  3 () 6x  15y  15
11y  11
y  1
Substitute 1 for y into either equation.
3x  2(1)  13
3x  2  13
3x  15
x  5
The solution is (5, 1).
53. 0.6m  0.2n  0.9
0.3m  0.45  0.1n
51.

58.


60.

()

()

 2y  8  2
 4y  0

1
x
6
1
x
2
4
x
6

1.

4

6

5
6


 b2
2
c  14  482
c2  196  2304
c2  2500
2c2  12500
c  50
The length of the hypotenuse is 50 units.

4.

c2  a2  b2
c2  ( 15 ) 2  ( 18 ) 2
c2  5  8
c2  13
2c2  113
c  113
The length of the leg is 113 or about 3.61 units.

 2(5  1) 2  (11  3) 2
 2(6) 2  (8) 2
 136  64
 1100
 10

or 0.83

 4

Chapter 11

Practice Quiz 2

6. d  2(x2  x1 ) 2  (y2  y1 ) 2

32,500 ft
739,200 ft
b

1.5
(1.5)
1.5
4.5
1
3 or 0.3

6

a2

 2(3  6) 2  (3  (12)) 2
 2(9) 2  (15) 2
 181  225
 1306
 17.49

32,500 ft
140 mi

57. a 

c2

or 1.1

5. d  2(x  x ) 2  (y  y ) 2
2
1
2
1

 0.044


c
a  c

c2  a2  b2
( 184 ) 2  a2  82
84  a2  64
20  a2
120  2a2
215  a
The length of the leg is 215 or about 4.47 units.

 232 ft/mi.
Alternatively, since there are 5280 ft per mile,
there is a horizontal gain of
(140) (5280)  739,200 ft.

56.

61.

3.

 4y  0

y  12
The solution is (6, 12).
55. The plane has a vertical decent of 32,500 ft and a
horizontal gain of 140 mi giving a slope of

6
1.5

c2  a2  b2
412  402  b2
1681  1600  b2
81  b2
181  2b2
9  b
The length of the leg is 9 units.

a
c

6(1.5)
5
9
5
9
or 1.8
5

2.

 4y  4

2  2y  8

Therefore m 

m 

ac
b

10
9

Page 621

 2y  8
1
y
2

5
(1.5)

6

59.

5

x6
Substitute 6 for x into either equation.
1
(6)
3

b
a  c

6  5
1.5
1
1.5
2
3 or 0.6

 4.5

Substitute 1.5 for m into either equation.


0.6(1.5)  0.2n  0.9
0.9  0.2n  0.9
0.2n  0
n0
The solution is (1.5, 0).
1
x
3
1
x
2




0.6m  0.2n  0.9


0.6m  0.2n  0.9
() 0.3m  0.1n  0.45  2 () 0.6m  0.2n  0.9
1.2m
 1.8
m  1.5

54.

a  b
c

528

PQ249-6481F-11[521-540] 26/9/02 8:15 PM Page 529 Sahuja Ahuja_QXP_06:Desktop Folder:Chandra:Algebra_FNL_Delivery:

Pages 627628

7. d  2(x  x ) 2  (y  y ) 2
2
1
2
1
 2(4  2) 2  (7  5) 2
 2(2) 2  (2) 2
 14  4
 18
 212
 2.83
8. d  2(x  x
2
1

)2

 (y2  y1

sin 50 

)2

BC
EF
a
2

CA

 FD


10.

10
1

a  20
BA
ED
c
1.5

BA

 ED


CA

BC
EF
12
8

10
1

c  15

cos 50 

BC  6.43

9
f

50

3. They are equal.


leg
4. sin Y  opposite
hypotenuse

CA

 FD

cos Y 

36
45

 0.8

 12

8b  144
b  18

Algebra Activity
(Preview of Lesson 11-7)

6.

cos Y 

10
26

Side Lengths

Ratios

7.

6.1

0.7

0.574

35

24
26

 0.9231

36
27

 1.3333
tan Y 


opposite leg
adjacent leg
10
24

 0.4167

SIN 60 ENTER .8660254038

55

90

10

12.2

0.7

0.574

35

55

90

20

24.4

0.7

0.574

35

55

90

28

40

48.8

0.7

0.574

35

55

90

35

50

61

0.7

0.574

35

55

90

42

60

73.2

0.7

0.574

35

55

90

KEYSTROKES:

COS 75 ENTER .2588190451

TAN 10 ENTER .1763269807


tan 10  0.1763
9. sin W  0.9848
1
KEYSTROKES: 2nd [SIN ] .9848 ENTER
79.99744219 sin W  80
10. cos X  0.6157
1
KEYSTROKES: 2nd [COS ] .6157 ENTER
8.

angle angle
B
C

14

KEYSTROKES:

51.99719884 cos X  52
11. tan C  0.3249
1
KEYSTROKES: 2nd [TAN ] .3249 ENTER
17.99897925 tan C  18
12. Let A  the angle to find.

1. All ratios and angle measures are the same for


any 7:10 right triangle.
2. 7:10
3. 55

sin A 

11-7 Trigonometric Ratios


Page 626

adjacent leg
hypotenuse

opposite leg
adjacent leg

cos 75  0.2588

Angle Measures

3.5

27
45

tan Y 

sin 60  0.8660

Steps 14. Sample answers are given in the table.


side side side
angle
BC:AC BC:AB
BC AC AB
A

KEYSTROKES:

adjacent leg
hypotenuse

 0.6

leg
5. sin Y  opposite
hypotenuse

 0.3846

Page 622

BC
10

10

12f  72
f6

 FD


BC
EF
12
8

AC
10

AC  7.66
A

 2(5  (2)) 2  [ 4  (9) ] 2


 2(3) 2  (13) 2
 19  169
 1178
 13.34
9.

Check for Understanding

1. If you know the measure of the hypotenuse, use


sine or cosine, depending on whether you know
the measure of the adjacent side or the opposite
side. If you know the measures of the two legs,
use tangent.
2. Sample answer: A  180  (90  50 ) or 40

opposite leg
hypotenuse

 13

2nd [SIN1] 7 13 ENTER


32.57897039 sin A  33

KEYSTROKES:

13. Let A  the angle to find.

Algebra Activity

opposite leg

1. See students work.


2. The angle measured by the hypsometer is not the
angle of elevation. It is the other acute angle
formed in the triangle. So, to find the measure of
the angle of elevation, subtract the reading on the
hypsometer from 90 since the sum of the
measures of the two acute angles in a right
triangle is 90.
3. See students work.

tan A  adjacent leg  15

2nd [TAN1] 6 15 ENTER


21.80140949 tan A  22
14. Let A  the angle to find.
KEYSTROKES:

cos A 

adjacent leg
hypotenuse

9.3

 9.7

1
KEYSTROKES: 2nd [COS ] 9.3 9.7 ENTER
16.5114644 cos A  17

529

Chapter 11

PQ249-6481F-11[521-540] 26/9/02 8:15 PM Page 530 Sahuja Ahuja_QXP_06:Desktop Folder:Chandra:Algebra_FNL_Delivery:

15. Find the measures of  A, AC, and BC.


Find the measure of  A.
The sum of the measures of the angles in a
triangle is 180.
180  90  30  60
 A  60
Find the measure of AC, which is the side opposite
B. Use the sine ratio.
sin 30 
0.50 

18. Draw a diagram.

0.8660 

AC
42
AC
42

opposite leg

tan A  adjacent leg


40

tan A  1000
Use a calculator.
1
KEYSTROKES: 2nd [TAN ] 40 1000 ENTER
2.290610043
 A  2.3

BC
42
BC
42

Pages 628630

36.4  BC or 36.4 in.


BC  36.4 in.
16. Find the measures of  A, AC, and AB.
Find the measure of  A.
180  90  35  55
 A  55
Find the measure of AC, which is the side opposite
B.
tan 35 
0.7002 

19. sin R 


cos R 

6
10

12
37

21. sin R 

AC
18
AC
18

opposite leg
hypotenuse


cos R 


 0.7241

22. sin R 


cos R 

16
34

leg
23. sin R  opposite
hypotenuse

18

0.8192  AB

21.97  AB
AB  22 m
17. Find the measures of B, AB, and BC.
Find the measure of B.
180  90  55  35
 A  35
Find the measure of AB, which is the hypotenuse.

7
1170

 0.5369

24. sin R 


4
4

0.5736  AB
6.973  AB
AB  7.0 in.
Find the measure of BC, which is the side opposite
 A.
BC
4
BC
4

5.71  BC
BC  5.7 in.

530

35
37

adjacent leg
hypotenuse
20
29

18
22

6
8

adjacent leg
hypotenuse
30
34

tan R 

11
1170

tan R 

4 110
22

 0.5750

26.
28.
30.
32.

21
20

opposite leg
adjacent leg
16
30

opposite leg
adjacent leg
7
11

 0.6364
tan R 


 0.8182

opposite leg
adjacent leg

 0.5333
tan R 


adjacent leg
hypotenuse

12
35

 1.05

adjacent leg
hypotenuse

opposite leg
adjacent leg

 0.3429

 0.8437
cos R 

tan R 


sin 30  0.5
cos 45  0.7071
tan 32  0.6249
tan 67  2.3559
cos 12  0.9781
cos V  0.5000
V  cos1 0.5000
 60
36. sin K  0.9781
K  sin1 0.9781
 78
38. tan S  1.2401
S  tan1 1.2401
 51
40. sin V  0.3832
V  sin1 0.3832
 23
25.
27.
29.
31.
33.
34.

cos 55  AB

opposite leg
hypotenuse

opposite leg
adjacent leg

 0.75

adjacent leg
hypotenuse

 0.8824
cos R 

tan R 


 0.6897

 0.4706

18

8
10

 0.9459

21
29

opposite leg
hypotenuse

adjacent leg
hypotenuse

 0.8
cos R 

 0.3243

cos 35  AB

Chapter 11

Practice and Apply

 0.6

Find the measure of AB, which is the hypotenuse.

1.4281 

opposite leg
hypotenuse

leg
20. sin R  opposite
hypotenuse

12.6  AC
AC  12.6 m

tan 55 

 0.04
1000 ft
 4%
To find the angle of elevation, note that the 40 ft
vertical rise is opposite the angle and the 1000 ft
horizontal change is adjacent the angle. This
suggests we use tangent.

21  AC
AC  21 in.
Find the measure of BC, which is the side adjacent
B. Use the cosine ratio.
cos 30 

40 ft

40

percent grade  1000

opposite leg
adjacent leg
18
4 110

 1.4230

sin 80  0.9848
cos 48  0.6691
tan 15  0.2679
sin 53  0.7986

35. cos Q  0.7658


Q  cos1 0.7658
 40
37. sin A  0.8827
A  sin1 0.8827
 62
39. tan H  0.6473
H  tan1 0.6473
 33
41. cos M  0.9793
M  cos1 0.9793
 12

PQ249-6481F-11[521-540] 26/9/02 8:15 PM Page 531 Sahuja Ahuja_QXP_06:Desktop Folder:Chandra:Algebra_FNL_Delivery:

42. tan L  3.6541


L  tan1 3.6541
 75
43. Let A  the angle that we need to find.
sin A 
sin A 
A

Now find the measure of C.


cos C 
cos C 
C

opposite leg
hypotenuse
10
16
10
sin1 16

1 2

opposite leg

tan A  adjacent leg


2

A  tan1

1142 2

 8
45. Let A  the angle that we need to find.
cos A 
cos A 
A

adjacent leg
hypotenuse
10
16
10
cos1 16

1 2

 51
46. Let A  the angle that we need to find.
sin A 
sin A 
A

opposite leg

tan A  adjacent leg

opposite leg
hypotenuse
21
25
21
sin1 25

24

tan A  16
A  tan1

1 2

cos A 
A

adjacent leg
hypotenuse
17
21
17
cos1 21

tan 45  8

1 2

18
8b

 36
48. Let A  angle that we need to find.

cos 45 

opposite leg

0.7071 

tan A  adjacent leg


tan A 

5
8

A  tan1

sin A 
A

8
c
8
c

0.7071c  8
c  11.3 ft
53. 180  90  27  63
The measure of A is 63 .

158 2

 32
49. Let A  the angle that we need to find.
sin A 

12416 2

 56
The measure of A is 56 .
52. 180  90  45  45
The measure of A is 45 .

 57
47. Let A  the angle that we need to find.
cos A 

1 2

 69
Now the measure of A is the difference between
the measures of C and B.
69  62  7
The measure of A is 7 .
51. Let A  the angle that we need to find. We can
use the Pythagorean Theorem to find the length
of the lower leg of the smaller right triangle.
a2  b2  c2
a2  242  252
a2  576  625
a2  49
a  149
a7
Now the length of the lower leg of the larger right
triangle is 23  7 or 16 units. Now find the
measure of A.

 39
44. Let A  the angle that we need to find.
tan A  14

adjacent leg
hypotenuse
16
45
16
cos1 45

sin 27  20

opposite leg
hypotenuse
9
15
9
sin1 15

0.4540  20

1 2

9.1 in  b
a

cos 27  20

 37
50. Let A  the angle that we need to find, B  the
upper angle in the smaller right triangle, and
C  the upper angle in the larger right triangle.
First find the measure of B.

0.8910  20
17.8 in  a

opposite leg

tan B  adjacent leg


30

tan B  16
B  tan1
 62

13016 2
531

Chapter 11

PQ249-6481F-11[521-540] 26/9/02 8:15 PM Page 532 Sahuja Ahuja_QXP_06:Desktop Folder:Chandra:Algebra_FNL_Delivery:

54. 180  90  21  69
The measure of B is 69 .

58.

13
c
13
c

cos 21 
0.9336 

0.9336c  13
c  13.9
AB is about 13.9 cm long.

tan A  6

tan 21  13

A  tan1

5.0  a
CB is about 5.0 cm long.
55. 180  90  40  50
The measure of B is 50 .
a
a

0.6428  16
10.3  a
BC is about 10.3 ft long.
b

cos 40  16
b

0.7660  16
12.3  b
AC is about 12.3 ft long.
56. 180  90  20  70
The measure of A is 70 .
0.3640 

tan A  12
A  tan1

33  b
AC is about 3.3 m long.
0.9397 

9
c
9
c

0.9397c  9
c  9.6
AB is about 9.6 m long.
57. 180  90  38  52
The measure of B is 52 .
tan 38 
0.7813 

sin A  6
A  sin1

24
b
24
b

0.6157 

626

61. sin A  4420


A  sin1

24
c
24
c

 8.1
62.

0.6157c  24
c  39
AB is about 39 in. long.

Chapter 11

136 2

 30
The measure of A is 30 .
180  90  30  60
The measure of B is 60 .

0.7813b  24
b  30.7
AC is about 30.7 in. long.
sin 38 

1125 2

 23
The measure of A is 23 .
180  90  23  67
The measure of B is 67 .
c2  a2  b2
60.
62  32  b2
36  9  b2
27  b2
127  2b2
5.2  b
AC is about 5.2 cm long.

b
9
b
9

cos 20 

186 2

 53
The measure of A is 53 .
180  90  53  37
The measure of B is 37 .
59.
c2  a2  b2
c2  122  52
c2  144  25
c2  169
2c2  1169
c  13
AB is 13 ft long.

sin 40  16

tan 20 

c2  a2  b2
c2  62  82
c2  36  64
c2  100
2c2  1100
c  10
AB is 10 ft long.

532

626
14420
2

c2  a2  b2
(4420) 2  a2  (626) 2
19,536,400  a2  391,876
19,144,524  a2
119,144,524  2a2
4375  a
The submarine traveled a horizontal distance of
about 4375 m.

PQ249-6481F-11[521-540] 26/9/02 8:15 PM Page 533 Sahuja Ahuja_QXP_06:Desktop Folder:Chandra:Algebra_FNL_Delivery:

3000

63. tan A  8000


A  tan1
 20.6
64.

c2

a2

Page 630

13000
8000 2

70.

b2



c2  80002  30002
c2  64,000,000  9,000,000
c2  73,000,000
2c2  173,000,000
c  8544
The distance is 8544 ft.
x

72.

0.7  x
x

sin 20  8
x

0.3420  8
2.74  x
from about 0.7 m to about 2.74 m
b

66. Let sin A  c and let cos A  c , where a and b are


legs of a right triangle and c is the hypotenuse.
a2
c2

b2
c2

a2  b2
.
c2

71.

KL
NO
3
4.5




KM
NP
9
o

5p  60
p  12

3o  40.5
o  13.5

LM
OP
5
10

KL
NO
3
4.5




KM
NP
3
o




LM
OP
k
12

4.5k  36
k8

d  2(x2  x1 ) 2  (y2  y1 ) 2
39  2(a  9) 2  (8  28) 2
39  2a2  18a  81  (36) 2
39  2a2  18a  81  1296
(39) 2  ( 2a2  18a  1377) 2
1521  a2  18a  1377
0  a2  18a  144
0  (a  24)(a  6)
or a  6  0
a  24  0
a  24
a  6

0.0872  8

Then sin2A  cos2 A 

Maintain Your Skills


KL
NO
6
p

5o  30
o6

65. sin 5  8

LM
OP
5
10

73.

Since the

d  2(x2  x1 ) 2  (y2  y1 ) 2
165  2(10  3) 2  (1  a) 2
165  272  1  2a  a2
( 165 ) 2  ( 250  2a  a2 ) 2
65  a2  2a  50
0  a2  2a  15
0  (a  5) (a  3)
a  5 a  3

Pythagorean Theorem states that a2  b2  c2, the


c2
expression becomes c2 or 1. Thus sin2A  cos2A  1.
67. If you know the distance between two points and
the angles from these two points to a third point,
you can determine the distance to the third point
by forming a triangle and using trigonometric
ratios. Answers should include the following.
If you measure your distance from the
mountain and the angle of elevation to the
peak of the mountain from two different
points, you can write an equation using
trigonometric ratios to determine its height,
similar to Example 5.
You need to know the altitude of the two
points you are measuring.
68. A; Use the Pythagorean Theorem to find RT.
c2  a2  b2
42  a2  22
16  a2  4
12  a2
112  2a2
213  a

74. c2 (c2  3c)  c4  3c3


75. s(4s2  9s  12)  4s3  9s2  12s
76. xy2 (2x2  5xy  7y2 )  2x3y2  5x2y3  7xy4
77.
a  3b  2
4a  7b  23
Substitute 3b  2 for a in the other equation.
4(3b  2)  7b  23
12b  8  7b  23
5b  8  23
5b  15
b3
Substitute 3 for b into either equation.
a  3(3)  2
92
 11
Checking values in both equations confirms the
solution is (11, 3).

RT is 213 units so TS is 213 units.


Use the Pythagorean Theorem to find RS.
c2  (213 ) 2  (213 ) 2
c2  12  12
c2  24
2c2  124
c  216
RS is 216 units.
2

69. D; cos Q  4
Q  cos1
 60

124 2
533

Chapter 11

PQ249-6481F-11[521-540] 26/9/02 8:15 PM Page 534 Sahuja Ahuja_QXP_06:Desktop Folder:Chandra:Algebra_FNL_Delivery:

p  q  10
3p  2q  5
Solve the first equation for p.
p  10  q
Substitute 10  q for p into the other equation.
3(10  q)  2q  5
30  3q  2q  5
30  5q  5
5q  35
q7
Substitute 7 for q into either equation.
p  10  7
3
Checking values in both equations confirms the
solution is (3, 7).
79. 3r  6s  0
4r  10s  2
Solve the first equation for r.
3r  6s  0
3r  6s
r  2s
Substitute 2s for r into the other equation.
4(2s)  10s  2
8s  10s  2
2s  2
s1
Substitute 1 for s into either equation.
r  2(1)
 2
Checking values in both equations confirms the
solution is (2, 1).

3a. Sample answer: dome, domestic, domicile


3b. Sample answer: eradicate, radicand, radius
3c. Sample answer: simile, similarity, similitude

78.

Chapter 11 Study Guide and Review


Page 632

2. true
4. true

6. true

7. false,

Pages 632636
9.

60
y2

Lesson-by-Lesson Review

160
1y2

14  115
0y 0

2 115
0y 0

10. 244a2b5  24  11  a2  b4  b
 14  111  2a2  2b4  1b
 2  111  0a 0  b2  1b
 2 0a 0b2 111b

11. (3  2112 ) 2  32  2(3) (2112 )  (2112 ) 2


 9  12112  48
 57  12(213 )
 57  2413
9
3  12

Reading Mathematics

1a. Sample answer: A circumstance that brings


about a result; any of two or more quantities
that form a product when multiplied together;
the quantities bring about a result, a product.
1b. Sample answer: The legs of an animal support
the animal; one of the two shorter sides of a
right triangle; the legs of a triangle support the
hypotenuse.
1c. Sample answer: To devise rational explanations
for ones acts without being aware that these are
not the real motives; to remove the radical signs
from an expression without changing its value;
to justify an action without changing its intent.
2a. Sample answer: Rank as determined by the sum
of a terms exponents; the degree of x2y2 is 4.
1
th of a circle; a semicircle measures 180 .
360
2b. Sample answer: The difference between the
greatest and least values in a set of data; the
range of 2, 3, 6 is 6  2 or 4. The set of all
y-values in a function; the range of {(2, 6), (1, 3)}
is {6, 3}.
2c. Sample answer: Circular or spherical; circles are
round. To abbreviate a number by replacing its
ending digits with zeros; 235,611 rounded to the
nearest hundred is 235,600.

Chapter 11

x 12xy
y

8. true

12.

Page 631

Vocabulary and Concept Check

1. false, 3  17
3. true
5. false, 3x  19  x2  6x  9

13.

14.

3

3  12
12

3 

9(3  12)
9  2

27  9 12
7

2 17
3 15  5 13

23a3b4
28ab10

12

2 17

3 15  5 13

 3 15  5 13  3 15  5 13


2 17(3 15  5 13)
45  75

2 17(3 15  5 13)
30

 17(3 15  5 13)
15

5 121  3 135
15

a3

b4

3 8  a  b10

3 8b6

13  2a2
18  2b6

3a2

0 a 0 13

 2 0 b3 0 12
0 a 0 13

12

 2 0 b3 0 12  12


0 a 0 16
4 0 b3 0

15. 213  815  315  313


 (2  3) 13  (8  3) 15
 513  515
16. 216  148  216  116  3
 216  413

534

PQ249-6481F-11[521-540] 26/9/02 8:16 PM Page 535 Sahuja Ahuja_QXP_06:Desktop Folder:Chandra:Algebra_FNL_Delivery:

17. 4127  6148  4(313)  6(413)


 1213  2413
 3613
18. 417k  717k  217k  (4  7  2) 17k
 17k
19. 5118  31112  3198
 5(312 )  3(417 )  3(712)
 1512  1217  2112
 (15  21) 12  1217
 6 12  1217
20. 18 

29.

Check:

2 12
1

12
4

8 12
4

12
4

9 12
4

21. 12(3  313 )  312  (313 )( 12 )


 312  316
22. 15(215  17 )  (215 )( 15 )  ( 17 )( 15 )
 10  135
23. ( 13  12 ) (212  13 )

13(5)  14  5  6

 ( 13) (212)  ( 13) ( 13)  12(212)  12( 13)


 2 16  3  4  16
 1  16

24. (6 15  2) (3 12  15 )
 (6 15) (3 12)  (6 15) ( 15)  2 (3 12)  2 15
 18 110  30  6 12  2 15

25. 10  21b  0

Check:

21b  10
1b  5
( 1b ) 2  (5) 2
b  25
No solution
26.

1a  4  6
( 1a  4 ) 2  62
a  4  36
a  32

27.

Check:
17x  1  5
2
2
( 17x  1 )  (5)
7x  1  25
7x  26
x

28.

4a

33

20
4a

33

2

1 3 4a3 22  22
4a
3

4
a3

Check:

Check:

31.

10  2125  0
?
10  2(5)  0
?
10  10  0
20  0

32.

132  4  6
?
136  6
66

37 1 7 2  1  5
?

26

126  1  5
?
125  5
55

26
7

4(3)
3

15  4  5  8

112  4  12  8
?

19  3
116  4
3  3
44
Since 5 does not satisfy the original equation, 12 is
the only solution.
30. 13x  14  x  6
13x  14  6  x
( 13x  14 ) 2  (6  x) 2
3x  14  36  12x  x2
0  x2  15x  50
0  (x  5) (x  10)
x  5  0 or x  10  0
x  10
x5
Check:

3 8  212  2 12


1x  4  x  8
( 1x  4 ) 2  (x  8) 2
x  4  x2  16x  64
0  x2  17x  60
0  (x  5) (x  12)
x  5  0 or x  12  0
x5
x  12

33.

20
?

14  2  0
?

220
34.

00

535

13(10)  14  10  6
?

116  10  6
11  5  6
?
?
156
4  10  6
66
14  6
Since 10 does not satisfy the original equation, 5 is
the only solution.
c2  a2  b2
c2  302  162
c2  900  256
c2  1156
2c2  11156
c  34
The hypotenuse is 34.
c2  a2  b2
c2  62  102
c2  36  100
c2  136
2c2  1136
c  11.66
The hypotenuse is 11.66.
c2  a2  b2
152  102  b2
225  100  b2
115  b2
1115  2b2
10.72  b
The length of the leg is 10.72.
c2  a2  b2
562  a2  42
3136  a2  16
3120  a2
13120  2a2
55.86  a
The length of the leg is 55.86.

Chapter 11

PQ249-6481F-11[521-540] 26/9/02 8:16 PM Page 536 Sahuja Ahuja_QXP_06:Desktop Folder:Chandra:Algebra_FNL_Delivery:

c2  a2  b2
302  182  b2
900  324  b2
576  b2
1576  2b2
24  b
The length of the leg is 24.
36.
c2  a2  b2
c2  (1.2) 2  (1.6) 2
c2  1.44  2.56
c2  4
2c2  14
c  2
The length of the hypotenuse is 2.

46. d  2(x  x ) 2  (y  y ) 2
2
1
2
1

35.

 2[5  (2) ] 2  (11  6) 2


 272  52
 149  25
 174
 8.60
d  2(x2  x1 ) 2  (y2  y1 ) 2

47.

5  2 [1  (3) ] 2  (a  2) 2
5  242  a2  4a  4
5  2a2  4a  20
52  ( 2a2  4a  20 ) 2
25  a2  4a  20
0  a2  4a  5
0  (a  5)(a  1)
a  5  0 or a  1  0
a5
a  1

37. no; 92  162  202


38. yes; 202  212  292
39. yes; 92  402  412

d  2(x2  x1 ) 2  (y2  y1 ) 2

48.

40. no; 182  ( 124 ) 2  302

5  2(4  1) 2  (a  1) 2
5  232  a2  2a  1
(5) 2  ( 2a2  2a  10 ) 2
25  a2  2a  10
0  a2  2a  15
0  (a  5)(a  3)
a  5  0 or a  3  0
a5
a  3

41. d  2(x  x ) 2  (y  y ) 2
2
1
2
1
 2(1  9) 2  [ 13  (2) ] 2
 2(8) 2  (15) 2
 164  225
 1289
 17
42. d  2(x  x ) 2  (y  y ) 2
2
1
2
1

d  2(x2  x1 ) 2  (y2  y1 ) 2

49.

 2(7  4) 2  (9  2) 2
 232  72
 19  49
 158
 7.62

1145  2(5  6) 2  [ a  (2) ] 2


1145  2(1) 2  a2  8a  4
( 1145 ) 2  ( 2a2  8a  5 ) 2
145  a2  8a  5
0  a2  8a  140
0  (a  14)(a  10)
a  14  0
or a  10  0
a  14
a  10

43. d  2(x2  x1 ) 2  (y2  y1 ) 2


 2(2  4) 2  [7  (6) ] 2
 2(6) 2  (13) 2
 136  169
 1205
 14.32

d  2(x2  x1 ) 2  (y2  y1 ) 2

50.

1170  2(a  5) 2  [ 3  (2) ] 2


1170  2a2  10a  25  1
( 1170) 2  ( 2a2  10a  26 ) 2
170  a2  10a  26
0  a2  10a  144
0  (a  18)(a  8)
a  18  0 or a  8  0
a  18
a  8

44. d  2(x2  x1 ) 2  (y2  y1 ) 2


 2(415  2 15) 2  (3  9) 2
 2(215) 2  (6) 2
 120  36
 156
 2114
 7.48

51.

45. d  2(x  x ) 2  (y  y ) 2
2
1
2
1
 2(7  4) 2  (12  8) 2
 2(11) 2  (4) 2
 1121  16
 1137
 11.70

AB
DE
16
9

AC

 DF


12
e

16e  108
e
AB
DE
16
9

27
4
BC

 EF


10
d

16d  90
d

Chapter 11

536

45
8

52.

AB
DE
10
12

AC

 DF


6
e

10e  72
e  7.2
AB
DE
10
12

BC

 EF
8

d

10d  96
d  9.6

PQ249-6481F-11[521-540] 26/9/02 8:16 PM Page 537 Sahuja Ahuja_QXP_06:Desktop Folder:Chandra:Algebra_FNL_Delivery:

53.

AB
DE
12
9

AC

 DF
b

 11

9b  132
b
AB
DE
12
9

AB
DE
15
6

54.

BC

AB
DE
15
6

d

12d  72
d6

BC

 EF


28
53

 0.8491
28

60. sin A  53

 1.6071
 0.5283
61. tan M  0.8043
62. sin I  0.1212
I  sin1 (0.1212)
M  tan1 (0.8043)
 39
 7
63. cos B  0.9781
64. cos F  0.7443
B  cos1 (0.9781)
F  cos1 (0.7443)
 12
 42
65. sin A  0.4540
66. tan Q  5.9080
A  sin1 (0.4540)
Q  tan1 (5.9080)
 27
 80

16
1

3 16
3

4 16
3

10
3

 3 30 


4  4

7  15(7)  14 2  15(2)  14
7  135  14
?

2  110  14
?

7  149
2  14
77
2  2
Since 2 does not satisfy the original equation, 7 is
the only solution.
15.
14x  3  6  x
( 14x  3 ) 2  (6  x) 2
4x  3  36  12x  x2
0  x2  16x  39
0  (x  3) (x  13)
x  3  0 or x  13  0
x  3 or
x  13

13

 13

16
3

6. 2112x4y6  216  7  x4  y6
 4  17  x2  |y3|
 4x2|y3|17
7.

4  216

16
3

4  26(4)  8

x  15x  14
x2  ( 15x  14 ) 2
x2  5x  14
2  5x  14  0
x
(x  7)(x  2)  0
x  7  0 or x  2  0
x7
x  2
Check:

12
12
13

2  26(2)  8

No solution
14.

3 3  16  13
 16 

10  1  11
11  11

2  24

b; sine
a; cosine
c; tangent
2127  163  413  2(313 )  317  413
 613  317  413
 213  317
2

1100  1  11

2  2

Page 637

5. 16 

Check:

Chapter 11 Practice Test


1.
2.
3.
4.

14(25)  1  11

14x  1  5 Check:
14(6)  1  5
?
( 14x  1 ) 2  52
124  1  5
?
4x  1  25
125  5
4x  24
55
x6
x  16x  8
13.
x2  ( 16x  8 ) 2
x2  6x  8
2
x  6x  8  0
(x  2)(x  4)  0
or x  4  0
x20
x  2
x  4

45

45
28

Check:

12.

28
45

58. cos A  53

 0.8491

110(40)  20
?
1400  20
20  20

( 14s ) 2  102
4s  100
s  25

 0.6222

45
53

Check:

14s  10

a
7

56. tan A 

110x  20
( 110x ) 2  202
10x  400
x  40

11. 14s  1  11

6a  105
a  17.5

 0.5283

59. tan B 

20
e

e8

 EF

57. sin B 

10.

15e  120

44
3

55. cos B 

AC

 DF

Check:

14(3)  3  6  3
?

112  3  3
?

19  3
33

40
90

14(13)  3  6  (13)
?

152  3  19
155  19

Since 13 does not satisfy the original equation,


3 is the only solution.

39
2

3
8. 16(4  112 )  416  172
 416  612
9. (1  13 )(3  12 )  3  12  313  16

537

Chapter 11

PQ249-6481F-11[521-540] 26/9/02 8:16 PM Page 538 Sahuja Ahuja_QXP_06:Desktop Folder:Chandra:Algebra_FNL_Delivery:

c2  a2  b2
c2  82  102
c2  64  100
c2  164
2c2  1164
c  1164
 12.81
The hypotenuse is about 12.81 units.
c2  a2  b2
17.
122  (612 ) 2  b2
144  72  b2
72  b2
172  2b2
172  b
8.49  b
The length of the leg is about 8.49.
c2  a2  b2
18.
172  a2  132
289  a2  169
120  a2
1120  2a2
1120  a
10.95  a
The length of the leg is about 10.95.
16.

19. d  2(x2  x1

)2

 (y2  y1

24.

BC
KH
4.5
j

25.

 12

26.

)2

15a  240
a  16
AB
JK
20
15

AC

 JH
b

 16

15b  320
b

64
3

Chapter 11

3
j
1

 42
AC

 JH


b
1

24
1

 108

c2  a2  b2
292  a2  212
841  a2  441
400  a2
1400  2a2
20  a

12021 2

A  44
B  180  90  44  46
c2  a2  b2
27.
c2  212  152
c2  441  225
c2  666
2c2  1666
c  25.8
15

tan A  21
A  tan1

28.

11521 2

A  36
B  180  90  36  54
A  180  90  42  48
b

sin 42  10

6
j

6.7  b
a

cos 42  10
7.4  a
c  a2  b2
29.
c2  92  122
c2  81  144
c2  225
2c2  1225
c  15 miles

AC

 JH

13
k

12k  130
k

A  tan1

BC

BC

 KH

20

12j  60
j5
AB
JK
12
10

7.5
5

tan A  21

 KH


 2 [21  (9) ] 2  (7  2) 2
 2302  52
 1900  25
 1925
 5137
 30.41
AB
JK
12
10

AC

 JH

b64

21. d  2(x2  x1 ) 2  (y2  y1 ) 2

23.

7.5
5

j1

 2(1  (1)) 2  (5  1) 2


 222  (6) 2
 14  36
 140
 2110
 6.32

BC

AB
JK
1
42
1
12
1
42 j

AB
JK
1
42
1
12
1
12 b

20. d  2(x2  x1 ) 2  (y2  y1 ) 2

 KH

7.5j  22.5
j3

 2(4 
 (2  7) 2
2
 20  (9) 2
 181
9

AB
JK
20
15

AC

 JH

7.5h  32.5
h  4.3

4) 2

22.

AB
JK
6.5
h

65
6

538

PQ249-6481F-11[521-540] 26/9/02 8:16 PM Page 539 Sahuja Ahuja_QXP_06:Desktop Folder:Chandra:Algebra_FNL_Delivery:

30. B; A  /w
 16 (2132  316 )
 2 1192  3(6)
 2(813 )  18
 1613  18 units2

10. 2y  2x  4  0
m


A
B
3
2
1
2

 3
m  3 since slopes of parallel lines are equal.

Chapter 11 Standardized Test Practice


11.

Pages 638639
1. D; y  2x  1
2. C;
/w6
2/  2w  92
3. C; Let x  cost of highway resurfacing project
y  cost of bridge repair project
x  y  2,500,000
y  2x  200,000
x  2x  200,000  2,500,000
3x  2,700,000
x  $900,000

12. Let x  first integer


y  second integer
x  y  66
1

y  18  2x

4. C; 32,800,000  3.28  107


n7
2
5. A; x  7x  18  0
(x  2)(x  9)  0
x  2  0 or x  9  0
x2
x  9

Substitute 18  2x for y into the other equation.


1

x  18  2x  66
1

12x  48
x  32
Substitute 32 for x into either equation.
32  y  66
y  34
The two integers are 32 and 34.

g  t2  t
132  t2  t
0  t2  t  132
0  (t  12)(t  11)
t  12  0
or t  11  0
t  12
t  11
12 teams are in the league.

6. B;

13. h(t)  16t2  v t  h


0
0
When the ball hits the ground h(t)  0.
0  16t2  60t  100
0  4t2  15t  25
0  (4t  5) (t  5)
4t  5  0 or t  5  0
4t  5

7. B; c2  a2  b2
Let x  length of shorter leg
x  4  length of longer leg
202  x2  (x  4) 2
400  x2  x2  8x  16
0  2x2  8x  384
0  2(x2  4x  192)
0  2(x  16)(x  12)
x  16  0
or x  12  0
x  16
x  12
The shorter leg is 12 in.
8. A; c2  a2  b2
c2  122  52
c2  144  25
c2  169
2c2  1169
c  13
The distance is 13 yards.
9. D; 4, one in each quadrant.

t5

t  4

It will take the ball 5 seconds.


14. x2  8x  6  0
x

b  2b2  4ac
2a

8  2(8) 2  4(1) (6)


2(1)

8  164  24
2

8  140
2

x

8  140
2

or

x

 7.16
3

8  140
2

 0.84
3

15. 23181  23(9)


3
 127
3

539

Chapter 11

PQ249-6481F-11[521-540] 26/9/02 8:16 PM Page 540 Sahuja Ahuja_QXP_06:Desktop Folder:Chandra:Algebra_FNL_Delivery:

1 2 1 2
3 4
2 3

16. x

2x
x

1 2

20. C; c2  a2  b2
c2  102  112
c2  100  121
c2  221
2c2  1221
c  1221

(x2 )
x2
1

x2
x1

1
2

x
 x
1

x22
 x

21a.

17.

 x2
 x 1x
A  /w
64  x

x3
 1

6
7
3

Value of Column B
12y  16  8y
4y  16
y  4

start
c2


 b2
2
c  7  92
c2  49  81
c2  130
2c2  1130
c  11.4 mi
21c. The sketch shows that the distance she is from
her starting point is the length of the
hypotenuse of a right triangle with legs 7 mi
and 9 mi long.
21b.

m  B
2

3
Value of Column B
Substitute 0 for x to find the y-intercept.
7(0)  4y  4
4y  4
y1

Chapter 11

finish

x  1
x

64  x2
164  2x2
8  x
18. B; Value of Column A
13x  12  10x  3
3x  12  3
3x  15
x  5
19. B; Value of Column A
2x  3y  10

( 1390 ) 2  a2  132
390  a2  169
221  a2
1221  2a2
1221  a

540

a2

PQ249-6481F-12[541-568] 26/9/02 6:32 PM Page 541 Sahuja Ahuja_QXP_06:Desktop Folder:Chandra:Algebra_FNL_Delivery:

Chapter 12
Page 641
y
9

1.

Rational Expressions and Equations

2.

16(y)  (9)(7)
16y  63
63

3
15

n

1.1
0.6

4.

9.

10.

11.

12.

2.7
3.6

x
8

8.47
n

6.

8.1
a

9
8

0.21
2

6

0.19
2

6m
6

 24

6c2d2

3x
3



10(5  x)  14(x  3)
50  10x  14x  42
50  10x  50  14x  42  50
10x  14x  92
10x  14x  14x  92  14x
4x  92
4x
4

92
4

x  23
23.
6

7n  1
6
7n  1
6

5

2  5(6)

7n  1  30
7n  1  1  30  1
7n  31
7n
7

3c2d(1


 2d)
3c 
6nm  15m2  3m(2n  5m)
x2  11x  24  (x  3)(x  8)
x2  4x  45  (x  9)(x  5)
2x2  x  21  2x2  6x  7x  21
 (2x2  6x)  (7x  21)
 2x(x  3)  7(x  3)
 (x  3)(2x  7)
18. 3x2  12x  9  3x2  3x  9x  9
 (3x2  3x)  (9x  9)
 3x(x  1)  9(x  1)
 (x  1)(3x  9)
 3(x  1)(x  3)
3x  2  5
19.
3x  2  2  5  2
3x  3
13.
14.
15.
16.
17.

m  10
11

149
6
149
m 6
5  x
14
 10
x  3

22.

2.7(a)  3.6(8.1)
0.19(24)  2(x)
2.7a  29.16
4.56  2x
a  10.8
2.28  x
30  2  3  5
42  2  3  7
GCF  2  3 or 6
60r2  2  2  3  5  r  r
45r3  3  3  5  r  r  r
GCF  3  5  r  r or 15r2
32m2n3  2  2  2  2  2  m  m  n n  n
12m2n  2  2  3  m  m  n
GCF  2  2  m  m  n or 4m2n
14a2b2  2  7  a  a  b  b
18a3b  2  3  3  a  a  a  b
GCF  2  a  a  b or 2a2b
2d

11(m  9)  5(m  10)


11m  99  5m  50
11m  99  99  5m  50  99
11m  5m  149
11m  5m  5m  149  5m
6m  149

9(6)  8(y)
54  8y
6.75  y
8.

m  9
5

21.

2(x)  8(0.21)
2x  1.68
x  0.84

1.1(n)  0.6(8.47)
1.1(n)  5.082
n  4.62
7.

2
10

x  20

3(n)  15(1)
3n  15
n5
5.

y
x

2(x)  4(10)
2x  40

y  16
3.

20. 5x  8  3x  (2x  3)
2x  8  2x  3
2x  8  2x  2x  3  2x
8  3
No Solution

Getting Started
7
16

n

24.
9

4t  5
9
4t  5
9

7

2  7(9)

4t  5  63
4t  5  5  63  5
4t  58

31
7
31
7

4t
4

58

 4

t  14.5

x  x  56  0
(x  8)(x  7)  0
x  8  0 or x  7  0
x8
x  7
x2  2x  8
26.
x2  2x  8  0
(x  4)(x  2)  0
or x  2  0
x40
x  4
x2
25.

3
3

x  1

541

Chapter 12

PQ249-6481F-12[541-568] 26/9/02 6:32 PM Page 542 Sahuja Ahuja_QXP_06:Desktop Folder:Chandra:Algebra_FNL_Delivery:

7.

12-1 Inverse Variation


Page 645

24
6

Check for Understanding

1. Sample answer: xy  8
2. Sample answer: Direct variation equations are in
the form y  kx and inverse variation equations
are in the form xy  k. The graph of a direct
variation is linear while the graph of an inverse
variation is nonlinear.
3. b; Sample answer: As the price increases, the
number purchased decreases.
4. Solve for k.
xy  k
(24) (8)  k
192  k
xy  192
Choose values for x and y whose product is 192.
Sample answer:
x
48
24
12
8
8
12
24
48

y
4
8
16
24
24
16
8
4

100

21.87
5.4

x
12
6
4
1
4
6
12

y
1
2
3
12
3
2
1

100 x

5120
8

Pages 645647
11.

8x

4
8

8y2
8

9  y2
Thus, y  9 when x  8.

Chapter 12

8y2
8

542

Practice and Apply

xy  k
(8)(24)  k
192  k
Choose values for x and y whose product is 192.
Sample answer:
x
24
12
8
8
12
24

x1y1  x2 y2
(6) (12)  (8)(y2 )
72  8y2
72
8

640  y2
Thus, the frequency of an 8-inch string is
640 cycles per second.

4
O

 x2

10. (length of string)  (frequency of vibrations)


x1y1  x2 y2
(10) (512)  (8)(y2 )
5120  8y2

xy  12

4

32x2
32

100

8

Thus, x  4 when y  32.

50

5.4x2
5.4

8  32x2

8
32
1
4

50
50

112 2 (16)  (x2)(32)

6y2
6

4.05  x2
Thus x  4.05 when y  5.4.
9.
x y x y
1 1
2 2

xy  192

100 50

4  y2
Thus, y  4 when x  6.
8.
x1y1  x2 y2
(8.1)(2.7)  (x2 )(5.4)
21.87  5.4x2

5. Solve for k.
xy  k
(2)(6)  k
12  k
xy  12
Choose values for x and y whose product is 12.
Sample answer:

6.

x1y1  x2 y2
(3)(8)  (6)(y2 )
24  6y2

y
8
16
24
24
16
8

100

y
xy   192

50
100 50 O
50
100

50

100 x

PQ249-6481F-12[541-568] 26/9/02 6:32 PM Page 543 Sahuja Ahuja_QXP_06:Desktop Folder:Chandra:Algebra_FNL_Delivery:

12.

xy  k
(4)(3)  k
12  k
Choose values for x and y whose product is 12.
Sample answer:
x
6
4
3
3
4
6

y
2
3
4
4
3
2

15.

x
8
4
2
2
4
8

xy  12
4
8

4

xy  k
(8)(9)  k
72  k
xy  72
Choose values for x and y whose product is 72.
Sample answer:

8x

4
8

y
9
18
36
36
18
9

y
xy  72

16
8

16 8 O

16 x

8
16

13.

xy  k
(15) (5)  k
75  k
xy  75
Choose values for x and y whose product is 75.
Sample answer:
x
25
15
5
5
15
25

y
3
5
15
15
5
3

xy  75

16.

y
40

x
8
4
2
2
4
8

20
20 10 O

10

20 x

20
40

14.

y
2
4
12
12
4
2

xy  48

17.

y
20
10
20 10 O
10

10

20x

xy  19.44

3y2
3

20  y2
Thus, xy  60 and y  20 when x  3.
18.
x1y1  x2 y2
(2)(7)  (7) (y2 )
14  7y2

10
10

x y x y
1 1
2 2
(5)(12)  (3) (y2 )
60  3y2
60
3

y
20

20 10 O

y
2.43
4.86
9.72
9.72
4.86
2.43

20

xy  k
(12) (4)  k
48  k
xy  48
Choose values for x and y whose product is 48.
Sample answer:
x
24
12
4
4
12
24

xy  k
(8.1)(2.4)  k
19.44  k
xy  19.44
Choose values for x and y whose product is 19.44.
Sample answer:

20 x

14
7

10

7y2
7

2  y2
Thus, xy  14 and y  2 when x  7.
19.
x1y1  x2 y2
(1)(8.5)  (x2 ) (1)
8.5  x2

20

8.5
1

x2
1

8.5  x2
Thus, xy  8.5 and x  8.5 when y  1.
20.
x y x y
1 1
2 2
(1.55)(8)  (x2 ) (0.62)
12.4  0.62x2
12.4
0.62

0.62x2
0.62

20  x
2
Thus, xy  12.4 and x  20 when y  0.62.

543

Chapter 12

PQ249-6481F-12[541-568] 26/9/02 6:32 PM Page 544 Sahuja Ahuja_QXP_06:Desktop Folder:Chandra:Algebra_FNL_Delivery:

21.

x1y1  x2 y2
(4.4)(6.4)  (x2 )(3.2)
28.16  3.2x2
28.16
3.2

28.

3.2x2
3.2

1.25
20

8.8  x2
Thus, xy  28.16 and x  8.8 when y  3.2.
22.
x1y1  x2 y2
(0.5)(1.6)  (x2 )(3.2)
0.8  3.2x2
0.8
3.2

3.2x2
3.2

16
7

 y2
12
5

14
3
14
3

when x  5.

108
15

 7y2

1 2  3(7y2)


21y2
21

 y2

Thus, xy 
26.

12
1
2

14
3

220
65
220
65

 32y2


t2 

32y2
32

t 
2

 y2

x1y1  x2 y2
(6.1)(4.4)  (x2 )(3.2)
26.84  3.2x2
26.84
3.2

3.2x2
3.2

8.3875  x2
Thus, xy  26.84 and x  8.3875 when y  3.2.

Chapter 12

15h2
15

65t2
65

 t2

220
h
65
(220) (60)
65

min

t  203 min
2
t2  3 h and 23 min
It takes about 3 hours and 23 minutes if they
drive 65 miles per hour.
33. 4 h 0 min S 3 h 60 min
 3 h 23 min
 3 h 23 min
37 min
If they drive at 65 miles per hour, they can save
about 37 min.

Thus, xy  8 and y  4 when x  32.


27.

or

8  32y2
8
32
1
4

1.6p2
1.6

(55) (4)  (65)(t2 )


220  65t2

and y  3 when x  7.

x y x y
1 1
2 2
(16)  (32) (y2 )
16
2

7.2  h2
It takes 7.2 hours, if 15 students hand out
1000 flyers.
32. Let r1  55, t1  4, and r2  65. Solve for t2.
r t r t
1 1
2 2

14  21y2
14
21
2
3

90w2
90

660  p2
The pitch of a tone with a wavelength of 1.6 feet
is 660 vibrations per second.
31. Let s1  12, h1  9, and s2  15. Solve for h2.
s h s h
1 1
2 2
(12) (9)  (15)(h2 )
108  15h2

5y2
5

x y x y
1 1
2 2
2
(7)

(7)
(y2 )
3

12

1056
1.6

Thus, xy  12 and y 
25.

when x  7.

x y x y
1 1
2 2
(2)(6)  (5)(y2 )
12  5y2
12
5
12
5

20x2
20

8  w2
The width of the second rectangle is 8 inches.
30. (pitch)  (wavelength)
Let p1  440, w1  2.4, and w2  1.6. Solve
for p2.
p1w1  p2w2
(440)(2.4)  (p2 )(1.6)
105.6  1.6p2

 y2

Thus, xy  16 and y 
24.

720
90

7y2
7

0.0625  x2
Thus, xy  1.25 and x  0.0625 when y  20.
29. A    w
Let 1  20, w1  36, and 2  90. Solve for w2.
1w1  2w2
(20) (36)  (90)(w2 )
720  90w2

0.25  x2
Thus, xy  0.8 and x  0.25 when y  3.2.
23. x1y1  x2 y2
(4)(4)  (7) (y2 )
16  7y2
16
7
16
7

x1y1  x2 y2
(0.5)(2.5)  (x2 )(20)
1.25  20x2

544

PQ249-6481F-12[541-568] 26/9/02 6:32 PM Page 545 Sahuja Ahuja_QXP_06:Desktop Folder:Chandra:Algebra_FNL_Delivery:

42. A; Graphs B and D can be ignored since they are


linear, showing direct variation. The point at
(2, 4) is one solution to the equation, so it
must be on the graph. Graph A contains this point
and Graph C does not. Therefore, the correct
choice is A.

34. If V is the volume of a gas at pressure P, then


PV  k.
or
If V1 is the volume at pressure P1, and V2 is the
volume at pressure P2, then P1V1  P2V2.
35.
P1V1  P2V2
(1) (60)  (3)(V2 )
60  3V2
60
3

Page 647

3V2
3

tan A 

20  V2
The new volume of the gas is 20 cubic meters.
36.
P1V1  P2V2
(1) (22)  (0.8)(V2 )
22  0.8V2
22
0.8

0.8V2
0.8

sin A 

The volume of the balloon at 0.8 atmosphere is


27.5 cubic meters.
37. Let w1  36, d1  8, and d2  20  d1. Solve
for w2.
w d w d
1 1
2 2
(36) (8)  (w2 )(20  8)
288  12w2


y1
2

Use a calculator.
2nd [SIN1] 10 12 ENTER
56.44269024
The measure of the missing angle is 56.
45. Let A represent the missing angle.

12w2
12

cos A 


46.

1
y
2 1

 y2

Thus, if the value of x1 is doubled, the value of y


1
will be 2 of what it was.
39. Let y2  3y1
x1y1  x2 y2
x1y1  x23y1
x1y1
3y
1

x1
3

adjacent leg
hypotenuse
3
10

Use a calculator.
1
KEYSTROKES: 2nd [COS ] 3 10 ENTER
72.54239688
The measure of the missing angle is 73.

2x1y2
2x

 y2 or

opposite leg
hypotenuse
10
12

KEYSTROKES:

24  w2
The second piece of art must weigh 24 kg.
38. Let x2  2x1
x1y1  x2 y2
x1y1  2x1y2
x1y1
2x

opposite side
adjacent leg
7
8

Use a calculator.
1
KEYSTROKES: 2nd [TAN ] 7 8 ENTER
41.18592517
The measure of the missing angle is 41.
44. Let A represent the missing angle.

27.5  V2

288
12

Maintain Your Skills

43. Let A represent the missing angle.

a
d
3
12

f

b
e
10
e

3
12

10
e

9
f

3(e)  (12) (10)


3e  120
3e
3

x23y1
3y
1

120
3

47.

a
d
a
21




a
21

Thus, if the value of y is tripled, the value of x


will be one third of what it was.
40. Sample answer: When the gear ratio is lower, the
pedaling revolutions increase to keep a constant
speed. Answers should include the following.
Shifting gears will require that the rider
increase pedaling revolutions.
Lower gears at a constant rate will cause a
decrease in speed, while higher gears at a
constant rate will cause an increase in speed.
41. B;
xy  k
(1.3)(4.25)  k
5.525  k

3
12

9
f

3(f )  (12) (9)


3f  108
3f
3

48.

545

108
3

f


4
f

 28

28(a)  (8) (21)


28a  168
28a
28

e  40

 x2 or 3 x1  x2

b
e
8
28

168
28

a6
8
28

4
f

8(f )  (4) (28)


8f  112
8f
8

112
8

f  36
f  14
A is 220  2  2  5  11
C is 264  2  2  2  3  11
C sharp is 275  5  5  11
GCF of A and C  2  2  11  44
GCF of A and C sharp  5  11  55
GCF of C and C sharp  11
Therefore, A and C sharp have the closest
harmony.
Chapter 12

PQ249-6481F-12[541-568] 26/9/02 6:32 PM Page 546 Sahuja Ahuja_QXP_06:Desktop Folder:Chandra:Algebra_FNL_Delivery:

49.

7(2y  7)  5(4y  1)
7(2y)  7(7)  5(4y)  5(1)
14y  49  20y  5
14y  49  49  20y  5  49
14y  20y  5  49
14y  20y  20y  20y  54
6y  54
6y
6

50.

36  2  2  3  3
15  3  5
45  3  3  5
GCF  3
56. 48  2  2  2  2  3
60  2  2  3  5
84  2  2  3  7
GCF  2  2  3  12
58. 17a  17  a
57. 210  10  3  7

55.

54

 6

y  9
w(w  2)  2w(w  3)  16
w2  2w  2w2  6w  16
2  2w  w2  2w2  6w  16  w2
w
2w  w2  6w  16
2w  2w  w2  6w  16  2w
0  w2  8w  16
0  (w  4)(w  4)
w40
w4404
w4

51.

330  10  3  11

34a2  2  17  a  a
GCF  17a

150  10  3  5
GCF  10  3  30
59. 12xy2  2  2  3  x  y  y
18x2y3  2  3  3  x  x  y  y  y
GCF  2  3  x  y  y  6xy2
60. 12pr2  2  2  3  p  r  r
40p4  2  2  2  5  p  p  p  p
GCF  2  2  p  4p

12-2 Rational Expressions


y  x  1

Page 651

Check for Understanding

1. Sample answer: Factor the denominator, set each


factor equal to 0, and solve for x.

y  3x  5

2. Sample answer:
52.

y  2x  3
x

2y  5x  14

53.

2x
2

y0
x

xy1

54.

7.
y
3x  2y  16

6
2

56x y
70x3y2

(14x y) (4)
(14x2y) (5xy)

(14x y) (4)
(14x2y) (5xy)
4
5xy

Exclude the values for which 70x3y2  0.


70x3  0 or y2  0
x3  0
y0
x0
The excluded values are x  0 and y  0.

x
x  4y  4

Chapter 12

5x  8y  8
O

1
x2  11x  28

x  3
The excluded value is 3.
6. Exclude the values for which n2  n  20  0.
(n  5)(n  4)  0
n50
or n  4  0
n  5
n4
The excluded values are 5 and 4.

xy3

or

3. Sample answer: You need to determine excluded


values before simplifying. One or more factors
may have been canceled in the denominator.
4. Exclude the values for which 3  a  0.
3a0
a  3
The excluded value is 3.
5. Exclude the values for which 2x  6  0.
2x  6  0
2x  6

1
(x  4) (x  7)

546

PQ249-6481F-12[541-568] 26/9/02 6:33 PM Page 547 Sahuja Ahuja_QXP_06:Desktop Folder:Chandra:Algebra_FNL_Delivery:

8.

x2  49
x  7

(x  7) (x  7)
x  7

(x  7 ) (x  7)
(x  7 )

13.

b2  3b  4
b2  13b  36

(b  1) (b  4)
(b  9) (b  4)

(b  1) (b  4 )
(b  9) (b  4 )

b  1
b  9

x7
Exclude the values for which x  7  0.
x70
x  7
The excluded value is 7.
9.

x  4
x2  8x  16

x  4
(x  4) (x  4)

x  4
(x  4 ) (x  4)

Exclude the values for which b2  13b  36  0.


b  9  0 or b  4  0
b9
b4
The excluded values are 9 and 4.
14. Let g  the number of guppies.

4
4  g

represents the fraction of neon fish in the


aquarium.
15. Let g  the number of guppies.
Double the guppy population, 2g, plus the four
neon fish, plus five new fish can be represented
4
by 9  2g.

x

1
 4

Exclude the values for which x2  8x  16  0.


or x  4  0
x40
x  4
x  4
The excluded value is 4.
10.

x2  2x  3
x2  7x  12

(x  1) (x  3)
(x  4) (x  3)

(x  1) (x  3 )
(x  4) (x  3 )

Pages 651653

x  1
 4

x

Exclude the values for which x2  7x  12  0.


(x  4)(x  3)  0
x  4  0 or x  3  0
x4
x3
The excluded values are 4 and 3.
11.

a2  4a  12
a2  2a  8

(a  6) (a  2)
(a  2) (a  4)

(a  6) (a  2 )
(a  2 ) (a  4)

a  6
a  4

Exclude the values for which a2  2a  8  0.


a2  2a  8  0
(a  2)(a  4)  0
a  2  0 or a  4  0
a2
a  4
The excluded values are 2 and 4.
12.

2x2  x  21
2x2  15x  28

(2x  7) (x  3)
(2x  7) (x  4)

(2x  7 ) (x  3)
(2x  7 ) (x  4)

x  3
 4

x

Exclude the values for which 2x2  15x  28  0.


(2x  7)(x  4)  0
2x  7  0 or x  4  0
2x  7
x4
7

x2
The excluded values are

7
2

Practice and Apply

16. Exclude the values for which m  2  0.


m20
m2
The excluded value is 2.
17. Exclude the values for which b  5  0.
b50
b  5
The excluded value is 5.
18. Exclude the values for which n2  36  0.
(n  6)(n  6)  0
n  6  0 or n  6  0
n6
n  6
The excluded values are 6 and 6.
19. Exclude the values for which x2  25  0.
(x  5)(x  5)  0
x  5  0 or x  5  0
x5
x  5
The excluded values are 5 and 5.
20. Exclude the values for which a2  2a  3  0.
(a  3)(a  1)  0
or a  1  0
a30
a  3
a1
The excluded values are 3 and 1.
21. Exclude the values for which x2  2x  15  0.
(x  5)(x  3)  0
or x  3  0
x50
x  5
x3
The excluded values are 5 and 3.
22. Exclude the values for which n2  n  30  0.
(n  6)(n  5)  0
or n  5  0
n60
n  6
n5
The excluded values are 6 and 5.
23. Exclude the values for which x2  12x  35  0.
(x  5)(x  7)  0
or x  7  0
x50
x  5
x  7
The excluded values are 5 and 7.

and 4.

547

Chapter 12

PQ249-6481F-12[541-568] 26/9/02 6:33 PM Page 548 Sahuja Ahuja_QXP_06:Desktop Folder:Chandra:Algebra_FNL_Delivery:

24.

35yz2
14y2z

(7yz) (5z)
(7yz) (2y)

(7yz ) (5z)
(7yz ) (2y)

Exclude the values for which 36mn3  12m2n2  0.


36mn3  12m2n2  12mn2 (3n  m)
12m  0 or n2  0 or 3n  m  0
m0
n0
3n  m
The excluded values are m  0, n  0, and 3n  m.

5z

 2y
Exclude the values for which 14y2z  0.
14y2  0 or z  0
y2  0
y0
The excluded values are y  0 and z  0.
25.

14a3b2
42ab3

(14ab2 ) (a2 )
(14ab2 ) (3b)

(14ab ) (a2 )
(14ab2 ) (3b)

a2
3b

30.

31.

(16qrs) (4r)
(16qrs) (q)

(16qrs ) (4r)
(16qrs ) (q)

4r
q

32.

(3xyz) (3x)
(3xyz) (8z)

(3xyz ) (3x)
(3xyz ) (8z)

3x
8z

(z  2 ) (z  8)
z  2

4x  8
x2  6x  8

4(x  2)
(x  4) (x  2)

4(x  2 )
(x  4) (x  2 )
1

x

33.

(7ab) (a2b)
(7ab) (3a  7b2 )

(7ab ) (a2b)
(7ab ) (3a  7b2 )

2(y  2)
(y  5) (y  2)
1
2(y  2 )
(y  5) (y  2 )
1

2
 5

Exclude the values for which y2  3y  0.


(y  5)(y  2)
or y  2  0
y50
y  5
y2
The excluded values are 5 and 2.

a2b

 3a  7b2

34.

Exclude values for which 21a2b  49ab3  0.


7a  0 or b  0 or 3a  7b2  0
a0
3a  7b2

m2  36
m2  5m  6




(m  6) (m  6)
(m  1) (m  6)
1
(m  6) (m  6 )
(m  1) (m  6 )
m  6

m1

a  3b2

Exclude the values for which m2  5m  6  0.


(m  1)(m  6)
m10
or m  6  0
m  1
m6
The excluded values are 1 and 6.

The excluded values are a  0, b  0, and a  3b2.


(3mn2 )mn
 4m(mn2 )

 12n(3mn2 )

(3mn2 )mn
 4m(3mn2 )

 12n

(3mn2 )mn
 4m(3mn2 )

 12n

y

3m2n3
36mn3  12m2n2

2y  4
y2  3y  10

Exclude the values for which 24xyz2  0.


24x  0 or y  0 or z2  0
x0
z0
The excluded values are x  0, y  0, and z  0.
7a3b2
21a2b  49ab3

4
 4

Exclude the values for which x2  6x  8  0.


(x  4)(x  2)  0
x40
or x  2  0
x  4
x  2
The excluded values are 4 and 2.

mn
 4m

 12n
Chapter 12

(z  2) (z  8)
z  2

29.

Exclude the values for which 16q2rs  0.


16q2  0 or r  0 or s  0
q2  0
q0
The excluded values are q  0, r  0, and s  0.

28.

z2  10z  16
z  2

z8
Exclude the values for which z  2  0.
z20
z  2
The excluded value is 2.

9x2yz
24xyz2

(x  5 ) (x  4)
x  5

27.

(x  5) (x  4)
x  5

x4
Exclude the values for which x  5  0.
x50
x  5
The excluded value is 5.

Exclude the values for which 42ab3  0.


42a  0 or b3  0
a0
b0
The excluded values are a  0 and b  0.
64qr s
16q2rs

26.

x2  x  20
x  5

548

PQ249-6481F-12[541-568] 26/9/02 6:33 PM Page 549 Sahuja Ahuja_QXP_06:Desktop Folder:Chandra:Algebra_FNL_Delivery:

35.

a2  9
a2  6a  27

(a  3) (a  3)
(a  9) (a  3)

1
(a  3) (a  3 )
(a  9) (a  3 )

2(2x2  3x  2)
2(x2  4x  4)
2(2x  1) (x  2)
2(x  2) (x  2)

a  3
a  9

1
1
2 (2x  1) (x  2 )
2 (x  2) (x  2 )

2x  1
x  2

40.

4x2  6x  4
2x2  8x  8

Exclude the values for which a2  6a  27  0.


(a  9)(a  3)
a90
or a  3  0
a  9
a3
The excluded values are 9 and 3.
36.

x2  x  2
x2  3x  2

(x  2) (x  1)
(x  2) (x  1)

1
(x  2) (x  1 )
(x  2) (x  1 )

x  2
x  2

Exclude the values for which 2x2  8x  8  0.


2(x  2)(x  2)
x20
x2
The excluded value is 2.
41.

3m2  9m  6
4m2  12m  8




4
Exclude the values for which 4m2  12m  8  0.
4(m  2)(m  1)
m20
or m  1  0
m  2
m  1
The excluded values are 2 and 1.
42. a  4500  1000  4.5
43. 3500  1000  3.5

(b  4) (b  2)
(b  4) (b  16)
(b  4) (b  2)
(b  4) (b  16)

Exclude the values for which b2  20b  64.


(b  4)(b  16)
b  4  0 or b  16  0
b4
b  16
The excluded values are 4 and 16.
38.

x2  x  20
x3  10x2  24x

t



1
(x  4 ) (x  5)
 4 ) (x  6)

 x(x

t

x  5
x(x  6)

n2  8n  12
n3  12n2  36n

40(37.95)

(n  2) (n  6)
 36)
(n  2) (n  6)
n(n  6) (n  6)

1
(n  2) (n  6 )
n(n  6) (n  6 )

 40.97
about 41 min to cook a potato.
45. The times are not doubled even though the
altitude is. The difference between the times is
12 minutes.

 n(n2  12n

n  2
 6)

40 [ 25  1.85(7) ]
50  1.85(7)

 50  12.95

Exclude the values for which x3  10x2  24x  0.


x(x  4)(x  6)
x  0 or x  4  0 or x  6  0
x  4
x  6
The excluded values are 0, 4, and 6.
39.

40 [ 25  1.85(3.5) ]
50  1.85(3.5)
40(25  6.475)
50  6.475
40(31.475)
43.525

 28.93
about 29 min
44. 7000  1000  7

(x  4) (x  5)
 4) (x  6)

 x(x

3(m  3m  2 )

 4(m2  3m  2 )

Exclude the values for which x2  3x  2  0.


(x  2)(x  1)
x  2  0 or x  1  0
x2
x1
The excluded values are 2 and 1.
b2  2b  8
b2  20b  64

3(m2  3m  2)

 4(m2  3m  2)
2

37.

46. MA 

s  r
r

47. MA 

17.5  0.4
0.4

MA  42.75
48. Force on lid  MA  force applied
 42.75  6
 256.5 lb
450  4n
49. 450  4n
50.
n

 n(n

Exclude the values for which n3  12n2  36n  0.


n(n  6) (n  6)
n  0 or n  6  0
n6
The excluded values are 0 and 6.

51. 15 

450  4n
n

15n  450  4n
11n  450
n  40.9
41 students must attend.
52.

54.

450  4(n  2)
n


4

53.

Area Circle
Area Square

x2
(2x) 2

x2
4x2


4

 0.785
 79%

549

Chapter 12

PQ249-6481F-12[541-568] 26/9/02 6:33 PM Page 550 Sahuja Ahuja_QXP_06:Desktop Folder:Chandra:Algebra_FNL_Delivery:

61.

55a. Sample answer: The graphs appear to be


identical because the second equation is the
simplified form of the first equation.
55b. Sample answer: The first graph has a hole at
x  4 because it is an excluded value of the
equation.
56. Sample answer: Use the rational expression for
light intensity to help determine the brightness of
the picture on the screen for the distance between
the projector and the screen. Answers should
include the following.
Find the solutions for the expression in the
denominator.
Use the light intensity expression to
determine the brightness of a search light.
57. C; The other expressions can be factored.
58. B;

7.5
8

4.374606646
The measure of V is 4.
65. sin A  0.7011
1
KEYSTROKES: 2nd [SIN ] .7011 ENTER
44.51532391
The measure of A is 45.

Maintain Your Skills

xy  k
(10) (6)  k
60  k
xy  60
x1y1  x2 y2
(10) (6)  (12) (y2 )
60  12y2
60
12

66.

12y2
12

112 2 (16)  k
k

8k
xy  8
x1y1  x2 y2
16
2

 (x2 )(32)
 32x2

Check:

8  32x2
8
32
1
4

12z  2  z  3
?

12(1)  2  1  3

32x2
32

14  2
2  2

 x2

The only solution is 7.

Thus, x  4 when y  32.

Chapter 12

1a  3  2
?
11  3  2
?
14  2
22
67.
12z  2  z  3
( 12z  2) 2  (z  3) 2
2z  2  z2  6z  9
2z  2  2z  2  z2  6z  9  2z  2
0  z2  8z  7
0  (z  1) (z  7)
z  1  0 or z  7  0
z1
z7

Thus, y  5 when x  12.


60.
xy  k

1
(16)
2

1a  3  2
( 1a  3) 2  22
a34
a1
Check:

5  y2

16
2

8y2
8

13.57393947
The measure of N is 14.
63. cos B  0.3218
1
KEYSTROKES: 2nd [COS ] .3218 ENTER
71.22818376
The measure of B is 71.
64. tan V  0.0765
1
KEYSTROKES: 2nd [TAN ] .0765 ENTER

x2  6x  5  0
(x  5)(x  1)  0
x  5 or x  1
5 and 1 are excluded values.

59.

0.9375  y2
Thus, y  0.9375 when x  8.
62. sin N  0.2347
1
KEYSTROKES: 2nd [SIN ] .2347 ENTER

x2  3x  2
x2  6x  5

Page 653

xy  k
(3)(2.5)  k
7.5  k
xy  7.5
x y x y
1 1
2 2
(3)(2.5)  (8) (y2 )
7.5  8y2

550

12z  2  z  3
?

12(7)  2  7  3
?

116  4
44

PQ249-6481F-12[541-568] 26/9/02 6:33 PM Page 551 Sahuja Ahuja_QXP_06:Desktop Folder:Chandra:Algebra_FNL_Delivery:

68.

74. A  /w
A  (2x  y) (x  y)
 2x2  2xy  xy  y2
 2x2  3xy  y2
The area is 2x2  3xy  y2 square units.

113  4p  p  8
113  4p  p  p  8  p
113  4p  8  p
13  4p  (8  p) 2
13  4p  64  16p  p2
13  4p  13  64  16p  p2  13
4p  51  16p  p2
4p  4p  51  16p  p2  4p
0  p2  20p  51
0  ( p  3)( p  17)
or p  17  0
p30
p  3
p  17
113  4p  p  8

Check:

113  4(3)  (3)  8

84 in.
1

76.

4.5 m
1

77.

113  4p  p  8

78.

181  17  8
?

538

9  17  8

88

12 in.
1 ft

84 in.
1

1 ft

12 in.  7 ft

100 cm
 m

4h
1

 450 cm

60 min
h
15 min
1

60 s

 min  14,400 s
60 s

 min  900 s

14,400 s  900 s  15,300 s

125  3  8

113  4(17)  (17)  8

79.

26  8

80.

The only solution is 3.


69.

75.

18 mi 5280 ft
 mi  95,040 ft
1
3 days 24 h
 day  72 h
1
220 mL
1000 mL
220 mL

 1
1
L

1000 mL 

220 L
1000

 0.22 L

23r2

 61  2r  1
 61) 2  (2r  1) 2
3r2  61  4r2  4r  1
0  4r2  4r  1  3r2  61
0  r2  4r  60
0  (r  6)(r  10)
r  6  0 or r  10  0
r6
r  10
( 23r2

23r 2  61  2r  1

Check:

23(6) 2

 61  2(6)  1

Page 654

1.

1108  61  12  1

3x  6
x2  7x  10

3(x  2 )
(x  2 ) (x  5)

3
x  5

When x  5, x  5  0. When x  2,


x  2  0. Therefore, x cannot equal 2 or 5
because you cannot divide by zero.

23r 2  61  2r  1
23(10) 2

Graphing Calculator Investigation


(Follow-Up of Lesson 12-2)

 61  2(10)  1
?

1300  61  20  1

1169  13

1361  21

13  13

19  21

The only solution is 13.


70. Common factor:

3
1

3

27 3  81
81 3  243
243 3  729
Thus, the next three terms are 81, 243, and 729.
71. Common factor:

24
6

[10, 10] scl: 1 by [10, 10] scl: 1

2.

4

(x  8 ) (x  1)
(x  8 ) (x  8)
1

x  1
 8

x

384 4  1536
1536 4  6144
6144 4  24,576
Thus, the next three terms are 1536, 6144, and
24,576.
1

x2  9x  8
x2  16x  64

When x  8, x  8  0. Therefore, x cannot equal


8 because you cannot divide by 0.

72. Common factor: 2  4  2


2 (2)  4
4 (2)  8
8 (2)  16
Thus, the next three terms are 4,8, and 16.
73. Common factor:
27
16
81
64
243
256

81

243

729

[5, 15] scl: 1 by [10, 10] scl: 1

3
4

4  64
4  256
4  1054

Thus, the next three terms are

81 243
,
,
64 256

729

and 1054.

551

Chapter 12

PQ249-6481F-12[541-568] 26/9/02 6:33 PM Page 552 Sahuja Ahuja_QXP_06:Desktop Folder:Chandra:Algebra_FNL_Delivery:

3.

5x2  10x  5
3x2  6x  3

5(x2  2x  1 )
 1)

 3(x2  2x

10.

1
24 feet
1 second

60 seconds
1 minute

60 minutes
1 hour

1 mile

 5280 feet
220

1  60  60  1 mile

3

 1  1  1 hour  220

When x  1, x  2x  1  0. Therefore, x


cannot equal 1 because you cannot divide by 0.

3600 miles
220 hour

 16.36 mph
11.

4,000
240,000 miles
1

1 hour

1 day

 60 minutes  24 hours
1

40
4,000  1  1  1 day
1  100  1  24
40 days
24
2
13 days





[10, 10] scl: 1 by [10, 10] scl: 1

1 minutes
100 miles

4.

2x  9
4x2  18x

2x  9
 9)

 2x(2x

Pages 657659

 2x

12.

a. Sample answer: Examine the values and verify


that they are identical.
b. It displays ERROR.

8
x2

2x

8x

 4x  4x3
1

 2x
13.

10r3
6n3

2 1
,
1 x

15.

2. Sample answer: When the negative sign in front


of the first expression is distributed, the
numerator is x  6.
3. Amiri; Sample answer: Amiri correctly divided by
the GCF.
4.

64y
5y

5y

 8y 

8 y
2

64y
5y

16.

17.
15 s2 t 3
12 s t
3 11

7.

m  4
3m

x2  4
2

 (m

x

1
3

2
4t

n2  16
n  4

1
m  4
3m

4m
3(m  5)

1
(x  2) (x  2 )
2

n  2
 16

m
4m2

19.

 (m  4) (m  5)

x

2
4
 2

20.

12 g

24 g2 h
5 1 b

12ag
5
(x  4) (x  3)
(x  8)

(n  1) (n  1)
(n  1)

 (n

(z  4) (z  6)
(z  6) (z  1)

1
(x  8 )
(x  8) (x  3)

x  4
x  8

(z  1) (z  5)
 4)

 (n

x  5
 5

x

n  2
 4) (n  4)
1

21.

1
x  5
(x  5) (x  2)
1

x2  25
9

y2  4
y2  1

y  1
 2

y

1
(z  4 ) (z  6)
(z  6) (z  1)

(z  6) (z  5)
(z  6) (z  3)

(x  1) (x  7)
(x  7) (x  4)

(x  1) (x  7)
(x  7) (x  1)

x  5
 5

x

(x  5) (x  5)
9
(x  5) 2
9

1
(y  2 ) (y  2)
(y  1) (y  1)

1
y  1
 2

y

y

(x  3) (x  2)
5

x  3
5

22.

1
x2  x  12

x  3
 5

x




552

(n  4)
 1) (n  4)
1

1
(x  4) (x  3)
1
1
(x  4) (x  5)

1
x  3
 5

x

1
(z  1 ) (z  5)
 4)

 (z  3) (z

1
(x  5) (x  5 )

 (n

n  4
n  4

y  2
 1

1
(x  4) (x  3 )
(x  8)
1

1
1
(n  1 ) (n  1 )
(n  1)

(x  4) (x  10)
 1) (x  10)

 (x

x  x  6
5

(x  1) (x  7)
(x  7) (x  4)

(n  4)
 1) (n  4)

 (z  3) (z

1
(n  4 ) (n  4)
n  4

Chapter 12

1 z2

 15 a b2

n  2
 4

(x  8)
(x  8) (x  3)

 n2  8n

x  5
x2  7x  10

2 s 1

n
9.

1 x

2t
s

3 a2 b
2gh

4 1 t

 2(x  2)
8.

1 a

24g2h

 15ab2 

16 s t2
 10 s3 t 3

18.

4m2
 4) (m  5)

4
 2

4wy
5xz2

5y

1 1

120 w2 x2 y3 z2
150 w x3 y2 z 4

 8y
3 1 1

16st2
 10s3t3

3a2b
2gh

1 1 1

 3  2s

6.

12w2x2
25y2z4

 8y
5.

15s2t2
12st

10y3z2
6wx3

2
n

Check for Understanding

1. Sample answer:

420 n2 r3


14.

42n2

 35r3  210 n3 r3

12-3 Multiplying Rational Expressions


Page 657

Practice and Apply

1
1
(x  4 ) (x  10 )
 1) (x  10)

 (x

PQ249-6481F-12[541-568] 26/9/02 6:33 PM Page 553 Sahuja Ahuja_QXP_06:Desktop Folder:Chandra:Algebra_FNL_Delivery:

23.

x  6
x2  4x  32

x  4
 2

x


24.

x  3
x  4

 x2  7x

 12

1
x  3
 4


n2

n
 8n  15

2n  10
n2

1
x  4
 2

x

36.

x  6
(x  8) (x  2)

x


25.

x  6
(x  8) (x  4)

37.

x
(x  4) (x  4)
x
(x  4) 2
n
(n  5) (n  3)

1
2(n  5 )
n2


26.

b2  12b  11
b2  9

b  9
 99

 b2  20b

1
(b  11 ) (b  1)
(b  3) (b  3)
b  1
(b  3) (b  3)




27.

28.

a2  a  6
a2  16

a2  7a  12
a2  4a  4

2.54 centimeters
1 inch

12 inches
 1 foot

(a  3) (a  2 )
(a  4) (a  4)

(a  3) (a  3)
(a  4) (a  2)

1
b  9

 (b  11) (b  9)
1

40. D;

42.

165

43.

14 feet
1

1 yard
3 feet
1

1 yard
3 feet
1

18 dollars
yard2

44.

 336
It will cost $336 to carpet the room.
33.

21.95 Canadian dollars


1
21.95 U.S. dollar

1.37

1 kilowatt

 1000 watts

8x2z2
2y3

41. A;

4a  4
a2  a

a2

 3a  3

104x3y2z3
8x2y4

 a(a  1)  3(a  1)

4(a  1)  a2

13xz3
y2

 3a(a

s  6
s2  36

4aa
 1)
4a
3(a  1)

Maintain Your Skills




s  6
(s  6) (s  6)

a2  25
a2  3a  10

(a  5) (a  5)
(a  2) (a  5)

Exclude the values for which a2  3a  10  0.


a  2  0 or a  5  0
a2
a  5
The excluded values are 2 and 5.

Exclude the values for which s2  36  0.


or s  6  0
s60
s  6
s6
The excluded values are 6 and 6.

13xyz
4x2y

Page 659
1 mile

 5280 feet

18 feet 3 feet 1 yard3


 1  27 feet3
1
20
540 feet3 1 yard3
 27 feet3
1

12 feet
1

1 hour

 60 minutes  1

 20 yard3
32.

60 watts
light
15 cents
1 dollar
 1 kilowatt  hour  100 cents

1 minute

 21.82 miles/hour
31.

1
32 feet 60 seconds 60 minutes

 1 hour
1 second 1 minute
1  60  60  1 mile
 1  1  1 hour  165
3600 miles
 165 hours

10 feet
1

1 vehicle
30 feet

 60 minutes  60 seconds

 16.67 m/s
30.

Sample answer: converting units of measure

3 feet
 1 yard

1 hour

176
5280 feet
1 mile

25 lights  h hours 

 91.44 cm/yd
29.

6864 vehicles 24 seconds 1 minute


 1 vehicle  60 seconds
1
3
6864  24  1  1 hour 1

8
1  1  60  60
20,592
 3600

(a  3) (a  4)

 (a  2) (a  2)

2.54  12  3 centimeters
yard

1
60 kilometers 1000 meters
 1 kilometer
1 hour
1  1000 meters  1  1
 1  1  1  60 seconds

13 miles
1 lane

 5.72
It will take about 5.72 hours.
38. c and e; Sample answer: The expressions each
have a GCF that can be used to simplify the
expressions.
39. Sample answer: Multiply rational expressions to
perform dimensional analysis. Answers should
include the following.

2n
 3)
2n
n  n(n  3)
2
n(n  3)

 n2 (n


 3  13  176  1 vehicle
 6864 vehicles
6864 vehicles are involved in the backup.

x
 4) (x  3)

 (x

3 lanes
1

x  3
x3  6x  9

x  3
(x  3) (x  3)

Exclude the values for which x2  6x  9  0.


x30
x  3
The excluded value is 3.

1 U.S. dollar

 1.37 Canadian dollars

 16.02
Johanna spent about $16.02 in U.S. dollars.
34.

18 miles
3 hours

6
18 hours
1

 108 miles

108 miles of streets can be cleaned.


35. 3 lanes 

13 miles
1 lane

5280 feet
1 mile

1 vehicle
30 feet

553

Chapter 12

PQ249-6481F-12[541-568] 26/9/02 6:33 PM Page 554 Sahuja Ahuja_QXP_06:Desktop Folder:Chandra:Algebra_FNL_Delivery:

45.

xy  k
(8) (9)  k
72  k
xy  72
x y x y
1 1
2 2
(8)(9)  (x2 )(6)
72  6x2
72
6

Check:

3.5r
3.5

712
79

54.

51.

24a3b4c7
6a6c2

g
8

52.
g
8

1 6

24a3b4c7  2
6a6  3c2

4b4c5
a3

3.375 6 3.5

3.625 3.5

Let r  2.1.

Let r  2.2.

3.5r 7.35

3.5r 7.35

3.5r 7.35

3.5(2.1) 7.35

3.5(2.2) 7.35

3.5(2.0) 7.35

9k 7

3
5
3 4
5 1
12
5

9k 7

12 1
5 9

Let r  2.0.

7.7 7.35

7 7.35

127 12
7

20p6
8p8

Check:

Let k  15 .

Let k  15 .

Let k  15 .

? 12
7 5

? 12
7 5

? 12
7 5

9k

115 2 7 125

3 4

12
5

12
5

9k

115 2 7 125

3 3

9
5

12
5

9k

115 2 7 125

3 5

15
5

12
5

55. Let m  money earned and d  number of days.


935  85x
m  kx
340  k(4)
11  x
85  k
It will take 11 days to earn $935.
56. x2  3x  40  (x  8) (x  5)
57. n2  64  (n  8)(n  8)
58. x2  12x  36  (x  6) (x  6) or (x  6) 2
59. a2  2a  35  (a  7) (a  5)
60. 2x2  5x  3  (2x  1) (x  3)
61. 3x3  24x2  36x  3x(x2  8x  12)
 3x(x  2) (x  6)

Page 659
1.

20p6

 8p8  6
2

 2p2

7
2
4
7 8

2 1
1

554

Practice Quiz 1

xy  k
(7)(28)  k
196  k
Choose values for x and y whose product is 96.
x
2
4
7
7
4
2

{g|g 6 28}

Chapter 12

7
2

7.35
3.5

12
12
4
5k|k 7 15 6

29 ? 7
6 2
8

3.2x2
3.2

50.

9k
4
9k 4
4 1

1
9

4y2
4

 73 or 343

7.35 7.35

3.6y2
3.6

 7129

7
2

{r|r 2.1}

8.8  x2
Thus, x  8.8 when y  3.2.
49.

Check:

48  y2
Thus, y  48 when x  4.
48.
xy  k
(4.4)(6.4)  k
28.16  k
xy  28.16
x1y1  x2 y2
(4.4)(6.4)  x2 (3.2)
28.16  3.2x2
28.16
3.2

g
8

3.5 3.5

5.4  y2
Thus, y  5.4 when x  3.6.
47.
xy  k
(8)(24)  k
192  k
xy  192
x1y1  x2 y2
(8)(24)  (4)(y2 )
192  4y2
192
4

Let g  27.

g
8

7
2

53. 3.5r 7.35

12  x2
Thus, x  12 when y  6.
46.
xy  k
(8.1)(2.4)  k
19.44  k
xy  19.44
x1y1  x2 y2
(8.1)(2.4)  (3.6) (y2 )
19.44  3.6y2


Let g  29.

g
8

28 ? 7
6 2
8

6x2
6

19.44
3.6

Let g  28.

y
98
49
28
28
49
98

y
160
120
80
40
8642O
80
120
160

2 4 6 8x

xy  196

PQ249-6481F-12[541-568] 26/9/02 6:33 PM Page 555 Sahuja Ahuja_QXP_06:Desktop Folder:Chandra:Algebra_FNL_Delivery:

2.

xy  k
(9) (6)  k
54  k
Choose values for x and y whose product is 54.
x
2
4
6
6
4
2

3.

y
27
13.5
9
9
13.5
27

4.

y  3y2
3y  1

4a
7b

a  7

2a

a  3
 7

2a

a7
6.

xy  54

3m  15
m  4

m  5

3m  15 6m  24
 m5
m  4
3(m  5) 6(m  4)
 m5
m  4

3(m  5 )
m  4

 6m  24 

2 4 6 8

7.

2x  6
x  5

y(1  3y )
(1  3y )




2(x  3 )
x  5

x
8.

k  3
k2  4k  4

2k  6
k  2

3m2
2m

k  3

5a  10
10x2

(3n  1 ) (n  2)
(3n  1 ) (n  4)

k  3
(k  2) (k  2 )

n  2
 4

 2(k  2)

4n  8
n2  25

18  m  m
9m

9.

2x  4
x2  11x  18

4x3




10.

x2  x  6
x2  9

5(a  2)
2x2  5

2x
 9
4(n  2)
(n  5) (n  5)
4
5(n  5)

x2  7x  12
x2  4x  4




x2  5x  6
x1
2(x  2)
(x  2) (x  3)

(x  9) (x  2)
x1

2(x  2)
(x  9) (x  2 )

2(x  2) (x  3)
(x  9) (x  1)

x1

2x  4

 x2  5x  6  x2  11x  18 

 a2  11a  18 
n  5
 10

k  2

 2(k  3 )

 5n

3mm
2m

k  2

 2(k  3)

2x2  2x

 (a  9) (a  2)

a

9.

k  2

 k2  4k  4  2k  6

 3m2
8.

k  3
(k  2) (k  2)

18m2
9m

1
 3

n
7.

x

2
 5

b  1
 9

3n2  5n  2
3n2  13n  4

6(m  4 )
m  5

2x  6
1

x  5 x  3
2(x  3)
1
x  3
x  5

 (x  3) 

b
6.

 18

(b  4 ) (b  1)
(b  9) (b  4 )

a  3
 7
1

y

b2  3b  4
b2  13b  36

2a

 a  3  a  3a
 a  3a

40
60
80

(7a ) (4a)
(7a ) (7b)

2a
a  3

8642O

28a2
49ab


5.

80
60
40
20

5.

n  5
 2)

 5(n

(x  3) (x  2)
(x  3) (x  3)
x  4
x  2

10.

85 kilometers
1 hour

(x  4) (x  3)
 2) (x  2)

 (x




1000 meters
1 kilometer

1 hour

(x  2 ) (x  3)
x  1

1 minute

 60 minutes  60 seconds

(85 kilometers) (1000 meters) (1 hour) (1minute)


(1 hour) (1 kilometer) (60 minutes) (60 seconds)
85  1000 meters  1  1
1  1  60  60 seconds
85,000 meters
3600 seconds

 23.61 m/s
11.

12-4 Dividing Rational Expressions

32 pounds
1 square foot

Page 662

15z
4y2

32 pounds

3x

 4y

 9 pound/square inch

2. Sometimes; Sample answer: 0 has no reciprocal.


3. Sample answer: Divide the density by the given
volume, then perform dimensional analysis.
4.

10n3
7

5n2
21

10n3
7

10n
7

2n

(32 pounds) (1 square foot)


(1 square foot) (144 square inches)

 144 square inches

Check for Understanding

1. Sample answer:

1 square foot

 144 square inches

12. There are 32 ounces in a quart,


The pan can hold 2.32  64 ounces.

21
 5n2
3

21

 5n2

If the pan is
candy in it.

2
3

128
3

3 

4

128
3

full, there are 64  3 


4

512
9

128
3

ounces of

 57

Latisha will make about 57 pieces of candy.

 6n

555

Chapter 12

PQ249-6481F-12[541-568] 26/9/02 6:33 PM Page 556 Sahuja Ahuja_QXP_06:Desktop Folder:Chandra:Algebra_FNL_Delivery:

Pages 662664
13.

a2
b2

a
b3




a2
b2
a
2

a
b2

b3
a

14.

n4
p2

n2
p3

b
3

8x2
y2

4x
y4
y2

n4
p2

y2

 8x2

16.

10m2
7n2

25m4
14n3

1
2

17.




a4bc3
g2h2

ab2c2
g3h2

19.

b2  9
4b

10m2
7n2
2

10m
7n2
1

x2y3z
s2t2

2n

14n

27.

 25m4

a4bc3
g2h2

a bc
g 2h2

1 mi

60 s

 5280 ft  1 min 

60 min
1h
1

(330 ft ) (1 mi) (60 s ) (60 min )


(1s ) (5280 ft ) (1 min ) (1 h)

1,188,000 mi
5280 h

 220 mi/h
1

28.

z2

330 ft
1s

cm3

s t
 x2yz3

y2s
z2

1m
1m
1m
1
(0.35 m3 ) (100 cm) (100 cm) (100 cm)
1 m3

5m2

s
3 2

x y z
s2t2

100 cm 100 cm 100 cm


m3 



 350,000

s3t2
 x2yz3

y2
2 3

1730 plants
1 km2

km

km

 1000 m  1000 m 

a3c
4 3
1
3

g3h2

 ab2c2

 0.00173 plants/m2

g
3 2

29.

g h

 ab2c2

2x  6
x  5

x

2
 5

2x  6
x  5

2 (x  3)
x  5




(b  3) (b  3 )
4b

30.

1
b  3

m  8
m  7

5d
d  3

m  8
m  8
1

m  8

23.

3x  12
4x  18

2x  8
x  4

(m  4 ) (m  4)
5m


31.

x2  2x  1
2

x  1
 1

x

m  4

d




(x  1) (x  1 )
2

(x  1(x  1)
2

32.

n2  3n  2
4

n  1
n  2

1
 1

33.

3(x  4 )
 9)

a2  8a  16
a2  6a  9

x  4
 4)

 2(x

2a  4
a  4

4a  8
2

a  4
 2 (a  2 )

34.

(n  2) 2
4

n  2
 1

n

a2  8a  16 3a 

a2  6a  9 2a 
(a  4) (a  4) 3(a

(a  3) (a  3) 2(a

(a  4 ) (a  4)
(a  3 ) (a  3)

3(a  4)
2(a  3)

9
8
 3)
 4)
1

b  2
b2  4b  4

2b  4
b  4

b  2

3(a  3 )
 4)

 2(a

b  4

 b2  4b  4  2b  4
1

a  4
 3

a

Chapter 12

(n  2) (n  1 )
4

a  4

 2a  6  2a  4
4 (a  2 )
2(a  3)

3(x  4)
 9)

4a  8
2a  6

n2  3n  2 n  2
n  1
4
(n  2) (n  1) n  2
n  1
4

2a  8
 9

 3a

 4(x
24.

x  1
 1

x

3x  12 x  4

 18 2x  8
3(x  4)
x  4

2(x  9) 2(x  4)

 2(x

1
(m  7) (m  1)

x2  2x  1 x  1
x  1
2
(x  1) (x  1) x  1
x  1
2

 4x


5d
(d  3) (d  1)

 m  7  (m  8 ) (m  1)

5d
 3

 m  7  (m  8) (m  1)

m2  16
1
m  4
5m
(m  4) (m  4)
1
m  4
5m

 (d  1)  d

 m2  7m  8  m  7  m2  7m  8

1
b  3

m  4
 5m
3k
1

k  1 k  2
3k
(k  1) (k  2)

 (k  2) 

x  5
2

x3

b  3
4b

 (m  4) 


22.

b2  9
1
b  3
4b
(b  3) (b  3)
4b

3k
k  1

x  5
2

a cg
b

21.

1730 plants

 (b  3) 

m2  16
5m

(1730 plants) (1 km ) (1 km )
(1 km2 ) (1000 m) (1000 m)

 1,000,000 m2

20.

ft3

14n3

 25m4

4n

 5m2

18.




x2yz3
s3t2

26. 0.35

(24 yd ) (3 ft) (3 ft) (3 ft)


1 yd3

 648

 8x2

3 ft

p
3

 n2

n2p

 2y2
x2y3z
s2t2

n
p2

3 ft

25. 24 yd3  1 yd  1 yd  1 yd 

p3
 n2

4x3
y4

3 ft

b
a

 ab
4x3
y4

15.

Practice and Apply

556

b  2
(b  2) (b  2)

b  4
2(b  2) 2

b  4

 2(b  2 )
1

PQ249-6481F-12[541-568] 26/9/02 6:33 PM Page 557 Sahuja Ahuja_QXP_06:Desktop Folder:Chandra:Algebra_FNL_Delivery:

35.

x2  x  2
x2  5x  6

x2  2x  3
 12

 x2  7x

x2  x  2 x2  7x  12

 6 x2  2x  3
(x  2) (x  1) (x  4) (x  3)

(x  3) (x  2) (x  3) (x  1)

(x  2 ) (x  1 )
(x  3) (x  2 )

x  4
x  3

36.

x2  2x  15
x2  x  30

x2  3x  18
x2  2x  24

45. Sample answer: Divide the number of cans


5
recycled by 8 to find the total number of cans
produced.
Answers should include the following.

 x2  5x

(x  4) (x  3 )

 (x  3 ) (x  1 )
1

46. B;

x2  2x  15 x2  2x  24

x2  x  30 x2  3x  18
(x  5) (x  3) (x  6) (x  4)

(x  6) (x  5) (x  6) (x  3)

(x  5 ) (x  3)
(x  6 ) (x  5 )

60 minutes
1 hour

V


18b
15c

3b
5c

3b
5c

 18b

(x  6 ) (x  4)

 (x  6) (x  3)

15c
6

A  /w
x2  4 

x2  4
x  1
 x2  x  2
1
(x  2) (x  2)
x  1
 (x  2) (x  1)
1
(x  2) (x  2)
x  1
 (x  2) (x  1)
1

60

 6.5  9.2

1 yd3

Page 664

39. An equation that represents the number of truck


loads.

48.

15 ft (18 ft2  15 ft)  9 ft  271 ydft 2


742.5 ft
1 yd
 20,000 yd3  1 1  27 ft 2
742.5 yd
 20,000 yd3  1 27 2

x  2
1

49.

x2  3x  10
x2  8x  15

x  5
(x  5 ) (x  2)

x2  5x  6
 4

rev

30 in.
1 rev

x  4
4y

2y

2y  x
2xy

(x  5) (x  2)
(x  5) (x  3)

(x  5 ) (x  2)
(x  5 ) (x  3 )

(x  3) (x  2)
 2)

 (x  2) (x
1

(x  3 ) (x  2 )
 2)

 (x  2) (x

x  4
4y
1

x  4
4y

16y
(x  4 ) (x  3)

16y

1 hr
 60 min

x

(12) (5280) (55)


(26) (60)

1 ft

51.

x2  8x  15
x  y

7x  14y
x  3

1 mi

 12 in.  5280 ft  63.5 mph

52.

c  6
c2  12c  36

2y  x
2xy

x  2y
.
x2  4y2
1

21

43. Volume of new block  x x  2 x  4

 (10  85 pounds) 

 3)

4
 3

(x  5) (x  3)
x  y

(x  5) (x  3 )
(x  y)

7(x  2y)
x  3

7(x  2y)
x  3

7(x  2y) (x  5)

x  y
c  6
(c  6) (c  6)

c  6
(c  6 ) (c  6)

is not equivalent to

44. Weight of original block

16y

 (x  4) (x

 2y  2xy  2xy


 x2  7x  12 

42. d; Sample answer:


1
x

55 mi
 1 hr

60 min
1h

x  2
1

x  2
 2

50.

x

 711 rpm
41. 1 revolution  circumference  d  30
711min 

x  2
1

27.5 yd3

5280 ft
 1 mi

 x2  4x

It will take 727.27 truck loads.


40. 1 revolution  circumference  d  26.
12 in.
 1 ft

w

x  5
(x  5) (x  2)

1

 727.27

1 rev
26 in.

w

 20,000 yd3  27.5 yd3




w

20,000 yd3
1
20,000
27.5

w

Maintain Your Skills

x  5
x2  7x  10

x2  x  2
x  1

x2w

 9 ft  27 ft3

15c

 18b

47. C;

(x  3) (x  4)
(x  6) (x  3)

n  20,000 yd3 

1 pound

 6 or 2

Jorge should ride at 9.2 miles per hour.


38. There are 27 cubic feet in 1 cubic yard.
d(a  b)
w
2
5 ft(18 ft  15 ft)
2

3b
5c

37. Convert minutes per mile into miles per hour.


1 mile
6.5 minutes

x  63,900,000 cans  8  33 cans

c
53.

25  x2
x2  x  30

1
 6

1(x  5) (x  5)
(x  6) (x  5)

1(x  5) (x  5 )
(x  6) (x  5 )

x  5
x  6

1x  12 21x  34 2 (x)

x3

557

Chapter 12

PQ249-6481F-12[541-568] 26/9/02 6:33 PM Page 558 Sahuja Ahuja_QXP_06:Desktop Folder:Chandra:Algebra_FNL_Delivery:

54.

a  3
a2  4a  3

a  3
(a  3) (a  1)

a  3
(a  3 ) (a  1)

61. The degree of z3  2z2  3z  4 is 3, since 3 is the


greatest sum of a terms exponents.
62. The degree of a5b2c3  6a3b3c2 is 10, since 10 is
the greatest sum of a terms exponents.
6  0.8 g
63.

a1
55.

n2  16
n2  8n  16

(n  4) (n  4)
(n  4) (n  4)

(n  4 ) (n  4)
(n  4 ) (n  4)

6
0.8

n  4
 4

n
56. 3y2  147
y2  49
y  7
{7}
Check:

15b
15

3(7) 2  147
3(49)  147
147  147
3(7) 2  147
3(49)  147
147  147
57.
9x2  24x  16
9x2  24x  16  0
(3x  4)(3x  4)  0
3x  4  0
3x  4

56

x

Check:

9
1

5b|b 7 6

Check:

1 2
1

12

24 4
1 3

0.049
0.07

15b 6 28

15b 6 28

1 2 6 28
28
15

15

1 2 6 28
29
15

15

29 6 28

12715 2 6 28

27 28

0.07x
0.07

0.049  0.07(0.6)
0.049  0.042
66.

3
h
7
7
3

3
49
3 7
49 3
1
7
1
7

5h|h 6 6

Check:

Let h  7.
3
h
7

3
49

1 2 6 493

3 1
7 7

3
49

2 ?

[ (6  114)  6]  14
?
(6  3.742  6) 2  14
14.003  14
?

is 0, since there are no

558

0.049  0.07(0.8)
0.049  0.056

137h21 2 6 1 21 2
6

h 6

[ (6  114)  6] 2  14
?
(6  3.742  6) 2  14
14.003  14

Chapter 12

15b 6 28

0.049  0.07(0.7)
0.049  0.049
Let x  0.6.
Let x  0.8.
0.049  0.07x
0.049  0.07x

 16

1
8

27

Let b  15 .

0.7  x
{x|x 0.7}
Let x  0.7.
Check:
0.049  0.07x

(15) 2  225  30(15)


?
225  225  450
450  450

60. The degree of 13 


exponents.

29

Let b  15 .

28 28

59. (n  6) 2  14
n  6  114
n  6  114
{6  114}
Check:

28

65. 0.049  0.07x

16  32  16
16  16
58.
a2  225  30a
a2  30a  225  0
(a  15) (a  15)  0
a  15  0
a  15
{15}
Check:

Let g  7.
6  0.8 g
?
6  0.8(7)
6  5.6

Let b  15 .

15

4
3

Let g  8.
6  0.8 g
?
6  0.8(8)
6  6.4

28
15
28
15
28
15

b 7

143 22  24143 2  16

9 16
1 9

0.8 g
0.8

7.5  g
{g|g 7.5}
Let g  7.5.
Check:
6  0.8 g
?
6  0.8(7.5)
66
64. 15b 6 28

4
3

49

Let h  7.
3
7

h 6

3
49

1 2 6 493

3 2
7 7

6
49

49

Let h  8.
3
7

h 6

3
49

1 2 6 493

3 1
7 8

3
56

3
49

PQ249-6481F-12[541-568] 26/9/02 6:33 PM Page 559 Sahuja Ahuja_QXP_06:Desktop Folder:Chandra:Algebra_FNL_Delivery:

67.

12r
4

3
20

70.

12r
4

3r 7
3r
3

3
20
3
20
3
20

6x2
x4

6  x2

 x 2  x2
1


72.

3
1
r 6 20

18a3
45a5

12

4
1

Let

12r
4
1
20

4
1
12 20
4
12
1

4
1

74.
7
?

3
20

29aaa
1

73.

b6c3
b3c6

3
20

4

3
20

4
1

1181 2 7 1220
?

12

12

Let r  22.
12r
4
1
22

4


12 22
4
12
1

1bb 21cc 2
6

75.

7x4z2
z3

b3
c3

 7(x4 )
 7(x

1zz 2
2
3

4 ) (z23 )

 7(x4 ) (z 1 )


7x4
z

7
?

7
?

7.

3x  6
9

9.

5x2  25x
10x

3
20




3
20
3
20

Reading Mathematics

4
1

10.

1221 2 7 1220

3(x  2)
9
x  2
3

4n  12
8




4(n  3)
8
n  3
2

12

8.

5x(x  5)
 10x
x  5
 2
x  3
x  3
 (x  4) (x  3)
x2  7x  12

x  3
(x  4) (x  3 )
1

12

22
20

1
 4
x  y
(x  y) (x  y)

x  y
(x  y) (x  y )

1
x  y

x

1 2 12  61

11.

x  y
x2  2xy  y2

y 3
{y|y 3}
Check:
Let y  3.
y
6

1. Sample answer: the quantity m plus two, divided


by 4
2. Sample answer: three x divided by the quantity x
minus 1,
3. Sample answer: the quantity a plus 2 divided by
the quantity a squared plus 8
4. Sample answer: the quantity x squared minus 25
divided by the quantity x plus 5
5. Sample answer: the quantity x squared minus 3 x
plus 18 divided by the quantity x minus 2
6. Sample answer: the quantity x squared plus 2 x
minus 35 divided by the quantity x squared
minus x minus 20

18 7 20

4
1

11228 21xx 21yy 2


3
 1 7 2 (x34 ) (y21 )
3
 1 7 2 (x1 ) (y)


Page 665

 7x

4
 1

m3
5

12 18

12

12x3y2
28x4y

5mmmm
55m

 (b63 ) (c36 )
 (b3 ) (c3 )

3y

12
12
20
20
1
r  18.
12r
3
7 20
4
1
12 18 ?
3
7 20
4

12
1

y
6
6 y
1 6

 59aaaaa

3
20

1201 2 7 1220

68.

6
x2

Let r  20.

5m4
25m

 5a2

5r|r 6 201 6

Check:

71.

3 ? 1
2
6
1
1
2
2

Let y  4.

Let y  2.

y
6

y
6

4 ? 1
2
6
2
1
2
3

12.

x2  16
x2  8x  16

(x  4) (x  4)
(x  4) (x  4)

(x  4 ) (x  4)
(x  4 ) (x  4)

2 ? 1
2
6
1
1
2
3

x  4
 4

x

69. Multiply 3250 by 12.


3250 covers
1 month

 12 months  39,000 covers

559

Chapter 12

PQ249-6481F-12[541-568] 26/9/02 6:33 PM Page 560 Sahuja Ahuja_QXP_06:Desktop Folder:Chandra:Algebra_FNL_Delivery:

12-5

Dividing Polynomials

Page 667

Algebra Activity

x2

1. (x2  3x  4)  (x  1)
Step 1:

x2

Step 3:

x2

1 1

x 3

x x

x
x
x

1 1
Step 2:

1
1
1

1
1
1

x1
The width of the array, x  3, is the quotient.
3. (x2  16)  (x  4)

x2

Step 1:

1

1 1 1 1 1 1
1 1 1 1 1

x2

1 1 1 1 1
Step 2:
Step 3:

x1
x4

x4

x2

x

x
x
x
x

1
1
1
1

x2
1 1 1 1

The width of the array, x  4, is the quotient.


2. (x2  5x  6)  (x  2)
Step 1:

x2

Step 3:

x4

x x x x x

x4

Step 2:

x x x x

x
x

1 1 1 1
1 1 1 1
1 1 1 1
1 1 1 1

x
x

x2

x2

x2

x x

The width of the array, x  4, is the quotient.


4. (2x2  4x  6)  (x  3)
Step 1:

x2

x2

x x x x

1 1 1
1 1 1

Chapter 12

560

PQ249-6481F-12[541-568] 26/9/02 6:33 PM Page 561 Sahuja Ahuja_QXP_06:Desktop Folder:Chandra:Algebra_FNL_Delivery:

Step 2:

5.

14a2b2  35ab2  2a2


7a2b2

14a2b2
7a2b2

14a2b2
7a2b2

35ab2
7a2b2

35ab2
7a2b2

x3

2

1 1 1

x3
x x x

x2

x x x

x
x

1 1 1
1 1 1

The quotient is r  3  r
8. 2m

The width of the array, 2x  2, is the quotient.


5. You cannot do it. There is a remainder.

Page 669

Check for Understanding

2x  3
b. x  32x2  9x  9
()2x2  6x
3x  9
()3x  9
0

x3
c. 2x  32x2  9x  9
()2x2  3x
6x  9
()6x  9
0

x6
d. 2x  32x2  9x  9
()2x2  3x
12x  9
()12x  18
27

2m2  3m  7
 0m2  5m  21
 6m2
6m2  5m
() 6m2  9m
14m  21
()14m  21
0

 34m3
()4m3

The quotient is b  2  2b  1.
10. Let p  0.75.
C

120,000(0.75)
1  0.75

 360,000
The company will have to pay $360,000.

Pages 669671

Practice and Apply

11. (x  9x  7)  3x 


x2  9x  7
3x
x2
9x
7

 3x
3x
3x
x
7
 3  3x
3

Therefore, b and c are the divisors that give a


remainder of 0.
2. Sample answer: A remainder of zero means that
the divisor is a factor of the dividend.

12. (a2  7a  28)  7a 

3. Sample answer: x3  2x2  8; x3  2x2  0x  8

4. (4x3  2x2  5)  2x 



4x3  2x2  5
2x
4x3
2x2
5

 2x
2x
2x
2x2

4x 3
2x
1


13.

2x2
2x
1

9
.
 9

The quotient is 2m2  3m  7.


b2
9. 2b  12b2  3b  5
()2b2  b
4b  5
()4b  2
3

1. Test each divisor to see which divisors give a


remainder of 0.
2x  15
a. x  32x2  9x  9
()2x2  6x
15x  9
()15x  45
54

7b2

2
7b2

r3
7. r  9r2  12r  36
()r2  9r
3r  36
() 3r  27
9

Step 3:

2x  2

5
a

2a2
7a 2b2

n4
6. n  3n2  7n  12
()n2  3n
4n  12
() 4n  12
0
The quotient is n  4.

x2

x2

x2

2a2

 7a2b2

 2x

9s3t2  15s2t  24t3


3s2t2

15s2t
3s2t2

 3s2t2 

15s2t
3s2t2

9s3t2
3s2t2

a2  7a  28
7a
a2
7a
28
 7a  7a
7a
a
4
1a
7

9s3t2

 2x2  x  2x

3

561

5
t

24t3

 3s2t2
8t

24t3

 3s2t2
s2

8t
s2

Chapter 12

PQ249-6481F-12[541-568] 26/9/02 6:33 PM Page 562 Sahuja Ahuja_QXP_06:Desktop Folder:Chandra:Algebra_FNL_Delivery:

14.

12a3b  16ab3  8ab


4ab

12a3b
4ab

12a3b
4ab

16ab3
4ab

16ab3
4ab

3a2

4b2

6n  3
24. 2n  512n2  36n  15
()12n2  30n
6n  15
()6n  15
0
The quotient is 6n  3.

8ab

 4ab
2

8ab

 4ab
1

 3a2  4b2  2
x8
x4
15. x  5x2  9x  20
16. x  2x2  6x  16
()x2  5x
()x2  2x
4x  20
8x  16
()4x  20
()8x  16
0
0
The quotient is x  4.
The quotient is x  8.
s2
n7
17. n  5n2  2n  35
18. s  9s2  11s  18
()n2  5n
()s2  9s
7n  35
2s  18
()7n  35
()2s  18
0
0
The quotient is n  7.
The quotient is s  2.

3x2  2x  3
25. x  2
 8x2  x  7
() 3x  6x2
2x2  x
() 2x2  4x
3x  7
() 3x  6
1
3x3
3

The quotient is 3x2  2x  3  x


5b2  3b  1
26. 4b
 27b2  13b  3
 15b2
12b2  13b
() 12b2  9b
4b  3
() 4b  3
0
The quotient is 5b2  3b  1.
 3 20b3
() 20b3

z9
19. z  7z2  2z  30
()z2  7z
9z  30
()9z  63
33
33

3x2
27. 2x  36x3  9x2  0x  6
() 6x3  9x2
6

The quotient is z  9  z  7.
a7
20. a  3a2  4a  22
()a2  3a
7a  22
()7a  21
1
The quotient is a  7  a

The quotient is 3x2  2x  3.


3g2  2g
28. 3g 
 0g2  5g
() 9g  6g2
6g2  5g
() 6g2  4g
9g
() 9g
29g3
3

1
.
 3

2r  7
21. r  52r2  3r  35
()2r2  10r
7r  35
()7r  35
0

3
8

8
6
2

The quotient is 3g2  2g  3  3g

The quotient is 2r  7.
3p  2
22. p  63p2  20p  11
()3p2  18p
2p  11
()2p  12
1

29. 2n

The quotient is 3p  2  p  6.

3n2  2n  3
 5n2  0n  12
 9n2
4n2  0n
() 4n2  6n
6n  12
() 6n  9
3

t2  4t  1
30. 4t
 17t2  0t  1
 t2
16t2  0t
() 16t2  4t
4t  1
() 4t  1
0
The quotient is t2  4t  1.
 14t3
() 4t3

562

2
.
 2

 36n3
() 6n3

The quotient is 3n2  2n  3  2n

t6
23. 3t  43t2  14t  24
()3t2  4t
18t  24
()18t  24
0
The quotient is t  6.

Chapter 12

1
.
 2

3
.
 3

PQ249-6481F-12[541-568] 26/9/02 6:33 PM Page 563 Sahuja Ahuja_QXP_06:Desktop Folder:Chandra:Algebra_FNL_Delivery:

31. Let W  150 and L  60.

210(72  20)
20

 546
He could lift a 546-lb rock.
33. Perimeter of bedroom  14(12)  12(12)  14(12)
 12(12)  34.5  34.5
 42  42
 471 inches
Convert 471 inches into yards and divide by 5.
5 yd
roll

64

10w3  23w2  5w  2
2w  1

64C
d2

15w2

35. Size
10-inch
Price
$4.99
Number of slices
5
Cost per slice
$1.02
$4.99

(10) 2
64

$12.99

1(18)
64 2
2

 $1.02

14-inch
$8.99
10
$0.93

14-inch:

 $0.82

18-inch
$12.99
16
$0.82

$8.99

1(14)
64 2
2

mass
volume

Concrete

Lead

Brass

Blood

 B

 9w
2w  110w3  23w2  5w
() 10w3  5w2
18w2  5w
() 18w2  9w
4w
() 4w

2
2

2
2
0
Area of the base: 5w2  9w  2

 $0.93

The area of a triangle is equal to 2bh.


1

5w2  9w  2  2 b(5w  1)
1

2(5w2  9w  2)  2  2 b(5w  1)

The 18-inch pizza offers the best price per slice.


36.

Iron

10w3  23w2  5w  2  B(2w  1)


(10w3  23w2  5w  2)  B(2w  1)

s  C  d2

18-inch:

6
4

38. The densities are clustered around 9.


39. Find the area of the base using the formula
V  Bh.

1d64 2

10-inch:

Aluminum

 2.62

Therefore, Anoki needs to buy 3 rolls of border.


C
34. s 
2

s

10

2
0

1 yd

471 in.  36 in.

12

Copper

Steel

W(L  x)
x

Density (g/cm3)

32. Let W  210 pounds, L  6  12  72 inches, and


x  20 inches.

20
18
16
14

Silver

37.

150(60  x)
x

Gold

W(L  x)
x

10w2  18w  4
5w  1

 density

b

2w
5w  110w2  18w
() 10w2  2w
20w
() 20w

4.15
Al  1.54  2.69 g/cm3
2.32
gold  0.12  19.33 g/cm3
6.30
silver  0.60  10.5 g/cm3
7.80
steel  1  7.8 g/cm3
15.20
iron  1.95  7.79 g/cm3
2.48
copper  0.28  8.86 g/cm3
4.35
blood  4.10  1.06 g/cm3
11.30
lead  1  11.3 g/cm3
17.90
brass  2.08  8.61 g/cm3
40
concrete  20  2 g/cm3

4
4
4
4
0

The base is 2w  4.
40.

x2  7x  12
x  k

(x  4) (x  3)
(x  k)

In order for the quotient to have no remainder, k


must be either 3 or 4.
41.

563

x5
x  2x2  7x  k
() x2  2x
5x  k
() 5x  10
k  10  2
In order for the quotient to have a remainder of 2,
k  10  2. The value of k is 12.

Chapter 12

PQ249-6481F-12[541-568] 26/9/02 6:33 PM Page 564 Sahuja Ahuja_QXP_06:Desktop Folder:Chandra:Algebra_FNL_Delivery:

x9
x  7x2  2x  k
() x2  7x
9x  k
() 9x  63
k  63  0
In order for the quotient to have a remainder of 0,
k  63  0. The value of k is 63.
43. Sample answer: Division can be used to find the
number of pieces of fabric available when you
divide a large piece of fabric into smaller pieces.
Answers should include the following.
The two expressions are equivalent. If you use
the Distributive Property, you can separate
the numerator into two expressions with the
same denominator.
When you simplify the right side of the
equation, the numerator is a  b and the
denominator is c. This is the same as the
expression on the left.
44. D; A    w
m2  4m  32    (m  4)
m8
m  4m2  4m  32
() m2  4m
8m  32
() 8m  32
0

51. 172  132  262  2  242  2


 612  412
 (6  4) 12
 1012
52. 112  118  148  222  3  232  2  242  3

x2  3x  14
45. B; x 
 0x2  5x  20
() x  3x2
3x2  5x
() 3x2  9x
14x  20
() 14x  42
22

58. (3x2  4xy  2y2 )  (x2  9xy  4y2 )


 (3x2  x2 )  (4xy  9xy)  (2y2  4y2 )
 4x2  13xy  2y2

42.

 213  312  413


 (2  4) 13  312
 613  312

53. d2  3d  40  d2  8d  5d  40
 (d2  8d)  (5d  40)
 d(d  8)  5(d  8)
 (d  8) (d  5)
54. x2  8x  16  x2  4x  4x  16
 (x2  4x)  (4x  16)
 x(x  4)  4(x  4)
 (x  4) 2
55. This polynomial is prime since t2  t  1 cannot
be factored.
56. Solve for x.
6x  11  150
6x  139
x  23 boxes
The greatest number of cards Mr. Martinez can
have printed is 23  100 or 2300 cards.
57. (6n2  6n  10m3 )  (5n  6m3 )
 [10m3  (6m3 ) ]  6n2  [ 6n  5n ]
 4m3  6n2  n

3x3
3

59. (a3  b3 )  (3a3  2a2b  b2  2b3 )


 [ a3  (3a3 ) ]  2a2b  b2  [b3  (2b3 ) ]
 2a3  2a2b  b2  3b3

60. (2g3  6h)  (4g2  8h)  2g3  4g2  [ 6h  (8h) ]

The quotient is x2  3x  14 

Page 671
46.

x  5x  6
x2  x  12

 2g3  4g2  2h

22
.
x  3

Maintain Your Skills


x  2
 20

 x2  x

x  5x  6
 12

 x2  x
1

x  x  20
x  2

(x  3 ) (x  2 )
(x  4 ) (x  3 )

Page 674

(x  5) (x  4)
x  2

m2  m  6
m 2  8m  15

m2  m  2
m 2  9m  20

m2  m  6
m 2  8m  15

(m  3 ) (m  2 )
(m  5 ) (m  3 )

m  4
m  1

b2  19b  84
b  3

(m  5 ) (m  4 )
 2 ) (m  1 )

 (m

b2  9

 b2  15b  36 

(b  12 ) (b  7)
b  3
1

(b  3 ) (b  3 )

 (b  12 ) (b  3 )
1

b7

49.

z2  16z  39
z2  9z  18

z  5

 z2  18z  65 

(z  13 ) (z  3 )
(z  6) (z  3 )
1

z  5

 (z  5 ) (z  13 )
1

5.

z6

a  2
4

a  2
4




50. 317  17  (3  1) 17
 217
Chapter 12

x  6
x  2

x  4
 2

x

 1.

2. Sample answer: When you add rational


expressions with like denominators, you combine
the numerators and keep the common
denominator. This is the same process as adding
fractions with like denominators.
3. Sample answer: Two rational expressions whose
sum is 0 are additive inverses, while two rational
expressions whose difference is 0 are equivalent
expressions.
4. Russell; sample answer: Ginger factored
incorrectly in the next-to-last step of her work.

m 2  9m  20
m2  m  2

48.

Check for Understanding

1. Sample answer:

x5
47.

Rational Expressions with Like


Denominators

12-6

564

a  2  a  2
4
2a
4
a
2

PQ249-6481F-12[541-568] 26/9/02 6:34 PM Page 565 Sahuja Ahuja_QXP_06:Desktop Folder:Chandra:Algebra_FNL_Delivery:

6.

3x
x  1

x

3
 1




3x  3
x  1
3(x  1)
x  1

23.

3
7.

8.

9.

11.

12.

2  n
n  1

24.

2  n  1

 n1
3  n
n1
4t  1
2t  3
4t  1  2t  3
 1  4t 
1  4t
1  4t
6t  2
 1  4t
5a
7a
5a  7a
7
4
 12  12
10. n  3  n  3
12
2a
 12
a
 6
3m
6
3m
6
 2  m  m  2  (m  2)
m  2
3m
6
m2m2
3m  6
 m2
2

x
x  y

1
n  1

x

y
 y




25.

26.



7  4
n  3
3
n  3

27.

x  y
x  y
(x  y) (x  y)
(x  y)





Pages 675676
14.

m
3

2m
3




45  29  10  12
960
96
960
1
10

15.

12z
7

5z
7





m
x  3
5

x  2
5




17.

n  7
2

n  5
2

a  5
6

18.

6
y  3




2y  6
y  3
2(y  3)
y  3

19.

3r
r  5

2
20.

k  5
k  1

k1


32.

21.

22.

4x  5
x  2

1
n  3




x  3
x  2




a  3
6

2
x  7

5
x  7

33.

4
z  2

34.

5
3x  5

r

15
 5







4n  2n
3
2n
3

(a  5)  (a  3)
6
a  5  a  3
6
2
6
1
3
2

2 
x 
7
x 

x


7
5
7

x

5
 7

6
z  2

4
6
z2z2
10
z2
3x
5  3x
 3x  5
3x  5
(3x  5)
 3x  5

 1

3r  15
r  5
3(r  5)
r  5

35.
36.

3

k  5  4
k  1
k  1
k  1

3x
7

1
n  2
n  3

n1
2y
y  3

11x  5  11x  12
2x  5
22x  7
2x  5

31.

5x
7

n  7  n  5
2
2n  2
2
2(n  1)
2

30.

x  3  x  2
5
2x  5
5

11x  12
2x  5

12z  (5z)
7
12z  5z
7
7z
7

z
16.

11x  5
2x  5

5x  3x
4n
2n
29. 3  3
7
2x
 7
x  4
x  2
(x  4)  (x  2)
 5 
5
5
x  4  x  2

5
2
5

28.

Practice and Apply

m  2m
3
3m
3

2a  3  a  2
a  4
3a  1
 a4
5s  1
3s  2
5s  1  3s  2
 2s  1 
2s  1
2s  1
8s  1
 2s  1
9b  3
5b  4
9b  3  5b  4
 2b  6 
2b  6
2b  6
14b  7
 2b  6
12x  7
9x  5
12x  7
9x  5
 2  3x  3x  2  (3x  2)
3x  2
12x  7
9x  5
 3x  2  3x  2
(12x  7)  (9x  5)

3x  2
12x  7  9x  5

3x  2
3x  2
 3x  2

xy
13.

a  2
 4

a

1

number of students absent


total number of students

2a  3
a  4

37.

n  2  (1)
n  3
n  3
n  3
4x  5  x  3
x  2
5x  2
x  2

565

4
7m
4  7m
 7m  2  7m  2
7m  2
2x
2x
2x
2x
 2  x  x  2  (x  2)
x  2
2x
2x
x2x2
4x
x2
5y
5y
5y
5y
 3  y  y  3  (y  3)
y  3
5y
5y
y3y3
5y  5y
 y3
10y
y3

Chapter 12

PQ249-6481F-12[541-568] 26/9/02 6:34 PM Page 566 Sahuja Ahuja_QXP_06:Desktop Folder:Chandra:Algebra_FNL_Delivery:

38.

8
3t  4

 3t

6t
 4

8  6t
 4
2(3t  4)
3t  4

47. For figure a:

 3t


area 

x2

 16

 2
39.

3
 1

15x
5x  1

 5x

15x
3
 5x  1
 1
15x  3
5x  1
3(5x  1)
5x  1

perimeter  x

 5x



40.

ratio 

41.

b  15
2b  12

For figure b:

10a
2a
10a
2a
10a

area 

x2

perimeter  x
ratio 

x
x
 18

For figure c:

x2

perimeter  x

x
x
 24

180
3n
60
n

48. Figure a has the greatest ratio since it has the


smallest number in the denominator.
49. Put the numbers in the same form.
a.

b.

area of one wall  area of trapezoid  area of triangle


area of one wall
1
1
(25  10)  2 (10)(20  5)  2 (15)(10)

25  10

250  125  75
250
50
1
or 5
250

2 1

45. There are 2 points in a quart and 4 quarts in a


gallon.
1 ft3
7.48 gal

12(1 qt)  4(1 pt) 121 ptqt 22  14gal


qt
3

1 ft

 7.48 gal (2 qt  2 qt) 


3

1 ft

k  2

1 gal
4 qt

3
 2
3
x  2

d. x

3
 2

3
 7

k  2  3
k  7
k  1
k  7

5r
52. B; perimeter of rectangle ABCD  2 1 2r 9r
 2 1 2r  6s 2
 6s 2

1gal

r

 7.48 ft3

 7.48 ft3  8.3 lb

Chapter 12

c. 2  x  x

62.4 lb
ft3

3
 2
3
x  2

 x

51. A; k  7  k

 7.48 gal (4 qt )  4 qt

46.

3
2  x
3
x  2

Number c is not equivalent to the others.


50. Sample answer: Since any rational number can be
expressed as a fraction, values on graphs can be
written as rational expressions for clarification.
Answers should include the following.
The numbers in the graphic are percents that
can be written as rational expressions with a
denominator of 100.
To add the rational expressions, factor 1 out
of either denominator so that it is like the
other.

fraction of walls painted red

124x 2
2

44. The area of a trapezoid is equal to 2h(b  b )


1
2
1
and the area of a triangle is 2bh.

1 x x
2 4 3

 24

The fraction of the trees that could be planted on


60
Monday, Tuesday, and Saturday is n .

1 21 2
1 21 2

1 3x 4x
12 12

area  2

ratio 

118x 2
2

8,695
269,817
140  20  20
n  2n

13x 216x 2

 18

42. 80 years or older: 8,634,000  61,000


total population: 77,525,000  79,112,000 
68,699,000  35,786,000 
8,634,000  61,000

43.

 16

 12
6a
 (2a  6)
 6
 12
6a

 2a  6
 6
 12  6a

2a  6
16a  12
 2a  6
2(8a  6)
 2(a  3)
8a  6
 a3
3b  8
(b  15)  (3b  8)

2b  12
2b  12
b  15  3b  8

2b  12
4b  23
 2b  12

6a
6  2a

116x 2
2

3
10a  12
2a  6

14x 22

r

9r  5r
r  3s

r

566

9r
 3s

14r
 3s

5r
 3s

PQ249-6481F-12[541-568] 26/9/02 6:34 PM Page 567 Sahuja Ahuja_QXP_06:Desktop Folder:Chandra:Algebra_FNL_Delivery:

Page 677

66.

Maintain Your Skills

x2  2x  3
53. x  2x3  0x2  7x  6
() x3  2x2
2x2  7x
() 2x2  4x
3x  6
() 3x  6
0
The quotient is x2  2x  3.

68.

69.

8x2
9
54. 7x  456x3  32x2  63x  36
() 56x3  32x2
63x  36
() 63x  36
0

70.

71.

The quotient is 8x2  9.


55.

b2  9
4b

 (b  3) 

b2  9
4b

b  3
1

56.

x
x  2

(b  3) (b  3 )
4b

b  3
4b

x2
x2  5x  6

b

x2  5x  6

x2

Page 677

x
x  2

x

1
 3

67.
45  3  3  5
55
10  2  5
6  23
6  23
15  3  5
LCM  2  3  3  5  90
LCM  2  3  5  30
8  222
9  33
12  2  2  3
LCM  2  2  2  3  3  72
16  2  2  2  2
20  2  2  5
25  5  5
LCM  2  2  2  2  5  5  400
36  2  2  3  3
48  2  2  2  2  3
60  2  2  3  5
LCM  2  2  2  2  3  3  5  720
9  33
16  2  2  2  2
24  2  2  2  3
LCM  2  2  2  2  3  3  144

1.

x
 2
1

a
a  3

Practice Quiz 2

a  11
a  3

a  3

a  3

 a  3  a  11
1

(x  3) (x  2 )
x2

 a  3  a  11

x  3
x

 a  11

57. a2  9a  14  a2  7a  2a  14
 a(a  7)  2(a  7)
 (a  2)(a  7)

2.

4z  8
z  3

4z  8
1

z  3 z  2
4(z  2)
1
z  2
z  3

4(z  2 )
z  3

 (z  2) 

58. p2  p  30  p2  6p  5p  30
 p( p  6)  5( p  6)
 ( p  5)( p  6)
3.

(2x  1) (x  2)
(x  2) (x  3)

(2x  1) (x  5)
(x  3) (x  1)

(2x  1 ) (x  2 )
(x  2 ) (x  3 )

(x  3) (x  1)
 1) (x  5)

 (2x

(x  3 ) (x  1)
 1 ) (x  5)

 (2x

x  1
 5

 (4x  9x)  7

4. (9xy2  15xy  3)  3xy 

61. (5x2  6x  14)  (2x2  3x  8)


 (5x2  2x2 )  (6x  3x)  (14  8)
 7x2  3x  22
62. 1 foot  12 inches
8

(2x  1) (x  2)
(x  2) (x  3)

x

 3x2  5x  7

riser
tread

60. (3x2  4x)  (7  9x)  (3x2  4x)  (7  9x)




z3

59. y2  11yz  28z2  y2  7yz  4yz  28z2


 y(y  7z)  4z(y  7z)
 (y  4z) (y  7z)
3x2

z  2

9xy2
3xy

15xy
3xy

 3xy
1

 3y  5  xy
x5
5. 2x  32x2  7x  16
() 2x2  3x
10x  16
() 10x  15
1

 12  3

4  22
9  33
12  2  2  3
LCM  2  2  3  3  36
64.
77
21  7  3
55
LCM  3  5  7  105
65.
6  23
12  2  2  3
24  2  2  2  3
LCM  2  2  2  3  24
63.

The quotient is x  5  2x  3.
y  15
6. y  4y2  19y  9
() y2  4y
15y  9
() 15y  60
51
The quotient is y  15  y

567

51
.
 4

Chapter 12

PQ249-6481F-12[541-568] 26/9/02 6:34 PM Page 568 Sahuja Ahuja_QXP_06:Desktop Folder:Chandra:Algebra_FNL_Delivery:

7.

8.

2  5
x  7
7
x7
6
2m
6
m3m3
m  3
2m  6
 m3
2(m  3)
 m3

2
x  7

x

2m
m  3

5
 7

9. The LCD is (y  5)(y  5).

2y
y2  25

2
9.

5x  1
3x  2

2x  1
3x  2





3
34

in.

11. The LCD is


3z
6w2

16.5 minutes

1


4a
2a  6

a

3
 3




13. The LCD is (b  4)(b  4).


b  8
b2  16

b

1
 4




a
a  4

a  4
a  4

b  4

b  8





b  4
(b  4) (b  4)
b  8  (b  4)
(b  4) (b  4)
b  8  b  4
(b  4) (b  4)
4
(b  4) (b  4)

14. The LCD is (x  2)(x  5).


1)2(n

x
x  2

 4)

 x2  3x

 10

x

x
 2

x  5
 5

x

 x2  3x

x2  5x
(x  2) (x  5)

x2  5x  3
(x  2) (x  5)

 10

3
(x  2) (x  5)

15. C; The LCD is ( y  4)( y  3).


2y
y2  7y  12

a  4a
(a  4) (a  4)

y  2
 4

y

2y

 y2  7y

 y2  7y

a  4
a  4

4a  16
(a  4) (a  4)

a  4a  4a  16
(a  4) (a  4)

a2  8a  16
(a  4) (a  4)

Pages 681683

568

y  2
 4

 12

y

 12

2y

4
a  4

16.

Chapter 12

b  8

 b2  16  b  4  b  4
 b2  16 

6 2x
7
  10x2
5x 2x
12x
7
 10x2
10x2
12x  7
10x2

4
a  4

4a
3
2
 a  32
 6
4a
6
 2(a  3)
2a  6
4a  6
2(a  3)
2(2a  3)
2(a  3)
2a  3
a  3

 2a


8. The LCD is (a  4)(a  4).


a
a  4

3w

3wz

x
5
,
2x  6 x  3

LCM  (n  1)(n  1)(n  4) or (n 


7. The LCD is 10x2.


12. The LCD is 2(a  3).

6. n2  3n  4  (n  4)(n  1)
(n  1) 2  (n  1)(n  1)

6z  3wz
12w2
3(2z  wz)
12w2
2z  wz
4w2

LCM  5  7  a  a  35a2
5. 2x  4  2(x  2)
3x  6  3(x  2)
LCM  (2  3)(x  2)  6(x  2)

a  2
6(a  1)
 (a  3) (a  1)
(a  3) (a  1)
a  2  6a  6
(a  3) (a  1)
7a  8
(a  3) (a  1)

12w2.
3z

Check for Understanding

7
10x2

y2  12y  25
(y  5) (y  5)

6z

5a2  5  a  a
7a  7  a

6
5x

 12w2  12w2

1. Sample answer: To find the LCD, determine the


least common multiple of all of the factors of the
denominators.
2. Sample answer: Multiply both the numerator and
denominator by factors necessary to form the
LCD.

4.

2y  y2  10y  25
(y  5) (y  5)

 4w  6w2  2  4w  3w

Rational Expressions with Unlike


Denominators

3. Sample answer:

Page 681

y2  10y  25
(y  5) (y  5)

2y
(y  5) (y  5)


1 revolution
1 minute
 4125 or about 12,959 in.

12-7

y  5
 5

y

333 revolutions

y  5
 5

2y

 y2  25  y

10. The LCD is (a  3)(a  1).


a  2
6
a  2
6
a  1
 a  3  a2  4a  3  a  3  a  1
a2  4a  3

(5x  1)  (2x  1)
3x  2
5x  1  2x  1
3x  2
3x
3x  2

10. Circumference of record  2r  2


2 34 in.

y  5
 5

y

y2  5y  6
(y  4) (y  3)

2y  y2  5y  6
(y  4) (y  3)

y2  7y  6
(y  4) (y  3)

Practice and Apply

a2b  a  a  b
ab3  a  b  b  b
LCM  a  a  b  b  b  a2b3

y  3
 3

y

7xy  7  x  y
21x2y  3  7  x  x  y
LCM  3  7  x  x  y  21x2y
18. LCM  (x  4)(x  2)
19. LCM  (2n  5)(n  2)

30. The LCD is 3(x  3).

17.

5
3x  9




31. The LCD is 3(3m  2).


m
3m  2

21. p2  5p  6  (p  6)(p  1)
LCM  (p  6)(p  1)
23. The LCD is a3.
22. The LCD is x2.


5
x

2
a3

 x2  x  x

 9m

2
 6

5x
x2
3  5x
x2

7a
a3
2  7a
a3

 a3 





3
5  a

4
5m2





3
5m
4

 5m2
7m 5m
15m
28
 35m2
35m2
15m  28
35m2

x

4
 4

3




n

3
 3

18
y2  9

7
7

n
 4

n  3
 3

n

3
 3

n

n2  3n  3n  12
(n  4) (n  3)

n2  12
(n  4) (n  3)
a  2
 2

 a  2  a  5a

x
x2  2x  1

7a2  14a
(a  5) (a  2)

7a2  14a  a2  5a
(a  5) (a  2)

8a2  9a
(a  5) (a  2)

x

x
 1

x

6x
 3

x  1
 1

x

x

x
 1

6x2  6x
(x  3) (x  1)

6x2  6x  x2  3x
(x  3) (x  1)

7x2  3x
(x  3) (x  1)

x

1
 1

 x2  2x

 1
x  x  1
(x  1) 2
2x  1
(x  1) 2




1
x  1

 1 x  1
x  1
(x  1) 2

x

 1

35. The LCD is (x  4)(x  1)2.

n  4
 4

n

2x  1
x  2
 x2  3x  4
(x  1) 2
2x  1 x  4
x  2
 (x  1) 2  x  4  (x  4) (x  1)

a  5

2x2  9x  4
(x  1) 2 (x  4)

2x2  9x  4  x2  3x  2
(x  1) 2 (x  4)

3x2  6x  6
(x  1) 2 (x  4)

x  1
 1

x

x2  3x  2
(x  1) 2 (x  4)

36. The LCD is (2x  3)(2x  3)2.

a2  5a
(a  5) (a  2)

7y  21
(y  3) (y  3)

 x2  2x

 a  2a  5

y  3
 3

y

34. The LCD is (x  1)2.

3n  12
(n  4) (n  3)

7
 3

7y  39
(y  3) (y  3)

x2
4x2  9

x
(2x  3) 2

29. The LCD is (x  3)(x  1).


6x
x  3

7
 3)

18

18

n2  3n
(n  4) (n  3)

7a

 y2  9  (y

 y2  9 

7
 y

18

3
x  4
4
x  5

 x  4x  5
 5 x  4
3x  12
4x  20
 (x  4) (x  5)
(x  5) (x  4)
3x  12  4x  20
(x  5) (x  4)
7x  8
(x  5) (x  4)

n

3

 y2  9  y

28. The LCD is (a  5)(a  2).


7a
a  5

a  5

33. The LCD is (y  3)(y  3).

27. The LCD is (n  4)(n  3).


n
n  4

3a  15  5
(a  5) (a  5)
3a  20
(a  5) (a  5)

x


 a2  25  (a  5)  a2  25
3

26. The LCD is (x  5)(x  4).


3
x  5

 3(3m  2)

 a  5  a  5  a2  25

7
5 2a
 3a  2a
6a2
7
10a
 6a2
6a2
7  10a
6a2

25. The LCD is 35m2.


3
7m

32. The LCD is (a  5)(a  5).

24. The LCD is 6a2.


5
3a

 9m  6

 x2 

m
3

 2 3
3m
3(3m  2)
3m  2
3(3m  2)
1
3

 3m


 a2  a3  a2  a

7
6a2

5
3
3
 x  33
 9
5
9
 3x  9
3x  9
5  9
3(x  3)
14
3(x  3)

 3x


20. x2  5x  14  (x  7)(x  2)
(x  2) 2  (x  2)(x  2)
LCM  (x  7)(x  2)(x  2) or (x  7)(x  2)2

3
x2

3
 3

x

x  3
 3

x

x2  3x
(x  3) (x  1)

x2
 9

 4x2

2x  3
 3

 2x

x
(2x  3) 2

2x3  3x2
(4x2  9) (2x  3)

2x3  3x2  2x2  3x


(2x  3) (2x  3) 2

2x3  5x2  3x
(2x  3) (2x  3) 2

2x  3
 3

 2x

2x2  3x
(2x  3) 2 (2x  3)

37. The LCD is (a  b)(a  b)2.


a2
a2  b2

569

a
(a  b) 2

a2

a  b

 a2  b2  a  b 

a
(a  b) 2

a3  a2b
(a2  b2 ) (a  b)

a3  a2b  a2  ab
(a  b) 2 (a  b)

a  b

a  b

a2  ab
(a  b) 2 (a  b)

Chapter 12

46. The LCD is x(x  5).

38. The LCD is 6x2.


7
3x

3
6x2

2x
x2  5x

7(2x)
3
 6x2
3x(2x)
14x
3
 6x2
6x2
14x  3
6x2





3x
 5

x




5(5x)

4  25x
15x2

7x
6y

3
a  6

11x(2)
7x(y)
 6y(y)
3y2 (2)
22x
7xy
 6y2
6y2
22x  7xy
6y2





3a

 21x2 





5a(3x)
7x(3x)

6

 a2  6a 


n
5  n

3a

 21x2

15ax
3a
 21x2
21x2
15ax  3a
21x2
3(5ax  a)
3(7x2 )
5ax  a
7x2

x2  1
x  1

3
 n2  25
 5)
n(n  5)
3
 n2  25
(n  5) (n  5)

n2  5n  3
(n  5) (n  5)

 n2  25  1(n

49. The LCD is 3(a  2)(a  2).


3a  2
6  3a

a  2

3a  2
a  2
 a2  4
 2)
3a  2(a  2)
(a  2) (3)
 (a2  4) (3)
 3(a  2) (a  2)

 a2  4  3(a


3a2  8a  4
 2) (a  2)

(x2  1) (x  1)
(x  1) (x  1)

(x2  1) (x  1)
(x  1) (x  1)

x3  x2  x  1
(x  1) (x  1)

x3  x2  x  1
(x  1) (x  1)

x3  x2  x  1  x3  x2  x  1
(x  1) (x  1)

2x2  2x
(x  1) (x  1)

2x(x  1)
(x  1) (x  1)

3a2  11a  10
 2) (a  2)
(3a  5) (a  2)
3(a  2) (a  2)
3a  5
3(a  2)




50. The LCD is (x  2)2(x  1).


3x
x2  3x  2

3x  6

 x2  4x  4 

43. The LCD is (k  5)(k  3).


3

k(k  3)
(k  5) (k  3)

3(k  5)
(k  3) (k  5)

k2  3k  3k  15
(k  5) (k  3)

k2  6k  15
(k  5) (k  3)

k

2
 2

k(k  2)
(2k  1) (k  2)

2(2k  1)
(k  2) (2k  1)

k2  2k
(2k  1) (k  2)

4k  2
(k  2) (2k  1)

k2  2k  4k  2
(k  2) (2k  1)

k2  2k  2
(2k  1) (k  2)

 2m  5 

(m  1) (2m  5)
(m  1) (2m  5)

3x  6
(x  2) 2

3x(x  2)
(x  2) (x  1) (x  2)

3x2  6x
(x  2) 2 (x  1)

3x2  6x  3x2  3x  6
(x  2) 2 (x  1)

9x  6
(x  2) 2 (x  1)

4(m  1)
(2m  5) (m  1)

4m  4
(2m  5) (m  1)

5a2  5a
(a  4) (a  1) (a  1)

5a2  5a  a2  3a  4
(a  4) (a  1) (a  1)

4a2  2a  4
(a  4) (a  1) (a  1)

(3x  6) (x  1)
(x  2) 2 (x  1)

3x2  3x  6
(x  2) 2 (x  1)

a2  3a  4
(a  4) (a  1) (a  1)

52. The LCD is (x  3)(x  1).




x2  4x  5
(x  3) (x  1)

2m2  3m  5  4m  4
(m  1) (2m  5)

x2  4x  5  2x  6
(x  3) (x  1)

2m2  m  9
(m  1) (2m  5)

x2  2x  1
(x  3) (x  1)

(x  1) 2
(x  3) (x  1)
x  1
x  3

2m  3m  5
(m  1) (2m  5)




x2  4x  5
x2  2x  3

x

2
 1

Chapter 12

5a
a  1
 a2  1
a2  3a  4
5a
a  1
 (a  4) (a  1)  (a  1) (a  1)
5a(a  1)
(a  1) (a  4)
 (a  4) (a  1) (a  1)  (a  1) (a  1) (a  4)

45. The LCD is (m  1)(2m  5).


m  1
m  1

3x
(x  2) (x  1)

51. The LCD is (a  1)(a  1)(a  4).

44. The LCD is (2k  1)(k  2).


k
2k  1

 2)

 3(a

2x
 1

k3

 3(a  2) (a

3a2  8a  4  3a  6
3(a  2) (a  2)

x
k
k  5

3a  6

 3(a

42. The LCD is (x  1)(x  1).


x2  1
x  1

3(a)
6
 a2  6a
(a  6) (a)
3a  6
3a  6
or a(a  6)
a2  6a

48. The LCD is (n  5)(n  5).

41. The LCD is 21x2.


5a
7x

2x  3x2
x2  5x
x(2  3x)
x(x  5)
2  3x
x  5

47. The LCD is a(a  6).

40. The LCD is 6y2.


11x
3y2

3x2

2x

 3x  15x2  3x(5x)


3x(x)
(x  5) (x)

 x2  5x  x2  5x

39. The LCD is 15x2.


4
15x2

2x

 x2  5x 

570

2(x  3)
(x  1) (x  3)

60. D; a2  2ab  b2  (a  b)(a  b)


a2  b2  (a  b)(a  b)
The LCD is (a  b)(a  b)(a  b) or
(a  b)2 (a  b).
61. C; The LCD is (x  3)(x  2)2.

53. The LCD is (m  4)2(m  4).


m  4
m2  8m  16

m  4
m  4

m  4(m  4)
(m  4) 2 (m  4)

(m  4) (m  4) 2
(m  4) (m  4) 2

m2  8m  16
(m  4) 2 (m  4)

m3  12m2  48m  64
(m  4) 2 (m  4)

m2  8m  16  m3  12m2  48m  64
(m  4) 2 (m  4)

m3  11m2  56m  48


(m  4) 2 (m  4)

x  4
(2  x) 2

54. Find the LCM of 2, 3, and 4.


22
33
4  22
The LCM is 2  2  3 or 12.
The least number of students is 12.
55. Find the number of miles each girl walks in an
hour.
60 minutes
12 minutes
60 minutes
15 minutes
60 minutes
20 minutes

Maya :
Makalla :
Monya :

Page 683
62.
63.

 1 mile  5 miles

64.

1 bag
12 days

total 

b
y

x2  x  12
(x  2) 2 (x  3)

x2  7x  10
(x  3) (x  2) 2

x2  x  12  x2  7x  10
(x  3) (x  2) 2

6x  22
(x  3) (x  2) 2

 3
 1
5
3

2y
5
 (y  3)
 3
2y
5
y3
y  3
2y  5
y  3

y

(b  10) (b  2)
b  2

 4t3
() t3




t2  4t  3
 19t  9

0t2
4t2

The quotient is t2  4t  3  t

3
.
 4

2m  3
67. 2m  74m2  8m  19
() 4m2  14m
6m  19
() 6m  21
2

x
7

 16  1.4875

The quotient is 2m  3  2m

2
.
 7

68. 2x2  10x  8  (2x2  8x)  (2x  8)


 2x(x  4)  2(x  4)
 (2x  2)(x  4)
 2(x  1)(x  4)
69. 5r2  7r  6  5r2  10r  3r  6
 5r(r  2)  3(r  2)
 (5r  3) (r  2)

 xy  yx


(x  5) (x  2)
(x  3) (x  2) (x  2)

4t2  19t
() 4t2  16t
3t  9
() 3t  12
3

x
 7 days

Therefore, she must buy 2 bags for one week.


57. Find the LCM of 3000, 6000, and 15,000.
3000  2  2  2  3  5  5  5
6000  2  2  2  2  3  5  5  5
15,000  2  2  2  3  5  5  5  5
The LCM is 2  2  2  2  3  5  5  5  5  30,000
Therefore, the cars odometer would read
36,000  30,000 or 66,000 miles.
58. Sample answer: This method will always work.
a
x

b  8b  20
b  2

66. t

7  16x
7
15

 b  10

7  15x

7
12

x

7
16

5
 y

7
x
15
1 bag
x

16 days days

third dog :

3

 7 days

1 bag
15 days

second dog :

2y
y  3

 1 mile  3 miles

7  12x
7
12

(x  4) (x  3)
(x  2) 2 (x  3)

Maintain Your Skills

65.

3m
3
3m
 2m  1  2m
2m  1
5
4x 
4x
 2x  3  2x 
2x  3

 1 mile  4 miles

Total miles  5  4  3  12 miles


The amount of money raised is 12  $2.50 or $30.
56. first dog :

x  5
 6

 x2  x

70. 16p2  4pq  30q2


 2(8p2  2pq  15q2 )
 2(8p2  10pq  12pq  15q2 )
 2[ (2p(4p  5q)  3q(4p  5q) ]
 2(2p  3q)(4p  5q)
71. amount spent on entertainment
 0.05(1782  525  120  40)
 0.05(1097)
 $54.85

ay
bx
 yx
xy
ay  bx
xy

59. Sample answer: You can use rational expressions


and their least common denominators to
determine when elections will coincide. Answers
should include the following.
Use each factor of the denominators the
greatest number of times it appears.
2012

571

Chapter 12

72.

x
2

3x
5

 2  3x

73.

a2
5b

a2

4a

 10b2  5b 

 2  3x
1

 5b 

a2

5b2
2a

 5b 
1


74.

x  7
x

6

x  7
 3

x

x  7
x

x  7
x

10b2
4a

9.

75.

3n
2n  5


12n2
 5

 2n

 2n

5b
2a

ab
2

10. Average 

2n  5
12n2

77.

x2  7x  12
x  6

3x
 2

x

3x
(x  2) (x  1)

 (x  3) 

x

1
 3

x

1
 3

11. 8 

(x  4) (x  3 )
x  6


15. 2m 

Mixed Expressions and


Complex Fractions

2
3

16. 3a 

Check for Understanding

4  m
m

a  1
2a

5
6

3(x)
4
3  x x
3x  4
 x
a  1
a  1
 2a  3a
3a

7
2
19
4

7
2

7 4

2 19
14
19

Chapter 12

2z
w

14. 6z 




2m(m)
m

6z(w)
2z
w
w
6wz  2z
w

4  m
m

6a2  a  1
2a
 b
 b

b2a  b3  a  b
a  b

r2 (r  3)
r  3

or

b3  b2a  a  b
a  b

r
18. r2  r

 4
 3

r  4
 3

19. 5n2 

r3  3r2  r  4
r  3
n  3
n  3
2

5n
n2  9
(n  3) (n  3)

r

4
x

4(a)
5
a
a
4a  5
a

3. Bolton; Lian omitted the factor (x  1).

7.

12. 4  a 

2m2  4  m
2m2  m  4
or
m
m
3a(2a)
a  1
 2a
2a

1
32
3
44

Practice and Apply

2x(y)
x
y
y
2xy  x
y

5

6.

204
20

a  b
b2 (a  b)
a
17. b2  a  b  a  b  a

 35

4.

165
20

1. Sample answer: Both mixed numbers and mixed


expressions are made up by the sum of an integer
or a monomial and a fraction or rational
expression.
2. Sample answer:
2
3
5
6

8(n)
3
n
n
8n  3
n

13. 2x  y 

x  4
 6

Pages 686687

3
n

x

12-8

Pages 687688

(x  4) (x  3)
x  6

130
20

729 1

20 5
29
7100 min

1
 1

90
20

4n

729
20

5
140
20


2n  5
12n2

7  42  62  84  105

a  b
x  y

(a  b) (a  b)
(x  y) (x  y)
1
(a  b) (a  b )
(x  y ) (x  y)

 4n
 (x  1)  x

x  y
1
x  y

x  3
x  7

3x
x  2

a2  b2

 a  b

x  3
 7

3n
 5

76.

x  y

 a  b  x2  y2
 a  b

x

3n
 5

 2n

x2  y2
a2  b2

x  3
x

x  y
a  b

5. 7 




7(6y)
5
 6y
6y
42y  5
6y

s  1

20. 3s2  s2  1 


2a(3a)
3a

a  1  6a2
3a

6a2  a  1
3a

n

1
 3

5n3  15n2  1
n  3
s  1
3s2  (s  1) (s  1)
3s2 (s  1)
s  1

s

1
 1

3s3  3s2  1
s  1
x  2
(x  5) (x  3)

x  3
x  3

21. (x  5) 

8.
19
4

5
6y

5n2 (n  3)
n  3

x3
y2
y3
x


x3

 y2 


x3
y2

x4
y5

y3
x

x  2
 3

x

x2  8x  15  x  2
x  3

x2  7x  17
x  3
p  1
(p  4) (p  4)

p  4
p  4

22. ( p  4) 

x
 y3

572

p  1

p4

p2  16  p  1
p  4

p2  p  15
p  4

23.

54
2

73

23
4
23
3
23
4

23
4

 23

24.

87
4

45

23
3

25.

a2
b

a2
b

a
b3

b
 a2

1
a

1
b

b2

26.

n3
m2
n2
m2

 b3  a2

x2
y2

n3
m2
n
n3
m2

m2
 n2
1
m2
 n2

27.

n3
m2

 ab2
x  4
y  2

145
84

x

x2

s
t2
s  t
s  t

x2  x  20
x  1
x2  2x  15
x  2

x2  x  20
x  1




(x  3) (x  1)
(x  2) (x  4)

34.

35
 12
63
n  2

nn
n

n(n  12)
n  12
n(n  2)
n  2

35
 12
63
n  2

n


n2  12n  35
n  12

s3
t2

s

 x2

y2

s3
t2

s

s  t
 t

n2  12n  35
n  12

y2 (x  4)
 2)

s3 (s  t)
t2 (s  t)




n2  12n  35
n  2
 n2  2n  63
n  12
1
(n  7 ) (n  5)
n  2
 (n  9) (n  7 )
n  12

(n  5) (n  2)
(n  12) (n  9)

x  4
 2

 y2

x  4
 2

y

y2  1
y2  3y  4

28.

y1

y2  1
 4

 y2  3y

s  t
 t

y  1
1

y
a2  2a  3
a2  1

a3

35.

aa

a  2a  3
a2  1

a2  2a  3
1
a  3
a2  1
1
1
(a  3 ) (a  1 )
1

(a  1) (a  1 ) a  3

a  3
1

n2  5n
n2  n  30

36.

n  2n
 18

 n2  9n



n  5n
 30

 n2  n

n2  2n
n2  n  30
 n2  5n
n2  9n  18
1
1
1
n (n  2)
(n  6 ) (n  5 )

(n  6 ) (n  3)
n (n  5 )
1

n  2
 3

x2  3x  28
x2  10x  24

b(b)
b
a(a)
a

b
1

a

b2  1
b
a2  1
a

b2  1
b

a2  1
a

b2  1
b

 a2  1

2b2
5c

4b3
2c
7b3
8c2

2b2
5c

2b2
5c

2b2
5c

32b2
35

14b2c

x2  3x  28
x2  10x  24

7b3
8c2

4c
2

1
16c
7

37. number of servings 

x2  4x  21
x2  9x  18

14b2c  7b8c 2
4

n
x2  4x  21
x2  9x  18

n2  2n  63
n  2

a(b2  1)

 b(a2  1)

a1
n2  2n
n2  9n  18

bb

1
 4
2

n2  2n  63
n  2

y2  1
1

 4 y  1
1
1
(y  1 ) (y  1 )
1

(y  4) (y  1 ) y  1

32.

x2  2x  15
x  2

 y2  3y

31.

15
 2
20
 1)

x2  2x  15
x  1
 x2  x  20
x  2
1
(x  5 ) (x  3)
x  1
 (x  5 ) (x  4)
x  2

n2
m2

x(x  2)
x
x  2
x(x  1)
 (x
(x  1)

 x2 (y

30.

15
 2
20
x  1

xx

n

y

29.

33.

12

a
b3

4
a
b3

58
7
24
5
58
24
 5
7
29
58 5

7 24

5  66 ounces
1
52

ounces

 330 
30

x2  4x  21 x2  10x  24

 18 x2  3x  28
1
1
1
1
(x  7 ) (x  3 ) (x  6 ) (x  4 )

(x  6 ) (x  3 ) (x  7 ) (x  4 )

 30  2
 60 servings

11
2

 330  11

330

11
2

 x2  9x
1

1

573

Chapter 12

38. h 






a

v
s

1

f
s
s

v
s

b.

a

1
3

a
b

fs
 v
370  760
760  65
281,200
695

4.799  1012
4,231,604,706

c.

4,651,208,508
4,231,604,706

1
2

 2a

b1


4.794  1012

 4,231,604,706


 1133 people per square mile


41. Let P  30 lb/in2 and V 

2
13

12

30 

5
3

153 2
k




(30)

50  k

Let k  50 and V  4 ft3. Solve for P.


P

12

50
3
4

P  50 
4

P

200
3
2
66 3

lb/in2

42. Simplify all expressions.


a.

a
1

3
a

a
3
a

1




a
a
a


a

3
a




a
3
a

a
a

a  3
3  a
a
a
a
a  3
a
 a  1
1

1
2 1  3 a a 2
a
a
a
a
 11  a  3 2  11  3  a 2


Chapter 12

a

a2
 3

3a

a

a2
 3

 (a  3)

3
b

3a  1
3




3a
3

3
b

3a  1
3

1
2

3a  1
3

b
1

3a  1
3a  1
 3b
3b
3a  1  3a  1
3b
2
0
3b

2a  2
1b

4a
2

b1

1  4a
2

b1




4a
2

2

1b

4a  1
2

1b

1  4a
2

1  4a
4a  1
 2  2b
2b  2
1  4a
4a  1
 (2b  2)
2b  2
1  4a
4a  1
 2b  2
2b  2
1  4a  4a  1
2b  2
8a  2
2b  2
2(4a  1)
2(b  1)
4a  1
0
b  1

Therefore, expression a is equivalent to 0.


43. Sample answer: Most measurements used in
baking are fractions or mixed numbers, which are
examples of rational expressions. Answers should
include the following.
You want to find the number of batches of
cookies you can make using the 7 cups of flour
you have on hand when a batch requires
1
12 cups of flour.
Divide the expression in the numerator of a
complex fraction by the expression in the
denominator.

3
4

P  50  3
P

3a
3

4a  1
 b  12  1 2  1  b2
1
1  4a
1
4a  1
1
 1 2  b  12  1 2  1  b2

ft3. Solve for k.

PV
k
30  2
13

5
(30)
3

a2  a2
a  3
0
a  3

3a  1
b
 2  1 3  12
1
3a  1 1
3a  1 1
 1 3  b2  1 3  b2

population of New Jersey


population of Alaska
 land area of Alaska
land area of New Jersey
8,414,350
626,932
 7419  570,374

 404.60 cycles /s
40.

1
3

hs


a2

a3

0

f
s  v
s
s  v
f s
s
fs  v
fs
s  v

39. Let s  760, v  65, and f  370.




a2
 3

a2

a2

574

44. C; Set up an equation.


P  4BC 
12  4BC 
6n
12  n  8
12(n  8)
6n
n8
n  8
12n  96
6n
n8
n  8
12n  96  6n
n  8
6n  96
n  8
6n  96 1
4
n  8
6n  96
4n  32

45. C;

6mn
5p
24n2
20mp

3n
2 n8
6n
n  8

 4BC

7
x2

51.

x
(x  3) 2

52.

2
t2  t  2

53.

2n
n2  2n  24

 x2 


 4BC

 20mp

6mn
5p

20mp
24n2

6mn
5p

20mp
24n2

4m2
4n

m2
n

4m

4n

p2 

Check:

36x

8(2y)
6y(2y)




16y

36x  16y
12y2
4(9x  4y)
4(3y2 )
9x  4y
3y2

57.

47. The LCD is (a  b)(2b  3a).


a
a  b

 2b

b
 3a

a(2b  3a)
(a  b) (2b  3a)

2ab  3a2  ab  b2
(a  b) (2b  3a)

3a2  3ab  b2
(a  b) (2b  3a)

b(a  b)
(a  b) (2b  3a)

48. The LCD is (3a  2)(a  4)2.

a2  a  12
(3a  2) (a  4) 2

a2  a  12  6a2  4a
(3a  2) (a  4) 2

7a2  3a  12
(3a  2) (a  4) 2

n  5
 6

 n2  n

9p2  64

183 22  64
64 ?
9 1 9 2  64
?

or

1 8 22  64
64 ?
9 1 9 2  64

9 3

64  64
64  64
z3  9z  45  5z2
z3  5z2  9z  45  0
3  5z2 )  (9z  45)  0
(z
z2 (z  5)  9(z  5)  0
(z  5)(z2  9)  0
or z2  9  0
z50
 5
z2  9
z  3
The solution set is {5, 3, 3}.
z3  9z  45  5z2
?

5  9(5)  45  5(5) 2
?
125  45  45  125
80  80
or
3
z  9z  45  5z2

6a2  4a
(3a  2) (a  4) 2

33  9(3)  45  5(3) 2
?
27  27  45  45
00
or

49. The LCD is (n  2)2(n  3).


n  4
(n  2) 2

Check:

a  3
2a
 a2  8a  16
3a2  10a  8
(a  3) (a  4)
2a(3a  2)
 (3a2  10a  8) (a  4)  (a2  8a  16) (3a  2)

 t2  t

64
9

 12y2  12y2


2  t
 2
2  t
 (t  2) (t  1)
(t  2)
 (t  2) (t  1)
1
 t  1
8
2n  8
 n2  2n  24
n2  2n  24
2(n  4)
 (n  6) (n  4)
2
n6
 2

64
9

46. The LCD is 12y2.


12x(3)
4y2 (3)

1
 3

p  3 or 3

Maintain Your Skills


8
6y

x

55. s2  16
s  116
s  4 or 4
s2  16
Check:
42  16
42  16
56. 9p2  64

p  3

12x
4y2

x  3
(x  3) 2

54. The answer is 100 multiplied by 2 ten times.


1000  210  1,024,000 bacteria.

24n2

6mn
5p

Page 689

 BC
 BC

 t2  t

 4BC
 4BC

 x2

3
(x  3) 2

 4BC

7  3
x2

50.

(n  4) (n  3)
(n  2) 2 (n  3)

(n  5) (n  2)
(n2  n  6) (n  2)

n2  n  12
(n  2) 2 (n  3)

n2  7n  10
(n  2) 2 (n  3)

n2  n  12  n2  7n  10
(n  2) 2 (n  3)

2n2  8n  2
(n  2) 2 (n  3)

z3  9z  45  5z2
?
3  9(3)  45  5(3) 2
?
27  27  45  45
00
3

575

Chapter 12

58. 160,140
53,310
27,990
22,980
18,120
15,750
11,190
10,800
59.

 16014  105
 5.331  104
 2.799  104
 2.298  104
 1.812  104
 1.575  104
 1.119  104
 1.08  104

amount spent on food


amount spent on clothing

12-9 Solving Rational Equations


Page 694

2.

amount spent on housing


total amount spent

Sample answer:

4.83  3.39
24  15
1.44
9

12  4

6.
10

4(12)  4
48  x
3
n
4

65.
3
n
4

14x 2

3
n
4
4 3
n
3 4

1 2

k(k  1)

3.2 
(7) (3.2) 

3n  (4)
6
3n  4
(6) 6

2  x(6)

x  1  x  4  6x
2x  5  6x
5  4x
x

k1

x  2
x  2

x

2
 2

 3

1xx  22  x 2 2 2  3(x  2) (x  2) 173 2

3(x  2) 2  6(x  2)  (x2  4) (7)


 4x  4)  6x  12  7x2  28
2
3x  12x  12  6x  12  7x2  28
3x2  6x  24  7x2  28
10x2  6x  4  0
2(5x2  3x  2)  0
(5x  2) (x  1)  0

x2  4

5x  2  0
5x  2

or

x10
x  1

x5
10.

32  b
128  b

 0.300

32  b  0.3(128  b)
32  b  38.4  0.3b
0.7b  6.4
b  9.14
Omar needs 10 more hits.

 9
 (6)(9)

50

 3
50

n  3

Chapter 12

3(x2

3n  4  54
3n  4  4  54  4
3n  50
3n
3

6

2  k(k  1) 1k 1 1 2

3(x  2) (x  2)

22.4  8  n
22.4  8  8  n  8
14.4  n
68.

5
7
k
k  1
5
7
k
k  1

9.

x  2

7x
10

x  1
x  4
 x
x
x  1
x  4
 x
x

4
(12)
3
8  n
7
8  n
7 7

7.

3  k

n  16

67.

7x

 10

5k  7(k  1)  k
5k  7k  7  k
2k  7  k
7  3k

66. 7x2  28

 12

a3

7(a  3)  5(a  1)
7a  21  5a  5
2a  26
a  13

2  101 2

8.

2.4  g

3393

3x
3
2
5
3x
3
2
5

7
a  1

5.

5
4

1.8  0.6  g  0.6  0.6

39

0

3
x  1

6x  15  7x
15  x

1.8  g  0.6

64.

x
4

2(x  1)  3x
2x  2  3x
x  2
x2

 0.16
Find the equation of the line using (15, 3.39).
y  mx  b
3.39  0.16(15)  b
3.39  2.4  b
0.99  b
Let y  C and x  m.
A linear equation is C  0.16m  0.99
62. Let m  9.
C  0.16(9)  0.99
C  1.44  0.99
C  2.43
The cost of a 9-minute call is $2.43.
63.

2
x

4.

 0.3328 or 33.3%
61. Find the slope of the line containing points
(15, 3.39) and (24, 4.83).
m

1

5.331  10
 105

 1.6014

n
3

3. The solution of a rational equation can never be


zero.

2.799  104
1.08  104

 2.59  100
60.

Check for Understanding

1. Sample answer: When you solve the equation,


n  1. But n  1, so the equation has no solution.

576

Pages 694695
4
a

11.

Practice and Apply


3

a2

4(a  2)  3(a)
4a  8  3a
a8
x  3
x

13.

3
x

12.

x

1
 2

p5

3(x  2)  x
3x  6  x
2x  6
x3

x  3
 6

x

21.

15(a  2)

x

6
x  4
3
x  2
2n
1
2n  3
2 6
3
2n
1
2n  3
2 6 6
3

4n  3  2n  3
2n  6

22.

16.

12

5
3y
 2
4
5
3y
 2
4

17.
(a  1) (a  1)

(4  c)

7y
6

2  121 2
15

a  1
a  1

2a
 1

x(x 





19.
(2x  3) (2x  3)

4x
2x  3

2x
2x  3

2x
2x  3

6
4  c
6
4  c
6
 c)

c
c

2  (4  c)(c)

2b  5
b  2

2b

3
 2

12bb  25  22  (b  2)(b  2) 1b 3 2 2

2b2  b  10  2b2  8  3b  6
b  2  3b  6
4b  4
b1
7
k  3

24.

4
x
4
x
4
x

2(k  3) (k  4)

2k

3
 4

1k 7 3  12 2  2(k  3) (k  4) 1k 3 4 2

14(k  4)  (k  3) (k  4)  6(k  3)
14k  56  (k2  7k  12)  6k  18

7  3x  4x  20
7x  27
4x
2x  3

c

c  4
c

(4  c)
c
 (4
(4  c)

(b  2)(2b  5)  2(b  2)(b  2)  (b  2)(3)

2  x(x  5) 14x 2
4
7  3x  (x2  5x) 1 x 2
x

5a  3a  6(a  2)
2a  6a  12
4a  12
a  3

(b  2)(b  2)

 (a  1) (a  1) (1)

(a  1) (a  1)  2a(a  1)  (a2  1) (1)


a2  2a  1  2a2  2a  a2  1
a2  4a  1  a2  1
4a  0
a0
7
3
5x
x2  5x
7
3
 (x  5)
x(x  5)
7
3
 (x  5)
x(x  5)
7
3
5) x(x  5)  (x  5)

5

2  25 (15(a  2) )

23.

2a

 a  1  1

a

5

y  4

a  1
a  1

a
a
 5a  10
3a  6
a
a
 5(a  2)
3(a  2)
a
a
 5(a  2)
3(a  2)

c  6  4c  c2
c  5c  6  0
(c  6)(c  1)  0
c  6  0 or c  1  0
c6
c1

7y
6

15  18y  14y
4y  15

n  3

18.

55  p2  8p  40
p  8p  15  0
p2  8p  15  0
Since p2  8p  15 does not factor, there are no
solutions.

x(x  1)  (x  6)(x  1)
x2  x  x2  5x  6
4x  6

15.

1 p 55 5  p p 5 2  ( p  5) (8)

x  6
 1

x
x  1

p2

 p  5  8

(x  3)(x  6)  (x  3)(x)
x2  9x  18  x2  3x
6x  18  0
6x  18
x3
14.

55
p  5

20.

k50
k5

27
7

25.

1

2  (2x  3) (2x  3)

k2  21k  68  6k  18
k2  15k  50  0
k2  15k  50  0
(k  5) (k  10)  0

x2  4
x  2
(x  2) (x  2)
x  2

or

k  10  0
k  10

 x2  4
 x2  4

x  2  x2  4
x2  x  2  0
(x  2)(x  1)  0
or x  1  0
x20
x  2
x1

4x2

(2x  3) (4x)  (2x  3) (2x) 


9
8x2  12x  4x2  6x  4x2  9
4x2  18x  4x2  9
18x  9
1

x2

577

Chapter 12

2n
n  1

26.
(n  1)(n  1)

n  5
 1

 n2

31. Find the distance for Jim.


d  rt

1

1n 2n 1  (n n1)(n5  1) 2  (n  1) (n  1)

d

(n  1) (2n)  n  5  (n  1) (n  1)
2n2  2n  n  5  n2  1
n2  3n  4  0
(n  4) (n  1)  0

Find the distance for Mateo.


d  rt
d

or n  1  0
n40
n  4
n1
The number 1 is an extraneous solution, since 1 is
an excluded value for n. Thus, 4 is the solution
of the equation.
27.
(z  4) (z  1)

3z
z2  5z  4
3z
(z  4) (z  1)

2
z  4

1803 2  21.82  0.82 mi


1101 2  21.82  2.2 mi

They will be 0.82 mile from the nearest shore.


32. R 
1

1 car
2 hr

112  12  13 2x  7
136  36  26 2x  7

3
z  1

8
x
6

3z  2(z  1)  3(z  4)
3z  2z  2  3z  12

x

14  2z
7z

(m  2) (m  6)

m

m
 2

x

m6

1 (m  2)4(m  6) 2  (m  2)(m  6) 1m m 2  m 1 6 2

x

4  m2  6m  m  2
0  m2  5m  6
0  (m  6)(m  1)
m10

or

m6

34. 600 ft3 

m  1

The number 6 is an extraneous solution, since 6 is


an excluded value for m. Thus, 1 is the solution
of the equation.
29. Let x  number of quizzes.
36  10x
5  x

35.

9

x2  x  2
x  5

x1
x

20

1
 1

20

(x  3) (x  2)
(x  2) (x  5)

20

(x  3) (x  2)
(x  2) (x  5)

 2  (x  2) (x  5) (0)

(x  3) (x  2)  2(x  2) (x  5)  0
x2  5x  6  2x2  6x  20  0
3x2  11x  14  0
(3x  14) (x  1)  0
3x  14  0
or x  1  0
3x  14
x1
14

x  3

r

The number 1 is an extraneous solution. Thus,


14
 3 is the only solution.
36. Sample answer: Rational equations are used in
solving rate problems, so they can be used to
determine traveling times, speeds, and distances
related to subways. Answers should include the
following.
Sample answer: Since both trains leave at the
same time, their traveling time is the same.
The sum of the distances of both trains is
equal to the total distance between the two
stations. So, add the two expressions to
represent the distance each train travels and
solve for time.

r
r

3
1
t  10 t
80
3
1
t  10 t
80

3

2  80(3)

3t  8t  240
11t  240
t  21.82
They will meet in about 22 minutes.

Chapter 12

(x  2) (x  1)
x  5

(x  2) (x  5)

Let t  time in minutes.

80

 4500 gal.

x  3
x  2
x  3
x  2

Find the rate for Mateo.


d  rt
3 mi  r  30 min
3 mi
30 min
1 mi
10 min

7.5 gal.
1ft3

It will take 4500 gallons to fill the pool.

36  10x  9(5  x)
36  10x  45  9x
x9
She must score 10 points on 9 quizzes to reach
her goal.
30. Find the rate for Jim.
d  rt
3 mi  r  80 min
3 mi
80 min

42
8
2
58
1
54

168 2

It will take them 5 hours and 15 minutes to clean


7 cars.
33. V  /  w  h
V  15 ft  10 ft  4 ft
 600 ft3

4  (m  6)(m)  (m  2)

m60

7

x  (7)

3z  5z  14

28.

1 car
3 hr

Let x  the number of hours.

2  (z  4)(z  1) 1z 2 4  z 3 1 2

4
m2  8m  12

and R2 

578

37. A;

a  2
a

a  3

a6a

a(a  6)

1a a 2  aa  36 2  a(a  6) 1a1 2

41.

2
 5
6
x  1

x2x
x6

1
n  2

n2  7n  8
 8

 3n2  2n

x2  7x  12
x  5

x2  7x  12
(x  5)

 x2  7x

3
2m  3

 6  4m 



1, 2, 3, 4, 6, 8, 12, 24


1

y
y2  2y  1

x2  8x  15
x2  x  6

(x  5) (x  3)
(x  3) (x  2)

(x  5 ) (x  3 )
(x  3 ) (x  2)

y

1
 1

a  2
a2  9

2a

 6a2  17a  3 

(x  3) (x  1)
 5) (x  3)
(x  3 ) (x  1)
 5 ) (x  3 )
1

x  1
 2

x
40.

a2  6a  5
a2  13a  42
2

a  4a  3
a2  3a  18

a  6a  5

a  4a  3

 a2  13a  42  a2  3a  18


(a  5) (a  1)
(a  6) (a  7)

(a  5) (a  1 )
(a  6 ) (a  7)

(a  5)
(a  7)

(a  6) (a  3)
 3) (a  1)

 (a

(a  6 ) (a  3 )
 3 ) (a  1 )

 (a

a  2
(a  3) (a  3)

2a
(a  3) (6a  1)

(a  2) (6a  1)
(a  3) (a  3) (6a  1)

2a(a  3)
(a  3) (6a  1) (a  3)

6a2  13a  2
(a  3) (a  3) (6a  1)

2a2  6a
(a  3) (a  3) (6a  1)

6a2  13a  2  2a2  6a


(a  3) (a  3) (6a  1)

4a2  7a  2
(a  3) (a  3) (6a  1)

45. 20x  8y  4(5x  2y)


46. 14a2b  21ab2  7ab(2a  3b)
47. 10p2  12p  25p  30  2p(5p  6)  5(5p  6)
 (2p  5)(5p  6)
48. Write two equations.
1. 0.5x  0.3y  0.45(100)
2. x  y  100
Solve for x in equation 1 and plug it into
equation 2.
0.5(100  y)  0.3y  45
50  0.5y  0.3y  45
0.2y  5
y  25
x  25  100
x  75
Therefore, there are 25 gallons of the 30% glycol
solution and 75 gallons of the 50% glycol solution.

y
y  1
 ( y  1) 2
( y  1) 2
y  y  1
( y  1) 2
1
1
or y2  2y  1
(y  1) 2

44. The LCD is (a  3)(a  3)(6a  1).

 (x
 (x




x2  2x  15
x2  2x  3

(x  1)
 12

3(2)
m
 2(2m  3)
(2m  3) (2)
6
m
 2(2m  3)
2(2m  3)
6  m
2(2m  3)

Maintain Your Skills

x2  2x  15
x2  2x  3

x2  7x  12
x  1

43. The LCD is (y  1)2.

Since n  2 is an extraneous solution, n  6 is


the solution.

39.

x2  7x  6  6
x  1

42. The LCD is 2(2m  3).

Therefore, n3  2n2  20n  24  (n  2)(n2  4n  12)


 (n  2)(n  6) (n  2)
n20
or n  6  0
n  2
n6

x2  8x  15
x2  x  6

x  7x  10  2
x  5

x  1
 5

Since (2)3  2(2)2  20(2)  24  0, 2 is a


solution.
By synthetic division,
2 1 2 20 24
2
8
24
1 4 12
0

Page 695

The possible factors of this polynomial must be




2
 5
6
x  1

x

x

1(3n2  2n  8)  (n  2) (n2  7n  8)
3n2  2n  8  n3  7n2  8n  2n2  14n  16
3n2  2n  8  n3  5n2  22n  16
3
n  2n2  20n  24  0
factors of 24
factors of 1

(x  2) (x  5)
x  5
(x  6) (x  1)
x  1
2

(a  2)(a  6)  a(a  3)  a  6
a2  8a  12  a2  3a  a  6
5a  12  a  6
6a  18
a3
38. D;

Chapter 12 Study Guide and Review


Page 696

Vocabulary and Concept Check

1. false, rational
3. true
5. false, x2  144

579

2. true
4. false,
6. true

5a  20
a  3

Chapter 12

Pages 696700

Lesson-by-Lesson Review

x1y1  xy
42  28  xy
1176  xy
1176  56y
21  y
9. x1y1  xy

8. x1y1  xy
5  15  xy
75  xy
75  3y
25  y
10. x1  y1  xy
175  35  xy
6125  xy
6125  75y
81.67  y

7.

8  18  xy
144  xy
144  x(3)
48  x

11.

x 1

3x2y
12xy3z

3x2y

 12x y3z

12.

4 y2

n2  3n
n  3

13.

a  25
a2  3a  10

14.

x  10x  21
x3  x2  42x

15.

7b2
9

6a2
b

6a2
b

7b2
9

14a2b
3

16.

25.

3a  6
a2  9

a  3

 a2  2a 


5x2y
8ab

19.

x2  x  12
x  2

26.

x  4
 6




20.

b2  19b  84
b  3

b2  9
 b2  15b  36

12a2b
25x

xy

5x2y
8ab

3axy
10

3a

12a2b
25x
5

10
 10(x  10 )

 (x

a  3
 2)

 a(a

3
a2  3a

22.

p3
2q

p2

p3

 (x

x  4
 3 ) (x  2)

29.

m  1
5




(x  4) 2
(x  2) 2
(b  7) (b  12 )
b  3

m  4
5

30.

(b  3 ) (b  3 )
(b  12 ) (b  3 )
1

5
2n  5

2n
 5

 2n

y2
y  4

 2q  p2

(a  b ) (a  b)
a  b

 2p

ab

y

y2
 4

3y

y2  16
3y

(y  4 ) (y  4)
3y

a2
a  b

b2
a  b

32.

7a
b2

5a
b2

7a  5a
b2

2a
b2

33.

2x
x  3

x

6
 3

3c2
b

2x  6
x  3

2(x  3 )
x  3

2

y(y  4)
3

 ( y2  6y  8) 

3y  12
y  4

34.

 y2  6y

 8

3(y  4 )
y  4
1

Chapter 12

6abc
2ab2

y2
 4

3y  12
y  4

 y2  16  y

4a
3 2

8a b c
2ab2

5  2n
2n  5
2n  5
2n  5

a2  b2
a  b

1
31.

23.

4q

6abc2
2ab2

m  4  m  1
5
2m  3
5

4q

 4q  2q  p2
p3

4a2b2c2
2ab2

8a3b2c
2ab2

x2  4x  2
x  3x3  7x2  10x  6
() x3  3x2
4x2  10x
() 4x2  12x
2x  6
() 2x  6
0
The quotient is x2  4x  2.

b7

21.

The quotient is 4b  1  12b  1.

(x  4) (x  3 )
x  2

4a2b2c2
2ab2

x2  2x  3
 0x2  7x  6
x
 2x2
2x2  7x
() 2x2  4x
3x  6
3x  6
0
The quotient is x2  2x  3.
28.
4b  1
12b  148b2  8b  7
() 48b2  4b
12b  7
() 12b  1
8

 x2  x

 2ac2  4a2c 

3(a  2 )
(a  3) (a  3 )

or

(4a2b2c2  8a3b2c  6abc2 )  2ab2 

2a

30
 10

3
a(a  3)

3m  2
 2) (3m  2)

 2x3
() x3

18.

 (3m

27.

3(x  10 )
1

x

3m  2
 4

 9m2

(2m  3) (m  5 )
m  5

10

17. (3x  30)  x2  100 

2m2  7m  15
m  5

(x  3) (x  7)
 42)
(x  3) (x  7)
x(x  7) (x  6)
x  3
x(x  6)

2m  3
 2

 x(x2  x


9m2  4
3m  2

 3m

(a  5) (a  5)
(a  5) (a  2)
a  5
a  2


2

n


2m2  7m  15
m  5

n(n  3)
n  3

 4y2z
2

24.

 (y

1
 4) (y  2)

m2
m  n

2mn  n2
m  n

m2  2mn  n2
m  n

(m  n) 2
m  n

mn

3
(y  4) (y  2)

580

35. The LCD is 6cd2.


2c
3d2

3
2cd

2c(2c)
3d2 (2c)

4c

44.

3(3d)
2cd(3d)

9d

x2
y3

3x
9y2

 6cd2  6cd2

3r
 3

r

r2  21r
(r  3) (r  3)

r2  21r  3r2  9r
(r  3) (r  3)

4r2  12r
(r  3) (r  3)

3x
9y2

31

x2

9y2
3x

36. The LCD is r 2  9.


r2  21r
r2  9

 y3 

4c2  9d
6cd2

x2
y3

45.

3x
y






46.

4r
 3

y9y

6
 4

y4y

2
 1

37. The LCD is (a  2) (a  1 ).


3a
a  2

a1

3a(a  1)
(a  2) (a  1)

5a(a  2)
(a  2) (a  1)

3a2  3a
(a  2) (a  1)

5a2  10a
(a  2) (a  1)

5a

8a  7a
(a  2) (a  1)

2.

38. The LCD is 21.




9n
7






39. The LCD is 6a

7n(7)
9n(3)
 7(3)
3(7)
49n
27n
 21
21
49n  27n
21
22n
21

7
3a

7(2a)

14a
3
 6a2
6a2
14a  3
6a2

4x
3

47.

40. The LCD is 5(x  4).


2x
2x  8

 5x

4
 20




x
 2

41. 4  x





12

2x
4
 5x  20
 4)
x
4
 5x  20
x  4
x(5)
4
 5x  20
(x  4) (5)
5x
4
 5x  20
5x  20
5x  4
5x  20

 2(x





43. 3 

x  y
x2  y2

9(y  4)
y  4

y

6
 4

y(y  1)
y  1

4(y  1)
y  1

y

2
 1

y2  4y  9y  36  6
y  4
y2  y  4y  4  2
y  1
y2  13y  30
y  4
y2  5y  6
y  1

48.
6x
3

49.

3x  3y  x  y
x2  y2

4x2  2y2
x2  y2

 (y

y  1
 3 ) (y  2)

(y  10) (y  1)
(y  4) (y  2)

or

y2  11y  10
y2  6y  8

7x

12 (7)
2

12 (7x)
12

12 (1)
4
1

2  6x116 2

6

6x (2)
3x
1

6x (1)
6
1

2
3r
r2
3r
2
3r
(3r)(r  2) 3r  r  2
(3r) (r  2) (2)
(3r) (r  2) (3r)

3r
r  2

 3

2  (3r)(r  2) (3)
 (3r)(r  2) (3)

2r  4  9r  9r2  18r
16r  4
4

r  16

x2  y2
x2  y2
2

(y  10) (y  3 )
y  4

33  4  x
29  x

y2  5y  6
y  1

 2  12  4

11
2
 3x
2x
11
2
 3x
2x

6x (11)
2x

1
2x2
2(x  2)
1
x2
x  2
2x  4  1
x  2
2x  5
x  2

16x  42  7x  3
9x  45
x  5

x  2
(x  2 ) (x  2)

3(x2  y2 )
x2  y2

y2  13y  30
y  4

14x3  72 2  121127x  14 2

x  2

12 (4x)
3

4(x  2)
x
x2
x  2
4x  8  x
x  2
5x  8
x  2

42. 2  x2  4  2 

5a  4
4a  6
 8
a
5a  4
8
 4a  6
a
5a  4
4
 2a  3
a
20a  16
2a2  3a

 6a2  3a(2a)  6a2




5a
4
a
a
a(4)
3(2)
 2(4)
2(4)
5a  4
a
4a  6
8

y(y  4)
y  4


7n
3

3a  3a  5a  10a
(a  2) (a  1)

4

3r(r  3)
(r  3) (r  3)

4r(r  3 )
(r  3) (r  3 )

r

a
2

11

5a

r  4

581

Chapter 12

50.
(x) (x  6)

x  2
x  3
x6
x
x  2
x  3
x6
x

1
x

8.

2  (x)(x  6) 11x 2

(x) (x  3)
3(x) (x  3)
x2  3x

3
x  2
x3
x2  3x
3
x  2
x3
x2  3x

The excluded values occur when c  7  0 or


3
c  7 and when 2c  3  0 or c  2.
9.

1
x

(n  4 ) (n  1) (1)
n  4

1
n  4
1
n  4

(x ) (x  3) (1)
(x )

n

n

1
 1

1
 1

(n  4) (n  1 ) (1)
n  1

 4

 (n  4) (n  1)
1

9
t

t2
81
 t2
t2
t  9
t




t  9
t

2
n2  3n  4

 (t

t2
 9) (t  9 )
1

t
t  9

The excluded values occur when t  0, t2  0,


t  9  0, and t  9  0.
or t  9  0
t  0 or t  9  0
t  9
t9

10.

(n  4 ) (n  1 ) (2)
(n  4 ) (n  1 )
1

 n2  3n

t
t

t2  81
t2
t  9
t2  81
 t2
t

3  x2  2x  x  3
x2  x  0
x(x  1)  0
x  0 or x  1  0
x  1
Since the value 0 gives a 0 in the denominator, 0
is an extraneous solution.
(n  4) (n  1)

1

9
t
81
t2

52.

1

2  (x)(x  3) 11x 2

(x) (x  3 ) (x  2)
x  3

(2c  3) 2
(2c  3) (c  7)
2c  3
c  7




(x  6) (x  2)  (x)(x  3)  x  6
x2  8x  12  x2  3x  x  6
5x  12  x  6
6x  18
x3
51.

4c2  12c  9
2c2  11c  21

5
u
t
6
2u
3
t

(5) (t)
6u
 6t
6t
2u
3t
 t
t
5t  6u
6t
2u  3t
t
5t  6u
2u  3t

6t
t

5t  6u
6t




(n  1)  (n  4)  2
5  2

There is no solution to this equation.

Chapter 12 Practice Test

5t  6u
 18t

The excluded values occur when t  0 and


12u  18t  0.
12u  18t  0
12u  18t

1. a; complex fraction
2. c; mixed expression
3. b; rational expression
4. xy  (40) (21)
5.
xy  (4)(22)
xy  840
xy  88
84y  840
x(16)  88
y  10
x  5.5

u
11.

5  2m
6m  15

5  2m

 3(5  2m )

5
 2
15
x  2

x4x
x6

 3
The excluded value occurs when 6m  15  0
5
or m  2.
3  x
2x2  5x  3

3  x
(2x  1) (x  3 )

(x  4) (x  2)  5
x  2
(x  6) (x  2)  15
x  2

(x  4) (x  2)  5
x  2

x2  2x  8  5
x  2

 x2  4x

x2  2x  3
x  2

x  2
 3

x2  2x  3
x2  4x  3

1
 1

The excluded values occur when 2x  1  0 and


x  3  0.
2x  1  0 or x  3  0
2x  1
x  3

12.

x2

2x
x  7

x

14
 7




2x  14
x  7
2(x  7)
x  7

2

Chapter 12

(x  6) (x  2)  15
x  2
x  2
 12  15

 x2  4x

The excluded values occur when x  2  0 and


x2  4x  3  0.
x  2  0 and
x2  4x  3  0
x2
(x  3)(x  1)  0
or x  1  0
x30
x  3
x  1

 2x

3t
2

7.

t
 3t

 12u

Page 701

6.

 2u

582

13.

n  3
2n  8

6n  24
2n  1

n  3
 4)

 2(n




14.

15.

(n  4)(n  5)

6n  18
4n  2
2(3n  9)
2(2n  1)
3n  9
2n  1

(n  4) (n  5) (2n)
n  4

x  3
 7x  12

 x2

z  2z  15
z2  9z  20

4n  56
n  14
3
x2  5x  6
3
(x  3) (x  2)

23.

(x  3)(x  2)

 (x

x  3
 4 ) (x  3 )
1

z  2z  15
1

x70
x7

z  3

4x2  11x  6
x2  x  6

x2  x  12
 16

18.

(10z4

5z3

z2 )

5z3

(4x  3) (x  2 )
(x  3 ) (x  2 )
1

10z4
5z3

10z4
5z3
1

5z3
5z3

6
6  3y

(x  4 ) (x  3 )
 4 ) (x  4)

 (x

z2
5z




y2  2y  14y  28
7( y  2) ( y  2)




15

1
5z

y
6
 3(y  2)
7(y  2)
y
2
y2
7(y  2)
y(y  2)
2(7) (y  2)
 7(y  2) (y  2)
7(y  2) (y  2)


21.

x2  1
x  1

A2

1x 36 y 21x

A

 y2
12

1x 36 y 21 (x  y12) (x  y) 2
3

3(x  y)
2

or 2 (x  y)

Chapter 12 Standardized Test Practice

(x2  1) (x  1)
(x  1) (x  1)

x3  x2  x  1
(x  1) (x  1)

x3  x2  x  1
(x  1) (x  1)

x3  x2  x  1  x3  x2  x  1
(x  1) (x  1)

2x2  2x
(x  1) (x  1)
2x(x  1)
(x  1) (x  1)
2x
x  1

or 18

A2

(x2  1) (x  1)
(x  1) (x  1)

15
8

25. B; Area of triangle  2bh

x  5
6(x  2)
 x2
 2
x  5  6x  12
x  2
7x  17
x  2

x2  1
x  1

2  15(1)

3t  5t  15
8t  15

6x


1

It will take them 18 hours to rake a lawn.

y2  16y  28
 2) (y  2)

x  5
x  2

1
1
t  3t
5
1
1
t  3t
5

t

 7(y
20.

1 lawn
.
3 hr

Let t  time for 1 lawn.

z2
5z3

1 lawn
.
5 hr

The rate Kalyn can rake a lawn is

5z3
1

or

24. The rate Scott can rake a lawn is

 5z3  5z3

 2z  1 
19.

 5x  14  0
(x  7) (x  2)  0
x20
x  2

4x  3
x  4

2z

y
7y  14

(1  x) (x  3) (x  2)
x  2

Since the value 2 gives a 0 in the denominator,


2 is an extraneous solution.

 x2  8x

x2

(z  5 ) (z  3 )
(z  5 ) (z  4)

1  x

3  7x  14  x  3  x2  3x

x2  8x  16
x2  x  12

 x  2

3  7(x  2)  (1  x)(x  3)

 (z  3)  z2  9z  20  z  3

7(x  3) (x  2)
x  3

z4
4x2  11x  6
x2  x  6

x  1

 x  2

7x  11  x2  2x  3

17.

7
 3
7
x  3

x

1 (x  3)3(x  2)  x 7 3 2  (x  3)(x  2) 11x  2x 2

3(x  3) (x  2)
(x  3) (x  2)

(x  8) (x  4 )
x  5

4(n  4) (n  5)
n  5

8n  40  4n  16

x  8
 5

16.

1n 2n 4  22  (n  4)(n  5) 1n 4 5 2

 2(n  4)(n  5) 

x
2

4
 5

2n2  10n  2n2  2n  40  4n  16

x  4x  32
x  5

2n

(n  5)(2n)  2(n  4)(n  5)  4(n  4)

5m  12
2m  310m2  9m  36
() 10m2  15m
24m  36
()24m  36
0
The quotient is 5m  12.
2

2n
n  4

22.

6(n  4)
2n  1

Pages 702703
1. D; Volume of a cylinder is r2h.
V  (2.5) 2 (8)
 157
2. B; Find the equation of the line.
Find the slope using points (3, 10) and (5, 14).
m

14  10
5  (3)

24
8

 3

y  mx  b
10  (3)(3)  b
1b
y  3x  1

583

Chapter 12

10. Let w  width.


length  5  w
Area of playground    w
750  (5  w)w
750  5w  w2
2
w  5w  750  0
(w  30)(w  25)  0
or w  25  0
w  30  0
w  30
w  25
The width is 25 meters since measurements must
be positive. Therefore, the length is 5  25 or
30 meters.
11. Find the discriminant.
b2  4ac  (24) 2  4(16)(9)
 576  576
0
Since the discriminant is 0, the equation has one
real root.

3. B; The graph contains the points (0,8) and


(10, 0).
Find the slope.
0  (8)
10  0

 10  5

y  mx  b
4

8  5 (0)  b
8  b
4
x
5

y

5
4. A;

4
y  5x
4
y  5x

8

 8

2  5(8)

5y  4x  40 or 4x  5y  40
y  mx  b

1 12

5  4 (12)  b
53b
2b

12.

y  4x  2

x2  9
x3  x

x

3x
 3

4y  x  8 or x  4y  8
5. D; Find the equation of the line. Find the slope
using points (1, 0) and (0, 2)
m

2
1

13.

 2

3(x  3)
x2  1

3x  9
x2  1

x
x  4

x

3x
 3

x

3x
 3

 x2  16  x

4x

x
 4

x

x
 4


14. B; Plug x 
1
x2  2x

1
y2  2y
1
8


7

12

1 2
4

1
4

x2  16
4x

(x  4 ) (x  4)
4x

x  4
4

or

1
x
4

1

and y  4 into both expressions.

1
2

12
1
4

1
(4) 2  2(4)
16
7

1
1
16

2

1
1
16

 16

1
7
16

16
7

 16  8  8

15. C; Simplify both expressions.


1500  120  1180  1720
 15  100  14  5  136  5  1144  5
 1015  215  615  1215
 215
1125  145
 125  5  19  5
 515  315
 215
215  215
16. A; Find the excluded value of both expressions.
32a  0
b60
a0
b  6
0 7 6

100

 144(1  0.5) 20
 144(0.5) 5
 4.5
9. Let x  miles driven on the third day.
 350

630  x  1050
x  420
They drove 420 miles on the third day.

Chapter 12

(x  3 ) (x  3)
x (x2  1)

y  mx  b
0  2(1)  b
2b
y  2x  2
Since the origin is not in the shaded area, the
inequality is y 2x  2.
6. B; Use elimination to solve the system of
equations.
3x  y  2
() 2x  y  8
5x
 10
x  2
Substitute 2 for x in either equation.
3(2)  y  2
y4
The solution is (2, 4).
7. B; Let w  width.
length  /  2.5 w
Area of rectangle  /  w
9750  (2.5 w)w
9750  2.5 w2
8. C; y  C(1  r) t

360  270  x
3

4 y  4x  4(2)

2  0
0  1

(x  3) (x  3)
x(x2  1)
1

y  4x  2
1

584

18a. The height increases.


18b. Sample answer: As the distance between the
bottom of the ladder and the base of the
building decreases, the height that the ladder
reaches increases.
18c. No; Sample answer: When the bottom of the
ladder is 6 feet from the base of the building, it
reaches a height of about 10.4 feet. When the
bottom of the ladder is 4 feet from the base of the
building, it reaches a height of about 11.3 feet. In
order to form an inverse variation 6  10.4 must
approximately equal 4  11.3. However,
6  10.4  63.6 and 4  11.3  45.2. Because the
products are not equal, this relationship does not
form an inverse variation.

17. D; Simplify both expressions.


5

3x
x  1

5(x  1)
3(x)
x1
x  1
5x  5  3x
x  1
8x  5
x  1

24y  15
3

3 (8y  5)
3

8y  5
y  1




24y  15
3
6y  6
6

6y  6
6

 6( y
1

6
 1)

Since x and y can be any number, the relationship


cannot be determined.

585

Chapter 12

PQ249-6481F-13[586-609] 26/9/02 6:37 PM Page 586 Sahuja Ahuja_QXP_06:Desktop Folder:Chandra:Algebra_FNL_Delivery:

Chapter 13
Page 707

Statistics
not be representative of the voting population.
Since readers voluntarily write letters, this is a
voluntary response sample.
5. The sample is the work from 4 students. The
population is work from all students in the 1st
period math class. The sample is a biased sample
because students who raise their hands first may
be expected to do better work. Since the students
volunteer to have their work shown at the open
house, this is a voluntary sample.
6. The sample is 25 nails. The population is all nails
on the store shelves. This is a stratified random
sample. Each of the nails was randomly selected
from each of the 25 boxes.
7. The sample is 12 pencils. The population is all
pencils in the school store. This sample is a
biased sample, since all of the pencils came from
the same box. Since Namid grabbed the closest
box of pencils and 12 pencils from the top of the
box, it is a convenience sample.

Getting Started

1. Sample answer: If a  5 and b  2, then c  3.


However, 5  3.
2. Sample answer: It could be a yellow rose.
3. Sample answer: The speed limit could be 55 mph,
and Tara could be driving 50 mph.
4. Sample answer: 6 is even, but not divisible by 4.
5. Order the set: 1, 7, 9, 15, 25, 59, 63
Find the middle value: 1, 7, 9, 15 , 25, 59, 63
The median is 15.
6. Order the set: 0, 2, 2, 2, 3, 3, 4, 5, 7, 8, 8, 9, 10
Find the middle value:
0, 2, 2, 2, 3, 3, 4 , 5, 7, 8, 8, 9, 10
The median is 4.
7. Order the set:
211, 218, 235, 317, 355, 395, 407, 407, 411, 726
Find the two middle values:
211, 218, 235, 317, 355 , 395 , 407, 407, 411, 726
Average the two middle values:

355  395
2

 375

The median is 375.

Pages 711713

8.
7 8 9 10 11 12 13 14 15

9.
15 16 17 18 19 20 21 22 23

10.
3

11.

13-1 Sampling and Bias


Page 710

Check for Understanding

1. All three are unbiased samples. However, the


methods for selecting each type of sample are
different. In a simple random sample, a sample is
as likely to be chosen as any other from the
population. In a stratified random sample, the
population is first divided into similar,
nonoverlapping groups. Then a simple random
sample is selected from each group. In a
systematic random sample, the items are selected
according to a specified time or item interval.
2. A convenience sample is a biased sample that is
determined based on the ease with which it is
possible to gather the sample. A voluntary sample
is a biased sample composed of voluntary
responses.
3. Sample answer: Ask the members of the schools
football team to name their favorite sport.
4. The sample is a group of readers of the
newspaper. The population is all readers of the
newspaper. The sample is a biased sample
because readers of a particular newspaper may

Chapter 13

Practice and Apply

8. The sample is 3 sophomores. The population is all


sophomores in the school. The sample was
randomly selected from all sophomores in the
school, so the sample is unbiased. This is a simple
random sample, since each name was equally
likely to be chosen.
9. The sample is 20 shoppers. The population is all
shoppers. Since shoppers at a fast-food restaurant
are more likely to name the cola sold in the
restaurant as their preference, this sample is
biased. Since only shoppers outside the fast-food
restaurant were selected for the sample, it is a
convenience sample.
10. The sample is people who are home between
9 A.M. and 4 P.M. The population is all people in
the neighborhood. Since people who are home
during the weekdays may tend to behave
similarly, the sample is biased. Since people were
interviewed only during the typical work day, it is
a convenience sample.
11. The sample is 860 people from a state. The
population is all people in a state. The people are
chosen at random, so the sample is unbiased.
Since the people were selected by county, which is
a sub-division of a state, it is a stratified random
sample.
12. The sample is 10 scooters. The population is all
scooters manufactured on the particular
production line during one day. The sample is
biased because samples were collected only on one
day, and it is possible that more manufacturing
mistakes occur early in the morning and late in
the afternoon. Since the scooters were selected
early at the beginning of the day and at the end
of the day, the sample is a convenience sample.

586

PQ249-6481F-13[586-609] 26/9/02 6:37 PM Page 587 Sahuja Ahuja_QXP_06:Desktop Folder:Chandra:Algebra_FNL_Delivery:

22. We know that the results are from a national


survey conducted by Yankelovich Partners for
Microsoft Corporation.
23. Additional information needed includes how the
survey was conducted, how the survey
respondents were selected, and the number of
respondents.
24. Sample answer: Get a copy of the schools list of
students and call every 10th person on the list.
25. Sample answer: Get a copy of the list of registered
voters in the city and call every 100th person.
26. The graph shows four phrases with a percent
associated with each phrase. We can assume that
the percents indicate the percent of respondents
who said the indicated topic was discussed during
family dinners. Based on the sum of the percents,
respondents must have been able to choose or
state more than one topic. We do not know how
many respondents there were, whether the
respondents selected topics from a list of choices
or stated their own topics, whether there were
any restrictions that may have existed about the
topics, and the time period of the family dinners
considered in this survey (a night, a week, a
month, or more).
27. Sample answer: Randomly pick 5 rows from each
field of tomatoes and then pick a tomato every 50 ft along each row.
28. Sample answer: Every hour pull one infant seat
from the end of the assembly line for testing.
29. It is a good idea to divide the school population
into groups and to take a simple random sample
from each group. The problem that prevents this
from being a legitimate stratified random sample
is the way the three groups are formed. The three
groups probably do not represent all students.
The students who do not participate in any of
these three activities will not be represented in
the survey. Other students may be involved in
two or three of these activities. These students
will be more likely to be chosen for the survey.
30. Usually it is impossible for a company to test
every item coming off its production lines.
Therefore, testing a sample of these items is
helpful in determining quality control. Answers
should include the following.
Sample answer: An unbiased way to pick the
CDs to be checked is to take every 25th CD off
the production line.
Sample answer: A biased way to pick the CDs
to be checked is to take the first 5 CDs coming
off the production line in the morning.
31. B; The most accurate result would be achieved
through an unbiased, random sample of the
population in question. Choices A, C, and D result
in biased samples. Therefore, the correct answer
is B, which samples the appropriate population.
32. D; The most accurate result would be achieved
through an unbiased, random sample. Choices A,
B, and C result in biased samples. Therefore, the
correct answer is D.

13. The sample is 3 students. The population is all


students in Ms. Finches class. The sample is
unbiased since each students number was equally
likely to be selected. For the same reason, the
sample is a simple random sample.
14. The sample is an 8-oz jar of corn. The population
is all corn in the storage silo. The sample is
biased because it was selected from corn near or
at the surface of all corn in the silo, which is
unlikely to represent all the corn in the storage
silo. Since only corn that was easily accessible
was sampled, the sample is a convenience sample.
15. The sample is a group of U.S. district court
judges. The population is all U.S. district court
judges. Since every 20th number is selected, the
sample is unbiased. Since the population was
divided into 11 federal districts before the sample
was selected, the sample is a stratified random
sample.
16. The sample is a group of people who watch a
television station. The population is all people
who watch a television station. The sample is
biased because people who watch a particular
television station may be more likely to have
similar viewpoints about building the golf course.
Since viewers decide whether to call the 1900
number, the sample is a voluntary sample.
17. The sample is 4 U.S. Senators. The population is
all U.S. Senators. The sample is biased because
the Presidents four closest colleagues are more
likely to be members of his political party and
share his political viewpoints. Since the President
discussed issues with people who are easily
accessible, the sample is a convenience sample.
18. The sample is a handful of Bing cherries. The
population is all Bing cherries in the produce
department. The sample is biased since the
cherries were not randomly selected. Since the
manager selected cherries at the edge of one case,
this sample is a convenience sample.
19. The sample is a group of high-definition television
sets. The population is all high-definition
television sets manufactured on one line on
one shift. The sample is unbiased since the sets
were selected randomly. Since every 15th set was
selected, the sample is a systematic random
sample.
20. The sample is a group of employees. The population
is all employees of the company. The sample is
unbiased, because employees were selected at
random. Since one employee was selected from
each department, the sample is a stratified random
sample.
21. The sample is a group of readers of a magazine.
The population is all readers of the magazine. The
sample is biased because the readers of a
particular magazine may be more likely to select a
particular actor as their favorite. Since the
readers mailed their inputs to the magazines
office, the sample is a voluntary sample.

587

Chapter 13

PQ249-6481F-13[586-609] 26/9/02 6:37 PM Page 588 Sahuja Ahuja_QXP_06:Desktop Folder:Chandra:Algebra_FNL_Delivery:

Page 713
33.

Maintain Your Skills




41.
6b2

1
4

2  12y114 2
4
6
3 1
10
5
(12y ) 1 3y 2  (12y ) 1 2y 2  12y 1 4 2
12y

10
5
 2y
3y
10
5
 2y
3y

6b2  15  19b
 19b  15  0
b



40  30  3y
10  3y
y

34.
r(r  4)
1

r(r  4) (3)
r  4

3
1
r
r  4
3
1
r
r  4

1
r

42.

r (r  4)
r

4m(m  3)
1

4m (m  3)
4m

1
2m
m3
4m
1
2m
m3
4m

44. (c  3)(c  7)  c2  7c  3c  21
 c2  10c  21
45. (x  4)(x  8)  x2  8x  4x  32
 x2  4x  32
46. 4.5  3.8  8.3
47. 16.9  7.21  24.11
48. 3.6  18.5  22.1
49. 7.6  3.8  3.8
50. 18  4.7  13.3
51. 13.2  0.75  12.45

4m(m  3)(2)

(m  3)  8m2  8m(m  3)
8m2  m  3  8m2  24m
25m  3
3
m  25
36.

2x  5
x
2x  5
6

2x  5
x

2x  5
x
6
x

 2x


37.

a(a  12)  35
a  12
a  7

38.

t2

a2  12a  35
a  12

(a  5) (a  7)
(a  12) (a  7)
a  5
a  12

t  2
 3

t

a  7

1
 2

1
 3

39. 516  3154  4124  516  319  6  414  6


 516  916  816
 2216 cm
40. x2  6x  40  0
x



Reading Mathematics

1a. This question will bias people toward answering


yes because it gives them a reason to think
that recycling will help alleviate a shortage in
resources.
1b. This question will bias people toward answering
no because most citizens are against the
government making laws that require certain
behaviors.
2a. This question is not biased. It does not influence
a person to answer one way or the other.
2b. This question will bias people toward answering
no because most people do not want taxes to
be raised.
3a. Sample answer: Since we had hamburgers at
the last party, would you prefer pizza for the
next party?
3b. Sample answer: Would you prefer hamburgers
or pizza for the class party?

6
 5

t
t

Page 714

2x  5
6


(t  2) (t  2)
(t  3) (t  2)

9  1105
4

 4.8, 0.3

2

4m(m  3) (2m)

m  3

(9)  2(9) 2  4(2) (3)


2(2)

43. (y  5)(y  7)  y2  7y  5y  35
 y2  12y  35

2  4m(m  3)(2)

d

3r  (r  4)  r  4
2r  4  r  4
r8
35.

19  11
12
19  1
12
20 18
, 12
12
2
1
13, 12

2d  9d  3
2d2  9d  3  0

r (r  4)
r


1
33

2  r(r  4) 11r 2

10
3

(19)  2(19) 2  4(6) (15)


2(6)

(6)  2(6) 2  4(1) (40)


2(1)

13-2 Introduction to Matrices

6  1196
2
6  14
2

Page 717718

 10, 4

Check for Understanding

1. A 2-by-4 matrix has 2 rows and 4 columns, and a


4-by-2 matrix has 4 rows and 2 columns.
2. Sample answer:
2 3 1
3
2
1 6 6
c
d  c
d
5 6 2 5
4 7 6 14

Chapter 13

588

PQ249-6481F-13[586-609] 26/9/02 6:37 PM Page 589 Sahuja Ahuja_QXP_06:Desktop Folder:Chandra:Algebra_FNL_Delivery:

3. Estrella; Hiroshi did not multiply each element of


the matrix by 5.
4. 3 by 3; first row, second column
5. 1 by 4; first row, first column
6. 4 by 1; third row, first column
7. 3 by 2; first row, second column
8. A is a 2 2 matrix, while C is a 1 2 matrix. To
add matrices, the matrices must have the same
dimension. Therefore, it is impossible to perform
A  C.
9. B  A  c

15 14
20 10
d  c
d
10
6
12
19

 c

15  20 14  (10)
d
10  12
6  19

 c

5
24
d
22 13

10. 2A  2 c
 c

0 0
26. 0 8
0 4
1
5
9
12
7 16
27. A  B  0 4 2 
5 10
13
3
7
6
20 11
8

2(10)
d
2(19)

40 20
d
24
38
11. 4C  4[5 7]
 [4(5) 4(7) ]
 [20 28]
12 10
F  11
8
14
8

3
13
8, R  1
10
8

12
5
11

11
11
8
6
10 , N  1
8 11
2
10 15 11

3
13 12 11
11
8  1
5 10  1
10
8 11
2
10

12  13  11
11  1  1
14  8  10

36 30
 13 21
32 34

10  12  8
858
8  11  15

8
6
8 11
15 11

3  11  6
8  10  11
10  2  11

17.
18.
19.
20.
21.
22.
23.
24.

2
3
3
2
3
4
2
2

25. c

by
by
by
by
by
by
by
by

2;
2;
1;
3;
3;
2;
3;
2;

34 91 63
52
9 70
d  c
d
81 79 60
49 8 45

 c

34  (52)
81  (49)

91  9 63  70
d
79  (8) 60  45

 c

18 100 133


d
32
71 105
34 91 63
52
9 70
d  c
d
81 79 60
49 8 45

 c

34  (52)
81  (49)

91  9 63  70
d
79  (8) 60  45

 c

86 82 7
d
130 87 15
12
7 16
1
5
9
5 10
13  0 4 2
20 11
8
3
7
6

12  (1)
75
16  9
5  0 10  (4) 13  (2)
20  3
11  7
86

11 2
5 14
17 4

25
15
2

1
5
9
31. 5A  5 0 4 2
3
2
6

20
29
23

5(1)
 5(0)
5(3)

15. The total sales for the weekend


16. The greatest number in the matrix is 36, which
represents small, thin crust pizzas. Therefore,
small, thin crust pizzas had the most sales over
the entire weekend.

Pages 718721

13 12 7
5
6 11
23 18 14

30. B  A 

13. No; the corresponding elements are not equal.


14. T  F  R  N
12 10
 11
8
14
8

29. C  D  c

 c

12.

1  (12)
5  7 9  (16)
0  5 4  10
2  13
3  20
7  11
68

28. C  D  c

20 10
d
12
19

2(20)
2(12)

5(5)
5(4)
5(2)

5(9)
5(2)
5(6)

5
25
45
 0 20 10
15
10
30
32. 2C  2 c

Practice and Apply

first row, first column


second row, first column
third row, first column
second row, second column
second row, third column
fourth row, first column
second row, third column
second row, first column

34 91 63
d
81 79 60

 c

2(34)
2(81)

 c

68 182 126
d
162 158 120

2(91)
2(79)

2(63)
d
2(60)

2 1 1
d
1 5 1

589

Chapter 13

PQ249-6481F-13[586-609] 26/9/02 6:37 PM Page 590 Sahuja Ahuja_QXP_06:Desktop Folder:Chandra:Algebra_FNL_Delivery:

41. 2F  V  S  C

1
5
9
34 91 63
d
33. A  C  0 4 2  c
81 79 60
3
7
6

 2[70 6 3 0.8]  [70 2 2 0.3]  [160 0 0 0]


 [185 2 11 3.9]
 [2(70)  2(6)  2(3)  2(0.8)]  [70 2 2 0.3]
 [160 0 0 0]  [185 2 11 3.9]
 [140  70  160  185 12  2  0  2 6  2  0
 11 1.6  0.3  0  3.9]
 [555 16 19 5.8]

Since the matrices do not have the same


dimension, it is impossible to add them.
34. B  D 

12
7 16
52
9 70
5 10
13  c
d
49 8 45
20 11
8

18
42. N  24
17

Since the matrices do not have the same


dimension, it is impossible to add them.
35. 2B  A  2


12 7 16
1
5
9
5 10
13  0 4 2
20 11
8
3
7
6

2(12)
2(5)
2(20)

2(16)
1
2(13)  0
2(8)
3

2(7)
2(10)
2(11)

43. 1.20
44.

5
9
4 2
7
6

4(1)
 4(0)
4(3)

12
7
5 10
20 11

45.

16
13
8

4  (12)
20  7 36  (16)
0  5 16  10
8  13
12  20
28  11
24  8

8
 5
8

37.

38.

2C  3D  2 c

34
81

13
52
26 21
17
16

2(34)
2(81)

 c

68
162

 c

68  (156)
162  (147)

 c

224 155 84


d
309 182 15

5D  2C  5 c

2(91)
2(79)
182
158

2(63)
3(52)
d  c
2(60)
3(49)

3(9)
3(8)

3(70)
d
3(45)

126
156
27 210
d  c
d
120
147 24 135
182  27 126  210
d
158  (24) 120  135

52
9 70
34 91
d  2c
49 8 45
81 79

 c

5(52)
5(49)

 c

260
45 350
68
d  c
225 40 225
162

 c

260  68
45  182
225  162 40  158

5(9)
5(8)

63
d
60

5(70)
2(34)
d  c
5(45)
2(81)
182
158

2(91)
2(79)

2(63)
d
2(60)

126
d
120

350  126
d
225  120

331
304
325
343

1.20(32)
1.20(45)
1.20(26)

1.20(24)
1.20(47)
1.20(30)

1.20(21)
1.20(25)
1.20(28)

38 29 25
54 56 30
31 36 34
4135
3840
4353
4436

26
24
41
36

15
571
14
473
T, B  D
13
347
15
533

4135
3840
4353
4436

26
24
41
36

15
571
14
473
TD
13
347
15
533

533  571
515  473
D
499  347
571  533

331  357
304  284
325  235
343  324

1104
988
D
846
1104

8548
7270
7782
8166

51. C; Since c

192 227 476


 c
d
63 118 345

39. V  [70 2 2 0.3], S  [160 0 0 0],


C  [185 2 11 3.9]
40. 2F  2[ 70 6 3 0.8]
 [2(70) 2(6) 2(3) 2(0.8) ]
 [ 140 12 6 1.6]
Chapter 13

331
304
325
343

1.20(28)
1.20(30)
1.20(19)

357
284
235
324

4413
3430
3429
3730

33
28
21
19

15
11
T
18
18

688
588
560
667

59
52
62
55

357
284
235
324

4413
3430
3429
3730

4135  4413 26  33
3840  3430 24  28
4353  3429 41  21
4436  3730 36  19

33
28
21
19

15
11
T
18
18

15  15
14  11
T
13  18
15  18

30
25
T
31
33

48. the total of the passing statistics for the 1999 and
2000 seasons
49a. sometimes
49b. always
49c. sometimes
49d. sometimes
49e. sometimes
49f. sometimes
50. Matrices can be used to organize data that can be
displayed in a rectangular array of numbers.
Answers should include the following:
A table is a rectangular array of numbers with
headings to indicate what each row and
column represents. A matrix is just the
rectangular array of numbers.
Sample answer: The grades from each of five
different tests for each student in a math class
can be organized in a matrix.

91 63
52
9 70
d
d  3c
79 60
49 8 45

 c

533
515
499
571

533
515
D
499
571

4
20 36
12
7 16
 0 16 8 
5 10
13
12
28 24
20 11
8


AD

21
25
28

46. 4 by 5, 4 by 5
47. T  A  B

4(9)
12
7 16
4(2) 
5 10
13
4(6)
20 11
8

4(5)
4(4)
4(7)

28 32 24
30 45 47
19 26 30

22 34
 29 36
20 23

25 19 23
 10 16
24
43 29
22
1

18
rN  1.20 24
17
1.20(18)
 1.20(24)
1.20(17)

24 14 32
1
5
9
 10 20
26  0 4 2
40 22
16
3
7
6

36. 4A  B  4 0 4 2 

28 32 24 21
30 45 47 25
19 26 30 28

590

1 3 5
2 7 0
d  c
d
7 2 0
13 1 8

 c

1  2 3  7 5  0
d
7  (13) 2  1 0  8

 c

3
6

4 5
d
1 8

PQ249-6481F-13[586-609] 26/9/02 6:37 PM Page 591 Sahuja Ahuja_QXP_06:Desktop Folder:Chandra:Algebra_FNL_Delivery:

65. a1  4, n  5, r  2

52. B; When adding two matrices, one must add each


of the corresponding elements. If M  N  M, the
0 0
d.
only way the equation is true is if N  c
0 0
53. c

0.7
1.6

17
54. c
12.1
55. c

5.3
2.4

0.4
4

2.3
d
2.4

66.
67.
68.

4.6
3.9
d
13.5 14.3
12.4
7.7

21.1
d
4

69.

14.22 9.72 12.24


56. c
d
10.62 7.92 13.86
57. c

3.92
3.12

0.48
2.04

Page 721

2.08
d
3.6

70.

Maintain Your Skills

58. The sample is 10 calendars.


59. Since the calendars examined each day are not
selected randomly, the sample is biased. The
sample is a convenience sample because the last
10 calendars printed each day are examined.
60. a  (a  1)

Page 721

1a 4 1  a3 2  a  (a  1)(1)

13x  x 4x 3 2  x  (x  3)(4)

3(x  3)  x(4x)  4x(x  3)


3x  9  4x2  4x2  12x
15x  9
9

x  15 or 5

62.

2(d  5) (d  9)

1dd  35  d 2 9 2  2(d  5) (d  9) 12d 5 10 2

2(d  3) (d  9)  2  2(d  5)  5(d  9)


2(d2  6d  27)  4(d  5)  5d  45
2d2  12d  54  4d  20  5d  45
2d2  13d  11  0
(2d  11) (d  1)  0
d

11
,
2

Practice Quiz 1

1. The sample is half of the households in a


neighborhood. The population is all households in
a neighborhood. Since each household is selected
randomly, the sample is unbiased. The sample is
a systematic random sample since every second
household is surveyed.
2. The sample is half of the households in a
neighborhood. Sample answer: voters in the state.
Since the sample was not randomly drawn from
all voters in the state, it is a biased sample. The
sample is a convenience sample because only
households in a particular neighborhood are
surveyed.
5 7
8
3
d  c
d
3. c
4 9
1
0

4a  3(a  1)  a  (a  1)
4a  3a  3  a2  a
a  3  a2  a
0  a2  2a  3
0  (a  3)(a  1)
a  3, 1
61. x  (x  3)

Since an  a1  r(n1)
a  4  (2) (51)
5
 4  (16)
 64
b2  7b  12  (b  3)(b  4)
a2  2ab 3b2  (a  b)(a  3b)
Since the trinomial d2  8d  15 cannot be
factored, it is prime.
Sample answer: Megan saved steadily from
January to June. In July, she withdrew money to
go on vacation. She started saving again in
September. Then in November, she withdrew
money for holiday presents.
In this graph, the domain is the months of the
year. The range is the amount of money in
Megans bank account.

4. c

d  5.5, 1

63. a1  4, n  5, r  3
Since an  a1  r(n1)

 c

8  5 3  (7)
d
4  (1)
9  0

 c

3 4
d
5 9

7 2 8
9 6 4
d  c
d
1 3 2
5 3 1
 c

9  7 6  (2)
1  5 3  (3)

 c

16 8 4
d
6 6
1

5. 3 c

(3) (51)

a5  (4) 
 4  81
 324
64. a1  2, n  3, r  7
Since an  a1  r(n1)

48
d
21

8 3 4
5
d
6 1
2 10

 c

3(8)
3(6)

 c

24 9 12 15
d
18 3
6 30

3(3)
3(1)

3(4)
3(2)

3(5)
d
3(10)

a3  (2)  (7) (31)


 (2)  49
 98

591

Chapter 13

PQ249-6481F-13[586-609] 26/9/02 6:37 PM Page 592 Sahuja Ahuja_QXP_06:Desktop Folder:Chandra:Algebra_FNL_Delivery:

12. For the men, the graph represents 5  6  4  2 


17 values. So, the median of the data is the 9th
value, which occurs in the range 220240
rebounds. For the women, the graph represents
1  2  5  5  1  1  15 values, so the median
of the data is the 8th value, which falls in the
range 240260 rebounds. The mens rebounding
data show a vast majority of values are between
200 and 240 rebounds. The womens rebounding
data are somewhat symmetrical. The top two
women have more rebounds than any of the men.
13. The graph representing age at inauguration has
8  24  9  41 values. The median of the data is
the 21st value, which occurs in the range
5060 years. The graph representing age at death
has 2  5  12  11  5  2  37 values. The
median of the data is the 19th value, which occurs
in the range 6070 years. Both distributions show
a symmetrical shape. The two distributions differ
in their spread. The inauguration ages are not
spread out as much as the death ages data.
14. Sample answer:

Histograms

13-3
Page 725

Check for Understanding

1. First identify the greatest and least values in the


data set. Use this information to determine
appropriate measurement classes. Using these
measurement classes, create a frequency table.
Then draw the histogram. Always remember to
label the axes and give the histogram a title.
2. 50
v 6 60
3. Sample answer:
1, 1, 2, 4, 5, 5, 8, 9, 10, 11, 12, 13, 22, 24, 41
4. $200250
5. There are no gaps. The data are somewhat
symmetrical.
6. In Group A, there are 31 values, so the median
value is the 16th. It falls in the 4045 range. In
Group B, there are 26 values, so the median value
is the average between the 13th and 14th values.
It also falls in the 4045 range.
7. The Group A test scores are somewhat more
symmetrical in appearance than the Group B test
scores. There are 25 of 31 scores in Group A that are
40 or greater, while only 14 of 26 scores in Group B
are 40 or greater. Also, Group B has 5 scores that
are less than 30. Therefore, we can conclude that
Group A performed better overall on the test.
8. Sample answer:

Semester Scoring
Average in Mathematics Class
Frequency

8
6
4
2
0
7075

8
6
4
2
0

75- 80- 85- 90- 9580 85 90 95 100


Weighted Average

15. Sample answer:

Raisins Counted
in Snack-Size Boxes
30- 40- 50- 60- 70- 8040 50 60 70 80 90
Passengers (millions)

Frequency

Frequency

Passenger Traffic at
U.S. Airports, 2000

9. B; The graph represents 32 employees, which


means that the median is the average of the 16th
and 17th values. Those values both fall in the
$3040 thousand range, so the median of the data
falls between $30,000 and $40,000. The statement
that is not correct is answer choice B.

14
12
10
8
6
4
2
0

50- 6060 70

70- 80- 90- 10080 90 100 110

Number of Raisins in a Box

Pages 726728

Practice and Apply

16. Sample answer:

10. The median occurs at the average of the 10th and


11th values. So it falls in the range 350600
thousand. There is a gap between 1100 and
1600 thousand. The histogram is highly skewed to
the left, with a vast majority of the data values in
the lowest measurement class. Over half of the
top 20 newspapers have circulations between 350
and 600 thousand.
11. The graph represents 8  5  3  4  3  1  24
values. So, the median of the data is the average
of the 12th and 13th values, so it is 34003800
championship points. There are no gaps. The data
appear to be skewed to the left.
Chapter 13

Frequency

Payrolls for Major League Baseball Teams


in 2000
12
10
8
6
4
2
0

median $54,000,000

10- 20- 30- 40- 50- 60- 70- 80- 90- 100- 11020 30 40 50 60 70 80 90 100 110 120
Team Payroll (millions of dollars)

592

PQ249-6481F-13[586-609] 26/9/02 6:37 PM Page 593 Sahuja Ahuja_QXP_06:Desktop Folder:Chandra:Algebra_FNL_Delivery:

27. Sample answer:

17. Sample answer: See graph above.


18. The data has 50 values, so the median is the
average between the 25th and 26th value in the
ordered set. The median, then, is
54.07  53.66
2

 53.865.
19. Sample answer:

Percent of Eligible Voters


Who Voted in the
2000 Presential Election

28. Sample answer:

Frequency

15
12
9
6
3
0
40- 45- 50- 55- 60- 6545 50 55 60 65 70

29. Sample answer:

Percent Eligible Voters Who Voted

Frequency

20. Sample answer: There are no gaps. The data are


somewhat symmetrical.
21. See students work.
22. Sample answer:
4

30. A is a 3 3 matrix. B is a 2 3 matrix. It is


impossible to add these matrices.

7 5 2
2
3 7
31. C  A  0
0 3  0 4 6
1
4 6
1 5 4

0
15- 20- 25- 30- 3520 25 30 35 40

4045

4550

5055

5560

6065

23. Histograms can be used to show how many states


have a median within various intervals. Answers
should include the following.
A histogram is more visual than a frequency
table and can show trends easily.

9 8 5
 0
4 3
2
9
2

Frequency

Year 2000 State Mean SAT


Mathematics Scores
20
15
10
5
0

7  (2)
5  3 2  7
0  0 0  (4) 3  6
1  1 4  (5) 6  4

32. 2B  2 c

480- 500- 520- 540- 560- 580- 600500 520 540 560 580 600 620
Score

8 1 1
d
2 3 7

 c

2(8)
2(2)

 c

16 2
4 6

2(1)
2(3)

2(1)
d
2(7)

2
d
14

2
3 7
33. 5A  5 0 4 6
1 5 4

24. C; According to the graph, the number of


employees represented in the graph is
9  12  11  6  6  2  46 employees.
25. B; The data has 46 values. The median is the
average of the 23rd and 24th values. Therefore,
the median falls in the range 46 days absent.
26. Sample answer:

5(2)
 5(0)
5(1)

5(3)
5(4)
5(5)

5(7)
5(6)
5(4)

10 15 35
 0
20 30
5
25 20
34. Since each CD player has an equal chance of
being selected, the sample is unbiased. The CD
players are selected at a regular time interval, so
the sample is a systematic random sample.

593

Chapter 13

PQ249-6481F-13[586-609] 26/9/02 6:37 PM Page 594 Sahuja Ahuja_QXP_06:Desktop Folder:Chandra:Algebra_FNL_Delivery:

35.

36.

s
s  7

s  5
 7

s

s

s
 7

s

s
 5

2m2  7m  15
m  2

Find the middle values:


1.6, 2.1, 3.901 , 5.21 , 7.4, 13.9
Average the values:

2m  3

 m2  5m  6

43. Order the set: 1.6, 2.1, 3.901, 5.21, 7.4, 13.9

s  7
 5

s

(m  2) (m  3)
2m  3
1

 (m  5)(m  3)

Page 730

37. 1y  3  5  9
1y  3  4
y  3  16
y  13
Check:

 4.56

The median is about 4.56.

(2m  3) (m  5)
m  2

3.901  5.21
2

Graphing Calculator Investigation


(Follow-Up of Lesson 13-3)

1. To make a scatterplot, enter the x values in L1


and the y values in L2. Use STAT PLOT to graph
the scatter plot. Use ZOOM

1(13)  3  5  9

9 to graph.

116  5  9
?

459
99
38. 1x  2  x  4
x  2  x2  8x  16
0  x2  9x  18
0  (x  6) (x  3)
x  6, 3
Check:

The graph appears to be an exponential regression.


To find the coefficient of determination, select

1(6)  2  (6)  4

ExpReg on the

14  6  4
22

STAT

KEYSTROKES: STAT

CALC menu.
0 ENTER

R2

 0.9969724389
2. To make a scatterplot, enter the x values in L1
and the y values in L2. Use STAT PLOT to graph

1(3)  2  (3)  4
?

11  1
1 1
x  3 is an extraneous solution.

the scatter plot. Use ZOOM

9 to graph.

39. 13  12w  5
169  2w  5
174  2w
87  w
Check:

13  12(87)  5
?

13  1174  5

The graph appears to be a linear regression.


To find the coefficient of determination, select

13  1169
13  13
40. Order the set: 2, 4, 7, 9, 12, 15
Find the middle values: 2, 4, 7 , 9 , 12, 15
Average the values:

7  9
2

16
2

LinReg(ax  b) on the
KEYSTROKES: STAT

8  10
2

CALC menu.

4 ENTER

R2  0.9389164209
3. To make a scatterplot, enter the x values in L1
and the y values in L2. Use STAT PLOT to graph

or 8

The median is 8.
41. Order the set: 1, 3, 6, 8, 10, 12, 15, 7
Find the middle values: 1, 3, 6, 8 , 10 , 12, 15, 17
Average the values:

STAT

the scatter plot. Use ZOOM 9 to graph.

9

The median is 9.
42. Order the set: 2, 4, 7, 7, 9, 19
Find the middle values: 2, 4, 7 , 7 , 9, 19
Average the values:

7  7
2

7

The median is 7.

The graph appears to be a linear regression.


To find the coefficient of determination, select
LinReg(ax  b) on the
KEYSTROKES: STAT

R2  0.9974802029

Chapter 13

594

STAT

CALC menu.

4 ENTER

PQ249-6481F-13[586-609] 26/9/02 6:37 PM Page 595 Sahuja Ahuja_QXP_06:Desktop Folder:Chandra:Algebra_FNL_Delivery:

4. To make a scatterplot, enter the x values in L1


and the y values in L2. Use STAT PLOT to graph

13-4 Measures of Variation

the scatter plot. Use ZOOM 9 to graph.

Pages 733734

The graph appears to be a quadratic regression.


To find the coefficient of determination, select
QuadReg on the

STAT

KEYSTROKES: STAT

Check for Understanding

1. Sample answer:
1, 4, 5, 6, 7, 8, 15 and 1, 2, 4, 5, 9, 9, 10
2. Outliers may make the mean much greater or
much less than the mean of the data excluding
the outliers.
3. Alonso; the range is the difference between the
greatest and the least values of the set.
4. Range: 85 25  60
median

CALC menu.

678

5 ENTER

25 58 59 62 67 69 69 73 75 76 77 77 82 85
64748
c
c
6973
Q2  2  71 Q3
Q1

R2  0.97716799
5. Enter the year in L1 and the cost in L2. Use
STAT PLOT to graph the scatter plot. Use

d Q  Q  IQR  77  62  15 S
3

ZOOM 9 to graph.

To find the outliers:


Q1  1.5(IQR)  62  1.5(15)
 39.5
Q3  1.5(IQR)  77  1.5(15)
 99.5
The data set consists of one value which is less
than 39.5 or greater than 99.5. Therefore, 25 is
the only outlier.
5. Range: 11.9  7.3  4.6

6. Find the coefficient of determination for linear,


quadratic, and exponential regression. Choose the
equation whose coefficient of determination is
closest to 1.
Linear: Select LinReg(ax  b) on the STAT CALC
menu.
R2  0.9312567132
Quadratic: Select QuadReg on the STAT CALC
menu.
R2  0.9880773362
Exponential: Select ExpReg on the STAT CALC
menu.
R2  0.9624472328
The value of R2 is closest to 1 for the quadratic
regression. The regression equation is

8.7  9.4
 9.05
2
10.0  10.1
Q :
 10.05
2
3
8.0  8.0
Q1:
 8.0
2

Median:

IQR  Q  Q
3
1

 10.05  8.0
 2.05
To find the outliers:
Q1  1.5(IQR)  8  1.5(2.05)
 4.925
Q3  1.5(IQR)  10.05  1.5(2.05)
 13.125
The data set has no values less than 4.925 or
greater than 13.125, so it has no outliers.
6. Range: 21 1  20 runs
7. There are 54 values in the data set, so the median
is the average of the 27th and 28th values. The

y  0.4107142857x2  1645.696429x  1,648,561.


The coefficient of determination is
R2  0.9880773362.
7. Use the quadratic regression equation with
x  2004.

5  5

median, then, is 2  5 runs.


8. The lower quartile is the 14th value, 4 runs. The
upper quartile is the 41st value, 10 runs.
9. IQR  Q  Q
3
1

y  0.4107142857x2  1645.696429x  1,648,561


y  0.4107142857(2004) 2  1645.696429(2004)
 1,648,561
y  20.5
The cost in 2004 is expected to be about 20.5.
8. No; the price will start to increase using the
quadratic model.
9. An exponential model may be more appropriate
for predicting cost beyond 2003, since it will
continue the pattern of decreasing prices where
the annual decrease is getting smaller.

 10  4
 4 runs.

595

Chapter 13

PQ249-6481F-13[586-609] 26/9/02 6:37 PM Page 596 Sahuja Ahuja_QXP_06:Desktop Folder:Chandra:Algebra_FNL_Delivery:

14. The greatest number is 5.3. The least number is


1. The range is 5.3  1 4.3.
Order the set:

10. To find the outliers:


Q1  1.5(IQR)  4  1.5(4)
 2
Q  1.5(IQR)  10  1.5(4)
3
 16 runs
The data set has no values less than 2, but it
does have two values greater than 16 runs. The
two values greater than 16 are 17 runs and
21 runs, which are outliers.

Pages 734736

Q1 

Q2 

 108 S

d 41  1.5(11.5)
 58.25 S

 30.6
d IQR  30.9  30.05  0.85 S
d 30.9  1.5(0.85)
 32.175 S

65  68
2
84  88
2

 66.5
 86

 8.65  6.35
 2.3
To find the outliers:
6.35  1.5(2.3)  2.9
8.65  1.5(2.3)  12.1
Since the data set has no values less than 2.9 and
no values greater than 12.1, it has no outliers.

Since the data set has no values less than 28.775


or greater than 32.175, the set has no outliers.

Chapter 13

 7.15 S

 221  202
 19
To find the outliers:
202  1.5(19)  173.5
221  1.5(19)  249.5
Since the data set has no values less than 173.5
and no values greater than 249.5, it has no outliers.
17. The greatest value in the set is 11.7. The least value
in the set is 5.0. The range is 11.7  5.0  6.7.
The data set has 25 values, so the median is the
13th value, or 7.6. The lower quartile is the
average of the 6th and 7th values, which is
6.3  6.4
 6.35. The upper quartile is the average
2
8.5  8.8
of the 19th and 20th values, or
 8.65.
2
IQR  Q  Q
3
1

30.8  31.0

 28.775 S

d 4.15  1.5(2)

 86  66.5
 19.5
To find the outliers:
66.5  1.5(19.5)  37.25 86  1.5(19.5)  115.25
Since the data set has no values less than 37.25
or greater than 115.25, it has no outliers.
16. The greatest value in the set is 232. The least value
in the set is 193. The range is 232  193  39.
The data set has 19 values, so the median is the
10th value, or 218. The lower quartile is the 5th
value, or 202. The upper quartile is the 15th
value, or 221.
IQR  Q  Q
3
1

 30.05 median Q 3 
 30.9
Q1 
2
2
144
4442444443 T
144442444443
29.9
30.0
30.1
30.5
30.7
30.8
31.0
31.0
6447448
30.5  30.7
Q 
2
2

d 30.05  1.5(0.85)

 3.1

IQR  Q  Q
3
1

Since the data set has no values less than 12.25


or greater than 58.25, the set has no outliers.
13. The greatest number is 31.0. The least number is
29.9. The range is 31.0  29.9  1.1.
Order the set:
30.0  30.1

3  3.2
2

 0.85 S

Q3 

d IQR  41  29.5  11.5 S


 12.25 S

 4.15

d 2.15  1.5(2)

Q 
1

Since the data set has no values less than 32 and


no values greater than 108, the data set has no
outliers.
12. The greatest number is 42. The least number is
28. The range is 44  28  16.
Order the set:
median
T
28 28 29 30 31 34 37 38 39 40 42 42 44
6447448
6447448
c
29  30
40  42
Q1 
 29.5 Q2 Q3 
 41
2
2
d 29.5  1.5(11.5)

3.4  4.9
2

Since the data set has no values less than 0.85


and no values greater than 7.15, it has no outliers.
15. The greatest value is 99. The least value is 53.
The range is 99  53  46.
The data set has 16 values. The median is the
77  77
average of the 8th and 9th values, or
 77.
2

d IQR  79.5  60.5  19 S

 32 S

Q 
3

d IQR  4.15  2.15  2 S

11. The greatest number is 92. The least number is


55, the range is 92  55  37.
Order the set:
median
T
55
58
59
62
67
69
73
75
77 77 82 448
85 92
644474
4448
6444744
c
62  59
77  82
 60.5 Q2 Q3 
 79.5
Q1 
2
2
d 79.5  1.5(19)

 2.15 median

6447448 T 144424443
1
2
2.3
3
3.2
3.4
4.9
5.3
6474
8

Practice and Apply

d 60.5  1.5(19)

2  2.3
2

596

PQ249-6481F-13[586-609] 26/9/02 6:37 PM Page 597 Sahuja Ahuja_QXP_06:Desktop Folder:Chandra:Algebra_FNL_Delivery:

29. The greatest value for a cable-stayed bridge is


1630 ft. The least value for a cable-stayed bridge
is 630 ft. The range for cable-stayed bridges is
1630  630  1,000 ft. The greatest value for a
steel-arched bridge is 1700 ft. The least value for
a steel-arched bridge is 730 ft. The range for
steel-arched bridges is 1700  730  970 ft.
30. For cable-stayed bridges:
1000  1000
Median:
 1000 ft
2

18. The greatest value in the set is 6.8. The least value
in the set is 0.0. The range is 6.8  0.0  6.8.
The data set has 20 values, so the median is the
average of the 10th and 11th values, or
2.6  3.3
 2.95. The lower quartile is the average
2
1.7  1.9
 1.8. The
of the 5th and 6th values, or
2
upper quartile is the average of the 15th and 16th
3.9  4.0
values, or
 3.95.
2
IQR  Q3  Q1

 3.95  1.8
 2.15
To find the outliers:
1.8  1.5(2.15)  1.425

3.95  1.5(2.15)  7.175

Since the data set has no values less than 1.425


and no values greater than 7.175, it has no outliers.
19. The greatest value on the chart is 588,641 visitors.
The least value on the chart is 117,080. The range
is 588,641  117,080  471,561 visitors.
20. Order the data:
117,080

123,947 138,600 155,533 167,552 242,938

343,149

395,604 465,978 474,082 571,775 588,641

31.

Average the middle two values to find the median:

21.

242,938  343,149
 293,043.5 visitors
2
138,600  155,533
Q 
 147,066.5 visitors
2
1
465,978  474,082
Q 
 470,030 visitors
2
3

32.

22. IQR  Q3  Q1
 470,030  147,066.5  322,963.5 visitors
23. To find any outliers:
Q1  1.5(IQR)  147,066.5  1.5(322,963.5)
 337,378.75
Q3  1.5(IQR)  470,030  1.5(322,963.5)
 954,475.25
Since the set has no data less than 337,378.75
or greater than 954,475.25, there are no outliers.
24. The greatest value is 304. The least value is 9.
The range of the data is 304  9  295 Calories.
25. Order the data:
9
9 10 13 14 17 17 17 20 25 28
30 35 60 60 66 89 304.
The median is the average of the middle two
20  25
values:
 22.5 Calories.
2

33.

26. Q  14 Calories, Q  60 Calories


1
3
27. IQR  Q  Q
3
1
 60  14
 46 Calories
28. To find any outliers:
Q1  1.5(IQR)  14  1.5(46)
 55
Q3  1.5(IQR)  60  1.5(46)
 129
The set has no values less than 55 Calories, but
one value, 304, that is greater than 129 Calories.
The outlier is 304 Calories (avocado).

34.

597

Q  760 ft
1
Q3  1280 ft
For steel-arch bridges:
Median: 980 ft
Q1  820 ft
Q3  1100 ft
For cable-stayed bridges:
IQR  Q3  Q1
 1280  760
 520 ft
For steel-arch bridges:
IQR  Q3  Q1
 1100  820
 280 ft
To find outliers for cable-stayed bridges:
Q1  1.5(IQR)  760  1.5(520)
 20 ft
Q3  1.5(IQR)  1280  1.5(520)
 2060 ft
Since the set has no values less than 20 ft and
no values greater than 2060 ft, there are no
outliers among the cable-stayed bridges.
To find the outliers for steel-arch bridges:
Q1  1.5(IQR)  820  1.5(280)
 400 ft
Q  1.5(IQR)  1100  1.5(280)
3
 1520 ft
The set has no values less than 400 ft, but it does
have two values1650 and 1700greater than
1520 ft. There are two outliers1650 ft and
1700 ftamong the steel-arch bridges.
Although the range of the cable-stayed bridges is
only somewhat greater than the range of the
steel-arch bridges, the interquartile range of the
cable-stayed bridges is much greater than the
interquartile range of the steel-arch bridges. The
outliers of the steel-arch bridges make the ranges
of the two types of bridges similar, but in general,
the data for steel-arch bridges are more clustered
than the data for the cable-stayed bridges.
The median, lower quartile, and upper quartile
will each decrease by 2 in., since all values will
decrease by 2 in. The range and interquartile
range will remain the same, since the difference
between values will not increase.

Chapter 13

PQ249-6481F-13[586-609] 26/9/02 6:37 PM Page 598 Sahuja Ahuja_QXP_06:Desktop Folder:Chandra:Algebra_FNL_Delivery:

35. Measures of variation can be used to discuss how


much the weather changes during the year.
Answers should include the following.
The range of temperatures is used to discuss
the change in temperatures for a certain area
during the year, and the interquartile range is
used to discuss the change in temperature
during the moderate 50% of the year.
The monthly temperatures of the local area
listed with the range and interquartile range
of the data.
36. B; The greatest value in the set is 65. The least
value is 40. The range is 65  40  25.
37. A; Order the set:
1 2 3 7 8 9 14 15 18 24
Average the two middle values to find the median:
8  9
2

45.
3 4 5 6 7 8 9 10 11 12 13

46.
10 12 14 16 18 20 22 24 26 28 30

47.
20 25 30 35 40 45 50 55 60 65 70

Page 736

 8.5

Page 736

Maintain Your Skills

38. Sample answer:


Data
3035
3540
4045
4550
5055
5560
6065

The median is

Frequency
2
6
2
5
0
0
3

4
2
0
3540

4045

4550

5055

5560

6065

39. 1 by 3; first row, first column


40. 3 by 2; third row, second column
41. 2 by 4; second row, second column
42.

15a
39a2

13-5

1
3a  5
3a  13a

t  3
t2  7t  12


1

t

m  3
m2  9

1
t  3
(t  3) (t  4)

1
;
 4

1
m  3
(m  3) (m  3 )
1

 m  3;

m2  9  (m  3)(m  3)
or m  3  0
m30
m  3
m3
The excluded values are 3 and 3.

Chapter 13

Check for Understanding

1. The extreme values are 10 and 50. The quartiles


are 15, 30, and 40. There are no outliers.
2. The scale must include the least and greatest
values.
3. Sample answer:
2, 8, 10, 11, 11, 12, 13, 13, 14, 15, 16

t2  7t  12  (t  3)(t  4)
t  3  0 or t  4  0
t3
t4
The excluded values are 3 and 4.
44.

 1075.

Box-and-Whisker Plots

Pages 739740

 13a; excluded value is 0


43.

1075  1075
2

Q1  1025
Q3  1125
IQR  Q3  Q1
 1125  1025
 100
5. To find the outliers:
Q  1.5(IQR)  1025  1.5(100)
1
 875
Q  1.5(IQR)  1125  1.5(100)
3
 1275
The set has one value, 835, less than 875 and no
values greater than 1275. The set has one
outlier835.

3035

Practice Quiz 2

1. There are 4  7  6  2  19 values in the data


set. As a result, the median occurs at the 10th
value. The median occurs in the $1020 range.
2. There is a gap in the $30$40 measurement class.
Most of the books cost less than $30. The
distribution is skewed to the left.
3. The greatest value is 1175. The least value is 835.
The range of the data is 1175  835  340.
4. Order the set:
835
975 1005 1025 1050 1055 1075
1075 1095 1100 1125 1125 1145 1175

598

PQ249-6481F-13[586-609] 26/9/02 6:37 PM Page 599 Sahuja Ahuja_QXP_06:Desktop Folder:Chandra:Algebra_FNL_Delivery:

4. Step 1

6. Step 1 Determine the quartiles and outliers for


each set of data.
A:
17 18 22 22 24 26 28 31 32
c
c
c
Q1  20 Q2  24
Q3  29.5
B:
23 24 27 27 28 30 30 32 45
c
c
c
Q1  25.5 Q2  28 Q3  31
Set B has one outlier, 45.
Step 2 Draw the box-and-whisker plots using
the same number line.

Determine the quartiles and any outliers.


Order the data from least to greatest.
Use this list to determine the quartiles.
15, 16, 17, 21, 22, 22, 24, 24, 28, 30
Q 
2

22  22
2

 22

Q1  17
Q3  24
Determine the interquartile range.
IQR  Q3  Q1
 24  17
7
Check to see if there are any outliers.
17  1.5(7)  6.5
24  1.5(7)  34.5
There are no values less than 6.5 or
greater than 34.5. There are no outliers.
Step 2 Draw a number line.
15 17

22 24

A
B

30
16 18 20 22 24 26 28 30 32 34 36 38 40 42 44 46

The B data have a greater range than


the A data. However, if you exclude the
one outlier in the B data, the B data are
less diverse than the A data.
7. Step 1 Determine the quartiles and outliers for
each set.
A:

14 16 18 20 22 24 26 28 30

Step 3

Complete the box-and-whisker plot.

14 16 18 20 22 24 26 28 30

5. Step 1

Determine the quartiles and any outliers.


Order the data from least to greatest.
Use this list to determine the quartiles.
64 65 66 66 67 67 68 69 71 74
Q 
2

67  67
2

8 11.4

14 14
c
Q1  14

 67

66 67

69

71

64

66

68

70

Step 2 Draw the box-and-whisker plots using


the same number line.

A
B
8

10 12 14 16 18 20 22 24

The A data are more diverse than the


B data.
8. Step 1 Determine the quartiles and outliers for
the data.

74

72

452 467 472 524 559 573 620 678 693 1397
Q1  472

74

66

68

70

72

c
Q2  566

c
Q3  678

The data has one outlier, 1397.


Step 2 Draw the box-and-whisker plot for the
data.

Complete the box-and-whisker plot.

64

16.7 19 24
c
Q3  16.7

13 14 15 15.8 16 16 18 20
c
c
c
Q1  13 Q2  15.4 Q3  16

Step 3

15.5 16
c
Q2  15.25

B:
9 12

Q1  66
Q3  69
Determine the interquartile range.
IQR  Q3  Q1
 69  66
3
Check to see if there are any outliers.
66  1.5(3)  61.5
69  1.5(3)  73.5
The set has no values less than 61.5 and
one value, 74, greater than 73.5. It is the
only outlier.
Step 2 Draw a number line.
64

15

74
400 500 600 700 800 900 1000 1100 1200 1300 1400
Million Dollars

599

Chapter 13

PQ249-6481F-13[586-609] 26/9/02 6:37 PM Page 600 Sahuja Ahuja_QXP_06:Desktop Folder:Chandra:Algebra_FNL_Delivery:

Complete the box-and-whisker plot.

9. Most of the data are spread fairly evenly from


about $450 million to $700 million. The one
outlier ($1397 million) is far removed from the
rest of the data.

0 2 4 6 8 10 12 14 16 18 20 22 24 26 28

Pages 740742

17. Order the data. Use this list to determine the


quartiles.

Practice and Apply

10. The greatest value is 130. The least value is 85.


The range of the data is 130  85  45.

23

11. From the graph, Q3  120 and Q1  90. As a


result, the interquartile range is 120  90  30.
12. Since Q  90,
1
13. Since Q  95,
2

1
4
1
2

of the data is less than 90.


of the data is greater than 95.

10

15

20

25

12

16

16

17

5.8

10

15

20

25

30

53 54
c
Q2  52

55

60

69

81

c
Q3  57.5

40

50

70

60

80

6.2

7.6 8.5
c
Q1

8.5

8.8 9.0
c
Q2  8.65

10.5 11.5
c
Q3

15.1

5 6 7 8 9 10 11 12 13 14 15 16

19. Order the data. Use the list to determine the


quartiles.
1.2 1.3 1.3 1.3 1.3 1.6 1.8 2 2.2 3.3 3.7 5.7 7.7 8.5 14
c
c
c
Q1
Q2
Q3

Determine the interquartile range.


IQR  5.7  1.3  4.4
Check to see if there are any outliers.
1.3  1.5(4.4)  5.3
5.7  1.5(4.4)  12.3
There is one value, 14, greater than 12.3. It is the
only outlier.
Complete the box-and-whisker plot.

30

16. Order the data. Use this list to determine the


quartiles.
1 1 1 2 2 4 4 5 5 5 6 8 10 10 14 15 27
c
c
c
Q1  2
Q2
Q3  10

Determine the interquartile range.


IQR  10  2  8
Check to see if there are any outliers.
2  1.5(8)  10
10  1.5(8)  22
There is one value, 27, greater than 22. It is the
only outlier.

Chapter 13

51

Determine the interquartile range.


IQR  10.5  7.6  2.9
Check to see if there are any outliers.
7.6  1.5(2.9)  3.25
10.5  1.5(2.9)  14.85
There is one value, 15.1, greater than 14.85. It is
the only outlier.
Complete the box-and-whisker plot.

c
c
c
Q1
Q2  6.5
Q3
Determine the interquartile range.
IQR  16  5  11
Check to see if there are any outliers.
5  1.5(11)  11.5
16  1.5(11)  32.5
There are no values less than 11.5 or greater
than 32.5, so there are no outliers.
Complete the box-and-whisker plot.

30

20

30

20

46 46
c
Q1  42.5

18. Order the data. Use the list to determine the


quartiles.

15. Order the data. Use this list to determine the


quartiles.
0

39

Determine the interquartile range.


IQR  57.5  42.5  15
Check to see if there are any outliers.
42.5  1.5(15)  20
57.5  1.5(15)  80
There is one value, 81, greater than 80. It is the
only outlier.
Complete the box-and-whisker plot.

14. Order the data. Use this list to determine the


quartiles.
1 1 2 3 4 5 6 8 10 15 27
c
c
c
Q2
Q3
Q1
Determine the interquartile range.
IQR  10  2  8
Check to see if there are any outliers.
2  1.5(8)  10
10  1.5(8)  22
Since 27 is greater than 22, it is the only outlier.
Complete the box-and-whisker plot.

27

0 1 2 3 4 5 6 7 8 9 10 11 12 13 14

20. The least value in both sets is 20. It is part of


data set B.
21. The greatest value in both sets is 70. It is part of
data set B.
22. The IQR for data set A is 60  30  30. The IQR
for data set B is 60  40  20. Data set A has the
greater IQR.

600

PQ249-6481F-13[586-609] 26/9/02 6:37 PM Page 601 Sahuja Ahuja_QXP_06:Desktop Folder:Chandra:Algebra_FNL_Delivery:

23. The range for data set A is 65  25  40. The


range for data set B is 70  20  50. Data set B
has the greater range.
24. Determine the quartiles and outliers for each
data set.
A:
15 16 17 19 22 24 26 28 32 38 40
c
c
c
Q1
Q2
Q3
B:
24 25 27 27 28 29 30 30 31 32 37
c
c
c
Q1
Q2
Q3
A: IQRA  32  17  15
17  1.5(15)  5.5; 32  1.5(15)  54.5;
no outliers
B: IQRB  31  27  4

26. Determine the quartiles and outliers for each


data set.
A:
1.5 3.5 3.8 3.9 4.0 4.0 4.1 4.1 4.2 4.4
{
c
c
c
outlier
Q1
Q2  4.0
Q3
B:
4.2
4.8
5.5
6.7
6.8
7.1
7.6
12.2
{
c
c
c
Q1  5.15 Q2  6.75 Q3  7.35 outlier
A: IQRA  4.1  3.8  0.3
3.8  0.3  3.5; 4.1  0.3  4.4; 3.1 is an
outlier.
B: IQRB  7.35  5.15  2.20
5.15  2.20  2.05; 7.35  2.20  9.55; 12.2 is
an outlier.
Draw the box-and-whisker plots using the same
number line.

27  1.5(4)  21; 31  1.5(4)  37;


no outliers
Draw the box-and-whisker plots using the same
number line.

A
B

A
0

10 11 12

Each set of data has an outlier. In general, the


B data are more diverse than the A data.
27. Determine the quartiles and outliers for each
data set.
A:
2.9 4.1 4.4 4.4 4.4 4.5 4.5 4.6 4.9
c
c
c
Q1  4.25
Q2
Q3  4.55
B:
3.9 4.1 4.2 4.3 4.5 4.5 4.9 5.1 5.2
c
c
c
Q1  4.15
Q2
Q3  5.0
A: IQRA  4.55  4.25  0.30
4.25  1.5(0.30)  3.80;
4.55  1.5(0.30)  5.00; 2.9 is an outlier.
B: IQRB  5.0  4.15  0.85
4.15  0.85  3.30; 5.0  0.85  5.85; no
outliers
Draw the box-and-whisker plots using the same
number line.

14 16 18 20 22 24 26 28 30 32 34 36 38 40

The A data are much more diverse than the


B data.
25. Determine the quartiles and outliers for each
data set.
A:
45 47 47 48 49 50 51 51 51 55 {
58
c
c
c
Q1
Q2
Q3
outlier
B:
35 37 37 38 39 41 41 41 45 {
48
c
c
c
Q1
Q2  40
Q3
outlier
A: IQRA  51  47  4
47  1.5(4)  41; 51  1.5(4)  57; 58 is an
outlier.
B: IQRB  41  37  4
37  1.5(4)  31; 41  1.5(4)  47; 48 is an
outlier.
Draw the box-and-whisker plots using the same
number line.

A
B

A
2

The A data have an outlier. Excluding the outlier,


the B data are more diverse than the A data.
34 36 38 40 42 44 46 48 50 52 54 56 58

The distribution of both sets of data are similar.


In general, the A data are greater than the
B data.

601

Chapter 13

PQ249-6481F-13[586-609] 26/9/02 6:37 PM Page 602 Sahuja Ahuja_QXP_06:Desktop Folder:Chandra:Algebra_FNL_Delivery:

IQR: 128  71  57
71  1.5(57)  14.5, 128  1.5(57)  213.5;
223 and 253 are outliers
Draw the box-and-whisker plot.

28. Determine the quartiles and outliers for the


data set.
3.5
3.5
4 4.5
4.5
5.5
10
17
c
c
c
Q1  3.75 Q2  4.5
Q3  7.75
A: IQR  7.75  3.75  4
3.75  1.5(4)  2.25; 7.75  1.5(4)  13.75;
no outliers.
Draw the box-and-whisker plot.

80

60

37.

100 120 140 160 180 200 220 240 260

Life-Time Scores for Top 50


U.S. Soccer Players
20

3 4 5 6 7 8 9 10 11 12 13 14 15 16 17

18
Frequency

29. The upper half of the data is very dispersed. The


range of the lower half of the data is only 1.
30. Determine the quartiles and outliers for the
data set.
35 35 35 36 36 36 36 38 38 38 39 39 40 43 44
c
Q1

123

56 56 57 58 58 60 64 66 76 89 98 100 181
c
outliers
Q3

60- 80- 100- 120- 140- 160- 180- 200- 220- 24080 100 120 140 160 180 200 220 240 260
Number of Scores

IQR  56  38  18
38  1.5(18)  11; 56  1.5(18)  83; outliers are
89, 98, 100, and 181
Draw the box-and-whisker plot of the data.

40

60

80

38. Both the box-and-whisker plot and the histogram


give a visual summary of the data. The box-andwhisker plot shows the intervals by quartile,
while the histogram shows the number of data
values for intervals of width 20.
39. Sample answer:
40, 45, 50, 55, 55, 60, 70, 80, 90, 90, 90
40. Box-and-whisker plots use a number line to show
the least value in the data, the greatest value in
the data, and the quartiles of the data. They also
indicate outliers. Answers should include the
following.

100 120 140 160 180

31. Top half; the top half of the data goes from
$48,000 to $181,000, while the bottom half goes
from $35,000 to $48,000.
32. The range appears to be 80  39  41 years. The
IQR appears to be 74  54  20 years.
33. Bottom half; the top half of the data goes from
70 yr to 80 yr, while the bottom half goes from
39 yr to 70 yr.
34. From the graph, the least value is 39, Q1 is 54,
Q2 is 70, Q3 is 74, and the greatest value is 80.
Therefore, three intervals of ages that contain
half the data are 39 to 70 years, 54 to 74 years,
and 70 to 80 years.
35. No; although the interval from 54 yr to 70 yr is
wider than the interval from 70 yr to 74 yr, both
intervals represent 25% of the data values.
36. Determine the quartiles and outliers for the
data set.
61

61

61

62

63

63

63

64

65

67

68

69

71 72
c
Q1

73

73

74

74

76

78

78

80

81

82

83

87

92

Least value that is not


Greatest value that is not
an outlier
an outlier
Lower
Upper
Quartile
Quartile
Outlier
Outlier
Median

Interquartile
range

Range

The students should draw a box-and-whisker


plot representing data they found in a
newspaper or magazine.
41 C; From the box-and-whisker plot. The median
appears to be 25.
42. D; According to the box-and-whisker plot, the
least value is 0, Q1 is 10, Q2 is 25, Q3 is 45, and
the greatest value is 50. The interval 045
represents the first three quartiles, or 75% of
the data.

96

c
Q2  85
100

101

102

107

108

118

119

124

126 128 129


c
Q3

131 131 135 137 138 150 152 189 193 223 253
14243
outliers

Chapter 13

12
10
8
6
4

45 46 46 47 47 48 48 49 50 51 52 54 55 55 55
c
Q2

20

16
14

602

PQ249-6481F-13[586-609] 26/9/02 6:37 PM Page 603 Sahuja Ahuja_QXP_06:Desktop Folder:Chandra:Algebra_FNL_Delivery:

Page 742

51. Find the measure of B.


180  90  39  51
The measure of B is 51 .

Maintain Your Skills

43. Range: 93  13  80
54  55
2

Median:

 54.5

Find the length of BC , which is the side


opposite A.

Frequency

Lower quartile: 45
Upper quartile: 67
Interquartile range: 67  45  22
To determine if there are outliers:
45  1.5(22)  12
67  1.5(22)  100
The data set has no values less than 12 and no
values greater than 100. It has no outliers.
44. Sample answer:

45.

tan 39 
0.8097 

9.7  BC
BC is about 9.7 meters long.
Find the length of AB, the hypotenuse.
12

cos 39  AB

4
2

ABcos 39  12
12

AB  cos 39

0
020

3
y  3

y

20- 40- 60- 8040 60 80 100

y
 4




12

AB  0.7771

3(y  4)  y(y  3)
(y  3) (y  4)

AB  15.4
AB is about 15.4 meters long.
52. Find the measure of A.
180  90  46  44
The measure of A is 51 .

3y  12  y  3y
(y  3) (y  4)

y2  6y  12
(y  3) (y  4)
2
3
2(r  2)  3(r  3)


r  3
r  2
(r  3) (r  2)
2r  4  3r  9
 (r  3) (r  2)
5r  5
 (r  3) (r  2)
w
4
3w  4
 15w  6  3(5w  2)
5w  2

46.

47.
48.

7a
5

15

 14a 

6r  3
r  6

Find the length of BC , which is the side


opposite A.
tan 44 
0.9657 


49.

BC
12
BC
12

14.5  BC

3  5  7  a  a
2  5  7  a

BC is about 14.5 feet long.


Find the length of AB, the hypotenuse.

3a
2

r2  9r  18
2r  1

3(2r1  1)
r  6

15

cos 44  AB

(r1  6) (r  3)
2r  1

ABcos 44  15

 3(r  3)
50. Find the measure of B.
180  90  42  48
The measure of B is 48 .
Find the length of BC , which is the side
opposite A.
sin 42 
0.6691 

15

AB  cos 44
15

AB  0.7193
AB  20.9
AB is about 20.9 feet long.
53. a2  7a  6  0
a2  7a  6

BC
22
BC
22

 6 

25
4

1a  2

BC is about 14.7 inches long.

49
4

25
4

a  2  3

Find the length of AC , which is the side


adjacent to A.
0.7431 

49
4
7 2
2

a2  7a 

14.7  BC

cos 42 

BC
15
BC
15

a  2  2

AC
22
AC
22

a2 
7

a22

16.3  AC

AC is about 16.3 inches long.

12
2

6
{1, 6}

603

or

a


5
2
7
2
2
2

2

1

Chapter 13

PQ249-6481F-13[586-609] 26/9/02 6:37 PM Page 604 Sahuja Ahuja_QXP_06:Desktop Folder:Chandra:Algebra_FNL_Delivery:

54. x2  6x  2  0
x2  6x  2
2  6x  9  2  9
x
(x  3) 2  7
x  3  17
x  3  17
x  3  17 or x  3  17
 5.6
 0.4
{0.4, 5.6}

8. Sample answer:
Weight
Interval
(pounds)
34
45
56
67
78
89
910
1011
1112

55. t2  8t  18  0
t2  8t  18
t2  8t  16  18  16
(t  4) 2  34
t  4  134
t  4  134
t  4  134 or t  4  134
 1.8
 9.8
{9.8, 1.8}
2

Weight of Babies
50
Cumulative Number
of Babies

56. (7p  p  7) 
 11)
 (7p2  p  7)  (p2  11)
 (7p2  p2 )  p  (7  11)
 6p2  p  18
57. (3a2  8)  (5a2  2a  7)
 (3a2  5a2 )  2a  (8  7)
 8a2  2a  1

s
160

90

 100

40
30
20
10
0

Algebra Activity
(Follow-Up of Lesson 13-5)

3-4 4-5 5-6 6-7 7-8 8-9 9-10 10-1111-12


Weight (pounds)

1. To find the number of students in Column 3, add


the number from the previous cell to the number
of students for the new interval.
2. The horizontal axes of both histograms represent
the test score intervals, and the vertical axes of
both histograms represent the number of
students. The bars for the second histogram are
taller than the bars for the first histogram
because each bar in the second histogram
indicates the cumulative number of students.
3. Sample answer: I prefer the first histogram
because you can see the number of students that
scored in each interval.
4. 40; 80; 120
5. Sample answer: greater than 600
6.

Cumulative
Number
of Babies
1
2
6
13
29
40
43
44
45

9. Sample answer:

(p2

Pages 743744

Number of
Babies
1
1
4
7
16
11
3
1
1

10. 89 lb

Chapter 13 Study Guide and Review


Page 745
1.
2.
3.
4.
5.
6.
7.
8.
9.
10.

7. 600  700

100s  14,400
s  144

Vocabulary and Concept Check

simple random sample


measures of variation
quartile
systematic random sample
biased sample
stratified random sample
interquartile range
voluntary response sample
outlier
range

Pages 745748

Lesson-by-Lesson Review

11. The sample is 8 test tubes with results of chemical


reactions. The population is all test tubes in the
laboratory. The sample is biased because not all
test tubes are equally likely to be selected. Since
the first 8 test tubes from Tuesday were selected,
the sample is a convenience sample.

Chapter 13

604

PQ249-6481F-13[586-609] 26/9/02 6:38 PM Page 605 Sahuja Ahuja_QXP_06:Desktop Folder:Chandra:Algebra_FNL_Delivery:

12. The sample is every 50th chocolate bar. The


population is all chocolate bars on the conveyor
belt. Since every chocolate bar is equally likely to
be selected, the sample is unbiased. The sample is
systematic, as every 50th chocolate bar is selected
for testing.

21. C  3D  c

1
3 1
1
1 3
13. A  B  C 2
0
4  C 2
3 1
1 1
3
1 2
0
11
3  1 1  (3)
C
22
03
4  (1)
1  (1) 1  (2)
30

3(1)
 C 3(2)
3(1)

3(1)
3(3)
3(2)

2(2)
2(2)

3(3)
3(1)
3(0)

 c

9 1
d
5 4

3(1)
d
3(0)

3 1
1
1 3
4  2
3 1
0
3
1 2
0
1
2(3)
2(0)
2(1)

2(1)
1
1
2(4)  2
3
2(3)
1 2

2
1
8  2
6
1

3
1
0

1 3
3 1
2
0

21
6  1 2  (3)
42
03
8  (1)
2  (1) 2  (2)
60

1
5
 2 3
1
0

1
9
6

23. Sample answer:

Cellular Minute Usage

2(1)
d
2(0)

3 2
2
16. C  D  c
d  c
2
1
4

3  2 2  1
d
1  (2)
40

 c

1 3
d
3
4

6
4
2
0

1
d
0

 c

0- 50- 100- 150- 200- 250- 30050 100 150 200 250 300 350
Cellular Minutes

24. Sample answer:

Coffee Consumption
16
14

3 2
2 1
d  c
d
1
4
2 0
Frequency

12

3  2 2  1
 c
d
1  (2)
40
5 1
 c
d
1
4

10
8
6
4

18. impossible

1
3 1
19. 5A  5 2
0
4
1 1
3
5(1)
 5(2)
5(1)

3  6 2  3
d
1  (6)
40

4 2
d
 c
4
0

17. C  D  c

 c

2
6
 4
0
2 2

Frequency

 c

3 2
6 3
d  c
d
1
4
6 0

2(1)
 2(2)
2(1)

1
d
0

2
2

 c

1

3
3 9
9 3
C 6
3 6
0
15. 2D  2 c

3 2
3(2)
d  c
1
4
3(2)

3
1
0

1
3
2

 c

22. 2A  B  2 2

2
4 4
C 4
3
3
2 3
3
1
14. 3B  3C 2
1

3 2
2 1
d  3c
d
1
4
2 0

5(3)
5(0)
5(1)

0
0-1 1-2 2-3 3-4
Cups

5(1)
5(4)
5(3)

5 15 5
 10
0 20
5 5 15
20. impossible

605

Chapter 13

PQ249-6481F-13[586-609] 26/9/02 6:38 PM Page 606 Sahuja Ahuja_QXP_06:Desktop Folder:Chandra:Algebra_FNL_Delivery:

25. Order the set of data from least to greatest.


30
40
50
60
70
80
90
100
c
c
c
Q2
Q3
Q1
The range is 100 30 or 70.
The median is

60  70
2

The lower quartile is

The interquartile range is 262 206 or 56. Check


to see if there are any outliers.
206  1.5(56)  122
262  1.5(56)  346
There is one outlier, 348.

or 65.
40  50
or 45.
2
80  90
or 85.
2

The upper quartile is


The interquartile range is 85 45 or 40.
The outliers would be less than 45 1.5(40) or
15 or greater than 85  1.5(40) or 145. There
are no outliers.
26. Order the set of data from least to greatest.
1 2 3 3.2 3.4 4 5 5 5.3 7 8 21 45 78
c
c
c
Q2
Q3
Q1
The range is 78  1 or 77.
The median is

5  5
2

60

60

Q2
77.1  82.3
2

50

 58.1 Q2 

67.0  69.8
2

 68.4 Q3 

78.1  79.1
2

 78.6

78.6  1.5(20.5)  109.35

60

70

80

90

Chapter 13 Practice Test


Page 749
1.
2.
3.
4.
5.
6.

b; column
a; element
c; row
e; scalar
d; dimensions
The sample is the first five dogs to run from the
pen. The population is all dogs in the pen. Since
each dog did not have an equal chance of being
selected for the sample, the sample is biased. The
first five dogs leaving the pen were selected;
therefore, it is a convenience sample.
7. The sample is a book for each hour the library is
open. The population is all book titles checked out
Wednesday. Since any book title checked out on
Wednesday has an equal chance of being selected,
the sample is unbiased. A book title is selected
every hour, so the sample is systematic.

125 199 200 212 220 230 239 240 240 250 274 327 348
c
c
c
250  274
2

100

There are no outliers.

64.3  68.6
or 66.45.
2
91.7  110.5
or 101.1.
2

Q3 

55.1  61.1
2

58.1  1.5(20.5)  27.35

The interquartile range is 101.1  66.45 or 34.65.


The outliers would be less than 66.45  1.5(34.65)
or 14.475 or greater than 101.1  1.5(34.65) or
153.075. There is one outlier, 254.8.
29. Order the set of data from least to greatest.

or 206 Q2

90

The interquartile range is 78.6  58.1 or 20.5.


Check to see if there are any outliers.

or 79.7.

The upper quartile range is

80

Q1 

Q3

The lower quartile range is

70

52.4 55.2 55.1 61.1 61.9 67.0 69.8 73.4 78.1 79.1 81.2 81.6

The range is 254.8  59.8 or 195.

Chapter 13

70

31. Order the data from least to greatest.

59.8 63.8 64.3 68.6 70.7 77.1 82.3 88.9 91.7 110.5 111.5 254.8
c
c
c

200  212
2

325

or 5.

The range is 92  55 or 37.


The median is the middle value or 73.
The lower quartile is 62.
The upper quartile is 77.
The interquartile range is 77 62 or 15.
The outliers would be less than 62  1.5(15) or
39.5 or greater than 77  1.5(15) or 99.5. There
are no outliers.
28. Order the set of data from least to greatest.

Q1 

275

70 75 80 85 85 90 95 100
c
c
c
80  85
Q
Q1 Q 

82.5
3
2
2
The interquartile range is 90  70 or 20. Check to
see if there are any outliers.
70  1.5(20)  40
90  1.5(20)  120
There are no outliers.

55 58 59 62 67 69 69 73 75 76 77 77 82 85 92
c
c
c
Q1
Q2
Q3

The median is

225

30. Order the data from least to greatest.

The lower quartile is 3.2.


The upper quartile is 8.
The interquartile range is 8 3.2 or 4.8.
The outliers would be less than 3.2  1.5(4.8) or
4 or greater than 8  1.5(4.8) or 15.2. There are
three outliers: 21, 45, and 78.
27. Order the set from least to greatest.

Q1

175

125

or 262

606

PQ249-6481F-13[586-609] 26/9/02 6:38 PM Page 607 Sahuja Ahuja_QXP_06:Desktop Folder:Chandra:Algebra_FNL_Delivery:

14. Sample answer:

2
3
1
4
2 1
8. W  X  1
0 1  2 2
0
2 2
0
0
1
2
3  2 1  (1)
0  (2)
1  0
2  1
02

14
12
Frequency

24
 1  (2)
20

Ages of Men
in Billiards Tournament

6
5
0
 3 2 1
2 1
2




3 2 1
3
1
6


1 2 4
4 1 1
33
1  4

2  1
2  (1)

16

4  (1)

 c

Number of Cards

16. Order the set of data from least to greatest.


975 1005 1025 1055 1075 1075 1095 1100 1125
1125 1145
The range is 1145  975 or 170.
The median is the middle value or 1075.
The lower quartile is 1025.
The upper quartile is 1125.
The interquartile is 1125  1025 or 100.
The outliers would be less than
1025  1.5(100) or 875 or greater than
1125  1.5(100) or 1275. There are no outliers.
17. Order the set of data from least to greatest.
0.1 0.2 0.2 0.3 0.4 0.4 0.4 0.5 0.5 0.5 0.6 0.7
0.8 0.9 1.9
The range is 1.9  0.1 or 1.8.
The median is the middle value or 0.5.
The lower quartile is 0.3.
The upper quartile is 0.7.
The interquartile range is 0.7  0.3 or 0.4.
The outliers would be less than
0.3  1.5(0.4) or 0.3 or greater than
0.7  1.5(0.4) or 1.3. There is one outlier, 1.9.

2(1)
2(1)

2(6)
d
2(1)

6 2 12
d
8
2
2
12. impossible
 c

13. Y  2Z  c

0- 20- 40- 60- 8020 40 60 80 100

3(1)
3(0)
3(2)

3
1
6
d
4 1 1

2(3)
2(4)

12
6 3
0
 6 6
3
6
0
11. 2Z  2 c

4
2
0

Number of Trading Cards


to Share

4
2 1
10. 3X  3C 2 2
0
0
1
2
3(2)
3(2)
3(1)

15. Sample answer:

0 3 5

5 1
5

3(4)
 3(2)
3(0)

65- 70- 75- 80- 85- 9070 75 80 85 90 95


Ages

Frequency

9. Y  Z 

10

3 2 1
3
1
6
d  2c
d
1 2 4
4 1 1

 c

3 2 1
2(3)
d  c
1 2 4
2(4)

 c

3 2 1
6
d  c
1 2 4
8

 c

36
1  8

 c

3 4 11
d
9
0
6

2  2
2  (2)

2(1)
2(1)

2(6)
d
2(1)

2 12
d
2 2
1  12
d
4  (2)

607

Chapter 13

PQ249-6481F-13[586-609] 26/9/02 6:38 PM Page 608 Sahuja Ahuja_QXP_06:Desktop Folder:Chandra:Algebra_FNL_Delivery:

7. D; Choices A, B, and C do not result in a sample


of students who represent the entire student body.
8. A; Consecutive entries in the Earth row are all
10 years apart. Consecutive entries in the Mars
row are all 5.3 years apart. Since the difference
between consecutive entries is constant in both
rows, the relationship is linear.
9. C; According to the graph, the least variation
occurs with Car C, which has the least range.
10. B; For Cars A and D, the median is 17. For Car C,
the median is 24. For Car B, the median is 26.

18. Order the data from least to greatest.


1 1 1 1 1 2 2 3 3 4 4 5 6 6 9 10 10
c
c
c
1  1
6  6
Q  2 6
Q1  2  1 Q2
3
The interquartile range is 6  1 or 5. Check to see
if there are any outliers.
1  1.5(5)  6.5
6  1.5(5)  13.5
There are no outliers.

11. x3  8x2  16x  x(x2  8x  16)


 x(x  4) (x  4)
 x(x  4) 2
2
12.
6x  x  2  0
(3x  2)(2x  1)  0
3x  2 or 2x  1

10

19. Order the data from least to greatest.


9 9 9 10 11 12 12 12 13 14 14 15 16 16 18 22
c
c
c
10  11
12  13
15  16
Q1 
 10.5 Q2 
 12.5 Q3 
 15.5
2
2
2

The interquartile range is 22  9  13. Check to


see if there are any outliers.
10.5  1.5(13)  9
15.5  1.5(13)  35
There are no outliers.

x  3

x2

13. 2419  14  3
 213
1

14. Each hour, Maren can complete 4 of the job.


1
Juliana can complete 6 of the job in the same
amount of time.

10 11 12 13 14 15 16 17 18 19 20 21 22

20. B; The interquartile range for A is 57  49 or 8.


The interquartile range for B is 45  28 or 17.
The interquartile range for C is 30  20 or 10.
B has the greatest interquartile range.

116 2  14 x  16 x  1
1
3

 4x  6x  1
1
x
4

 6x  3

3x  2x  8
5x  8

Chapter 13 Standardized Test Practice

x5
8

It will take 5 hours for the two to complete the job


working together.

Pages 750751
1. B; The equation y  4x  6 has a slope of m  4.
1

The equation y  4 x  2 has a slope of m 


making it perpendicular.
2. B; 3x  5x  8  32
8x  40
x5
5x  5(5)
 25

15. (AB) 2  (AC) 2  (BC) 2


(5) 2  (3) 2  (BC) 2
25  9  BC2
BC2  16
BC  4 miles
The distance between points B and C is 4 miles.
16. B; Column A: The next three terms of the
sequence are 749, 1082, and 1415. Their sum is
749  1082  1415  3246.
Column B: The 67th term of the arithmetic
sequence is

1
,
4

3. B; (x  8) 2  (x  8)(x  8)
 x2  8x  8x  64
 x2  16x  64
4. D; The least value of the graph occurs when
x  0. As a result,
y  x2  4
 (0) 2  4
 4

a67  a1  (67  1)  r,
where r  2  (49)  51.
Therefore, a  49  66  51  3317.
67
Column B is greater.

5. C; 3172  312  314  9  2  312


 3  2  312  312
 1812  312
 1512
6. B;

12
9

 27

9x  324
x  36 m

Chapter 13

608

17. A; Column A: The root of y  0.25x2  x  1 is

19.

0.25x  x  1  0
x2  4x  4  0
(x  2) 2  0
x2
Column B: The roots of b 

3a2

 5a  2 are

a3
2

The sum of the roots is 1  3  13.


Column A is greater.
3x  1
14

18. C; Column A:

4

3  4x  4  14x
12x  4  14x
4  2x
2x
Column B:

45
4y  1

6
4
2
0

4050

50- 60- 70- 80- 9060 70 80 90 100

20. There are 1  0  4  5  7  3  20 pieces of


data. The interval 80  90 has 7 pieces of data,
7
representing 20  0.35  35% of the data.
21. Order the data from least to greatest. Use this list
to determine the quartiles.
22, 24, 25, 30, 32, 34, 36, 38, 38, 39, 40, 40, 40, 41,
42, 45, 45, 47, 47, 48, 49
Q1  33
Q2  40
Q3  45
Determine the interquartile range.
IQR  45  33  12
Check to see if there are any outliers.
Q1  1.5(12)  33  18  15;
Q3  1.5(12)  45  18  63
There are no outliers.
Complete the box-and-whisker plot.

3a2  5a  2  0
(3a  2) (a  1)  0
3a  2 or a  1
2

Frequency

PQ249-6481F-13[586-609] 26/9/02 6:38 PM Page 609 Sahuja Ahuja_QXP_06:Desktop Folder:Chandra:Algebra_FNL_Delivery:

10

 y
45y  4y  10  10
45y  40y  10
5y  10
y2

So, x  y.

20

609

30

40

50

Chapter 13

Chapter 14 Probability
Page 753

3. 5! is the product of 5 and all positive integers less


than 5.
5!  5  4  3  2  1
 120
4. Spin 1 Spin 2 Spin 3
Outcomes

Getting Started

1. There are 6 red cubes and 14 cubes total.


6

P(red)  14 or 7
2. There are 3 blue cubes and 14 cubes total.
3

P(blue)  14
3. There are 4 yellow cubes and 14 cubes total.
4

P(yellow)  14 or 7
4. There are 8 cubes that are not red and 14 cubes
total.
8
14

P(not red) 

or

5.

4
5

3
4

4
5

3
4

6.

5
12

6
 11

8.

4
32

13
52
13

 256

9.

 32  32  32

4
 52

 19  20  19
 95

13
52

6
 11

 22
1

7
20

5
12
5

5
7.

4
7

14

4
 52

10.

56
100

24
 100

1
52

56
100

84
625

25

24
 100
25

11. 0.725  (0.725  100) %


 72.5%
12. 0.148  (0.148  100) %
 14.8%
13. 0.4  (0.4  100) %
 40%
14. 0.0168  (0.0168  100) %
 1.68%
15.

7
8

17.

107
125

 0.875

B
Y

16.

33
80

18.

625
1024

 87.5%

 0.4125

 41.3%

 0.856
 85.6%

 0.6103515625

 61%
Y

14-1 Counting Outcomes


Page 756

Check for Understanding

1. Sample answer: choosing 2 books from 7 on a


shelf
2. Toss 1
Toss 2 Toss 3
Outcomes

H
H
T

H
T
H
T

HHH
HHT
HTH
HTT

H
T
H
T

THH
THT
TTH
TTT

B
G
Y

H
T
T

Chapter 14

610

R
B
Y
G
R
B
Y
G
R
B
Y
G
R
B
Y
G
R
B
Y
G
R
B
Y
G
R
B
Y
G
R
B
Y
G
R
B
Y
G
R
B
Y
G
R
B
Y
G
R
B
Y
G
R
B
Y
G
R
B
Y
G
R
B
Y
G
R
B
Y
G

RRR
RRB
RRY
RRG
RBR
RBB
RBY
RBG
RYR
RYB
RYY
RYG
RGR
RGB
RGY
RGG
BRR
BRB
BRY
BRG
BBR
BBB
BBY
BBG
BYR
BYB
BYY
BYG
BGR
BGB
BGY
BGG
YRR
YRB
YRY
YRG
YBR
YBB
YBY
YBG
YYR
YYB
YYY
YYG
YGR
YGB
YGY
YGG
GRR
GRB
GRY
GRG
GBR
GBB
GBY
GBG
GYR
GYB
GYY
GYG
GGR
GGB
GGY
GGG

13. 11!  11  10  9  8  7  6  5  4  3  2  1
 39,916,800
14. 13!  13  12  11  10  9  8  7  6  5  4  3  2  1
 6,227,020,800
15. There are 6 possible outcomes for each die.
Multiply.
red white blue
possible
die
die
die
outcomes
123
123
123
14
424
43

5. The tree diagram shows that there are 64 possible


outcomes.
6. Of the 64 possible outcomes shown on the tree
diagram, we observe that 18 involve both green
and blue.
7. 8!  8  7  6  5  4  3  2  1
 40,320
8. Multiply to find the number of ways students can
choose to do their assignments.
project
paper
presentation
number
choices choices
choices
of ways
14243
14243
144
424
443
14243

6 
6
 6 
216
There are 216 possible outcomes.
16. Multiply to find the number of outfits.
5  3  3  4  180
There are 180 possible outfits.
17. Multiply to find the number of ways.
Seattle
Denver
number
to
to
of
Denver St. Louis
ways
14243
14243
14243

15

6

8

720
There are 720 different ways that students can
choose to do their assignments.

Pages 757758
9. English

Practice and Apply


Math

Science
A
B
C
A
B
C
A
B
C
A
B
C
A
B
C
A
B
C
A
B
C
A
B
C
A
B
C

A
A

B
C
A

B
C
A

Outcomes

B
C

4
 (4  2) 
24
A traveler can book a flight from Seattle to
St. Louis in 24 ways.
18. There are ten digits. Five of these, 1, 3, 5, 7, and 9,
are odd. Multiply to find the number of area codes.
odd 0 or 1 any digit
area
codes
123
123
14243
1442443

AAA
AAB
AAC
ABA
ABB
ABC
ACA
ACB
ACC
BAA
BAB
BAC
BBA
BBB
BBC
BCA
BCB
BCC
CAA
CAB
CAC
CBA
CBB
CBC
CCA
CCB
CCC

5 
2

10

100
There are 100 possible area codes.

19.

0
4
1

0
5
1

Monitor 1
CD-ROM
Monitor 2
Monitor 1
CD recorder
Monitor 2
Monitor 1
DVD
Monitor 2

Printer
Scanner
Printer
Scanner
Printer
Scanner
Printer
Scanner
Printer
Scanner
Printer
Scanner

407
408
409
417
418
419
507
508
509
517
518
519

20. Let C represent a win for Columbus Crew and D


a win for D.C. United.
If either team wins all of the first three games,
the 4th and 5th games will not be played. Two
possible outcomes are C-C-C and D-D-D.
If one of the teams achieves its third win in the
4th game, the 5th game will not be played.
More possible outcomes are
C-C-D-C, C-D-C-C, D-C-C-C, D-D-C-D, D-C-D-D,
and C-D-D-D.
If all five games are played, the possible outcomes
are C-C-D-D-C, C-D-C-D-C, C-D-D-C-C,
D-C-D-C-C, D-D-C-C-C, D-C-C-D-C, D-D-C-C-D,
D-C-D-C-D, D-C-C-D-D, C-D-C-D-D, C-C-D-D-D,
and C-D-D-C-D.
21. From the list of possible outcomes, we see that
6 outcomes require that exactly four games be
played to determine the champion.

The tree diagram shows that there are 27 possible


outcomes.
10.

7
8
9
7
8
9
7
8
9
7
8
9

CD-M1-P
CD-M1-S
CD-M2-P
CD-M2-S
CDR-M1-P
CDR-M1-S
CDR-M2-P
CDR-M2-S
DVD-M1-P
DVD-M1-S
DVD-M2-P
DVD-M2-S

The tree diagram shows that there are 12 possible


outcomes.
11. 4!  4  3  2  1
12. 7!  7  6  5  4  3  2  1
 24
 5040

611

Chapter 14

33. There are 8 data values greater than the median.


The 4th and 5th of these are 71 and 76, respectively.
71  76
The upper quartile is
or 73.5.
2

22. From the list of possible outcomes, we see that


there are 10 ways D.C. United can win the
championship.
23. Since there are five days in a school week, Tucker
walked 3 days (60% of 5 days), rode his bike 1 day
(20% of 5 days), and rode with a friend 1 day.
Tyler could have taken his bicycle any one of the
five days of the week, leaving any one of the
remaining four days as the day he could have
gone with a friend. The remaining three days he
walked; there are no choices here.
choices for
choices for
number
day he rode
day he rode
of
bike
with friend
outcomes
144
424
443

144
424
443

There are 8 data values less than the median. The


4th and 5th of these are 35 and 44, respectively.
35  44
The lower quartile is
or 39.5.
2
The interquartile range is 73.5  39.5 or 34.0.
34. An outlier must be 1.5(34.0) or 51 less than the
lower quartile, 39.5, or 51 greater than the upper
quartile 73.5. There are neither data values less
than 39.5  51 or 11.5 nor greater than
73.5  51 or 124.5. There are no outliers.
35.

14
424
43

5

4

20
This situation is represented by 20 outcomes.
24. Sample answer: You can make a chart showing all
possible outcomes to help determine a football
teams record. Answers should include the
following.
a tree diagram or calculations to show
16 possible outcomes
25. A; 9!  9  8  7  6  5  4  3  2  1
 362,880
26. C; Multiply the number of model choices, the
number of package choices, and the number of
color choices.
4  6  12  288
There are 288 possibilities for this car.

Page 758

36.

37.

Maintain Your Skills

Chapter 14

56  59
2

x  4
 2

x

(2x  1) (x  2)
(x  4) (3x  1)
 (x  2) (3x  1)
(3x  1) (x  2)
(2x  1) (x  2)  (x  4) (3x  1)
(3x  1) (x  2)

2x2  3x  2  3x2  11x  4


(3x  1) (x  2)

5x2  8x  6
(3x  1) (x  2)
4n
3
4n
3
 n  3  2(n  3)  n  3
2n  6
2n
3
n3n3
2n  3
 n3
3z  2
z  2
3z  2
z  2
 z2  4  3(z  2)  (z  2) (z  2)
3z  6
3z  2
1
 3(z  2)  z  2
3z  2
3
 3(z  2)  3(z  2)
3z  2  3
 3(z  2)
3z  1
 3(z  2)
3z  1
 3z  6
m  n
1
m  n
1
 m2  n2  m  n  (m  n) (m  n)
m  n
(m  n) (m  n)
1
 (m  n) (m  n)  (m  n) (m  n)
(m  n) (m  n)  1
 (m  n) (m  n)

38.

27. For plot A, the least value is 32, the greatest


value is 88, the lower quartile is 44, the upper
quartile is 85, and the median is 60.
For plot B, the least value is 38, the greatest
value is 86, the lower quartile is 48, the upper
quartile is 74, and the median is 64.
28. The least data value in either set is 32, which is
contained in set A.
29. For plot A, the interquartile range is
85  44 or 41.
For plot B, the interquartile range is
74  48 or 26.
Plot B has the smaller interquartile range.
30. For plot A, the range is 88  32 or 56.
For plot B, the range is 86  38 or 48.
Plot A has the greater range.
31. The greatest data value is 109. The least data
value is 30. Thus, the range of the data is
109  30 or 79.
32. There are 16 data values. In ascending order, the
8th and 9th data values are 56 and 59, respectively.
The median is

2x  1
3x  1


39.

m2  2mn  n2  1
m2  n2

522n2  28  20
1
(522n2
5

 28)  5 (20)

22n2  28  4
( 22n2  28) 2  42
2n2  28  16
2n2  44
n2  22
n  122
The solutions are 122 and 122.

or 57.5.

612

40.

44. x2  11x  17  0
x2  11x  17

25x2  7  2x
( 25x2  7) 2  (2x) 2
5x2  7  4x2
x2  7  0
x2  7
x  17

121
4
11 2
2

x2  11x 

1x  2
x

25x  7  2x

Check:

x

15(7)  7  217

11
2

11
2

189
4

189
4

45.

25x2  7  2x
?

189
4

11
2

189
4

 1.4

2p2  10p  3  0
1
(2p2
2

25( 17) 2  7  2( 17)

x

or

 12.4
{1.4, 12.4}

128  217
217  217

 10p  3)  0
3

p2  5p  2  0

25(7)  7  217
?

p2  5p   2

128  217
217  217

25
4
5 2
2

p2  5p 

1p  2

Since 17 does not satisfy the original equation,


17 is the only solution.
1x  2  x  4
( 1x  2) 2  (x  4) 2
x  2  x2  8x  16
0  x2  9x  14
0  (x  7)(x  2)
x  7  0 or x  2  0
x7
x2
Check:

 3

x

25(17)  7  2(17)

41.

189
4

11
2

121
4

 17 

 2 


25
4

19
4

p  2  3

19
4

p  2 
5

19

p  2 

34

or

19

34
5

p  2 

19

34

 0.3
 4.7
{4.7, 0.3}
46. There are four 10s and a total of 52 cards.

1x  2  x  4
?

P(10)  52 or 13

17  2  7  4
?

19  3
33
1x  2  x  4

47. There are four aces and a total of 52 cards.


4

P(ace)  52 or 13
48. There are two red 5s and a total of 52 cards.

12  2  2  4

P(red 5)  52 or 26

14  2
2  2
Since 2 does not satisfy the original equation, 7 is
the only solution.
42. b2  6b  4  0
b2  6b  4
2  6b  9  4  9
b
(b  3) 2  5
b  3  15
b  3  15
b  3  15 or b  3  15
 5.2
 0.8
{0.8, 5.2}

49. There is one queen of clubs and a total of 52 cards.


1

P(queen of clubs)  52
50. There are 20 even numbers, four each of 2, 4, 6, 8,
and 10, and a total of 52 cards.
20

P(even number)  52 or 13
51. There are four 3s and four kings. Thus, there are
eight ways to get a 3 or king and a total of 52 cards.
8

P(3 or a king)  52 or 13

Page 759

43. n2  8n  5  0
n2  8n  5
2  8n  16  5  16
n
(n  4) 2  21
n  4  121
n  4  121
n  4  121 or n  4  121
 0.6
 8.6
{8.6, 0.6}

Algebra Activity
(Follow-Up of Lesson 14-1)

1. Yes; Sample answer: Front St., Main St., Second


Ave., State St., Elm St., First Ave., Town St.
2. Besides sample answer in Exercise 1:
(2) Second Ave., Main St., Front St., State St.,
Elm St., First Ave., Town St.
(3) Second Ave., Main St., Front St., State St.,
Town St., First Ave., Elm St.
(4) Front St., Main St., Second Ave., State St.,
Town St., First Ave., Elm St.
There are 4 traceable routes that begin at Aleks
house.

613

Chapter 14

3. Also:
(5) Town St., First Ave., Elm St., State St.,
Front St., Main St., Second Ave.
(6) Elm St., First Ave., Town St., State St.,
Front St., Main St., Second Ave.
(7) Town St., First Ave., Elm St., State St.,
Second Ave., Main St., Front St.
(8) Elm St., First Ave., Town St., State St.,
Second Ave., Main St., Front St.
There are 8 traceable routes now.
4. Yes; all nodes can be connected without retracing
an edge.
5. No; all nodes cannot be connected without
retracing an edge.
6. Yes; all nodes can be connected without retracing
an edge.
7a.

n!
(n  r)!
8!
(8  5)!
8!
3!

87654321
321

6. P 
n r
P 

8 5

 8  7  6  5  4 or 6720
7. C 
n r
C 

7 5

n!
(n  r)! r!
7!
(7  5)! 5!
7!
2! 5!
1

7654321

 2154321
1

76

 2  1 or 21
10!

3!

8. ( P )( P )  (10  5)!  (3  2)!


10 5 3 2
10! 3!
 5!  1!


10  9  8  7  6
1

321
1

 181,440
6!

4!

9. ( C ) ( C )  (6  2)! 2!  (4  3)! 3!
6 2 4 3
6!
4!
 4! 2!  1! 3!

7b. Yes; start at any node and first follow the edges
to adjacent nodes until you come back to where
you started. Then follow the edges that form a
star until you again come back to where you
started.
7c. Yes; starting at any building you can follow the
sidewalk to the building two buildings away in
the clockwise direction. If you repeat this, you
will eventually reach all five buildings without
using any sidewalk more than once.
8. Sample answer: If you follow the edges of a graph,
you should cover each edge only once.

65
2!

1

 60
10. Permutation; the order of the digits is important.
11. We find the number of permutations of 10 digits
taken 3 at a time.

n!
(n  r)!
10!
(10  3)!
10!
7!

10  9  8  7  6  5  4  3  2  1
7654321

P 

n r

P 

10 3

 10  9  8 or 720
720 different codes are possible.
12. We begin by finding the number of codes whose
digits are all odd. Since there are five odd digits,
we find the number of permutations of 5 odd
digits taken 3 at a time.

14-2 Permutations and Combinations


Page 764

Check for Understanding

1. Sample answer: Order is important in a


permutation but not in a combination.
Permutation: the finishing order of a race
Combination: toppings on a pizza
2. C 
n n
P 
n n

n!
(n  n)! n!
n!
(n  n)!

n!
0!

or n!, since 0!  1

54321
21

3. Alisa; both are correct in that the situation is a


combination, but Alisas method correctly
computes the combination. Erics calculations find
the number of permutations.
4. Combination; order is not important.
5. Permutation; order is important.

Chapter 14

P 

5 3

n!

 0!n! or 1, since 0!  1.

n!
(n  r)!
5!
(5  3)!
5!
2!

P 

n r

 5  4  3 or 60
There are 60 codes whose digits are all odd and a
total of 720 possible codes.
60

P(all odd)  720 or 12

614

13. B; Since order is not important, we find the number


of combinations of 8 items taken 2 at a time.
C 

n r

C 

8 2

27.

C 

n r

C 

n!
(n  r)!r!
8!
(8  2)!2!
8!
6!2!

20 8





1
87654321

 65432121
87

28.

 2  1 or 28

P 

n r

P 

15 3

Pages 764766
14.
15.
16.
17.
18.
19.
20.
21.
22.

Practice and Apply

P 

P 

12 3




29.

P 



P 

4 1

7!




C 

7 3

20!

24. nCr 
C 

6 6




n!
(n  r)!r!
6!
(6  6)!6!
6!
0!6!
6!
1  6!

 1

1






16  15  14  13
1
7!
 4)!4!

765
3!

 105
8!

5!

33. ( C ) ( P )  (8  5)!5!  (5  5)!


8 5 5 5
8! 5!
 3!5!  0!


876
3!

5!
1

 6720
34. Permutation; order is important.
35. We find the number of permutations of 9 players
taken 9 at a time.

1
765

C 

3!

 3  2  1 or 35

15 3

20  19
1

32. ( C ) ( C )  (3  2)!2!  (7
3 2 7 4
3!
7!
 1!2!  3!4!

1
7654321

C 

16!

 16,598,400

n!
(n  r)!r!
7!
(7  3)!3!
7!
4!3!

n r

1

 35,280

 4321321

26.

7!
1

31. ( P )( P )  (20  2)!  (16  4)!


20 2 16 4
20! 16!
 18!  12!

4
25. C 
n r

7!

30. ( P )( P )  (7  7)!  (7  1)!


7 7 7 1
7! 7!
 0!  6!

1
12  11  10  9  8  7  6  5  4  3  2  1
987654321
1

4!
(4  1)!
4!
3!
1
4321
321
1

n!
(n  r)!
16!
(16  5)!
16!
11!
16  15  14  13  12  11!
11!

 524,160

n!
(n  r)!
12!
(12  3)!
12!
9!

n!
(n  r)!

P 

n r

16 5

 12  11  10 or 1320
23. P 
n r

n!
(n  r)!
15!
(15  3)!
15!
12!
15  14  13  12!
12!

 2730

Combination; order is not important.


Permutation; order is important.
Combination; order is not important.
Permutation; order is important.
Permutation; order is important.
Combination; order is not important.
Combination; order is not important.
Combination; order is not important.
n r

n!
(n  r)!r!
20!
(20  8)!8!
20!
12!8!
20  19  18  17  16  15  14  13  12!
12!8!
5,079,110,400
or
125,970
40,320

P 

n!
(n  r)!r!
15!
(15  3)!3!
15!
12!3!
15  14  13  12!
12!3!
15  14  13
3!
2730
or 455
6

n r

P 

9 9

n!
(n  r)!
9!
(9  9)!
9!
0!

 9! or 362,880
The manager can make 362,880 different lineups.
36. Combination; order does not matter.

615

Chapter 14

There are 3780 ways to choose 5 women and


177,100 possible outcomes.

37. We find the number of combinations of 12 people


taken 4 at a time.
C 

n r

C 

12 4




n!
(n  r)!r!
12!
(12  4)!4!
12!
8!4!
12  11  10  9
4!

3780

45. We find the number of permutations of 8 girls


taken 3 at a time.
P 

8 3

 495
Thus, 495 different groups of students could be
selected.
38. We find the number of permutations of 12 people
taken 4 at a time.
P 

n r

P 

12 4




( 4P2 )( 4P1 ) 



 11,880
Thus, 11,880 different groups of students could be
selected.
39. There are 12 students, one of which will be the
chairperson.

48

P(1st, 2nd: West; 3rd: Central)  336


1

 7 or about 14%
47. There are 4 choices of entree, 3 choices of side
dish, and 3 choices of beverage. Use the
Fundamental Counting Principle.
4  3  3  36
There are 36 possible meal combinations.
48. There are 3 possible side dishes, one of which is
soup.

40. Permutation; order matters.


41. Use the Fundamental Counting Principle. On
each die, you have 6 possible outcomes.
6  6  6  6  6  7776
There are 7776 possible outcomes.
42. There are 6 different ways that all five dice could
show the same number and 7776 possible
outcomes.
6
7776

P(soup)  3 or about 33%.


49. There are 4  3 or 12 ways that a student can
choose an entree and a side dish, one of which is
to choose a sandwich and soup.

1
1296

P(all same number) 


or
43. We find the number of combinations of 15 men
taken 4 at a time and 10 women taken 2 at a time
and multiply.

P(sandwich and soup) 

C 



P(tap shoe) =

C 

6 4




 177,100
Next, we find the number of ways to choose
1 man and 5 women.



1
30,240

6!
(6  4)! 4!
6!
2! 4!
65
or 15
2!

The coach can form 15 different teams.


53. We find the number of permutations of
4 swimmers taken 4 at a time.

15!
10!

(15  1)!1! (10  5)!5!
15!
10!

14!1! 5!5!
15 10  9  8  7  6

1
5!

P 

4 4

4!
(4  4)!
4!
0!

 4! or 24
The four swimmers had to swim 24 relays.

 3780
Chapter 14

or about 8%

52. We find the number of combinations of


6 swimmers taken 4 at a time.

25!
(25  6)!6!
25!
19!6!
25  24  23  22  21  20
6!

( 15C1 ) ( 10C5 ) 

1
12

50. A two-word arrangement is composed of the seven


letters and one space. There are 7! ways to arrange
the letters and 6 possible positions for the space, so
there are 7!  6 or 30,240 two-word arrangements.
51. Tap shoe is one possible arrangement and there
are a total of 30,240 possible arrangements.

15!
10!

(15  4)!4! (10  2)!2!
15!
10!

11!4! 8!2!
15  14  13  12 10  9
 2!
4!

 61,425
This can be done in 61,425 ways.
44. First, we find the number of ways 6 people can be
chosen from the total of 25 men and women.
25 6

4!
4!

(4  2)! (4  1)!
4! 4!

2! 3!
43 4
 1 or 48
1

There are 48 ways for the described event to


happen and 336 possible outcomes.

P(being chairperson)  12

8!
(8  3)!
8!
5!

 8  7  6 or 336
The runners can place first, second, and third in
336 ways.
46. First, we find the number of permutations of
4 girls taken 2 at a time and 4 girls taken 1 at a
time and multiply.

n!
(n  r)!
12!
(12  4)!
12!
8!
12  11  10  9
1

( 15C4 ) ( 10C2 ) 

27

P(5 women)  177,100 or 1265

616

54. There are positions, one of which is the third leg.


1
4

P(third leg) 

66. d  2(x  x ) 2  (y  y ) 2
2
1
2
1

or 25%

 2(16  12) 2  (34  20) 2

55. Sample answer: Combinations can be used to


show how many different ways a committee can
be formed by various members. Answers should
include the following.
Order of selection is not important.
Order is important due to seniority, so you
need to find the number of permutations.
56. B; We find the number of permutations of
12 songs taken 10 at a time.
P

12 10




 242  142
 1212
 2153 or about 14.56 units
67. d  2(x  x ) 2  (y  y ) 2
2
1
2
1
 2(2  (18)) 2  (15  7) 2
 2202  82
 1464
 4129 or about 21.54 units

12!
(12  10)!
12!
2!

68. d  2(x  x ) 2  (y  y ) 2
2
1
2
1

P 

4 4




s

4  242  4(1) (2)


2(1)
4  18
2
4  2 12
2

m  2  12
 3.41

b  2b2  4ac
2a

1  212  4(2) (15)


2(2)
1  1121

4
1  11

4
1  11
1  11
s
or s 
4
4
10
12
 4
 4
5
2
 3
5
3, 2

71.

2n2

2n2  n  4
n40

n

b  2b2  4ac
2a

1
 3

1
(x  7) (x  7 )
(x  7 ) (x  5)


n

x  7
 5

(1)  2(1) 2  4(2) (4)


2(2)
1  133
4
1  133
1  133
or n 
4
4

 1.69
{1.19, 1.69}

b  2b2  4ac
2a

70. 2s2  s  15  0

x
65.

 2  12
m  2  12 or
 0.59
{3.41, 0.59}

n2  n  20
n2  9n  20

25

34

x  3
(x  3) (x  3 )

m

x2  49
x2  2x  35

 (2) 2

69. m2  4m  2  0

50 60 70 80 90 100 110 120130140150

64.

 (3  5) 2

34  4
5

60. The greatest data value is $148,000. The least


data value is $55,800. Thus, the range of the data
is $148,000  $55,800 or $92,200.
61. There are ten data values. Of the five smallest
data values, $56,700 is in the middle. Thus, the
lower quartile is $56,700. Of the five largest data
values, $91,300 is in the middle. Thus, the upper
quartile is $91,300.
62. The interquartile range is $91,300  $56,700 or
$34,600. An outlier must be 1.5($34,600) or
$51,900 less than the lower quartile or greater
than the upper quartile. No data values are less
than $56,700  $51,900 or $4800. One data value,
$148,000, is greater than $91,300  $51,900 or
$143,200. Thus, $148,000 is the only outlier.

x

 2 or 22 units

Maintain Your Skills

3 2

58. There are two possibilities, boy or girl, for each


year and there are four years. Use the
Fundamental Counting Principle.
2  2  2  2  16
There are 16 different ways.
59.

x  3
x2  6x  9

3 12 2

 4! or 24

63.

4!
(4  4)!
4!
0!

Page 767

3 1 2  (2) 2

 12  11  10  9  8  7  6  5  4  3
 239,500,800
57. C; We find the number of permutations of
4 numbers taken 4 at a time.

(n  5) (n  4)
(n  5) (n  4)

72.

n  5
 5

8
52

12

 1.19
3

 52  52 or 13

n

617

Chapter 14

73.

7
32

20

Page 768

 8  32  32
27

 32
74.
75.

76.

77.

5
6
2
9
3
 15  15  15 or 5
15
15
11
3
15
11
18
 24  4  24  24  24
24
8
1
 24 or 3
2
15
1
24
15
9
 36  4  36  36  36
3
30
5
 36 or 6
16
3
1
64
30
25
 10  4  100  100  100
25
69
 100

Page 767

Practice Quiz 1

1. There are 6 possible outcomes for the die and


2 possible outcomes for each of the coins. Use the
Fundamental Counting Principle.
6  2  2  24
There are 24 possible outcomes.
2. Use the Fundamental Counting Principle.
5  4  2  5  200
There are 200 possible mountain bikes.
3.

C 

n r

C 

13 8





n!
(n  r)! r!
13!
(13  8)!8!
13!
5! 8!
13  12  11  10  9  8!
5! 8!
13  12  11  10  9
5!

14-3 Probability of Compound Events


Pages 772773

 1287
4. P 
n r
P 

9 6




 987654
 60,480
5. First, find the number of combinations of
14 flowers taken 4 at a time.
C 



14!
(14  4)!4!
14!
10!4!
14  13  12  11
4!

 1001
Now, find the number of ways to choose 2 roses
out of 6 and 2 lilies out of 3.
( 6C2 ) ( 3C2 ) 



6!
(6  2)! 2!
6!
3!

4!2! 1!2!
65 3
1
2!

3!

 (3  2)! 2!

Chapter 14

 21  21
30

10

 441 or 147
6. Since the first chip is not replaced, the events are
dependent.
P(yellow, yellow)  P(yellow)  P(yellow)

 45
There are 45 four-flower bouquets with two roses
and two lilies and 1001 possible four-flower
bouquets.
P(two roses and two lilies) 

Check for Understanding

1. A simple event is a single event, while a compound


event involves two or more simple events.
2. Sample answer: The probability of rolling a
number less than or equal to six on a number
cube and tossing heads or tails on a coin.
3. Sample answer: With dependent events, a first
object is selected and not replaced. With
independent events, a first object is selected and
replaced.
4. Chloe; sample answer: Since it is possible for the
person chosen to be a girl and a senior, the events
are inclusive. So, add the probability that a girl
14
is chosen, 34, and the probability that a senior is
15
chosen, 34, then subtract the probability that a
6
senior girl is chosen, 34.
5. Since the first chip is replaced, the events are
independent.
P(red, green)  P(red)  P(green)

n!
(n  r)!
9!
(9  6)!
9!
3!
9  8  7  6  5  4  3!
3!

14 4

Reading Mathematics

1. Sample answer: Yes; combine can mean placing


many things together, as you do in a combination.
A mutation is a change in genes and the order in
which they appear as in a permutation.
2. Sample answer: Both permutations and
combinations involve selecting items. However, a
permutation considers the order of the selected
items.
3. Sample answer:
factorialthe product of all the positive integers
from 1 to nsymbol n!
factorany of the numbers or symbols in
mathematics that when multiplied together form
a product
factorizationthe operation of resolving a
quantity into factors
The meanings all involve products.
4. probabilitythe quality or state of being
probable; something (as an event or circumstance)
that is probable
probusupright, liberal, generous
probareto test, approve, prove
Sample answer: The words all involve something
being true or approved.

 21  20
2

 420 or 210

45
1001

618

17. Since the first marble is not replaced, the events


are dependent.
P(blue, then red)  P(blue)  P(red)

7. Since the chips are replaced, the events are


independent.
P(green, blue, red)  P(green)  P(blue)  P(red)
6

 18  17

80

 306 or 51

 21  21  21
240

12

 9261 or 3087

 21  20  19
240

12

126

5
8

84

or

20. P(3 and D)  P(3)  P(D)


1

 65




12
24
24
24

22. P(a prime number and A)


 P(a prime number)  P(A)
3

7
8

 65

12
24

 30 or 10

or 1

23. P(2 and A, B, or C)


 P(2)  P(A, B, or C)

12. Since there are students who are both male and
not 11th graders, these events are inclusive.
P(male or not 11th grader)
 P(male)  P(not 11th grader)
 P(male and not 11th grader)
21
24

 30 or 5

11. Since a student cannot be both male and female,


the events are mutually exclusive.
P(male or female)  P(male)  P(female)

18

 65
3

 30 or 10
24. P(both even)  P(even)  P(even)

 24  24  24


or

15

210

 840 or 4

25. P(both 7 20 and 6 30)


 P( 7 20 and 6 30)  P( 7 20 and 6 30)
9

 30  28
81

27

 840 or 280

 5  5  5 or 5
15. The last one tested is tested third out of
3 possibilities, first, second, or third.
P(last) 

14

 30  28

13. These are independent events, since each DVD


1
player has the same probability, 5, of being the
defective player.
14. P(defective, defective, or defective)
 P(defective)  P(defective)  P(defective)
1

21. P(an odd number and a vowel)


 P(an odd number)  P(vowel)
6

12

 6  5 or 30

 24  24  24


 4896 or 408

10. Since there are students who are both 10th


graders and females, these events are inclusive.
P(10th grader or female)
 P(10th grader  P(female)
P(10th grader and female)
15
24

 18  17  16

 24 or 2

12

19. Since the marbles are not replaced, the events are
dependent.
P(blue, then yellow, then red)
 P(blue)  P(yellow)  P(red)

 24  24

 4896 or 272

9. Since a student cannot be both a 9th and 12th


grader, the events are mutually exclusive.
P(9th or 12th grader)  P(9th grader)
 P(12th grader)
6

 18  17  16

 7980 or 133

18. Since the marbles are not replaced, the events are
dependent.
P(2 yellows in a row then orange)
 P(yellow)  P(yellow)  P(orange)

8. Since the chips are not replaced, the events are


dependent.
P(green, blue, red)  P(green)  P(blue)  P(red)
6

26. P(first 7 10, second 6 40 or odd)


 P( 7 10)  P( 6 40 or odd)
20

 30 

1
3

11928  1428  1028 2

23

 3  28
46

Pages 773776

Practice and Apply

27. P(first 7 12 or prime, second multiple of 6 or 4)


 P(7 12 or prime)  P(multiple of 6 or 4)

16. Since the first marble is not replaced, the events


are dependent.
P(2 orange)  P(orange)  P(orange)
3

11830  1030  305 2  1285  287  283 2

23

 30  28

 18  17
6

23

 84 or 42

207

69

 840 or 280

 306 or 51

619

Chapter 14

40. In the 1624 age group, 1145  2080 or 3225


thousand earn no more than minimum wage.
P(no more than minimum wage)

28.
A

B
0.90

0.06

3225

 15,793 or about 0.20

0.02

In the 25 age group, 970  2043 or


3013 thousand earn no more than minimum wage.
P(no more than minimum wage)
0.02

3013

 55,287 or about 0.05

29. P(A or B)  P(A)  P(B)  P(A and B)


 0.96  0.92  0.90
 0.98 or 98%
30. From the Venn diagram, we see that this
probability is 0.02 or 2%.
31. No; P(A and B)  P(A)  P(B)
P(A and B)  0.90
P(A)  P(B)  0.96  0.92 or 0.8832
32. There is one combination of genes, bb, for blue
eyes and four combinations.

A worker in the 1624 age group is more likely to


earn at most minimum wage, since the
probability is greater for this group.
41. There are 6C2 or 15 ways to choose 2 of the six
angles.
150, 30
150, 130
150, 50
150, 20
150, 160
30, 130
30, 50
30, 20
30, 160
130, 50
130, 20
130, 160
50, 20
50, 160
20, 160
Every pair listed above contains either an angle
inside ABC or an obtuse angle.
P(inside or obtuse)  1
42. Among the six angles, there is no straight angle,
and there is no right angle inside ABC.
P(straight or right)  0
43. P(20 or 130)
 P(20)  P(130)  P(20 and 130)

P(blue eyes)  4
33. The eye colors of the children are independent
events.
P(both brown eyes)
 P(brown eyes)  P(brown eyes)
3

 4  4 or 16
34. P(both brown)  P(first or second blue)  1
9
16

 P(first or second blue)  1


P(first or second blue) 

44. Since there are regions that are both a triangle


and red, these events are inclusive. The area of
the dartboard is 12  10 or 120 in2. The area of
1
each trapezoid is 2 (6  12)(5) or 45 in2. The area
1

of each triangle is 2 (3)(5) or 7.5 in2.


The total area of the four triangles is 4  7.5 or
30 in2. The total area of the red regions is
45  7.5  7.5 or 60 in2. The total area that is
both a triangle and red is 7.5  7.5 or 15 in2.
P(a triangle or a red region)
 P(triangle)  P(red)  P(triangle and red)
30

60

75

15

 120  120  120


 120 or 8

2115
71,080

45. Since there is a region that is both a trapezoid


and blue, these events are inclusive. The total
area of the two trapezoids is 45  45 or 90 in2.
The total area of the blue regions is 45  7.5 
7.5 or 60 in2. The total area that is both a
trapezoid and blue is 45 in2.
P(a trapezoid or a blue region)

 0.03
There are 2080  2043 or 4123 thousand who
earn less than minimum wage.
4123

P(less than minimum wage)  71,080


 0.06
39. These are mutually exclusive events.
P(less than or equal to minimum wage)
 P(less than)  P(equal to)
 0.06  0.03 or 0.09

Chapter 14

 15 or 5

35. See students work.


36. These events are mutually exclusive.
P(public transportation or walks)
 P(public transportation)  P(walks)
 0.049  0.015
 0.064 or 6.4%
37. We estimate that 88.9% of the 400 employees will
use a motor vehicle.
88.9% of 400  0.889(400)
 355.6
There should be at least 356 parking spaces.
38. There are 1145  970 or 2115 thousand who earn
minimum wage and a total of 15,793  55,287 or
71,080 thousand hourly workers.
P(earn minimum wage) 

 15  15  15

7
16

 P(trapezoid)  P(blue)  P(trapezoid and blue)


90

60

105

45

 120  120  120


 120 or 8

620

46. Since no region is both blue and red, these events


are mutually exclusive.
P(a blue triangle or a red triangle)
 P(blue triangle)  P(red triangle)



15
120
30
120

Page 776

C 

n r

15
 120
1
or 4

C 

5 3

47. Since no region is both a square and a hexagon,


these events are mutually exclusive. There are no
regions that are squares. There is one hexagon
that is formed by the two trapezoids; its area is
45  45 or 90 in2.
P(a square or a hexagon)
 P(square)  P(hexagon)




90

4

C 

48. Find the sum of the numbers in the regions of the


Venn diagram.
36  38  8  25  2  5  3  3  120
Thus, 120 students were surveyed.
49. Find the sum of the numbers in the circle
representing Event A.
36  38  25  2  101
Thus, 101 students said that they drive a car to
school.
50. There are two students in the overlap of the three
circles, indicating the students who do all three.
There is a total of 120 students.

n r

C 

4 2





57. Use the Fundamental Counting Principle. There


are 10 choices for the first store, 9 choices for the
second store, and so on.
10  9  8  7  6  5  4  604,800
There are 604,800 different arrangements.
3 6
2 4
3  (2) (6)  4
58. c
d  c
d  c
d
1
2
1 5
(1)  1
25

or

 c

39
40

59. c

52. Sample answer: Meteorologists use probabilities


to forecast the weather. Answers should include
the following.
You can use compound probabilities to forecast
the weather over an extended period of time.
80%
53. C; Since the marbles are not replaced, these
events are not independent.
P(first three red)
 P(first red)  P(second red)  P(third red)
8

60.

2m2  7m  15
m  5

5
2
d
4 1

9m2  4
3m  2

2m2  7m  15 3m  2
 9m2  4
m  5
(2m  3) (m  5)
3m  2
 (3m  2) (3m  2)
m  5

(2m  3) (m  5)
m  5

1 2
d
0
7

4 5
9 7
(4)  (9) (5)  (7)
d c
d c
d
8
8
4
9
84
89
c

 24  23  22
336

P(any one person)  10 or 5

51. Find the sum of the numbers within the three


circles.
36  38  8  25  2  5  3  117
There are 117 students who drive or are involved
in activities or have a job. There is a total of 120
students.
P(A or B or C) 

n!
(n  r)!r!
4!
(4  2)!2!
4!
2!2!
4  3  2!
2!2!
43
or 6
2!

Thus, there are 6 different possible committees


with our particular person and 10 possible
committees.

P(all three)  120 or 60

117
120

n!
(n  r)!r!
5!
(5  3)!3!
5!
2!3!
5  4  3!
2!3!
54
or 10
2!

There are 10 possible committees.


56. Consider a particular person. Any committee for
which this person is selected must also have two
of the remaining four people. Find the number of
combinations of the remaining four people taken
two at a time.

 0  120

Maintain Your Skills

55. Find the number of combinations of five people


taken three at a time.

3m  2

 (3m  2) (3m  2)
1

2m  3

 3m  2

 12,144 or 253

61. 145  13  3  5
 232  15
 315

54. A; We can eliminate answer choices B, C, and D


with the following observation. The probabilty of
Yolanda making a free throw in one attempt is
0.8. Three attempts would increase the
probability that she makes a free throw. Thus,
the probability of Yolanda making at least one
free throw in three attempts is greater than 0.8
or 80%.

62. 1128  18  8  2
 282  12
 812

63. 240b4  223  5  b4


 222  12  15  2b4
 2  12  15  b2
 2b2 110

621

Chapter 14

There are 5 ways to roll a sum of 6 and 36


distinct rolls.

64. 2120a3b
 223  3  5  a3  b
 222  12  13  15  2a2  1a  1b
 2  12  13  15  0a 0  1a  1b

P(X  6)  36
6. First, find the probability that the sum is greater
than 6 on a single roll.
P(X 7 6)
 P(X  7)  P(X  8)  P(X  9) 
P(X  10)  P(X  11)  P(X  12)

 2 0a 0 130ab

65. 317  612  (3  6)( 17  12)


 1817  2
 18114

67.

9
24

69.

63
128

 9 24

68.

2
15

70.

5
52

 0.375
 0.492
71.

8
36

73.

81
2470

74.

18
1235

 0.222

75.

 18 1235
 0.015

14-4 Probability Distributions


Check for Understanding

1. The probability of each event is between 0 and 1


inclusive. The probabilities for each value of the
random variable add up to 1.
2. Sample answer: The probability of tossing a coin
and getting a head versus getting a tail is the
same. No matter what is tossed, the same
probability is multiplied three times.
3. Sample answer: the number of possible correct
answers on a 5-question multiple-choice quiz, and
the probability of each
1
2
3
4
5
6
7

2
3
4
5
6
7
8

3
4
5
6
7
8
9

4
5
6
7
8
9
10

5
6
7
8
9
10
11

Pages 779780

Practice and Apply

10. There are two possible outcomes on each spin, red


(R) and blue (B). Thus, for three spins, the
possible outcomes are RRR, RRB, RBR, RBB,
BRR, BRB, BBR, and BBB.
11. Each spin is an independent event.
P(X  0)  P(RRR)
 P(R)  P(R)  P(R)
1

 444

6
7
8
9
10
11
12

 64
P(x  1)  P(RRB)  P(RBR)  P(BRR)
1

 64
P(X  2)  P(RBB)  P(BRB)  P(BBR)
1

 444  444  444

 64  64  64

There are 4 ways to roll a sum of 5 and 36


distinct rolls.

 64

27

P(X  3)  P(BBB)

P(X  5)  36 or 9

 444
27

 64
Chapter 14

 64  64  64

P(X  4)  36 or 12

 444  444  444

5. There are 3 ways to roll a sum of 4 and 36


distinct rolls.
3

7. For each value of X, the probability is greater


than or equal to 0 and less than or equal to 1.
0.05  0.10  0.40  0.40  0.05  1, so the
probabilities add up to 1.
8. The probability of passing the course is the sum
of the probabilities of earning an A, B, C, or D.
P(X
1.0)  P(X  1.0)  P(X  2.0) 
P(X  3.0)  P(X  4.0)
 0.10  0.40  0.40  0.05
 0.95
9. P(B or better )  P(X
3.0)
 P(X  3.0)  P(X  4.0)
 0.40  0.05
 0.45

 0.036

4. Dice
1
2
3
4
5
6

343

 128 3570

Page 779

 1728

 11 38

 0.033
128
3570

 0.289

 81 2470

 12  12  12

 5 52

11
38

P(X 7 6 on three rolls)


 P(X 7 6 on first roll)
 P(X 7 6 on second roll)  P(X 7 6 on third roll)

 2 15

72.

21

Each roll is an independent event.

 0.096

 8 36

 36 or 12

 0.133

 63 128

 36  36  36  36  36  36

66. 13( 13  16)  13  13  13  16


 232  13  13  12
 3  232  12
 3  312

622

12.

27
64

P(X )

32
64
24
64
16
64

23a. Let G represent a girl and B a boy. For a single


1
1
birth, P(G)  2 and P(B)  2.
If X  1, then the first child is a girl.
P(X  1)  P(G)

27
64

9
64

8
64

2

1
64

1
2
X = Blue

If X  2, then the first child is a boy and the


second is a girl.
P(X  2)  P(BG)
 P(B)  P(G)

If X = 3, then the first two children are boys and


the third is a girl.
P(X  3)  P(BBG)
 P(B)  P(B)  P(G)
1

 2  2  2 or 8
If X  4, then the first three children are boys
and the fourth is a girl.
P(X  4)  P(BBBG)
 P(B)  P(B)  P(B)  P(G)
1

 2  2  2  2 or 16
23b. P(X  1)  P(X  2)  P(X  3) 
P(X  4)  P(X 7 4)  1
1
2

 4  8  16  P(X 7 4)  1
15
16

 P(X 7 4)  1
1

P(X 7 4)  16
24. Sample answer: A pet store owner could use
probability distributions to plan sales and special
events. Answers should include the following.
Determine the probability of each outcome of
an event and list them in a table.
The owner could look at the probability of a
customer owning more than one pet and
create special discounts for larger purchases.
25. A; P(X 2)  P(X  0)  P(X  1)  P(X  2)
 0.0625  0.25  0.375
 0.6875
26. B; P(X 6 5)  P(X  2)  P(X  3)  P(X  4)

0.3
0.25
P(X )

 2  2 or 4

13. No; the probability of X  1 is less than the


probability of X  2, indicating that X  1 is less
likely to occur.
14. Let X  the number of CDs. X can have values of
100, 200, 300, 400, and 500.
15. For each value of X, the probability is greater
than or equal to 0 and less than or equal to 1.
0.10  0.15  0.40  0.25  0.10  1,
so the probabilities add up to 1.
16. P(X 400)  P(X  100)  P(X  200) 
P(X  300)  P(X  400)
 0.10  0.15  0.40  0.25
 0.90
17. P(X 7 200)  P(X  300)  P(X  400) 
P(X  500)
 0.40  0.25  0.10
 0.75
18. P(at most some college)
 P( some high school)  P(high school
graduate)  P(some college)
 0.167  0.333  0.173
 0.673
19.
0.35

0.2
0.15
0.1
0.05

 36  36  36

0
me

ee
gr
De e
re
ed
nc D e g
va
e
A d l o r ' s egre
e
sD
ch
Ba
t e'
c i a e ge
so
ll
As
Co
m e o o l ool
So
ch Sch
S
g h i gh
H

Hi

So

 36 or 6 or about 0.17

Page 781

Maintain Your Skills

27. These are mutually exclusive events.


P(ace or ten)  P(ace)  P(ten)

20. Sample answer: Add the values for the bars


representing bachelors and advanced degrees.
21. No; 0.221  0.136  0.126  0.065  0.043  0.591.
The sum of the probabilities does not equal 1.
22. We would expect about 6.5% of the 35 women to
say they watch figure skating.
6.5% of 35  0.065(35)
 2.275
We would expect about 2 women to say they
watch figure skating.

 52  52
 52 or 13
28. These are inclusive events.
P(3 or diamond)  P(3)  P(diamond)
 P(3 and diamond)
4

13

16

 52  52  52
 52 or 13

623

Chapter 14

 612  712
 1312

29. These are inclusive events.


P(odd number or spade)
 P(odd number)  P(spade)  P(odd number and spade)

30.

16
52

25
52

C 

n r

C 

10 7





13
52

38. 213  112  213  222  3


 213  ( 222  13)
 213  213
 413
39. 317  2128  317  2222  7
 317  2( 222  17)
 317  2(217)
 317  417
 17

4
52

n!
(n  r)!r!
10!
(10  7)!7!
10!
3!7!
10  9  8  7!
3!7!
10  9  8
3!

C 

31.

n r

C 

12 5





 120
32. ( P ) ( P ) 
6 3 5 3



n!
(n  r)!r!
12!
(12  5)!5!
12!
7!5!
12  11  10  9  8  7!
7!5!
12  11  10  9  8
5!

 792

 c

2 4
d
3 12

34. B  A  c
 c

1.44
1.8

0.0825 12(4)
12

 900(1.006875) 48
 1250.46
The balance is $1250.46.
42. Sample answer: Monthly; the balance after four
years is greater with monthly compounding.

40
d
75

3 0
1 4
d
d  c
5 7
2 5

43.

16
80

45.

30
114

 0.2

44.

20
52

46.

57
120

 20%

3  1 0  4
d
2  5 5  7

 0.3846
 38%

 0.2632
 26%

47.

72
340

 0.2118

Page 781

 0.475
 48%

48.

54
162

 21%

 0.3333
 33%

Practice Quiz 2

1. For each value of X, the probability is greater


than or equal to 0 and less than or equal to 1.
0.25  0.32  0.18  0.15  0.07  0.02 
0.01  1, so the probabilities add up to 1.
2. P(X
4)  P(X  4)  P(X  5) 
P(X  6)  P(X  7)
 0.15  0.07  0.02  0.01
 0.25 or 25%
3.
0.35

or 0.8

0.3

P(X )

0.25

4

 3

0.2
0.15
0.1

x3

0.05

x  3 when y  3.

37. 318  712  3222  2  7 12


 3( 222  12)  712
 3(212)  712
Chapter 14

A  900 1 

y  0.8 when x  1.8.


36. First find k.
xy  k
(1) (4)  k
4  k
The equation is xy  4.
Now find x when y  3.
xy  4
x(3)  4
3x
3

r
41. A  P 1  n nt

4 4
 c
d
7 2
35. First find k.
xy  k
(0.6) (2.4)  k
1.44  k
The equation is xy  1.44.
Now find y when x  1.8.
xy  1.44
1.8y  1.44
y

0.0825 4(4)
4

 900(1.020625) 16
 1247.68
The balance after 4 years is $1247.68.

 7200
3 0
1 4
33. A  B  c
d  c
d
2 5
5 7
1  (3)
5  (2)

A  900 1 

6!
5!

(6  3)! (5  3)!
6! 5!

3! 2!
654 543
 1
1

 c

r
40. A  P 1  n nt

1 2 3 4 5 6 7
X  Number of People

624

5. Sample answer: 5 marbles of two colors where


three of the marbles are one color to represent
making a free throw, and the other two are a
different color to represent missing a free throw.
Randomly pick one marble to simulate a free
throw 25 times.
6. See students work.
7. See students work.
8. See students work.
9. Yes; 70% of the marbles in the bag represent
water and 30% represent land.
10. Since 30% of the Earths surface is land, the
theoretical probability that a meteorite reaching
the Earths surface hits land is 30%.
11. Brown represents hitting land. There are 19 and
a total of 56  19 or 75 occurrences.

4. These events are inclusive.


P(odd or greater than 4)
 P(odd)  P(greater than 4)  P(odd and greater than 4)
5

 10  10  10
 10 or 5

5. These events are mutually exclusive.


P(less than 3 or greater than 7)
 P(less than 3)  P(greater than 7)
2

 10  10
 10 or 2

14-5 Probability Simulations


Page 783

19

P(hits land)  75
 0.25 or about 25%
12. We would expect 25% of the next 500 meteorites
to hit land.
25% of 500  0.25(500)
 125
We would expect 125 of the next 500 meteorites to
hit land.

Algebra Activity

1. Rolling a 2 is one of six possible outcomes.


P(rolling a 2) 

1
6

2. These events are mutually exclusive.


P(rolling a 1 or a 6)
 P(rolling a 1)  P(rolling a 6)
1

66
 6 or 3

Pages 785787

666
 6 or 2
4. Sample answer: The classs probability is closer
since there are more trials.
5. Sample answer: The classs probability should be
closer to the theoretical probability.
6. Sample answer: The more times an experiment is
performed, the closer the experimental probability
is to the theoretical probability.

Page 785

Practice and Apply

13. Sample answer: a coin tossed 15 times


14. Sample answer: a spinner divided into 3 sections
1
1
where 2 represents cola, 3 represents diet cola,
1
and 6 represents root beer
15. Sample answer: a coin and a die since there are
12 possible outcomes
16. Sample answer: toss a coin and roll a die 100
times each
17. See students work. 18. See students work.
19. See students work. 20. See students work.
21. As you roll the dice more times, the experimental
probabilities should get closer to the theoretical
probabilities. Consider the following
theoretical probabilities.

3. P(rolling a value less than 4)


 P(rolling a 1)  P(rolling a 2)  P(rolling a 3)

Check for Understanding

P(sum of 5)  36 or about 0.11

1. An empirical study uses more data than a single


study, and provides better calculations of
probability.
2. As the number of trials increases, the
experimental probabilities tend toward the
theoretical probabilities.
3. Sample answer: a survey of 100 people voting in a
two-person election where 50% of the people favor
each candidate; 100 coin tosses
4. If you flip a coin, the theoretical probability of
1
getting heads is 2. If you flip a coin twice and get
1 head and 1 tail, the experimental probability of
1
getting heads is also 2. However, if you flip a coin
twice and get two heads, then the experimental
2
probability of heads is 2 or 1. Thus, the theoretical
and experimental probabilities of an event are
sometimes the same.

P(sum of 9)  36 or about 0.11


Since these theoretical probabilities are close to
10%, we would expect to get a sum of 5 about 10%
of the time and a sum of 9 about 10% of the time.
22. There are a total of 787 people enrolled.
47

P(Preschool)  787 or about 0.060


46

P(Kindergarten)  787 or about 0.058


378

P(Elementary School)  787 or about 0.480


201

P(High School)  787 or about 0.255


115

P(College)  787 or about 0.146

625

Chapter 14

45. These events are independent.


P(3 dimes)  P(dime)  P(dime)  P(dime)

23. These events are mutually exclusive.


P(Elementary or High School)
 P(Elementary)  P(High School)
 0.480  0.255
 0.735 or about 0.74 or 74%
24. We would expect about
be in kindergarten.

46
787

25

29.
31.
32.
33.

34.

15,625

18

125

46. These events are dependent.


P(nickel, quarter, dime)
 P(nickel)  P(quarter)  P(dime)

of the new students to

12

25

 55  54  53

We would expect about 105 of the new students to


be in kindergarten.
Sample answer: 3 coins 26. See students work.
See students work.
MMMMM, MMMMF, MMMFM, MMMFF,
MMFMM, MMFMF, MMFFM, MMFFF,
MFMMM, MFMMF, MFMFM, MFMFF, MFFMM,
MFFMF, MFFFM, MFFFF, FMMMM, FMMMF,
FMMFM, FMMFF, FMFMM, FMFMF, FMFFM,
FMFFF, FFMMM, FFMMF, FFMFM, FFMFF,
FFFMM, FFFMF, FFFFM, FFFFF
See students work.
30. See students work.
See students work.
No; there were 181 heads out of the 300 tosses. The
experimental probability of heads is about 60%.
Sample answer: Probability can be used to
determine the likelihood that a medication or
treatment will be successful. Answers should
include the following.
Experimental probability is determining
probability based on trials or studies.
To have the experimental probability more
closely resemble the theoretical probability the
researchers should perform more trials.
D; These are independent events.
P(2 tails and a 3)  P(tail)  P(tail)  P(3)
1

25

 166,375 or 1331

46
1787
2 (1800)  105.2

25.
27.
28.

25

 55  55  55

5400

20

 157,410 or 583
47. Find the number of combinations of 55 coins
taken 3 at a time.
C 

55 3




55!
(55  3)!3!
55!
52!3!
55  54  53
3!

 26,235
Now find the number of combinations of 2 dimes
and 1 quarter.
( 25C2 )( 12C1 ) 



25!
12!

(25  2)!2! (12  1)!1!
25!
12!

23!2! 11!1!
25  24 12
 1
2!

 3600
There are 3600 ways to pick 2 dimes and
1 quarter and a total of 26,235 ways to choose
3 coins.
3600

48.

2a  3
2
a  3
2a  3
2
a  3

35. B; There are three ways that the one head can
occurfirst, second, or thirdand there are eight
possible outcomes for flipping a coin three times.

(a  3) (a  3)
1

(a  3) (2a  3)  2(a  3)(a  3)  12(a  3)


(2a2  3a  9)  (2a2  18)  12a  36
2a2  3a  9  2a2  18  12a  36

P(exactly one head)  8

3a  9  12a  36

37. See students work.


39. See students work.
41. See students work.

9a  45
a5

The solution is 5.
r2
r  7
r2
r  7

49.

Maintain Your Skills

42. For each value of X, the probability is greater


than or equal to 0 and less than or equal to 1.
0.579  0.276  0.107  0.038  1, so the
probabilities add up to 1.
43. P(X
2)  P(X  2)  P(X
3)
 0.107  0.038
 0.145
44. P(X 1)  P(X  0)  P(X  1)
 0.579  0.276
 0.855

Chapter 14

12
 3

 24

Page 788

a

2  (a  3)(a  3) 1a 12 3 2
1 (a  3)1(a  3)  2aa  33 2  1 (a  3)1(a  3)  21 2
(a  3)(a  3)

 226

36. See students work.


38. See students work.
40. See students work.

80

P(2 dimes and 1 quarter)  26,235 or 583

(r  7)
1

r  7
1

r

r2
 7
1

50
 r
50
r  7

7

 14

 14

1r r 7  r 50 7 2  (r  7)14

2 1


r  7
1

r

50
 7
1

2  14(r  7)

r2  50  14r  98
r2  14r  48  0
(r  6)(r  8)  0
r  6  0 or r  8  0
r6
r8
The solutions are 6 and 8.

626

12

a  3
1

50.

x(x  6)
1

x (x  6)
1

x  2
x
1

2 1


x  2
x  3
x6
x
x  2
x  3
x6
x
1

x(x  6)
1

x  3
 6

x

1
x

x (x  6)
1

54. Let c be the length of the ladder which will be the


hypotenuse of the right triangle formed by the
house, the ground, and the ladder. The angle
formed by the ladder and the ground can be at
most 75; this angle is opposite the side of the
triangle formed by the house, which is 24 feet.

2  x(x  6) 11x 2
2

x
1

(x  6)(x  2)  x(x  3)  x  6
(x2  8x  12)  (x2  3x)  x  6
x2  8x  12  x2  3x  x  6
5x  12  x  6
6x  18
x3
The solution is 3.
51.
14

2x  3
x
2
7
2x  3
x
2
7

x  3
14
x  3
14 14

2 1

sin75 
sin75 

c sin75  24
24

c  sin75
c  24.8466
The shortest ladder he should buy is 25 feet long.
55. Since the measure of the longest side is 9, let
c  9, a  5, and b  7.

1141  2x 7 3 2  1141  2x 2  141  x 14 3


2

c2  a2  b2
?
92  52  72
?
81  25  49
81  74
Since c2  a2  b2, the triangle is not a right
triangle.

2(2x  3)  7x  x  3
4x  6  7x  x  3
3x  6  x  3
4x  9
9

x  4

56. Since the measure of the longest side is 3134, let


c  3134, a  9, and b  15.
c2  a2  b2

The solution is 4.


52.

n(n  1)
1

n(n  1)
1

5n
n  1
1

21

5n
1
n
n  1
5n
1
n
n  1

5

2  n(n  1)(5)

n (n  1)
1

1
n
1

?
(3134) 2  92  152
?

2  5n(n  1)

306  81  225
306  306
Since c2  a2  b2, the triangle is a right triangle.
57. Since the measure of the longest side is 93.6, let
c  93.6, a  36, and b  86.4.
c2  a2  b2
?
93.62  362  86.42
?
8760.96  1296  7464.96
8760.96  8760.96
Since c2  a2  b2, the triangle is a right triangle.
Exercises 5863 For checks, see students work.
58. (x  6) 2  4
x  6  14
x  6  2
x6  2
x62 x62
8
4
{4, 8}

n(5n)  (n  1)  5n2  5n
5n2  n  1  5n2  5n
n  1  5n
1  4n

56

1
4

n

7
3

1
4

53.

a  2
a  2

3(a  2) (a  2)

a

2
 2

1aa  22  a 2 2 2  3(a  2) (a  2) 173 2

13(a  2)1(a  2)  aa  22 2  13(a  2)1(a  2)  (a 2 2) 2


1

opposite leg
hypotenuse
24
c

3 (a  2) (a  2)
1

7
3
1

3(a  2) (a  2)  3(a  2) (2)  (a  2)(a  2) (7)


3(a2  4a  4)  6(a  2)  7(a2  4)
3a2  12a  12  6a  12  7a2  28
3a2  6a  24  7a2  28
10a2  6a  4  0
2(5a2  3a  2)  0
2(5a  2) (a  1)  0
5a  2  0 or a  1  0
5a  2
a  1

59.

x2  121  22x
x  22x  121  0
x2  2(x)(11)  112  0
(x  11) 2  0
x  11  0
x  11
{11}
2

a5
2

The solutions are 1 and 5.

627

Chapter 14

60.

4x2  12x  9  0
(2x)  2(2x) (3)  32  0
(2x  3) 2  0
2x  3  0
2x  3

11. List the possible outcomes. Let J represent a win


for the Jackals and T a win for the Tigers.
If either team wins the first three games, the last
two games will not be played. Thus, two possible
outcomes are JJJ and TTT.
If either team achieves its third victory in the
fourth game, the fifth game will not be played.
Six more possible outcomes are JJTJ, JTJJ, TJJJ,
TTJT, TJTT, and JTTT.
If all five games are played, the possible outcomes
are JJTTJ, JTJTJ, JTTJJ, TJJTJ, TJTJJ, TTJJJ,
TTJJT, TJTJT, TJJTT, JTTJT, JTJTT, and JJTTT.
There are 20 possible outcomes.

61.

x  2

532 6

25x2  20x  4
25x  20x  4  0
(5x) 2  2(5x) (2)  22  0
(5x  2) 2  0
5x  2  0
5x  2
2

62.

12. P 
n r

P 

x  5

525 6

4 2




49x2  84x  36  0
(7x)  2(7x) (6)  62  0
(7x  6) 2  0
7x  6  0
7x  6
2

C 

4 4

6
7


2

63. 180x  100  81x


0  81x2  180x  100
0  (9x) 2  2(9x)(10)  102
0  (9x  10) 2
0  9x  10
10  9x

5 6

654
3!
7!

7!

16. ( P )( P )  (7  3)!  (7  2)!


7 3 7 2
7! 7!
 4!  5!
 76576
 8820
3!

2.
4.
6.
8.

4!

17. ( C ) ( P )  (3  2)!2!  (4  1)!


3 2 4 1
3! 4!
 1!2!  3!
3

 14

factorial
1
mutually exclusive
Theoretical

 12
18. P(blue, red, green)
 P(blue)  P(red)  P(green)
22

30

22

 74  73  72
14,520

605

 388,944 or 16,206

Lesson-by-Lesson Review

19. P(red, red, blue)


 P(red)  P(red)  P(blue)

9. There are a total of 10 videos, so she has


10 choices for the first video. Then after she has
chosen the first video, she has only 9 videos from
which to choose the second video. Then she has
8 choices for the third video. Use the
Fundamental Counting Principle.
10  9  8  720
There are 720 possible outcomes.
10. Use the Fundamental Counting Principle.
12  8  10  5  4800
There are 4800 possible outcomes.
Chapter 14

6!
 3)!3!

 140

Vocabulary and Concept Check

Pages 789792

n!
(n  r)!r!
4!
(4  4)!4!
4!
0!4!
4!
or 1
4!

 1

x

permutation
independent
are not
1

7!

Chapter 14 Study Guide and Review


1.
3.
5.
7.

15. ( C ) ( C )  (7  1)!1!  (6
7 1 6 3
7!
6!
 6!1!  3!3!

10
9

Page 789

C 

8 3

n!
(n  r)!r!
8!
(8  3)!3!
8!
5!3!
8  7  6  5!
5! 3!
876
3!

 56
14. C 
n r

10
9

13. C 
n r

 4  3 or 12

x7

56

n!
(n  r)!
4!
(4  2)!
4!
2!
4  3  2!
2!

30

29

22

 74  73  72
19,140

1595

 388,944 or 32,412
20. P(red, green, not blue)
 P(red)  P(green)  P(not blue)
30

22

50

 74  73  72
33,000

1375

 388,944 or 16,206

628

21. P(diamond or club)


 P(diamond)  P(club)
13

13

26
52

1
2

Chapter 14 Practice Test

 52  52


or

Page 793
1. permutation
3. random variable
4.

22. P(heart or red)


 P(heart)  P(red)  P(heart and red)
13

26

Clifton-2
Bakersville-1
Clifton-3

 52 or 2

Clifton-4
Ashville

23. P(10 or spade)


 P(10)  P(spade)  P(10 and spade)



4
52
16
52

13
 52
4
or 13

Clifton-1
Clifton-2
Bakersville-2

1
52

Clifton-3
Clifton-4

Probability

Derry1
Derry2
Derry1
Derry2
Derry1
Derry2
Derry1
Derry2
Derry1
Derry2
Derry1
Derry2
Derry1
Derry2
Derry1
Derry2

A-B1-C1-D1
A-B1-C1-D2
A-B1-C2-D1
A-B1-C2-D2
A-B1-C3-D1
A-B1-C3-D2
A-B1-C4-D1
A-B1-C4-D2
A-B2-C1-D1
A-B2-C1-D2
A-B2-C2-D1
A-B2-C2-D2
A-B2-C3-D1
A-B2-C3-D2
A-B2-C4-D1
A-B2-C4-D2

5. We can either count the outcomes from the tree


diagram or use the Fundamental Counting
Principle.
2  4  2  16
There are 16 different routes from Ashville to
Derry.
6. Combination; order is not important. We find the
number of ways the students could choose 6 of
9 chairs.

24. For each value of X, the probability is greater


than or equal to 0 and less than or equal to 1.
0.04  0.12  0.37  0.30  0.17  1, so the
probabilities add up to 1.
25. P(1 X 3)
 P(X  1)  P(X  2)  P(X  3)
 0.12  0.37  0.30
 0.79 or 79%
26. Extracurricular Activities
0.4
0.3
0.2
0.1
0

Clifton-1

13

 52  52  52
26

2. experimental

C 

n r

C 

9 6




0
1
2
3 4
X  Number of Activities

n!
(n  r)!r!
9!
(9  6)!6!
9!
3!6!
9  8  7  6!
3!6!
987
3!

 84
There are 84 ways to choose 6 of the 9 chairs.
7. Permutation; order is important. We find the
number of permutations of 10 participants taken
4 at a time.

27. Let R represent red and P represent pink. For a


single flower produced, P(R)  0.75 and
P(P)  0.25.
P(4 red, 1 pink)  P(RRRRP)  P(RRRPR)
 P(RRPRR)  P(RPRRR)
 P(PRRRR)
 P(R)P(R)P(R)P(R)P(P)
 P(R)P(R)P(R)P(P)P(R)
 P(R)P(R)P(P)P(R)P(R)
 P(R)P(P)P(R)P(R)P(R)
 P(P)P(R)P(R)P(R)P(R)
 (0.75)(0.75)(0.75)(0.75)(0.25)
 (0.75)(0.75)(0.75)(0.25)(0.75)
 (0.75)(0.75)(0.25)(0.75)(0.75)
 (0.75)(0.25)(0.75)(0.75)(0.75)
 (0.25)(0.75)(0.75)(0.75)(0.75)
 0.07910  0.07910  0.07910
 0.07910  0.07910
 0.396 or 39.6%
28. Sample answer: There are six possible outcomes,
so you could use a die.
29. Of the 80 total observations, 23 have 3 red and
2 pink.

P 

n r

P 

10 4




n!
(n  r)!
10!
(10  4)!
10!
6!
10  9  8  7  6!
6!

 10  9  8  7 or 5040
There are 5040 different ways that 10 participants
can finish first, second, third, and fourth.
8. Permutation; since each has a separate
responsibility, order is important. We find the
number of permutations of 15 girls taken 2 at a
time and 19 boys taken 2 at a time and use the
Fundamental Counting Principle.

115P22119P22  (1515! 2)!  (1919! 2)!


15!

19!

 13!  17!


23

P(3 red, 2 pink)  80

15  14
1

19  18
1

 71,820
There are 71,820 possible committees.

 0.2875 or about 28.8%

629

Chapter 14

9. These are dependent events.


P(blue, green)  P(blue)  P(green)
6

19. First find P(queen or red). These events are


inclusive.
P(queen or red)
 P(queen)  P(red)  P(queen and red)

 16  15
12

 240 or 20
10. These are dependent events.
P(yellow, yellow)  P(yellow)  P(yellow)
4

 240 or 20

20. First find P(ace or heart) after a black 10 has


been selected and not replaced.
P(ace or heart)
 P(ace)  P(heart)  P(ace and heart)

 3360 or 35
12. These are dependent events.

16

P(black 10, ace or heart)


 P(black 10)  P(ace or heart)

 3360 or 35

13. These are independent events.


P(yellow, 4)  P(yellow)  P(4)
1

32

21. To find the probability for each number of heads,


divide the number of outcomes given by 16, the
total number of outcomes.

 48
14. These are independent events.
P(red, even)  P(red)  P(even)

Four Coins Tossed


X  Number of Heads Probability
0
0.0625
1
0.25
2
0.375
3
0.25
4
0.0625

 86
1

 48 or 16
15. These are independent events.
P(purple or white, not prime)
 P(purple or white)  P(not prime)
2

 86
6

22. From the probability distribution, we see that


P(X  0)  0.0625 or 6.25%.
23. P(X
2)  P(X  2)  P(X  3)  P(X  4)
 0.375  0.25  0.0625
 0.6875 or 68.75%
24. When there are two tails, there are also two heads.
P(two tails)  P(two heads)
 P(X  2)
 0.375 or 37.5%
25. D; Three numbers a, b, and c can be arranged in
6 different ways.
a, b, c
a, c, b
b, a, c
c, a, b
b, c, a
c, b, a

 48 or 8
16. These are independent events.
P(green, even or less than 5)
 P(green)  P(even or less than 5)
1

 86
5

 48
17. These are dependent events.
P(club, heart)  P(club)  P(heart)
13

13

 52  51
169

13

 2652 or 204
18. These are dependent events.
P(black 7, diamond)  P(black 7)  P(diamond)
2

13

 52  51
26

 2652 or 102

Chapter 14

 2652 or 663

16

 52  51

 86

 51

12

 16  15  14
288

13

 51  51  51

P(blue, red, not green)  P(blue)  P(red)  P(not green)


6

 663

 16  15  14
96

 13  51

11. These are dependent events.


P(red, blue, yellow)  P(red)  P(blue)  P(yellow)
6

28

P(queen or red, jack of spades)


 P(queen or red)  P(jack of spades)

26

 52 or 13

 16  15
12

 52  52  52

630

9. D; There are 26 choices for each character. Use


the Fundamental Counting Principle.
26  26  26  17,576
There are 17,576 possible passwords.
10. Solve the system of equations.
x  4y  0
2x  3y  11
Solve the first equation for x.
x  4y  0
x  4y
Substitute 4y for x in the second equation.
2x  3y  11
2(4y)  3y  11
8y  3y  11
11y  11
y  1
Use x  4y to find x.
x  4y
 4(1) or 4
The intersection point is (4, 1).

Chapter 14 Standardized Test Practice


Pages 794795
1. D; Since the average of a and b is 20, we have
a  b
 20 or a  b  40. Since the average of a,
2
a  b  c
b, and c is 25, we have
 25 or
3
a  b  c  75. Substitute 40 for a  b and solve
for c.
a  b  c  75
40  c  75
c  35
2. D; Since the formula for the volume of a cube is
V  s3 and the volume of the cube is 27 cubic
inches, s3  27 where s is the length of an edge of
the cube. Thus, s  3.
surface area  6s2
 6(3) 2
 6(9) or 54
The surface area of the cube is 54 square inches.
3. B; Assuming that the truck is traveling at a
constant speed, the truck is halfway between the
1
two towns after 2 hour. Thus, the car has traveled
1
for 2 hour when the truck reaches Newton.
d  rt

 4x  4x  1  1
 4x2  4x
The equation is true for all values of x.
12. Determine whether c2  a2  b2.
c2  a2  b2

 60  2
 30
The car has traveled 30 miles.
4. A; One point on the graph appears to have
coordinates (70, 50). Check (70, 50) in each
equation. The only equation given that has
1
(70, 50) as a solution is y  2x  15.
5. C; There are eight possible outcomes. Three of the
outcomes have exactly one boy, since the boy
could be born first, second, or third.

?
( 174) 2  52  72
?

74  25  49
74  74
The triangle is a right triangle with hypotenuse c.
Thus, the angle opposite side c measures 90.
13. There are 9 possible digits for each of the first
and last digits of the four digits. There are 10
possible digits for each of the remaining two
digits. Use the Fundamental Counting Principle.
9  10  10  9  8100
There are 8100 telephone numbers available.

P(exactly one boy)  8


6. C; 52  52 or 25
1

1
1
11. 4 x  2 2  1  4 x2  x  4  1

x2  x  20
x  x  20  0
(x  5)(x  4)  0
or x  4  0
x50
x  5
x4
The solutions are 5 and 4.
8. B; The ordered pairs (2, 2) and (7, 8) describe the
locations of the planes relative to the airport. Use
the distance formula to find the distance between
the planes.

7. D;

14. On a single roll, P(left)  6 or 3 and


4
2
P(right)  6 or 3. These are independent events.
There are two ways to reach the goal in two turns:
first go left then right or first go right then left.
P(reach goal in two turns)
 P(left the right)  P(right then left)
 P(left) P(right)  P(right) P(left)

 33  33
2

 9  9 or 9

d  2(x2  x1 ) 2  (y2  y1 ) 2

15. A; 3x  15 7
3x 7
x 7
2y  3 7
2y 7
y 6
Thus, x 7 10

d  2(8  2) 2  (7  2) 2
d  262  52
d  161 or about 7.8
The planes are about 7.8 miles apart.

631

45
30
10
17
20
10
7 y.

Chapter 14

18a. 10 Random Specials are possible


You can list all of the combinations, using
letters for each topping.
PS PO PM PG
SO SM SG
OM OG
MG
There are 10 possible combinations.
18b. Of the ten combinations, four include mushrooms.
The probability of mushrooms on a Random
4
Special pizza is 10 or 40%.

12!
 4)!

16. A; 12P4  (12


12!
 8!

 12  11  10  9 or 11,880
C 

10 6




10!
(10  6)!6!
10!
4!6!
10  9  8  7
4!

or 210

Thus, 12P4 7 10C6.


17. B; These are dependent events.
P(blue, green)  P(blue)  P(green)
7

 14  13
14

 182 or about 0.08


P(red, red)  P(red)  P(red)
5

 14  13
20

 182 or about 0.11


Thus, P(blue, green) P(red, red).

Chapter 14

632

PQ249-6481F-PR[633-642] 26/9/02 8:20 PM Page 633 Sahuja Ahuja_QXP_06:Desktop Folder:Chandra:Algebra_FNL_Delivery:

Prerequisite Skills
Page 799

Operations with Fractions: Adding


and Subtracting

1.

2
5

5

3.

4
3

4
3







5.

5
16

2  1
5
3
5


7.

6
9

9.

28
40

11.

27
99

13.

2
9

2
3

7
2

10.

27  9

 99  9

12.

24
180

3
11

14.

2
15

16.

17.

1
9




19.

1
2

4
3

9


23.

1
4

20.

5
4

3
4

2
3

26.

8
9

27.

3
7

28.

15.




31.

3
4

30.

3
5

32.

4
15

33.

4
15





55
60
39
60
13
20

34.

1
8

 15 


3
2

15
2
7
2

9
4

12.

 15
1

or 32

3  23

1
3

5  35
1

5
1

 18  4  18

14.

11
3

 44 

11
3

 44

4

2
7

14
3

9
10
3
10

24
88
35
88

 7
1

4
3

10

14
3

or

1
3

16.

2
11

110
17

1
13

 11 
1

12

20
17

18.

1
3

15
2

4
19

30

 3
1

10

30
11

110
17
3

or 117

12

 19  3  19

 3  11  3

20.

6
5

10

5
2

15
2
1

or 22

10

 12  5  12

10

2

 11

1
21.


6
10

45

7
6

1

3
2

23.

11
88

1
22

22
1

3
2

is 3.

14
23

1
1
22

is 22.

14
23

is 14 or 114.

23

 14  1

The reciprocal of

633

is 6 or 16.

The reciprocal of
24.

6
7

3  1

The reciprocal of

6
7

The reciprocal of
22.

16

15
2

1
9


19.

61

3
11

10.

 60 or 160
16
60

17.

 4  60  60

 12

21

 11  3  11
2

13

 3  12  12

11
12

9
10

2
3

or 54

 33

2  92

29. 1 

2
9

 20
1
19

8.

4
19
19
18
19

12

21
4

5  35

 5  20  20

11

73

2  22

 14
1
19

1
3

399

13
20

7
2

8

6.

13.

9

 14  14  14

51

1
6

2

 4  12  12
2

23

7  57

2

11

2
5

 35

4  24

0
24.

4.

 12

22

5
2


11.

3
2

3
5

22. 12  2  2  2

 5  20  20

13

 20  5  20

9.

366

 20
25.

 21

1
2
2
4
1
2

or 19

21

3  73

 15

9

7.

12
9
17
9

2
7

3
4

2.

8

88

1
2

5.

 15  15

1
5

2
15

444


21.

18.

3.

24  12

3
9
1
3

31

5  45
3

 180  12

1
4
9

3
4

4
25

7
8

125

Operations with Fractions: Multiplying


and Dividing

 20

16  4

18

143

 6  150  150
 150

 100

1
3

33

1.

 100  4

99

Page 801

1
2

3
25

36.

7  7

7
10

44

2

 14  7

16
100

94
50

7  4
2
3
1
or 12
2

7
14

11

 25  100  100
 100

3  4
9
7
9

2

28  4


15.

8.

 40  4

4
9

94
100

2  1
7
1
7

6  3
 3
2
3

4.

3
9

6.

9


7

5  4
16
1
16

2
7

4  4
3
8
2
or 23
3

 16 

2.

35.

23

Prerequisite Skills

PQ249-6481F-PR[633-642] 26/9/02 8:20 PM Page 634 Sahuja Ahuja_QXP_06:Desktop Folder:Chandra:Algebra_FNL_Delivery:

24 

25.
11
4

11
4

11. The part is 16, and the base is 40. Let p represent
the percent.

 11  1
3

a
b
16
40

The reciprocal of 24 is 11.


26.

1
53
16 3

3 16

16
3

2
3

1600
40

 3  31

28.

16
9

30.

3
7

3
2

 2  21

5

 2 or 3
31.

9
10

 7  10  3

32.

1
2

5

63

 30
21

16 9

9 4
144
 36 or 4
3 5
 71
15
1
 7 or 27
1 5
 23
5
6

a
b
14
b

33.

35.

37.

1
2

9
1
 42
9 2
 41
18
 4
9
1
 2 or 42
11
2
11
5
 13  12  3
12
11 3
 12  5
33
11
 60 or 20
1
1
1
6
 15  3  5
3
1 5
 36
5
 18

Page 803
1.
3.
5.

2
3





36.

38.

3
8

3
25

1
4

1400
20

4
2
3
3
4 3

3 2
12
or 2
6

3 4
 81
12
 8
3
 2 or
2
3
 25
15
45
 50

0.9

2.
4.
6.

8.

9
 1000
1400
1400%  100

a
b
80
b

8000
50

a
b
a
18

60
3
60%  100 or 5
120
6
120%  100 or 5
2.5
2.5%  100
25
1
 1000 or 40
0.4
0.4%  100
4
1
 1000 or 250

100a
100

 100
50

 100

50b
50

 100
25

 100

450

 100

a  4.5
4.5 is 25% of 18.
15. The percent is 10, and the base is 95. Let a
represent the part.
a
b
a
95

or 14

 100
10

 100

100a  95(10)
100a  950

100a
100

 100

950

 100

a  9.5
9.5 is 10% of 95.

125p
125

20  p
25 is 20% of 125.

Prerequisite Skills

20b
20

100a  18(25)
100a  450

 100

160  b
80 is 50% of 160.
14. The percent is 25, and the base is 18. Let a
represent the part.

or 10

25(100)  125p
2500  125p
2500
125

20

 100

80(100)  50b
8000  50b

1
12
15
 2

10. The part is 25, and the base is 125. Let p


represent the percent.
a
b
25
125

 100

70  b
14 is 20% of 70.
13. The part is 80, and the percent is 50. Let b
represent the base.

The Percent Proportion

5
1
5%  100 or 20
11
11%  100
78
39
78%  100 or 50

7. 0.9%  100

9.

34.

1
13

40p
40

14(100)  20b
1400  20b

 10 or 210
1
24

40  p
16 is 40% of 40.
12. The part is 14, and the percent is 20. Let b
represent the base.

9

 3 or 2
29.

 100

16(100)  40p
1600  40p

1

The reciprocal of 53 is 16.


27.

 100

634

PQ249-6481F-PR[633-642] 26/9/02 8:20 PM Page 635 Sahuja Ahuja_QXP_06:Desktop Folder:Chandra:Algebra_FNL_Delivery:

21. The part is 49, and the percent is 200. Let b


represent the base.

16. The part is 30, and the base is 48. Let p represent
the percent.
a
b
30
48

a
b
49
b

 100
p

 100

30(100)  48p
3000  48p
3000
48

4900
200

62.5  p
30 is 62.5% of 48.
17. The percent is 150, and the base is 32. Let a
represent the part.
a
b
a
32

200

 100

49(100)  200b
4900  200b

48p
48

 100

200b
200

24.5  b
49 is 200% of 24.5.
22. The part is 15, and the base is 12. Let p represent
the percent.

 100

a
b
15
12

100a  32(150)
100a  4800

15(100)  12p
1500  12p

100a
100

 100
150

1500
12

4800
100

a
b
48
32

 100
5

 100

4800
32

5b
5

a
b
a
20

 100
p

 100

100a
100

400p
400




a
b
36
40

p
100
0.5
100

32p
32

 100
85

 100


1700
100

 100
p

 100

3600  40p

100a  250(0.5)
100a  125
100a
100

 100

a  17
Madeline will likely make 17 of the 20 shots.
25. The part is 36, and the base is 40. Let p represent
the percent.

0.25  p
1 is 0.25% of 400.
20. The percent is 0.5, and the base is 250. Let a
represent the part.
a
b
a
250

 100

100a  1700

1(100)  400p
100  400p
100
400

12p
12

150  p
48 is 150% of 32.
24. The percent is 85, and the base is 20. Let a
represent the part.

70  b
3.5 is 5% of 70.
19. The part is 1, and the base is 400. Let p represent
the percent.
a
b
1
400

48(100)  32p
4800  32p

3.5(100)  5b
350  5b
350
5

 100

125  p
15 is 125% of 12.
23. The part is 48, and the base is 32. Let p represent
the percent.

a  48
48 is 150% of 32.
18. The part is 3.5, and the percent is 5. Let b
represent the base.
a
b
3.5
b

 100

3600
40

40p
40

90  p
Brian answered 90% of the questions correctly.

125

 100

a  1.25
1.25 is 0.5% of 250.

635

Prerequisite Skills

PQ249-6481F-PR[633-642] 26/9/02 8:20 PM Page 636 Sahuja Ahuja_QXP_06:Desktop Folder:Chandra:Algebra_FNL_Delivery:

26. The part is 4, and the percent is 80. Let b


represent the base.
a
b
4
b

7.

3
10

80

 100


25

13. Let N  0.6 or 0.666 p .


Then 10N  6.666 p
10N  6.666 p
 1N  0.666 p
9N  6
6

So, 0.6  3.

15

 100

34

17

14. 0.0034  10,000 or 5000


8

15. 2.08  2100 or 225

15b
15

16. 0.004  1000 or 250

20  b
The recommended number of grams of saturated
fat is 20.
29. The part is 470, and the percent is 20. Let b
represent the base.
p

 100
20

 100

47,000  20b
20b
20

2350  b
The recommended daily value of sodium is
2350 mg or 2.35 g.
30. The part is 110, and the base is 250. Let p
represent the percent.

17.
19.
21.
23.
25.
27.
29.
31.

0.4  40%
2.5  250%
0.065  6.5%
0.005  0.5%
45%  0.45
68%  0.68
200%  2
5.2%  0.052

18.
20.
22.
24.
26.
28.
30.
32.

0.08  8%
0.33  33%
5  500%
0.3  33.3%
3%  0.03
115%  1.15
0.1%  0.001
10.5%  0.105

33.

3
4

34.

9
20

 0.75
 75%

35.

1
2

37.

1
3

 0.5

39.

11,000  250p

 0.3333 p

6
5

 1.2

40.

70

43. 52%  0.52

Expressing Fractions as Decimals


and Percents

38

2.

2
5

 0.375
3.

2
3

23
 0.666 p or 0.6

Prerequisite Skills

25

4.

 1000

34
 0.75

636

 0.76

 100
25

 100 or 4
46. 135%  1.35

3
50

47. 0.1%  0.001

 0.4
3
4

or

19
25

44. 25%  0.25


13

45. 6%  0.06
6
100

 0.875

 76%
42. 3%  0.03

 100 or 10
52

1.

38.

7
8

 0.1666 p

 87.5%

 100 or 25

3
8

1
6

 16.7%

 120%
41. 70%  0.70

250p
250

44  p
44% of the Calories come from fat.

Page 805

36.

 33.3%

 100

 0.45
 45%

 50%

 100

N  9 or 3

11,000
250

So, 0.45  11.

 100

a
b
110
250

45

120p
120

N  99 or 11

47,000
20

56

10. 0.25  100 or 4

 100

a
b
470
b

5
6

12. Let N  0.45 or 0.4545 p .


Then 100N  45.4545 p
100N  45.4545 p
 1N  0.4545 p
99N  45

8.

 0.8333 p or 0.83

9
10
24

300  15b
300
15

59

11. 5.24  5100 or 525

5p
The solution is 5% glucose.
28. The part is 3, and the percent is 15. Let b
represent the base.
a
b
3
b

5
9

 0.555 p or 0.5

 3  10

9. 0.9 

80b
80

 100

6.

 0.3

600  120p
600
120

12
 0.5

5b
Jos played 5 games of solitaire.
27. The part is 6, and the base is 120. Let p represent
the percent.
a
b
6
120

1
2

 100

400  80b
400
80

5.

35

 1100 or 120
48. 0.5%  0.005


5
1000

or 200

PQ249-6481F-PR[633-642] 26/9/02 8:20 PM Page 637 Sahuja Ahuja_QXP_06:Desktop Folder:Chandra:Algebra_FNL_Delivery:

Page 807

Making Bar and Line Graphs

Page 809

1. A line graph is the better choice since we are


looking at how one quantity, the size of the plant,
changes over time.
2. A bar graph is the better choice since we are
comparing three different quantities, the
populations of Idaho, Montana, and Texas.
3. A bar graph is the better choice since we are
making a comparison of the number of students
in several classes.
4. A line graph is the better choice since we are
looking at how a single quantity, your height, has
changed over time.
5. A bar graph is the better choice since we are
comparing the number of people in two categories,
those who shower in the morning and those who
shower at night.
6.
Hours of Sleep
10
Number of Hours

Making Circle Graphs

1. Find the number of degrees for each continent


or region.
North America:
7.9% of 360  0.079  360
 28
South America:
5.7% of 360  0.057  360
 21
Europe:
12.0% of 360  0.12  360
 43
Asia:
60.7% of 360  0.607  360
 219
Africa:
13.2% of 360  0.132  360
 48
Australia:
0.5% of 360  0.005  360
 2
Antarctica:
0% of 360  0
Due to rounding, the angles do not total 360.
Make the circle graph.

8
6
4
2

World Population, 2000

0
Alana Kwam Tomas Nick
Student

Kate Sharla

Lawn Care Profits

7.

Asia
60%

North
America
8%
Europe
South
12%
America
6%
2. Find the number of degrees for each part of the
body. There are a total of 206 bones in the body.

Profits ($)

200
150
100
50
0
1 2 3 4 5 6 7 8 9 10111213 1415
Week

8.

29
 360  51
206
26
Spine: 206  360  45

Skull:

Play Attendance
Number of People

Africa Australia
13%
1%

25
206

 360  44

50

Ribs and Breastbone:

40

Shoulders, Arms, and Hands:


64
206

30

 360  112

Pelvis, Legs, and Feet:

20

62
206

 360  108

Make the circle graph.


10

Bones in Parts of the Human Body

0
Under 20-39 40-59 60 and
20
over
Age

Shoulders,
Arms,
Hands,
64

Ribs, Breast bones


25

637

Pelvis,
Legs,
Feet,
62
Skull,
29
Spine,
26

Prerequisite Skills

PQ249-6481F-PR[633-642] 26/9/02 8:20 PM Page 638 Sahuja Ahuja_QXP_06:Desktop Folder:Chandra:Algebra_FNL_Delivery:

Page 811

Identifying Two-Dimensional Figures

Page 814

1. The triangle has all acute angles and no


congruent sides. It is an acute scalene triangle.
2. The triangle has all acute angles and all sides
congruent. It is an acute equilateral triangle.
3. The triangle has one obtuse angle and two
congruent sides. It is an obtuse isosceles triangle.
4. The triangle has one right angle and no
congruent sides. It is a right scalene triangle.
5. The figure has six sides. It is a hexagon.
6. The figure has five sides. It is a pentagon.
7. The figure has four sides with opposite sides
congruent and four right angles. It is a
quadrilateral, a parallelogram, and a rectangle.
8. The figure has four sides with opposite sides
parallel, four congruent sides, and four right
angles. It is a quadrilateral, a parallelogram, a
rectangle, a rhombus, and a square.
9. The figure has four sides with opposite sides
parallel. It is a quadrilateral and a parallelogram.
10. The figure has four sides with one pair of opposite
sides parallel. It is a quadrilateral and a trapezoid.
11. The figure has four sides with opposite sides
congruent and four right angles. It is a
quadrilateral, a parallelogram, and a rectangle.
12. The figure has four congruent sides. It is a
quadrilateral, a parallelogram, and a rhombus.
13. The figure has eight sides. It is an octagon.
14. The figure has four sides, none of which are
either parallel or congruent. It is a quadrilateral.
15. The figure has five sides. It is a pentagon.

Page 812

1. P  2(/  w)
 2(3  2)
 2(5)
 10
A  /w
 32
6
The perimeter is 10 centimeters, and the area is
6 square centimeters.
2. P  4s
 4(1)
4
A  s2
 12
1
The perimeter is 4 inches, and the area is
1 square inch.
3. P  2(/  w)
 2(1  7)
 2(8)
 16
A  /w
 17
7
The perimeter is 16 yards, and the area is
7 square yards.
4. P  4s
 4(7)
 28
A  s2
 72
 49
The perimeter is 28 kilometers, and the area is
49 square kilometers.
5. P  2(6  4)
 2(10)
 20
A  64
 24
The perimeter is 20 feet, and the area is
24 square feet.
6. P  2(12  9)
 2(21)
 42
A  12  9
 108
The perimeter is 42 centimeters, and the area is
108 square centimeters.
7. P  4(3)
 12

Identifying Three-Dimensional Figures

1. The figure has two parallel, congruent faces that


are rectangles. It is a rectangular prism.
2. The figure has one base that is a triangle and three
faces that are triangles. It is a triangular pyramid.
3. The figure is a sphere.
4. The figure has two parallel, congruent faces that
are triangles. It is a triangular prism.
5. The figure has one base that is a rectangle and
four faces that are triangles. It is a rectangular
pyramid.
6. The figure has a circular base and one vertex. It
is a cone.
7. The figure is a rectangular prism in which all of
the faces are squares. It is a cube.
8. The figure has two parallel, congruent faces that
are triangles. It is a triangular prism.
9. The figure has one base that is a rectangle and
four faces that are triangles. It is a rectangular
pyramid.

Prerequisite Skills

Perimeter and Area of Squares


and Rectangles

A  32
9
The perimeter is 12 meters, and the area is
12 square meters.

638

PQ249-6481F-PR[633-642] 26/9/02 8:20 PM Page 639 Sahuja Ahuja_QXP_06:Desktop Folder:Chandra:Algebra_FNL_Delivery:

8. P  4(15)
 60

Page 816

A  152
 225
The perimeter is 60 inches, and the area is
225 square inches.

1  112
1
 2 1 192 2
39
 21 2 2

9. P  2

1
82

 39
1

A  82  11



17 11
 1
2
187
or
2

932

The perimeter is 39 inches, and the area is


1
932 square inches.
1
1 1
2
1
2
 2 1 124  144 2
3
 2 1 264 2
107
 21 4 2

10. P  2 124  142

107
2

or 532

49 29
 2
4
1421
or
8

is about 18.8 meters.

is about 31.4 inches.

is about 75.4 centimeters.

is about 9.4 kilometers.

is about 3.1 yards.

1 12
1
 1 54 2 

A  124  142


Area and Circumference of Circles

1. C  2r
 2(3)
 6
 18.8
The circumference
2. C  d
 (10)
 10
 31.4
The circumference
3. C  2r
 2(12)
 24
 75.4
The circumference
4. C  2r
 2(1.5)
 3
 9.4
The circumference
5. C  d
 (1)

 3.1
The circumference
6. C  d
  54

1778

 16.5
The circumference is about 16.5 feet.
7. C  2r

The perimeter is 532 feet, and the area is


5
1778 square feet.

1 12

11. P  4(2.4)
 9.6
A  (2.4) 2
 5.76
The perimeter is 9.6 centimeters, and the area is
5.76 square centimeters.
12. P  4(5.8)
 23.2
A  (5.8) 2
 33.64
The perimeter is 23.2 meters, and the area is
33.64 square meters.
13. P  2(18  90)
 2(108)
 216
A  18  90
 1620
The perimeter for each plot of the garden is
216 feet, and the area is 1620 square feet.

 2 242

 49
 153.9
The circumference is about 153.9 inches.
8. A  r2
 (5) 2
 25
 78.5
The area is about 78.5 square inches.
9. A  r2
 (2) 2
 4
 12.6
The area is about 12.6 square feet.
10. The radius is one-half times the diameter, or
1 km.
A  r2
 (1) 2

 3.1
The area is about 3.1 square kilometers.

639

Prerequisite Skills

PQ249-6481F-PR[633-642] 26/9/02 8:20 PM Page 640 Sahuja Ahuja_QXP_06:Desktop Folder:Chandra:Algebra_FNL_Delivery:

11. The radius is one-half times the diameter, or 2 yd.


A  r2
 (2) 2
 4
 12.6 yd2
The area is about 12.6 square yards.

Page 817

 252

2.

12. A  r2
 (1) 2

 3.1
The area is about 3.1 square meters.
13. A  r2
 (1.5) 2
 2.25
 7.1
The area is about 7.1 square feet.
14. The radius is one-half the diameter, or 7.5 cm.
A  r2
 (7.5) 2
 56.25
 176.7
The area is about 176.7 square centimeters.
15. C  2r
25,000  2r
25,000
2

Volume

1. V  /  w  h

3.

4.

5.

6.

2r
2

7.

3979  r
The tunnel would be about 3979 miles.
16. C  d
 (27)
 27
 84.82
The bicycle will travel about 84.82 inches for one
rotation of the tire. Multiply by 10.
10  84.82  848.2
The bicycle will travel about 848.2 inches in
10 rotations of the tire.
17. A  r2
 (2) 2
 4
 13
The area of the region that will benefit from the
system is about 13 square miles.
18. The radius is one-half times the diameter, or
125 feet.
A  r2
 (125) 2
 15,625
 49,087.4
The grass and sidewalk cover about
49,087.4 square feet of area.

8.

5
The volume is 5 cubic inches.
V  /wh
 12  3  2
 72
The volume is 72 cubic centimeters.
V  /wh
 621
 12
The volume is 12 cubic yards.
V  /wh
 100  1  10
 1000
The volume is 1000 cubic meters.
V  /wh
 225
 20
The volume is 20 cubic meters.
V  /wh
 12  6  2
 144
The volume is 144 cubic inches.
V  /wh
 8  5  5.5
 220
The volume of the tank is 220 cubic feet.
V  /wh
1

 18  10  112
 2070
The volume of the microwave oven is 2070 cubic
inches.
9. V  /  w  h
 222
8
The volume of the cube is 8 cubic meters.
10. 8  4  4  128
The volume of a full cord of firewood is 128 cubic
feet.
1

11. 8  4  22  80
1

The volume of a short cord of 22-foot logs is


80 cubic feet.
12. 12  2  h  128
24  h  128
h
h

128
24
16
or
3

53 feet or 5 ft 4 in.

The stack will be 5 feet 4 inches high.

Prerequisite Skills

640

PQ249-6481F-PR[633-642] 26/9/02 8:20 PM Page 641 Sahuja Ahuja_QXP_06:Desktop Folder:Chandra:Algebra_FNL_Delivery:

Page 819

The numbers 201 and 199 each occur twice,


and the number 200 occurs only once. The
numbers that appear most often in the set are
201 and 199. The modes are 201 and 199.
7. {4, 5, 6, 7, 8}

Mean, Median, and Mode

1. {1, 2, 3, 5, 5, 6, 13}
mean 


1  2  3  5  5  6  13
7
35
or 5
7

mean 

The set is arranged in order. The middle number


in the set is 5. The median is 5.
The number that appears most often in the set is
5. The mode is 5.
2. {3, 5, 8, 1, 4, 11, 3}
mean 


The set is arranged in order. The middle number


in the set is 6. The median is 6.
All numbers in the set occur exactly once. There
is no mode.
8. {3, 7, 21, 23, 63, 27, 29, 95, 23}

3  5  8  1  4  11  3
7
35
or
5
7

mean 

Order the numbers from least to greatest.


1, 3, 3, 4, 5, 8, 11
The middle number in the set is 4. The median is 4.
The number that appears most often in the set is
3. The mode is 3.
3. {52, 53, 53, 53, 55, 55, 57}
mean 


52  53  53  53  55  55  57
7
378
or
54
7

mean 


8  7  5  19
4
39
or 9.75
4

15
2

2  2.5
2

or 7.5

mean 


or 2.25

5  7  5  4  3  3  8
7
35
or
5
7

Order the numbers from least to greatest.


3, 3, 4, 5, 5, 7, 8
The middle number in the set is 5. The median is 5.
The numbers 3 and 5 each occur twice, and all
other numbers occur only once. The numbers that
appear most often in the set are 3 and 5. The
modes are 3 and 5.
11. 17.6, 16.0, 14.1, 13.7, 13.5, 12.9, 12.3, 11.6, 11.4, 11.4
mean

3  11  26  4  1
5
45
or
9
5

Order the numbers from least to greatest.


1, 3, 4, 11, 26
The middle number in the set is 4. The median is 4.
All numbers in the set occur exactly once. There
is no mode.
6. {201, 201, 200, 199, 199}
mean 

4.5
2

The median is $2.25.


The amount that appears most often is $2.00. The
mode is $2.00.
10. 5, 7, 5, 4, 3, 3, 8

The median is 7.5.


All numbers in the set occur exactly once. There
is no mode.
5. {3, 11, 26, 4, 1}
mean 

0.5  2  2  1.25  5.25  3  2.5  3.5


8
20
or $2.50
8

Order the numbers from least to greatest.


0.5, 1.25, 2, 2, 2.5, 3, 3.5, 5.25
There is an even number of items. Find the mean
of the middle two.

Order the numbers from least to greatest.


5, 7, 8, 19
There is an even number of items. Find the mean
of the middle two.
7  8
2

3  7  21  23  63  27  29  95  23
9
291
1
or
32
9
3

Order the numbers from least to greatest.


3, 7, 21, 23, 23, 27, 29, 63, 95
The middle number in the set is 23. The median
is 23.
The number that appears most often in the set is
23. The mode is 23.
9. $0.50, $2.00, $2.00, $1.25, $5.25, $3.00, $2.50, $3.50

The set is arranged in order. The middle number


in the set is 53. The median is 53.
The number that appears most often in the set is
53. The mode is 53.
4. {8, 7, 5, 19}
mean 

4  5  6  7  8
5
30
or 6
5

201  201  200  199  199


5
1000
or
200
5

17.6  16.0  14.1  13.7  13.5  12.9  12.3  11.6  11.4  11.4
10

134.5
10

or 13.45

Order the numbers from least to greatest.


11.4, 11.4, 11.6, 12.3, 12.9, 13.5, 13.7, 14.1, 16.0, 17.6
There is an even number of items. Find the mean
of the middle two.

Order the numbers from least to greatest.


199, 199, 200, 201, 201
The middle number in the set is 200. The median
is 200.

12.9  13.5
2

26.4
2

or 13.2

The median is 13.2.


The number that appears most often is 11.4. The
mode is 11.4.

641

Prerequisite Skills

PQ249-6481F-PR[633-642] 26/9/02 8:20 PM Page 642 Sahuja Ahuja_QXP_06:Desktop Folder:Chandra:Algebra_FNL_Delivery:

12. Let x represent Bills score on the fifth test.


mean 
88 
88 

14. mean 

sum of the first four scores  fifth score


5
86  90  84  91  x
5
351  x
5





440  351  x
89  x
Bill must earn 89 on the fifth test.

Olivias new average score will be about 92.7.

13. Let x represent Sues score for the tenth game.


mean 
110 
110 

9(average score of first 9 games)  tenth score


10
9(108)  x
10
972  x
10

1100  972  x
128  x
Sue needs to score 128 in the tenth game.

Prerequisite Skills

5(average score of first 5 tests)  sixth score


6
5(92)  96
6
460  96
6
556
6
2
923 or about 92.7

642

PQ249J-6481F-15-19[643-685] 26/9/02 9:49 PM Page 643 Sahuja Ahuja_QXP_06:Desktop Folder:Chandra:Algebra_FNL_Delivery:

Extra Practice
Page 820

5. (8  1)  3  (7)  3
 21

Lesson 1-1

1. The word sum implies add, so the expression can


be written as b  21.
2. The word product implies multiply, so the
expression can be written as 7x.
3. The words increased by imply add, so the
expression can be written as t  6.
4. Sum implies add, and times implies multiply. So
the expression can be written as 4  6z.
5. Increased by implies add, and times implies
multiply. So the expression can be written as
10  4a.
6. Sum implies add, and times implies multiply. So
the expression can be written as 8  (2n).
7. The word cube means to raise to the third power,
1
so the expression can be written as 2x3.
8. The word square means to raise to the second
4
power, so the expression can be written as 5m2.

6. 4(5  3) 2  4(2) 2
 4(4)
 16
7. 3(12  3)  5  9  3(15)  5  9
 45  45
0
8. 53  63  52  125  216  25
 341  25
 316
9. 16  2  5  3  6  8  5  3  6
 40  3  6
 120  6
 20
10. 7(53  32 )  7(125  9)
 7(134)
 938

9. 24  2  2  2  2
 16
2
10. 10  10  10
 100
3
11. 7  7  7  7
 343
12.

203

11.





36  12
64
48
64
48
24

2
12. 25 

 20  20  20
 8000

13. 36  3  3  3  3  3  3
 729

13.

14. 45  4  4  4  4  4
 1024
15. Sample answer: two times n
16. Sample answer: ten to the seventh power
17. Sample answer: m to the fifth power
18. Sample answer: the product of x and y
19. Sample answer: five times n squared minus 6
20. Sample answer: nine times a cubed plus 1
21. Sample answer: x cubed times y squared
22. Sample answer: c to the fourth power times d to
the sixth power
23. Sample answer: three times e plus 2 times
e squared

Page 820

94  26
64

14.

15.

16.

1
(18
3

 9)  25  3 (27)

 25  9
 16
8a  b  8  2  5
 16  5
 21
48  ab  48  2  5
 48  10
 58
a(6  3n)  2(6  3  10)
 2(6  30)
 2(24)
 48
bx  an  5  4  2  10
 20  20
 40

17. x2  4n  42  4  10
 16  4  10
 16  40
 24
18. 3b  16a  9n  3  5  16  2  9  10
 15  32  90
 47  90
 43

Lesson 1-2

1. 3  8  2  5  3  4  5
75
2
2. 4  7  2  8  4  14  8
 18  8
 26
3. 5(9  3)  3  4  5(12)  3  4
 60  12
 48

19. n2  3(a  4)  102  3(2  4)


 102  3(6)
 100  3(6)
 100  18
 118

4. 9  32  9  9
0

643

Extra Practice

PQ249J-6481F-15-19[643-685] 26/9/02 9:49 PM Page 644 Sahuja Ahuja_QXP_06:Desktop Folder:Chandra:Algebra_FNL_Delivery:

4. Replace y in 5y  4  11 with each value in the


replacement set for y.

20. (2x) 2  an  5b  (2  4) 2  2  10  5  5
 (8) 2  2  10  5  5
 64  2  10  5  5
 64  20  25
 84  25
 59

5y  4  11

y
1
3

21. [a  8(b  2) ] 2  4  [2  8(5  2) ] 2  4


 [2  8(3) ] 2  4
 [2  24] 2  4
 [26] 2  4
 676  4
 169

5
7
9

5  1  4  11 S 1  11
5  3  4  11 S 11  11
5  5  4  11 S 21  11

0
2
4
6
8

0  4  4 S 4  4
?

2  4  4 S 2  4
?

444 S 04
?

644 S 24
?

844 S 44

5  7  4  11 S 31  11

25  y  18
?

25  1  18 S 24  18

25  3  18 S 22  18

5
7
9

?
?

25  5  18 S 20  18
?

25  7  18 S 18  18
?

25  9  18 S 16  18

True or False?
false
false
false
false

5  9  4  11 S 41  11

3x  1  25

0
2

3  0  1  25 S 1  25
?

3  2  1  25 S 7  25
?

3  4  1  25 S 13  25

3  6  1  25 S 19  25

3  8  1  25 S 25  25

?
?

14 

true

96
x

96
2

96
4

96
6

96
x

 2 with each value in the

2

True or False?

 2  14 S undefined

false

 2  14 S 50  14

false

 2  14 S 26  14

false

 2  14 S 18  14

false

96
8

 2  14 S 14  14

The solution of 14 

96
x

true

 2 is 8.

y
3

6. Replace y in 0   3 with each value in the


replacement set for y.
y

033

True or False?
false
false
false
true
false

True or False?

1
3

30 S

3
3

 3  0 S 2  0

5
3

7
3

9
3

2
23

0

false

?
?

false

false

0

false

 3  0 S 13  0
?

30 S

2
3

true

30 S 00
y

The solution of 0  3  3 is 9.
7. x 

True or False?

x

27  9
2

8.

36
2

x  18
The solution is 18.

false
false

6(5)  3
 3
30  3
8  3
33
11

false

9. n  2(4)

false

n
n

true

11.

72  9(2  1)
2(10)  1
49  9(3)
20  1
49  27
19
76
19

t

y

11
11

y

5(4)  6
22  3
20  6
4  3
14
7

z
z
z

2z
The solution is 2.
33  52

12. a  2(3  1)

t

a

t

a

t
4t
The solution is 4.

644

18  7
13  2

1y
The solution is 1.
10.

n3
The solution is 3.

The solution of 3x  1  25 is 8.

Extra Practice

false

96
0

The solution of 25  y  18 is 7.
3. Replace x in 3x  1  25 with each value in the
replacement set for x.
x

false

The solution of x  4  4 is 8.
2. Replace y in 25  y  18 with each value in the
replacement set for y.
y

false

replacement set for x.

1. Replace x in x  4  4 with each value in the


replacement set for x.
x44

true

The solution of 5y  4  11 is 3.

Lesson 1-3

false

5. Replace x in 14 

Page 820

True or False?

27  25
2(2)
52
4

a  13
The solution is 13.

PQ249J-6481F-15-19[643-685] 26/9/02 9:49 PM Page 645 Sahuja Ahuja_QXP_06:Desktop Folder:Chandra:Algebra_FNL_Delivery:

13. Replace x in x  2  7 with each value in the


replacement set for x.
x

x27

42 7 7 S 67

52 7 7 S 77

62 7 7 S 8 7 7

72 7 7 S 9 7 7

8  2 7 7 S 10 7 7

17. Replace x in 3 6 2 with each value in the


replacement set for x.

True or False?
false

6 2 S 2 2

true

7
3

6 2 S 23 2

8
3

6 2 S 23 2

true

true

true

true

71 6 8 S 6 6 8

81 6 8 S 7 6 8

2x  15

True or False?

2  4 15 S 8 15

true

true

6
7
8

2  5 15 S 10 15
?

true

true

2  6 15 S 12 15
2  7 15 S 14 15
?

2  8 15 S 16  15

3y  36

10

3  10
36 S 30 36

12
14
16

true

true

3  14
36 S 42
36
3  16
36 S 48
36

false

5y
4

x
3

6 2 is {4, 5}.

 20

True or False?

10

5  10 ?

20 S 12.5 20

false

12

5  12 ?

20 S 15 20

false

14

5  14 ?

20 S 17.5 20

false

16

5  16 ?

20 S 20
20

true

Page 821

5y
4

20 is {16}.

Lesson 1-4

1. Reflexive Property of Equality


n  3, since 4  3  4  3.
2. Additive Identity Property
5

n  4, since

5
4

 4  0.

3. Multiplicative Identity Property


n  1, since 15  15  1.
4. Multiplicative Inverse Property
3

n  2, since 3  2  1.
5. Commutative Property of Addition
n  1.3, since 2.7  1.3  1.3  2.7.
6. Substitution Property of Equality, Multiplicative
Inverse Property, and Multiplicative Identity
Property

false
true

The solution set for

True or False?

3  12
36 S 36
36

false

false

5y

The solution set for 2x 15 is {4, 5, 6, 7}.


16. Replace y in 3y
36 with each value in the
replacement set for y.
y

false

The solution set for

The solution set for x  1 8 is {4, 5, 6, 7, 8}.


15. Replace x in 2x 15 with each value in the
replacement set for x.
x

18. Replace y in 4
20 with each value in the
replacement set for y.

True or False?
true

true

6
3

61 6 8 S 5 6 8

51 6 8 S 4 6 8

true

true

The solution set for x  2  7 is {6, 7, 8}.


14. Replace x in x  1 8 with each value in the
replacement set for x.

6 2 S 13 6 2
6 2 S 13 6 2

41 6 8 S 3 6 8

True or False?

5
3

2

true

x18

4
3

false

x
3

1
n  4, since 4 62  36  4.

The solution set for 3y


36 is {12, 14, 16}.

7. Multiplicative Property of Zero


n  0, since 8  0  0.
8. Multiplicative Inverse Property
1
n  1, since 1  9  9.
9. Reflexive Property of Equality
n  7, since 5  7  5  7.
10. Substitution Property of Equality
n  2, since (13  4)(2)  9(2).

645

Extra Practice

PQ249J-6481F-15-19[643-685] 26/9/02 9:49 PM Page 646 Sahuja Ahuja_QXP_06:Desktop Folder:Chandra:Algebra_FNL_Delivery:

1 12

2
[15
3

34

 10] Substitution; 12  2  10

2 3
2

3

Substitution; 15  10  2

1

12.

7
4

3 4  118  824


1 12

Multiplicative Inverse; 3  2  1

Multiplicative Inverse; 8  8  1

7
[4 ]
4

Multiplicative Identity; 4  1  4

 4 [4  1 ]

16.

Substitution; 4  4  7

13. [ (18  3)  0]  10
 [6  0 ]  10 Substitution; 18  3  6
Multiplicative Property of
 [0 ]  10
Zero; 6  0  0
Multiplicative Property of
0
Zero; 0  10  0

Page 821

17.
18.
19.
20.

8. 32 x  8  32(x)  32

 36(5)  36
 180  9
 189

114 2

 (4)18 

1181 218

 72  1
 73
13a  5a  (13  5)a
 18a
21x  10x  (21  10)x
 11x
8(3x  7)  8(3x)  8(7)
 24x  56
There are no like terms. 4m  4n is simplified.
3(5am  4)  3(5am)  3(4)
 15am  12

22. 9y2  13y2  3  (9  13)y2  3


 22y2  3
23. 11a2  11a2  12a2  (11  11  12)a2
 12a2
24. 6a  7a  12b  8b  (6  7)a  (12  8)b
 13a  20b

Page 821

Lesson 1-6

1. 23  8  37  12  23  37  8  12
 (23  37)  (8  12)
 60  20
 80
2. 19  46  81  54  19  81  46  54
 (19  81)  (46  54)
 100  100
 200
3. 10.25  2.5  3.75  10.25  3.75  2.5
 (10.25  3.75)  2.5
 14  2.5
 16.5
4. 22.5  17.6  44.5  22.5  44.5  17.6
 (22.5  44.5)  17.6
 67  17.6
 84.6

118 2

 32x  4
9. c(7  d)  c(7)  c(d)
 7c  cd
10. 6  55  6(50  5)
 6(50)  6(5)
 300  30
 330
11. 15(108)  15(100  8)
 15(100)  15(8)
 1500  120
 1620
12. 1689  5  (1700  11)5
 (1700)5  (11)5
 8500  55
 8445
13. 7  314  7(300  14)
 7(300)  7(14)
 2100  98
 2198

Extra Practice

21. 15x2  7x2  (15  7)x2


 22x2

Lesson 1-5

1. 5(2  9)  5(2)  5(9)


 10  45
 55
2. 8(10  20)  8(10)  8(20)
 80  160
 240
3. 20(8  3)  20(8)  20(3)
 160  60
 100
4. 3(5  w)  3(5)  3(w)
 15  3w
5. (h  8)7  (h)7  (8)7
 7h  56
6. 6(y  4)  6(y)  6(4)
 6y  24
7. 9(3n  5)  9(3n)  9(5)
 27n  45

15. 418  18  4  18 18

7

14. 36 54  36 5  4

11. 3 [ 15  (12  2) ]

5. 23  6  33  4  23  33  6  4

 23  33  (6  4)
 6  10
 16
6

6. 57  15  47  25  57  47  (15  25)
 10  40
 50
7. 6  8  5  3  6  5  8  3
 (6  5)  (8  3)
 30  24
 720

646

PQ249J-6481F-15-19[643-685] 26/9/02 9:49 PM Page 647 Sahuja Ahuja_QXP_06:Desktop Folder:Chandra:Algebra_FNL_Delivery:

8. 18  5  2  5  (18  5)  (2  5)
 90  10
 900
9. 0.25  7  8  0.25  8  7
 (0.25  8)  7
 27
 14
10. 90  12  0.5  90  (12  0.5)
 90  6
 540
1

21. 12b3  12  12b3  12b3  12b3  12


 (12b3  12b3 )  12
 (12  12)b3  12
 24b3  12
22. 7  3(uv  6)  u  7  3(uv)  3(6)  u
 7  3uv  18  u
 7  18  3uv  u
 (7  18)  3uv  u
 11  3uv  u
23. 3(x  2y)  4(3x  y)
 3(x)  3(2y)  4(3x)  4(y)
 3x  6y  12x  4y
 3x  12x  6y  4y
 (3x  12x)  (6y  4y)
 (3  12)x  (6  4)y
 15x  10y
24. 6.2(a  b)  2.6(a  b)  3a
 6.2(a)  6.2(b)  2.6(a)  2.6(b)  3a
 6.2a  6.2b  2.6a  2.6b  3a
 6.2a  2.6a  3a  6.2b  2.6b
 (6.2a  2.6a  3a)  (6.2b  2.6b)
 (6.2  2.6  3)a  (6.2  2.6)b
 11.8a  8.8b
25. 3  8(st  3w)  3st
 3  8(st)  8(3w)  3st
 3  8st  24w  3st
 3  (8st  3st)  24w
 3  (8  3)st  24w
 3  11st  24w
26. 5.4(s  3t)  3.6(s  4)
 5.4(s)  5.4(3t)  3.6(s)  3.6(4)
 5.4s  16.2t  3.6s  14.4
 5.4s  3.6s  16.2t  14.4
 (5.4s  3.6s)  16.2t  14.4
 (5.4  3.6)s  16.2t  14.4
 9s  16.2t  14.4
27. 3[4  5(2x  3y) ]  3[ 4  5(2x)  5(3y) ]
 3[ 4  10x  15y ]
 3[4]  3[ 10x]  3[ 15y]
 12  30x  45y

11. 53  4  6  53  6  4

 53  6  4
 32  4
 128
5

12. 46  10  12  46  12  10

 46  12  10
 58  10
 580
13. 5a  6b  7a  5a  7a  6b
 (5a  7a)  6b
 (5  7)a  6b
 12a  6b
14. 8x  4y  9x  8x  9x  4y
 (8x  9x)  4y
 (8  9)x  4y
 17x  4y
15. 3a  5b  2c  8b  3a  5b  8b  2c
 3a  (5b  8b)  2c
 3a  (5  8)b  2c
 3a  13b  2c
2

16. 3x2  5x  x2  3x2  x2  5x

123x2  x22  5x
2
 1 3  1 2 x2  5x


 3x2  5x

17. (4p  7q)  (5q  8p)  4p  7q  5q  8p


 4p  8p  5q  7q
 (4p  8p)  (5q  7q)
 (4  8)p  (5  7)q
 4p  (2q)
 4p  2q
18. 8q  5r  7q  6r  8q  7q  5r  6r
 (8q  7q)  (5r  6r)
 (8  7)q  (5  6)r
 1q  (1r)
qr
19. 4(2x  y)  5x  4(2x)  4(y)  5x
 8x  4y  5x
 8x  5x  4y
 (8x  5x)  4y
 (8  5)x  4y
 13x  4y

Page 822

Lesson 1-7

1. Hypothesis: an animal is a dog


Conclusion: it barks
2. Hypothesis: a figure is a pentagon
Conclusion: it has five sides
3. Hypothesis: 3x  1  8
Conclusion: x  3
4. Hypothesis: 0.5 is the reciprocal of 2
Conclusion: 0.5  2  1
5. Hypothesis: a figure is a square
Conclusion: it has four congruent sides.
If a figure is a square, then it has four congruent
sides.
6. Hypothesis: a  4
Conclusion: 6a  10  34
If a  4, then 6a  10  34.

20. 9r5  2r2  r5  9r5  r5  2r2


 (9r5  r5 )  2r2
 (9  1)r5  2r2
 10r5  2r2

647

Extra Practice

PQ249J-6481F-15-19[643-685] 26/9/02 9:49 PM Page 648 Sahuja Ahuja_QXP_06:Desktop Folder:Chandra:Algebra_FNL_Delivery:

7. Hypothesis: it is night
Conclusion: the video store is open
If it is night, then the video store is open.
8. Hypothesis: it is Thursday
Conclusion: the band does not have practice
If it is Thursday, then the band does not have
practice.
9. It can snow in May in some locations.
10. You may live in Portland, Maine.
11. y  3;
2(3)  4  6  4
 10
but 3 3
If y  3, then 2y  4  10 is true, but y 3 is
false.

Page 823
1. The
The
2. The
The
3. The
The
4. The
The
5. The
The
6. The
The
7.

12. Sample answer: a  1; (1) 2  1 7 0


but 1  0

Page 822

6

Lesson 1-8

3

2

1

6 5 4 3 2 1

2 1

10.
10 9 8 7 6 5 4 3 2 1

11. 22 is twenty-two units from zero in the positive


direction.
022 0  22
12. 2.5 is two and five-tenths units from zero in the
negative direction.
02.5 0  2.5
13.

2
3

is two-thirds unit from zero in the positive


direction.

0 23 0  23
7

14. 8 is seven-eighths unit from zero in the negative


direction.

0 78 0  78

Lesson 1-9

Page 823

Lesson 2-2

1. 3  16  19
2. 27  19  ( 027 0  019 0 )
 (27  19)
 8
3. 8  (13)  ( 013 0  08 0 )
 (13  8)
 5
4. 14  (9)  ( 014 0  09 0 )
 (14  9)
 23
5. 25  47  ( 047 0  025 0 )
 (47  25)
 22
6. 97  (79)  ( 097 0  079 0 )
 (97  79)
 18

 0.16

About 16% of U.S. Presidents have been born in


Ohio.

Extra Practice

4

9.

1. The bar for Virginia shows 8 presidents and the


bar for Texas shows 2 presidents. Since 8 equals
4 times 2, there were 4 times more presidents
born in Virginia than in Texas.
2. The bar for Massachusetts shows 4 presidents
and the bar for New York shows 4 presidents. So,
Massachusetts and New York have the same
number of presidents.
3. No; you need to have the birthplaces of all
presidents to compare parts to the whole in a
circle graph.
4. The bar for Ohio shows 7 presidents. What
percent of 43 is 7?
7
43

5

8.

1. Sample answer: The temperatures increase from


January through the summer and then begin to
decrease again.
2. Sample answer: The roller coaster goes down a
small hill, coasts at about the same speed,
increases in speed on the way down the hill,
decreases again on the way up the hill, increases
down another hill, and then slows down for the
end of the ride.
3. Sample answer: The jogger increases in speed,
runs about the same speed, increases again, runs
at a faster pace for a while, decreases, maintains
a speed, and finally slows down at the finish of
the run.
4. Sample answer: The hiker walks away from the
camp, stops for a rest, hikes a little further, and
then returns to camp.

Page 822

Lesson 2-1

dots indicate each point on the graph.


coordinates are {3, 2, 1, 0, 1, 2, 3, 4}.
dots indicate each point on the graph.
coordinates are {2, 0, 2, 3, 6}.
dots indicate each point on the graph.
coordinates are {2, 3, 4}.
dots indicate each point on the graph.
coordinates are {7, 8, 9, 10, 11, 12, 13}.
dots indicate each point on the graph.
coordinates are {6, 4, 2, 0}.
dots indicate each point on the graph.
coordinates are {2, 1, 0, 1, 2, 3, 4}.

648

PQ249J-6481F-15-19[643-685] 26/9/02 9:49 PM Page 649 Sahuja Ahuja_QXP_06:Desktop Folder:Chandra:Algebra_FNL_Delivery:

7. 4.8  3.2  ( 04.8 0  03.2 0 )


 (4.8  3.2)
 1.6
8. 1.7  (3.4)  ( 01.7 0  03.4 0 )
 (1.7  3.4)
 5.1
9. 0.009  0.06  ( 00.06 0  00.009 0 )
 (0.06  0.009)
 0.051

1 72

11

22.

5
11

 11
23.

2
7

 14  14  14
1

 14

1 0 11 0 0 7 0 2
11
7
 1 9  9 2

28

1 0 28 0 0 2560 0 2
28
25
  1 60  60 2
  60 

25

11. 5  6  30  30

1 0 2530 0  0 1830 0 2
25
18
  1 30  30 2


12.

3
8

14

 12  24  24

 60
1

 20

10 0 0 02
14
9
  1 24  24 2
14
24



25

 60  60

 2

 30

1 25 2
28
25
 60  1 60 2
28

24. 15  12  60  60

18

18

1 0 0  0 115 0 2
6
5
  1 11  11 2

 9
5

6
11



10.  9  9    9  9

 11  11  11

Page 823

Lesson 2-3

1. 5(12)  60
2. (6)(11)  66
3. (7)(5)  35
4. (6)(4)(3)  (24)(3)
 72

9
24

 24

13. 27  14  13
14. 8  17  8  (17)
 ( 017 0  08 0 )
 (17  8)
 9
15. 12  (15)  12  (15)
 12  15
 27
16. 35  (12)  35  (12)
 35  12
 ( 035 0  012 0 )
 (35  12)
 23
17. 2  (1.3)  2  (1.3)
 2  1.3
 ( 02 0  01.3 0 )
 (2  1.3)
 0.7
18. 1.9  (7)  1.9  (7)
 1.9  7
 8.9
19. 4.5  8.6  4.5  (8.6)
 ( 04.5 0  08.6 0 )
 (4.5  8.6)
 13.1
20. 89.3  (14.2)  89.3  (14.2)
 89.3  14.2
 103.5
21. 18  (1.3)  18  (1.3)
 18  1.3
 ( 018 0  01.3 0 )
 (18  1.3)
 16.7

5.
7.

178 2113 2  247


1
42

21 2  1 21 2
1
23





9 7
2 3
63
6
21
2
1
102

1 22

10

6. (5) 5   5
8.

2
17

 2

21359 2  197 21329 2






288
63
32
7
4
47

9. (5.34)(3.2)  17.088
10. (6.8)(5.415)  36.822
11. (4.2)(5.1)(3.6)  (21.42)(3.6)
 77.112
12. (3.9)(1.6)(8.4)  (6.24)(8.4)
 52.416
13. 5(3a)  6a  5(3)a  6a
 15a  6a
 (15  6)a
 21a
14. 8(x)  3x  8(1)x  3x
 8x  3x
 (8  3)x
 5x
15. 2(6y  2y)  2(6y)  2(2y)
 2(6)y  2(2)y
 12y  4y
 (12  4)y
 8y
16. (c  7c)(3)  c(3)  7c(3)
 1(3)c  7(3)c
 3c  (21c)
 [3  (21)]c
 24c

649

Extra Practice

PQ249J-6481F-15-19[643-685] 26/9/02 9:49 PM Page 650 Sahuja Ahuja_QXP_06:Desktop Folder:Chandra:Algebra_FNL_Delivery:

17. 3n(4b)  2a(3b)  3(4)nb  2(3)ab


 12nb  6ab
 12bn  6ab
18. 7(2m  3n)  7(2m)  (7)(3n)
 7(2)m  (7)(3)n
 14m  (21n)
 14m  (21n)
 14m  21n

18.

2a  10b
2

 (2a  10b)  (2)

1 12
1
1
 2a 1 2 2  10b 1 2 2

 (2a  10b) 2

 1a  (5b)
 a  5b
19.

27c  (99b)
9

 [ 27c  (99b) ]  9

119 2
1
1
 27c 1 9 2  (99b) 1 9 2

 [27c  (99b) ]

Page 824
1.
3.
5.
6.

Lesson 2-4

49  (7)  7
2. 52  (4)  13
66  (0.5)  132 4. 25.8  (2)  12.9
55.25  (0.25)  221
82.1  (16.42)  5
2

7. 5  5  5  5
2

 25

8.

10

9. 4  10  4   7

10.

3
2




1 12

40
7
5
57

1 22

 2  2  1

7
8

12.




13.

32a
4

1 52

 32a 
 8a
15.

5n  15
5

64

114 2

1 82

Page 824

14

 225




14.

12x
2






 12x  (2)

1 12

 12x  2
 6x


4

1 12
1
1
 5n 1 5 2  15 1 5 2

1 12
1
1
 2b 1 2 2  10 1 2 2

60

148



152

156

12


16

20




64

68






 


76

80

72

4. The lowest value is 111, and the highest value is


175, so use a scale that includes those values.
Place an  above each value for each occurrence.

 1b  (5)
b5

 (65x  15y)  (5)

12
1
1
 65x 1 5 2  15y 1 5 2

 (65x  15y)










 (2b  10)  (2)

x x

x
x xx

xx

x x x

110 115 120 125 130 135 140 145 150 155 160 165 170 175

1
5

5. The greatest common place value is tens, so the


digits in the tens place are the stems.
Stem
Leaf
1
0 7 7 8 8 9 9
2
2 2 3 3
3
5 5 5 9
4
0 4 5 7
1|0  10

 13x  3y

Extra Practice

144

3. The lowest value is 62, and the highest value is


80, so use a scale that includes those values.
Place an  above each value for each occurrence.

 (2b  10) 2

65x  15y
5

140

2. The lowest value is 4, and the highest value is 19,


so use a scale that includes those values. Place an
 above each value for each occurrence.

 1n  (3)
 n  3

17.







136

 (5n  15)  (5)

2b  10
2

Lesson 2-5

1. The lowest value is 134, and the highest value is


156, so use a scale that includes those values.
Place an  above each value for each occurrence.

 (5n  15) 5

16.

1 12
1
1
 3n 1 3 2  (3m) 1 3 2

 1n  1m
nm

11. 5  8  5  5
 25

 [ 3n  (3m) ]  (3)


 [3n  (3m) ] 3

13 25
15  3
325
 45
65
9
2
79

 32a  4

3n  (3m)
3

 32

1 12

 2

3
25

20.

 (4)  8  4

 3
13
15

 3c  (11b)
 3c  11b

650

PQ249J-6481F-15-19[643-685] 26/9/02 9:49 PM Page 651 Sahuja Ahuja_QXP_06:Desktop Folder:Chandra:Algebra_FNL_Delivery:

9. There are 3 letters that are s and 7 total letters.

6. The greatest common place value is ones, so the


digits in the ones place are the stems.
Stem
Leaf
1
0 1 2 2 3 8 9
2
0 1 2 5
3
0 2
4
1 1 5
5
6
1|0  1.0
7. The greatest common place value is hundreds, but
the hundreds digit in every number is a one.
Thus, we use the digits in the tens place as the
stems.
Stem
Leaf
10
1 1 4 5 8 9
11
1
12
1 3 9
13
0 3 4 5 7 9 10|1  101

P(the letter s)  7  0.43


3

The probability of selecting the letter s is 7 or


about 43%.
10. There is no letter b in the word success and
7 total letters.
0

P(the letter b)  7  0
The probability of selecting the letter b is 0 or 0%.
11. There are 2 letters that are vowels and 7 total
letters.
2

P(vowel)  7  0.29
2

The probability of selecting a vowel is 7 or about


29%.
12. There are 3 letters that are u or c and 7 total
letters.
3

P(the letters u or c)  7  0.43

Page 824

Lesson 2-6

The probability of selecting the letters u or c is


or about 43%.
13. There is 1 outcome that is a 4 and 6  1 or
5 outcomes that are not a 4.

1. There is one way for the coin to land tails up and


two ways for the coin to land.
1

P(tails up)  2 or 0.50

odds of a 4  5

The probability a coin lands tails up is 2 or 50%.


2. At some point during this month you will eat
something. Therefore, the probability you eat this
month is 1 or 100%.
3. There is one outcome that is a girl and two
possible outcomes.
P(a girl) 

1
2

The odds of rolling a 4 are 1:5.


14. There are 3 outcomes that are numbers greater
than 3 and 6  3 or 3 outcomes that are numbers
not greater than 3.
3

odds of a number greater than 3  3

or 0.50

1

1
2

The probability a baby will be a girl is or 50%.


4. There are no blue elephants. Therefore, the
probability you will see a blue elephant is 0 or
0%.
5. You are studying algebra from an algebra book,
and the statement is written in that book.
Therefore, the probability this is an algebra book
is 1 or 100%.
6. There is 1 day in the week that is Wednesday and
7 days in a week.

The odds of rolling a number greater than 3 are


1:1.
15. There are 2 outcomes that are numbers that are a
multiple of 3 and 6  2 or 4 outcomes that are
numbers that are not a multiple of 3.
2

odds of a multiple of 3  4
1

2
The odds of rolling a number that is a multiple of
3 are 1:2.
16. There are 4 outcomes that are numbers less than
5 and 6  4 or 2 outcomes that are numbers that
are not less than 5.

P(a day is Wednesday)  7  0.14


The probability a day is Wednesday is
14%.
7. There is 1 e and 7 total letters.

1
7

or about

odds of a number less than 5  2


2

1

P(the letter e)  7  0.14

The odds of rolling a number less than 5 are 2:1.


17. There are 3 outcomes that are odd numbers and
6  3 or 3 outcomes that are not odd numbers.

The probability of selecting the letter e is 7 or


about 14%.
8. There are 5 letters that are not c and 7 total
letters.
P(not c) 

5
7

3
7

odds of an odd number  3

 0.71

1

The probability of selecting a letter that is not c is


5
or 71%.
7

The odds of rolling an odd number are 1:1.


18. There is 1 outcome that is a 6 and 6  1 or
5 outcomes that are not a 6.
5

odds against a 6  1
The odds against rolling a 6 are 5:1.

651

Extra Practice

PQ249J-6481F-15-19[643-685] 26/9/02 9:50 PM Page 652 Sahuja Ahuja_QXP_06:Desktop Folder:Chandra:Algebra_FNL_Delivery:

Page 825

18. You can use a calculator to find an approximation


for 115.
3.88  3.88
115  3.87298334
Therefore, 3.88 7 115.
19. You can use a calculator to find the value of
1529.
1529  23.0
20  20.0
Therefore, 1529 6 20.
20. You can use a calculator to find the value of
10.25.
10.25  0.5
0.5  0.55555555
Therefore, 10.25 7 0.5.

Lesson 2-7

1. 1121 represents the positive square root of 121.


121  112 S 1121  11
2. 136 represents the negative square root of 36.
36  62 S 136  6
3. 12.89 represents the positive square root of 2.89.
2.89  1.72 S 12.89  1.7
4. 1125 represents the negative square root of
125.
125  11.182 S 1125  11.18
5.

81

81

3 100 represents the positive square root of 100.


81
9 2
81
9
 1 10 2 S 3 100  10
100
36

6.  3 196 represents the negative square root of


36
196

36
6
 14
1146 22 S 3 196

36
.
196

21. You can use a calculator to find an approximation


13
for 3 .

 7

1
3
13
3

7. 19.61 represents the positive and negative


square roots of 9.61.
9.61  3.12 and 9.61  (3.1)2
19.61  3.1
7

roots of 8.
 0.94 and

7
8

 (0.94)

1
13
13
3

7
8

 3  0.94

13
.
3

 0.577350269
1
13

23
.
3
1

23. You can use a calculator to find the value of  3 4.


1

 3 4  0.5
1

4  0.25
1

Therefore,  3 4 6 4.

3 2  2, this number is a natural

24. You can use a calculator to find an approximation


1
for  16.

number, a whole number, an integer, and a


rational number.
12. Because 66 and 55 are integers and (66  55) 
1.2 is a terminating decimal, this number is a
rational number.
13. Because 1225  15, this number is a natural
number, a whole number, an integer, and a
rational number.

6  0.16666666
1

 16  0.408248290
1

Therefore, 6 7  16.

Page 825

Lesson 3-1

is the
divided
1. A number
z
times 2 minus 6 same as m
by
3.
1442443 123 { 1
424
3 { 14243 { 14243 {
z
 2  6

m

3
The equation is 2z  6  m  3.
2. The cube decreased the square
c.
by
of a
of b 3 is
equal to
14243 1442443 1
44244
1
44244
3{

14. Because  3 4  0.866025403, which is not a


repeating or terminating decimal, this number is
irrational.
15. Because 1 and 7 are integers and 1  7 
0.142857142 is a repeating decimal, this
number is a rational number.

a3

b2

c
The equation is a3  b2  c.
3.
the product is the
Twenty-nine decreased by of x and y same as z .
144424443 144424443 1442443 14243 {
29

xy

z
The equation is 29  xy  z.

16. Because 10.0016  0.04 is a terminating


decimal, this number is a rational number.
17. Write the numbers as decimals.
6.16  6.16161616
6  6.0
Therefore, 6.16 7 6.0.

Extra Practice

 0.577350269

Therefore,

9. Because 1149  12.2065556, which is not a


repeating or terminating decimal, this number is
irrational.
10. Because 5 and 6 are integers and 5  6 
0.8333333 is a repeating decimal, this number
is a rational number.
11. Because

1
3

22. You can use a calculator to find approximations


1
13
for 13 and 3 .

 0.577350269

Therefore,

8.  3 8 represents the positive and negative square

7
8

 0.33333333

652

PQ249J-6481F-15-19[643-685] 26/9/02 9:50 PM Page 653 Sahuja Ahuja_QXP_06:Desktop Folder:Chandra:Algebra_FNL_Delivery:

4. The perimeter
P
P

144424443

3.

the sum of twice the length of the


leg a and the length of the base b.
is
{ 14
4444444424444444
443
2a  b


The equation is P  2a  b.
the quotient
Thirty
increased
by
of s and t is equal to v .
1
424
3 1442443 1442443 1442443 {
v
30

(s  t)

The equation is 30  (s  t)  v.
6. The area half the product of lengths
is of the diagonals a and b.
A
14243 { 144444424444443
1
A

ab

5.

4  (5)  9
9  9
The solution is 5.
4.
m62
m6626
m  4
m62
Check:
?
4  6  2
22
The solution is 4.
t  (4)  10
5.
t  (4)  4  10  4
t  14
t  (4)  10
Check:

The equation is A  2ab or 0.5ab.


7. Sample answer:
0.5x  3

14243

10

123

The sum of five-tenths


times x and three
8. Sample answer:
n

is equal to

negative ten.

6
123

14  (4)  10
10  10
The solution is 14.
6.
v  7  4
v  7  7  4  7
v3
v  7  4
Check:
?
3  7  4
4  4
The solution is 3.
7.
a  (6)  5
a  (6)  (6)  5  (6)
a  11
a  (6)  5
Check:

2n  1

14243

The quotient of n is the same as the sum of two


and negative six
times n and one.
9. Sample answer:
18

123

5h

123

13h

123

Eighteen decreased five times h is the thirteen


by
same as times h.
10. Sample answer:
n2

16
{
123

123

The square of n is equal to sixteen.


11. Sample answer:
2x2  3

21
{
123
14243

The sum of three


is equal to twenty-one.
and twice x squared
12. Sample answer:
m

4

n
14243

The sum of four and


the quotient of m and n

Page 825
1.

4  y  9
4  y  4  9  4
y  5
Check:
4  y  9

11  (6)  5
5  5
The solution is 11.
8.
2  x  8
2  x  x  8  x
2  8  x
2  8  8  x  8
6x
Check:
2  x  8
?
2  6  8
8  8
The solution is 6.
9.
d  (44)  61
d  (44)  44  61  44
d  17
Check:
d  (44)  61
?
17  (44)  61
61  61
The solution is 17.
10.
e  (26)  41
e  (26)  (26)  41  (26)
e  15
e  (26)  41
Check:
?
15  (26)  41
41  41
The solution is 15.

12

123

is equal to twelve.

Lesson 3-2

2  g  7
2  g  2  7  2
g9
2  g  7
Check:
?

2  9  7
77
The solution is 9.
2.
9  s  5
9  s  9  5  9
s  14
9  s  5
Check:
?
9  (14)  5
5  5
The solution is 14.

653

Extra Practice

PQ249J-6481F-15-19[643-685] 26/9/02 9:50 PM Page 654 Sahuja Ahuja_QXP_06:Desktop Folder:Chandra:Algebra_FNL_Delivery:

11.

p  47  22
p  47  47  22  47
p  69
Check:
p  47  22

18.

69  47  22
22  22
The solution is 69.
12.
63  f  82
63  f  f  82  f
63  82  f
63  82  f
19  f
Check:
63  f  82

t  (46.1)  3.673
t  (46.1)  (46.1)  3.673  (46.1)
t  49.773
t  (46.1)  3.673
Check:
?
49.773  (46.1)  3.673
3.673  3.673
The solution is 49.773.
7
10

19.
7
10

aa2a
7
10
7
1
2
10
2
10
1
5

63  19  82
82  82
The solution is 19.
13.
c  5.4  11.33
c  5.4  5.4  11.33  5.4
c  16.73
c  5.4  11.33
Check:
?
16.73  5.4  11.33
11.33  11.33
The solution is 16.73.
14.
6.11  b  14.321
6.11  b  6.11  14.321  6.11
b  20.431
6.11  b  14.321
Check:

Check:

2a
1

2a2
a
a
7
10

a2

7
10

5

1 ? 1
2
5 ? 1

20.

10
1
2
1
.
5

The solution is

2
1
2

1 12 3
1
1
3
1
f  1 8 2  1 8 2  10  1 8 2
f  8  10
7

f  40

6.11  20.431  14.321


14.321  14.321
The solution is 20.431.
5  y  22.7
15.
5  22.7  y  22.7  22.7
17.7  y
5  y  22.7
Check:
5  17.7  22.7
5  5
The solution is 17.7.
16.
5  q  1.19
5  q  q  1.19  q
5  1.19  q
5  1.19  1.19  q  1.19
6.19  q
5  q  1.19
Check:
?
5  (6.19)  1.19
1.19  1.19
The solution is 6.19.
17.
n  (4.361)  59.78
n  (4.361)  4.361  59.78  4.361
n  64.141
Check:
n  (4.361)  59.78

Check:

1 12 3
7
1 ? 3
 1 8 2  10
40
f  8  10

12 ? 3
 10
40
3
3
 10
10

21.

7
.
40

The solution is

1 12
5
1
1
1
412  1 1036 2  t  1 1036 2  1 1036 2
5

412  t  1036
16

1436  t
4

149  t
Check:

412 
5

412 
The solution is

654

412  t  10 36
5 ?
412 
5 ?

64.141  (4.361)  59.78


59.78  59.78
The solution is 64.141.

Extra Practice

a2

4
149
15
436
5
412

4
149.

 10 36

PQ249J-6481F-15-19[643-685] 26/9/02 9:50 PM Page 655 Sahuja Ahuja_QXP_06:Desktop Folder:Chandra:Algebra_FNL_Delivery:

3. 2y  3

x84

22.

2y
2

x8848

y

x  8

y  12 or 1.5
2y  3
Check:

3 ? 1
4
2 ? 1

8  8 
8
1
4

23.

4
1
4

2(1.5)  3
3  3
The solution is 1.5.
4. 62y  2356

62y
62

1
The solution is 8.
7
9
116  s  8
7
7
9
7
116  s  116  8  116
5
s  16

? 9
8
? 9

9
8

8
9
8

5.

116  16 

The solution is
8

116 


6

8
17 9

5
26

13
20 18

d

8 ?
179 
8 ?

13
2018

8
179

8
179

16

179  1718

6.

5
26

59

2

 7

1 2  59(7)
c  413

c
59

 7

413 ?

59

7

7  7
The solution is 413.

7.

Lesson 3-3

f
14
f
14

 63

1 2  14(63)
f  882

1. 7p  35

f
14

Check:

35
7

 63

882 ?

14

p5
Check:

63

63  63
The solution is 882.

7p  35
?
7(5)  35
35  35
The solution is 5.
2. 3x  24


c
59
c
59

Check:

14

3x
3

 2

2  2
The solution is 12.

13

a
6

12 ?

6

The solution is 2018.

7p
7

1 2  6(2)

Check:

1 2
5
5
 d  1 26 2  26
179  d  26

Page 826

 2

5
16.

Check:

a
6
a
6

a  12

17 9  d  26

24.

62y  2356

62(38)  2356
2356  2356
The solution is 38.

2356
62

y  38
Check:

116  s  8

Check:

3
2
3
2
1

x84

Check:

8.

84  97
97(84)  97
8148  x

24
3

Check:

x8
Check:

197x 2

84 

197x 2

? 8148
97

84 

3x  24
?
3(8)  24
24  24
The solution is 8.

84  84
The solution is 8148.

655

Extra Practice

PQ249J-6481F-15-19[643-685] 26/9/02 9:50 PM Page 656 Sahuja Ahuja_QXP_06:Desktop Folder:Chandra:Algebra_FNL_Delivery:

9.
5

w
5
w
5

3

1 2  5(3)

26
3

12

26
26
5
26
5
12
5

j
j

33
The solution is 15.
q
9
q
9

55

w
5

15 ?

5

10.

26 j

w  15

Check:

14. 26 j  55

j

13
6


6

 13
2

or 25
1

1 22 1
13 12 ?
1
 55
6152

q  27
q

9
27 ?

9

26 ?

5
1
55 

3
3

3  3
The solution is 27.
11.

2
x
5
5 2
x
2 5

3
7

11

3  18  q
11
6
5
16

33
5

16. 14 p  8
5

14p
3

14

8
3

14

1 72
5
4
p  8  1 7 2
5

p  8  4

1 12
9 15
r  5 1 2 2

p  14

5 9r  5 72

27

r   2 or 132

135 ?

18
1
72 
1
132.

The solution is

5
.
14

17. 57k  0.1824

72
1
72

8  8

9r  72

9 132  72

The solution is

1 2  58
?
7 5
5
4 1 14 2  8
3 5
14

14

135

14p  8

Check:

r   10

Check:

The solution is 16.

9r  72
5

1 52
? 18 11
3  11 1 6 2
?

3  111q

3  111 16

The solution is 22.


9

q

q

Check:

12  12

13.

111

18

1 2

111q

3  11  q

or 17

2
4
x7
5
?
2 3
4
1 7
5 7
4
4
7
7
3
The solution is 17.
z
5
 12
6
z
5
6 6  6 12
5
1
z  2 or 22
z
5
Check:
 12
6
1
22 ?
5
 12
6

12

111

20

x  14

12.

3  111q

15.

1 2  52 147 2

Check:

55

10
7

55

The solution is 25.

7

x

26 25  55

 9(3)

Check:

26 j  55

Check:

 3

57k
57

0.1824
57

k  0.0032
Check:

57k  0.1824
?

57(0.0032)  0.1824
0.1824  0.1824
The solution is 0.0032.
Extra Practice

656

PQ249J-6481F-15-19[643-685] 26/9/02 9:50 PM Page 657 Sahuja Ahuja_QXP_06:Desktop Folder:Chandra:Algebra_FNL_Delivery:

18. 0.0022b  0.1958


0.0022b
0.0022

24. 184  2.5x

0.1958
0.0022

184

b  89
0.0022b  0.1958
Check:
?
0.0022(89)  0.1958
0.1958  0.1958
The solution is 89.
19. 5j  32.15
5j
5

2.5
18.25
2.5

7.3  x
1 ?
1 ?

184  18.25
1

w
2
w
2

The solution is 7.3.

Page 826

 2.48

1 2  2(2.48)
w

2
4.96 ?

2

Check:

2.48

2x
2

2.48

1 2  2.8(6.2)
z  17.36

z

2.8
17.36 ?

2.8

4.

6.2
6.2

0.063

x
0.063
x
0.063

Check:

2  0.063(0.015)

23.

32
4

t8

8g
8

3c
3

0.015
6.

0.015

 5p

15
3

c5
5k  7  52
5k  7  7  52  7
5k  45
5k
5

45
5

k  9
7. 5s  4s  72
9s  72

5p

5
5
123
1
 5  p
8

1 2

9s
9

340  p
3

158  5p

3 ?

158  5 340
3 ?

15

158  1540

72
9

s  8
8.
3x  7  2
3x  7  7  2  7
3x  9

123

 40  p

Check:

x  0.000945

0.015  0.015
The solution is 0.000945.
3
158
3
158

5  g
5.
3c  9  24
3c  9  9  24  9
3c  15

 0.015

x

0.063
0.000945 ?

0.063

4t
4

a  6
47  8g  7
47  7  8g  7  7
40  8g
40
8

6.2  6.2
The solution is 17.36.
22.

2

x4
3.
7a  6  36
7a  6  6  36  6
7a  42
7a
42
 7
7

 6.2

Check:

Lesson 3-4

Exercises 121 For checks, see students work.


1.
2.
2x  5  3
4t  5  37
2x  5  5  3  5
4t  5  5  37  5
2x  8
4t  32

2.48  2.48
The solution is 4.96.
z
2.8
z
2.8

184  184

5j  32.15

w  4.96

2.8

184  2.5x
184  2.5(7.3)

21.

x

Check:

5(6.43)  32.15
32.15  32.15
The solution is 6.43.
2

2.5x
2.5

32.15
5

j  6.43
Check:

20.

3x
3

3

x3

9.

8  3x  5
8  3x  8  5  8
3x  3
3x
3

3
3

x  1

158  158
3

The solution is 340.

657

Extra Practice

PQ249J-6481F-15-19[643-685] 26/9/02 9:50 PM Page 658 Sahuja Ahuja_QXP_06:Desktop Folder:Chandra:Algebra_FNL_Delivery:

10.

3y  7.569  24.069


3y  7.569  7.569  24.069  7.569
3y  16.5
3y
3

21. 4.8a  3  1.2a  9


6a  3  9
6a  3  3  9  3
6a  12

16.5
3

6a
6

y  5.5
11.
7  9.1f  137.585
7  9.1f  7  137.585  7
9.1f  130.585
9.1f
130.585
 9.1
9.1
12.

Page 826

Lesson 3-5

Exercises 121 For checks, see students work.


1.
5x  1  3x  3
5x  1  3x  3x  3  3x
2x  1  3
2x  1  1  3  1
2x  4

2.4m
2.4

4.75  m
e
5

13.
e
5

 6  2

 6  6  2  6

e
5
e
5

d
4

14.
d
4

 8  5

 8

e  40

15.

2x
2

d
4
d
4

3

1 2  4(3)
d  12

13n
13

76

13y  7  7  6  7
13

3.

13y  13
4

13

 4 13y   4 (13)

16.
10

169
4

y

p  3
10
p  3
10

or 424

4

2  10(4)

17.
6

p  3  40
p  3  3  40  3
p  37
18.
8

5f  1
8
5f  1
8

 3

2  8(3)

19.
2

25
5

3t  4
2
3t  4
2

2
h
3

1
a
2
1
a
2

3
a
4

 4  3  4a
1

4434
7

1 2  43 (7)
a

6.

20

 3
2

t   3 or 63

28
3

or 93

6(y  5)  18  2y
6y  30  18  2y
6y  30  2y  18  2y  2y
8y  30  18
8y  30  30  18  30
8y  48
8y
8

48
8

y6

658

43

3
a
4
4 3
a
3 4

8

3t  4  16
3t  4  4  16  4
3t  20

Extra Practice

 4  4a  3  4a  4a
3
a
4

16
4

n  4

20

 5  3h  4  3h  3h

5.

2  2(8)

3t
3

 5  4  3h

h  5  4
h  5  5  4  5
h  9

 12

2
h
3

4.

2  2(12)

4n
4

 5

z0

4n  8  24
4n  8  8  24  8
4n  16

f  5
20.

13

 13

5z
5

2  6(1)

4n  8
2
4n  8
2

4
2

n  1
3z  5  2z  5
3z  5  2z  2z  5  2z
5z  5  5
5z  5  5  5  5
5z  0

1

h76
h7767
h  13

5f  1  24
5f  1  1  24  1
5f  25
5f
5

h  7
6
h  7
6

x  2
2.
6  8n  5n  19
6  8n  5n  5n  19  5n
6  13n  19
6  13n  6  19  6
13n  13

 8  8  5  8

1 2  5(8)

4
13y

12
6

a2

f  14.35
6.5  2.4m  4.9
6.5  4.9  2.4m  4.9  4.9
11.4  2.4m
11.4
2.4

PQ249J-6481F-15-19[643-685] 26/9/02 9:50 PM Page 659 Sahuja Ahuja_QXP_06:Desktop Folder:Chandra:Algebra_FNL_Delivery:

7.

14. 3(x  1)  5  3x  2
3x  3  5  3x  2
3x  2  3x  2
Since the expressions on each side of the equation
are the same, this equation is an identity. The
statement 3(x  1)  5  3x  2 is true for all
values of x.

28  p  7( p  10)
28  p  7p  70
28  p  7p  7p  70  7p
28  6p  70
28  6p  28  70  28
6p  42
6p
6

42
6

15.

p7
1
(b  9)
3
1
b3
3

8.

1
b
3

x
2

b9

b9
x
6

3bb9b

x
2
1
3
x
6
1
3

16.

2 3b  2 (12)

b  18
9.
4x  6  0.5(x  30)
4x  6  0.5x  15
4x  6  0.5x  0.5x  15  0.5x
4.5x  6  15
4.5x  6  6  15  6
4.5x  9
4.5x
4.5

x
6
x
6

 6

1 2  6116 2

6v  9
3
6v  9
3

2  3(v)

9
3

17.
4

3t  1
4
3t  1
4

 4t  5

3t  1  3t  20
3t  1  3t  3t  20  3t
1  20
Since 1  20 is a false statement, this equation
has no solution.
18.
0.4(x  12)  1.2(x  4)
0.4x  4.8  1.2x  4.8
0.4x  4.8  1.2x  1.2x  4.8  1.2x
0.8x  4.8  4.8
0.8x  4.8  4.8  4.8  4.8
0.8x  0
0.8x
0.8

 0.8

x0

s  1
12.
2.85y  7  12.85y  2
2.85y  7  12.85y  12.85y  2  12.85y
10y  7  2
10y  7  7  2  7
10y  5

3y 

19.
3y 

4
5

4
5

 3y 
4

5 

1 42

8 5 

3
10

 10

y  0.5
13.
2.9m  1.7  3.5  2.3m
2.9m  1.7  2.3m  3.5  2.3m  2.3m
0.6m  1.7  3.5
0.6m  1.7  1.7  3.5  1.7
0.6m  1.8


3v
3

2  4134t  52

2.9
2.9

0.6m
0.6

3v

9
4.5

10y
10

v

6v  9  3v
6v  9  6v  3v  6v
9  3v

x  2
10. 4(2y  1)  8(0.5  y)
8y  4  4  8y
8y  4  8y  4
Since the expressions on each side of the equation
are the same, this equation is an identity. The
statement 4(2y  1)  8(0.5  y) is true for all
values of y.
11.
1.9s  6  3.1  s
1.9s  6  s  3.1  s  s
2.9s  6  3.1
2.9s  6  6  3.1  6
2.9s  2.9
2.9s
2.9

x  1

3b  12
2

 3  2  3

3b  3  3  9  3

 3  2

3323

3b  3  9
2

332

3
x
4

20.
3
x
4

y

 4  7  2x
1

 4  2x  7  2x  2x
1
x
4

1
x
4

47

4474

1.8
0.6

m3

1
y
3
1
y  3y
3
8
3y
3
8
8 3y

1
x
4
1
x
4

 11

1 2  4(11)
x  44

659

Extra Practice

PQ249J-6481F-15-19[643-685] 26/9/02 9:50 PM Page 660 Sahuja Ahuja_QXP_06:Desktop Folder:Chandra:Algebra_FNL_Delivery:

21.

0.2(1  x)  2(4  0.1x)


0.2  0.2x  8  0.2x
0.2  0.2x  0.2x  8  0.2x  0.2x
0.2  8
Since 0.2  8 is a false statement, this equation
has no solution.

Page 827
4
5

1.

2y
2

 20

2.

5x
5

7b
7

y
5

4

y
y
5.

10.

4.

7
4

7a
7

a

12
7
12
7

x
9

or 17

4a
4

12.

4n
4

22
2

13p
13

n  4
7

12
4

14.

q

210
168
5
4 or

65

 13

k  3
2

9k
9

k
 k or 2.5  k
5m  3
4

5m  3
6

(5m  3)6  4(5m  3)


30m  18  20m  12
30m  18  20m  20m  12  20m
10m  18  12
10m  18  18  12  18
10m  30

30

 14

2
9

23
9
23
9
5
29

12q(14)  7(30)
168q  210
168q
168

5p  7
8

2(2)  9(k  3)
4  9k  27
4  27  9k  27  27
23  9k

n3
12q
7

p5
13.

 3(n  4)
 3n  12
 3n  12  3n
 12


36
4

(6p  2)8  7(5p  7)


48p  16  35p  49
48p  16  35p  35p  49  35p
13p  16  49
13p  16  16  49  16
13p  65

2.16
3

n(7)
7n
7n  3n
4n

8.

6p  2
7

0.24
3

n
3

a9

x  0.72
7.

4

(a  3)4  8(3)
4a  12  24
4a  12  12  24  12
4a  36

x(3)  9(0.24)
3x  2.16
3x
3

34.84

 17.42
a  3
8

11.

t  11
6.

 17.42

x2

a

(t  5)2  4(3)
2t  10  12
2t  10  10  12  10
2t  22
2t
2

x
8.71

17.42x
17.42

7(a)  4(3)
7a  12

15
4
15
4
3
34 or 3.75
t  5
3
2
4

4

 2

x(17.42)  8.71(4)
17.42x  34.84

189
7

b  27

y(4)  5(3)
4y  15
4y
4

3
 5

y2

7

b(7)  63(3)
7b  189

16  x
3.

b
63

y

1(y  5)  (y  3)3
y  5  3y  9
y  5  3y  3y  9  3y
2y  5  9
2y  5  5  9  5
2y  4

Lesson 3-6

4(20)  5(x)
80  5x
80
5

1
y  3

9.

14

10m
10

30

 10

m3

Extra Practice

660

PQ249J-6481F-15-19[643-685] 26/9/02 9:50 PM Page 661 Sahuja Ahuja_QXP_06:Desktop Folder:Chandra:Algebra_FNL_Delivery:

w  5
4

15.

w  3
3

2. Find the amount of change. Since the new


amount is greater than the original, the percent
of change is a percent of increase.
98  62  36
Find the percent using the original number, 62, as
the base.

(w  5)3  4(w  3)
3w  15  4w  12
3w  15  4w  4w  12  4w
w  15  12
w  15  15  12  15
w  27
w
1

36
62

27

 1

36(100)  62(r)
3600  62r

w  27
16.

96.8
t

3600
62

12.1
7

12.1t
12.1

56  t
r  1
r  1

17.

5

(r  1)5  (r  1)3
5r  5  3r  3
5r  5  3r  3r  3  3r
2r  5  3
2r  5  5  3  5
2r  8
2r
2

33
322

3300
322

2n  7
7

(4n  5)7  5(2n  7)


28n  35  10n  35
28n  35  10n  10n  35  10n
18n  35  35
18n  35  35  35  35
18n  0
18n
18

42
78

 18

4200
78

Lesson 3-7

 100

33(100)  100(r)
3300  100r
3300
100

322r
322

 100

78r
78

54  r
The percent of decrease is about 54%.
5. Find the amount of change. Since the new
amount is greater than the original, the percent
of change is a percent of increase.
230  212  18
Find the percent using the original number, 212,
as the base.

1. The percent of change is a percent of decrease


because the new amount is less than the original.
Find the change.
100  67  33
Find the percent using the original number, 100,
as the base.
33
100

42(100)  78(r)
4200  78r

n0

Page 827

 100

10  r
The percent of decrease is about 10%.
4. The percent of change is a percent of decrease
because the new amount is less than the original.
Find the change.
78  36  42
Find the percent using the original number, 78, as
the base.

r4
18.

62r
62

33(100)  322(r)
3300  322r

2

4n  5
5

58  r
The percent of increase is about 58%.
3. The percent of change is a percent of decrease
because the new amount is less than the original.
Find the change.
322  289  33
Find the percent using the original number, 322,
as the base.

96.8(7)  t(12.1)
677.6  12.1t
677.6
12.1

 100

18
212

 100

18(100)  212(r)
1800  212r

100r
100

33  r
The percent of decrease is 33%.

1800
212

212r
212

8r
The percent of increase is about 8%.

661

Extra Practice

PQ249J-6481F-15-19[643-685] 26/9/02 9:50 PM Page 662 Sahuja Ahuja_QXP_06:Desktop Folder:Chandra:Algebra_FNL_Delivery:

The tax is 4% of the discounted price of the


jacket.
4% of $45.50  0.04  45.50
 1.82
Add this amount to the discounted price.
$45.50  $1.82  $47.32
The total price of the jacket is $47.32.
12. The discount is 10% of the original price.
10% of $28.95  0.10  28.95
 2.895
Subtract $2.90 from the original price.
$28.95 $2.90  $26.05
The discounted price of the backpack is $26.05.
The tax is 5% of the discounted price of the
backpack.
5% of $26.05  0.05  26.05
 1.3025
Round $1.3025 to $1.31 since tax is always
rounded up to the nearest cent. Add this amount
to the discounted price.
$26.05  $1.31  $27.36
The total price of the backpack is $27.36.

6. Find the amount of change. Since the new


amount is greater than the original, the percent
of change is a percent of increase.
65  35  30
Find the percent using the original number, 35, as
the base.
30
35

 100

30(100)  35r
3000  35r
3000
35

7.

8.

9.

10.

11.

35r
35

86  r
The percent of increase is about 86%.
The discount is 20% of the original price.
20% of $299  0.20  299
 59.80
Subtract $59.80 from the original price.
$299.00  $59.80  $239.20
The discounted price of the television is $239.20.
The tax is 7% of the price of the book.
7% of $15.95  0.07  15.95
 1.1165
Round $1.1165 to $1.12 since tax is always
rounded up to the nearest cent. Add this amount
to the original price.
$15.95  $1.12  $17.07
The total price of the book is $17.07.
The tax is 6.25% of the price of the software.
6.25% of $36.90  0.0625  36.90
 2.30625
Round $2.30625 to $2.31 since tax is always
rounded up to the nearest cent. Add this amount
to the original price.
$36.90  $2.31  $39.21
The total price of the software is $39.21.
The discount is 15% of the original price.
15% of $49.99  0.15  49.99
 7.4985
Subtract $7.50 from the original price.
$49.99  $7.50  $42.49
The discounted price of the boots is $42.49.
The tax is 3.5% of the discounted price of the
boots.
3.5% of $42.49  0.035  42.49
 1.48715
Round $1.48715 to $1.49 since tax is always
rounded up to the nearest cent. Add this amount
to the discounted price.
$42.49  $1.49  $43.98
The total price of the boots is $43.98.
The discount is 30% of the original price.
30% of $65  0.3  65
 19.50
Subtract $19.50 from the original price.
$65.00  $19.50  $45.50
The discounted price of the jacket is $45.50.

Extra Practice

Page 827

Lesson 3-8

xrq
xrrqr
xqr
The value of x is q  r.
2.
ax  4  7
ax  4  4  7  4
ax  3
1.

ax
a

a
3

xa
3

The value of x is a. Since division by 0 is


undefined, a  0.
3.

2bx  b  5
2bx  b  b  5  b
2bx  5  b
5  b
2b
5  b
x  2b
5  b
of x is 2b .

2bx
2b

The value
Since division by 0 is
undefined, 2b  0 or b  0.
4.
(c  a)

x  c
c  a
x  c
c  a

a

2  (c  a)a

x  c  ca  a2
x  c  c  ca  a2  c
x  ca  a2  c or a2  ac  c
The value of x is a2  ac  c.

662

PQ249J-6481F-15-19[643-685] 26/9/02 9:50 PM Page 663 Sahuja Ahuja_QXP_06:Desktop Folder:Chandra:Algebra_FNL_Delivery:

5.
c

x  y
c
x  y
c

d

11.

2  c(d)

2
(A)
h
2A
h
2A
y
h
2A
y
h

x  y  cd
x  y  y  cd  y
x  cd  y
The value of x is cd  y.
6.
2

ax  1
2
ax  1
2

b

12.

2b  1
a
2b  1
x a
2b  1
value of x is a .

The
undefined, a  0.

Page 828

d 
n 
d 
n
d  a
.
n  b

Since division by 0 is

a
b
a
b

0.60x
0.60

85(4)  95(1)
4  1

9
(y)
5
9
y
5
9
y
5
9
y
5

9 5

 5 9 (x  32)
 x  32




1.5

 0.60

340  95
5
435
5

 87
The students average is 87%.

The value of
division by 0 is
undefined, 3  r  0 or 3  r.
5

Amount of
Grape Juice
0.10 (5)
1.00 x
0.40(5  x)

x  2.5
2.5 qt of pure grape juice should be added.
2. The students average is determined using a
weighted average.

3

y  9 (x  32)

Lesson 3-9

0.10(5)  1.00x  0.40(5  x)


0.50  x  2  0.40x
0.50  x  0.40x  2  0.40x  0.40x
0.50  0.60x  2
0.50  0.60x  0.50  2  0.50
0.60x  1.5

2r
 r
2r
x3r
2r
x is 3  r. Since

10.

x

Amount of
amount of
amount of
grape juice
grape juice
grape juice
in 10% sol plus in 100% sol equals in 40% sol.
1442443 123 1442443 1
424
3 1442443
0.10(5)

1.00x
 0.40(5  x)

The value of x is
Since division by 0 is
undefined, n  b  0 or n  b.
3x  r  r(3  x)
9.
3x  r  3r  rx
3x  r  rx  3r  rx  rx
3x  r  rx  3r
3x  r  rx  r  3r  r
3x  rx  2r
(3  r)x  2r
(3  r)x
3  r

2rx
2r

Amount of
Solution
10% grape juice
5
100% grape juice
x
40% grape juice
5x

nx  a  bx  d
nx  a  bx  bx  d  bx
nx  a  bx  d
nx  a  bx  a  d  a
nx  bx  d  a
(n  b)x  d  a


1. Let x  the amount of 100% grape juice to be


added.

The value
undefined, d  0.

A  2r2  2rx
A  2r2  2r2  2rx  2r2
A  2r2  2rx

d(x  3)  5
dx  3d  5
dx  3d  3d  5  3d
dx  5  3d

(n  b)x
n  b

x

The value of x is 2r  r. Since division by 0 is


undefined, 2r  0 or r  0.

Since division by 0 is

5  3d
d
5  3d
x d
5  3d
of x is d .

8.

xyy

A  2r2
2r
A
r
2r

dx
d

xy

The value of x is h  y. Since division by 0 is


undefined, h  0.

ax  1  2b
ax  1  1  2b  1
ax  2b  1

7.

2 1

 h 2h(x  y)

2A

2  2(b)

ax
a

A  2h(x  y)

 32  x  32  32
 32  x
9

The value of x is 5y  32.

663

Extra Practice

PQ249J-6481F-15-19[643-685] 26/9/02 9:51 PM Page 664 Sahuja Ahuja_QXP_06:Desktop Folder:Chandra:Algebra_FNL_Delivery:

3. Let x  the number of adult tickets sold.


Number of
Tickets

Cost of
Tickets

Total
Cost

$5.50

5.50x

21  x

$3.50

3.50(21  x)

Adult
Tickets
Child
Tickets

Page 828

1.
2.
3.
4.
5.
6.
7.
8.
9.

Total cost
total cost
of adult
of child
equals
tickets plus
tickets43 14
$83.50 .
1442443 123 14
44244
424
3 14243
83.50
5.50x
 3.50(21  x)

5.50x  3.50(21  x)  83.50
5.5x  73.5  3.5x  83.5
2x  73.5  83.5
2x  73.5  73.5  83.5  73.5
2x  10
2x
2

10
2

Peanuts
Chocolate
Mixture

Price per
Unit
$4.50
$6.50
$5.25

Total
Price
4.50(5)
6.50x
5.25(5  x)

Price of
price of
price of
peanuts
plus
chocolate
equals
mixture.
14243 123 14
4244
31
424
3 1442443
4.50(5) 
6.50x
 5.25(5  x)
4.50(5)  6.50x  5.25(5  x)
22.5  6.5x  26.25  5.25x
22.5  6.5x  5.25x  26.25  5.25x  5.25x
22.5  1.25x  26.25
22.5  1.25x  22.5  26.25  22.5
1.25x  3.75
1.25x
1.25

3.75

 1.25

x3
3 lb of chocolate should be mixed with 5 lb of
peanuts.
5. Let t  the number of hours until they meet.
Sheila
Casey

r
55
65

t
t
t

d  rt
55t
65t

Distance
distance
traveled
traveled
by
Sheila
plus
by
Casey
equals
210
miles.
14
4244
3 123 14
4244
31
4
424
4
3 14
424
43
55t

65t

210
55t  65t  210
120t  210
120t
120

210

 120

t  1.75
65t  65(1.75) or 113.75
They will meet in 1.75 hours, and they will be
113.75 miles from Bozeman.
Extra Practice

xPoint Coordinate
B
1
T
5
P
6
Q
0
A
2
K
4
J
2
L
4
S
3

yCoordinate
2
0
2
6
2
5
5
0
5

Ordered
Pair
(1, 2)
(5, 0)
(6, 2)
(0, 6)
(2, 2)
(4, 5)
(2, 5)
(4, 0)
(3, 5)

Quadrant
I
none
IV
none
III
I
IV
none
II

10. A(2, 3)


Start at the origin.
Move right 2 units and down 3 units.
Draw a dot and label it A.
(See coordinate plane after Exercise 18.)
11. B(3, 6)
Start at the origin.
Move right 3 units and up 6 units.
Draw a dot and label it B.
(See coordinate plane after Exercise 18.)
12. C(4, 0)
Start at the origin.
Move left 4 units.
Since the y-coordinate is 0, the point is on the
x-axis.
Draw a dot and label it C.
(See coordinate plane after Exercise 18.)
13. D(4, 3)
Start at the origin.
Move left 4 units and up 3 units.
Draw a dot and label it D.
(See coordinate plane after Exercise 18.)
14. E(5, 5)
Start at the origin.
Move left 5 units and down 5 units.
Draw a dot and label it E.
(See coordinate plane after Exercise 18.)
15. F(1, 1)
Start at the origin.
Move left 1 unit and up 1 unit.
Draw a dot and label it F.
(See coordinate plane after Exercise 18.)
16. G(0, 2)
Start at the origin.
Since the x-coordinate is 0, the point is on the
y-axis.
Move down 2 units.
Draw a dot and label it G.
(See coordinate plane after Exercise 18.)

x5
21  x  21  5 or 16
5 adult tickets and 16 child tickets were sold.
4. Let x  the number of pounds of chocolate in the
mixture.
Units
5
x
5x

Lesson 4-1

664

PQ249J-6481F-15-19[643-685] 26/9/02 9:51 PM Page 665 Sahuja Ahuja_QXP_06:Desktop Folder:Chandra:Algebra_FNL_Delivery:

5b.

17. H(2, 3)
Start at the origin.
Move right 2 units and up 3 units.
Draw a dot and label it H.
(See coordinate plane after Exercise 18.)
18. J(0, 3)
Start at the origin.
Since the x-coordinate is 0, the point is on the
y-axis.
Move up 3 units.
Draw a dot and label it J.
1018.
y

A' A

S' S

Q'

Q
x

6a. To dilate the triangle by a scale factor of 2,


multiply the coordinates of each vertex by 2.
(x, y) S (2x, 2y)
R(2, 1) S R(2  2, 2  1) S R(4, 2)
E(3, 1) S E(2  (3), 2  (1) ) S E(6, 2)
D(2, 4) S D(2  2, 2  (4) ) S D(4, 8) )

B
D

U'

6b.

R'

F
C

R
x

E'

Page 828

D'

Lesson 4-2

7a. To reflect the pentagon over the x-axis, multiply


the y-coordinate of each vertex by 1.
(x, y) S (x, y)
B(3, 5) S B(3, 5)
L(4, 5) S L(4, 5)
A(4, 1) S A(4, 1)
C(0, 4) S C(0, 4)
K(4, 1) S K(4, 1)
7b.
y
B'
L'

1. The figure has been flipped over a line. This is a


reflection.
2. The figure has been shifted horizontally to the
left. This is a translation.
3. The figure has been increased in size. This is a
dilation.
4a. To translate the quadrilateral 1 unit up, add 1
to the y-coordinate of each vertex. To translate
the quadrilateral 2 units right, add 2 to the
x-coordinate of each vertex.
(x, y) S (x  2, y  1)
A(2, 2) S A(2  2, 2  1) S A(4, 3)
B(3, 5) S B(3  2, 5  1) S B(1, 6)
C(4, 0) S C(4  2, 0  1) S C(2, 1)
D(2, 2) S D(2  2, 2  1) S D(4, 1)
4b.
y
B'
B

A'

K'
C'
B

8a. To rotate the triangle 90 counterclockwise


about the origin, switch the coordinates of each
vertex and then multiply the new first
coordinate by 1.
(x, y) S (y, x)
A(2, 1) S A(1, 2)
N(4, 1) S N(1, 4)
G(3, 4) S G(4, 3)

C'
x

A'

D'
D
5a. To reflect the square over the y-axis, multiply
the x-coordinate of each vertex by 1.
(x, y) S (x, y)
S(1, 1) S S(1, 1)
Q(4, 1) S Q(4, 1)
U(4, 4) S U(4, 4)
A(1, 4) S A(1, 4)

665

Extra Practice

PQ249J-6481F-15-19[643-685] 26/9/02 9:51 PM Page 666 Sahuja Ahuja_QXP_06:Desktop Folder:Chandra:Algebra_FNL_Delivery:

8b.

2. Table

N'

x
4
2
0
2

G'
A'
N

y
2
0
2
4

Graph

Mapping
y

9a. To translate the parallelogram 2 units down,


add 2 to the y-coordinate of each vertex. To
translate the parallelogram 1 unit left, add 1
to the x-coordinate of each vertex.
(x, y) S (x  1, y  2)
G(3, 2) S G(3  1, 2  2) S G(4, 4)
R(4, 2) S R(4  1, 2  2) S R(3, 4)
A(6, 4) S A(6  1, 4  2) S A(5, 2)
M(1, 4) S M(1  1, 4  2) S M(2, 2)
9b.

A'

x
7
2
4
5
9

G
R'

G'

4
2
0
2

0
2
4

The domain of this relation is {4, 2, 0, 2}. The


range is {0, 2, 4}.
3. Table

M'

y
5
3
0
7
2

Graph
y

Page 829

Lesson 4-3

1. Table
x
5
0
9

y
2
0
1

Graph
y

Mapping

Mapping

5
0
9

2
0
1

7
2
4
5
9

5
3
0
7
2

The domain of this relation is {9, 2, 4, 5, 7}.


The range is {7, 3, 0, 2, 5}.

The domain of this relation is {9, 0, 5}. The


range is {1, 0, 2}.

Extra Practice

666

PQ249J-6481F-15-19[643-685] 26/9/02 9:51 PM Page 667 Sahuja Ahuja_QXP_06:Desktop Folder:Chandra:Algebra_FNL_Delivery:

4. Table
x
3.1
4.7
2.4
9

Page 829
1.

y
1
3.9
3.6
12.12

Graph
y

Lesson 4-4

x
0

y
1

y  3x  1
1  3(0)  1
1  1
2  3(4)  1
2  11
4  3(2)  1
45
5  3(2)  1
55

True or False?
true
false
false
true

The solution set is {(0, 1), (2, 5)}.


2.

x
1

y
8

3y  x  7
3(8)  1  7
24  8
3(7)  0  7
21  7
3(3)  2  7
99
3(4)  5  7
12  12

True or False?
false
false
true
true

The solution set is {(2, 3), (5, 4)}.


3.

Mapping

X
3.1
4.7
2.4
9

5.
6.
7.
8.
9.

10.

Y
1
3.9
3.6
12.12

The domain of this relation is {9, 4.7, 2.4, 3.1}.


The range is {3.6, 1, 3.9, 12.12}.
relation: {(1, 3), (2, 4), (3, 5), (4, 6), (5, 7)}
inverse: {(3, 1), (4, 2), (5, 3), (6, 4), (7, 5)}
relation: {(4, 1), (2, 3), (0, 1), (2, 3), (4, 1)}
inverse: {(1, 4), (3, 2), (1, 0), (3, 2), (1, 4)}
relation: {(1, 5), (2, 5), (2, 4), (2, 1), (6, 1)}
inverse: {(5, 1), (5, 2), (4, 2), (1, 2), (1, 6)}
relation: {(3, 7), (5, 2), (9, 1), (3, 2)}
inverse: {(7, 3), (2, 5), (1, 9), (2, 3)}
relation: {(4, 3), (2, 2), (2, 1), (0, 0), (1, 1),
(2, 3), (2, 1)}
inverse: {(3, 4), (2, 2), (1, 2), (0, 0), (1, 1),
(3, 2), (1, 2)}
relation: {(3, 1), (3, 3), (2, 2), (2, 0),
(1, 3), (1, 1), (0, 2), (1, 3), (1, 1), (2, 0),
(2, 2), (3, 1), (3, 3)}
inverse: {(1, 3), (3, 3), (2, 2), (0, 2),
(3, 1), (1, 1), (2, 0), (1, 1), (3, 1), (0, 2),
(2, 2), (1, 3), (3, 3)}

x
2

y
0

4

12

4x  8  2y
4(2)  8  2(0)
88
4(0)  8  2(4)
00
4(0)  8  2(2)
04
4(4)  8  2(12)
16  16

True or False?
true
true
false
true

The solution set is {(2, 0), (0, 4), (4, 12)}.


4.

3x  10  4y
True or False?
3(10)  10  4(5)
false
30  10
1
3(2)  10  4(1)
true
66
0.25
3(3)  10  4(0.25)
true
99
5
3(5)  10  4(5)
false
15  10

x
y
10 5
2
3
5

The solution set is {(2, 1), (3, 0.25)}.


5. First solve the equation for y in terms of x.
xy3
xyx3x
y3x
x
2
1
0
1
2

3
3
3
3
3

3x
 (2)
 (1)
0
1
2

y
5
4
3
2
1

(x, y)
(2, 5)
(1, 4)
(0, 3)
(1, 2)
(2, 1)

The solution set is


{(2, 5), (1, 4), (0, 3), (1, 2), (2, 1)}.

667

Extra Practice

PQ249J-6481F-15-19[643-685] 26/9/02 9:51 PM Page 668 Sahuja Ahuja_QXP_06:Desktop Folder:Chandra:Algebra_FNL_Delivery:

6.

x
2
1
0
1
2

10. First solve the equation for y in terms of x.


2x  y  4
2x  y  2x  4  2x
y  4  2x

(x, y)
(2, 2)
(1, 1)
(0, 0)
(1, 1)
(2, 2)

y
2
1
0
1
2

x
2
1
0
1
2

The solution set is


{(2, 2), (1, 1), (0, 0), (1, 1), (2, 2)}.
7.

x
2
1
0
1
2

5x  1
5(2)  1
5(1)  1
5(0)  1
5(1)  1
5(2)  1

y
9
4
1
6
11

(x, y)
(2, 9)
(1, 4)
(0, 1)
(1, 6)
(2, 11)

y

11.

13  4x
3

2

13  4(2)
3

1

13  4(1)
3

17
3

13  4(0)
3

13
3

13  4(1)
3

13  4(2)
3

y
7

(x, y)

5
3

5 (2, 7), 11, 173 2, 10, 133 2, (1, 3), 12, 53 26.

11, 173 2
10, 133 2
12, 53 2

2

8  4(2)
5

16
5

1

8  4(1)
5

12
5

8  4(0)
5

8
5

8  4(1)
5

4
5

8  4(2)
5

y
2
3
4
5
6

(x, y)
(2, 2)
(1, 3)
(0, 4)
(1, 5)
(2, 6)

10  2x
3
10  2x
3

10  2x
3

2

10  2(2)
3

1

10  2(1)
3

y
14
3

(x, y)

12, 165 2
11, 125 2
10, 85 2
11, 45 2
(2, 0)

512, 165 2, 11, 125 2, 10, 85 2, 11, 45 2, (2, 0) 6.


668

(x, y)

12, 143 2
(1, 4)

10  2(0)
3

10
3

10  2(1)
3

8
3

10  2(2)
3

512, 143 2, (1, 4), 10, 103 2, 11, 83 2 (2, 2) 6.

The solution set is

Extra Practice

4x
 (2)
 (1)
0
1
2

The solution set is

8  4x
5
8  4x
5

y

9. First solve the equation for y in terms of x.


5y  8  4x
5y
8  4x
 5
5

4
4
4
4
4

3y
3

(1, 3)

The solution set is

y

x
2
1
0
1
2

(2, 7)

(x, y)
(2, 8)
(1, 6)
(0, 4)
(1, 2)
(2, 0)

y
8
6
4
2
0

The solution set is


{(2, 2), (1, 3), (0, 4), (1, 5), (2, 6)}.
12. First solve the equation for y in terms of x.
2x  3y  10
2x  3y  2x  10  2x
3y  10  2x

13  4x
3
13  4x
3

4  2x
 2(2)
 2(1)
 2(0)
 2(1)
 2(2)

The solution set is


{(2, 8), (1, 6), (0, 4), (1, 2), (2, 0)}.

The solution set is


{(2, 9), (1, 4), (0, 1), (1, 6), (2, 11)}.
8. First solve the equation for y in terms of x.
4x  3y  13
4x  3y  4x  13  4x
3y  13  4x
3y
3

4
4
4
4
4

10, 103 2
11, 83 2
(2, 2)

PQ249J-6481F-15-19[643-685] 26/9/02 9:51 PM Page 669 Sahuja Ahuja_QXP_06:Desktop Folder:Chandra:Algebra_FNL_Delivery:

16. First solve the equation for y in terms of x.


x  4y  2
x  4y  x  2  x
4y  2  x

13. First solve the equation for y in terms of x.


2y  3x  1
2y
2

y

3x  1
2
3x  1
2

4y
4

3x  1
2

2

3(2)  1
2

2

1

3(1)  1
2

3(0)  1
2

3(1)  1
2

1

12,  52 2

(1, 1)

10, 12 2

1
2

3(2)  1
2

y

(x, y)

(1, 2)
7
2

512, 52 2, (1, 1), 10, 12 2, (1, 2), 12, 72 26.

The solution set is

12, 72 2

x1
2  1
1  1
01
11
21

y
3
2
1
0
1

2  x
4

8

2  (8)
4

2.5

(8, 2.5)

4

2  (4)
4

1.5

(4, 1.5)

2  0
4

0.5

(0, 0.5)

2  4
4

0.5

(4, 0.5)

2  8
4

1.5

(8, 1.5)

(x, y)

(x, y)
(2, 3)
(1,2)
(0, 1)
(1, 0)
(2, 1)

17. First solve the equation for y in terms of x.


y3x
y33x3
yx3

Graph the solution set


{(2, 3), (1, 2), (0, 1), (1, 0), (2, 1)}.

x
5
1
3
7
9

2  x
4
2  x
4

Graph the solution set


{(8, 2.5), (4, 1.5), (0, 0.5), (4, 0.5), (8, 1.5)}.

14. First solve the equation for y in terms of x.


xy1
x1y11
x1y
x
2
1
0
1
2

x3
5  3
1  3
33
73
93

(x, y)
(5, 2)
(1, 2)
(3, 6)
(7, 10)
(9, 12)

y
2
2
6
10
12

Graph the solution set


{(5, 2), (1, 2), (3, 6), (7, 10), (9, 12)}.
y

15.

x1
3  1
1  1
01
11
31

x
3
1
0
1
3

y
2
0
1
2
4

(x, y)
(3, 2)
(1, 0)
(0, 1)
(1, 2)
(3, 4)

Graph the solution set


{(3, 2), (1, 0), (0, 1),(1, 2), (3, 4)}.
O

669

Extra Practice

PQ249J-6481F-15-19[643-685] 26/9/02 9:51 PM Page 670 Sahuja Ahuja_QXP_06:Desktop Folder:Chandra:Algebra_FNL_Delivery:

20. First solve the equation for y in terms of x.

18. First solve the equation for y in terms of x.


x  y  2
x  y  x  2  x
y  2  x
2  x
2  (4)
2  (3)
2  0
2  1
2  3

x
4
3
0
1
3

3y  3x  4
1
(3y)
3

y

(x, y)
(4, 2)
(3, 1)
(0, 2)
(1, 3)
(3, 5)

y
2
1
2
3
5

2
x
9

4
3

4
3

8
3

9 x  3

x
6

2
(6)
9

3

2
(3)
9

3

Graph the solution set


{(4, 2), (3, 1), (0, 2), (1, 3), (3, 5)}.
y

2
(0)
9

2
(1)
9

2
(3)
9

3

(x, y)

2

16, 83 2

(3, 2)

4
3

4
3

3

9

3

10
2

10, 43 2
11, 109 2
13, 23 2

516, 83 2, (3, 2), 10, 43 2, 11, 109 2, 13, 23 26.

Graph the solution set

1 2

 3 3x  4

19. First solve the equation for y in terms of x.


2x  3y  5
2x  3y  2x  5  2x
3y  5  2x
3y
3

y

5

5  2(5)
3

3

3

5  2(3)
3

3

5  2(0)
3

5  2(5)
3
5  2(6)
3

y
5
1
5
3

y  4 

(x, y)

15, 53 2
13, 13 2
10, 53 2

17
3

515, 53 2, 13, 13 2, 10, 53 2, (5, 5), 16, 173 26.

Graph the solution set

(5, 5)

16, 173 2

3
x
4

4  4x

4

4  4 (4)

4  4 (0)

4  4 (4)

4 

3
(6)
4

0.5

4 

3
(8)
4

(x, y)

y
7

(4, 7)

4

(0, 4)

1

(4, 1)
(6, 0.5)
(8, 2)

Graph the solution set


{(4, 7), (0, 4), (4, 1), (6, 0.5), (8, 2)}.
y

Extra Practice

2 (2y)  2 8  2x

2y  8  2x

5  2x
3

21. First solve the equation for y in terms of x.

5  2x
3
5  2x
3

670

PQ249J-6481F-15-19[643-685] 26/9/02 9:51 PM Page 671 Sahuja Ahuja_QXP_06:Desktop Folder:Chandra:Algebra_FNL_Delivery:

Page 829

Lesson 4-5

1. First rewrite the equation so that the variables


are on one side of the equation.
3x  2y
3x  2y  2y  2y
3x  2y  0
The equation is now in standard form where A  3,
B  2, and C  0. This is a linear equation.
2. Since the term y2 has an exponent of 2, the equation
cannot be written in the form Ax  By  C.
Therefore, this is not a linear equation.
3. First rewrite the equation so that the variables
are on one side of the equation and a constant is
on the other side.
4x  2y  8
4x  2y  2y  8  2y
4x  2y  8
Since the GCF of 4, 2, and 8 is not 1, divide
each side by 2.
2(2x  y)  8
2(2x  y)
2

8. Select five values for the domain and make a table.


3x  1
3(2)  1
3(1)  1
3(0)  1
3(1)  1
3(2)  1

x
2
1
0
1
2

(x, y)
(2, 5)
(1 2)
(0, 1)
(1, 4)
(2, 7)

2

1
x

9. Solve the equation for y.


3x  2y  12
3x  2y  3x  12  3x
2y  12  3x
2y
2

y

12  3x
2
3x  12
2

Select five values for the domain and make a table.

5
y

y
7
4
1
2
5

6. Since each of the terms and has a variable in


the denominator, the equation cannot be written
in the form Ax  By  C. Therefore, this is not a
linear equation.
7. Solve the equation for y.
3x  y  4
3x  y  3x  4  3x
y  4  3x
Select five values for the domain and make a table.
4  3x
4  3(1)
4  3(0)
4  3(1)
4  3(2)
4  3(3)

y
5
2
1
4
7

Graph the ordered pairs and draw a line through


the points.

2x  y  4
The equation is now in standard form where A  2,
B  1, and C  4. This is a linear equation.
4. First rewrite the equation so that the variables
are on one side of the equation and a constant is
on the other side.
5x  7y  2x  7
5x  7y  2x  2x  7  2x
3x  7y  7
The equation is now in standard form where A  3,
B  7, and C  7. This is a linear equation.
5. First simplify. Then rewrite the equation so that
the variables are on one side of the equation.
2x  5x  7y  2
7x  7y  2
7x  7y  7y  2  7y
7x  7y  2
The equation is now in standard form where A  7,
B  7, and C  2. This is a linear equation.

x
1
0
1
2
3

3x  12
2

(x, y)

1

3(1)  12
2

7.5

(1, 7.5)

3(0)  12
2

6

3(1)  12
2

4.5

3(2)  12
2

3

3(4)  12
2

(0, 6)
(1, 4.5)
(2, 3)
(4, 0)

Graph the ordered pairs and draw a line through


the points.
y

(x, y)
(1, 7)
(0, 4)
(1, 1)
(2, 2)
(3, 5)

Graph the ordered pairs and draw a line through


the points.

671

Extra Practice

PQ249J-6481F-15-19[643-685] 26/9/02 9:51 PM Page 672 Sahuja Ahuja_QXP_06:Desktop Folder:Chandra:Algebra_FNL_Delivery:

12. The only value in the range is 2. Since there is


no x in the equation, the value of x does not
depend on y. Therefore, x can be any real number.
Select five values for the domain and make a
table.

10. Solve the equation for y.


2x  y  6
2x  y  2x  6  2x
y  6  2x
1(y)  1(6  2x)
y  2x  6
Select five values for the domain and make a table.
2x  6
2(1)  6
2(0)  6
2(1)  6
2(2)  6
2(3)  6

x
1
0
1
2
3

y
8
6
4
2
0

x
3
1
0
1
4

(x, y)
(1, 8)
(0, 6)
(1, 4)
(2, 2)
(3, 0)

y
2
2
2
2
2

(x, y)
(3, 2)
(1, 2)
(0, 2)
(1, 2)
(4, 2)

Graph the ordered pairs and draw a line through


the points.

Graph the ordered pairs and draw a line through


the points.

y
x

13. To find the x-intercept, let y  0.


y  5x  7
0  5x  7
0  7  5x  7  7
7  5x

11. Solve the equation for y.


2x  3y  8
2x  3y  2x  8  2x
3y  8  2x
3y
3

y

7
5
7
5

8  2x
3
2x  8
3

2x  8
3

2

2(2)  8
3

(x, y)

y
4

(2, 4)

2(0)  8
3

2(1)  8
3

2(2)  8
3

3

2(4)  8
3

3
2

10, 83 2

(1, 2)
4

12, 43 2

x

(4, 0)

Extra Practice

5x
5

17 2

Graph the ordered pairs and draw a line through


the points.

The graph intersects the x-axis at 5, 0 .


To find the y-intercept, let x  0.
y  5x  7
y  5(0)  7
y  7
The graph intersects the y-axis at (0, 7).
Plot these points and draw the line that connects
them.

Select five values for the domain and make a table.


x

672

PQ249J-6481F-15-19[643-685] 26/9/02 9:51 PM Page 673 Sahuja Ahuja_QXP_06:Desktop Folder:Chandra:Algebra_FNL_Delivery:

16. To find the x-intercept, let y  0.


5x  2y  8
5x  2(0)  8
5x  8

14. The only value in the domain is 4. Since there is


no y in the equation, the value of y does not
depend on x. Therefore, y can be any real number.
Select five values for the range and make a table.
x
4
4
4
4
4

y
4
2
0
1
3

5x
5

(x, y)
(4, 4)
(4, 2)
(4, 0)
(4, 1)
(4, 3)

x5
The graph intersects the x-axis at
To find the y-intercept, let x  0.
5x  2y  8
5(0)  2y  8
2y  8

Graph the ordered pairs and draw a line through


the points.

2y
2

185, 02.

 2

y  4
The graph intersects the y-axis at (0, 4).
Plot these points and draw the line that passes
through them.
x

5

15. To find the x-intercept, let y  0.


1

x  3y  2
1

x  3 (0)  2
x2
The graph intercepts the x-axis at (2, 0).
To find the y-intercept, let x  0.

17. To find the x-intercept, let y  0.


4.5x  2.5y  9
4.5x  2.5(0)  9
4.5x  9

x  3y  2
1

0  3y  2

1
y
3
1
y
3

1 2

4.5x
4.5

2

 4.5

x2
The graph intersects the x-axis at (2, 0).
To find the y-intercept, let x  0.
4.5x  2.5y  9
4.5(0)  2.5y  9
2.5y  9

 3(2)

y6
The graph intersects the y-axis at (0, 6).
Plot these points and draw the line that passes
through them.

2.5y
2.5

 2.5

y  3.6
The graph intersects the y-axis at (0, 3.6).
Plot these points and draw the line that passes
through them.
y

673

Extra Practice

PQ249J-6481F-15-19[643-685] 26/9/02 9:51 PM Page 674 Sahuja Ahuja_QXP_06:Desktop Folder:Chandra:Algebra_FNL_Delivery:

18. To find the x-intercept, let y  0.


1
x
2
1
x
2

8. Graph the equation. For each value of x, a vertical


line passes through no more than one point on the
graph. Thus, the line represents a function.

 3y  12

 3(0)  12

1
x
2
1
x
2

 12

1 2  2(12)

x  24
The graph intersects the x-axis at (24, 0).
To find the y-intercept, let x  0.
1
x
2

 3y  12

1
(0)
2

 3y  12

9. Graph the equation. For each value of x, a vertical


line passes through no more than one point on the
graph. Thus, the graph represents a function.

3y  12
3y
3

12
3

y4
The graph intersects the y-axis at (0, 4).
Plot these points and draw the line that passes
through them.

12
9
6
3

12 9 6 3
3
6
9
12

10.
x

Page 830

O
3 6 9 12x

f(x)  2x  5
f(4)  2(4)  5
 8  5
 3

11. g(x)  3x2  1


g(2)  3(2) 2  1
 3(4)  1
 12  1
 11
12.
f(x)  2x  5
f(3)  5  [ 2(3)  5]  5
 (6  5)  5
 11  5
6

Lesson 4-6

1. Since an element of the domain is paired with


more than one element in the range, the relation
is not a function. Both 3 and 7 in the range are
paired with x  1.
2. The mapping does not represent a function since
the element 2 in the domain is paired with both
4 and 2 in the range. Also, 3 in the domain is
paired with both 2 and 0 in the range.
3. For each value of x, a vertical line passes through
no more than one point on the graph. Thus, the
relation graphed represents a function.
4. Since an element of the domain is paired with
more than one element in the range, the relation
is not a function. Both 3 and 4 in the range are
paired with x  1.
5. Since each element of the domain is paired with
exactly one element of the range, the relation is a
function.
6. Since each element of the domain is paired with
exactly one element of the range, the relation is a
function.
7. Since an element of the domain is paired with
more than one element in the range, the relation
is not a function. Both 7 and 5 in the range are
paired with x  4.

Extra Practice

g(x)  3x2  1
g(2)  4  [3(2) 2  1]  4
 [3(4)  1]  4
 (12  1)  4
 11  4
 15
14. f(x)  2x  5
f(b2 )  2(b2 )  5
 2b2  5
13.

15.

g(x)  3x2  1
g(a  1)  3(a  1) 2  1
 3(a2  2a  1)  1
 (3a2  6a  3)  1
 3a2  6a  2

16. f(x)  2x  5 and g(x)  3x2  1


f(0)  g(3)  [ 2(0)  5]  [ 3(3) 2  1]
 (0  5)  [ 3(9)  1]
 5  (27  1)
 5  26
 31

674

PQ249J-6481F-15-19[643-685] 26/9/02 9:51 PM Page 675 Sahuja Ahuja_QXP_06:Desktop Folder:Chandra:Algebra_FNL_Delivery:

17. f(x)  2x  5 and g(x)  3x2  1


f(n)  g(n)  [2(n)  5]  [3(n) 2  1]
 (2n  5)  (3n2  1)
 3n2  2n  4

Page 830
1. 2

2. 3

12

3. 2

16


2
4
8

4. 21 16 11 6


5

5

5

5. 0 0.25 0.5 0.75

1
3

1
9

1
81

1
27

9

27

13

81

23

33

10

10

10

10

11.

8

9
8



1
1
1
1
1
1
8

5
4

8

11
8

8

8

8

8

3
2

8

12. 3

6

11

3

6






1
1
1
1
1
1
2

2

2

2

2

1
2.

The common difference is


Add 2 to the last
1
term of the sequence and continue adding 2
until the next three terms are found.
11

6

17

3

10

6

3




1
1
1
2

2

2

17

10

The next three terms are 3,  6 ,  3 .


13. Use the formula for the nth term of an arithmetic
sequence with a1  3, d  6, and n  12.
an  a1  (n  1)d
a12  3  (12  1)6
a12  3  66
a12  69

10

The 12th term of the arithmetic sequence is 69.


14. Use the formula for the nth term of an arithmetic
sequence with a1  2, d  4, and n  8.
an  a1  (n  1)d
a8  2  (8  1)4
a8  2  28
a8  26

2

The 8th term of the arithmetic sequence is 26.


15. Use the formula for the nth term of an arithmetic
sequence with a1  1, d  3, and n  10.
an  a1  (n  1)d
a10  1  (10  1) (3)
a10  1  (27)
a10  28

   
0.6

8

5 11 3
, .
8 2

The next three terms are 12, 14, 16.


9. 2 1.4 0.8 0.2
?
?
?
0.6

7
8

8




2
2
2

0.6

8

The next three terms are 4,

The common difference is 2. Add 2 to the last


term of the sequence and continue adding 2
until the next three terms are found.
10 12 14 16

0.6

8


1
1
1

2

3
4

9
8

   
2

The next three terms are 43, 53, 63.


8. 4 6 8 10
?
?
?
2

The common difference is 8. Add 8 to the last term


1
of the sequence and continue adding 8 until the
next three terms are found.


10 10 10

2

8

8

The common difference is 10. Add 10 to the last


term of the sequence and continue adding 10 until
the next three terms are found.
33 43 53 63

2

The next three terms are 37, 45, 53.



10

29



8
8
8

This is not an arithmetic


sequence because the
difference between terms is
not constant.



2
2
2

7. 3

8

This is an arithmetic
sequence because the
difference between terms is
constant. The common
difference is 0.25.


0.25 0.25 0.25

21

The common difference is 8. Add 8 to the last


term of the sequence and continue adding 8 until
the next three terms are found.
29
37
45
53

This is an arithmetic
sequence because the
difference between terms
is constant. The common
difference is 1.
This is not an arithmetic
sequence because the
difference between terms
is not constant.
This is not an arithmetic
sequence because the
difference between terms
is not constant.
This is an arithmetic
sequence because the
difference between terms is
constant. The common
difference is 5.


2
3
4

13



Lesson 4-7
1



1
1
1

6.

10. 5

0.6 0.6

The 10th term of the arithmetic sequence is 28.

The common difference is 0.6. Add 0.6 to the last


term of the sequence and continue adding 0.6
until the next three terms are found.
0.2
0.4 1.0
1.6



0.6 0.6 0.6

The next three terms are 0.4, 1.0, 1.6.

675

Extra Practice

PQ249J-6481F-15-19[643-685] 26/9/02 9:51 PM Page 676 Sahuja Ahuja_QXP_06:Desktop Folder:Chandra:Algebra_FNL_Delivery:

20. 25

16. Use the formula for the nth term of an arithmetic


sequence with a1  2.2, d  1.4, and n  5.
an  a1  (n  1)d
a5  2.2  (5  1)(1.4)
a5  2.2  5.6
a5  7.8

40



5
5

The first term is 2. The common difference is 5.


Use the formula for the nth term of an arithmetic
sequence with a1  2, d  5, and n  12.
an  a1  (n  1)d
a12  2  (12  1)(5)
a12  2  (55)
a12  57

15n  10

an

(n, an)

1
2
3
4
5

15(1)
15(2)
15(3)
15(4)
15(5)

 10
 10
 10
 10
 10

25
40
55
70
85

(1,
(2,
(3,
(4,
(5,

The 12th term of the arithmetic sequence is 57.


1

28

90
80
70
60
50
40
30
20
10

14

18


3
3
3
8

8

8

The first term is 22. The common difference is 8.


Use the formula for the nth term of an arithmetic
1
3
sequence with a1  22, d  8, and n  10.
an  a1  (n  1)d
1

1 32

a10  22  (10  1) 8
1

a10  22  38
7

a10  8

21. 9

The first term is 3. The common difference is 4.


Use the formula for the nth term of an equation
with a1  3 and d  4.
an  a1  (n  1)d
an  3  (n  1)4
an  3  4n  4
an  3  4n  4
an  4n  7

20
15
10
5
O
5

6n  15

an

(n, an)

6(1)  15
6(2)  15
6(3)  15
6(4)  15
6(5)  15

9
3
3
9
15

(1, 9)
(2, 3)
(3, 3)
(4, 9)
(5, 15)

4n  7

an

(n, an)

7
7
7
7
7

3
1
5
9
13

(1, 3)
(2, 1)
(3, 5)
(4, 9)
(5, 13)

15
12
9
6
3
O
3
6
9

an

1 2 3 4 5n

Extra Practice

1 2 3 4 5n

3

1
2
3
4
5

4(1)
4(2)
4(3)
4(4)
4(5)

an

The first term is 9. The common difference is 6.


Use the formula for the nth term to write an
equation with a1  9 and d  6.
an  a1  (n  1)d
an  9  (n  1)6
an  9  6n  6
an  6n  15


4
4
4

1
2
3
4
5

25)
40)
55)
70)
85)




6
6
6

The 10th term of the arithmetic sequence is 8.


19. 3
1
5
9

70

The first term is 25. The common difference is 15.


Use the formula for the nth term to write an
equation with a1  25, d  15.
an  a1  (n  1)d
an  25  (n  1)15
an  25  15n  15
an  15n  10

The 5th term of the arithmetic sequence is 7.8.


17. 2 7 12

18. 22

55



15 15 15

676

an

1 2 3 4 5n

PQ249J-6481F-15-19[643-685] 26/9/02 9:51 PM Page 677 Sahuja Ahuja_QXP_06:Desktop Folder:Chandra:Algebra_FNL_Delivery:

22. 3.5

2 0.5

3. 12

The first term is 3.5. The common difference


is 1.5. Use the formula for the nth term to write
an equation with a1  3.5 and d  1.5.
an  a1  (n  1)d
an  3.5  (n  1)1.5
an  3.5  1.5n  1.5
an  1.5n  5
n

1.5n  5

an

(n, an)

1
2
3
4
5

1.5(1)  5
1.5(2)  5
1.5(3)  5
1.5(4)  5
1.5(5)  5

3.5
2
0.5
1
2.5

(1, 3.5)
(2, 2)
(3, 0.5)
(4, 1)
(5, 2.5)

3
2
1
O
1
2
3
4

23

34

45


11 11 11



1.5 1.5 1.5

The difference between each pair of terms is


always 11. The sequence is arithmetic with a
common difference of 11. Each term is 11 more
than the term before it. Add 11, 11, and 11.
45
56
67
78


11 11 11

The next three terms are 56, 67, and 78.


4. 39
33
27
21



6
6
6

The difference between each pair of terms is


always 6. The sequence is arithmetic with a
common difference of 6. Each term is 6 less than
the term before it. Add 6, 6, and 6.
21
15
9
3


6
6
6

an

The next three terms are 15, 9, and 3.


5. 6.0
7.2 8.4
9.6



1.2 1.2 1.2

1 2 3 4 5n

The difference between each pair of terms is


always 1.2. The sequence is arithmetic with a
common difference of 1.2. Each term is 1.2 more
than the term before it. Add 1.2, 1.2, and 1.2.
9.6 10.8 12.0 13.2



1.2 1.2 1.2

Page 830

The next three terms are 10.8, 12.0, and 13.2.


6. 86
81.5 77
72.5

Lesson 4-8




4.5
4.5 4.5

1. The pattern consists of circles with one-tenth


shaded. The section that is shaded is the fourth
section in a clockwise direction from the
previously-shaded section. The next two figures in
the pattern are shown:

The difference between each pair of terms is


always 4.5. The sequence is arithmetic with a
common difference of 4.5. Each term is 4.5 less
than the term before it. Add 4.5, 4.5, and
4.5.
72.5
68 63.5
59



4.5 4.5
4.5

The next three terms are 68, 63.5, and 59.


7. 4
8
16
32

2. The pattern consists of a figure that is alternately


flipped (or reflected) then enlarged and flipped.
Continue the pattern by flipping the last figure
without changing its size or shape, then enlarging
and flipping the figure you just sketched. The
next two figures in the pattern are shown.



4
8
16

The difference between each pair of terms doubles


for each successive pair. Continue doubling each
successive difference. Add 32, 64, and 128.
32
64
128
256




32
64 128

The next three terms are 64, 128, and 256.


8. 3125
625 125
25


100

2500 500

The difference between each pair of successive


1
terms is 5 of the difference between the previous
1
pair of terms. Continue taking 5 of each
successive difference. Add 20, 4, and 0.8.
25
5
1
0.2



20
4
0.8

The next three terms are 5, 1, and 0.2.

677

Extra Practice

PQ249J-6481F-15-19[643-685] 26/9/02 9:51 PM Page 678 Sahuja Ahuja_QXP_06:Desktop Folder:Chandra:Algebra_FNL_Delivery:

9. 15

16

18

21

25

14. Make a table of ordered pairs for several points


on the graph.

30


1
2
3
4
5

1

The difference between each pair of terms


increases by 1 for each successive pair. Continue
increasing each successive difference by 1. Add 6,
7, and 8.
30
36
43
51

1

2

1

2


1

2

1

1
2
1
2

2

1

1

3
2

2

y is always 1
1
less than 2x.

1
 2x  1.
1

If x  2, then y  2 (2) 1 or 0.


1

If x  1, then y  2 (1)  1 or 2.


1

1

Thus, y  2x  1 describes this relation. Since


this relation is also a function, we can write the
1
equation in function notation as f(x)  2x  1.
15. Make a table of ordered pairs for several points
on the graph.



1

1

1
1

1

 
0
0



1
1

1

1
1

2
2

3


1

x
y

The difference of the x values is 1, and the


difference of the y values is 1. The difference in
y values is the opposite of the difference in x
values. This suggests y  x.
Check:
If x  1, then y  (1) or 1.
If x  2, then y  (2) or 2.
Thus, y  x describes the relation. Since this
relation is also a function, we can write the
equation in function notation as f(x)  x.
13. Make a table of ordered pairs for several points
on the graph.
1

1
3


0



0

1

0
3



0

3
0

0
2

3
4


2
2

The difference of x values is 3, and the difference of


2
y values is 2. The difference of the y values is 3 the
2
difference of the x values. This suggests y  3x.
2

If x  3, then y  3 (3) or 2. But


the y value for x  3 is 0. This is a
difference of 2. Try other values in the
domain to see if the same difference
occurs.

Check:

2

1
3

3



3
3

3

2
x
3

2


0

Check y 

2
x
3

y is always 2 more than


2
x.
3

2

 2.
2

If x  3, then y  3 (3)  2 or 0.

There is no difference in y values, no matter what


the difference of x values. The value of y remains
constant at 3. This suggests y  3.
Check:
If x  1, then y  3.
If x  3, then y  3.
Thus, y  3 describes the relation. Since this
relation is also a function, we can write the
equation in function notation as f(x)  3.
Extra Practice

2

Check y

The next three terms are 7, 4, and 5.


12. Make a table of ordered pairs for several points
on the graph.

3
3

2

 2x



1
3
1

x
y

2


2

The difference between successive pairs of terms


alternates between 3 and 1. Continue
alternating the difference. Add 1, 3, and 1.
6
7
4
5

If x  2, then y  2 (2) or 1.


But the y value for x  2 is 0. This is
a difference of 1. Try other values in
the domain to see if the same
difference occurs.

Check:


3
1
3
1
3

2
2

2

The difference of x values is 1, and the difference


1
of y values is 2. The difference of the y values is
1
the opposite of the difference of the x values.
2
1
This suggests y  2x.

The next three terms are w  10, w  12, and


w  14.
11. 13
10
11
8
9
6

x
y


2


2

1

2

2

The difference between each pair of terms is


always 2. The sequence is arithmetic with a
common difference of 2. Each term is 2 less than
the term before it. Add 2, 2, and 2.
w8
w  10
w  12
w  14


2

1

The next three terms are 36, 43, and 51.


10. w  2
w4
w6
w8


2



6
7
8

2

1

If x  3, then y  3 (3)  2 or 4.
2

Thus, y  3x  2 describes this relation. Since


this relation is also a function, we can write the
2
equation in function notation as f(x)  3x  2.

678

PQ249J-6481F-15-19[643-685] 26/9/02 9:51 PM Page 679 Sahuja Ahuja_QXP_06:Desktop Folder:Chandra:Algebra_FNL_Delivery:

Page 831

11. Let (1, r)  (x1, y1 ) and (1, 4)  (x2, y2 ).

Lesson 5-1

y2  y1

1. Let (4, 4)  (x1, y1 ) and (2, 0)  (x2, y2 ).

mx

y2  y1

mx

5 

 x1

0  (4)
 (4)
4
or 2
2

m  2
m

5 

5(2)  4  r
10  4  r
10  4  4  r  4
6  r
(1)(6)  (1) (r)
6r
12. Let (r, 2)  (x1, y1 ) and (7, 1)  (x2, y2 ).

2. Let (4, 2)  (x , y ) and (0, 1)  (x , y ).


1 1
2 2
y2  y1

mx

 x1

1  2
(4)
3
3
or 4
4

m0
m

y2  y1

3. Let (2, 2)  (x1, y1 ) and (3, 3)  (x2, y2 ).

mx

y2  y1

mx

1
4
1
4

 x1

3  2
(2)
5
or 1
5

m3
m

y2  y1
2

m
m

 x1

4  (8)
1  (2)
12
or 4
3
y2  y1
2

 x1

m  1 or 2
6. Let (5, 4)  (x1, y1 ) and (1, 11)  (x2, y2 ).

26
3
26
3

y2  y1
2

m
m

 x1

11  4
1  (5)
7
4

7. Let (18, 4)  (x1, y1 ) and (6, 10)  (x2, y2 ).

Page 831

m
m

 x1

m

r

y2  y1
2

m

 x1

2  0
3  0

or 3
3

2. The constant of variation is 2.


y2  y1

y2  y1

m

3r
3

Lesson 5-2

mx

8. Let (4, 6)  (x1, y1 ) and (4, 8)  (x2, y2 ).


2

1. The constant of variation is 3.

10  (4)
6  18
6
1
or 2
12

mx

y2  y1

mx

r  2
(3)
r  2
10

7

2(10)  3(r  2)
20  3r  6
20  6  3r  6  6
26  3r

6  4

m43

mx

2
3
2
3

5. Let (3, 4)  (x , y ) and (4, 6)  (x , y ).


1 1
2 2
mx

 x1

1  (2)
7  r
1
7  r

1(7  r)  4(1)
7r4
7r747
r  3
13. Let (3, 2)  (x1, y1 ) and (7, r)  (x2, y2 ).
y2  y 1
mx x

4. Let (2, 8)  (x , y ) and (1, 4)  (x , y ).


1 1
2 2
mx

 x1

4  r
1  (1)
4  r
2

mx

 x1

8  (6)
4  (4)
2
0

m

 x1

3  0
2  0

or 2
1

3. The constant of variation is 5.


y2  y1

mx

Since division by zero is undefined, the slope is


undefined.

m

9. Let (0, 0)  (x , y ) and (1, 3)  (x , y ).


1 1
2 2

 x1

1  0
5  0

or 5

y2  y1

mx

 x1

3  0
 0

m  1

or 3

10. Let (8, 1)  (x1, y1 ) and (2, 1)  (x2, y2 ).


y2  y1

mx

m

 x1

1  1
2  (8)

or 0

679

Extra Practice

PQ249J-6481F-15-19[643-685] 26/9/02 9:52 PM Page 680 Sahuja Ahuja_QXP_06:Desktop Folder:Chandra:Algebra_FNL_Delivery:

8. Find the value of k.


y  kx
7  k(1)
(1)(7)  (1)(k)
7k
Therefore, y  7x.
Find x when y  84.
y  7x
84  7x

4. Write the slope as a ratio.


5

5
1

Graph (0, 0). From the point (0, 0) move up


5 units and right 1 unit. Draw a dot. Draw a line
containing the points.
y

y  5x

84
7

450
6

5. Write the slope as a ratio.


6

k(6)
6

75  k
Therefore, y  75x.
Find y when x  10.
y  75x
y  75(10)
y  750
10. Find the value of k.
y  kx
6  k(48)

6  1
Graph (0, 0). From the point (0, 0) move up
6 units and left 1 unit. Draw a dot. Draw a line
containing the points.
y

6
48
1
8

y  6x
x

7x
7

12  x
9. Find the value of k.
y  kx
450  k(6)

k(48)
48

k
1

Therefore, y  8x.
Find y when x  20.

6. Write the slope as a ratio.

y  8x

4

3  3
Graph (0, 0). From the point (0, 0) move down
4 units and right 3 units. Draw a dot. Draw a line
containing the points.

y  8 (20)
y  2.5

Page 831
4
3

y x
O

7. Find the value of k.


y  kx
45  k(9)
45
9

k(9)
9

4. Replace m with 3 and b with 3.

5k
Therefore, y  5x.
Find y when x  7.
y  5x
y  5(7)
y  35

Extra Practice

Lesson 5-3

1. Replace m with 5 and b with 15.


y  mx  b
y  5x  (15)
y  5x  15
2. Replace m with 6 and b with 3.
y  mx  b
y  6x  3
3. Replace m with 0.3 and b with 2.6.
y  mx  b
y  0.3x  (2.6)
y  0.3x  2.6
y  mx  b
4

y  3x  3
2

5. Replace m with 5 and b with 2.


y  mx  b
2

y  5 x  2

680

PQ249J-6481F-15-19[643-685] 26/9/02 9:52 PM Page 681 Sahuja Ahuja_QXP_06:Desktop Folder:Chandra:Algebra_FNL_Delivery:

6. Replace m with

7
4

11. The y-intercept is 3. So, graph (0, 3). The slope is


2
2 or 1 . From (0, 3), move down 2 units and
right 1 unit. Draw a dot. Draw a line connecting
the points.

and b with 2.

y  mx  b
7

y  4x  (2)
7

y  4x  2

7. Find the slope.


y2  y1

mx

m
m

 x1

1  3
2  0
2
or
2

y  2x  3

1

The line crosses the y-axis at (0, 3). So, the


y-intercept is 3.
y  mx  b
y  1x  3
y  x  3
8. Find the slope.

12. Solve for y to find the slope-intercept form.


3x  y  6
3x  y  3x  6  3x
y  3x  6
(1)(y)  (1) (3x  6)
y  3x  6
The y-intercept is 6. So, graph (0, 6). The slope
3
is 3 or 1. From (0, 6), move up 3 units and right
1 unit. Draw a dot. Draw a line connecting the
points.

y2  y1

mx

m
m

 x1

2  (3)
2  0
1
1
or 2
2

The line crosses the y-axis at (0, 3). So, the


y-intercept is 3.
y  mx  b

y  2x  (3)
y

1
2x

3

9. Find the slope.


y2  y1

mx

3x y  6

 x1

2  1
(3)

m0
1

m3
The line crosses the y-axis at (0, 2). So, the
y-intercept is 2.
y  mx  b

Page 832

y  3x  2
10. The y-intercept is 1. So, graph (0, 1). The slope
5
is 5 or 1. From (0, 1), move up 5 units and right
1 unit. Draw a dot. Draw a line connecting the
points.
y

y  5x  1
O

Lesson 5-4

1. The point (0, 0) lies on the y-axis.


The y-intercept is 0.
y  mx  b
y  2x  0
y  2x
2. Find the y-intercept.
y  mx  b
2  4(3)  b
2  12  b
2  12  12  b  12
14  b
Write the slope-intercept form.
y  mx  b
y  4x  14
3. The point (0, 5) lies on the y-axis.
The y-intercept is 5.
y  mx  b
y  1x  5
y  x  5

681

Extra Practice

PQ249J-6481F-15-19[643-685] 26/9/02 9:52 PM Page 682 Sahuja Ahuja_QXP_06:Desktop Folder:Chandra:Algebra_FNL_Delivery:

4. Find the y-intercept.


y  mx  b

8. Find the slope.


y2  y1

mx

m

32b
1

5
2

b

9. Find the slope.


y2  y1

5
2

mx

5. Find the y-intercept.


y  mx  b

m
m

5  3 (1)  b
2
2

5  3  3  b  3
17

3  b
Write the slope-intercept form.
y  mx  b
2

17

y  3x   3
y

2
x
3

17
3

y2  y1

mx

6. Find the y-intercept.


y  mx  b
1
4
1
4
1
4

m

1 2b
1
2

8

44b4
15

4  b
Write the slope-intercept form.
y  mx  b

15

y  8x 

15
4

m
m

m
m

 x1

7  3 (5)  b

2  7
8  (1)
9
or 1
9

7
7

Find the y-intercept.


y  mx  b
7  1(1)  b
71b
711b1
6b
Write the slope-intercept form.
y  mx  b
y  1x  6
y  x  6

Extra Practice

 x1

3  7
1  5
4
2
or 3
6

Find the y-intercept.


y  mx  b

y2  y1
2

or 1

y2  y1

mx

7. Find the slope.


mx

 x1

1  0
0  1

The point (0, 1) lies on the y-axis.


The y-intercept is 1.
y  mx  b
y  1x  1
y  x  1
11. Find the slope.

4b

y  8x   4

 x1

1  (1)
7  8
0
or 0
1

Find the y-intercept.


y  mx  b
1  0(8)  b
1  b
Write the slope-intercept form.
y  mx  b
y  0x  (1)
y  1
10. Find the slope.

5  3  b
2

or 4

y  4x  5

Write the slope-intercept form.


y  mx  b
y

5  0
0  4

The point (0, 5) lies on the y-axis.


The y-intercept is 5.
y  mx  b

322b2

1
4x

 x1

3  4 (2)  b

10
3
11
3

10
3
10
3

b
b

10
3

b

Write the slope-intercept form.


y  mx  b
2

y  3x 

682

11
3

PQ249J-6481F-15-19[643-685] 26/9/02 9:52 PM Page 683 Sahuja Ahuja_QXP_06:Desktop Folder:Chandra:Algebra_FNL_Delivery:

12. Find the slope.

Write the slope-intercept form.


y  mx  b

y2  y1

mx

 x1

m
m

16. Find the slope of the line containing the points


(2, 0) and (0, 1).
y2  y1

Find the y-intercept.


y  mx  b

mx

y  2x  1
17. Find the slope of the line containing the points
(1, 0) and (0, 4).
y2  y1

y  3x  (10)

mx

y  3x  10

m

13. Find the slope.

m

 x1

3  3
1  (2)
0
or 0
3

Find the y-intercept.


y  mx  b
3  0(2)  b
3b
Write the slope-intercept form.
y  mx  b
y  0x  3
y3
14. Find the slope.

3  0
 0

m  4

Write the slope-intercept form.


y  mx  b
y  1x  5
y  x  5
19. Find the slope of the line containing the points
(1, 0) and (0, 3).
y2  y1

mx

or 4

m

The point (0, 0) lies on the y-axis.


The y-intercept is 0.
y  mx  b
3
3

y  4x

y2  y1

m

m

m

 x1
3
4

1 12

 2

1
4
3
4

or 3

1
1

y  4x  1

1
2

21. Find the slope of the line containing the points


(3, 0) and (0, 3).
y2  y1

1 12

mx

 3 2  b

m

 6  b
1

or 4

y  4x  (1)

Find the y-intercept.


y  mx  b
1
2
1
2
1
6
2
3

 x1

1  0
0  (4)

Write the slope-intercept form.


y  mx  b

2

1
4

or 3

y2  y1

mx

15. Find the slope.


2

 x1

3  0
0  (1)

Write the slope-intercept form.


y  mx  b
y  3x  3
20. Find the slope of the line containing the points
(4, 0) and (0, 1).

y  4x  0

mx

 x1

5  0

m  0  5 or 1

 x1

or 4

y2  y1

mx

y2  y1

mx

 x1

4  0
0  1

Write the slope-intercept form.


y  mx  b
y  4x  (4)
y  4x  4
18. Find the slope of the line containing the points
(5, 0) and (0, 5).

y2  y1

m

Write the slope-intercept form.


y  mx  b

5  5  b
5  5  5  b  5
10  b
Write the slope-intercept form.
y  mx  b

 x1

1  0

m  0  2 or 2

5  3 (3)  b

mx

y  3x  3

15  (5)
3  (3)
10
5
or 3
6

 x1

3  0
0  3

or 1

Write the slope-intercept form.


y  mx  b
y  1x  (3)
yx3

 6  b  6
b

683

Extra Practice

PQ249J-6481F-15-19[643-685] 26/9/02 9:52 PM Page 684 Sahuja Ahuja_QXP_06:Desktop Folder:Chandra:Algebra_FNL_Delivery:

Page 832
1.

2.
3.

4.

5.

11.

Lesson 5-5

y  y1  m(x  x1 )
y  (2)  3(x  5)
y  2  3(x  5)
y  y  m(x  x )
1
1
y  4  5(x  5)
y  y1  m(x  x1 )
y  6  2(x  0)
y  6  2x
y  y1  m(x  x1 )
y  1  0 [x  (3) ]
y10
y  y  m(x  x )
1
1

12.

y  0  3 [x  (1) ]
2

y  3 (x  1)
6.

y  y  m(x  x )
1
1
3
y  (4)  4 [ x  (2) ]

13.

y  4  4 (x  2)
7.

8.

y  3  2(x  4)
y  3  2x  8
y  3  3  2x  8  3
y  2x  11
y  2x  2x  11  2x
2x  y  11
(1)(2x  y)  (1) (11)
2x  y  11

14.

15.

y  3  2 (x  6)

1 12

y  1  1.5(x  3)
2(y  1)  2(1.5) (x  3)
2y  2  3(x  3)
2y  2  3x  9
2y  2  2  3x  9  2
2y  3x  11
2y  3x  3x  11  3x
3x  2y  11
(1)(3x  2y)  (1)11
3x  2y  11
y  6  3.8(x  2)
5(y  6)  5(3.8)(x  2)
5y  30  19(x  2)
5y  30  19x  38
5y  30  30  19x  38  30
5y  19x  8
5y  19x  19x  8  19x
19x  5y  8
y  1  2(x  5)
y  1  2x  10
y  1  1  2x  10  1
y  2x  9
y  3  4(x  1)
y  3  4x  4
y  3  3  4x  4  3
y  4x  7
y  6  4(x  2)
y  6  4x  8
y  6  6  4x  8  6
y  4x  14
4

y  1  5 (x  5)

16.

2(y  3)  2 2 (x  6)

y  1  5x  4

2y  6  1(x  6)
2y  6  x  6
2y  6  6  x  6  6
2y  x  12
2y  x  x  12  x
x  2y  12
9.

y4
3(y  4) 

2
3 (x
2
3 3

y  1  1  5x  4  1
4

y  5x  3

 5)

y
y

1
4
1
4

2
x
3
2
x
3
2
x
3

1
4




y

143 2 (x  6)

y43 x2

18.

Page 832

3y  6  4(x  6)
3y  6  4x  24
3y  6  6  4x  24  6
3y  4x  30
3y  4x  4x  30  4x
4x  3y  30
(1)(4x  3y)  (1)(30)
4x  3y  30

Extra Practice

y  4x  2

y  2  3 (x  6)
3(y  2)  3

y  2  2  4x  2  2

3y  12  2(x  5)
3y  12  2x  10
3y  12  12  2x  10  12
3y  2x  22
3y  2x  2x  22  2x
2x  3y  22
10.

y  2  4x  2

1 2 (x  5)

y  2  4 (x  2)

17.





1
3
1
1
4
3
1
12

Lesson 5-6

1. The line parallel to y  4x  2 has the same


slope, 4.
y  y1  m(x  x1 )
y  6  4(x  1)
y  6  4x  4
y  6  6  4x  4  6
y  4x  2

684

PQ249J-6481F-15-19[643-685] 26/9/02 9:52 PM Page 685 Sahuja Ahuja_QXP_06:Desktop Folder:Chandra:Algebra_FNL_Delivery:

2. The line parallel to y  2x  7 has the same


slope, 2.
y  y1  m(x  x1 )
y  6  2(x  4)
y  6  2x  8
y  6  6  2x  8  6
y  2x  2

8. Find the slope of the given line.


6x  y  4
6x  y  6x  4  6x
y  6x  4
The slope of the line perpendicular to this line is
1
the opposite reciprocal of 6 or 6.
y  y1  m(x  x1 )

3. The line parallel to y  3x  1 has the same


2
slope, 3.
y  y1  m(x  x1 )

y  3  6 [ x  (2) ]
1

y  3  6 (x  2)
1

10
3

y  3  3  6x  3  3
y  6x 

y  3x  2

9. The slope of the given line is 4. So, the slope of


the line perpendicular to this line is the opposite
3
4
reciprocal of 4 or 3.

4. The line parallel to y  3x  7 has the same


slope, 3.
y  y1  m(x  x1 )
y  (2)  3(x  5)
y  2  3x  15
y  2  2  3x  15  2
y  3x  13
5. Solve the given equation for y.
3x  8y  4
3x  8y  3x  4  3x
8y  3x  4
8y
8

y

y  y1  m(x  x1 )
4
y  0  3 (x  0)
4

y  3x
10. Find the slope of the given line.
4x  3y  2
4x  3y  4x  2  4x
3y  4x  2

3x  4
8
3
1
8x  2

3y
3
1

y  0  4 (x  4)

4

y

y  4 (x  4)
3

y  4x  3
11. Find the slope of the given line.
3x  5y  1
3x  5y  3x  1  3x
5y  3x  1

x  7
5
1
7
x
5
5
1

5y
5

The line parallel to y  5x  5 has the same


1
slope, 5.
y  y1  m(x  x1 )

y

3x  1
5
3
1
x
5
5

The slope of the line perpendicular to this line is


3
5
the opposite reciprocal of 5 or 3.

y  3  5 (x  2)
1

y  y  m(x  x )
1
1

y  7  3 (x  6)

1
x
5

13
5

y  3  5x  5

y  3  3  5x  5  3
y

4x  2
3
4
2
x3
3

The slope of the line perpendicular to this line is


4
3
the opposite reciprocal of 3 or 4.
y  y  m(x  x )
1
1

6. Solve the given equation for y.


x  5y  7
x  5y  x  7  x
5y  x  7
5y
5

y
3

The line parallel to y  8x  2 has the same


3
slope, 8. The point (0, 4) lies on the y-axis. The
y-intercept is 4.
y  mx  b
y

y  3  6x  3

y  3 (x  3)

3
8x

y  0  3 [x  (3) ]

y  7  3  10
5

y  7  7  3x  10  7

3
5.

7. The slope of the given line is


So, the slope of
the line perpendicular to this line is the opposite
3
5
reciprocal of 5 or 3. The point (0, 1) lies on the
y-axis, so the y-intercept is 1.
y  mx  b

y  3x  17

y  3x  (1)
5

y  3x  1

685

Extra Practice

PQ249J-6481F-20-23[686-725] 26/9/02 9:41 PM Page 686 Sahuja Ahuja_QXP_06:Desktop Folder:Chandra:Algebra_FNL_Delivery:

2. d  (3) 6
d3 6
d33 6
d 6
The solution

12. Find the slope of the given line.


8x  4y  15
8x  4y  8x  15  8x
4y  8x  15
4y
4

y

8x  15
4
15
2x  4

The slope of the line perpendicular to this line is


1
the opposite reciprocal of 2 or 2.

3.

y  y1  m(x  x1 )
1
y  (1)  2 (x  5)
1

1
x
2
1
x
2

5
2
7
2

y  1  2x  2
y11
y

Page 833




Lesson 5-7

1. The graph shows a negative correlation. As the


age of the car increases, the value decreases.
2. The graph shows a positive correlation. As the
decade increases, the number of students increases.
3. The graph shows no correlation.
4. The graph shows a positive correlation. As the
year increases, the number of metric tons of fish
caught in China increases.
5. Find the slope.

6.

38  24

m  1998  1994
m

14
4

or 2
1

7.

y  24  2 (x  1994)
7

y  24  2x  6979
7

y  24  24  2x  6979  24
7

y  2x  6955
7

6. Use the equation y  2x  6955.


y
y

7
x  6955
2
7
(2005) 
2

6955

y  62.5
There should be about 62.5 millions of metric tons
of fish caught in China in 2005.

Lesson 6-1

Exercises 116 For checks, see students work.


1.
c93
c9939
c  6
The solution set is {c|c  6}.
6

Extra Practice

4

2

10

16

20

24

4

2

4

2

2x 7 x  3
2x  x 7 x  3  x
x 7 3
The solution set is {x|x 7 3}.
2

2x  3  x
2x  3  x  x  x
x30
x3303
x3
The solution set is {x|x  3}.
0

16  w 6 20
16  w  16 6 20  16
w 6 36
The solution set is {w|w 6 36}.
40 32 24 16 8

9.

Page 833

12

6

8

2

8.

2
2
2  7
9
set is {h|h 7 9}.
6

8

4

Use the point-slope form.


y  y  m(x  x )

11 7 d  4
11  4 7 d  4  4
7 7 d
7 7 d is the same as d 6 7.
The solution set is {d|d 6 7}.
10

 x1

10

5.

y2  y1

mx

z  4 7 20
z  4  4 7 20  4
z 7 24
The solution set is {z|z 7 24}.

4. h  (7) 7
h7 7
h77 7
h 7
The solution

1

13
13
13  3
10
set is {d|d 6 10}.

2

686

14p 7 5  13p
14p  13p 7 5  13p  13p
p 7 5
The solution set is {p|p 7 5}.
0

PQ249J-6481F-20-23[686-725] 26/9/02 9:41 PM Page 687 Sahuja Ahuja_QXP_06:Desktop Folder:Chandra:Algebra_FNL_Delivery:

10.

7 6 16  z
7  z 6 16  z  z
7  z 6 16
7  z  7 6 16  7
z 6 23
The solution set is {z|z 6 23}.
0

11.

12

16

20

12.
1
t
2

11
12

1
1
t4
2
1
1
 2t
4
1
4
1
2
3
4
11
12




3
t
2
3
t
2

4
2
3
2
3




5j
5

t33

11

3

2

1

11
12

6.

3.

1

4.

4

2

p
5
p
(5) 5

8f
8

7 12
7 (3) (12)

6 8
6 (5)8

48
8

f 7 6
The solution set is { f|f 7 6}.
6. 0.25t  10
0.25t
0.25

10

 0.25

t  40
The solution set is {t|t  40}.
7.

g
8
g
(8) 8

6 4
7 (8)4

g 7 32
The solution set is { g|g 7 32}.
8. 4.3x 6 2.58

5z  6 7 4z
5z  6  4z 7 4z  4z
z6 7 0
z66 7 06
z 7 6
The solution set is {z|z 7 6}.
2

w
3
w
(3) 3

p 6 40
The solution set is { p|p 6 40}.
5. 8f 6 48

a  2.3  7.8
a  2.3  2.3  7.8  2.3
a  5.5
The solution set is {a|a  5.5}.
6

60
5

w 7 36
The solution set is {w|w 7 36}.

2  9n  10n
2  9n  9n  10n  9n
2  n
2  n is the same as n  2.
The solution set is {n|n  2}.
2

49
7

j 7 12
The solution set is { j|j 7 12}.

t

1

b  7
The solution set is {b|b  7}.
2. 5j 6 60

9x 6 8x  2
9x  8x 6 8x  2  8x
x 6 2
The solution set is {x|x 6 2}.

3

16.

7b
7

4

15.

t3
2

Lesson 6-2

Exercises 116 For checks, see students work.


1. 7b  49

 t is the same as t  12.

2

Page 833

 2t

The solution set is t|t 

14.

24

1.1v  1 7 2.1v  3
1.1v  1  1.1v 7 2.1v  3  1.1v
1 7 v  3
1  3 7 v  3  3
2 7 v
2 7 v is the same as v 6 2.
The solution set is {v|v 6 2}.
2

13.

17. Let n  the number.


n  (6) 7 9
n6 7 9
n66 7 96
n 7 15
The solution set is {n|n 7 15}.
18. Let n  the number.
5n 6 6n  12
5n  6n 6 6n  12  6n
n 6 12
The solution set is {n|n 6 12}.

4.3x
4.3

2.58
4.3

x 7 0.6
The solution set is {x|x 7 0.6}.
9. 4c  6
4c
4

6
4

c  1.5
The solution set is {c|c  1.5}.

687

Extra Practice

PQ249J-6481F-20-23[686-725] 26/9/02 9:41 PM Page 688 Sahuja Ahuja_QXP_06:Desktop Folder:Chandra:Algebra_FNL_Delivery:

10.

6  0.8n
6
0.8

Page 834

0.8n
0.8

7.5  n
7.5  n is the same as n  7.5.
The solution set is {n|n  7.5}.
11.

2
m
3
3 2
m
2 3

12

3y
3

 22

132 2 (22)

7s
7

0.05a
0.05

28
15
28
or
15

a 7

13

3.

9x 6 42

197 2 179x2 7 197 242

y

32
0.375
256
or
3

6v
6

The solution set is y|y  853 .


16. 7y  91
7y
7

 10


153 2 (10)
50

n   3 or 163

100

400
3

or 1333

The solution set is n|n  1333 .

Extra Practice

30
6

18
2

12
4

t 7 3
The solution set is {t|t 7 3}.

 0.75

n

4t
4

The solution set is n|n  163 .


19. Let n  the number.
0.75n  100
0.75n
0.75

15
5

k 7 9
The solution set is {k|k 7 9}.
6.
2x  1 6 16  x
2x  1  2x 6 16  x  2x
1 6 16  x
1  16 6 16  x  16
15 6 x
15 6 x is the same as x 7 15.
The solution set is {x|x 7 15}.
7.
15t  4 7 11t  16
15t  4  11t 7 11t  16  11t
4t  4 7 16
4t  4  4 7 16  4
4t 7 12

91

y  13
The solution set is {y|y  13}.
17. Let n  the number.
1n 7 7
(1)(n) 6 (1)(7)
n 6 7
The solution set is {n|n 6 7}.
18. Let n  the number.

12

2k
2

 7

3
n
5
5 3
n
3 5

v5
The solution set is {v|v  5}.
5.
2k  12 6 30
2k  12  12 6 30  12
2k 6 18

853

e 6 3
The solution set is {e|e 6 3}.
4.
6v  3  33
6v  3  3  33  3
6v  30

x 7 54
The solution set is {x|x 7 54}.
15. 0.375y  32
0.375y
0.375

5e  9 7 24
5e  9  9 7 24  9
5e 7 15
5e
5

The solution set is a|a 7 115 .


14.

or 37

The solution set is s|s 6 37 .

13

115

25
7
25
7

s 6

500 6 a
500 6 a is the same as a 7 500.
The solution set is {a|a 7 500}.
13. 15a 6 28
15a
15

33
3

y 7 11
The solution set is {y|y 7 11}.
2.
7s  12 6 13
7s  12  12 6 13  12
7s 6 25

m  33
The solution set is {m|m  33}.
12. 25 7 0.05a
25
0.05

Lesson 6-3

Exercises 120 For checks, see students work.


1.
3y  4 7 37
3y  4  4 7 37  4
3y 7 33

688

PQ249J-6481F-20-23[686-725] 26/9/02 9:41 PM Page 689 Sahuja Ahuja_QXP_06:Desktop Folder:Chandra:Algebra_FNL_Delivery:

8.

13  y  29  2y
13  y  y  29  2y  y
13  29  3y
13  29  29  3y  29
16  3y
16
3
16
3
16
3

The
9.

14.

2q
2

15
5

18
6

4
2

5w
5

6t
6

6 (3)(9)

11
4
3
24
3
24

32
2

12
6

8k
8

4y
4

6 y
3

6 y is the same as y 7 24 or 2.75.

The solution set is y|y 7 24 .

t 6 16
The solution set is {t|t 6 16}.

19.

 7  5

 7  7  5  7
z
4
z
(4) 4

1  k
1  k is the same as k  1.
The solution set is {k|k  1}.
17.
9m  7 6 2(4m  1)
9m  7 6 8m  2
9m  7  8m 6 8m  2  8m
m  7 6 2
m  7  7 6 2  7
m 6 9
The solution set is {m|m 6 9}.
18. 3(3y  1) 6 13y  8
9y  3 6 13y  8
9y  3  9y 6 13y  8  9y
3 6 4y  8
3  8 6 4y  8  8
11 6 4y

6 9

r 7 8
set is {r|r 7 8}.
7 c  17
7 c  17
7 c  17
7 c  17  c
7 17
7 17  5
7 12

8
8

2t  5 6 27
2t  5  5 6 27  5
2t 6 32

z
4

48
6

c 7 2
The solution set is {c|c 7 2}.
16.
5(k  4)  3(k  4)
5k  20  3k  12
5k  20  5k  3k  12  5k
20  8k  12
20  12  8k  12  12
8  8k

3 7 t
3 7 t is the same as t 6 3.
The solution set is {t|t 6 3}.

z
4

6c
6

3w
3  w is the same as w  3.
The solution set is {w|w  3}.
11.
4t  5 7 2t  13
4t  5  4t 7 2t  13  4t
5 7 6t  13
5  13 7 6t  13  13
18 7 6t

13.

6r
6

The solution
15. 8c  (c  5)
8c  c  5
7c  5
7c  5  c
6c  5
6c  5  5
6c

1
53.

q  2
The solution set is {q|q  2}.
10.
2(w  4)  7(w  1)
2w  8  7w  7
2w  8  2w  7w  7  2w
8  5w  7
8  7  5w  7  7
15  5w

2t
2

7r  37
7r  37  7r
37
37  11
48

y

16
 y is the same as y   3 or
1
solution set is y|y  53 .

2t  5
3
2t  5
(3) 3

7
7
7
7
7

3y
3

5q  7  3(q  1)
5q  7  3q  3
5q  7  3q  3q  3  3q
2q  7  3
2q  7  7  3  7
2q  4

12.

13r  11
13r  11  7r
6r  11
6r  11  11
6r

5x  10(3x  4)
5x  30x  40
5x  30x  30x  40  30x
25x  40
25x
25

 12

40

 25
8

x  5 or 15 or 1.6

 (4)(12)

The solution set is x|x  15 .

z  48
The solution set is {z|z  48}.

689

Extra Practice

PQ249J-6481F-20-23[686-725] 26/9/02 9:42 PM Page 690 Sahuja Ahuja_QXP_06:Desktop Folder:Chandra:Algebra_FNL_Delivery:

4.

3a3 a1

20.

3a  2  a  1
3a  2  a  a  1  a
2a  2  1
2a  2  2  1  2
2a  3
2a
2

a

3
2
3
2

4v  1  3v
4v  1  4v  3v  4v
1  v
(1)(1)  (1) (v)
1v

and

The solution set is the intersection of the two


graphs.
Graph 2  v or v  2.

or 12

2v  2  3v
2v  2  2v  3v  2v
2  v

Graph 1  v or v  1.

The solution set is a|a  12 .

Find the intersection.

Page 834
1.

or
2x 7 5
2  x 6 5
2x2 7 52
2  x  2 6 5  2
x 6 7
x 7 3
The solution set is the union of the two graphs.
8

2.

Lesson 6-4

16  4b
16
4

10

10

10

12

12

Graph
t  11.

12

Find the
union.

2g
2

2g
2

9 6 2z  7
9  7 6 2z  7  7
16 6 2z
16
2

4  b or b  4.

Find the
intersection.

2z  7 6 10
2z  7  7 6 10  7
2z 6 3

and

2z
2

2z
2

8 6 z

3
2

z 6 1.5

The solution set is the intersection of the two


graphs.

2

Graph
8 6 z or z 7 8.
Graph z 6 1.5.

2  g or g  2.
Graph g  4.

Find the
intersection.

The solution set is {z|8 6 z 6 1.5}.

Find the
intersection.

The solution set is {g|2  g  4}.

Extra Practice

The solution set is {b|4  b  4}.


6. First express 9 6 2z  7 6 10 using and.

Graph

Graph b  4.

2  g
g4
The solution set is the intersection of the two
graphs.
4

b4

4b
4

Graph

The solution set is {t|t is a real number.}.


and
2g  7  15
3  2g  7
3.
2g  7  7  15  7
3  7  2g  7  7
4  2g
2g  8
4
2

32
8

The solution set is the intersection of the two


graphs.

The solution set is {x|x 6 7 or x 7 3}.


or
4  t 6 7
4  t 7 5
4  t  4 6 7  4
4  t  4 7 5  4
t 7 1
t 6 11
The solution set is the union of the two graphs.
Graph
t  1.

4  b

Find
the
union.

8b
8

4  12  4b  12  12

Graph
x  3.

3b  4  7b  12
and
8b  7  25
3b  4  3b  7b  12  3b
8b  7  7  25  7
4  4b  12
8b  32

Graph
x  7.

The solution set is {v|v  1}.


5.

690

PQ249J-6481F-20-23[686-725] 26/9/02 9:42 PM Page 691 Sahuja Ahuja_QXP_06:Desktop Folder:Chandra:Algebra_FNL_Delivery:

5m  8  10  m

7.

5m  11 6 9

or

6m  8  10

5m
5

6m  18


20
5

4p  6 6 8

4p  8  6
4p  8  8  6  8

4p
4

m3

Graph m  3.
2

2
4

p 6

1
2

4p
4

2 1 0 1

Graph p 6 2.

2 1 0 1

2 1 0 1

Graph p  2.

Find the union.


2

Find the intersection.

The solution set is {m|m 6 4 or m  3}.


8.

25
5

14
7

c2

11.

5c
5

7
7
7
7
7

16  2r
and
16  2r  2r
16
16  8
8

4r
4

8
4

5  c

The solution set is {c|c is a real number.}.


9. First express 2h  2  3h  4h  1 using and.
and
2h  2  3h
2h  2  2h  3h  2h
2  h

7r  21
7r  21  r
6r  21
6r  21  21
6r

6
6
6
6
6

r9
r9r
9
9  21
30

6r
6

30
6

r 6 5

Graph r 7 2.

Graph
5  c or c  5.
Find the union.

6.

r 7 2

1
2

The solution set is the intersection of the two


graphs.

Graph c  2.

2r  8
2r  8  2r
4r  8
4r  8  8
4r

The solution set is the union of the two graphs.


6

The solution set is p|2  p 6

12c  4  5c  10
4c  1  c  24
or
12c  4  5c  5c  10  5c
4c  1  4c  c  24  4c
7c  4  10
1  5c  24
7c  4  4  10  4
1  24  5c  24  24
7c  14
25  5c
7c
7

2
4

p  2

Graph m 6 4.

The solution set is the intersection of the two


graphs.

The solution set is the union of the two graphs.


0

4p  2

4p 6 2

18
6

5p  8  p  6
5p  8  p  p  6  p

4p  6  6 6 8  6

m 6 4

and

3p  6 6 8  p
3p  6  p 6 8  p  p

5m 6 20

6m  8  8  10  8

6m
6

10.

5m  11  11 6 9  11

5m  8  m  10  m  m

Graph r 6 5.
Find the
intersection.

Since the graphs do not intersect, the solution set


is the empty set .

3h  4h  1
3h  4h  4h  1  4h
h  1
(1) (h)  (1) (1)
h1

12.

The solution set is the intersection of the two


graphs.

4j  3
4j  3  4j
3
3  22
19

6
6
6
6
6

j  22
and
j  22  4j
5j  22
5j
5j

19
5

5j
5

Graph 2  h or h  2.

j3
j3j
3
3  15
12

6
6
6
6
6

2j  15
2j  15  j
j  15
j  15  15
j

3.8 6 j

2 1 0 1 2 3

The solution set is the intersection of the two


graphs.

Graph h  1.
2 1 0 1 2 3

Graph

Find the intersection.

12

12

12

2 1 0 1 2 3

The solution set is {h|h  1}.

3.8 6 j or j 7 3.8.
Graph
12 6 j or j 7 12.
Find the intersection.

The solution set is { j|j 7 12}.

691

Extra Practice

PQ249J-6481F-20-23[686-725] 26/9/02 9:42 PM Page 692 Sahuja Ahuja_QXP_06:Desktop Folder:Chandra:Algebra_FNL_Delivery:

13.

2(q  4)  3(q  2)

16. First express (2x  5)  x  5  2x  9 using


and.

q84q

or

q8q4qq

2q  8  3q  6

2q  8  4

2q  8  2q  3q  6  2q

(2x  5)  x  5

2q  8  8  4  8

8  q  6
8  6  q  6  6

2q  12
2q
2

14  q

10  3x
10
3

3x
3

33  x

4 8

The solution set is the intersection of the two


graphs.

Graph q  6.
0

14  x

5  5  3x  5  5

Graph 14  q or q  14.

16 12 8 4

5x9
59x99

5  3x  5

The solution set is the union of the two graphs.


0

x  5  x  2x  9  x

2x  5  2x  x  5  2x

12
2

q6

16 12 8 4

x  5  2x  9

and

2x  5  x  5

4 8

Graph

Find the union.


16 12 8 4

12

16

12

16

14  x or x  14.

12

16

Find the
intersection.

33  x or x  33 .

4 8

Graph

The solution set is {q|q is a real number.}.


1
w
2

14. First express


and.
1
w
2
1
w
2

5w2

5w2
1

The solution set is {x|x  14}.

w  2  2w  2w  9  2w

 5  2w  w  2  2w
1

1
w
2

5  2w  2
1

1
w
2

5  2  2w  2  2
1

29

1
(2) 2 w

1
(2) 2 w

6w

Page 834

2292
1
w
2

3  2w
(2)3 

 9 using

w  2  2w  9

and
1

1
w
2

7
 (2)7

w  14

The solution set is the intersection of the two


graphs.
5

6 7 8

9 10 11 12 13 14 15

Graph
6  w or w  6.

6 7 8

9 10 11 12 13 14 15

Graph w  14.

6 7 8

9 10 11 12 13 14 15

Find the
intersection.

12 8 4

or

n  6  n 7 10
2n  6 7 10

3n
3

12

16

20

24

28

30

The solution set is { g|g 6 14 or g 7 2}.


3. Write |t  5|  3 as t  5  3 and t  5  3.
Case 1:
Case 2:
t53
t  5  3
t5535
t  5  5  3  5
t8
t2

21
3

n 6 7

2n 7 16
2n
2

16 14 12 10 8 6 4 2

3n  1 7 20
3n  1  1 7 20  1
3n 7 21

2n  6  6 7 10  6

The solution set is { y|10 6 y 6 28}.


2. Write |g  6| 7 8 as g  6 7 8 or g  6 6 8.
Case 1:
Case 2:
g6 7 8
g  6 6 8
g66 7 86
g  6  6 6 8  6
g 7 2
g 6 14

Since the graphs do not intersect, the solution set


is the empty set .
15. n  (6  n) 7 10

Lesson 6-5

1. Write |y  9| 6 19 as y  9 6 19 and
y  9 7 19.
Case 1:
Case 2:
y  9 6 19
y  9 7 19
y  9  9 6 19  9
y  9  9 7 19  9
y 6 28
y 7 10

16
2

n 7 8

The solution set is the union of the two graphs.

10

The solution set is {t|2  t  8}.


4. Write |a  5|  0 as a  5  0 or a  5  0.
Case 1:
Case 2:
a50
a50
a5505
a5505
a  5
a  5

Graph n 7 8.
8

Graph n 6 7.
Find the union.

3

The solution set is {n|n 6 7 or n 7 8}.

2

1

The solution set is {a|a is a real number.}.


Extra Practice

692

PQ249J-6481F-20-23[686-725] 26/9/02 9:42 PM Page 693 Sahuja Ahuja_QXP_06:Desktop Folder:Chandra:Algebra_FNL_Delivery:

5. Write |14  2z|  16 as 14  2z  16 or


14  2z  16.
Case 1:
Case 2:
14  2z  16
14  2z  14  16  14

14  2z  16
14  2z  14  16  14
2z  30

2z  2
2z
2

2z
2

 2

z  1
0

10. Write |3p  5|  23 as 3p  5  23 and


3p  5  23.
Case 1:
Case 2:
3p  5  23
3p  5  23
3p  5  5  23  5
3p  5  5  23  5
3p  18
3p  28

30
2

3p
3

12

16

20

10

The solution set is {1, 15}.


6. |a  5|  3 means that the distance between a
and 5 is 3 units. Since distance cannot be
negative, there is no real number that makes this
statement true. The solution set is the empty set .
3

2

1

2m
2

8 6 4 2

8 4

20 24

5

12

y1
1

y

The solution set is 1,

1
45

6.

21
5
21
or
5

p6 .

6b  12  36
6b  12  12  36  12
6b  24

48
6

6b
6

24
6

b  4

3x
3

x 7

20
3
2
63

The solution set is

3x
3

30
3

x 6 10
8
2
x|63

10

12

6 x 6 10 .

13. Write |7  8x| 7 39 as 7  8x 7 39 or


7  8x 6 39.
Case 1:
Case 2:
7  8x 6 39
7  8x 7 39
7  8x  7 6 39  7
7  8x  7 7 39  7
8x 7 32
8x 6 46

28 32 36

5y
5

 5

1
p|93

28
3
1
93

The solution set is {b| 4  b  8}.


12. Write |25  3x| 6 5 as 25  3x 6 5 and
25  3x 7 5.
Case 1:
Case 2:
25  3x 6 5
25  3x 7 5
25  3x  25 6 5  25 25  3x  25 7 5  25
3x 6 20
3x 7 30

The solution set is {w|w  6 or w  34}.


9. Write |13  5y|  8 as 13  5y  8
or 13  5y  8.
Case 1:
Case 2:
13  5y  8
13  5y  8
13  5y  13  8  13 13  5y  13  8  13
5y  5
5y  21
5y
5

6 4 2

14  w  20
14  w  14  20  14
w  34
(1) (w)  (1) (34)
w  34

12 16

2

b8

The solution set is {m|m 6 4 or m 7 9}.


8. Write |14  w|  20 as 14  w  20 or
14  w  20.
Case 1:
Case 2:
14  w  20
14  w  14  20  14
w  6
(1) (w)  (1) (6)
w  6

4

6

6b
6

8
2

10

p

6b  12  36
6b  12  12  36  12
6b  48

m 6 4

m 7 9

8

11. Write |6b  12|  36 as 6b  12  36 and


6b  12  36.
Case 1:
Case 2:

18
2

3p
3

The solution set is

7. Write |2m  5| 7 13 as 2m  5 7 13 or
2m  5 6 13.
Case 1:
Case 2:
2m  5 7 13
2m  5 6 13
2m  5  5 7 13  5
2m  5  5 6 13  5
2m 7 18
2m 6 8
2m
2

18
3

p6

z  15

8x
8

32
8

8x
8

46
8

x 6 5.75

x 7 4
6 4 2

The solution set is {x|x 6 5.75 or x 7 4} .

45

693

Extra Practice

PQ249J-6481F-20-23[686-725] 26/9/02 9:42 PM Page 694 Sahuja Ahuja_QXP_06:Desktop Folder:Chandra:Algebra_FNL_Delivery:

14. Write |4c  5|  25 as 4c  5  25 or


4c  5  25.
Case 1:
Case 2:
4c  5  25
4c  5  25
4c  5  5  25  5
4c  5  5  25  5
4c  20
4c  30
4c
4

20
4

4c
4

c5

18. Write

Case 1:

30
4

2b
2

5s
5

42
5

s 6 8.4
8 6 4 2

4

2

17. Write

Case 1:
2n  1
3
2n  1
(3) 3

0  10 as

2n  1
3
2n  1
(3) 3

 10
 (3)10

2n
2

8

4

10

12

2b
2

 2

6 4 2

13
2

b  6.5
2

The solution set is {b|b  3.5 or b  6.5}.

 10
 (3)(10)

Page 835

12

Lesson 6-6

1. Use a table to substitute the x and y values of


each ordered pair into the inequality.

29
2

n  14.5

n  15.5
16 12

 10.

2n  1  30
2n  1  1  30  1
2n  29

31
2

13
2

b  6.5

b  3.5

2n  1
3

Case 2:

2n  1  30
2n  1  1  30  1
2n  31
2n
2

 10 or

2b
2

2n  1
3

The solution set is {x| 2  x  12}.


20. |3  (2b  6)|  10
|3  2b  6|  10
|3  2b|  10
Write |3  2b|  10 as 3  2b  10 or
3  2b  10.
Case 1:
Case 2:
3  2b  10
3  2b  10
3  2b  3  10  3
3  2b  3  10  3
2b  7
2b  13

The solution set is {x|x  13 or x  7}.


2n  1
3

2b
2

10

The solution set is {s|s 6 8.4 or s 7 10}.


16. |4  (1  x)|  10
|4  1  x|  10
|3  x|  10
Write |3  x|  10 as 3  x  10 or
3  x  10.
Case 1:
Case 2:
3  x  10
3  x  10
3  x  3  10  3
3  x  3  10  3
x7
x  13
16 12 8

1

 (2) (3)

7  2b  6
7  2b  7  6  7
2b  13

 2

 3

The solution set is {b|0.5  b  6.5}.


19. |2  (x  3)|  7
|x  5|  7
Write |x  5|  7 as x  5  7 and x  5  7.
Case 1:
Case 2:
x57
x  5  7
x5575
x  5  5  7  5
x  12
x  2

50
5

 (2)3

b  0.5

s 7 10
0

7  2b
2
7  2b
(2) 2

3

7  2b  6
7  2b  7  6  7
2b  1

The solution set is {c|c  7.5 or c  5}.


15. Write |4  5s| 7 46 as 4  5s 7 46 or
4  5s 6 46.
Case 1:
Case 2:
4  5s 7 46
4  5s 6 46
4  5s  4 7 46  4
4  5s  4 6 46  4
5s 7 42
5s 6 50
5s
5

Case 2:

7  2b
2
7  2b
(2) 2

c  7.5

8 6 4 2

0 7 2 2b 0  3 as 7 2 2b  3 and 7 2 2b  3.

16

The solution set is {14.5, 15.5}.

x
0

y
0

3

3

xy0
000
00
1  (3)  0
2  0
220
40
3  (3)  0
00

True or False
true
false
true
true

The ordered pairs {(0, 0), (2, 2), (3, 3)} are part
of the solution set.

Extra Practice

694

PQ249J-6481F-20-23[686-725] 26/9/02 9:42 PM Page 695 Sahuja Ahuja_QXP_06:Desktop Folder:Chandra:Algebra_FNL_Delivery:

2. Use a table to substitute the x and y values of


each ordered pair into the inequality.
x
0

y
0

1

1

2

2x  y  8
2(0)  0  8
08
2(1)  (1)  8
3  8
2(3)  (2)  8
48
2(8)  0  8
16  8

6. Since the boundary is not included in the


solution, draw a dashed line.
Test the point (0, 0).
x 6 4
0 6 4 true
The half plane that contains (0, 0) should be
shaded.

True or False
true
true

true

x4

false

x
0
2
3
2

y
0
0
4
1

y
0
0
4
1

x
7 0
7 2
7 3
7 2

True or False
false
false
true
false

7. Since the boundary is not included in the


solution, draw a dashed line.
Test the point (0, 0).
x  y 6 2
0  0 6 2
0 6 2 false
The half plane that does not contain (0, 0) should
be shaded.

The ordered pair {(3, 4)} is part of the solution


set.
4. Use a table to substitute the x and y values of
each ordered pair into the inequality.
x
0

y
0

4

5

3x  2y  1
3(0)  2(0) 6
0 6
3(3)  2(2) 6
5 6
3(4)  2(5) 6
2 6
3(0)  2(6) 6
12 6

True or False
1
1
1
1
1
1
1
1

true
x

false
x y  2

true
true

8. Since the boundary is included in the solution,


draw a solid line.
Test the point (0, 0).
x  y  4
0  0  4
0  4 true
The half plane that contains (0, 0) should be
shaded.

The ordered pairs {(0, 0), (4, 5), (0, 6)} are part
of the solution set.
5. Since the boundary line is included in the
solution, draw a solid line.
Test the point (0, 0).
y  2
0  2 false
The half plane that does not contain (0, 0) should
be shaded.

y
x y  4

The ordered pairs {(0, 0), (1, 1), (3, 2)} are
part of the solution set.
3. Use a table to substitute the x and y values of
each ordered pair into the inequality.

y  2

695

Extra Practice

PQ249J-6481F-20-23[686-725] 26/9/02 9:42 PM Page 696 Sahuja Ahuja_QXP_06:Desktop Folder:Chandra:Algebra_FNL_Delivery:

12. Since the boundary is not included in the


solution, draw a dashed line.
Test the point (1, 3).
x 6 y
1 6 3 true
The half plane that contains (1, 3) should be
shaded.

9. Since the boundary is not included in the


solution, draw a dashed line.
Test the point (0, 0).
y 7 4x  1
0 7 4(0)  1
0 7 1 true
The half plane that contains (0, 0) should be
shaded.

y
y  4x  1

xy

13. Since the boundary is included in the solution,


draw a solid line.
Test the point (0, 0).
3x  y  4
3(0)  0  4
0  4 true
The half plane that contains (0, 0) should be
shaded.

10. Since the boundary is not included in the


solution, draw a dashed line.
Test the point (0, 0).
3x  y 7 1
3(0)  0 7 1
0 7 1 false
The half plane that does not contain (0, 0) should
be shaded.

y
3x  y  4

3x y  1

14. Since the boundary is not included in the


solution, draw a dashed line.
Test the point (0, 0).
5x  y 6 5
5(0)  0 6 5
0 6 5 true
The half plane that contains (0, 0) should be
shaded.

11. Since the boundary is included in the solution,


draw a solid line.
Test the point (0, 0).
3y  2x  2
3(0)  2(0)  2
0  2 true
The half plane that contains (0, 0) should be
shaded.

y
O
5x  y  5
O

3y  2x  2

Extra Practice

696

PQ249J-6481F-20-23[686-725] 26/9/02 9:43 PM Page 697 Sahuja Ahuja_QXP_06:Desktop Folder:Chandra:Algebra_FNL_Delivery:

18. Since the boundary is included in the solution,


draw a solid line.
Test the point (0, 0).
3x  8y  4
3(0)  8(0)  4
0  4 true
The half plane that contains (0, 0) should be
shaded.

15. Since the boundary is included in the solution,


draw a solid line.
Test the point (0, 0).
2x  6y  12
2(0)  6(0)  12
0  12 false
The half plane that does not contain (0, 0) should
be shaded.
y

2x  6y  12

3x  8y  4

19. Since the boundary is included in the solution,


draw a solid line.
Test the point (0, 0).
5x  2y  6
5(0)  2(0)  6
0  6 false
The half plane that does not contain (0, 0) should
be shaded.

16. Since the boundary is included in the solution,


draw a solid line.
Test the point (0, 0).
x  3y  9
(0)  3(0)  9
0  9 true
The half plane that contains (0, 0) should be
shaded.
y

O
x  3y  9
O

5x  2y  6

17. Since the boundary is not included in the


solution, draw a dashed line.
Test the point (0, 0).
y 7 3x  7
0 7 3(0)  7
0 7 7 false
The half plane that does not contain (0, 0) should
be shaded.

Page 835

Lesson 7-1

1. The graphs appear to intersect at (3, 9). Check in


each equation.
y  3x
4x  2y  30
Check:
?
?
4(3)  2(9)  30
9  3(3)
?
99
12  18  30
30  30

y  3x  7

4 x  2 y  30

12

y  3x

(3, 9)

4
O

x
O

12

There is one solution. It is (3, 9).

697

Extra Practice

PQ249J-6481F-20-23[686-725] 26/9/02 9:43 PM Page 698 Sahuja Ahuja_QXP_06:Desktop Folder:Chandra:Algebra_FNL_Delivery:

2. The graphs appear to intersect at (2, 1). Check


in each equation.
x  2y
xy1
Check:
?
?
2(1)
2

(1)  1
2
22
11
y

x  2 y

5. The graphs of the equations are parallel lines.


Since they do not intersect, there are no solutions
to this system of equations.
y
x y 6

x y 1
x

O
(2, 1)

O
3x  3y  3

6. The graphs appear to intersect at (2, 6). Check


in each equation.
Check:
y  3x
4x  y  2
?
?
4(2)  (6)  2
6  3(2)
?
6  6
862
22

There is one solution. It is (2, 1).


3. The graphs appear to intersect at (2, 6). Check in
each equation.
yx4
3x  2y  18
Check:
?
?
624
3(2)  2(6)  18
?
66
6  12  18
18  18

y
4x  y  2

y
(2, 6)

y  3 x

3 x  2 y  18

y x4

There is one solution. It is (2, 6).


7. The graphs appear to intersect at (4, 0). Check in
each equation.
2x  y  8
xy4
Check:
?
?
404
2(4)  0  8
88
44

(2, 6)

There is one solution. It is (2, 6).


4. The graphs appear to intersect at (4, 2). Check in
each equation.
Check:
xy6
xy2
?
?
422
426
66
22

y
2x  y  8

y
(4, 0)

xy 6

x y 4

(4, 2)

There is one solution. It is (4, 0).

x y 2

There is one solution. It is (4, 2).

Extra Practice

698

PQ249J-6481F-20-23[686-725] 26/9/02 9:43 PM Page 699 Sahuja Ahuja_QXP_06:Desktop Folder:Chandra:Algebra_FNL_Delivery:

8. The graphs appear to intersect at (3, 3). Check


in each equation.
1
x
5

Check:
1
(3)
5

12
5
? 12
 (3)  5
? 12
3
5  3  5
12
12
 5
5

y

11. The graphs appear to intersect at (2, 2). Check in


each equation.

3x  5y  6
?

2

3(3)  5(3)  6

?
1
(2) 
2
?

9  15  6

264
22

(2, 2)

1
2

x y 3
x

(3, 3) 1 x  y  12
5

2  3(2)  4

y
O

y  3x  4

213
33

9  (15)  6
66

3x  5y  6

x  2y  3

Check:

y  3x  4

There is one solution. It is (2, 2).


12. The graphs of the equations coincide. Since every
point is a point of intersection, there are infinitely
many solutions to this system of equations.

There is one solution. It is (3, 3).


9. The graphs appear to intersect at (6, 3). Check
in each equation.
x  2y  0
y  3  x
Check:
?
?
3  3  (6)
6  2(3)  0
?
66
6  6  0
00

y
2
1
x y 2
3
2

4 x  3 y  12

y
x

(6, 3)

x  2y  0
x

13. The graphs appear to intersect at (3, 1). Check


in each equation.

y  3  x

yx4

Check:

1  3  4

? 5
2
? 5
2
5
2

3  2 (1) 
1

1  1

There is one solution. It is (6, 3).


10. The graphs appear to intersect at (1, 5). Check
in each equation.
x  2y  9
xy6
Check:
?
?
1(5)  6
1  2(5)  9
?
?
156
1  (10)  9
9  9
66

x  2y  2
32
5
2

(3, 1)

y  x4

y
1
2

x y

x  2 y  9

5
2

There is one solution. It is (3, 1).


x y  6

(1, 5)

There is one solution. It is (1, 5).

699

Extra Practice

PQ249J-6481F-20-23[686-725] 26/9/02 9:43 PM Page 700 Sahuja Ahuja_QXP_06:Desktop Folder:Chandra:Algebra_FNL_Delivery:

2. Since y  7  x, substitute 7  x for y in the


second equation.
2x  y  8
2x  (7  x)  8
2x  7  x  8
3x  7  8
3x  7  7  8  7
3x  15

14. The graphs of the equations coincide. Since every


point is a point of intersection, there are infinitely
many solutions to this system of equations.
y
2x  y  3

4x  2y  6

3x
3

15. The graphs appear to intersect at (2, 3). Check


in each equation.
Check:
1
2
x  3 y  3
12x  y  21
2
?

1
(2)
2

12(2)  (3)  21


?

24  3  21
21  21

 3 (3)  3
?

1  (2)  3
3  3

y
12 x  y  21

6y
6

y

1
2
x  y  3
2
3

(2, 3)

15
3

x5
Use y  7  x to find the value of y.
y7x
y75
y2
The solution is (5, 2).
3. Since x  5  y, substitute 5  y for x in the
second equation.
3y  3x  1
3y  3(5  y)  1
3y  15  3y  1
3y  16  3y
3y  3y  16  3y  3y
6y  16

16
6
8
3

Use x  5  y to find the value of x.


x5y
8

x53
7

There is one solution. It is (2, 3).

x3
The solution is

Page 835

4. Since y  2  x, substitute y  2 for x in the first


equation.
3x  y  6
3(y  2)  y  6
3y  6  y  6
4y  6  6
4y  6  6  6  6
4y  0

Lesson 7-2

1. Since y  x, substitute x for y in the second


equation.
5x  12y
5x  12(x)
5x  12x
5x  5x  12x  5x
0  7x
0
7

4y
4

7x
7

4

y0
Use y  2  x to find the value of x.
y2x
02x
2x
The solution is (2, 0).

0x
Use y  x to find the value of y.
yx
y0
The solution is (0, 0).

Extra Practice

173, 83 2.

700

PQ249J-6481F-20-23[686-725] 26/9/02 9:43 PM Page 701 Sahuja Ahuja_QXP_06:Desktop Folder:Chandra:Algebra_FNL_Delivery:

7. Solve the first equation for x since the coefficient


of x is 1.
x  2y  10
x  2y  2y  10  2y
x  10  2y
Since x  10  2y, substitute 10  2y for x in the
second equation.
x  y  2
(10  2y)  y  2
10  2y  y  2
10  3y  2
10  3y  10  2  10
3y  12

5. Solve the first equation for x since the coefficient


of x is 1.
x  3y  3
x  3y  3y  3  3y
x  3  3y
Since x  3  3y, substitute 3  3y for x in the
second equation.
2x  9y  11
2(3  3y)  9y  11
6  6y  9y  11
6  15y  11
6  15y  6  11  6
15y  5
15y
15

3y
3

 15

x3

1
3

for y in either equation to find

113 2  3

x13
x1131
x4

1 12

The solution is 4, 3 .
6. Solve the second equation for x since the
coefficient of x is 1.
x  3y  4
x  3y  3y  4  3y
x  4  3y
Substitute 4  3y for x in the first equation.
3x  18  2y
3(4  3y)  18  2y
12  9y  18  2y
12  9y  9y  18  2y  9y
12  18  11y
12  18  18  11y  18
30  11y
30
11
30
11

6
3

y

Now substitute
the value of x.
x  3y  4
x3

11y
11

30
11

13011 2  4
90
90

90

x  11  11  4  11
x

46
11

The solution is

46 30
11, 11

3x
3

2  x
Now substitute 2 for x in either equation to find
the value of y.
2y  12  x
2y  12  (2)
2y  12  2
2y  14

for y in either equation to find

x  11  4
90

12
3

y4
Now substitute 4 for y in either equation to find
the value of x.
x  2y  10
x  2(4)  10
x  8  10
x  8  8  10  8
x2
The solution is (2, 4).
8. Solve the first equation for y since the coefficient
of y is 1.
2x  3  y
2x  y  3  y  y
2x  y  3
2x  y  2x  3  2x
y  3  2x
Since y  3  2x, substitute 3  2x for y in the
second equation.
2y  12  x
2(3  2x)  12  x
6  4x  12  x
6  4x  4x  12  x  4x
6  12  3x
6  12  12  3x  12
6  3x

y3
Now substitute
the value of x.
x  3y  3

2y
2

14
2

y7
The solution is (2, 7).

2.

701

Extra Practice

PQ249J-6481F-20-23[686-725] 26/9/02 9:43 PM Page 702 Sahuja Ahuja_QXP_06:Desktop Folder:Chandra:Algebra_FNL_Delivery:

x  6y  1
x  6(1)  1
x  (6)  1
x61
x6616
x7
The solution is (7, 1).
12. Solve the second equation for y since the
coefficient of y is 1.

9. Since y  3x, substitute 3x for y in the first


equation.
6y  x  36
6(3x)  x  36
18x  x  36
19x  36
19x
19

36

 19
36

x  19
Use y  3x to find the value of y.
y  3x
y  3
y

13619 2

108
 19

The solution is

3
x
2
3
x
2

3
(10
4
15
2

.
13619, 108
19 2

15
2

1
15
2

13

 2


11213 21132 2

2x
2

y  6
Now substitute 6 for y in either equation to find
the value of x.
x  y  10
x  (6)  10
x  6  10
x  6  6  10  6
x4
The solution is (4, 6).
11. Solve the first equation for x since the coefficient
of x is 1.
x  6y  1
x  6y  6y  1  6y
x  1  6y
Since x  1  6y, substitute 1  6y for x in the
second equation.
3x  10y  31
3(1  6y)  10y  31
3  18y  10y  31
3  28y  31
3  28y  3  31  3
28y  28
28y
28

5  3y
2
5
3

y
2
2

x
5

Since x  2  2y, substitute


second equation.
4x  9y  9
4

5
2

 2y for x in the

152  32y2  9y  9

10  6y  9y  9
10  15y  9
10  15y  10  9  10
15y  1
15y
15

1

 15
1

y  15
1
15

Now substitute
the value of x.
2x  3y  5
2x  3

for y in either equation to find

1151 2  5
1

2x  5  5
1

2x  5  5  5  5
24
5
1
1 24
2x  2 5
2
12
x 5
12 1
solution is 5 , 15

12

28

 28

y  1
Now substitute 1 for y in either equation to find
the value of x.

Extra Practice

3x  3x  6  12
6  12
The statement 6  12 is false. This means that
there is no solution to the system of equations.
13. Since no coefficient of x or y is 1 or 1, solve for
either variable in either equation. For example,
solve for x in the first equation.
2x  3y  5
2x  3y  3y  5  3y
2x  5  3y

 4y  3y  1

1 2

13

1

y

3x  2 2x  3  12

 y)  3y  1

3y3

3  y, substitute 2x  3 for y in the


first equation.
3x  2y  12

15
13
 12y
2
13
15
y 2
12
13
y
12
12 13
y
13 12

3y

3
x3
2
3
x3
2
3
Since 2x 

 3y  1

yy3y
3
x
2

10. Solve the second equation for x since the coefficient


of x is 1.
x  y  10
x  y  y  10  y
x  10  y
Since x  10  y, substitute 10  y for x in the
first equation.
3
x
4

y3

The

702

2x 

12

2.

PQ249J-6481F-20-23[686-725] 26/9/02 9:43 PM Page 703 Sahuja Ahuja_QXP_06:Desktop Folder:Chandra:Algebra_FNL_Delivery:

2. Since the coefficients of the y terms, 1 and 1,


are additive inverses, you can eliminate the y
terms by adding the equations.
2x  y  32
() 2x  y  60
4x
 92

14. Since x  4  8y, substitute 4  8y for x in the


second equation.
3x  24y  12
3(4  8y)  24y  12
12  24y  24y  12
12  12
The statement 12  12 is true. This means that
there are infinitely many solutions to the system
of equations.
15. Since no coefficient of x or y is 1 or 1, solve for
either variable in either equation. For example,
solve for x in the first equation.
3x  2y  3
3x  2y  2y  3  2y
3x  2y  3
3x
3

x

4x
4

Since x 
 1, substitute 3y  1 for x in the
second equation.
25x  10y  215

12

25 3 y  1  10y  215
50
y
3

2x
2

11
2
11
x 2
11
substitute 2

 25  10y  215
80
y
3

80
y
3

 25  215

 25  25  215  25
80
y
3
3 80
y
80 3

1 2

1803 2240

y

y9
Now substitute 9 for y in either equation to find
the value of x.
3x  2y  3
3x  2(9)  3
3x  18  3
3x  18  18  3  18
3x  15
3x
3

y

5

The solution is

11
2

1112, 12 2.

4. Since the coefficients of the t terms, 2 and 2, are


additive inverses, you can eliminate the t terms
by adding the equations.
s  2t  6
() 3s  2t  2
4s
8

15
3

4s
4

4

s2
Now substitute 2 for s in either equation to find
the value of t.
s  2t  6
2  2t  6
2  2t  2  6  2
2t  4

1. Since the coefficients of the y terms, 1 and 1,


are additive inverses, you can eliminate the y
terms by adding the equations.
xy7
() x  y  9
2x
 16


5
1

Lesson 7-3

2x
2

11
2

11
2
11
2

for x in either equation to find

y  2

x5
The solution is (5, 9).

Page 836

Now
the value of y.
yx5

 240


92
4

x  23
Now substitute 23 for x in either equation to find
the value of y.
2x  y  60
2(23)  y  60
46  y  60
46  y  46  60  46
y  14
The solution is (23, 14).
3. Since the coefficients of the y terms, 1 and 1,
are additive inverses, you can eliminate the y
terms by adding the equations.
y  x  6
() y  x  5
2x  11

2y  3
3
2
y1
3

2
y
3

2t
2

16
2

2

t2
The solution is (2, 2).

x8
Now substitute 8 for x in either equation to find
the value of y.
xy7
8y7
8y878
y  1
The solution is (8, 1).

703

Extra Practice

PQ249J-6481F-20-23[686-725] 26/9/02 9:43 PM Page 704 Sahuja Ahuja_QXP_06:Desktop Folder:Chandra:Algebra_FNL_Delivery:

8. Since the coefficients of the y terms, 1 and 1,


are additive inverses, you can eliminate the y
terms by adding the equations.
xy 8
() 2x  y  6
3x
 14

5. Since neither the coefficients of the x terms nor


the coefficients of the y terms are the same or
additive inverses, you cannot use elimination by
addition or subtraction to solve this system. Use
substitution.
Since x  y  7, substitute y  7 for x in the
second equation.
2x  5y  2
2(y  7)  5y  2
2y  14  5y  2
3y  14  2
3y  14  14  2  14
3y  12
3y
3

3x
3

14
3
14
x 3
14
substitute 3

Now
the value of y.
xy8
14
3

12

 3

14
3

y  4
Use x  y  7 to find the value of x.
xy7
x  4  7
x  11
The solution is (11, 4).
6. Since the coefficients of the x terms, 3 and 3, are
the same, you can eliminate the x terms by
subtracting the equations.
3x  5y  16
() 3x  2y  2
7y  14
7y
7

14
3

8

14
3

10
3

1143, 103 2.

9. Rewrite the second equation so the system is in


column form.
s  7  3t
s  3t  7  3t  3t
s  3t  7
Since the coefficients of the t terms, 3 and 3, are
additive inverses, you can eliminate the t terms
by adding the equations.
2s  3t  4
() s  3t  7
3s
 3

14
7

3s
3

3

s1
Now substitute 1 for s in either equation to find
the value of t.
s  7  3t
1  7  3t
1  7  7  3t  7
6  3t

6
3

6
3

3t
3

2t
The solution is (1, 2).
10. Rewrite the second equation so the system is in
column form.
6x  42  16y
6x  42  42  16y  42
6x  16y  42
6x  16y  16y  42  16y
6x  16y  42
Since the coefficients of the x terms, 6 and 6,
are additive inverses, you can eliminate the x
terms by adding the equations.
6x  16y  8
() 6x  16y  42
0  34
The statement 0  34 is false. This means that
there is no solution to the system of equations.

2

x3
Now substitute 3 for x in either equation to find
the value of y.
xy3
3y3
3y333
y  0
(1)(y)  (1)0
y0
The solution is (3, 0).

Extra Practice

y

The solution is

x  2
The solution is (2, 2).
7. Since the coefficients of the y terms, 1 and 1,
are additive inverses, you can eliminate the y
terms by adding the equations.
xy3
() x  y  3
2x
6
2x
2

for x in either equation to find

y8
y

y  2
Now substitute 2 for y in either equation to find
the value of x.
3x  5y  16
3x  5(2)  16
3x  (10)  16
3x  10  16
3x  10  10  16  10
3x  6
3x
3

704

PQ249J-6481F-20-23[686-725] 26/9/02 9:43 PM Page 705 Sahuja Ahuja_QXP_06:Desktop Folder:Chandra:Algebra_FNL_Delivery:

Since the coefficients of the y terms, 1 and 1,


are additive inverses, you can eliminate the y
terms by adding the equations.
xy0
() x  y  7
2x
7

11. Rewrite the second equation so the system is in


column form.
3x  0.4y  4
3x  0.4y  0.4y  4  0.4y
3x  0.4y  4
Since the coefficients of the x terms, 3 and 3, are
the same, you can eliminate the x terms by
subtracting the equations.
3x  0.2y  7
() 3x  0.4y  4
0.6y  3
0.6y
0.6

2x
2

x  3.5
Now substitute 3.5 for x in either equation to find
the value of y.
xy
3.5  y
The solution is (3.5, 3.5).
1
1
14. Since the coefficients of the y terms, 3 and 3, are
additive inverses, you can eliminate the y terms
by adding the equations.

 0.6

y5
Now substitute 5 for y in either equation to find
the value of x.
3x  0.2y  7
3x  0.2(5)  7
3x  1  7
3x  1  1  7  1
3x  6
3x
3

4x  3y  8
()

x

14
9
14
x 9
14
substitute 9 for

234
7

39
10.5
26
7

56
9

 26 

 3y  8

56
9
1
3y
1
3y

8


56
9

16
9

2  (3)169
16

y  3

1149, 163 2.

15. Since the coefficients of the y terms, 1 and 1,


are the same, you can eliminate the y terms by
subtracting the equations.
2x  y  3

234
7

52

2y   7

112 22y  112 21527 2

()

2
x
3
4
x
3

 y  1

12

26

y  7

The solution is

56
9
1
y
3

The solution is

 2y  26
234
7

1149 2  13y  8

(3)

1267 2  2y  26

 2y 

x in either equation to find

4x  3y  8

Now substitute for x in either equation to find


the value of y.
9x  2y  26
234
7

Now
the value of y.

26
7

 14
9x
9

x2
The solution is (2, 5).
12. Since the coefficients of the y terms, 2 and 2,
are additive inverses, you can eliminate the y
terms by adding the equations.
9x  2y  26
() 1.5x  2y  13
10.5x
 39


5x  3y  6
9x

3

10.5x
10.5

2

1267, 267 2.

3 4
x
4 3




1 24
3
4

x3
Now substitute 3 for x in either equation to find
the value of y.
2x  y  3
2(3)  y  3
6y3
6y636
y  3
(1)(y)  (1) (3)
y3
The solution is (3, 3).

13. Rewrite the first equation so the system is in


column form.
xy
xyyy
xy0

705

Extra Practice

PQ249J-6481F-20-23[686-725] 26/9/02 9:44 PM Page 706 Sahuja Ahuja_QXP_06:Desktop Folder:Chandra:Algebra_FNL_Delivery:

Page 836

Lesson 7-4

4. Multiply the first equation by 5 so the


coefficients of the y terms are additive inverses.
Then add the equations.

1. Multiply the second equation by 2 so the


coefficients of the y terms are additive inverses.
Then add the equations.
3x  2y  10
2x  y  5

Multiply by 2.

3x  2y  10
() 4x  2y  10
7x


7x
7

1
x
3

 y  1

1
x
5

 5 y  1

Multiply by 5 .

( )

0
0

 7

1
x
3

 y  1

1
(9)
3

 y  1

39

 13

3x  5y  8

Multiply by 4.

4x  7y  10

Multiply by 3.

12x  21y 

y2

16

37y  74


x  0.5y 

74
37

y  2

x  0.5y 
Multiply by 0.5.

()

0.2x  0.5y  1


1.2x
1.2x
1.2

0
0
1.2

x0

Now substitute 0 for x in either equation to find


the value of y.
0.4x  y  2
0.4(0)  y  2
y  2
The solution is (0, 2).

10
5

x2
The solution is (2, 2).

Extra Practice

18
3

0.4x  y  2

Now substitute 2 for y in either equation to find


the value of x.
5x  3y  4
5x  3(2)  4
5x  6  4
5x  6  6  4  6
5x  10
5x
5

x6
The solution is (6, 2).
6. Multiply the second equation by 0.5 so the
coefficients of the y terms are additive inverses.
Then add the equations.

20x  25y  90


37y
37

30

Now substitute 2 for y in either equation to find


the value of x.
3x  5y  8
3x  5(2)  8
3x  10  8
3x  10  10  8  10
3x  18
3x
3

20x  12y 

1 32

(1) (y)  (1) (2)

x  1
The solution is (1, 3).
3. Eliminate x.
()

y  2

2
2

Multiply by 5.

 5

12x  20y  32


()

y3

Multiply by 4.

1
x
15

3  y  1
3  y  3  1  3
y  2
(1)(y)  (1)2
y  2
The solution is (9, 2).
5. Eliminate x.

Now substitute 3 for y in either equation to find


the value of x.
2x  5y  13
2x  5(3)  13
2x  15  13
2x  15  15  13  15
2x  2

4x  5y  18

x  9

13y  39

5x  3y  4

 5 y  1

Now substitute 9 for x in either equation to find


the value of y.

2x  5y  13 Multiply by 2.
4x  10y  26
4x  3y  13
() 4x  3y  13

1
x
5

Now substitute 0 for x in either equation to find


the value of y.
2x  y  5
2(0)  y  5
y  5
(1)(y)  (1)(5)
y5
The solution is (0, 5).
2. Multiply the first equation by 2 so the
coefficients of the x terms are additive inverses.
Then add the equations.

2x
2

2
5

(15) 15 x  (15) 5

x0

13y
13

15 x  5 y 

706

PQ249J-6481F-20-23[686-725] 26/9/02 9:44 PM Page 707 Sahuja Ahuja_QXP_06:Desktop Folder:Chandra:Algebra_FNL_Delivery:

7. Multiply the first equation by 4 so the


coefficients of the x terms are additive inverses.
Then add the equations.
x  8y  3 Multiply by 4.
4x  2y  7

y  5  2

34y  5

y52

y
5
34

Now substitute
the value of x.
x  8y  3

1 32

4x  32y  12


( ) 4x  2y 
34y
34

x8

Now substitute 5 for x in either equation to find


the value of y.
yx2
3

5
34
5
34

y5

3 7

The solution is 5, 5 .

for y in either equation to find

10. Multiply the first equation by 2 so the


coefficients of the x terms are additive inverses.
Then add the equations.

1345 2  3

x  4y  30 Multiply by 2.
2x  8y  60
2x  y  6
() 2x  y  6

20

x  17  3
20

y5525

20

20

9y  66

x  17  17  3  17
x

9y
9

31
17

13117, 345 2.

The solution is

y

9x
9

x

x4

44
9

4
8

y  9

1223 2  30

x

11
9
11
9

x

88
3

88
3
88
3

 30
 30 
2

x3
The solution is

for x in either equation to find

44
9

The solution is

1 82

Now substitute
the value of y.
4x  4y  5

1119, 89 2.

Multiply by 3.

 10

x2

9. Multiply the second equation by 3 so the


coefficients of the y terms are additive inverses.
Then add the equations.
3y  8x  9
yx2

123, 223 2.

10x
10

(1)(y)  (1) 9
y9

88
3

11. Multiply the first equation by 2 so the coefficients


of the y terms are additive inverses. Then add the
equations.
3x  2y  0 Multiply by 2.
6x  4y  0
4x  4y  5
() 4x  4y  5
10x
5

y4

y

22
3

22

1119 2  y  4
44
9

44
9

11
9

66
9

Now substitute 3 for y in either equation to find


the value of x.
x  4y  30

8. Multiply the first equation by 2 so the coefficients


of the y terms are additive inverses. Then add the
equations.
4x  y  4 Multiply by 2.
8x  2y  8
x  2y  3
() x  2y  3
9x
 11

Now substitute
the value of y.
4x  y  4

()

5x
5

for x in either equation to find

112 2  4y  5

2  4y  5
2  4y  2  5  2
4y  3

3y  8x  9
3y  3x  6
5x 

1
2

4y
4

4
3

 5

y4

x  5

The solution is

707

112, 34 2.

Extra Practice

PQ249J-6481F-20-23[686-725] 26/9/02 9:44 PM Page 708 Sahuja Ahuja_QXP_06:Desktop Folder:Chandra:Algebra_FNL_Delivery:

15. Eliminate y.

12. Multiply the second equation by 3 so the


coefficients of the y terms are additive inverses.
Then add the equations.
9x  3y  5
9x  3y  5
xy1
Multiply by 3. () 3x  3y  3
12x
8
12x
12

6x  3y  9
8x  2y  4

2
3
2
3

Now substitute 2 for x in either equation to find


the value of y.
6x  3y  9

for x in either equation to find

The solution is
13. Eliminate x.
2x  7y  9
3x  4y  6

3y
3

123, 13 2.

Multiply by 3.
Multiply by 2.

()

6x  21y  27
6x  8y  12

Page 836

39

 13

y  3

y 6

x 3

12
2

2x  6y  16

Multiply by 5.

5x  7y  18

Multiply by 2.

2. The solution includes the ordered pairs in the


intersection of the graphs of y  2 and
y  x  2. The region is shaded. The graphs of
y  2 and y  x  2 are boundaries of this
region. The graphs of both boundaries are dashed
and are not included in the solution.

10x  30y  80


( )

10x  14y 

36

44y  44
44y
44

44
44

y1

Now substitute 1 for y in either equation to find


the value of x.
2x  6y  16
2x  6(1)  16
2x  6  16
2x  6  6  16  6
2x  10

y 2

y  x  2

10
2

x  5
The solution is (5, 1).

Extra Practice

Lesson 7-5

x  6
The solution is (6, 3).
14. Eliminate x.

112, 42.

1. The solution includes the ordered pairs in the


intersection of the graphs of x  3 and y  6. The
region is shaded. The graphs of x  3 and y  6
are boundaries of this region. The graphs of both
boundaries are dashed and are not included in
the solution.

Now substitute 3 for y in either equation to find


the value of x.
2x  7y  9
2x  7(3)  9
2x  (21)  9
2x  21  9
2x  21  21  9  21
2x  12

2x
2

12
3

y4

13y
13

The solution is

13y  39

2x
2

112 2  3y  9

3  3y  9
3  3y  3  9  3
3y  12

y313
1

6

 12
1

y1
y3

 6

x2

 12

12x  6y  18
24x  6y  12
12x
12

2
3

()

12x

x3
Now substitute
the value of y.
xy1

Multiply by 2.
Multiply by 3.

708

PQ249J-6481F-20-23[686-725] 26/9/02 9:44 PM Page 709 Sahuja Ahuja_QXP_06:Desktop Folder:Chandra:Algebra_FNL_Delivery:

6. The solution includes the ordered pairs in the


intersection of the graphs of y  x  3 and
y  x  2. The region is shaded. The graphs of
y  x  3 and y  x  2 are boundaries of this
region. The graphs of both boundaries are solid
and are included in the solution.

3. The solution includes the ordered pairs in the


intersection of the graphs of x  2 and y  3  5.
The region is shaded. The graphs of x  2 and
y  3  5, or y  2, are boundaries of this region.
The graphs of both boundaries are solid and are
included in the solution.
y

y
y 2

yx 3
yx 2

x 2

x
O

7. The solution includes the ordered pairs in the


intersection of the graphs of x  3y  4 and
2x  y  5. The region is shaded. The graphs of
x  3y  4 and 2x  y  5 are boundaries of this
region. The graph of x  3y  4 is solid and is
included in the graph of x  3y  4. The graph of
2x  y  5 is dashed and is not included in the
graph of 2x  y  5.

4. The solution includes the ordered pairs in the


intersection of the graphs of x  y  1 and
2x  y  2. The region is shaded. The graphs of
x  y  1 and 2x  y  2 are boundaries of this
region. The graphs of both boundaries are solid
and are included in the solution.
y

2x  y  2

x  y  1

x  3y  4

2x  y  5

5. The solution includes the ordered pairs in the


intersection of the graphs of y  2x  2 and
y  x  1. The region is shaded. The graphs of
y  2x  2 and y  x  1 are boundaries of this
region. The graphs of both boundaries are solid
and are included in the solution.
y  x  1

8. The solution includes the ordered pairs in the


intersection of the graphs of y  x  1 and
y  2x  10. The region is shaded. The graphs of
y  x  1 and y  2x  10 are boundaries of this
region. The graph of y  x  1 is dashed and is
not included in the graph of y  x  1. The graph
of y  2x  10 is solid and is included in the
graph of y  2x  10.

y  2x  2
O

y  2 x  10

y x1

709

Extra Practice

PQ249J-6481F-20-23[686-725] 26/9/02 9:45 PM Page 710 Sahuja Ahuja_QXP_06:Desktop Folder:Chandra:Algebra_FNL_Delivery:

12. The solution includes the ordered pairs in the


intersection of the graphs of 4x  10y  5 and
2x  5y  1. The region is shaded. The graph
of 2x  5y  1 is a boundary of this region.
The graph of this boundary is dashed and is not
included in the solution. (Note: The graph of
4x  10y  5 is not included in the solution
since it is not a boundary of the shaded region.)

9. The solution includes the ordered pairs in the


intersection of the graphs of 5x  2y  15 and
2x  3y  6. The region is shaded. The graphs of
5x  2y  15 and 2x  3y  6 are boundaries of
this region. The graphs of both boundaries are
dashed and are not included in the solution.
y
x

O
2x  3y  6

y
4x  10y  5

5 x  2 y  15

O
2x  5y  1

10. The solution includes the ordered pairs in the


intersection of the graphs of 4x  3y  4 and
2x  y  0. The region is shaded. The graphs of
4x  3y  4 and 2x  y  0 are boundaries of this
region. The graphs of both boundaries are dashed
and are not included in the solution.

13. The solution includes the ordered pairs in the


intersection of the graphs of y  x  0, y  3,
and x  0. The region is shaded. The graphs of
y  x  0, y  3 and x  0 are boundaries of this
region. The graphs of these boundaries are solid
and are included in the solution.

y
y3

4x  3y  4
O

2x y  0

x0

yx0

11. The solution includes the ordered pairs in the


intersection of the graphs of 4x  5y  20 and
y  x  1. The region is shaded. The graphs of
4x  5y  20 and y  x  1 are boundaries of this
region. The graphs of both boundaries are solid
and are included in the solution.

14. The solution includes the ordered pairs in the


intersection of the graphs of y  2x, x  3, and
y  4. The region is shaded. The graphs of y  2x,
x  3, and y  4 are boundaries of this region.
The graphs of these boundaries are dashed and
are not included in the solution.

yx1

y 4

x  3
x

4x  5y  20

Extra Practice

710

y  2x

PQ249J-6481F-20-23[686-725] 26/9/02 9:45 PM Page 711 Sahuja Ahuja_QXP_06:Desktop Folder:Chandra:Algebra_FNL_Delivery:

15. The solution includes the ordered pairs in the


intersection of the graphs of y  x, x  y  4 and
y  3. The region is shaded. The graphs of
y  x, x  y  4, and y  3 are boundaries of
this region. The graph of x  y  4 is dashed and
is not included in the graph of x  y  4. The
graphs of y  x and y  3 are solid and are
included in the graphs of y  x and y  3,
respectively.
y

15.

 4 w6 w8
1

 4w68
1

 4w14
16. [ (23 ) 3 ] 2  [ (8) 3 ] 2
 (8) 6
 262,144

yx

17.
xy4
x

Lesson 8-1

1. No; it shows subtraction, not multiplication of


variables and numbers.
2. Yes; it is a real number and therefore a monomial.
3. Yes; it is a product of a number and two variables.
4. No; it shows subtraction, not multiplication of
variables and numbers.
5. a5 (a)(a7 )  a517
 a13
3
4
4
4
6. (r t ) (r t )  (r3 r4 )(t4 t4 )
 (r34 )(t44 )
 r7t8
3
4
3
7. (x y )(xy )  (x3 x)(y4 y3 )
 (x31 )(y43 )
 x4y7
8. (bc3 )(b4c3 )  (b b4 )(c3 c3 )
 (b14 )(c33 )
 b5c6
2
9. (3mn )(5m3n2 )  (3 5)(m m3 )(n2 n2 )
 15(m13 ) (n22 )
 15m4n4
3
2
2
6
2
10. [ (3 ) ]  [3 ]
 312 or 531,441
3
2
11. (3s t )(4s3t2 )  3(4) (s3 s3 )(t2 t2 )
 12(s33 )(t22 )
 12s6t4
12. x3 (x4y3 )  (x3 x4 )y3
 (x34 )y3
 x7y3
2
4
13. (1.1g h ) 3  (1.1) 3 (g2 ) 3 (h4 ) 3
 1.331g6h12
14.

3
4a(a2b3c4 )





123y32 (3y2)3  123y32 (3)3(y2)3


2
 1 3y3 2 (27) (y6 )
2
 1 3 27 2 (y3 y6 )

 18(y36 )
 18y9
3
18. (10s t)(2s2t2 ) 3  (10s3t) (2) 3 (s2 ) 3 (t2 ) 3
 (10s3t) (8) (s6 ) (t6 )
 10(8) (s3 s6 ) (t t6 )
 80(s36 ) (t16 )
 80s9t7
3
4
3
19. (0.2u w )  (0.2) 3 (u3 ) 3 (w4 ) 3
 0.008u9w12

y  3

Page 837

112w322(w4)2  112 22(w3)2(w4)2

Page 837
10

Lesson 8-2
107

1.

6
67

2.

b c
b3c2

6

 63 or 216
6 5

1bb 21cc 2
6

63

 (b
) (c52 )
3 3
b c
8

3.

(a) 4b
a4b7

4 8

a b

 a4b7


1aa 21bb 2
4

44

 (a
) (b87 )
0 1
a b
 1b or b
4.

(x) 3y
x3y6



1xx 21yy 2
3

 (x33 ) (y36 )
 (x0 ) (y3 )
 (1)

5

5.

12ab
4a4b3

1
y3

1y1 2
3

1124 21aa 21bb 2


5

3(a14 ) (b53 )


 3a3b2

1 21 2

3 1 b2
a3 1

1

5

3
4 (a a2 )b3c4
3
4 (a12 )b3c4
3
4a3b3c4

6.

24x
8x2

3b2
a3

24 x
18
21x 2
5
2

3(x52)


 3x3

711

Extra Practice

PQ249J-6481F-20-23[686-725] 26/9/02 9:45 PM Page 712 Sahuja Ahuja_QXP_06:Desktop Folder:Chandra:Algebra_FNL_Delivery:

7.

9b2k4
18h5j3k4

h
1 k
19
18 21 h 21 j 21 k 2
1
1
 2 (h25 ) 1 j 2 (k44 )
1
1
 2 (h3 ) 1 j 2 (k0 )
1 1 1
 2 1 h 21 j 2 (1)
2

15.

1ab 23  (a(b ))


3

3 3

2 3

1a1 21b1 2
1 b
 1 a 21 1 2

8.

(2a b ) 2
(3a3b) 2

2 4
2a b 2
3a3b

9.

9a b c
2a5b4c5

 a9

16.

(2) 2 (a ) 2 (b ) 2
(3) 2 (a3 ) 2 (b) 2

4a4b8
9a6b2




4b
9a2

10.

17.

(r)s
r3s4

18.

4 0

28a b
14a3b1




2 3

19.

x
y
z
115
10 21 x 21 y 21 z 2
5

4

4

(j k m)
(jk4 ) 1

 2 (x1(4) )(y56 )(z7(4) )


3

 2x5y11z11
3

1 21y1 21z1 2
5

11

3 x
1

2

4

11. 3
12.

156 2

2

1
34

or

11

3x z
2y11

21.

1
81

151 2161 2
1 6
 1 5 21 1 2
2

2

22.

5b0a7

(u v )
(u3v) 3

j k m
j1k4

12

1

4

8 12

1 jj 21kk 21m1 2
4

1

3

2

3

122aab b 21  (2(2aab b) )


3 2

3 2

(u ) (v )
(u3 ) 3 (v) 3
v

1uu 21vv 2
9

3

1

2

2 (a ) 1 (b ) 1
(21 ) 1 (a5 ) 1 (b3 ) 1

21a3b2
2a5b3

6 6

1 5 3 1
3

122 21aa 21bb 2


1

3

3

 (211 ) (a35 ) (b2(3) )


 22a8b5

3 2

6

3 2 1

1 5 3

121 21a1 21b1 2


5

b5

 4a8

23.

(u6(9) )(v6(3) )


 u3v9

Extra Practice

2 3

 u9v3


18x a
18 x
a
 1 6 21 x 21 a 2
16x
a 2


 (a2 ) (1)

2b
a7

0 3

(a5(7) )(b0 )

121 21a1 21b1 2

14.

1

(j ) 4 (k ) 4 (m) 4
(j) 1 (k4 ) 1

 a2
3 3 2

 ( j8(1) )(k12(4) ) (m4 )


 j9k16m4

36
25

or

4


 3x2a0
 3x2

52

62
52

12814 21aa 21bb 2

 62

4

3(x0(2) ) (a3(3) )

13. a

y
12x
4y 2
4

20.

5 11

3

 2(a43 )(b0(1) )
 2a7b1

1 21 21c1 2


1rr 21ss 2

5 7

 (r1(3) )(s5(4) )
 r4s9

1 21 21 21 2

15xy z
10x4y6z4

9 25
(a
)(b74 )(c35 )
2
9 3 3 2
a b c
2

9b3
2a3c2

2

9 a b c
2 a5 b4 c5

9 1 b
a3 1

2

 4x2y6

x
y6

121 21x1 21y1 2


1 1 1
 1 2 21 x 21 y 2

1 21 2
2

2 2

2


3 2

4 1 b
9 a2 1

2

3

1 21 2
4

1y2x 22  (y(2x))




4 a b
a6 b2
4 46
(a
)(b82 )
9
4 2 6
a b
9

2 7 3

2 4

9

6

9

1
2h3j3

9

a
b6

712

15n2nmm 20  1
1

2

PQ249J-6481F-20-23[686-725] 26/9/02 9:45 PM Page 713 Sahuja Ahuja_QXP_06:Desktop Folder:Chandra:Algebra_FNL_Delivery:

(3ab c) 3
(2a2bc2 ) 2
2

24.




25. (3.1
104 ) (4.2
105 )  (3.1
4.2)(104
105 )

3

(a) 3 (b2 ) 3 (c) 3


22 (a2 ) 2 (b) 2 (c2 ) 2

 13.02
101
 1.302
101
101
 1.302
100 or 1.302

33a3b6c3
22a4b2c4

131 2121 21aa 21bb 21cc 2


1 1
 1 27 21 4 2 (a34 )(b62 )(c34 )
3

3

6

3

26. (78
106 ) (0.01
105 )  (78
0.01)(106
105 )
 0.78
1011
 7.8
101
1011
 7.8
1010 or 78,000,000,000

 108a7b8c7
1
1

 108

1a1 21b1 21c1 2


7

27.

 108a7b8c7

Page 837

2

2.31
10
3.3
103

10
12.31
3.3 21 10 2
2
3

 0.7
102(3)
 0.7
101
 0.7
101
101
 7
100 or 7

Lesson 8-3

1. 2.6
105  260,000
2. 4
103  0.004

Page 838

3. 6.72
103  6720
4. 4.93
104  0.000493
5. 1.654
106  0.000001654
6. 7.348
107  73,480,000
7. 6500  6.5
103
8. 953.56  9.5356
102
9. 0.697  6.97
101

3.

12. 0.0000269  2.69


105
13. 0.121212  1.21212
101
14. 543
104  5.43
102
104
 5.43
106
15. 739.9
105  7.399
102
105
 7.399
103
2
16. 6480
10  6.48
103
102
 6.48
101
7
17. 0.366
10  3.66
101
107
 3.66
108
3
18. 167
10  1.67
102
103
 1.67
105

7. The polynomial

10
14.8
1.6 21 10 2
3
1

10. The polynomial

 3
1031
 3
102 or 300
21. (4
102 ) (1.5
106 )  (4
1.5)(102
106 )
 6
108 or 600,000,000
2

22.

8.1
10
2.7
103

and

7.8
105
1.3
107

has only one term, whose


a3

1
,
5

x2
3

10
18.1
2.7 21 10 2

 2  5 has three terms,

x2
,
3

2,

whose degrees are 2, 1, and 0, respectively.

Thus, the degree of

x2
3

 2  5 is 2, the greatest of

2, 1, and 0.
11. The polynomial 6 has only one term, whose
degree is 0. Thus, the degree of 6 is 0.
12. The polynomial a2b3  a3b2 has two terms, a2b3
and a3b2, whose degrees are both 5. Thus, the
degree of a2b3  a3b2 is 5.
13. 2x2  3x  4x3  x5
 2x2  3x1  4x3  x5
 3x  2x2  4x3  x5

3

 3
102(3)
 3
105 or 300,000
23.

a3
4

degree is 3. Thus, the degree of 4 is 3.


8. The polynomial 10 has only one term, whose
degree is 0. Thus, the degree of 10 is 0.
9. The polynomial 4h5 has only one term, whose
degree is 5. Thus, the degree of 4h5 is 5.

19. (2
105 ) (3
108 )  (2
3) (105
108 )
 6
103 or 0.006
3

 k2y

The term k is not a monomial. No, the expression


is not a polynomial.
4. 3a2x  5a  3a2x  (5a)
This expression is the sum of two monomials.
Yes, it is a polynomial; it is a binomial.
5. The polynomial a  5c has two terms, a and 5c,
whose degrees are both 1. Thus, the degree of
a  5c is 1.
6. The polynomial 14abcd  6d3 has two terms,
14abcd and 6d3, whose degrees are 4 and 3,
respectively. Thus, the degree of 14abcd  6d3 is
4, the greater of 4 and 3.

11. 568,000  5.68


105

4.8
10
1.6
101

5
k

10. 843.5  8.435


102

20.

Lesson 8-4

1. 5x2y  3xy  7  5x2y  3xy  (7)


This expression is the sum of three monomials.
Yes, it is a polynomial; it is a trinomial.
2. 0
This expression is a monomial. Yes, it is a
polynomial.

10
17.8
1.3 21 10 2
5
7

 6
105(7)
 6
102 or 600
24. (2.2
102 ) (3.2
105 )  (2.2
3.2) (102
105 )
 7.04
103 or 7040

713

Extra Practice

PQ249J-6481F-20-23[686-725] 26/9/02 9:45 PM Page 714 Sahuja Ahuja_QXP_06:Desktop Folder:Chandra:Algebra_FNL_Delivery:

14. x3  x2  x  1
 x3  x2  x1  1x0
 1  x  x2  x3
15. 2a  3ax2  4ax
 2ax0  3ax2  4ax1
 2a  4ax  3ax2
16. 5bx3  2bx  4x2  b3
 5bx3  2bx1  4x2  b3x0
 b3  2bx  4x2  5bx3
8
17. x  2x2  x6  1
 x8  2x2  x6  1x0
 1  2x2  x6  x8
18. cdx2  c2d2x  d3
 cdx2  c2d2x1  d3x0
 d3  c2d2x  cdx2
19. 5x2  3x3  7  2x
 5x2  3x3  7x0  2x1
 3x3  5x2  2x  7
20. 6x  x5  4x3  20
 6x1  x5  4x3  20x0
 x5  4x3  6x  20

7. (4a2  10b2  7c2 )  (5a2  2c2  2b)


 (4a2  5a2 )  (10b2 )  (7c2  2c2 )  2b
 a2  10b2  9c2  2b
2  6z  8)  (4z2  7z  5)
8. (z
 (z2  6z  8)  (4z2  7z  5)
 (z2  4z2 )  (6z  7z)  (8  5)
 3z2  13z  3
9. (4d  3e  8f )  (3d  10e  5f  6)
 (4d  3e  8f )  (3d  10e  5f  6)
 (4d  3d)  (3e  10e)  (8f  5f )  6
 7d  7e  3f  6
10. (7g  8h  9)  (g  3h  6k)
 (7g  g)  (8h  3h)  9  6k
 6g  5h  9  6k
11. (9x2  11xy  3y2 )  (x2  16xy  12y2 )
 (9x2  11xy  3y2 )  (x2  16xy  12y2 )
 (9x2  x2 )  (11xy  16xy)  (3y2  12y2 )
 8x2  5xy  15y2

12. (3m  9mn  5n)  (14m  5mn  2n)


 (3m  14m)  (9mn  5mn)  (5n  2n)
 11m  4mn  7n
13. (4x2  8y2  3z2 )  (7x2  14z2  12)
 (4x2  8y2  3z2 )  (7x2  14z2  12)
 (4x2  7x2 )  8y2  (3z2  14z2 )  12
 3x2  8y2  11z2  12
14. (17z4  5z2  3z)  (4z4  2z3  3z)
 (17z4  5z2  3z)  (4z4  2z3  3z)
 (17z4  4z4 )  2z3  5z2  (3z  3z)
 13z4  2z3  5z2
15. (6  7y  3y2 )  (3  5y  2y2 )  (12  8y  y2 )

21. 5b  b3x2  3bx


2

 5bx0  b3x2  3bx1


2

 b3x2  3bx  5b
22. 21p2x  3px3  p4
 21p2x1  3px3  p4x0
 3px3  21p2x  p4
23. 3ax2  6a2x3  7a3  8x
 3ax2  6a2x3  7a3x0  8x1
 6a2x3  3ax2  8x  7a3
1

 (3y2  2y2  y2 )  (7y  5y  8y)  (6  3  12)


 2y2  20y  3

16.

24. 3 s2x3  4x4  5 s4x2  4 x


1

2 4 2
s x
5

1
x
4

 (7c 2  16c 2  9c 2 )  (2c  9c  7c )  (5  6  3  7 )

 3 s2x3  4x4  5 s4x2  4 x1


4

 4x 

1 2 3
s x
3

(7c 2  2c  5 )  (9c  6 )  (16c 2  3 )  (9c 2  7c  7 )


 1

Page 838
Page 838

Lesson 8-5

1. (3a2  5)  (4a2  1)  (3a2  4a2 )  (5  1)


 7a2  4
2. (5x  3)  (2x  1)  (5x  2x)  (3  1)
 3x  2
3. (6z  2)  (9z  3)  (6z  2)  (9z  3)
 (6z  9z)(2  3)
 3z  1
4. (4n  7)  (7n  8)  (4n  7)  (7n  8)
 (4n  7n)  (7  8)
 3n  15
5. (7t2  4ts  6s2 )  (5t2  12ts  3s2 )
 (7t2  5t2 )  (4ts  12ts)  (6s2  3s2 )
 12t2  8ts  3s2
6. (6a2  7ab  4b2 )  (2a2  5ab  6b2 )
 (6a2  7ab  4b2 )  (2a2  5ab  6b2 )
 (6a2  2a2 )  (7ab  5ab)  (4b2  6b2 )
 4a2  12ab  10b2

Extra Practice

Lesson 8-6

1. 3(8x  5)  3(8x)  (3) (5)


 24x  15
2. 3b(5b  8)  3b(5b)  3b(8)
 15b2  24b
3. 1.1a(2a  7)  1.1a(2a)  1.1a(7)
 2.2a2  7.7a
1

4. 2x(8x  6)  2x(8x)  2x(6)


 4x2  3x
5. 7xy(5x  y2 )  7xy(5x2 )  7xy(y2 )
 35x3y  7xy3
6. 5y(y2  3y  6)  5y(y2 )  5y(3y)  5y(6)
 5y3  15y2  30y
2

7. ab(3b2  4ab  6a2 )


 ab(3b2 )  ab(4ab)  ab(6a2 )
 3ab3  4a2b2  6a3b
8. 4m2 (9m2n  mn  5n2 )
 4m2 (9m2n)  4m2 (mn)  4m2 (5n2 )
 36m4n  4m3n  20m2n2

714

PQ249J-6481F-20-23[686-725] 26/9/02 9:45 PM Page 715 Sahuja Ahuja_QXP_06:Desktop Folder:Chandra:Algebra_FNL_Delivery:

11(n  3)  5  2n  44
11(n)  11(3)  5  2n  44
11n  33  5  2n  44
11n  28  2n  44
9n  28  44
9n  72
n8
24.
a(a  6)  2a  3  a(a  2)
a(a)  a(6)  2a  3  a(a)  a(2)
a2  6a  2a  3  a2  2a
a2  4a  3  a2  2a
4a  3  2a
2a  3

9. 4st2 (4s2t3  7s5  3st3 )


 4st2 (4s2t3 )  4st2 (7s5 )  4st2 (3st3 )
 16s3t5  28s6t2  12s2t5
1

23.

10. 3x(9x2  x  5)  3x(9x2 )  3x(x)  3x(5)


1

 3x3  3x2  3x
11. 2mn(8m2  3mn  n2 )
 2mn(8m2 )  2mn(3mn)  2mn(n2 )
 16m3n  6m2n2  2mn3
3

11

12. 4ab2 3b2  9b  1





1 2

3
1
3
4
4 ab2 3b2  4 ab2 9b
1
1
3
4ab4  3ab3  4ab2

2  34 ab2(1)

a  2 or 1.5

13. 3a(2a  12)  5a  3a(2a)  3a(12)  5a


 6a2  36a  5a
 6a2  41a
14. 6(12b2  2b)  7(2  3b)
 6(12b2 )  6(2b)  7(2)  7(3b)
 72b2  12b  14  21b
 72b2  33b  14
15. x(x  6)  x(x  2)  2x
 x(x)  x(6)  x(x)  x(2)  2x
 x2  6x  x2  2x  2x
 2x2  6x
16. 11(n  3)  2(n2  22n)
 11(n)  11(3)  2(n2 )  2(22n)
 11n  33  2n2  44n
 2n2  55n  33
17. 2x(x  3)  3(x  3)
 2x(x)  2x(3)  3(x)  3(3)
 2x2  6x  3x  9
 2x2  3x  9
18. 4m(n  1)  5n(n  1)
 4m(n)  4m(1)  5n(n)  5n(1)
 4mn  4m  5n2  5n
19. 7xy  x(7y  3)  7xy  x(7y)  x(3)
 7xy  7xy  3x
 3x
20. 5(c  3a)  c(2c  1)
 5(c)  5(3a)  c(2c)  c(1)
 5c  15a  2c2  c
 2c2  6c  15a
21. 9n(1  n)  4(n2  n)
 9n(1)  9n(n)  4(n2 )  4(n)
 9n  9n2  4n2  4n
 5n  13n2
22.
6(11  2x)  7(2  2x)
6(11)  6(2x)  7(2)  7(2x)
66  12x  14  14x
66  26x  14
26x  52
x2

25.

q(2q  3)  20  2q(q  3)
q(2q)  q(3)  20  2q(q)  2q(3)
2q2  3q  20  2q2  6q
3q  20  6q
9q  20  0
9q  20
20

q  9

w(w  12)  w(w  14)  12


w(w)  w(12)  w(w)  w(14)  12
w2  12w  w2  14w  12
12w  14w  12
2w  12
w  6
x(x  3)  4x  3  8x  x(3  x)
27.
x(x)  x(3)  4x  3  8x  x(3)  x(x)
x2  3x  4x  3  8x  3x  x2
x2  x  3  11x  x2
x  3  11x
10x  3  0
10x  3
26.

x  10
28.

3(x  5)  x(x  1)  x(x  2)  3


3(x)  3(5)  x(x)  x(1)  x(x)  x(2)  3
3x  15  x2  x  x2  2x  3
4x  15  x2  x2  2x  3
4x  15  2x  3
6x  15  3
6x  12
x  2

29.

n(n  5)  n(n  2)  2n(n  1)  1.5


n(n)  n(5)  n(n)  n(2)  2n(n)  2n(1)  1.5
n2  5n  n2  2n  2n2  2n  1.5
2n2  3n  2n2  2n  1.5
3n  2n  1.5
n  1.5
n  1.5

Page 839

Lesson 8-7

1. (d  2)(d  5)  d(d)  d(5)  2(d)  2(5)


 d2  5d  2d  10
 d2  7d  10
2. (z  7)(z  4)  z(z)  z(4)  7(z)  7(4)
 z2  4z  7z  28
 z2  3z  28

715

Extra Practice

PQ249J-6481F-20-23[686-725] 26/9/02 9:45 PM Page 716 Sahuja Ahuja_QXP_06:Desktop Folder:Chandra:Algebra_FNL_Delivery:

3. (m  8) (m  5)  m(m)  m(5)  8(m)  8(5)


 m2  5m  8m  40
 m2  13m  40
4. (a  2) (a  19)  a(a)  a(19)  2(a)  2(19)
 a2  19a  2a  38
 a2  17a  38
5. (c  15) (c  3)  c(c)  c(3)  15(c)  15(3)
 c2  3c  15c  45
 c2  12c  45
6. (x  y)(x  2y)  x(x)  x(2y)  y(x)  y(2y)
 x2  2xy  xy  2y2
 x2  xy  2y2
7. (2x  5)(x  6)  2x(x)  2x(6)  5(x)  5(6)
 2x2  12x  5x  30
 2x2  7x  30
8. (7a  4) (2a  5)  7a(2a)  7a(5)  4(2a)  4(5)

19. (x  2)(x2  2x  4)
 x(x2  2x  4)  2(x2  2x  4)
 x3  2x2  4x  2x2  4x  8
 x3  8
20. (3x  5)(2x2  5x  11)
 3x(2x2  5x  11)  5(2x2  5x  11)
 6x3  15x2  33x  10x2  25x  55
 6x3  5x2  8x  55
21. (4s  5)(3s2  8s  9)
 4s(3s2  8s  9)  5(3s2  8s  9)
 12s3  32s2  36s  15s2  40s  45
 12s3  47s2  4s  45
22. (3a  5)(8a2  2a  3)
 3a(8a2  2a  3)  5(8a2  2a  3)
 24a3  6a2  9a  40a2  10a  15
 24a3  34a2  19a  15
23. (a  b)(a2  ab  b2 )
 a(a2  ab  b2 )  b(a2  ab  b2 )
 a3  a2b  ab2  a2b  ab2  b3
 a3  b3
24. (c  d)(c2  cd  d2 )
 c(c2  cd  d2 )  d(c2  cd  d2 )
 c3  c2d  cd2  c2d  cd2  d3
 c3  d 3

 14a2  35a  8a  20
 14a2  43a  20

9. (4x  y)(2x  3y)


 4x(2x)  4x(3y)  y(2x)  y(3y)
 8x2  12xy  2xy  3y2
 8x2  10xy  3y2
10. (7v  3)(v  4)  7v(v)  7v(4)  3(v)  3(4)
 7v2  28v  3v  12
 7v2  31v  12
11. (7s  8) (3s  2)
 7s(3s)  7s(2)  8(3s)  8(2)
 21s2  14s  24s  16
 21s2  38s  16
12. (4g  3h)(2g  5h)
 4g(2g)  4g(5h)  3h(2g)  3h(5h)
 8g2  20gh  6gh  15h2
 8g2  14gh  15h2
13. (4a  3) (2a  1)
 4a(2a)  4a(1)  3(2a)  3(1)
 8a2  4a  6a  3
 8a2  2a  3
14. (7y  1) (2y  3)
 7y(2y)  7y(3)  1(2y)  1(3)
 14y2  21y  2y  3
 14y2  23y  3
15. (2x  3y)(4x  2y)
 2x(4x)  2x(2y)  3y(4x)  3y(2y)
 8x2  4xy  12xy  6y2
 8x2  16xy  6y2
16. (12r  4s)(5r  8s)
 12r(5r)  12r(8s)  4s(5r)  4s(8s)
 60r2  96rs  20rs  32s2
 60r2  76rs  32s2
17. (a  1)(3a  2)
 a(3a)  a(2)  1(3a)  1(2)
 3a2  2a  3a  2
 3a2  a  2
18. (2n  4)(3n  2)
 2n(3n)  2n(2)  4(3n)  4(2)
 6n2  4n  12n  8
 6n2  8n  8

Extra Practice

25. (5x  2)(5x2  2x  7)


 5x(5x2  2x  7)  2(5x2  2x  7)
 25x3  10x2  35x  10x2  4x  14
 25x3  20x2  31x  14
26. (n  2)(2n2  n  1)
 n(2n2  n  1)  2(2n2  n  1)
 2n3  n2  n  4n2  2n  2
 2n3  5n2  3n  2
27. (x2  7x  4)(2x2  3x  6)
 x2 (2x2  3x  6)  7x(2x2  3x  6)  4(2x2  3x  6)
 2x4  3x3  6x2  14x3  21x2  42x  8x2  12x  24
 2x4  17x3  23x2  30x  24

28. (x2  x  1) (x2  x  1)


 x2 (x2  x  1)  x(x2  x  1)  1(x2  x  1)
 x4  x3  x2  x3  x2  x  x2  x  1
 x4  x2  2x  1

29.

(a2  2a  5)(a2  3a  7)
 a2 (a2  3a  7)  2a(a2  3a  7)  5(a2  3a  7)
 a4  3a3  7a2  2a3  6a2  14a  5a2  15a  35
 a4  a3  8a2  29a  35

30.

(5x4  2x2  1)(x2  5x  3)


 5x4 (x2  5x  3)  2x2 (x2  5x  3)  1(x2  5x  3)
 5x6  25x5  15x4  2x4  10x3  6x2  x2  5x  3
 5x6  25x5  13x4  10x3  5x2  5x  3

Page 839

Lesson 8-8

1. (t  7) 2  t2  2(t) (7)  72
 t2  14t  49
2. (w  12)(w  12)  w2  122
 w2  144
2
2
3. (q  4h)  q  2(q) (4h)  (4h) 2
 q2  8hq  16h2

716

PQ249J-6481F-20-23[686-725] 26/9/02 9:45 PM Page 717 Sahuja Ahuja_QXP_06:Desktop Folder:Chandra:Algebra_FNL_Delivery:

28. (x  2)(x  2) (2x  5)


 (x2  22 ) (2x  5)
 (x2  4) (2x  5)
 x2 (2x)  x2 (5)  4(2x)  4(5)
 2x3  5x2  8x  20
29. (4x  1)(4x  1) (x  4)
 [ (4x) 2  12 ] (x  4)
 (16x2  1) (x  4)
 16x2 (x)  16x2 (4)  1(x)  1(4)
 16x3  64x2  x  4
30. (x  5)(x  5) (x  4) (x  4)
 (x2  52 ) (x2  42 )
 (x2  25)(x2  16)
 x2 (x2 )  x2 (16)  25(x2 )  25(16)
 x4  16x2  25x2  400
 x4  41x2  400
31. (a  1)(a  1) (a  1) (a  1)
 (a  1) (a  1) (a  1) (a  1)
 (a2  12 ) (a2  12 )
 (a2  1) 2
 (a2 ) 2  2(a2 ) (1)  12
 a4  2a2  1
32. (n  1)(n  1) (n  1)
 (n2  12 ) (n  1)
 (n2  1) (n  1)
 n2 (n)  n2 (1)  1(n)  1(1)
 n3  n2  n  1
33. (2c  3)(2c  3) (2c  3) (2c  3)
 (2c  3) (2c  3) (2c  3) (2c  3)
 [ (2c) 2  32 ] [ (2c) 2  32 ]
 (4c2  9) (4c2  9)
 (4c2  9) 2
 (4c2 ) 2  2(4c2 ) (9)  92
 16c4  72c2  81
34. (4d  5e)(4d  5e) (4d  5e) (4d  5e)
 (4d  5e) (4d  5e) (4d  5e) (4d  5e)
 [ (4d) 2  (5e) 2 ] [ (4d) 2  (5e) 2 ]
 (16d2  25e2 ) (16d2  25e2 )
 (16d2  25e2 ) 2
 (16d2 ) 2  2(16d2 ) (25e2 )  (25e2 ) 2
 256d2  800e2d2  625e4

4. (10x  11y)(10x  11y)  (10x) 2  (11y) 2


 100x2  121y2
2
2
5. (4e  3)  (4e)  2(4e)(3)  32
 16e2  24e  9
6. (2b  4d)(2b  4d)  (2b) 2  (4d) 2
 4b2  16d2
2
2
7. (a  2b)  a  2(a)(2b)  (2b) 2
 a2  4ab  4b2
8. (3x  y) 2  (3x) 2  2(3x)(y)  y2
 9x2  6xy  y2
9. (6m  2n) 2  (6m) 2  2(6m) (2n)  (2n) 2
 36m2  24mn  4n2
2
10. (3m  7d)  (3m) 2  2(3m) (7d)  (7d) 2
 9m2  42md  49d2
11. (5b  6) (5b  6)  (5b) 2  62
 25b2  36
12. (1  x) 2  1  2(1)(x)  x2
 1  2x  x2
13. (5x  9y) 2  (5x) 2  2(5x)(9y)  (9y) 2
 25x2  90xy  81y2
14. (8a  2b)(8a  2b)  (8a) 2  (2b) 2
 64a2  4b2
15.

114 x  422  114 x22  2114 x2 (4)  (4)2


1

 16 x2  2x  16

16. (c  3d) 2  c2  2(c)(3d)  (3d) 2


 c2  6cd  9d2
17. (5a  12b) 2  (5a) 2  2(5a)(12b)  (12b) 2
 25a2  120ab  144b2
18.

112 x  y22  112 x22  2112 x2 (y)  y2


1

 4x2  xy  y2

19. (n2  1) 2  (n2 ) 2  2(n2 ) (1)  (1) 2


 n4  2n2  1
2
2
20. (k  3j)  (k2 ) 2  2(k2 )(3j)  (3j) 2
 k4  6k2j  9j2
2  5) (a2  5)  (a2 ) 2  (5) 2
21. (a
 a4  25
3  7)(2x3  7)  (2x3 ) 2  (7) 2
22. (2x
 4x6  49
3
3
23. (3x  9y)(3x  9y)  (3x3 ) 2  (9y) 2
 9x6  81y2
2
2
24. (7a  b) (7a  b)  (7a2 ) 2  (b) 2
 49a4  b2
25.

Page 839

112x  102112x  102  112x22  (10)2


1

 4x2  100

26.

Lesson 9-1

1. List all pairs of numbers whose product is 23.


1
23
factors: 1, 23
prime
2. List all pairs of numbers whose product is 21.
1
21
3
7
factors: 1, 3, 7, 21
composite
3. List all pairs of numbers whose product is 81.
1
81
3
27
9
9
factors: 1, 3, 9, 27, 81
composite

113n  m2113n  m2  113n22  m2


1

 9n2  m2

27. (a  1)(a  1)(a  1)


 (a  1) [ a2  2(a)(1)  12 ]
 (a  1) [a2  2a  1]
 a(a2  2a  1)  1(a2  2a  1)
 a3  2a2  a  a2  2a  1
 a3  3a2  3a  1

717

Extra Practice

PQ249J-6481F-20-23[686-725] 26/9/02 9:45 PM Page 718 Sahuja Ahuja_QXP_06:Desktop Folder:Chandra:Algebra_FNL_Delivery:

21. Factor each monomial and circle the common


prime factors.
45  3 3 5
80  2 2 2 2 5
GCF: 5
22. Factor each monomial and circle the common
prime factors.
29  1 29

4. List all pairs of numbers whose product is 24.


1
24
2
12
3
8
4
6
factors: 1, 2, 3, 4, 6, 8, 12, 24
composite
5. List all pairs of numbers whose product is 18.
1
18
2
9
3
6
factors: 1, 2, 3, 6, 9, 18
composite
6. List all pairs of numbers whose product is 22.
1
22
2
11
factors: 1, 2, 11, 22
composite
7. 42  2 21
 2 3 7
8. 267  3 89
9. 72  1 72
 1 2 36
 1 2 3 12
 1 2 3 3 4
 1 23 32
10. 164  2 82
 2 2 41
 22 41
11. 57  1 57
 1 3 19
12. 60  1 60
 1 2 30
 1 2 2 15
 1 22 3 5
13. 240mn  2 120 m n
 2 2 60 m n
 2 2 2 30 m n
 2 2 2 2 15 m n
 2 2 2 2 3 5 m n
14. 64a3b  1 2 32 a a a b
 1 2 2 16 a a a b
 1 2 2 2 8 a a a b
 1 2 2 2 2 2 2 a a a b
15. 26xy2  1 2 13 x y y
16. 231xy2z  1 3 77 x y y z
 1 3 7 11 x y y z
17. 44rs2t3  2 22 r s s t t t
 2 2 11 r s s t t t
18. 756m2n2  1 2 378 m m n n
 1 2 2 189 m m n n
 1 2 2 3 63 m m n n
 1 2 2 3 3 21 m m n n
 1 2 2 3 3 3 7 m m n n
19. Factor each monomial and circle the common
prime factors.
16  2 2 2 2
60  2 2 3 5
GCF: 2 2 or 4
20. Factor each monomial and circle the common
prime factors.
15  3 5
50  2 5 5
GCF: 5

Extra Practice

23.

24.

25.

26.

27.

28.

29.

30.

31.

718

58  2 29
GCF: 29
Factor each monomial and circle the
prime factors.
55  5 11
305  5 61
GCF: 5
Factor each monomial and circle the
prime factors.
126  2 3 3 7
252  2 2 3 3 7
GCF: 2 3 3 7 or 126
Factor each monomial and circle the
prime factors.
128  2 2 2 2 2 2 2
245  5 7 7
GCF: 1
Factor each monomial and circle the
prime factors.
7y2  7 y y
14y2  2 7 y y
GCF: 7 y y or 7y2
Factor each monomial and circle the
prime factors.
4xy  2 2 x y
6x  1 2 3 x
GCF: 2x
Factor each monomial and circle the
prime factors.
35t2  5 7 t t
7t  7 t
GCF: 7t
Factor each monomial and circle the
prime factors.
16pq2  2 2 2 2 p q q
12p2q  2 2 3 p p q
4pq  2 2 p q
GCF : 2 2 p q or 4pq
Factor each monomial and circle the
prime factors.
5 5 1
15  3 5
10  2 5
GCF: 5
Factor each monomial and circle the
prime factors.
12 mn  2 2 3 m n
10 mn  2 5 m n
15 mn  3 5 m n
GCF: mn

common

common

common

common

common

common

common

common

common

PQ249J-6481F-20-23[686-725] 26/9/02 9:45 PM Page 719 Sahuja Ahuja_QXP_06:Desktop Folder:Chandra:Algebra_FNL_Delivery:

17. d(d  11)  0


d  0 or d  11  0
d  11
{11, 0}
18. z(z  2.5)  0
z  0 or z  2.5  0
z  2.5
{0, 2.5}
19. (2y  6)(y  1)  0
2y  6  0 or y  1  0
y  3
y1
{3, 1}
20. (4n  7)(3n  2)  0
4n  7  0 or 3n  2  0

32. Factor each monomial and circle the common


prime factors.
14xy  2 7 x y
12y  2 2 3 y
20x  2 2 5 x
GCF: 2
33. Factor each monomial and circle the common
prime factors.
26jk4  2 13 j k k k k
16jk3  2 2 2 2 j k k k
8j2  2 2 2 j j
GCF: 2j

Page 840

523, 74 6

10a  40a  10a(a  4)


15wx  35wx2  5wx(3  7x)
27a2b  9b3  9b(3a2  b2 )
11x  44x2y  11x(1  4xy)
16y2  8y  8y(2y  1)
14mn2  2mn  2mn(7n  1)
25a2b2  30ab3  5ab2 (5a  6b)
2m3n2  16mn2  8mn  2mn(m2n  8n  4)
2ax  6xc  ba  3bc
 (2ax  6xc)  (ba  3bc)
 2x(a  3c)  b(a  3c)
 (2x  b)(a  3c)
10. 6mx  4m  3rx  2r
 (6mx  4m)  (3rx  2r)
 2m(3x  2)  r(3x  2)
 (2m  r)(3x  2)
11. 3ax  6bx  8b  4a
 (3ax  6bx)  (8b  4a)
 3x(a  2b)  4(2b  a)
 3x(a  2b)  4(1) (2b  a)
 3x(a  2b)  (4)(a  2b)
 (3x  4)(a  2b)
12. a2  2ab  a  2b  (a2  2ab)  (a  2b)
 a(a  2b)  1(a  2b)
 (a  1)(a  2b)
13. 8ac  2ad  4bc  bd
 (8ac  2ad)  (4bc  bd)
 2a(4c  d)  b(4c  d)
 (2a  b)(4c  d)
14. 2e2g  2fg  4e2h  4fh
 (2e2g  2fg)  (4e2h  4fh)
 2g(e2  f )  4h(e2  f )
 (2g  4h)(e2  f )
 2(g  2h)(e2  f )
2  xy  xy  y2  (x2  xy)  (xy  y2 )
15. x
 x(x  y)  y(x  y)
 x(x  y)  y(1)(x  y)
 (x  y)(x  y)
 (x  y) 2
Exercises 1627 For checks, see students work.
16. a(a  9)  0
a  0 or a  9  0
a9
{0, 9}
1.
2.
3.
4.
5.
6.
7.
8.
9.

n4

Lesson 9-2

n  3

21. (a  1)(a  1)  0
a  1  0 or a  1  0
a  1
a1
{1, 1}
22. 10x2  20x  0
10x(x  2)  0
10x  0 or x  2  0
x0
x2
{0, 2}
23. 8b2  12b  0
4b(2b  3)  0
4b  0 or 2b  3  0
3
b0
b2

50, 32 6

or {0, 1.5}

24. 14d  49d  0


7d(2d  7)  0
7d  0 or 2d  7  0
d0

d  2

572, 06 or {3.5, 0}

15a2  60a
15a2  60a  0
15a(a  4)  0
15a  0 or a  4  0
a0
a4
{0, 4}
33x2  22x
26.
2  22x  0
33x
11x(3x  2)  0
11x  0 or 3x  2  0
25.

x0

27.

523, 06

x  3

32x2  16x
32x  16x  0
16x(2x  1)  0
16x  0 or 2x  1  0
2

x0

50, 12 6
719

x2

Extra Practice

PQ249J-6481F-20-23[686-725] 26/9/02 9:45 PM Page 720 Sahuja Ahuja_QXP_06:Desktop Folder:Chandra:Algebra_FNL_Delivery:

Page 840

18. k2  27kj  90j2  k2  (27j)k  90j2

Lesson 9-3

1. Among all pairs of factors of 14, choose 7 and


2, the pair of factors whose sum is 9.
x2  9x  14  (x  7)(x  2)
2. Among all pairs of factors of 36, choose 12 and
3, the pair of factors whose sum is 9.
a2  9a  36  (a  12)(a  3)
3. Among all pairs of factors of 15, choose 5 and
3, the pair of factors whose sum is 2.
x2  2x  15  (x  5)(x  3)
4. Among all pairs of factors of 15, choose 5 and
3, the pair of factors whose sum is 8.
n2  8n  15  (n  5)(n  3)
5. Among all pairs of factors of 21, choose 21 and 1,
the pair of factors whose sum is 22.
b2  22b  21  (b  21)(b  1)
6. Among all pairs of factors of 3, choose 3 and 1,
the pair of factors whose sum is 2.
c2  2c  3  (c  3)(c  1)
7. Among all pairs of factors of 24, choose 8 and
3, the pair of factors whose sum is 5.
x2  5x  24  (x  8)(x  3)
8. Among all pairs of factors of 7, choose 7 and 1,
the pair of factors whose sum is 8.
n2  8n  7  (n  7)(n  1)
9. Among all pairs of factors of 39, choose 13 and
3, the pair of factors whose sum is 10.
m2  10m  39  (m  13)(m  3)
10. Among all pairs of factors of 36, choose 12 and 3,
the pair of factors whose sum is 15.
z2  15z  36  (z  12)(z  3)
11. s2  13st  30t2  s2  (13t)s  30t2
Among all pairs of factors of 30t2, choose 15t
and 2t, the pair of factors whose sum is 13t.
s2  13st  30t2  (s  15t) (s  2t)
12. Among all pairs of factors of 35, choose 5 and
7, the pair of factors whose sum is 2.
y2  2y  35  ( y  5)( y  7)
13. Among all pairs of factors of 40, choose 5 and
8, the pair of factors whose sum is 3.
r2  3r  40  (r  5)(r  8)
14. Among all pairs of factors of 6, choose 1 and 6,
the pair of factors whose sum is 5.
x2  5x  6  (x  6)(x  1)
15. x2  4xy  5y2  x2  (4y)x  5y2
Among all pairs of factors of 5y2, choose y and
5y, the pair of factors whose sum is 4y.
x2  4xy  5y2  (x  y)(x  5y)
16. Among all pairs of factors of 63, choose 9 and 7,
the pair of factors whose sum is 16.
r2  16r  63  (r  9) (r  7)
17. Among all pairs of factors of 52, choose 26 and
2, the pair of factors whose sum is 24.
v2  24v  52  (v  26)(v  2)

Extra Practice

Among all pairs of factors of 90j2, choose 30j


and 3j, the pair of factors whose sum is 27j.
k2  27kj  90j2  (k  30j) (k  3j)
Exercises 1933 For checks, see students work.
19.
a2  3a  4  0
(a  4)(a  1)  0
a  4  0 or a  1  0
a  4
a1
{4, 1}
20.
x2  8x  20  0
(x  2)(x  10)  0
x  2  0 or x  10  0
x  2
x  10
{2, 10}
21. b2  11b  24  0
(b  3)(b  8)  0
b  3  0 or b  8  0
b  3
b  8
{8, 3}
22.
y2  y  42  0
(y  7)(y  6)  0
y  7  0 or y  6  0
y  7
y6
{7, 6}
23. k2  2k  24  0
(k  4)(k  6)  0
k  4  0 or k  6  0
k4
k  6
{6, 4}
24. r2  13r  48  0
(r  16)(r  3)  0
r  16  0 or r  3  0
r  16
r  3
{3, 16}
25.
n2  9n  18
2
n  9n  18  0
(n  6)(n  3)  0
n  6  0 or n  3  0
n6
n3
{3, 6}
26.
2z  z2  35
2  2z  35  0
z
(z  7)(z  5)  0
z  7  0 or z  5  0
z  7
z5
{7, 5}
27.
20x  19  x2
2
x  20x  19  0
(x  1)(x  19)  0
x  1  0 or x  19  0
x1
x  19
{1, 19}
28.
10  a2  7a
2
a  7a  10  0
(a  5)(a  2)  0
a  5  0 or a  2  0
a  5
a  2
{5, 2}

720

PQ249J-6481F-20-23[686-725] 26/9/02 9:45 PM Page 721 Sahuja Ahuja_QXP_06:Desktop Folder:Chandra:Algebra_FNL_Delivery:

29.

z2  57  16z
z  16z  57  0
(z  3)(z  19)  0
z  3  0 or z  19  0
z  3
z  19
{3, 19}
x2  14x  3
2  14x  33  0
x
(x  11) (x  3)  0
x  11  0 or x  3  0
x  11
x  3
{11, 3}
22x  x2  96
2  22x  96  0
x
(x  6)(x  16)  0
x  6  0 or x  16  0
x6
x  16
{6, 16}
144  q2  26q
2  26q  144  0
q
(q  8)(q  18)  0
q  8  0 or q  18  0
q8
q  18
{8, 18}
x2  84  20x
2  20x  84  0
x
(x  6)(x  14)  0
x  6  0 or x  14  0
x6
x  14
{6, 14}

6. Among all positive pairs of factors of 24, choose 3


and 8, the pair of factors whose sum is 11.
4y2  11y  6  4y2  3y  8y  6
 y(4y  3)  2(4y  3)
 (4y  3) (y  2)
7. Among all pairs of factors of 18, choose 2 and
9, the pair of factors whose sum is 7.
6n2  7n  3  6n2  2n  9n  3
 2n(3n  1)  3(3n  1)
 (2n  3) (3n  1)
8. Among all pairs of negative factors of 70, choose
7 and 10, the pair of factors whose sum is
17.
5x2  17x  14  5x2  7x  10x  14
 x(5x  7)  (2) (5x  7)
 (5x  7) (x  2)
9. Among all pairs of negative factors of 26, there
are no pairs whose sum is 11. The trinomial is
prime.
10. Among all pairs of factors of 24, choose 2 and
12, the pair of factors whose sum is 10.
8m2  10m  3  8m2  12m  2m  3
 4m(2m  3)  1(2m  3)
 (2m  3) (4m  1)
11. Among all pairs of factors of 12, there are no
pairs whose sum is 2.
6y2  2y  2  2(3y2  y  1)
12. Among all pairs of factors of 28, choose 4 and
7, the pair of factors whose sum is 3.
2r2  3r  14  2r2  4r  7r  14
 2r(r  2)  7(r  2)
 (2r  7) (r  2)
13. Among all pairs of negative factors of 75, there
are no pairs whose sum is 3. The trinomial is
prime.
14. Factor out the GCF, 6.
18v2  24v  12  6(3v2  4v  2)
Now factor 3v2  4v  2. Among all pairs of
positive factors of 6, there are no pairs whose sum
is 4.
Thus, 18v2  24v  12  6(3v2  4v  2) .
15. Factor out the GCF, 2.
4k2  2k  12  2(2k2  k  6)
Now factor 2k2  k  6. Among all pairs of factors
of 12, choose 3, and 4, the pair of factors
whose sum is 1.
(2k2  k  6)  2k2  3k  4k  6
 k(2k  3)  2(2k  3)
 (2k  3) (k  2)
Thus, 4k2  2k  12  2(2k  3) (k  2) .

30.

31.

32.

33.

Page 840

Lesson 9-4

1. Among all pairs of factors of 252, choose 14


and 18, the pair of factors whose sum is 4.
4a2  4a  63  4a2  14a  18a  63
 2a(2a  7)  9(2a  7)
 (2a  7) (2a  9)
2. Among all pairs of factors of 18, choose 9 and
2, the pair of factors whose sum is 7.
3x2  7x  6  3x2  9x  2x  6
 3x(x  3)  2(x  3)
 (3x  2)(x  3)
3. Among all pairs of negative factors of 24, choose
1 and 24, the pair of factors whose sum is
25.
4r2  25r  6  4r2  r  24r  6
 r(4r  1)  (6)(4r  1)
 (4r  1)(r  6)
4. Among all pairs of negative factors of 30, choose
5 and 6, the pair of factors whose sum is 11.
2z2  11z  15  2z2  5z  6z  15
 z(2z  5)  (3) (2z  5)
 (2z  5)(z  3)
5. Among all pairs of factors of 63, choose 7 and
9, the pair of factors whose sum is 2.
3a2  2a  21  3a2  9a  7a  21
 3a(a  3)  7(a  3)
 (3a  7) (a  3)

721

Extra Practice

PQ249J-6481F-20-23[686-725] 26/9/02 9:45 PM Page 722 Sahuja Ahuja_QXP_06:Desktop Folder:Chandra:Algebra_FNL_Delivery:

16. Factor out the GCF, 10.


10x2  20xy  10y2  10(x2  2xy  y2 )
Now factor x2  2xy  y2. Among all pairs of
factors of y2, choose y and y, the pair of factors
whose sum is 2y.
10(x2  2xy  y2 )  10[x2  yx  yx  y2 ]
 10[x(x  y)  y(x  y) ]
 10[ (x  y)(x  y) ]
 10(x  y) 2
2  20xy  10y2  10(x  y)(x  y)
Thus, 10x
or 10(x  y)2.
17. Among all pairs of factors of 60, choose 4 and
15, the pair of factors whose sum is 11.
12c2  11cd  5d2  12c2  15cd  4cd  5d2
 3c(4c  5d)  d(4c  5d)
 (3c  d)(4c  5d)
18. Among all pairs of factors of 30, choose 5 and
6, the pair of factors whose sum is 1.
30n2  mn  m2  30n2  5mn  6mn  m2
 5n(6n  m)  m(6n  m)
 (5n  m)(6n  m)
Exercises 1933 For checks, see students work.
19. 8t2  32t  24  0
8(t2  4t  3)  0
t2  4t  3  0
(t  3)(t  1)  0
t  3  0 or t  1  0
t  3
t  1
{3, 1}
20. 6y2  72y  192  0
6(y2  12y  32)  0
y2  12y  32  0
(y  4)(y  8)  0
y  4  0 or y  8  0
y  4
y  8
{8, 4}
21.
5x2  3x  2  0
(5x  2)(x  1)  0
5x  2  0 or x  1  0
2

x5

51, 6

24.

n2

25.

516, 32 6

1
6

n

12x2  x  35  0
 21x)  (20x  35)  0
3x(4x  7)  5(4x  7)  0
(4x  7)(3x  5)  0
4x  7  0 or 3x  5  0
(12x2

x4

26.

553, 74 6

x  3

18x2  36x  14  0
2(9x2  18x  7)  0
9x2  18x  7  0
9x2  21x  3x  7  0
(9x2  21x)  (3x  7)  0
3x(3x  7)  1(3x  7)  0
(3x  7)(3x  1)  0
3x  7  0 or 3x  1  0
7

x  3

27.

573, 13 6

x3

15a2  a  2  0
 5a)  (6a  2)  0
5a(3a  1)  2(3a  1)  0
(3a  1)(5a  2)  0
3a  1  0 or 5a  2  0
(15a2

a3

28.

x  1

2
5

22. 9x2  18x  27  0


9(x2  2x  3)  0
x2  2x  3  0
(x  1)(x  3)  0
x  1  0 or x  3  0
x1
x  3
{3, 1}
23. 4x2  4x  4  4
4x2  4x  8  0
4(x2  x  2)  0
x2  x  2  0
(x  1)(x  2)  0
x  1  0 or x  2  0
x  1
x2
{1, 2}

Extra Practice

12n2  16n  3  0
 18n)  (2n  3)  0
6n(2n  3)  1(2n  3)  0
(2n  3)(6n  1)  0
2n  3  0 or 6n  1  0
(12n2

525, 13 6

a  5

14b2  7b  42  0
7(2b2  b  6)  0
2b2  b  6  0
2b2  4b  3b  6  0
2b(b  2)  3(b  2)  0
(2b  3)(b  2)  0
2b  3  0 or b  2  0
3

b2

29.

52, 32 6

b  2

13r2  21r  10  0
(13r  5r)  (26r  10)  0
r(13r  5)  2(13r  5)  0
(13r  5)(r  2)  0
13r  5  0 or r  2  0
2

r  13

52, 6
5
13

722

r  2

PQ249J-6481F-20-23[686-725] 26/9/02 9:45 PM Page 723 Sahuja Ahuja_QXP_06:Desktop Folder:Chandra:Algebra_FNL_Delivery:

30.

31.

35y2  60y  20  0
5(7y2  12y  4)  0
7y2  12y  4  0
2
7y  14y  2y  4  0
7y(y  2)  2(y  2)  0
(7y  2)(y  2)  0
7y  2  0 or y  2  0

y  7
2
7,

y2

13. 9x2  100y2  (3x) 2  (10y) 2


 (3x  10y) (3x  10y)

16x2  4x  6  0
2(8x2  2x  3)  0
8x2  2x  3  0
2
8x  6x  4x  3  0
2x(4x  3)  1(4x  3)  0
(2x  1) (4x  3)  0
2x  1  0 or 4x  3  0
1

x  2

32.

9. 75r2  48  3(25r2  16)


 3[ (5r) 2  (4) 2 ]
 3(5r  4) (5r  4)
10. x2  36y2  (x) 2  (6y) 2
 (x  6y) (x  6y)
11. 3a2  16 prime
12. 12t2  75  3(4t2  25)
 3[ (2t) 2  (5) 2 ]
 3(2t  5) (2t  5)

512, 34 6

14. 49  a2b2  (7) 2  (ab) 2


 (7  ab) (7  ab)
15. 5a2  48 prime
16. 169  16t2  (13) 2  (4t) 2
 (13  4t) (13  4t)
17. 8r2  4  4(2r2  1)

x4

18. 45m2  5  5(9m2  1)


 5[ (3m) 2  (1) 2 ]
 5(3m  1) (3m  1)
Exercises 1933 For checks, see students work.
4x2  16
19.
2  16  0
4x
4(x2  4)  0
x2  4  0
2  (2) 2  0
x
4(x  2)(x  2)  0
or x  2  0
x20
x  2
x2
{ 2}
2x2  50
20.
2  50  0
2x
2(x2  25)  0
x2  25  0
x2  52  0
(x  5)(x  5)  0
x  5  0 or x  5  0
x5
x  5
{ 5}
9n2  4  0
21.
(3n) 2  (2) 2  0
(3n  2)(3n  2)  0
3n  2  0 or 3n  2  0
3n  2
3n  2

28d2  5d  3  0
28d  12d  7d  3  0
4d(7d  3)  1(7d  3)  0
(4d  1)(7d  3)  0
4d  1  0 or 7d  3  0
2

d4

33.

537, 14 6

d  7

30x2  9x  3  0
3(10x2  3x  1)  0
10x2  3x  1  0
2  5x  2x  1  0
10x
5x(2x  1)  1(2x  1)  0
(5x  1) (2x  1)  0
5x  1  0 or 2x  1  0
1

x  5

515, 12 6
Page 841

x2

Lesson 9-5

1. x2  9  x2  32
 (x  3)(x  3)
2. a2  64  a2  82
 (a  8)(a  8)
3. 4x2  9y2  (2x) 2  (3y) 2
 (2x  3y) (2x  3y)
4. 1  9z2  12  (3z) 2
 (1  3z)(1  3z)
5. 16a2  9b2  (4a) 2  (3b) 2
 (4a  3b) (4a  3b)
6. 8x2  12y2  4(2x2  3y2 )
7. a2  4b2  (a) 2  (2b) 2
 (a  2b)(a  2b)
8. x2  y2  (x  y)(x  y)

22.

n3

5 23 6

n  3
25

a2  36  0

156 22  0
1a  56 21a  56 2  0
a2 

a60
5

a6

5 56 6
723

or

a60
a

5
6

Extra Practice

PQ249J-6481F-20-23[686-725] 26/9/02 9:45 PM Page 724 Sahuja Ahuja_QXP_06:Desktop Folder:Chandra:Algebra_FNL_Delivery:

23.

16
9
4 2
3

1 2  b2  0
143  b2143  b2  0

4
3

b0
b

24.

5 43 6

72  2z2  0
2(36  z2 )  0
36  z2  0
(6) 2  z2  0
(6  z)(6  z)  0
6  z  0 or 6  z  0
z6
z  6
{ 6}
30.
25a2  1
2
25a  1  0

 b2  0

4
3

or

29.

b0

4
3

b  3
1

18  2x2  0

1
(36
2

1
 25
1 2
a2  5

36  x2  0
(6) 2  x2  0
(6  x)(6  x)  0
6  x  0 or 6  x  0
x6
x  6
{ 6}
25.
20  5g2  0
5(4  g2 )  0
4  g2  0
(2) 2  g2  0
(2  g)(2  g)  0
2  g  0 or 2  g  0
g2
g  2
{ 2}

a50

 p2 )  0

1 2
c
4

1
4

90

1c2  169 2  0
16
9
4 2
3

c2 

c2

12

0
0

1c  43 21c  43 2  0
4

c30

or

c  3

28.

5 43 6

or

5 15 6

a50

a  5

c30

Page 841

c3

Lesson 9-6

1. Yes;
The first term is a perfect square: x2
The last term is a perfect square: 62
The middle term is 2(x)(6).
x2  12x  36  x2  2(x) (6)  62
 (x  6) 2
2. No; the middle term is not 2(n)(6).
3. Yes;
The first term is a perfect square: a2
The last term is a perfect square: 22
The middle term is 2(a)(2).
a2  4a  4  a2  2(a) (2)  22
 (a  2) 2

3z2  48  0
3(z2  16)  0
z2  16  0
2
z  (4) 2  0
(z  4)(z  4)  0
z  4  0 or z  4  0
z4
z  4
{ 4}

Extra Practice

0

2q3  2q  0
2q(q2  1)  0
2q(q  1)(q  1)  0
or q  1  0
2q  0 or q  1  0
q0
q  1
q1
{1, 0, 1}
32.
3r3  48r
3  48r  0
3r
3r(r2  16)  0
3r [ r2  (4) 2 ]  0
3r(r  4)(r  4)  0
3r  0 or r  4  0 or r  4  0
r0
r4
r  4
{4, 0, 4}
33.
100d  4d3  0
4d(25  d2 )  0
4d [ (5) 2  d2 ]  0
4d(5  d)(5  d)  0
4d  0 or 5  d  0 or 5  d  0
d0
d5
d  5
{5, 0, 5}
31.

64  p2  0
(8) 2  p2  0
(8  p)(8  p)  0
8  p  0 or 8  p  0
p8
p  8
{ 8}
27.

12

0

a5

1
(64
4

a2

1a  15 21a  15 2  0

16  4p2  0

26.

1
25 a2  25  0

 x2 )  0

724

PQ249J-6481F-20-23[686-725] 26/9/02 9:46 PM Page 725 Sahuja Ahuja_QXP_06:Desktop Folder:Chandra:Algebra_FNL_Delivery:

4. No; the last term, 100, is not a perfect square.


5. No; the first term, 2n2, is not a perfect square.
6. Yes;
The first term is a perfect square: (2a)2
The last term is a perfect square: 52
The middle term is 2(2a)(5).
4a2  20a  25  (2a) 2  2(2a)(5)  52
 (2a  5) 2
2
2
7. 3x  75  3(x  25)
 3(x  5)(x  5)
8. n2  8n  16  (n) 2  2(4)(n)  (4) 2
 (n  4) 2
9. 4p2  12pr  9r2
 (2p) 2  2(2p)(3r)  (3r) 2
 (2p  3r) 2
10. 6a2  72  6(a2  12)
11. s2  30s  225
 (s) 2  2(15)(s)  (15) 2
 (s  15) 2
12. 24x2  24x  9  3(8x2  8x  3)
13. 1  10z  25z2
 (1) 2  2(1)(5z)  (5z) 2
 (1  5z) 2
14. 28  63b2  7(4  9b2 )
 7(2  3b)(2  3b)
2
15. 4c  2c  7 is prime.
Exercises 1621 For checks, see students work.
x2  22x  121  0
16.
2
x  2(x)(11)  (11) 2  0
(x  11) 2  0
x  11  0
x  11
{11}
17. 343d2  7

20.

21.

594 6

573 6

p3

9p2  42p  20  29


9p2  42p  49  0
2  2(3p) (7)  72  0
(3p)
(3p  7) 2  0
3p  7  0
3p  7
7

Lesson 10-1

1. y  x2  6x  8
Sample answer:
x
0
1
2
3
4
5
6

y
8
3
0
1
0
3
8

y  x 2  6x  8

2. y  x2  3x
Sample answer:

d2  49

5 17 6

s4

Page 841

d2  343
d

16s2  81  72s
16s  72s  81  0
(4s) 2  2(4s) (9)  92  0
(4s  9) 2  0
4s  9  0
4s  9
2

x
1
0
1
2
3
4
5

1
7

18. (a  7)  5
a  7  15
a  7 15
{7 15}
c2  10c  36  11
19.
c2  10c  25  0
c2  2(c)(5)  52  0
(c  5) 2  0
c50
c  5
{5}

725

y
4
0
2
2
0
4
10

y
y  x 2  3x
O

Extra Practice

PQ249J-6481F-24-26[726-749] 26/9/02 9:02 PM Page 726 Sahuja Ahuja_QXP_06:Desktop Folder:Chandra:Algebra_FNL_Delivery:

3. y  x2
Sample answer:
x
3
2
1
0
1
2
3

7. In y  x2  2x  3, a  1 and b  2.
x

y
9
4
1
0
1
4
9

b
2a
(2)
2(1)

or 1 is the equation of the axis of


x
symmetry.
y  (1) 2  2(1)  3
y  1  2  3
y  2
The vertex is at (1, 2).
Since the coefficient of the x2 term is negative, the
vertex is a maximum.

x
y  x 2

y
O

y  x 2  2x  3

4. y  x2  x  3
Sample answer:
x
3
2
1
0
1
2
3

y
9
5
3
3
5
9
15

8. In y  3x2  24x  80, a  3 and b  24.


x
y  x2  x  3

5. y  x2  1
Sample answer:
y
10
5
2
1
2
5
10

Extra Practice

y
40
25
16
13
16
25
40

140
120
100
80
60
40
20

y  x2  1
x

y  3x 2  24x  80
8

6. y  3x2  6x  16
Sample answer:
x
4
3
2
1
0
1
2

b
2a
(24)
2(3)

or 4 is the equation of the axis of


x
symmetry.
y  3(4) 2  24(4)  80
y  48  96  80
y  32
The vertex is at (4, 32).
Since the coefficient of the x2 term is positive, the
vertex is a minimum.

x
3
2
1
0
1
2
3

y
28
20
12 y  3x 2  6x  16
4
2

726

6

4

2

2x

PQ249J-6481F-24-26[726-749] 26/9/02 9:02 PM Page 727 Sahuja Ahuja_QXP_06:Desktop Folder:Chandra:Algebra_FNL_Delivery:

9. In y  x2  4x  4, a  1 and b  4.
x

Since the coefficient of the x2 term is positive, the


vertex is a minimum.

b
2a
(4)
2(1)

x
or 2 is the equation of the axis of
symmetry.
y  (2) 2  4(2)4
y484
y  8
The vertex is at (2, 8).
Since the coefficient of the x2 term is positive, the
vertex is a minimum.

y  3x 2  6x  3
x

12. In y  2x2  12x, a  2 and b  12.


x
x

b
2a
(12)
2(2)

x
or 3 is the equation of the axis of
symmetry.
y  2(3) 2  12(3)
y  18  36
y  18
The vertex is at (3, 18).
Since the coefficient of the x2 term is positive, the
vertex is a minimum.

y  x 2  4x  4

10. In y  5x2  20x  37, a  5 and b  20.


x

12
8
4

b
2a
(20)
2(5)

x
or 2 is the equation of the axis of
symmetry.
y  5(2) 2  20(2)  37
y  20  40  37
y  17
The vertex is at (2, 17).
Since the coefficient of the x2 term is positive, the
vertex is a minimum.
70
60
50
40
30
20
10
2

6

y  2x 2  12x

x

2x

b
2a
(6)
2(1)

x
or 3 is the equation of the axis of
symmetry.
y  (3) 2  6(3)  5
y  9  18  5
y  4
The vertex is at (3, 4).
Since the coefficient of the x2 term is positive, the
vertex is a minimum.

y  5x 2  20x  37
4x

11. In y  3x2  6x  3, a  3 and b  6.


x

2 4 O
8
12
16
20

13. In y  x2  6x  5, a  1 and b  6.

4

b
2a
(6)
2(3)

or 1 is the equation of the axis of


x
symmetry.
y  3(1) 2  6(1)  3
y363
y0
The vertex is at (1, 0).

y  x 2  6x  5

727

Extra Practice

PQ249J-6481F-24-26[726-749] 26/9/02 9:02 PM Page 728 Sahuja Ahuja_QXP_06:Desktop Folder:Chandra:Algebra_FNL_Delivery:

14. In y  x2  6x  9, a  1 and b  6.
x

Since the coefficient of the x2 term is positive, the


vertex is a minimum.

b
2a
(6)
2(1)

x
or 3 is the equation of the axis of
symmetry.
y  (3) 2  6(3)  9
y  9  18  9
y0
The vertex is at (3, 0).
Since the coefficient of the x2 term is positive, the
vertex is a minimum.

x
y  4x 2  1

17. In y  2x2  2x  4, a  2 and b  2.


x
x

b
2a
(2)
2(2)

or 2
1

The equation of the axis of symmetry is x  2.

1 2

1 2

1
1
y  2 2 2  2 2  4

y

y  x 2  6x  9

y

15. In y  x2  16x  15, a  1 and b  16.


x

y
y  2x 2  2x  4

2
y y  x  16x  15

18. In y  6x2  12x  4, a  6 and b  12.


x
4

12

b
2a
(12)
2(6)

x
or 1 is the equation of the axis of
symmetry.
y  6(1) 2  12(1)  4
y  6  12  4
y  10
The vertex is at (1, 10).
Since the coefficient of the x2 term is positive, the
vertex is a minimum.

16. In y  4x2  1, a  4 and b  0.


x

Since the coefficient of the x2 term is negative, the


vertex is a maximum.

x
or 8 is the equation of the axis of
symmetry.
y  (8) 2  16(8)  15
y  64  128  15
y  49
The vertex is at (8, 49).
Since the coefficient of the x2 term is negative, the
vertex is a maximum.

O
10
20
30

14

The vertex is at 2, 42 .

b
2a
(16)
2(1)

50
40
30
20
10

1
2
1
42

b
2a
(0)
2(4)

x
or 0 is the equation of the axis of
symmetry.
y  4(0) 2  1
y01
y  1
The vertex is at (0, 1).

y
O

y  6x 2  12x  4

Extra Practice

728

PQ249J-6481F-24-26[726-749] 26/9/02 9:02 PM Page 729 Sahuja Ahuja_QXP_06:Desktop Folder:Chandra:Algebra_FNL_Delivery:

19. In y  x2  1, a  1 and b  0.


x

Since the coefficient of the x2 term is negative, the


vertex is a maximum.

b
2a
(0)
2(1)

x
or 0 is the equation of the axis of
symmetry.
y  (0) 2  1
y01
y  1
The vertex is at (0, 1).
Since the coefficient of the x2 term is negative, the
vertex is a maximum.

y  5x 2  3x  2
2
1
1

1x

O
1

22. In y  x2  x  20, a  1 and b  1.

x
y  x 2  1

x

b
2a
(1)
2(1)

or 2 is the equation of the axis of


x
symmetry.
y

112 22  112 2  20

y  4  2  20
1

20. In y 
x

x2

b
2a
(1)
2(1)

y  204

 x  1, a  1 and b  1.

The vertex is at

Since the coefficient of the x2 term is negative, the


vertex is a maximum.

or 2 is the equation of the axis of


x
symmetry.
y

112 22  12  1

20
15
10
5

y  4  2  1
1

y  14
The vertex is at

112, 114 2.

4

Since the coefficient of the x2 term is negative, the


vertex is a maximum.
y

2

y  x 2  x  20

23. In y  2x2  5x  2, a  2 and b  5.


x

y  x 2  x  1

b
2a
(5)
2(2)

or 1.25 is the equation of the axis of


x
symmetry.
y  2(1.25) 2  5(1.25)  2
y  3.125  6.25  2
y  5.125
The vertex is at (1.25, 5.125).
Since the coefficient of the x2 term is positive, the
vertex is a minimum.

21. In y  5x2  3x  2, a  5 and b  3.


x

112, 2014 2.

3
2
1

b
2a
(3)
2(5)

x
or 0.3 is the equation of the axis of
symmetry.
y  5(0.3) 2  3(0.3)  2
y  .45  0.9  2
y  2.45
The vertex is at (0.3, 2.45).

3

729

2

1

1x
1
2
3
y  2x 2  5x  2
4
5

Extra Practice

PQ249J-6481F-24-26[726-749] 26/9/02 9:02 PM Page 730 Sahuja Ahuja_QXP_06:Desktop Folder:Chandra:Algebra_FNL_Delivery:

24. In y  3x2  18x 15, a  3 and b  18.


x

3. Graph d2  36  0.
Sample answer:

b
2a
(18)
2(3)

d
2
1
0
1
2

x
or 3 is the equation of the axis of
symmetry.
y  3(3) 2  18(3)  15
y  27  54  15
y  12
The vertex is at (3, 12).
Since the coefficient of the x2 term is negative, the
vertex is a maximum.
y  3x 2  18x  15

6

4

2

15
10
5

5
10
20
25
30

f (d )
60
50
40
30
20 f (d )  d 2  36
10
O
4321
1 2 3 4d

The graph has no d-intercept. Thus there are no


real number solutions for this equation.
4. Graph b2  18b  81  0.
Sample answer:

b
7
8
9
10
11

2x

f(d)
40
37
36
37
40

f(b)
4
1
0
1
4

f (b )  b 2  18b  81
5
4
3
2
1
O

Page 842

Lesson 10-2

f(a)
0
16
24
25
24
16
0

15
10
5
5

5
10
15
20
25

f (a )

x
3
2
1
0

5a

f(n)
0
12
15
16
15
12
0

16
12
8
4
2
4
8
12
16

6

4

2

f (x )
40
35
30
25
20 f (x )  x 2  3x  27
15
10
5
x
O
2

The graph has no x-intercept. Thus there are no


real number solutions for this equation.
6. Graph y2  3y  10  0.
Sample answer:

f (n)

y
7
5
3
2
1
0
2

O
2 4 6 8 10 n

f (n )  n 2  8n

The n-intercepts of the graph are 0 and 8. Thus,


the solutions of the equation are 0 and 8.

Extra Practice

f(x)
27
25
25
27

f (a )  a 2  25

The a-intercepts of the graph are 5 and 5. Thus,


the solutions of the equation are 5 and 5.
2. Graph n2  8n  0.
Sample answer:
n
0
2
3
4
5
6
8

b
2 4 6 8 10 12

The graph has one d-intercept, 9. Thus the


solution of the equation is 9.
5. Graph x2  3x  27  0.
Sample answer:

1. Graph a2  25  0.
Sample answer:
a
5
3
1
0
1
3
5

f (b )

f(y)
18
0
10
12
12
10
0

f (y )  y 2  3y  10
f (y )
12
10
8
6
4
2
54321O

1 2y

The y-intercepts of the graph are 5 and 2. Thus,


the solutions of the equation are 5 and 2.

730

PQ249J-6481F-24-26[726-749] 26/9/02 9:02 PM Page 731 Sahuja Ahuja_QXP_06:Desktop Folder:Chandra:Algebra_FNL_Delivery:

7. Graph x2  2x  3  0.
Sample answer:
x
5
3
2
1
0
1
3

11. Graph 3x2  6x  9  0.


Sample answer:

f(x)
12
0
3
4
3
0
12

x
1
0
1
2
3

f (x )

f (x )  x 2  2x  3

f(x)
21
5
3
4
3
5
21

c
3
2
1
0
1
2
4

2
( )
f (x ) f x  x  6x  5

f(a)
21
5
3
4
3
5
21

t
2
1
0
1
2

f (a )
f (a )  a 2  2a  3

f(r)
15
5
1
3
1
5
15

f (c )

2
1
2

1

1c

f (c )  c 2  c 1

f(t)
14
5
2
5
14

f (t )

f (t )  3t 2  2
O

The graph has no t-intercept. Thus, there are no


real number solutions for this equation.
14. Graph b2  5b  2  0.
Sample answer:
b
1
0
1
2
3
4
5
6

f (r )

The a-intercepts of the graph are 3 and 1. So,


the solutions of the equation are 3 and 1.
10. Graph 2r2  8r  5  0.
Sample answer:
r
1
0
1
2
3
4
5

f(c)
6
2
0
0
2
6
20

The c-intercepts of the graph are 0 and 1.


So, the solutions of the equation are 0 and 1.
13. Graph 3t2  2  0.
Sample answer:

The x-intercepts of the graph are 1 and 5. So, the


solutions of the equation are 1 and 5.
9. Graph a2  2a  3  0.
Sample answer:
a
6
4
2
1
0
2
4

O f (x )
21
1 2 3x
2
4 f (x )  3x 2  6x  9
6
8
10
12
14
16

The graph has no x-intercept. Thus, there are no


real number solutions for this equation.
12. Graph c2  c  0.
Sample answer:

The x-intercepts of the graph are 3 and 1.


So, the solutions of the equation are 3 and 1.
8. Graph x2  6x  5  0.
Sample answer:
x
2
0
2
3
4
6
8

f(x)
18
9
6
9
18

f(b)
4
2
6
8
8
6
2
4

8
6
4
2
1
2
4
6
8

f (b )

f (b )  b 2  5b  2

O
1 2 3 4 5 6b

The b-intercepts of the graph are between 1 and


0 and between 5 and 6. So, one root is between
1 and 0, and the other is between 5 and 6.

f (r )  2r 2  8r  5

The r-intercepts of the graph are between 0 and 1


and between 3 and 4. So, one root is between 0
and 1 and the other is between 3 and 4.

731

Extra Practice

PQ249J-6481F-24-26[726-749] 26/9/02 9:03 PM Page 732 Sahuja Ahuja_QXP_06:Desktop Folder:Chandra:Algebra_FNL_Delivery:

15. Graph 3x2  7x  1.


x
4
3
2
1
0
1
2

f(x)
19
5
3
5
1
9
25

19. Graph a2  12a  36  0.


Sample answer:

f (x )

a
8
7
6
5
4

f (x )  3x 2  7x  1

f(x)
0
18
28
30
30
28
18
0

f(n)
8
1
4
7
8
7
4
1

O f (x )
8

4

x
3
2
1
0
1
2
3

4x

5
10
15
20
25
30
f (x )  x 2  5x  24

f (n )  8  n 2

f(x)
0
24
30
30.25
30
24
0

10
5
2
5
10
15
20
25
30

x
3
2
1
0
1
2
3
n

z
4
3
2
1
0
1
2

O a

f(x)
55
60
63
64
63
60
55

f (x )  64  x 2

60
50
40
30
20
10

f (x )

8642O

2 4 6 8x

20

f(x)
41
19
5
1
1
11
29

f (x )  4x 2  2x  1
1
1

f (x )

1
2

f(z)
47
20
3
4
1
12
35

f (z )
4
3
2
1
2

1

1
2
3
4
f (z )  5z 2  8z  1

1z

The z-intercepts of the graph are between 2 and


1 and between 0 and 1. So, one root is between
2 and 1, and the other is between 0 and 1.

f (x )  x 2  7x  18

The x-intercepts of the graph are 2 and 9.


So, the solutions of the equation are 2 and 9.
Extra Practice

2

The x-intercepts of the graph are between 1 and


0 and between 0 and 1. So, one root is between
1 and 0, and the other is between 0 and 1.
22. Graph 5z2  8z  1.
Sample answer:

f (x )
O
2 4 6 8

4

The x-intercepts of the graph are 8 and 8. So,


the solutions of the equation are 8 and 8.
21. Graph 4x2  2x  1.
Sample answer:

f (n )

6

The graph has only one a-intercept, 6, So, the


solution of the equation is 6.
20. Graph 64  x2  0.
Sample answer:

The n-intercepts of the graph are between 3 and


2 and between 2 and 3. So, one root is between
3 and 2, and the other is between 2 and 3.
18. Graph x2  7x  18.
Sample answer:
x
2
1
3
3.5
4
6
9

6
5
4
3
2
1

f (a )  a 2  12a  36

The x-intercepts of the graph are 8 and 3.


So, the solutions of the equation are 8 and 3.
17. Graph 8  n2  0.
Sample answer:
n
4
3
2
1
0
1
2
3

f (a )

8

The x-intercepts of the graph are between 3 and


2 and between 0 and 1. So, one root is between
3 and 4 and the other is between 0 and 1.
16. Graph x2  5x  24  0.
Sample answer:
x
8
6
4
3
2
1
1
3

f(a)
4
1
0
1
4

732

PQ249J-6481F-24-26[726-749] 26/9/02 9:03 PM Page 733 Sahuja Ahuja_QXP_06:Desktop Folder:Chandra:Algebra_FNL_Delivery:

3. b2  10b  25  11
(b  5) 2  11
2(b  5) 2  111
0b  5 0  111
b  5  111
b  5  5  111  5
b  5  111
b  5  111 or b  5  111
 1.7
 8.3
The solution set is {8.3, 1.7}.
4. a2  22a  121  3
(a  11) 2  3
2(a  11) 2  13
0a  11 0  13
a  11  13
a  11  11  13  11
a  11  13
a  11  13 or a  11  13
 9.3
 12.7
The solution set is {9.3, 12.7}.
5. x2  2x  1  81
(x  1) 2  81
2(x  1) 2  181
0x  1 0  181
x  1  181
x  1  1  181  1
x  1  181
x  1  181 or x  1  181
8
 10
The solution set is {10, 8}.
6. t2  36t  324  85
(t  18) 2  85
2(t  18) 2  185
0t  18 0  185
t  18  185
t  18  18  185  18
t  18  185
t  18  185 or t  18  185
 27.2
 8.8
The solution set is {8.8, 27.2}.
7. a2  20a  c

23. Graph p  27  p2.


Sample answer:
p
7
6
5
4
3
1
0
1
2
3
4
5
6

f(p)
15
3
7
15
21
27
27
25
21
15
7
3
15

15
10
5

f (p )

8 642
5
10
15
20
25

2 4 6p

f (p )  p 2  p  27

The p-intercepts of the graph are between 6 and


5 and between 4 and 5. So, one root is between
6 and 5, and the other is between 4 and 5.
24. Graph 6w  15  3w2.
Sample answer:
x
3
2
1
0
1

f(x)
24
15
12
15
24

f (w )
21
18
15
12
9
f (w )  3w 2  6w  15
6
3
321O

1w

The graph has no w-intercept. Thus, there are no


real number solutions for this equation.

Page 842

Lesson 10-3

1. x2  4x  4  9
(x  2) 2  9
2(x  2) 2  19
0x  2 0  19
x  2  19
x  2  2  19  2
x  2  19
x  2  19 or x  2  19
5
 1
The solution set is {1, 5}.
2. t2  6t  9  16
(t  3) 2  16
2(t  3) 2  116
0t  3 0  116
t  3  116
t  3  3  116  3
t  3  116
t  3  116 or t  3  116
 1
7
The solution set is {1, 7}.

Find
c

1
of
2
20 2
2

1 2

20 and square the result.

 102 or 100
8. x  10x  c
2

Find
c

1
of
2
10 2
2

1 2

10 and square the result.

 52 or 25
9. t  12t  c
2

Find
c

1
of
2
12 2
2

1 2

12 and square the result.

 62 or 36

733

Extra Practice

PQ249J-6481F-24-26[726-749] 26/9/02 9:03 PM Page 734 Sahuja Ahuja_QXP_06:Desktop Folder:Chandra:Algebra_FNL_Delivery:

10. y2  9y  c
Find

1
2

16.
2y2

of 9 and square the result.

1 9 22

2y2  7y  4  0
 7y  4  4  0  4
2y2  7y
2
7y
y2  2
7y
49
y2  2  16
7
y42
7
y4
7
7
y44

c  2


81
4

or 20.25

11. p2  14p  c
1
of 14
2
14 2
2
196
or 49
4

Find
c


1 2

and square the result.

1
of 13 and
2
13 2
2
169
or 42.25
4

c

13.

1 2

2
49

 2  16

81

 16
9

 4

7
9
 4
4
7  9
y
4
7  9
7  9
y 4
or y  4
2
1
16
 4 or 2
  4 or
1
The solution set is 4, 2 .

12. b2  13b  c
Find

2

square the result.

a2  8a  84  0
2  8a  84  84  0  84
a
a2  8a  84
2  8a  16  84  16
a
(a  4) 2  100
a  4  10
a  4  4  4  10
a  4  10
a  4  10 or a  4  10
 14
 6
The solution set is {6, 14}.

17. t2  3t 

1t

9
4

3 2
2
3
t2
3
3
2
2

 40 

9
4

169
4
13
 2
3
13
t
 2  2
3  13
t
2
3  13
3  13

or
t
2
2

t5
t  8
The solution set is {8, 5}.
x2  8x  9  0
18.
2
x  8x  9  9  0  9
x2  8x  9
2
x  8x  16  9  16
(x  4) 2  25
x  4  5
x  4  4  4  5
x  4  5
x  4  5 or x  4  5
1
 9
The solution set is {9, 1}.

c2  6  5c
c2  6  6  5c  5c  5c  6
c2  5c  6
c2  5c  6.25  6  6.25
(c  2.5) 2  0.25
c  2.5  0.5
c  2.5  2.5  2.5  0.5
c  2.5  0.5
c  2.5  0.5 or c  2.5  0.5
 2
 3
The solution set is {3, 2}.
15.
p2  8p  5  0
2  8p  5  5  0  5
p
p2  8p  5
2  8p  16  5  16
p
( p  4) 2  11
p  4  111
p  4  4  4  111
p  4  111
p  4  111 or p  4  111
 0.7
 7.3
The approximate solution set is {0.7, 7.3}.

14.

19.

y2  5y  84  0
y  5y  84  84  0  84
y2  5y  84
2

25
4
5 2
2
5
y2
5
5
2
2

y2  5y 

1y

y
y

5  19
2

or

 84 

25
4

361
4
19
 2
5
19
 2  2
5  19
y
2
5  19
y
2

y  12
y7
The solution set is {12, 7}.

Extra Practice

4

734

PQ249J-6481F-24-26[726-749] 26/9/02 9:03 PM Page 735 Sahuja Ahuja_QXP_06:Desktop Folder:Chandra:Algebra_FNL_Delivery:

t2  12t  32  0
t  12t  32  32  0  32
t2  12t  32
2  12t  36  32  36
t
(t  6) 2  4
t  6  2
t  6  6  6  2
t  6  2
t  6  2 or t  6  2
 4
 8
The solution set is {8, 4}.
2x  3x2  8
21.
3x2  2x  8

20.

23.

3x2  2x
3
2x
x2  3
2x
4
x2  3  36
2 2
x6
2
x6
2
2
x66

z

 3  36


3

2
25

 2  16
57

 16



 157
4
5  157
4
5  157
4

z

or

5  257
4

8
1

08
1

8
1

t

02

3  211
4

11

 16



or

 1.6

2

 8  16

t

 111
4
3  111
4
3  111
4

t

3  211
4

 0.08

The approximate solution set is {1.6, 0.08}.

 2  16
73

 16

Page 842

173
4
1
173

4
4
1  173
4




y
y

5  257
4

8t2  12t  1
8
12t
1
1
2
t  8 88
3t
t2  2
3t
9
t2  2  16
3
t42
3
t4
3
3
t44

2

02

 3.1
 0.6
The approximate solution set is {0.6, 3.1}.
24. 8t2  12t  1  0

8
8

2

z

3

2y2  y  9
2
y
9
9
y2  2  2  2
y
y2  2
y
1
y2  2  16
1
y42
1
y4
1
1
y44

1  273
4

8

2y2  y  9  0

2z2  5z  4
2
5z
z2  2  2  2
5z
z2  2
5z
25
z2  2  16
5
z42
5
z4
5
5
z44

 3

100
36
10
 6
2
10
6  6
2  10
x 6
2  10
2  10
x 6
or x  6
8
2
 6 or
4
The solution set is 3, 2 .

22.

2z2  5z  4  0

or

y

Lesson 10-4

1. x2  8x  4  0
x



1  273
4

 2.4
 1.9
The approximate solution set is {1.9, 2.4}.

b  2b2  4ac
2a
(8)  2(8) 2  4(1) (4)
2(1)
8  164  16
2
8  180
2
8  280
8  280
or x 
2
2

x
 8.5
 0.5
The approximate solution set is {0.5, 8.5}.
2. x2  7x  8  0
x




x

b  2b2  4ac
2a
(7)  2(7) 2  4(1) (8)
2(1)
7  149  32
2
7  181
2
7  9
2
7  9
7  9
or x  2
2

1
 8
The solution set is {8, 1}.

735

Extra Practice

PQ249J-6481F-24-26[726-749] 26/9/02 9:03 PM Page 736 Sahuja Ahuja_QXP_06:Desktop Folder:Chandra:Algebra_FNL_Delivery:

3. x2  5x  6  0
x




x

7. m2  4m  2  0

b  2b2  4ac
2a
(5)  2(5) 2  4(1) (6)
2(1)
5  125  24
2
5  11
2
5  1
2
5  1
5  1
or x  2
2

m



m

 0.6
 3.4
The approximate solution set is {3.4, 0.6}.
8. 2t2  t  15  0

3
2
The solution set is {2, 3}.
4. y2  7y  8  0
y




y

t

b  2b2  4ac
2a
(7)  2(7) 2  4(1) (8)
2(1)
7  149  32
2
7  181
2
7  9
2
7  9
7  9
or y  2
2





t

m2  2m  35  35  35
m2  2m  35  0





x

x

b  2b2  4ac
2a

(2)  2(2) 2  4(1) (35)


2(1)
2  14  140
2
2  1144
2
2  12
2
2  12
2  12
or x  2
2




x





y

b  2b2  4ac
2a
(0)  2(0) 2  4(5) (125)
2(5)
0  10  2500
10
 12500
10
50
10
50
50
or x  10
10

5
 5
The solution set is {5, 5}.
10. t2  16  0

7
 5
The solution set is {5, 7}.
6. 4n2  20n  0
y

b  2b2  4ac
2a
(1)  2(1) 2  4(2) (15)
2(2)
1  11  120
4
1  1121
4
1  11
4
1  11
1  11
or t  4
4

 2.5
3
The solution set is {2.5, 3}.
9.
5t2  125
2  125  125  125
5t
5t2  125  0

 1
8
The solution set is {1, 8}.
5.
m2  2m  35

x

b  2b2  4ac
2a
(4)  2(4) 2  4(1) (2)
2(1)
4  116  8
2
4  18
2
4  18
4  18
or m 
2
2

t


b  2b2  4ac
2a
(20)  2(20) 2  4(1) (0)
2(4)
20  1400  0
8
20  1400
8
20  20
8
20  20
20  20
or y 
8
8




b  2b2  4ac
2a
(0)  2(0) 2  4(1) (16)
2(1)
 10  64
2
 164
2

The solution set is .


11.
4x2  8x  3
4x2  8x  3  3  3
4x2  8x  3  0
x

5
0
The solution set is {0, 5}.




x

b  2b2  4ac
2a
(8)  2(8) 2  4(4) (3)
2(4)
8  164  48
8
8  1112
8
8  1112
8  1112
or x 
8
8

 0.3
 2.3
The approximate solution set is {0.3, 2.3}.

Extra Practice

736

PQ249J-6481F-24-26[726-749] 26/9/02 9:03 PM Page 737 Sahuja Ahuja_QXP_06:Desktop Folder:Chandra:Algebra_FNL_Delivery:

12.

16. s2  8s  7  0

3k2  2  8k
3k  2  2  8k  8k  8k  2
3k2  8k  2  0
2

k



k

s


b  2b2  4ac
2a
(8)  2(8) 2  4(3) (2)
2(3)
8  164  24
6
8  140
6
8  140
8  140
or k 
6
6




s

 7
 1
The solution set is {7, 1}.
17. d2  14d  24  0

 0.3
 2.4
The approximate solution set is {2.4, 0.3}.
13. 8t2  10t  3  0

14.

5x
1
 4 20
b  2b2  4ac

2a





x

1 52

5 2

 12
2
The solution set is {2, 12}.
18.
3k2  11k  4
2
3k  11k  4  4  4
3k2  11k  4  0
k

1 12

 4(3) 2

2(3)

25
16

3 1 4 2

 4 
5
4

5
4

(11)  2(11) 2  4(3) (4)


2(3)
11  1121  48

6
11  1169

6
11  13
11  13
k
or k 
6
6
1
3
 4
1
The solution set is 4, 3 .

6

121

3 16
6
11
4

6
5
4


6

11
4

or

x

3
The solution set is

5
4

19.

11
4




n2

 3n  1  0

d

6
1
 4

523, 14 6.




15. 5b  3b2  1  0


b

b  2b2  4ac
2a

6
5
4

b  2b2  4ac
2a
(14)  2(14) 2  4(1) (24)

2(1)
14  1196  96

2
14  1100

2
14  10

2
14  10
14  10

or s 
2
2

d

b  2b2  4ac
2a
(10)  2(10) 2  4(8) (3)

2(8)
10  1100  96

16
10  14

16
10  2

16
10  2
10  2
t  16
or t  16
1
3
 2
 4
3
1
The solution set is 4, 2 .

t

3x2

b  2b2  4ac
2a
(8)  2(8) 2  4(1) (7)
2(1)
8  164  28
2
8  136
2
8  6
2
8  6
8  6
or s  2
2

b  2b2  4ac
2a
(3)  2(3) 2  4(5) (1)
2(5)
3  19  20
10
2  111
1

d

b  2b2  4ac
2a
(3)  2(3) 2  4(1) (1)
2(1)
3  19  4
2
3  15
2
3  15
3  25
or d 
2
2

 2.6
 0.4
The approximate solution set is {0.4, 2.6}.


The solution set is .

737

Extra Practice

PQ249J-6481F-24-26[726-749] 26/9/02 9:03 PM Page 738 Sahuja Ahuja_QXP_06:Desktop Folder:Chandra:Algebra_FNL_Delivery:

20. 2z2  5z  1  0
z



z

27.

 0.2
 2.7
The approximate solution set is {2.7, 0.2}.
21.

25t2  30t  9
25t  30t  9  9  9
25t2  30t  9  0
b2  4ac  (30) 2  4(25) (9)
0
Since the discriminant is 0, the equation has one
real root.
2

b  2b2  4ac
2a
(5)  2(5) 2  4(2) (1)
2(2)
5  125  8
4
5  132
4
5  132
5  232
or z 
4
4

Page 842

3h2  27
3h  27  27  27
3h2  27  0
2

h



h

b  2b2  4ac
2a
(0)  2(0) 2  4(3) (27)
2(3)
0  2324
6
18
6
18
18
or h  6
6

2

72

70
71
72
73

3f 2  2f  6
3f  2f  6  6  6
3f 2  2f  6  0
b2  4ac  (2) 2  4(3) (6)
 76
Since the discriminant is positive, the equation
has two real roots.

2. y 

1
7

1
7
49
343

1

y
y  7x

2x

113 2x, 113 25.6

Sample answer:
x
3
2

2x2  0.7x  0.3


2
2x  0.7x  0.3  0.7x  0.7x  0.3  0.3
2x2  0.7x  0.3  0
b2  4ac  (0.7) 2  4(2)(0.3)
 2.89
Since the discriminant is positive, the equation
has two real roots.

1
0
1
2

113 2x
113 23
113 22
113 21
113 20
113 21
113 22

y
14
12
10
8
6
1
4 y 3
2

y
27
9
3
1

( )

2 1 O

1 2x

1
3
1
9

113 25.6  0.002


3
3
3. y  1 5 2 x, 1 5 2 4.2

The y-intercept is 1.

Sample answer:

25. 4r2  12r  9  0


b2  4ac  (12) 2  4(4) (9)
0
Since the discriminant is 0, the equation has one
real root.

x
3
2

x2  5x  9
2  5x  9  9  9
x
x2  5x  9  0
b2  4ac  (5) 2  4(1)(9)
 11
Since the discriminant is negative, the equation
has no real roots.

1
0
1
2

135 2x
135 23
135 22
135 21
135 20
135 21
135 22

135 24.2  8.5

738

14
12
10
8
6
4
2

125
27
25
9
5
3

1
3
5
9
25

The y-intercept is 1.

Extra Practice

70
60
50
40
30
20
10

1
49

The y-intercept is 1.
71.5  18.5

24. 3w2  2w  8  0
b2  4ac  (2) 2  4(3)(8)
 92
Since the discriminant is negative, the equation
has no real roots.

26.

7x

0
1
2
3

23.

1 71

3
 3
The solution set is {3, 3}.
22.

Lesson 10-5

1. y  7x, 71.5
Sample answer:

( 3)

y 5

4

2

1x

PQ249J-6481F-24-26[726-749] 26/9/02 9:03 PM Page 739 Sahuja Ahuja_QXP_06:Desktop Folder:Chandra:Algebra_FNL_Delivery:

4. y  3x  1
Sample answer:
x

3x  1

8. y 

Sample answer:
y

2 32  1

10
9

1 31  1

4
3

0
1
2
3

30  1
31  1
32  1
33  1

14
12
10
8
6
4
2

2
4
10
28

x
2
1
y

1O

2x  5

2

22  5

19
4

1

21  5

9
2

3x 

1 2 3x

1
2
3

20  5
21  5
22  5
23  5

10
8
6
4
2

4
3
1
3

8

4

O
4x
2
4
6 y  2x  5

2
1

1
y
28
24
20
16
12
8
4 y  2x  3
4

2

2
3

2x

7. y  3x + 1
3x  1

2

31

1
0
1
2
3

30
31
32
33
34

1
3

1
3
9
27
81

4

2

3
2

( 2)

y 3

1
2

2
3

1

2x

4
9
8
27

125 2x

125 2x
2
5 1 5 2 2
2
5 1 5 2 1
2
515 20
2
515 21
2
515 22
2
515 23

5(3x)

14
12
10
8
6
4
2

125
4
25
2

2

( 2)

y 5 5

4
5
8
25

2 5(32 )

5
9

1 5(31)

5
3

0 5(30 )
1 5(31)
2 5(32)

y
14
12
10
8
6
4
2 y  3x  1

9
4

The y-intercept is 5.
10. y  5(3x )
Sample answer:

The y-intercept is 8.

Sample answer:

6

The y-intercept is 1.
9. y  5

The y-intercept is 4.


6. y  2x  3
Sample answer:
x 2x  3 y
2
21
2
1
22
4
3
0
2
8
1
24 16
2
25 32
3
26 64

123 2x
123 22
123 21
123 20
123 21
123 22
123 23

The y-intercept is 2.
5. y  2x  5
Sample answer:

0
1
2
3

123 2x

5
15
45

16
14
12
10
8
6
4
2

y  5(3)x

The y-intercept is 5.

3 21O

1 2x

11. y  4(5)x
Sample answer:

2x

x
The y-intercept is 3.

4(5)x

2 4(5)2

4
25

1 4(5)1

4
5

0 4(5)0
1 4(5)1
2 4(5)2

4
20
100

35
30
25
20
15
10
5
2

y  4(5)x
2x

The y-intercept is 4.

739

Extra Practice

PQ249J-6481F-24-26[726-749] 26/9/02 9:03 PM Page 740 Sahuja Ahuja_QXP_06:Desktop Folder:Chandra:Algebra_FNL_Delivery:

12. y  2 (5)x  1
x
2

Page 843

2(5)  1
2(5)2

1

1 2(5)1  1
0 2(5)0  1
1 2(5)1  1
2 2(5)2  1

7
5

3
11
51

21O

x
2
1
0
1
2

112 2x1

112 2x1 y
112 221 2
112 211 1
112 201 12
112 211 14
112 221 18

 8500(1.00604) 48
 11,349.73
He will have about $11,349.73.

1 2x

The y-intercept is
14. y 

118 2x

y
7
6
5
4
3
1
2 y 2
1

2a. V  C(1  r) t
V  21,500 (1  0.08) 5
2b. V  21,500(0.92) 5
 14,170.25
The value of the car will be $14,170.25.
3a. P  C(1  r) t
P  3422(1  0.049) 8

x1

3b. P  3422(1.049) 8
 5017
The population was 5017.

( )

2

Page 843
1
.
2

1. 12

2
1
0
1
2

118 2x
118 22
118 21
118 20
118 21
118 22

y
14
12
10
8
6
1
4 y 8
2

y
64
8



1.2
1.2

2

No; the difference between consecutive terms is


constant. This sequence is arithmetic, not
geometric.
4. 86
68.8
55.04
44.032

2x

1
64


0.8

2
1
0
1
2

1 2 2
134 22  2
134 21  2
134 20  2
134 21  2
134 22  2


0.8




2
2
2

y
2
9

2
3

1
5
4

4
3 x

( )

2

y  4 2

Yes; the common ratio is 2.


6. 13
10
11
8
9






3
1
3
1
3

No; the difference between consecutive terms is a


constant pattern. This sequence is arithmetic, not
geometric.
7. 3125, 625, 125, 25, ...
Divide the second term by the first.

2x

1

23
16

625
3125

The y-intercept is 1.


16. No; the domain values are at regular intervals
and the range values have a common difference
of 4.
17. Yes; the domain values are at regular intervals
and the range values have a common ratio of 5.

Extra Practice


0.8

Yes; the common ratio is 0.8.


5. 4
8
16
32

Sample answer:
x


1.2




3
3
3

134 2x  2
3 x
4

45

Yes; the common ratio is 1.2.


3. 39
33
27
21

The y-intercept is 1.
15. y 

34

No; the difference between consecutive terms is


constant. This sequence is arithmetic, not
geometric.
2. 6
7.2
8.64
10.368

( )

1
1
8

Lesson 10-7
23




11
11
11

Sample answer:
x

M  8500 1 

The y-intercept is 3.
13. y 

12(4)
1 0.0725
12 2
0.0725
1b. M  8500 1 1  12 2 12(4)

14
12
10
8
6
4
2 y  2(5)x  1

27
25

Lesson 10-6

r
1a. M  P 1  n nt

 0.2

Multiply by 0.2 three more times. The next three


terms are 5, 1, and 0.2.
8. 15, 45, 135, 405, ...
Divide the second term by the first.
45
15

 3

Multiply by 3 three more times. The next three


terms are 1215, 3645, and 10,935.

740

PQ249J-6481F-24-26[726-749] 26/9/02 9:03 PM Page 741 Sahuja Ahuja_QXP_06:Desktop Folder:Chandra:Algebra_FNL_Delivery:

22. an  a1  rn1
a3  a1  r31

9. 243, 81, 27, 9, ...


Divide the second term by the first.
81
243

3

81  1  r2
81  r2
9  r
The geometric mean is 1(9)  9 or 1(9)  9.

Multiply by 3 three more times. The next three


1
terms are 3, 1, and 3.
10. 15, 7.5, 3.75, 1.875, ...
Divide the second term by the first.
7.5
15

23. an  a1  rn1
a3  a1  r31

 0.5

9  81  r2

Multiply by 0.5 three more times. The next


three terms are 0.9375, 0.46875, and 0.234375.
11. 25, 15, 9, 5.4, ...
Divide the second term by the first.
15
25

9
81
1
9
1
3

 0.6

12.

14
504
1
36
1
6

terms are

three more times. The next three

4
8
,
,
625 3125

16

and 15,625.

13. an  a1  rn1
a10  1  6101

15. an  a1  rn1
a4  6  0.441
a4  6  0.43
a4  0.384

25.

a  1  (4) 6
7
a7  4096

504r2
504

 r2
r

1 2

116 2  84

an  a1  rn1
a3  a1  r31

162  0.5  r2
162
0.5

16. an  a1  rn1
a10  100  0.1101

17. an  a1  rn1
a5  750  (1.5) 51

The geometric mean is 504


1
or 504 6  84.

14. an  a1  rn1
a7  1  (4) 71

a10  1  69
a10  10,077,696

113 2  27

14  504  r2

3

Multiply by

r

24. a  a  rn1
n
1
a3  a1  r31

2
3

 r2

1 2

Divide the second term by the first.


1
10
1
4

81r2
81

The geometric mean is 81


1
or 81 3  27.

Multiply by 0.6 three more times. The next three


terms are 3.24, 1.944, and 1.1664.
1 1 1
2
, , ,
,
4 10 25 125

0.5r2
0.5

324  r2
18  r
The geometric mean is 0.5(18)  9 or
0.5(18)  9.

a10  100  0.19


a10  0.0000001

26. an  a1  rn1
a3  a1  r31

a5  750  (1.5) 4
a5  3796.875

4  1  r2
4
1

 rn1

18. an  a1
a5  64  851

1r2
1

4  r2
2  r
The geometric mean is 1(2)  2 or 1(2)  2.

a5  64  84
a5  262,144

27.

 rn1

19. an  a1
a9  0.5  (10) 91
8

a9  0.5  (10)
a9  50,000,000

 rn1

20. an  a1
a5  20  (2.5) 61

an  a1  rn1
a3  a1  r31

0.36  0.25  r2

a5  20  (2.5)
a5  1953.125

0.36
0.25

0.25

 0.25r2

1.44  r2
1.2  r
The geometric mean is 0.25(1.2)  0.3 or
0.25(1.2)  0.3.

n1

21. an  a1  r
a4  350  (0.9) 41
a  350  (0.9) 3
4
a4  255.15

741

Extra Practice

PQ249J-6481F-24-26[726-749] 26/9/02 9:03 PM Page 742 Sahuja Ahuja_QXP_06:Desktop Folder:Chandra:Algebra_FNL_Delivery:

28. a  a  rn1
n
1
a3  a1  r31
1
8
1
8
1
2
1
4
1
2

11.

 2  r2
1 2
r
2
1
2

 r2
r
 rn1

29. an  a1
a3  a1  r31
32
27
32
27
2
3
16
9
4
3

1 2

1 1
2 2

1
4

or

1
2

1 2
1
2

1 2  89 or 23 143 2  89.

an  a1  rn1
a3  a1  r31
6.25

 6.25r2

21.

13
15

115
5

2. 1200  1100  2
 1012
4. 1700  1100  7
 1017

23.

24.

72
6

9.

25.

17

8.

14  2  7
7

2 114
7

3 3

2
6

5
24

15
124

15
2 16

130
12

Extra Practice

232c5

132  2c5
19  2d2

4 12  c2 1c
30d0

3 7  17  17

5
8

32c5

3 9d2

15
15

18

3 16
0g0

22. 299x3y7  199  2x3  2y7


 3111  x1x  y3  1y
 3 0 xy3 0 111xy

 112
 14  3
 213
7.

154

54

3 g2  2g2


Lesson 11-1

1. 150  125  2
 512
3. 1162  181  2
 912

172
16

150
2z2
125  2
0z0

5 12
0z0

15. 612  13  616

20. 2175a4b6  1175  2a4  2b6


 517  a2  0 b3 0
 5a2 0 b3 0 17

0.36  r2
0.6  r
The geometric mean is 6.25(0.6)  3.75 or
6.25(0.6)  3.75.

6.

19. 212ts3  112  1t  2s3


 213  1t  s1s
 2 0 s 0 13st

2.25  6.25  r2

5.

18. 2200m2y3  1200  2m2  2y3


 1100  2  0 m 0  2y2  1y
 10  12  0 m 0  y  1y
 10 0 my 0 12y

2 4
3

The geometric mean is 3

13
15

115
115

17. 24x4y3  14  2x4  2y2  1y


 2x2 0 y 0 1y

r

Page 844

115x
15

50

3 z2 

16. 516  213  10118


 1019  2
 10(312)
 3012

 r2

2.25
6.25

1x
115

12.

14. 17  13  121
1
4.

2
 r2
3
2
 r2
3
2
3

13. 110  120  1200


 1100  2
 1012

The geometric mean is

30.

2x

3 30  3 15

10.

17

27p4

3 3p2

1
3  15

3 32  132

2
3

 19  2p2
 30p0
1

3





17
4 12

114
8

3
2

 11

1

3 3


26.

16
16

742

2
13  5

4c2 12c
30d0

 29p2

17
116  2
12
12

29d2

15
3  15
9  5
3  15
4

3  15
15

3 

2
13  5

13  5 13  5
2( 13  5)
3  25
2( 13  5)
22
13  5
11

PQ249J-6481F-24-26[726-749] 26/9/02 9:03 PM Page 743 Sahuja Ahuja_QXP_06:Desktop Folder:Chandra:Algebra_FNL_Delivery:

27.

13
13  5





28.

16
7  2 13




15
 3 15  12 1 19 2
1 11
15 2
1
15
15
 10 1 15  15 2  3 15 12 1 3 2
15
 10 1 5 2  3 15  4 15

19. 10 3 5  145  12 3 9  10

16
7  2 13

 2 13 7  2 13
16(7  2 13)
49  12
7 16  2 118
37
7 16  6 12
37

7


Page 844

18. 4175  6127  4(513)  6(313)


 2013  1813
 3813

13
13  5

13  5 13  5
13( 13  5)
3  25
3  5 13
22

 2 15  3 15  4 15
 5 15

20. 115 

13

3 5  115  15
 115 

Lesson 11-2

1. 3111  6111  2111  (3  6  2) 111


 7111
2. 6113  7113  (6  7) 113
 13113
3. 2112  513  2(213)  513
 413  513
 913
4. 917  412  312  517
 (9  5) 17  (4  3) 12
 1417  12
5. 315  513 in simplest form
6. 418  315  4(212)  315
 812  315
7. 2127  4112  2(313)  4(213)
 613  813
 213
8. 8132  4150  8(412)  4(512)
 3212  2012
 5212
9. 145  6120  315  6(215)
 315  1215
 1515
10. 2163  6128  8145
 2(317)  6(217)  8(315)
 617  1217  2415
 6 17  2415
11. 1413t  813t  2213t
12. 716x  1216x  516x
13. 517  3128  517  3(217)
 517  617
 17
14. 718  118  7(212)  312
 1412  312
 1112
15. 7198  5132  2175
 7(712)  5(412)  2(513)
 4912  2012  1013
 6912  1013
16. 416  312  215 in simplest form
17. 3120  2145  17  3(215)  2(315)  17
 615  615  17
 17


1
3

21. 3 3  9 3

1
12

5 115
5
4 115
5

13 15

15 15
115
5

11
 9 1 112 2  913
1 11
13 2
1
13
1
13
 3 1 13  13 2  9 1 2 13  13 2  913
13
13
 3 1 3 2  9 1 6 2  913

 1243  3

 13 

3 13
2

 913

2 13
2

3 13
2

17 13
2

18 13
2

22. 13( 15  2)  115  213


23. 12( 12  315)  2  3110
24. ( 12  5) 2  ( 12) 2  2( 12) (5)  52
 2  1012  25
 27  1012
25. (3  17)(3  17)  32  ( 17) 2
97
2
26. ( 12  13) ( 13  12)  16  14  19  16
 216  2  3
 216  5
27. (417  12) ( 13  315)
 4121  12135  16  3110

Page 844
1.

Lesson 11-3

15x  5
( 15x) 2  52
5x  25
x5
?

15(5)  5
?
125  5
55
2.
417  1m
(417) 2  ( 1m) 2
112  m
112  m
Check:

Check:

743

417  1(112)
?
417  1112
417  417

Extra Practice

PQ249J-6481F-24-26[726-749] 26/9/02 9:03 PM Page 744 Sahuja Ahuja_QXP_06:Desktop Folder:Chandra:Algebra_FNL_Delivery:

3. 1t  5  0
1t  5
( 1t) 2  52
t  25

9. 1a  2  0
1a  2
( 1a) 2  22
a4

125  5  0
?
550
00

Check:

Check:

10. 12j  4  8
12j  12
( 12j) 2  122
2j  144
j  72

4. 13b  2  0
13b  2
( 13b) 2  (2) 2
3b  4
4

b3

3 31

Check:

4
3

220

Check:

?
?

14  2  0
?
220
40

Since does not satisfy the original equation,


there is no solution.
1x  3  6
( 1x  3) 2  62
x  3  36
x  39

Check:

12.

139  3  6
?
136  6
66

Check:

6. 5  13x  1
13x  4
(13x) 2  (4) 2
3x  16
16
3

x

5

Check:

3 31 3 2  1
?

5  116  1
?
541
11
7. 2  31y  13
31y  11

c5
Check:

11
3
11 2
3

121 ?

9
11 ?

14.
13

1 3 2  13
?

2  11  13
13  13
13g  6

13(12)  6
?

136  6
66

Extra Practice

215t  10
15t  5
( 15t) 2  52
5t  25
t5
Check:

( 13g) 2  62
3g  36
g  12
Check:

2  33
23

3 515 2  9

7  14  9
?
729
99

121
9

y

8.

7

1 2

( 1y) 2 

Check:

15(9)  4  7
?
145  4  7
?
149  7
77

13. 7  15c  9
15c  2
( 15c) 2  22
5c  4

1y 

5  116  9
?
549
99

15y  4  7
( 15y  4) 2  72
5y  4  49
5y  45
y9
Check:

16

12(72)  4  8
?
1144  4  8
?
12  4  8
88

11. 5  1x  9
1x  4
( 1x) 2  42
x  16

4
3

5.

14  2  0
?
220
00

744

215(5)  10
?
2125  10
?
2(5)  10
10  10

PQ249J-6481F-24-26[726-749] 26/9/02 9:03 PM Page 745 Sahuja Ahuja_QXP_06:Desktop Folder:Chandra:Algebra_FNL_Delivery:

15.

144  21p
2111  21p
111  1p
( 111) 2  ( 1p) 2
11  p

144  2111
2111  2111
41x  5  15

Check:
16.

20.

1x  5 
( 1x  5) 2 
x5
x
Check:

225 ?

16
15 ?

4

1 2

15
15

15  15

17. 4  1x  3  9
1x  3  5
(1x  3) 2  52
x  3  25
x  28

22(2 13) 2  12  213

12(12)  12  213

12(12)  12  213

124  12  213

124  12  213

?
?

Check:

110(5)  5  315

110(5)  5  315

145  315
315  315

145  315
315  3 15

22a2  144  a
( 22a2  144) 2  a2
2a2  144  a2
a2  144
2a2  1144
a  12
?

22(12) 2  144  12

12(144)  144  12

1288  144  12

1144  12

12  12

2( 12) 2  16  2( 12)  512


?
12  16  212  512
?
118  212  512
?
312  212  512
512  512

2(12) 2  16  2(12)  5(12)


?
12  16  212  512
?
118  212  512
?
312  212  512
12  512
Since 12 does not satisfy the original equation,
12 is the only solution.

Since 15 does not satisfy the original equation,


15 is the only solution.

1144  12

112  213
213  213

22. 2b2  16  2b  5b
2b2  16  3b
( 2b2  16) 2  (3b) 2
b2  16  9b2
8b2  16
b2  2
2b2  12
b  12

210( 15) 2  5  315 210(15) 2  5  3(15)

1288  144  12

Since 213 does not satisfy the original


equation, 213 is the only solution.

210x2  5  3x
( 210x2  5) 2  (3x) 2
10x2  5  9x2
x2  5
2x2  15
x  15

12(144)  144  12

112  213
213  213

22(12) 2  144  12

22(213) 2 12  213


?

4  128  3  9
?
4  125  9
?
459
1  9
Since 28 does not satisfy the original equation,
there is no solution.

Check:

124  1  5
?
125  5
55

Check:

19.

21. 22x2  12  x
22x2  12  x2
2x2  12  x2
x2  12
2x2  112
x  213
Check:

 5  15

Check:

Since 1 does not satisfy the original equation, 8 is


the only solution.

43

18.

13(8)  1  8  3

13  1  2
?
14  2
2  2

1 2

305
16

13(1)  1  1  3

Check:

15
4
15 2
4
225
16
305
16

43

13y  1  y  3
( 13y  1) 2  (y  3) 2
3y  1  y2  6y  9
0  y2  9y  8
0  ( y  1) (y  8)
y  1  0 or y  8  0
y1
y8

?
?
?
?

12  12

Since 12 does not satisfy the original equation,


12 is the only solution.

745

Extra Practice

PQ249J-6481F-24-26[726-749] 26/9/02 9:04 PM Page 746 Sahuja Ahuja_QXP_06:Desktop Folder:Chandra:Algebra_FNL_Delivery:

23. 1m  2  m  4
1m  2  4  m
( 1m  2) 2  (4  m) 2
m  2  16  8m  m2
0  m2  9m  14
0  (m  2)(m  7)
m  2  0 or m  7  0
m2
m7
?

c2  a2  b2
( 1200) 2  a2  102
200  a2  100
100  a2
1100  2a2
10  a
The length of the leg is
5.
c2  a2  b2
(312) 2  32  b2
18  9  b2
9  b2
19  2b2
3  b
The length of the leg is
6.
c2  a2  b2
142  62  b2
196  36  b2
160  b2
1160  2b2
12.65  b
The length of the leg is
7.
c2  a2  b2
( 147) 2  ( 111) 2  b2
47  11  b2
36  b2
136  2b2
6  b
The length of the leg is
4.

12  2  2  4
17  2  7  4
?
?
14  2  4
19  7  4
?
?
224
374
44
10  4
Since 7 does not satisfy the original equation, 2 is
the only solution.
24. 13  2c  3  2c
13  2c  2c  3
( 13  2c) 2  (2c  3) 2
3  2c  4c2  12c  9
0  4c2  10c  6
0  2(2c2  5c  3)
0  2(2c  3)(c  1)
2c  3  0 or c  1  0
2c  3
c1
Check:

c2
Check:

3 3  212 2  3  212 2
3

13  3  3  3

10  3  3
33

13  2(1)  3  2(1)
?

13  2  3  2
?

11  3  2
42

Since 1 does not satisfy the original equation,


the only solution.

Page 845

3
2

is

about 12.65 units.

6 units.

b2



c2  ( 113) 2  b2
c2  13  36
c2  49
2c2  149
c  7
The length of the hypotenuse is 7 units.

9.

c2  a2  b2
c2  ( 16) 2  32
c2  6  9
c2  15
2c2  115
c  3.87
The length of the hypotenuse is about 3.87 units.

1.

Extra Practice

a2

3 units.

8.

Lesson 11-4

c2  a2  b2
292  a2  202
841  a2  400
441  a2
1441  2a2
21  a
The length of the leg is 21 units.
c2  a2  b2
2.
c2  72  242
c2  49  576
c2  625
2c2  1625
c  25
The length of the hypotenuse is 25 units.
c2  a2  b2
3.
c2  22  62
c2  4  36
c2  40
2c2  140
c  6.32
The length of the hypotenuse is about 6.32 units.

c2

10 units.

10.

c2  a2  b2
102  a2  ( 175) 2
100  a2  75
25  a2
125  2a2
5  a
The length of the leg is 5 units.

11.

746

c2  a2  b2
( 1130) 2  a2  92
130  a2  81
49  a2
149  2a2
7  a2
The length of the leg is 7 units.

PQ249J-6481F-24-26[726-749] 26/9/02 9:04 PM Page 747 Sahuja Ahuja_QXP_06:Desktop Folder:Chandra:Algebra_FNL_Delivery:

12.

13.

14.

15.

c2  a2  b2
152  92  b2
225  81  b2
144  b2
1144  2b2
12  b
The length of the leg is 12 units.

3. d  2(x  x ) 2  (y  y ) 2
2
1
2
1
 2(1  4) 2  (2  (2) ) 2
 2(3) 2  42
 19  16
 125
5
4. d  2(x  x ) 2  (y  y ) 2
2
1
2
1

c2  a2  b2
112  a2  52
121  a2  25
96  a2
196  2a2
9.80  a
The length of the leg is about 9.80 units.
c2

a2

 2[4  (2) ] 2  (2  4) 2


 262  (6) 2
 136  36
 172
 612 or about 8.49
5. d  2(x  x ) 2  (y  y ) 2
2
1
2
1

b2



c2  ( 133) 2  42
c2  33  16
c2  49
2c2  149
c  7
The length of the hypotenuse is 7 units.
c2

a2

 2(2  3) 2  (1  1) 2
 2(5) 2  (2) 2
 125  4
 129 or about 5.39
6. d  2(x  x ) 2  (y  y ) 2
2
1
2
1
 2[7  (2) ] 2  (8  4) 2

b2



 52  b2
34  25  b2
9  b2
19  2b2
3  b
The length of the leg is 3 units.
Yes; 142  482  502
No; 202  302  402
Yes; 212  722  752
Yes; 52  ( 1119) 2  122
Yes; 152  362  392
2
2
2
No; ( 15)  12  13
2
2
2
No; 10  ( 122)  12

 292  (12) 2
 181  144
 1225
 15

( 134) 2

7. d  2(x  x ) 2  (y  y ) 2
2
1
2
1
 2[ 9  (5) ] 2  (6  0) 2
 2(4) 2  62
 116  36
 152
 2113 or about 7.21

16.
17.
18.
19.
20.
21.
22.
23. No; 22  32  42
2
2
2
24. Yes; ( 17)  8  ( 171)

Page 845

8. d  2(x  x ) 2  (y  y ) 2
2
1
2
1
 2(5  5) 2  (13  (1) ) 2
 202  142
 1196
 14
9. d  2(x2  x1 ) 2  (y2  y1 ) 2
 2(10  2) 2  [ 8  (3) ] 2

Lesson 11-5

 282  112
 164  121
 1185 or about 13.60

1. d  2(x2  x1 ) 2  (y2  y1 ) 2
 2(2  4) 2  (10  2) 2
 2(6) 2  82
 136  64
 1100
 10

10. d  2(x2  x1 ) 2  (y2  y1 ) 2


 2[2  (7) ] 2  (7  5) 2
 292  (12) 2
 181  144
 1225
 15

2. d  2(x2  x1 ) 2  (y2  y1 ) 2
 2(7  (5)) 2  (6  1) 2
 2122  52
 1144  25
 1169
 13

11. d  2(x2  x1 ) 2  (y2  y1 ) 2


 2[ 5  (6) ] 2  [ 4  (2) ] 2
 212  62
 11  36
 137 or about 6.08

747

Extra Practice

PQ249J-6481F-24-26[726-749] 26/9/02 9:04 PM Page 748 Sahuja Ahuja_QXP_06:Desktop Folder:Chandra:Algebra_FNL_Delivery:

12. d  2(x  x ) 2  (y  y ) 2
2
1
2
1

20. d  2(x2  x1 ) 2  (y2  y1 ) 2

 2(3  8) 2  [ 2  (10) ] 2

 2[3  (9) ] 2  (3  2) 2

 2(5) 2  122
 125  144
 1169
 13

 2122  (5) 2
 1144  25
 1169
 13

13. d  2(x  x ) 2  (y  y ) 2
2
1
2
1
2

 2(7  4)  [ 9  (3) ]
2

21. d  2(x2  x1 ) 2  (y2  y1 ) 2


2

 2(512  312) 2  (9  7) 2

 23  (6)
 19  36
 145
 315 or about 6.71

 2(212) 2  22
 18  4
 112
 213 or about 3.46

14. d  2(x  x ) 2  (y  y ) 2
2
1
2
1

22. d  2(x2  x1 ) 2  (y2  y1 ) 2

 2(9  6) 2  (7  3) 2
232

 2(10  6) 2  (0  3) 2

42



 19  16
 125
5

 242  (3) 2
 116  9
 125
5
23. d  2(x2  x1 ) 2  (y2  y1 ) 2

15. d  2(x  x ) 2  (y  y ) 2
2
1
2
1

 2(5  3) 2  (5  6) 2

 2(9  10) 2  (7  0) 2
2

 222  (11) 2
 14  121
 1125
 515 or about 11.18

 2(1)  7
 11  49
 150
 512 or about 7.07

24. d  2(x2  x1 ) 2  (y2  y1 ) 2

16. d  2(x  x ) 2  (y  y ) 2
2
1
2
1
 2(3  2) 2  (3  (1)) 2

 2[5  (4) ] 2  (4  2) 2

 2(5) 2  42
 125  16
 141 or about 6.40

 292  22
 181  4
 185 or about 9.22

17. d  2(x2  x1 ) 2  (y2  y1 ) 2

25.

d  2(x2  x1 ) 2  (y2  y1 ) 2

 2 [3  (5) ] 2  (2  4) 2

5  2(a  0) 2  (3  0) 2

 282  (6) 2
 164  36
 1100
 10

5  2a2  32
(5) 2  ( 2a2  9) 2
25  a2  9
16  a2
116  2a2
4  a

18. d  2(x2  x1 ) 2  (y2  y1 ) 2


 2(0  0) 2  [ 7  (9) ] 2
26.

 202  162
 1256
 16

10  2(6  2) 2  (a  (1) ) 2
10  2(8) 2  (a  1) 2

19. d  2(x2  x1 ) 2  (y2  y1 ) 2


 2 [8 
292

(1) ] 2

 (4 

10  264  a2  2a  1

7) 2

102  ( 2a2  2a  65) 2


100  a2  2a  65
0  a2  2a  35
0  (a  7)(a  5)
a70
or a  5  0
a  7
a5

(3) 2



 181  9
 190
 3110 or about 9.49

Extra Practice

d  2(x2  x1 ) 2  (y2  y1 ) 2

748

PQ249J-6481F-24-26[726-749] 26/9/02 9:04 PM Page 749 Sahuja Ahuja_QXP_06:Desktop Folder:Chandra:Algebra_FNL_Delivery:

d  2(x2  x1 ) 2  (y2  y1 ) 2

27.

6.

161  2(a  1) 2  (6  0) 2
161  2a2  2a  1  62
( 161) 2  ( 2a2  2a  37) 2
61  a2  2a  37
0  a2  2a  24
0  (a  6)(a  4)
a  6  0 or a  4  0
a6
a  4
185  2(5 

(2)) 2

 (10 

1274  2(0  15)  (4  a)

AC
DF
18
e

8.

AB
DE
c
17.5

AC
DF
b
11

BC

 EF

9.

1.7

 8.5

10.

1274  2(15) 2  16  8a  a2

d  2(x2  x1 )  (y2  y1 )

15
d

AB

 DE
20

 10

AC
DF
5.6
7

1.7

BC

 EF
4

d

5.6d  28
d5
AB
DE
7.2
f

 8.5

BC
EF
80
100

AC

 DF


5.6
7

5.6f  50.4
f9

AC

 DF
b

 125

100b  10,000
b  100

( 1274) 2  ( 2225  16  8a  a2 ) 2
274  a2  8a  241
0  a2  8a  33
0  (a  11)(a  3)
or a  3  0
a  11  0
a  11
a  3
2

BC

 EF

8.5b  18.7
b  2.2

BC

 EF

20e  180
e9

8.5c  29.75
c  3.5

d  2(x2  x1 )  (y2  y1 )
2

AB
DE
20
10

20d  150
d  7.5

1
93
BC
EF
6
4.5

4.5c  45
c  10

a) 2

30.

7.

 4.5

b
AB
DE
c
7.5

185  272  100  20a  a2


( 185) 2  ( 2149  20a  a2 ) 2
85  a2  20a  149
0  a2  20a  64
0  (a  4)(a  16)
a  4  0 or a  16  0
a4
a  16
29.

BC

 EF

4.5b  42

d  2(x2  x1 ) 2  (y2  y1 ) 2

28.

AC
DF
b
7

BC
EF
80
100

AB

 DE
c

 218.75

100c  17,500
c  175

1136  2(a  3) 2  (9  3) 2

Page 846

1136  2a2  6a  9  62
2

( 1136)  ( 2a  6a  9  36)
136  a2  6a  45
0  a2  6a  91
0  (a  7)(a  13)
a70
or a  13  0
a  7
a  13

Lesson 11-7

1. sin N 

opposite leg
hypotenuse
36
39

 0.9231
cos N 


adjacent leg
hypotenuse
15
39

 0.3846

Page 845

tan N 

Lesson 11-6

1. No; corresponding angles do not have equal


measures.
2. Yes; corresponding angles have equal measures.
3. Yes; corresponding angles have equal measures.
4.

AB
DE
7
f




BC
EF
5
10

5f  70
f  14
AC
DE
8
e




BC
EF
5
10

5e  80
e  16

5.

AB
DE
4
f




 2.4
3. sin N 

BC
EF
2
3




opposite leg
hypotenuse
14
50

 0.28
cos N 


adjacent leg
hypotenuse
48
50

 0.96
opposite leg

tan N  adjacent leg


14

 48
 0.2917

opposite leg
hypotenuse
40
50

 0.8
adjacent leg
cos N  hypotenuse

2f  12
f6
AC
DF
3
e

opposite leg
adjacent leg
36
15

2. sin N 

30

 50

BC
EF
2
3

 0.6
opposite leg

tan N  adjacent leg

2e  9
e  4.5

40
30

 1.3333
4. cos 25  0.9063
6. sin 71  0.9455

749

5. tan 31  0.6009
7. cos 64  0.4384
Extra Practice

PQ249-6481F-P27-28[750-765] 26/9/02 8:58 PM Page 750 Sahuja 79:PQ249:PQ249J:PQ249J-ch15-28-Repro:

8. tan 9  0.1584
10. tan B  0.5427
B  tan1 (0.5427)
 28
11. cos A  0.8480
A  cos1 (0.8480)
 32
12. sin J  0.9654
J  sin1 (0.9654)
 75
13. cos Q  0.3645
Q  cos1 (0.3645)
 69
14. sin R  0.2104
R  sin1 (0.2104)
 12
15. tan V  11.4301
V  tan1 (11.4301)
 85
16. 180  90  60  30
The measure of A is 30.

9. sin 2  0.0349

Page 846
1.

Lesson 12-1

xy  k
(7.5)(10)  k
75  k
y

75
x

5
15

x
y

3
25

1
75

1
75

1
15

3
5

1
12

1
12

3
25

5
15

2
6

3
4

y
50
2

50

2.

xy  k
(3)(5)  k
15  k
y
3
5

x
y

15

tan 60  BC
BC(1.7321)  15
BC  8.7 cm
15
sin 60  AB

15

AB(0.8660)  15
AB  17.3 cm
17. 180  90  55  35
The measure of Z is 35.

15
x

1
15
y

5
2

x
O

15

21

sin 55  XZ
XZ(0.8192)  21
XZ  25.6 ft

3.

21

tan 55  XY

y

XY(1.4281)  21
XY  14.7 ft
18.

12

2
6

4
2

16
8

M  tan1 (2)
 63
180  90  63  27
The measure of  L is 27.

Extra Practice

12
x

3
4

x
y

c2  a2  b2
LM 2  162  82
LM 2  256  64
LM 2  320
2LM 2  1320
LM  17.0 m
tan M 

xy  k
(2)(6)  k
12  k

4
12

750

PQ249-6481F-P27-28[750-765] 26/9/02 8:58 PM Page 751 Sahuja 79:PQ249:PQ249J:PQ249J-ch15-28-Repro:

4.

xy  k
(0.5) (1)  k
0.5  k
y
x
y

9.

y

0.5
x

1
0.5

9

0.5
1

1.0

0.5
1

1
0.5

x

12.

x1y1  x2 y2

113 2 (27)  x2 y2
y

y

y  6
13.

xy  k
(3)(2.5)  k
7.5  k
y
x
y

7.5
x

3
2.5
15

y

1.5
5

1
7.5

1
7.5

1.5
3
5 2.5

15.

y

15

6.

Page 846

x
y

2
x

2
1

1
2

1
2

2
1

2
1

2x

2
4

7.

x1y1  x2 y2
(4) (54)  x2 y2
y
27 
x

216
x
216
x
216
27

4
16.

18.75
x
18.75
2.5

x  2.25
x1y1  x2 y2
(3.2)(0.4)  x2 y2
y
0.2 

8.

x1y1  x2 y2
(6)(18)  x2 y2
y
12 

1.28
x
1.28
x

x  6.4

Lesson 12-2

3. Exclude the values for which c2  4  0.


(c  2)(c  2)  0
or c  2  0
c20
c  2
c2
The excluded values are 2 and 2.
4. Exclude the values for which b2  8b  15  0.
(b  5)(b  3)  0
b  5  0 or b  3  0
b5
b3
The excluded values are 5 and 3.

y
4

9
x
9
x

1. Exclude the values for which x  1  0.


x10
x  1
The excluded value is 1.
2. Exclude the values for which n  0.
n0
The excluded value is 0.

xy  k
(1)(2)  k
2k
y

3
4

x1y1  x2 y2
(3) (3)  x2 y2

y  7.5

y

y  12
x1y1  x2 y2
(2.5)(7.5)  x2 y2

14.

24
x
24
2

y

5
2

x1y1  x2 y2
(8)(3)  x2 y2
y

9
x

y  12

1.0

5.

64
x
64
16

y
y4

24
4

y

0.5

x1y1  x2 y2
(8) (8)  x2 y2
y

(8)(3)  x2 y2
24
y x

x
1

10.

288
x
288
x
288
9

x  32
x1y1  x2 y2

11.

0.5
2

x1y1  x2 y2
(24) (12)  x2 y2

5.

13a
39a2

13a
(13a) (3a)

13a
(13a ) (3a)

 2x (21y)

 21y

108
x
108
x

 3a

x9

a0

x8

751

38x2
42xy

6.

2x(19x)

 2x(21y)
1

2x (19x)
1

19x

x  0 and y  0

Extra Practice

PQ249-6481F-P27-28[750-765] 26/9/02 8:58 PM Page 752 Sahuja 79:PQ249:PQ249J:PQ249J-ch15-28-Repro:

p  5
2( p  5)

7.

13.

p  5
 5)

 2(p


r1

8.

14.

a  b
(a  b) (a  b)

a  b
(a  b ) (a  b)
1

ab
2

15.

Exclude the values for which a  b  0.


(a  b)(a  b)  0
or a  b  0
ab0
a  b
ab
a  b
y  4
y2  16

y  4
(y  4) (y  4)

y  4
(y  4 ) (y  4)

4t2  8
4t  4

4(t2  2)
4(t  1)

t2  2
t  1

Exclude the values for which 4t  4  0.


4t  4  0
4t  4
t1
t1

9.

6y3  12y2
12y2  18

12y2  18  0
6(2y2  3)  0
2y2  3  0
2y2  3
3

y2  2
3

1
 4

y  3 2

Exclude the values for which y2  16  0.


( y  4)(y  4)  0
or y  4  0
y40
y  4
y4
y  4
c2  4
c2  4c  4

(c  2) (c  2)
(c  2) (c  2)

(c  2 ) (c  2)
(c  2 ) (c  2)

c  2
c  2

y

16.

Exclude the values for which c2  4c  4  0.


(c  2)(c  2)  0
c20
c  2
c  2
a  a
a  1

a(a  1)
a  1

a(a  1 )
a  1

Page 847
1.

a2b
b2c

a2bc

a2  b

 bbd
a2

 bd

a
Exclude the values for which a  1  0.
a1
12.

Lesson 12-3

 d  b2cd

x2  4
x4  16

2.

6a2n
8n2

12n
9a

3234aann
2433nna

a

x2  4
(x2  4) (x2  4)

3.

2a d
3bc

9b c

 16ad2 

x2  4

(x2  4 ) (x2  4)
1

 x2  4

4.

10n3
6x3

5.

120n5x4
5

6m3n
10a2

4a2m
9n3




752

3ab
8d
4

12n2x4

(x2  4)(x2  4)  0
x2  4  0
x2  4  0
2
x  4
x2  4
x  2
not possible
x  2

2aad9bbc
3  b  c  16  a  d  d

 25n2x2  150n2x5

Exclude the values for which x4  16  0.

Extra Practice

5(x2  2x  1)
 1)
5
3

 3(x2  2x

Exclude the values for which 3x2  6x  3  0.


3x2  6x  3  0
x2  2x  1  0
(x  1) 2  0
x  1
x  1

11.

16
2
5x2  10x  5
3x2  6x  3

16
2

y

y2 (y  2)
2y2  3

Exclude the values for which 12y2  18  0.

10.

6y2 (y  2)

 6(2y2  3)


y

r2 (r  1)
r  1

 r2

1
2

2( p  5)  0
p50
p  5
p  5.
a  b
a2  b2

r3  r2
r  1

4n3
5x
24a2m4n
90a2n3
4m4
15n2

PQ249-6481F-P27-28[750-765] 26/9/02 8:58 PM Page 753 Sahuja 79:PQ249:PQ249J:PQ249J-ch15-28-Repro:

6.
7.
8.

(a  5) (a  1) (a  7) (a  6)
a

a
(a  1) (a  7) (a  8) (a  5)
x  1
x  2

 (x
(x  2) (x  3) (x  3) (x  1)
5n  5
3

n

9
 1

5(n  1)
3

n

 6
 8
1
 3) 2

4.

9.

a
a  b

3a  3b
 a

x2y
6

9
 1

 12z2

x4

p2
14qr3

2r2p
7q

2(a  2b)
5

25

 6a  8b 

 4r5

11.
12.

3
x  y
x  5
3x

(x  y) 2
6

12x

 10

x  5
3x
1

 (x

5e  f
5e  f

7.

t2  2t  15
t  5

12x
 5) (x  2)

8.

4x
 2

x

13.

a2  b2
4

a  b 

14.

4a  8
a2  25

a  5
 5a  10

15.

r2
r  s

16

(a  b) (a  b)
4

a

16
 b

 4a  4b

6.

x  y
2

 x2  7x

25

 2(3a  4b)

5a  10b
3a  4b


1

r2  s2
s2

10.

4(a  2)
a  5

(a  5) (a  5) 5(a  2)
4
 5a  25
r2
(r  s) (r  s)

r  s
s2

r3  r2s
s2
a2  b2
7
(a  b) (a  b)
a  b 
a  b
a  b

a

17.

x2  3x  2
 x2  7x  6

12.

7
 b

(x  9) (x  1)
(x  9) (x  2)

(x  2) (x  1)
 6) (x  1)

 (x

x  1
 6

x

18.

x2  6x  5
x2  7x  12

Page 847
1.

5m2n
12a2

x2  14x  40
 x2  5x  50

13.

(x  5) (x  1)
(x  4) (x  3)

x  1
x  3

(x  4) (x  10)
(x  10) (x  5)

Lesson 12-4
30m4
18an

5m2n
12a2
2

14.

31

18an

 30m4
6

ammnn
4aammmm

15.

n2

 4am2
2.

25g7h
28t3

5g5h2
42s2t3

63

25g7h
28t3
4

16.

15g7hs2t3
2g5h2t3

15g2s2
2h
5

3.

6a  4b
36

3a  2b
45

2(3a  2b)
36

5
2

18

t5

5x  10

(x  2)
x  2
5(x  2)
1
 x  2 x  2
5
x2
3v2  27
v  3

v2
15v
3(v2  9)
v2
 15v  v  3

9.

11.

(v  3) (v  3)  v2
5v (v  3)
v(v  3)

5
b2  9

(b  3)
4b
(b  3) (b  3)
1

b  3
4b
b  3
 4b
p
p2

2y
y2  4
p2
2  y
 (y  2) (y  2)  p
p2
1(y  2)
 (y  2) (y  2) 
p
p
y2
k2  81
k  9

k6
k2  36
(k  9) (k  9) k  6
 (k  6) (k  6)  k  9
k  9
k6
2a3
a2

a1
a  1
2a3
a  1
 a  1  a2

9
 3
3d
2d  3
 d(2d  3)  9
3
1
3
g  5
3g2  15g

g2
4
3g(g  5)
g2

g  5
4

3d
2d2  3d

2d

3g3
4

 2a

42
 5g5h2

7
x2  10x  9
x2  11x  18

5e  f
1

 f 25e2  f 2
5e  f
1
 5e  f  (5e  f ) (5e  f )
1
 (5e  f ) 2
t  3
(t  5) (t  3) t  5

t  3
t  5
t  5

(25e2  f 2 )  5e

16.

7q

p2

2a  4b
5

p2

 14qr3  2r2p
 4r5p

 3a
10.

3x2

 18z  2yz
x4y

3(a  b)

a

a
a  b

2yz
3x2

 12yz2

5.
2

 15
2

x2y
18z

45

 3a  2b

753

7
x2  16

x
16  x2
(x  4) (x  4) x
 (x2  16)  7
(x  4) (x  4)
x
 (x  4) (x  4)  7
x
 1  7
x
 7

Extra Practice

PQ249-6481F-P27-28[750-765] 26/9/02 8:58 PM Page 754 Sahuja 79:PQ249:PQ249J:PQ249J-ch15-28-Repro:

17.

18.

y
5

y2  25
5  y
y
5  y
 5  y2  25
y
1(y  5)
 5  (y  5) (y  5)
y
 5(y  5)
y
 5y  25

3m
m  1

t6
8.

3m

(2r2

 3r  35)
(2r  7) 


9.

1
 2

12n2  36n  15
6n  3

20.

21.

a2  3a  10
a2  3a  10

a2  2a  3
a2  3a  2
(a  5) (a  2) (a  3) (a  1)
 (a  2) (a  1)  (a  5) (a  2)
a  3
a2
x2  x  2
x2  6x  8

x2  x  12
x2  4x  3
(x  2) (x  1) (x  4) (x  3)
 (x  3) (x  1)  (x  4) (x  2)

1

Lesson 12-5

x  7  2x
1. 2x  32x2  11x  20
() 2x2  3x
14x  20
() 14x  21
1

1
 3

x2  2x  35
x  7
(x  7) (x  5)
x  7

c2  6c  27
c  9
(c  9) (c  3)
c  9

c3
y  13 
2  6y  25
y

7y
6.
() y2  7y
13y  25
() 13y  91
66

Extra Practice

4
 2

4b2  b  b  2
16. b  24b3  7b2  2b  4
() 4b3  8b2
b2  2b
() b2  2b
04

x5
5. (c  6c  27)
(c  9) 

(2m  3) (2m  5)


2m  3
 2m  5

3c  2  9c
2
15. 9c  227c  24c  8
() 27c2  6c
18c  8
() 18c  4
4

m2  4m  5
m  5
(m  5) (m  1)
m  5

m1

4m2  4m  15
2m  3

3. (m2  4m  5)
(m  5) 

13.

(5x  7) (2x  3)

3t2  2t  3  2t  3
3
14. 2t  36t  5t2  0t  12
() 6t3  9t2
4t2  0t
() 4t2  6t
6t  12
() 6t  9
3

a 7
2. a  3a2  10a  21
() a2  3a
7a  21
() 7a  21
0

4. (x2  2x  35)
(x  7) 

10.

10x2  29x  21
5x  7


5x  7
 2x  3
t2  4t  1
11. 4t  14t3  17t2  0t  1
() 4t3  t2
16t2  0t
() 16t2  4t
4t  1
() 4t  1
0
2a2  3a  4
12. a  32a3  9a2  5a  12
() 2a3  6a2
3a2  5a
() 3a2  9a
4a  12
() 4a  12
0

m2  6m  16
2m  16

m2  m  6
m  2
2(m  8) (m  3) (m  2)
 m  2  (m  8) (m  2)
2m  6
 m2

Page 847

(6n  3) (2n  5)
6n  3

 2n  5

3m

 m2  m  2
19.

2r2  3r  35
2r  7
(2r  7) (r  5)
2r  7

r5

(m  2)

 m  1m

3t2  14t  24
3t  4
(3t  4) (t  6)
3t  4

7. (3t2  14t  24)


(3t  4) 

66
y  7

754

PQ249-6481F-P27-28[750-765] 26/9/02 8:58 PM Page 755 Sahuja 79:PQ249:PQ249J:PQ249J-ch15-28-Repro:

17. t

t2  4t  3  t
 19t  9

 4t3  0t2
() t3  4t2

3
 4

12.

18. 3x




6y
 y

6x  6y
x  y

6(x  y)
x  y

6(x  y)
x  y
1

6
13.

3x2  2x  2  3x
0x2
6x2

x

4t2  19t
() 4t2  16t
3t  9
() 3t  12
3

 29x3
() 9x3

6x
x  y

5x
24

3x

 24 


6
 2

5x  3x
24
2x
24
1

 2x  10

2x

 24
12

6x2  2x
() 6x2  4x
6x  10
() 6x  4
6

 12
14.

7p
3

8p
3




7p  8p
3
p
3

15.

8k
5m

3k

 5m 


8k  3k
5m
5k
5m
k

m

Page 848
1.

4
z

16.

Lesson 12-6

3
z

4  3
z
7
z




2.

a
12

2a
12




a  2a
12
3a
12

17.

7
2t





5  7
2t
2
2t

4.

y
2

18.

a
4
y  y
2
2y
2

2


6.

y
2

2
x




y  6
2

20.




2a
2a  5
1
4z  1

 2a

7.

x
x  1

1
x  1




x
x
x
x






1
1
1
1

22.

8.





2n
2n 
2n
2n 
2n 
2n 

3a
a  2
n
n  1

3a
 2

a
1

1
 n

5
5
5
5




23.

5
(2n  5)
5
2n  5

x  y
2  y

x  y
 2

y

10.

r2
r  s

11.

12n
3n  2

r

s2
 s

 3n




r2  s2
r  s

8
 2

7
 7

a  7

a7
1
24.

2a
6a  3

3

(1)
 6a

2a

2a

 6a  3  3  6a
 6a  3  6a  3
2a  1

 6a  3

x  y
 2

y

2a  1

 3(2a  1)
1

2a  1

 3(2a  1 )

12n  8
3n  2
4(3n  2)
3n  2

4(3n  2)
3n  2

(7)

7aa77a
a7a

y  x
 2
2y
y  2

y

a
a  7

n
1
n1
 1
n  (1)
n  1
n  1
n  1

n


1

0

2n  5
 5

 2n

9.

1  4z

(4z)

5
5  2n

 2a

 4z  1  4z  1

1

y3
2n
2n  5

2a  5
 5

5
 5

4z  1

21.

y  (y  6)
2
2y  6
2
2(y  3)
2

a  2  a  3
6
1
6

 4z  1
1

b  2
x

a  3
6

y

b
x

 t
5.

19.

2y
2

a  2
6

21
2t

y  2y
b  6
y
b  6

2y

b6



5
2t

y
b  6

3a
12
4

3.

8  6

m2m2
m2

8
m  2

3

4

755

Extra Practice

PQ249-6481F-P27-28[750-765] 26/9/02 8:58 PM Page 756 Sahuja 79:PQ249:PQ249J:PQ249J-ch15-28-Repro:

Page 848

11. 3t  2  3t  2
t2  4  (t  2)(t  2)
LCD  (3t  2)(t  2) (t  2)

Lesson 12-7

27a2bc

 333aabc
36ab2c2  2  2  3  3  a  b  b  c  c
LCM  2  2  3  3  3  a  a  b  b  c  c  108a2b2c2
2. 3m  1  3m  1
6m  2  2(3m  1)
LCM  2(3m  1)  6m  2
1.

t  2
3t  2
 t2  4
3t  2
(3t  2) (t  2) (t  2)
(t  2) (3t  2)
 (3t  2) (t  2) (t  2)  (3t  2) (t  2) (t  2)
(t  2) [ (3t  2) (t  2)  (3t  2) ]

(3t  2) (t  2) (t  2)
(3t  2) (t  2)  3t  2

(3t  2) (t  2)
3t2  6t  2t  4  3t  2

3t2  6t  2t  4
3t2  t  6
 3t2  8t  4

3. x2  2x  1  (x  1) (x  1)
x2  2x  3  (x  3) (x  1)
LCM  (x  1) (x  1) (x  3)  (x  1) 2 (x  3)

4. LCD  21
s
3

2s
7





7(s)
3(2s)
 21
21
7s  6s
21
13s
21

12. a  5  a  5
a2  5a  a(a  5)
LCD  a(a  5) or a2  5a
3
a  5

5. 2a  2a
6a  3(2a)
LCD  3(2a) or 6a
5
2a

3
6a

3(5)

 3(2a) 

6


2n
5

3
6a
15  3
6a
12
2
or a
6a

3m
4

15

7
10x2




14. 7w  7  w  w
ww
LCD  7  w  w or 7w2
3z
7w2




t  3
s




 2b

s
t2

a

3
 3

a
a2  4




Extra Practice

4(a  2)
a2  4
a  (4a  8)
a2  4
3a  8
a2  4
a

 a  2  a2  4 



17. m  n  m  n
mm
LCD  m(m  n)

a(2b  3a)
b(a  b)
 3a2  ab  2b2
 2b2
2ab  3a2  ab  b2
3a2  ab  2b2
3a2  3ab  b2
3a2  ab  2b2

m
m  n

m(m)

5(m  n)

 m  m(m  n)  m(m  n)



m2  (5m  5n)
m(m  n)
m2  5m  5n
m(m  n)

18. y  5  y  5
y2  25  (y  5)(y  5)
LCD  (y  5)(y  5) or y2  25

4a
3(2)
 2a  6
 6
4a  6
2a  6
2(2a  3)
2(a  3)
2a  3
a  3

 2a


3(s)
t(r)
 3t2
3t2
3s  rt
3t2

16. a  4  (a  2)(a  2)
a2a2
LCD  (a  2)(a  2) or a2  4

10. 2a  6  2(a  3)
a3a3
LCD  2(a  3) or 2a  6
4a
2a  6

 3t 


 3a2  ab


7w(2z)
7w2
3z  14wz
7w2

2(s)
t(t  3)
 st
st
2
2s  t  3t
st

b
 3a

3z

 7w2 

15. t  t  t
3t  3  t
LCD  3  t  t  3t2

9. a  b  a  b
2b  3a  2b  3a
LCD  (a  b)(2b  3a)
 2ab  3a2  2b2  3ab
 3a2  ab  2b2
a
a  b

2z
w

8. LCD  st
2
t

2x(6)
7
 10x2
10x2
12x  7
10x2

5(z)
6(x)
 xyz
xyz
5z  6x
xyz

 yz 

4(2n)
5(3m)
 20
20
8n  15m
20




7. LCD  xyz
5
xy

3a  6
a2  5a

6. 5x  5x
10x2  2  5  x  x
LCD  2  5  x  x or 10x2
6
5x

6

13. LCD  20

3
6a

 6a 


3(a)

 a2  5a  a2  5a  a2  5a

y  5
y  5

2y

 y2  25 



756

(y  5) (y  5)
2y
 y2  25
y2  25
y2  10y  25  2y
y2  25
y2  8y  25
y2  25

PQ249-6481F-P27-28[750-765] 26/9/02 8:58 PM Page 757 Sahuja 79:PQ249:PQ249J:PQ249J-ch15-28-Repro:

19. t2  100  (t  10)(t  10)


10  t  1(t  10)
LCD  1(t  10)(t  10)
t  10
t2  100

 10

1
 t

10.

1(t  10)
 10) (t  10)

 1(t

x  3
x  1
x2
y2

t  10
 10) (t  10)

 1(t

t  10  t  10
 10) (t  10)

x  3 y2

 1 x2
y2 (x  3)
x2 (x  1)

1(2t  20)
1(t  10) (t  10)

t

y2

x

 1(t


x2

x  3
 1

x

11.

2
 10

y
3

2y  5
6
2y  5
y
2y  5
6

2y  5
6

6

2

20. a2  3a  10  (a  5)(a  2)
a2  4a  12  (a  6)(a  2)
LCD  (a  2)(a  5)(a  6)

5
y

2a2  6a  12a  36  (3a2  5a  15a  25)


(a  2) (a  5) (a  6)

2a2  18a  36  3a2  10a  25


(a  2) (a  5) (a  6)

a2  8a  61
(a  2) (a  5) (a  6)

1. 4 


5

2. 8  3t 

3.

b  1
2b

12.

1
x
1
y




1
y
1
x

y  x
xy
x  y
xy

y  x
xy

x  y
xy

y  x
xy

x

xy
 y

y  x
x  y

4x
2
x
x
4x  2
x


4. 3z 

z  2
z




5.

2
a  2

7.

44




t2

3z2
z  2
 z
z
3z2  z  2
z

14.

aa1
3

aa2

7
2
19
4
7
19

4
2

Page 849
1.

14

 19
x2
y

x3

x2
y
x5
y2

x3
y

a2 (a  2)
a  2
2  a3  2a2

a  2
a3  2a  2

a  2
4
3r2 (2r  1)
4

 2r  1
2r  1
2r  1
6r3  3r  4
 2r  1

9.

t  2
t  3

t
t3

 2  19

8.

t2
t  2
 3
t  3
2t  2

t2

(t  2) (t  2)
(t  2) (t  3)

t2
t

b  1
3b  2b
 2b
2b
6b2  b  1
2b

x2
y
y
x3

t2  4
t2  5t  6

 a2  a  2 

6. 3r2 

32

13.

8  3t
5
 3t
3t
24t  5
3t

 3b 

Lesson 12-8


 2y  5

6

2
x

2y  5
y

3a  5
2a  6
 a2  4a  12
a2  3a  10
(a  6) (2a  6)
(a  5) (3a  5)
 (a  2) (a  5) (a  6)  (a  2) (a  5) (a  6)

Page 848

t4
u
t3
u2

t4
u

t3

u2
3.
1

t
u
1

 tu

u
t3

a(a  1)  2
a  1
a(a  2)  3
a  2
a2  a  2
a  1

a2  2a  3
a  2
a2  a  2
a2  2a  3

a  1
a  2

a2  a  2
a  1

(a2  a  2) (a  2)
(a  1) (a2  2a  3)

Lesson 12-9

k
2k
 3
6
k
2k
 3
6

5
2
5
2

2  1 26

2.

4.

70

k  4k  15
5k  15
k  3
18
b
18
b

b3

1 21

3
b

3b

18  3  3b
15  3b
5b

757

a  2
 3

 a2  2a

2x
27
 10
7
2x
27
 10
7

4x
5
4x
5

2  1 270

20x  189  56x


189  36x
5.25  x

10x

3
7
 2x
5x
3
7
 2x
5x

1

2  10x(1)

6  35  10x
41  10x
41
x
10

Extra Practice

PQ249-6481F-P27-28[750-765] 26/9/02 8:58 PM Page 758 Sahuja 79:PQ249:PQ249J:PQ249J-ch15-28-Repro:

5.
6

2a  3
6
2a  3
6

2a
3

2  61

2
2a
3

1
2

2a  3  4a  3
6  2a
3  a

7.
14

2b  3
b
2
7
2b  3
b
2
7

b  3
14
b  3
14 14

2(2b  3)  7b  b  3
4b  6  7b  b  3
3b  6  b  3
9  4b

8.
5(y  4)

x  3
x
4x  5
x

5

13.

5

4x  5  5x
5x

2 1

9
4

3x  2
x

6.

(x  3)

t(t  3)

3

15.

2

2  t(t  3) (2)

x(x  1)

5x
1
x
x  1
5x
1
x
x  1

(r  2)(r  9)

(3x  10) (x  5)

(m  1) (m  1)

2x
x  5

2

2  (3x  10)(x  5) (2)

 5x  100  0
x2  x  20  0

(x  5)(x  4)  0
x  5 or x  4
2a  3
a  3

17.
(a  2) (a  3)

12

2a2

12aa 33 22  (a  2)(a  3) 1a 12 2 2

2a2  4a  3a  6  2a2  6a  4a  12  12a  36

6

3a  6  12a  36
42  9a

5
 1

5
m  1

18.
(z  1) (z  3)

1

2  (m  1) (m  1) (1)

m

z  3
z  1
z  3
z  1

z  1
z  3

z  1
z  3

42
9

a

14
3

a

2

2  2(z  1) (z  3)

(z  3) (z  3)  (z  1)(z  1)  (2z  2) (z  3)
z2  9  z2  1  2z2  6z  2z  6
2z2  10  2z2  8z  6
8z  16
z2

m(m  1)  5(m  1)  m2  1
m2  m  5m  5  m2  1
4m  5  1
4m  6

Extra Practice

2x
 5

11x2  45x  6x2  50x  100

2  (r  2) (r  9)6

5x
3x  10

x

(a  2) (2a  3)  2(a  2) (a  3)  (a  3)12

r  2
2r
r9
r  2
r  2
2r
r9
r  2

m
m  1

5x
3x  10

5x2

x

m

5x2  25x  6x2  20x  6x2  30x  20x  100

5

m
m  1

 3  2

5x(x  5)  2x(3x  10)  (6x  20)(x  5)

(r  9) (r  2)  2r(r  2)  6(r2  11r  18)


r2  7r  18  2r2  4r  6r2  66r  108
r2  3r  18  6r2  66r  108
0  7r2  63r  126
0  r2  9r  18
0  (r  6) (r  3)
r  6 or r  3
12.

16.

x(5x)  x  1  5x(x  1)
5x2  x  1  5x2  5x
x  1  5x
1  4x

11.

1b 14 6 2  2(b  6)(b  8) 112  b 6 8 2

a
4a  15

2  x(x  1)(5)

1
4

2b8

(4a  15) 4a  15  3  (4a  15)(2)


a  3(4a  15)  8a  30
a  12a  45  8a  30
11a  45  8a  30
15  3a
5  a

t(2t)  3(t  3)  2t(t  3)


2t2  3t  9  2t2  6t
3t  9  6t
9  3t
3t
10.

14
b  6

2(b  8) (14)  (b  6) (b  8)  2(b  6)(6)


28(b  8)  b2  14b  48  12(b  6)
28b  224  b2  14b  48  12b  72
28b  224  b2  2b  24
0  b2  30b  200
0  (b  10)(b  20)
b  10 or b  20

2  5(y  4)(3)

2t
3
t
t  3
2t
3
t
t  3

 12

2x  4x  12x  36
6x  12x  36
6x  36
x6

2(b  6) (b  8)

10y  3(y  4)  15( y  4)


10y  3y  12  15y  60
7y  12  15y  60
72  8y
9y
9.

 12

2  12(x  3)

14.

b

2y
3
5
y  4
2y
3
5
y  4

2x
4x
3x
x  3
2x
4x
x3
x  3
2x
4x
x3
x  3

3
2

758

PQ249-6481F-P27-28[750-765] 26/9/02 8:58 PM Page 759 Sahuja 79:PQ249:PQ249J:PQ249J-ch15-28-Repro:

Page 849

4. The matrix has 1 row and 1 column. Therefore, it


is a 1 by 1 matrix.
5. Matrices A and B have different dimensions;
therefore, it is impossible to add them.
2 4
1 0
6. A  C  c
d  c
d
3
5
0 1

Lesson 13-1

1. The sample is 10 dogs from a county. The


population is all dogs from a county. The sample
is biased because not every dog in the county has
an equal chance of being selected. Since the
sheriff checked the first ten dogs near his office,
the sample is a convenience sample.
2. The sample is 25 students. The population is all
students at the school. The sample is biased, since
the students are self-selecting. The sample is a
voluntary response sample, since the students
respond to the school bulletin.
3. The sample is ice cream cones made during three
shifts. The population is all ice cream cones made
during three shifts. The sample is unbiased since
every ice cream cone has an equal chance of being
selected. The sample is a systematic random
sample since every tenth cone is selected for
weighing.
4. The sample is cars from an assembly line. The
population is all cars from an assembly line. The
sample is unbiased since every car is equally
likely to be selected. Since the cars are identified
by the day of the week, the sample is a stratified
random sample.
5. The sample is people who return the survey. The
population is all people entering the department
store. The sample is biased since the respondents
are self-selecting. Since people can decide
whether to return the survey, the sample is a
voluntary response sample.
6. The sample is some residents in a community.
The population is all residents of the community.
The sample is unbiased since every resident is
equally likely to be selected. The sample is a
systematic random sample since every twentieth
person is surveyed.
7. The sample is malformed frogs identified by
residents and reported. The population is all
malformed frogs in the states lakes. The sample
is biased, since not every frog has an equal chance
of being selected. The sample is a voluntary
response sample, since residents choose to report
sightings of malformed frogs.
8. The sample is 10 cartons of strawberries. The
population is all cartons of strawberries in the
store. The sample is biased since not every carton
of strawberries has an equal chance of being
selected. Since the top ten cartons of strawberries
were taken from the shelf, the sample is a
convenience sample.

Page 849

 c

2  1 4  0
d
3  0
51

 c

3 4
d
3
6
1 1
4
5 1 4
d  c
d
0
3 2
3 0
2

7. B  D  c
 c

1  (5)
0  (3)

 c

4 0 0
d
3 3 0
5 1 4
1 1
4
d  c
d
3 0
2
0
3 2

8. D  B  c
 c

5  1 1  (1)
4  4
d
3  0
0  3 2  (2)

 c

6
2 8
d
3 3
4

9. 2B  2 c

1
3

1
0

2(1)
2(0)

 c

2 2
8
d
0
6 4

3(0)
d
3(1)

 c

3(1)
3(0)

 c

3 0
d
0 3
2 4
1 0
d  c
d
3
5
0 1

 c

2  1 4  0
d
3  0
51

 c

1 4
d
3
4

12. 5c  5 c

1 0
d
0 1

 c

5(1)
5(0)

 c

5
0
d
0 5

13. 2A  C  2 c

759

2(4)
d
2(2)

2(1)
2(3)

1 0
d
0 1

11. A  C  c

1. The matrix has 1 row and 4 columns. Therefore,


it is a 1 by 4 matrix.
2. The matrix has 2 rows and 2 columns. Therefore,
it is a 2 by 2 matrix.
3. The matrix has 3 rows and 3 columns. Therefore,
it is a 3 by 3 matrix.

4
d
2

 c

10. 3c  3 c

Lesson 13-2

1  1 4  (4)
d
30
2  2

5(0)
d
5(1)

2 4
1 0
d  c
d
3
5
0 1
2(4)
1 0
d  c
d
2(5)
0 1

 c

2(2)
2(3)

 c

4  1 8  0
d
6  0 10  1

 c

5 8
d
6 11

Extra Practice

PQ249-6481F-P27-28[750-765] 26/9/02 8:59 PM Page 760 Sahuja 79:PQ249:PQ249J:PQ249J-ch15-28-Repro:

5
3

1 4
1 1
4
d  c
d
0
2
0
3 2

5. Sample answer:

3(5)
3(3)

 c

15  1 3  (1) 12  4


d
9  0
0  3 6  (2)

 c

16
9

3(1)
3(0)

Number of Defective
Light Bulbs Per Shift

3(4)
1 1
4
d  c
d
3(2)
0
3 2

 c

Frequency

14. 3D  B  3 c

4 16
d
3
8

15. Matrices B and C have different dimensions,


therefore, it is impossible to add them.
1 0
2 4
16. 2C  3A  2 c
d  3c
d
0 1
3
5
2(1)
 c
2(0)

2(0)
3(2)
d  c
2(1)
3(3)

26
 c
0  (9)

10
8
6
4
2
0
02

35

68

911

1214

Number of Bulbs

3(4)
d
3(5)

Page 850

0  (12)
d
2  15

Lesson 13-4

1. Order the data from least to greatest.


10 33 34 37 43 45 45 50
The range is 5610 or 46.
The median is the middle value, or 43.

8 12
 c
d
9
17

56

33  34
or 33.5.
2
45  50
is
or 47.5.
2

The lower quartile is

Page 850

Lesson 13-3

The upper quartile

1. The data set has a total of 2  6  1  2  0.5 


11.5 values, so the median is the average of the
sixth and seventh values. Both values occur in
the range 10001500, so the median occurs in
that range as well. The data appears to be skewed
to the left.
2. The data set has 2  4  7  5  14  11  4 
4  51 values, so the median is the 26th value.
The median occurs in the range 7075. About half
of the data lie in the 7080 percent range.
3. Sample answer:

The interquartile range is 47.5  33.5 or 14.


The outliers would be less than 33.5  1.5(14) or
12.5 or greater than 47.5  1.5(14) or 68.5. Since
10  12.5, 10 is the only outlier.
2. Order the data from least to greatest.
65 65 68 77 78 84 84 95 96 99
The range is 99  65 or 34.
The median is

Frequency

Prices of Notebooks

2
1
0
2741

4256

5771

7286

87101

The median is

Price in Cents

The

4. Sample answer:

The

Frequency

Number of Fish in Each Tank

4
2
0
916

1724

2532

3340

4148

4956

Number of Fish

Extra Practice

60  70
2

or 65.

40  50
lower quartile is
or 45.
2
80  90
or 85.
upper quartile is
2

The interquartile range is 85  45 or 40.


The outliers would be less than 45  1.5(40) or
15 or greater than 85  1.5(40) or 145. There
are no outliers.
4. Order the data from least to greatest.
1 1.3 2.4 3 3.7 4 5.2 6 7.1 8 9
The range is 9  1 or 8.
The median is the middle value or 4.
The lower quartile is 2.4.
The upper quartile is 7.1.
The interquartile range is 7.1  2.4 or 4.7.
The outliers would be less than 2.4  1.5(4.7) or
4.65 or greater than 7.1  1.5(4.7) or 14.15.
There are no outliers.

14
12
10
8
6

18

or 81.

The lower quartile is 68.


The upper quartile is 95.
The interquartile range is 95  68 or 27.
The outliers would be less than 68  1.5(27) or
27.5 or greater than 95  1.5(27) or 135.5. There
are no outliers.
3. Order the data from least to greatest.
30
40
50 60
70
80
90 100
The range is 100  30 or 70.

5
4
3

1226

78  84
2

760

PQ249-6481F-P27-28[750-765] 26/9/02 8:59 PM Page 761 Sahuja 79:PQ249:PQ249J:PQ249J-ch15-28-Repro:

3. Order the set from least to greatest.

5. Order the data from least to greatest.


0 4 13 25 29 31 37 44 56 66 73
The range is 73  0 or 73.
The median is the middle value or 31.
The lower quartile is 13.
The upper quartile is 56.
The interquartile range is 56  13 or 43.
The outliers would be less than 13  1.5(43) or
51.5 or greater than 56  1.5(43) or 120.5.
There are no outliers.
6. Order the data from least to greatest.
112 234 268 369 400 406 527 648 775
The range is 775  112 or 663.
The median is the middle value or 400.

1.2

Q1 

234  268
or 251.
2
527  648
or 587.5.
is
2

55

49  55
2

3.2

1.7  1.8
2

 1.75 Q2 

3.5

4.2

5.5

2.2  2.6
2

 2.4 Q3 

3.2  3.5
2

 3.35

0 10 20 30 40 50 60 70 80 90 100

55

c
Q 

3.0

35  35

5. Order the set from least to greatest.


A: 21 24 27 34 40 46 50 58 61 67 70 72
c
c
c
Q1 

59
c

 52 Q 
2

55  59
2

63

27  34
2

Q1 

69

69

69

69  69
2

46  50
2

 48 Q3 

61  67
2

 76.5 Q3 

81  82
2

 64

69  72
2

 70.5 Q2 

75  78
2

 81.5

The interquartile range for set B is 81.5  70.5 


11. Check to see if there are any outliers.
70.5  1.5(11)  54
81.5  1.5(11)  98
There are no outliers.

89

c
 57 Q3 

 30.5 Q2 

The interquartile range for set A is 64  30.5 


33.5. Check to see if there are any outliers.
30.5  1.5(33.5)  19.75 64  1.5(33.5)  114.25
There are no outliers.
Order the set from least to greatest.
B: 67 69 69 72 74 75 78 79 81 82 83 83
c
c
c

2. Order the data from least to greatest.


55

2.6

Q1
Q2 
 35 Q3
2
The interquartile range is 52  22 or 30. Check
to see if there are any outliers.
22  1.5(30)  23
52  1.5(30)  97
There is one outlier, 99.

0 2 4 6 8 10 12

49

2.2

4. Order the set from least to greatest.


15 15 18 22 25 25 35 35 37 50 52 65 69 99
c
c
c

Lesson 13-5

45

1.8

1. Order the data from least to greatest.


1 2 2 3 3 4 4 5 7 8 8 9 10 11 12
c
c
c
Q2
Q3
Q1
The interquartile range is 9  3 or 6. Check to see
if there are any outliers.
3  1.5(6)  6
9  1.5(6)  18
There are no outliers.

40

1.8

The interquartile range is 3.35  1.75 or 1.6.


Check to see if there are any outliers.
1.75  1.5(1.6)  0.65
3.35  1.5(1.6)  5.75
There are no outliers.

The interquartile range is 587.5  251 or 336.5.


The outliers would be less than 251  1.5(336.5)
or 253.75 or greater than 587.5  1.5(336.5) or
1092.25. There are no outliers.

Page 850

1.7
c

The lower quartile is


The upper quartile

1.2

 69

The interquartile range is 69  52 or 17. Check to


see if there are any outliers.
52  1.5(17)  26.5
69  1.5(17)  94.5
There are no outliers.

A
B
20

30

40

50

60

70

80

90

The A data are much more diverse than the B


data. In general, the B data are greater than the
A data.

40 50 60 70 80 90

761

Extra Practice

PQ249-6481F-P27-28[750-765] 26/9/02 8:59 PM Page 762 Sahuja 79:PQ249:PQ249J:PQ249J-ch15-28-Repro:

6. Order the set from least to greatest.

8. Order the set from least to greatest.

A: 60 65 70 72 75 76 83 85 88 91 92 92 92 99 100
c

Q1

Q2

Q3

A:

Q1 

5.25

6.25

6.50

3.25  4.25
2

 3.75

7.95

8.25

Q3 

6.50  7.95
2

 7.225

7.225  1.5(3.475)  12.4375

There are no outliers.


Order the set from least to greatest.
B:

3.25

4.25

4.50

5.25

5.50

c
Q1 

4.25  4.50
2

5.75

6.95

c
 4.375

8.65

9.50

Q3 

6.95  8.65
2

 7.8

The interquartile range is 7.8  4.375  3.425.


Check to see if there are any outliers.
4.375  1.5(3.425)  0.7625

7.8 + 1.5(3.425)  12.9375

There are no outliers.

A
B
70

60

80

90

100

3.00 4.00 5.00 6.00 7.00 8.00 9.00

The B data are more diverse than the A data. The


data sets have approximately the same range.
7. Order the set from least to greatest.
0.5

0.8

0.8

1.1

1.5

c
Q1 

0.8  0.8
2

2.2

2.2

2.3

c
 0.8 Q2 

3.0

3.6

The distribution of both sets are similar. The


values for B are somewhat greater than the
values for A.

3.8

1.5  2.2
2

 1.85 Q  2.3
3

 3.0
2

 2.65

Page 851

The interquartile range is 2.65  0.8  1.85.


Check to see if there are any outliers.
0.8  1.5(1.85)  1.975
2.65  1.5(1.85)  5.425
There are no outliers.
Order the set from least to greatest
0.9

4.75

3.75  1.5(3.475)  1.4625

B:

4.25

The interquartile range is 7.225  3.75  3.475.


Check to see if there are any outliers.

0.4

3.25
c

The interquartile range is 92  72  20. Check to


see if there are any outliers.
72  1.5(20)  42
92  1.5(20)  122
There are no outliers.
Order the set from least to greatest.
B: 62 65 73 77 77 81 82 85 85 88 91 92 95 98 99
c
c
c
Q1
Q2
Q3
The interquartile range is 92  77  15. Check to
see if there are any outliers.
77  1.5(15)  54.5
92  1.5(15)  114.5
There are no outliers.

A:

2.65

1.2

1.6

1.8

1.9

2.5

c
Q1 

3.3

3.8

4.0

1.6  1.8
2

 1.7 Q2 

5.4

5.7

Lesson 14-1

1.

ice cream
chicken

cookies
ice cream
6.0

lettuce

beef

2.5  3.3
2

 2.9 Q3 

4.0  5.4
2

pudding

pudding
cookies

 4.7

The interquartile range is 4.7  1.7  3.0. Check


to see if there are any outliers.
1.7  1.5(3)  2.8
4.7  1.5(3)  9.2
There are no outliers.

ice cream
fish

pudding
cookies
ice cream

chicken

pudding
cookies

ice cream
0

coleslaw

beef

The B data are more diverse than the A data. In


general, the B data are greater than the A data.

pudding
cookies
ice cream

fish

pudding
cookies

The tree diagram shows that there are 18 possible


outcomes.

Extra Practice

762

PQ249-6481F-P27-28[750-765] 26/9/02 8:59 PM Page 763 Sahuja 79:PQ249:PQ249J:PQ249J-ch15-28-Repro:

2.

H
H
T
H
H
T
T
H
H
T
T
H
T
T

4.

H
T
H
T
H
T
H
T
H
T
H
T
H
T
H
T

M
C
B

S
M

The tree diagram shows that there are 12 possible


outcomes.
5. There are ten digits and 26 letters. Multiply to
find the number of possible outcomes.
10  10  26  26  67,600
There are 67,600 possible license plates.
6. Multiply to find the number of ways.

The tree diagram shows that there are 16 possible


outcomes.
3.

W
P
W
P
W
P
W
P
W
P
W
P

W
Y
B
R
G
W
Y
B
R
G
W
Y
B
R
G
W
Y
B
R
G
W
Y
B
R
G

rocking
or
non-rocking
1442443

swivel
or
non-swivel
1442443

cotton, leather
or plush
cover
144424443

green, blue,
maroon,
or black
1442443

2
2
3



There are 48 possible lounge chairs.
7. Multiply to find the number of ways.
boots skis poles
123 123 123

 48

3  4  5  60
There are 60 possible selections.
8. There are 4 possible outcomes for each die.
Multiply.
red die blue die white die
1
424
3 14243 14
424
43

9.
10.
11.
12.
13.
14.

The tree diagram shows that there are 25 possible


outcomes.

15.
16.

4
4
4


 64
There are 64 possible outcomes.
8!  8  7  6  5  4  3  2  1
 40,320
1!  1
0!  1
5!  5  4  3  2  1
 120
2!  2  1
2
9!  9  8  7  6  5  4  3  2  1
 362,880
3!  3  2  1
6
14!  14  13  12  11  10  9  8  7  6  5  4  3  2  1
 87,178,291,200

Page 851

Lesson 14-2

1. This is a combination because order is not


important.
2. This is a permutation because order of runners
can make a difference.

763

Extra Practice

PQ249-6481F-P27-28[750-765] 26/9/02 8:59 PM Page 764 Sahuja 79:PQ249:PQ249J:PQ249J-ch15-28-Repro:

3. This is a combination because order is not


important.
4. This is a permutation because order of winning is
important.
5. This is a permutation because order is important
with letters.
6. This is a permutation because order is important
with a lock.
7. This is a combination because order is not
important.
8. 5P2 



8!

3!

3!

6

7!
(7  7)!
7!
0!

Page 851




40

39

1560

dependent. P(2 black):  55  54  2970 or

52
99

8. Since the first duck is not replaced, the events are


2

dependent. P(2 yellow)  55  54  2970 or

1
1485

9. Since the first duck is not replaced, the events are


40
40 1
dependent. P(black, then gold)  55  54  2970
4
or 297

13!
(13  13)!13!
13!
13!

40

39

38

474,240

10. P(3 blacks, then red)  55  54  53  52  8,185,320


304
or 5247

1
9!

11. P(yellow, then blue, then gold)


2
4
1
8
4
 55  54  53  157,410 or 78,705
1

12. P(2 gold)  55  54  0

987654
1

24

13. P(4 blue)  55  54  53  52  8,185,320 or

 60,480
7!

4!

16. ( P )( P )  (7  3)!  (4  2)!


7 3 4 2
7! 4!
 4!  2!
765
1

1
341,055
24

14. P(4 blue, then gold)  55  54  53  52  51  417,451,320


1
or 17,393,805

43
1

 2520

Extra Practice

7. Since the first duck is not replaced, the events are

18!
(18  10)!10!
18!
8!10!
18  17  16  15  14  13  12  11
8!

1
18

40  55 ducks. P(red, then gold)  55  54  2970


4
or 1485

15. P  (9  6)!
9 6
9!
 3!


6. Since the first duck is not replaced, the events are


dependent. There are a total of 8  2  1  4 

 43,758
13 13

1
4

5. P(red greater than 2, blue greater than 3)


4 3
1
12
 6  6  36 or 3

 56

14.

1
4

4. P(red 6, blue greater than 4)  6  6  36 or

8!
(8  2)!
8!
6!
87
1

3. There are 3 prime numbers (2, 3, 5) on a die.


3 3
9
P(red prime number, blue even)  6  6  36 or

6!
(6  5)!5!

10  9
2!

C 
18 10

2. P(red even, blue even)  6  6  36 or

6

13.

Lesson 14-3

1. P(red 1, blue 1)  6  6  36

 45

10!

19. ( P )( C )  (3  2)!  (10  10)!(10!)


3 2 10 10
3! 10!
 1!  10!

10!

10!

 3  10!
 10,886,400

10. 10C2  (10  2)!2!


10!
 8!2!

12. 8P2 

76
2!

18. ( C ) ( P )  (3  2)!2!  (10  10)!


3 2 10 10
3! 10!
 2!  0!

 7!
 5040

11. 6C5 

 28  21
 588

5!
(5  2)!
5!
3!
54
1

87
2!

 20
9. P 
7 7

7!

17. ( C ) ( C )  (8  6)!6!  (7  5)!5!


8 6 7 5
8!
7!
 2!6!  2!5!

764

PQ249-6481F-P27-28[750-765] 26/9/02 8:59 PM Page 765 Sahuja 79:PQ249:PQ249J:PQ249J-ch15-28-Repro:

Page 852
1.


1
2
3
4
5
6

4. See students work.

Lesson 14-4
1
1
2
3
4
5
6

2
2
4
6
8
10
12

3
3
6
9
12
15
18

4
4
8
12
16
20
24

5
5
10
15
20
25
30

5.

6
6
12
18
24
30
36

1
36

There are 4 ways to roll a product of 12.


P(X  12) 

4
36

or

2
36

or

Ways

Product

Ways

18

20

10

24

12

25

15

30

16

36

P(product of 12) 

1
18

P(product greater than 15) 

11
36

11

4
36
4
36

or about 0.11
or about 0.11

Since these theoretical probabilities are close to


10%, we would expect to get a product of 6 about
10% of the time and a product of 12 about 10% of
the time.

3. There are 1  2  2  2  1  2  1  11
different ways to roll a product greater than 15.
121

 36  1296

6. P(product is 2) 

4. Let X  the number of customers.


X can have the values of 500, 1000, 1500, 2000,
and 2500.
5. For each value of X, the probability is greater
than or equal to 0 and less than or equal to 1.
0.05  0.25  0.35  0.30  0.05  1,
So the probabilities add up to 1.
6. P(X 1000)  P(0 X 500)  P(501 X 1000)

2
36

or

1
18

7. There are a total of 1351 households.


P(1) 
P(2) 
P(3) 
P(4) 

172
1351
293
1351
482
1351
256
1351

or about 12.7%
or about 21.7%
or about 35.7%
or about 18.9%

P(5 or more) 

 0.05  0.25
 0.30

7.

Product

P(product of 6) 

1
9

There are 2 ways to roll a product of 24.


P(X  24) 

Ways

As you roll the dice more times, the experimental


probabilities should get closer to the theoretical
probabilities. Consider the following theoretical
probabilities.

2. There is 1 way to roll a product of 9.


P(X  9) 

Product

8. P(5 or more) 

P(X 7 500)  P(501 X 1000)  P(1001 X 1500) 

148
1351
148
1351

or about 11.0%
or about 0.11 or 11%

9. These events are mutually exclusive.


P(1 or 2)  P(1)  P(2)

P(1501 X 2000)  P(2001 X 2500)


 0.25  0.35  0.30  0.05

172

293

 1351  1351

 0.95

465

 1351 or about 0.34 or 34%

Page 852

Lesson 14-5

1. See students work.


2. See students work.
1

3. P(4 heads)  2  2  2  2  16

765

Extra Practice

PQ249J-6481F-P01-14[766-785] 26/9/02 9:07 PM Page 766 Sahuja Ahuja_QXP_06:Desktop Folder:Chandra:Algebra_FNL_Delivery:

Mixed Problem Solving


Page 853

8. Sample answer:
4 tank tops
8 shorts
4 t-shirts
at $6.99
at $7.99 plus at $4.99 plus

Chapter 1 The Language of Algebra

1. Sum implies add, and product implies multiply.


So the expression can be written as r2  r/.

14243 123 14
42443 123 1442443

2. Replace / in r2  r/ with r.

3.

8(7.99)

4(4.99)

4(6.99)

r2  r/  r2  r  r


 r2  r2
 (1  1)r2
 2r2

8 shorts
4 t-shirts
4 tank tops
at
$5.99
plus
at
$8.99
plus
at 2.99
14243 123 14243 123 1442443
8(5.99)

4(8.99)

4(2.99)

Money for
money for
money for
small tables
large tables
small tables
for first market plus for first market plus for second market
144424443 123 144424443 123 14444244443

14243 123 14243 123 14243 123 1442443

25(5)

10(8.5)

4 tank tops
4 shorts
4 shorts
4 t-shirts
at $7.99 plus at $5.99 plus at $4.99 plus at $2.99
4(7.99)

35(5)

money for
large tables
plus for second market
123 14444244443


12(8.5)

The expression is 25(5)  10(8.5)  35(5)  12(8.5).


4. 25(5)  10(8.5)  35(5)  12(8.5)
 125  85  175  102
 487
$487 was collected at the two markets.
5. Let a represent adult price, c represent child
price, and g represent observer price.
The cost of
the cost of
the cost of is less than
2 adults plus 2 children plus 1 observer or equal to $55.


2(c)

1(g)

2a  2c  g  55

16.95, 12.95,
4.95

2(16.95)  2(12.95)
?
 4.95  55
S 64.75  55

4(4.99)

4(2.99)

8(5.99)

8(2.99)
8(5.99)  8(2.99)  47.92  23.92
 71.84
The discount is 15% of the purchase.
15% of $71.84  0.15  71.84
 10.776
Subtract $10.78 from the purchase.
$71.84  $10.78  $61.06
The least amount you can spend is $61.06.
amount of sales
11. amount of sales
of
25
blankets
plus
of 25 rabbits
144424443 123 144424443

True or
False?
false

No, the family cannot go for a full day.

Mixed Problem Solving

8(7.99)

8(8.99)
8(7.99)  8(8.99)  63.92  71.92
 135.84
The discount is 15% of the purchase.
15% of $135.84  0.15  135.84
 20.376
Subtract $20.38 from the purchase.
$135.84  $20.38  $115.46
The greatest amount you can spend is $115.46.
8144
shorts
at4443
$5.99 1
plus
8 tops at $2.99
4424
23 144424443

55

The inequality is 2a  2c  g  55.


6. To find the cost for a full day, evaluate
2a  2c  g if a  16.95, b  12.95, and g  4.95.
2a  2c  g  2(16.95)  2(12.95)  4.95
 33.90  25.90  4.95
 64.75
A full day would cost $64.75.
To find the cost for a half day, evaluate
2a  2c  g if a  10.95, b  8.95, and g  3.95.
2a  2c  g  2(10.95)  2(8.95)  3.95
 21.9  17.9  3.95
 43.75
A half day would cost $43.75.
7. Replace a, c, and g in 2a  2c  g  55 with the
prices for a full day.
a, c, g

4(5.99)

144
4424
4443 123 144424443

1442443 123 1442443 123 1442443 1442443 123

2(a)

9. Sample answer:
8(7.99)  4(4.99)  4(6.99)
 63.92  19.96  27.96
 111.84
8(5.99)  4(8.99)  4(2.99)
 47.92  35.96  11.96
 95.84
4(7.99)  4(5.99)  4(4.99)  4(2.99)
 31.96  23.96  19.96  11.96
 87.84
The cost of the 16 items could be $111.84, $95.84,
or $87.84. All totals end in $0.84.
10. 8 shorts at $7.99 plus 8 tops at $8.99

25(28)

25(18)
25(28)  25(18)  25(28  18)
 25(46)
 1150
The total amount of sales was $1150.

766

PQ249J-6481F-P01-14[766-785] 26/9/02 9:07 PM Page 767 Sahuja Ahuja_QXP_06:Desktop Folder:Chandra:Algebra_FNL_Delivery:

12. 10 tickets

10 tickets
10 tickets
at $18.95 plus at $12.95 plus at $9.95 plus 30 combos

Page 854

10(18.95)  10(12.95) 

10(9.95)

30(5.50)

10(18.95)  10(12.95)  10(9.95)  30(5.50)


 189.5  129.5  99.5  165
 583.5
The total cost is $583.50.
13. If two lines are perpendicular, then they meet to
form four right angles. If two lines meet to form
four right angles, then they are perpendicular.
14. The domain contains the number of lawns Laurie
mows each week. She mows at most 30 lawns
each week. Therefore, a reasonable domain would
be values from 0 to 30 lawns. The range contains
the weekly profit from $0 to 30  $15 or $450.
Thus, a reasonable range is $0 to $450. Graph the
ordered pairs (0, 0) and (30, 450). Since she can
mow any amount of lawns up to 30 lawns, connect
the two points with a line to include those points.
Profit ($)

(30, $450)

400
300
200
100

x
5 10 15 20 25 30
Number of Lawns

15. Sample answer: The second graph is misleading


because the intervals for the years are not equal
and the intervals for the y-axis are not equal.

x
1980
2000

1960

0
1920
1940

Population Density

y
6
5
3
2
1

144424443

y
6
5
4

1980
2000

1960

x
1920

Population Density

14243

144424443

29,035
(1312)

29,035  (1312)  29,035  (1312)
 29,035  1312
 30,347
The difference is 30,347 feet.
5. To find the change in temperature, multiply the
number of 1000-foot increments in 10,000 feet by
the temperature drop in 1000 feet.
10(3.6)  36
The change in temperature is 36
F.
6. To find the altitude, find the difference of 70 and
38, then divide by 3.6.
[70  (38) ] 3.6  [ 70  (38) ] 3.6
 [ 70  38] 3.6
 108 3.6
 30
The altitude is 30  1000 or 30,000 feet.

Year

Chapter 2 Real Numbers

1. Since 18 15 13 11 2 14


19 31 34 38 39, the monthly normal
temperatures ordered from least to greatest are
18, 15, 13, 11, 2, 2, 14, 19, 31, 34, 38, 39.
2. 18 is eighteen units from 0 in the negative
direction. |18|  18
15 is fifteen units from 0 in the negative
direction. |15|  15
13 is thirteen units from 0 in the negative
direction. |13|  13
11 is eleven units from 0 in the negative
direction. |11|  11
2 is two units from 0 in the negative direction.
|2|  2
2 is two units from 0 in the negative direction.
|2|  2
14 is fourteen units from 0 in the positive
direction. |14|  14
19 is nineteen units from zero in the positive
direction. |19|  19
31 is thirty-one units from zero in the positive
direction. |31|  31
34 is thirty-four units from zero in the positive
direction. |34|  34
38 is thirty-eight units from zero in the positive
direction. |38|  38
39 is thirty-nine units from zero in the positive
direction. |39|  39
3. No; the lowest temperatures should be at the
beginning and end of the table for January and
December.
4. Write 29,035 as a positive number to represent
above sea level, and 1312 as a negative number to
represent below sea level. Subtract to find the
difference.
elevation of
elevation of
Mount Everest minus the Dead Sea

14
424
43 123 14
424
43 123 14
424
43 123 14
424
43

Year

767

Mixed Problem Solving

PQ249J-6481F-P01-14[766-785] 26/9/02 9:07 PM Page 768 Sahuja Ahuja_QXP_06:Desktop Folder:Chandra:Algebra_FNL_Delivery:

14. There are 20  39  39  39  44  35  40


or 256 million people predicted to not be
1524 years old and 299 million people predicted
as a total.

7. By finding the values obtained by using all twodigit whole numbers, we conclude that the least
result is obtained from the number 19.
19
1  9

19

 10

256,000,000

P(not be 1524)  299,000,000

 1.9
8. The greatest common place value is ones, so the
digits in the ones place are the stems.
Stem
Leaf
7
1 7
8
1 6 9
9
0 1 1 6
10
2 4 5
11
0 9
12
5 8
7|1  7.1
9. The least value is 7.1 and the greatest value
is 12.8.
12.8  7.1  5.7
10. mean 

256

 299
 0.86
The probability that a person selected will not be
256
1524 years old is 299 or about 86%.
15. 2202  452  1400  2025
 12425
 49.24
The hypotenuse of the garden is about 49 ft.
16. Let x  the length of the other leg. The situation
can be represented by the equation
55  2x2  452. Estimate to find reasonable
values for the replacement set. Start by letting
x  25 and then adjust values up or down as
needed.

sum of wind speeds


number of speeds
8.9  7.1  9.1  9.0  10.2  12.5  11.9  11.0

 12.8  10.4  10.5  8.6  7.7  9.6  9.1  8.1


16

 9.78125
The mean is about 9.8.
To find the median, order the numbers from least
to greatest. The median is in the middle. 7.1, 7.7,
8.1, 8.6, 8.9, 9.0, 9.1, 9.1, 9.6, 10.2, 10.4, 10.5, 11.0,
14243
11.9, 12.5, 12.8
9.1  9.6
2

55  2x2  452

55 S 51.5 55

too low

?
452 

55 S 54.1 55

too low

35 235 

?
452 

55 S 57.0 55

too high

25
30

2302
2

2 ?

31 2312  45  55 S 54.6 55

 9.35

2 ?

32 2322  45  55 S 55.2  55

The median is 9.35.


The mode is the number that occurs most
frequently. 9.1 occurs twice and all the other
speeds occur once. The mode is 9.1.
11. Sample answer: Yes, many of the data values are
around 9.1.
12. There are 20 million people predicted to be under
age 5 and 299 million people predicted as a total.

Reasonable?

?
452 

2252

almost
reasonable

The length of the other leg should be about 32 ft.


17. To find the speed, divide the number of meters by
the number of seconds.
400 44  9.1
The speed of the 400 meter run was about
9 meters per second.
18. To find the speed, divide the number of meters by
the number of seconds.
400 3 min 41 sec  400 221 sec
 400 221
 1.8
The speed of the 400-meter freestyle was about
2 meters per second.
19. Since 2  4.5  9, the running speed is 4.5 times
faster than the swimming speed.

20,000,000

P(under age 5)  299,000,000


20

 299
 0.07
The probability that a person selected will be
20
under 5 is 299 or about 7%.
13. There are 40 million people predicted to be 65 or
over and there are 299  40 or 259 million people
predicted to be under 65.
40,000,000

odds of 65 or over  259,000,000

Page 855

40

Chapter 3 Solving Linear Equations

 259
1.

The odds of selecting a person that will be 65 or


over are 40:259.

The lateral
surface area is

the product of the


two times  times radius and the height.

144424443 123 123 123 123 123 144444


42444
4443

The formula is L  2rh.


2. L  2rh
 2(3.14)(4.5)(7)
 197.82
The lateral area is about 197.8 in2.

Mixed Problem Solving

768

rh

PQ249J-6481F-P01-14[766-785] 26/9/02 9:07 PM Page 769 Sahuja Ahuja_QXP_06:Desktop Folder:Chandra:Algebra_FNL_Delivery:

3.

the area
the lateral
The total
surface area equals two times of the base plus surface area.

The Wilson family bought 2 (30)  1 or 16 bags.


1
The Martinez family bought 2 (30  16)  1 or
8 bags. The Brightfeather family bought
1
(30  24)  1 or 4 bags. The Wimberly family
2
bought 2 bags.
11. Let n  the least even integer. Then n  2  the
next greater even integer, and n  4  the
greatest of the three even integers.

144424443 14243 123 123 1442443 123 144


424443
2

r

2rh

The formula is T  2r2  2rh.


4. T  2r2  2rh
 2(3.14)(4.5) 2  2(3.14)(4.5)(7)
 127.17  197.82
 324.99
The total surface area is about 325.0 in2.
5. Let n  the length of the Niger River.

Five times the


twice the sum of
greatest even integer is equal to the other two integers plus 42.

44244
3 1444442444443 123 123
1444442444443 1

310

2900

The equation is n  310  2900.


6.
n  310  2900
n  310  310  2900  310
n  2590
The length of the Niger River is 2590 miles.
7. Let g  the length of an average common
goldfish.
the length of an
the average length of a
4.8 times average common goldfish equals yellow-banded angelfish.

123 123 144444424444443 1


4244
444443
424
3 1444444

4.8

12

The equation is 4.8g  12.


8. 4.8g  12
4.8g
4.8

12

 4.8

x  (x  10)  2[ x  (x  10) ]

g  2.5
The length of an average common goldfish is
2.5 in.
9. Let x  the number of registered Labrador
Retrievers.
The number of
the number of
Labrador Retrievers was fifteen times Great Danes plus 6997.


15

9860

6x
6

6997

Undo the Statement


2

The Wimberlys bought


half the remaining
bags plus one.
The Brightfeathers
bought half the
remaining bags
plus one.
The Martinezes bought
half the remaining
bags plus one.
The Wilsons bought
half the remaining
bags plus one.
The starting number
of bags

2  2x  1 S x  2

42

180

150
6

x  25
x  10  25  10 or 35
2[x  (x  10)  2[ 25  (25  10) ] or 120
The measures of the three angles are 25
, 35
,
and 120
.
13. Let c represent the amount of chemicals needed.

x  15(9860)  6997
x  147,900  6997
x  154,897
There were 154,897 registered Labrador
Retrievers.
10. Start at the end of the problem and undo each step.
Statement
The Wimberlys bought
2 bags.

x  (x  10)  2[ x  (x  10) ]  180


2x  10  2[ 2x  10]  180
2x  10  4x  20  180
6x  30  180
6x  30  30  180  30
6x  150

14
444424444
43 123 1
424
3 123 144424443 123 123

2[ n  (n  2) ]

5(n  4)  2[n  (n  2) ]  42
5n  20  2[ 2n  2]  42
5n  20  4n  4  42
5n  20  4n  46
5n  20  4n  4n  46  4n
n  20  46
n  20  20  46  20
n  26
n  2  26  2 or 28
n  4  26  4 or 30
The consecutive even integers are 26, 28, and 30.
12. Let x  the measure of the second angle. Then
x  10  the measure of the first angle, and
2[x  (x  10)]  the measure of the third angle.
The sum of the measures of
the three angles of a triangle equals 180.

1444444442444444443 14243 123

1444
4244443 123 1442443 14243 1444
4244443

5(n  4)

the length of
The length of
the Niger River plus 310 miles equals the Congo river.

1.5
5000

 12,500

1.5(12,500)  5000(c)
18,750  5000c

18750
5000

5000c
5000

3.75  c
The amount of chemicals needed is 3.75 pounds.
14. Find the change.
2999  1999  1000
Find the percent using the original number, 2999,
as the base.

2(2  1)  6

2(6  1)  14

1000
2999

 100

1000(100)  2999(r)
100,000  2999r

2(14  1)  30

100,000
2999

2999r
2999

33  r
The percent of decrease was about 33%.

30

769

Mixed Problem Solving

PQ249J-6481F-P01-14[766-785] 26/9/02 9:07 PM Page 770 Sahuja Ahuja_QXP_06:Desktop Folder:Chandra:Algebra_FNL_Delivery:

3.

15. The tax is 7% of the cost of the camp.


7% of $1254  0.07  1254
 87.78
Add this amount to the original cost.
$1254.00  $87.78  $1341.78
The total cost of the camp is $1341.78.
16. The refund is
1
2

of $87.78 

Since it has five sides, the design is in the shape


of a pentagon.
4. To dilate the pentagon by a scale factor of 0.75,
multiply the coordinates of the vertices by 0.75.

Prt
rt

(x, y) S (0.75x, 0.75y)


(1, 1) S (0.75  1, 0.75  (1)) S (0.75, 0.75)
(2, 2) S (0.75  2, 0.75  2) S (1.5, 1.5)
(0, 3) S (0.75  0, 0.75  3) S (0, 2.25)
(2, 2) S (0.75  (2), 0.75  2) S (1.5, 1.5)
(1, 1) S (0.75  (1), 0.75  (1) ) S (0.75, 0.75)

P
I

18. P  rt
1848.75

P  0.0725(6)
P  4250
The amount of money invested is $4250.
19. Let x  the amount of 40% iodine solution to be
added.
Amount of
Solution
40
x
40  x

15% Solution
40% Solution
20% Solution

5. The area of the original design can be found by


dividing the pentagon into five polygons as
follows:
The area of the rectangle with vertices
(1, 1), (1, 2), (1, 2), and (1, 1) is
bh  2(3) or 6 units2.
The area of the triangle with vertices (1, 1),
1
1
(2, 2), and (1, 2) is 2 bh  2(1)(3) or
2
1.5 units . The area of the triangle with
vertices (1, 1), (1, 2), and (2, 2) is also
1.5 units2.
The area of the triangle with vertices (2, 2),
1
1
(0, 3), and (0, 2) is 2bh  2(2)(1) or 1 unit2. The
area of the triangle with vertices (2, 2), (0, 3),
and (0, 2) is also 1 unit2.
The total area of the original design is the sum of
the areas of the rectangle and four triangles
described above. This is 6  2(1.5)  2(1) or
11 units2.
Similarly, the area of the dilated design can be
found by dividing the dilated pentagon into five
polygons as follows:
The area of the rectangle with vertices
(0.75, 0.75), (0.75, 1.5), (0.75, 1.5), and
(0.75, 0.75) is bh 1.5(2.25) or 3.375 units2.
The area of the triangle with vertices
(0.75, 0.75), (1.5, 1.5), and (0.75, 1.5) is
1
1
bh  2(0.75)(2.25) or 0.84375 units2. The area
2
of the triangle with vertices (0.75, 0.75),
(1.5, 1.5), and (0.75, 1.5) is also 0.84375 units2.
The area of the triangle with vertices
(1.5, 1.5), (0, 2.25), and (0, 1.5) is
1
1
bh  2 (1.5) (0.75) or 0.5625 units2. The area
2
of the triangle with vertices (1.5, 1.5),
(0, 2.25), and (0, 1.5) is also 0.5625 units2.
The total area of the dilated design is the sum of
the areas of the rectangle and four triangles
described above. This is
3.375  2(0.84375)  2(0.5625) or 6.2 units2.

Amount of
Iodine
0.15(40)
0.40x
0.20(40  x)

amount of
Amount of
amount of
iodine in
iodine in
iodine in
15% solution plus 40% solution equals 20% solution.
44244
43
14
4424443 123 144424443 1
4243 14
0.15(40)

0.40x

0.20(40  x)

0.15(40)  0.40x  0.20(40  x)


6  0.4x  8  0.2x
6  0.4x  0.2x  8  0.2x  0.2x
6  0.2x  8
6  0.2x  6  8  6
0.2x  2
0.2x
0.2

 0.2

x  10
Isaac should add 10 gallons of the 40% solution to
the 40 gallons of the 15% solution.

Page 856

1
of the tax.
2
1
 87.78
2

 43.89
The amount of the refund is $43.89.
17. I  Prt
I
rt
I
rt

Chapter 4 Graphing Relations


and Functions

1. Start at the origin (the building).


Move right (east) 2 units and down (south)
1 unit.
The x-coordinate is 2, and the y-coordinate is 1.
The coordinates of the pool are (2, 1).
2. Start at the origin (the building).
Move left (west) 3 units and up (north) 5 units.
The x-coordinate is 3, and the y-coordinate is 5.
The entrance to the community has coordinates
(3, 5).

Mixed Problem Solving

770

PQ249J-6481F-P01-14[766-785] 26/9/02 9:07 PM Page 771 Sahuja Ahuja_QXP_06:Desktop Folder:Chandra:Algebra_FNL_Delivery:

6. Graph the ordered pairs (60, 102), (62, 109),


(64, 116), (66, 124), (68, 131), (70, 139), (72, 147),
and (74, 155).

Graph the ordered pairs and draw a line through


the points.
c
100

Weight

160

120

Cost

140

75
50
25

100
0

64

68
72
Height

76

7. No; for each 2 unit increase in height, the


increase in weight is not constant.
8. Sample answer: 171 pounds; from 72 to 74 inches,
there is an increase of 8 pounds, so 8  2  16 and
added to 155 is 171.

y
230
228

9.

Distance from
Sun, d

d
93,000,000

AU

36,000,000

36,000,000
93,000,000

0.387

Mars

141,650,000

141,650,000
93,000,000

1.523

Jupiter

483,750,000

483,750,000
93,000,000

5.202

222

39.223

220

Mercury

Pluto

3,647,720,000
93,000,000

3,647,720,000

226
Time

Planet

To the nearest thousandth, Mercury is 0.387 AU


from the Sun, Mars is 1.523 AU from the Sun,
Jupiter is 5.202 AU from the Sun, and Pluto is
39.223 AU from the Sun.
10. If the number of AU is less than 1, the planet is
closer than Earth. If the number of AU is greater
than 1, the planet is farther than Earth.
11.

x
2
4
6
8
Number of Blinds

13. If Pam has 8 blinds installed, then x  8.


Substituting into the given equation, we have
c  25  6.5(8) or 77. Thus, the cost of 8 blinds
is $77.
14. Graph the ordered pairs (0, 229), (4, 227), (8, 218),
(12, 230), (16, 224), and (20, 222).

x
60

c  25  6.5x

d
93,000,000
d
(93,000,000) 93,000,000

218
0

 270,000

C
25
38
51
64
77

8
16
24
Years Since 1980







5
5
5
5
5
5

 (93,000,000)270,000

25  6.5x
25  6.5(0)
25  6.5(2)
25  6.5(4)
25  6.5(6)
25  6.5(8)

15. Yes; each value of x is paired with only one


value of y.
16. Sample answer: 218 min
17. 1
6
11
16
21
26
31

d  25,110,000,000,000
So Alpha Centauri is about 25,110,000,000,000 mi
from the Sun.
12. Select five values for the domain and make a
table.
x
0
2
4
6
8

224

The first term is 1. The common difference is 5.


Use the formula for the nth term to write an
equation with a1  1 and d  5.
an  a1  (n  1)d
an  1  (n  1)5
an  1  5n  5
an  5n  4

x, C
(0, 25)
(2, 38)
(4, 51)
(6, 64)
(8, 77)

18. Use the formula from Exercise 17 with n  20.


an  5n  4
a20  5(20)4
a20  100  4
a  96
20
Row 20 would contain 96 beads.

771

Mixed Problem Solving

PQ249J-6481F-P01-14[766-785] 26/9/02 9:07 PM Page 772 Sahuja Ahuja_QXP_06:Desktop Folder:Chandra:Algebra_FNL_Delivery:

19. Side
1
2
3
4
5
6
7
8
9
10

7. The graph passes through (0, 453,589) with


slope 2890.

Area
1
4
9
16
25
36
49
64
81
100

Population of Wyoming
Population
(thousands)

500

Chapter 5 Analyzing Linear Equations

change in price
change in time

2.00  0.67

 1950  1940


m
m

4820x
4820

rate of
number of years
population
Population
equals change times
after 1990
plus in 1990.
1442443 14243 14243 123 14444244443 123 1442443
y

2890

(0, 453,589)

3 4 5 6 7 8
Years Since 1990

9 10

 x1

136  112
66  60
24
or 4
6

Find the y-intercept.


y  mx  b
112  4(60)  b
112  240  b
112  240  240  b  240
128  b
Write the slope-intercept form.
y  mx  b
y  4x  (128)
y  4x  128
10. y  4x  128
y  4(72)  128
y  160
A person who is 72 inches tall should be
160 pounds.
11. Write the point-slope form of the equation for a
line that passes through (1990, 3,600,000) with
slope 300,000.
y  y1  m(x  x1 )
y  3,600,000  300,000(x  1990)
12.
y  3,600,000  300,000(x  1990)

15  x
It will take 15 seconds for sound to travel
72,300 feet.
6.

450

y2  y1

r
4820


1
Solve for the rate.
4820  r(1)
4820  r
Therefore, the equation is y  4820x.
5.
y  4820x
72,300  4820x


460

mx

1.33
10

 0.133
Over this 10-year period, the price increased by
$1.33, for a rate of change of $0.133 per year.
3. The slope of the line segment that represents the
10-year period from 1980 to 1990 is negative. This
represents a drop in price.
rate times time.
4. Distance equals 1
14243 14243 23 123 123

72,300
4820

470

8. The year 2005 is 15 years after 1990.


y  2890x  453,589
y  2890(15)  453,589
y  496,939
The population of Wyoming should be about
496,939 in 2005.
9. Let x represent the height of a person. Let y
represent the weight of the person. Write an
equation of the line that passes through (60, 112)
and (66, 136).
Find the slope.

1. The greatest rate of change occurred in the 10-year


period from 19701980, which is represented by
the steepest segment. The least rate of change
occurred in the 9-year period from 19901999,
which is represented by the least steep segment.
2.

(10, 482,489)

480

The values for the area are the squares of


consecutive integers.

Page 857

490

453,589

Therefore, the equation is y  2890x  453,589.

y  3,600,000  300,000x  597,000,000


y  3,600,000  3,600,000  300,000x  597,000,000
 3,600,000
y  300,000x  593,400,000

13. y  300,000x  593,400,000


y  300,000(2010)  593,400,000
y  9,600,000
The number of people who will take a cruise in
2010 should be about 9,600,000.

Mixed Problem Solving

772

PQ249J-6481F-P01-14[766-785] 26/9/02 9:07 PM Page 773 Sahuja Ahuja_QXP_06:Desktop Folder:Chandra:Algebra_FNL_Delivery:

14.

17. We use the points (0, 324) and (2, 1530).


Find the slope.

y
y  12 x  6

y2  y1

mx

m

y  6  2x
y  2x

m

The y-intercept is 324.


y  mx  b
y  603x  324
18. The year 2005 is 13 years after 1992.
y  603x  324
y  603(13)  324
y  8163
There should be about 8163 children adopted
from Russia in 2005.

y  12 x  92

Solve each equation for y to find the slopeintercept form.


Eq. 1: y  2x
2x  y  6
Eq. 2:
2x  y  2x  6  2x
y  2x  6

Page 858

Eq. 3: y  2x  6
Eq. 4:

y

x  9
2
1
9
x
2
2

The slope of y  2x is 2.


The slope of y  2x  6 is 2.
Therefore, these two lines are parallel.
1

The slope of y  2x  6 is 2.
1

The slope of y  2x  2 is 2.
Therefore, these two lines are parallel.
1
Since 2 is the opposite reciprocal of 2, y  2x
and y  2x  6 are both perpendicular to
1
1
9
y  2x  6 and y  2x  2. Therefore, the figure is
a rectangle.
15. Opposite sides have the same slopes, so they are
parallel. Consecutive sides have slopes that are
opposite reciprocals, so they are perpendicular.
16. Draw a line that passes close to the points.
Sample answer:

0
4

Number of Children

4w
4

40
4

0 6 w  10
A female bustard can weigh up to 10 lb.
6. 1.50a  0.30a 75
7. 1.50a  0.30a 75
1.20a 75

Adopted Russian Children


4000

1.20a
1.20

3000

75

1.20

a 62.50
8. Jennie must make and sell 63 or more apples to
make at least $75.

2000
1000

Chapter 6 Solving Linear Inequalities

1. Let a  the amount Scott can spend after buying


a concert ticket.
a  26  50
a  26  26  50  26
a  24
Scott can spend no more than $24.
2. Let a  the amount Scott can spend after buying
a concert ticket, lunch, and a CD.
a  26  2.99  12.49  50
a  41.48  50
a  41.48  41.48  50  41.48
a  8.52
Scott can spend no more than $8.52.
3. Let   the length of a male bustard.
0 6 /4
4. Let w  the weight of a male bustard.
0 6 w  40
5. Let w  the weight of a female bustard. Then 4w
represents the weight of a male bustard.
0 6 4w  40

x  2y  9
x  2y  x  9  x
2y  x  9
2y
2

 x1

1530  324
2  0
1206
or 603
2

2 3 4 5 6 7
Years Since 1992

773

Mixed Problem Solving

PQ249J-6481F-P01-14[766-785] 26/9/02 9:07 PM Page 774 Sahuja Ahuja_QXP_06:Desktop Folder:Chandra:Algebra_FNL_Delivery:

9. Let p  the price of a pair of shoes for a customer


with a $15 coupon. Then p  15 represents the
original price of a pair of shoes.
24.95  p  15  149.95
First express 24.95  p  15  149.95 using and.
24.95  p  15
24.95  15  p  15  15
9.95  p

and

The half plane that contains (72, 72) should be


shaded.
The dimensions cannot be negative. Therefore,
the domain and range contain only nonnegative
numbers.

p  15  149.95
p  15  15  149.95
p  134.95

The solution set is {p|9.95  p  134.95}.


The price for a pair of shoes for a customer who has
a coupon is between $9.95 and $134.95, inclusive.
10. Let p  the price of the pair of shoes using a
$15 coupon.
p  109.95  15
p  94.95
Using a coupon, the price of the shoes is $94.95.
Let d  the price of the pair of shoes after a
15% discount on the price.
d  109.95  0.15(109.95)
(to the nearest cent)
d  109.95  16.49
d  93.46
With a 15% discount, the price of the shoes is
$93.46. You should choose the 15% discount.
11. Let p  the regular price of a pair of shoes.
p  0.15p  p  15
0.85p  p  15
0.85p  p  p  15  p
0.15p  15
0.15p
0.15

100
50

w
O

4/
4

p  100
A 15% discount is the same as $15 off if the
regular price is $100.
12. Let m  the mean of the temperatures.
m

927
12

m  77.25

13.
14.

15.

16.

17.

To the nearest degree, the mean of the


temperatures is 77
.
mean  lowest temperature  77  73 or 4
The lowest temperature varies by 4
from the mean.
highest temperature  mean  81  77  4
The highest temperature varies by 4
from the
mean.
Let t  the temperature.
The difference between the actual temperature
and the mean is within 4 degrees, so |t  77|  4.
Let w  the width of the quilt. Let /  the length
of the quilt. Since the perimeter of a rectangle is
given by P  2/  2w, the inequality
2/  2w  318 represents this situation.
Since the boundary is included in the solution,
draw a solid line. Test the point (72, 72).
2w  2/  318
2(72)  2(72)  318
144  144  318
288  318 true

Mixed Problem Solving

100

150

318
4

/  79.5
So, the area is greatest when the length / and
width w are each 79.5 in. Since A  /w, the
greatest area would be found by substituting 79.5
for both / and w.
A  /w
A  (79.5)(79.5)
A  6320.25
The greatest area, 6320.25 in2, occurs when the
dimensions of the quilt are 79.5 in. by 79.5 in.
19. Let m  the area of a state in square miles,
1045  m  570,473
20. Let m  the area of a state in acres.
677,120  m  365,481,600
21. There are 640 acres in one square mile. So, the
number of square miles multiplied by 640 is the
number of acres.
Alaska: 570,473 (640)  365,102,720
Rhode Island: 1045 (640)  668,800
Alaska is listed in the table as having
365,481,600 acres, but 570,473 mi2 is
365,102,720 acres. While this difference of
378,880 acres is large, it actually represents
only about a 0.1% difference.
Rhode Island is listed in the table as having
677,120 acres, but 1045 mi2 is 668,800 acres.
While this difference of 8320 acres is significantly
less than Alaskas difference of 378,800 acres, it
actually represents about a 1.2% difference.
Thus, Alaska is very close, but Rhode Island is a
little off.

15

73  73  74  76  78  79  81  81  81  80  77  74
12

50

Sample answers: Since (70, 75) is in the shaded


region, the quilt could be 70 in. by 75 in.
Similarly, since (72, 72) is in the shaded region,
the quilt could be 72 in. by 72 in.
18. The area of the quilt is greatest when the length
/ and width w are equal.
2/  2w  318
2/  2/  318
4/  318

 0.15

m

2w  2  318

150

774

PQ249J-6481F-P01-14[766-785] 26/9/02 9:07 PM Page 775 Sahuja Ahuja_QXP_06:Desktop Folder:Chandra:Algebra_FNL_Delivery:

4. Since there are 60 min in each hour and 60 sec in


each min,
1:48:29 is about 1 h 48 min
 60 min  48 min, or 108 min.
1:51:41 is about 1 h 52 min
 60 min  52 min, or 112 min.
2:04:58 is about 2 h 5 min
 120 min  5 min, or 125 min.
1:54:43 is about 1 h 55 min
 60 min  55 min, or 115 min.

Page 859 Chapter 7 Solving Systems of Linear


Equations and Inequalities
1. Since the year 1980 is 0, the year 1990 is 10. Use
the ordered pairs (0, 19.3) and (10, 17.6) to write
the equation. First find the slope.
y2  y1

mx

m
m

 x1

17.6  19.3
10  0
1.7
or 0.17
10

Since the first coordinate of the ordered pair


(0, 19.3) is 0, the y-intercept is 19.3. Use the
slope-intercept form to write the equation.
y  mx  b
y  0.17x  19.3
2. Use the ordered pairs (0, 8.2) and (10, 8.4) to
write the equation. First find the slope.

Year
1995
2000

m
m

Womens
125
115

5. Since the year 1995 is 0, the year 2000 is 5. Use


the ordered pairs (0, 108) and (5, 112) to write the
equation. First find the slope.

y2  y1

mx

Mens
108
112

y2  y1

mx

 x1

8.4  8.2
10  0
0.2
or 0.02
10

m
m

Since the first coordinate of the ordered pair


(0, 8.2) is 0, the y-intercept is 8.2. Use the slopeintercept form to write the equation.
y  mx  b
y  0.02x  8.2
3. Solve the system of equations.
y  0.17x  19.3
y  0.02x  8.2
Substitute 0.17x  19.3 for y in the second
equation.
y  0.02x  8.2
0.17x  19.3  0.02x  8.2
0.17x  19.3  0.17x  0.02x  8.2  0.17x
19.3  0.19x  8.2
19.3  8.2  0.19x  8.2  8.2
11.1  0.19x
11.1
0.19

 x1

112  108
5  0
4
or 0.8
5

Since the first coordinate of the ordered pair


(0, 108) is 0, the y-intercept is 108. Use the
slope-intercept form to write the equation.
y  mx  b
y  0.8x  108
6. Use the ordered pairs (0, 125) and (5, 115) to
write the equation. First find the slope.
y2  y1

mx

m
m

 x1

115  125
5  0
10
or 2
5

Since the first coordinate of the ordered pair


(0, 125) is 0, the y-intercept is 125. Use the
slope-intercept form to write the equation.
y  mx  b
y  2x  125
7. Solve the system of equations.
y  0.8x  108
y  2x  125
Substitute 0.8x  108 for y in the second
equation.
y  2x  125
0.8x  108  2x  125
0.8x  108  2x  2x  125  2x
2.8x  108  125
2.8x  108  108  125  108
2.8x  17

0.19x
0.19

58.4  x
Therefore, the percent of working men and
women will be the same about 58 yr after 1980, or
during the year 2038.

2.8x
2.8

17

 2.8

x  6.1
Therefore, you might expect the mens and
womens winning times to be the same about 6 yr
after 1995, or in 2001.

775

Mixed Problem Solving

PQ249J-6481F-P01-14[766-785] 26/9/02 9:07 PM Page 776 Sahuja Ahuja_QXP_06:Desktop Folder:Chandra:Algebra_FNL_Delivery:

8. Let t  the time it took Mrs. Sumner to travel to


Fullerton in hours. Let d  the distance between
Mrs. Sumners home and Fullerton.
Rate
40
56

To Fullerton
Return trip

Time
t
t2

14. Solve the system of equations.


g  s  64
3g  2s  168
Multiply the first equation by 3 so the
coefficients of the g terms are additive inverses.
Then add the equations.
3g  3s  192
() 3g  2s  168
s  24
(1)(s)  (1) (24)
s  24
Now substitute s  24 in either equation to find
the value of g.
g  s  64
g  24  64
g  24  24  64  24
g  40
So, the U.S. won 40 gold and 24 silver medals.
15. Since the total points scored for gold and silver
medals was 168, and the total number of points
scored was 201, then the number of points earned
for bronze medals was 201  168 or 33 points.
Since each bronze medal is worth 1 point, the
U.S. had 1 bronze medal for each point, or
33 bronze medals.
16. The cost
the number
the number
the cost

Distance
d
d

Use the formula rate  time  distance, or rt  d.


40t  d
56(t  2)  d
Substitute 40t for d in the second equation.
56(t  2)  d
56(t  2)  40t
56t  112  40t
56t  112  56t  40t  56t
112  16t
112
16

16t
16

7t
Use d  40t to find the value of d.
d  40t
d  40(7)
d  280
Therefore, Mrs. Sumner lives 280 mi from
Fullerton.
9. Let t  the number of $2 bills and let f  the
number of $50 bills.
t  f  1,500,888,647
f  t  366,593,903
10. Substitute t  366,593,903 for f in the first
equation.

of a
child
ticket times
14243 123
15

t  f  1,500,888,647

c
7
So, the inequality is c  2a.
18. a 0; c 0
19.
c

2t  366,593,903  366,593,903  1,500,888,647  366,593,903


2t  1,134,294,744
1,134,294,744
2

t  567,147,372

Use f  t  366,593,903 to find the value of f.


f  t  366,593,903
f  567,147,372  366,593,903
f  933,741,275
Therefore, there were 567,147,372 $2 bills and
933,741,275 $50 bills in circulation.
11. 567,147,372 $2 bills are worth 567,147,372  2 or
$1,134,294,744. 933,741,275 $50 bills are worth
933,741,275  50 or $46,687,063,750. So the total
in circulation in $2 bills and $50 bills was
$1,134,294,744  $46,687,063,750 or
$47,821,358,494.
12. Let g represent the number of gold medals, and
let s represent the number of silver medals.
g  s  64
13. 3g  2s  168

Mixed Problem Solving

20

1442443

is no
more
than $800.
123 123


800

1442443 144424443 14444244443

2t  366,593,903  1,500,888,647

of adult
tickets
sold

So, the inequality is 15c  20a  800.


17. The number
of child
twice the number
tickets
is greater than of adult tickets.

t  (t  366,593,903)  1,500,888,647

2t
2

of child
of an
tickets
adult
sold
plus ticket times
1442443 123 14243 123

2a

60

c  2a
40

15c  20a  800

20
O

20

40

60

20. Sample answer: The station can buy 5 adult and


40 child tickets, or 10 adult and 30 child tickets,
or 12 adult and 32 child tickets since the graphs
of the ordered pairs (5, 40), (10, 30), and (12, 32)
lie in the region that represents the intersection
of the inequalities.

776

PQ249J-6481F-P01-14[766-785] 26/9/02 9:07 PM Page 777 Sahuja Ahuja_QXP_06:Desktop Folder:Chandra:Algebra_FNL_Delivery:

Page 860

11. R  N  (133.5x  10,278.5)  (37.9x  1315.9)

Chapter 8 Polynomials

 (133.5x  10,278.5)  (37.9x  1315.9)


 (133.5x  (37.9x))  (10,278.5  1315.9)
  95.6x  8962.6

1. Yes; each is the product of variables and/or a real


number.
2. Volume  s3
 33
 27
The volume is 27 ft3.

12. Since 2015 is 18 years after 1997, substitute 18


for x.
  95.6x  8962.6
 95.6(18)  8962.6
 10,683.4
There will be about 10,683 such stations in 2015.
13. 0.5n(n  3)  0.5n(n)  0.5n(3)
 0.5n2  1.5n
2
14.
3: 0.5(3)  1.5(3)  0
4: 0.5(4)2  1.5(4)  2
5: 0.5(5)2  1.5(5)  5
6: 0.5(6)2  1.5(6)  9
7: 0.5(7)2  1.5(7)  14
8: 0.5(8)2  1.5(8)  20
9: 0.5(9)2  1.5(9)  27
10: 0.5(10)2  1.5(10)  35
15. You add one more each time to the previous
number. For example, 0  2  2, 2  3  5,
5  4  9, and so on.
16. V  /wh
 x(x  3) (2x  5)
 (x2  3x) (2x  5)
 x2 (2x)  x2 (5)  3x(2x)  3x(5)
 2x3  5x2  6x2  15x
 2x3  11x2  15x
17. See students work.
18. 4000 is the amount of the investment, 1 will add
the amount of the investment to the interest, 0.05
is the interest rate as a decimal, and 2 is the
number of years of the investment.
19. 4000(1  0.05)2  $4410
20. 10,000(1  0.0625)4  $12,744

Surface Area  6s2


 6(3) 2
 6(9)
 54
The surface area is 54 ft2.
3.

s3  6s2
s  6s2  0
s2 (s  6)  0
3

s2  0 or s  6  0
s  0 or
s6
When the side length is 6 units, the volume and
surface area have the same numerical value.
4.

r2h
(2r) 2 (2h)

r2h

   22  r2  2h


1 212 1 2 21rr 21hh 2


2
2

1
8


or 1:8
5.
vacuum: 3.00  108  300,000,000
air: 3.00  108  300,000,000
ice: 2.29  108  229,000,000
glycerine: 2.04  108  204,000,000
crown glass: 1.97  108  197,000,000
rock salt: 1.95  108  195,000,000
6.

3.00  108
1.95  108

10
13.00
1.95 21 10 2
8
8

 (1.54)(100 )
 1.54
Light travels about 1.54 times faster through a
vacuum.
2.29  108
 108

7. ice; 1.95

10
12.29
1.95 21 10 2
8
8

Page 861

 (1.17)(100 )
 1.17
8. 0.003y2  0.086y  0.708
 0.003y2  (0.086y)  0.708
This polynomial is the sum of three monomials; it
is a trinomial.
9. The degrees of the terms, 0.003y2, 0.086y, and
0.708, are 2, 1, and 0, respectively. Thus, the
degree of the polynomial is 2, the greatest of 2, 1,
and 0.
10. Since 2010 is 90 years after 1920, substitute 90
for y and simplify.
0.003y2  0.086y  0.708
 0.003(90)2  0.086(90)  0.708
 17.3
The population should be about 17.3 people/square
mile.

Chapter 9 Factoring

1. 1-foot by 1-foot squares can be used since both


10 and 12 are divisible by 1.
2-foot by 2-foot squares can be used since both
10 and 12 are divisible by 2.
2-foot by 3-foot squares can be used since 10 is
divisible by 2 and 12 is divisible by 3.
1 by 1, 2 by 2, and 2 by 3; The 3-foot squares will
not cover the 10-foot dimension without cutting.

777

Mixed Problem Solving

PQ249J-6481F-P01-14[766-785] 26/9/02 9:07 PM Page 778 Sahuja Ahuja_QXP_06:Desktop Folder:Chandra:Algebra_FNL_Delivery:

10  12

11. Let x  the number of feet being added and


subtracted to 24.
(24  x)(24  x)  512
576  x2  512
x2  64
x2  64
x8
24  8  16
24  8  32
The dimensions of the deck are 16 ft by 32 ft.
12. The area in the plans is 24 ft  24 ft  576 sq ft.
1
The new area is 2 (576 sq. ft)  288 sq ft.

2. For 1-foot by 1-foot squares, there are 1  1 or


120 squares.
The cost is 120  $3.75  $450.
10  12
For 2-foot by 2-foot squares, there are 2  2 or
30 squares.
The cost is 30  $15  $450.
For 2-foot by 3-foot rectangular pieces, there are
10  12
or 20 squares.
2  3
The cost is 20  $21  $420.
Therefore, the 2  3 tiles are the least expensive
at a total cost of $420.
3. The height of the rocket is 0 when it returns to
the ground.
4. 125t  16t2  h
125t  16t2  0
t(125  16t)  0
t  0 or 125  16t  0
125  16t
t
5.
6.

7.

8.

125
16

y2  288
y  1288 or about 16.97
Since the new area is not a perfect square, the
equation is not factorable, so there are no whole
number solutions.
13. When the object hits the ground, it will have
dropped 1483 ft. Let h  1483.
1483  16t2
92.6875  t2
9.6  t
It will take about 9.6 seconds for the object to hit
the ground.
14. Let h  386.
386  16t2
24.125  t2
4.9  t
It will take the object dropped from the Petronas
Tower I building about 9.6  4.9  4.7 seconds
longer to hit the ground.
15. Let t  12.

or 7.8125

{0, 7.8125}
Since h  0 when the rocket returns to the
ground, it will take about 7.8 seconds.
Let w  the width of the field; therefore,
2w  60  the length of the field. The area is
length times width, or (2w  60)w  w(2w  60).
w(2w  60)  36,000
2w2  60w  36,000
2  60w  36,000  0
2w
2(w2  30w  18,000)  0
w2  30w  18,000  0
(w  120) (w  150)  0
or w  150  0
w  120  0
w  120
w  150
The width must be 120 feet since it is a
measurement. The length is
2w  60  2(120  60), or 300 feet.
The dimensions of the field are 120 feet by
300 feet.
There are 3 feet in a yard. Divide the length and
width by 3.
120
3

 40 yd

300
3

h  16(12) 2
h  16(144)
h  2304 ft.
The building is 2304 feet tall.
16. The height of the pool in feet is
V  /w  h
42

1750  /w  12
500  /w
The area of the water that is exposed to the air is
500 ft2.
17. If w is the width of the pool, the length is equal to
w  5.
w(w  5)  500
w2  5w  500  0
(w  25)(w  20)  0
w  25  0
or w  20  0
w  25
w  20
The width must be 20 ft since it is a measurement.
The length is 20  5  25 ft.
The dimensions of the pool are 20 ft by 25 ft.
18. Model B holds 1750  2  3500 cubic feet of
water.
  w  h  3500
Sample answer: 20 ft by 50 ft by 42 in.

 100 yd

The dimensions of the field are 40 yd by 100 yd.


9. 48 is the initial velocity of the ball and 506 is the
height from which the ball is thrown, which is
500 feet plus 6 feet, Terils height.
10. 16t2  48t  506  218
16t2  48t  288  0
16(t2  3t  18)  0
t2  3t  18  0
(t  3)(t  6)  0
or t  6  0
t30
t6
t  3
The ball was in the air for 6 seconds.

Mixed Problem Solving

42
.
12

778

PQ249J-6481F-P01-14[766-785] 26/9/02 9:07 PM Page 779 Sahuja Ahuja_QXP_06:Desktop Folder:Chandra:Algebra_FNL_Delivery:

7. Solve 0  16t2  210.

19. The measurements for Model C are


width  2(20)  40 ft
length  2(25)  50 ft
height  42 in.
42
Volume of Model C  40  50  12  7000 ft3.
Model A
Model C

1750

0  16t2  210
16t2  210
t2 

t  3

 7000  4

Chapter 10 Quadratic and


Exponential Functions

1. In h  16t2  112t  6, a  16 and b  112.


b

t  2a
112

t  2(16) or 3.5

2.

3.

4.

5.

The equation of the axis of symmetry is t  3.5.


h  16t2  112t  6
h  16(3.5) 2  112(3.5)  6
h  196  392  6
h  202
The vertex is at (3.5, 202).
Since the coefficient of the x2 is negative, the
vertex is a maximum point. The h-coordinate of
the vertex represents the maximum height. The
maximum height the ball reaches is 202 feet.
The t-coordinate of the vertex represents the time
when the ball reaches its maximum height. The
ball reaches its maximum height after 3.5 seconds.
The ball hits the ground when h  0. When t  7,
h  16(7)2  112(7)  6 or 6. When t  8,
h  16(8)2  112(8)  6 or 122.
Thus, the ball hits the ground between 7 and
8 seconds after it is thrown. The ball is in the air
about 7 seconds.
Replace h with 40 in the equation.

1
(10x2
10

x2  16.4x  19.8
x  16.4x  67.24  19.8  67.24
(x  8.2) 2  47.44
x  8.2  147.44
x  8.2  147.44
x  8.2  147.44 or x  8.2  147.44
 15.1
 1.3
{1.3, 15.1}
10. The poster board is not wide enough to have a
side margin of 15.1 inches. Thus, the side
margins are each about 1.3 inches wide. The top
margin is about 3  1.3 or 3.9 inches, and the
bottom margin is about 2  1.3 or 2.6 inches.
11.

4.95  116.8861
0.22

4.95  116.8861

f (t )  16t 2  210 f (t )
200
100
2

4.95  2(4.95) 2  4(0.11) (17.31)


2(0.11)

4.95  116.8861

x
or x 
0.22
0.22
 41.2
 3.8
The values mean that 3.8 years and 41.2 years
after 1977, 30% of households had cable.
12. No; the parabola only reaches a maximum height
of about 68, meaning that no more than 68% of
homes will ever have cable, which is not realistic.

Graph f(t)  16t2  210.

4

30  0.11x2  4.95x  12.69


30  30  0.11x2  4.95x  12.69  30
0  0.11x2  4.95x  17.31
x

40  16t2  250
40  40  16t2  250  40
0  16t2  210

f(t)
46
66
194
210
194
66
46

 164x)  10 (198)

h  16t2  250
40  16t2  250
6. Rewrite the equation.

t
4
3
1
0
1
3
4

210
16

t  3.6
Ignore the negative solution. It takes about
3.6 seconds to complete the ride.
8. The width of the part covered is 22  2x inches
and the height is 27  5x. Thus, the area of
the part covered can be represented by
2
(22  2x)(27  5x). This area is 3 the area of the
2
poster board, 3 (22)(27) or 396 square inches.
Write the equation.
(22  2x)(27  5x)  396
9.
(22  2x) (27  5x)  396
594  110x  54x  10x2  396
10x2  164x  594  396
2
10x  164x  594  594  396  594
10x2  164x  198

The ratio of the volume of Model A to Model C


is 1:4.

Page 862

210
16

4t

The equation has two roots. One is between


4 and 3, and the other is between 3 and 4.

779

Mixed Problem Solving

PQ249J-6481F-P01-14[766-785] 26/9/02 9:07 PM Page 780 Sahuja Ahuja_QXP_06:Desktop Folder:Chandra:Algebra_FNL_Delivery:

13. P(x)  335(1.007)x


x
0
100
200
300
400

Page 863

P(x)
335
673
1352
2716
5455

P (x )

6000
5000
4000
3000
2000
1000

1. 200 km  200,000 m
Thus, the radius of the satellites orbit is
6,380,000  200,000 or 6,580,000 meters.
P (x )  355(1.007)x

200

400

The y-intercept is 335.


14. The y-intercept represents the population of Asia
in year 0 or 1650. Thus, the population of Asia in
1650 was 335,000,000.
15. The year 2050 is 2050  1650 or 400 years after
1650. Thus, x  400.

16. A  P 1  n

A  10,000 1 

9
32

27
64

0.0745 1(4)
1

81
128

243
256

729
512

2187
1024

      
3

2 2

2 2
 rn1

2

2

2

6561
2048


3

2

(6.67  10  11 ) (5.97  1024 )


6.58  106

39.8199  1013
6.58  106

39.8199
6.58

 107

2r
v
2(6,580,000)
7779

4. d 

2hv2
g

2(10) (10) 2
9.8

2000
9.8

5. d 

2hv2
g

2(10) (20) 2
9.8

8000
9.8

 28.57
d is about 28.57 meters.
This value is twice as great as the value in
Exercise 4.
6. Yes; because 12  1.4

19,683
4096

7. r 


3

2

3 h
162

3 (8.25)

 2.5
The radius of the container is about 2.5 inches.
8. First convert 98 acres to square miles.
98
98 acres  640 or 0.153125 mi2

2187
or about
1024
6561
sequence is 2048 or

21. The eighth term of the sequence is

2.1. The ninth term of the


about 3.2. Laurie will first exceed 3 miles during
her ninth session.
22. No; since Laurie is running every other day, her
ninth session will be on the 17th day.

Mixed Problem Solving

 14.29
d is about 14.29 meters.

20. an  a
1
1 3
an  8 2 n1

12

 5315
The orbital period of the satellite is about
5315 seconds, which is about 5315 3600 or
1.5 hour.

In 2003, T(4)  2575(1.06) 4  3250.88.


In 2004, T(5)  2575(1.06) 5  3445.93.
In 2005, T(6)  2575(1.06) 6  3652.69.
In 2006, T(7)  2575(1.06) 7  3871.85.
The total cost of his tuition will be
$3250.88  $3445.93  $3652.69  $3871.85
or $14,221.35.
18. Sample answer: Aaron could reinvest the money
he does not need each year and earn additional
interest during the time he is attending college.
3
16

GmE
r

17. y  C(1  r) t
y  2575(1  0.06) t
y  2575(1.06) t

1
8

3. T 

A  10,000(1.0745) 4
A  13,329.86
Aaron will have $13,329.86 at the end of 4 years.

19.

2. v 

 7779
The orbital velocity of the satellite is about
7779 meters per second.

P(400)  335(1.007) 400


 5455.48076
There will be about 5,455,480,760 people in Asia
in 2050.
r nt

Chapter 11 Radical Expressions


and Triangles

The area of the square is 0.153125 square miles.


A  s2
0.153125  s2
10.153125  s
0.3913  s
The length of a side of the square is about
0.3913 mile, which is 0.3913  5280 or about
2066 feet.

780

PQ249J-6481F-P01-14[766-785] 26/9/02 9:07 PM Page 781 Sahuja Ahuja_QXP_06:Desktop Folder:Chandra:Algebra_FNL_Delivery:

9. c2  a2  b2
c2  (2066) 2  (2066) 2
c2  8,536,712
c  18,536,712
c  2922
The diagonal distance is about 2922 feet.
10. Tyrones distance from the store is the length of the
hypotenuse of a right triangle with legs 32 and 45.

16. Let A represent the angle of elevation.

c2  a2  b2
c2  (32) 2  (45) 2
c2  3049
c  13049
c  55
No; the distance is about 55 blocks which is about
55 12 (there are 12 blocks per mile) or 4.6 mi.
11. Sample answer: 40 blocks south and 45 blocks
west; 38 blocks north and 47 blocks west

Page 864

377

sin A  745
A  sin1

 30

The angle of elevation is about 30


.

1. p 

Chapter 12 Rational Expressions


and Equations

1
f

2

1

1
2

1

1
2

P
2
1

12. d  2(x  x ) 2  (y  y ) 2
2
1
2
1
side 1: d  2(3  1) 2  (2  1) 2

4

 2(4) 2  12
 117

2

1m

 (5 

 2(8)  (6)
 164  36
 1100
 10

2. f  20 cm  100 cm  0.2 m

1) 2

p  0.2  5 diopters

1m

3. f  40 cm  100 cm  0.4 m


1

p  0.4  2.5 diopters


4. Sample answer: One value is negative and the
other is positive.

The perimeter is 10  117  165 or about


22.19 units.
13. The new vertices are (2, 2), (6, 4), and (14, 10).
d  2(x2  x1 ) 2  (y2  y1 ) 2
side 1: d  2(6 

2) 2

 (4 

5. y  x2

2) 2

2

1

1
4

1
4

 2(8) 2  22
 168 or 2117

4
3
2
1

side 2: d  2(14  (6)) 2  (10  4) 2


 2(8) 2  (14) 2
 1260 or 2165

2

side 3: d  2(14  2) 2  (10  2) 2

2x

The graph looks like an inverse variation graph


except it lies in Quadrants I and II, consecutive
quadrants, not opposite quadrants.
6. The values of x must be positive since x
represents distance.

 2(16) 2  (12) 2
 1400 or 20
The perimeter is 20  2117  2165 or about
44.37 units.
14. Yes; the ratio of each pair of corresponding sides
is 1:2.
15.
745 ft

4f

2

 2(4) 2  (7) 2
 165
side 3: d  2(7 

O
1

side 2: d  2(7  (3) ) 2  (5  2) 2

1) 2

1377
745 2

8.

d
t
d
t

9.

157 ft 60 min
1 mi
 1 h  5280 ft
1 min

7.

250
5

 50  157 ft/min


 1.8 mi/h

377 ft

781

Mixed Problem Solving

PQ249J-6481F-P01-14[766-785] 26/9/02 9:07 PM Page 782 Sahuja Ahuja_QXP_06:Desktop Folder:Chandra:Algebra_FNL_Delivery:

10. 0.232x  13,063,000


x  56,306,034 vehicles
11.

5.79  1010 mi

5.79

Mercury: 1.86  105 mi/s  1.86  105 s 


Venus:
Earth:
Mars:

1.08  1011 mi
1.86  105 mi/s
11

1.496  10 mi
1.86  105 mi/s

2.28  1011 mi
1.86  105 mi/s

1.08
1.86

1.496
1.86

11

7.78  10 mi
 105 mi/s

Jupiter: 1.86

2.28
1.86

 13,405 min
 20,430 min

7.78

 1.86  106 sec 

1.43  1012 mi

1.43

2.87  10

mi

2.87

Pluto:

12.

4.50
1.86

1 min
60 s

1 min
60 s

Uranus: 1.86  105 mi/s  1.86  107 s 


4.50  1012 mi
1.86  105 mi/s

 9677 min

1 min
106 sec  60 s

1 min
60 s

1 min
107 s  60 s

3.

226,338
257,808
191,310
215,180
95,195
91,738
181,596
198,484
77,691
78,282
Y
Y; B  I
AI
96,946
100,377
111,486
106,621
291,079
321,586
370,522
436,790
85,487
80,785
4. Each matrix has ten rows and one column. Both
matrices are 10  1.
5. P  A  B

 69,713 min

 128,136 min
 257,168 min

 403,226 min

9400  6900
9400  6900  3800  17,400  11,700  3300  5400
16,300
163
 57,900  579

13. 35  5  7
50  2  5  5
75  3  5  5
LCM  2  3  5  5  7 or 1050
Celeste should order 1050 bricks.
14.

1
2

257,808
226,338
215,180
191,310
91,738
95,195
198,484
181,596
78,282
77,691
I
YI
Y
100,377
96,946
106,621
111,486
321,586
291,079
436,790
370,522
80,785
85,487

yd  8 yd  14 yd
4

8  8  4



4  5  10
8
19
3

28 yd
8
3

15. No; they need 28 yd for one of each type. Then


3

30 28 yd is not a whole number, so they cannot

257,808  226,338
215,180  191,310
91,738  95,195
198,484  181,596
78,282  77,691
I
Y
100,377  96,946
106,621  111,486
321,586  291,079
436,790  370,522
80,785  85,487

use the entire bolt by making an equal number of


each type.
16. Rick rate:
Phil rate:
1
x
6
2
x
12

1 house
6 days
1 house
4 days

 4x  1
3

 12x  1
5
x
12

1

x

31,470
23,870
3457
16,888
591
Y
I
3431
4865
30,507
66,268
4702
6. Matrix P represents the difference in the
populations of the ten cities between 1999
and 1990.

12
5

x  2.4
It will take 2.4 days.

Mixed Problem Solving

Chapter 13 Statistics

1. Sample answer: The sample contained students


ages 1418.
2. Sample answers: How were the students chosen?
Was it random or were they volunteers? Were the
students given only certain careers as choices?
How many students responded?

 5188 min

1 min
60 s

Saturn: 1.86  105 mi/s  1.86  107 s 


12

1 min
60 s

1 min
106 s  60 s

 106 s 

Page 865

782

PQ249J-6481F-P01-14[766-785] 26/9/02 9:07 PM Page 783 Sahuja Ahuja_QXP_06:Desktop Folder:Chandra:Algebra_FNL_Delivery:

9a. Order the low temperatures from least to


greatest.
80, 70, 69, 66, 61, 60, 60, 60, 58,
54, 54, 52, 51, 50, 50, 50, 48, 48,
47, 47, 46, 45, 42, 40, 40, 40, 39,
37, 37, 36, 36, 35, 34, 34, 32, 32,
30, 29, 29, 27, 27, 25, 23, 19, 19,
17, 17, 16, 2, 12
The greatest temperature is 12, and the least
temperature is 80. Thus, the range is
12  (80) or 92
F.
9b. There are 50 temperatures total. The lower
quartile is the median of the 25 smallest
temperatures. The lower quartile is 51
F. The
upper quartile is the median of the 25 greatest
temperatures. The upper quartile is 29
F.
9c. The interquartile range is 29  (51) or 22
F.
9d. An outlier would be 1.5(22) or 33
F less than the
lower quartile or greater than the upper
quartile. There are no temperatures in the list
less than 51  33 or 84
F. There is one
temperature, 12
F, greater than 29  33 or
4
F. There is one outlier, 12
F.
10. Low Temperatures:
Since there are an even number of temperatures,
the median is the mean of the 25th and 26th
temperatures.

7. The matrix P shows that three cities had a


decrease in population. For each of these cities,
find the percent decrease using the absolute value
of the number from matrix P as the change and
the 1990 population as the base.
Elmira, NY:
3457
95,195

 100

345,700  95,195r
345,700
95,195

r

3.6  r
Lawton, OK:
4865
111,486

 100

486,500  111,486r
486,500
111,486

r

4.4  r
Pine Bluff, AR:
4702
85,487

 100

470,200  85,487r
470,200
85,487

r

5.5  r
Pine Bluff, AR has the greatest percent decrease.
8a. Order the high temperatures from least to
greatest.
100, 100, 104, 105, 105, 106, 106, 107, 108, 108,
109, 110, 110, 110, 110, 111, 111, 112, 112, 112,
112, 113, 113, 114, 114, 114, 114, 114, 115, 116,
117, 117, 117, 118, 118, 118, 118, 118, 118, 119,
120, 120, 120, 120, 121, 121, 122, 125, 128, 134
There are an even number of temperatures, so
find the mean of the 25th and 26th temperatures,
which are both 114.
114  114
2

40  (40)
2

The median is 40.


We found the other information necessary to
construct the plot for the low temperatures in
Exercise 9.
High Temperatures:
We found the median in Exercise 8.
The greatest temperature is 134, and the least
temperature is 100. Thus, the range of the high
temperatures is 134  100 or 34
F.
The lower quartile is the median of the 25
smallest temperatures. The lower quartile is
110
F. The upper quartile is the median of the
25 greatest temperatures. The upper quartile is
118
F.
The interquartile range is 118  110 or 8
F.
An outlier would be 1.5(8) or 12
F less than the
lower quartile or greater than the upper quartile.
There are no high temperatures less than
110  12 or 98
F. There is one high temperature,
134
F, greater than 118  12 or 130
F.
Complete the parallel box-and-whisker plot.

 114

The median high temperature is 114


F.
8b. Sample answer:

Frequency

Record High Temperatures


in United States
25
20
15
10
5
0
100109

109118

118127

 40

127136

Temperature (F)

8c. Sample answer: More than half of the states had


high temperatures of less than 118
F. Only two
states had temperatures over 126
F.
8d. Yes; to find the median, you must order the data
from least to greatest. Then it is easier to place
the values into measurement classes.

80 60 40

783

20

20

100

120

140

Mixed Problem Solving

PQ249J-6481F-P01-14[766-785] 26/9/02 9:07 PM Page 784 Sahuja Ahuja_QXP_06:Desktop Folder:Chandra:Algebra_FNL_Delivery:

3. The four most expensive arrangements are listed.


deluxe, velvet, orchids, large
deluxe, velvet, orchids, small
deluxe, satin, orchids, large
deluxe, satin, orchids, small
Find the cost of each of these.
$12.00  $3.00  $35.00  $2.50 or $52.50
$12.00  $3.00  $35.00  $1.75 or $51.75
$12.00  $2.00  $35.00  $2.50 or $51.50
$12.00  $2.00  $35.00  $1.75 or $50.75
4. This problem involves combinations since the
order of the letters is not important. Find the
number of combinations of 12 letters taken 4 at a
time.

10a. Sample answer: The range of the low


temperatures is much greater than the range of
the high temperatures. Both data sets have one
outlier at the high end of the data set.
10b. The range of the low temperatures is almost
3 times greater than the range of the high
temperatures.
11. AL
139
LA
130
OH
152
AK
180
ME
153
OK
147
AZ
157
MD
149
OR
173
AR
149
MA
142
PA
153
CA
179
MI
163
RI
129
CO
179
MN
174
SC
130
CT
138
MS
134
SD
178
DE
127
MO
158
TN
145
FL
110
MT
187
TX
143
GA
129
NE
165
UT
186
HI
88
NV
175
VT
155
ID
178
NH
152
VA
140
IL
153
NJ
144
WA
166
IN
152
NM
172
WV
149
IA
165
NY
160
WI
168
KS
161
NC
144
WY
180
KY
151
ND
181
11a. Sample answer:

C 

n r

C 

12 4





P 

n r

Frequency

P 

10
8
6

4 3

130139

140- 150149 159

160169

170179

180189

Temperature (F)

11b. Sample answer: More than half of the states had


temperature differences between 129
F and 170
F.

Page 866

Chapter 14 Probability

1. There are 3 choices for the vase, 2 choices for the


ribbon, 3 choices for the flowers, and 2 choices for
the card. Use the Fundamental Counting
Principle.
3  2  3  2  36
There are 36 possible floral arrangements.
2. The most expensive vase is $12.00, the most
expensive ribbon is $3.00, the most expensive
flowers are $35.00, and the most expensive card is
$2.50. So, the most expensive arrangement costs
$12.00  $3.00  $35.00  $2.50 or $52.50.
The least expensive vase is $5.00, the least
expensive ribbon is $2.00, the least expensive
flowers are $12.00, and the least expensive card is
$1.75. So, the least expensive arrangement costs
$5.00  $2.00  $12.00  $1.75 or $20.75.

Mixed Problem Solving

n!
(n  r)!
4!
(4  3)!
4!
1!

 4! or 24
There are 24 different three-letter arrangements
of the four letters.
6. The 24 different arrangements of the four letters
are listed.
ATR
ART
ATE
AET
ARE
AER
TAR
TRA
TAE
TEA
TRE
TER
RAT
RTA
RAE
REA
RTE
RET
EAT
ETA
EAR
ERA
ETR
ERT
Of these, 9 are words: ART, ATE, ARE, TAR, TEA,
RAT, EAT, EAR, and ERA.
7. Hidalgo had 175 hits and a total of 558 times at
bat.

4
2
0
120129

495

Melissa can choose the four letters in 495


different ways.
5. This problem involves permutations since the
order of the letters is important. Find the number
of permutations of 4 letters taken 3 at a time.

Difference in High and Low Temperatures

80119

n!
(n  r)!r!
12!
(12  4)!4!
12!
8!4!
12  11  10  9  8!
8!4!
12  11  10  9
or
4!

P(Hidalgo gets hit) 

175
558

or about 0.31

8. Compute the probabilities for the other three


players.
161

P(Alou gets hit)  454 or about 0.36


68

P(Ward gets hit)  264 or about 0.26


73

P(Cedeno gets hit)  259 or about 0.28


Alou has the greatest probablity of getting a hit
his next time at bat.
9. These events are independent.
P(both get hit)
 P(Ward gets hit)  P(Cedeno gets hit)
 0.26  0.28 or about 0.07

784

PQ249J-6481F-P01-14[766-785] 26/9/02 9:07 PM Page 785 Sahuja Ahuja_QXP_06:Desktop Folder:Chandra:Algebra_FNL_Delivery:

14. There are six possible outcomes for each of the


five digits. Use the Fundamental Counting
Principle.
6  6  6  6  6  7776
There are 7776 possible numbers.
15. Sample answer: Roll five dice and record the
results each time. Each roll represents one person
picking a winning number.
16. See students work. The theoretical probability is
1
or about 0.00013.
7776

10. He should choose Alou and Hidalgo, the two


players with the highest individual probabilities
of getting the next hit.
P(both get hit)
 P(Alou gets hit)  P(Hidalgo gets hit)
 0.36  0.31 or about 0.11
11. Yes; the probability for each value of X is greater
than or equal to 0 and less than or equal to 1, and
0.053  0.284  0.323  0.198  0.142  1, so
the probabilities add up to 1.
12. P(under 20)  0.053
13. P(50 or older)  P(50  64)  P(65 and over)
 0.198  0.142 or 0.34

785

Mixed Problem Solving

You might also like